You are on page 1of 689

About Pearson

Pearson is the world’s learning company, with presence across 70 countries


worldwide. Our unique insights and world-class expertise comes from a long
history of working closely with renowned teachers, authors and thought
leaders, as a result of which, we have emerged as the preferred choice for
millions of teachers and learners across the world.
We believe learning opens up opportunities, creates fulfilling careers and
hence better lives. We hence collaborate with the best of minds to deliver you
class-leading products, spread across the Higher Education and K12 spectrum.
Superior learning experience and improved outcomes are at the heart of
everything we do. This product is the result of one such effort.
Your feedback plays a critical role in the evolution of our products and you
can contact us - reachus@pearson.com. We look forward to it.

A01_MADAN 07_65901_FM.indd 1 09/01/23 3:38 PM


This page is intentionally left blank

A01_MADAN 07_65901_FM.indd 2 09/01/23 3:38 PM


NTA—UGC

TEACHING AND
RESEARCH APTITUDE
PAPER 1
NET | SET | JRF

2023
Seventh Edition

KVS MADAAN

A01_MADAN 07_65901_FM.indd 3 09/01/23 3:38 PM


To
my revered gurus, my parents
Smt. Devika Rani and Late Shri Prem Singh Madaan

*The 2022 Question paper set contains memory-based questions, lawfully curated solely by the author relying upon
the memory of past attending candidates of the 2022 NTA-UGC examination. The copyright in the original question
papers is owned and vested with the concerned owners and Pearson does not claim any ownership in such copyright.
The answers to these questions have also been solely developed by the author herein, and Pearson shall not be liable for
any claims arising out of usage/accuracy of the content herein”.

Copyright © 2023 Pearson India Education Services Pvt. Ltd

Copyright © 2022, 2021, 2020, 2019, 2016, 2014 Pearson India Education Services Pvt. Ltd

Published by Pearson India Education Services Pvt. Ltd, CIN: U72200TN2005PTC057128.

No part of this eBook may be used or reproduced in any manner whatsoever without the publisher’s
prior written consent.

This eBook may or may not include all assets that were part of the print version. The publisher
reserves the right to remove any material in this eBook at any time.
ISBN 978-93-560-6590-1

EISBN 978-93-570-5140-8

Head Office: 1st Floor, Berger Tower, Plot No. C-001A/2, Sector 16B, Noida–201 301,
Uttar Pradesh, India.
Registered Office: 7th Floor, SDB2, ODC 7, 8 & 9, Survey No.01 ELCOT IT/ ITES - SEZ,Sholinganallur,
Chennai – 600119, Tamilnadu, India.
Phone: 044-66540100
Website: in.pearson.com, Email: companysecretary.india@pearson.com

A01_MADAN 07_65901_FM.indd 4 09/01/23 3:38 PM


Contents
Prefacevii
About NTA ix
About the Author x
Syllabusxi
Strategy about NTA-UGC NET Exam xiii
Memory-based UGC NET Question Paper 2022 - Set 1 xv
Memory-based UGC NET Question Paper 2022 - Set 2 xxxiii

1 Teaching Aptitude 1.1

2 Research Aptitude 2.1

3 Comprehension 3.1

4 Communication 4.1

5 Mathematical Reasoning and Aptitude 5.1

6 Logical Reasoning 6.1

7 Data Interpretation 7.1

8 Information and Communication Technology (ICT) 8.1

9 People, Development and Environment 9.1

10 Higher Education System 10.1

MOCK TEST M.1


NTA-UGC NET/JRF Paper 1 December 2019  Z.3
NTA-UGC NET/JRF Paper 1 2020 Z.12
NTA-UGC NET/JRF Paper 1 2021 - Set 1 Z.24
NTA-UGC NET/JRF Paper 1 2021 - Set 2 Z.38

A01_MADAN 07_65901_FM.indd 5 09/01/23 3:38 PM


This page is intentionally left blank

A01_MADAN 07_65901_FM.indd 6 09/01/23 3:38 PM


Preface

The NET-JRF aspirants are most welcome go through this edition of NTA UGC NET/SET/JRF Paper 1–Teaching and
Research Aptitude title which has helped lakhs of candidates to qualify the NET JRF exam and pursue career in academ-
ics and research in higher education institutions. As marks obtained in Paper 1 are counted towards making the final list,
so this paper becomes automatically important. Our main objective has been to make our dear students ‘atmanirbhar’
in qualifying the exam. Pearson Education has always helped in this endeavour. This is an updated version of the book,
with a lot of new and updated practice questions.
This title covers not just teaching and research aptitude but also comprehension, mathematical ability, logical
reasoning, ICT and awareness relating to environment science and higher education. Thus, the syllabus has been divided
into ten units accordingly by UGC. In 2019, some ancient education topics were added in the syllabus so that we get in
touch with our ancient roots. We have given Sanskrit or Hindi translation of important words from Indian logic in Unit 6
and ancient education in Unit 10 so that students feel comfortable in grasping the topics. These are actually in continuity
with New Education Policy, 2020 so that it works for the holistic development of the students.
The author has taken special care so that the dynamic aspect of exam pattern is maintained, we have given space to
previous year papers as well as to future questions. We have taken special care of subjective knowledge and comprehensive
skills in the form of assertion reason and multiple statements type of questions.
This tile may be helpful to PhD admissions as per new UGC Guidelines, 2022. Many have travelled their successful
journey towards Fellowship programmes of IIMs with the help from this book.

Key Features
The language of this book has been kept very simple so that students are able to understand the topics in a lucid manner.
The discussion is supported by diagrams, figures, tables, updated data and pervious years’ questions as ‘stopovers’ so
that students can prepare smoothly and move ahead with confidence. There are more than 2600 questions for regular
practice. There is inclusion of two July–October 2022 papers as well.
For suggestions and feedback, a NET aspirant may write to madaankvs@gmail.com and at Telegram Link–t.me/@
KVSPaper1.

Acknowledgments
I feel indebted to Prof. Surindra Lal, retired Principal of Multani Mal Modi College, Patiala for always being a mentor and
continuous source of inspiration. I am grateful to my faculty colleagues Prof Sunita Mehta from Department of English,
CMK National Girls College (Sirsa), Prof Mahesh Mehta from Department of English, MM College, Fatehabad, Prof SP
Sajjan from Department of Computers Sciences, Govt First Grade College for Women, Jamkhandi (Karnataka) and Prof
Debasree Paul from Department of Political Science, Rabindra Bharati University (West Bengal). Prof S P Sajjan from,
Computer Sciences for their active support in academic and language matters. There is a great role of Prof Harish Jhamb,
Director, Daffodils College of Education (Fatehabad), Prof. (Dr) Pankaj Thakur, JNU (New Delhi), Prof Aditi Vinod Arya,
Unacademy, Prof Preeti Mandyal, Unacademy, Prof Manisha Kalra, SSD College (Bathinda), Prof Abhilasha N, Mulund
College of Commerce, Mumbai and Research Scholar Priya Gupta for their enormous contribution in this endeavour.
I am indebted to Shreya Malik, Convenor, Shodh Delhi, Prof Rahul Mishra, Prof Purnachander Konderi, and Yasar
Arafat, HOPE, WB for their online active support. Parv Luthra, a doctoral student of Pravara Institute of Medical Sciences,
Loni, (Maharashtra) always provided me the motivational support.
My special gratitude is always there for Shukti Mukherjee, Ruchira Dash, Bhupesh Sharma and Ashi Jain my esteemed
colleagues from Pearson Education for the successful implementation of this project.
Our sales and distribution magnets Vishal Dhawan, Binit Shukla, Desh Das Bandhu, Raman Pruthi and Srinivasan did
a great social service by taking this valuable product to emerging professors of India.
The best kind of help always comes from my better half Anshu Bala for the required motivation for this best product.
I always feel a divine power supporting my efforts when my kids Jiya, Srijan and Jappan are around. They actually made
me work harder and smarter on the project.

KVS Madaan

A01_MADAN 07_65901_FM.indd 7 09/01/23 3:38 PM


This page is intentionally left blank

A01_MADAN 07_65901_FM.indd 8 09/01/23 3:38 PM


About NTA

The National Testing Agency (NTA) has been established as a premier, specialist, autonomous and ­self-sustained test-
ing organization to conduct entrance examinations for admission or fellowship in higher educational ­institutions. It has
been set up with the following specific objectives:
1. To conduct tests in an efficient, transparent and international standard, in order to assess the competency of candi-
dates for admission and recruitment purposes
2. To undertake research on educational, professional and testing systems in order to identify gaps in the knowledge
system and take steps for bridging them
3. To identify experts and institutions for formulating examination questions
4. To produce and disseminate information and research on education and professional development standards.
The NET exam is conducted on behalf of UGC to determine the eligibility of candidates opting for the post of Assistant
Professor only or Junior Research Fellowship (JRF) and Assistant Professor in Indian universities and colleges. The JRFs
selected acquire an essential stipend while pursuing PhD/Fellowship.
Until recently, the CBSE conducted NET exam in 84 subjects at 91 selected cities across the country. From December
2018, the UGC-NET (Eligibility for Assistant Professor only or Junior Research Fellowship and Assistant Professor) is
now conducted by NTA.
The NTA intends to create a question bank for all subjects using modern techniques. The administration of NTA has
been entrusted to the governing body, with Shri R. Subrahmanyam, Secretary, Department of Higher Education, MHRD,
being its first chairman.

Eligibility
Candidates who have secured at least 55% marks in master’s degree or equivalent examination from recognized uni-
versities in humanities (including languages) and social sciences, computer science and applications, electronic sci-
ence, etc., are eligible to appear for this test. A relaxation of 5% for other reserved categories has also been entitled to
candidates.
Candidates who are pursuing their master’s degree or equivalent course should comply to all the eligibility criteria.
It is essential for the candidates to complete their master’s degree or equivalent examination within two years from the
date of NET result with the required percentage of marks, failing which they will be treated as disqualified.
A Ph.D. degree holders whose master’s–level examination was completed by 19th September 1991 (­irrespective of
the date of result declaration) shall be eligible for a relaxation of 5% in aggregate marks, i.e., from 55% to 50%, for
appearing in NET.
Candidates are advised to appear in the subject of their post-graduation only. Candidates whose post-graduation
subject is not covered in the list of subjects may appear in a related subject.

A01_MADAN 07_65901_FM.indd 9 09/01/23 3:38 PM


About the author

KVS Madaan, Author and Director, KVS Madaan Online Classes/KvsMadaan.com has
been a Former Associate Director, Arni School of Business Management, Arni University,
Himachal Pradesh is a renowned author and academician. He is post graduate in Business
Administration from Haryana School of Business, GJUST, Hisar. He has helped thousands
of career aspirants realize their goals. He worked at top positions in many Universities
and institutions. He has been a keynote speaker at UGC and AICTE sponsored faculty
development programmes. He has authored a book for Commerce, NTA UGC NET, Paper
2.
He has active channel on YouTube – KVS Madaan Online Classes and a website kvsmadaan.
com.

A01_MADAN 07_65901_FM.indd 10 09/01/23 3:38 PM


Syllabus

UNIVERSITY GRANTS COMMISSION


NET BUREAU

PAPER-I
Subject: General Paper on Teaching and Research Aptitude

The main objective is to assess the teaching and research capabilities of the candidates. The test aims to assess teaching
and research aptitude as well. Candidates are expected to possess and exhibit cognitive abilities, which include com-
prehension, analysis, evaluation, understanding the structure of arguments, deductive and inductive ­reasoning. The
candidates are also expected to have a general awareness about teaching and learning processes in the higher educa-
tion system. Further, they should be aware of interaction between people, the environment, natural resources and their
impact on the quality of life.
The details of syllabus are as follows:

I. Teaching Aptitude
• Teaching: Concept, Objectives; Levels of teaching (Memory, Understanding and Reflective), Characteristics and
basic requirements
• Learner characteristics: Characteristics of adolescent and adult learners (Academic, Social, Emotional and
Cognitive), Individual differences
• Factors affecting teaching, related to the teacher, learner, support material, instructional facilities, learning environ-
ment and institution
• Methods of teaching in institutions of higher learning: Teacher-centred vs. Learner-centred methods, Off-line vs
On-line methods (Swayam, Swayamprabha, MOOCs etc.)
• Teaching Support Systems: Traditional, Modern and ICT based
• Evaluation Systems: Elements and Types of evaluation, Evaluation in choice-based credit system in higher educa-
tion, Computer-based testing, Innovations in evaluation systems

II. Research Aptitude


• Research: Meaning, Types and Characteristics, Positivism and Post-positivistic approach to research
• Methods of research: Experimental, Descriptive, Historical, Qualitative and Quantitative methods
• Steps of Research
• Thesis and article writing: Format and styles of referencing
• Application of ICT in research
• Research ethics

III. Comprehension
• A passage of text will be given. Questions will be asked from the passage to be answered.

IV. Communication
• Communication: Meaning, types and characteristics of communication
• Effective communication: Verbal and Non-verbal, Inter-cultural and group communications, Classroom
communication
• Barriers to effective communication
• Mass-Media and Society

A01_MADAN 07_65901_FM.indd 11 09/01/23 3:38 PM


xii Syllabus

V. Mathematical Reasoning and Aptitude


• Types of reasoning
• Number series, Letter series, Codes and relationships
• Mathematical Aptitude (Fraction, Time & Distance, Ratio, Proportion and Percentage, Profit and Loss, Interest and
Discounting, Averages etc.)
VI. Logical Reasoning
• Understanding the structure of arguments: argument forms, structure of categorical propositions, Mood and
Figure, Formal and Informal fallacies, Uses of language, Connotations and denotations of terms, Classical square of
opposition.
• Evaluating and distinguishing deductive and inductive reasoning
• Analogies
• Venn diagram: Simple and multiple use for establishing validity of arguments
• Indian Logic: Means of knowledge
• Pramanas: Pratyaksha (Perception), Anumana (Inference), Upamana (Comparison), Shabda (Verbal ­testimony),
Arthapatti (Implication) and Anupalabddhi (Non-apprehension)
• Structure and kinds of Anumana (inference), Vyapti (invariable relation), Hetvabhasas (fallacies of inference)
VII. Data Interpretation
• Sources, acquisition and classification of data
• Quantitative and qualitative data
• Graphical representation (Bar-chart, Histograms, Pie-chart, Table-chart and Line-chart) and mapping of data.
• Data Interpretation.
• Data and governance.
VIII. Information and Communication Technology (ICT)
• ICT: General abbreviations and terminology
• Basics of Internet, Intranet, E-mail, audio and video-conferencing
• Digital initiatives in higher education
• ICT and Governance
IX. People, Development and Environment
• Development and environment: Millennium development and sustainable development goals
• Human and environment interaction: Anthropogenic activities and their impacts on environment
• Environmental issues: Local, regional and global; Air pollution, water pollution, soil pollution, noise pollution, waste
(solid, liquid, biomedical, hazardous, electronic); Climate change and its socio-economic and political dimensions.
• Impacts of pollutants on human health
• Natural and energy resources: Solar, Wind, Soil, Hydro, Geothermal, Biomass, Nuclear and Forests
• Natural hazards and disasters: Mitigation strategies
• Environmental Protection Act (1986), National Action Plan on Climate Change, International agreements/efforts:
Montreal Protocol, Rio Summit, Convention on Biodiversity, Kyoto Protocol, Paris Agreement, International Solar
Alliance
X. Higher Education System
• Institutions of higher learning and education in ancient India
• Evolution of higher learning and research in post-independence India
• Oriental, Conventional and Non-conventional learning programmes in India
• Professional, Technical and Skill-Based education.
• Value education and environmental education.
• Policies, Governance and Administration.
NOTE:  (i) Five questions, each carrying 2 marks, are to be set from each Module.
(ii) Whenever graphical/pictorial question(s) are set for sighted candidates, a passage followed by equal num-
ber of questions and weightage be set for visually impaired candidates.

A01_MADAN 07_65901_FM.indd 12 09/01/23 3:38 PM


Strategy about NTA Exam

The National Eligibility Test is a national-level exam conducted in India by NTA on behalf of the University Grants
Commission (UGC). This test enables post-graduate students to qualify for university-level teaching jobs in India and
also to seek admission to Ph.D level programmes. The test ensures that a candidate qualifies for the minimum standards
in the teaching profession and research. The test is conducted in Humanities (including languages), Social Sciences,
Environmental Sciences, Geography, Commerce, Management to name a few.
Once you clear UGC NET, you are eligible to become an Assistant Professor or pursue Junior Research Fellowship in
distinguished universities. The vacancies are announced from time to time and you have to apply for the same.
Once you clear NET, you can apply for the post of Assistant Professor as and when the vacancies are announced by
the colleges and universities. While clearing NET is a mandatory criterion in many colleges and universities, the final
appointment will depend on how well you fare in the interview conducted by the concerned university.
Junior Research Fellowship is for those who have their heart set on research. You become eligible for the Junior
Research Fellowship if you have applied for it in your application form and you have subsequently cleared NET. Once you
get selected, you can then pursue research in any prestigious university and get the tuition fees reimbursed.

Tips and Techniques to Prepare for UGC NET


❍ Preparing well in time: The preparation for UGC Net exam starts while you are undergoing post-­graduate course.
Once post-graduation is complete, a focused preparation of three to six months is mandatory. Preparation should be
planned and implemented in such a manner that it leaves scope for revision as well.
❍ Refer to past questions: The previous years’ questions help the career aspirant get familiarized with the pattern of
the exam. Go through them and try to solve at least a few of them. This will give you an idea of what to expect in the
exam, and it also builds confidence in you. Take help from the teachers or seniors if required. It is better if you can
make a list of important topics, though the test is of an objective pattern now.
❍ Gaining familiar with the syllabus: Run through the syllabus thoroughly. It is important that you understand the
syllabus completely and prepare for each topic accordingly. As all papers are objective now, do not leave out any
topic.
❍ Prioritising the subjects: Out of the two papers, Paper II is based on the subject of your choice, whereas Paper I will
be based on general awareness and aptitude. It is very important that you have a complete hold over your subject, as
you can score well there. While focusing your attention on the subject papers, start preparing for the general aware-
ness test by solving sample papers.
For UGC Net Paper I, which is not merely qualifying now, marks are calculated towards the final score. Therefore,
students take this paper seriously now. Earlier, Paper I was only qualifying in nature: one had to score only 40% in
this paper, only then Paper II would be evaluated.
❍ Making personal notes: This is applicable for all the subjects and Paper I as well. These notes work as a kind of
‘ready reckoner’ once the exam nears fast. In general, go for big page notebooks or registers, divide the page into
three columns—on the left-hand side, write the main topics, titles, serial number, etc., the middle column is for
main discussion of the topic, and right-hand side for any addition, updations, references, etc., that may be required
to be done at some later date. The notes help in revision. The same may be done in case of Paper I also.
❍ Build upon speed and quality of preparation: While your aim should be on completing all the topics mentioned
in the syllabus, quality must also be kept in mind. While preparing, concentrate on improving the quality of your
preparation, which means you can concentrate more on zeroing down on what exactly and how much has to be
studied.

A01_MADAN 07_65901_FM.indd 13 09/01/23 3:38 PM


xiv Strategy about NTA-UGC NET Exam

❍ Time management: Time management depends upon the nature of the subject. Some subjects are purely theoreti-
cal, some are technical and some are both. Utilise time in an efficient manner. Have a strategy in place as to how to
complete all the questions within the given time. It is best if you time yourself while preparing the subjects.
❍ Be confident: There is no substitute of confidence. Confidence is the outcome of your degree of preparation.
Once you decide that you carve to eke out a distinguished career in teaching, then UGC Net Exam is the stepping
stone.
❍ If we take into account December 2021 / January 2022 UGC NTA NET JRF Exams, it is moving towards surprise ele-
ments, innovations and updations. Thus, it needs more efforts as well.

“All The Best and Do Your Best”

A01_MADAN 07_65901_FM.indd 14 09/01/23 3:38 PM


Memory-based UGC NET
Question Paper 2022 - Set 1 *
INSTRUCTIONS
Time Allowed: 1 hour  50 × 2 = 100 Marks

1. This paper consists of Fifty (50) objective type questions of Two (2) marks each.
2. All questions are compulsory.
3. Each item has four alternative responses marked (a), (b), (c) and (d). You have to darken the circle as indicated below on the
correct response against each item.
Example: a b c d where (c) is the correct response.
4. Your responses to the items are to be indicated in the OMR Sheet given at the end of the book.
5. Read instructions provided with each question carefully.
6. There are no negative marks for incorrect answers.

Directions for questions 1 to 5: Based on the data in 3. If 25% diesel engine cars in state ‘C’ are Air-Condi-
table, answer the five questions that follow: tioned (AC), then what is the number of diesel engine
Consider the following table that shows the percentage cars which are non-AC in State ‘C’?
distribution of cars and ratio between diesel and petrol (a) 150 (b) 90
engine cars in four different states (A-D).
(c) 190 (d) 210
Total number of cars in all the four states is 1400.
State wise distribution of cars 4. What is the difference between the total number of
cars in State ‘C’ and the number of petrol engine cars
Ratio in State ‘B’?
Percentage
State Diesel Engine Cars: (a) 212 (b) 224
Distribution of cars
Petrol Engine Cars
(c) 204 (d) 196
A 14% 3:4
5. What is the sum of petrol engine cars in all the states
B 28% 5:9 together?
C 32% 5:3 (a) 710 (b) 712
D 26% 1:1 (c) 714 (d) 716
6. Which of the following types of learners learn with
1. What is the difference between the number of die-
the help of flow charts or diagrams?
sel engine cars in state ‘B’ and the number of petrol
(a) Tactile (b) Visual
engine cars in state ‘D’?
(c) Auditory (d) Rote
(a) 24 (b) 42
7. Match List I with List II.
(c) 56 (d) 68
List I List II
2. Number of petrol engine cars in state ‘C’ is what percent Concepts Main proponent
more than the number of diesel engine cars in state ‘A’? A. Integral education I. John Dewey
(a) 100% (b) 125% B. Focus on spiritual II. Sri Aurobindo
(c) 200% (d) 120% aspects of Indian
philosophy

*The 2022 Question paper set contains memory-based questions, lawfully curated solely by the author relying upon the memory of past attending
candidates of the 2022 NTA-UGC examination. The copyright in the original question papers is owned and vested with the concerned owners and
Pearson does not claim any ownership in such copyright. The answers to these questions have also been solely developed by the author herein, and
Pearson shall not be liable for any claims arising out of usage/accuracy of the content herein”.

A01_MADAN 07_65901_FM.indd 15 09/01/23 3:38 PM


xvi Memory-based UGC NET Question Paper 2022 - Set 1

List I List II (c) C, D and E only


Concepts Main proponent (d) A, B and C only
C. Learning to take place III. Dr. Sarvepalli 14. Which of the following factors influence the sample
in nature and from Radhakrishan size for a project?
nature A. Type of project
D. Experiential learning IV. Rabindranath Tagore B. Purpose of project
Choose the correct answer from the options given
C. Previous research in the area
below: D. Researcher’s personality
(a) A – II, B – IV, C – III, D – I E. His/her standing as a researcher
(b) A – I, B – II, C – III, D – IV Choose the correct answer from the options given

(c) A – III, B – II, C – I, D – IV below:
(d) A II, B – III, C – IV, D – I (a) A, B and C only
8. In order to identify individual differences of learners in a (b) B, C and D only
class, which of the following can be used by a teacher? (c) C, D and E only
(a) Summative Assessments (d) A, C and E only
(b) Formative Assessments
(c) Diagnostic Assessments 15. Match List I with List II.
(d) Peer Assessments List I List II
Research method Characteristic
9. The e-content generated under e-PG Pathshala project
for Non-Technical Post-Graduate courses in hosted on A. Content analysis I. Summarization
web by B. Social network II. Use of bipolar
(a) NPTEL (b) INFLIBNET analysis adjectives
(c) SWAYAM (d) IGNOU
C. Semantic differential III. Focus on dyadic
10. Which of the following is NOT an affective domain? relations
(a) Receiving (b) Analysing
D. Meta analysis IV. Context unit
(c) Valuing (d) Organizing
11. When a variable leads to a spurious interpretation of Choose the correct answer from the options given

research results, it is called below:
(a) Control variable (a) A – II, B – III, C – IV, D – I
(b) External variable (b) A – IV, B – III, C – II, D – I
(c) Conflicting variable (c) A – I, B – II, C – III, D – IV
(d) Confounding variable (d) A – III, B – IV, C – I, D - II

12. The ‘Age’ of a respondent is an example of 16. In a classroom, an effective listener


(a) Parallel variable (a) Resists distractions
(b) Monadic variable (b) Seeks diversions
(c) Dyadic variable (c) Prefers message filtering
(d) Simple variable (d) Judges the message delivery style

13. Which of the following are disadvantages of qualita- 17. The two way, face-to-face communication is techni-
tive research? cally referred to as
A. Too sensitive to the context (a) Level one (b) Level two
B. Open to debate (c) Level three (d) Level four
C. Too subjective 18. From the communication perspective, which of the
D. Difficult to generalize following variables distinguish one culture from
E. Issue of replication another?
Choose the correct answer from the options given
A. Learning skills
below: B. Time sense
(a) A, D and E only C. Level of gratitude
(b) B, C and D only D. Level of honesty

A01_MADAN 07_65901_FM.indd 16 09/01/23 3:38 PM


Memory-based UGC NET Question Paper 2022 - Set 1 xvii

Choose the correct answer from the options given


(a) 12 (b) 15
below: (c) 20 (d) 10
(a) A and B only (b) B and C only 26. If the statement ‘No birds are mammals’ is given as
(c) C and D only (d) A and D only true, what could be immediately inferred from it?
19. Given below are two statements: A. ‘All birds are mammals’ is false
B. ‘Some birds are mammals’ is false
Statement I: Small group communication fulfills the
relationship needs. C. ‘Some birds are not mammals’ is false
D. ‘Some birds are not mammals’ is true
Statement II: However, small group communication
does not serve task related needs. Choose the correct answer from the options given

below:
In light of the above statements, choose the correct
(a) A, B and C only
answer from the options given below:
(a) Both statement I and statement II are true (b) A, B and D only
(b) Both statement I and statement II are false (c) A and D only
(c) Statement I is true but statement II is false (d) A and C only
(d) Statement I is false but statement II is true 27. “XYZ Home Finance offers best financial product in
the country because like a family member and a good
20. Which of the following are the functions of media as friend, XYZ Home Finance fulfills your need to have
related to their audiences? your own sweet home”. Which of the following falla-
A. Creation of passive longing for media products cies is committed in this argument?
B. Omnivorous devouring of media content (a) Ad Populum
C. Compensation through identification (b) Ad Hominem
D. Vicarious wish fulfillment (c) Hasty Generalisation
E. Sources of suggestion for appropriate role (d) Red Herring
Choose the correct answer from the options given
28. Given below are two statements:
below:
Statement I: ‘Some S is P’ is contrary to ‘Some S is not
(a) A, B and C only (b) B, C and D only
P’.
(c) A, B and E only (d) C, D and E only
Statement II: ‘Some S is not P’ is contradictory to ‘All
21. Find the missing term in the series given as: S is P’.
4, 6, 12, 14, 28, 30, ?
In light of the above statements, choose the correct
(a) 32 (b) 36
answer from the options given below:
(c) 52 (d) 60
(a) Both Statement I and Statement II are true
22. In a certain code, O is written as E, A as C, M as I, S as (b) Both Statement I and Statement II are false
O, N as P, E as M, I as A, P as N and C as S, then how (c) Statement I is true but Statement II is false
will COMPANION be written in that code? (d) Statement I is false but Statement II is true
(a) SEINCPAEP (b) SENCIAPEP
(c) SENCPIAPE (d) SNEIPCAPE 29. Given below are two statements :
Statement I: For the Indian thinkers, inference (anu-
23. In a class of 43 students, Aashman’s rank is 16th from
mana) means only a syllogistic inference based on the
the bottom. What is his rank from the top?
relation of invariable concomitance between middle
(a) 28 (b) 27
term and major term.
(c) 26 (d) 29
Statement II: There should be absence of the middle
24. A pipe can fill a water tank three times as fast as term wherever there is an absence of the major
another pipe. If both are used together, these can fill term.
the tank in 36 minutes. How much time will be taken
by the slower pipe alone to fill the tank? In light of the above statements, choose the correct
(a) 192 minutes (b) 108 minutes answer from the options given below:
(c) 144 minutes (d) 160 minutes (a) Both Statement I and Statement II are true
(b) Both Statement I and Statement II are false
25. A retailer buys 40 pens at the marked price of 36 pens
from a wholesaler. If he sells these pens giving a dis- (c) Statement I is true but Statement II is false
count of 1%, what is the profit percentage? (d) Statement I is false but Statement II is true

A01_MADAN 07_65901_FM.indd 17 09/01/23 3:38 PM


xviii Memory-based UGC NET Question Paper 2022 - Set 1

30. Given below are two statements: (a) Both Statement I and Statement II are true
Statement I: According to Classical Indian Logicians (b) Both Statement I and Statement II are false
(Naiyayikas), the inference from the effect of the (c) Statement I is true but Statement II is false
cause is called Purvavat. (d) Statement I is false but Statement II is true
Statement II: According to Naiyayikas, the inference 35. Match List I with List II.
of cause from the effect is not possible.
In light of the above statements, choose the correct List I List II
answer from the options given below: Computer Term Definition

(a) Both Statement I and Statement II are true A. Spyware I. W


 ireless technology used
for transmitting data over
(b) Both Statement I and Statement II are false short distances
(c) Statement I is true but Statement II is false B. Firewall II. Software that allows the
(d) Statement I is false but Statement II is true user to gain information
31. Given below are two statements, one is labeled as about another person’s
computer activity
Assertion A and the other is labeled as Reason R.
C. Bluetooth III. An attack on a network
Assertion A: In computer networking a MODEM is
which involves flooding the
considered as both input and output device. network with useless traffic
Reason R: MODEM sends data (upload/output) and D. Denial of service IV. H
 ardware or software
receives data (download/input) at the same time. designed to protect a
In light of the above statements, choose the correct network by controlling what
answer from the options given below: data can and cannot enter
(a) Both A and R are true and R is the correct expla-
nation of A Choose the correct answer from the options given

(b) Both A and R are true but R is NOT the correct below:
explanation A (a) A – III, B – IV, C – I, D – II
(c) A is true but R is false (b) A – IV, B – II, C – II, D – I
(d) A is false but R is true (c) A – I, B – IV, C – III, D – III
32. Programs that come into a computer system disguised (d) A – II, B – IV, C – I, D - III
as something else, are called:
36. Which of the following air pollutants is known to be a
(a) Trojan horses (b) Viruses
human carcinogen?
(c) Web bugs (d) Zombies (a) Carbon monoxide
33. You have been instructed to install additional RAM (b) Asbestos fiber
into a computer by your teacher. Identify the correct (c) Nitrogen dioxide
order of the following steps A-C that you should fol- (d) Sulphur dioxide
low to install the RAM safely.
A. Install RAM 37. Which region of India experiences the maximum
B. Isolate computer from the power supply number of cyclones?
C. Connect anti-static strap to wrist (a) East Coast
Choose the correct answer from the options given
(b) West Coast
below: (c) Central India
(a) A, B, C (b) C, A, B (d) Northern India
(c) B, A, C (d) B, C, A
38. Which of the following statements about biodiversity
34. Given below are two statements: are correct?
Statement I: SWAYAM initiative is a significant step A. Biodiversity can support ecosystem stability.
towards providing financial and technical assistance B. Human activities have sharply increased species
to young start-up entrepreneurs. extinctions.
Statement II: The tagline of Digilocker logo says “My C. Habitat fragmentation has no adverse impact on
Documents, Anytime, Anywhere”. biodiversity.
In light of the above statements, choose the correct D. About 70% of all known species are invertebrates.
answer from the options given below:

A01_MADAN 07_65901_FM.indd 18 09/01/23 3:38 PM


Memory-based UGC NET Question Paper 2022 - Set 1 xix

(a) A, B and C only (c) National Scientists and Social Scientists


(b) A, B and D only Foundation
(c) B, C and D only (d) National Space Science and Society Forum
(d) A, C and D only 44. As per NEP 2020, National Educational Technology
Forum (NETF) will have following functions:
39. Identify the major sources of methane emissions. A. Provide independent evidence–based advice to
A. Landfills B. Mountain rivers central and state government agencies in technol-
C. Paddy fields D. Wetlands ogy–based interventions
Choose the correct answer from the options given
B. Articulate new directions for research and
below: innovation
(a) A, B and C only C. Envision strategic thrust area in the educational
(b) B, C and D only technology domain
(c) A, B and D only D. Develop curriculum for technical courses
(d) A, C and D only E. Build intellectual and institutional capacities in
40. Match List I with List II educational technology
Choose the correct answer from the options given
List I List II
below:
Objectives/Provisions International
Treaties/Agreements
(a) A, B, C and D only
(b) B, C, D and E only
A. Emissions Trading I. Paris Agreement
(c) A, B, C and E only
B. Sustainable Management II. K
 yoto Protocol (d) A, B, D and E only
of Resources
45. The following are the main objectives of curriculum
C. Clean affordable energy III. International Solar revision:
Alliance A. To align it better with vision and mission of the
D. Intended Nationally IV. Rio Summit institution
Determined B. To make it learning outcome based
Contributions
C. To make it more effective in improving attain-
Choose the correct answer from the options given
ment levels against programme outcomes
below: D. To develop capacities of students for competitive
(a) A – II, B – IV, C – I, D – III examination
(b) A – II, B – IV, C – III, D – I E. To meet the expectations of all stakeholder
(c) A – I, B – IV, C – III, D – II Choose the correct answer from the options given
(d) A – I, B – IV, C – II, D - III below:
41. Who is fully credited with the introduction of English (a) A, B, C and D only
education system in India, officially through the nec- (b) A, B, C and E only
essary order issued by Bentinck, the Governor Gen- (c) A, C, D and E only
eral of India on 7th March 1835? (d) B, C, D and E only
(a) Zachary Macaulay 46. Read the given passage carefully and answer the five
(b) Thomas Babington Macaulay questions that follow:
(c) Alexander Duff Many great inventions are initially greeted with ridi-
(d) Charles Trevelyan cule and disbelief. The invention of the airplane was
42. Which University did Xuang Zang and I-Quing visit in no exception. Although many people who heard about
India? the first powered flight on December 17, 1903, were
(a) Nalanda University excited and impressed, others reacted with peals of
(b) Takshashila University laughter. The idea of flying an aircraft was repulsive
to some people. Such people called Wilbur and Orville
(c) Banaras Hindu University
Wright, ‘the inventors of the first flying machine’,
(d) Vikramshila University
impulsive fools. Negative reactions, however, did not
43. NSSF, as envisaged by the National Knowledge Com- stop the Wrights. Impelled by their desire to succeed,
mission, stands for they continued their experiments in aviation.
(a) National Science and Social Science Foundation Orville and Wilbur Wright always had a compel-
(b) National Scientific and Social Science Foundation ling interest in aeronautics and mechanics. As young

A01_MADAN 07_65901_FM.indd 19 09/01/23 3:38 PM


xx Memory-based UGC NET Question Paper 2022 - Set 1

boys, they earned money by making and selling kites half an hour at a time. Others had flown in balloons
and mechanical toys. Later, they designed a newspa- and hang gliders, but the Wright brothers were the
per-folding machine, built a printing press, and oper- first to build a full-size machine that could fly under
ated a bicycle repair shop. In 1896, when they read its own power. As the contributors of one of the most
about the death of Otto Lilienthal, the brothers’ inter- outstanding engineering achievements in history, the
est in flight grew into a compulsion. Wright brothers are accurately called the fathers of
Lilienthal, a pioneer in hang-gliding, had con- aviation.
trolled his gliders by shifting his body in the desired 47. The major interest of Wright brothers was in
direction. This idea was repellent to the Wright broth- (a) Making kites
ers, however, they searched for more efficient meth- (b) Producing mechanical toys
ods to control the balance of airborne vehicles. In (c) Designing newspaper–folding machines
1900 and 1901, the Wrights tested numerous glid-
(d) Methods to control airborne vehicles
ers and developed control techniques. The brothers’
inability to obtain enough lift power for the gliders 48. The Wright brothers succeeded in
almost led them to abandon their efforts. (a) Testing numerous gliders
After further study, the Wright brothers con- (b) Proving previous air pressure tables wrong
cluded that the published tables of air pressure on (c) Setting up a wind tunnel
curved surfaces must be wrong. They set up a wind (d) Finding desired direction for gliders
tunnel and began a series of experiments with model 49. Which of the following was responsible for the inven-
wings. Because of their efforts, the old tables were tion of the first flying machine?
repealed in time and replaced by the first reliable fig- (a) People’s support
ures for air pressure on curved surfaces. This work,
(b) Negative reactions of the people
in turn, made it possible for the brothers to design
a machine that would fly. In 1903, the Wrights built (c) Compelling desire of the inventors
their first airplane, which cost less than $1,000. They (d) Previous experiments in aviation
even designed and built their own source of propul- 50. The Wright brothers were ultimately successful in fly-
sion – a lightweight gasoline engine. When they ing their plane when they built their own
started the engine on December 17, the airplane pul- (a) Air pressure system
sated wildly before taking off. The plane managed to (b) Model wings
stay aloft for 12 seconds, however, and it flew 120 (c) Gasoline engine
feet. (d) Curved surfaces
By 1905, the Wrights had perfected the first air-
plane that could turn, circle, and remain airborne for

A01_MADAN 07_65901_FM.indd 20 09/01/23 3:38 PM


Memory-based UGC NET Question Paper 2022 - Set 1 xxi

Answer Keys
1. (b) 2. (a) 3. (d) 4. (d) 5. (c) 6. (b) 7. (d) 8. (b) 9. (b) 10. (b)
11. (d) 12. (b) 13. (c) 14. (a) 15. (b) 16. (a) 17. (a) 18. (b) 19. (c) 20. (d)
21. (d) 22. (a) 23. (a) 24. (c) 25. (d) 26. (b) 27. (a) 28. (d) 29. (a) 30. (c)
31. (a) 32. (a) 33. (d) 34. (b) 35. (d) 36. (b) 37. (a) 38. (b) 39. (d) 40. (b)
41. (b) 42. (a) 43. (a) 44. (c) 45. (b) 46. (c) 47. (d) 48. (b) 49. (c) 50. (a)

A01_MADAN 07_65901_FM.indd 21 09/01/23 3:38 PM


xxii Memory-based UGC NET Question Paper 2022 - Set 1

Hints and Solutions


From the question, it seems that we should try to get basic information first of all by making use of
some direct or shortcut methods. There are multiple options.
State Percentage distribution Absolute number Ratio of Diesel Diesel cars Petrol cars
of cars (Total 1400) of cars and Petrol cars
A. 14% 196 3:4 3/(3 + 4) of 196 = 84 196 – 84 = 112
B. 28% 392 5:9 5/(5 + 9) of 392 = 140 392 – 140 = 252
C. 32% 448 5:3 5/8 × 448 = 280 448 – 280 = 168
D. 26% 364 1:1 364 / (1+1) = 182 182

Basic calculations which apply in all the questions: document business, specifically by Frank and Lillian
14 Gilbreth. They are used in any field such as education,
Cars in State A = 14% of 1400 =
100
× 1400 = 196 business, manufacturing, engineering, architecture,
etc. Few examples have been shown below
Cars in State B = 28% of 1400, that means double of 196
= 2 × 196 = 392 Terminator
Cars in State C = Cars in State B + 4% of 1400 = 392 +
56 = 448
Cars in State D = Cars in State B – 2% of
1400 = 392 – 28 = 364 Process Document
Decision Data
We can take summation of the numbers to see that it
equals 1400
1. (b): The number of diesel engine cars in State
B = 182 Database

The number of petrol engine cars in State D = 140


Difference = 182 – 140 = 42 7. (d): Integral Education–The concept of Integral
Education was given by Sri Aurobindo that is based
2. (a): The number of petrol engine cars in State
on the belief that the education of a human being
C = 168
should begin at birth and continue throughout his
The number of diesel engine cars in State A = 84 life. The main objective of any kind of education is
Difference = 168 – 84 = 84 to ensure the realization of the inner potential of the
84 learner. Thus, this concept is quite dynamic and flex-
Percentage difference = 84 × 100 = 100% ible, it takes into account the whole personality of
the learner, through the concept of dealing with the
3. (d): The number of AC cars in State C = 25% of
‘whole child’. This takes into account the individual
280 = 70
curiosity, passion and compassion, and this applies
The number of non-AC cars = 280 – 70 = 210 to all situations in a beautiful manner. There are
4. (d): integral schools at many places in India as enunci-
The number of total cars in State C = 448 ated by Sri Aurobindo and his spiritual companion
Mirra Alfassa – known as mother.
The number of petrol engine cars in State B = 252
Focus on spiritual aspects of Indian philosophy–
The difference = 448 – 252 = 196
This is associated with Dr. Sarvapalli Radhakrishnan’s
5. (c): metaphysics that is based on Sankara’s Advaita (non-
The number of Petrol engine cars in all states = 112 + dual) Vedanta tradition (sampradaya). He was asso-
252 + 168 + 182 = 714 ciated with Prasthanatraya (main primary texts of
6. (b): A flowchart is a diagram that depicts a process, Vedanta), U ­ panishads, Brahma Sutra, and the Bhaga-
system or computer algorithm. It is used for visual vad Gita. He was greatly influenced by the writings
representation of data flow. Flowcharts use rectangles, and work of Swami Vivekananda.
ovals, diamonds and potentially numerous other shapes Dr. Sarvapalli Radhakrishnan was actively involved
to define the type of step, along with connecting arrows in the works of UNESCO. He was known for being a
to define flow and sequence. Flowcharts were used to bridge between eastern and western philosophies to

A01_MADAN 07_65901_FM.indd 22 09/01/23 3:38 PM


Memory-based UGC NET Question Paper 2022 - Set 1 xxiii

promote world unity and universal fellowship. To Dr of a certain topic. These assessments are done at the
Radhakrishnan – ‘The idea of God is an interpreta- start of the topic to understand how much students
tion of experience’. already know about the subject.

Learning to take place in nature and from nature– Peer Assessment: This assessment provides a struc-
Sri Rabindranath Tagore believed in complete free- tured learning process for students to critique and
dom of every kind for the students in terms of his provide feedback to each other on their work. Thus,
intellectual level, decision making, feelings, knowl- students develop lifelong skills. It equips them with
edge, etc. The independence means normalcy or skills to self-assess and improve their own work.
the fact of being natural. There should be harmony 9. (b): e-PG Pathshala–The UGC has launched an e-PG
between man, God and nature. There is need to prac- Pathshala Project for standardizing the textbook mate-
tice equanimity, harmony and balance. rials for MA, MSc and MCom courses in different uni-
Experiential Learning–In John Dewey’s experien-
versities of India. In India, e-PG Pathshala is such an
tial learning theory, everything occurs within a social innovation maintained by UGC-INFLIBNET and funded
environment. Knowledge is socially constructed and by Ministry of Education. As part of its National Educa-
based on experiences. This knowledge should be tion Mission through ICT (NME-ICT), it is a wide e-learn-
organized in real-life experiences that provide a con- ing platform in terms of its subject material and breadth
text for the information. The names of other great per- of use as well.
sonalities such as David Kolb, Kurt Lewin, and Jean On the other hand, Ministry of Education (earlier

Piaget can be associated with experiential learning. MHRD) launched ‘e-pathshala’ web portal to host
educational resources for students, teachers, par-
8. (b): Every learner (in fact, an individual) is unique. This
ents, researchers and educators. This is a single point
also gives rise to the concept of ‘individual differences’.
repository of e- resources containing NCERT text-
Every learner has her/his own characteristics. For a
books and various other learning resources.
teacher, the individual differences in the form of vari-
ables such as physical characteristics, intelligence, per- 10. The affective domain is one of the three domains in
ception, gender, ability, learning styles are important. A Bloom’s Taxonomy. The affective domain as described by
more effective and productive learning-teaching process Krathwohl, Bloom, Masia in the year 1973, includes the
can be planned by a teacher with this kind of knowledge. manner in which we deal with things emotionally, such
Assessments are a useful way of maintaining qual-
as feelings, values, appreciation, enthusiasms, motiva-
tions, and attitudes. As per question, they have been put
ity. Assessments assess students’ understanding,
in the following five categories (for future questions):
knowledge, and takeaways from the educational
experience. Assessments make learning process more 1. Receiving

effective and enhance student development. Usually, (a) awareness of the action
the assessments make use of empirical data. (b) desire to achieve action
Summative Assessment–This evaluates the overall
(c) controlled attention to action
learning of the students at the end of a unit or course. 2. Response (responding)

This assessment compares against some standardized (a) In response to an agreement
criteria or benchmark. These are basically formal in (b) Desire to respond
nature which help in awarding a certificate also. This (c) Response Contentment
assessment can be written assessment (writing an orig- 3. Evaluation (valuing)

inal piece, such as a narrative or analytical essay), per- (a) Acceptance of value (valuing)
formance assessment (to do an activity or task that will (b) Priority of value
showcase the abilities of the learners), standardized (c) Commitment
assessment (an exam created for a given curriculum) 4. Organizing (managing)

or oral assessment (such as a speech or presentation). (a) Concept of values
Formative Assessment–This assessment measures
(b) Price system organization
how a student is learning during a course of study. 5. Specialization

The goal is to monitor student learning to provide (a) Higher evaluation by peers to do
ongoing feedback that can be used by instructors to (b) Normal group
improve their teaching and by students to improve (c) Specialization
their learning. This word can be linked with actual In option (b), analysing means dividing a topic into parts

formation of career of a student in the class. for the sake of understanding and also synthesising
Diagnostic Assessment–A diagnostic assessment is
means integrating the parts into a consolidated form.
an evaluation of a child’s knowledge and skill level

A01_MADAN 07_65901_FM.indd 23 09/01/23 3:38 PM


xxiv Memory-based UGC NET Question Paper 2022 - Set 1

These acts relate consistently to an internal


belief. They may articulate a philosophy
or worldview. They break down complex
Characterizing
situations and respond on the basis of our
value system. They improve our lives by a The values become more systematic. We can
code of personal behavior. compare and contrast our values and choices.
Organizing The values can be ordered and prioritized.
We choose to commit to certain values.
We are motivated to invest our time
and behave in a certain manner
frequently. We begin to identify Valuing
with a behavior and commit to that.
We participate willingly and try
to become obedient, and find
Responding satisfaction. We get ready to
We want to get ready for participate.
the setting or situation.
We give attention by
choice and be ready Receiving/Attending
for a new experience.

11.(d): A variable is any quantity that a researcher is able 12. (b): Monadic variable is basically used in the sur-
to measure in some way. This could be temperature, vey research that offers individual concepts to sur-
height, age, etc. Basically, a variable is anything that vey respondents in isolation–one at a time. The
contributes to the outcome or result of your experi- independent findings are required for each stimu-
ment in any way. lus. They provide clear detailed feedback. This is
Control Variable–A control variable is anything that available with every automated tool. This can be
is held constant or limited in a research study. It’s a compared with polyadic variables. In comparison
variable that is not of interest to the study’s aims, but testing several stimuli are tested simultaneously.
is controlled because it could influence the outcomes. 13. (c): There are two kinds of variables-Qualitative and
For example, one control variable in the plant growth Quantitative
experiment could be temperature. Assume that this Qualitative Variables–This is also called as a categor-

should be kept uniform at 25° C, so it is the duty of ical variable. As the name indicates, it is not numeri-
researcher to keep it at this uniform level. cal. This describes data that fits into categories. These
External variable–These variables are factors that are all qualitative variables as they have no natural
are not manipulated as part of an experiment, but order.
they may exert some influence on the dependent vari-
The examples are

able under study.
Confounding Variable–This is an unmeasured third Skin colours–White, Yellow, Brown, Black

variable that ‘confounds’ (or influences) the relation- States–Haryana, Punjab, Manipur, Kerala

ship between an independent and a dependent vari- Quantitative variables–They have a value and they

able. It actually suggests the p ­ resence of a spurious can be added, subtracted, divided or multiplied.
correlation. Confounding is a causal concept, and as
such, cannot be described in terms of correlations or Now look at the detailed discussion in context of its

associations. disadvantages.
Controls  oo sensitive to context–The main objective of
A. T
qualitative or any kind of research is to connect
Independent Condition A Dependent with the theme or basic idea. It is basically behav-
Influences
Variable
Condition B
Variable ioural in nature.
Confounding B. Open To Debate–It is also positive reflection as it

is linked with the transparency.
Extraneous
Variables C. Too subjective–Qualitative can always be some-

what subjective but being too much subjective is
Controls its negative characteristic as the personal view-
source-www.simplypsychology.org
point becomes very important.

A01_MADAN 07_65901_FM.indd 24 09/01/23 3:38 PM


Memory-based UGC NET Question Paper 2022 - Set 1 xxv

D.  Difficult to generalise–Though generalisation case studies) may be different and a researcher need
is the main objective of any kind of research but to find the basic idea or description. This is called con-
qualitative lacks in that. tent analysis.
E. Issue of replication–There are always less and less Semantic Differential–A semantic differential can

chances of replication so that the generalisation also be used for the structuring of attitudes, most likely
becomes difficult. Thus, this can be taken as the dis- in qualitative studies. This survey or questionnaire
advantage of qualitative research. rating scale asks people to rate a product, event, or
14. (a): The word ‘sample’ mostly applies in the case of any ‘entity’ within the frames of a multi-point rating
research. Thus, the word project can be taken as research option. These survey answering options are gram-
project. Sample helps us in finding the truth or getting matically on opposite adjectives at each end. As they
us very close to the truth. Sample size calculation and are opposite to each other on the two ends of a con-
determination are imperative to the researcher to deter- tinuum, this may be called as bipolar adjective.
mine the right number of respondents, keeping in mind Social Network Analysis–The dyad definition is the

the research study’s quality. close relationship of two people over a long period
The sample is the miniature version of the population
with many interactions in different settings. The most
that reflects the desired features. That helps us in sav- basic requirement for dyad sociology is that there are
ing time and associated costs. two members of the group. The dyadic communica-
tion definition pertains to how the two group mem-
In qualitative studies, the study is mostly subjective,

bers interact with each other. German sociologist
and the sample size may have to be kept small as we
Georg Simmel studied how group dynamics changed
want to do their detailed analysis.
based on the size of the group.
Thus, the population size matters, so is the confidence

16. (a): The word effective in communication means that
level (90%, 95% or 99% to reflect upon the accuracy
a message has been sent and received properly with
and confidence of results), the margin of error (confi-
an understanding. That understanding is reflected
dence interval), standard deviation from mean (also
by ‘feedback’. In a classroom, an effective listener is
called as sigma that reflects upon the variability of a
completely focused and thus tries to resist any kind of
distribution of population) – all these things matter to
distraction (or noise) that exists in the environment.
a researcher.
Message filtering in communication may actually be
Necessary Sample Size = (Z-score)2 * StdDev*(1-

perceived as a communication, as some fact is kept
StdDev) / (margin of error)2
hidden, distorted or manipulated with an objective to
The precision of obtaining the results from popula-
manage receiver’s reaction. For example, a manager
tion may ask for changing the sample size. Thus, the tries to hide the poor sales figures of his division so
population variability is important. If the popula- that the receiver (boss) does not get angry.
tion is homogeneous, a researcher can manage with
17. (a): This question is about face to face communica-
smaller sample and vice versa. We have to determine
tion but different levels have been given by different
if the increased accuracy is more important than the
authors. In two way communication, the sender of
cost.
message gets the feedback from the receiver, that ful-
There can be probability (random and objective) or
fils the basic objective of communication. The main
non-probability (non random and more subjective) objective of communication is understanding the
sampling techniques. message in the intended form.
The choices D and E are not or least important for a
Our answer is Level 1 as the communication is two

researcher as research should always be objective. way and Face to Face between two persons. There can
be higher channels as per the channels involved.
15. (b): Meta-analysis intends to take a quick review
of multiple research studies on basically the same or The verbal, physical, auditory, emotional and
closely related problem or issue. combines the results energetic are also described as the five levels of
of multiple scientific studies. It is basically a quanti- communication.
tative, formal, epidemiological study design that is 18. (b): Time Sense–This is linked with temporal aspect
based on randomized, controlled trials. linked with culture. Western cultures, even South East
Content Analysis–this is basically used in qualitative Asia tend to measure their time by the clock, keeping
studies where the answers may be subjective which in view that each activity should have a precise begin-
are based on the personal opinion. The language used ning and end that may be termed as linear timing. To
by the respondents in the qualitative research (such as them, the word ‘deadline’ is very important. Western-

A01_MADAN 07_65901_FM.indd 25 09/01/23 3:38 PM


xxvi Memory-based UGC NET Question Paper 2022 - Set 1

ers also see that time has a limited supply, so they the media and audience and products for audi-
structure that accordingly. ence as dynamic and active.
In India, there is a concept of rebirth (punrjanam), we

B. Omnivorous devouring of media content –
assume that if something could not be completed in Here, omnivorous means to consume all kinds
this birth, will be completed in next birth. India has of food and devouring means consuming them.
the concepts of satya yug, dwapar yug, treta yug and Omnivorous devouring means that all types of
kaliyug, according to different time spans and culture media content is being taken in, but media has to
makes a shift accordingly. In most parts of India, Paki- show the contents to the audience or public also,
stan, Bangladesh. etc. a culture is event or personality- so that it is appreciated (or depreciated).
related. The culture also travels if we look at history.
C. Compensation through identification–Here,
Some practices which were part of Greece travelled identification means connectivity with the audi-
to India and vice versa. Thus, the institutions such as ence or public so that some kind of rewards
marraige become institutional. (through advertisements) can be generated.
Level of Gratitude–This is an integral part of cul-
That is basically being done by YouTube, Face-
ture According to Robert Emmons as he expressed book, Instagram, etc. They develop loyalty and
in his book Thanks, ‘Gratitude is literally one of the CRM (Customer Relationship Management).
few things that can measurably change people’s D. Vicarious Wish Fulfilment–Vicarious means a

lives’. Gratitude can improve our health and rela- feeling developed or experienced by watching or
tionships—making it one of the most well-studied reading about somebody else doing something,
and effective ways to increase our well-being in rather than by doing it yourself. An example of
life. This level of gratitude varies according to our vicarious is when a mother who always wanted
culture such as India, China, Russia, Japan, Ger- to be a dancer continually pushes her children to
many, etc. There can be high context, low context, do ballet.
temporal context, emotive context, directive, phatic
contexts of different cultures which have been men- Freud’s most well-known theory, wish fulfilment, is

tioned in Pearson Unit 3. the idea that when wishes can’t or won’t be fulfilled in
They found some similarities across cultures, as well our waking lives, they are carried out in dreams. Even
anxious or punishing dreams have their roots in wish
as some differences—an initial glimpse at how our
fulfilment. Freud’s dream theory is rooted in the idea
early steps toward gratefulness might be shaped by
that we all need a way to express or vicariously fulfil
larger societal forces. There can be Verbal or oral grat-
all of our wishes and desires.
itude, concrete gratitude (offering toffee, chocolate,
etc) and connective gratitude (offer of friendship or Now, we see that people are led by their dreams and

help, etc). fantasies. The social media generates the need for
19. (c): Statement 1–Small group communication refers product which were not felt by public till they see
to interactions among three or more people who are them on social media.
connected through a common purpose, mutual influ- Sources of suggestion for appropriate roles–this point
ence, and a shared identity which indicate the rela- is vey important. Even the children of today want to
tionship needs. The upper number of a small group is empathize the roles of adult members. This role per-
taken between 15 to 20. ception means that they are becoming more assertive
Statement 2–A person who is well informed about and challenging. They have almost complete access to
the group’s task and/or highly motivated as a group media that enables them to challenge the roles being
member may emerge as a leader and set into motion played by elders.
internal decision-making processes, such as recruit- 21. (d): 4 6 12 14 28 30 ….
ing new members or assigning group roles, that affect The numbers are in the order of addition of TWO and
the structure of a group. Different members will also multiplied by TWO
gravitate toward different roles within the group and
will advocate for certain procedures and courses of 4+2=6
action over others. External factors such as group 6 × 2 = 12
size, task, and resources also affect group structure. 12 + 2=14
20. (d): The options seem to be very close ones so that we 14 × 2=28
need to get some idea about all of them. 28 + 2=30
A. Creation of passive longing for media prod- 30 × 2=60
ucts–It has the word ‘passive’, otherwise we see

A01_MADAN 07_65901_FM.indd 26 09/01/23 3:38 PM


Memory-based UGC NET Question Paper 2022 - Set 1 xxvii

22. (a): Given that Universal A All S are P All Birds are
Actual O A M S N E I P C Affirmative Mammals
Word Universal E No S are P No Birds are
Given E C I O P M A N S Negative Mammals
Form Particular I Some S are P Some Birds are
Affirmative Mammals
Accordingly
Particular O Some S are Some Birds are
Given C O M P A N I O N
Word Negative not P Not Mammals

Coded S E I N C P A E P We need to look at traditional square of opposition



and the Truth Table.
Form T F
Thus, (a) is the correct answer. F D
Contrary
23. (a): That means that there are 15 students with below D T A E F F
ranks in the class and Aashman is 16th student from
bottom. Considering his position in the class, there are s
rie
tra
(43 – 16), that is 27 students above him in the class. Sub
Con Sub
Thus, his own position is 28th in the class. alternation ies, alternation
ct or
24. (c): di
ntra
Let the slower pipe alone fill the tank in x minutes. Co
The tank filled by slower pipe in 1 minute = 1/x
T T I O D F
x Sub-Contrary
Then, faster pipe will fill it in 3 minutes. T D
F T
The tank filled by faster pipe in 1 minute = 1/x/3 =

3/x We need to look for separate discussion of contradic-

1 3 1 tories, contraries, sub contraries and sub alternation
∴ x + x = 36
for questions linked with square of opposition.
A E I O
4 1
⇒ x = 36
T F T F
F T F T
⇒ x = 144 minutes
D F T D
25. (d): ATQ, Cost Price of 40 Pens = Marked Price of 36 F D D T
Pens
Now look at the question statement – ‘No Birds

Let the Cost Price of 40 Pens = Marked Price of 36
are Mammals’ is Universal Negative that is E.
Pens = Rs 36
As it is given to be true, come to the second row where
Taking them separately,
A is False–it means that All Birds are Mammals is

Cost Price of 40 Pens = Rs 36
False.
Marked price of 36 pens = Rs 36
I is False–it means that Some Birds are Mammals is

Marked Price of 40 Pens = Rs 40 False
Discount is 1% on Marked Price. O is True–Some Birds are Not Mammals is True

Discount = 1% of Rs 40 = 40 paise
Thus, as per question A, B and D are part of answer.
Selling Price = Rs 40 – 40 Paise = Rs 39.60 Thus, option B is the correct answer.
Thus Profit = Rs 39.60 – Rs 36 = Rs 3.60 27. (a): Ad Populum: This can also be termed as ‘Appeal
Profit Percentage = 3.60 / 36 × 100 = 10% to Popularity’. This argument supports a position by
26. (b): As we know, a categorical statement lines up appealing to the shared opinion of a large group of
two distinct categories or classes against one another, people such as the majority, the general public.
measuring degrees of comparison, and makes an In this statement, a housemaker, XYZ Home Finance
assertion about the categories. This always involves a appeals to be like a family member and thus, to fulfil
subject term and a predicate term: the need to have your own sweet home – most likely

A01_MADAN 07_65901_FM.indd 27 09/01/23 3:38 PM


xxviii Memory-based UGC NET Question Paper 2022 - Set 1

by financing through maximum finance, compara- is obtained after some perception, testimony, etc.
tively less rate of interest, long duration and easy to According to question, we can discuss the following
pay system of Equated Monthly Instalments (EMIs). three aspects.
28. (d): Statement I–‘Some S is P’ belongs to ‘I-category’ 1.  Purvavat–This is basically cause to effect rela-
AND ‘Some S is not P’ belongs to ‘O-category’. That tionship. This can also be termed as unperceived
has been explained in the explanation given under effect from a perceived cause, thus the perceived
question 26. cause comes first and thereafter, the unperceived
By looking at first diagram in question 26, we can say effect (anuman). For example, we can infer the
that I and O are sub-contrary to each other, they are future rain from a present dark clouds.
not contrary to each other. So, Statement I is a false 2. Sesavat–An inference in which the unperceived
statement. cause is known from a perceived effect. For
Statement II–Again, ‘Some S is/are not P’ is a ‘O-cat- example, we come to know about the rain from a
egory’ statement and ‘All S is P’ is a ‘A-category’ muddy flow of water in the river.
statement. As given in square of opposition diagram, 3. Samanyatodrasta Anumana–There are many
both O and A are contradictory to each other. Thus, universal positions in this world such as different
Statement II is a true statement. positions of moon around the earth. From these,
Thus, (d) is the correct answer. we can draw inferences.
29. (a): Statement I–The anumana is basically about 31. (a): MODEM functions as both – input and output
attaining inferential knowledge. That is further linked device. MODEM is a contraction of two words - MOdulator
with invariable concomitance or vyapti. Thus, the anu- / DEModulator which functions simultaneously.
mana is based on syllogistic inference that is basically 1. When sending communications out of the net-
invariable concomitance (vyapti) between middle work, the modem converts (modulates) a digital
term and major term. Concomitance means the fact of signal into an analog signal for transmission over
existing or occurring together with something else. analog media such as cable or telephone line.
Thus, we are basically concerned with the definition 2. When signals are coming into the modem from
the internet, it converts (demodulates) the analog
of Vyapti that is the non-presence of middle term
signal back to a digital signal for transmission on
(hetu) in the locus of the non-existence of the major
the local network.
term (sadhya). Vyapti is an invariable and uncon-
ditional relation between the middle term and the 32. (a):
major term. A. Trojan Horse–This is a type of malware that dis-
For example, the hill is full of fire because it is full guises itself as legitimate code or software. Once
of smoke. Here, smoke which is the middle term is they get inside the network, these attackers are
absent from all regions outside the region of fire (the able to carry out any action that a legitimate user
major term). could perform, such as exporting files, modify-
ing data, deleting files or otherwise altering the
Statement II–Vyapti may also be defined as the co- contents of the device.
presence of the middle term with the major term
when there is no condition attached to the middle B. Virus–A computer virus is a type of computer
term. Vyapti is the possession of that nature which program that, when executed, replicates itself by
establishes the connection of the middle term with modifying other computer programs and insert-
the major term. ing its own code. If this replication succeeds, the
affected areas are then said to be “infected” with a
We can show the relation between minor term computer virus, a metaphor derived from biologi-
(paksh), middle term (hetu) and major term (sad- cal viruses.
hya) with the help of following diagram.
C. Web bug–This is also known as a Web beacon, is
Paksha Hetu Sadhya a file object that is placed on a Web page or in an
Minor Term Middle Term Major Term
e-mail message to monitor user behaviour.
(Mountain) (Reason) vyapya (Fire) vyapaka D. Zombies–This is actually a computer connected
Characteristic- (smoke) Factum factum to the Internet that has been compromised by a
linga probans probandum hacker via a computer virus, computer worm, or
trojan horse program and can be used to perform
30. (c): Anumana is knowledge that is obtained after malicious tasks under the remote direction of the
a proof. Anumana is not a direct knowledge but it hacker.

A01_MADAN 07_65901_FM.indd 28 09/01/23 3:38 PM


Memory-based UGC NET Question Paper 2022 - Set 1 xxix

33. (d): B, C, A is the correct answer. C. Bluetooth – This is a short-range wireless tech-
Random Access Memory (RAM) – This is essentially nology standard that is used for exchanging data
a short term memory where data is stored as the pro- between fixed and mobile devices over short dis-
cessor needs it. tances and building personal area networks.
D. A Denial of Service – This form of attack is
Anti-static Strap – This is a key piece of safety gear
meant to shut down a machine or network,
that helps to prevent the build up of static electricity
making it inaccessible to its intended users.
near sensitive electronics or other projects where static
charge could damage electronics or cause safety issues. 36. (b): Asbestos was nicknamed “the magic mineral”
34. (b): Statement 1–Let’s look at some basics which as it has unique chemical and physical properties.
have been offered officially. The asbestos is used in thousands of products and
industries such as roofing, thermal and electrical
SWAYAM is basically Study Webs of Active-Learn- insulation, cement pipe and sheets, flooring, plas-
ing for Young Aspiring Minds that is part of MOOC tics, textiles, paper, etc. Tiny asbestos fibres collect
Platform. It was developed indigenously by AICTE in the lungs. Asbestos causes mesothelioma, that is
in 2016 to facilitate hosting of online courses which a relatively rare cancer of the thin membranes that
could be accessed by anyone, anywhere at any time line the chest and abdomen. It causes cancers of the
free of cost. The main objective is to achieve access, lung, larynx and ovary. Mesothelioma treatments
equity and quality in context of education. This covers include surgery, radiation and chemotherapy.
subjects from high school onwards till higher educa-
tion including Skill based courses to ensure that every Asbestosis causes permanent lung damage. In Pleu-
student benefits from learning material through ICT. ral effusions, fluid collects around the lungs.
The courses hosted on SWAYAM are in 4 quadrants
Asbestos deposits are found in Andhra Pradesh, Bihar,
– video lectures, specially prepared reading material Jharkhand, Karnataka, Rajasthan and Manipur.
that can be downloaded/printed, self-assessment 37. (a): The occurrence of cyclones is a sea linked phe-
tests through tests and quizzes and an online discus- nomena so there are no chances of its origin from
sion forum for clearing the doubts. The objective is to Northern and Central India. It originates in the seas
make system inclusive by using audio-video, multi- due to low air pressure conditions. The East coast
media and state of the art pedagogy/ technology. receives cyclones not only from those that originated
Statement 2–DigiLocker is basically linked with good
in Andaman Sea but also from the Pacific Ocean
governance. This Indian digitization online service is (typhoons). For transfer of typhoons, there is no big
provided by Ministry of Electronics and Information land mass to stop this. The Bay of Bengal receives
Technology (MeitY) under Digital India initiative. more rainfall in comparison to west coast. The winds
This intends to provide services to every Aadhaar are slower and weaker over the ocean. This part is also
holder for accessing driving license, vehicle registra- fed by rivers Ganga and Brahmaputra.
tion, academic mark sheet in digital format from the The surface sea temperature and humidity also play
original issuers of these certificates. It also provides an important part. The average temperature is around
1GB storage space for the same. 28°C that is intensified by warm air currents.
The structure of DigiLocker includes My Certificates
Odisha state suffers the most from cyclones. The ratio
(digital and uploaded documents), My Profile, My between cyclones from east coasts and west coasts is
Issuer, My Requester and Directories. 4:1.
The tropical cyclones are less in west coast due to dry
35. (d):
air intrusion as dry winds flow from west to east. The
A. Spyware – This malicious software enters our
Arabian Peninsula is already dry. There is a strong
computer device, gathers data, and forwards it to
wind shear which hampers cyclone development.
a third-party without our consent. This legitimate
software monitors our data for commercial pur- The naming of cyclones is done in a schematic manner
poses like advertising. by World Meteorological Organisation (WMO) that
B. Firewall – A Firewall in a computer system is basi- was set up in the year 1950, the roots of which were
cally a network security system which controls planted at the 1873 Vienna International Meteorolog-
the flow of data between two or more networks, ical Congress. WMO maintains rotating lists of names
and manages the links between the networks. A which are appropriate for each Tropical Cyclone
firewall can consist of both hardware and soft- basin. If a cyclone is particularly deadly or costly, then
ware elements. The examples are router (physical its name is retired and replaced by another one.
firewall), anti virus, Java (software firewall).

A01_MADAN 07_65901_FM.indd 29 09/01/23 3:38 PM


xxx Memory-based UGC NET Question Paper 2022 - Set 1

38. (b): of Threatened Species of the International Union for


A. Greater biodiversity in ecosystems, species, and Conservation of Nature (IUCN) is an important ref-
individuals leads to greater stability. For exam- erence for the threat of species, but it is still heavily
ple, species with high genetic diversity and many biased towards vertebrates, with invertebrates being
populations that are adapted to a wide variety particularly underrepresented.
of conditions are more likely to be able to with- 39. (d): Landfills–Landfill gases produced during bio-
stand weather disturbances, disease, and climate logical degradation of buried organic wastes include
change. In other words, the two key compo- methane, which when released to the atmosphere,
nents of ecosystem stability are resilience and can contribute to global climate change. This can be
resistance. Resistance is an ecosystem’s ability taken as a part of manure management.
to remain stable when confronted with a distur- Methane is the second-most abundant greenhouse
bance. Resilience is the speed at which an ecosys- gas in the atmosphere, after carbon dioxide.
tem recovers from a disturbance.
According to the IPCC, more than 40 per cent of meth-
B. Extinction is the death of all members of a species ane emissions come from farms or are an outcome of
of plants, animals, or other organisms. The three peatland destruction. A quarter of the global emissions
types of extinction are mass extinction, back- are products of agriculture and land-use changes. The
ground extinction, and human led extinction. pledge by more than 80 nations, helped by the US and
Earth could lose 16% of its species if the average EU, to cut emissions of this GHG (greenhouse gas) by
global temperature rise exceeds 4.3 °C. 30 per cent is, therefore, an important step in global
warming mitigation efforts. Methane is 80 times
Human activities in the form of anthropogenic more powerful than carbon dioxide in its first 20
activities are basically human interference. The years in the atmosphere. While much of the conten-
main modern causes of extinction are the urban- tious aspects of curbing emissions of this GHG pertain
isation, industrialisation, loss and degradation to agriculture, the oil and gas industry — especially
of habitat (mainly deforestation), over exploita- natural gas, whose popularity as a relatively cleaner
tion (hunting, overfishing), invasive species, cli- fossil fuel has led to a 50 per cent increase in its use in
mate change, and nitrogen pollution. Our global the past 10 years — is the second-highest emitter of
food system is the primary driver of biodiversity this gas.
loss, with agriculture alone being the identified
threat to 24,000 of the 28,000 (86%) species at THE EMISSIONS SHARE
risk of extinction.
Manufacturing Industrial
Industries and Transport processes Enteric
C. Habitat fragmentation actually leads to with- Construction (13%) and product fermentation
drawal of support systems to many other species (18%) use (8%) (54.6%)
which put their survival at stake. Agricultural
soils (19%)
D.
Waste (3%) Rice
Vertebrates Invertebrates cultivation
69, 276 species 1,305,250 Agriculture (17.5%)
species (14%) Manure
management
Electricity (6.9%)
production
(44%) Crop residue
burning
Flowering (2.1%)
Plants
Fungi & Global Methane Pledge–During Glasgow COP meet-

268, 000
Protists
species
52, 280 species
ing, US and EU floated the idea of Global Methane
Pledge so as to contribute towards global tempera-
ture rise limit to 1.5°C. Participants joining the Pledge
agree to take voluntary actions to contribute to a col-
lective effort to reduce global methane emissions
at least 30 percent from 2020 levels by 2030, which
could eliminate over 0.2˚C warming by 2050. This is
Invertebrates are central to the functioning of ecosys-
a global pledge, and not a national reduction target.
tems, and are important part of biodiversity, yet they Russia, India and China have not signed the Global
are underappreciated and understudied. The Red List Methane Pledge.

A01_MADAN 07_65901_FM.indd 30 09/01/23 3:38 PM


Memory-based UGC NET Question Paper 2022 - Set 1 xxxi

40. (b): studying Conference of Parties 26th Meet at Glasgow


A. Emission Trading – Kyoto Protocol – This proto- in 2021.
col was adopted in Kyoto, Japan, on 11 December 41. (b): On this day in 1835, Thomas Babington Macaulay
1997 and entered into force on 16 February successfully westernised education in India; English
2005. The main objective of Kyoto Protocol was was made the official language for the government
to reduce the emission of gases. This was put into and courts, and was adopted as the official medium
practice by committing industrialized countries of instruction. This point is specifically important in
and economies in transition to limit and reduce Macaulay Minutes - ‘We must at present do our best
greenhouse gases (GHG) emissions. The follow- to form a class of persons, Indian in blood and col-
ing were the key objectives of Kyoto Protocol. our, but English in taste, in opinions, in morals and in
1. International Emissions Trading intellect’.
2. Clean Development Mechanism (CDM) 42. (a): Nalanda University is the correct answer. Nal-
3. Joint implementation (JI) anda is an ancient center of higher learning in Bihar
There were annexed nations for the purpose of
from 427 to 1197. Nalanda was established in the 5th
reduction of gases. The key concept was ‘com- century AD in Bihar, India.
mon but differentiated responsibilities’ (CBDR). A new Nalanda University has been recently devel-

The main GHGs were Carbon dioxide (CO2), oped in Bihar in 2010.
Methane (CH4), Nitrous oxide (N2O), Hydro- Xuan Zang (or call him Hiuen Tsang) visited Nalanda
fluorocarbons (HFCs), Perfluorocarbons (PFCs); University in 7th century. He stayed and travelled in
and Sulphur hexafluoride (SF6). India for sixteen years. He translated Buddhist scrip-
B. Sustainable Management of Resources – Rio tures. He also studied logic, Sanskrit grammar, Yogac-
Summit (1992) - Agenda 21 of Rio Summit ara in India. He opposed Samkhya and Vaiseshika
(1992) talked about it. The ‘Earth Summit’ con- that he found to be opposing Budhhist philosophy.
cluded that the concept of sustainable develop- According to I-Quing, there were followers of both
ment was an attainable goal for all the people of “vehicles” - Hinayana and Mahayana. Northern India
the world, regardless of whether they were at the and most of the islands of the South Seas (such as
local, national, regional or international level. It Sumatra, Java – parts of present Indonesia) practiced
also recognized that integrating and balancing – –
Hi nayana. The Buddhists in China and Malayu prac-
economic, social and environmental concerns in – –
ticed Maha yana.
meeting our needs is vital for sustaining human 1. For future questions, note that Fa-Hien was the
life on the planet and that such an integrated first Chinese traveller to visit India in the early
approach is possible. fifth century. The purpose of his visit was to look
C. Clean Affordable Energy – International Solar for texts sacred to Buddhism.
Alliance (ISA) – ISA is a treaty based inter-gov- 2. Taxila university was one of the oldest universi-
ernmental organization working to create a global ties of the world.
market system to tap the benefits of solar power
3. Nalanda University is a public central/union uni-
and promote clean energy applications. This was
versity located in Rajgir in Nalanda district in the
primarily promoted by India and France. It was
state of Bihar. It is designated as an Institute of
started in 30 November 2015.
National Importance (INI) and excellence. This
D. Intended Nationally Determined Contributions international university is supported by 18 mem-
– Paris Agreement – This agreement was adopted ber countries was established by an Act of the
in 2015. They are essence of the Paris Agreement Indian Parliament in 2010. The President of India
so that they can facilitate the achievement of serves as the Visitor to Nalanda University.
long-term objectives of emission reduction, bet-
ter energy measures, forest resources, etc. INDCs 43. (a): National Knowledge Commission (NKC) was con-
embody efforts by each country to reduce national structed in 2006 as a high-level advisory body to the
emissions and adapt to the impacts of climate Prime Minister of India, with the objective of trans-
change. forming India into a knowledge society. It was brain
child of Sam Pitroda. NKC submitted around 300
The long term objective is to hold the increase in
recommendations on 27 focus areas. NKC focused
global average temperature to well below 2°C, to on five key areas of the knowledge paradigm – access
pursue efforts to limit the increase to 1.5°C, and to to knowledge, knowledge concepts, knowledge crea-
achieve net zero emissions in the second half of this tion, knowledge application and development of bet-
century. We can get the basic idea of revised goals by

A01_MADAN 07_65901_FM.indd 31 09/01/23 3:38 PM


xxxii Memory-based UGC NET Question Paper 2022 - Set 1

ter knowledge services. 9. To categorize emergent technologies based on


NKC has recommended setting up of a National Sci- their potential and estimated frame for disrup-
ence and Social Science Foundation (NSSF) for pro- tion, and periodically present this analysis to
ducing research work that can fetch a Nobel Prize for Ministry of Education.
an Indian scientist. In July, 2021, the Prime Minister stated that
National Digital Education Architecture (NDEAR)
44. (c): National Education Policy, 2020 envisages the
and National Education Technology Forum
setting up of an autonomous body – National Edu-
(NETF) would be working to make digital and
cational Technology Forum (NETF) to provide a
economic framework across the country.
platform for free exchange of ideas on the use of tech-
nology to enhance learning, assessment, planning, Let us look at the few other initiatives started by
administration and so on, both for School and Higher Government of India in education sector.
Education. Academic Bank of Credit: This aims to pro-
vide multiple entry and exit options for stu-
Aim of NETF: The aim of the NETF will be to facili-
dents in Higher education and also the first year
tate decision making on the induction, deployment
Engineering Programmes in Regional Languages
and use of technology by providing to the leadership
and Guidelines for Internationalization of Higher
of education institutions, State and Central Govern-
Education.
ments and other stakeholders the latest knowledge
Vidya Pravesh: The initiative is a three-month
and research as well as the opportunity to consult and
play-based school preparation module for Grade
share best practices. The NETF will have following
functions: 1 students.
Indian Sign Language: The Indian Sign
1. To provide independent evidence-based advice to Language has been launched as a subject at the
Central and State Government agencies on tech- secondary level.
nology-based interventions. NISHTHA 2.0: This is an integrated programme
2. To build intellectual and institutional capacities of teacher training designed by NCERT.
in education technology. SAFAL (Structured Assessment For Analyzing
3. To envision strategic thrust areas in this domain. Learning Levels): SAFAL is a competency-based
4. To articulate new directions for research and assessment framework, which has been launched
innovation. for Grades 3, 5 and 8 in CBSE schools.
5. To lay down standards of content, technology, Artificial Intelligence Website: A website dedi-
and pedagogy for online/digital teaching-learn- cated to Artificial Intelligence has also been
ing. These standards will help to formulate guide- launched.
lines for e-learning by States, Boards, Schools, 45. (b): Curriculum revision means to modify the existing
HEIs, etc. curriculum using data from the review. Review and
6. To maintain regular flow of authentic data from revision are important because they enable teachers
multiple sources including educational technol- to consider the ways curriculum interacts with actual
ogy innovators and will engage with diverse set students in a real teaching learning environment. The
of researchers to analyse the data. vision and mission also undergo a change. The social,
7. To conduct multiple regional and national con- political, economic and above all the philosophical
ferences, workshops, etc. to solicit inputs from environment are always at a change so as to make
national and international educational technol- teaching learning process more effective. Thus, there
ogy researchers, entrepreneurs, and practitioners. are different stakeholders. The demands of industry
8. To identify technological interventions for the also change, so that the curriculum produces more
purpose of improving teaching-learning and eval- outcome-based citizens.
uation process, supporting teacher preparation 46. (c)
and professional development, enhancing educa-
47. (d)
tional access, and streamlining educational plan-
ning, management, and administration including 48. (b)
process related to admissions, attendance, assess- 49. (c)
ments, etc. 50. (a)

A01_MADAN 07_65901_FM.indd 32 09/01/23 3:38 PM


Memory-based UGC NET
Question Paper 2022 - Set 2 *
INSTRUCTIONS
Time Allowed: 1 hour  50 × 2 = 100 Marks

1. This paper consists of Fifty (50) objective type questions of Two (2) marks each.
2. All questions are compulsory.
3. Each item has four alternative responses marked (a), (b), (c) and (d). You have to darken the circle as indicated below on the
correct response against each item.
Example: a b c d where (c) is the correct response.
4. Your responses to the items are to be indicated in the OMR Sheet given at the end of the book.
5. Read instructions provided with each question carefully.
6. There are no negative marks for incorrect answers.

Directions for questions 1 to 5: The following table 3. The number of overweight men in the year 2021 was
presents data about the number of men, women and chil- what percent of the number of men who were not
dren and percentage (%) of overweight men, overweight overweight in the same year?
women and overweight children in a city during the last (a) 55% (b) 60%
six years from 2016 to 2021.
(c) 50.5% (d) 65.5%
Year-wise Distribution of Population in a city
4. What was the difference between the number of over-
Number of Percentage (%) of weight women and overweight children together in
Year Overweight the year 2018 and the number of overweight men in
Men Women Children Men Women Children the same year?
(a) 150 (b) 450
2016 27000 19000 7500 15% 36% 30%
(c) 345 (d) 865
2017 37500 32000 10500 7% 35% 28%
5. What was the ratio of the number of overweight
2018 31500 30000 6000 30% 25% 35%
women in the year 2018 to the number of overweight
2019 33000 27000 8000 16% 30% 30% men in the year 2020?
2020 35000 34000 10000 12% 27% 32% (a) 6 : 7 (b) 21 : 65
2021 39000 37500 22500 37.50% 22% 36% (c) 15 : 73 (d) 25 : 14
6. Which of the following are the principal components
1. What was the total number of children who were not of the CPU of a computer system?
overweight in the year 2016 and 2017 together? A. ALU (Arithmetic-Logic Unit)
(a) 4530 (b) 4350 B. CU (Control Unit)
(c) 12810 (d) 14820 C. Processor Registers
2. What was the average of the number of men, women D. SSD (Solid-State Drive)
and children who were overweight in the year E. VRAM (Video RAM)
2019? Choose the correct answer from the options given below:
(a) 4812 (b) 5016 (a) A, B and D only (b) A, B and C only
(c) 5395 (d) 5260 (c) C, D and E only (d) B, C and D only
*The 2022 Question paper set contains memory-based questions, lawfully curated solely by the author relying upon the memory of past attending
candidates of the 2022 NTA-UGC examination. The copyright in the original question papers is owned and vested with the concerned owners and
Pearson does not claim any ownership in such copyright. The answers to these questions have also been solely developed by the author herein, and
Pearson shall not be liable for any claims arising out of usage/accuracy of the content herein”.

A01_MADAN 07_65901_FM.indd 33 09/01/23 3:38 PM


xxxiv Memory-based UGC NET Question Paper 2022 - Set 2

7. Which among the following pesticides would have the 14. Given below are two statements.
least chance of bioaccumulation in food chains? Statement I: The new system of communication has
(a) Chlordane (b) Aldrin facilitated the emergence of an interactive society.
(c) Carbaryl (d) Endrin Statement II: The new system of communication is
8. In a certain code, ‘467’ means ‘leaves are green’, ‘485’ digital and has integrated multiple modes of commu-
means ‘green is good’ and ‘639’ means ‘they are play- nication through networks.
ing’, Which digit stands for ‘leaves’ in that code? In light of the above statements, choose the
(a) 6 (b) 7 correct answer from the options given below
(c) 4 (d) 9 (a) Both Statement I and Statement II are true
(b) Both Statement I and Statement II are false
9. According to the revised Bloom’s taxonomy, which of
(c) Statement I is true but Statement II is false
the following categories is considered to be highest
for the learner? (d) Statement I is false but Statement II is true
(a) Understand (b) Apply 15. Which of the following have been considered as threat
(c) Analyse (d) Create to internal validity in research?
A. Randomisation B. Generalisability
10. Consider the following spreadsheet:
C. Maturation D. History
A B C E. Instrumentation
1 4 3
Choose the correct answer from the options given

2 5 2 below:
3 7 (a) A, B and C only (b) B, C and D only
4 8 9 (c) C, D and E only (d) A, B and E only
5 9 16. Some of the most notable universities that evolved
6 during ancient period in India were situated at:
A. Takshashila B. Vikramshila
If the formula = $A$3 + B2 in tell C4 is- copied to cell
C. Jagaddala D. Odantapuri
C5, then what is the value in cell C5? E. Nalanda
(a) 9 (b) 8 Choose the correct answer from’ the options given below:

(c) 7 (d) 17 (a) A, B, C, D and E (b) A, B and E only
11. Dynamic approach to teaching means (c) A, B, C and E only (d) A, B, D and E only
(a) Teaching should be lecture based. 17. Which of the following is not an essential component
(b) Teachers should be energetic. required for video-conferencing system?
(c) Teachers should be student friendly. (a) Video Camera (b) Display Device
(d) Students should learn through activities. (c) Telephone (d) Microphone
12. Suggestions for communication with people from dif- 18. The average age of students, in a class was 15 years,
ferent cultures can be: When 5 new boys whose average age was 12 years 6
A. Emphasise your interpretation months were admitted in the class, the average age
B. Forget your own cultural identity was reduced by 6 months. How many students were
C. Promote an eco-system of mutual respect there in the class originally?
D. Know the cultural context of other people (a) 15 (b) 20
E. Always state facts (c) 18 (d) 16
Choose the correct answer from the options given below:
19. Given below are two statements, one is labelled as
(a) A, B and C only (b) C, D and E only Assertion A and the other is labelled as Reason R
(c) B, C and E only (d) A, D and E only Assertion A: MOOCs on the SWAYAM portal can be
13. A wise sage has said, ‘The hunger for knowledge is used for earning upto 40% credits per semester by
more difficult to remove than the hunger for food, students.
Therefore, one should stick to the food,’ Which fallacy Reason R: All MOOCs on the SWAYAM portal are
is committed in this argument? credit-based MOOCs.
(a) Begging the Question In light of the above statements, choose the correct
(b) False Dilemma answer from the options given below
(c) Ignoratio elenchi (irrelevant conclusion) (a) Both A and R are true and R is the correct expla-
(d) Appeal to Emotion nation of A

A01_MADAN 07_65901_FM.indd 34 09/01/23 3:38 PM


Memory-based UGC NET Question Paper 2022 - Set 2 xxxv

(b) Both A and R are true but R is NOT the correct (c) Physical noise
explanation of A (d) Symbolic representaion
(c) A is true but R is false. 27. Which of the following is NOT characteristic of a good
(d) A is false but R is true question paper?
20. Given below are two statements. (a) Objectivity (b) Subjectivity
Statement I: An ethical framework of sustainable (c) Reliability (d) Validation
development provides normative guidance regarding 28. Given below are two statements, one is labelled as
our relationship with environment. Assertion A and the other is labelled as Reason R
Statement II: Anthropocentric model is misleading Assertion A: According to classical Indian Logicians
as evident from its environmental consequences. (Naiyayikas), all fallacies are material fallacies.
In light of the above statements, choose the correct Reason R: According to Nyaya, Syllogism is deduc-
answer from the options given below: tive-inductive and formal-material.
(a) Both Statement I and Statement II are true In light of the above statements, choose the
(b) Both Statement I and Statement II are false correct answer from the options given below
(c) Statement I is true but Statement II is false (a) Both A and R are true and R is the correct expla-
(d) Statement I is false but Statement II is true nation of A
21. Number of bacteria in a container doubles in every one (b) Both A and R are true but R is NOT the correct
minute, If the container gets completely filled with explanation of A
bacteria in 30 minutes, then in how many minutes (c) A is true but R is false
1/4th of the container shall be filled with bacteria? (d) A is false but R is true
(a) 15 minutes (b) 16 minutes 29. If the statement ‘Some plants are not carnivorous’ is
(c) 28 minutes (d) 25 minutes given as false, then which of the following could be
22. SWAYAM tries to take the best teaching-learning immediately inferred from it?
resources to all students through A. ‘Some plants are carnivorous’ is true
(a) Seminars (b) Video lectures B. ‘Some plants are carnivorous’ is false
(c) Classroom teaching (d) Conferences C. ‘No plants are carnivorous’ is false
23. ‘We should value the Moon more than the Sun because D. ‘All plants are carnivorous’ is true
the Moon shines when it is dark whereas Sun shines Choose the correct answer from the options given

when there is light anyway,’ This statement commits below:
the fallacy of (a) B, C and D only (b) A and D only
(a) Appeal to Emotion (b) Fallacy of False Cause (c) A and C only (d) A C and D only
(c) False Dilemma (d) Equivocation 30. Given below are two statements
24. The term ‘Hermeneutics’ is drawn from Statement I: The qualitative data are powerful
(a) Sociology (b) Theology because they are collected from very sensitive social,
(c) Anthropology (d) Economics historical and temporal context
Statement II: Context sensitivity cannot be com-
25. Which among the following hazards can occur due to
pletely removed from the qualitative data.
anthropogenic causes?
A. Cyclones In light of the above statements, choose the correct
answer from the options given below
B. Nuclear Disaster
(a) Both Statement I and Statement II are true
C. Floods
(b) Both Statement I and Statement II are false
D. Volcanoes
(c) Statement I is true but Statement II is false
E. Forest Fires
(d) Statement I is false but Statement II is true
Choose the correct answer from the options given

31. Given below are two statements, one is labelled as
below:
Assertion A and the other is labelled as Reason R
(a) A, B and D only (b) B, C and D only
Assertion A: Most cyclones cause widespread dam-
(c) B, D and E only (d) B, C and E only
age in coastal area.
26. In face-to-face communication, the non-verbal cues Reason R: Energy of most cyclones increases after
provided by the receiver, can be considered as landfall.
(a) Feedback In light of the above statements, choose the correct
(b) Decoding answer from the options given below:

A01_MADAN 07_65901_FM.indd 35 09/01/23 3:38 PM


xxxvi Memory-based UGC NET Question Paper 2022 - Set 2

(a) Both A and R are correct and R is the correct


(a) A – I, B – II, C – III, D – IV
explanation of A (b) A – II, B – III, C – IV, D – I
(b) Both A and R are correct but R is NOT the correct (c) A – III, B – IV, C – I, D – II
explanation of A (d) A – IV, B – I, C – II, D – III
(c) A is correct but R is not correct 37. Which of the following is NOT correct regarding
(d) A is not correct but R is correct Digilocker, a key initiative under ‘Digital India pro-
32. In pursuance of the National Council of Teacher Edu- gramme of Government of India?
cation Act,1993, NCTE was established in the year: (a) Indian citizens who sign up for a Digilocker
(a) 1995 (b) 1993 account get a dedicated cloud storage space that
(c) 1994 (d) 1996 is limited to their Aadhar number.
(b) It aims to eliminate the use of physical documents
33. Arrange the following water bodies in increasing and provide access to their digital documents
order of the quantity of freshwater they hold: anytime, anywhere and share it online.
A. Ice and snow (c) Citizens can self-upload documents and digitally
B. Freshwater lakes and reservoirs sign them using the e-sign facility.
C. Groundwater (d) The slogan of Digilocker is ‘My documents,
D. Rivers and streams Anytime, Anywhere.’
Choose the correct answer from the options-given
38. Find the missing number in the series:
below 2, 10, 30, 68, 130, 222, ?
(a) D, B, C, A (b) B, C, D, A (a) 350 (b) 352
(c) B, D, A, C (d) D, B, A, C (c) 194 (d) 104
34. Historical research has the limitation of 39. Communication compatibility between individuals is
(a) Problem identification rooted in
(b) Analytical synthesis (a) Stewardship (b) Interest in media
(c) Universal generalisation (c) Empathy (d) External influence
(d) Probable conclusions 40. Following are the characteristics of Non-Conventional
learning:
35. Which of the following is envisaged in the Sustainable A. It is teacher oriented.
Development Goals? B. It is for improvement of quality.
(a) By 2020, conserve at least 50% of coastal and C. It is cost effective.
marine areas. D. It is linked to employment.
(b) By 2030, reduce by one-third of the per capita E. It is on campus.
global food waste at the retail and consumer Choose the correct answer from the options given
levels. below:
(c) By 2030, double the global rate of improvement (a) A, B and C only (b) B, C and D only
in energy efficiency. (c) A, C and D only (d) C, D and E only
(d) By 2030, reduce by one-fifth the premature mor- 41. Certain number of birds are sitting on two trees A and
tality from non-communicable diseases. B. Birds sitting on tree A, tell to the birds sitting on
tree B, that if 2 birds from tree B shift to the tree A
36. Match List I with List II
then we are equal in number, However, the birds sit-
List I List II ting on tree B, tell those sitting on A that if two birds
Research Perspective Discipline from your side (tree A) shift to our side (tree B), then
A. Phenomenology I. Anthropology
we become double than you (tree A). How many birds
are sitting on the two trees A and B, respectively?
B. Ethnography II. Sociology (a) 8,1 2 (b) 10,14
C. Ethnomethodology III. Social psychology (c) 14,10 (d) 5, 7
D. Symbolic interactionism IV. Philosophy 42. Which of the following are the advantages of grape-
vine communication?
Choose the correct answer from the options given

A. Smooth float of adverse comments
below:
b. Knowing the morale in the organisation

A01_MADAN 07_65901_FM.indd 36 09/01/23 3:38 PM


Memory-based UGC NET Question Paper 2022 - Set 2 xxxvii

C. Spead of propagandist views Together with her husband, Pierre, she discovered
D. To know the important issues faced by the radium, an element widely used for treating can-
employees cer, and studied uranium and other radioactive
E. Assessment of employee anxities substances. Pierre and Marie’s amicable collabora-
Choose the correct answer from the options given tion later helped to unlock the secrets of the atom.
below: Together with her husband Pierre, she was awarded
(a) A, B and C only (b) B, C and D only the Noble Prize for Physics in 1903.
(c) D, C and E only (d) B, D and E only Marie was born in Warsaw, Poland on 7 November

43. Which of the following statements are true regarding 1867, the daughter of a Physics teacher. At an early
Nyaya (Classical Indian School of Logic) view of hetra – age, she displayed a brilliant mind and a blithe per-

bhasa (fallacy) sonality. Her great exuberance for learning prompted
A. It means that middle term appears to be a her to continue with her studies after high school. She
reason but is not a valid reason. became disgruntled, however, when she learned that
B. All fallacies are material fallacies. the university in Warsaw was closed to women. Deter-
C. When an inference is based not on causation but mined to receive a higher education, she defiantly left
on uniformity of co-existence, it leads to a fallacy. Poland and in 1891 entered the Sorbonne, a French
D. Fallacies occur when any of the five characteris- University, where she earned her master’s degree and
tics of a middlle term are violated. doctorate in Physics.
Choose the correct answer from the options given Marie was fortunate to have studied at the Sorbonne

below: with some of the greatest scientists of her day, one of
(a) A, B and C only (b) A, B and D only whom was Pierre Curie. Marie and Pierre were mar-
(c) B, C and D only (d) A, B, C and D ried in 1895 and spent many productive years work-
ing together in the physics laboratory. A short time
44. Many computer users are worried about malware.
after they discovered radium, Pierre was killed by a
Which of the following are two examples of malware?
horse-drawn wagon in 1906. Marie was stunned by
A. Firewall B. Worm
the horrible misfortune and endured heartbreaking
C. Router D. Virus
anguish. Despondently she recalled their close rela-
Choose the correct answer from the options given
tionship and the joy that they had shared in scientific
below:
research. The fact that that she had two young daugh-
(a) A and C only (b) A and B only
ters to raise by herself greatly increased her distress.
(c) B and D only (d) C and D only
Curie’s feeling of desolation finally began to fade when
45. Match List I with List II
she was asked to succeed her husband as a Physics
List I List II professor at the Sorbonne. She was the first woman to
Concepts Main in Proponent be given a professorship at the world-famous univer-
A. Self-education through I. Sri Aurobindo sity. She received her second Noble Prize in Chemistry
development of individuality for isolating radium, in 1911. Curie’s eldest daughter,
B. Integral education II. Mahatma Gandhi Irene, was herself a scientist and winner of the Noble
C. Education to transform III. Maria Prize for Chemistry.
human mind Montessori
Although Marie Curie eventually suffered a fatal ill-
D. Basic education (Wardha IV. J. Krishnamurti
Education System)
ness from her long term exposure to radium, she
never became disillusioned about her work. Regard-
Choose the correct answer from the options given
less of the consequences, she had dedicated herself to
below: science and to revealing the mysteries of the physical
(a) A – I, B – III, C – IV, D – II world.
(b) A – III, B – I, C – II, D – IV 46. At what age did Marie Curie receive her first Nobel
(c) A – III, B – I, C – IV, D – II prize?
(d) A – I, B – IV, C – II, D – III (a) 36 years (a) 46 years
(b) 56 years (c) 44 years

Read the given passage and answer the questions
that follow: 47. After how many years of her marriage did Marie Curie
lose her husband?
Marie Curie was a Polish-born physicist and chem-

(a) 11 years (b) 39 years
ist and one of the most famous scientists of her time.
(c) 28 years (d) 6 years

A01_MADAN 07_65901_FM.indd 37 09/01/23 3:38 PM


xxxviii Memory-based UGC NET Question Paper 2022 - Set 2

48. The passage shows Marie Curie to be a great scientist 50. What made Marie Curie study at Sorbonne University?
who (a) University in Warsaw did not allow her admission.
(a) was amiable in nature. (b) She wanted to work with Pierre Curie.
(b) had indomitable spirit and dedication to science. (c) She wanted to work on Radium.
(c) was emotionally vulnerable. (d) She did not enjoy learning at an early age in
(d) became disillusioned after her husband’s death. Warsaw.
49. Which of the following elements is widely used in
treatment of Cancer?
(a) Uranium (b) Radium
(c) Thorium (d) Plutonium

Answer Keys
1. (c) 2. (d) 3. (b) 4. (a) 5. (d) 6. (b) 7. (c) 8. (b) 9. (d) 10. (c)
11. (d) 12. (b) 13. (c) 14. (a) 15. (c) 16. (a) 17. (c) 18. (b) 19. (c) 20. (a)
21. (c) 22. (b) 23. (b) 24. (b) 25. (d) 26. (a) 27. (b) 28. (a) 29. (d) 30. (a)
31. (c) 32. (a) 33. (a) 34. (c) 35. (c) 36. (d) 37. (d) 38. (a) 39. (c) 40. (b)
41. (b) 42. (d) 43. (b) 44. (c) 45. (c) 46. (a) 47. (a) 48. (b) 49. (b) 50. (a)

A01_MADAN 07_65901_FM.indd 38 09/01/23 3:38 PM


Memory-based UGC NET Question Paper 2022 - Set 2 xxxix

Hints and Solutions


1. (c): The percentage of children who were not over- 6. (b): Central Processing Unit (CPU): This part of a
weight in 2016 = 100 – 30 = 70% computer is comprised with digital circuits called
Their number = 70/100 × 7500 = 5250 arithmetic logic units (ALU) that are capable of per-
forming billions of arithmetic and logic operations
The percentage of children who were not overweight
every second.
in 2017 = 100 – 28 = 72%
ALU is a digital circuit used to perform arithmetic and
Their number = 72/100 × 10500 = 7560
logic operations. This makes the fundamental build-
The total number = 5250 + 7560 = 12810 ing block of the CPU of a system. The modern CPUs
Thus, (c) is the correct answer. contain powerful and complex ALUs.
2. (d): For the year 2019 Control Unit (CU) - Modern CPUs contain a CU, in
Number of overweight men = 16% of 33000 = addition to ALUs. CU tells the ALU what operation is
16/100 × 33000 = 5280 to be performed on the data. ALU stores the result in
an output register. CU moves the data between these
Number of overweight women = 30% of 27000 =
registers, the ALU, and memory.
30/100 × 27000 = 8100
Process Registers – ALUs load data from input regis-
Number of overweight children = 30% of 8000 =
ters. A register is a small amount of storage available
30/100 × 8000 = 2400
as part of a CPU.
The total of above three = 5280 + 8100 + 2400 =
The information in a computer is stored and manipu-
15780
lated in the form of binary numbers, i.e. 0 and 1 which
Their average = 15780 / 3 = 5260 are managed by Transistor switches which are open or
Thus (d) is the right answer. closed.
3. (b): If we assume that total number of men in 2021
CPU
was 100
Control
Then the number of overweight men = 37.5 Unit Instructions
The number of not overweight men = 100 – 37.5 =
Processor
62.5
The required percentage = 37.5 / 62.5 × 100 = 3 / 5 Registers
× 100 = 60%
Thus, (b) is the correct answer Input
Combinational
Output
Logic
4. (a): For the year 2018
The number of overweight women = 25% of 30000 =
25/100 × 30000 = 7500 Main
Memory
The number of overweight children = 35% of 6000 =
35 /100 × 6000 = 2100 Block diagram of a basic computer with uniprocessor
The total of overweight men and women = 7500 + CPU. Black lines indicate data flow, whereas red lines
2100 = 9600 indicate control flow. Arrows indicate the direction of
The number of overweight men = 30% of 31500 = flow.
30/100 × 31500 = 9450 7. (c): Bioaccumulation describes the accumulation and
The required difference = 9600 – 9450 = 150 enrichment of contaminants in organisms, relative to
that in the environment. Bioaccumulation is the net
5. (d): The required ratio between overweight women result of all uptake and loss processes. Uptakes are
in 2018 and number of overweight men in 2020 respiratory and dietary. Loss happens by egestion,
= 25% of 30000 : 12% of 35000
passive diffusion, metabolism, transfer to offspring
= 750 : 420
and growth.
= 25 : 14
8. (b): ‘Leaves are Green’ and ‘Green is Good’ are coded
Thus, (d) is the correct answer.
by 467 and 485, respectively. The word Green is there
in both sentences and so is 4. Thus, ‘Green’ is indi-

A01_MADAN 07_65901_FM.indd 39 09/01/23 3:38 PM


xl Memory-based UGC NET Question Paper 2022 - Set 2

cated by 4 ………..(1) 1. C3 will get the formula ‘= A3 + B3’


Similarly, in ‘Leaves are Green’ and ‘They are play-
2. D2 will get the formula ‘= B2 + C2’
ing’, coded by 467 and 639 respectively, the word
Example 2 - C2 contain the formula ‘= $A2 + B$2’

‘are’ is common. Their codes also have 6 in com-
mon……………..(2) 1. C3 will get the formula ‘= $A3 + B$2’

Comparing (1) and (2) and Leaves are Green with
2. D2 will get the formula ‘= $A2 + C$2’

each other, we can say that leaves are indicated by 7. In the above example, the column A in the first refer-

Thus, (b) is the right answer.
ence and row number 2 in the second reference were
made absolute using the ‘$’ sign in the original for-
9. (d): There are different phases of human learning:
mula of cell C2. Thus, when C2 was copied over to
1. Cognitive - knowing, or head C3 and D2, the absolute part of both cell references
2. Affective - emotions, feelings, or heart and didn’t change in the formula.
3. Psychomotor - doing, or kinesthetic, tactile, hap- In the given question, the value of the formula in cell

tic or hand/body C4 is ‘= $A$3 + B2’. When this formula is copied over
Bloom’s Cognitive Taxonomy in 1956 had remained
to the cell C5, its value would become ‘= $A$3 + B3’,
an essential element in teacher training. The updated as per the behaviour laid out above. Thus, value of C5
version was done by Anderson and Krathwohl in comes out to be 7.
2001. Affective skills were added in 1961 by David 11.(d): Learning allows us to make sense of the world
Krathwohl. around us, the world inside of us, and where we fit
1956 2001 within the world. The life and society have become
Higher Level
very complex, the situations are diverse, we have to
Evaluation
deal with new situations every time. Thus, engage-
Create
ment of students in different activities is very
Synthesis Evaluate important so that there is wholesome development
Analysis Analyze of personality. Education helps us get exposure to
new ideas and concepts.
Application Apply
12. (b):
Comprehension Understand
A. Emphasize your interpretation approach indi-
Knowledge Remember
cates that we may be highly subjective. It is not
Lower Level Noun to verb Form relevant as we have to deal with open and diverse
society. The globalization has happened all

The revised taxonomy has these elements –
around. This type of approach also indicates the
1. Remember - Recognizing and Recalling
feeling of ‘ethnocentrism’.
2. Understand - Interpreting, Exemplifying,
B. Forgetting your own culture means that we as a
Classifying, Summarizing, Inferring, Comparing,
culture are not participating so that establishing
Explaining
contact with others may not be fruitful.
3. Apply - Executing, Implementing C. An ecosystem of mutual respect creates the feel-
4. Analyze - Differentiating, Organizing, Attributing ing of trust so that one culture can communicate
better with other cultures.
5. Evaluate - Checking, Critiquing
D. Knowing the cultural context of other people pro-
6. Create - Generating, Planning, Producing motes the feeling of ‘cultural relativism’, so that
10. (c): In a spreadsheet application, whenever a cell with we emphasize that every culture is equally impor-
a formula is copied over to another cell, any cell refer- tant. This helps in better communication with
ences in the formula will shift as per the relative shift other cultures.
in the references of the copied cell and its destination. E. Stating facts means that we believe in transpar-
ency and trustworthiness. We are not hiding
The above behaviour can be explicitly disabled by anything.
using the ‘$’ character before the column letter or row
Thus, (b) is the correct answer.
number or both in the cell references in a formula.
13. (c): Ignoratio elenchi, is also called as ‘ignorance
We can take a look at what happens to the formula in
cells C3 and D2, when they are copied over from C2. of the refutation’. Refute means overthrow by argu-
ment, evidence, or prove to be false or incorrect or
Example 1 - C2 contain the formula ‘= A2 + B2’ controvert. It is broadly defined as any incorrect

A01_MADAN 07_65901_FM.indd 40 09/01/23 3:38 PM


Memory-based UGC NET Question Paper 2022 - Set 2 xli

argument which reaches an evidentially irrelevant External validity refers to the generalizability and

conclusion. Historically, the fallacy is also more also to randomisation of the treatment/condition
NARROWLY defined as a counterargument to an outcomes.
argument which does not attempt to prove the con-
A. Randomisation - We need unbiased and objec-
tradictory of what was intended to be proved.
tive in our research, and randomisation helps in
In this statement, hunger of knowledge and hunger
that. In simple terms, randomization eliminates
of food are narrowly linked, without any substantial predictability. It follows the theory of probability.
value. Practically, it functions as a ‘catch-all’ category. This removes the possibility of any kind of human
The Traditional Form of Ignoratio Elenchi (fallacy of interruption by the researcher. Randomization
irrelevancy) is occurring here whenever the conclu- eliminates accidental bias, including selection
sion of an argument is irrelevant to its premises. The bias.
fallacy of non sequitur is often identified with this Assume that a teacher has to select 10 students
version. out of 100 for a research purpose, they can be
14. (a): Both statements are true. any ten. Everyone in the class stands 1/100
Statement 1 - Here, we can speak about three mod- chance of selection which are equal for anyone.
els of communication – linear, transactional and This equal chance of being selected is called as
interactive. randomisation.
The linear model is transmission of communication - B. Generalisation – It means that results of a
mostly one way communication. So, called as trans- research study are applicable to the outer world.
mission model of communication. That is called as the external validity.
C. Maturation – The processes within subjects act
Transactional Model takes into account social and
as a function of the passage of time. This happens
relational aspects and thus social context.
when study is conducted on people which are
Interactive Models talk about channel (the way mes- base of research findings. If that element changes
sages are sent and received), field of experience, phys- over a period of time, it threatens the internal
ical context and psychological context. We can take validity of the research.
the examples of social media, interactive and online D. History – This refers to any event other than
marketing, user generated contents, chat rooms etc. the ‘independent variable’. History--the specific
for interactive communication. We always look for events which occur between the first and second
immediate feedback. Here, the communication is measurement.
a process where participants alternate positions as E. Instrumentation – The reliability of the instru-
sender and receiver and generate meaning by send- ment used to gauge the dependent variable or
ing messages and receiving feedback within physical manipulate the independent variable may change
and psychological contexts. in the course of an experiment. Thus, internal
1. The continuous feedback is the main element in validity may be threatened.
interactive communication. We are quickly able Both A and B facilitate the objectivity, unbiasedness

to understand if a message is received or under- and validity of research. The question is about the
stood. WhatsApp may be taken as an example. threat to internal validity in research. In this case,
2. There are continuous updates and communica- these factors also matter for future questions.
tion with audiences.
1. Testing - first test affecting the subsequent tests
3. There is more transparency in our interactive
2. Statistical regression - candidates with extreme
communication.
scores, for example, choosing the poorest or the
4. There are additional alignment opportunities.
most intelligent students in the class
That was it seems to be extrovert as well.
3. Experimental mortality - that is the loss of
5. We can share information with large audiences.
subjects
For statement 2 - The interested candidates may look 4. Selection of subjects - the biases which may
for Bloom Taxonomy once again that comes for digital result in selection of comparison groups.
iconic communication. Randomisation can help in minimisation of this
15. (c): Internal validity is concerned with correctly con- effect.
cluding that an independent variable and not some When the sample size is small, randomization may

extraneous variable is responsible for a change in the lead to Simpson Paradox. Similarly, the students can
dependent variable. study John Henry effect for additional knowledge

A01_MADAN 07_65901_FM.indd 41 09/01/23 3:38 PM


xlii Memory-based UGC NET Question Paper 2022 - Set 2

that is not conducive to internal validity. 19. (c): Assertion (A): According to government sources,
The NET aspirants may remember the following
courses offered through SWAYAM (Study Webs of
aspects which apply on external validity for future Active Learning for Young Aspiring Minds) are fully
questions. recognized and eligible for credit transfer as per the
UGC (Credit Framework for Online Learning Courses
1. Reactive or interaction effect of testing through Study Webs of Active Learning for Young
2. Interaction effects of selection biases and the Aspiring Minds) Regulations, 2021. These regula-
experimental variable tions, now facilitate an institution to allow upto 40%
3. Reactive effects of experimental arrangements of the total courses being offered in a particular pro-
4. Multiple treatment interference gramme in a semester, through the online learning
16. (a): All these universities are actually ancient uni- courses offered through the SWAYAM platform. The
versities. India is one of the oldest civilizations in the credits/marks obtained by the candidate enrolled in
world. It was a centre for higher learning in ancient universities for SWAYAM Certificate will be counted in
times. India contributed greatly to the field of Math- the transcript of the candidate, only if the University
ematics, Astrology, Astronomy, and other sciences. has adopted MOOCs (Massive Open Online Course)
Vedic science was so advanced that it could be com- offered on SWAYAM Platform for Credit Transfer as
pared with the present day’s scientific inventions. per the UGC (Credit Framework for Online Learning
Courses through Study Webs of Active Learning for
Taxila university was one of the oldest universities of
Young Aspiring Minds) Regulations, 2021.
the world but that was not completely centralized.
The government has tried to revive the concept of Reason (R): SWAYAM is a part of MOOC. SWAYAM is
Nalanda University in Bihar. India’s own MOOCs platform, which offers free online
courses on almost all the discipline to students/
17. (c): With video conferencing we can conduct meet-
learners. The objective is to ensure access to the best
ings in two or more locations by audio and video trans-
teaching learning resources to all, including the most
mission. VC specifically uses web cameras, display
disadvantaged. The courses delivered on SWAYAM
devices, microphones (for voice communication),
are expected to reduce the digital divide by providing
high-speed Internet connectivity and personal com-
access to best content to all. These courses are devel-
puters. With VC, those who participate in the meeting
oped by reputed teachers in the country and available
can see, hear and speak to one another, regardless of
free of cost. By integrating SWAYAM MOOCs with
their geographical location. This is useful, can be used
conventional education the learning outcomes of the
both in home and business.
students are expected to improve in coming days.
During recent times, the availability of cloud-based
20. (a): Statement 1 – The agenda of sustainable devel-
services has facilitated the to – do video conferenc-
opment is being carried forward with the help of
ing with minimal upfront investment. Now there is
Sustainable Development Goals (SDGs) which aim
advantage of rapidly emerging Artificial Intelligence
to bring peace and prosperity to all people by 2030.
(AI) powered features to improve both the audio and
They started in the year 2015. This Agenda is a plan of
video performance.
action for people, planet and prosperity. It also seeks
18. (b): Let the original number of students = X to strengthen universal peace in larger freedom. Ear-
Average age = Total age / Number of students
lier there were Millennium Development Goals which
Total original age = 15X years
were set for a period between 2000 to 2015.
There are the 17 SDGs which are an urgent call for
New total age = 15X + 5(12.5) = 15X + 62.5 years =

action by all countries - developed and developing.
15X + 125/2
In simple terms, sustainable development means
New number of students = X + 5
integrating the economic, social and environmental
New average age = 15 years – 6 months = 14.5 years
objectives of society, in order to maximise human
= 29/2 years well-being in the present, without compromising the
ability of future generations to meet their needs.
New average age = 29/2 = (15X + 125/2) / (X + 5)

Statement 2 – Anthropogenic model is the root cause
29/2 (X + 5) = 15X + 125/2

for taking sustainable development agenda more seri-
29X + 145 = 30X + 125
ously. Anthropogenic activities is the human interrup-
X = 145 – 125 = 20
tion in nature for the sake of development to make the
human life comfortable but these activities bring mis-
Thus, the original number of students was 20.

ery also in the form of pollution, hazards etc. We need
Thus (2) is the correct answer.
to have a look at the following diagram.

A01_MADAN 07_65901_FM.indd 42 09/01/23 3:38 PM


Memory-based UGC NET Question Paper 2022 - Set 2 xliii

Forest and other Fires

New species of Unsustainable


cultivated plants deforestations

Genetic Modification (GM) Anthropogenic Activities contributing Constructions (Concretization)


to Environmental Degradation

Variety of Pollutants Unplanned Waste Management

Increase in Food intake

21. (c): The correct answer is (c). 3.  UGC (University Grants Commission) for non
If the container is completely filled with bacteria in 30 technical post-graduation education
minutes, then at the end of 29 minutes, it was filed by 4. CEC (Consortium for Educational Communication)
half. for under-graduate education
5. NCERT (National Council of Educational
If we move back to 28 minutes, it was filled by ¼ th.
Research and Training) for school education
22. (b): According to www.swayam.gov.in - SWAYAM 6. NIOS (National Institute of Open Schooling) for
conducts programmes through a platform that facil- school education
itates hosting of all the courses, taught in the class- 7. IGNOU (Indira Gandhi National Open University)
rooms from Class 9 till post-graduation to be accessed for out-of-school students
by anyone, anywhere at any time. All the courses are 8. IIMB (Indian Institute of Management,
interactive, prepared by the best teachers in the coun- Bangalore) for management studies
try and are available, free of cost to any learner. More 9. NITTTR (National Institute of Technical Teachers
than 1,000 specially chosen faculty and teachers from Training and Research) for Teacher Training
across the country have participated in preparing programme
these courses.
23. (b): The fallacy of false cause (non causa pro causa)
The courses hosted on SWAYAM are in 4 quadrants mislocates the cause of one phenomenon in another
1. video lecture that is only seemingly related. The example taken in
2. specially prepared reading material that can be the question statement are very much natural. The
downloaded/printed moon is able to shine during the night as it gets light
3. self-assessment tests through tests and quizzes from sun that is reflected back.
and 24. (b): Hermeneutics is the art of understanding and
4. an online discussion forum for clearing the interpretation of texts, specifically religious ones.
doubts. Steps have been taken to enrich the learn- Hermeneutics is philosophical and theological in
ing experience by using audio-video and multi- nature. The history of hermeneutics stretches across
media and state of the art pedagogy / technology. epochs, methods and all the disciplines in the human-
For future questions - In order to ensure that best
ities, social sciences, and even the natural sciences. It
quality content is produced and delivered, nine can be identified with four major strands: conserva-
National Coordinators have been appointed. They are tive, critical, radical, and moderate.
the following Sociology is linked with society, groups and social insti-
1. AICTE (All India Council for Technical Education) tutions. Anthropology is the study of human beings
for self-paced and international courses in its entirety. Economics deals with wealth related
2.  NPTEL (National Programme on Technology issues in the form of micro and macro- economics.
Enhanced Learning) for Engineering

A01_MADAN 07_65901_FM.indd 43 09/01/23 3:38 PM


xliv Memory-based UGC NET Question Paper 2022 - Set 2

25. (d): Anthropogenic means human interruption in The feedback can be of following types:

environment for the sake of development. It is basi- 1. Negative feedback: It reflects about mistakes,
cally based on environment possibilism so that errors during the process of communication, the
humans are to determine which way the environment areas which have not been understood properly
should go. Earlier there was concept of environmen- and improvements required.
tal determinism that said that ultimately the environ- 2. Positive feedback: There is proper understand-
ment should decide its own way. ing of the message conveyed by sender.
We need to look at all the events. 3. Negative feedforward: These are basically cor-
Cyclones – They are linked with sea water tem- rective comments about future behaviour of the
perature and pressure conditions. The more about work and communication.
cyclones has been discussed in the earlier paper. The 4. Positive feedforward: These affirm about the
frequency of cyclones has increased during recent future events.
decades.
27. (b): The question papers are made with the perspec-
Nuclear Disasters – In a major nuclear disaster, a
tive of evaluation and assessment of candidates.
reactor core is damaged and significant amounts of
Objectivity – This can be taken as the main element
radioactive isotopes are released which may result
of a good question paper. They should be constructed
in loss of lakhs of lives. The major such examples are
for the sake of qualifying an exam or the ranking of
Chernobyl disaster in 1986 and Fukushima nuclear
candidates.
disaster in 2011. The largest nuclear power plant in
Subjectivity – This means that question paper setter
Europe, the Zaporizhzhia Nuclear Power Plant, is
may be biased so that a particular set of candidates
located in Ukraine, and has been a major flashpoint
get benefited from this.
for nuclear disaster.
Reliability – This refers to the consistency of a meas-
Floods – The frequency of floods has increased during
ure (whether the results can be reproduced under the
the recent times as the natural way of water has been
same conditions).
disrupted due to concretization of natural resources
in the form of buildings, roads, factories etc. Defor- Validition – This refers to the accuracy of a measure
estation has happened in a big way. There are more (whether the results really do represent what they are
and more flash floods which cause more destruction. supposed to measure).
Volcanoes – They erupt from within the earth as per From a research perspective, objectivity, reliability
the temperature and pressure conditions of the earth, and validity are important characteristics.
so anthropogenic activities do not play any role in it. 28. (a): Material Fallacies
Forest Fires – That is basically burning of forest trees There are five characteristics of a valid middle term;
on a massive scale. they are as following:
26. (a): Let’s discuss the main aspects of communication 1. Paksadharmata - The must be present in the
and feedback. minor term. For example, smoke must be present
Communication is the main source of interaction in the hill.
between people by which they share their ideas, 2. Sapaksasattva - It must be present in all positive
and thoughts with each other. Communication is instances in which the major terms are present.
important in every aspect of life. The communica- For example, the smoke must be present in the
tion can be verbal, nonverbal or through by the use kitchen where fire exists.
of sign language or body language. Feedback can be 3. Vipaksasattva - It must be absent in all nega-
referred as ‘fed-back’. Feedback means that response tive instances in which the major terms is absent.
is generated. For example smoke must be absent in the lake in
which fire does not exist.
Feedback can be in the form of non-verbal cues pro-
4. Abadhita - It must be non-incompatible with the
vided by the receiver. Thus, feedback fulfils the basic
minor term. For example, it must not prove the
objective of two-way communication. The importance
coolness of fire.
of feedback can be gauzed from the following facts.
5. Aviruddha - It must be qualified by the absence
1. The element of feedback should always be there
of counteracting reasons which lead to a contra-
to make the communication effective.
dictory conclusion. For example, ‘the fact of being
2. Feedback is effective listening.
caused’ should not be used to prove the ‘eternal-
3. Feedback can motivate.
ity’ of sound.
4. Feedback can improve performance.
5. Feedback is a tool for continued learning. Whenever, one of these characteristics is violated, the
fallacies take its place. It also says that the validity of

A01_MADAN 07_65901_FM.indd 44 09/01/23 3:38 PM


Memory-based UGC NET Question Paper 2022 - Set 2 xlv

an inference depends on the validity of hetu or rea- Similarly E as False, that is option C is a part of answer.
son. It is because of this fact that Naiyayikas consider A as True, that is option D is also part of answer.
fallacies of inference as fallacies of the reason (hetv-
That means that A, C and D are part of answer. Thus
abhasa). Fallacies of inference thus are all MATERIAL
overall, (d) is the answer.
fallacies.
30. (a): Statement I – Qualitative data is powerful as the
Fallacies of reason are of five types – Asiddha, Savyab-
most of this is to be collected from human resources
hichara, Satpratipaksha, Badhita and Virudha. They
whose behaviour is always complex and dynamic.
have been discussed as the last topic in Unit 6 of Pear-
Even the social context is so dynamic and changeable.
son book.
The behavioural context may even change from one
Material fallacies are also known as fallacies of point of time to another. Qualitative data is basically
presumption, because the premises ‘presume’ too non numeric, and it holds conceptual information.
much—they either covertly assume the conclusion or We use their feedback or response to capture their
avoid the issue in view. themes and patterns. The answering research ques-
Reason – The Nyaya system is formal and is deduc- tions is difficult. Thus, the whole scene is based on
tive-inductive can be understood through the follow- sensitive social, historical and temporal (time based)
ing example context.
(For such fallacies, we can refer to the last topic of Statement II – Obtaining right kind of information
discussion in Unit 6.) in a set of complex situations through customer inter-
29. (d): This question is quite similar to question number views, surveys, and feedback. Context sensitivity
26 in the previously solved pervious year paper. This takes the surroundings into consideration while plan-
is the basic information ning for the research. It takes into account community
values and assets.
Universal A All S are P All Birds are
Affirmative Mammals
We can look at the following types of approaches to
deal with the qualitative data so that we get the right
Universal E No S are P No Birds are
kind of information. This may be useful for the future
Negative Mammals
questions.
Particular I Some S are P Some Birds are
Affirmative Mammals 1. Content analysis – This element examines and
Particular O Some S are Some Birds are quantifies the presence of certain words, subjects,
Negative not P Not Mammals and concepts in text, image, video, or audio mes-
sages. The method transforms qualitative input
Now look at the question statement – ‘Some plants
into quantitative data to help you make reliable
are not carnivorous’, that is basically O – type. This is conclusions. This can be used for quantitative
given to be false. data as well.
A. ‘some plants are carnivorous’ denotes I-type – 2. Thematic analysis – This can only be applied to
given to be True in question. qualitative data, and focuses on identifying pat-
B. ‘some plants are carnivorous’ again denotes I- terns and ‘themes’. This can be done with innova-
type – given to be False tive tools such as Dovetail and Thematic.
C. No plants are carnivorous E-type – given to be 3. Narrative analysis – This method used to inter-
False pret research participants’ stories—things like
D. All plants are carnivorous denotes A-type – given testimonials, case studies, interviews, and other
to be True text or visual data.
4. Grounded theory analysis – This is a method of
A E I O
conducting qualitative research to develop theo-
T F T F ries by examining real-world data.
F T F T 5. Discourse analysis – This is the act of research-
D F T D ing the underlying meaning of qualitative data. It
F D D T involves the observation of texts, audio, and vid-
eos to study the relationships between the infor-
If O-type is false, then looking horizontally, we can
mation and its context.
say that 31. (c): Assertion – Cyclones cause an amazing rise in
I-type is True – that denotes that option A is a part of
the sea level. Cyclones are composite phenomena,
answer. consisting of winds, multiple storm waves, storm

A01_MADAN 07_65901_FM.indd 45 09/01/23 3:38 PM


xlvi Memory-based UGC NET Question Paper 2022 - Set 2

surges (which generally last from several hours up to The problem identification is the first thing that

a day) and inland flooding generated by heavy rain- should be done in any research. So, this can be taken
fall. This inland flooding often causes more fatalities as the main objective.
than the wind. Cyclones generally cause massive Analytical synthesis means first probing all the

erosion or deposition – both long- and cross-shore. aspects of research study and then integrating them
The forestations and green belts such as mangroves in a desirable manner.
usually protect us from the damage.
Universal Generalisation is an act of reasoning that
Reason – There are four stages of a cyclone - Forma-
involves drawing broad inferences from particular
tive Stage, Immature Stage, Mature Stage and Decay-
observations. It is widely-acknowledged as a qual-
ing Stage.
ity standard in quantitative research. The chances of
A landfall happens when the centre of the storm
generalisation are limited in qualitative research.
moves across the coast. It often carries heavy winds,
lashing rains, increasing sea level that can pose threat 35. (c):
to people living in the neighbouring region. Energy is the golden thread that connects economic
Once a cyclone makes landfall, it gets separated from growth, increased social equity, and environmental
its ocean energy sources. When tropical cyclones sustainability that allows the world to thrive. The
reach a land surface, they begin to lose their energy indicator for Energy Intensity is in terms of total pri-
and die out. This is because they are no longer receiv- mary energy supply and GDP at PPP.
ing heat energy and moisture from the ocean, which Under target number 7 of Sustainable Development
is needed to drive them. Goals, we need to double the global rate of improve-
32. (a): The National Council for Technical Educa- ment in energy efficiency by the year 2030.
tion was enacted in 1993 and was set up in 1995. 36. (d): Phenomenology - This is a qualitative research
According to Government sources - the main objec- approach that seeks to understand and describe the
tive of the NCTE is to achieve planned and coordi- universal essence of a phenomenon. The approach
nated development of the teacher education system investigates the everyday experiences of human
throughout the country, the regulation and proper beings while suspending the researchers’ precon-
maintenance of Norms and Standards in the teacher ceived assumptions about the phenomenon.
education system and for matters connected there-
Ethnography – In this qualitative research, a
with. It includes the whole gamut of teacher educa-
researcher or an ethnographer study a particular
tion programmes including research and training of
social/cultural group with the aim to understand it
persons for equipping them to teach at pre-primary,
better. They actively participate in the group in order
primary, secondary and senior secondary stages in
to gain an insider’s perspective of the group. He may
schools, and non-formal education, part-time edu-
try to get the equivalent experience that is similar to
cation, adult education and distance (correspond-
the group members. There are experiences relating
ence) education courses.
to process, participation, data collection, interviews,
33. (a): Earth is known as ‘Water Planet’, as our planet and the analysis of documents and artifacts.
has more water than land.
Ethnomethodology – This is the study of how social
1. In reality, less than three percent of Earth’s water
order is produced in and through processes of social
is fresh water, and most of that three percent is
interaction. It generally seeks to provide an alterna-
inaccessible.
tive to mainstream sociological approaches.
2. Over 68 percent of the FRESH WATER on Earth is
found in ice and snow (including glaciers). Symbolic interactionism – This counsels us that a
3. Almost 30 percent of FRESH WATER is found in complex social situation like triangulated research
ground water. involves much more than simply forging a rational
3. Only about 0.3 percent of FRESH WATER is found design among fairly like-minded colleagues, since
in the surface water of lakes, rivers, and swamps. participants must work hard to create a working,
4. In totality, more than 99 percent of Earth’s water shared understanding of their research.
is unusable by humans and many other living 37. (d): According to Government sources - DigiLocker
things. is a flagship initiative of Ministry of Electronics & IT
As the question is in the form of increasing quantity, (MeitY) under Digital India programme. DigiLocker
(a) is the correct answer. aims at ‘Digital Empowerment’ of citizen by providing
34. (c): Historical research is basically a descriptive study, access to authentic digital documents to citizen’s digi-
which mostly is a qualitative study. tal document wallet. The issued documents in Dig-

A01_MADAN 07_65901_FM.indd 46 09/01/23 3:38 PM


Memory-based UGC NET Question Paper 2022 - Set 2 xlvii

iLocker system are deemed to be at par with original (b) Interest in media for two individuals may not
physical documents as per Rule 9A of the Information develop in compatibility. Between two individu-
Technology (Preservation and Retention of Informa- als, one may be interested in football matches,
tion by Intermediaries providing Digital Locker facili- and other one in serials.
ties) Rules, 2016 notified on February 8, 2017 vide (c) Empathy is shown in how much compassion and
G.S.R. 711(E). understanding we can give to another. Sympathy

Benefits to Citizens is more of a feeling of pity for another. Empathy
1. Important Documents Anytime, Anywhere! is our ability to understand how someone feels
2.  Authentic Documents, Legally at Par with while sympathy is our relief in not having the
Originals. same problems.
3. Digital Document Exchange with the consent of (d) External influence may help in temporary asso-
the citizen. ciation, and not in compatibility.
4.  Faster service Delivery- Government Benefits,
40. (b): Non Conventional Learning Programmes-
Employment, Financial Inclusion, Education,
Health. The education system in which teaching-learning
activities is offered other than the on-campus with

Benefits to Agencies fixed time classrooms. For examples, evening learn-
Reduced Administrative Overhead: Aimed at the
ing, distance learning, vocational studies, skill-based
concept of paperless governance. It reduces the courses, online learning, etc.
administrative overhead by minimizing the use of Non-conventional education is inspired by the P.H.
paper and curtailing the verification process. Coombs, and Ahmed who has worked on non-formal
Digital Transformation: Provides trusted issued doc-
education for the poor.
uments. Issued Documents available via DigiLocker Characteristics of Non-conventional learning
are fetched in real-time directly from the issuing
• Learners oriented
agency.
• No fixed curriculum
Secure Document Gateway: Acts as a secure docu-
• Cost-effective
ment exchange platform like payment gateway • Linked to employment
between trusted issuer and trusted Requester/Verifier • Continuous
with the consent of the citizen. • For improvement of Quality
Real Time Verification: Provides a verification mod-
Target groups of non-conventional education are
ule enabling government agencies to verify data unemployed youths, the school dropped out, under-
directly from issuers after obtaining user consent. privileged group, women and girls, Tribal and Minor-
ity population. This type of education is also for
38. (a): We will use the formula: N3 + 1 where N is 1, 2,
literacy programmes.
3, 4, …………
41. (b): Let the number of birds on Tree A = A
13 + 1 = 2
And birds of Tree B = B
23 + 2 = 10
In first situation – if two birds shift from B to A, then
33 + 3 = 30
birds on A and B will be equal.
43 + 4 = 68
B–2=A+2
53 + 5 = 130

A – B = –4 ……(1)
63 + 6 = 222

In second situation – If 2 birds shift from A to B, then
73 + 7 = 350
birds on B will be double than those on A. Here, we
Thus, (a) is the correct answer.
are doubling the birds on A so that both on A and B
are equal.
39. (c): The natural ability to live or work together in
harmony because of well-matched characteristics is 2(A – 2) = B + 2
called as compatibility between individuals. 2A – 4 = B + 2
(a) Stewardship is an ethical value that embodies 2A – B = 2 + 4 = 6
the responsible planning and management of 2A – B = 6 ……(2)
resources.

A01_MADAN 07_65901_FM.indd 47 09/01/23 3:38 PM


xlviii Memory-based UGC NET Question Paper 2022 - Set 2

Deducting 2 from 1 4. Abadhita - It must be non-incompatible with the


A – B – (2A – B) = – 4 – 6 minor term. For example, it must not prove the
coolness of fire.
–1A = –10
5. Aviruddha - It must be qualified by the absence
A = 10 of counteracting reasons which lead to a contra-
Putting A = 10 in equation (1) dictory conclusion. For example, ‘the fact of being
10 – B = –4 caused’ should not be used to prove the ‘eternal-
–B = –4 –10 ity’ of sound.
B = 14 The above five are linked with Hetvabhasa or types

Now, A=10, B= 14, thus (b) is the answer. of it. In Indian logic a fallacy is called hetvahasa. It
The question can be solved directly from the answer means that middle term appears to be a reason but is
by adding and reducing the values. not a valid reason. All fallacies are material fallacies.
We have mentioned the five characteristics of a valid
42. (d): Grapevine communication is basically informal middle term. When these are violated, we have falla-
in nature so that more and more people feel comforta- cies. Five kinds of fallacies are recognised.
ble in it. They get connected in it, they feel connected 1. Assiddha or sadhyasama - This is the fallacy of
to informal groups where they express themselves. unproved middle.
Thus, informal communication boosts the morale of
2. Savyabhicara - This is the fallacy of irregular
the employees.
middle.
In the grapevine, employees discuss various issues. 3. Satpratipaksa - Here the middle term is contra-
The anxieties of the employees are also expressed. dicted by another middle term.
The formal communication moves in a set direction, 4. Badhita - It is the non-inferentially contradicted
so employees are bound by rules and regulations. middle.
43. (b): Basically, criterion of valid knowledge is called as 5. Viruddha - It is the contradictory middle.
pramana. The inference of one proposition from two 44. (c): Malware is actually worrisome, it is any pro-
premises is called as syllogism. Any error of reason- gram or file that is intentionally harmful to a com-
ing is called as fallacy. In nyaya system, it is called as puter, network or server. Malware comes in so many
hetvabhasa. variants, there are numerous methods to infect
The aspects A and B have been discussed in an earlier computer systems. The malware includes computer
question. viruses, worms, Trojan horses, ransomware and
C. hetu (causation) is missing, (it may seem more spyware.
like inductive). 45. (c):
D. Linga paramarsa: The Nyaya syllogism has five A. Montessori Method: This was developed by
terms. Among them, middle term works as a Dr. Maria Montessori (1907) is based on the
bridge between the major and the minor terms. idea of self-education. The child does not sit still
Therefore, the middle term has main responsibil- and idle to the lessons taught by the teacher,
ity to prove a syllogism valid or invalid. How a here, in this system, the child educates himself
middle term is related to major term is lingapara- or herself, but with the teacher as a guide. The
marsha. There are five characteristics of a middle child uses hands-on materials and specially
term: prepared classrooms to develop both intellect
1. Paksadharmata - it must be present in the minor and character. This type of self-actualization of
term. For example, smoke must be present in the students is called “normalization.” This classroom
hill. system is suitable in theory for all ages but mostly
2. Sapaksasattva - It must be present in all positive practiced by preschoolers and kindergartners.
instances in which the major terms are present. There is respect for feelings of child, the way
For example, the smoke must be present in the he/she wants to educate matters.
kitchen where fire exists. B. Integral Education, as envisioned by the Mother
3. Vipaksasattva - It must be absent in all negative and Sri Aurobindo, regards the child as a grow-
instances in which the major terms are absent. ing soul and helps him to bring out all that is
For example smoke must be absent in the lake in best, most powerful, most innate and living in his
which fire does not exist. nature. It helps the child develop all facets of his

A01_MADAN 07_65901_FM.indd 48 09/01/23 3:38 PM


Memory-based UGC NET Question Paper 2022 - Set 2 xlix

personality and awaken his latent possibilities so parents. Every experience becomes a learning tool for
that he acquires the child in its growth. IE helps the child to integrate
– a strong, supple, healthy, beautiful body with its true Self, its surroundings, its society, its
– a sensitive, emotionally refined, energetic country and humanity; in other words, to become the
personality complete being, the integral being that the child is
– a wide-ranging, lively intelligence and will meant to be.
– the subtler spiritual qualities that unify and Answers for Comprehension Passage
harmonise the being around the child’s inmost 46. (a)
Truth or Soul. 47. (a)
The focus and emphasis in Integral Education (IE)
48. (b)
is not just information and skills acquisition but
49. (b)
also self-development, triggered from within the
child and supported and nourished by teachers and 50. (a)

A01_MADAN 07_65901_FM.indd 49 09/01/23 3:38 PM


This page is intentionally left blank

A01_MADAN 07_65901_FM.indd 50 09/01/23 3:38 PM


CHAPTER

1 Teaching Aptitude

01 Education: Basic Elements

02 Teaching: Concept, Objectives, Levels of


Teaching (Memory, Understanding and Reflective),
Characteristics and Basic Requirements.

03 Learner’s Characteristics:
Characteristics of Adolescent and Adult
Learners (Academic, Social, Emotional
and Cognitive), Individual Differences.

04 Factors Affecting
LEARNING Teaching Related to:
Teacher, Learner, Support
OBJECTIVES Material, Instructional
Facilities, Learning
Environment and Institution.

05 Methods of Teaching in Institutions


of Higher Learning: Teacher-Centred
vs. Learner-Centred Methods;
Off-line vs. Online Methods (Swayam,
Swayamprabha, MOOCs, etc.).
06 Teaching Support System:
Traditional, Modern and ICT Based.

07 Evaluation Systems: Elements and Types of Evaluation,


Evaluation in Choice Based Credit System in Higher
Education, Computer Based Testing, Innovations in
Evaluation Systems.

M01_MADAN 07_65901_C01.indd 1 27/12/22 8:14 PM


1.2 Chapter 1

John Dewey defines education as the power by which


Education a man is able to control his environment and fulfill his
possibilities.
Definitions and Meaning of Education According to Frobel, ‘Education is a process by which
the child develops its inner potential in a manner so as to
“Education is not the preparation for life,
participate meaningfully in the external environment’.
education is life itself ”
United Nations Educational, Scientific and Cultural
– John Dewey
Organization (UNESCO) celebrates October 5 as the
“Education is simply the soul of a society as it passes World Teachers’ Day, while India celebrates Teacher’s Day
from one generation to another” on September 5 thats is birth anniversary of our second
– G.K. Chesterton President Dr. Sarvepalli Radhakrishnan.

According to New Education Policy, 2020, education Important Learning T heories and A pproaches
is a fundamental for achieving full human potential, Philosophy is a very vast subject, and education is closely
developing an equitable and just society, and promoting linked with it. All aspects of education, such as aims, objec-
national development. tives, curriculum, teaching methods, teacher, textbooks
The Global Education Development agenda reflected in the and discipline are influenced by philosophy. There have
Sustainable Development Goal-4 (SDG-4) of the 2030 agenda been more and more questions from learning theories in a
for sustainable development, adopted by India in 2015 seeks direct and in assertion reason form. So, their discussion is
to “ensure inclusive and equitable quality education and pro- important for us.
mote lifelong learning opportunities for all by 2030”. The concept of learning is very dynamic. The educa-
Every human being often reflects the learnings that he tional process has three types of questions.
has received from his elders - parents, family members (a) ‘Why’ is decided by philosophy?
and teachers. That’s why we spend 20–25 years of life in (b) ‘How’ is decided by psychology?
getting formal education. We want to become good citi- (c) ‘What’ is decided by the social needs?
zens as a result. Education is closely related to the appli-
cation part of philosophy. Philosophy gives ideals, values Hence, education is based on philosophical, psycho-
and principles and our education system works them out. logical, and social aspects. As per Western philosophy,
Education without philosophy is like a tourist who knows Socrates is considered to be the father of education.
the name of the place where he wishes to go but does not
1. Behaviourist School of Thought: This approach was
know how to find the place.
developed in the 1900s, it was quite dominating in
Thus,
the early 20th century. Learning basically consists of
(a) The main task of education is the transfer of our a change in behaviour as a result of stimuli (S) from
knowledge to the next generation so as to make environment and response (R) of the individual. Such
our society a better place. changes do happen in sequential manner as a result
(b) In a formal manner, teacher functions as the of use of reward and punishment. Naturally, we get
facilitator of knowledge. Teaching is the activity interested in measurable changes in behaviour.
directed at enabling learning. It is an activity usu- For example, when a teacher asks student a question
ally conducted by an expert in their subject. in accountancy. In case, the student gives the cor-
(c) Learning is the final outcome of teaching process. rect answer, the teacher may appreciate the student
(d) Evaluation is also done to assess the progress of by saying ‘commendable’, ‘excellent’, ‘very good’, etc
students. This acts as reinforcement to the student’s response,
that gains strength with passage of time. In teaching-
Thus, philosophy of society, education, teaching, learning process, the focus is on drill and practice and
learning and evaluation are closely related. Research tutorials.
adds to the existing stock of knowledge.
We are starting our discussion with education and The following perspectives are important in the context
gradually shifting towards teaching. of behavioural theory.
According to Tagore, the aim of education is self real- E L Thorndike: There are two perspectives, firstly, a
ization. It means the realization of universal soul in one’s response to a stimulus is reinforced when followed by
self. It is a process which cannot be realized without edu- a positive rewarding effect. Secondly, a response to a
cation. He synthesizes the ancient Vedantic. stimulus becomes stronger by exercise and repetition.
Swami Vivekananda defines education is the manifes- This view of learning is akin to the “drill-and-practice”
tation of perfection already in a man. programmes.
Aristotle defined education as a ‘creation of a sound
The behaviourists have put forward three main laws of
mind in a sound body’.
learning:
According to Heinrich Pestalozzi, ‘Education is the nat-
ural harmonious and progressive development of man’s (a) Law of Exercise: This law relates to strengthen-
innate powers’. ing the connection through practice.

M01_MADAN 07_65901_C01.indd 2 27/12/22 8:14 PM


Teaching Aptitude 1.3

  The main principle of learning is repetition, it Discovery Learning: This is based on the constructiv-
is termed as ‘drill’ or ‘drill and practice’ as done in ist approach, with no or little guidance from teacher.
subjects such as mathematics. It was introduced by Jerome Bruner and is a method
(b) Law of Readiness: This indicates the student’s of inquiry-based instruction. This popular theory
willingness to make S-R connection. encourages learners to build on past experiences and
(c) Law of Effect: The focus is on the effect of a knowledge, use their intuition, imagination and cre-
‘response’. The satisfying results reinforce the ativity, and search for new information to discover
response, and result in a reward. The annoying feel- facts, correlations and new truths. A simulation in
ings weaken the same, and result in ‘punishment’. which the teacher assists the students in the recall or
the application of relevant principles, is called ‘guided
Type of conditioning: Conditioning is of two types. discovery method’.
(a) Classical conditioning: This concept was set by 4. Transformative Learning Theory (TLT): TLT is a
Ivan Pavlov, that is linked with feeding of a dog learning approach for adult education and young
that salivates on a bell’s voice that is followed by adult learning. It was suggested by Jack Mezirow.
food. Two stimuli are linked together to produce TLT focuses on the idea that learners can adjust their
a new learned response. It is called as ‘generaliza- thinking based on new information. Adults has impor-
tion’, unconscious or automatic learning. It is also tant teaching and learning opportunities connected
termed as Reflexive or Involuntary conditioning. to their past experiences and that critical reflection
(b) Operant Conditioning: The organism learns by and review could lead to a transformation of their
way of modification in behaviour through rein- understanding.
forcement or punishment. It is termed as ‘volun-   This approach works well for adult students.
tary’ and ‘instrumental behaviour.(Skinner sug- Children don’t have the same kind of transformation
gested the concept of ‘programmed instruction’). with their learning experiences—and with life expe-
2. Cognitive School of Thought: This approach rience. Teachers can employ this learning theory by
started in late 1950s. The acquisition of knowledge is encouraging their students to learn new perspectives
through brain: the learner is an information-proces- while questioning their assumptions and open the floor
sor who absorbs information, undertakes cognitive for discourse to cement their new train of thought.
operations on it, and stocks it in memory. The ability
of brain to experience, senses, and thought is known 5. Social Learning Theory (SLT): This is meant to deal
as cognition. It is linked with lecturing and reading with difficult students who like to disrupt the class-
textbooks. The learner is a passive recipient of knowl- room and cause trouble. This theory focuses on the
edge. The learners can be influenced by both internal concept of children learning from observing others
and external elements. The cognitive theory broke by acting on or not acting on what they see exhibited
off into sub-theories that focus on unique elements of by their classmates. This learning theory was founded
learning and understanding. by Albert Bandura. He conducted an experiment
called the Bobo doll experiment in the early ’60s, dur-
  Jean Piaget worked a lot on cognitive psychology. ing which he studied children’s behaviour after they
His work focuses on environments and internal struc- watched an adult act aggressively with a doll-like toy.
tures and how they impact learning. He noted how the children reacted when the adult got
  Teachers can give students opportunities to ask rewarded, punished, or suffered no consequences after
questions (probing), and think out loud. The teach- they attacked the doll. Bandura wrote about his find-
ing should be done in a more organized manner, that ings in 1977, detailing social learning theory and how
help students to know their thought process works, it affected the behavioural development of students.
and utilize this knowledge to construct better learn-   There are four elements to SLT:
ing opportunities.
  Vygotsky claimed that we are born with four (a) Attention: which calls upon different or unique
‘elementary mental functions’: Attention, Sensation, lessons or activities to help children focus.
Perception, and Memory. (b) Retention: focusing on how the student will
3. Constructivist School of Thought: This school internalize information and recall it later on.
believes that learning is not passive. They began to (c) Reproduction: drawing on previously learned
consider learning as personal. Everyone constructs behavior and when it’s appropriate to use it.
his or her own knowledge and skills as a result of (d) Motivation: which can extend from seeing other
undergoing experiences. The learning or knowledge classmates being rewarded or punished for their
continuously gets constructed as new knowledge is actions.
acquired. It means new experience is integrated with 6. Experiential Learning Theory (ELT): ELT focuses
previous experience. The focus is on Independent on ‘learning by doing’. Using this theory, students
learning, experiential learning and programming are encouraged to learn through experiences that
etc. The focus is on ‘generalisable skills’ that are upon can help them retain information and recall facts.
individual discovery. ELT, was identified by David Kolb in 1984. Though

M01_MADAN 07_65901_C01.indd 3 27/12/22 8:14 PM


1.4 Chapter 1

his influence came from other theorists such as John word ‘utility’, whatever is useful is good, and whatever
Dewey, Kurt Lewin, and Jean Piaget, Kolb was able to is good is useful. A pragmatist lives in a world of facts.
identify four stages of ELT. The first two stages, con-   Pragmatism focuses on activity or doing. There are
crete learning and reflective observation, focus on no absolute values of life. Truth is created during the
grasping an experience. The latter two, abstract con- course of experience. Humans are active beings and
ceptualization and active experimentation are about have the ability to solve their problems through the
transforming an experience. To Kolb, effective learn- logic of experiments and scientific methods.
ing is seen as the learner goes through the cycle of   The thinkers John Dewey, Kilpatrick, Mead are
experiential learning theory. Students can enter the some of the main exponents of this philosophy.
cycle in any way and at any point. 4. Rationalism: Rationalists claim that there are signifi-
  The “Situated Learning Theory” and “Community cant ways through which our concepts and knowledge
of Practice” draw many of the ideas of the learning gain independently through our sense experience.
theories considered above. They have been developed 5. Empiricists: A theory which states that knowledge
by Jean Lave and Etienne Wenger. comes only or primarily from sensory experience. Our
five senses are eyes, ears, nose, tongue and skin.
7. Connectivism Learning Theory: This is one of the 6. Existentialism: A good education emphasizes indi-
newest educational learning theories. It focuses on viduality, thus the first step in any education then is to
the idea that people learn and grow when they form understand ourselves. Every individual is unique and
connections, as per their roles and obligations in the education must cater to the individual differences, thus
society. They influence learning. Teachers can utilize developing our unique qualities, to harness his potenti-
connectivism in their classrooms to help students alities and cultivate our individualities. Socrates is con-
make connections to things that excite them, helping sidered to be the first existentialist through his work
them learn. Teachers can use digital media to make ‘Know thyself’. In Hindu philosophy, there is focus on
good, positive connections to learning. They can help the knowledge of the self, Janna. Buddhist philosophy
create connections and relationships with their stu- seeks enlightenment, being a ‘bodhi’.
dents and with their peer groups to help students feel 7. Gestalt Psychology: It believes that the whole is
motivated about learning. The e-learning or digital greater than the sum of its parts.
learning is based on this.   For example, in the human body, there are cells, tis-
sues, organs, systems, etc., and the sum of all these
Some Other Philosophies components (human body) is greater than the sum of
its parts. This is because the parts are interrelated to
1. Idealism: This aspect is closely linked with philosophy. each other. There is synergy created. Further, Gestalt
The word ‘idealism’ has been derived from ‘ideal’. It is psychology demonstrated the significance of percep-
actually about ‘Mind and Self’, that reflects ‘spiritual- tion. It also showed that complex learning need not
ism’. The universal mind or God is central in under- occur gradually through lengthy practice but may
standing of the world. God is the source of all creation, develop through insight.
and knowledge, spirit and mind constitute reality.   This theory states that the transfer of learning can
(a) Values are absolute, eternal and unchanging. be best achieve when an individual is in the very best
(b) Real knowledge is perceived in mind that is more of the frame of mind; in the times that he or she is
important than knowledge gained through the aware of the meanings of a particular situation or
senses. experiences and to their practical application to one’s
(c) Man has a superior nature that is expressed in the daily life. It believes in the concept of ‘synergy’ that
form of intellectual culture, morality and religion. believes that 2 + 2 = 5. That is positive synergy. The
 Fröbel, Kant, Plato, Swami Dayanand, concept of negative synergy is 2 + 2 = 3.
Vivekananda and Sri Aurobindo the main propo- 8. Eclectic Philosophy: Eclecticism is nothing but the
nents of idealism. fusion of knowledge from all sources. It is a peculiar
type of educational philosophy that combines all good
2. Naturalism: Contrary to idealism, naturalism is a ideas and principles from various philosophies.
philosophy that believes nature alone represents the 9. Yoga philosophy: This speaks about the theory and
entire reality. It takes into account natural matter, practice for the realization of the ultimate truth con-
natural force and natural laws. cerning human being and the world. In Vedanta, yoga
  Our senses are the gateway to knowledge, and is understood as a spiritual union of the individual
nature is the source of all knowledge. The mind is soul with the supreme soul. Yoga is a spiritual effort to
subordinate to nature. The educative process must be attain perfection through the control of sense organs,
pleasurable and set in natural surroundings. gross body, subtle mind, intellect and ego. It guides
  The main protagonists of naturalism are Tagore, to achieve the highest wisdom through spiritual
Rousseau, and Herbert Spencer. realization. The first scripture of Yoga is Patanjali’s
3. Pragmatism: Pragmatism is basically a Greek word ‘Yoga-sutras’, followed by Vaisya’s “Yoga–bhasya” and
that means practice or action. Here, we focus on the Vacavpati Mishra’s ‘Tattva-vaisaradi”.

M01_MADAN 07_65901_C01.indd 4 27/12/22 8:14 PM


Teaching Aptitude 1.5

  The Yoga Philosophy is closely associated with 1. Concrete Experience (CE): The concrete experience
Samkhya philosophy (discussed in detail in Unit 6). can be in the form of a new experience or situation, or
The Yoga presents a practical path for the realiza- a reinterpretation of existing experience in the light
tion of the self whereas the Samkhya emphasizes the of new concepts. Assume that you get a job of Assis-
attainment of knowledge of self by means of concen- tant Professor and get some concrete experiences
tration and meditation. Thus, yoga is the practice and while teaching in the class.
Samkhya is its theory. 2. Reflective Observation of the New Experience (RO):
We may add to or modify our knowledge (new learn-
Advanced O rganizer Model ing) in the light of their existing knowledge. Though
you may teach very well in the class but still learn cer-
(Ausubel’s Model) tain things for betterment of classes in future. For a
Advance Organizer Model is given by great education- teacher the learning is continuous. This step may be
ist psychologist, David Ausubel. The primary concern of termed as ‘review’ also.
this model is to help teachers organize and convey large 3. Abstract Conceptualization (AC): The reflection
amounts of information in a very meaningful and efficient gained in step 2 may settle in our mind in the form of
manner. This model is taken from verbal learning princi- an abstract concept. Some principles to teach better
ple. As we understand, any subject is a chain of concepts, get settled in our minds.
when we accept these facts, that is also settled as a chain in 4. Active Experimentation (AE): We look to imple-
our mind. The new concept should be related with the old ment abstract concept into the future situations. The
one. We need to explore big ideas with students before this. teacher applies his idea/s in the class to teach the stu-
dents in a better manner.
Mental Model: Closely linked with advanced organizer
model, a ‘mental model’ is any organized pattern (such as Concrete
visual representations) that consist of both structure and Experience
process (a kind of flow chart) that helps a student com- Feeling
prehend content knowledge. They may help to solve the
problems.

Continuum
This theory of meaningful verbal learning. It has three
major concerns: Diverging
Accommodating
(a) How knowledge (curriculum content) is feel and do feel and watch
organized; Active Processing Continuum Reflective
(b) How the mind works to process new information Experimentation Observation
(learning); and Doing Watching
Perception
(c) How teacher can apply these ideas about curricu- Converging Assimilating
lum and learning when they present new material think and do think and watch
to students (instruction). This model is designed
to strengthen student’s cognitive structure.
Under ‘syntax’, Advance Organizer Model has three Abstract
phases of activity. Conceptualisation
Phase 1: The presentation of the advance organizer Thinking
(mental model)
Phase 2: The presentation of the learning task or
learning material; and Source: https://www.simplypsychology.org/learning-kolb.html
Phase 3: The strengthening of cognitive organiza-
tion. In this, the relationship of the learning material Thus, these four steps complete the outer part of the fol-
is to be done with the existing ideas to bring about an lowing diagram.
active learning process.
Active Reflective
Kolb’ s Learning Cycle Experimentation Observation
(Doing) (Watching)
We have just discussed the concepts of cognitivism,
behaviourism, and constructivism. That way David Kolb’s Concrete Accommodating Diverging
Learning Experience become automatically important as Experience (CE/AE) (CE/RO)
it is the combination of above. This model is in the form (Feeling)
of a few processes. This is also called as the reflective or
experiential learning model. And few questions have Abstract Converging Assimilating
been asked in NET JRF exam. Though it may seem to be Conceptualization (AC/AE) (AC/RO)
initially tough, but our discussion will help. It is a practi- (Thinking)
cal topic. The main objective of teaching is learning. The
learning cycle can be divided into the following steps. Source: https://www.simplypsychology.org/learning-kolb.html

M01_MADAN 07_65901_C01.indd 5 04/01/23 4:50 PM


1.6 Chapter 1

Now we move towards the combination of the two ele- Havighurst and Neugarten have given two important
ments that have been discussed earlier. Thus, we get four functions of the education system:
steps further—Diverging, Assimilating, Converging and 1. A mirror that reflects society as it is or to be the stabi-
Accommodating (DACA). Similarly, the feeling, watching, lizer of the society.
thinking and doing (FWTA) are also important. The inter- 2. An agent of social change or a force directed towards
ested students may mug up the parts for future questions. implementing the ideas of society.
Some of the important philosophies of education have
been mentioned below: George Payne, a sociologist, has given three main
­functions of education:
Table 1.1 Important Concepts in Education and
Teaching and their Proponents 1. Assimilation of traditions
2. Development of new social patterns
Concepts Proponents 3. Creative and constructive role
Basic education (Wardha Education Mahatma Emile Durkheim worked on structural functionalism that
System)–This initiative was put in Gandhi means how does the society function. He focused on the
words by Dr Zakir Hussain in 1937, transmission of society’s norms and values as the major
this is also called as Nai or Buniyadi function of society. The education was recognized as a
Taleem. Its objective is to make training for specialized roles (division of labour) that
individuals self sufficient. also includes adopting some occupation for livelihood.
Education is required to perform the function of
Learning must take place in nature Rabindranath
and from nature Tagore (a) cultural transmission and enrichment
(b) acceptance and reformulation
Integral education Sri Aurobindo (c) change and reconstruction

Focus on the spiritual aspects of Dr. Sarvepalli Two terms can be discussed here.
Indian philosophy Radhakrishnan Enculturation: It is the process why the young generation
learns the traditional ways of society. This practice differs
Education to transform human mind J. Krishnamurti from one society to another. Enculturation is formalized
through education.
Experiential learning John Dewey
Acculturation: It is the process through which a person
Self-education through development Maria or group from one culture comes to adopt the practices
of individuality Montessori and values of another, while still retaining their own dis-
tinct culture. This concept has become important in an
Kindergarten focus on self-activity, Fröbel increasingly globalized society.
creativeness, and social cooperation
Factors Determining Educational Aims
No formal learning; nature is the Rousseau
only teacher Education is mostly a planned and purposeful activity.
Educational aims are necessary in giving direction to
unique activities which are determined by the following
Forms of Education factors.
We can divide the education into three parts. 1. Philosophy: Philosophy and education are the two
1. Formal sides of the same coin. Philosophy is the main factor that
2. Informal determines the aim of education. Education is termed
3. Non-formal to be the best means for propagation of philosophy.
2. Human nature: It is closely linked with philosophy.
Integration of these three makes the education holistic For example, idealists regard unfolding of the divine
and comprehensive. These forms of education have been in man as the aim of education.
discussed in Figure 1.1 3. Socio-cultural factors and problems: Education has
to preserve and transmit the cultural heritage and tra-
Major Aims and ditions from one generation to another.
4. Religious factors: In ancient India, Buddhism
Objectives of Education emphasized the inculcation of the ideals of r­ eligion,
such as ahimsa and truth into the prevailing educa-
“We want that education by which character is tional system.
formed, strength of mind is increased, the intellect is 5. Political ideologies: There can be authoritarianism,
expanded and by which one stands on one’s own feet.” conservatism, liberalism, moderates, feminism, totali-
– Swami Vivekanand tarianism, corporatism, democracy, environmental-
ism, and so on. The society develops accordingly.

M01_MADAN 07_65901_C01.indd 6 27/12/22 8:14 PM


Teaching Aptitude 1.7

Pre-planned and organized


Formal
Imparted through institutions like

FormS of Education
Schools Colleges, etc.

Indirect and Spontaneous


Informal
Imparted through day-to-day
activities from home and community

Flexible, Open system


Non-formal
Imparted through conscious and
deliberate efforts

Figure 1.1 Forms of Education

6. Exploration of knowledge: Knowledge is must for drawing out of the best in child and man-body, mind
good interpersonal relationships, healthy adjustment and spirit’.
in life, modification of behaviour, self-awareness and 10. Moral and character development: This is consid-
for social growth, it is also a source of happiness. The ered as the best goal of education by Indian education
pace of creation of knowledge has become very fast. system. According to Herbert Spencer, education must
7. Vocational: Education prepares the child to earn his enable the child to cultivate moral values and virtues,
livelihood and make him self-sufficient and efficient such as truthfulness, goodness, purity, courage, rever-
in both economic and social factors. ence and honesty.
8. Self-actualization and total development: Self actu- 11. Citizenship: To become better citizens of society, to
alization is the ultimate realization of the potential do our duties and responsibilities.
gifted to us by God. It helps in holistic development 12. Education for leisure: Leisure time is meant for
that includes physical, mental, emotional, social and enjoyment and recreation. It helps us to take rest and
spiritual-moral developments. regain energy. It gives birth to physical and mental
9. Harmonious development: According to Mahatma balance. It helps us developing artistic, moral and aes-
Gandhi,” ‘By education, we mean an all round thetic elements.

Physical Cognitive

Intellectual

Holistic
Emotional Development
Co-Cognitive

Social

Affective Psychomotor
Moral

M01_MADAN 07_65901_C01.indd 7 27/12/22 8:14 PM


1.8 Chapter 1

Some Specified Aims of Education in India: When 4. Teaching is a planned activity and effective teaching
India gained freedom there was a need for reorientation depends on the following factors.
and restructuring of all our existing social, political and (a) How clearly the students understand what they
educational systems, in order to meet the socio-economic, are expected to learn.
political and educational needs of the country. (b) How accurately their learning can be measured.
Since independence, various committees and commis- 5. Teaching is a process in which the learner, teacher
sions were appointed to lay down the aims and objectives and other variables are organized in a systematic way
of education in India. to attain some predetermined goals.
6. Teaching is an activity that influences a child to learn
National Educational Policy (1986): This policy specified and acquire desired knowledge and skills and also
the f­ ollowing aims and objectives of our education: their desired ways of living in the society.
1. All-round material and spiritual development of all
people
2. Cultural orientations and development of interest in Basic Teaching Models
Indian culture
There is no basic model of teaching that fits into all situa-
3. Scientific temper tions. Basically, we have discussed two models-pedagogy
4. National cohesion and andragogy-that are almost extremes of a teaching
5. Independence of mind and spirit, that furthers the continuum. Then there is heutagogy approach.
goals of socialism, secularism and democracy
6. Manpower development for different levels of economy
Pedagogy
7. Promoting research in all areas of development Pedagogy as a conventional approach is also called as
8. Education for equality the art and science of teaching. The teacher assumes
the primary role in deciding the main aspects of teach-
New Education Policy (2020): This is the first policy in ing. Friedrich Herbart, in his work Universal Pedagogy
21st century. We can see a remarkable shift from 1968 and (1906), advocated five formal steps in teaching.
1986 policies. It is built upon the foundational pillars of 1. Preparation: Linking new material to relevant exist-
Access, Equity, Quality, Affordability and Accountability. ing ideas (memories).
The new policy is aligned to the 2030 Agenda for 2. Presentation: Present the new lesson after taking
Sustainable Development. It aims to transform India into account the actual experience of concrete objects.
into a vibrant knowledge society and global knowledge 3. Association: Comparison of the new idea with exist-
superpower by making both school and college educa- ing ideas with an objective to implant new ideas in the
tion more holistic, flexible, multidisciplinary, more suit- mind of learner.
able to current and future needs. It aims to bring out the 4. Generalization: To take learning beyond perception
unique capabilities of each student. It has been discussed and experience to develop absrtract ideas.
in details in the last unit of Higher Education. 5. Application: To make the newly acquired knowledge
become an integral part of the life of the student.
Concept of Teaching
The main objective of effective teaching is learning. The Andragogy
teachers are facilitators of knowledge. The progress and Here, the learner is mostly self-directed. He is mostly
prosperity of a nation depends upon the development of responsible for his own learning. The students not only
its human resources. We always need highly competent receive knowledge but they interpret through ‘discovery’.
teachers who can give direction to the society and nation. The students set the pace of their own learning.
The name of this unit, teaching aptitude is all about The instructors facilitate the learning by offering
evaluating candidates who want to enter this successful opportunities to learn themselves and acquire new knowl-
career of teaching profession on the basis of their qualities edge. Thus, students develop new skills.
of knowledge and skills. This also includes qualification, Self-evaluation is also the characteristic of this
intelligence, attitude, etc. to name a few. approach. Andragogical approach is also identified with
Teaching can be defined in the following ways: ‘adult learning.’
1. Teaching is the purposeful direction and manage-
ment of the learning process. Heutagogy
2. Teaching is a process of providing opportunities for stu- It is basically self-determined learning. The learners
dents to produce relatively permanent change through are highly autonomous. The emphasis is placed on
engagement in e­ xperiences provided by the teachers. development of learner capacity and capability with
3. Teaching is a skilful application of knowledge, experi- the goal of producing learners who are well-prepared
ence and scientific principles with an objective to set for the complexities of today’s workplace. ‘Double Loop
up an environment to facilitate learning. Learning’ is considered as a part of heutagogy.

M01_MADAN 07_65901_C01.indd 8 27/12/22 8:14 PM


Teaching Aptitude 1.9

Table 1.2 Major Shifts in Teaching 3. Teaching is closely related to education, learning,
instruction and training.
From Pedagogy To Andragogy 4. Teaching is essentially an intellectual activity.
5. Teaching is an art as well as a science.
Teacher-centered, Learner-centered, flexible 6. Teaching tends towards self-organization.
fixed designs process 7. Teaching is a social service.
8. Teaching includes lengthy periods of study and
Teacher’s direction and Learner’s autonomy
training.
decisions
9. Teaching has a high degree of autonomy.
Teacher’s guidance and Teacher’s facilitation, 10. Teaching is a continuous process.
monitoring of learning support and 11. Teaching is a profession.
encouragement for learning These features have been discussed later in the coming
paragraphs.
Passive reception in Active participation in
learning learning Levels of T eaching
Learning within the four Learning in the wider The main objective of teaching is to ensure learning that
walls of the classrooms social context takes place during interaction between an experienced
person (the teacher) and an inexperienced person (the
Knowledge as ‘given’ and Knowledge evolves and is student).
‘fixed’ created There are basically three different levels of teaching
to ensure learning - memory level, understanding level
Disciplinary focus Multidisciplinary, and reflective level. Morris L. Biggie, in 1976, suggested
educational focus ‘least thoughtful’, ‘thoughtful’ and ‘the most thoughtful’
Linear exposure Multiple and divergent for these three levels respectively. The autonomous devel-
exposure opmental stage was added at a later stage, but has been
discussed first as it appears at the earliest stage of life.
Assessment is short Assessment is
multifarious, continuous Autonomous Development Level
This level is student centered. Intellectual development
Source: NCERT Pedagogy just happens in a natural manner. There is no felt need for
any kind of formal teaching. Thus, the presence or role of
Pedagogy is also called engagement, andagogy as culti- a teacher is assumed to be more negative than positive.
vation and heutagogy as realization. There should be little or no leadership, direction, coer-
cion, prescription, or imposition of student thought or
behaviour.
Nature of Teaching Some educators assume that this level may prevail
The nature of teaching includes the following: throughout the learning, even during tertiary level. The
assistance of teachers is still crucially needed. The suc-
1. Teaching works at different levels in life. cessful teachers are required to not only teach students
2. Teaching takes place in a dynamic environment. in academic areas but also understand students’ personal

The 5 pillars of Educational Psychology

Biological Cognitive Developmental Social and Mental and


personality physical health
Biopsychology/ Perception Learning Social Abnormal
Neuroscience
Thinking Lifespan Personality Therapies
Sensation development
Intelligence Emotion Stress,
Consciousness lifestyle and
Memory Motivation health

Figure 1.2 Five Pillars of Psychology

M01_MADAN 07_65901_C01.indd 9 27/12/22 8:14 PM


1.10 Chapter 1

6. The subject material is well-organized and simple in


nature. The subject matter is simple.
7. The knowledge delivered is definite, structured and
observable.
Teaching Method
Reflective level The teaching is subject-centered and so are the teaching
(Most thoughtful) methods – drill, review, revision and asking ­questions.
Drill (and practice) means repetition or practice to attain
proficiency in memorizing.
Understanding level Review or revising the elements relates learners to new
(Thoughtful) experiences and form new associations.
The question technique is used to examine whether
Memory Level of Teaching or not knowledge-level objectives of teaching have been
(Less Thoughtful) achieved.
Teacher’s Role
Autonomous Development Level The teacher is the stage-setter, resource manager as well
as the evaluator in the classroom.
Figure 1.3 Teaching Pyramid Role of Learner
The role of the learner is a passive one, as the subject
and cultural characteristics. For this to happen, teachers content, teaching techniques and teaching methods are
need to be flexible in teaching and creating a supportive decided by the teacher.
learning environment.
Teaching Equipment
Different kinds of teaching aids, such as visual, audio
Memory Level of Teaching (MLT) and audio-visual aids models, charts, maps, pictures, TV,
Herbart is the main proponent of the memory level of radio etc. develop curiosity within the learners towards
teaching. Good memory includes rapidity in learning, the learning process.
stability of retention, rapidity in recalling and the ability Nature of Motivation
to bring only desirable contents to the conscious level.
The teaching–learning process is basically a ‘Stimulus– The motivation by a teacher drives students to learn better
Response’ (S–R). that should result in the development of an ‘intrinsic’
MLT is actually the initial stage of formal teaching feeling, it should not be a forced one. The focus is on
where a learner puts factual material to memory. memorization of facts. Hence, the nature of motivation at
this level of teaching is purely extrinsic.
1. MLT is the initial stage of teaching.
2. MLT induces the habit of rote memorization of facts Evaluation System for Learners
and bites of information. 1. The evaluation system mainly includes oral, written
3. MLT enables the learner to retain and also to repro- and essay-type examination.
duce the learnt material whenever required. 2. The evaluation is done on predetermined objectives
4. The evaluation system mainly includes oral, written to check the power of memorization of the students.
and essay-type examination. 3. For the written test, short-type, recall-type, recognition-
type, multiple-choice and matching-type test items are
Thus, the three major aspects are:
used.
1. Learning of the material 4. Here, the learners are required to attempt the maxi-
2. Retention of the material mum number of questions.
3. Reproduction of the material as and when required
Psychological Bases of Memory L evel
O bjectives of Memory Level of T eaching of T eaching
1. The main objective is imparting knowledge and infor- 1. According to theory of ‘cognitive development’, Jean
mation to the learner. Piaget stated that memory level is meant for starters.
2. Knowledge gained is basically factual that has been The learners are at the pre-operational level of cogni-
acquired through memorization or rote learning. tive development in the school. They cannot operate
3. It covers only the knowledge-based objectives of upon abstract concepts. As per their mental develop-
Bloom’s taxonomy. ment, they can learn simple concepts without analyz-
4. Teaching is subject-centered. ing their true meaning and nature. Normally, they are
5. Simple memorable things are taught to students. not expected to reflect upon the learnt facts.

M01_MADAN 07_65901_C01.indd 10 27/12/22 8:14 PM


Teaching Aptitude 1.11

2. Herbartian theory of apperception suggests that the classification, comparison, inference of instructional
young pupil mind gets ready at this stage for perceiv- messages.
ing themselves and the world around them. The brain 2. Application objectives include the use of a proper
gets ready for a big mass of factual information. It procedure.
includes concepts, elements, structures, models and 3. The subject matter is wider and more detailed. It
theories. They can acquire and retain information includes the application of principles and generaliza­
about a large number of things, objects, and material tions of real-life situations.
through memorization.
Teaching Methods and Equipments: They include the
3. In conditioning theory, Pavlov and Skinner advocated
following:
that underlines the concept that correct response
made by an individual is strengthened by reinforce- 1. Lecture (cum demonstration)
ment to retain the learned subject-matter longer and 2. Discussion method
to facilitate ease in further learning. 3. Inductive-deductive (discussed in Unit 6).
4. Pavlov’s classical conditioning applies the mechanical 4. Exemplification and explanation
process as applied in memory level of teaching; and
thorough review or retrieval, correct learning by the Exemplification makes use of specific instances (exam-
student is rewarded and retained for future use. ples) in classroom to clarify a point, to add interest, or
to persuade. Classroom climate is more motivational.
Suggestions for Memory-Level T eaching Learners are actively engaged in the learning process.
Motivation at the understanding level of teaching is
1. The teaching material should be objective and useful. extrinsic as well as intrinsic in nature.
2. The teaching material should be finite and progress Teaching equipment includes the following:
from simple to complex.
3. Teaching aids should be adequate, and parts of the 1. Models
content should be integrated and well-sequenced. 2. Charts
4. The subject matter should be presented in a system- 3. Flash cards
atic and organized manner. 4. Pictures
5. There should be scope for continuous evaluation to mea-
The learners can comprehend the concepts more easily with
sure improvement in the memory power of students.
the help of teaching aids. The teacher must use the right aid
6. The retention of the material in memory of students
and equipment as per the need and level of students.
can be increased by practice and exercise.
7. There should be scope for continuous reinforcement
during the course of instruction. Evaluation
The evaluation system mainly includes both essay and
objective-type questions. Understanding level needs a
Understanding-Level of Teaching more comprehensive evaluation. The tests and tools need
(ULT) proper planning. Planned tests could evaluate the stu-
dent’s ability to comprehend, grasp, analyze, synthesize
Morrison is the main proponent of the understanding and discriminate.
level of teaching. It is ‘memory plus insight’ as it goes These abilities can be examined using oral and writ-
beyond just memorizing facts. It focuses on mastery of ten tests. There should be some scope for testing practical
the subject. knowledge.
It makes pupil understand the generalizations, prin-
ciples, facts and some application parts also. It provides
more and more opportunities for the students to develop Role of the T eacher and Learner
‘intellectual behaviour’. The teacher plays a significant role. He presents learning
The teaching at this level stands higher as compared to material and generates interest in them. The sequence of
that of memory level. presentation of content, mode of instruction and methods
There is higher development of cognitive abilities. used in evaluation remain with the teacher.
They become more capable of thinking, presenting the The role of the learner is more active and they have
facts in a more logical manner, analyzing them properly to work hard at this level. The participants at this level
and drawing inferences. They are able to evaluate the are secondary learners. The framework is set by the
relationships between the principles and facts and also teacher.
provide space for the assimilation of facts.

O bjectives Reflective Level of Teaching (RLT)


According to the revised Bloom’s Taxonomy, understand-
ing-level teaching aims at the following objectives: “True teachers are those who help us think for
ourselves.”
1. It aims at the understanding of instructional mes- – Dr. Sarvepalli Radhakrishnan
sages by means of interpretation, exemplification,

M01_MADAN 07_65901_C01.indd 11 27/12/22 8:14 PM


1.12 Chapter 1

Hunt is the main proponent of RLT. This level is the high- Which level of teaching is reflected through this?

est and most practical level of teaching. Society expects (a) Autonomous development level
that a person should be able to understand the whole (b) Memory level
­scenario and apply his knowledge. This is also termed (c) Understanding level
as the ‘introspective level’. It also means thinking deeply (d) Reflective level
about something. The correct option is (d).
1. It is highly thoughtful and useful. The pupils occupy Offering positive and negative examples means making
the primary place, and teachers assume the second- the students familiar with actual life situations.
ary place.
2. A learner can achieve this level only after attaining Dynamic Environment
memory level and understanding level.
Effective communication is the essence of teaching
3. Reflective level makes the learners to solve real problem
that itself is very dynamic in nature. Teaching changes
situations in life.
according to time and place. Its environment consists of
4. At this level, the student is made to face a real prob- interaction among three variables:
lem situation.
1. Independent variables: Teachers are largely assumed
5. Classroom environment is to be sufficiently ‘open and to independent as they are in a position to manipulate
independent’. the behaviours of students.
6. Here, the problem is identified, defined, and then a 2. Dependent variables: Students depend upon teach-
solution is found. ers for learning.
7. The student’s original thinking and capabilities 3. Mediating or Intervening variables: The examples
develop at this level. are teaching methods, teaching instructional facili-
8. The teacher needs to be democratic. The knowledge ties and motivational techniques. A mediator variable
should not be enforced on students. hypothesize that the independent variable impacts
9. The students becomes active, and they need to the mediating variable, which in turn impacts the
become innovative and imaginative. dependent variable.
10. Essay-type test is used for evaluation. Attitude, belief According to Ronald T. Hyman, the dynamic triad of
and involvement are also evaluated. teaching involves the teacher, the learner, and the subject
matter. Education is also assumed to be a tripolar process
Merits of R eflective Level T eaching of pupil, teacher and social environment.
1. The teaching at this level is leaner-centered. Teaching is a Complex whole
2. There is more interaction between the teacher and
Teaching makes use of all sorts of techniques, methods, and
the learner.
media and also entails skills such as questioning, probing,
3. This level of teaching is appropriate for the higher class.
exemplifying, etc. Then teaching also makes use of media
Demerits of Reflective Level T eaching such as audio-visual media, human interaction, print media,
realia, electronic media, etc. Success in teaching, to a great
1. It is mostly suitable for mentally mature children. extent, depends upon selection and use of appropriate tech-
2. The study material is neither organised nor pre- niques, methods and media that makes it complex.
planned or systematic.
Teaching Visualizes Change in Behaviour
The concepts of cognition and metacognition as they are
linked with reflective practices. The changes as discussed in cognitive domain (knowledge),
affective domain (attitude, motivation) and psycho motor
Metacognition: This is a subdivision of cognition, or domain (physical skills) are tentatively permanent. The
a type of cognition. Metacognition is defined as the changes that occur in learners need not be performative but
­scientific study of an individual’s cognitions about his or be potential ability of the learners.
her own cognitions. It is basically application part.
Teaching can be Direct or Indirect
Cognition: This is a mental process that includes memory,
attention, producing and understanding language, reason- (a) Direct methods: Here, teachers use methods,
ing, learning, problem-solving and decision-making. such as lecture, demonstration, etc. and engages
It is often referred to as information processing, apply- students in face-to-face interaction.
ing knowledge, and changing preferences. It is something (b) Indirect teaching: Here, active teaching meth-
that is more basic. ods, such as role-play, project, assignment,
inquiry or other such activities are applied. They
are basically learner-centred methods where the
involvement of learners is higher.
Stopover
A teacher offers many positive and negative examples
These methods have been discussed in the concept
in the classroom to support his or her presentation. box.

M01_MADAN 07_65901_C01.indd 12 27/12/22 8:14 PM


Teaching Aptitude 1.13

Training, Development, Instruction and


Concept Box Indoctrination
In case we want to look at the teacher, learner, Training: Training is a short-term process utilizing a sys-
curriculum, physical conditions (climate), the following tematic and organized procedure by which students or
scenarios emerge, starting from bipolar to quadrilateral. non-managerial personnel learn technical knowledge
skills for a definite purpose. It is for a short duration and
1. It is a bipolar process. for a specific job related purpose. When a teacher joins
some institution, the authorities make them familiar with
Teacher Learner rules, regulations and policies of the institution, that is
called as ‘induction training’.
The Inquiry Training Model was developed by Richard
figure 1.4 Bipolar Process Schumann to teach students a process for investigating
and explaining unusual phenomena.
2. It is a tripolar process.
Development: This is a long-term continuous educational
Teacher process utilizing a systematic and organized procedure
which managerial personnel learn conceptual and
Teacher theoretical knowledge for general purpose. It involves
Learning philosophical and theoretical educational concepts and
process its purpose is long-term development for managers.
Learner Curriculum
Instruction: Instruction is the process of teaching and
engaging students with content–how does a teacher
figure 1.5 Tripolar Process organize time and activities in implementing that content
(curriculum) and plan.
3. It is a quadri-polar process.
Indoctrination: Indoctrination can be termed as the
highest order of teaching. In indoctrination, beliefs and
Teacher ideas are impressed upon others and can be included in
teaching. Teaching can be done without indoctrination,
but indoctrination is not possible without teaching.

Physical Teaching
Curriculum
inteLLectuaL activity
climate Learning
Teaching is essentially a cognitive activity. It requires
Process
conscious and continuous organization of learning activi-
ties. It entails the creation of a conductive and supportive
learning environment. A teacher has to evolve a suitable
Learner plan of action to achieve desired changes in the behaviour
of a group of learners. The learners constitute the raw
figure 1.6 Quadripolar Process material. The expectations of learners are always varied.

teaching is a science as WeLL as an art


Teaching can be Vertical or Horizontal The teaching profession is based upon a systematic body
In vertical teaching, teachers may lead students deep into of knowledge, which has been derived from social, psy-
the topic. They teach learners higher-order thinking skills chological, historical, political and economical spheres
such as analysis, synthesis, evaluation and creation. of life. It is also influenced by the religious, spiritual and
Conversely, in horizontal teaching, more areas spread ethical beliefs of the society. Teaching techniques are sys-
over several topics instead of going deeper into one topic tematic; it has; they have definite steps to be followed and
are covered. are easily communicable. On the basis of assumptions of
science, a teacher can be trained. There are definite steps
Teaching may be Planned or Unplanned that are followed in training a teacher.
Traditionally, teachers plan for instruction before they go Teaching is an art as well. It takes places in a dynamic
to the classroom for teaching-content analysis and task environment. The teacher has to deal with individual dif-
analysis is performed, the techniques and methods to be ferences in a class in a tactful manner. This requires a lot
used for teaching are decided, and then evaluation is per- of individualized approach and discretion.
formed.
However, with the coming of active leaning methods, seLf o rganization
no strict planning is possible as one is not clear in advance The people in teaching profession are sensitive towards
what could be the possible way of organizing teaching- growth and development because it is self-organized.
learning activities. They evolve a definite mechanism to sustain and promote

M01_MADAN 07_65901_C01.indd 13 27/12/22 8:15 PM


1.14 Chapter 1

the standards of the teaching profession. The growth in emphasize on the output rather than the process of
teaching profession is organic in nature, i.e., the growth instruction) and measurable.
happens in a spontaneous manner. The objectives of teaching and learning must integrate
at the end of the instruction.
Social Service There are two main ways of classifying instructional
It has been accepted that education is a potent tool to objectives. One classification is given by Bloom, whereas
bring about changes in any nation. It is useful to develop another classification is given by Gagne and Briggs.
the society.
Bloom’s Classification of Teaching and
High Degree of Autonomy Instructional Objectives
This autonomy is different from autonomous level that is According to this classification, instructional objectives
applicable ib = n case of students. There is a high degree fall under one of the following three categories:
of autonomy in the teaching profession, right from curric-
ulum development, planning activities for a year, identi- 1. Cognitive Domain: It is the core learning domain. It
fying instructional objectives, deciding upon the method is related to the development of intellectual capa-
of teaching, media, evaluation criteria and techniques to bility that is basically thinking or knowledge. Other
divide the admission and promotion rules and autonomy domains (affective and psycho-motor) also need
in planning and execution of co-curricular activities. the cognitive domain as basic platform. The fol-
lowing points are important in context of cognitive
Teaching as a Profession domain.
Teaching is the profession that makes other professions (a) Knowledge: It is basically about recalling infor-
possible. There are many courses, such as B.Ed, M.Ed, mation or contents.
etc., which impart knowledge and skills that establish the (b) Comprehension: It is the ability to grasp the
foundation for a successful pathway to a career in teach- meaning of a material.
ing. It entails a number of years of study and intermittent (c) Application: It converts abstract knowledge into
practice.
training periods. A teacher has to improve his or her quali-
(d) Analysis: It breaks communication into its con-
fication for advancements in the teaching career.
stituent parts so that it can be understood better.
(e) Synthesis: It basically combines the constituent
parts to convert them into a whole. It is the anto-
Objectives of Teaching nym of analysis.
An objective describes an intended result of instruction (f) Evaluation: It involves judgement made about
rather than the process of instruction itself. A good objec- the value of methods and materials for particular
tive should be specific, outcome based (i.e., it should purposes.

Comparative Table Bloom’s 1956 vs. Anderson and Krathwohl, 2001


1956 2001

Complex Evaluation Create


form

Synthesis Evaluation

Cognitive Analysis Analyze


functions

Application Apply

Comprehension Understand

Simple
form Knowledge Remember

Noun 1956 Verb Form


Source: www.theresearchgate.com

M01_MADAN 07_65901_C01.indd 14 27/12/22 8:15 PM


Teaching Aptitude 1.15

  Anderson, a former student of Bloom, and the rational brain is yet to start functioning. The term
David Krathwohl rearranged the the six levels in ‘Emotional Intelligence’, given by Daniel Golemann
the following manner by making changes in ‘eval- enables the individual to deal intelligently with various
uation’ and ‘synthesis’. social problems that one faces in life situations.
  An individual’s affective behaviour or learning is
  (i)  Remembering: Recall or retrieve previous influenced by both emotional intelligence and cogni-
learned information. tive learning.
  (ii) 
Understanding: Comprehending the mean-
  Therefore, the implication for the educational pro-
ing, translation, interpolation and interpre-
cess is that cognitive learning and affective learning
tation of instructions and ­problems. State a
should be planned to go hand in hand.
problem in one’s own words.
(iii) Applying: Use a concept in a new situa-   Affective flattening is a kind of schizophrenia.
tion or unprompted use of an abstraction. This means that the person doesn’t have the full
Applies what was learned in the classroom range of emotional expression that others do. They
into novel situations in the work place. are relatively immobile, show unresponsive facial
  (iv)  Analysing: It separates a material or con- expressions, have poor eye contact. Disorders such
cepts into component parts so that its organ- as catatonia and alogia show inability to spea.
izational structure may be understood. It 3. Psychomotor Domain: It is mainly concerned with
distinguishes between facts and inferences. the acquisition of technical skills. There are five levels
  (v) Evaluating: Make judgments about the of instructional levels under it.
value of ideas or materials. (a) Imitation: It includes the demonstration of a skill
  (vi)  Creating: Builds a structure or pattern from by a skilled person, and the learner tries to follow
diverse elements. Put parts together to form the same.
a whole, with emphasis on creating a new (b) Manipulation: A learner tries to experiment
meaning or structure. various aspects, such as manipulating machinery,
  In Figure 1.6, the ‘synthesis’ (at point number 5) equipment, etc.
in the earlier model should be assumed to be (c) Precision: Accuracy in performing various acts
shifted to point number 6 of ‘creating’ in the increases with practice.
Anderson’s revised model. (d) Articulation: Achieving a desired level of effi-
  The top model ‘evaluation’ goes on the second ciency and effectiveness through practice.
last position in the revised model. (e) Naturalization: Skill is internalized, and an indi-
2. Affective domain: Man is not only rational but emo- vidual is able to adapt, modify or design new tech-
tional as well. Affective is closely linked with emotions, niques, methods or procedures according to the
motivation, feelings, sympathy, co-operation, fellow- requirements of a situation.
feeling, willingness to participate. It entails preference The three domains of learning are not mutually exclusive,
for some object, issue, notion, etc. Affect is also treated the differentiation among them is warranted because of
as a response to different social, political and economic the nature of the behavioural outcomes.
issues in the form of attitudes. An individual has to It is clear that cognitive as well as affective learning takes
develop and nurture desirable positive attitudes and place simultaneously and with the same content of learning.
interests for his or her better adjustment in the society.
  Affective also means valuing what is being learned
and ultimately incorporating the values of discipline Gagne and Briggs Classification of T eaching
into a way of life. It asks for better student participa- and I nstructional O bjectives
tion and includes the following sub-levels: According to this classification, the learning outcomes fall
(a) Receiving: willingness to listen under one of the following categories.
(b) Responding: willingness to participate 1. Intellectual Skills: These skills are crucial for dealing
(c) Valuing: willingness to be involved with the environment. They include concept learning,
(d) Organizing: willingness to be an advocate of an idea rule learning and problem solving.
(e) Characterization: willingness to change one’s 2. Cognitive Strategies: These include methods and
behaviour or way of life techniques for one’s own learning, remembering and
  Affective education takes a long time to achieve the thinking skills.
objectives as it deals with the behaviour. An individu- 3. Verbal Information: It refers to organized bodies of
al’s emotional and rational components of the brain are knowledge that an individual acquires.
somewhat independent of each other and operate sep- 4. Motor Skills: They are basically about motions car-
arately. But at times, when they work in harmony with ried out when the brain, nervous system and muscles
each other. When the individual is faced with a prob- work together.
lem or dilemma and is required to make a decision, 5. Attitudes: They refer to an internal state of an indi-
the emotional center of the brain functions first while vidual.

M01_MADAN 07_65901_C01.indd 15 27/12/22 8:15 PM


1.16 Chapter 1

The teaching objectives can be put in the following forms 9. From Analysis to Synthesis: Initially, the students
also: have little or vague knowledge about the topics.
Analysis means dividing problems into its constituent
Cultural context parts, and then those are studied.
Synthesis means understanding by connecting the
Philosophy of people knowledge acquired through analysing the parts. A
teacher should use analytic–synthetic method.
National goals 10. Follow Nature: It means to regulate the education of
a pupil according to his nature.
Aims of education
11. Training of Senses: The five senses, sight, sound,
taste, smell and touch are gateways to knowledge.
It is better if all or maximum of these senses can be
Objectives of content applied in teaching. Montessori and Fröbel are the
main proponents of this maxim.
Translated into behaviours 12. Encouragement to Self-study: Dalton’s system is
based on self-study.
figure 1.7 Hierarchy of Formulation of Objectives
principLes of t eaching
They are closely related to maxims. Teaching methods are
Effective Teaching Practices based on two types of principles, general principles and
psychological principles.
MaxiMs of teaching
A maxim is a ground rule or a fundamental principle General Principles
that has evolved over a period of time. It is a guide for 1. Principle of Direction: The direction of develop-
future action or behaviour. Teaching also has its own ment from head to feed is called as cephalo-caudal.
set of maxims, which have been discussed below. When this happens from centre (spinal cord) towards
1. From Simple to Complex: Simple things be explained peripheries (heart, limbs etc.) is called as proximodis-
first. This can be done with day to day examples. Then tal sequence).
gradually, a teacher can move towards concepts and 2. Principle of Activity (Learning by Doing): Fröbel’s
technical terms. This creates interest among learners Kindergarten (KG) system is based on this principle. It
to acquire new knowledge. This is helpful for better includes both physical and mental activities. For exam-
retention. ple, students are asked to make charts and models.
2. From Known to Unknown: This is related to the 3. Principle of Interest: By generating genuine interest
first maxim. Retention is always better if new among the learner’s community, the effectiveness of
knowledge can be linked with what is known. the teaching-learning process can be increased.
3. From Seen to Unseen: Students should be imparted 4. Principle of Linking with Life: Life is a continuous
knowledge about the present, and then they can experience, and learning linked with life can be more
understand the past and the future better. enduring.
4. From Concrete to Abstract: The mental develop- 5. Principle of Definite Aim: This is important for opti-
ment of students happen better with the concrete mum utilization of teaching resources and making
objects they become familiar with and define micro- learning more focused.
words for them at a later stage. 6. Principle of Recognizing Individual Differences:
5. From Particular to General: The students should be Every student is unique in terms of intelligence,
presented with examples first and then general laws attitude, abilities and potentialities, and socio-eco-
and their derivations can be explained to them. The nomic background. The teaching method should be
experiments and demonstrations serve this purpose. devised in such a manner that it makes all students
6. From Whole to Part: Gestalt psychologists have avail equal opportunities in life.
proved that we first see the whole object and then its 7. Principle of Selection: The horizon of knowledge is
parts. For example, we first perceive the tree and then expanding every single day. The teacher should be
its trunk, branches, leaves, etc. Thus, the introduction able to pick contents that can be more relevant and
or overview of the topics is important. Linked with it updated to the learners’ objectives.
is the principle of integration during learning. 8. Principle of Planning: Every teacher has certain
7. From Indefinite to Definite: The teacher should help time-bound objectives, and hence, teaching should be
transform indefinite knowledge into definite knowl- systematic, to make optimum use of resources within
edge and aim to clarify the doubts of students. the time limit.
8. From Psychological to Logical: During initial stages, 9. Principle of Division: To make learning easier, the
psychological order is more important, whereas for subject matter should be divided into units and there
grown-up learners, logical order is emphasized more. should be links between the units.

M01_MADAN 07_65901_C01.indd 16 27/12/22 8:15 PM


Teaching Aptitude 1.17

10. Principle of Revision: To make learning enduring,


the acquired knowledge should be revised immedi- Concept Box
ately and repeatedly.
Microteaching
11. Principle of Creation and Recreation: This principle
is a must to make the classroom environment humor- Microteaching is one of the most recent innovations in
ous and creative. teacher training program. It is a professional develop-
mental tool in pre-service or in-service teacher training
12. Principle of Democratic Dealing: It involves students programs. It helps teachers to better understand the
in planning and executing different activities; it helps processes of teaching and learning and provides the
in developing self-confidence and self-respect among opportunity to learn teaching skills, to study their own
the learners. teaching, and to study the teaching of others.
Psychological Principles This is an organized, scaled-down teacher training
program where a trainee teacher plans a short lesson,
1. Principle of Motivation and Interest: A teacher teaches it to a reduced group of students (3 to 10) in a 5
needs to understand that every student is a unique to 20 minute lesson, and then reflects on their teaching
psychological entity, and a student can be motivated afterwards. The lesson may be video recorded for either
after identifying his or her motives and needs. individual or peer review. The trainee teacher’s micro-
2. Principle of Recreation: Recreation is necessary to lesson is reviewed, discussed, analyzed, and evaluated
tackle fatigue after attending lengthy classes. This to give a feedback. Based on this feedback, the trainee
breaks monotony and prepares students for learning teacher re-teaches the micro-lesson, incorporating those
again. points raised during the discussion and analysis.
3. Principle of Repetition and Exercise: This is espe- Thus, microteaching has the potential to improve
cially true in case of small children. the teachers’ pedagogic skills, competencies, self-con-
4. Principle of Encouraging Creativity and Self- fidence, beliefs, and attitudes with minimum available
expression: This is specifically applicable in subjects facilities. It provides students with valuable teaching
such as mathematics and languages. experiences and make them aware of the benefits and
5. Principle of Sympathy and Cooperation: This relationships between theory and practice.
principle is required for the motivation of students.
6. Principle of Reinforcement: Students should be 1. Plan
suitably rewarded for their desired behaviour. 2.
k
ac

Te
7. Principle of Remedial Teaching: This principle is a
db

ch
ee

necessary for the teacher to identify mistakes and


-F

suggest better answers to the problems.


Re
6.

Microteaching Cycle
Stopover

ck
5.

The feedback is an important element in classroom


ba
Re

teaching. Which of the following elements does not ed


-T

Fe
ea

apply in the context of feedback?


3.
ch

(a) It should be available in a time bound manner 4. Re-Plan


(b) Feedback should not be educative in nature
(c) Feedback should reference a skill or specific
knowledge rationale behind them. They should be connectivity
(d) Be sensitive to the individual needs of the student between new material with earlier learned material.
The correct option is (b). 3. Scaffolded Instruction: The teachers initially pro-
vide the students with the required support. This sup-
port is gradually reduced as students demonstrate
competence in working independently.
Effective Teaching Behaviour
The following four dimensions have been identified for
The following aspects make teaching more effective. effective teaching behaviour:
1. Teach for understanding rather than exposure:
Most often, the teachers teach for exposure rather
than in-depth understanding of students. The teach-
Methods of Teaching
ers should focus on less number of big ideas that are
important to critical understanding. ‘If a child can’t learn the way we teach, we
2. Explicit Instruction: Teachers should share those big should teach the way they learn’
ideas with students and repeat them often. Students —Ignacio Estrada
should learn what are they going to learn and

M01_MADAN 07_65901_C01.indd 17 27/12/22 8:15 PM


1.18 Chapter 1

Clarity Variety Task orientation, mostly Engagement in learning


(For Effective Teaching) (Newness, Motivation, in class task
Sustaining attention) (approach towards (self-directed goals and
practical and better life) drawing new experiences)
1. Objectives 1. Intensity of stimulus – 1. Goal directed 1. Make learning a
2. Knowledge speech pattern/bright light 2. Action directed ­developmental activity
3. Individual Differences 2. Contrast – sudden change 3. Achievement/ 2. Develop concentration
4. Integration of Facts 3. Teacher’s bodily gestures completion 3. Complete specified
5. Pinpointedness 4. Teacher’s movement 4. In-built motivation assigned homework
6. Levels of Teaching in class 5. Need fulfillment or task
7. Techniques of Evaluation 5. Self activity of participants 6. Relaxation 4. Reinforce the process of
8. Reflection 6. Audio-visual aids Example: Karma Yoga development of concep-
7. Teacher’s personal tual understanding and
9. Balance
behaviour/interaction application
10. Sharpness
8. Pausing

The term teaching method refers to the general principles, The teacher is an active participant, the students are
pedagogy and management strategies used for classroom assumed to be passive listeners. It may become one way
instruction. They become important after setting objectives. communication if students don’t converse during the
As per NTA-NET syllabus, we have the following two class.
extreme set of methods for institutes of higher learning: It can be made a two-way communication if the teacher
1. Teacher-centred methods allows students to ask few questions to clarify a point. It
2. Learner-centred methods is continuous in nature. The class listens, writes and notes
facts and ideas for remembering and to think them over
These can be assumed to be two extreme approaches. In at a later stage.
between, we can have a third method approach, which is
called mixed approach. Basic Features
1. It is formal and narrative in nature.
Teacher-Centred T eaching Methods 2. It presents a series of events or facts.
3. It explores a problem.
Lecture Method
This is the most conventional and dominating teaching Advantages
method. Teaching is primarily meant to be delivered through 1. It is cost effective as the number of students can be
lecture method. The facts, principles or relationships are taught simultaneously. It becomes more objective as
explained in the class. the syllabus can be covered in a prescribed time.

Table 1.3 Important Methods of Instruction


Teacher-centred strategy Mixed strategy Learner-centred strategy
Large group methods Small group methods Individualized methods
1. Lectures 1. Group discussion 1. Tutorials
2. Team teaching 2. Seminar 2. Assignments
3. TV or video presentation 3. Panel discussion 3. Project work
4. Dialogic Teaching and Direct Teaching 4. Brainstorming 4. Case study
5. Project method or work 5. Programmed instruction
6. Tutorials 6. Computer-assisted learning
7. Case study 7. Interactive video
8. Role-play 8. Open learning
9. Simulation 9. Personalized system of instruction (PSI)
10. Demonstration 10. Heuristic method

M01_MADAN 07_65901_C01.indd 18 27/12/22 8:15 PM


Teaching Aptitude 1.19

6. What
3. Verbal 5. What
students
message students
say & do
think

1. Visual 4. Students
message notes

2. Physical presence

Figure 1.8 Lecture Method

2. It simplifies the task of the teacher. stimulating and problem-solving questions. Leave
3. It imparts factual information. The attention of time at the end for clarifications and questions.
students can be drawn to important points.
The lectures can be made interactive with the following
4. During lecture, interruptions and distractions are
innovative techniques:
usually avoided.
Limitations 1. Write a Question–To motivate all the students to
spend a minute or two reflecting on the lecture thus
1. It provides very little opportunity for student ­activity, far and then writing down one or two questions on
as the teacher takes special care to make the class paper.
interesting. 2. Think-Pair-Share–A sufficiently difficult question can
2. It usually does not provide opportunities to ­learners be asked in the class. Then ask students to pair. The stu-
to solve problems. dents can be asked to share their perspectives with the
3. It offers limited opportunities for checking learning whole class.
progress, whether the students are attending and
3. Brainstorming–The students should be motivated
understanding all that the teacher is explaining.
to share any new idea on (new) topic. This can be
A phrase or expression in which the same thing is said
recorded on the board.
twice in different words is called ‘tautology’.
4. The interests, abilities and intelligence of students are 4. Classroom Response Systems (Clickers)–These
not taken care of. techniques allow instructors to collect and analyze
5. It does not allow individual pace of learning. student responses to multiple-choice (and sometimes
free-response) questions during class.
Suggestions to make lecture method more effective: 5. Backchannel–This refers to the student-to-student
1. Avoiding tendency to read out material from lecture and student-to-instructor conversations that can
notes and that too word by word. occur during lectures and presentations.
2. To maintain eye contact with the students to seek 6. Just-in-Time Teaching (JiTT)–It’s not uncommon to
their continuous attention. expect students to have “done the reading” in smaller
3. There should be good lesson planning with introductory seminar that lays the foundation for in class discus-
remarks, main headings, sub-headings, figures and sions. It is basically a teaching and learning strategy
important data and concluding remarks. The students designed to promote the use of class time for more
should get the opportunity to make notes. active learning. This was developed by Educationist
4. The simple language should be used so that students Gregor Novak in 1999.
are able to understand. The main points should be 7. Team-Based Learning (TBL)–This well-developed
repeated in alternative language. teaching method is similar to JiTT in that it involves
5. To make effective use of audio-visual instructional leveraging pre-class student assignments.
facilities to improve the communication of ideas.
6. To make appropriate use of illustration and examples. Team Teaching Method
There is a need to ensure fair presentation of different This is an innovative approach in teaching large groups
views and theories. in which two or more teachers are involved in planning,
7. There should be provision of short breaks during executing and evaluating the learning experiences for a
the lecture period for asking thought-provoking, group of students.

M01_MADAN 07_65901_C01.indd 19 27/12/22 8:15 PM


1.20 Chapter 1

INSTRUCTIONS / PLANNING ARE EQUALLY Dialogic Teaching and Direct Teaching


SHARED BETWEEN TECAHERS Dialogic teaching was developed by Robin Alexander in
early 2000s. It harnesses the ‘power of talk’ to stimulate
and extend students’ thinking and advance their learning
and understanding. It helps the teacher more precisely to
Teachers
diagnose students’ needs, frame their learning tasks and
Must Trust assess their progress.
Each Other Direct teaching is the use of straightforward, explicit
teaching techniques, usually to teach a specific skill. In
teacher-directed method, the teacher stands in front of a
classroom and presents the information. It is mostly used
for the languages where there is no need to make use of
native language.
Here, a teacher gets focused on explanations, reviews
Both teachers are Equally respected and feedback.
Students Feel a Greater presence when both teachers
work together Mastery Learning Model (MLM): This is basically an
educational strategy that was first introduced by educator
figure 1.9 Team Teaching Method Benjamin Bloom in 1968. This strategy uses multi-tiered
lessons, multiple assessments, and instructor feedback to
help students master learning material. This strategy main-
Advantages tains that students are capable of learning if given the time
1. Sharing the best faculty by more students and the right combination of tools to meet their particular
2. Optimum use of multiple teaching techniques and learning needs. MLM is also called as the Competency-based
devices learning or Learning for Mastery (LFM).
3. Improvement of teaching quality

Limitations Stopover
1. Finding teachers with special competencies is a diffi- Which of the following should be termed as the prin-
cult task. ciples of dialogic teaching?
2. More teachers are required for this method. (i) Collective
3. Not useful for teaching all subjects. (ii) Reciprocal
4. Requires much time for planning and scheduling. (iii)Supportive
(iv) Cumulative
(v) Purposeful
TV or Video Presentation Codes:
Television or video presentation is an improved pres- (a) (i),(ii), (iv) and (v)
entation of radio or audio presentation, and it can (b) (ii),(iii), (iv) and (v)
virtually bring the whole world inside the classroom. (c) (i),(ii), (v) and (vi)
Screening of video presentation is followed by a discus- (d) All of the above
sion or task. The correct option is (d).
Collective means being together, reciprocal means
Advantages listening to each other, supportive means free expres-
1. Many important personalities and experts are brought sion of ideas, cumulative means chaining of ideas,
to the classroom through video presentation. purposeful means being objective.
2. Specifically useful for adult learners.
3. Illustrated lectures and demonstrations can be sup-
plemented by other teaching instructional facilities, MixeD group t eaching MethoDs
such as slides, models, specimens, etc. This may be primarily meant for 3 to 12 students. Some
4. Easily accessible for learners in remote areas. of these methods are group discussion, seminar, project
5. Specifically useful for subjects such as geography, work, tutorials, role playing, etc. These are briefly
astronomy, etc. discussed below.

Limitations Group Discussion (GD)


1. Less possibility for two-way communication. This may be counted under small group teaching meth-
2. It can be difficult to adjust complicated schedules for ods. It is one of the oldest methods used by Greek scholars
the period of the telecast. and scholars from Nalanda University.

M01_MADAN 07_65901_C01.indd 20 27/12/22 8:15 PM


Teaching Aptitude 1.21

Group Discussion method considers various facts 5. Panel Discussion: The panel discussion method of
about the topic under discussion. It tries to understand teaching is a method in which four to six or eight per-
the relationships between facts and draws out conclu- sons or students discuss assigned topic/issue/prob-
sions of the facts. This democratic method stimulates stu- lem creatively among themselves in front of an audi-
dents’ thinking process to analyses and integrate facts and ence which may be too large.
help in developing their abilities in presentation of their
ideas and facts clearly and fluently. This is useful in the
following situations.
1. For teaching a particular subject, and for supplement-
ing a lecture.
2. It is important in connection with an observation visit
or case presentation, and for sharing information.
It helps in analysis and initiation of facts, ideas, and
concepts.
3. It is a stimulating thinking process pooling of knowl-
edge, ideas, and feelings of several persons.
4. It may rationalize the facts and thus promotes intel-
ligent learning.
5. Learning together, sharing responsibilities and inter-
ests develops a team spirit.
6. It may help in discovering talented students.
It helps in stimulating thinking process, pooling of knowl-
edge, ideas and feelings of a group, rationalization of Figure 1.10 Panel Discussion
facts and developing hidden talents.
6. Conference: Conference as a method of teaching is
some what similar to class discussion but the number
Forms of Discussion of students is more limited. Usually the teacher assigns
The types of discussion can be classroom discussion, for- large class into small conference groups. A formal
mal group discussion, discussion in terms of a sympo- lecture may be conducted on the problem/issue so
sium, panel, seminar and conference. that participants get opportunities for discussion. The
method is also used before and after an observation
1. Classroom Discussion: This is basically an informal visit - pre-visit conference and post-visit conference
method of discussion. This can be undertaken when respectively. The lecturer may involve other, teachers
class size is not too large. A topic may be selected and to lead the conference groups. He needs to explain
the whole class participates in that. The main points the objectives of the conference to all the teachers
may be written on the black board by the teacher or the involved and function as a team.
leader of the group. The group discussion may be planned, partly planned
2. Formal Group Discussion: The basic process will and unplanned.
remain the same. The groups may be of 3-5 stu-
dents. At the end of smaller group discussion, they Advantages
combine into big group again at the end of allotted 1. This method gives more independence, which leads
time of 30-40 minutes. All points may be discussed to the development of presentation skills of the
again. participants.
3. Symposium: The symposium is a form of discus- 2. It provides opportunity for the trainees to prepare and
sion in which a systematic presentation of papers or contribute to a particular topic thoroughly.
speeches concerning a problem or various aspects of 3. It provides opportunity to the trainees towards
a problem or topic are done by two or more expert practical group leadership and allows them to use
persons to an audience under the direction of a chair- ­analytical skills, research on conclusions, solve a
man followed by general discussion. The debating problem, etc.
points are avoided here. It is systematic, well planned.
Symposium method is more formal than a panel dis- Limitations of Discussion Method
cussion and the participation of the audience may not
1. It may require more time and efforts of both teachers
always play a great part.
and students. The participants may not come directly
4. Seminar: Seminar as a method of group discussion
to facts.
is similar in many ways than symposium. It is usually
2. It may create emotional stress and unpleasant feelings.
used with students in the schools, colleges and uni-
3. Only a few students may dominate in discussion.
versities. It gives training in self learning, promotes
4. Discussion may go off the track.
independent thinking, and team spirit and co-opera-
tive attitude. Thus it is not suitable for all the topics.

M01_MADAN 07_65901_C01.indd 21 27/12/22 8:15 PM


1.22 Chapter 1

Brainstorming Advantages
Brainstorming is a creative group work in which the 1. It has the semblance of real-life situations.
group members produce a large number of ideas quickly 2. Interactive and interesting, it entails the participation
on a given topic or problem for subsequent evaluation. of every member of the group.
In this method, anyone can exchange remarks with 3. It gives immediate feedback.
anyone except that the participants are not allowed to 4. It develops social, decision-making, problem-­solving,
criticize the ideas at the time when views are invited. negotiating and manipulating skills.
Sometimes the quantity of ideas is more important than 5. It is effective to change the attitude of the participants.
quality. Spontaneity is the hallmark of brainstorming
sessions. Limitations
Several rounds of brainstorming are conducted till all 1. It has unpredictable outcomes.
the ideas are exhausted. Participants are then asked to 2. Real-life situations are usually more complex.
evaluate all ideas and list the best one. 3. It requires a considerable amount of resources.

Advantages Simulation
1. It encourages creativity that helps trainers to p
­ roduce, Simulation means creating conditions that are quite
think and explore ideas. similar to actual conditions and then training is provided
2. Scope for larger participation. under those conditions. For example, the training of pilots
3. It is economical as it does not require much and astronauts takes place in conditions that are quite
preparation. similar to actual flight conditions. Simulation is specifi-
cally used for training purpose.
Limitations
1. It is not a very systematic way of studying a subject. Advantages
2. There is a possibility of some trainees being reluctant 1. They are economical in the long run.
to participate. 2. Safety aspects are taken care of.

Project Method Limitation


This can be described as both a small group and an 1. It entails high initial investment in machinery, equip-
individualized instruction. In this method, students are ment, etc.
allowed to explore and experience their environment
through their senses and direct their own learning by Tutorials
their individual interests. Very little is taught from the The tutorial method is employed for teaching small
textbooks and the emphasis is on experiential learning groups for developing skills for solving numerical prob-
rather than rote learning and memorization. A project lems, providing individual guidance and sorting personal
method classroom focuses on democracy and collabora- problems. It is appropriate for taking care of individual
tion to solve purposeful problems. differences and guiding the students as per their needs,
mastery, learning, comprehending concepts, principles
Advantages and their applications and for remedial exercises.
1. Students are likely to develop the habit of critical
thinking. Advantages
2. They develop the habit of working in teams. 1. Focused attention
2. Generates more ideas
Limitations 3. Better control over pace of teaching and learning
1. Continuous monitoring may be required. process.
2. Additional resources may be required.
Limitations
Role Playing 1. It is difficult to find a suitable pace if the trainees vary
greatly in ability.
Role playing has been used effectively by many teachers 2. It can be time consuming.
to help solve classroom interpersonal problems and to
teach human-relations skills in the classroom. Role play- Demonstration Method
ing has also been used to facilitate subject matter learning
This method is based on the principle of learning by doing
through the dramatization of literary and historical works and learning from concrete to abstract. The term demon-
and historical or current events. In all these uses, role stration means to show. It is adopted in the classroom for
playing provides the student with a dramatic confronta- the achievement of cognitive, affective and psychomotor
tion and clarification. objectives.

M01_MADAN 07_65901_C01.indd 22 27/12/22 8:15 PM


Teaching Aptitude 1.23

do not merely copy from the books. The assignments


should be open-ended and should promote creativity
among the students.
Advantages
1. It helps students work independently.
2. It helps sharpen the student’s comprehension, analytical
and problem-solving abilities.
3. It helps in the inculcation of creativity among students.

Limitation
1. Students may copy each other’s material unless the
assignments are carefully planned.

Case Study
Case study approach is an innovative student centred
approach where students gain hands-on experience of the
real world. It is important to bring real-world problems
into a classroom or a workshop.
A case is usually a ‘description of an actual situation,
commonly involving a decision, a challenge, an opportu-
figure 1.11 Demonstration Method nity, a problem, or an issue faced by a person or persons
in a social set-up such as an organization’. In case stud-
ies, the students must deal with situations described in
Demonstration can be defined as a combination of ver-
the case, that is basically in the role of a decision-maker
bal explanation coupled with a live display of using appa-
facing the situation. This method has been adopted in dis-
ratus for presenting important facts, ideas or processes. It
ciplines, such as psychology, management, biology, law,
may entail audio–visual explanation.
sociology, history, etc.
Advantages The cases can be short, from brief classroom discus-
sions, to long and elaborate semester-long projects. This
1. It is effective in explaining materials, objects and ensures active participation and may lead to innovative
ideas. solutions to the problems.
2. It is effective for explaining abstract concepts.
3. It is useful for achieving objectives in cognitive, psy-
Programmed Instruction (PI):
chomotor and affective domains as there is mental
and physical participation of students. PI is a general term for a highly structured system of
learning, which is based on logical sequence of self-paced,
Limitation learning steps with feedback between each step. The
1. Only few get opportunities to participate in the exper- learner gets immediate feedback after each step.
imental process.

Learner-centreD t eaching MethoDs Concept Box


The learner-centred teaching methods try to accommo-
date the differences displayed between the learners. The Brainstorming Session: Flow of Ideas
main teaching methods include assignments, case-study
approach, computer-based learning, open learning, per- Ideas
sonalized system of instruction and programmed instruc-
tions, which are discussed below. Strategies
Situation
Assignments
Problems Needs Objectives
Assignments are given to students for a number of pur-
poses, such as for acquiring additional information, • The Idea you want to develop should flow from the
surveying, application of knowledge and solving numeri- strategies you identify to achieve the objectives.
cal problems. Although the main role is of the learner, the • Objective is what you want to achieve.
teacher too has a crucial role. The teacher has to plan the • Strategy is how you propose to achieve the objective
assignments and guide the student regarding references • Ideas can be all over the place. To develop ideas
for collecting relevant information. efficiently, you need a strategy that addresses the
Assignments can be prepared on any type of topic, but identified problem.
the nature of assignment should be such that the students

M01_MADAN 07_65901_C01.indd 23 27/12/22 8:15 PM


1.24 Chapter 1

Advantages learning. For open learning, the learning packages are


to be developed making use of multimedia. Open learn-
1. There is regular feedback.
ing instruction is, however, not suitable for the rapidly
2. This ensures active participation of the learner and it changing nature of content as this involves time, exper-
can be used for any subject. tise and resources.
Limitation Advantage
1. Learner motivation may get diminished after sometime. 1. It offers flexibility to the learner.
Personalized System of Instruction (PSI) Limitations
PSI can be used for all subject matters except where the 1. It is not suitable for achieving psychomotor and affec-
students are to select the content. Learners must achieve tive learning objectives.
mastery on a series of written mastery units, assisted 2. It requires time, expertise, resources and is hence not
by teachers, proctors and enriching lectures before pro- suitable for subjects of rapidly changing nature.
ceeding to the final test. PSI consists of five basic ele-
ments as: Interactive Video
1. Mastery Learning Model (MLM) The interactive video approach to teaching can be
2. Self-pacing employed to achieve cognitive, psychomotor and affec-
3. Stress on written material tive objectives. It allows the learner to randomly access
4. Proctors any piece of information and provide immediate feed-
5. Lectures back regarding the consequences of their action.
The essence of the interactive video experience is video
It is best-suited for content that is usually conveyed simulation with a greater number of video presentations
through written material. of real images.
Advantages Advantage
1. It is based on mastery learning. 1. Interactive video approach enhances the decision-
2. It facilitates self-paced learning. making power of an individual.
Limitations Limitation
1. It is not suitable for rapidly changing course contents. 1. This method is time consuming and requires resources
2. It is not suitable for psychomotor and affective and expertise.
domains.

Computer-Assisted Learning (CAL)


CAL is concerned with the use of a computer to mediate Concept Box
the flow of information in a learning process. A computer Reflective Teaching
has the ability to process information very quickly,
accurately and to adapt and respond to the learner’s Classroom teaching depends on many factors, such as
needs, difficulties, and progress, which is much greater individual differences, class environment, teacher’s
than that of a book or video tape. and learner’s abilities and the subject matter of the les-
son. There is no standard and perfect teaching method
Advantages to follow but teachers should seek a suitable approach
1. It has more flexibility and better control in comparison to present their lessons. Reflective teaching is specifi-
to other methods. cally useful for pre-service teachers.
2. It can be effectively used for drilling and practising, A reflexive teaching approach involves the use of
simulation and modelling. Experience Based Learning (EBL) techniques, which
engage the whole person and stimulate reflection on expe-
Limitation rience, whilst opening up the learner to new experiences.
1. It is impersonal and costly. a. The steps involved in reflective teaching are ‘sugges-
tions, problems, hypothesis, reasoning and testing’.
Open Learning Reflective teaching is also seen as the attitude of
It is a flexible method of delivering the instruction, where questioning the practice of the teaching profession.
the learner has open access to learning resources of peo- b. The peer observation, written account of
ple, material, equipment and accommodation, although experiences, self-reports, auto-biographies, journal
regular class attendances are not necessary. There are no writing, collaborative diary keeping and recording
or minimal restrictions on admissions. lessons, student’s feedback, teacher’s stories are all
The face-to-face interaction between teachers and common examples.
students through tutorials should form a part of open

M01_MADAN 07_65901_C01.indd 24 27/12/22 8:15 PM


Teaching Aptitude 1.25

Heuristic Method
Stopover
This method was advocated by Prof. Armstrong. In
this method, the student has to find out the answer Below given are two columns. Column–A lists meth-

to his/her own problem by unaided efforts. Thus, the ods of teaching and Column–B lists the focus of a
child becomes a discoverer of knowledge by develop- method.
ing a spirit of inquiry. The main aim of teaching by
this method is not to provide much facts about science, Match Column–A with Column–B.
mathematics, grammar, etc., but to teach how knowl-
edge of these can be obtained. Column–A Column–B
(A) Lecture method (i) Participative procedure
Advantage
1. Self-learning approach (B) Demonstration (ii) Experience-based
method learning
Limitation
1. Not much focus on factual knowledge (C) Project method (iii) Large amount of
knowledge
Differentiated Instruction (DI) (D) Collaborative (iv) Learning together
Differentiated instruction is a dynamic, proactive method method
of teaching. It means that the teacher plans and uses a vari-
ety of ways to teach learning. It is a ­combination of whole
Select your answer from the options given below:
group, small group and individual instruction methods.
In this method, the qualitative aspects are given more (a) (A)–(i); (B)–(iii); (C)–(ii); (D)–(iv)
weightage than quantitative aspects. It uses multiple (b) (A)–(ii); (B)–(iv); (C)–(i); (D)–(iii)
approaches to accommodate multiple intelligences. It is
student centred, meaning that the lessons are engaging, (c) (A)–(iii); (B)–(i); (C)–(ii); (D)–(iv)
relevant, interesting and active. It is an organized and (d) (A)–(iii); (B)–(ii); (C)–(iv); (D)–(i)
planned method of teaching.
The correct option is (d).
Flexibility Point of View
This includes the following types.
Autocratic Style: It refers to the control of a teacher over Gagne’s Teaching – Learning
the teaching methodologies. This includes Lecture, Lesson Process
demonstration, Tutorials and Programmed instruction.
Robert Gagne (1916–2002) was an educational psy-
Permissive Style: They are mainly student-centred. The chologist who pioneered the science of instruction in the
students choose what and how to learn. They may be sup- 1940s. His book “The Conditions of Learning” first pub-
ported by the teacher. They include Question-Answer, lished in 1965, identified the mental conditions that are
Heuristics, Projects, Review, Group discussion, Role- necessary for effective learning. Gagne created a nine-
playing, Assignment, Discovery, CAI, Brain storming, step process that detailed each element required for
Open—Independent study, Leaders group or T-group, structuring of lectures that results in effective teaching.
and Sensitivity Training.
In T-group method, the participants learn about them- 1. Level 1: Gaining Attention (Reception): The fore-
selves through their interaction with each other. The feed- most task for teachers before starting teaching is to
back, problem solving, and role play techniques are used. gain the attention of students. This change in stimu-
Thus, it an experience-based learning in the small groups lus alerts the group that learning will soon take place.
of 8-14 people. This is meant over an extended period. This can be done by raising the volume of voice, ges-
The sensitivity training is linked with T- group. This turing, showing a short video.
aims at making people more aware of their own goals as 2. Level 2: Informing Learners of the Objective
well as their prejudices, and more sensitive to others and (Expectancy): Now the class is ready to learn the
to the dynamics of group interaction. This includes the new information. This should be explained to the
following steps. learners that what are the objectives and benefits of
the lecture or session.
1. Unfreezing the old values – awareness about the 3. Level 3: Stimulating Recall of Prior Learning
inadequacy of the old values. This includes coming (Retrieval): There may be a need to establishing link-
out of dilemma of old values vs new values. ing or connectivity with the previous learning.
2. Development of new values – through training etc. 4. Level 4: Presenting the Stimulus (Selective
3. Refreezing the new ones – through practicing new Perception): Present the new information to the
role and behaviour. group in an effective manner. That also means in a

M01_MADAN 07_65901_C01.indd 25 27/12/22 8:15 PM


1.26 Chapter 1

logical and easy-to-understand manner. Try to use a 4. Verbal association: The links between the items
variety of different media and styles (such as visual being connected are described as verbal in nature.
cues, verbal instruction, and active learning) to suit This association helps in the development of language
people with different learning styles. skills. The simplest example is of naming an object,
5. Level 5: Providing Learning Guidance (Semantic which involves a chain of two links.
Encoding): The team should be provided alterna- Example: Someone is named as Ishwar Chander that
tive approaches so that learners are able to learn and is association of Ishwar and Chander.
retain the information. This aspect can be illustrated 5. Discrimination learning: This means developing the
by citing examples, case studies, graphics, storytell- ability to make appropriate (different) responses to a
ing, or analogies. series of similar stimuli that differ in a systematic way.
6. Level 6: Eliciting Performance (Responding): At For example, differentiating an object by its colour,
this stage, a teacher needs to ensure the learning of shape etc.
students. This can be done by making them to demon- 6. Concept learning: This means developing an abil-
strate their knowledge gained so far. There can be role ity to make a consistent response to different stimuli
playing exercises. that form a common class or category of some sort.
7. Level 7: Providing Feedback (Reinforcement): This forms the basis of the ability to generalize, clas-
After the demonstration of their knowledge, provide sify etc. For example, we respond to stimuli in terms
them the feedback and reinforce the necessary points. of abstract features such as color, shape, position etc.,
8. Level 8: Assessing Performance (Retrieval): The instead of on the basis of their concrete physical prop-
learners should complete the test assigned to them erties such as boiling point, melting point etc.
in an independent manner, without any explicit help 7. Rule learning: This is a very high level cognitive
or coaching of the teacher. The students can be given process. It involves the ability to learn relationships
tests, short questionnaires, or even essays. between concepts which can be applied in different
9. Level 9: Enhancing Retention and Transfer situations which may not be previously encountered.
(Generalization): The learners must be in a position Thus, we relate two or more concepts. Rules are, in
to retain information by transferring their new knowl- effect, chains of concepts. The knowledge may be rep-
edge or skill to situations that are different from the resented as a hierarchy of rules.
ones that are taught on in the class. Repeated practice 8. Problem solving: This is the highest level of cog-
is the best approach. This helps in making them profi- nitive process. It involves developing the ability to
cient and desired generalizing. invent a complex rule, algorithm or procedure for
the purpose of solving one particular problem, and
Learning Principles of Robert G agne then using the method to solve other problems of a
The eight basic types have been arranged in a hierarchy. similar nature.
If arranged in a pyramid, the first four in the pyramid are 5E Model: Atkin and Karplus suggested creation
‘behaviourial’ and rest up to problem solving are ‘cognitive’. of the 5E Model. It focuses on allowing students to
understand a concept over time through a series
1. Signal learning: This is the simplest form of learn-
of established steps, or phases - Engage, Explore,
ing. This mainly includes the classical conditioning
Explain, Elaborate, and Evaluate.
that was first developed by the behavioral psycholo-
gist Pavlov. The dog linked experiment is divided into
conditioned (with food) and unconditioned (with Stopover
food). The animal (or individual) acquires a condi-
tioned response to a given signal. 1. In Gagne’s hierarchy of learning styles which type
Example: When there is a red signal (conditioned of learning immediately precedes ‘Rule learning’?
response) on the road, people stop their vehicles. (2021)
2. Stimulus-response learning: This is also called as (a) Problem solving learning
‘operand conditioning’. It was originally developed by (b) Concept learning
Skinner. This learning is all about getting a response (c) Discrimination learning
to a Stimuli.
(d) Chain learning
Example: When a teacher instructs the students to
raise their hands (stimuli), the students immediately The correct answer is (b).
raise their hands (response).
3. Chaining: This is the ability to connect two or more
previously-learned stimulus-response bonds into a
linked sequence. This is of two types. Blended Learning
(i) Motor chaining: learning a skill such as riding a bike This model has been adopted by UGC now. This term
(ii) Verbal chaining: rote learning such as saying or ‘blended learning’ originated in USA. There is no clear
a time table. These skills help in the development single definition available for it. Blended learning
of language skills. combines online learning with face-to-face learning. It is

M01_MADAN 07_65901_C01.indd 26 27/12/22 8:15 PM


Teaching Aptitude 1.27

also defined as the combination of multiple approaches everybody top level higher learner to go on fast track
to pedagogy or teaching, for example, self-paced, while the slow learner would require extra time and
collaborative and inquiry-based study. It improves assistance to meet their academic targets. Keeping this
research skills, self-learning, self-engagement, self- mind, the ABC can mainly be understood through six
driving force, better decision making and also adds to building blocks:
computer literacy. It offers a larger sense of responsibility.
The overall goal of blended learning is to provide the
most efficient and effective instruction experience by
combining delivery modalities. The different types have Any Subject Bachelor of
been mentioned below. Combinations Liberal Education

Blended face-to-face class: This is based in the classroom, Notion of Specialization Notion of Branches broken or
broken more interdisciplinary types of Degrees to change
although there are many online activities.

Blended online class: Sometimes referred to as the Merging of Regular, Academic Convert Credits
“online driver model,” this class is the inverse of the Distance, Online Bank of into Degrees
& Virtual Modes Credit & Diplomas
blended face-to-face class. The class is mostly conducted
online, but there are some required in-person activities Notion of Mode of Education Notion of Educational
broken–Flexible education
such as lectures or labs. Currency broken

The flipped classroom: This reverses the traditional class Life Long
Study in any National
structure of listening to a lecture in class and completing or International
Learning Enabler
homework activities at home. Students in flipped classes Institutions
watch a short lecture video online and come into the Notion of Time of Education Notion of Physical Campus
classroom to complete activities such as group work, broken–Fast/Slow Degrees & Locations broken
Multiple Entry Multiple Exit
projects or other exercises. The flipped classroom model
can be seen as a sub-model of the blended face-to-face or Figure 1.12 ABC Building Blocks
blended online class.

The rotation model: Here, the students in a course


rotate between various modalities, one of which is online There is another linked approach, also called as Active
learning. There are various sub-models: station rotation, Blended Collaborative (ABC).
lab rotation and individual rotation. This requires
students in a course to rotate among locations on campus The Treasure Within: Jacques Delors from France is the
(at least one of which is an online learning lab). In the main proponent of the four pillar approach. He stated
individual rotation model, a student rotates through that lifelong learning is essential for gaining self-esteem
learning modalities on a customised schedule. and the ability to take control of one’s own life.
The self-blend model: Learners using this model are
enrolled in a school but take online courses in addition
to their traditional face-to-face courses. They are not
directed by a faculty member and choose which courses
they will take online and which they will take in person.
Learning to Live Together

The blended MOOC: This is a form of flipped classroom


Learning to Know

using in-person class meetings to supplement a massive


Learning to Do

Learning to Be

open online course. In 2012, San Jose State University


piloted such a blended MOOC.
Flexible-mode courses: This course offers all instruction
in multiple modes - in person and online and students
choose how to take their course. An example of this is
San Francisco State University’s hybrid flexible (HyFlex)
model.

Academic Bank of Credit (ABC): ABC is a carefully


laid out set of building blocks that has flexibility for Figure 1.13 Four Pillars of Learning

M01_MADAN 07_65901_C01.indd 27 27/12/22 8:15 PM


1.28 Chapter 1

Offline vs Online Methods Offline Teaching Online Teaching


Methods Methods
tabLe 1.4 Differences Between Offline Classroom Less chance for motivation There are greater chances
and Online Classroom and self leaning for self learning and
motivation.
Offline Teaching Online Teaching
Methods Methods More use of traditional Makes use of innovative
styles of teaching techniques of instruction
Teacher is the central Learner is the key focus
focus of teaching learning of the teaching–learning Use of technological Use of technological
process. process instruments is less instruments is more
Learner is more passive, Learning is more active
there are less roles for with more roles in the
him. process. Duration and period of Duration and period of
study is fixed study may be flexible
Teacher is the facilitator Teacher helps in
of knowledge and subject construction of knowledge
matter.
The teaching strategy There is less focus on Massive o pen o nLine courses (Moocs)
is verbal oriented and instructions and more on Since last one decade, MOOCs have become important.
traditional. styles and methods. The learners come from diverse geographical areas, aca-
demic backgrounds, and professional pursuits.
There may be some use of Multimedia may be used India has become the second largest user of MOOCs
multimedia. in a variety of way. after USA. MOOCs courses are being run by pri-
vate houses, and also by Government with help from
There is less student More interaction with National Program me on Technology Enhanced Learning
interaction with technology (NPTEL), IITs in India. SWAYAM is also a part of it. They
technology. help in making education reach some target group,
Focus on face-to-face Face-to-face interaction achieve desired literacy level, and sharpen the skills of
interaction between between teacher and youth.
teacher and learners learners is less MOOCs are further divided into two categories – cMOOCs
and xMOOCs.

Concept Box

X
FOCUS ON
m o o c
SCALABILITY

C
MASSIVE OPEN ONLINE COURSE
What is massive? 1. Open 1. Real-time 1. Self-paced
It may be 100 or Registration interaction? 2,. Start and end dates
1,000 or 10,000 2. Open content 2. Open content 3. College credits
or 100,000 or any 3. Free of charge 4. Badges
FOCUS ON number. or not 5. Role of the instructor
COMMUNITY AND 4. Affordable? 6. Learning community
7. Formal assessments
CONNECTIONS
and feedback

M01_MADAN 07_65901_C01.indd 28 27/12/22 8:15 PM


Teaching Aptitude 1.29

1. cMOOCs are MOOCs that allow for dynamic develop- be taken as a combo of open and distributed as shown
ment of study material. That is, instead of having a below:
pre-planned set of reading materials and courseware,
the material is developed through online discussions
and collaborations among learners taking the course
across the globe.
2. xMOOCs believe in the conventional approach where MOOCs
the courses are well-structured with pre-selected Flexible
reading and reference materials. Open Learning Distributed
MOOCs were first started in 2008 by George Siemens Environments
and Stephen Downs. It was called as “Connectivism
and Connective Knowledge 2008” or simplified
as CCK08. It was created as a credit course for the
University of Manitoba.
There were effective changes in 2012 when Professors
Sebastian Thrun and Peter Norvig of Stanford University
started the online course called “Introduction to MOOCs Open and distributed learning Environment
Artificial Intelligence”. After its success, both Professors Pedagogical benefits of MOOCs are given in Table 1.5:
started Udacity, a business model for online knowledge
sharing. Coursera and EdX were very popular initially.
Table 1.5  ink between MOOC features and
L
Most MOOC courses have a schedule with start and end
Pedagogical benefits
date. There are some self-paced courses that are without
time restrictions of joining and enrolment. They are only MOOC Features Pedagogical Benefits
six percent of all the MOOCs offered.
In MOOCs, instructions are delivered in the form Online mode of delivery Efficacy of online learning
of multimedia content. The mixed mode approach,
‘blended learning’ brings together the online and the Online quizzes and Retrieval learning
face-to-face classroom components. This approach has assessments
been recommended by University Grant Commission.
Short videos and quizzes Mastery learning
This needs a multiple language platform. Government
promotes open technologies. Peer and self-assessment Enhanced learning
Now more users are accessing the courses via mobile through this assessment
devices including tablets, smart phone, than ever
before. These applications can support multiple plat- Short videos Enhanced attention
forms such as, android and iOS, allowing the learners and focus
to use the mobile devices to get enrol, access to course
content, and participate in all course activities. Online forums Peer assistance,
out-of-band learning
The following features are mostly adopted for MOOC
courses.
Process of MOOC Courses
1. Course Format–self-paced or scheduled course
1. Choose the Platform: There are multiple of plat-
2. Learning Model–online or blended. forms. A MOOC should be useful, consistent and cred-
3. Number of Courses ible. There can be following platforms.
4. Number of users already registered in any course of
platform. Provider Course Format Learning Model
5. Institutional Credits: Whether other institution
NPTEL Scheduled Online
provides credit for courses completed on the MOOC
platforms. mooKIT Scheduled Blended
6. Platform Language: What are the languages in
which the platform is provided? IITBX Scheduled Self-Pace Online
7. Mobile App: Do the platform have an App? SWAYAM Scheduled Self-Pace Blended
8. App Platform: For which platform, android and/or
iOS the app is provided. 2. Create an Account
MOOCs are driven by pedagogical techniques, knowl- 3. Decide how we want to Learn: There are three types
edge sharing and globalization. Courses are offered for of courses:
free or at very nominal cost. Many MOOC courses may (a) Audited course–learning course for our own
offered by professors at the top schools. MOOCs can also benefit

M01_MADAN 07_65901_C01.indd 29 27/12/22 8:15 PM


1.30 Chapter 1

(b) Certified course–to receive a certificate (veri- Important MOOC linked I nstitutions


fied or unverified) The following institutions are important from examina-
(c) Specialisation course–consists of series of tion point of view.
course leading to a specialization
4. Enrolling in the MOOC A. National Program me on Technology
5. Complete the course
Enhanced Learning (NPTEL)
The Learner-Centric MOOCs (LCMs) have also become 1. NPTEL was started by the Ministry of Education in
very important during recent times. 2003 that is basically a joint initiative of seven IITs and
Microcredentials: Microcredentials are a series of related the Indian Institute of Science (IISC), Bengaluru for
MOOCs that allows a person to gain a deeper understand- different streams such as computer science; electrical,
ing of a specific subject. mechanical, and ocean engineering; management;
humanities, music, etc., free of cost. The certificates
1. MicroMasters—such as edX’s Artificial Intelligence may be issued at a nominal cost. Open technology is
MicroMasters used for such courses.
2. Specializations—such as Coursera’s Deep Learning 2. The courses are powered by Google’s open-source
Specialization platform Course Builder that runs on app engine
3. Nanodegrees—such as Udacity’s Self-Driving Car and Compute Engine. The course content is offered
Engineer Nanodegree. through conventional video lectures or with PPT
slides to share the content.
Class Central is a search engine for online courses. Class
3. NPTEL is the largest single repository of technical
Central has been keeping track of SWAYAM’s progress since
courses through video format and with text meta-data
the platform was first announced back in August 2014.
form videos, text transcription and subtitling. There is
use of multiple Indian languages. With hundreds of
MOOC Platforms in I ndia new courses, they look like MOOC providers.
India is among the leading countries in terms of enrolments
in courses provided edX, Coursera, Udacity, SWAYAM B. mooKIT
, NPTEL. Thus MOOC is an eminent alternative for the 1. This open source lightweight MOOC management
learners in India to have access to higher education, and system was developed by IIT-Kanpur in 2014. It is
to improve quality of their education with online learning. used from micro to massive. It is designed to offer
Government has taken many steps to provide ‘open edu- cMOOC (connectivist MOOC). It is specially designed
cation’ for repositories, libraries, educational media files, to deal with low-bandwidth and low-computing
e-books, etc. They have been made accessible to all through power situations.
technology with participation of following institutions. 2. mooKIT provides an indicator to help in solution of
1. IGNOU’s National Digital Repository problems. It gives a visual indication of bad connec-
tion to the learner and they can use other content
2. Sakshat providing e-content delivery options that mooKIT provides. There is the
3. CBSE’s Shishya for XI-XII Standards provision of phone calls that helps learners from rural
4. Vidya Vahini integrating IT into the curriculum of areas. This feature is very helpful for learners from
rural schools rural areas, which don’t have smart phone, laptop,
internet connectivity, high bandwidth.
5. Education and Research Network (ERNET) connect-
3. It has a very powerful analytics interface that allows
ing various colleges and schools by providing network
the learners to view their course activities.
connectivity
6. EDUSAT - a satellite launched for education in India It offers four types of solutions based on the requirement:
7. Consortium for Educational Communication (CEC)– 1. mooKIT Standard: It is used to run a single course,
the use of TV even YouTube videos can be used. Example, moocon-
8. Information and Library Network Centre (INFLIBNET)- mooc.org.
autonomous Inter-University Centre for connecting 2. mooKIT Enterprise: It is suitable to run a large num-
university libraries. ber of online courses. Users are a part of a portal and
will be able to enroll in the courses they’re interested
9. e-PG Pathshala for postgraduate course – it is man-
in. Example, mooc4dev.org, and agmoocs.in.
aged by INFLIBNET of UGC. It is more of a repository
3. mooKIT Replicated: It is suitable for low bandwidth
of e-content and assessment than a MOOC.
areas, by allowing the content to be cached on local
e-PG Pathshala is an initiative of Ministry of Education servers. The serves will sync periodically for updates.
under its National Mission on Education through ICT 4. mooKIT Personal or Mobi-mooKIT: It can run on the
(NME-ICT). It is being executed by the UGC. devices with low computation and low storage capa-
e-Adhyayan (e-Books), UGC MOOCs and e-Pathya bility as of mobile devices. It does not provide forum
(Offline Access) are closely linked with e-PG Pathshala. or social networking due to the device constraint.

M01_MADAN 07_65901_C01.indd 30 27/12/22 8:15 PM


Teaching Aptitude 1.31

Concept Box

Higher School
Education Education

Flow of Flow of
resources resources

IITs, ITSc, Universities, National E-library NCERT, CBSE, NIOS,


and other stake (Containing all the media resources) SIETs, SIEs, and other
holders in Higher stake holders in School
education Education
Flow of
Login

Login
Researchers, Researchers,
Educators Teachers, resources Educators Teachers,
etc. etc.

MOOCs platform

Higher School
Education Education

Source: Ministry of Education, Government of India

C. IIT-BombayX More Discussion a bout Mooc


IIT-BombayX is a non-profit MOOC platform developed technicaL issues
by IIT Bombay using the open-source platform Open edX,
1. Coursebuilder: NPTEL courses are run on open-source
in 2014.
platform CourseBuilder that was created by Google in
The funding was done by National Mission on
2012. Anyone can offer their own courses using course
Education through Information and Communication
builder platform. Yet, the platform does not incorpo-
Technology (NMEICT), Ministry of Education. It offers
rate social networking as such. This course is built on
courses on different subjects from multiple disciplines.
Google app engine that is written in Python.
It uses blended learning. This model is named as “Blended
Learning - MOOC Model of IIT Bombay (BLMM)”. In this 2. Drupal: Drupal base code is written in PHP. It is
system prime universities from India are offering MOOC hosted on Apache Web Server as an application.
courses to Indian local college learners. For developing mooKIT, Drupal is used as open
source Content Management System (CMS) that
Institution is the core part of platform. Drupal is also used as a
Year of Behind back-end system for many websites relating to per-
Provider Launch Platform Website Link sonal blogs to corporate, political, and government
sites. It is also used for knowledge management and
NPTEL 2003 IIT Madras nptel.ac.in/ business collaboration.
These modules are written mainly in PHP and Java
mooKIT 2012 IIT Kanpur www.mookit.co/ Script.
IITBX 2014 IIT Bombay iitbombayx.in/ 3. Open EdX: IITBX is powered by open source Open
edX that was developed in 2013. EdX is founded by
SWAYAM 2016 MHRD and swayam.gov.in Harvard University and Massachusetts Institute of
Microsoft Technology (MIT).

M01_MADAN 07_65901_C01.indd 31 27/12/22 8:15 PM


1.32 Chapter 1

  It is used by educational providers to host their own by the best faculty and available free of cost. There are
MOOCs. It can also used to host smaller classes and more than 1,000 specially chosen faculty members.
training modules. The courses hosted on SWAYAM are in 4 quadrants:
  Open EdX is mostly based on python with Django
(a) Video tutorials covering a whole course (normally
as the web framework. It is a web-based platform for
of 20 hours, each lecture not exceeding 30 minutes)
creating, delivering, and analyzing online courses.
(b) E-Content (added to the learning imparted
  LMS Module, Insight and CMS or Studio Module
through the video tutorials)
are also linked with it.
(c) Self assessment (Quizzes/assignments that
4. SWAYAM: This independent and microsoft supported intersperse the course)
MOOC type programme has been discussed sepa- (d) Discussion forum for posting queries
rately in this unit.
Features of SWAYAM
Challenges for MOOC in I ndia
(a) High quality learning experience using multime-
Some of the major concerns regarding the implementa- dia on an anytime, anywhere basis.
tion of MOOCs in India have been discussed below: (b) One-stop web location for interactive ­e-content
1. Technological Infrastructure: There can be digital for all courses from school to university level.
divide issues. MOOC needs the high speed internet (c) State-of-the-art system that allows easy access,
connections. monitoring and certification.
2. Investment: There is a need to liberalise conventional (d) Peer group interaction and discussion forum to
regulations and encourage public private partnership clarify doubts
for creating MOOCs in this country. (e) Hybrid model that adds to the quality of class
3. Diversified Needs: India is a well diversified nation room teaching.
in terms of culture, languages, and rural urban divide. Thus, all this happens through using audio-video, multi
We need to minimize such ‘digital divide’ through media and State-of-the-art pedagogy/technology.
uniform policy measures to ensure better quality.
4. Adoption of MOOC among Learners: There is a need Nine National Coordinators are:
for improvement in communication skills – both in oral
1. AICTE-self-paced and international courses
and written forms – between a teacher and learners.
2. NPTEL-engineering
5. Quality: There is a need for better quality teachers and
3. UGC-Non-technical PG education
technical staff to create and deliver quality of MOOC
4. CEC-UG education
programmes.
5. NCERT
6. NIOS-school education
Stopover 7. IGNOU-out of school students
Massive Open Online Courses (MOOCs) are:
8. IIMB-Management studies
(a) flexible and open form of self-directed, online 9. NITTTR-Teacher Training programme
learning designed for mass participation
(b) flexible and open form of teacher-directed, online
learning designed for mass participation
(c) flexible and open form of selfdirected, off-line Ministry of Education - Guidlines for
learning designed for mass participation MOOC guidelines Developing online
(d) flexible and open form of teacher-directed, off- courses for
line learning designed for mass participation SWAYAM
The correct option is (a).

SWAYAM
SWAYAM is an indigenous (Made in India) IT Massive
Open Online Courses (MOOCs) Platform for provid-
University Grants All India Council for
ing best quality education that can be accessed by any-
Commission Technical Education
one, anytime and anywhere using an IT system. It was gazette: Credit gazette:
launched by the Government of India to achieve the framework for Credit framework
three cardinal principles of Education–access, equity online learning for online learning
and quality. Access means taking the best teaching learn- courses through courses through
ing even to the most disadvantaged. It seeks to bridge SWAYAM SWAYAM
the digital divide for the economically disadvantaged
students at all levels. It is taught in classrooms from
9th class till post-graduation to be accessed by anyone, Figure 1.14 SWAYAM Framework
anywhere and at any time. The sessions are developed Source: Ministry of Education

M01_MADAN 07_65901_C01.indd 32 27/12/22 8:15 PM


Teaching Aptitude 1.33

Students looking for certifications shall be registered and 4. Assist students (class 11th and 12th) prepare for com-
offered a certificate on successful completion of the course, petitive exams.
with a small fee. The assessment takes place through proc-
tored examination and the marks/grades secured in this Teaching Support System
exam could be transferred to the academic record of the stu- Teacher Support System is basically a set of tools that
dents. UGC has already issued the UGC (Credit Framework helps to improve student achievement by building newer
for online learning courses through SWAYAM) Regulation capacities in the teachers. It can be taken as kind of process
2016 advising the Universities to identify courses where as well, it influences the way decisions are made and what
credits can be transferred on to the academic record of the information is passed on. This helps us to know that how
students. teachers acquire new skills and increase student achieve-
SWAYAM platform is indigenously developed by ment in areas they are underperforming. Though they
Ministry of Education and AICTE with the help of are happening simultaneously, the movement has taken
Microsoft, and would be ultimately capable of hosting place from traditional to modern to ICT (Information and
2000 courses and 80000 hours of learning, covering communication technology)-based education. It happens
school, under-graduate, post-graduate, engineering, with the help of technology also.
law and other professional courses.
Traditional and Modern Methods
Stopover Let’s first differentiate between traditional and modern
approaches:
Which programme or course provide advance cur-

riculum and professional certification under a unified Traditional Methods Modern Methods
scheme and also meets the demands of Choice based
credit system in India? Traditional is basically Modern is a student-
(a) SWAYAM teacher-centred instruction centric approach that
(b) NPTEL that reflects educational reflects educational
(c) UGC essentialism and education progressivism.
(d) AICTE perennialism.
The correct option is (a).
Memorization of facts, The understanding of
objective information and facts, application of facts,
correct knowledge are analysis, evaluation,
Swayam Prabha paramount. innovation and critical
The SWAYAM PRABHA is a group of 34 DTH ­channels thinking is paramount
devoted to telecasting of high-quality educational pro-
grammes on 24 × 7 basis using the GSAT-15 ­satellite. Every This aims at high test This aims at learning,
day, there will be new content for at least 4 hours which scores, grades and retention, accumulation
would be repeated 5 more times in a day, allowing the stu- ultimately degrees. of valuable knowledge
dents to choose the time of their convenience. The channels Subjects are individual and and skills. Subjects
are uplinked from BISAG, Gandhinagar. The contents are independent. are integrated and
provided by NPTEL, IITs, UGC, CEC, IGNOU, NCERT and multidisciplinary.
NIOS. The INFLIBNET Centre maintains the web portal.
The students are matched The students are matched
The DTH Channels shall cover the following:
by age, ability, etc. by interest or ability for
1. Higher Education: Curriculum-based course contents each project or subject.
at post-graduate and under-graduate levels covering It can be multiage also.
diverse disciplines, such as arts, science, commerce,
performing arts, social sciences and humanities, This method has direct Practicality, discoveries,
engineering, technology, law, ­medicine, agriculture, instruction and lectures, group activities are the
etc. All courses would be certification-ready in their seatwork. Here, students main pillars. Focus is on
detailed offering through SWAYAM, the platform learn through listening and the Internet, library and
being developed for offering MOOCs courses. observation. The teacher outside experts.
relies on textbooks,
2. School Education (9-12 Levels): These are basic mod- lectures, and individual
ules for teacher’s training, teaching and learning instruc- written assignments, etc.
tional facilities for our children to help them understand
the subjects in better manner. It also helps learners to The presentation and The context learning
prepare themselves for various competitive exams to get testing methods favour integrates personal
admissions into professional degree courses. students who have prior knowledge within the
3. Curriculum-based courses that can meet the needs of exposure to the material school environment.
life long learners among Indian citizens in India and or exposure in multiple
abroad. contexts.

M01_MADAN 07_65901_C01.indd 33 27/12/22 8:15 PM


1.34 Chapter 1

ICT-Based T eaching Learning and Learner’s


COVID 19 pandemic has actually changed the situation.
During this situation, ICT has actually became the only Characteristics
way for teaching and learning. This trend should become
more permanent. Learning Defined
ICT based teaching support is an approach to facilitate It appears quite simple to define the term ‘learning’, when
and enhance learning through, and based on, both com- we all have spent our entire lives learning new things.
puter and communication technology. It refers to the use Learning is basically psychological in nature. A few defini-
of computer-based electronic technologies of the Internet, tions offered by psychologists are:
e-mail, websites and CD-ROMS to deliver, facilitate and
enhance both formal and informal learning and knowl- 1. Gates: Learning is the modification of behaviour
edge sharing from any place at any time. The communica- through experience.
tion devices can also include digital television, Personal 2. Skinner: Learning is a process of progressive behav-
Digital Assistants (PDAs) and mobile phones. iour adaptation.
ICT-based learning is also called Computer-Based 3. Crow and Crow: Learning involves the acquisition of
Training (CBT). Generally, CBT and e-learning are habits, knowledge and attitude.
treated as synonyms, but CBT is an older term, dating Learning has also been defined as permanent change in
from the 1980s. The term ICT evolved from CBT along the capacity for performance. The learning is basically
with the maturation of the internet, CDs and DVDs. It fundamental, so for this, there are behaviourist school,
includes Internet-based learning, web-based learning cognitivist school and constructivist school to deal with
and online learning. learning. These theories have been discussed in the begin-
ICT is significant in many ways. It enables flexible ning of this unit.
learning with just-in-time, effective and efficient learn-
ing. The pace is determined by the learner. Principles of Learning
ICT facilitates collaborative internet and web-based We have already discussed different theories and maxims
learning opportunities to the learners. of teaching that enhance learning. Now we will discuss
ICT supports distance learning with wide area net- some other principles of learning.
works (WAN) and by creating multimedia CD-ROMs or
websites. 1. Principle of Association: A teacher must be aware
In ICT teaching methods, there is advantage of ­having about the learner’s mind and capacity. A teacher must
hyperlinking. There are interactive parts that illustrate dif- be associated with the learner on a continuous basis,
ficult things. Here, doing some exercises is also possible; so that learning is better and innovative. The current
it allows a wider range of learning experiences, such as learning must be linked with the previous learning.
there is educational animation to online learners. It also 2. Principle of Clarity: Learning practice must be con-
imparts e-training through the asynchronous and syn- tinuously evaluated and redirected. Objectives must
chronous communication modes. Thus, it permits learn- be clear to a teacher and to the learner. Ultimately,
ers the convenience of flexibility. Learners may look at ‘seeing is believing’.
many other options to learn. 3. Principle of Self Activity: Learning is more effective
Specialised training is rendered through custom- if it engages the maximum number of senses.
ised software, which addresses the particular needs of 4. Principle of Rewards: Learning must be challenging
the clientele mostly through the synchronous mode on and satisfying as well.
dedicated broadband Internet connectivity. Equally, it 5. Principle of Practice: Learning should always result
also renders training to the learners through the generic in functional understanding of facts.
software displaying universal contents in asynchronous 6. Principle of Nurturing Environment: It can be in
mode to the learners through a shared network with congenial environment that may be physical and
limited Internet access or on the world wide web; and social environment.
enhances teaching by professional development of teach- 7. Principle of Variable Learning Ability: Every
ers through training on usage of ICT in education. World individual is unique so they need different techniques
Links enable teachers to integrate technology into teach- to learn as well. This is linked as the concept of
ing, and thus create dynamic student-centred learning individual differences. Some learners may be
environment in classrooms. The faculties can also inter- slow, some may be fast, so learning depends upon
act with their peer groups in the world and exchange communication and learning ability.
ideas and notes on the subject. 8. Principle of Multiple Exposure: Different methods of
ICT is a planned effort towards providing interactive teaching, teaching aids and lot of practice is required to
and experiential learning, flexibility in terms of time, make students learn.
place and pace, participation and accessibility, exper- 9. Principle of Learning Capacity: The rate of learn-
tise and qualitative subject matter, best resource at the ing declines at the rate of about one percent a year
learners’ doorsteps and personalised training and centres after the age of 35. The main reasons attributed for
round the trainees. this decline are physical problems, low external

M01_MADAN 07_65901_C01.indd 34 27/12/22 8:15 PM


Teaching Aptitude 1.35

motivation, habits and the impact of a particular depends on our ability to select the most appropriate
ideology. method, not the best one, looking into the nature of
10. Principle of Active Process: Learning is an individ- matter to be learned and the objectives for which it is
ual or personal choice, so some practices are required. learned.
Audio-visual aids may also help. 9. Probing: The art of asking questions is called
11. Principle of Theory and Practice: The ‘why’ and probing. In theory the perfect convergent (closed-
‘how’ of an idea are explained by theory. So a teacher ended) question would have only one answer, and
should balance theory and practice for ­better learning the perfect divergent (open-ended) question would
by the learners. have infinite answers. Questions can be factual,
12. Principle of Effective Communication: Better explanatory, analysing, hypothetical, decisional,
learning can be achieved by integrating suitable and so on.
audio-visual aids in the teaching-learning process.
‘Self Learning is also called as the conative learning’.
Learning Methods Anecdotal notes: They should be used to record the day-
Learning can take different forms. Our sensory organs to-day development of students, as well as their specific
help in it. Apart from teaching, some of the ways how chil- behaviors, especially those that are a cause for concern,
dren learn have been explained below: speech patterns, language development, social/emotional
development, peer interactions, etc.
1. Imitation: We see that small girls usually imitate
their mothers and small boys their fathers. Even
adults do imitations of others. Specifically less Features of A Motivated Classroom
­confident or less assertive people look for confident This must have the following virtues for the students.
and assertive people. With practice and passage of 1. Belongingness: This is people’s innate need to estab-
time, they also become confident or assertive. lish close relationships with others through motiva-
2. Observation: Observation is an interesting way tion and mutual respect.
to learn. In case we want to learn about the social 2. Meaningfulness: This is developing an interest in or
behaviour of our leaders or boss or teacher, we appreciation for academic content, specially in sub-
need to go to their office or habitat to observe them jects such as mathematics.
closely. 3. Competence: The need to be successful in meeting
3. Experience: Learning is based on and follows from goals and interacting with the environment through
experience. We follow many things from our experi- challenging tasks and formative feedback. This fos-
ences. That is called as experiential learning. We need ters effort and persistence in students.
to raise questions and reflect over them so that learn- 4. Accountability: This refers to the structures and rou-
ing is continuous. tines that oblige students to report, explain, or justify
4. Teaching: The teachers select a topic, subject it to their activities. It is a kind of discipline.
content and task analysis, and expose students to the 5. Autonomy: The need to behave according to one’s
topic through lectures. The students listen to the lec- interests and values in an independent manner.
ture and, as a result, they learn.
5. Instruction: Instruction is more linked with physical
settings rather than cognitive skills. Usually the system Learning Characteristics
involves demonstration with supporting explanation. Some of the important characteristics of learning are:
As a result, the learners acquire knowledge and skills.
6. Trial and Error: The amount of knowledge is expand- 1. Learning is unitary: It implies that the learner reacts
ing. This world is already so complex, there is greater as a whole person to the whole situation in a unified
amount of risk while dealing with the problems, way. It means that the learner responds intellectually,
sometimes we don’t find the suitable answer. Thus, emotionally, physically and spiritually at the same
the method of trial and error is also required to learn time. This attitude helps in the achievement of educa-
in major part of life. The learning of mathematics can tional goals.
be specially good with ‘drill and practice’. 2. Learning is a complex whole: The different dimen-
7. Reflection: The reflective person has a tendency to sions are knowledge, skills, insights, values, attitudes
consider the whole situation and deliberate upon the and habits. Three main dimensions have been dis-
alternative solutions to problems. The cognitive pow- cussed below.
ers, such as reasoning and analysis to make sense of Development of knowledge: According to cogni-
things is used. Conversely, an impulsive person tends tive psychologists such as Bruner and Ausubel,
to respond spontaneously without deliberation, each discipline has a structure consisting of key
especially in case of uncertain situations. concepts. Knowledge is organized systematically.
8. Experimentation: We need to resort to reality, thus Expository methods, such as lecture or demonstra-
experimentation is going to help us. This may be tion help in better understanding in finite (defi-
more linked with deductive method and quantitative nite) form. Logical organized contents easily fall
techniques. Our success in learning, to a great extent, into patterns–critical thinking abilities, such as

M01_MADAN 07_65901_C01.indd 35 27/12/22 8:15 PM


1.36 Chapter 1

analysis and synthesis. Patterns give meaning. We


need to take learners to the higher levels of cogni-
Unitary
tive abilities, such as analysis, synthesis, evaluation
and creating.
Individual and social
Development of skills: A skill is a learned activity
that one develops through practice and reflection.
It is the ability to perform a learned activity well Characteristic Self-active
and at will. Skill, as an ability to perform something, of learning
includes proficiency, competence, and expertise in Purposive
the activity. Skill refers to learning of psycho-motor
behaviours required in the activities such as driving
a car or swinging a tennis racket. The development of Learning Creative
skills entails the following stages:
(a) Cognitive stage: Achieved through declarative Transferable
knowledge
(b) Associative stage: Combining individual steps Affects the conduct
into larger units of the learner
(c) Automated stage: Where the whole procedure
can be accomplished without much attention. In
the last stage, the brain process shifts from reflec- Figure 1.15 Multifaceted Learning
tive to reflexive.
2. Practice: Practice is the repetition of a response.
Development of Attitudes: Attitude is a mental Practice of Stimulus-Response (S-R) is required, such
state held by an individual which affects the way that as in classical conditioning, operant conditioning and
person responds to events and organizes responses. skills learning.
Attitudes are commonly held to have three essential 3. Reinforcement: We can use reinforcement in differ-
components or dimensions: ent ways to produce different effects.
(a) Cognitive dimension: Beliefs and rationaliza- 4. Feedback is providing the knowledge that the
tions which explains the holding of the attitudes responses are correct or that they require amend-
(b) Affective dimension: Emotional aspects of atti- ments, and it also functions as reinforcement in
tude, such as likes, dislikes, feeling of distaste strengthening the responses to be learned. It increases
(c) Conative or behavioural dimension: This the learning efficiency.
involves the extent to which an individual is pre- 5. Generalization and Discrimination: Both gener-
pared to act on the attitude that they hold. alisation and discrimination (for example, among
3. Learning may be planned or unplanned. colours) are perhaps better defined as phenomena
4. Learning can be active as well as passive. rather than as conditions of learning.
5. Learning is usually individual, but it can also be col-
lectively generated in groups. Learner’s Response to Learning
6. Learning is treated both as a process and as an out- Singh and Sharma (1987) have suggested five component
come. Learning is a life long process. behaviours of skill for managing students’ responses in
7. Learning may be incremental–it may add cumula- the classroom to enhance participation:
tively to the prior learning or transformation.
8. Learning can be stimulated or triggered by any expe- 1. Prompting (when there is no answer/response):
rience, failure, success, and anything else. Prompting in the form of clues, hints, a partial answer,
9. Learning outcomes may be undesirable as well as etc. to the students. Clues given for a correct answer
desirable. are known as prompts.
10. Learning always has a moral dimension. 2. Redirection: When a student is not able to give the
11. Learning is self-active, creative and transferable. right answer to the question, then it is directed to a
number of students. Through redirection, a teacher
Thus, we can say that learning is always multifaceted. can ensure greater participation, more time to think
and initiate brainstorming for the students.
Basic Conditions of Learning 3. Seeking Further Information: Additional informa-
tion related to a question or answer can be asked to
External conditions responsible for learning are very
ensure whether a student has properly understood
important in various types of learning. A brief descrip-
the concept or not.
tion of the basic conditions of learning are:
4. Refocusing: In case the students’ answer is correct,
1. Contiguity: This basic condition is almost simultane- he can be asked to relate knowledge gained in similar
ous occurrence of the stimuli and of the responses to or new situations. This helps in better application of
them. It applies to teaching also. the acquired knowledge.

M01_MADAN 07_65901_C01.indd 36 27/12/22 8:15 PM


Teaching Aptitude 1.37

5. Increasing Critical Awareness: In case a student Family, neighborhood community, peer groups and
gives the correct answer, the teacher puts higher market-places are the prime stakeholders.
order questions in order to increase critical thinking 3. Informal Learning: incidental and individual­ized, it
or awareness among students. is not organized. In a country, one can find a number
of initiatives and programmes, which are non-formal
by nature.
Stopover In India, adult education programmes, basic edu-
Which among the following features describe the
cation programmes, lit­ eracy programmes, health
emo­ tional intelligence of learners in an optimum awareness programme, etc. are certain examples.
manner? Curricular experiences are gen­ erally formulated
(i) A better understanding of others’ and our own keeping in mind the specific needs of the target
emotions group. These are more indi­vidualised rather than
(ii) Strong expression of one’s own emotions broad-based and general in nature. The transaction
(iii) Manipulating one’s own emotions of learning experiences is centred around commu-
(iv) Developing rational thinking nity resources.
(v) Being innovative and accepting criticism
(vi) Better accepting of other people
Importance of Creating a Positive L earning
Choose your answer from the options given below:
(a) (i), (iv) and (v) E nvironment
(b) (iv), (v) and (v) Some constituents of an institution’s environment are dis-
(c) (i), (ii) and (iii) cussed as follows:
(d) (ii), (iii) and (iv)
1. Physical Environment: ‘The classroom is the basic
The correct option is (a). structural unit of our educational system.’ ­Learners
constantly interact with the physical environment of
their institutions during scheduled or unscheduled
time, consciously or unconsciously. There is strong,
Learning Environment consistent evidence of the effect of basic physical vari-
Learning is an ongoing dynamic process. It is the result of the ables (air quality, temperature, noise) on learning.
interaction between individuals and environment in which Overall, it is an important part of teaching learning
they operate. Learning environment may be defined ‘as a com- process.
posite of natural conditions, ­circumstances and influences,   Different room arrangements, chairs, tables, etc.
and sociocultural contexts in which an individual is situated’. serve different purposes. It is necessary for class-
Learning environment is the sum total of the surroundings in rooms to have some degree of flexibility as change is
which individuals interact to enrich experiences, thus leading preferred sometimes. ‘Ownership’ and ‘engagement
to learning. There are many theories at play while describing of space and equipment is important.
learning environment. 2. Psychological Environment: If building, space and
Bandura, a social psychologist, in his ‘social learning the- facilities provide physical environment to a school, its
ory’ proposed that new behaviour can be learned through philosophy and practices that provide a psychologi-
the process of observation and imitation. It is the result of cal environment. This environment mostly refers to
interactions among personal characteristics, behaviour and the stimuli that influence upon the learner’s psyche
environmental factors. Besides social environment, physical in the school. For example, imagine a situation in a
environment also influences learning. school where a learner is kept in a threatening situa-
Vygotsky, a constructivist thinker, believed that the tion. This situation will result in the development of
mastery over the culturally important tasks (such as lan- anxiety in the learner. Anxiety may lead to undesirable
guage) can take place only through social interactions. behaviours, such as escaping from tests, blaming the
Variations in learning environment lead to different teachers for faulty questions, engaging in malpractice
learning outcomes. and so on.
Based on the different types of settings, UNESCO 3. Social Environment: Through constant interactions
(1984) classified the following types of learning: with the peer group, a learner learns and practices
the social virtues of cooperation, self-sacrifice and
1. Formal Learning: loyalty. The outdoor activities offer learners opportu-
• Institutionalized to transmit knowledge. nities for developing qualities like courage, resource-
• Highly structured and there is contin­uous interac- fulness, self-reliance and initiative among students.
tion that lead to recognised qualification. Good personal relationships develop stability, trust,
• Teachers and learners are the prime stakeholders. sense of belongingness, self-respect and self-accep-
2. Non-formal Learning: It is not structured and it is tance among learners and creates a positive environ-
organised with flexibility. There is no certification. ment for learning.

M01_MADAN 07_65901_C01.indd 37 27/12/22 8:15 PM


1.38 Chapter 1

As a teacher, you are expected to fulfill at least two Select your answer from the options given below:
cognitive demands of the learners: (a) (i), (ii), (iii) and (vi)
(a) academic task demands (understanding and (b) (iii), (iv), (v) and (vi)
working with content) and (c) (i), (ii), (iii) and (vi)
(b) social task demands (interacting with others who (d) (ii), (iii), (iv) and (vi)
are concerned with that content). The correct option is (d).
4. Motivational Climate: The motivational climate
must be created in the very beginning. The stu-
dents must see the worth of the work that ‘they are inDiviDuaL Differences in Learning –
doing’ and the ‘work others do’. Value and effort are t ypes of Learners
the two main methods for creating a motivational Learners have both common points and differences at dif-
climate. ferent ages and stages of life. Teachers need to understand
(a) The duty of teacher is to demonstrate the value of both the commonalities and the differences in order to
the task to be performed in the class. This can be meet the students’ needs as no two individuals are alike.
done by telling them the value of the task in daily There are variations among learners with respect to their
life. age, cultural environment, past experiences, physical,
(b) Effort involves the time, energy and creativity a mental and emotional make up, goals, needs, etc.
student uses to develop the “work”. This can be Furthermore, different learners have different learn-
done through adequate rewards and praise. ing styles and as heterogeneity is increasing day by day,
5. Understanding Individual Differences: A teacher they may perceive, interpret and evaluate the same learn-
must understand the diversity element in the class and ing event in different ways.
create a conducive element for the same. He needs to Learners’ characteristics, therefore, merit consideration
adopt a customized approach to cater to individual when selecting the media. Learners benefit from those
needs to the extent possible. media that match their individual learning styles. Therefore,
it is understandable why a variety of methods, resources and
paths should be provided for different students to achieve a
Stopover particular objective. Thus, while designing an instructional
Below are listed some learners’ characteristics. Iden- plan, the important task for the designer is to identify the
tify those that help in effective teaching. most critical characteristics for the attainment of instruc-
(i) Learner’s respect for teacher tional objectives.
(ii) Learner’s level of mental ability
(iii) Learner’s previous experiences types of Learners
(iv) Learner’s level of interest to relation There are three main categories of learner characteristics,
(v) Learner’s level of interpersonal relation such as general characteristics, specific entry competencies
(vi) Learner’s view about the society and learning styles.
Learner Characteristics
1. On the Basis of Personal and Social Attributes:
Concept Box They help in planning instructional objectives as it
Main Steps in Learning Process may reveal physical characteristics that are relevant to
In NET Exam, there may be direct or indirect ques- training or instructional decisions. The social factors
tions on learning process. mainly include the following.
(a) Age and maturity level
1. Reception: Gaining attention by making some (b) Motivation and attitude towards the subject
abrupt changes in stimulus or stimuli. (c) Expectations and vocational aspirations
2. Expectancy: Informing learners of the objective (d) Special talents
and what they will be able to do after learning.
(e) Mechanical dexterity
3. Retrieval to working memory: Stimulating recall
of prior knowledge. (f) Ability to work under various environmental
4. Selective perception: Displaying contents with conditions.
distinct features. Some of the differences in learner characteristics
5. Semantic encoding: Learning guidance. between the adolescence and the adults has been
6. Responding: Asking learner to perform. described further.
7. Reinforcement: Providing feedback to the learner. 2. Field Independent vs. Field Dependent: ‘Field’ here
8. Retrieval and reinforcement: Additional perfor- means context or surroundings.
mance by learner and it entails feedback also. Field-independent learners tend to rely less on the
9. Generalization: More practice of varied problems teacher or other learners for support. They perceive
so as to increase retention. the things analytically. They see objects separately
from the surrounding field, they prefer to work in

M01_MADAN 07_65901_C01.indd 38 27/12/22 8:15 PM


Teaching Aptitude 1.39

self-structured situation and have self-defined goals. (iii) Deductive: From theory to individual facts
In the classroom activities, such as extensive reading   (iv) Reflectively: Introspection
and writing, which learners can carry out alone are 5. Learner Characteristics on the Basis of Listening
useful for field-independent learners. Skills: Listening is an important skill and there are
  On the other hand, field-dependent learners often four types of listening styles, which are as follows:
work well in teams as they tend to be better at inter- (a) Active Listening: It is listening with a purpose.
personal relationships. They perceive globally. They (b) Empathic Listening: It is a form of active listening in
prefer to work in existing structure or context, they which you attempt to understand the other person.
require externally defined goals and reinforcements (c) Evaluative Listening or Critical Listening: In
and are more aware of their surroundings. this type, the listener evaluates the accuracy,
  In the classroom, activities that connect different meaningfulness and utility of speaker’s message.
parts of a lesson are useful for field-dependent learn- (d) Appreciative Listening: Listening for enjoyment
ers. For example, learners can discuss what they know involves seeking situations involving relaxing, fun
about a topic, predict content or look at and listen to or emotionally stimulating information.
related material. 6. Learner Characteristics on the Basis of Thinking
3. Reflectivity and Impulsivity: When a question is Styles: There are different thinking styles of learners,
posed, some students take long time to respond while which are mentioned below.
others are quick in response. The speed with which (a) Reflective thinkers
the respondents make a response to the task and the   (i) View new information with respect to the
number of errors they make is termed as conceptual subject.
tempo. Those students who respond quickly and make   (ii) Relate new information to past experiences.
a fair number of mistakes are said to have a fast concep- (iii) Always ask ‘why?’
tual tempo. They are said to possess impulsive style of   (iv) Examine their feelings about what they are
learning. learning.
  Learners who are slow in response and tend to (b) Creative thinkers
make fewer mistakes are called reflective. In problem-   (i) Like to play with new information.
solving situations, the impulsive learner collects less   (ii) Always ask ‘why?’
data, they are less systematic and does not look for (iii) Create their own solutions and shortcuts.
alternative solutions. Reflective learner spends more (c) Practical thinkers
time collecting information and analyzing the data   (i) Always look for factual information.
before offering a response.   (ii) Seek the simplest and the most efficient
4. Classroom based Learning Styles: Learning styles way to do their work.
are traits that refer to how learners receive and pro- (iii) Not satisfied until they know how to apply
cess information. their new skills to their job or other interest.
(a) Visual learners learn easily and are better (d) Conceptual thinkers
through sight. Brightness, size, colour, distance,   (i) Accept new information only after seeing
clarity, frame and symmetry are important to vis- the big picture.
ual learners. Visual learners must see so that they   (ii) Want to know how things work, not just the
may learn easily. Visual learners may be catego- final outcome.
rized as verbalists (they see words and letters) or (iii) Learn the concepts that are presented but
imagists (they see images, i.e., pictures). also want to know the related concepts that
(b) Auditory learners acquire information through may not have been included.
sound, i.e., the ear gate. Various aspects of sound,
for example, pitch, volume, tempo, rhythm, Characteristics of Adolescent and Adult
resonance are important for auditory learners. Learners: Academic, Social, Emotional and
Auditory learners may be aural (they learn by
listening to others) or oral (they learn by talking
Cognitive
and hearing themselves).
‘Live as if you were to die tomorrow’
(c) Motor learners learn through motor activity.
‘Learn as if you were to live forever’
Various aspects of action, for example, frequency,
— Mahatama Gandhi
duration, intensity, pressure, etc., are important
for them. Motor learners may be kinesthetic
(they learn through the use of gross motor mus- Learning is a fundamental, continuous and lifelong process.
cles) or mechanical (they use fine motor muscles It’s for our successful adaptation of human beings to inter-
to support their learning). nal and external environment. It should be according to the
Apart from above, the classroom style learners physical and intellectual ability of the learner. Both teachers
can be of the following types. and learners must be aware of this fact. Here, we are first
   (i) Intuitive: Insights and hunches going to discuss concepts of adolescent and adult learning
  (ii) Inductive: From facts to generalization and thereafter compare both.

M01_MADAN 07_65901_C01.indd 39 27/12/22 8:15 PM


1.40 Chapter 1

Nowadays, children know more, learn more, and want


Adolescence Learners more. This may lead to arguments, friction and tears, all
Adolescence means ‘to emerge’ to achieve ‘identity’. It is a leading to a ‘cultural gap’ from their parents.
time for the maturing of mind and behaviours. It is not an Adults learn voluntarily. They require more time
age, but a stage. It is divided into three major stages: to practice new skills. They have many responsibili-
ties and have less time to learn. They prefer to learn by
1. Early Adolescence: (10 to 12 years) growth of hands, participation.
feet and later the limbs. There is demand for indepen- As they have their own self-esteem and ego, they evalu-
dence and privacy, so there are chances of ­conflict. ate learning in terms of results, and its utility to their life
2. Middle Adolescence: (12 to 16 years) there are fur- situations.
ther bodily and genetic developments, specifically
in girls. Social Changes
  Girls develop personal skills quicker, and loyalty
and commitment matter more. The decisions of voca- Margaret Mead highlighted the role of cultural factors
tions and education are made. The physical effect of in the development of personality of the adolescent.
pubertal development becomes incorporated into the Mead observed “storm and stress” as a serene and grad-
self-image. ual transition from childhood to adulthood and an easy
3. Late Adolescence: (16 to19 years) It is transforma- acceptance of adult roles. It is relatively stress- free in a
tion towards the phase of adulthood. In late adoles- society.
cence, career decisions are finally traced. The child Harold W. Bernard also subscribes it as a cultural phe-
gradually returns to the family on a new footing. nomenon. The cultural aspect of adolescence states that
two main aspects:
WHO defines adolescence in terms of age, spanning
the ages between 10 and 19 years. Hall describes ado- 1. A rapidly widening life
lescence as a “storm and stress” period that reflects the 2. An increasing overlapping between the roles of the
unsettling growth period in modern societies. This con- child and adult.
cept was recognized by Margaret Mead also. Adolescents may feel bad while facing conflicts, values,
Academically, adolescence is the time spent in high emotional tension and extreme attitudes.
schools and early colleges. With industrialization, urbanization and individualiza-
Psychologically, it is a period of transition, during tion, the incidents of juvenile delinquency also increase.
which cognitive, physical, personality and social changes With fast growth and structural bodily changes, new
occur. attitude towards oneself and others, rising awareness of
Sociologically, it is a period that fills the gap between one’s rights and duties, adolescence is a transitional period.
dependent childhoods to self-sufficient adulthood. From Adolescence is affected by the following factors.
a medical point of view, adolescence begins with hor-
monal changes and the growth of the body. 1. There is search for self-concept or self-identity.
In India, the adolescent is dependent on his parents for 2. There is a demand for more independence to make
many more years in comparison to the Western nations. decisions.
The emotional dependence is also termed as ‘Delayed 3. They think more about right values and wrong values.
Adolescence’ that can go upto 21 years and even up to 4. Peer pressure also increases.
25 years. 5. They communicate in different ways–through inter-
net, cell phones and social media.
Academic Achievements The process of ‘socialisation’ is also affected during this
Adolescents spend more waking time in school. Academic stage.
achievement during this stage is predicted by interper- During ‘Homophily’, an adolescent spends more time
sonal (parental engagement), intrapersonal (intrinsic with friends. The peer groups evolve from primarily sin-
motivation), and institutional factors. It can set the stage gle-sex to mixed-sex.
for future career opportunities. Sports, games, arts and The ‘deviant peer contagion’, under which peers rein-
crafts also play some role. Parents put greater efforts dur- force problem behaviour by laughing or showing other
ing this stage. signs of approval, increase the likelihood of future prob-
Malcom Knowles has identified the following charac- lematic behaviour. Negative peer pressure leads to vices
teristics of adult learners. and crime. Friends may provide support mechanism.
Crowds refer to different groups of people, such as
1. More autonomous and self-directed
‘­theater kids’ or ‘environmentalists’. Friendships are recip-
2. Goal-oriented and practical rocal dyadic relationships. Cliques refer to frequently
3. Relevance-oriented and see a reason for learning interacting groups of individuals. They enjoy ‘shared rep-
something. utations’ than actual interactions, such as when the whole
4. Adults must be shown respect. The adult trainers group is famous or notorious for an activity.
must acknowledge the wealth of experiences that Romantic relationships are usually short-lived rather
adult participants bring to the classroom. than long-term commitments during this stage.

M01_MADAN 07_65901_C01.indd 40 27/12/22 8:15 PM


Teaching Aptitude 1.41

Self-efficacy: It is a person’s belief in their ability to suc- Mayer and Salovey (1997) proposed that EI was a
ceed in a particular situation. This reflects confidence in cognitive ability which is separate but also associated to,
the ability to exert control over one’s own motivation, general intelligence. Their model consists of four different
behavior, and social environment. Psychologist Albert abilities:
Bandura described these beliefs as determinants of how
1. Perception of emotion: in yourself and others
people think, behave, and feel.
2. Emotional facilitation: the ability of emotions to
Emotional Changes help thinking
3. Understanding emotions: to judge the emotional
Research indicates that emotions cannot be separated
content of several stimuli such as faces, designs and
from the intellect (learning). If the learner is stressed,
colours.
over anxious, he/she will not be able to learn. Role confu-
4. Management of emotions
sion is an indicator of not successfully meeting the task of
adolescence. Mayer and Salovey offered us a test, called as Mayer
Adolescents face problems of morality and being very Salovey Emotional Intelligence Test (MSCEIT).
ambitious. They favour freedom and democratic life. They The inability to recognize or describe one’s own
like permissive atmosphere so that parents and teachers emotions is called as ‘alexithymia’.
to be lenient towards them. They tend to be rebellious by Baron’s (2006) Model of Emotional Social Intelligence
nature. (ESI) consists of intrapersonal (emotional awareness,
‘Conscience formation’ takes place during this stage. assertiveness, independence, self regard, self actual-
Adolescents possess a self-owned yearning for religion, ization); interpersonal (empathy, social responsibility,
God, worship, prayer and spiritual values. interpersonal relationships), stress management (stress
Hall says that the major physical changes during this tolerance, impulse control); and general mood (happi-
phase cause major psychological changes. ness and optimism).
Adolescent years are more important for the forma-
tion of personality. Anne believed that the libido, which Multiple intelligences: This tries to differentiate human
is quieted during the latency years, reawakens in adoles- intelligence into specific ‘modalities’, rather than focus
cence and threatens to upset the delicate balance of ego on a single general ability. This concept was described by
and id. Howard Gardner in his 1983 book ‘Frames of Mind’: The
According to Erik Erickson, adolescence resolves the Theory of Multiple Intelligences. According to the theory,
conflict of identity vis-à-vis identity confusion. an intelligence ‘modality’ must fulfill eight established cri-
Early puberty and cognitive changes come with worse teria which are Linguistic, Logical/Mathematical, Spatial,
outcomes for girls than boys. It impacts decision making Bodily-Kinesthetic, Musical, Interpersonal, Intrapersonal,
controls also. and Naturalist.
The emotional changes with the unique combination
of genes, brain, environment, experiences, and culture Cognitive Development
shape development. There is more self-consciousness We have discussed its definition earlier as well. This
about physical appearance and changes. It is basically refers to intelligence, thinking and imagination about
an “invincible” stage of thinking and acting. our environment. Our innate power (heredity) also plays
The egoistic needs are in the form of dominance, an important role in it.
achievement, retention, attention, autonomy, acquisi- Piaget mentioned the following stages for cognitive
tion, cognizance and destruction. Moffitt regards ado- development:
lescent-limited antisocial behaviour as resulting from a
“maturity gap”. 1. Sensory period (0–2 years)
The genetic changes to environmental factors are 2. Pre-operational period (2–7 years)
called the differential susceptibility model. These varia- 3. Concrete operation period (7–11 years)
tions are considered riskier than ­others. Individual differ- 4. Formal operation period (11–15 years)
ences play an important role in adolescent development. The main characteristics displayed here are:
The ‘unholy triad’ sums up these as substance abuse,
violence and early sexual experimentation. 1. Systematic analysis of a problem
2. Logical approach towards the solution of problem – to
Emotional Intelligence: The concept of ‘emotional intel- move away from rote learning.
ligence’ was introduced by Daniel Goleman in 1995. EI is 3. Ability to use higher order structure to solve a problem
a set of skills for the purposes of 4. Systematic analysis of a problem
5. Moral maturity
1. The accurate appraisal and expression of emotions in
oneself and in others It has been observed that the type of language used in
2. The effective regulation of emotion in self and others, uneducated homes is mostly of commands, whereas in
and educated homes, it is mostly of explanations. Individuals
3. The use of feelings to motivate, plan, and achieve in struggle through environmental changes. Through this
one’s life. process of adaptation, cognitive development takes place.

M01_MADAN 07_65901_C01.indd 41 27/12/22 8:15 PM


1.42 Chapter 1

The purpose of this process of adjustment is to bring about Autonomous


a ‘state of equilibrium’ in the life of individuals. and self directed
At adolescence stage, social interaction plays a very Lift experience
significant role in learning. Readymade solutions of prob- Self respect
and knowledge
lems should be discouraged. The teachers should provide
Characteristics
such type of education that helps to form minds which
of learning
can be critical, can verify and not accept everything that
is offered. Practical Goal oriented
At the stage of formal operation, the child displays
three new qualities: Relevance
oriented
1. Systematic analysis (with all possible solutions) of the
Figure 1.17  haracteristics of Learning by an
C
problem
Adult Learner
2. Logical approach and
3. Ability to use higher order structure.
There is no knowledge development without relating
objects within the environment. An individual acquires
knowledge not by passively copying objects in the envi- Leadership
ronment but by acting upon it.

Adult L earners Principles of


Novel styles Experience
Life is busy, and adults are seldom easy. They are actively adult learning
engaged in the process of life. To take time out of this busy
process, adults may ask:
“How will this benefit me? What makes learning this
Respect Appeal
worth the effort?”
Figure 1.18 Principles of Adult Learning
Mature

Prefer to be self-directed
Important Characteristics of Adulthood
Adult learner
Adults are not just grown up children. Adults learn differ-
Independent
ently from children.
With the maturity, the self-concept of a person moves
Experienced from being a dependent personality towards one of being
a self-directed person. Adulthood is the stage where this
transition occurs.
Figure 1.16 Adult Learner as per Malcolm S. Knowles Adults are experiential learners. The person accumu-
lates a growing wealth of experience that is used to make
Adult education is based on a philosophy called ‘­andragogy’, sense of the environment. Adults may know more than
which is the art and science of helping adults learn. The the teacher.
guiding principles of adult learning aim at ­bringing: With maturity, the readiness to learn becomes ori-
1. Changes in what people know ented to the development tasks of social roles, but the
2. Changes in what can do content must be relevant and legitimate. Life application
3. Changes in what people think is critical.
4. Changes in what people actually do. With maturity, the time perspective moves from one of
postponed application to one of immediate application.
In other words, it must emphasize change in knowledge, The shift is from subject-centeredness to problem-cen-
attitude and skills of the learner. teredness. Adults enjoy solving problems.
Adulthood is mostly defined on the basis of age or Here, the motivation to learn is increasingly internal.
cognitive maturity. While in India, adulthood is defined Personality responsibility is significant. Adult learners
between 15 to 35 years, UNESCO and Organisation want to meet the requirements of their lives. They want
for Economic Cooperation and Development defines it to be successful. In India, an adult learner is a person who
between 24 to 65 years. has had no opportunity of formal education in their early
Adult learning process may be systematic learning pro- years of life.
cess, be it formal or non formal or informal, it may be self- Once we have gone through both adolescence and
mentored or corporate-sponsored, may be undertaken as adult learners, we need to focus on difference between
fulltime or a part time learner. the two:

M01_MADAN 07_65901_C01.indd 42 27/12/22 8:15 PM


Teaching Aptitude 1.43

Table 1.6 Differences Between the Adolescence and the Adults


Elements Adolescent Learners Adult Learners
Learner Dependence on teacher for learning The learner is self motivated and self directed
Role of The learner has little experience that is to be The learners are rich in experience. The adults
learner’s built upon, with the help of instructor. are a rich resource for one another also. The
experience experience becomes the source of self identify.
Orientation to Learning is a process of acquiring the Learning must have relevance to real-life tasks
learning prescribed subject matter. The contents that is organized around life/work situations
are sequenced according to the logic of the rather than subject matter units.
subject.
Motivation for  rimarily motivated by external rewards and
P  otivation happens by internal incentives:
M
learning punishment, competition for grades, and the recognition, better quality of life, self confidence,
consequences of failure self actualization. Learning is required in order to do
good in one or some aspect/s of life.
Demand for  earner must balance life responsibilities
L  earner can devote more time to the demands of
L
learning with the demands of learning learning because responsibilities are minimal
Permanence of  earning is self initiated and tends to last a
L  earning is compulsory and tends to disappear
L
learning long time shortly after instruction.
Climate • Tense, low trust • Relaxed, trusting
• Formal, cold, aloof • Mutually respectful
• Authority-oriented • Informal, warm
• Competitive, judgmental • Collaborative, supportive
Diagnosis of Primarily by teaching system and teacher Mutual assessment by learner and facilitator
needs
Planning of Primarily by teaching system and teacher Mutually by learners and facilitator
curriculum
Objectives of Objectives are set by the teaching system, Objectives are set by mutual negotiations; they are
learning they are predetermined and inflexible. predetermined and inflexible.
Designing • Teachers’ content plans • Learning contracts
learning plans • Fixed course syllabus • Learning projects
• Logical sequence • Sequenced by readiness
Learning  assive teaching methods like transmitted
P Active training methods are used
activities techniques, assigned readings, etc., are used
Pace of learning Teachers control timing and pace of learning Learners influence timing and pace of learning
Evaluation Evaluation is done by teachers, in Evaluation is done by learner-collected evidence
‘norm-referenced tests’ with grades validated by peers, facilitators, and experts. The
‘criterion-referenced’ test is used.

M01_MADAN 07_65901_C01.indd 43 27/12/22 8:15 PM


1.44 Chapter 1

(e) follow cooperative learning


Stopover (f) adopt interactive approach
As per the cognitive development theory of Jean

Piaget, adolescent students are in which stage of At different stages of teaching, with added experience, a
development? teacher goes on enriching a higher style of teaching. This
(a) Sensory motor stage is indicative of the teacher’s growth in higher professional
(b) Concrete operational stage pursuits.
(c) Operational stage Metacore skills and Polycrest skills: A teacher needs to
(d) Formal operational stage focus on clarity, variety, task orientation, engagement in
The correct option is (b). learning task in classroom environment. A teacher, like a
doctor, pilot, engineer or a counselor, is supposed to pos-
sess a repertoire of teaching skills so that he can perform
Teaching and Learning Factors his teaching well. These ‘metacore skills’ help during pro-
fessional coaching. The greater variety of sub-skills are
According to new NTA-NET Exam pattern, teaching and
called ‘Polycrest skills’.
learning are important for effective teaching
Learning F actors
Teaching Process Learning In effective learning, we mostly notice these factors:
1. More intensity of learning
2. More retention
Spirit of New 3. Joyful learning
Realization 4. More scope of cognitive development
joy and experience
of objectives 5. Self-directed learning
creativity (Learning)
6. Self-motivation for further learning
Figure 1.19 Effective Teaching Keeping the above in consideration, the following f­ actors
become automatically important:
Teaching F actors 1. Background of the learner (repertoire)
A teacher should have the following qualities: 2. Nature of learning material (easy or difficult)
3. Environmental factors (space, physical condition and
1. Personal qualities like warmth, affection, s­ ympathy, psychological support)
democracy, optimism, dynamism, etc. 4. Motivation (intrinsic and extrinsic)
2. Professional competencies like command over the 5. Learning support can be human (teachers and par-
subject matter, effective communication, proper use ents) or electronic (audio, video and computer).
of teaching instructional facilities, classroom man-
agement and evaluating students learning also. Certain qualities and traits of learners, such as their
level of intelligence, their attitudes, their motivation,
From the training point of view, we classify teaching skills their learning styles, aptitudes, their readiness to take
into three broad categories: risks, etc. can impact the way they learn. Many factors
1. Core teaching skills: common for all subjects. have been defined under the learning topic itself.
2. Specific teaching skills: for specific subject areas
such as language, social science, science, maths, etc. Teacher Support Material
3. Target group specific skills: for exceptional children The support materials aim to support teachers and stu-
dents in achieving the learning outcomes of any subject.
Some factors as practised by teachers in classroom are as The ideas and resources are neither prescriptive nor
follows: exhaustive. Teachers and students can discover many
1. A teacher with limited exposure and experience in other ways of reaching the learning outcomes. They can
teaching is prone to relate to any book, practical sessions, some specific activi-
(a) follow textbook reading by the students ties etc. We can take the example from a book here:
(b) transmit information through lecture 1. Teacher solutions manual: They are designed to
(c) dictate notes assist teachers in effective teaching, such as sugges-
(d) impart required information tions on how to teach a topic. They are kind of com-
2. A teacher with professional training and reflective prehensive supplementary resources like an end-to-
thinking is tempted to end solution.
(a) adopt new ways to teach 2. Lecture slides: While teachers develop their own
(b) involve students in teaching instructional facilities to deliver a lecture in the class,
(c) generate new ideas through problem-solving the lecture slides provide a firm base for instructors to
(d) teach through group activities build on.

M01_MADAN 07_65901_C01.indd 44 27/12/22 8:15 PM


Teaching Aptitude 1.45

3. Extra practice questions: Students always demand 5. Better Organization of Classroom Teaching: The
extra practice. The texts are bundled with extra exercise planned sequential organization of things is always
questions, case studies, and other such materials used better for better delivery of contents, better learn-
by teachers to frame homework, quizzes and tests. ing and better retention. This helps to overcome
shortcomings in verbal or visual communication.
Learning Environment and Institutions 6. To Facilitate Holistic Learning: There are varied
Learning environment means developing an ecosystem learning objectives in cogni­tive, affective and psycho-
for the learning that itself is linked with the teaching and motor domains. There is need to cater to individual
education. Education seeks its origin from philosophy. differences as well.
These are linked with democracy, autocracy, laissez faire 7. Promotion of Scientific Temper: Teaching instruc-
and then with political, cultural, legal environments. On tional facilities promotes scientific temper, which is
internal levels teachers, teaching methods, learning prin- one of the main goals of education.
ciples, evaluation process play an important part. The 8. Practical Applications: These instructional facilities
institutional building is also crucial for creation of direct show application of theoretical knowledge into prac-
learning environment. As social, political and industrial tical applications.
environment is dynamic, thus learning environment has 9. Making Learning Fun: Learners enjoy the novelty of
to be dynamic. handling new objects and learn new concepts through
them.
I nstructional F acilities 10. Concept Formation: Teaching instructional facilities
facilitate the formation and attainment of concepts
‘I hear and I forget, I see and I believe, I do and among ­children. They concretize the abstract con-
I understand’ cepts. Thus, children are able to understand them and
 —Confucius not resort to rote learning.
Hersey-Blanchard Model of Leadership: There have
The instructions are based on ‘give’ and ‘take’ relation- been some questions in NET-JRF Exam that are based on
ships in the teaching-learning process. The instructional this model. It focuses on the ability and willingness of indi-
facilities that assist an instructor in the teaching–learn- vidual employees. It’s also referred to as the Situational
ing process. They supplement teaching methods and are Leadership Model to deal with the members or learners.
themselves not as self-supporting as teaching methods. There are following D and S categories.
The teaching instructional facilities include audio-
visual instructional facilities. They follow the assumption Nature of learner (or member):
that learning originates from senses’ experience. They D1 - Unable and unwilling
help in better learning, retention and recall, thinking and
D2 - Unable willing
reasoning, activity, interest, imagination, better assimila-
tion and personal growth and development. D3 - Able unwilling
D4 - Able willing
The Main Benefits of Instructional Facilities Approaches to be adopted:
Instructional facilities are also known as teaching learn-
ing materials TLMs. This association makes teaching more S1 - Directing
effective, and also learners have better retention. S2 - Coaching
They are used to make the teaching-learning process S3 - Supporting
effective. They also help learners achieve the learning S4 - Delegating
outcomes after classroom teaching and learning. Some
of the reasons to use teaching instructional facilities in For example, if a candidate is able and unwilling, he must
classroom are of various types as described below. be supported. Able and willing candidate must be delegated
some task to show the results.
1. Motivation of Learners: The main objective is to
capture the attention of learners through ‘variety of
stimuli’. Stopover
2. Based on Maxims of Teaching: The use of teaching The teaching strategy that is best suited to deal with

instructional facilities is based on maxims of teaching, students who in terms of performance readiness level
thus it is a systematic exercise. are in the category of ‘able but unwilling’, will be,
3. Better Retention of Information: The more is the (a) Assigning challenging tasks and delegation of roles
use of sensory channels (and five sensory organs) (b) Involving in tasks and providing socio-emotional
in interactions, the longer will be the retention of support with scope for recognition
information. (c) Mentoring and guiding with close supervision
4. Teaching Instructional Facilities Facilitate Change (d) Keeping the tasks specific and soft
in Attitude: Pictures, models, etc. help in creating a The correct answer is (b).
positive attitude among learners.

M01_MADAN 07_65901_C01.indd 45 27/12/22 8:15 PM


1.46 Chapter 1

Educational T echnology seeing at 87% and hearing at 7% are the major ones
Educational technology can be divided into two categories: to attract attention and increase learning. Examples of
audio-visual instructional facilities include television,
1. Hardware approach: This refers to the use of video films, documentary films, etc.
machines and other devices in the teaching learning
process to reach maximum number of students and Functions of A udio-visual I nstructional
also to lessen the cost. There is application of mecha- F acilities
nized “physical science” in the process through teach-
ing machines, radio, television, tape recorder, video- These perform the following functions.
tape, projectors, etc. 1. More clarity and understanding
2. Software approach: This approach is linked mostly 2. Better attention, interest and retention
with behavioral aspects such as skill and knowl- 3. It helps in faster and comprehensive learning
edge. Educationist Skinner is also associated with it. 4. Better access
It is concerned with teaching objectives, principles 5. Save the instructor’s time
of teaching, methods of teaching, reinforcement, 6. Supplement the spoken words by combining audio
feedback, reviews, evaluation. Newspapers, books, and visual stimuli
magazines, games, flash cards etc. are also impor-
tant. It tries to develop Input, Process and Output of Limitations of Audio-visual
technology. Instructional Facilities
1. The learners may form distortions in case of any lack
Types of I nstructional F acilities in instructional facilities.
According to the senses involved, educational technology 2. Teaching may be narrowed down to only a few big
can be divided into audio, visual and audio-video instruc- ideas, not giving the complete picture of a subject.
tional facilities. 3. The practice of being a mere spectator rather than
taking any active part is called as ‘spectatorism’. There
1. Audio Instructional Facilities: The quality of voice
should be thoughtful inquiry.
is assumed to be very important as it has direct effect.
4. Multimedia could include several forms of media,
We spend more than 50% of our time in hearing. This
such as audios, texts, still images, animations, graph-
reflects the importance of the audio media in our
ics, videos and films so their right combination is
life. Examples of audio instructional facilities include
important for effective learning.
language labs, radio sets, sound distribution sets, etc.
2. Visual Instructional Facilities: This is linked with Types of I nstructional Facilities A ccording
‘seeing’. Examples include posters, flashcards, charts,
­bulletin boards, maps, models, photographs, etc. to P rojection or S how
3. Audio-visual instructional facilities: These ‘heard Teaching instructional facilities according to projection
and seen’ messages create a tripolar process of motiva- or show are divided into projected and non-­ projected
tion, classification and stimulation. Out of five senses, instructional facilities that have been depicted in table 1.7.

Table 1.7 Projected and Non-projected Instructional Facilities


Projected Non-projected instructional facilities
instructional
facilities Graphic Display boards 3-D Audio Activity

Films Charts Blackboard Models Radio Field trips


Slides Flash cards Whiteboard Mock-ups Recordings Experimentation
Overhead Posters Bulletin board Objects and Digital Audio Dramatics
projector specimens Player
Epidiascope Pictures and Flannel board Puppets Television Teaching machines
photographs
Video projectors Graphs Magnetic board Telephone and Programmed
mobile instructions
Film strips Map diagrams Peg board

M01_MADAN 07_65901_C01.indd 46 27/12/22 8:15 PM


Teaching Aptitude 1.47

Projected Visual Instructional overheaD projector (ohp)


An overhead projector, like a film or slide projector, uses
Facilities light to project an enlarged image on a screen, allowing
Any visual instructional facilities that is used for magnifi- the view of a small document or picture to be shared with
cation of image on a screen in dark or semi-dark conditions a large audience. Their use is very less these days.
can be called a projected visual instructional facility. There
are three important methods of projection and they are: hanDheLD projector
It is also known as a pocket projector, a mobile projec-
1. Direct Projection: Slide and film projectors tor or a pico-projector. It is an emerging technology that
2. Indirect Projection: Overhead projector applies the use of a projector in a handheld device. It is a
3. Reflected Projection: Opaque projector and response to the emergence of compact portable devices,
epidiascope such as mobile phones, personal digital assistants and
digital cameras, which have sufficient storage capacity to
sLiDes handle presentation materials with an attached display
A slide is a transparent-mounted picture that is projected screen.
by focusing light through it. The projection may be made
on a screen or on a white wall. Slides of 35 mm films viDeo projector
mounted on individual cardboard or plastic frames are A video projector is also known as a digital projector,
common and are extensively used in extension work dur- which is now popular for many applications for extension
ing training programmes, seminars, workshops, group and development. All video projectors use a very bright
meetings, campaigns, exhibitions, etc. light to project the image.

opaQue projector (epiDiascope or episcope)


Concept Box It is a device that displays opaque materials by shining a
bright lamp onto the object from above. The material can be
Mind Mapping book pages, drawings, mineral specimens, leaves, etc.
This method of concept creation was developed by
Tony Buzan in 1960. A mind map is a diagram used fiLM strips
to visually organize information into a hierarchy, The film strip was a common form of still image instruc-
showing relationships among pieces of the whole. It tional multimedia. It was once commonly used by educa-
helps students to visualize and externalize concepts. tors in primary and secondary schools, now overtaken by
It is often created around a single concept, drawn newer and increasingly low-cost, full-motion videocas-
as an image in the center of a blank page, to which settes and DVDs, since 1940s till 1980s.
associated representations of ideas such as images,
words and parts of words are added. It is commonly
used in presentations, critical thinking, brainstorming, Concept Box
decision making and project management.
Microsoft PowerPoint Presentations
Sub-idea Sub-idea Microsoft PowerPoint is a presentation program that
− Idea Idea − was basically created by Robert Gaskins and Dennis
Sub-idea Sub-idea
Austin in 1987. This creates a ‘slide’ show of important
Subject information, charts, designs, images, etc. Now record-
Sub-idea Sub-idea ing, video options are also available on it. The better
− Idea Idea −
Sub-idea Sub-idea presentable and motivational contents created on it are
used for business and school presentations.
Microsoft Powerpoint, copyright © 2003

Fairness Quality

Success

Teamspirit Honesty
Microsoft Corporation.

M01_MADAN 07_65901_C01.indd 47 27/12/22 8:15 PM


1.48 Chapter 1

Non-projected Visual Instructional fLash carDs


Flash cards are brief visual messages on poster board. The
Facilities cards are flashed (turned over at short intervals) before
These facilities are used without projection or the help the audience to emphasize the important points in a pres-
of any projector. Advantages include easy availability, entation. Flash cards are held like a pack of cards and are
no specific power supply requirement, economical and flashed to the audience one at a time in a sequence along
ease in handling. They can be useful in small group sit- with the talk.
uations. Many of them can be converted into projected
instructional facilities. For example, charts, flannel poster
graphs and flash cards can be photographed or scanned A poster is displayed in a public place with the purpose of
and converted into slides. Some of them can be projected creating awareness among people. A poster is generally
through an opaque projector. seen from a distance, and the person glancing at it seldom
has the time or inclination to stop and read. The job of the
poster is to stop the persons hurrying past and thrust the
charts message upon them.
Charts (and diagrams) are especially helpful, as they
enable students to see ideas visually laid out in an organ- 1. Posters give only an initial idea and cannot furnish
ized way. The visual tools can help the students process detailed information. They need to be reported for
content and to make connections more easily. The charts further information by another instructional facilities
can be displayed in the room as per lesson plan. They are or method (Examples: Leaflets and demonstration).
visually interesting, they put direct emphasis to highlight 2. The production of good posters is a technical job and
key findings. requires skill and time.
3. It cannot be repeated, so for each occasion, a new
poster has to be made.
An attractive poster with appealing text to indulge the
audience is known as ‘caption’. Usually, a caption con-
Concept Box veys the important message and the visual is to attract
attention and therefore, to support the message to be
Types of Charts conveyed. Posters can be of themes such as ‘Save Earth’,
1. Process charts are used to show steps in a process. ‘Swachh Bharat Abhiyan’, ‘Rural Health’ etc.
For example, charts can show life cycles of insects,
energy cycles, etc. pictures anD photographs
2. Organizational charts are used to represent
hierarchal relationships, flow of communication A picture is a representation made by drawing, paint-
among different departments in an organization. ing or photography, which gives an accurate idea of an
3. Time charts are used to represent events, occur- object. A good picture may tell a story without using a
rences in chronological sequences, such as evolu- single word. Pictures may be in black and white or in
tion of man, political empires, etc. colour. Nowadays, digital cameras are popularly used to
4. Tabular chart represents data in tabular form for take many photos and eye-catching images.
easy comparison and understanding. For example,
types of plantations, etc. are represented in tabular graphs
form, which makes comprehension easier. A graph is an image that represents data symbolically. A
5. Tree chart shows the growth and development graph is used to present complex information and numer-
from single source to many branches, like in a tree. ical data in a simple, compact format. Bar graphs, line
For example, a family tree is a familiar example. graphs, scatter graphs, and pictographs are some types
6. Stream chart is opposite to a tree chart wherein of graphs. In a two-dimensional graph, the information
many branches come together to converge into a is represented along two co-ordinates: X coordinate and
single stream. For example: Many rivers like Yamuna Y coordinate. An independent variable is shown along X
fall in Ganga, which then flows down to the sea. axis and dependent along Y axis. More about graphs is
7. Sequence charts or flip charts are collection of discussed in Chapter 7 on data interpretation.
charts like flip charts used to show many events or
series of events in succession. Maps
The flip chart is like a calendar, with a sheet each
for twelve months. As the month changes, the A map is a visual representation of an area. It is a sym-
sheet is flipped over. Actually, flip chart consists of bolic depiction highlighting the relationship between
several charts arranged in a sequential order and elements of that space, such as objects, regions and
fastened together at one end with a spiral, metal or themes. Cartography or map-making is the study and
wooden strip. practice of making representations of the earth upon a
flat surface.

M01_MADAN 07_65901_C01.indd 48 27/12/22 8:15 PM


Teaching Aptitude 1.49

Maps are a useful tool in every discipline. In social Peg Board


studies, it is very important for learning geographical, Perforated hardboard is a tempered hardboard that is
historical and economical concepts. pre-drilled with evenly spaced holes. The holes are used
to accept pegs or hooks to support various items, such as
Diagrams tools in a workshop.
A plan, sketch, drawing or outline designed to demon-
strate or explain how something works or to clarify the Three -dimensional Models
relationship between the parts of a whole is called a
Real things may not be available all the times and in
­diagram.
the desired form. Hence, models help to tide over this
problem. A model is a recognizable representation of
Display Boards real things in three-dimensional view, such as its height,
width and depth. This makes the understanding better
Blackboard or Chalkboard
and easy.
Blackboard is one of the oldest teaching instructional Models can be of three types, such as (i) solids, (ii) cut
facilities, and the it is probably the simplest, most inex- away or cross sections and (iii) working models. They
pensive, convenient, and widely used non-projected vis- have the advantage of reality depiction, illustration and
ual instructional facility in extension teaching. It is a vehi- are complex and intricate. They are long-lasting and
cle for a variety of visual materials. inexpensive.
The chalkboard is suitable for use in lectures, training
programmes, group meetings, etc. Objects, S pecimens , and Globe
It facilitates a step-by-step presentation of the topic,
creates a dramatic impact and sustains audience inter- Objects are collections of real things for instructional
est. Presentations may be adjusted according to the use. Specimen is any typical object representing a class
receptivity of the audience. It helps the audience to or group of things. A globe is the spherical model of the
take notes. It helps in comprehension and retention of Earth.
knowledge.
Audio I nstructional Facilities
White Board
Radio
A whiteboard is also known as marker board,
multipurpose board, dry-erase board, dry-wipe board, Radio has been a popular mass medium for close to a
and pen-board etc, They became popular in 1990s. They century. These days, many of us tune in to the radio
are glossy, usually white (or black surface) for making through FM channels. Radio became popular due to its
non-permanent markings. They have a smoother surface easy access, speed and immediacy. With its introduction
allowing rapid marking and erasing of markings on their in 1917, radio was visualized as a source of mass edu-
surface. A white board can be used for display on it with cation. In India, the first radio station was established
the help of projectors. in Mumbai (Bombay) in July 1927. Two more radio sta-
tions in Calcutta and Delhi were established in 1936.
Bulletin Board All India Radio (AIR) broadcasted radio programmes
A bulletin board displays messages. It is a surface in which for the country. In 1937, Calcutta station broadcasted
bulletins, news, information and announcements of spe- school programmes for the first time, and it continues
cific or general interest can be displayed. Bulletin boards till date.
are of different sizes with provisions to hold pins, book School educational programmes are still in demand
exhibits and other materials. and are used by teachers to generate interest of stu-
dents. Gyan Vani is a dedicated FM channel for edu-
Flannel Board and Flannel Graph cational broadcasts. It is used to broadcast educational
A flannel board is a visual instructional facilities in which programmes from Educational Media Production Centre
messages are written or drawn on thick paper and pre- (EMPC) of Indira Gandhi National Open University
sented step-by-step to the audience to synchronize with (IGNOU), New Delhi. Audio programmes developed by
the talk. The board is a flannel-covered flat surface. Central Institute of Educational Technology (CIET) of
Flannel is stretched and then glued to a piece of plywood NCERT for school children are also broadcast by Gyan
or heavy cardboard. Vani.

Magnetic Board Podcast


A magnetic board can be a sheet of tinplate and simply Radio is a mass broadcast medium, whereas podcasts are
a type of chalkboard, the surface of which is treated or personalized broadcast.
coated with a porcelain-like substance. The base of the Podcasts are prepared for and made available to a tar-
board is steel, and pictures and objects can be pasted or get group for specific learning objectives. Podcast is the a
mounted with small magnets and can easily be moved portmanteau of the words ‘pod’ from iPod and ‘cast’ from
about. broadcasting.

M01_MADAN 07_65901_C01.indd 49 27/12/22 8:15 PM


1.50 Chapter 1

Recordings opportunity to review their work. If the machines are


A tape recorder or any other kind of audio recording is used in a classroom, they relieve teachers of some of
suitable for extension work in meetings, training pro- the time-consuming aspects of drilling students and
grammes, campaigns, recording radio programmes, etc. allowing them to give more attention to individuals
It facilitates on-the-spot recording of sound. It is easy to with specific problems or to concentrate on some par-
operate and preserve and has low operational cost as the ticularly difficult area of instruction.
same tape may be used again. 5. Programmed Instructions: They are also useful
instrument.
Digital Audio Player
A digital audio player is sometimes referred to as an MP3
player and has the primary function of storing, organ-
izing and playing audio files. Some digital audio play- Factors Influencing the Selection of
ers are also referred to as portable media players as they Instructional Facilities
have image viewing and video-playing support. An ideal
example is iPod (fourth generation audio instructional No single rule-of-thumb can be given for the selection
facilities). and use of various audio-visual instructional facilities
to ensure effectiveness in all situations. In order to get
Telephone and Mobile the most effective results, the following aspects are
Usually, two people can communicate at a time through important:
a telephone and the system serves many people in a 1. Selection of appropriate instructional facilities
given area if a speaker is attached to it, like Cell Phone- 2. Suitable combination of the selected instructional
Operated Mobile Audio Communication and Conference facilities
System (COMBACCS). This technology is seeing a phe- 3. Their use in proper sequence
nomenal growth in many developing countries. Short
message service (SMS) and wireless application pro- Audio-visual instructional facilities are used individually
tocol (WAP)-enabled cell phones with cameras can be or in combination, thereby taking into consideration the
effective in offering an available extension between following factors.
experts and people. COMBACCS can help community 1. Nature of Audience: Printed media are meant for lit-
members at different locations build relationships and erate people, whereas exhibits, pictures and symbols
understanding. are for less literate people.
Television 2. Size of Audience: A video show or whiteboard can-
Television is an effective tool in expressing abstract not be used effectively when the number of partici-
concepts or ideas. Abstract concepts are usually pro- pants exceeds 30, and internet can be used for large
duced and conveyed with words. Besides this, in mak- audiences.
ing an abstract concept concrete, the role of animation 3. Teaching Objective or Expected Nature of Change:
and visual experimentation is very ­important. Select the audio-visual instructional facilities based
on the objective of extension teaching, i.e., to bring
Activity I nstructional Facilities about a change in
1. Field Trip: A field trip is a structured activity that (a) Thinking or knowledge
occurs outside the classroom. It can be a brief observa- (b) Attitude or feeling
tional activity or a longer, more sustained investigation (c) Actions or skills.
or project. Field trips offer an opportunity for students   If you merely want to inform or to slightly influence
to get exposure to real people, events and opportunity a large number of people, then use mass media such
to make connections with others. as radio or television.
2. Experimentation: The experiments are specifi- 4. Nature of Subject Matter: In case a new practice is
cally useful in science subjects to relate theory with simple and familiar, a news article, a radio ­message, or
practice. a circular letter is effective, whereas complex or unfa-
3. Dramatics: They can convey some message to society miliar practices will require audio-visual instructional
or the public at large. These are usually theme-based facilities.
and the students are assigned different roles. 5. Availability of Instructional Facilities: Despite the
4. Teaching Machines: There are many types of teach- availability of the Internet more than two decades
ing machines. In general, they all work on the same back, it was not being used on a large scale. With
method, which is to present a question, have the user the availability of speed, due to better technology,
indicate the answer and then provide the user with and cost effectiveness, more people are now using
the correct answer. They are usually programmed. Internet-based technologies as teaching instructional
They are particularly useful in subjects that require facilities.
drill, such as arithmetic or a foreign language. Users 6. Relative Cost: Effective instructional facilities need
can proceed at their own pace and also have an not necessarily be costly. The amount expended on

M01_MADAN 07_65901_C01.indd 50 27/12/22 8:15 PM


Teaching Aptitude 1.51

Dale’s Cone of Experience


People generally remember Learners are able to (Learning Outcomes)

10% of what they Read Define


Read
text Describe
List
Listen to Explain
20% of what they Hear
lecture (Hear)

30% of what they See Watch still pictures


Demonstrate
Watch moving pictures Apply
Practice
50% of what they See and Hear VIew exhibit

Watch demonstration

Participate in hands-on workshop Concrete


70% of what they Say and Write
Role-play a situation Analyze
Design
Model or simulate a real experience Create
Evaluate
90% of what they Do
as they perform the task Direct Purposeful Experience - Go through the real experience

Figure 1.20 Dale’s Cone of Experience

audio-visual instructional facilities, in relation to the they use perceptual learning styles and these learning
extent of effectiveness is also an important consider- styles are sensory based.
ation in their selection and use. The more sensory channels possible in interacting
with a resource, the better chance that many students can
learn from it. According to Dale, the instructors should
Dale’s Cone of Experience design instructional activities that build upon more real-
Dale’s Cone of Experience is a model that incorporates sev- life experiences. Dales’ cone of experience is a tool to help
eral theories related to instructional design and learning instructors make decisions about resources and activities.
processes. During 1960s, Edgar Dale theorized that learn-
ers retain more information by what they ’do’ as opposed
to what is ‘heard’, ‘read’, or ‘observed’. His research led to Important Tips for Better
the development of the ‘Cone of Experience’. Today, this
‘learning by doing’ has become known as ‘experiential Classroom Management
learning’ or ‘action learning’. In NET examination, there are questions about class indis-
How can instructors use the cone of experience? cipline and how to deal with the situation. There are a
According to Dale’s research, the least effective method number of things a teacher must keep in mind when deal-
at the top involves learning from information presented ing with students who do not behave in a disciplined man-
through verbal symbols, i.e., listening to spoken words. ner in class. There is a basic rule that the teacher must
The most effective methods at the bottom involve direct, consider that he or she does not hurt them physically or
purposeful learning experiences, such as hands-on or emotionally. This would prove psychologically harmful to
field experience. Direct purposeful experiences repre- the student and our purpose is surely not to harm them
sent reality or the closet things to real, everyday life. The but to modify their behaviour as individuals.
cone charts the average retention rate for various meth- Punishments, if any, should be seen as reasonable and
ods of teaching. The further you progress down the cone, fair, and never vicious. The ability to control a group of
the greater the learning and the more information likely students depends on the personality of the teacher and
to be retained. It also suggests that when choosing an also the rapport that he or she develops with them. There
instructional method, it is important to remember that are some tips to be kept in mind.
involving students in the process strengthens knowledge
retention. It reveals that ‘action–learning’ techniques 1. Immediate Action: In case of deviant behaviour, a
result in up to 90% retention. People learn best when teacher must take immediate action.

M01_MADAN 07_65901_C01.indd 51 27/12/22 8:15 PM


1.52 Chapter 1

the main purpose of instruction is conceived in terms of


Concept Box helping students achieve a set of learning outcomes that
include changes in the cognitive, psychomotor and affec-
Teaching with Sense of Humour tive domains.
● Laughter is a natural, universal phenomenon, and has
There is a direct relationship among the four impor-
beneficial effects, both physical and psychological. tant factors of the educational system, such as objec-
● Everyone loves a teacher with an infectious sense of tive, curriculum, method and evaluation. As Indian
humour. Education Commission (1944–1966) has remarked, ‘It
● It builds cordial relationship. is now agreed that evaluation is a continuous process,
● It has the ability to relax people and reduce tension. forms an integral part of the total system of education,
● It is an effective advertising strategy. and is intimately related to educational objectives. It
● Teaching with the help of cartoon is a very exercises a great influence on the pupil’s study habits
effective way. and the teacher’s method of instruction, and this helps
● When there is a willingness to change, there is hope
not only to measure educational achievement but also
to improve it’.
for progress in any field.
● Students enjoy humour in forms of funny anecdotes.
evaLuation
Evaluation is a systematic process of collecting, analysing
and interpreting information to determine the extent to
which instructional objectives are being achieved.
Perhaps the most comprehensive definition of evalua-
2. Stop Teaching in Case of Misbehaviour: The tion has been given by Beeby, which says, ‘Evaluation is
moment the teacher stops teaching, it is clear that the the systematic collection and interpretation of evidence
teacher means business and will not tolerate misbe- leading as a part of process to a judgement of value with
haviour in the class. a view to action.’
3. Change Seats: If few students disrupt the class, From this definition, it is clear that the following four
change their seats. Separating the troublemakers is key elements constitute the process of evaluation.
quite effective in controlling indiscipline. 1. Systematic collection of evidence (Example: score)
4. Adapt and be Sensitive: In case the entire class 2. Its interpretation
is gradually getting out of control, then it’s a signal 3. Judgement of value
that the activity is boring. The best way of control- 4. A view to action
ling them is by changing the activity. For example, if
they are reading, immediately switch over to a writing
task, which would keep them all quiet and involved.
The teacher must learn to adapt and be sensitive to
the mood of the class.
5. Counsel after Class: One of the most effective ways Curriculum standards
of tackling a student is by giving counselling after • Frameworks
class. The teacher should also clearly explain the con- • Syllabi
sequences of not improving. • Guides
6. Talk to the Parents: In several cases, a talk with the • Blueprints
parents will improve the behaviour of the student. It • Benchmarks
Correlation
would also give a better insight into the reasons for
indiscipline by individual students.
Validity
Instructional program
• Instructional styles
Evaluation Systems • Print materials
Assement-evaluation • Equipment
i nterDepenDence of teaching, L earning, anD • Facilities
system
e vaLuation • Objective tests • Technologies
Alignment • Communities
Teaching has been defined as the process of facilitating • Performance
learning, and the term learning is broadly defined as the assessments
process of acquiring knowledge, attitude, skills, habits • Portfolios
and abilities. To determine whether teaching has facili- • Teacher observations
tated learning, and if yes, upto what extent, evaluation • Program evaluations
is carried out. In other words, teaching, learning and
evaluation are three inter-dependent aspects of the edu-
cative process. This interdependence is clearly seen when figure 1.21 Congruence Triangle – Reynolds (1996)

M01_MADAN 07_65901_C01.indd 52 27/12/22 8:15 PM


Teaching Aptitude 1.53

Desirable Characteristics of Evaluation


Evaluation
1. Comprehensiveness: It must try to assess all aspects
of a child’s development. Thus, different techniques
might be used by the teachers to evaluate the perfor-
Assessment
mance of the child.
2. Continuous: Evaluation is a continuous process in
education. It is not just an examination but a part Measurement
of the evaluation process. There is no fixed time
limit for the completion of evaluation work, but it
is a continuous process. CBSE’s Continuous and
Comprehensive Evaluation (CCE) is based on the
above two parameters. CCE helps in improving the
student’s performance by identifying his or her
learning difficulties at regular time intervals right Figure 1.22 Comparison Between Measurement,
from the beginning of the academic session and Assessment and Evaluation
employing suitable remedial measures for enhanc-
ing their learning performance. Functions of Evaluation
Difference between Measurement, Assessment Evaluation does not end with the summarization of results.
and Evaluation It has direct bearing on the improvement of the system as a
whole. The functions of evaluation are as ­follows.
Measurement is the quantitative description of one’s
performance. For example, a student scored 92 in 1. Feedback: To assess strengths and weaknesses.
Mathematics, 75 in Science, 65 in Social Science, 64 in 2. Motivation: The mere realization that you would be
Hindi and 68 in English. evaluated propels a student to work hard.
3. Better Guidance: Crucial for the growth of pupils.
Gathering information Measurement 4. Remediation: It helps in locating the areas that
require remedial measures.
5. Facilitates Planning: It helps the teacher in planning,
Assessment is the second step of evaluating student’s per-
organizing and implementing learning activities.
formance. The description that a student stood first in
the class represents the concept of assessment. It makes 6. Revision of curriculum.
student’s performance more meaningful. Unless we inter- 7. Inter-institutional comparison.
pret, analyse, rank-order and compare one’s individual 8. Educational Decision-making: It relates to selec-
score with the average score of the group, we cannot find tion, classification, placement, promotion, etc.
out one’s relative position in a group. 9. Submission of progress report to parents.

Comparison of two or
Assessment Types of Evaluation
more sets of information A good evaluation device is one which secures valid evi-
dence regarding the desired change of behaviour. A teacher
If in case of a student, we find that she has ‘improved signifi- needs to know the various devices that are helpful in gath-
cantly in half-yearly ­examination’ in comparison to her per- ering evidence on the changes taking place in a pupil.
formance in the earlier examinations, what does it mean?
We can conclude that this type of judgement carries Quantitative Techniques
certain value and adds to the performance of student to 1. Written Examination: It is also known as paper pen-
make it more meaningful. cil test. In this technique, the answers are to be writ-
While forming a judgement like ‘improved signifi- ten as per the instruction of questions.
cantly’, the earlier performance of the student in the pre- 2. Oral Examination: They supplement the written
vious examinations has been taken into consideration examination. Examples are tests of reading ability,
by the teacher. When we add value to the assessment of and pronunciation and viva voce.
student performance, we carry out evaluation of their 3. Practical Examination: These tests are necessary to
performance. test experimental and manipulative skills of a learner,
particularly in subjects such as science, technology,
Comparison of two agriculture, craft, and music.
sets of information Placement
+ = Evaluation
(Assessment) of value Qualitative Techniques
1. Observation and Interviews: Observation should be
The comparison can be depicted with the help of ­following done in a systematic manner to evaluate the behaviour
diagrams as well. of the students. It helps in controlled and uncontrolled

M01_MADAN 07_65901_C01.indd 53 27/12/22 8:15 PM


1.54 Chapter 1

situations. Thus its recording is also important. designing instruction and it continues till the end of
Interview is sometimes superior to other devices. It is the course/instruction. Conducting unit end exami-
because of the fact that pupils are usually more willing nation, monthly examination, quarterly examina-
to talk than write. tion, half-yearly examination, etc. are the examples
2. Checklist: A checklist is an instrument that is used for of formative evaluation. It provides feedback to the
collecting and recording evidence regarding signifi- teachers to know effectiveness of their teaching and
cant behavioural tendencies of the pupils or specific modification required and the learners to know the
problems they present in the classroom. progress of their learning. It also provides scope for
3. Rating Scale: Rating is a term applied to the expres- diagnostic evaluation.
sion of opinion or judgement regarding some situa- 3. Diagnostic Evaluation: The key word in diagnos-
tion, object or character. A rating scale is a device by tic evaluation is identifying of ‘learning difficulties’.
which judgements can be quantified. This evaluation is specially conducted to identify
4. Cumulative Records: Anecdotal records, cumula- and remove the learning difficulties of learner if it is
tive record cards and diaries of pupils are some other observed and found during the formative evaluation.
devices used in evaluation process to know the details For example, if a learner couldn’t understand certain
about a child’s behaviour. concepts in a particular subject and continuously
performing poorly in that subject, a teacher should
type of evaLuation on the basis of phase of conduct a diagnostic test to know the causes of the
instruction difficulties and accordingly provide them remedial
treatment to overcome the difficulties. Diagnostic
In the various phases of instruction, evaluation is inte-
evaluation also identifies and provides remedies for
grated. The four types of evaluation are placement, form-
personal, physical and psychological problems.
ative, diagnostic and summative.
4. Summative Evaluation (External Evaluation): As
1. Placement Evaluation: This can be termed as the name indicates, it is done at the end or comple-
‘assessment before entrance’. This evaluation is tion of the course.
conducted when a candidate is seeking admission It determines the extent to which the objectives
in an institution, so it is called as ‘entry behaviour’. of instruction have been achieved and is used for
Thus, it starts before the actual teaching in the class. assigning course grades. Summative evaluation
An institution or teacher is assured of some standard generally includes oral reports, projects, term
of skills and knowledge that a learner possess at the papers and teacher-made achievement tests and it
time of admission. shows how good or how satisfactory the student is
in accomplishing the objectives of instruction.

Areas of function Types of evaluation and their


Concept Box functions
Deeksharambh After instructions Summative evaluation
This is basically a Student Induction Programme (SIP). (To certify the learner)
This helps new students adjust and feel comfortable in
the new environment, inculcate in them the ethos and During instructions (a) Diagnostic evaluation
culture of the institution, help them build bonds with (To solve learning difficulties)
other students and faculty members, and expose them (b) Formative evaluation
to a sense of larger purpose and self-exploration. (To provide feedback on the
teaching-learning process and
2. Formative Evaluation: The key word in forma- to have mastery in content)
tive evaluation is learning progress (evaluation
for learning). It is the second stage of evaluation. Before instructions Placement evaluation
This evaluation starts from the very beginning of (To know entry behaviour)

On the basis of On the basis of phase On the basis of nature On the basis of On the basis
Approaches of Instruction of reference purpose of grades
Quantitative techniques 1. Placement 1. Norm referenced 1. Diagnostic tests Direct
2. Formative 2. Criterion referenced 2. Aptitude tests
Qualitative techniques 3. Diagnostic 3. Achievement tests Indirect
4. Summative 4. Proficiency tests

M01_MADAN 07_65901_C01.indd 54 27/12/22 8:15 PM


Teaching Aptitude 1.55

evaLuation accorDing to nature of In the above statements, the person’s performance


is compared to others of their group and the relative
reference standing position of the person in his/her group is
Norm-referenced testing and criterion-referenced testing mentioned. We compare an individual’s performance
are the two complimentary approaches to educational with similar information about the performance of
testing. There are some similarities and differences others.
between them. Norm-referenced tests are mostly easy but can be
1. Criterion-referenced Evaluation: Glasar (1963) first tough as well
used this test to describe the learner’s achievement on Reflective Prompts
a performance continuum.
Reflective prompts is a technique in which the teacher
Consider the following Statements: provides a set of flexible questions to the students that
(a) Amit scored 95 or 95% marks in Mathematics. prompt them to reflect on their own learning. In this
(b) The typing speed of Davinder is 58 words per technique, each student answers some questions such
minute. as given below after completion of a lesson/unit by the
A criterion-referenced test is used to ascertain teacher. If the test scores are interpreted in terms of an
an individual’s status with respect to a defined individual, then they are known as self-referenced.
achievement domain. In the above statements,
there is no reference to the performance of other types of evaLuation tests of the basis of
members of the group. Thus, criterion-referenced p urpose
evaluation determines an individual’s status with
reference to well-defined criterion behaviour. Though there is some overlap with the evaluation tech-
There are clearly defined learning outcomes niques as discussed earlier, purpose-specific category
which serve as referents (criteria). Success of cri- includes tests designed to achieve a specific purpose of
terion-reference test lies in the delineation of all evaluation. Generally four test-types are identified in this
defined levels of achievement which are usually category. Let us briefly present the features of each of these.
specified in terms of behaviourally stated instruc- Diagnostic Test
tional objectives.
The purpose of criterion-referenced evaluation These tests help us in identifying ‘area of learning’ in
or test is to assess the objectives and that’s why it which a learner may need a remedial course and they pro-
is termed objective-based test. The objectives are vide us a profile of what the learner knows and does not
assessed, in terms of behavioural changes among know. A diagnostic test may consist of a battery of a num-
students. Hively and Millman (1974) suggested ber of sub-tests to cover sub-areas.
the new term domain-referenced, which has a
Aptitude Test
wider connotation. A criterion referenced test can
measure one or more assessment domain/s. These basically serve a predictive function. They help
2. Norm-referenced Evaluation: This test is used to us identify potential talents and desirable character-
ascertain an individual’s status with respect to the istics which are essential for one to be competent to
performance of other individuals on that test. It is perform a specific task. These tests are generally used
normally used in competitive exams. while selecting people for special courses.
Consider the following statements:
(i) Amit stood third in Mathematics test. Achievement Test
(ii) Rajesh scored 98 percentile, which means only As the name indicates, such tests aim to measure the
2% candidates scored better than him. This is extent to which the objectives of a course have been
used in CAT for admission into IIMs and some achieved. The usual end-of-course exam may be taken as
other top-notch institutes in India. a typical example of an achievement test.

Proficiency Tests
Concept Box These tests aim to assess the general ability of a person at
a given time.
Scholastic Assessment Their scope is governed by a reasonable exception of
Scholastic assessment refers to the assessment of cog- what abilities learners of a given status (say, matriculates
nitive abilities of learners in various academic activi- or graduates) should possess.
ties that are associated with various subjects.
Therefore, all those abilities in cognitive domain, graDing systeM of evaLuation
namely, knowledge, understanding, application, anal-
ysis, synthesis, evaluation and creativity come under The word ‘grade’ is derived from the Latin word Gradus
scholastic abilities. Continuous and comprehensive which means ‘step’. In educational measurement, grading
evaluation is one such example. involves the use of a set of symbols to communicate the
level of achievement of the students.

M01_MADAN 07_65901_C01.indd 55 27/12/22 8:15 PM


1.56 Chapter 1

Types of Grading (C) Evaluating student in grading system


1. Direct Grading: In direct grading, the performance (D) Providing feedback while teaching
exhibited by an individual is assessed in qualitative (E) Encouraging student reflect more
terms, and the impression so obtained by the examiner Select your answer from the options given below:
is directly expressed in terms of letter grades. (a) (A), (B) and (C)
The advantage of direct grading is that it minimizes (b) (B), (C) and (D)
the inter-examiner variability. Moreover, it is easier to
(c) (B), (D) and (E)
use in comparison to indirect grading. Direct grading
lacks transparency. (d) (B), (C) and (E)
2. Indirect Grading: In this method, the performance The correct option is (c).
of an examinee is first assessed in terms of marks and 2. Given below are two statements (2021)
subsequently transformed into letter grades by using Statement (I): Achievement and aptitude tests
different modes. This transformation may be carried designed to assess the upper limits of the examinees’
out in terms of both ‘absolute grading’ and ‘relative knowledge and abilities.
grading’ as discussed below. Statement (II): Objective personality tests and projec-
(a)  Absolute Grading: Absolute grading is a conven- tive personality tests are designed to measure the typi-
tional technique of evaluation. It is based on a pre- cal behaviour and characteristics of the examinees.
determined standard that becomes a reference
point for assessment of students’ performance. It Which of the following options is correct?
involves direct conversion of marks into grades, (a). Both statement (I) and statement (II) are correct.
irrespective of the distribution of marks in a sub- (b). Both statement (I) and statement (II) are incorrect.
ject. For example, the categorization of students (c). Statement (I) is correct but statement (II) is incorrect.
into five groups, namely, (d). Statement (I) is incorrect but statement (II) is correct.

Distinction - 75% and above The correct answer is (a).


First division - 60% and less than 75%
Second division - 45% and less than 60%
Third division - 33% and less than 45%
Unsatisfactory - Below 33% Choice Based Credit System (CBCS)
The Ministry of Education has brought out New Education
(b) Relative Grading: Relative grading is generally Policy (NEP) to bring out reforms in Indian education sys-
used in public examination. In this system, the tem. UGC has earlier asked for active participation, already
grade of a student is decided not by her perfor- initiating several steps to bring equity, efficiency and aca-
mance alone but the performance of the group. demic excellence in National Higher Education System.
This type of grading is popularly known as ‘grad- Majority of institutions have entered recently into semes-
ing on the curve.’ ter system to match the international educational pattern.
There is need to allow flexibility in education sys-
O bjective and Projective P ersonality T ests tem, so that students depending upon their interests
Objective Tests: This is a self-reporting system that and aims can choose interdisciplinary, intra-disciplinary
measures personality traits on a “yes” or “no” scale. and skill-based courses. This can only be possible when
These tests are considered objective as there is no inter- choice-based credit system (CBCS), an internationally
vention by the test maker to manipulate the answer. acknowledged system, is adopted. The choice-based
   Locus of Control refers to an individual’s percep- credit system not only offers opportunities and avenues
tion about the underlying main causes of events in to learn core subjects but also explore additional avenues
his/her life. of learning beyond the core subjects for holistic develop-
Projective Tests: This test is designed to let a person ment of an individual.
respond to ambiguous stimuli, presumably revealing
hidden emotions. The internal conflicts may also be Advantages of CBCS
projected by the person into the test. 1. Shift in focus from the teacher-centric to student-cen-
   Kindly look at the following questions. tric education.
2. Student may undertake as many credits as they can
Stopover cope with (without repeating all courses in a given
semester if they fail in one/more courses).
1. Below are listed some activities performed by a 3. CBCS allows students to choose inter-disciplinary,
teacher. Which activities are of the nature of forma- intra-disciplinary courses, skill-oriented papers (even
tive evaluation? from other disciplines according to their learning
(A) Giving a mastery test needs, interests and aptitude) and more flexibility for
(B) Conducting quiz session students).

M01_MADAN 07_65901_C01.indd 56 27/12/22 8:15 PM


Teaching Aptitude 1.57

4. CBCS makes education broad based and at par with


global standards. One can take credits by combin- Computer-Based Testing (CBT)
ing unique combinations. For example, Physics CBT seems to be a catalyst for changes in pedagogical meth-
with Economics, Microbiology with Chemistry or ods. It brings about a transformation in learning, pedagogy
Environment Science etc. and curricula in educational institutions. The setting is the
5. CBCS offers flexibility for students to study at differ- basis of both computer-based and paper-based testing.
ent times and at different institutions to complete one
course (ease mobility of students). Benefits of Computer-Based T esting (CBT)
6. Credits earned at one institution can be transferred. 1. More frequent testing opportunities
Though difficult to adopt, the uniform grading system will 2. Data-rich results
also enable potential employers assess the performance 3. Increasing candidate reach
of the candidates. In order to bring uniformity in evalu- 4. Streamlined logistics
ation system and computation of the cumulative grade
point average (CGPA) based on student’s performance in There are two types of CBT which include:
examinations, the UGC has formulated the guidelines to 1. Linear Test: This involves a full-length examination
be followed. in which the computer selects different questions for
individuals without considering their p ­ erformance
O utline of C hoice Based Credit System level.
2. Adaptive Test: Here, the computer selects a range
1. Core Course: A course, which should compulsorily of questions based on the individual’s performance
be studied by a candidate as a core requirement is level. These questions are taken from a very large pool
termed as a core course. of possible questions categorized by content and dif-
2. Elective Course: Generally a course which can be ficulty.
chosen from a pool of courses and which may be very
specific or specialized or advanced or supportive to Using the Waterfall model, CBT’s SDLC, was split into
the discipline/subject of study or which provides a number of independent steps. The previous stage is
an extended scope or which enables an exposure to always completed before moving to the next stage of the
some other discipline/subject/domain or nurtures life cycle.
the candidate’s proficiency/skill is called an elective 1. Requirements analysis and definition
course. 2. System and software design
3. Discipline-Specific Elective (DSE) Course: 3. Implementation and unit testing
Elective courses may be offered by the main dis- 4. Integration and system testing
cipline/subject of study is referred to as Discipline 5. Operation and maintenance
Specific Elective. The University/Institute may also
offer discipline related elective courses of interdis- In India, CBT has been started for number of exams such
ciplinary nature (to be offered by main discipline/ as by IBPS for banking exams and currently by National
subject of study). Testing Agency (NTA) to conduct exams for UGC, NEET,
4. Dissertation/Project: An elective course designed to GPAT, GMA, etc.
acquire special/advanced knowledge, with an advi-
sory support by a teacher/faculty member is called Stopover
dissertation/project. Which of the following is an example of maximum

5. Generic Elective (GE) Course: An elective course performance test?
chosen generally from an unrelated discipline/sub-
ject, with an intention to seek exposure is called a (a) Personality tests
Generic Elective. (b) Projective personality tests
6. Ability Enhancement Courses (AEC): This may (c) Aptitude tests
be of two kinds: Ability Enhancement Compulsory (d) Interest and attitude scales
Courses (AECC) and Skill Enhancement Courses The correct option is (c).
(SEC). “AECC” courses are those based upon the
content that leads to Knowledge enhancement; Skill
Enhancement Courses (SEC): These courses may be
chosen from a pool of courses designed to provide Curriculum Framework, Curriculum
value-based and/or skill-based knowledge. and Syllabus
Central/state universities have a lot of flexibility in decid-
Though many Constitution related aspects have been dis-
ing common minimum syllabi of the core papers and at
cussed in unit 10, we need to look at the following points.
least follow common minimum curriculum as fixed by the
UGC. This allows deviation from the syllabi, 20 % being 1. Till 1976, state governments used to take decisions
the maximum. on all matters pertaining to education, including

M01_MADAN 07_65901_C01.indd 57 27/12/22 8:15 PM


1.58 Chapter 1

curriculum. The role of centre was to provide guid- facilitate learning and are designed to implement specific
ance to the states on policy issues. educational aims.
2. National Council for Education Research and Training It is a plan to explain what concepts are to be trans-
(NCERT) developed the National Curriculum acted, what knowledge, skills and attitudes are to be
Framework (NCF) in 1975 following the recommen- deliberately developed among learners. It includes state-
dations of Education Policy on 1968. ments of criteria for selection of content and choice of
3. In 1976, the constitution was amended to include methods for transaction of content as well as evaluation.
education in the concurrent list. It is concerned with the following factors:
4. In 1986, National Policy on Education (NPE) envisioned 1. The general objectives of education at a particular
National Curriculum Framework (NCF) that aimed at stage or class.
modernizing education, and focused on India’s geo- 2. Subject-wise learning objectives and content.
graphical and cultural diversity while setting core val-
ues and standards. NPE focused on a relevant, flexible 3. Course of studies and time allocation.
and learner-centred curriculum. NCF was subsequently 4. Teaching-learning experiences.
revised in the years 1988, 2000 and 2005. 5. Teaching-learning instructional facilities and materials.
5. In September 2021, the Union Ministry of Education 6. Evaluation of learning and feedback to learners.
constituted a 12-member national steering commit-
tee to develop a new National Curriculum Framework In reference to the discussion given above, it would mean
(NCF) in line with the National Education Policy 2020 that the curriculum core and syllabus put together form
(NEP). the curriculum.

Curriculum Framework: It is a plan that interprets Syllabus


educational aims with regard to both the individual and Syllabus is a document that gives the details of the content
society. This plan leads to an understanding of the kinds of subjects to be transacted and the skills, knowledge and
of learning experiences that an educational institute must the attitude which are to be deliberately fostered together
provide to children. with the stage (level)-specific objectives.
Curriculum: This is perhaps best thought of as the Syllabus is a descriptive list of subjects to be covered and
sum total of all deliberately planned set of activities that a summary of their contents. It describes and summarizes

Curriculum framework

Assumptions concerning Aims of education


human beings and society

Stage-specific objectives Details of the syllabus


Epistemological assumptions
Principles of content
selection and organization

Recommended
Assumptions about learning Criteria for good methods classroom practices

Criteria for good material Textbooks and TLM

Assumed understanding of
the child and her context
Principles of evaluation Evaluation scheme

Foundations of curriculum Curriculum core Curriculum details

Source: NCERT Pedagogy


figure 1.23 Curriculum Framework

M01_MADAN 07_65901_C01.indd 58 27/12/22 8:15 PM


Teaching Aptitude 1.59

Connecting knowledge to life outside school.

Ensuring that learning is shifted away from rote methods

Guidelines
for Enriching the curriculum to provide for overall development
curriculum of children rather than remaining textbook-centric.
development

Making examination more flexible and integrated into


classroom life.

Nurturing overriding identity informed by caring concers


within the democratic policy of the country.

Figure 1.24 Curriculum Development Guidelines

what should be taught to the students; it may have details, for action, identified as the ‘successful performance
such as schedule, assessments, assignments, projects, etc. of tasks’. The focus is to observe the change in behav-
Thus, it may highlight the schedule of assignments, proj- iour. The following types fall within the scope of such
ects and exams, etc. curriculum.

Main Differences Between Syllabus and (a) Competency-based curriculum: According


to UNESCO, competency is the ability to apply
C urriculum learning resources and outcomes (knowledge,
1. The syllabus is the summary of the topics to be taught skills, values,and attitudes) adequately in a
in the particular subject where as curriculum refers to defined context (education, work, personal,or
the overall content taught in an educational system or professional development). A competency-based
a course. curriculum is a curriculum that emphasizes what
2. The curriculum is wider in scope when compared learners are ‘expected to do’ rather than mainly
with syllabus. Syllabus is descriptive in nature, but focusing on what they are ‘expected to know’. In
the curriculum is prescriptive. principle, such a curriculum is learner-centred
3. Syllabus varies from teacher to teacher while the cur- and adaptive to the changing needs of students,
riculum is same for all teachers. teachers, and society.
4. The syllabus is accessible to the learners at the begin- (b) Criterion-referenced curriculum: In such situ-
ning of course. This can used as a guide for their stud- ations, the results are compared to a set standard
ies. Curriculum is not available to the learners unless or criteria and testers are ranked in relation to the
it is specifically ask for. body of tested knowledge.
5. Syllabus is mostly prepared by the teachers. (c) Mastery learning: In Mastery Learning curricu-
Conversely, a curriculum is decided at a higher level. lum, a student’s aptitude is based on how long
6. The duration of syllabus is one year only. The curricu- they need to master the content, and all stu-
lum lasts till the completion of the course. dents (given enough time and intervention) are
assumed to be able to eventually master the con-
Basic A pproaches to Curriculum tent. Mastery learning is usually divided into five
1. Subject-centred Curriculum: The knowledge is stages: pre-assessment, instruction, formative
transferred from possessors of knowledge to those assessment, correction or enrichment instruc-
who don’t possess it. Sometimes it is termed as give tion, and summative grading or assessment.
(teacher) and take (learner) relationship. Lecture is (d) Programmed learning: This curriculum is in the
the main method for it. The focus to imbibe the top- process format where we can move from arrang-
ics is through rote memorization. The knowledge is ing the material to be learned into a series of
tested through evaluation. sequential steps that is from known to unknown.
2. Behaviourist Curriculum: Here, learning is viewed B.F.Skinner and his companions had first started
as change in behaviour. Knowledge is the capability ‘programmed learning’ in 1943.

M01_MADAN 07_65901_C01.indd 59 27/12/22 8:15 PM


1.60 Chapter 1

  Here, the teachers are facilitators of knowledge   ‘Constructivist curriculum’ is based on the follow-
and learning, they assume active role. The learn- ing assumptions:
ers are assumed to be passive (or silent) receivers of (a) Knowledge is constructed in an active manner, thus
knowledge, and their cognitive aspects and behav- it is invented, created or discovered by learners. It is
iour changes under guidance from a teacher. The not passively received and stored by learners.
chalk and talk (lecture) method is assumed to be the (b) Knowledge cannot be separated from the process
best. The critics say that it does not focus on all round of learning. It is based on learner’s conceptual
development of the student. structures and prior experiences.
3. Learner-centred Curriculum: Here, motivation is (c) There is need to construct and reconstruct cogni-
used to impart and nurture knowledge in a stimulat- tive structures by learners, this may happen as a
ing environment. The engaging curriculum also plays result of new knowledge or reflection of students
a role in it. The learning is viewed from a constructivist on previously acquired knowledge.
perspective. (d) Social interactions help in construction and evo-
  The students need to become creative, get involved lution of knowledge.
in inquiry and make meaning for themselves out of (e) Concept formation progresses from concrete to
interactions with the environment. The focus is ‘what abstract slowly.
is happening within’. The learner stands between the (f) The learning styles of learners are different and they
stimulus and the response. must be duly accommodated by the teacher.

Table 1.8 Important Dates Related to Education

Dates Important days


January 04 World Braille Day—A form of written language for blind people, in which the characters are
represented by patterns of raised dots that are felt with the fingertips.
January 24 National Girl Child Day, also known as Balika Divas.
February 21 International Mother Language Day
February 28 National Science Day—To commemorate the invention of the Raman Effect in India by the Indian
physicist Sir Chandrasekhara Venkata Raman on the same day in 1928.
June 21 International Day of Yoga— United Nations proclaimed 21 June as International Yoga Day.
September 5 Teachers’ Day is celebrated on 5th September every year, which is also the birthday of Dr. Sarvepalli
Radhakrishnan, the first Vice President of independent India and the second President of the country.
September 8 International Literacy Day—To highlight the importance of literacy in life and remind ourselves of
the status of literacy and adult learning worldwide.
September 14 Hindi Day
October 5 World Teachers’ Day—UN World Teachers’ Day commemorates the work of teachers and their
contributions to society.
October 11 International Day of Girl Child
October 20 World Statistics Day
November 1 National Education Day—It is also the birthday of Maulana Abul Kalam Azad, eminent educationist
and the first Education Minister of independent India.
November 14 Children’s Day—It is also the birthday of independent India’s first Prime Minister Pundit Jawaharlal
Nehru.
November 20 Universal Children’s Day

M01_MADAN 07_65901_C01.indd 60 27/12/22 8:15 PM


Teaching Aptitude 1.61

A s s e s s Yo u r L e a r n i n g

CONCEPTS, NATURE, AND CHARACTERISTICS


1. Given below are two statements, one labelled as 7. In the sets given below, Set I provides levels of teach-
Assertion (A) and the other labelled as Reason (R). ing while Set II gives their focus of concern:
Assertion (A): Philosophy helps in determining
aims of education. Set I Set II
Reason (R): Education depends mostly on (Levels of Teaching) (Focus of Concern)
Philosophy. (A) Autonomous (1) Problem rising and
In the context of the two statements, which one of development level
the following is correct? problem solving
Codes:
(B) Memory level (2) Affects and feelings
(a) Both (A) and (R) are true.
(b) Both (A) and (R) are false. (C) Understanding (3) Recall of facts and
(c) (A) is true, but (R) is false. level informations
(d) (A) is false, but (R) is true. (D) Reflective level (4) Seeing of relationship
2. Given below are two statements: among facts and their
Statement I: One of the basic principles of progres- examples
sive view of teaching is that education should be life (5) Peer learning
itself rather than a preparation for living.
Statement II: According to reconstructionist view of Select the correct answer from the options given below.
education the new social order must be ‘genuinely
(a) A-1, B-2, C-4, D-5
democratic’.
(b) A-1, B-4, C-3, D-2
In the light of the above statements. Choose the cor-
(c) A-2, B-3, C-4, D-1
rect answer from the options given below:
(d) A-5, B-4, C-3, D-2
(a) Both Statement I and Statement II are true.
(b) Both Statement I and Statement II are false. 8. Who among the following thinkers propagated the idea
(c) Statement I is true but Statement II is false. of education of the most talented? [June 2019]

A S S E S S YO U R L E A R N I N G
(d) Statement I is false but Statement II is true. (a) Emanuel Kant (b) John Paul Sartre
(c) Aristotle (d) Plato
3. The term hermeneutics is drawn from [July 2022]
(a) Sociology (b) Anthropology 9. A teacher is faced with the problem of choosing the
(c) Theology (d) Economics appropriate teaching method for the transaction of a
course content. Which branch of philosophy will most
4. Things as they are and as they are likely to be encoun-
appropriately describe his/her predicament?
tered in life rather than words” was the slogan of the
[June 2019]
(a) Pragmatists (b) Realists
(a) Metaphysics (b) Epistemology
(c) Idealists (d) Existentialists
(c) Axiology (d) Logic
5. An existentialistic teacher should emphasize on
10. Which of the following can be taken as the right state-
(A) Freedom
ments in context of teaching?
(B) Responsibility
1. A teacher should have the ample subject knowl-
(C) Subjective feelings
edge as per the standard being taught.
(D) Cooperative living
2. Teachers are always born in a particular section of
In the above, which combination is correct?
society.
[December 2012]
3. The duty of a teacher in the class should be to
Codes: teach his/her subject in the artistic manner.
(a) A and B are correct. 4. The main objective of a teacher is to make the stu-
(b) A and C are correct. dent understand what has been taught in the class.
(c) A, B and C are correct. 5. The prime duty of a teacher is to always seek guid-
(d) B, C and D are correct. ance from the authorities while teaching in the class.
6. Behaviour pattern that increases in frequency when Codes:
followed by a reward is known as
(a) 1, 3, and 5 (b) 1, 2, 3, and 5
(a) Shaping (b) Classical Conditioning
(c) 1, 3 and 4 (d) 1, 2, 3, 4 and 5
(c) Generalization (d) Operant Conditioning

M01_MADAN 07_65901_C01.indd 61 27/12/22 8:15 PM


1.62 Chapter 1

11. Which of the following statements are true? 15. Development of language in children, according to
1. Behaviourism pedagogical approach would say B.F. Skinner, is the result of
that learning is teacher centred. Teacher is sole (a) Training in grammar
authority figure and leads the lesson. The class (b) Imitation and reinforcement
activities are visible. (c) Innate abilities
2. The theory of Behaviourism in a classroom setting (d) Maturation
came from pedagogical research by Thorndike, 16. Techno-Pedagogic competency is
Pavlov and Skinner. (a) A science of using technology in teaching.
3. In behaviourist pedagogical approach, we can (b) A technique of combining principles of technol-
expect a mixture of lecturing, modelling and dem- ogy and principles of teaching.
onstration, rote learning, and choral repetition. (c) A set of skills of interweaving technology into
4. The process of imbibing one’s own culture in one’s teaching and learning both scientifically and
personality is called as enculturation. aesthetically.
Codes: (d) A competence to develop techno pedagogic sys-
(a) 1, 2, 3 and 4 tems in education.
(b) 3 and 4 17. Which of the following can be taken as the main
(c) 2 and 4 approaches in andragogy?
(d) 1, 2 and 3 1. The focus of andragogical approach is on self-
12. An almost simultaneous occurrence of the stimuli and directed, cooperative and two-way learning process.
of the responses to them, it is called as 2. In andragogy, motivation for learners is based on
(a) Generalisation and discrimination intrinsic factors such as; self-esteem, self-confi-
(b) Contiguity dence, desire for the better quality of life, curios-
(c) Reinforcement ity, self-development etc.
(d) Practice 3. Andragogy is motivational approach.
13. Which of the following chains represents the change Codes:
processes underlying educational system of Free (a) 1 and 2 (b) 2 and3
India? (c) 1 and 3 (d) 1, 2 and 3
(a) Psychological change – Social change – Political 18. Match the following.
change – Educational change.
(b) Political change – Social change – Psychological Type of variable Basic Feature
change – Educational change.
(A) Independent Variable (i) Predictor variable
(c) Social change – Psychological change – Political
(B) Dependent Variable (ii) Tells relationship
A S S E S S YO U R L E A R N I N G

change – Educational change.


(d) Educational change – Social change – Psychological between two
change. variables
14. According to Kolb (1984), match the learning style (C) Intervening Variable (iii) Outcome variable
with their respective characteristics as given in List-I
and List-II [December 2021] (a) A–(ii), B–(iii), C–(i)
(b) A–(i), B–(iii), C–(ii)
List-I List-II (c) A–(iii), B–(i), C–(ii)
(Learning Style) (Characteristics) (d) A–(i), B–(ii), C–(iii)
(A) Accommodators (i) Real life experience 19. Match List-I with List-II.
and discussion
List-I List-II
(B) Converger (ii) Theories and facts (Teaching maxims) (Main proponents)
(C) Diverger (iii) Hands-on learning (A) From whole to part (i) Gestalt
(D) Assimilator (iv) Hands-on learning psychologists
and theory (B) Self-study (ii) Dalton
Choose the correct answer from the options given (C) Training of senses (iii) Montessori and
below: Fröbel
Codes: Codes:
(a) (A)–(iv), (B)–(ii), (C)–(i), (D)–(iii) (a) (A)–(i), (B)–(ii), (C)–(iii)
(b) (A)–(iii), (B)–(i), (C)–(iv), (D)–(ii) (b) (A)–(i), (B)–(iii), (C)–(ii)
(c) (A)–(i), (B)–(iii), (C)–(ii), (D)–(iv) (c) (A)–(ii), (B)–(iii), (C)–(i)
(d) (A)–(iii), (B)–(iv), (C)–(i), (D)–(ii) (d) (A)–(ii), (B)–(i), (C)–(iii)

M01_MADAN 07_65901_C01.indd 62 27/12/22 8:15 PM


Teaching Aptitude 1.63

20. The principle of ‘the child, the school and education (a) Learning emerges from the sequence of events in
itself are shaped largely by social and cultural forces’ a class.
was enunciated by [June 2019] (b) Learning occurs through the search for epistemo-
(a) reconstructionism (b) existentialism logical structure of understanding.
(c) pragmatism (d) perennialism (c) Learning is contingent upon the socio-emotional
21. Who among the following Indian thinkers has con- environment in the class.
tributed towards the concept of integral education? (d) Learning is perceived as a change in behaviour as
 [June 2019] a result of teacher motivation.
(a) Gijubhai Badhera 29. A teacher gives lot of positive and negative examples
(b) Swami Vivekananda to support his/her presentations in the classroom.
(c) Mahatma Gandhi This will be related to which level of teaching?
(d) Sir Aurobindo  [June 2019]
22. Pitch of the teacher’s voice in the classroom is (a) Autonomous development level.
described as [December 2019] (b) Memory level.
(a) Linguistic (b) Paralinguistic (c) Understanding level.
(c) Non-linguistic (d) Macro-linguistic (d) Reflective level.
23. In developing seven ‘multiple intelligences’, Gardner 30. Which of the following statements are true in context
made some important observations. Identify such of teaching?
observations from the list given below: [June 2019] 1. The method mainly used by a teacher is
1. Intelligence in not entirely genetic analytical-synthetic.
2. Intelligence is a part of acquired behaviour 2. The books are good source if contents are illustrative.
3. Intelligence is not fixed at birth 3. The objective of teacher should be to develop an
4. There is little or no hope for developing intelligence inquiring mind.
5. Intelligence can be nurtured and grown 4. A teacher motivates students for self learning
Give your answer by selecting from the options given 5. A teacher should be compassionate and disciplinarian
below: Codes:
(a) 1, 2 and 4 (b) 1, 3 and 5 (a) 1, 2 and 3
(c) 3, 4 and 5 (d) 1, 4 and 5 (b) 2, 3 and 5
24. From the list of the effective teaching behaviours. (c) 1, 3 and 4
1. Direct, audible and oral delivery to all students (d) 1, 2, 3, 4 and 5
2. Encouraging students to elaborate on an answer 31. Effectiveness of teaching has to be judged in terms of
3. Intelligence is not fixed at birth  [January 2017]

A S S E S S YO U R L E A R N I N G
4. Varying modes of presentation (a) Course coverage
5. Preventing misbehaviour with a minimum of class (b) Students’ interest
disruption (c) Learning outcomes of students
6. Organising what is to came and summarising (d) Use of teaching aids in the classroom
what has gone before 32. In emphasis in memory level, teaching is on which of
Select your answer from the options given below: the following? [December 2019]
(a) 1, 4 and 5 (b) 1, 2 and 3 1. Organisation of thoughts and ideas
(c) 2, 3 and 4 (d) 4, 5 and 6 2. Seeing of relationship among facts
25. What do we mean by curriculum? 3. Systematic presentations to enable quick
(a) All the experiences which students get in school. reproduction
(b) Subject that are transferred by the faculty. 4. Critical thinking
(c) Syllabus prescribed for the course. 5. Mastery of concept sequencing of facts
(d) Class experiences, sports and games. 6. Segregating one feature from another
26. Family is the main agency of Choose the answer from the following options
(a) Informal education (a) 1, 2 and 3 (b) 1, 3 and 5
(b) Formal education (c) 2, 3 and 4 (d) 4, 5 and 6
(c) Non-formal education
33. Given below are two statements [December 2021]
(d) None of the above
Statement I: The core of the teaching process is the
27. Delinquency is committed by the children of the age arrangement of environments within which stu-
group dents can interact and study how to learn.
(a) 8 – 18 years (b) 6 – 14 years Statement II: A model of teaching is not a descrip-
(c) 7 – 15 years (d) 9 – 19 years tion of a learning environment.
28. Five E’s instructional model of constructivism has In light of the above statements, choose the most
been developed on the basis of which of the following appropriate answer from the options given below:
theoretical basis of learning? [June 2019]

M01_MADAN 07_65901_C01.indd 63 27/12/22 8:15 PM


1.64 Chapter 1

Codes: 37. The order of activity in advance organiser model of


(a) Both Statement I and Statement II are correct. teaching is: [December 2021]
(b) Both Statement I and Statement II are incorrect. A. presentation of learning task/material
(c) Statement I is correct but Statement II is incorrect. B. strengthening cognitive organisation
(d) Statement I is incorrect but Statement II is correct. C. presentation of advance organiser
34. Which among the following constitute personal D. clarification of students problems
competence? Choose the correct answer from the options given
1. Empathy below:
2. Motivation (a) A, B, D, C (b) C, A, B, D
3. Social skills (c) C, B, A, D (d) B, A, C, D
4. Self awareness 38. Which of the following are the highest taxonomic cat-
5. Self regulation egories in the cognitive, affective and psychomotor
Choose the correct answer from the options given domains of learning? [December 2019]
below: A. Analysis
Codes: B. Evaluation
(a) 1, 2 and 3 only (b) 2, 3 and 4 only C. Awareness
(c) 2, 4 and 5 only (d) 3, 4 and 5 only D. Characterization
35. As suggested by UGC, Student Induction Programme, E. Naturalization
also called as ‘deeksharambh’ engages with the new Choose the correct answer from the options given below:
students as soon as they come into the institution, (a) A, B and C only (b) B, D and E only
before regular classes start. The students learn about (c) B, C and D only (d) C, D and E only
the institutional policies, processes, practices, culture 39. The variability or flexibility of delivery during the
and values, and their mentor groups are formed. presentation of a lesson by a teacher is called as
Which of the following policies are covered under  [December 2021]
‘deeksharambh’ ? (a) Thinking curriculum
(1) Socializing (b) Instructional variety
(2) Associating (c) Comprehension
(3) Governing (d) Procedural knowledge
(4) Experiencing 40. Which of the following statements are true about
(5) Small-group activities coherent curriculum?
Codes: 1. It is well organized and purposefully designed to
(a) 1, 2 and 4 (b) 2, 3 and 5 facilitate learning
(c) 3, 4 and 5 (d) 1, 2, 3, 4 and 5
A S S E S S YO U R L E A R N I N G

2. It is free of academic gaps and needless repetitions


36. Match List-I with List-II. [December 2021] 3. It is aligned across lessons, courses, subject areas,
and grade levels
List-I List-II Codes:
(Training types) (Major Objective) (a) 1 and 3 (b) 1 and 2
(c) 2 and 3 (d) 1, 2 and 3
(A) Pre-service (i) acquaint new entrants 41. Which of the following sequences correctly reflects
teacher training with duty and Kolb’s experiential teaching-learning? [June 2019]
responsibility (a) Think, do, feel and watch
(B) Practice (ii) upgradation of (b) Watch, feel, do and think
teaching knowledge (c) Do, watch, feel and think
(C) Induction (iii) to develop teaching skills (d) Feel, watch, think and do
training 42. Which of the following is not a characteristic of a good
question paper? [July 2022]
(D) In-service (iv) understanding aims & (a) Subjectivity
teacher training objectives of education (b) Objectivity
(c) Reliability
Choose the correct answer from the options given (d) Validation
below: 43. Consider the following assertion and reason.
Codes:
(a) (A)–(i), (B)–(ii), (C)–(iii) Assertion (A): Pedagogy is defined simply as the
(b) (A)–(i), (B)–(iii), (C)–(ii) method, and practice, of teaching.
(c) (A)–(ii), (B)–(iii), (C)–(i) Reason (R): The delivery of contents by a teacher
(d) (A)–(ii), (B)–(i), (C)–(iii) depends upon his teaching preferences, his experi-
ence, and the context that he teaches in.

M01_MADAN 07_65901_C01.indd 64 27/12/22 8:15 PM


Teaching Aptitude 1.65

(a) A is true and R is also true, and R explains the A (b) creating, remembering, understanding, evaluat-
correctly ing, applying, and analyzing
(b) Both A and R are correct, but R is not the exact (c) 
understanding, remembering, applying, analyz-
explanation of A ing, evaluating, and creating
(c) Only A is correct (d) 
remembering, applying, understanding, analyz-
(d) Only R is correct ing, evaluating, and creating
44. In context of dynamic teaching environment, which 51. Epistemology is the branch of philosophy which deals
of the following statement is true? with the theories of
(a) Teacher is a dependent variable and student is (a) Reality (b) Existence
an independent variable. (c) Knowledge (d) Values
(b) Teacher is an independent variable and student
is a dependent variable. 52. Match List-I with List-II
(c) Both teacher and student are intervening variables.
List-I List-II
(d) Teaching is not linked with any variable.
(Characteristic) (Philosophy)
45. At which of the following teaching levels is classroom (A) Learning by doing (i) Naturalism
environment required to be sufficiently ‘open and
independent’? (B) Education through (ii) Idealism
(a) Memory level (b) Understanding level environment
(c) Reflective level (d) All of the above (C) Realization of truth, beauty, (iii) Pragmatism
46. Dynamic approach to teaching means and goodness
(a) Teaching should be lecture based (D) ‘World as it is here and now’ (iv) Realism
(b) Teacher should be energetic
(c) Teacher should be student friendly Codes:
(d) Student should learn through activities (a) A–(i), B–(iv), C–(ii), D–(iii)
47. Assertion (A): Emotionally intelligent children are (b) A–(iii), B–(i), C–(ii), D–(iv)
found to be good in problem solving. [June 2019] (c) A–(i), B–(iii), C–(ii), D–(iv)
Reason (R): Emotional intelligence implies the (d) A–(iii), B–(ii), C–(iv), D–(i)
capacity to integrate and manage the emotion. 53. Constructivist approach to teaching – learning pro-
In the light of the above statements choose the cor- cess is being emphasized at all levels of education.
rect option:
(a) Both (A) and (R) are correct, and (R) is the cor- Which of the following methods is mainly based on
rect explanation of (A). this approach?

A S S E S S YO U R L E A R N I N G
(b) Both (A) and (R) are correct, but (R) is not the (a) Lecture Method (b) Project Method
correct explanation of (A). (c) Discussion Method (d) All of the above
(c) (A) is true, but (R) is false. 54. Which of the following assists students in managing
(d) (A) is false, but (R) is true their own learning by modeling a problem solving
48. Which of the strategies in the list given below will be technique?
associated with direct teaching? (a) Drill and Practice (b) Mental Modeling
1. Providing example and explanation (c) Discovery Learning (d) Direct instruction
2. Promoting inquiry 55. The students or individuals may develop beliefs (posi-
3. Focusing on concepts tive or negative) about their own ability to cope effec-
4. Giving review and recapitulation tively in a variety of situations. This can be termed as
5. Offering practice and feedback (a) Self-efficacy (b) Self-regulation
6. Problem raising and problem-solving (c) Ego (d) Confidence
Choose the correct answer from the options given below: 56. When the learning outcomes of students are ensured
(a) 1, 2 and 3 (b) 2, 3 and 4 with employment of less resources and effort on the
(c) 1, 4 and 5 (d) 4, 5 and 6 part of a teacher and more initiative for self-learning
49. To make use of previously learned material in new is evident, which of the following expression will
situation is describe the teacher most appropriately?
 [June 2019]
(a) Comprehension (b) Application
(a) Teacher is successful
(c) Knowledge (d) Analysis (b) Teacher is effective
50. There are six levels of cognitive learning according to (c) Teacher is intelligent
the revised version of Bloom’s Taxonomy. Each level is (d) Teacher is practical
conceptually different. The six levels are 57. Which among the following is NOT a component of
(a) 
remembering, understanding, applying, analyz- effective teaching? [December 2019]
ing, evaluating, and creating

M01_MADAN 07_65901_C01.indd 65 27/12/22 8:15 PM


1.66 Chapter 1

(a) Questioning to check for the understanding of Select your answer from the options given below:
students (a) 1, 2 and 3 (b) 3, 4 and 5
(b) Providing students with feedback (c) 2, 3 and 5 (d) 4, 5 and 6
(c) Being flexible about how long it takes to learn
(d) Reactive mode of student’s behaviour 63. Which among the following best describes the emo-
tional intelligence of learners?
58. Identify the factors that do not contribute to the effec-
tiveness of teaching from the options given below: 1. Understand the emotions of other people and
 [December 2020] your own
1. Socio–economic background of Teacher 2. Aggressive expression of feelings
2. Teacher’s skill in pleasing the students 3. Human beings are simply information processors
3. Teacher’s subject knowledge 4. Self regulation as per the situation
4. Teacher’s personal contact with students 5. Being creative and open to criticism
6. Empathy
Choose the correct option:
(a) 1, 3 and 4 (b) 2, 3 and 4 Choose your answer from the options given below:
(c) 1, 2 and 3 (d) 1, 2 and 4 (a) 1, 4, 5, 6 (b) 3, 4 and 5
59. Which of the following skills refer to metacognition (c) 1, 2 and 3 (d) 2, 3 and 4
skills? 64. Which of the following potential factors affecting
1. It is thinking about thinking. teaching are related to instructional facilities and
2. It enhances a student’s learning for immediate learning environment? [June 2020]
outcomes and for understanding own learning. 1. Praise and encouragement by teacher‘
3. It is a teachable skill that is central to other skills 2. Non availability of text books / reading materials
sets such as problem solving, decision-making, for use
and critical thinking. 3. Proactive moves in the interactive process
4. It is knowing your own strengths and weaknesses 4. Shortage of highly qualified teachers
as a learner. 5. Availability of smart classes
(a) Only 1, 2 and 3 (b) Only 2, 3 and 4
Choose the correct answers from the following options
(c) Only 1, 2 and 4 (d) All of the above
(a) 1, 2 and 3 (b) 2, 3 and 5
60. Choose two factors that do NOT affect the academic
performance of students adversely: (c) 1, 4 and 5 (d) 3, 4 and 5
 [December 2019] 65. The phrase ‘giving personal attention to each student
1. Low self– efficacy belief individually’ is an example of. [January 2018]
2. Belief in God (a) succinctness
A S S E S S YO U R L E A R N I N G

3. Indifferent attitude towards politics (b) circumlocution


4. Teacher’s low expectation from students (c) wordiness
Choose the correct option: (d) tautology
(a) 1 and 2 only (b) 2 and 3 only 66. Assertion (A): All classrooms communication is pre-
(c) 3 and 4 only (d) 2 and 4 only decided by social and institutional demands.
61. Identify the characteristics of a learner that help in Reason (R): Positive educative actions should guide
the effective teaching. [June 2019] learners to acquire knowledge of social significance.
Choose the correct answer from the following code:
1. The overall respect for a teacher
(a) Both (A) and (R) are true, and (R) is the correct
2. The mental ability level of the learner
explanation of (A).
3. The previous experiences of the learner
(b) Both (A) and (R) are true, but (R) is not the cor-
4. The interest of the learner
rect explanation of (A).
5. The interpersonal relation of the learner
(c) (A) is true, but (R) is false.
6. The societal perspective of the learner
(d) (A) is false, but (R) is true.
Select your answer from the options given below:
67. Which of the following statements is true in the case
(a) 1, 2, 3 and 4 (b) 3, 4, 5 and 6 of a test prepared by you as a teacher?
(c) 1, 2, 3 and 6 (d) 2, 3, 4 and 6 (a) If a test is reliable, it is objective
62. Identify the teaching methods that are relevant for (b) If a test is valid, it is reliable also
dialogic discourses. (c) If a test is reliable, it is valid
1. Formal Lecture (d) If a test is valid and reliable, it is usable
2. Team teaching 68. Consider the following statements.
3. Tutorials (1) It is the use of straightforward, explicit teaching
4. Project methods techniques, usually to teach a specific skill.
5. Group discussions (2) It is a teacher-directed method.

M01_MADAN 07_65901_C01.indd 66 27/12/22 8:15 PM


Teaching Aptitude 1.67

Which of the following teaching methods is being dis- (3) It affects areas of the brain that process language.
cussed here? The choices are
(a) Discovery learning
(a) Alzheimer’s disease
(b) Constructivist approach
(b) Dyslexia
(c) Direct instruction
(c) Parkinson’s disease
(d) Hit and trial teaching
(d) None of the above
69. Which among the following bases can be counted to
75. The theory of multiple intelligences differentiates
judge the effectiveness of teaching? [June 2019]
human intelligence into specific ‘modalities’, rather
(A) Marks obtained by students in overall subjects
than seeing intelligence as dominated by a single gen-
(B) Level of realization of instructional objectives
eral ability. Who among the following developed the
(C) Time taken by the students to learn
theory of ‘Multiple Intelligence’?
(D) Resources used by the teacher in teaching
(E) Regularity of students in class (a) Alfred Binet
(b) L. Thurstone
Select your answer from the options given below: (c) Charles Spearman
(a) (A), (B) and (C) (d) Howard Gardner
(b) (A), (B) and (D) 76. Which of the following statement explain the con-
(c) (C), (D) and (E) cepts of inclusive teaching?
(d) (B), (C) and (D)
(a) Teacher facilitates the learning of the gifted students
70. Who among the following has given the concept of (b) Teacher facilitates the learning of the weak students
masterly learning? (c) Teacher takes support of parents of the student to
(a) Carlton Washburne (b) Albert Bandura make them learn
(c) David Ausubel (d) Edward Thorndike (d) Teacher makes the student of different back-
71. Consider the following statements: ground to learn together in the same class
(1) The theory is used for teaching abstract relationship. 77. Which of the following applies in the case of discovery
(2) The theory helps students to retain important & learning?
large information for longer duration.
(3) The theory was discovered by David Ausubel. 1. It is a constructivist approach based
2. It was introduced by Jerome Bruner
Identify the theory. 3. It is a method of Inquiry-Based Instruction
(a) Theory of mastery learning Codes:
(b) Theory of receptive learning (a) 1 and 3 (b) 2 and 3
(c) Theory of social learning (c) 1 and 2 (d) 1, 2 and 3
(d) Theory of discovery learning

A S S E S S YO U R L E A R N I N G
78. As learning is continuous, a teacher must start teach-
72. Consider the following statements. ing at the level of students’ mind for their better learn-
(1) Learning is an active process ing. This is called as
(2) Learners construct new ideas or concepts based
upon their current or past knowledge. (a) Principle of Clarity
(3) This is basically a constructivist approach. (b) Principle of Association
(c) Principle of Active Process
Which of the following theories is according to these (d) Principle of Multiple Exposure
statements?
79 The rate of learning declines at the rate of about one
(a) Receptive learning by David Ausubel percent a year after the age of
(b) Social learning theory Albert Bandura
(c) Mastery learning by Carlton Washburne (a) 30 years (b) 35 years
(d) Discovery Learning by Jerome. S. Brune (c) 42 years (d) 45 years
73 Which of the following disease shows the irreversible, 80. Which of the following are the methods of learning?
progressive brain disorder that slowly destroys mem- 1. Teaching
ory and thinking skills and, eventually, the ability to 2. Trial and Error
carry out the simplest tasks? 3. Imitation
(a) Alzheimer’s disease (b) Dyslexia 4. Experimentation
(c) Parkinson’s disease (d) None of the above 5. Questioning
6. Reflection
74. Which of the diseases refer to following problems?
Codes:
(1) A learning disorder that involves difficulty in read-
ing due to problems identifying speech sounds. (a) 1, 2, 3 and 5 (b) 2, 3, 4 and 6
(2) It deals with learning how they relate to letters (c) 1, 3 and 5 (d) All of the above
and words that is decoding.

M01_MADAN 07_65901_C01.indd 67 27/12/22 8:15 PM


1.68 Chapter 1

81. From the list of the effective teaching behaviours, Codes:


identify those which are called key behaviours. (a) (A)–(i), (B)–(ii), (C)–(iii)
1. Direct, audible and oral delivery to all students (b) (A)–(ii), (B)–(i), (C)–(iii)
2. Encouraging students to elaborate on an answer (c) (A)–(ii), (B)–(iii), (C)–(i)
3. Warm and nurturing relationships with learners (d) (A)–(iii), (B)–(ii), (C)–(i)
4. Varying modes of presentation 86. Which of the following statements applies in the
5. Preventing misbehaviour with a minimum of context of online teaching methods?
class disruption 1. Teacher does not help in construction of knowledge
6. Organizing what is to come and summarizing 2. There is more role of learners in the instructional
what has gone before process
3. The delivery of instruction is always verbal
Select your answer from the options given below:
4. There are more chances of self-learning and
(a) 1, 4 and 5 (b) 1, 2 and 3
motivation
(c) 2, 3 and 4 (d) 4, 5 and 6
5. It is rigid in nature
82. From the list given below, identify those questions Codes:
which are called process rather than content based (a) 1, 2 and 4 (b) 2 and 4
questions? [June 2019] (c) 2, 3 and 5 (d) 1, 3 and 5
1. Convergent questions
87. Which of the following statements is/are correct in
2. Divergent questions
the context of learning process?
3. Fact based questions
(a) The context can be both field independent and
4. Concept based questions
field dependent
5. Open questions
(b) The learners are always reflective in nature
6. Closed questions
(c) There is no scope for practical thinkers
Choose the correct answer from the options given (d) All of the above
below:
88. Which of the following can be described as the emo-
(a) 1, 2 and 3 (b) 2, 3 and 4
tional changes during adolescence stage?
(c) 4, 5 and 6 (d) 2, 3 and 5
1. It is stage of conscience formation
83. Which of the learner characteristics will influence 2. It is invincible stage of thinking and acting
his/her perspective in a course of study? 3. The genetic changes to environmental factors
1. Learner’s commitment are called as a differential susceptibility model
2. Parents interest in the learner 4. There is reawakening of libido
3. Prior knowledge of the learner 5. The outcomes are worse for girls than in case of
4. Skill of the learner in the concerned area boys
A S S E S S YO U R L E A R N I N G

5. Family size of the learner Codes:


6. Socio-economic background of the family to (a) Only 1, 2 and 3
which the learner belongs (b) Only 2, 3 and 4
(c) Only 1, 3 and 5
Choose the correct answer from the options given (d) All of the above
below:
89. Which of the following stages have been defined by
(a) 1, 3 and 4 Piaget for the cognitive development in the ascend-
(b) 1, 2 and 3 ing order?
(c) 3, 4 and 5 (a) Sensory period, Pre-operational Period,
(d) 4, 5 and 6 Concrete operation period, Formal Operation
84. Teaching can be defined as period
(a) Bipolar process (b) Pre-operational Period, Sensory period,
(b) Tripolar process Concrete operation period, Formal Operation
(c) Quadrilateral process period
(d) All of the above (c) Sensory period, Concrete operation period, Pre-
85. Match the following items operational Period, Formal Operation period
(d) Sensory period, Formal Operation period,
Development of skill Description ­Pre-operational Period, Concrete operation period
(A) Cognitive Stage   (i) combining steps into 90. During which of the following stages, the purpose
larger units of process of adjustment is to bring about a ‘State of
(B) Associative Stage (ii) declarative ­Equilibrium’ in the life of individuals is more specific?
knowledge (a) Emotional Changes
(b) Cognitive development
(C) Automated Stage (iii) without much
(c) Social changes
attention
(d) Physical changes

M01_MADAN 07_65901_C01.indd 68 27/12/22 8:15 PM


Teaching Aptitude 1.69

91. Which of the following changes are more prominent 97. Consider the following statements:
during the social changes of adolescence? 1. The curriculum covers the whole spectrum of
(a) It is period of storm and stress during transition teaching-learning activities in the educational
­
(b) There is feeling of homophily as there is mix up institution.
transition from single sex to mixed sex. 2. The syllabus is not restricted to examinable por-
(c) There is deviant peer contagion that may show tions of the curriculum
other signs of approval Which of the above is/are true?
(d) All of the above Codes:
92. Which of the following types of evaluation applies (a) Only 1 (b) Only 2
more during adult stage? (c) Both 1 and 2 (d) None of the above
1. By learner-collected evidence validated by peers, 98. Consider the following statements
facilitators, experts 1. It is self determined learning
2. It is basically criterion-referenced 2. It is student centered instructional strategy
Codes: 3. It emphasize the development of autonomy, capac-
(a) Only 1 (b) Only 2 ity, and capability
(c) Both 1 and 2 (d) None of the above The following teaching strategy is indicated as a result
93. Consider the following statements: of these statements.
1. A problem is identified. (a) Pedagogy (b) Heutagogy
2. Small groups are formed. (c) Andragogy (d) Tautalogy
3. All members are encouraged to find the solution 99. Match List-I with List-II.
and express their ideas.
4. No idea is criticized, however ideas can be modified. List-I List-II
Which of the following teaching techniques is being (Teaching maxims) (Main proponents)
referred here? (A) From whole to part (i) Gestalt
(a) Brainstorming (b) Task Group psychologists
(c) Inquiry Group (d) Role Play
(B) Self-study (ii) Dalton
94. Microteaching is a technique aiming to prepare
teacher candidates to the real classroom setting. (C) Training of senses (iii) Montessori and
Which of the following is correct order of phases of Fröbel
microteaching?
Codes:
1. Knowledge acquisition Phase
(a) (A)–(i), (B)–(ii), (C)–(iii)
2. Skill acquisition Phase
(b) (A)–(i), (B)–(iii), (C)–(ii)
3. Transfer Phase

A S S E S S YO U R L E A R N I N G
(c) (A)–(ii), (B)–(iii), (C)–(i)
Codes:
(d) (A)–(ii), (B)–(i), (C)–(iii)
(a) 1, 2, 3 (b) 1, 3, 2
(c) 2, 1, 3 (d) 3, 1, 2 100. Match The Following List.
95. From the following list of statements, select those
Bloom’s Classification Main Characteristic
which indicate the characteristics and basic require-
ments of effective teaching. (A) Cognitive Domain (i) Technical Skills
1. Teaching effectiveness depends upon (B) Affective Domain (ii) Love, motivation
communication and attitude
2. Effective teaching entails ability to develop rela-
tionships with their students. (C) Psycho-motor (iii) Describe a specific
3. An effective teacher has to be a good seller of ideas Domain topic
4. Effective teaching is always formal in nature
5. Effective teacher who are most successful in help- Codes:
ing students to learn (a) A–(iii), B–(i), C–(ii)
Choose the correct answer from the code given (b) A–(i), B–(ii), C–(iii)
below: (c) A–(iii), B–(ii), C–(i)
(a) 1, 2, 3 and 5 (b) 2, 3 and 4 (d) A–(ii), B–(iii), C–(i)
(c) 2, 3 and 5 (d) 1, 2 and 5 101. Which of the following is/are the teaching maxims?
96. When presenting materials, which of the following 1. From psychological to logical
should the teachers consider? 2. From analysis to synthesis
(a) Structuring (b) Balancing 3. From concrete to abstract
(c) Motivation (d) All of the above 4. Follow nature
(a) 1, 2 and 3 (b) 1, 2 and 4
(c) 1, 3 and 4 (d) All of the above

M01_MADAN 07_65901_C01.indd 69 27/12/22 8:15 PM


1.70 Chapter 1

102. Analysis means (c) Mastery over appropriate use of media and tech-
(a) Ability to break a problem into its constituent nology in teaching.
parts (d) All of the above
(b) To combine the constituent parts 109. Which of the following activities can help a teacher
(c) Both (a) and (b) inculcate social and moral values among the
(d) It is always used in combination with synthesis students?
103. The rules of presenting the contents to make them (a) Deliver lectures on values
easy are called (b) Show TV programmes
(a) Methods of teaching (c) Involve students actively in co-curricular activities
(b) Maxims of teaching (d) Observe religious festivals
(c) Techniques of teaching 110. According to the revised Bloom’s taxonomy, which
(d) Teaching strategies of the following categories is considered to be high-
104. With smaller classes, teachers are much more able to est for the learner? [July 2022]
(a) Identify learning problems (a) Understand (b) Create
(b) Provide individual attention (c) Analyse (d) Apply
(c) Adapt instruction to individual differences 111. Following are the characteristics of non–conventional
among students. learning [July 2022]
(d) All of the above 1. It is teacher oriented
105. The single most important factor in the beginning of 2. It is for improvement of quality
the teaching career is 3. It is cost effective
(a) Meritorious academic record 4. It is linked to employment
(b) Communication skills 5. It is on campus
(c) One’s personality and ability to relate to class Choose the correct answer from the options given
and students below:
(d) Organizing ability
(a) 1, 2 and 3 (b) 1, 3 and 4
106. In List-I, the names of the philosophers are given and (c) 2, 3 and 4 (d) 3, 4 and 5
in List-II, the statements regarding the philosophy
112. Which of the following is the least important aspect
are given. Match List-I with List-II in correct order.
of the teacher’s role in the guidance of learning?
List-I List-II (a) The development of insight to overcome pitfalls
(Characteristic) (Philosophy) and obstacles.
(b) The development of insight into what consti-
(A) Tagore (i) Harmonious development tutes an adequate performance.
A S S E S S YO U R L E A R N I N G

of personality (c) The provision of encouragement and moral


(B) Vivekananda (ii) The doctrine of karma support.
(d) The provision of continuous diagnostic and
(C) Mahatma (iii) Child is more important
remedial help.
Gandhi than all kinds of books
113. From the following list of statements, select those
(D) Buddhism (iv) Yoga as a method of
which indicate the characteristics and basic require-
education
ments of teaching.
(v) The doctrine of dharma 1. Effective teaching must include some feedback
Codes: 2. Teaching means organizing and supervising.
(a) A–(v), B–(i), C–(ii), D–(iv) 3. Teaching implies making others learn.
4. Teaching requires convincing others.
(b) A–(iii), B–(iv), C–(ii), D–(v)
5. There can be teaching without formal
(c) A–(iii), B–(iv), C–(i), D–(ii) communication
(d) A–(iv), B–(ii), C–(i), D–(iii) (a) 1, 2, 3 and 5 (b) 1, 3 and 5
107. Who said that ‘Education is man-making. It is that (c) 2, 3, 4 and 5 (d) All of the above
by which character is formed, strength of mind is 114. Which of the following are the underlying purposes
increased, intellect is expanded and by which man of teaching aids?
can stand on his own feet’? 1. To motivate students to learn
(a) Swami Vivekananda 2. Teaching aids are helpful in meeting individual
(b) Rabindranath Tagore differences
(c) Swami Dayanand Saraswati 3. Teaching aids makes the abstract ideas concrete
(d) Jawahar Lal Nehru 4. To make learning permanent
108. The basic requirement of teaching efficiency is Codes:
(a) Mastery on teaching skills (a) 1, 2 and 3 (b) 2, 3 and 4
(b) Mastery over use of different techniques of (c) 1, 3 and 4 (d) All of the above
teaching.

M01_MADAN 07_65901_C01.indd 70 27/12/22 8:15 PM


Teaching Aptitude 1.71

115. Arrange the following activities of classroom teach- (a) (i), (iv), (v), (ii), (iii)
ing in a logical order. (b) (iv), (v), (i), (ii), (iii)
1. Analysis of the work done (c) (iv), (i), (ii), (v), (iii)
2. Planning and preparation (d) (iv), (i), (v), (ii), (iii)
3. Presentation of material 122. Which one of the following is a product of learning?
4. Modification and improvement (a) Intelligence (b) Maturity
Codes: (c) Skills (d) Memory
(a) 2, 3, 1 and 4 (b) 3, 2, 1 and 4
123. Match List I with List II [July 2022]
(c) 2, 3, 4 and 1 (d) 2, 1, 3 and 4
116. In intuitive thinking, the learners typically focus on List I List II
one characteristic of someone or something, and (Concepts) (Main proponent)
base their decisions or judgment on that one charac-
teristic rather than considering multiple characteris- (A) I ntegral Education I. John Dewey
tics. This is called as (B) F
 ocus on spiritual II. Sri Aurobindo
(a) Logical thinking (b) De-centering aspects of Indian
(c) Centration (d) None of the above philosophy
117. Consider the following terms:
1. Logical Thinking (C) Learning to take III. D
 r. Sarvepalli
2. Critical Thinking place in nature and Radhakrishnan
3. Objectivity from nature
4. Suscepticism (D) Experiential IV.  Rabindranath
5. Preservance Learning Tagore
Which of the above are required for development of
scientific attitude and scientific temper among the Choose the correct answer from the options given
learners? below.
(a) 2, 3, 4 and 5 (b) 1, 3, 4 and 5 (a) A-II, B-IV, C-III, D-I
(c) 1, 2, 3 and 5 (d) 1, 2, 3, 4 and 5 (b) A-I, B-II, C-III, D-IV
118. Cognitive domain is related to development of intel- (c) A-III, B-II, C-I, D-IV
lectual capabilities of learners that functions at dif- (d) A-II, B-III, C-IV, D-I
ferent levels. Which of the following level is about 124. The following are the main objectives of curriculum
making use of previously learned material in new revision: [July 2022]
situation according to Bloom’s Classification of A. To align it better with vision and mission of the
Teaching and Instructional objectives? institution

A S S E S S YO U R L E A R N I N G
(a) Knowledge (b) Comprehension B. To make it learning outcome based
(c) Application (d) Analysis C. To make it more effective in improving attain-
119. Reinforcement is provided by any factor that increases ment levels against programme outcomes
the probability that a response will be repeated. Which D. To develop capacities of students for competitive
of the following can be factor(s) of reinforcement? examination
(a) Praise E. To meet the expectations of all stakeholders
(b) Token award Choose the correct answer from the options given
(c) Simply succeeding in a task below:
(d) All of the above (a) A, B, C and D only
120. While comparing hearing and listening, we can say that (b) A, B, C and E only
(a) Hearing is a physical process; listening is a psy- (c) A, C, D and E only
chological process (d) B, C, D and E only
(b) Listening is a physical process; hearing is a psy- 125. The ability to locate, evaluate, and effectively use
chological process information is an important trait known as
(c) Both are purely physical processes (a) Critical thinking
(d) Both are purely biological processes (b) Information literacy
121. Arrange the following teaching process in order (c) Hearing
  (i) Relating the present knowledge with the previ- (d) Selective attention
ous knowledge 126. Consider the following statements:
(ii) Evaluation 1. The curriculum covers the whole spectrum of
(iii) Re-teaching teaching-learning activities in the school
  (iv) Formulation of teaching objectives 2. The syllabus is not restricted to examinable por-
(v) Presentation of materials tions of the curriculum

M01_MADAN 07_65901_C01.indd 71 27/12/22 8:15 PM


1.72 Chapter 1

Which of the following statement/s of the above is / 135. What are the components of Tyler’s model of
are true? curriculum?
(a) Only 1 (b) Only 2 (a) Aims, subject content, teaching, evaluation
(c) Both 1 and 2 (d) Neither 1 nor 2 (b) Purpose, educational experiences, effective
127. Which theory of learning has found that knowledge organization of experiences, verification of goal
of internal processes is crucial for the understanding (c) Aims of education, organization of content, test-
of learning? ing, feedback
(a) Cognitive Theory (d) Subject content, teaching, learning, testing
(b) Stimulus–response theorists 136. The experienced teachers do not require the detailed
(c) Operant conditioning theorists lesson plan of a topic because
(d) Classical conditioning theorists (a) They can teach more effectively without its help
128. Arrange the following activities of interaction in log- (b) There are just few curious students in the class
ical order. (c) The teacher is not likely to face any challenges
1. Analysis of the work done from students even if they are wrong
2. Planning and preparation (d) They can equip themselves with brief outline as
3. Presentation of material they gain specialization in it through experience
4. Modification and improvement 137. The field of education is permeated by conflicts and
(a) 1, 2, 3 and 4 (b) 2, 3, 1 and 4 misconceptions because of the
(c) 4, 1, 2 and 3 (d) 1, 3, 4 and 2 (a) Subjectivity of interpretation
129. Effective teaching means all of the following except (b) Problems that are not amenable to rigorous sci-
(a) A teacher teaches with enthusiasm entific investigation
(b) A teacher finds fault in his students (c) Lack of good teaching methods
(c) A teacher puts emphasis more on teaching than (d) All of the above
on class control 138. With the development of technology, the role of a
(d) A teacher is interested in making the subject teacher in future will be
matter understood rather than on completing (a) To provide information
the course (b) To develop new textbooks
130. If some students fail in the examination, it is the (c) To guide students
fault of (d) To use the Internet in teaching
(a) The teacher 139. Which of the following skills are needed for the
(b) The principal present-day teacher to adjust effectively with class-
(c) Students themselves room teaching?
A S S E S S YO U R L E A R N I N G

(d) It cannot be generalized 1. Knowledge of technology


131. A teacher who is not able to draw the attention of his 2. Use of technology in teaching learning
students should 3. Knowledge of students’ needs
(a) Evaluate his teaching method and improve it 4. Content mastery
(b) Resign from the post Codes:
(c) Find fault in his pupils (a) 1 and 3 (b) 2 and 3
(d) Start dictating (c) 2, 3 and 4 (d) 2 and 4
132. Which of the following teacher’s behaviour suggests 140. Which one of the following statements is correct?
a dimension of ‘unsuccessful’ teacher behaviour? (a) Syllabus is an annexure to the curriculum
A teacher who is (b) Curriculum is the same in all educational
(a) Always motivating students institutions
(b) Business-like and friendly approach (c) Curriculum includes both formal and informal
(c) Aloof and focused on routine tasks education
(d) Understanding and sympathetic (d) Curriculum does not include methods of
evaluation
133. Which of the following is desirable from a new
teacher as his/her professional responsibility? 141. To whom the responsibility of organization of cur-
(a) Changing the course curriculum ricular activities should be stored with?
(b) Cooperate with the fellow teacher despite the (a) The principal
differences (b) The teacher who is appointed for this work.
(c) Follow the procedures of the institute (c) The teachers who take interest in it.
(d) None of the above (d) All the teachers
134. ‘Mirambika’—The school based on ideas of free 142. The aim of education should be
­progress education was advocated by (a) To develop vocational skills in the students.
(a) M. K. Gandhi (b) Vivekananda (b) To develop social awareness among students.
(c) John Dewey (d) Sri Aurobindo (c) To prepare the students for examination.
(d) To prepare the students for practical life.

M01_MADAN 07_65901_C01.indd 72 27/12/22 8:15 PM


Teaching Aptitude 1.73

143. The contribution of taxpayers in primary education 152. ‘Man is born free but is everywhere in chains.’ This
is in the form of statement was given by
(a) Income tax (a) Abraham Maslow
(b) Tuition fee (b) Jean-Jacques Rousseau
(c) Paying money for individual tuition (c) John Dewey
(d) Educational cess (d) W. H. Kilpatrick
144. The success of integrated education depends on 153. Which of the following statements are true in context
(a) The support of community. of social constructivism?
(b) The excellence of textbooks. (a) This is considered to be a blend of teacher guided
(c) The highest quality of teaching-learning and student centred.
material. (b) This concept was given by Lev Vygotsky
(d) The attitudinal changes in teachers. (c) The learning was a collaborative process between
145. The purpose of ICT integration is for student and teacher.
(a) optimising learning outcomes (d) All of the above
(b) enhancing students’ involvement in learning 154. Which of the following are the features of construc-
(c) promoting concems for economy in use of tivism ?
resources 1. People learn through experiences and reflection.
(d) reducing workload of teachers in terms of trans- 2. The child is put at the centre of learning, and
actional hours thus it may be called as the ‘invisible pedagogy’.
146. The quality of school education exclusively depends 3. This approach incorporates project work, inquiry
upon based learning, and might adopt a Montessori or
(a) Infrastructural facilities Steiner method.
(b) Financial provisions 4. We make a shift from behaviourism and towards
(c) International support social constructivism.
(d) The quality of teacher’s education 5. This approach was mainly propounded by Piaget
who wrote extensively about ‘schemas’.
147. The in-service teacher training can be made more
effective by Codes:
(a) Using training package which is well prepared in (a) 1, 2 and 3 (b) 2, 3 and 4
advance. (c) 1, 2, 3, 4 and 5 (d) 1, 3 and 5
(b) Making it a residential programme. 155. When the children become mischievous and disobe-
(c) Using cooperative approach. dient in the classroom, the teacher should examine
(d) Practicing training follow-up procedures. (a) Home background of the students.

A S S E S S YO U R L E A R N I N G
148. Who developed the interaction analysis category (b) Influence of outside elements in class.
system in education for increasing the teacher’s (c) Teaching methods and subject knowledge.
effectiveness? (d) Co-curricular and other attractions in the school.
(a) Flander 156. Which of the following set of activities will be con-
(b) Rayon sidered most relevant for developing curricular
(c) Amidon and Simon framework in writing skill?
(d) Richard Over (a) Construct, Reflect, Communicate, Construct
149. One of the important theories of moral development and Assess.
was proposed by (b) Plan, Assess, Build and Reconstruct.
(a) Lawrence Kohlberg (b) Erich Fromm (c) Plan, Construct, Communicate, Reflect and
(c) Daniel Coleman (d) Benjamin Bloom Assess.
(d) Formulate, Construct, Prepare and Reflect.
150. Spare the rod and spoil the child. This assumption
is related to the type of discipline which has been 157. Assertion (A): Teacher education curriculum has to
advocated give enough scope to learners for developing their
(a) By naturalist philosophy own logical thinking and problem solving ability.
(b) By pragmatist philosophy Reason (R): The learners today need considerable
(c) In Victorian era freedom to explore, enquire and investigate.
(d) In democratic era Select the correct option:
151. Which of the following is an approach to educational (a) Both (A) and (R) are true, and (R) is the correct
planning? explanation of (A).
(a) Manpower approach (b) Both (A) and (R) are true, but (R) is not the cor-
(b) Social demand approach rect explanation of (A).
(c) Both (a) and (b) (c) (A) is true, but (R) is false.
(d) None of the above (d) (A) is false, but (R) is true.

M01_MADAN 07_65901_C01.indd 73 27/12/22 8:15 PM


1.74 Chapter 1

TEACHING METHODS AND TEACHING INSTRUCTIONAL FACILITIES


158. Micro teaching is more effective 168. Educational technology is useful because
(a) During preparation for teaching practice (a) It is the need of the hour.
(b) During teaching practice (b) It is adopted by famous institutions.
(c) After the teaching practice (c) It makes teaching effective and efficient.
(d) Always (d) It attracts students towards teaching and learn-
159. Microteaching is useful for students of ing activities.
(a) Primary classes only 169. Which component(s) might be included in a lesson
(b) Junior classes only plan?
(c) 10 + 2 classes only (a) Development or outline of a lesson.
(d) Both for primary and higher classes (b) Varied materials and media to supplement and
160. Which of the following university was the pioneer in clarify content.
microteaching concept started in 1961? (c) Specific objectives of the lesson.
(a) Stanford University (d) All of the above
(b) Oxford University 170. One of the most popular forms of drill and practice is
(c) Delhi University (a) Questioning
(d) JNU University, New Delhi (b) Direct instruction
161. In which of the following is instructional procedure (c) Experiential instruction
the main component? (d) Discussion orientation
(a) Synectics teaching model 171. Practice is made in
(b) Basic teaching model (a) Inductive method (b) Deductive method
(c) Inductive model (c) Drill method (d) Discussion method
(d) Social stimulation 172. In which of the following subjects, role playing can
162. In education, John Dewey stressed on be mainly useful for teaching?
(a) Learning by doing (a) History (b) Science subjects
(b) Authoritarian teaching methods (c) Mathematics (d) Language
(c) Rote learning 173. Below given are two columns. Column–A lists methods
(d) None of the above of teaching and Column–B lists the focus of a method.
163. Symposium is a type of
(a) Discovery method Column–A Column–B
(b) Discussion method
A S S E S S YO U R L E A R N I N G

(A) Lecture method (i) D


 ynamic, proactive
(c) Lecture method method of teaching
(d) Demonstration method
164. Questioning skill in teaching is most useful in (B) Demonstration (ii) E
 xperience-based
(a) Ensuring students’ active participation in method learning
learning (C) Heuristic method (iii) Imparting large
(b) Memorizing the facts by students amount of knowledge
(c) Making students disciplined
(d) Preparing students for examination (D) Differentiated (iv) T
 he student has to
165. Which of the following statement is incorrect about Instruction solve own problem
microteaching? by unaided efforts
(a) It is a method of teaching.
(b) It consists of core teaching skills. Select your answer from the options given below:
(c) Each skill is practised separately. (a) (A)–(i); (B)–(iii); (C)–(ii); (D)–(iv)
(d) Questioning is one component of microteaching. (b) (A)–(ii); (B)–(iv); (C)–(i); (D)–(iii)
166. Armstrong was the main exponent of (c) (A)–(iii); (B)–(i); (C)–(ii); (D)–(iv)
(a) Problem-solving method (d) (A)–(iii); (B)–(ii); (C)–(iv); (D)–(i)
(b) Project method 174. Which of the following condition must be met to make
(c) Discussion method the discussion method in teaching more effective?
(d) Heuristic method 1. We need to ask each participant so that an easy
167. Discussion method is useful if topic is picked.
(a) The topic is very easy. 2. The topic should be declared in advance
(b) The topic is difficult. 3. The topic should be of common interest
(c) The topic is very difficult. 4. Availability of more than one teacher
(d) In all situations 5. Language facility of participants

M01_MADAN 07_65901_C01.indd 74 27/12/22 8:15 PM


Teaching Aptitude 1.75

Select appropriate answer from the options given 4. Team teaching based method
below: 5. Interactive presentation
(a) 2, 3 and 5 (b) 1, 2 and 3 6. Tutorials
(c) 1, 2 and 5 (d) 3, 4 and 5 Select your answer from the options given below:
175. According to Bruner discovery approach is a must for (a) 1, 2 and 3 (b) 2, 3 and 4
learning with components of which of the following? (c) 3, 4 and 5 (d) 2, 3 and 6
(a) Activation, maintenance and direction
184. When presenting materials, teachers should con-
(b) Stimulus situation, response and reinforcement
sider which of the following?
(c) Assimilation, accommodation and adaptation
(a) Structuring (b) Balancing
(d) Self-motivation, beliefs, self-control and
(c) Motivating (d) All of the above
self-reaction
185. The technology in education is making teaching
176. Which is not the advantage of team teaching?
(a) Learner oriented
(a) Better utilization of resources
(b) Teacher oriented
(b) Better planning
(c) Both (a) and (b)
(c) Better use of teaching techniques
(d) Social media oriented
(d) Better financial benefits of teachers
186. Which of the following is/are true of whole-group
177. Peer group interaction in a classroom helps in
instruction?
(a) Concept understanding
(a) Permits students to progress at their own pace
(b) Realization of One’s misunderstanding
(b) Convenient for teaching the same skills or con-
(c) Rapport Building
tent to the entire class
(d) Questioning
(c) Gives the teacher a chance to introduce new
178. A teacher performs practically and explains in skills at a level suited to particular students
(a) Lecture method (d) All of the above
(b) Discovery method
187. Which of the following methods of communication
(c) Demonstration method
is the most effective?
(d) Problem-solving method
(a) Verbal communication
179. Given below are two statements–one is labelled as (b) Oral Communication
Assertion (A) and the other is labelled as Reason (R). (c) Multimedia method
Assertion (A): Defining the objectives of the topic (d) Difficult to generalize and depends upon the
of discussion is not necessarily the first step in class- situation
room communication. 188. Maximum participation of students is possible in
Reason (R): Clear definitions of objectives make teaching through

A S S E S S YO U R L E A R N I N G
students understand the topic well. (a) Lecture method
(a) Both (A) and (R) are true and (R) is the correct (b) Discussions
explanation of (A). (c) Textbook method
(b) Both (A) and (R) are true but (R) is not the cor- (d) Audio-visual instructional facilities
rect explanation of (A). 189. In the two lists given below, List-I provides the list of
(c) (A) is true, but (R) is false. teaching methods, while List-II indicates the factors
(d) (A) is false, but (R) is true. helpful in rendering them effective. Match the two
180. The main assumption underlying team teaching is lists and choose the correct answer from the code
(a) Teachers feel bored while working alone. given below.
(b) Teachers are not competent.
List-I List-II
(c) The best teachers can be shared by more students.
(Teaching (Factors helpful in making
(d) The single teacher cannot control the class.
Methods) them effective)
181. Dialogue method of discovering the truth was given
by (A) Expository (i) Promotes student
(a) Plato (b) Socrates method autonomy and enhance
(c) Froebel (d) Herbart learning
182. Which is not the mode of CAI? (B) Discovery (ii) Open-ended and
(a) Tutorial mode (b) Drill mode method collaborative exchange
(c) Simulation mode (d) Question mode of ideas
183. In the spectrum of teaching methods which of the (C) Discussion (iii) Learning by doing
following will be called ‘dialogic’? method
1. Demonstration method (D) Personalised (iv) Systematic, step-by-step
2. Problem solving method method presentation
3. Chalk and talk method

M01_MADAN 07_65901_C01.indd 75 27/12/22 8:15 PM


1.76 Chapter 1

Codes: 196. Which of the following cannot be a good way of com-


(a) (A)–(iii), (B)–(iv), (C)–(ii), (D)–(i) munication in promoting literacy among villagers?
(b) (A)–(iv), (B)–(iii), (C)–(i), (D)–(ii) (a) Demonstration
(c) (A)–(iv), (B)–(iii), (C)–(ii), (D)–(i) (b) Reading and writing
(d) (A)–(iv), (B)–(ii), (C)–(iii), (D)–(i) (c) Providing material on TV and film projector
190. Below is the list of teaching methods and approaches. (d) Large group discussion
Which among them denotes the individualized 197. The main advantage of giving home assignments to
teaching approach? students is
(A) Demonstration method (a) Keeping them busy in studies all the time
(B) Modular approach based teaching (b) To stop them from watching TV
(C) Programmed learning (c) To develop the habit of self-study
(D) Personalized teaching (d) All of the above
(E) Collaborative method 198. From the list given below, identify those teach-
Select your answer from the options given below: ing methods which have a great scope for dialogic
discourses.
(a) (A), (B) and (C)
(b) (B), (C) and (D) 1. Lecture with audio–visual aids
(c) (C), (D) and (E) 2. Team teaching
(d) (A), (D) and (E) 3. Tutorials
4. Problem solving methods
191. The teacher presents the subject matter and directs
5. Chalk and talk method
the students through the lesson. A rule is presented
6. Group discussions
with an example and then practice is provided. This
is termed as Select your answer from the options given below:
(a) Expository Teaching (a) 1, 2 and 3 (b) 3, 4 and 6
(b) Direct instruction (c) 2, 3 and 5 (d) 4, 5 and 6
(c) Guided exploration 199. In which teaching method, learner’s participation is
(d) Problem based learning made optimal and proactive? [January 2017]
192. The knowledge that is acquired without inference (a) Discussion method
and/or use of reason is called as (b) Buzz session method
(a) Demonstrative knowledge (c) Brainstorming session method
(b) Logical knowledge (d) Project method
(c) Experiential knowledge 200. The main task of educational computer is
(d) Intuitive knowledge (a) Scoring the answers
A S S E S S YO U R L E A R N I N G

193. Concept-mapping is used to organize related infor- (b) Preserves the information
mation in a visual manner, it demonstrates hierar- (c) Analysis of data
chical relationships among the topic, main ideas, (d) All of the above
and supporting details or pertinent course material. 201. The computer-based teaching model has been devel-
Concept mapping can be used as oped by
(a) both for learning and assessment tool (a) Gilbert (b) Stolorow and Davis
(b) for memorising only (c) Robert Gagne (d) Mecner
(c) for learning only
(d) for assessment only 202. Which of the following expectation students have
from group learning?
194. In which of the following techniques does a teacher (a) To get appreciation from the group
provide a set of flexible questions to the students (b) To distribute the work equally
that prompt them to reflect on their own learning? (c) To ignore individual view point
(a) Reflective prompts (d) To motivate isolated students to become members
(b) Self assessment of the group
(c) Peer assessment
(d) None of the above 203. Which of the following combines scopes of large
group, small group and individualized teaching
195. Which type of teaching paradigm would focus on a methods?
technical or ‘one right way to teach’ approach to pre- (a) Group discussion
senting content? (b) Differentiated instruction
(a) Learning paradigm (c) Brainstorming
(b) Instructional paradigm (d) Direct instruction
(c) Value-added paradigm
(d) Attitudinal paradigm 204. Which combination of teaching methods listed below
would encourage the learner-centred paradigm?
(a) Individualized instruction and lecture method.

M01_MADAN 07_65901_C01.indd 76 27/12/22 8:15 PM


Teaching Aptitude 1.77

(b) Simulation and demonstration. Choose your answer from the following options:
(c) Lecture method and experimentation. (a) 2, 4 and 6 (b) 1, 2 and 3
(d) Projects and direct experiences. (c) 3, 4 and 6 (d) 4, 5 and 6
205. What is the limitation of the project method of 215. Peer group interaction in a classroom helps in
teaching? (a) Concept understanding
(a) It is learner-centred. (b) Realization of one’s misunderstanding
(b) Learners get practical experience. (c) Rapport building
(c) Learners are usually not properly supervised. (d) Questioning
(d) None of the above
216. Inclusive education implies
206. The heuristic approach is based on (a) Ensuring learning outcome of every child to be
(a) Rote memorization (b) Home work the same
(c) Spirit of inquiry (d) None of the above (b) Including the disabled in the main stream
207. A teacher uses the learning instructional facilities to (c) Provides compulsory education for children
make learning below 14 years
(a) Simple (b) More knowledgeable (d) Ensuring that no child is left behind in education
(c) Quicker (d) Interesting 217. The modality of teaching has to be differentiated
208. Which of the following is a teaching instructional from training or instruction in terms of which of the
facilities? following learning outcome?
(a) LCD projector (b) Green board (a) Disciplined or patterned behaviour
(c) Tape recorder (d) All of the above (b) Critical and creative thinking
209. Teacher uses visual instructional facilities to make (c) Firm and durable association of ideas
learning (d) Quick feedback
(a) Interesting (b) Passive 218. A university teacher plans to study the effect of
(c) Quicker (d) Complex level of aspiration of students in terms of their
210. Which of the following is not true about projects as a socio-economic background on their scholastic
learning activity? performance at the term end evaluation conducted
(a) It is a purposeful activity by an external testing agency. What is the dependent
(b) It is proceeds in social environment variable in this study?
(c) It is accomplished in real-life situations (a) Level of aspiration of students
(d) It is teacher-centred activity (b) Socio-economic background of students
(c) Scholastic performance of students
211. Use of telecast materials facilitates (d) Term end evaluation
(a) Better concentration and learning

A S S E S S YO U R L E A R N I N G
(b) To reach large number of people 219. From the list given below identify the instructional
(c) Better retention of topics taught events which form part of the structure of a lesson
(d) All of the above plan. Select your answer from the options given
below the list:
212. Which of the following is a benefit associated with List of instructional events:
the overhead projector? 1. Gaining attention of students
(a) They are relatively inexpensive 2. Prior knowledge of students
(b) Overhead transparencies can be made relatively 3. Informing the learner about the objectives
quickly 4. Stimulating recall of pre-requisite learning
(c) They offer teachers the option of writing on 5. Checking the availability of reading material in
transparencies during the class activity the library.
(d) All of the above 6. Eliciting the desired response
213. What is most important while writing on blackboard? (a) 1, 2, 3, 4 (b) 1, 3, 4, 6
(a) Good handwriting (c) 2, 3, 5, 6 (d) 1, 3, 4, 5
(b) Clarity in writing
(c) Writing in big letters 220. Which of the following statements are true in the
(d) Writing in small letters context of Brain storming?
1. Brainstorming is a group creativity technique
214. Which of the following teaching method(s) is/are con- 2. Brainstorming is a problem solving technique
tributive to ‘direct learning’? [December 2019] 3. Brainstorming helps at individual level and
1. Team Teaching and lecturing group level
2. Simulation and role playing 4. The top priority of brainstorming is quantity over
3. Lecture with or without use of audio—visuals quality
4. Participation in workshop and assignments Codes:
5. MOOCs and heuristics (a) Only 1, 2 and 3 (b) Only 2, 3 and 4
6. Interactive presentation (c) 1, 3 and 4 only (d) All of the above

M01_MADAN 07_65901_C01.indd 77 27/12/22 8:15 PM


1.78 Chapter 1

221. For optimizing self-learning which of the following 5. Enable students to have some control over the
motivational principles will be the most helpful? speed or topics through the tools used by them
(a) Using reward and punishment Select your answer from the options given below:
(b) Creating scope for fulfilling relatedness need
(a) 1, 2 and 3
(c) Providing scope for satisfying need for
(b) 2, d and 5
competence
(c) 2, 3 and 4
(d) Promoting concerns for meeting status needs
(d) 1, 3 and 5
222. Why do teachers use teaching instructional facilities?
230. Consider the following statements:
(a) To make teaching fun filled
(b) To teach within the understanding level of Assertion (A): In a flipped classroom programme,
students students can access digital learning materials via a
(c) For students’ attention cloud-based platform
(d) To make students attentive Reason (R): Resources such as video lectures, pod-
223. Which among the following gives more freedom to casts, recordings and articles would be provided
the learner to interact? by teacher to students prior to each class on cloud-
(a) Use of film based platform.
(b) Small group discussion (a) Both the (A) and (R) are true, and (R) provides
(c) Lectures by experts the correct explanation of (A).
(d) Viewing country-wide classroom programme on (b) Both the (A) and (R) are true, and (R) does not
TV provide the correct explanation of (A).
224. Which of the following is more interactive and stu- (c) (A) is true but (R) is false.
dent centric? (d) (A) is false but (R) is true.
(a) Seminar (b) Workshop 231. The student centricity means
(c) Lecture (d) Group discussion (a) Availability of multiple entry and exit points
225. An effective teaching instructional facilities is one (b) Promotion of the English and other foreign
which languages
(a) Is colourful and good looking (c) Reforming the examination systems with open
(b) Activates all faculties book testing and group exams
(c) Is visible to all students (d) Both (a) and (c)
(d) Easy to prepare and use 232. The important features of Blended Learning envi-
226. Which of the following belongs to a projected ronment are:
instructional facilities? (a) Improved student learning outcomes
A S S E S S YO U R L E A R N I N G

(a) Blackboard (b) More flexible teaching and learning environment


(b) Diorama (c) Better opportunities for experiential learning
(c) Epidiascope (d) All of the above
(d) Globe 233. Which of the following is not the advantage of
227. The students valued the blended learning because blended learning:
(a) Boost the creativity level of students (a) Repeatable, reliable and reproducible learning
(b) It provides both immediate and asynchronous material.
access to learning resources (b) Increase in interaction
(c) Solve the problems of students (c) Rigid to change
(d) Keep students relaxed while learning (d) Enhancement in learning skills
228. Blended Mode is an 234. Which of the following are essential characteristics
(a) instructional methodology, of experiential learning ?
(b) professional methodology (a) Learning is a continuous process grounded in
(c) research methodology experience.
(d) none of these (b) Learning is a holistic process of creating
knowledge.
229. Blended learning involves
(c) Learning involves transactions between the per-
1. A teaching and learning approach that combines
son and the environment.
face to face classroom methods with computer
(d) All of the above
mediated activities to deliver instruction
2. A mixture of face-to-face and teacher centred 235. Correct sequence of the steps of constructivist learn-
activities ing is :
3. It is an integration of synchronous and asyn- (a) New information> experiential activity>
chronous learning tools Analyse >Solve> Examine >Explore> Assess.
4. It does not provide effective learning processes (b) Experiential activity> Examine >Analyse >
new information> assess > Explore

M01_MADAN 07_65901_C01.indd 78 27/12/22 8:15 PM


Teaching Aptitude 1.79

(c) New information> Analyse >experiential activ- 239. Match the following
ity> assess > Explore> Examine
(d) Examine > Analyse >Solve> New information 1. The self-blend    (I) Massive open online
>Explore> Experiential activity > Assess Model course
236. Which is not feature of blended learning from below 2. The flipped (II) Self blending is a
mentioned. classroom programme-level model
(a) classroom instruction has largely been 3. The rotation (III) K–12 education.
teacher-directed Model
(b) privacy that keep teachers continuously engaged
and motivated. 4. The blended (IV) Reversal of the
(c) classroom instruction has largely been more MOOC traditional class
student-driven structure
(d) provides an appropriate balance between online
instructions as tech-based learning (a) 1-II, 2-IV, 3-III, 4-I
(b) 1-I, 2-IV, 3-II, 4-III
237. Which is not a role of a Learner in the Blended learn- (c) 1-II, 2-III, 3-IV, 4-I
ing Environment (d) 1-I, 2-IV, 3-IV, 4-III
(a) Increase student interest and Promote student
ownership: 240. Experiential Learning Theory has been given by
(b) Keep students focused for longer and Prepares (a) David Kolb (b) Morris L. Biggie
students for the future (c) John Dewey (d) Jean Piaget
(a) Instil a disposition of self-advocacy and Enables 241. Which of the following define Experiential Learning
students to learn at their own pace Theory Approach?
(d) keep students continuously engaged and (a) Learning is the process whereby knowledge
motivated is created through the transformation of
238. Blended learning offers a multitude of real-world experience
skills, that directly translate into life skills, from not (b) This theory is majorly concerned with the learn-
(a) Research skills and Computer literacy er’s internal cognitive processes
(b) Self-learning and Self-engagement (c) This happens through questioning, critical
(c) Helps to develop a ‘self-driving force’ and better analysis, application of the concept and by,
decision making reflection-on-action.
(d) Solve problems and create new knowledge (d) Both (a) and (b)

A S S E S S YO U R L E A R N I N G
LEARNER’s CHARACTERISTICS
242. Understanding theories and principles of children’s 246. Below are listed some learners’ characteristics. Iden-
learning is of fundamental importance tify those that help in effective teaching.
(a) For effective teaching 1. Learner’s respect for teacher
(b) For effective curriculum planning 2. Learner’s level of mental ability
(c) For motivation of the learner 3. Learner’s previous experiences
(d) All of the above 4. Learner’s level of interest to study
243. The most appropriate meaning of learning is 5. Learner’s level of interpersonal relation
(a) Inculcation of knowledge 6. Learner’s view about the society
(b) Modification of behaviour Select your answer from the options given below:
(c) Personal adjustment (a) 1, 2, 3 and 6 (b) 3, 4, 5 and 6
(d) Acquisition of skills (c) 1, 2, 3 and 4 (d) 2, 3, 4 and 6
244. Teachers knowledge on students’ needs and interests 247. The psychological aspects of the classroom are best
are covered by the subject managed by
(a) Philosophy of education (a) The class teacher (b) The subject teacher
(b) Psychology of education (c) The principal (d) The student themselves
(c) Sociology of education
248. Students can be classified into four types on the basis
(d) Politics of education
of their learning. Which one of the following seeks
245. The most important challenge before a teacher is meaning and reasoning to the learning?
(a) To maintain discipline in the class (a) Innovative learner
(b) To make students do their homework (b) Analytic learner
(c) To prepare question paper (c) Common sense learner
(d) To make teaching-learning process enjoyable (d) Dynamic learner

M01_MADAN 07_65901_C01.indd 79 27/12/22 8:15 PM


1.80 Chapter 1

249. Instruction that takes into account various types of (a) 1, 2, 3 and 4 (b) 4, 3, 2 and 1
learners and learning styles and is adapted accord- (c) 2, 3, 4 and 1 (d) 3, 1, 2 and 4
ingly is said to be 258. We usually say that no two students are alike. They
(a) Teacher-centred (b) Differentiated may differ from each other in terms of
(c) Direct instruction (d) None of the above (a) Upbringing and social status
250. A heavy emphasis on measurable outcomes leads to (b) Aptitude
(a) Rote learning (b) Memorization (c) Attitude
(c) Both (a) and (b) (d) None of the above (d) All of the above
251. Teachers who are enthusiastic in the classroom 259. Reinforcement is provided by any factor that
teaching increases the probability that a response will be
(a) Lack proficiency in the subjects, which stays hid- repeated. Which of the following can be factor(s) of
den under their enthusiasm reinforcement?
(b) Simply dramatize to hold the student’s attention (a) Praise
(c) Involve their students in the teaching-learning (b) Token reward
process (c) Simply succeeding in a task
(d) All of the above (d) All of the above
252. A teacher organizes and reorganizes the presentation 260. When you complete your math’s exercise, you can
in a discussion session being conducted by her/him. play the computer game. Using such kind of rein-
This will be placed at which stage of the teaching act? forcement wherein student’s favourite activity can
(a) Preactive preparation stage be used to reinforce a student’s engagement in a less
(b) Preactive planning stage popular activity is termed as
(c) Interactive stage (a) Premack principle (b) Thorndike law
(d) Post active stage (c) Pavlov experiment (d) None of the above
253. Diversity in student demographics requires common 261. Morphographs is the term associated with
academic standards to (a) Corrective spelling
(a) Provide a common benchmark for assessment. (b) Corrective reading
(b) Promote genuine learning for high-performing (c) Corrective learning
students. (d) Corrective behaviour
(c) Ensure that all states have the same require- 262. Which of the following description(s) apply in con-
ments as required by the Central government. text of constructivist approach and cognitive theory
(d) None of the above of learning?
254. Intuitive thinking (a) Learners as active participants in learning
A S S E S S YO U R L E A R N I N G

(a) Is part of the process of discovery process


(b) Has been encouraged by traditional teaching (b) Seeking to interpret
(c) Is not a cognitive process (c) Using multiple sources of information
(d) None of the above (d) All of the above
255. While presenting your ideas in a classroom, it is bet- 263. Organized bodies of knowledge that we build up
ter to about particular objects, situations or phenomena
1. Recognize that there can be other views are termed as
2. Recognize that students are not a homogenous (a) Schemas or mental schemata
mass (b) Memory
3. Take dissenting views also in consideration (c) Cognition
Codes: (d) Metacognition
(a) Both 1 and 3 (b) Only 3 264. The taking in of new information is termed as
(c) Only 2 and 3 (d) 1, 2 and 3 (a) Assimilation (b) Accommodation
256. ‘Individual differences’ in learning process are given (c) Cognition (d) Reception
the least importance in 265. The term which is used to refer to the variety of ways
(a) Naturalism (b) Realism in which teachers and others help or support learn-
(c) Idealism (d) Pragmatism ers to move beyond their current level of under-
257. Arrange the following experimental learning activi- standing by giving them cues, suggestions or even
ties adopted by a teacher in cyclic order. direct guidance at appropriate moments in their
1. Accommodation investigations or activities is known as
2. Converging (a) Scaffolding
3. Assimilation (b) Cognition
4. Diverging (c) Reinforcement
Codes: (d) None of the above

M01_MADAN 07_65901_C01.indd 80 27/12/22 8:15 PM


Teaching Aptitude 1.81

266. The changes in behaviour (learning) are the net result (c) Both (a) and (b)
of environmental influences, interacting with innate (d) To hear everything possible
predispositions and processes within the learner 276. According to Francis Gallon, heredity does not go
(a) Neo-behavioural theory to immediate parents but to remote ancestors. Only
(b) Cognitive theory 50% of the heredity is due to
(c) Behavioural theory (a) Parents
(d) None of the above (b) Great-grandparents
267. The students or individuals may develop beliefs (c) Grand-parents
(positive or negative) about their own ability to (d) None of the above
cope effectively in a variety of situations. This can be 277. Arrange the following teaching processes in order.
termed as 1. Relating the present knowledge with the previous
(a) Self-efficacy (b) Self-regulation knowledge
(c) Ego (d) Confidence 2. Evaluation
268. The ability of an individual to think about one’s own 3. Re-teaching
thought processes, self-monitor and modify one’s 4. Formulation of teaching objectives
learning strategies as necessary is termed as 5. Presentation of materials
(a) Metacognition (b) Self-regulation Codes:
(c) Simple cognition (d) None of the above
(a) 1, 2, 3, 4, 5
269. Teachers blame a student’s learning problems on (b) 2, 1, 3, 4, 5
(a) Lack of motivation (c) 5, 4, 3, 1, 2
(b) Lack of intelligence (d) 4, 1, 5, 2, 3
(c) Casual parental attitude
278. Assertion (A): Learning is a lifelong process.
(d) None of the above
Reason (R): Learning to be useful must be linked
270. The memory that refers to our memory of meaning-
with lifelong processes.
ful facts, rules, definitions, concepts and principles
is termed as Choose the correct answer from the following code:
(a) Semantic memory (a) Both (A) and (R) are true, and (R) is the correct
(b) Episodic memory explanation of (A).
(c) Procedural memory (b) Both (A) and (R) are true, but (R) is not the cor-
(d) None of the above rect explanation of (A).
271. An individual’s awareness of his or her own mem- (c) (A) is true, but (R) is false.
ory processes and the ways in which storage and (d) (A) is false, but (R) is true

A S S E S S YO U R L E A R N I N G
retrieval of information can be enhanced is termed 279. The process of communication enhances through
as (a) The feeling of belongings and commonness
(a) Semantic memory (b) Episodic memory (b) Security and freedom to make choices
(c) Procedural memory (d) Meta memory (c) Informal environment
272. Which of the following can impact process listening (d) All of the above
in an adverse manner? 280. Which one of the following is a product of learning?
(a) Excess of listened material—message overload (a) Intelligence (b) Maturation
(b) Very high speed of speaking (c) Skills (d) Memory
(c) A good amount of hearing loss 281. Which of the following explains the mental growth
(d) All of the above most suitably?
273. The most important aspect of communication, which (a) A growth pattern runs parallel to the physical
is listening can be improved by growth
(a) Linking listening to a monetary reward system (b) It is an erratic pattern
(b) Making the contents interesting and need based (c) It is not an erratic pattern
(c) Enhancing voice effectiveness and impressiveness (d) Uniform rise to the middle teens and gradual
(d) All of the above levelling off during middle twenties
274. Listening to a lecture is basically 282. While comparing hearing and listening, we can say
(a) Informational listening that
(b) Evaluative listening (a) Hearing is a physical process and listening is a
(c) Emphatic listening psychological process
(d) Dynamic Listening (b) Hearing is a psychological process and listening
275. The evaluative listening is basically about is a physical process
(a) To accept or reject an idea given to the listener (c) Both are mainly physical processes
(b) To evaluate the speaker’s credibility and personality (d) Both are biological processes

M01_MADAN 07_65901_C01.indd 81 27/12/22 8:15 PM


1.82 Chapter 1

283. Which of the following statements is true? (c) Giving examples all the time
(a) Human relationships are not affected by listening (d) Delivering speech in class
(b) When communicating, college students spend 292. The best way a teacher can try to inculcate good val-
over half of their lives listening ues among students is by
(c) Listening constitutes just a small fraction of our (a) Storytelling
overall communication (b) Developing sense of discipline
(d) None of the above (c) Ideal behaviour of teacher themselves
284. When a person listens and attempts to understand (d) Taking their parents into confidence
the other person’s viewpoint, he/she can be termed as 293. How the students should be motivated to get success
(a) Active listening in life?
(b) Empathetic listening (a) Selected study (b) Incidental study
(c) Critical listening (c) Intensive study (d) Rote learning
(d) Passive listening
294. The problem of absenteeism can be tackled in a bet-
285. Which gender tends to listen in order to solve prob- ter way through
lems, is less attentive to nonverbal cues and inter- (a) Reduction of the weight of curriculum.
rupts to switch topics? (b) Sympathy of teachers.
(a) Men (b) Women (c) Attractive environment of the school.
(c) Both genders (d) Neither gender (d) Motivation of the students.
286. Critical thinking 295. The best remedy of a student’s problems related with
(a) Focuses solely on the details instead of the main learning is
point (a) Suggestion for hard work
(b) Ignores the context in which communication is (b) Supervised study in library
occurring (c) Suggestion for private tuition
(c) Is important when making judgments about the (d) Diagnostic teaching
message being presented
296. Emotional adjustment of students is effective in
(d) Is only associated with listening
(a) Personality formation
287. Asking questions to clarify information, paraphras- (b) Class teaching
ing messages and identifying confusing areas are (c) Discipline
basically (d) All of the above
(a) Barriers to listening
297. Who among the following is described as ‘Father of
(b) Listening for fun
Psychoanalysis’?
(c) Techniques for checking your understanding of
(a) Erik H. Erikson (b) Jean Piaget
A S S E S S YO U R L E A R N I N G

a message
(c) Jerome S. Bruner (d) Sigmund Freud
(d) None of the above
298. The main proponent of the cognitive theory of teach-
288. Suggestions for lecture listening include
ing is
(a) Focusing on the delivery part and avoiding sum-
(a) N. L. Gage (b) Shiv Kumar Mitra
marizing and reviewing the information
(c) B. F. Skinner (d) McDonald
(b) Poor attention in order to think creatively, listen-
ing for details and ignoring lecture cues 299. Practical knowledge of language is learnt at
(c) Not making notes so as to focus better on lecture (a) School
and message delivery (b) Language laboratory
(d) Finding areas of interest to you, avoiding dis- (c) Language teaching
tractions and listening for main ideas (d) Language instruction
289. The ability to locate, evaluate and effectively use 300. In which domain does the following objective fall?
information is an important trait known as At the end of the lesson, the learner should be able to
(a) Critical thinking hit the football using the head.
(b) Information literacy (a) Affective domain
(c) Hearing (b) Cognitive domain
(d) Selective attention (c) Psychomotor domain
290. At primary level, it is better to teach in mother lan- (d) Both (a) and (c)
guage because 301. Planning or arranging the student’s environment
(a) It develops self-confidence in children in order to predict the consequences of a student’s
(b) It makes learning easy behaviour is referred to as
(c) It is helpful in intellectual development (a) Prompting (b) Reinforcement
(d) It helps children to learn in natural atmosphere (c) Shaping (d) Stimulus control
291. The best approach to motivate students is by 302. Which theory of learning has found that knowledge
(a) Giving them suitable prizes of internal processes is crucial to the understanding
(b) Providing them proper guidance of learning?

M01_MADAN 07_65901_C01.indd 82 27/12/22 8:15 PM


Teaching Aptitude 1.83

(a) Cognitive theory 312. If the students do not understand what is taught in
(b) Stimulus–response theory the class, the teacher should
(c) Operant conditioning theory (a) Repeat the lesson once again
(d) Classical conditioning theory (b) Teach the lesson again citing suitable examples
303. Which aspect of evaluation is used when a teacher (c) Check the previous knowledge of the students in
ensures that students complete an exercise in math- the topic
ematics and also makes sure that instructions are (d) Proceed to the next topic
clear and specific? 313. An ideal situation in a classroom would be where
(a) Validity (b) Practicality (A) A teacher comes fully prepared to deliver his
(c) Reliability (d) Consistency lecture
304. Which of the following is not a characteristic of a (B) Students come fully prepared and discuss the
slow learner? subject with each other in teacher’s presence.
(a) Limited vocabulary (C) Teachers and students discuss the subject
(b) Short span of attention (D) The teacher uses audio-visual instructional facil-
(c) Abstract thinking ities while learning
(d) Limited range of interests (a) Both (A) and (C)
305. A teacher used the following statement to change the (b) Both (C) and (D)
behaviour of a student who was a smoker. ‘Smoking (c) Only (B)
is healthy for the nation’. This is an example of: (d) Only (D)
(a) Cognitive dissonance 314. Which of the following categories of teachers tend
(b) Conceptual conflict to favour the traditional, formal seating pattern of
(c) Meaningful learning rows of students directly facing the teacher at the
(d) Challenge front of the classroom?
306. The best way to memorize is (a) Direct instruction
(a) To study for long time (b) Indirect instruction
(b) To understand the concept (c) Student-centred
(c) To read loudly (d) None of the above
(d) To write the concept 315. Which of the following teacher can be identified
307. Attitudes, concepts, skills, and knowledge are prod- with authoritarian teaching style?
ucts of (a) Democratic teacher
(a) Learning (b) Research (b) Indirect teacher
(c) Heredity (d) Explanation (c) Laissez-faire teacher

A S S E S S YO U R L E A R N I N G
308. Which of the following is not a product of learning? (d) Direct instruction teacher
(a) Attitudes (b) Concepts 316. What is more desirable in a classroom?
(c) Knowledge (d) Maturation (a) A teacher delivering a lecture on the basis of the
309. Some students in a class exhibit great curiosity for text and his own research
learning. It may be because such children (b) A teacher delivering a lecture on the basis of
(a) are gifted course content and standard books
(b) come from rich families (c) A teacher answering questions raised by
(c) show artificial behaviour students
(d) create indiscipline in the class (d) A teacher maintaining strict discipline and tak-
ing attendance regularly
310. The best way to react to a wrong answer given by a
student is 317. Which process of communication is the best for con-
(a) To scold him for not having learnt the lesson trolling noise in a classroom?
(b) To explain why the answer is wrong (a) Saying ‘do not talk’
(c) To ask another student to give the correct answer (b) Raising one’s voice above students’ voice
(d) To ignore the wrong answer and pass on the (c) Remaining calm and just looking at the students
next question (d) Continue teaching without caring for noisy class
311. When a student asks a question to which the teacher 318. As a teacher, what will you do if students do not
has no direct, correct answer, he should attend your class?
(a) Give some vague answer and satisfy the student (a) Blame students for their absence from the class
(b) Tell the student not to ask much irrelevant (b) Ponder over the present attitude of students in a
questions calm manner
(c) Tell the student that he would give the correct (c) Think about using some interesting techniques
answer later of teaching
(d) Ask the student to find out the answer himself (d) Try to understand the reasons and try to elimi-
from the books in the library nate them

M01_MADAN 07_65901_C01.indd 83 27/12/22 8:15 PM


1.84 Chapter 1

319. A teacher is strict in maintaining discipline in the 327. With specific reference to classroom environment, all
class both in curricular and extracurricular activi- except one of the major components of listening is
ties. However, there is always room for discussion (a) Hearing
regarding clarification of doubts in teaching–learn- (b) Being attentive
ing and conducting other activities. What is the (c) Answering
approach of teacher towards students? (d) Understanding and remembering
(a) Authoritative (b) Democratic 328. Which of the following will not hamper effective
(c) Flexible (d) Rigid communication in the classroom?
320. Better classroom management means (a) A lengthy statement
(a) Effective group work and interaction among the (b) An ambiguous statement
students (c) A precise statement
(b) Proper planning and preparation in developing (d) A statement which allows the listener to his own
suitable teaching instructional facilities conclusions.
(c) Punctuality of the teachers and ability to com- 329. If backbenchers are always talking in the classroom,
plete course in time a teacher should
(d) All of the above (a) Let them do what they are doing.
321. Which of the following can be described as the most (b) Punish them
probable characteristic of an ineffective teacher? (c) Ask them to sit on the front benches
(a) Always focus on achievement of instructional (d) None of the above
objectives 330. If a teacher is not able to answer the question raised
(b) Always focus on observance of teaching standards by a student in the classroom, he should
(c) Always focus upon control of immediate (a) Say that he will answer after consultation
situation (b) Rebuke the student
(d) None of the above (c) Try to manipulate the students
322. If majority of students in your class are weak, as a (d) Feel shy of his ignorance
teacher you should 331. If students are not able to follow, you should
(a) Not care about the intelligent students (a) Give them prompt explanation
(b) Keep your speed of teaching fast so that stu- (b) Make the matter easy
dents’ comprehension level may increase (c) Illustrate with examples
(c) Keep your teaching slow (d) All of the above
(d) Keep your teaching slow along with some extra 332. If students pass remarks on you while you are work-
guidance to bright pupils ing as a teacher, you will
A S S E S S YO U R L E A R N I N G

323. If some of the students misbehave with the teacher (a) Punish them
in the college campus, which kind of solution can (b) Expel them from the college
help in the long run? (c) Take revenge while evaluating internal test copies
(a) Reporting to their parents (d) Be impartial at the time of evaluation
(b) Teacher should improve their behaviour by their 333. Discussion in class will be more effective if the topic
own character and scholarship of discussion is
(c) Reporting the matter to the principal (a) Not introduced
(d) Mobilize other teachers against these guys (b) Stated at the start of the discussion
324. A teacher in the class should keep the pitch of his voice (c) Written on the board without introducing it
(a) High enough (d) Informed to the students in advance
(b) Low 334. Failure of the teacher to communicate his ideals well
(c) Moderate to the students may result in
(d) Sometime low and sometime high (a) Classroom indiscipline
325. Which of the following should a teacher adopt in a (b) Loss of students’ interest in the topic being
lecture? taught
(a) Precise and low tone (c) Increased number of absentees in the class
(b) Elongated tone (d) All of the above
(c) Precise and high tone 335. Which of the following is the most important sin-
(d) Moderate tone gle factor underlying the success of beginning as a
326. If students do not understand what is taught in the teacher?
classroom, the teacher should feel (a) Scholarship
(a) Terribly bored (b) Communicative ability
(b) That she needs to explain it in a different way (c) Personality and the ability to relate to the class
(c) That he is wasting time and the pupils
(d) Pity for the students (d) Organizational ability

M01_MADAN 07_65901_C01.indd 84 27/12/22 8:15 PM


Teaching Aptitude 1.85

336. All of the following are the characteristic features of (c) Discipline
an effective teacher except (d) Entertaining
(a) Emphasis upon maintaining standards of 345. Research has shown that the most frequent symptom
education of nervous instability among teachers is
(b) Emphasizing group discussion for the purpose (a) Digestive upsets
of clarifying the objectives. (b) Explosive behaviour
(c) Emphasis upon providing solution to immediate (c) Fatigue
problems. (d) Worry
(d) Differential treatment meted out to students of
346. Which one of the following is appropriate with
his class.
respect to teacher-student relationship?
337. Some students send a greeting card to you on Teach- (a) Very informal and intimate
er’s Day. As a teacher, you will (b) Limited to classroom only
(a) Not respond at all. (c) Cordial and respectful
(b) Say thanks to them. (d) Indifferent
(c) Ask them to not waste money.
347. In a lively classroom situation, there is likely to be
(d) Reciprocate wishes to them.
(a) Occasional roars of laughter
338. A student comes late in your class. As a teacher you (b) Complete silence
will (c) Frequent teacher–student dialogue
(a) Inform the principal and parents. (d) Loud discussion among students
(b) Punish him to set an example.
348. For maintaining effective discipline in the class, the
(c) Try to know the reason.
teacher should
(d) It is not worth paying attention.
(a) Allow students to do what they like
339. When a number of students regularly fail in the (b) Deal with the students strictly
exams, it can be understood that it is (c) Give the students some problems to solve
(a) The system‘s failure (d) Deal with them politely and firmly
(b) The teacher’s failure
349. Those teachers are popular among students who
(c) The textbooks failure
(a) Develop intimacy with them
(d) The individual student’s failure
(b) Help them solve their problems
340. If a student asks questions on some unrelated topic (c) Award good grades
in the class, as a teacher you will (d) Take classes for extra tuition fee
(a) Allow him to ask unrelated questions
350. The essence of an effective classroom environment is
(b) Not allow him to ask unrelated questions
(a) A variety of teaching instructional facilities

A S S E S S YO U R L E A R N I N G
(c) Answer the question after the class
(b) Lively student–teacher interaction
(d) Consider it an act of indiscipline
(c) Pin-drop silence
341. A guardian never comes to see you in school. As a (d) Strict discipline
teacher, you will
351. On the first day of his class, if a teacher is asked by
(a) Ignore the child
the students to introduce himself, he should
(b) Write a letter to the guardian
(a) Ask them to meet after the class
(c) Go to meet him yourself if possible
(b) Tell them about him in brief
(d) Start punishing the child
(c) Ignore the demand and start teaching
342. To maintain interest among students in class, a (d) Scold the student for this unwanted demand.
teacher should
352. Moral values can be effectively inculcated among the
(a) Make maximum use of teaching instructional
students when the teacher
facilities
(a) Frequently talks about values
(b) Discuss
(b) Himself practices them
(c) Ask questions intermittently
(c) Tells stories of great people
(d) All of the above
(d) Talks of gods and goddesses
343. A teacher asks the questions in the class to
353. Suppose a student wants to share his problems with
(a) Keep students busy
his teacher and visits the teacher’s house for the pur-
(b) Maintain discipline
pose. The teacher should
(c) Attract the student’s attention
(a) Contact the student’s parents and solve his
(d) Teach
problem
344. What quality do students like the most in a teacher? (b) Suggest him that he should never visit his house
(a) Idealist philosophy (c) Suggest him to meet the principal and solve the
(b) Compassion problem
(d) Extend reasonable help and boost his morale

M01_MADAN 07_65901_C01.indd 85 27/12/22 8:15 PM


1.86 Chapter 1

354. To keep a check on the habit of absenteeism among (c) Affective processes
students (d) Both (a) and (b)
(a) The principal and parents should get worried 359. In totality, the teacher helps student the most in
(b) The officials should put notice against absentee which of the following way?
students on the notice board (a) Integrated development of the child
(c) The teachers should take it as a serious problem (b) Physical growth of the child
(d) They should be given less priority in the class- (c) For socio-cultural
room as compared to regular students (d) Development of the child
355. When the students try to solve the questions in some 360. If students are not able to follow, the teacher in the
different way not taught by the teacher from pre- class should
scribed books, then these students should be (a) Give them prompt reply
(a) Always discouraged to consult some other books (b) Illustrate with suitable examples
on the subject (c) Make the contents easier
(b) Always encouraged to consult other books on (d) All of the above
the subject
361. If the students are not taking interest in your teach-
(c) Suggested to seek permission of their respective
ing, then you will
class teachers before referring to other books
(a) Ignore them
(d) No action required
(b) Leave the class
356. Students who ask questions in the class should be (c) Ask them to pay attention
(a) Advised to meet the teacher after the class (d) Review the teaching method
(b) Encouraged to participate in the discussion in
362. When some students are deliberately attempting to
the class
disturb the discipline of the class by making mis-
(c) Encouraged to continue asking questions
chief, what will be your role as a teacher?
(d) Encouraged to search answers independently
(a) Expelling those students.
357. In order to modify the undesirable behaviour of a (b) Isolate those students.
student, the most effective method is (c) Reform the group with your authority.
(a) To punish the student (d) Giving them an opportunity for introspection
(b) To bring it to the notice of parents and improve their behaviour.
(c) To find out the reasons for the undesirable
363. A teacher is said to be fluent in asking questions if he
behaviour and provide remedies
can ask
(d) All of the above
(a) Meaningful questions
358. A majority of classroom tasks initiated by teachers in (b) As many questions as possible
A S S E S S YO U R L E A R N I N G

traditional classrooms are usually (c) Maximum number of questions in a fixed time
(a) Low-level cognitive processes (d) Many meaningful questions in a fixed time
(b) High-order cognitive processes

LEARNER’S EVALUATION
364. In the role of a test constructor, how should a teacher 366. Given below are the two statements:
proceed? Statement I: In norm referenced testing of students,
A. Prepare test items that match the instructional test data helps us determine a student’s place or rank.
objectives
B. List the major course or unit objectives Statement II: In criterion referenced testing, the test
C. Prepare a table of specifications data tells us about a student’s level of proficiency.
D. Specify the course or unit content In the light of above statements, choose the correct
E. Discard unrealistic objectives answer from the options given below.
Choose the correct answer from the options given (a) Both statements I and Statement II are true.
below: (b) Both statements I and Statement II are false.
(a) A, B, C, D, E (b) B, E, D, C, A (c) Statement I is true and Statement II is false.
(d) Statement I is false and Statement II is true.
(c) C, A, E, D, B (d) D, B, E, C, A
367. Which of the following is not applicable to the inter-
365. The most significant approach of evaluation is
nal assessment?
(a) Continuous and comprehensive evaluation
(a) Integration of teaching and evaluation
(b) Conducting objective term-end examination
(b) Testing of skills and abilities
(c) Maintaining cumulative records of students
(c) Periodic and continuous
(d) Semester system evaluation
(d) Use of only achievement tests

M01_MADAN 07_65901_C01.indd 86 27/12/22 8:15 PM


Teaching Aptitude 1.87

368. What type of test is most effective when trying to test 376. Teachers use placement evaluation in order to
memorization? (a) Find out what knowledge and skills students
(a) True or false (b) Multiple choices have mastered.
(c) Fill in blanks (d) Both (b) and (c) (b) Discover the causes of students’ learning or
369. Essay-type tests are not reliable because behavioural problems.
(a) Their answers are different (c) Both (a) and (b)
(b) Their results are different (d) None of the above
(c) Their checking is affected by examiner’s mood 377. Summative evaluation is used for which of the fol-
(d) Their responding styles are different lowing purposes?
370. Assertion (A): Formative learning tends to acceler- (a) To monitor student’s progress during the learn-
ate the process of learning. ing process.
(b) Primarily to certify or grade students.
Reason (R): As against summative evaluation, for- (c) To find out the students interests and work
mative evaluation is highly reliable.[January 2017] habits.
Choose the correct answer from the following code: (d) To assign students to specific learning groups.
(a) Both (a) and (R) are true, and (R) is the correct 378. Which among the following is the main objective of
explanation of (A). assessment?
(b) Both (a) and (R) are true, but (R) is not the cor- (a) Collect information systematically to judge stu-
rect explanation of (A). dents’ characteristics.
(c) (a) is true, but (R) is false. (b) To find out the weakness of teaching-learning
(d) (a) is false, but (R) is true system.
371. Below are listed some activities performed by a (c) To relate various collected information about
teacher. Which activities are of the nature of forma- students.
tive evaluation? (d) To obtain data about learning difficulties.
A. Giving a mastery test 379. For the homework to be effective in accomplishing
B. Conducting quiz session its purposes, which of the following suggestions for
C. Evaluating students in grading system teachers is correct?
D.Providing feedback while teaching (a) Give homework as punishment.
E. Encouraging students reflect more (b) Make up spur-of-the-moment homework assign-
Select your answer from the options given below: ments for student practice.
(a) A, B and C (b) B, C and D (c) Do not expect students to always have their
(c) B, D and E (d) B, C and E homework assignments completed.
(d) Do not listen to what students say about their expe-

A S S E S S YO U R L E A R N I N G
372. The best method of checking a student’s homework is riences in completing homework assignments.
(a) To assign it to intelligent students of the class.
(b) To check the answers in the class in group manner. 380. Which of the following is an example of maximum
(c) To check them with the help of specimen answer. performance test? [December 2019]
(d) To check by the teacher himself in a regular way. (a) Projective Personality Test
(b) Intertest Inventory
373. A time-bound testing programme for students (c) Aptitude Test
should be implemented in schools so that (d) Attitude Test
(a) The progress of the students should be informed
to their parents 381. Which among the following factors doesn’t contrib-
(b) A regular practice can be carried out ute to assessment bias?
(c) The students can be trained for final examinations (a) When language of the test and the tester is dif-
(d) The remedial programme can be adopted on the ferent from the languages of the students.
basis of the feedback from the results (b) Answers that support middle–class value.
(c) To assessment procedures are flexible and diverse
374. Which of the following evaluation systems belongs to make disadvantaged students comfortable.
to the category of being an innovative practice? (d) Objective test for assessing abstract reasoning of
(a) Semesterized examinations the student.
(b) Performance evaluation
(c) Portfolio based evaluation 382. Which type of evaluation is carried out at the end of
(d) Learning outcome based evaluation a course of study?
(a) Summative (b) Assessment
375. The most important indicator of quality of education (c) Formative (d) Both (a) and (b)
in an educational institute is
(a) Infrastructural facilities of a school 383. Which test is carried out to determine the ability of a
(b) Classroom system learner?
(c) Textbooks and teaching-learning material (a) Aptitude (b) Attitude
(d) Student achievement level (c) Achievement (d) Scholastic

M01_MADAN 07_65901_C01.indd 87 27/12/22 8:15 PM


1.88 Chapter 1

384. The verbs write, list, label and name when used in an repeated and responses that bring displeasure or
examination, test the pain are likely to weaken and fade, it is called
(a) Comprehension level (a) Thorndike’s law of effect
(b) Application level (b) Thorndike’s law of exercise
(c) Knowledge level (c) Both (a) and (b)
(d) Synthesis level (d) None of the above
385. Good evaluation of written material should not be 391. Given below are two statements, one is labelled as
based on Assertion (A) and the other is labelled as Reason
(a) Linguistic expression (R). [July 2022]
(b) Logical presentation Assertion (A): MOOCs on the SWAYAM portal can
(c) Ability to reproduce whatever is read be used for earning upto 40% credits per semester
(d) Comprehension of subject by the students.
386. By which of the following methods, the true evalua- Reason (R): All MOOCs on the SWAYAM portal are
tion of the students is possible? credit based MOOCs.
(a) Evaluation at the end of the course.
In light of above statements, choose the correct
(b) Evaluation twice in a year
answer from the options given below:
(c) Continuous evaluation
(a) Both A and R are true, and R is the correct expla-
(d) Formative evaluation
nation of A.
387. In order to identify individual differences of learners (b) Both A and R are true, but R is not the correct
in a class, which of the following can be used by a explanation of A.
teacher? [July 2022] (c) Only A is true, but R is false.
(a) Formative Assessments (d) A is false, but R is true.
(b) Summative Assessments
392. SWAYAM tries to take the best teaching learning
(c) Diagnostic Assessments
resources to all students through [July 2022]
(d) Peer Assessments
(a) Seminars
388. Which of the following can be termed as the main (b) Classroom teaching
benefits of Choice-based Credit System (CBCS)? (c) Video lectures
1. Shift in focus from the teacher-centric to student- (d) Conferences
centric education.
393. Which of the following represent ICT integrated
2. Student may undertake as many credits as they
strategy?
can cope with.
3. There is more flexibility to the students in terms A. Flipped teaching-learning
B. Blended teaching-learning
A S S E S S YO U R L E A R N I N G

of completion of different courses.


4. The education becomes broad based and at par C. Edusat-based teaching-learning
with global standards. D.Video conferencing
5. Credits earned at one institution can be transferred. E. Collaborative teaching-learning
F. Peer tutoring
Codes:
Indicate your answer from the codes given below:
(a) 1, 2 and 4 (b) 2, 3, 4 and 5
(c) 1, 2, 3, 4 and 5 (d) 2, 3 and 5 (a) B, C and D (b) B, C and E
(c) A, B and D (d) A, B, C and D
389. Consider the following statements.
1. In direct grading, the performance of an individ- 394. Which of the following instructional designs is part
ual is assessed in qualitative terms. This is easier of SWAYAM launched by Government of India?
to apply and it minimizes the inter examiner 1. E-Tutorial 2. E-Content
variability. 3. Assessment 4. Discussion Forum
2. In indirect grading, the performance of an 5. Classroom teaching
examinee is first assessed in terms of marks and Choose the correct option from below
subsequently transformed into letter grades by (a) 1 and 2 only (b) 1, 3, 4 and 5 only
using different modes. (c) 1, 2, 3 and 4 only (d) 3 and 5 only
3. The indirect grading can be absolute as well as
395. Kindergarten system of education was constituted by
relative grading.
(a) T. P. Nunn (b) Spencer
Codes: (c) Fröbel (d) Montessori
(a) 1 and 2 (b) 1, 2 and 3 396. Given below are two statements–one is labeled as
(c) 2 and 3 (d) 1 and 3 Assertion (A) and the other is labeled as Reason (R):
390. When an action or response produces a pleasant or Assertion (A): Through on-line teaching a large
rewarding outcome that behaviour is likely to be number of students can be taught by very competent
faculty.

M01_MADAN 07_65901_C01.indd 88 27/12/22 8:15 PM


Teaching Aptitude 1.89

Reason (R): On-line teaching helps students in mined standard, learning goal, performance level,
developing critical thinking more than the Off–line or other criterion.
teaching can do. Read the statements and choose the correct answer
In the light of the above two statements choose the using the code given below:
correct option: (a) Both (a) and (R) are true and (R) is the correct
(a) Both (A) and (R) are true, and (R) is the correct explanation of (A)
explanation of (A). (b) Both (a) and (R) are true, but (R) is not the cor-
(b) Both (A) and (R) are true, but (R) is not the cor- rect explanation of (A)
rect explanation of (A). (c) (a) is true, but (R) is false
(c) (A) is true, but (R) is false. (d) (a) is false, but (R) is true
(d) (A) is false, but (R) is true 401. Asking questions to clarify information, paraphras-
397. Statement I: National Council of Educational ing messages, and identifying confusing areas are
Research and Training (NCERT) to assist and advise basically
the Central and State Governments on policies and (a) Barriers to listening
programmes for qualitative improvement in school (b) Listening for fun
education was established in 1961. (c) Techniques for checking your understanding of
a message
Statement II: National Council for Teacher Educa- (d) None of the above
tion (NCTE) that plans and co-ordinates the devel-
opment of teacher education system was set up in 402. In many cases, slow learning is caused by factors
the year 1995. other than low intellectual ability. Which of the fol-
lowing is not a characteristic of a slow learner?
(a) Both statements I and Statement II are true. (a) Abstract Thinking
(b) Both statements I and Statement II are false. (b) Short span of attention
(c) Statement I is true and Statement II is false. (c) Limited range of interests
(d) Statement I is false and Statement II is true. (d) Limited vocabulary
398. Which of the following are the main objectives of 403. Which of the following activities indicate the nature
New Education Policy, 2020 ? of formative evaluation?
1. No hard separations between different subjects (1) Conducting a quiz test
and streams (2) Conducting an aptitude test
2. Multidisciplinarity and a holistic education across (3) The evaluating of student in a grading system
all subjects (4) Providing feedback while teaching
3. Emphasis on conceptual understanding rather (5) Motivating students for innovative thinking in
than rote learning and learning-for-exams

A S S E S S YO U R L E A R N I N G
the class
4. Creativity and critical thinking to encourage logi-
cal decision-making and innovation Select your answer from the options given below:
5. Promoting multilingualism and the power of lan- (a) (1), (2) and (3) (b) (2), (3) and (4)
guage in teaching and learning (c) (1), (4) and (5) (d) (2), (3) and (5)
6. More focus on summative assessment than forma- 404. The performance of a student is compared with
tive evaluation another student in which type of testing?
Codes: (a) Criterion referenced testing
(b) Diagnostic testing
(a) 2, 3, 4 and 5 (b) 1, 2, 3, 4 and 5 (c) Summative testing
(c) 1, 2, 3 and 4 (d) 3, 4, 5 and 6 (d) Norm-referenced testing
399. A teacher presents information by accessing from 405. When the ‘test data’ tell us about a student’s level of
E-learning portals while displaying confidence in proficiency in a defined area. The procedure of eval-
the domain of concerned knowledge. This may uation is called  (June 2020)
involve which of the following? (a) Formative evaluation
(a) Non-observance of ethicality (b) Summative evaluation
(b) Inappropriate information coverage (c) Norm referenced testing
(c) Lack of harmony and relevance (d) Criterion referenced testing
(d) Lack of technical skill 406. Diagnostic evaluation ascertains
400. Given below are two statements, one labelled as (a) Students performance at the beginning of
Assertion (a) and the other labelled as Reason (R). instructions
Assertion (A): In Criterion-referenced tests, an indi- (b) Cause and remedies of persistent learning prob-
vidual’s score, and how that score is categorized, is lems during instructions
not affected by the performance of other students. (c) Degree of achievement of instructional objec-
Reason (R): Criterion-referenced tests compare tives at the end
a person’s knowledge or skills against a predeter- (d) Learning progress and failure after instructions

M01_MADAN 07_65901_C01.indd 89 27/12/22 8:15 PM


1.90 Chapter 1

407. Which of the given learner characteristics are of a specific subject. Which of the following can be
associated with field dependent (holistic/visual) termed as popular microcredentials ?
learning style? (June 2020) (a) MicroMasters (b) Specializations
1. Focusing on facts and principles (c) Nanodegrees (d) All of the above
2. Relating concepts to personal experience 410. e-PG Pathshala is an initiative of the MHRD under its
3. Formal interaction with teacher related to task at National Mission on Education through ICT (NME-ICT)
hand being executed by the UGC. Which of the following
4. Preferring to compete rather than cooperate institutions are closely associated with e-PG Pathshala?
5. Preferring to cooperate rather than compete (a) e-Adhyayan (e-Books)
6. Perceiving global aspects of concepts and materials (b) UGC MOOCs
Choose the correct answer from the option given below: (c) e-Pathya (Offline Access)
(a) 1, 2 and 3 (b) 2, 3 and 5 (d) All of the above
(c) 2, 5 and 6 (d) 4, 5 and 6 411. Which of the following can be termed as the major
408. Massive Open Online Courses (MOOCs) are challenges for MOOCs in India?
(a) flexible and open form of selfdirected, online 1. Technical Infrastructure
learning designed for mass participation 2. Diversified needs
(b) flexible and open form of teacher-directed, 3. Adoption of MOOC among learners
online learning designed for mass participation 4. Low investment
(c) flexible and open form of selfdirected, off-line 5. Low training
learning designed for mass participation Codes:
(d) flexible and open form of teacher-directed, (a) 1, 2 and 4 (b) 2, 3 and 5
offline learning designed for mass participation (c) 1, 2, 3 and 4 (d) 1, 2, 3, 4 and 5
409. Microcredentials are the series of related MOOCs
that allows a person to gain a deeper understanding
A S S E S S YO U R L E A R N I N G

M01_MADAN 07_65901_C01.indd 90 27/12/22 8:15 PM


Teaching Aptitude 1.91

Answer Keys

Concepts, Nature, and Characteristics


1. (a) 2. (a) 3. (c) 4. (b)  5. (c) 6. (d)  7. (a)  8. (d) 9. (b) 10. (c)
11. (a) 12. (b) 13. (b) 14. (d) 15. (b) 16. (c) 17. (d) 18. (b) 19. (a) 20. (a)
21. (d) 22. (b) 23. (b) 24. (a) 25. (a) 26. (a) 27. (c) 28. (b) 29. (c) 30. (d)
31. (c) 32. (b) 33. (c) 34. (c) 35. (d) 36. (d) 37. (b) 38. (b) 39. (b) 40. (d)
41. (d) 42. (a) 43. (a) 44. (b) 45. (c) 46. (d) 47. (a) 48. (c) 49. (b) 50. (a)
51. (a) 52. (b) 53. (a) 54. (b) 55. (a) 56. (b) 57. (d) 58. (d) 59. (d) 60. (b)
61. (d) 62. (b) 63. (a) 64. (b) 65. (d) 66. (a) 67. (b) 68. (c) 69. (d) 70. (a)
71. (b) 72. (d) 73. (a) 74. (b) 75. (d) 76. (d) 77. (d) 78. (b) 79. (b) 80. (d)
81. (a) 82. (d) 83. (a) 84. (d) 85. (b) 86. (b) 87. (a) 88. (d) 89. (a) 90. (b)
91. (d) 92. (c) 93. (a) 94. (a) 95. (d) 96. (d) 97. (b) 98. (b) 99. (c) 100. (c)
101. (d) 102. (a) 103. (b) 104. (d) 105. (c) 106. (c) 107. (a) 108. (d) 109. (c) 110. (b)
111. (c) 112. (a) 113. (b ) 114. (b) 115. (a) 116. (c) 117. (d) 118. (c) 119. (d) 120. (a)
121. (d) 122. (c) 123. (d) 124. (b) 125. (d) 126. (a) 127. (a) 128. (b) 129. (b) 130. (d)
131. (a) 132. (c) 133. (b) 134. (d) 135. (b) 136. (d) 137. (b) 138. (c) 139. (c) 140. (a)
141. (a) 142. (d) 143. (d) 144. (d) 145. (a) 146. (d) 147. (d) 148. (a) 149. (a) 150. (c)
151. (c) 152. (b) 153. (d) 154. (c) 155. (c) 156. (c) 157. (a)

Teaching Methods and Teaching Instructional Facilities


158. (a) 159. (d) 160. (a) 161. (b) 162. (a) 163. (b) 164. (a) 165. (a) 166. (d) 167. (d)
168. (c) 169. (d) 170. (b) 171. (c) 172. (a) 173. (d) 174. (a) 175. (a) 176. (d) 177. (a)
178. (c) 179. (d) 180. (c) 181. (b) 182. (d) 183. (d) 184. (d) 185. (a) 186. (b) 187. (d)
188. (b) 189. (c) 190. (a) 191. (a) 192. (d) 193. (a) 194. (a) 195. (b) 196. (d) 197. (c)
198. (b) 199. (d) 200. (d) 201. (b) 202. (d) 203. (b) 204. (d) 205. (c) 206. (c) 207. (a)
208. (d) 209. (a) 210. (d) 211. (d) 212. (d) 213. (a) 214. (a) 215. (a) 216. (d) 217. (b)
218. (c) 219. (b) 220. (d) 221. (c) 222. (b) 223. (b) 224. (d) 225. (b) 226. (c) 227. (b)
228. (a) 229. (d) 230. (a) 231. (d) 232. (d) 233. (c) 234. (d) 235. (a) 236. (b) 237. (d)
238. (d) 239. (a) 240. (a) 241. (d)

A S S E S S YO U R L E A R N I N G
Learner’s Characteristics
242. (d) 243. (b) 244. (b) 245. (d) 246. (d) 247. (a) 248. (b) 249. (b) 250. (c) 251. (c)
252. (c) 253. (a) 254. (a) 255. (d) 256. (c) 257. (d) 258. (d) 259. (d) 260. (a) 261. (a)
262. (d) 263. (a) 264. (a) 265. (a) 266. (a) 267. (a) 268. (a) 269. (a) 270. (a) 271. (d)
272. (d) 273. (c) 274. (b) 275. (a) 276. (a) 277. (d) 278. (b) 279. (d) 280. (c) 281. (d)
282. (a) 283. (b) 284. (b) 285. (a) 286. (c) 287. (c) 288. (d) 289. (d) 290. (d) 291. (b)
292. (c) 293. (c) 294. (d) 295. (d) 296. (d) 297. (d) 298. (a) 299. (b) 300. (c) 301. (d)
302. (a) 303. (a) 304. (c) 305. (a) 306. (b) 307. (a) 308. (d) 309. (a) 310. (b) 311. (c)
312. (b) 313. (a) 314. (a) 315. (d) 316. (a) 317. (c) 318. (d) 319. (c) 320. (d) 321. (c)
322. (d) 323. (b) 324. (a) 325. (c) 326. (a) 327. (c) 328. (c) 329. (c) 330. (a) 331. (d)
332. (d) 333. (d) 334. (d) 335. (c) 336. (d) 337. (b) 338. (c) 339. (a) 340. (c) 341. (c)
342. (d) 343. (c) 344. (b) 345. (b) 346. (c) 347. (c) 348. (d) 349. (b) 350. (b) 351. (b)
352. (b) 353. (d) 354. (c) 355. (b) 356. (c) 357. (c) 358. (d) 359. (a) 360. (b) 361. (d)
362. (d) 363. (d)

Learner’s Evaluation
364. (d) 365. (a) 366. (a) 367. (d) 368. (d) 369. (c) 370. (c) 371. (c) 372. (d) 373. (d)
374. (d) 375. (d) 376. (a) 377. (b) 378. (a) 379. (c) 380. (c) 381. (c) 382. (a) 383. (a)
384. (c) 385. (c) 386. (c) 387. (a) 388. (c) 389. (b) 390. (a) 391. (a) 392. (c) 393. (d)
394. (c) 395. (c) 396. (c) 397. (a) 398. (b) 399. (a) 400. (b) 401. (c) 402. (a) 403. (c)
404. (d) 405. (d) 406. (b) 407. (c) 408. (a) 409. (d) 410. (d) 411. (d)

M01_MADAN 07_65901_C01.indd 91 27/12/22 8:15 PM


This page is intentionally left blank

M01_MADAN 07_65901_C01.indd 92 27/12/22 8:15 PM


ChAPTER

2 Research Aptitude

01 Research: Meaning, Types and Characteristics

02 Positivism and Post-positivistic Approach to Research

03 Methods of Research: Experimental,


Descriptive, Historical

04 Qualitative and Quantitative Methods

05 Steps of Research
lEARnIng
OBJECTIVES 06 Thesis and Article Writing

07 Format and Styles of Referencing

08 Application of ICT in Research

09 Research Ethics

M02_MADAN 04_65901_C02.indd 1 23/12/22 7:14 PM


2.2 Chapter 2

The scientific method consists of systematic observa-


Research: Concept and Meaning tion, classification, and interpretation of data. Research
According to a famous Hudson Maxim, “All progress is is basically scientific in nature, so as to achieve the objec-
born out of inquiry. Our doubts lead to inquiries, and an tives that are based upon unbiased evaluation of data.
inquiry leads to an invention”. Research is not synonymous There is nothing like unscientific research approach, even
with common sense. The difference between research and in case of social sciences. A good researcher must be hard-
common sense lies in procedures and methods. working, diligent, open-minded, and should adopt a criti-
We acquire knowledge through different sources. The cal way of thinking.
process of acquiring knowledge has become very fast now. Here, we should know what is a scientific method.
There are basically five basic methods to acquire knowl- According to Random House Dictionary, in scientific
edge that gradually become research oriented. method, a problem is identified, relevant data are gath-
ered, a hypothesis is formulated, and that is empirically
1. Intuition: This is the first method to know. When we tested.
use our intuition, we rely on our guts, our emotions, Research is like carrying out a journey, and one must
and/or our instincts to guide us. be aware about its final destination. We should be clear
2. Authority: It means accepting new ideas because about the route to be adopted. The sequence of steps
some authority figure states that they are true. during the journey is not absolute. At every step, there
3. Rationalism: It involves using logic and reasoning to is multiplicity of methods, approaches, and procedures.
acquire new knowledge. Here, the experience of researcher helps our actions to
4. Empiricism: It involves acquiring knowledge through achieve our objectives.
observation and experience. The research can be conducted on issues such as
5. Scientific method: It is a process of systematically
collecting and evaluating evidence to test ideas and 1. How the COVID-19 disease has erupted and spread
answer questions. The pure basic form of scientific throughout the world.
method is called deduction that has been discussed 2. What are the common conditions prevalent among
later under positivism also. rural students?
3. How the New Education Policy has evolved after
Research is a structured enquiry that utilizes the accept- independence?
able scientific methodology to solve problems and create 4. What is the change in the level of understanding of
new knowledge that is generally applicable. The enquiry students with a change in the method of teaching?
is aimed at understanding an object or phenomenon or
solving a problem. Thus, the list of questions may be endless.
The term research comprises two words ‘re’ and ‘search’.
Generally, ‘re’ means again and ‘search’ means to find out.
Research is a careful investigation or inquiry, specially to
search for new facts in any branch of knowledge.
Research Objectives
According to Creswell, ‘research is a process of steps We need to narrow down to essentials in the research.
used to collect and analyze information to increase our
understanding of a topic or issue.’ Thus, we can say that 1. To gain familiarity or to develop a new insight with
research some phenomenon.
2. To review and synthesize the existing knowledge.
1. is a process of enquiry and investigation, 3. To investigate some existing situation or problem.
2. is systematic and methodical, and 4. To offer some solution to a problem.
3. increases knowledge 5. To explore and analyze more general issues.
6. To construct or create a new procedure or system.
Cook has beautifully outlined research as an honest, exhaus-
7. To generate new knowledge.
tive, and intelligent searching for facts and their meanings or
implications, with reference to a problem. To him, research The actual research may encompass one or a combination
is an acronym of the following that defines its essence. of any of the above objectives.
Motivational Factors in Conducting Research
R = Rational way of thinking
Usually research takes long time so its motivational aspect
E = Expert and exhaustive treatment
is very important.
S = Search and solution
E = Exactness 1. To acquire a research degree along with its consequen­
A = Analysis tial benefits.
R = Relationship of facts 2. To Face the challenges in solving unsolved problems.
C = Critical observation, careful planning, constructive 3. There is an intellectual satisfaction in doing innova-
attitude, and condensed generalization tive work.
H = Honesty and hard work However, the list for motivating people to undertake
research studies is not exhaustive.

M02_MADAN 04_65901_C02.indd 2 23/12/22 7:14 PM


Research Aptitude 2.3

(b) Standardization: It means using uniform,


White Hat 1. Information • Facts
consistent procedures in all phases of data
• Figures
collection.
• Data
(c) Operationalization of Concepts: It is basically
Red Hat 2. Emotions • Gut feel standardizing the meaning of concepts.
• Intuitions   An operational definition of a concept defines
• How it makes you feel that concept in terms of how it is measured or
what operations produced it.
Black Hat 1. Negative • Why it can’t be done (d) Avoiding Bias: Bias from external influences,
Aspects • What’s so hard in this personal beliefs, observers’ perspectives, and
task human expectations can distort all data.
• What’s won’t work
here 2. Reliability: Reliability in the context of research is
consistency. It refers to the extent to which an inves-
Yellow Hat 2. Positive • Hope tigation produces consistent results. It can also be
Aspects • The bright side termed as verifiability.
of the pic   If any research yields similar results each time, that
• Why it will work is undertaken with a similar population in the given
context and with similar procedures, it is said to be a
Green Hat 1. Creative • New ideas reliable research.
Things • Different perspective   Let us take an example, assume that a research
• Lateral thinking is conducted on the effects of watching television
Blue Hat 2. Process • Cool and calm on the class performance of the children and if the
• Control results conclude that watching TV causes low grades
• Organisation and in class, and if another sample taken from the popu-
process lation shows the same results with the same research
procedure, then we can say that the research proce-
dure and the outcomes are reliable.
  More the similarity in results, the more the reliabil-
Desirable Characteristics of Research ity of the research. The coefficient of determination is
also termed as reliability coefficient.
Before starting nature of research, there is a word of
caution. Which states that there is an overlapping in the Let us look the required criteria in a formal manner from
meaning and scope of these characteristics. The research exam point of view.
should be free of biases, prejudice and subjective errors. (a) Stability: Will the measure employed repeatedly
We make use of measurement as the assignment of on the same individuals yield similar results?
numerical values to objects or events according to rules. Test-retest correlation provides an indication
Then we are able to quantify the data in exact manner, of stability over time, its definition has been dis-
though ‘random errors’ do happen in measurements. cussed below also.
1. Objectivity: In simple words, being objective means (b) Equivalence: Will the measure employed by dif-
‘without any biased approach’. It should be without any ferent investigators yield similar results?
prejudices. We need to be objective so that it always (c) Homogeneity: Will a set of different ‘operational
reflects scientific and systematic aspect. All other char- definitions’ of the same concept (such as poverty,
acteristics are built around it. Researchers usually take richness, job satisfaction etc.) employed on the same
utmost precautions that results are not affected by their individuals, using the same data-collecting tech-
own presence, behaviour, and attitude. They critically nique, give us a highly correlated result? OR will all
examine the research methods to avoid any bias. items of the measure be internally consistent?
  The following measures can be adopted to ensure   Split-half correlation is related to homogene-
objectivity during research process: ity. The group is divided into half, there are two
(a) Procedural Safeguards: The processes involved in groups of two questions, and see if the two sub-
procedural safeguards are as follows: scales are highly correlated.
  (i) 
Keeping complete records of observations There are four general classes of reliability estimates, each
and data analyzes in a form that other of which estimates reliability in a different way. They are:
researchers can understand and evaluate.
(ii) Most scientific reports are written in a simi- 1. Inter-Rater or Inter-Observer Reliability: Used to
lar form and published by organizations assess the degree to which different raters/observers
of scientists. These reports communicate give consistent estimates of the same phenomenon.
ideas to the entire scientific community and 2. Test-Retest Reliability: Used to assess the consis-
open those ideas to criticism. tency of a measure from one time to another.

M02_MADAN 04_65901_C02.indd 3 23/12/22 7:14 PM


2.4 Chapter 2

3. Parallel-Forms Reliability: Used to assess the con- (d) Content Validity: The indicator measures all
sistency of the results of two tests constructed in the aspects of the construct or concept and not just a
same way from the same content domain. part of it.
4. Internal Consistency Reliability: Used to assess the (e) Criterion Validity: Criterion validity measures
consistency of results across items within a test. how well one measure (or indicator) predicts an
  Types of Errors: There can be two types of errors outcome for another measure. It is of three types.
that have been put in brief here. (i) Predictive Validity: If the test accurately pre-
dicts what it is supposed to predict.
(a) Random error: This can be attributed to a set of (ii) Concurrent Validity: When the predictor
unknown and uncontrollable external factors that and criterion data are collected at the same
randomly influence some observations but not oth- time. For example, when a test replaces
ers. Random error is seen as noise in measurement, another test that might be cheaper.
it may always happen, so it is usually ignored. (iii) Postdictive validity: If the test is a valid
(b) Systematic error: This error is introduced by measure of something that happened before.
factors that systematically affect all observations For example, does a test for adult memories
of a construct across the entire sample. This error of childhood events work.
is considered as a bias in measurement, it should (f) Construct Validity: The indicator measures the
be corrected to yield better results of the sample. construct in a manner that is convergent with
3. Validity: Validity in research mainly stands for the accu- other measures in terms of direction.
racy of procedures, research instruments, tests, etc. For example: The level of education and income
  The concept of validity can also be understood by level converge. The indicator also allows discrimi-
posing the question, ‘are we measuring or able to nation of opposing constructs.
measure what we originally intended to measure?’   Reliability is about the consistency of a mea-
  Validity means that research must be unbiased sure, and validity is about the accuracy of a
and free from any systematic errors, as these may measure.
impact the applicability of research. Without v­ alidity, 4. Accuracy: It is closely related to validity. It is also
research goes in the wrong direction. the degree of accuracy to which research processes,
  Generally, validity is termed to be much more instruments, and tools are related to each other.
important than reliability. To keep research on the Accuracy also measures whether the research tools
right track, it is imperative that the concepts are have been selected in the best possible manner and
defined in the best possible ­manner so that no error whether the research procedures suit the research
occurs during measurement. Different types of valid- problem or not.
ity are given as follows:   Rigorous scientific methods and procedures have
(a) Internal Validity: With higher internal valid- been adopted in research and each step in the research
ity, a researcher is able to establish better causal is tested for accuracy. Thus, choosing the best data
relationship between two or more variables. This collection tool improves the accuracy of research.
is specifically true in case of laboratory experi- 5. Credibility: It is the use of the best source of infor-
ments where cause and effect relationship is sup- mation and the best procedures in the research. The
posed to be more clearly established. use of secondary data saves time and reduces cost.
(b) External Validity: External validity means that However, the excessive reliance on secondary data
research findings study should be applied to the when the option of primary data is available entails
external situations. It refers to the extent to which the risk of reducing the credibility of the research.
the research outcome can be ‘generalized’ and Hence, it has to be a trade-off between primary data
applied to other cases that are not under study. The and secondary data.
external factors that may affect the study must be   The accurate references in research enhances
controlled. For example, the response of a respon- the credibility of research, but fake references also
dent in social sciences surveys could be affected by decrease the credibility of research.
the mere presence of a non-participant observer. 6. Generalizability: It is closely related to validity. It
  Thus, internal validity is called as credibility, and refers to the degree to which research findings can
external validity is called as generalizability (or be applied to a larger population. The sample con-
transferability). Generalization also means devel- sidered is representative of the whole population, so
oping a theory that applies to other situations. the findings should also be applicable to the whole
(c) Face Validity: By valid, we mean that survey population.
and questionnaire accurately measure what   Research adopts both quantitative and qualitative
they are supposed to measure. techniques. Social sciences have not been able to
  For example, all participants who filled a ques- establish generalizations equivalent to theories of
tionnaire meant for measuring certain personality the natural sciences or, to predict events accurately.
traits agree that this exercise appears to measure Perhaps, social sciences will never realize the objective
those traits and not something very different. of science as completely as natural sciences (physical

M02_MADAN 04_65901_C02.indd 4 23/12/22 7:14 PM


Research Aptitude 2.5

11. Logical: The statement, a good research is logical, implies


that research is guided by the rules of logical reasoning.
Induction and deduction are of great value in research,
sample which have been discussed under types of research.
selection 12. Replicable: This characteristic allows the results of
the research to be verified by replicating the study
and thereby building a sound basis for decisions.

Stopover
Which of the following can be defined as the most

desirable characteristic/s of research?
(a) Objectivity (b) Reliability
(c) Validity (d) All of the above
The correct option is (d).

N| n Positivism and Post-positivistic


Generalization Paradigms to Research
m| M

s| s 1. Paradigm: A paradigm is a comprehensive belief sys-


tem, world view or framework that guides research
and practice.
and biological phenomena) do. In fact, there are (a) A paradigm helps in making number of assump-
several limitations involved in the application of the tions about fundamental issues relating to nature
scientific approach in social sciences. and characteristics of truth or reality (ontology)
7. Empirical Research: It is based on real-life experi- and the theory of knowledge dealing with how
ences, direct experiences, or observation by the can we know the things that exist (epistemology).
researcher. It implies that research is related basi- (b) Paradigms guide research and practices used in
cally to one or more aspects of a real situation and the field.
deals with concrete data that provides a basis for (c) Paradigms are general methodological prescrip-
external validity to the results of the research. tions including tools for data collection and data
8. Systematic: For a research to be effective, it has to be analysis to conduct the work within the paradigm.
systematic. It is the only approach to undertake any
research work, and each step must follow the other. Paradigms are of three types:
There are a set of procedures that have been tested (i) Realism–to get information from external
over a period of time and are, thus, suitable for use world or sources.
in research. Therefore, each research should follow a (ii) C onstructivism–everyone has unique expe-
definite procedure. rience, there is nothing important outside
9. Controlled Factors: In real-life experience, there is from this type of experience.
always more than one factor that affects an outcome (iii) P
 ragmatism–Realism and constructivism
of an event. Similarly, in research, various factors may are termed as the two alternative ways to
affect the outcome, and some are taken as controlled fac- understand this world.
tors, whereas the others are tested for possible outcome.
  The concept of control implies that, in exploring 2. Metaphysics is the branch of philosophy that deals
causality in relation with two variables (factors), we with abstract concepts, such as being, knowing, iden-
set up a study in a way that minimizes the effects of tity, time and space. It is intimately connected with
other factors affecting the relationship. The controlled epistemology.
factors or variables have to be controlled rigorously. 3. Ontology refers to the branch of metaphysics deal-
  In physical sciences, it is easier to control such fac- ing with the nature of being and reality. In short,
tors as the experiments are conducted in laboratories. ontology refers to our ideas of reality and how it is
In social sciences, it is extremely difficult, as research is constituted.
carried out on the issues related to human beings living 4. Epistemology is the study of knowledge. It deals
in society, where exerting such controls is not possible. with the origin, nature, scope and methods to acquire
Moreover, within social sciences, the level of control knowledge. This term was first used by Frederick
may vary significantly from one discipline to another. Ferrier. There are basically two ways to acquire
10. Cyclical: Research is a cyclical process because it knowledge and they are rationalism and empiricism.
starts with a problem and ends with a problem that (a) Rationalism: Rationalism tends to believe
may become basis for further research. that logic and reason as the means of acquiring

M02_MADAN 04_65901_C02.indd 5 23/12/22 7:14 PM


2.6 Chapter 2

knowledge. Mind is given the authority over Comte divided the theological stage into three parts -
senses. This is b
­ asically a prior use of logic and rea- Animism, Polytheism and Monotheism.
son comes first to conclude something before expe- Positivists believe that sociology can and should use the
rience. Rationalism is associated with deduction. same methods and approaches to study the social world
(b) Empiricism: Empiricists claim that sensing expe- that “natural” sciences such as biology and physics use
rience is the ultimate starting point for all our to investigate the physical world. By adopting “scientific”
knowledge. The senses give us all our raw data techniques sociologists should be able, eventually, to
about the world and without this raw material, uncover the laws (social actions) that govern societies
there would be no knowledge at all. This is termed just as scientists have discovered the laws that govern
as a p
­ osteriori. It is related to induction. the physical world. Thus, the focus is on social actions.
5. Axiology: This is a branch of philosophy that studies Max Weber’s social action theory argues that individual
judgements about the value. Specifically, axiology is human beings engage in unique series of actions (social
engaged with assessment of the role of research­er’s actions) informed by cause and effect in social contexts.
own value system on all stages of the research pro­ We should say that social actions make the society to
cess. Axiology primarily refers to the ‘aims’ of the function. The methods being adopted in ‘natural sciences’
research. It checks whether actions are blamewor­thy should be adopted in the ‘social sciences’ also.
or praiseworthy. Accordingly, Bacon proposed that philosophers should
6. Theory is a set of systematically related statements, not attempt to wander beyond the ‘limits of nature’.
including some law-like generalizations that can be Saint–Simon applied the term positive in his Essay on the
tested empirically. These generalizations provide Science of Man to the sciences which were based on facts
hypothesis and these hypothesis determine what which have been observed and analyzed.
must be measured. Positivism relies on the following facts of science:
7. Research paradigms are models of the functions and
1. Science is deterministic as it explains cause and effect
interrelationships of a process, a ‘way of thinking’ about
relationships. Positivists believe the social sciences
something and how to study it. There is a difference
can be as rigorously scientific as natural ­sciences.
between natural sciences and social sciences and so is
2. Science is mechanistic as researchers develop hypoth-
the difference between research approaches relating
eses to be proved or disproved via applica­tion of spe-
to them. Hence, there are two competing paradigms
cific research methods.
to acquire knowledge. The paradigms are grouped as
3. Science uses methods such as selection of sample,
positivist paradigm and interpretive paradigms.
measurements, analysis, and reaching conclusions
about hypotheses.
4. Science deals with empiricism, where it is assessed
Epistemology:
Theoretical
as objective, as seen, or measured. Science must be
framework,
value free.
literature and Research Methodology 5. Positivists usually adopt a deductive approach and
research practice quantitative research aspects, wherein the concentra-
tion is on facts.
6. Positivism depends on quantifiable observations that
Ontology: lead to statistical analyzes. So, accordingly, the role
Assumptions Axiology: of the researcher is limited to data collection and
about the Value systems interpretation in an objective way. The researcher
nature of reality and ethical is independent and external in his research stud-
and knowledge principles ies, that also means that there is minimal interac-
tion with participants and the research is purely
objective.
7. We need to be ‘factual’ and ‘objective’ in positivism,
Figure 2.1 Research Methodology approaches so ontology in positivism is ‘realist’ and epistemol-
ogy is ‘objectivist’. ‘Factual’ knowledge is obtained
through observation (senses and measurement) in
Positivism is aligned with the hypothetico-deductive positivism.
model of science that builds on verifying a priori hypoth- 8. Positivists usually adopt deductive approach, the
eses and experimentation by operationalizing variables focus is on the facts. There is one reality, knowable
and measures; results from hypothesis testing are used to within probability. There is less scope for alternative
inform and advance science. methods in the research.
Positivism Approach
The term positivism was coined by the French philosopher Drawbacks of Positivism
Auguste Comte in 19th century and reflected through by Positivism as an epistemology is associated with the fol-
Francis Bacon, John Locke, Isaac Newton etc. Auguste lowing set of disadvantages.

M02_MADAN 04_65901_C02.indd 6 23/12/22 7:14 PM


Research Aptitude 2.7

It believes in the unity of method. Sociology is not dif- research design should be flexible and unstructured,
ferent from the natural sciences as far as the method of the methods should be valid, and the research design
enquiry is concerned. should generate small-scale and intensive data, using
insider accounts and based on descriptions of what is
1. Positivism relies on experience as a valid source of
seen and heard.
knowledge.
2. Verstehen: It is closely associated with the work
2. The principle of objectivity may be compromised to
of the German sociologist Max Weber. In social
some extent as there can be variations of actions of
sciences, such as anthropology and sociology,
individuals or relationships between individuals.
verstehen means a systematic interpretive process
3. Adoption of positivism in business studies and other
in which an outside observer of a culture attempts
studies can be criticized for reliance on status quo.
to relate to it and understand others. Verstehen
4. The principle of metaphysics may also be compromised
roughly translates to ‘meaningful understanding’ or
to some extent.
‘putting yourself in the shoes of others to see things
from their perspective’.
Post-positivistic A pproach
The overall society (including natural sciences) is not The method of the natural sciences (Physics, Chemistry,
static as assumed in positivism paradigm. There are and Biology) is explanation (erklären), whereas that of
traditions of ‘conflict’. The human beings are rational. history is understanding (verstehen).
According to Durkheim, society is a complex system that Note: As positivism is deductive in nature, so in research,
consists of inter-related and interdependent parts. There in positivism we want to move to inductive. Similarly
are interactions at micro-relational level. Thus, all the for post positivism (or realism) it is deduction, for criti-
above complications demand that there is requirement cal realism it is retroduction and for interpretativism it is
of different approach that was called as post-positivism abduction.
paradigm.
Thus, the focus of research has shifted from ‘reality’
to ‘critical reality’. This approach is more relevant to deal Stopover
with the problems of a more ‘complicated world’, and thus The research approach which is based on the assump-

it is more ‘innovative’ also to deal with more dynamic tion that social phenomena can be explained by
situations. Research is broader than set conditions. Post- observing cause and effect is:
positivist research has the following characteristics: (a) Positivism
1. Research is much broader rather than being special- (b) Interpretivism
ized. There are lots of different things which qualify (c) Qualitative
as a research; We need not to focus on ‘just the facts’; (d) None of the above
We should not always look for ‘correct techniques’ for
research. It represents a less stricter form of positiv- The correct option is (a).
ism. It is also known as ‘logical positivism’. Now the
social scientists and natural scientists share the same
goals for research and employ similar methods of Methods of Research
investigation.
2. Post-positivism paradigm is more constructivist that Sometimes positivism and post positivism approaches
considers that new knowledge is built on the previous can also be counted as types of research. Research itself is
knowledge. becoming more and more interdisciplinary.
3. The researcher’s motivations for research are cen- Table 2.1 gives an idea about the main basis adopted
tral and crucial to the project. This kind of research for the classification of research. Here, it is important
puts more focus on values, passion, and politics to mention that these approaches are not exclusive.
in research. It is basically qualitative or ‘mixed’ Research is usually interdisciplinary. In the follow-
research. ing classification, positivism may be identified more
4. The focus of ‘positivism’ was on reality. With more with the experimental, quantitative and deductive
involvement of individualism, and human beings in researches. Post positivistic approach should be identi-
a more direct manner, post-positivism becomes ‘criti- fied more with descriptive, qualitative and inductive
cal reality. Thus, its ontology is ‘critical reality’ and its approaches. Practically, most of the researches are
epistemology becomes ‘modified objectivist’. ‘mixed’ in nature. The use of word ‘more’ in the above
sentences is being done with a caution. Thus, research-
The following terms are more closely linked with post- ers should be innovative while categorizing the differ-
positivism approach. ent types of approaches.
1. Interpretive Paradigm: It is usually associated with
qualitative research strategies. It is specifically appli- Classification on the Basis of O bjectives
cable in social sciences, such as sociology, political On the basis of objectives, a research can be classified as
science, etc. According to interpretive approach, the follows:

M02_MADAN 04_65901_C02.indd 7 23/12/22 7:14 PM


2.8 Chapter 2

Table 2.1 Methods of Research 3. There is little possibility of using statistical tools to
verify an issue.
S. No. Basis of classification Types 4. There is a possibility that presence of an observer
could interfere with a respondent’s ability to answer
1. Objectives • Descriptive accurately. This effect is called as the “observer
• Correlational effect.”
• Exploratory It includes surveys and fact-finding enquiries with
• Experimental ­adequate interpretation.
The main examples are National Sample Surveys and
2. Outcome • Fundamental Census methods.
• Applied ‘The study of socio-economic status of distance educa-
tion students in India’ describes the gender composition,
3. Logic • Deductive economic status of students, rural-urban composition,
• Inductive etc. The findings (description) of one such study, say in
2009, can be different than what one would find in a sim-
4. Process • Quantitative ilar study in 2019. The aim of descriptive research is to
• Qualitative describe ‘what exists’ with respect to variables or condi-
tions in a situation.
5. Inquiry mode • Structured
• Unstructured
Applications of Descriptive Research
6. Idea or concept • Conceptual This helps in understanding the behaviours, traits and
• Empirical patterns of the participants. Close-ended questions can
reveal their opinions and attitudes. These may help in
making various decisions. This can be designed to meas-
ure trends as well. This can be used to do comparisons in
Descriptive Research terms of differences and similarities. An analysis can also
be conducted.
The main objective of descriptive research is describing Depending on the number of times the data are col-
the state of affairs as it prevails at the time of study. It aims lected, descriptive research can be of two types:
to accurately and systematically describe a population,
situation or phenomenon. Descriptive research answers 1. Cross-sectional Study: One-time interaction or one-
the questions who, what, where, when, and how. In time data collection.
simple words, descriptive research is all about describing 2. Longitudinal Study: A study that collects data more
the phenomenon, observing, and drawing conclusions than once from the same individuals.
from it.
Types of Descriptive Research
Benefits There are mainly three types of descriptive studies that
1. Descriptive studies mostly deal with large sample help in better understanding of the phenomena.
sizes, so their data collection is efficient and cost
effective. 1. Survey research
2. The variety of observational method, case study 2. Correlational studies
method and survey method can be used in it. 3. Causal-comparative studies
3. Descriptive study can use both qualitative and quan- Ex post facto, historical, exploratory, and analytical
titative data, thus it provides a holistic view of the research are the other variants of descriptive research.
subject.
4. As descriptive research is conducted in a respondent’s 1. Survey Studies: We have discussed many aspects of
own environment, so data integrity and accuracy are survey research under descriptive research.
assured. There is no manipulation of ‘environment’,
(a) The survey research is a non-experimental (that
so it is not a true experimental study.
is, it does not involves any observation under con-
trolled conditions).
Limitations (b) This is again a quantitative method used for
1. As the most of descriptive studies rely on observa- studying of large sample. The researcher collects
tional data, their research process is difficult to rep- data with the help of standardized questionnaires
licate. This makes it different from experimental or interviews which is administered on a sample
research. of respondents from a population.
2. The poorly framed questions can compromise the (c) This technique is used in social sciences that does
results of a descriptive study. data collection through direct (such as face to

M02_MADAN 04_65901_C02.indd 8 23/12/22 7:14 PM


Research Aptitude 2.9

face interview) and indirect observation (such Ex Post Facto Research


as opinions on online education facility of a 1. It is used in social sciences and business organizations.
university). 2. It is conducted in context of a phenomenon after that
  There is some scope of biased approach while has occurred or at the time of its occurrence.
compiling results though we make use of random 3. It basically deals with non-manipulated variables of a
sampling. phenomenon.
  There is description of sampling, randomization,
preparing questionnaire, mails, structured inter- Historical Research
views, unstructured interviews used in context of sur-
vey research in the coming paragraphs. 1. It is another dimension of descriptive research and
somewhat similar to ex post facto research.
  Random sampling is vital to the generalizability of
2. It usually focuses on the historical aspect of an issue of
a survey’s results.
interest or problem.
2. Correlational Studies: A correlational research 3. Examples are growth of trade unions in India, evolu-
design investigates relationships between two vari- tion of modern education system in India, etc.
ables (or more) without the researcher controlling
or manipulating any of them. It’s a non-experimen- Analytical Research
tal type of quantitative research. The correlation can
1. In this method, the researcher uses facts or informa-
be positive, negative or zero. It varies between -1 to
tion already available.
+1 including zero.
2. It attempts to make critical evaluation of the material.
Correlational studies are generally intended to answer
the following questions. Exploratory Research
Is there a relationship between two variables (or two 1. It is generally done at the beginning of a research.
sets of data)? If ‘yes’, then two other questions ­follow: It is undertaken to explore an area where little is
(a) What is the direction of the relationship, and is it known or to investigate the possibilities of undertak-
positive or negative? ing a particular research study, and is akin to feasibil-
(b) What is the magnitude of the relationship as indi- ity study or pilot study. A ‘small-scale study’ is under-
cated by the coefficient of correlation? taken to decide whether it is worth carrying out a
detailed investigation.
The correlational statistics will help test researchers’ 2. It attempts to clarify why and how there is a relation-
hypothesis about the relationship between two variables ship between two or more aspects of a situation or
and assess the magnitude of the relationship. phenomenon.
3. The purpose of exploratory research is to gain back-
Causal-comparative research: This method used to ground information, define terms, clarify the prob-
identify the cause–effect relationship between a depend- lems, develop hypothesis, establish research priorities
ent and independent variable. This relationship is usually and objectives, and develop questions to be answered.
a suggested relationship as a researcher can’t control an 4. It makes use of secondary data (mainly literature
independent variable completely. The researcher com- review), experience surveys, case studies, interviews
pares two groups to find out whether the independent (mainly focus group interviews), projective tech-
variable affected the outcome or the dependent variable. niques, and Delphi techniques.
(The dependent and independent variables have been
discussed under research process). This research tries
to answer ‘why’ also, thus it tries to find out the possible
causes of certain occurrences or their non-occurrence. Experimental Research
Then we draw conclusions. It is descriptive research, as
variables can’t be manipulated (at least in a big manner).
For example, ability variables (independent variable) “Men do not think they know a thing till they have
effect the achievement (dependent variable) in the exam. grasped the ‘why’ of it”
Similarly, personality variables lead to the development – Aristotle
of self concept. This research is of two types:

1. Retrospective causal comparative research: Earlier, we discussed causal-comparative research but that
Retrospective means looking backwards. Thus, was without any manipulation. In experimental research,
this research starts with effects (that have already there is scope of manipulation on independent variable
occurred) and it investigates causes. This type of so that we get an effect (that is dependent variable). As
research is more common. the manipulation is done only on independent variable,
2. Prospective causal comparative research: Prospective other variables are held constant.
means that is expected in future. The researcher begins The variable upon which the effects of changes are
with the causes and it investigates effects. observed is called the dependent variable, which is

M02_MADAN 04_65901_C02.indd 9 23/12/22 7:14 PM


2.10 Chapter 2

observed but not manipulated by the experimenter. v­ ulnerable to internal and external validity. They have no
The dependent variable is so named because its value is control group for contrast. They are of following types:
hypothesized, to depend upon and vary with the value of
1. One-shot case study research design: In this kind of
the independent variable.
research, one dependent variable is considered. It’s a
For example, to examine the effect of different teach-
post-test study as it’s carried out after treating what
ing methods upon achievement in reading, an investi-
presumably caused the change. It is difficult to draw
gator would manipulate the method, the independent
conclusions as there is no typical cause and effect
variable, by using different teaching methods in order to
relationship between the intervention and outcome.
ascertain their effect upon achievement, the dependent
  Educational institutions follow the pre-experimen-
variable.
tal research design to administer exams and evaluate
Examples are–why examination-related stress leads
students at the end of a semester. Students are the
to rote learning? Why and how does stress lead to heart
dependent variables and lectures are independent.
disease?
Since exams are conducted at the end and not the
beginning of a semester, it’s easy to conclude that it’s
Three Characteristics of Experimental a one-shot case study research.
R esearch 2. One-group pretest-posttest design: This test is
somewhat more structured. This research combines
These can also be termed as conditions for claim of
both pretest and posttest studies by testing a single
­causality.
group before and after administering the treatment.
1. Manipulation: Manipulation is defined as the first This design has minimal internal validity. It has no
feature of experimental design. Manipulated condi- external validity that means that it has less chances
tion (Independent variable) can also be termed as the for ‘generalization’. It has the minimal control.
‘treatment’ or ‘intervention’. Manipulation of an inde- 3. Static-group comparison: It is the best out of three.
pendent variable must involve the active intervention This research involves studying two groups by sub-
of the researcher. jecting one to treatment while the other remains
2. Control: We understand and able to identify the dif- static. After post-testing, all groups the differences
ferent factors affecting the internal and external are observed.
validity of an experimental design. In order to make   This design is practical but lacks in certain areas of
experimental design both internally and externally true experimental criteria.
valid, a check has to be put on the sources of error
affecting dependent variable. This is called ‘Control’. True Experiments
3. Random Assignment: Random assignment intends True experiments are a kind of ‘gold standard’ as they
to produce equivalent groups. It ensures group simi- help in evaluating other experiment designs. It is planned
larity. It means each participant has an equal chance to depict cause and effects relationships among variables
of being selected in a group and they are independent through manipulation, control, and random assignment
of selection of other participants. to the groups.
  Reflexivity refers to the circular relationships The true experiment is basically study of one population.
between cause and effect. A reflexive relation- True experiments take care of all the three aspects such
ship is bidirectional, with both the cause and the as manipulation, control and random assignment. The
effect affecting one another in a relationship in randomization is done to increase the equivalence of
which neither can be assigned as causes or effects. the groups. The control group does not receive any
For example, poverty is the main cause of unem- experimental treatment.
ployment, and unemployment is the main cause of
poverty. 1. The Pretest Post Test Design: This results of this
research depend on statistical analysis to approve or
Types of Experimental Research disregard a hypothesis. This accurate design can be
According to Campbell and Stanley, experimental conducted with or without a pretest on a minimum
research refers to that portion of research in which vari- of two dependent variables assigned randomly. It is
ables are manipulated and their effects upon other vari- further classified into three types:
ables are observed. 2. The posttest-only control group design: This
involves randomly selecting and assigning subjects
to two groups: experimental and control. Only the
Pre-Experimental Research Design experimental group is treated, while both groups
Pre-experiments refer to experiments for the purpose of are observed and post-tested to draw a conclusion
comparison. Experimental designs refer to true experi- from the difference between the groups. To evalu-
ments, quasi-experiments and some static experiments. ate the teaching methods of two teachers, they can
Pre-experiments are not typical experimental designs. be assigned two student groups. After teaching their
They are mainly used as exploratory tools and for com- respective groups on the same topic, a posttest can
parison with other experimental designs. They are determine which group scored better and who is

M02_MADAN 04_65901_C02.indd 10 23/12/22 7:14 PM


Research Aptitude 2.11

better at teaching. This method can have its draw- 1. The sample size is small so there are difficulties in
backs as certain human factors, such as attitudes of randomization. Quasi-experiments have outcome
students and effectiveness to grasp a subject, may measures, treatments, and experimental units.
negatively influence results. 2. They are more suitable for real natural world. They
3. Pretest-posttest control group design: In this test, are easier to be designed than true experimental
two groups are randomly assigned subjects. Both designs, which require random assignment of sub-
groups are presented, the experimental group is jects. They are more feasible to conduct.
treated and both groups are post-tested to measure 3. The criterion for assignment is selected by the
how much change happened in each group. researcher, while in a natural experiment the assign-
  For example, this research method can be used to ment occurs ‘naturally,’ without the researcher’s
evaluate employees’ skills. The candidates are asked intervention.
to take tests before filling a post for the purpose of 4. There may be threats to internal validity.
screening. That is done from a pool of candidates.
This allows organizations to identify skills at the time 1. Time-series design: Here, a dependent variable
of employment. After training employees on the job, is measured at many different points in time in one
organizations further evaluate them to test impact and group before and after a treatment is administered,
improvement. In this control group example employ- which is manipulated by the researcher.
ees are ‘subjects’ and the training is ‘treatment’. 2. Multiple time series: This design adds a control
4. Solomon four-group design: This design is a com- group to the study for the purpose of comparison.
bination of the previous two methods. Subjects are
randomly selected and assigned to four groups. Two Statistical Groups
groups are tested using each of the previous methods.
  True experimental research design should have a Statistical groups basically deal with the random aspect.
variable to manipulate, a control group and random The randomized designs provide flexibility in deal-
distribution, which is administered after exposure to ing with numerous variables in the field s­ettings. Such
a treatment. designs result from random assignment of participants to
treatment groups or are based on randomization.
Quasi-experimental Designs 1. Randomized Block Design: A randomized block
The word ‘quasi’ signifies that it resembles experimental design is an experimental design where the experi-
research. There is scope for manipulating the indepen- mental units are in groups called blocks. The treat-
dent variable before the dependent variable. ments are randomly allocated to the experimental
The quasi-experiments have the element of direc- units inside each block. When all treatments appear
tionality in them, but the participants are not randomly at least once in each block, we have a completely ran-
assigned. The confounding variables that affect partici- domized block design.
pant selection are still there. These designs are used for 2. Latin Square Design: It is the arrangement of t
their convenience. treatments, each one repeated t times, in such a way

Experimental
Research Designs

Pre- True Quasi


Experimental Experimental Statistical
Experimental

One-Shot Pre-Test -
Time Series Randomized
Case Study Post-Test
Blocks
Control Group

One Group Post-Test Multiple Time Latin Square


Pre-Test- Only Control Series Design
Post-Test Group

Solomon Four Factorial


Static Group
Group Design

Figure 2.2 Experimental Research

M02_MADAN 04_65901_C02.indd 11 23/12/22 7:14 PM


2.12 Chapter 2

that each treatment appears exactly one time in each Examples


row and each column in the design. 1. All men are mortal (general fact, applies to all men).
3. Factorial Design: It is used to examine treatment 2. Socrates is a man.
variations and can combine a series of independent 3. (Therefore,) Socrates is mortal (specific).
studies into one, for efficiency. Inductive Approach
It is also termed as bottom–up approach.
In inductive research, we move from specific to general.
Stopover This approach also involves the following three steps.
Consider the following statements:
1. True experiments must have a control group. 1. Observe different phenomena in the world.
2. Control group is a group of research participants 2. Search for a pattern in what is observed.
that resemble the experimental group, but do 3. Make a generalization about what is occurring.
not receive the experimental treatment. Examples
3. The control group provides a reliable baseline
data to which you can compare the experimental 1. Socrates is mortal (specific).
results. 2. Alexander is mortal (specific), Pluto is mortal and so
on (specific).
Which of the above are the correct statements? 3. All men are mortal (general).
(a) Only 1 and 2
(b) Only 2 and 3 Take another example: 3 + 5 = 8. Here, eight is an even
(c) Only 1 and 3 number. 7 + 59 = 66, and the result is again an even num-
(d) All of the above ber. Therefore, the conclusion is when an odd number is
added to another odd number, the result will be an even
The correct option is (d).
number.
Figure 2.3 shows the main components that form a
part of inductive and deductive approach.
Classification on the Basis of Application
On the basis of application, research is of two types,
namely pure (or basic research) and applied research. Generalization —
laws and theories
Table 2.2  lassification of Research on the Basis
C Induction Deduction
of Application
Fundamental Applied research Observations Explanation and
research and facts predictions
Addition to knowledge Solution to existing problems
Figure 2.3 Induction and Deduction
Discovery or invention Innovation or application
Mostly academic Practical use in solving a There is more discussion on inductive and deductive
problem approaches in Unit 6.

Extensive in nature Intensive in nature Classification on the Basis of I nquiry Mode


Basically, the process adopted to find answers to research
questions involves two approaches, structured and
Classification on the Basis of Logic unstructured.
In research, conclusions are based on two approaches, Structured Approach
deduction and induction.
1. The structured approach to inquiry is usually ­classified
Deductive Approach as quantitative research.
2. Everything that forms the research process, such
It is also termed as top–down or general-to-specific approach.
as objectives, design, sample, and the questions
In deduction, we start from a theory and try to prove
that a researcher plans to ask of respondents, is
it with the help of available information. The deductive
predetermined.
method involves the following three steps.
3. It is more appropriate to determine the extent of a
1. State the hypothesis (based on theory or research problem, issue, or phenomenon by quantifying the
­literature). variation. For example, how many people have a par-
2. Collect data to test the hypothesis. ticular problem? How many people hold a particular
3. Make decision to accept or reject the hypothesis. attitude?

M02_MADAN 04_65901_C02.indd 12 23/12/22 7:14 PM


Research Aptitude 2.13

Unstructured Approach in the data. Data collection and data analysis occur
1. The unstructured approach to inquiry is usually clas- simultaneously.
sified as qualitative research. 5. Ethnography: Ethnographic studies involve the col-
2. It allows flexibility in all aspects of the research lection and analysis of data about cultural groups.
process. Ethnography has been described as “encountering dif-
3. It is more appropriate to explore the nature of a prob- ferent worlds and making sense of them”. It means that
lem, issue, or phenomenon without quantifying it. one world (of researcher) making sense of actions of
other world (of participants from some tribal society).
Classification on the Basis of Process Ethnography means ‘learning from people’, where
the researcher frequently lives with the people and
There are two types of researches—quantitative and becomes a part of their culture. The researcher explores
qualitative. with the people their rituals and customs. An entire cul-
Quantitative research: This basically helps in deduc- tural group may be studied or a subgroup in the culture.
tive approach, it deals with numbers and statistics. We can cite example of tribal society in Madhya Pradesh.
Quantitative methods allow you to systematically meas- Ethnographers interview people who are most knowl-
ure variables and test hypotheses. edgeable about the culture. These people are called key
informants. Data are generally collected through partic-
Qualitative research: This deals with words and mean- ipant observation and interviews. Data collection and
ings. Qualitative methods allow a researcher to explore analysis occur simultaneously. As understanding of the
the concepts and experiences in an exhaustive manner. data occurs, new questions emerge. The end purpose
of ethnography is the development of cultural theories.
We have tried to differentiate between the two researches Ethnographers study how people live and how they
with the help of table. communicate with each other.
1. In-depth Interview: This is usually a one-to-one 6. Semiotics: The world is full of signs and symbols
interview, with one participant at a time. Though it broadcast by brands and consumers in the mass
is systematically planned, it may have unstructured media. Semiotics is a theory that can be used to
elements as well. The researcher prepares questions uncover hidden meanings in the messages through
in advance to make sure that only the most important ‘Decoding. It also reflects that how meaning is made
questions are asked to the participant. The interview within a culture.
can last anywhere between twenty minutes to half an 7. Transformative/Emancipatory paradigm: This
hour, during which the researcher tries to collect as research focuses on the view that reality is shaped
much meaningful data as possible from the partici- by culture, politics, economics, race, gender, ethnic-
pants to draw inferences. ity and disability. Values are considered to be impor-
2. Focus Group: A focus group comprises around 6–10 tant, particularly as values and beliefs differ from
participants who are usually subject matter experts. one culture to the next. Knowledge and understand-
A moderator, usually an experienced person, is ing are aimed at critical praxis.
assigned to a focus group to facilitate the discussion. 8. Case Study Research: A case study may be consid-
The role of a moderator is to probe the participants by ered as quantitative or qualitative research depend-
asking the correct research questions so as to collect ing on the purpose of the study and the design cho-
research-related information. sen by the researcher. Case studies are not used to
3. Narrative Research: It is an approach to review the test hypotheses, but hypotheses may be generated
literature. Sometimes, it is contrasted with a system- from case studies. Data may be collected in case stud-
atic review. It tends to be less focused than a system- ies through various means such as questionnaires,
atic review and seeks to arrive at a critical interpreta- interviews, observations, or written accounts by the
tion of the literature that it covers. subjects. This type of research is used in the areas of
4. Phenomenology: This research examines human management, education, philosophy, and psychol-
experiences through the descriptions provided by the ogy. This method involves deep digging into the
people involved. These experiences are called ‘lived developments and collecting data.
experiences’. The researcher must first identify what 9. Content Analysis: This is closely linked with case
she or he expects to discover and then deliberately put study. Content analysis is also known as text analysis.
aside these ideas; this process is called ‘bracketing’. This method is a bit different from other qualitative
A researcher wants to explore areas where there is research methods. It is used to analyze social life by
little knowledge. For example, interviewing the wives decoding words, texts, etc., through any available
of 10 prisoners of war and asking them to describe form of documentation. The researcher studies and
their experiences. In phenomenological research, understands the context in which the documents are
respondents are asked to describe their experiences furnished with the information and then tries to draw
as they perceive them. They may write about their meaningful inferences from it.
experiences, but information is generally obtained 10. Grounded Theory: Grounded theory is a qualitative
through interviews. Themes and patterns are sought research approach that was developed by two

M02_MADAN 04_65901_C02.indd 13 23/12/22 7:14 PM


2.14 Chapter 2

sociologists, Glaser and Strauss in 1967. The data is


collected and analyzed and then a theory is developed Action Research
that is grounded in the data. Data are gathered in Action research refers to a wide variety of evaluative,
naturalistic settings (field settings). Data collection investigative, and analytical research methods designed to
primarily consists of participant observation diagnose problems with the objective to help researchers
and interviews. The data are recorded through to find practical solutions to address them in a quick and
handwritten notes and tape recordings. A process efficient manner. Action research is basically ‘learning by
called ‘constant comparison’ is used, in which data doing’.
are constantly compared to data that have already The term ‘action research’ was coined by Kurt Lewin
been gathered. It reflects the fundamental patterns in during 1940s. He is considered as father of ‘action
all social life. These patterns are called ‘basic social research’. He defined Action Research as ‘a spiral of steps,
processes’. Adjustments are to be made to modify the each of which is composed of a circle of planning, action,
theory. Role play, simulation, and diary methods are and fact-finding about the result of the action’. Cohen and
also used in research. Manion (1989) described action research as, ‘a small-
11. Hermeneutics: This is the theory and methodology scale intervention in the functioning of the real world and
of interpretation, especially the interpretation of a close examination of the effects of such intervention’. A
biblical texts, wisdom literature, and philosophical lot of research has taken place on ‘action research.’
texts.
Once the candidates go through the research process, Features of Action Research: The main features of
they can have a look at the following table again: ‘action research’ have been described below.

Table 2.3 Difference between Quantitative and Qualitative Research


Quantitative Research Qualitative Research
It is very clear to researcher, what to do, what not to Here, the main objective is to develop an understanding
do by employing statistical, logical and mathematical on human beings/social sciences to know what people
techniques. feel and think.
It adopts ‘objective’ approach that describes, explain and This is basically a ‘subjective research’ that explores and
quantify the problem. gains an understanding of the problems.
The approach is top–down. The objective is to confirm The approach is bottom–up. It explores to know ‘How’
the ‘what’ and ‘when’. and ‘When’.
This follows general to specific approach. This basically This is actually follows specific to general approach.
follows ‘deductive reasoning’ that starts from the theory, This mainly follows inductive reasoning that starts from
it forms a hypothesis, then goes for observation and observation, it tries to develop a pattern, then it develops
finally confirm the hypothesis. a ‘tentative hypothesis’ and finally forms a theory.
The data collected mainly includes experiments, The data is mainly verbal (such as words and images),
numbers and statistics, polls, controlled observations, in-depth interviews, focus group discussion, observation
longitudinal studies, structured interviews, surveys and and document reviews. We make use of ‘content analysis’ to
statistical records. The sample sizes are big in number. obtain and interpret information. The sample sizes are less
in number.
Random sampling is a type of probability sampling Non-random sampling is a sampling technique where
in which the researcher ‘randomly’ selects a subset of the sample selected will be based on factors such as
participants from a population. Each member of the convenience, judgement and experience of the researcher
population has an equal chance of being selected. This and not on probability. All units of the population do not
‘randomization’ is meant for objective and unbiased an equal chance of participating in the survey. Therefore,
approach in scientific methods. the results cannot be generalized for the population.
Result-oriented inquiry. Process-oriented inquiry.
The results are specific and thus findings are more The results are descriptive and thus findings are less
generalizable. generalizable.

M02_MADAN 04_65901_C02.indd 14 23/12/22 7:14 PM


Research Aptitude 2.15

1. Situational and problem solving perspective: It Now, we need to look at the main steps of ‘action
usually emerges out of situational needs and a solu- research’.
tion to a problem is also designed with respect to the
1. Planning: First thing to do is to analyze the problem
situation.
scientifically in the specific perspective in which
2. Intervention in real world: As problem emerges in
the problem has emerged. Planning covers the ini-
practical real-life situation, so action is to be taken in
tial reflection.
real world as well.
Planning entails identification of the problem and
3. Adoption of alternative practices: Through action
changes a teacher can make to overcome the problem.
research, we intend to discover new and alternative
2. Action: After the planning stage in which all the
ways to achieve our objectives, be it teaching or man-
procedures of investigation have been determined,
agement sector.
comes the action stage. This stage is time bound.
4. Immediate problematic situation: The focus may be
The researcher has to administer tools to collect data
more on problems that need urgent attention.
and information. Systematic analysis has to be done.
5. Goals of social science: It is mostly in social situa-
Results have to be recorded.
tions, such as in educational institutions.
3. Observation: During ‘action research’, observation
6. Collaborative and participatory.
of tools has to be done cautiously. Observation has
7. Co-learning: As action research is collaborative
to be objectively done without any presuppositions.
approach, co-learning is also the outcome.
The detailed observations, monitoring, and record-
8. Self-evaluative: Just as action research is self-initi-
ing enables you to report your findings to others.
ated since it evolves out of the perception of problems
Those involved in action research should also keep a
by the practising individual or group, it becomes self-
detailed diary or journal.
evaluative where the action research team evaluates
4. Reflection: Once the results have been obtained and
the outcome of the exercise.
analyzed and conclusions drawn, you are ready to ini-
9. Action research is a process: It has been discussed in
tiate changes in your teaching strategy. This change
the following paragraphs.
or modification in the style of teaching is the result
of action-research aimed at improving the teaching-
Action Research Process or Cycle learning process. It is also aimed at adopting a new
Action research is a process by which change and under- method. You would also reflect on the efficacy of the
standing can be pursued at one time. It is usually described changes you are bringing in.
as cyclic, with action and critical reflection taking place in
turn. The reflection is used to review the previous action Scope of action research in education: We can identify
and plan the next one. It is commonly done by a group of the following areas in the scope of action research.
people, though sometimes individuals use it to improve 1. Curriculum planning and course material development
their practice. 2. Programme delivery and learning strategies
Stephen Kemmis has developed a simple model of 3. Student assessment and evaluation
the cyclical nature of the typical action research ­process, 4. Staff development
which consists of four steps: such as planning, acting, 5. Management and administration
observing, and reflecting. 6. Behavioural changes, like attitudes, values, staff
motivation, etc.
If required Revised If required
Plan Cycle 3 Thus, almost all areas have the potentiality of using action
plan research for solving problems and for improvement of
practices.
Reflect Act Reflect Act

Mixed Research
Observe Observe
Both quantitative and qualitative researches are not
Cycle 1 Cycle 2 exclusive. Qualitative research may end in a hypothesis
that can be quantitatively tested later.
Figure 2.4 Action Research Quantitative research may involve qualitative research
elements.
In action research, development and research take Quantitative research may answer questions, such as
place in a simultaneous manner. For example, a teacher the extent and pattern of poverty in India, but it may not
may face many problems and he or she tries to find an be efficient for answering questions, such as what is the
instant solution based on previous experiences, but experience of facing poverty, hardships, consequences,
many times such solution is either partial or temporary. and circumstances that lead to poverty. This may be
Thus, a teacher needs to find a solution which is based answered by qualitative research. As quantitative research
on research, so that the solution obtained really solves is generally well known, it may be useful to outline when
the problem. Here, action research comes handy. qualitative research is needed.

M02_MADAN 04_65901_C02.indd 15 23/12/22 7:14 PM


2.16 Chapter 2

Classification on the Basis of Concept Stopover


Conceptual Research Which of the following is a form of explanatory

Conceptual research is generally used by philosophers research in which the researcher develops a theoreti-
and thinkers to develop new concepts or to reinterpret cal model and empirically tests the model to deter-
existing concepts. It is related to some abstract idea or mine how well the model fits the data?
theory.
(a) Causal modelling
Empirical Research (b) Predictive research
Empirical research relies on experience or observation (c) Descriptive research
alone, which is without due regards for system and the- (d) Exploratory research
ory. It is a data-based research coming up with conclu- The correct option is (a).
sions that are capable of being verified by observation or
experiment.
In this research, the researcher must formulate a
working hypothesis. He collects data to prove or disprove
his hypothesis. The researcher is in ­control over the facts. Steps Of Research
Empirical research is appropriate when proof is sought
that certain v­ariables affect other variables in one way or The research process usually adopts the following three
the another. criteria of undertaking a research.

The Concept of Triangulation 1. It is conducted within a framework of a set of


approaches. It may be qualitative or quantitative. It
We know that being ‘objective’ is the most desirable fea- depends upon the academic discipline as well.
ture in research. 2. The logical sequencing of steps undertaken in order to
1. Triangulation is mostly used in qualitative or social find the answers to the research questions is termed
research to deal with the issue of multiple realities in as research methodology. It adopts procedures, meth-
society. ods, and techniques that are tested for their validity
2. It also solves the research problem in multiple ways to and reliability.
increase the validity of the research. 3. It is unbiased and objective.
3. It overcomes the problem of subjectivity and biasness. According to the definition given by Creswell, research
4. Triangulation makes use of multiple data sources, consists of three basic steps:
multiple investigators, and most importantly, mul-
tiple methods (such as participant observation, focus 1. Formulating a research question or posing a problem,
groups, case studies and so on) to get a complete to which the researcher wants to find answers to.
understanding of the social phenomena. 2. Collecting data to answer the questions.
3. Present an answer to the question.
Hermeneutics is the theory and methodology of
interpretation, especially the interpretation of biblical These basic steps have been elaborated further in Table 2.4
texts, wisdom literature, and philosophical texts. and discussed in detail in the following paragraphs.

Table 2.4 Research Process—A Snapshot


Step 1: Formulation of Step 2: Preparing Step 3: Developing Data Step 4: Selecting Samples
Research Problem Research Design Collection Instrument Types of samples
• Literature review • Problem structure (Research Instrument) • Probability
• Formulation of objectives • Study design • Types of data • Non-probability
• Identifying research • Experimental study • Methods of data • Determining sample
variables and measuring • Non-experimental collection design
scales study • Designing research tool
• Formulating hypothesis
Step 5: Writing a Step 6: Collecting Step 7: Processing and Step 8: Writing a
Research Proposal Data Analyzing Data Research Report
Main elements • Observations • Editing and Coding • Title page
– Need • Interviews • Classification of data • Table of contents
– Benefits • Questionnaire • Tabulation • Chapters
– Types of data • Schedules • Analysis • Bibliography
– Justification for funds and • Hypothesis testing • Appendices
other resources • Generalizations

M02_MADAN 04_65901_C02.indd 16 23/12/22 7:14 PM


Research Aptitude 2.17

4. It helps contextualize the findings. It means how


Step 1: Formulation of the Research value addition has been done by the researcher to the
Problem existing stock of knowledge.
The manner in which a problem is formulated determines The procedure for reviewing literature covers searching
almost every step that follows. the existing literature, reviewing it, and developing a the-
oretical and conceptual framework.
Steps in F ormulation of a Research Problem The main sources of literature review are books and
journals. In both cases, specifically in journals, there can
During this initial phase of ‘discovery of context’, we be a gap of two to three years between the completion of a
take help from our observations, belief, information research project and the publication in a journal.
and general knowledge, etc. We are guided towards As with books, the researcher needs to prepare a list
a new idea or a different way of thinking about some of journals for identifying the literatures relevant to his
phenomenon. research. Nowadays, researchers make extensive use of
A reasonable level of knowledge in the broad subject internet sources for literature survey and review, and at
area is required to work through these steps. Thus, follow- the same time, the researcher should be careful about the
ing steps are required to formulate a research problem. authenticity of the content.
Step 1: Identifying a Broad Area of Interest: What Bibliography given at the end of a project gives a clear
really interests me as a professional? As a teacher, I might and complete description of the sources that were used
be interested in the area of teaching methodology or while preparing the report.
exploring the reasons behind increasing acts of violence
among students. Formulation of O bjectives
Step 2: Dividing Broad Areas Into Subareas: ­Suppose I Objectives are the goals you set out to attain in your study.
want to study acts of violence among students. It can have They inform the reader what the researcher wants to
various subareas: (1) ­profile of families these ­students accomplish through the research work. The wording of
come from, (2) ­ profile of perpetrators, (3) causes of the objective should be very ­precise and specific.
­violence, (4) role of politicization of education, (5) impact Objectives can be written under two headings:
on society and so on.
1. Main objectives or aims
Step 3: Focusing on and Selecting an Area of I­ nterest: 2. Sub-objectives
Here, we follow the concept of ‘selective perception’. We
may not be in a position to focus on each and every area The main objective is an overall statement of the study.
from research point of view. It also states the main associations and relationships that
we want to establish. The sub-objectives are the specific
Step 4: Identifying the Gaps and Raising Research aspects of the topic that you want to investigate within the
Questions: Within an area, list all the questions the main framework.
researcher wants to find answers to.
1. They should be listed numerically.
Step 5: Formulation of Broad Objectives: ­Objectives 2. The wording should be clear, complete, and specific.
grow out of the questions. 3. Each objective should contain only one aspect of the
study.
Step 6: Assessing and Reviewing Objectives: As a 4. Use action-oriented words or verbs when writing
researcher narrows the research problem, the specific objectives.
identification of study population is crucial in order to
select the appropriate respondents. The objectives should start with words such as ‘to deter-
mine’, ‘to find out’, ‘to ascertain’, ‘to m
­ easure’, ‘to explore’,
Extensive L iterature R eview etc. The wording of objectives determines the type of
research (descriptive, correlational, and experimental)
Literature review is an integral part of the entire research and the type of research design you need to adopt to
process. It makes significant contribution to each and achieve them. For example, in case of descriptive studies,
every operational step at a later stage. After passing the objective can be stated as, ‘To describe the types of
through this stage, a researcher is able to acquaint one- incentives provided by the organizations in Chandigarh
self with the available body of knowledge in the area of to their employees in the IT industry.’
interest. The main objectives of literature review are as In correlational studies, it may state, ‘To ascertain the
follows. impact of coaching classes on students’ performance.’
1. It broadens the knowledge of researcher about the
research problem. Main C onsiderations in S electing a R esearch
2. It brings better clarity and focus to the research
problem and also helps to improve the authenticity Problem
of research. These help ensure that your study remains m
­ anageable
3. It helps improve the research methodology. and that you remain motivated.

M02_MADAN 04_65901_C02.indd 17 23/12/22 7:14 PM


2.18 Chapter 2

You can find innumerable research problems in any For example, if richness (or say poverty) is a concept
area. Obviously, you cannot study all the problems. or construct, then assets and income are its indicators.
The asset values and annual incomes are variables.
1. Interest: Research is usually time-consuming and ‘Concept Mapping’ is a visual representation of the
entails the use of resources. Many unforeseen prob- relationship between ideas generated by an individual
lems may crop up. So the topic should be of interest to or group.
the researcher to sustain the desired motivation level.
2. Manageable Magnitude: The topic should be man- Types of Variables
ageable within the available time and resources. The
broader topic should be broken down to be a manage- The variables are classified into categorical and quanti-
able level. It should be specific and clear to the high- tative variables. Quantitative variables vary in degree or
est extent possible. The cardinal principle is to choose amount, such as annual income, and categorical variables
a research problem that is not too small to be insig- vary in type or kind, such as gender. Categorical variables
nificant but not too big to be impossible. have been discussed in subsequent discussion.
3. Concept Measurement: The clarity about indicators On the basis of causation, the variables are basically of
and measurement of concepts is required. The idea of two types, namely independent and dependent variables.
construct is important here. In an experiment, one discovers and confirms a rela-
4. Level of Expertise: The adequate level of skills for tionship between an independent variable and a depen-
the task is required. dent variable.
5. Relevance: Though relevance is again a subjective 1. Independent Variables (IV) are also known as
term, the research should add to the existing stock manipulated, experimental, or treatment ­variables.
of knowledge and bridge the current knowledge   They become the cause of another variable. It is the
gaps. variable that is manipulated by the researcher in an
6. Availability of Data: The availability of data sources experiment.
is to be ensured before finalizing the topic. 2. Dependent Variables (DV) are also called the out-
7. Ethical Issues: The ethical issues and their rem- come or response variables. The dependent variable
edies must be anticipated before formulating the is the variable that changes as a result of changes
problem. made on the independent variable.
  For example, in the study about impact of ­coaching
Example of Main Objective on student performance, coaching is independent vari-
The main objective is to explore the relationship between able and student performance is dependent variable.
the use of modern teaching techniques and student
performance. In addition, there can be intervening variables and extra-
neous variables.
Sub-objectives 1. Intervening Variables: These are also termed as
The sub-objectives are as follows: mediator variables. They establish a link between IV
1. To find out the extent of relationship between the use and DV. These are variables through which one vari-
of modern teaching techniques and student perfor- able affects another. These are helpful to understand
mance. the process.
2. To compare the use of modern teaching techniques in   For example, tissue damage is an intervening vari-
government and private schools. able in smoking and lung cancer relationship. We can
3. To study the impact of modern teaching and the level use arrows (which mean causes or affects) and draw
of motivation of students to learn. the relationship that includes an intervening variable
like shown in the figure.
Concepts and Variables Smoking Tissue damage Lung cancer
The meanings of terms, such as teaching effective-
ness, class performance, job satisfaction etc. may vary 2. Extraneous Variable:
from one person to another, and from one place to (a) In real-life situations, there can be many factors or
another. Concepts (or constructs) are mental images variables that may affect the outcome. These vari-
and thus, are not directly measurable. For research, ables are termed as extraneous variables.
we have to define concepts so that they are under- (b) Extraneous variables also affect the dependent
stood in the same sense by respondents in case we variable, although these are not manipulated by
collect data. the researcher.
The concepts have to be made measurable, in a form (c) They may mask the relationship between inde-
so that data can be collected in an easy manner. It pendent variable and dependent variable.
means that concepts should be capable of assuming dif-   Extraneous variables may directly affect the
ferent values. Here, the terms indicators and variables dependent variable or may combine with the inde-
come into picture. Anything capable of assuming differ- pendent variable to produce an affect. Therefore,
ent values is known as variable. extraneous variables must be controlled so that

M02_MADAN 04_65901_C02.indd 18 23/12/22 7:14 PM


Research Aptitude 2.19

Independent, dependent, and extraneous variables in a causal relationship

Smoking Coronary heart disease

(Assumed cause) (Assumed effect)

Independent variable Dependent variable

Affect the relationship

• The age of the person


• The extent of his/her smoking
• The duration of smoking
• The extent of daily exercise, etc.

Extraneous variables

Figure 2.5 Relationship between Independent, Dependent, and Extraneous Variables

the experimenter can determine whether the   (ii) Dichotomous Variable: Two categories, such as
dependent variable changes in relation to a varia- male and female, rich and poor, etc.
tion in the independent variable. (iii) Polytomous Variable: More than two catego-
  They actually compete with the independent ries, such as below average, average, above aver-
variable in explaining the outcome. age, etc.
(d) If an extraneous variable is the real reason for an 2. Continuous variables: These variables are variables
outcome instead of independent variables, then that can take on any value within a range. Continuous
it is also known as confounding variable because variables are also considered metric or quantitative
it has confused or confounded the relationship variables, where the variable can have an infinite
we are interested in. number or value between two given points. Example
Take an example from teaching. Speed of learning is height, weight etc that can take any possible value.
depends on meaningfulness of topic–the greater the 1. Constant Variable: Only one value, such as flower,
meaningfulness, the faster the learning. Therefore, the tree, etc.
speed of learning is called dependent variable and mean- 2. Dichotomous Variable: Two categories, such as male
ingfulness is an independent variable. and female, rich and poor, etc.
A mediating variable (or mediator) explains the pro- 3. Polytomous Variable: More than two categories,
cess through which two variables are related, while a such as below average, average, above average, etc.
moderating variable (or moderator) affects the strength
and direction of that relationship. Measuring Variables
On the basis of study design: The variables can be of Measurement of variables is central to research ­ studies.
two types. According to Stevens, measurement scales can be of four types:
1. Active variables: They can be manipulated or con- Scale: A scale is a device or an object used to measure or
trolled during the study. quantify any event or another object.
2. Attribute variables: These cannot be changed,
controlled, or manipulated. such as gender, age, etc. Levels of Measurements
On the basis of Unit of Measurement: The variables can There are four different scales of measurement. The four
be categorical or continuous. types of scales are:
1. Categorical variables: These are measured on
nominal or ordinal scale and can be further classified 1 st–Nominal Scale
as follows. It is basically ‘named variables’. A nominal scale is also
(i) Constant Variable: Only one value, such as termed as classificatory scale, a variable is classified into
flower, tree, etc. two or more categories. These tags or labels helps in iden-

M02_MADAN 04_65901_C02.indd 19 23/12/22 7:14 PM


2.20 Chapter 2

tification of objects. Thus, a nominal scale usually deals Ratio scale has unique feature such that it allows
with the non-numeric variables or the numbers that do unit conversions like kilogram – calories, gram – calo-
not have any value. ries, etc.
It is qualitative in nature as it helps in identification of
Example: For percentage score in 12th standard.
objects. They don’t define the object features. The only
permissible aspect of numbers in the nominal scale is Less than 60 percent
“counting.” Between 60 – 75 percent
For example, a gender can be classified into three cat- Between 76 – 90 percent
egories – Male, Female and Transgender. Between 86 – 95 percent
More than 95 percent
2 nd–Ordinal Scale
Attitudinal Scale
It is named + ordered. The ordinal scale reports the
If you want to ascertain the attitude of students towards
ordering and relative ranking of data without establishing
a teacher, the questionnaire framed may be open-ended
the degree of variation between them. It identifies and
or close-ended. If the ­questionnaire is open-ended, it
describes the magnitude of a variable. This data is again
may invite respondents to describe the attitude they hold
qualitative data or categorical data. It can be grouped,
towards teaching quality. If the researcher has framed
named and also ranked. As it is qualitative, the interval
close-ended questions, the respondent is given options
properties are not known.
such as strongly agree (SA), agree (A), undecided (U),
Examples: disagree (D), and strongly disagree (SD).
1. Ranking of university students: 1st, 2nd, 3rd, 4th,
5th etc. Types of Attitudinal Scale
2. Evaluating the frequency of occurrence: Very As the attitudinal scales are very important in qualitative
often, Often, Not so often, Not present at all research, the different types of attitudinal scales are as
3. Degree of agreement: Totally agree, Agree, Neutral, follows:
Disagree, Totally disagree 1. Likert Scale: It is also termed as summated rating
scale. It is the easiest one to construct. It is based on the
3rd–Interval Scale assumption that each statement or item on the scale
It is named + ordered + proportionate interval between has equal attitudinal value, importance, or weight. The
variables. It is a quantitative measurement scale. It can quality of a teacher may have many dimensions, such as
quantify the difference between the values. Thus, vari- use of knowledge base, communication skills, presenta-
ables are measured in an exact manner, not as in a ‘rela- tion of contents, organization of material, promptness
tive’ way in which the presence of zero is arbitrary. It to solve student problems, etc. The respondents may
allows calculating the mean and median of the variables. have different attitudes towards different aspects.
It is useful in statistical calculations, as it helps to assign
any numerical values to arbitrary assessment such as feel-
ings, calendar types, etc. For example, 60°C is colder than Statements About Teacher SA A U D SD
65°C. The temperature difference of 5°C is same as that
between 50°C and 55°C. In other words, the difference of 1. Knows the subject well (+)
5°C in both intervals shares the same interpretation and 2. Willing to solve ­students’
meaning. problems (+)
Examples are Likert Scale, Net Promoter Score (NPS)
and Bipolar Matrix Table 3. Have poor ­communication
skills (-)

4th–Ratio Scale 4. Is hard to approach (-)


It is named + ordered + proportionate interval between 5. Good teaching skills (+)
variables + can accommodate absolute ‘zero’.
It is again quantitative. It combines the properties of 6. Liking/disliking (+/-)
nominal, ordinal, and interval scales. Ratio scale consists
of equidistant points. It allows researchers to compare the
differences or intervals. It possesses the feature of absolute Some statements may be positive, some negative,
zero. As it has absolute zero feature, it does not have nega- and some neutral. Then, the scores may be assigned
tive numbers. It has its own property of fixed starting point. to different responses and the score of each respon-
It offers wide options in statistical analysis. The vari- dent is calculated. Some respondents may have more
ables can be orderly added, subtracted, multiplied, positive attitude than others. There can be a numeri-
divided. Mean, median, and mode can be calculated using cal scale as well. Instead of SA, A, U, D, and SD, it
the ratio scale. will have values 5, 4, 3, 2, and 1.

M02_MADAN 04_65901_C02.indd 20 23/12/22 7:14 PM


Research Aptitude 2.21

2. Thurstone Scale: It may assign weightages to 3. It makes the study more objective.
the different statements, such as willingness to 4. It facilitates the formulation of a theory.
solve student problem may have a weight of 1, the
Hypothesis can be of the following types:
statement about subject knowledge has weight of
1.5, and statement in context of teaching skill can 1. Descriptive Hypothesis: It is formulated to describe
have weight of 2, and the scores are calculated the characteristics. For example, the present rate of
accordingly. unemployment in urban areas of India is 10 per cent.
3. Guttman Scale: It is mainly based on ratio scale. It is 2. Relational Hypothesis: It indicates the relationship
quite difficult to develop. between two variables. For example, parents residing
in urban areas spend more money on the education of
Factor Analysis their children.
Factor analysis allows researchers to describe many vari- 3. Explanatory Hypothesis: It guides the cause-
ables using few factors, thus reducing the number of vari- and-effect relationship between two variables. For
ables to a manageable level in terms of factors, which can example, when salaries increase, the spending on
be analyzed further. food items also increase. However, the reverse may
not be true.
Formulating Hypothesis In statistical hypothesis testing, you have a null hypoth-
(Plural – Hypotheses) esis against which you are testing an alternative. The
hypothesis concerns one or more characteristics of the
Researchers may have one or more hypotheses. These are distribution.
the questions that they want to address, which include
predictions about possible relationships between the
things they want to investigate (variables). Concepts of Null and Alternative Hypotheses
As a researcher, we do not know the exact truth, but Professor R. A. Fisher was the first to use such an
have a hunch about the outcome, and thus, we make ­experiment for testing the hypothesis by scientific investi-
some prediction about the outcome. This hunch or pre- gation. He talked about the principle of replication, rand-
diction about the outcome is called a hypothesis. It can omization, and local control in research.
also be termed as an educated guess or assumption about Hypothesis testing or significance testing is a method
some phenomenon. This assumption is tested by collect- for testing a claim or hypothesis about a parameter in a
ing information that will enable us to conclude if our population, using data measured in a sample. We know
hunch was right. Thus, defining hypothesis has the fol- that data about population are called as population
lowing features: parameter and about sample are called as sample statistic.
1. It is a tentative proposition. We always assume sample mean as an unbiased estimator
2. The validity of a hypothesis is unknown. of the population mean.
3. In most cases, formulating a hypothesis specifies the
logical relationship between two variables. Null Hypothesis
4. It must be generalizable. The Null Hypothesis (Ho) is a statement about a popu-
5. It should be kept simple. lation parameter, such as the population mean, that is
assumed to be true. The null hypothesis is a starting
Six Thinking Hats point. We will test whether the value stated in the null
This concept explores the cognitive aspect, how do we hypothesis is likely to be true. Null means no difference
think in different directions. This approach was created between variables, such as between population mean
by Edward de Bono in 1985 under the book of the same and sample mean or between two sample means and so
name. He wanted it to be a practical tool for everyday on. Let us look at some simple examples of setting null
problem solving. He was also the inventor of “lateral hypothesis.
thinking,” a method of solving problems indirectly, often
in creative and surprising ways.
Chance Accept the
Main Functions of a Hypothesis Outcomes Null Hypothesis
Middle 95%
Formulating a hypothesis, though important, is not abso-
lutely necessary for a research. A perfectly valid research
can be conducted even without formulating any hypoth-
esis. In general, formulation performs the following Critical Region
Critical Region
functions: Reject the Null
Non-Chance
Outcomes Two-Tailed Test
1. It brings focus, clarity, and specificity to the research (Non-Directional)
study.
Outer 2.5% Outer 2.5%
2. It helps in making the sample design.

M02_MADAN 04_65901_C02.indd 21 23/12/22 7:15 PM


2.22 Chapter 2

Research design achieves the following purposes:


Question Null Hypothesis
1. It makes research efficient
Are teens better at math Age has no effect on 2. Optimum utilization of resources—maximum
than adults? mathematical ability. information with minimal expenditure, time, and
Does taking aspirin every Taking aspirin daily does money
day reduces the chance of not affect heart attack 3. Flexibility
having a heart attack? risk. 4. Minimization of bias
5. Reliability and objectivity
Do teens use cell phones Age has no effect no cell
to access the internet phones being used for
more than adults? internet access. Study Design on the Basis of
Number of Contacts
We use the word ‘no’ or ‘no difference’ with every null On the basis of contacts, research can be cross-sec-
hypothesis so as develop a platform to be tested through tional or longitudinal.
‘sample’.
1. Cross-sectional studies: In cross-sectional studies,
Alternative Hypothesis (HA): If ‘null hypothesis’ is data are collected only once during the research pro-
rejected as sample mean falls in the ‘rejection region’ cess. The data are not necessarily gathered simulta-
on left or right, then alternative hypothesis is accepted. neously and data collection may spread over a period
It is actually contradiction of null hypothesis. It means of time, such as one week, one month or so. It may
that actual value of a population parameter is less than, also take a longer period. For example, data may be
greater than, or not equal to the value stated in null collected about holiday preference of software profes-
hypothesis. sionals in India.
After setting hypothesis, we collect data with the help 2. Longitudinal Studies: In longitudinal studies, data
of samples. We apply different statistical tests on the sam- would be collected at several points of time. For exam-
ple data to arrive at some findings. Then those findings ple, when a drug has been administered to a group
are interpreted. In the mean while we will discuss other of patients to check the efficacy of the drug, the data
aspects in the research process. Once familiar with these may have to be collected many times to check their
aspects, we will try to discuss the things in the form of health conditions. The HR department of an organi-
‘testing of hypothesis’ on page 2.29. zation may collect data about employee satisfaction
level before and after a raise in pay or promotion. The
marketer may be interested in getting information
Step 2: Preparing Research Design about the impact of advertisement on sales.
and Study Design The relation between research design and problem
structure has been shown in the Figure 2.6.
Now, when the setting up of hypothesis is done, the next
step is research design. It is a roadmap to carry out the Research design Problem structure
research. It is a step-by-step approach addressing basic
questions like what is the scope of research study or
what type of data are to be collected or something like
Exploratory Unstructured
what methods should be used to collect the data and to
analyze them. Justification is required at every step as Descriptive Structured
the resources are at stake. In fact, many of the research
Causal Structured
methods are basically research designs or closely linked
with them. Figure 2.6 Research Design
Research design is also defined as a plan, structure,
and strategy of investigation to get answers to research Study Design on the Basis of
questions. Reference Period
Therefore, research design involves the following
consideration: 1. Retrospective Study Design: It is meant for a phe-
nomenon or a situation that has occurred in the
1. Objectives of research study past.
2. Selection of method of data collection 2. Prospective Study Design: It pertains to likely preva-
3. Source of information—sample design lence of a phenomenon in the future.
4. Tools for data collection Study design based on reference period can be a
5. Data analysis—qualitative and quantitative ­combination of both retrospective and prospective studies.

M02_MADAN 04_65901_C02.indd 22 23/12/22 7:15 PM


Research Aptitude 2.23

Study Design on the Basis of reality, he is being given only sugar pills. It can play
an important role in his recovery. There can be two
N ature of Investigation groups, where one receiving the actual treatment and
For example, a pharmaceutical company wants to test the second receiving placebo treatment. The control
the impact of a drug in treating people. There is a cause group can also be used in this design. The first group
and effect relationship between the two variables. The may receive the actual treatment, the second group
research can be broadly classified into two for the pur- receives placebo treatment, and the third group (con-
pose of study design, namely experimental study and trol group) receives nothing.
non-experimental study.
Experimental Study Step 3: Developing Data Collection
If a study is done in a manner that we start from cause to
establish the effects, the independent variable can be manip- Instrument (Research Instrument)
ulated by the researcher to see the effect of change in inde- Data are defined as the information recorded to represent
pendent variable (cause) on dependent variable (effect). facts. Some important points about data are worth men-
The treatment groups (not in terms of medical ­science) tioning here.
are of two types, experimental group and control group.
1. Data represent facts about hypothesized variables.
1. Experimental Group: The group receiving the treat- 2. Data are analyzed to determine consistency with
ment is the experimental group. prediction. Prediction is in the form of setting up of
  Example: Students of a class being taught with hypothesis.
a new teaching method, a group of patients being 3. If data and prediction are consistent, then null hypoth-
administered a new medicine, etc. esis is supported.
2. Control group: The group not receiving the treat- 4. If data and prediction are inconsistent, hypothesis is
ment is the control group. not supported and is rejected.
  Example: Students being taught with same con-
ventional method, a group of patients with no medi- The quality and validity of the output are solely dependent
cine or no new medicine, etc. on the tools used for data collection. The data can be clas-
  Control in an experiment means that the researcher sified as primary and secondary, objective and subjective,
wants to investigate the effect of various factors one and qualitative and quantitative.
at a time in that ­experiment. Primary Data and Secondary Data
Randomization Primary data are collected for the purpose of current
It covers the following aspects: research project, whereas secondary data are collected
for some other research purpose. They are collected fresh
1. The selection of a group as experimental or control by the researcher and are based on surveys, observations,
group is random. and experimentation. They are expensive and difficult
2. All participants have equal chance of being chosen for to acquire. They are reliable as they have been obtained
experimental group or control group. directly with a specific problem in view. Figure 2.7 shows
3. The larger the number of participants, the greater the various ways in which primary data can be collected.
chance that groups will represent the population.
Secondary data are collected from external sources, such
Non-experimental Study as TV, radio, the Internet, magazines, books, and newspa-
It is a retrospective study. Thus, we start from the effects pers. These data might have been collected for different
to trace the cause. Here, the assumed cause has already purposes. They are an inexpensive and quick method to
occurred. The variables are still referred to as IV and DV. obtain information. Sometimes, it is the only way when
Example: Comparing opinions from natural groups. the original source is inaccessible. It should be ascertained
There can be other types of study designs as well. A few (i) whether the data are relevant to your study and
important study designs are as follows. (ii) if they are credible.

1. Action research: Action research has been discussed


earlier. It is a research initiated to solve an immediate Primary Data
problem. It was coined by Kurt Lewin.
2. Cohort Studies: It is based on the existence of some
common characteristics, such as year of birth, gradu-
Observations Interviews
ation, or marriage. For example, the researcher wants
to study the pattern of employment of MBAs who grad-
uated in 1991, when India was facing economic crisis,
or a study of people born between 1995 and 2000. Mail Phone Personal
3. Placebo Design: A patient, for example, may have
an impression that he is undergoing treatment, but in Figure 2.7 Sources for Collection of Primary Data

M02_MADAN 04_65901_C02.indd 23 23/12/22 7:15 PM


2.24 Chapter 2

O bjective Data and Subjective Data Reactivity refers to when a measure changes participants’
Objective data are independent of any single person’s behaviour.
opinion, whereas subjective data can be an individual’s Vignettes: They comprise stimuli that portray elements
opinion or dependent upon the researcher. of reality to which research participants readily respond.
They reflect perceptions, attitudes, and behaviours in
Q ualitative Data and Quantitative Data qualitative research.
Qualitative data are the description of things made with-
Limitations
out assigning numeric values. For example, facts gener-
ated from unstructured interview. It needs the research- Initially, there are many behavioural aspects that may
er’s interpretation. not be observable directly. For example, marketer as a
Quantitative data entail measurements in which the researcher cannot measure the feelings, beliefs, and atti-
numbers are used directly to represent properties of tudes that motivate buying behaviour, and infrequent
things. It is ready for statistical analysis. A larger sample behaviour cannot be observed.
is required in quantitative data; and with proper sampling Secondly, the observation method is quite expensive.
design, the ability to generalize is also high.
Survey Method
O bservation Method Sometimes, the observation method is supplemented
This is used in behavioural sciences. It is about collecting with survey method. This approach is most suited for
primary data by investigator’s own direct observation of gathering descriptive information, and this research may
relevant people, actions, and situations without asking be direct or indirect. It is of two types and these are struc-
the respondent. tured and unstructured surveys.
1. Structured surveys: They use formal lists of questions
Types of Observations to be asked from all respondents in the same manner.
1. Structured—for descriptive research 2. Unstructured surveys: They give the interviewer the
2. Unstructured—for exploratory research flexibility to probe respondents and direct the inter-
3. Participant observation view according to their answers.
4. Non-participant observation Advantages
5. Disguised observation
1. Quick and lower cost in comparison to observation
The most important approach for data collection in method
observational research is participant observation. In 2. Survey method can be administered to collect many
participant observation, researchers become active different types of information.
participants in the group or situation they are studying.
Participant observation is very similar to naturalis- Limitations
tic observation in that it involves observing people’s 1. Privacy issues
behaviour in the environment in which it typically 2. Reluctance on the part of respondents
occurs. 3. Biases
One other observational method is called as structured
observation. Here, the investigator makes careful obser- Contact Methods
vations of one or more specific behaviours in a particular Information may be collected by mail, telephone, ­personal
setting that is more structured than the settings used in interview, etc.
naturalistic and participant observation.
In social sciences or qualitative studies, we may use Mail Questionnaires
‘naturalistic observation’ to collect data from samples. It
is an observational method that involves observing peo- Advantages
ple’s behaviour in the environment in which it typically 1. It includes collecting large amounts of information at
occurs. Thus, naturalistic observation is a type of field a low cost per respondent.
research (as opposed to a type of laboratory research). 2. Respondents may give more honest answers to per-
Jane Goodall’s famous research on chimpanzees is a clas- sonal questions to a mail questionnaire.
sic example of naturalistic observation. 3. It is unbiased as no interviewer is involved.
Sometimes, researchers usually make their observa- 4. Convenient for respondents who can answer when
tions as unobtrusively as possible so that participants are they have time.
not aware that they are being studied. Such an approach 5. Good way to reach people who travel.
is called disguised naturalistic observation.
In undisguised naturalistic observation the partici- Limitations
pants are made aware of the researcher presence and 1. It is not flexible.
thus, monitoring of their behaviour. The reactivity may 2. It takes a longer time to complete than telephonic or
happen as a result of undisguised natural observation. personal interviews.

M02_MADAN 04_65901_C02.indd 24 23/12/22 7:15 PM


Research Aptitude 2.25

3. It has a low response rate. Focus Group Interviewing


4. Little control of researcher over the process. Advantages
Telephonic Interviewing 1. It is usually conducted by inviting 6–10 people to
gather for a few hours with a trained moderator to
Advantages talk about a product, service, or organization. The
1. It is a quick method. meeting is held in a pleasant place and refreshments
2. It gives greater flexibility to interviewer as he can are served to create a relaxed environment.
explain questions not understood properly by the 2. The moderator needs objectivity, knowledge of the
respondent. subject and the industry, and some understanding of
3. It has greater sample control. the group and individual behaviour.
4. It has a higher response rate. 3. The moderator starts with a broad question before mov-
ing to more specific issues, encouraging open and easy
Limitations discussion to bring out the true feelings and thoughts.
1. High cost per respondent as interviewer should be At the same time, the interviewer focuses on the dis-
more skilled. cussion, hence the term focus group interviewing.
2. Privacy issues 4. It is often held to help determine the subject areas
3. Complete standardization is not possible. on which questions should be asked in a later, large-
4. Wrong entry is possible due to the lack of time. scale, structured, direct interview.
5. Comments are recorded through note taking or
Personal Interviewing ­videotaped and studied later (content analysis).
It is very flexible and can be used to collect large amounts Limitations
of information. Trained interviewers can hold the 1. It is more expensive than a telephonic survey.
respondent’s attention and are available to clarify difficult 2. Group interview studies keep the sample size small
questions. enough to keep the time and cost down. Therefore, it
They can guide interviews, explore issues, and probe may be difficult to generalize from the results.
as the situation requires. Personal interviews can be 3. There is some possibility of interviewer bias.
used in any type of questionnaire and can be conducted
fairly quickly. Interviewers can also show actual prod- Construction of a Research T ool
ucts, advertisements, or packages, and observe and
record their reactions and behaviour. This takes two (Q uestionnaire)
forms, namely intercept interviewing (for individuals) The questionnaire depends upon research ­objectives.
and focus group interviewing (for groups). For each objective or research questions, all the
­associated questions that a researcher wants to answer
Intercept Interviewing through the study are listed. Then, the informa-
Advantages tion required to answer them is listed and finally the
questions are listed.
1. It is widely used in marketing research such as tourism.
2. It allows the researcher to reach known people in a
short period of time. Questionnaire-Concept and types
3. It is the only method of reaching people whose names A questionnaire consists of a set of questions presented to
and addresses are unknown. a respondent for answers. The questionnaire is used dur-
4. It involves talking to people at homes, offices, on the ing structured surveys or interviews. The respondents
street, or in shopping malls. read the questions, interpret what is expected, and then
5. The interviewer must gain the interviewee’s cooperation. write down the answers themselves. It is also called an
6. It is time-consuming and may range from a few min- interview schedule when the researcher asks the ques-
utes to several hours (for longer surveys, compensa- tions and records the respondent’s reply on the interview
tion may be offered). schedule. Here, the researcher may have to explain ques-
7. It involves the use of judgemental sampling, i.e., the tions to the respondents.
interviewer has guidelines as to whom to ­intercept, There are many options before the researchers to
such as 25 per cent under twenty years of age and adopt this method, but questionnaires should be devel-
75 per cent over sixty years of age. oped and tested carefully before being administered on
a large scale. There are three basic types of question-
Limitations naires, closed-ended, open-ended, and a combination
1. There is a greater room for error and bias on the part of both.
of the interviewer who may not be able to correctly 1. Closed-ended Questionnaire: Closed-ended question-
judge age, race, etc. naires generally include multiple choice questions or
2. Interviewer may not be comfortable talking to c­ ertain scale questions. This type of questionnaire can be admin-
ethnic or age groups. istered to a large number of respondents or sample size.

M02_MADAN 04_65901_C02.indd 25 23/12/22 7:15 PM


2.26 Chapter 2

As there is a set format, the data generated from ques- 2. Variation in Population: The greater the variation in
tionnaire can be easily fed into a computer system for the population, the greater will be the uncertainty of out-
purpose of analysis. come. The higher the consistency in population, the
2. Open-ended Questionnaire: Open-ended question- more confident we are about the quality of outcome.
naires offer the flexibility to respondents to answer in The higher the variations in the population, the larger
their own words. It may leave a blank section to write should be the sample size.
an answer.
  Closed-ended questionnaires might be used to
find out how many people use metro rail service in Types of Sampling T echniques
New Delhi, but open-ended questionnaires might be 1. Probability or Random Samples: Each person in
used to find out what people think about the quality the population has equal, independent, and known
of service. chances of being selected. In case, there are 100
3. Combined Questionnaire: In this method, it is possi- elements in a population, every element has 1/100
ble to find out how many people use a service and what chance of being selected in a sampling exercise. Here,
they think of the service in the same form. The com- independence means that selection of one element is
bined questionnaire may begin with a series of closed- neither affected by the selection of other elements nor
ended questions, with boxes to tick or scales to rank will it affect the other elements.
and then finish with a section of open-ended questions   Though probability or random sampling is mostly
or a more detailed response. correct, still some error, technically known as ‘Margin
of Error’ cannot be avoided. It can be calculated statis-
tically and accounted for in the results. NET aspirants
Step 4: Selecting Samples can refer to terms such as ‘level of significance’ for a
better idea.
If the population under study is small or manageable, then   Now we can discuss the important types of
the data should be collected on each item or entity under ­probability sampling.
study. But this is rarely the situation in a survey research.
(a) Simple Random Sample: Every element or
Sampling is required if the universe of population under
member of the population has a known and equal
study is too large.
chance of being selected.
A sample may be defined as a representative subset
(b)  Stratified Random Sample: In case the popula-
or cross-section of the population in miniature. It should
tion is heterogeneous, it can be divided into dif-
homogeneously represent the entire field.
ferent strata. The population within a stratum is
The validity of research results depends upon the qual-
homogeneous with respect to the characteristics
ity of the sample drawn. There are scientific, logical, or
under study. Population is divided into mutually
statistical techniques for formulating a sample.
exclusive groups, such as age groups, and random
If the sample is biased or lopsided, then the results can-
samples are drawn from each group. The popula-
not be trusted or generalized.
tion in a particular stratum may be proportionate
The main benefits of sampling are as follows:
to its population. Suppose there are 1000 students
1. Reduction in overall cost of research. in a college, 600 of them study humanities and 400
2. Less time-consuming and, in certain cases, it is desir- pupils study commerce. In a sample of 100, sixty
able as well. students will be from humanities and forty from
3. In case the population is consistent, this becomes commerce, i.e., in the same ratio as in the overall
even more desirable. population.
(c) Cluster Sample: A simple and stratified sam-
Ideally, a representative sample should be an unbiased pling is adopted in situations where the popu-
indication of what the population is like. Some of the fac- lation size is small and units are identifiable.
tors that researchers consider when selecting a represent- But if the population is larger, the researcher
ative sample include sex, age, educational level, socio- can go for cluster sampling. The population
economic status, and marital status. is divided into mutually exclusive groups and
For example, if roughly half of the total population the researcher draws a sample of the group to
of interest is female, then a sample should be made interview. For example, in a national level sur-
up of approximately 50 per cent women in order to be vey, at the first few levels, a few states may be
representative. selected. Within the states, a few districts may
In research, the population does not mean only human be selected, then within each district, some
population all the times, and it can be factories, schools, etc. blocks may be selected and then villages and
Population is denoted by N and the sample as n. so on. This is termed as ‘multistage cluster
The factors affecting inferences drawn from a ­sample sampling’.
are dependent on the following.
2. Non-probability or Non-random Sampling: It is a
1. Sample Size: The larger the sample the more the ­non-structured sample and items are included in the
accuracy. study due to some convenience of the researcher,

M02_MADAN 04_65901_C02.indd 26 23/12/22 7:15 PM


Research Aptitude 2.27

etc. This sampling takes less time and is handy. As


all members of population do not get equal chance Step 5: Writing A Research Proposal
of being selected, non-probability sampling may be A research proposal is a document of usually three to
­lopsided, loaded with biases and have higher ­margin seven pages that informs others of a ­proposed piece of
of error. research. This proposed research is usually a master’s or
  The types of non-probably sampling have been dis- doctorate thesis, but it can also be a research for a corpo-
cussed as follows: rate purpose.
(a) Convenience Sampling: The researcher selects A research proposal is a document written by a
the easiest population members from which to researcher that provides a detailed description of the
obtain information. proposed program. It is like an outline of the entire
(b) Judgement or Purposive Sampling: The research process that gives the reader a summary of
researcher uses his or her judgement to select information discussed in the ­project. Research propos-
population members who are likely to provide als are written for v­ arious reasons, such as a budget
accurate information. This can be used for his- request for the research, certification requirements for
torical research or descriptive research. the research, etc.
(c) Quota Sampling: The researcher finds and
interviews a prescribed number of people in Main Elements of Research Proposal
each of several categories. Here, the main crite- 1. Need of a specific research project.
rion used by the researcher is the ease to access 2. Benefits and beneficiaries.
sample population. The sample is selected 3. The type of data to be collected and the means to col-
from a location convenient to him or her. Here, lect the data.
there are some possibilities to include people 4. Types of analysis.
with some visible characteristics. However, 5. Help required from other organization, if any.
the results may not be generalized to larger 6. Duration, facilities, and requirement of funds.
populations. 7. Profile and credentials of the proposers.
(d) Accidental Sampling: It is akin to quota ­sampling,
but it is used in market research (in market places)
where a researcher can come across any person
who may not have any information. Step 6: Collecting Data
Snowball Sampling: In this kind of sampling,
(e)  There are many alternatives available to collect relevant
the information may be selected from few indi- data. The researcher should select one of these methods
viduals and they may identify other people for of collecting data taking into account the nature of the
the purpose of gathering information. They may investigation, scope and objective of inquiry, financial
also become a part of the sample. This creates a cost, availability of time, and the desired accuracy.
network of sample elements. Primary data can be collected either through experi-
ment or through survey. If the researcher conducts an
Determining Sample Design experiment, he will take some quantitative measure-
Designing the sample calls for three decisions: ments (data). The data are analyzed further to test the
hypothesis.
1. The sample itself: Who will be surveyed? It further In case of a survey, data can be collected by any one or
depends upon what type of information is required. more of the following ways.
2. Sample size: How many people should ideally be sur-
veyed? Large samples are likely to give more reliable 1. By Observation: This entails the collection of infor-
results than small samples. However, it has to be a mation by way of the investigator’s own obser-
trade-off between cost and accuracy. vation without interviewing the respondents.
3. Sampling: How should the sample be chosen? Sam- The information obtained relates to the current
pling can be done by adopting either probability or ­happenings. This method is very expensive, and
non-probability method. the information gathered in such a manner is
limited. This method is not suitable for research
The sample size for any research study depends upon where large samples are required, as is the case
four Ps: with quantitative research.
1. Purpose: The required precision of study. 2. Personal Interviews: The investigator follows struc-
2. Population: The size and nature of the population tured approach, and the questions are pre-conceived.
under study. Here, the output also depends upon the ability of the
3. Procedure: The time, budget, and resources available. interviewer to a large extent.
4. Publishing: The importance of the study. 3. Telephonic Interviews: This method of collecting
information involves contacting the respondents over
Note: The more heterogeneous or diverse the population, telephone itself. It is used when the survey has to be
the bigger should be the sample size. accomplished in a very limited time.

M02_MADAN 04_65901_C02.indd 27 23/12/22 7:15 PM


2.28 Chapter 2

4. Mailed Questionnaires: Questionnaires are mailed to Research Biases


the respondents with a request to return after filling in The most formidable challenge in research is to remain
the same. It is the most commonly used method in eco- objective and free from biases. There can be a variety of
nomic and business surveys. A pilot study may be con- biases to distort people’s impressions of collected data.
ducted for testing the questionnaire to check its appro-
priateness for the purpose. 1. External Influences: One’s culture or opinion cre-
5. Schedules: In this method, the enumerators are spe- ated by media (say social media) can influence people
cially appointed and trained for gathering informa- to accept a particular world view.
tion. Schedules consist of relevant questions. The enu- 2. Personal Bias: This may happen due to personal
merators visit respondents with these schedules, and beliefs, attributes, or past experiences.
are filled up by the enumerators on the basis of replies 3. Observer Bias: Some events are taken as meaning-
given by the respondents. ful by some and not taken meaningful by others.
Researchers themselves were raised in certain cul-
Some random checking by the supervisors may be tures and societies. They also have role expectations.
required to ensure the validity of the research process. These background factors can affect all the way that
Although attention should be given to all these factors, researchers observe and interpret events in their lives.
much depends upon the ability and experience of the 4. Expectancy Bias: Researchers sometimes expect to
researcher. find specific outcomes, they may see (or note) what
they expect to see rather than remain objective.
5. Placebo Bias: It operates when people strongly want
Step 7: Processing and Analyzing Data to believe a treatment is successful. For example,
many people may claim to feel better after taking a
After collecting the data, the next step is to analyze it. placebo such as a sugar pill.
It requires a number of closely related operations, such as
establishment of categories and the application of these Classification of Data
categories to raw data through coding, tabulation, and Classification of data is a process of arranging data in
then drawing statistical inferences. groups or classes on the basis of common characteristics.
The large data should be condensed into a few manage- It can be done in the following ways.
able groups and tables for further a ­ nalysis. This is done
with the help of classification of data into more relevant, Classification According to Attributes
purposeful, and usable ­categories.
The data can be descriptive (Example: Literacy, sex,
1. Editing: The process of cleaning data is called religion, etc.) or numerical (Example: Weight, height,
editing. The purpose of editing is to identify and income, etc.). Further classification can be either simple
minimize errors, miscalculations, misclassifica- classification or manifold classification.
tion, or any gap in information provided by the
respondent. Editing improves the quality of the 1. Simple Classification: In this classification, we con-
data for coding. sider only one attribute and divide the universe into
2. Coding: It depends upon how a variable has been two classes, where one class consists of items possess-
measured in your research instrument. For coding, the ing the given attribute and the other class consists of
first step is to ensure the nature of data, i.e., whether items that do not possess the given ­attribute.
it is quantitative or qualitative. The qualitative data   Example: The number of candidates with MBA
may be descriptive about the f­ ollowing details or case degree is as follows.
history. For example, categorical or discrete, gender
(male or female), income (below average or above Yes No Total
average), and attitude (strongly favourable, favour-
MBA degree 21 09 30
able, or unfavourable).
Quantitative and categorical information is processed 2. Manifold Classification: In this classification, we
to be converted into numerical values called codes. It is consider two or more attributes simultaneously and
usually done at this stage through which the categories divide the data into a number of classes.
of data are transformed into symbols that may be tabu-   Example: The educational qualification of ­faculty
lated and counted. After coding is completed, the data members is given as follows:
are tabulated.
The descriptive information goes through a process Yes No Total
called content analysis with the motive of getting an idea
about the ‘themes’ of the descriptive information, such Gender M F M F M F
as an interview. In descriptive or qualitative data, the
researcher may go through the transcription of all inter- PG degree
views in which people may use different words to express
Ph.D.
the same phenomenon.

M02_MADAN 04_65901_C02.indd 28 23/12/22 7:15 PM


Research Aptitude 2.29

Classification According to Class Intervals 2. Assign Codes to the Main Themes: Assigning codes
Classification is done with data relating to income, age, is required when the researcher wants to count the
weight, tariff, production, occupancy, etc. Such quanti- number of times a theme has emerged in an interview.
tative data are known as the statistics of variables and 3. Classify Responses Under the Main Themes:
are classified on the basis of class intervals. For example, Having identified the themes, the next step is to go
people whose income is within `2001 and `4000 can through the transcripts of all the interviews and clas-
form one group or class, and those with income within sify the responses under different themes.
`4001 and `6000 can form another group or class and so 4. Integrate Themes and Responses into the Text of
on. The number of items that fall in a given class is known Your Report: Having identified the responses that fall
as the frequency of the given class. under different themes, the next step is to integrate
the themes and responses into the text of your report.
Tabulation It entirely depends upon the way the researcher wants
to communicate the findings to the readers.
Tabulation is a part of the technical procedure wherein
the classified data are put in the form of tables. It is the Quantitative Data Analysis
process of summarizing raw data and displaying the This method is most suitable for large, well-designed and
same in a compact form for further analysis. Mechanical well-administered surveys using properly constructed
devices can also be used for this purpose. When data and worded questionnaire. Data can be analyzed either
are really large, computers can be used for tabulation. It manually or with the help of a computer.
also makes it possible to study large number of ­variables
affecting a problem simultaneously. It is an orderly 1. Manual data analysis: This can be done if the num-
arrangement of data in columns and rows. It is essential ber of respondents is reasonably small and if there
because of the following reasons: are not many variables to analyze. However, this is
useful only for calculating frequencies and for sim-
1. It conserves space and reduces explanatory and
ple cross-tabulations. Manual data analysis is time
descriptive statement to a minimum.
consuming. The easiest way to do this is to code it
2. It facilitates the process of comparison.
directly onto a large graph paper in columns.
3. It facilitates the summation of items and the detection
2. Data analysis using a computer: If you want to ana-
of errors and omissions.
lyze the data using computer, you should be familiar
4. It provides the basis for various statistical
with the appropriate program. In this area, knowledge
­computations.
of computer and statistics plays an important role.
Tabulation may also be classified as simple and complex The most common software is SPSS for w ­ indows.
tabulations. Simple tabulation generally results in a one- However, data input can be a long and laborious
way table that supplies answers to questions about one process, and if data are entered incorrectly, it will
characteristic of data only. Complex tabulation usually influence the final results.
results in two-way tables, which give information about
two interrelated characteristics of data. Three-way tables In the process of analysis, relationships or differences,
or still higher order tables are also known as manifold supporting or conflicting with original or new hypothesis,
tables. should be subjected to tests of significance to determine
with what validity the data can be said to indicate any
Analysis of Data conclusions.
After tabulation, analysis is done with the help of differ-
ent mathematical and statistical techniques, such as per- Testing of Hypothesis
centages, averages, coefficients of correlation, regression We have discussed all major aspects of research. In step
and so on. It largely depends upon whether the data are 1, the focus was on basics. Now, being familiar with all
qualitative or quantitative. concepts in the process, in step 7, we have tried to explain
Qualitative Data Analysis the concept with an example. The questions in NTA NET
Exam have been asked on hypothesis, errors, skewness,
The analytical approach may be personalized, and there normalisation etc. So, we need to be more comprehen-
may be few rigid rules and procedures. ­Generally, the sive, but without being too much technical as that kind of
researcher needs to go through a process called content discussion is beyond the scope of syllabus.
analysis. Content ­analysis means analyzing of the con-
tents of an interview in order to identify the main themes Population Parameter and Sample Statistic: The value
that emerge from responses given by the respondents. assumed about population is called as ‘population param-
This process involves the following steps: eter’. The value of a statistic obtain from the sample can
be taken as an estimate of the population parameter. The
1. Identify the Main Themes: People use different
value obtained about the sample can be termed as ‘sample
words and languages to express themselves, so we
statistic’. The population standard deviation is used if it is
need to understand the main theme. The researcher
known, otherwise the sample standard deviation is used
needs to understand the descriptive responses given
for calculation purposes. We not putting formulas as they
by respondents to each question.
are not part of syllabus.

M02_MADAN 04_65901_C02.indd 29 23/12/22 7:15 PM


2.30 Chapter 2

Example: We are comparing past performance of stu- 2. Directional hypothesis measures the direction of vari-
dents who have taken coaching with the performance of ation of two variables. This effect of one variable on
present students who have completed it now. The ‘sam- the other variable can be in positive direction or in
ple’ of present students will be collected. The past marks negative direction. Non-directional hypothesis does
of students were actually 80 (or they might have been not indicate the kind of effects but only shows the
assumed to be 80). In general, we know that coaching has relation between two variables.
positive impact on performance of students. Null hypoth-
Sampling Errors: When we talk about samples, errors
esis assumes that there is ‘NO’ such benefit (impact) of
some may erupt into testing of hypothesis. There are two
coaching as the word ‘null’ itself indicates this as per the-
types of errors.
ory discussion earlier.
Null hypothesis (Ho) = 80 marks 1. a(Alpha) is the probability of Type I error in any test
The data of 80 marks may be available actually or may be of hypothesis. It incorrectly rejects the null hypoth-
assumed (set hypothetically) as per situation. esis. Alpha is also called as ‘Level of Significance’. It
Alternative hypothesis (Ha): It states that marks of may be set as 1%, 5% or 10%. For example, a signifi-
present students are not equal to 80 marks. cance level of 0.05 indicates 5% risk (probability) of
It means that coaching has an impact on the performance concluding that a difference exists when there is no
of students, in positive or negative direction. actual difference.
We collect data from a sample of 40 students in the 2. b(Beta) is the probability of Type II error in any test-
school. As per ‘standard deviation’ and ‘level of significance’, ing of hypothesis. It incorrectly fails to reject the null
we keep a range of +/– 3 from 80 marks. It means 77 to 83 hypothesis.
is the ‘flexible’ range set. If students get between 77 to 83
marks, then coaching has not benefited them. They actually The errors can be of categorised into following types
fall within ‘acceptance region’, and thus null hypothesis is also.
accepted. It means null hypothesis is accepted, and marks 1. Sampling Errors: Errors caused by the act of taking a
of the students tally with the past or assumed performance. sample. It makes samples inaccurate.
If marks are ‘less than 77’OR‘more than 83’, they will 2. Random Sampling Errors: These errors are caused
fall in the ‘rejection region’. It means null hypothesis by a ‘chance factor’ in selecting a ‘random sample’.
is rejected (or ‘alternative hypothesis’ is accepted) and The random sampling is basically done to ensure
coaching has an impact on the performance of students. objectivity and unbiasedness in samples.
The impact may be positive (if marks > 83) or negative 3. Non-sampling Errors: These errors are not related
(if marks < 77). We make use of some statistical tests to to the act of selecting a sample from the population.
make such calculations. They can be present in a census.
conclusion from Sample Size:
statistical analysis
Small Sample Test: If sample size is less than 30, it is
Accept the Null Reject the Null called as small sample test. The best example is ‘student’s
Null hypothesis t-test’.
is true Type I Error
Large Sample Test: If sample size is more than 30, it is
Correct called as large sample test. The best example is ‘z-test’.
reject a true null
Conclustion
hypothesis The basic difference between these two test is in
changes in formulas.
the true state
of nature Parametric and Non-parametric Tests:
Type II Error
1. Parametric Tests: A parametric statistical test is
Correct
accept a false null one that makes ‘assumptions’ about the population
Conclustion
Null hypotheis hypothesis parameters. For example, the value of arithmetic
is false mean was assumed to be 80 in the example. The value
of 80 about the population can be actual or assumed.
The sample data is collected on the same basis.
Directional and Non directional Tests: The above cal-   T-test, Z-test, ANOVA, and Correlation Coefficient
culation was done as per ‘two-tailed’ test. They reflect ‘=’ are the main examples of parametric tests.
sign. These can be stated as ‘non-directional’ tests.   The value assumed about population is called as
There can be one-tailed tests also to show the situation ‘population parameter’. The value of a statistic obtain
with < or > signs. These can be termed as ‘directional’ from the sample can be taken as an estimate of the
tests. population parameter. The value obtained about the
In a formal manner, we can say that sample can be termed as ‘sample statistic’.
2. Non-parametric Tests: A non-parametric test is one
1. A directional test makes assumption about one tailed that makes no such assumption about population
test, while a non-directional test is two-tailed test. parameter. Non-parametric tests are ‘distribution-free’

M02_MADAN 04_65901_C02.indd 30 23/12/22 7:15 PM


Research Aptitude 2.31

and, as such, can be used for non-normal variables. In


Mode
such cases, skewness and kurtosis may deviate a lot Mode
Median Median
from the normal distribution.
Mean Mean
 Chi-square test, Wilcoxon signed rank test,
Friedman test, Goodman Kruska’s Gamma, Mann–
Whitney test, and Spearman Rank Correlation are the
main examples of non-parametric tests.
  Non-parametric tests are more suitable for qualita- Left-Skewed (Negative Skewness) Right-Skewed (Positive Skewness)
tive research studies.
2. Kurtosis: While skewness focuses on the overall shape,
Difference between Parametric and Non-parametric Kurtosis focuses on the tail shape. Kurtosis is a measure
Tests of how differently shaped are the tails of a distribution
as compared to the tails of the normal distribution.
Non-parametric Parametric Kurtosis is the fourth standardized central moment,
• C
 an be used on ordinal • U
 sed mainly on interval thus it is always positive. Kurtosis values can range from
and nominal scale and ratio scale data. 1 to +∞. The kurtosis of a normal distribution is 3.
data (although also on There are three situations in Kurtosis.
internal and ratio scale).
• C
 an be used on small • T
 end to need larger 1. Platykurtic: When K< 3, it is called as platykurtic
samples. samples. (less ‘peakedness’ and weaker ‘tails’)
2. Leptokurtic: When K > 3, it is leptokurtic (more peak-
• C
 an be used on data • D
 ata should fit a edness and heavy tails)
that are not normally particular distribution; 3. Mesokurtic: When k = 3, it is called as mesokurtic
distributed. the data can be (moderate in height and breadth)
transformed to that
distribution. Leptokurtic
Distribution
• C
 an be used where the • S
 amples should be
samples are not selected drawn randomly from Mesokurtic
Distribution
randomly. the population.
• H
 ave less power than the • More powerful than non- Platykurtic
Distribution
equivalent parametric test. parametric equivalent.
Source: Saga research methods

Requirement of Normal distribution:


If we look at the first figure, it has the ‘normally distri- Stopover
bution’, where mean, median and mode carry the same
A researcher administers an achievement test to assess

value. It is symmetric in nature. Somehow errors come
and indicate the possible effect of an independent
in due to different reasons. Statistical theory assume
variable in his/her study. The distribution of scores
that the underlying data is normally distributed. Thus,
on the tests is to be negatively skewed. On the basis of
researchers try to achieve normal distribution. It helps us
this, what can be stated with regard to difficulty level
in rectifying many errors.
of the test ? (June 2020).
Skewness and Kurtosis: The ‘Skewness’ and ‘Kurtosis’ show (a) The test level is very easy.
the different aspects of same distribution.The interested stu- (b) The test is very difficult.
dents may go through this piece of information, though their (c) The test is neither easy nor difficult.
detailed discussion is beyond the scope of syllabus. (d) The test is easy and needs normalisation.
1. Skewness: It helps us know by how much the ‘overall The correct answer is (d).
shape of a distribution’ deviates from the ‘normal dis-
tribution’, it is actually lack of symmetry from the nor- In this scenario, we need to ensure ‘normalisation’.It
mal distribution. Skewness isthe third standardized means that normal ‘symmetrical’distributionis required
central moment. Skewness of the normal distribution to get better results. As given in question statement, there
is zero. The term skewness means lack of symmetry are independent (anddependent) variables, it should be
from the normal distribution. The study of measure of more standardised to see the effect of one set of variables
skewness, thus becomes is important to guard against on other. Even if it is negatively skewed (or positively
any variation from normal distribution. If the value of skewed), some mathematical formula and techniques
mean is greater than that of mode, skewness is posi- should be adopted to remove skewness and bring in nor-
tive in nature. If the value of mean is less than that of malisation. We can’t say that negative skewness is worse
mode, it is negative in nature. than the positive skewness or vice versa in this case. It’s

M02_MADAN 04_65901_C02.indd 31 23/12/22 7:15 PM


2.32 Chapter 2

a question of direction only. The method for converting Chapter II Research Design
negative or positive skewed distribution into the ‘normal’ Chapter III Data Analysis and Interpretation
one may be different. Chapter IV Summary and Conclusion.
Suggestions for Further Research
G eneralizations and I nterpretation
References/Bibliography
If a hypothesis is tested and upheld several times, it may Appendices
be possible for the researcher to arrive at generalization, Appendix I Questionnaire for Employees
i.e., building a theory. As a matter of fact, the real value Appendix II Questionnaire for Managers
of research lies in its ability to arrive at certain generali-
zations. If the researcher had no hypothesis to start with, List of Tables
he might seek to explain his findings on the basis of some This section includes the title and page numbers of
theory. It is known as interpretation. The process of inter- all tables.
pretation may trigger new questions, which in turn may
lead to further researches. Example
Table No. Title Page No.
Step 8: Writing A Research Report 1. Income levels of ­respondents
Writing a report is the last and, for many, the most dif- 2. Age distribution of respondents
ficult step of the research process. The report informs
the world what you have done, what you have dis-
covered, and what conclusions you have drawn from List of Figures
your findings. The report should be written in an aca- This section contains the title and page numbers of all
demic style. The language should be formal and not graphs, pie charts, etc.
journalistic.
Example
Stopover Figure No. Title Page No.
The process not needed in experimental research is:

1. Pie chart showing the age
(a) Observation
distribution of respondents
(b) Manipulation and replication
(c) Controlling 2. Bar graph showing the
(d) Reference collection popularity of menu items
The correct option is (b).
Acknowledgement
In this section, the researcher may ­acknowledge the insti-
Research Report Format tute, principal, faculty guides (both research guide and
Traditional written reports tend to be produced in the fol- technical guide), research participants, friends, etc.
lowing format.
Introduction
Title Page This section introduces the research, setting out the
1. Title of the research project main aims and objectives. It is actually a rationale for the
2. Name of the researcher research.
3. Purpose of the research project
Theoretical Framework and Review of Literature
For example, ‘A research project submitted on p
­ artial
fulfilment of the requirements of XYZ University, This section includes all the background research informa-
New Delhi, for the degree of ____________’. tion that has been obtained from the literature review. You
4. Date of publication must indicate from where all the information was obtained.
Thus, it is mandatory to keep a complete record of every-
thing the researcher has read. Otherwise, there are chances
Table of Contents that the researcher could be accused of plagiarism, which is
In this section, the contents of the report, either in chap- akin to intellectual theft.
ters or in subheadings, are listed.
Research Design
Example This section includes all practical details followed for the
Contents Page No. research. After reading this, any interested party should
Introduction be able to replicate the research study. It includes the
Chapter I Theoretical Framework and Review of Related methods used for data collection, sampling procedures,
Literature tools used for data collection, and analysis of data.

M02_MADAN 04_65901_C02.indd 32 23/12/22 7:15 PM


Research Aptitude 2.33

Critical values for a


non-directional test
(two-tailed test with a = .05

Acceptance region

Rejection region Rejection region


a = .0250 a = .0250

-3 -2 -1 0 1 2 3
Arithmetic Mean
-1.96 = Median 1.96
= Mode

We expect the
sample mean to
HA: Students be equal to the HA: Students
scoring less population mean. scoring more
than 77 marks. than 83 marks.

m = 80 m = 80

m = 80
HA: Students
who do not
score 80
Normal Symmetric Distribution

Data Analysis and Interpretation Suggestions for Further Research


If you have conducted a large quantitative survey, this sec- Research is a continuous process. This section shows
tion may contain tables, graphs, pie charts, and associ- how research could be continued. This could hap-
ated statistics. If you have conducted a qualitative piece of pen as some results are inconclusive or the research
research, this section may be a descriptive prose. itself has thrown up many more research questions
that need to be addressed. It also shows the honesty
Summary and Conclusion and integrity of the researcher–that he has a wider
In this section, you sum up your findings and draw conclu- perspective and has actually not tried to cover up the
sions from them, perhaps in relation to other research or shortcomings.
literature.
List of References/Bibliography
Recommendations The list of references contains only the details of those
If the research has been conducted for any client organi- works that have been cited in the text. It includes sources
zation, this section could be treated as the most important not cited in the main text, but those that are relevant to
part of the report. Sometimes, this section is included at the subject of study, specifically in case of larger disserta-
the beginning of the report. tions or theses. Small research projects may need just a

M02_MADAN 04_65901_C02.indd 33 23/12/22 7:15 PM


2.34 Chapter 2

reference section to include all the literature that has been Certificate
referred to in the report.
Certified that this research project titled _____________
For Books _____________________ is the bona fide record of work
1. Authors’ surname (alphabetically), followed by their carried out by __________ for final year __________.
initials.
2. Date of publication. Technical guide Research coordinator Principal
3. Title of the book in italics.
Place __________  Date __________
4. Place of publication and publisher.

For Journal Articles


The title of the article appears in inverted commas and
the name of the journal comes in italics, followed by the Format and Styles of Referencing
volume number and pages of the article. A referencing style is a set of rules that tell you how to
acknowledge the thoughts, ideas, and works of others in
Example a particular way.
Madaan. KVS. ‘Influence of British Rule on Indian Culture’. Referencing plays a crucial role in the following:
Journal of Tourism. 10–18.
1. Successful academic writing.
Appendices 2. Avoiding plagiarism.
3. Key to your assignments and research.
This is specifically required in case of questionnaires or
interview schedules made for conducting the research; it Earlier, after the last step of research steps, we discussed
may be useful to include them in the report as an appendix. ‘Research Report Format’ that may vary somewhat
Appendices do not count towards your total number according to university as well. Different types of sources
of pages or words. It is a useful way of including relevant have different formatting requirements in the bibliogra-
material so that the examiner can gain a deeper under- phy. Background research plan and bibliography work-
standing of your work by reading it. sheet help in the development of bibliography.

Dissertation and Thesis


A dissertation culminates in a postgraduate degree such as MS/M.Tech./M.Sc./M.Phil., whereas a t­ hesis leads
to a doctoral degree (Europe and India). In American universities, a dissertation leads to a Ph.D. degree and a thesis
leads to a master’s degree. We will adhere to the former one.
In a dissertation, it is adequate if one has a decent knowledge of the new discoveries in order to arrive at the con-
clusion effectively. In a thesis, one has to substantiate the hypothesis with original research work. The hypothesis or
the ‘synopsis’ should contain the gist of the new findings one has made on the subject of research. The written thesis
should contain all the details of original research work that one has done on the subject. A thesis may be subjected to
scrutiny for any plagiarism to determine the originality of the effort. Another finite difference between the two is that,
in a thesis, analysis of the existing literature is added, whereas a dissertation by itself is an analysis of the existing
literature. The differences between a dissertation and a thesis are given below.
1. A researcher has to utilize the already collected information in order to prepare a dissertation, whereas thesis is
based on the research conducted entirely by the researcher.
2. A thesis is lengthier and takes more time to be completed, whereas a dissertation is short. Therefore, it does not
consume too much time to be completed.
3. In a thesis, the researcher has to include a hypothesis based on the research work. In contrast to a thesis, in a
dissertation, the researcher should have a decent knowledge of the new discoveries in order to infer conclusions
effectively.
4. In a thesis, the researcher has to focus on the primary argument in order to prove the standpoint to the readers.
Conversely, a dissertation focuses on background work.
5. In a master’s dissertation, the researcher has to utilize the research work in order to prove his point. In case of a
Ph.D. thesis, the researcher has to add novel findings to the existing literature.
6. A thesis is written as an academic research paper, whereas a dissertation is more like an academic book.
7. The data collected in a dissertation is a based upon the hypothetical analysis of contents, whereas a thesis is com-
prised of theory and argumentation based on original research.
The structure of a dissertation and thesis writing is normally described in a university’s students’ handbook.

M02_MADAN 04_65901_C02.indd 34 23/12/22 7:15 PM


Research Aptitude 2.35

There are standards for documenting sources of infor- The other popular styles are given as follows:
mation in research papers. Even though different journals
may use a slightly different format for the bibliography, Abbreviation Full Name
they all contain the same basic information. The most
basic information is that each reference should have the ACS American Chemical Society
author’s name, the title, the date, and the source. AGLC Australian Guide to Legal Citation
In research areas, the two most commonly used
guidelines for this formatting are published by the MLA AMA American Medical Association
(Modern Language Association) and the APA (American AMJ Academy of Management style
Psychological Association). The MLA guidelines call the
bibliography ‘works cited’. CMS Chicago Manual of Style
The APA guidelines call the bibliography Reference List.
CSE (CBE) Council of Science Editors/Council of
Biology Editors
APA article Author, A. A., Author, B. B., and Harvard
from a Author, C. C. (Year). Title of article.
database Title of Periodical, volume number IEEE Institute of Electrical and Electronics
(issue number), pages. DOI Engineers
Vancouver
APA Website Author, A. A., and Author, B. B. (Date
of publication). Title of document.
Retrieved date, from name of Website, Chicago/Turabian style is generally used for business,
http://Web address history, and the fine arts.
The Indian National Bibliography has been conceived as
an authoritative bibliographical record of current Indian
publications in Assamese, Bengali, English, Gujarati,
MLA article Author, First name. ‘Title of Article.’ Hindi, Kannada, Malayalam, Marathi, Oriya, Punjabi,
from a Title of Journal Volume. Issue (Year): Sanskrit, Tamil, Telugu, and Urdu languages, received in
database pages. Name of database. Web. Date the National Library, Kolkata, under the Delivery of Books
of access. and Newspapers (Public Libraries) Act, 1954.
The main entries are in Roman script and the collations
MLA Website Editor, author, or compiler name. and annotations, if any, are in English. The classified por-
Name of Site. Version number. tion follows the Dewey Decimal Scheme of Classification,
Institution/organization affiliated but the numbers from the Colon Classification scheme are
with the site, date of resource creation. assigned to each entry at the bottom right hand to facili-
Web. Date of access. tate the use of the bibliography and libraries arranged
according to the Colon Schemes of Classification. The
Indian National Bibliography and Central Reference
Library fall under the supervision of Ministry of Culture,
APA Style MLA Style Government of India.

Used in social sciences Used in humanities

The title is in italics – only the The title is underlined, Main Terms Used in Context of Footnotes
most necessary words are put all major words are
and Reference Writing
in capital letters. capitalized.
1) Ibid is the abbreviation for the Latin word, Ibidem,
The source page is titled as Source page is titled which means the same. It refers to the same author and
‘references’ or reference links. ‘works cited’ source (Examples: Book and journal) in the immedi-
ately preceding reference.
Here, only the last name of Here, it is the full 2) op. cit. is the abbreviation for the Latin term, opus
author is used. name. citatum, meaning the work cited. It refers to the refer-
ence listed earlier by the same author.
Author’s name and year of Here, it is author’s 3) Loc. cit. is the abbreviation for the Latin origin loco
publication is used for ‘in-text’ name and page citato, meaning in the place cited. It is a footnote or
citations. numbers. endnote term used to repeat the title and page number
for a given work.
Commas are used for ‘in-text’ No use of commas. 4) et. al. refers to and others, where it is used for refer-
citations. ring to a number of people.

M02_MADAN 04_65901_C02.indd 35 23/12/22 7:15 PM


2.36 Chapter 2

Microsoft Excel is a popular spreadsheet software. The


other spreadsheet packages are Lotus 1-2-3,
Shodhganga and Shodhgangotri: Shodhganga is Quattro Pro, Javelin Plus, Multiplan, VisiCalc,
the name coined to denote the digital repository SuperCalc, Plan Perfect, etc.
of Indian electronics thesis and dissertations set up by
Information and Library Network (INFLIBNET) Centre, Other Statistical Tools
an autonomous inter-university centre (IUC) of the SAS, S-PLUS, LISREL, EViews, etc.
University Grants Commission. Now, the candidates
will have to store the thesis in a compact disc (CD) and Word Processor Packages
upload each chapter in a separate PDF file using the A word processor (document preparation system) is an
naming convention as prescribed by Shodhganga. The ICT application used for the production (including com-
CD must be authenticated by the Supervisor/Head of position, editing, formatting, and possibly printing) of
Department. any sort of printable material.
INFLIBNET introduced Shodhgangotri, which has been The word processing packages are Microsoft Word,
built to maintain a database of synopsis of on-going WordStar, WordPerfect, Softward, AKHAR (Gujarati),
M.Phil./Ph.D. in Indian universities and institutions. Ami Pro, etc.
Turnitin is an American commercial, Internet-based
plagiarism detection service. Presentation Software
A presentation program is a computer software package
used to display information, normally in the form of a slide
show. It typically includes three major functions: an editor
that allows text to be inserted and formatted, a method for
Application of ICT Tools on Research inserting and manipulating graphic images, and a slide
Process show system to display the content.
The presentation packages are Microsoft Powerpoint,
Use of ICT in research is very extensive, where nowadays Lotus Freelance Graphics, Corel Presentations, Apple
it is difficult to conceive a scientific research project with- Keynote, etc.
out it.
Many research studies cannot be carried out without the Database Management Packages (dbms)
use of computers and specifically ICT that entails complex Database is an organized collection of information. DBMS
computations, data analysis, and modelling. Computer in is a software designed to manage the database.
scientific research is used at all stages of study, from pro- The various desktop databases are Microsoft Access,
posal/budget stage to submission/presentation of findings. Paradox, dBase or dBase III+, FoxBASE, FoxPro/ Visual
Statistical Analysis Tool: The acronym SPSS stands for FoxPro, and FileMaker Pro.
Statistical Package for Social Sciences. The latest version The commercial database servers that supports
of SPSS is IBM SPSS STATISTICS 20 (purchased by IBM multiuser are Oracle, MS SQL Server, Sybase, Ingres,
after version 19). It provides the following attributes: Informix, DB2 UDB (IBM), Unify, Integral, etc.
The open source database packages are MySQL,
1. Provides data view and variable view measures of PostgreSQL, Firebird, etc.
central tendency and dispersion
2. Statistical inference Browsers
3. Correlation and regression analysis A web browser is a software application which enables
4. Analysis of variance a user to display and interact with text, images, vid-
5. Non-parametric test eos, music, games, and other information typically by
6. Hypothesis tests: t-test, chi-square, z-test, ANOVA, accessing a web page found on a website which is col-
Bipartite variable, etc. lectively provided in the world wide web or a local area
7. Multivariate data analysis network.
8. Frequency distribution Some examples of browsers are Microsoft Internet
9. Data exposition by using various graphs, like line, Explorer, Mozilla Firefox, Opera, Netscape Navigator,
scatter, bar, Ogive, histogram, pie chart…. Chrome (Google browser), Safari, etc.
Data A nalysis T ool: Spreadsheet Packages
Tools T hrough Internet
Since ICT has become an indispensable tool for research,
various content has been picked from w­ ebsites to throw • Search engines (To search the information)
some information on it. A spreadsheet is a computer • Google (Popular search engine)
application that simulates a paper worksheet. It dis- • Yahoo!
plays multiple cells that together make up a grid con- • WebCrawler
sisting of rows and columns, each cell containing either • Excite
alphanumeric text or numeric values. • AltaVista

M02_MADAN 04_65901_C02.indd 36 23/12/22 7:15 PM


Research Aptitude 2.37

Online Data/Documentation Management confidentiality, present loss versus future benefits and
(To Manage Documents Online) so on. Each decision made in research involves a poten-
• Dropbox tial compromise of one value for another. However,
• Google Drive researchers must still try to minimize the risk to par-
• Google Docs ticipants, colleagues, and society while trying to opti-
• MS SkyDrive (Free) mize the quality of outcome. Research ethics help us to
• Microsoft 365 (Paid version) reconcile value conflicts.
The benefits of observing ethics in research studies are
as follows:
Online Data Collection
(To Collect Data Online from Different Users) 1. It helps in promoting the aims of research, such as
• Online forms - Online questionnaires bringing out the truth and avoidance of errors.
• Online surveys 2. It promotes the values that are essential to collab-
• Collaboration tools orative work, such as trust, accountability, mutual
• Skype : Voice and video conferencing respect, and fairness.
• Google Hangouts : Voice and video conferencing 3. It holds the researcher accountable to the public and
society.
Modern Research Tools 4. It helps in building public support for research, which
in return can help in getting participants who take
Modern electronic research tools, like Zotero and part in the research willingly.
Evernote, make the collection of research data and col-
laboration between colleagues possible, which in the past
would have been difficult, expensive, or even impossible. Main A pproaches to Research Ethics
They also save a great amount of time citing and creating The following are the three major approaches to ethics:
bibliographies. Evernote allows the user to capture digi-
tal content, including web pages, PDF files, or snippets of 1. Deontological Approach: We should identify and
web pages, organize them, annotate them, share them, use a universal code in making ethical decisions. This
publish them, and search them. is an absolutist approach.
2. Ethical Scepticism Approach: It states that ethical
standards are not universal, but are relative to one’s
own particular culture and time. This is based on
Research Ethics relativism.
Ethics are the principles and guidelines that help us 3. Utilitarianism Approach: This can be taken as the
uphold the things we value. Ethics and law are differ- modern version of the end justifying the means.
ent aspects, although laws of the land are intended to be Research decisions are based on utilitarian cost
based on certain ethics. Almost all societies have legal -benefit analysis. Some comparison may be done.
rules to govern certain behaviour in a country or society, If the expected benefits exceed the expected risks,
but ethical norms tend to be broader and more infor- the study is presumed to be ethical. Sometimes
mal than laws. An action may be legal, but unethical the issue may be of potential harm to human life
or illegal, but ethical. Ethics aim to achieve two funda- or animal life, thus such qualitative issues become
mental objectives, i.e., to tell us how we ought to act in crucial for the purpose of taking research decisions.
a given situation and to provide us with strong reasons
for doing so. Some Desirable Elements to Ensure
Ethics always emerge from a conflict between values,
and research ethics are not an exception. In research, E thics in Research
these conflicts may take different forms, such as par- The following is a general summary of some research
ticipant’s concern for privacy versus some justifica- ethical guidelines and principals that various codes
tion for manipulation, openness and replication versus address.

M02_MADAN 04_65901_C02.indd 37 23/12/22 7:15 PM


2.38 Chapter 2

Ethical Issues Relating to Participants


Honesty in reporting data, results, methods and
1 procedures, and publication status. There are many ethical issues in relation to participants
of a research activity. One of the most commonly cited
Objectivity to avoid bias in experimental design, ethical principles is that we should not cause harm to
2 data analysis, interpretation, and peer review. our research participants. The issue of ethics in research
mainly caught the attention of policy makers as a result of
Integrity, acting with sincerity, and striving for
many gruesome instances, few of which have been men-
3 consistency of thought and action. tioned below.
1. Medical experiments conducted by Nazi doctors in Ger-
Carefulness to avoid careless errors and man concentration camps in the 1930s. Nazi doctors in
4 negligence and proper documentation of German concentration camps killed twin gypsy teenag-
all aspects. ers while conducting experiments in order to determine
why some of them had differently coloured eyes.
Openness in sharing data, results, ideas, tools, 2. A South African oncologist experimented with women
5 and resources and openness to criticisms and suffering from cancer, administering excessive dos-
new ideas. ages of chemotherapeutics without informing the
patients and taking their due consent.
Respect for intellectual property rights, such 3. Ethical issues during organ transplant, s­terilization
6 as patents, copyrights and other forms of and so on.
intellectual property. 4. Experiments on animals.
In research, specifically in medical sciences, the obser-
Confidentiality in context of communications,
7 ­personal records, and privacy issues.
vance of ethics is very crucial. Even now, when phar-
maceutical companies want to conduct clinical trials in
underdeveloped or developing countries, this issue crops
Responsible publication, with the aim to serve up again and again. It is a general understanding that eth-
8 the society. Avoiding wasteful and duplicative ical research issues are more moral than legal.
publication. Apart from physical injury, the psychological distress
or emotional harm, loss of self-esteem, being persuaded
Responsible mentoring in terms of guiding to conduct morally ­reprehensible acts, and hampering
9 research students. one’s physical, intellectual, or emotional development are
other important concerns.
Respect for colleagues translates to extending We must also be careful about the security of our
10 fair treatment to colleagues. research records, so that respondents may not be identi-
fied or otherwise harmed through loss of confidentiality.
Social responsibility means to serve the society During the initial phase of medical research history,
11 and different stakeholders. people participating in trials were referred to as research
subjects. Now, they are known as trial participants. Now,
Non-discrimination against colleagues or their role has transformed from a passive subject to that of
12 students on the basis of sex, race, or factors that an active participant. Thus, we can see that research eth-
are not related to their scientific competence and ics are basically about means of ensuring that vulnerable
integrity. people are protected from exploitation and other forms of
harm. The ethical issues are to be observed at every stage
Enhancing competence for own professional of a research process.
13 advancement or lifelong learning and taking steps
to promote competence in science as a whole. 1. Collecting Information: Before a researcher actually
collects information, his request for information may put
Ensuring legality of the whole process by pressure or create anxiety on a respondent, and this may
14 obeying relevant laws, i.e., institutional and not be ethical. However, without research, there will be
governmental policies. no intellectual progress or development in the society. A
researcher is required to improve the conditions by con-
sidering various points in the ensuing discussion.
15 Animal care through proper experimental designs. 2. Seeking Consent of Participants: Informed consent
refers to an individual’s willingness to participate in
a study. Individuals who provide informed consent
have been made aware of the design and procedures
Stakeholders in Research with enough detail to exercise a rational decision to
There are three stakeholders in the research process, participate.
namely the participants or subjects, the researcher, and 3. Providing Incentives: Most people do not partici-
the funding organization. pate in a study because of incentives, but they are

M02_MADAN 04_65901_C02.indd 38 23/12/22 7:15 PM


Research Aptitude 2.39

motivated because of the importance of the study. 1. Fabricating Behaviour: Creation of spurious data by
Giving a gift before data collection is not ethical on researcher, their recording, and drawing inferences.
the part of a researcher. 2. Falsification: It manipulates the research material,
4. Seeking Sensitive Information: Some pieces of equipment, and processes or changes or omits data or
information can be regarded as sensitive or confi- results such that the research is not accurately repre-
dential by some people. This may be akin to invading sented in the research records.
their privacy. Seeking such information may upset 3. Plagiarism: It is the act of appropriating somebody
them. Questions on income, age, marital status, etc., else’s ideas, thoughts, pictures, theories, words,
may be considered intrusive. or stories and presenting those as your own. If a
  However, it may not be unethical to enquire if the researcher plagiarizes the work of others, the integ-
participants need explanation before the research rity, ethics, and trustworthiness of the sum total of his
and if they are given sufficient time to decide if they or her research becomes questionable. Plagiarism is
want to participate without any major inducement. both illegal and a punishable act and is considered to
5. Possibility of Causing Harm to the Participants: be on the same level as stealing from the author who
When you collect data from the respondents or involve originally created it. It can take the following forms:
subjects in an experiment, you need to examine care- (a)  Intra-corpal: A case of plagiarism where one
fully whether their involvement is likely to harm them student has copied from another in the same sub-
in any way. Harm may include the use of chemicals, mission is known as intra-corpal plagiarism.
drugs, discomfort, anxiety, harassment, invasion of (b)  Extra-corpal: It is an instance of plagiarism where
privacy, or demeaning or dehumanizing procedures. a student has copied the material from an external
Even after the consent, the researcher must make sure source (Example: Books, journal article, world
that the risk is minimal. wide web, etc.).
6. Maintaining Confidentiality: In case the researcher (c)  Autoplagiarism: It is citing one’s own work with-
has to identify the respondent as information needs to out acknowledgement.
be sought more than once, sharing information about 4. Multiple Authorship: There can be many impropri-
a respondent with others for purposes other than eties in authorship. Improper assignment of credit,
research is not ethical, and the information provided such as excluding other authors, inclusion of others
by the respondent should at least be kept anonymous. as authors who have not made a definite contribution
towards the work published, or submission of multi-
Ethical Issues Relating to the Researcher authored publication without the knowledge of all
1. Avoiding Bias: Objectivity in research means to avoid the authors.
bias in the research process as it is considered unethi- 5. Peer Review: It is the process in which an author
cal. Bias means a deliberate attempt to either hide submits a written manuscript or an article to a jour-
facts or to under-represent or over-represent them. It nal for publication. The journal editor distributes
may undermine the truth. the article to experts or reviewers. The peer review
2. Provision or Deprivation of a Treatment: This is process seldom proceeds in a straight line. The entire
specifically true in case of medical research. Is it ethi- process may involve several rounds of communica-
cal to provide a study population with an intervention tion between the editor, the reviewers, and the origi-
or treatment that has not yet been conclusively proven nal author before an article is ready for publication.
effective? Thus, it imposes an ethical dilemma before   The two most important ethics in the process are
researchers. Informed consent, minimum risk, and maintaining confidentiality and the protection of
frank discussion can help resolve the ethical issues. intellectual property. Reviewers and author should
3. Using Inappropriate Research M ­ ethodology: The not know the names of each other. Only then, the
deliberate use of a highly biased sample, method, or peer review process can be genuinely open and ben-
procedure is unethical. eficial. None in the process can publicly disclose the
4. Incorrect Reporting: This can be done to advance information in the article or use the information in a
the interests of the researcher. submitted article for personal gain.
5. Inappropriate Use of the Information: Sometimes, 6. Duplicate and Partial Publication: It is publishing the
it is possible to harm individuals in the process of same data and same results in more than one publica-
achieving benefits for the organization. An example tion or journal. This is unethical, but may be acceptable
would be a study to help in the formulation of a pol- in certain cases, such as publishing results in a jour-
icy by the organization. New policy may not serve nal to provide research participants with a summary
the interests of certain individuals, but may be good of the results. Partial publication involves publishing
for the organization as such. Should you ask respon- parts of your results in different journals. It is specifi-
dents for information that is likely to be used against cally unethical for a small, focused study. However, in
them? case of large studies with many variables, this may be
acceptable as different publications involve different
Some of the key terms used in the context of ethical issues research questions and different data, and this actually
concerning researchers are as follows: advances the ­interest of the study.

M02_MADAN 04_65901_C02.indd 39 23/12/22 7:15 PM


2.40 Chapter 2

Important Measures to Make 3. Journal selection


Research More Ethical 4. Journal submission
1. Informed Consent: The provision of informed con- 5. Manuscript tracking
sent also includes the knowledge that the informed 6. Peer review
participation is voluntary and that participants can 7. Manuscript rejection
withdraw from the study at any time. 8. Post publication
2. Protective Research Design: This involves estimat-
A review article or review paper summarizes the findings
ing the probability of happening of harmful effects,
of existing literature. So, the readers can develop an idea
their severity, and the likely duration of these effects.
about the existing knowledge on a topic without having to
3. Screening: It is an attempt to select only those indi- read all the published works in the field. It does not report
viduals for study who show a high tolerance for poten- original research. Review articles generally summarize
tial risks. the existing literature on a topic in an attempt to explain
4. Pilot Studies: When the potential harms the current state of understanding on the topic. Review
are ­uncertain, a useful precaution involves a pilot articles can be of three kinds.
study with follow-up diagnostic interviews to assess
the effects and request advice from the participants. 1. Narrative Review: It explains the existing knowledge
5. Outside Proposal Review: Requesting others to on a topic based on all the published research avail-
review research proposals is a helpful precaution for able on the topic.
minimizing risks. 2. Systematic Review: It searches for the answer to a
particular question in the existing scientific literature
6. Professional Codes: Two features of professional
on a topic.
codes are important for discussion. Firstly, profes-
3. Meta-analysis: It compares and combines the find-
sional codes have been developed inductively from
ings of previously published studies, usually to assess
the wide research experiences of professionals.
the effectiveness of an intervention or mode of
Secondly, professional codes place strong emphasis
treatment.
on researchers’ responsibility for their research.
7. Government Regulations: Government regulations Most reputed journals publish review articles. If published
such as state and central laws are designed to protect in a good peer-reviewed journal, the review articles often
or advance the interests of society and its individu- have a high impact and receive a lot of citations.
als. Thus, the researchers are required to take certain
precautions. Difference Between a Thesis and an Article
A researcher is always under pressure to publish, where
one good way to do this is to convert doctoral thesis into
Articles, Workshop, Seminar, a journal article, during or after Ph.D. It is essential to
know how a thesis differs from a journal article. Here are
Conference, And Symposium some of the elements that you will need to work on to
successfully create a journal article from your thesis.
‘The whole of science is nothing more than a
refinement of everyday thinking’ Thesis Article
 — Albert Einstein
Purpose The purpose is The purpose is
education as it advancement
shows how much a to enhance
Article or J ournal A rticle person knows. credibility and
Since we are discussing articles within research, an contribution in
article is also to be discussed in the manner of research the field.
only. This topic can be divided into research article and
review paper. Audience Educational Here, person may
A research article is based on original research. The kind committee and look to become
of research varies depending upon the field or the topic professors decide a scientist or
(experiments, survey, interview, questionnaire, etc.). Here, whether a person is conduct further
the authors need to collect and analyze raw data and con- worthy of degree. research.
duct an original study. The research paper will be based on
Abstract Longer, up to 500 Shorter, up to
the analysis and interpretation of these data.
words 150–250 words.
The various steps followed to show the process have
been given as: Introduction More detailed More concise, only
1. Conducting research absolutely required
2. Manuscript writing information.

M02_MADAN 04_65901_C02.indd 40 23/12/22 7:15 PM


Research Aptitude 2.41

Thesis Article W ebinars or Web Conferences


Webinars or web conferences are presentations that
Length Longer, as the It is shorter, involve an audio and video component. The audio portion
page count can be between 3000 of the event is delivered via phone or over the internet, so
up to fifty pages and 6000 words. that participants can listen via their computer speakers.
and around 20000 It is better to The video portion of the event is delivered via the inter-
words. avoid copying, net, giving participants a presentation to watch while lis-
rewriting, or tening to the instructor.
paraphrasing.
Seminar
Material and Extensive Controlled
The word seminar is derived from the Latin word semi-
method presentation presentation
narium, meaning seed plot. It is a formal presentation by
Discussion Detailed Clear and concise one or more experts to a small group of audience. It can be
interpretation of presentation of conducted on recurring or regular basis, monthly or even
results results. weekly. There is an invited speaker, and the audience is
much more technically versed or specific in nature.
References Exhaustive list Selective list The motive behind the seminar system is to familiarize
students extensively with vital aspects of their study and
Appendices Inclusion Inclusion optional allow them to interact with examples of practical prob-
mandatory lems that always occur during study or research work.
Thus, a seminar is a form of academic instruction either
at an academic institution or offered by a commercial or
professional organization.
Meeting Seminars focus on some particular subject in which
A meeting is an assembly or coming together of people, everyone present is requested to actively participate.
be it a symposium, workshop, conference or so. In a very Colloquia and seminars both happen in an ­academic
remote sort of a way, all of them convey the same mean- setting. Phenomena such as global warming and climate
ing, i.e., people coming together for a purpose. change and nuclear power accidents are discussed but
from the perspective of a scientist. However, a well-edu-
Symposium cated audience is able to understand it.
It is usually a formal meeting in which specialists deliver Teleseminars are seminars delivered via a conference
short addresses on a topic or on related topics and then call over the telephone and/or through the internet.
answer the questions relating to these topics. It is espe-
cially one in which the participants form an audience and W orkshops
make presentations. Workshops tend to be smaller and more intense than semi-
A symposium is also defined as a collection of writings nars. This format involves students practising their new skills
on a particular topic, as in a magazine. during the event under the watchful eye of an instructor.
Hands-on workshops typically involve participants
Colloquium doing work on a particular issue during the program. The
promise is that when they leave, they will have at least a
It is usually an academic meeting in which specialists
rough plan or tools in place to address the challenge.
deliver addresses on a topic or on related topics and then
answer the questions relating to these topics. A colloquium Impact Factor
is targeted to a well-educated but not specialized audience.
The impact factor of an academic journal is a measure
reflecting the average number of citations to recent arti-
Conference cles published in the journal. It reflects the relative impor-
A conference is a meeting of people who confer about a tance of a journal within its field. Journals with higher
topic. It is a meeting where people come for discussion. impact factors are deemed to be more important than
It features keynotes and presentations delivered to those with lower ones.
all attendees, as well as multiple break-out sessions. The h-index is an index that attempts to measure both
Attendees expect to receive information about industry the productivity and the impact of the published work of
trends and developments. a scientist or a scholar.
It can be an academic conference (a formal event The g-index is like the h-index, and it has an averaged
where researchers present results), a business confer- citation count.
ence (organized to discuss business-related matters), The i10-index indicates the number of academic
or a parent–teacher conference (meeting with a child’s publications an author has written that has at least ten
teacher to discuss grades and school performance), a citations from others. It was introduced in July 2011 by
peace conference (a diplomatic meeting to end conflict) Google as part of their work on Google Scholar, a search
and so on. engine dedicated to academic and related papers.

M02_MADAN 04_65901_C02.indd 41 23/12/22 7:15 PM


2.42 Chapter 2

A s s e s s Yo u r L e a r n i n g

RESEARCh – BASIC COnCEPTS


1. Research is 8. A scientific approach in social science research should
(a) A purposeful, systematic activity. take into account
(b) Conducted for purely academic purposes. A. Metaphysical elements
(c) Conducted to answer questions about practical B. Spurious relations
issues. C. The factor of co-variation
(d) A random, unplanned process of discovery. D.Time order
2. Which of the following can be termed as the main E. The possibility of theorisation
objectives of research ? Choose the most appropriate answer from the options
1. To achieve research objectives which can be gen- given below:
eral as well as specific. (a) A, B, C only
2. To get valid, reliable, accurate, timely and com- (b) B, C, D only
plete information. (c) C, D, E only
3. To solve the problems of society. (d) A, B, E only
4. To expand our knowledge base.
5. To gain familiarity with new phenomena. 9. Consider the following definitions.
1. Scientism: Science is qualitatively distinct from,
Codes: superior to and ideal for all other areas of human
(a) 1, 2 and 3 endeavor.
(b) 2, 3 and 4 2. Methodologism: The distinction, superiority and
(c) 2, 3, 4 and 5 idealhood that Science enjoys is traceable. to its
(d) 1, 2, 3, 4 and 5 possession of method.
3. A grand theory 3. Methodological monism: There is only one
(a) Explains interrelationships among concepts. method common to all sciences, irrespective of
(b) Is highly abstract. their subject matter.
(c) Broad explanation of phenomenon in a discipline. 4. Inductivism: A method that is common to all sci-
(d) All of the above ences, natural or human.
A S S E S S YO U R L E A R N I N G

4. The idea that knowledge comes from experience is Which of the above definition are correct?
(a) Rationalism (a) 1 and 2
(b) Deductive reasoning (b) 2 and 3
(c) Logic (c) 1, 2 and 3
(d) Empiricism (d) All are correct
5. Which of following descriptions is true in context of 10. Consider the following statements
defining ‘theory’? 1. The nature of reality is external to human mind.
(a) An organized body of concepts and principles 2. The focus is on critical reality.
intended to explain a particular phenomenon. 3. The purpose of research is to find universals.
(b) Tentative explanations that new data either sup- 4. The acceptable method and data is basically
port or do not support. ‘scientific method’.
(c) Apt to drive further research. 5. This method is nearer to inductive approach.
(d) It is always contradictory to related generalization. Which of the following approach is working in above
6. Epistemology refers to statements?
(a) A term specifically used in the social sciences. (a) Positivism
(b) A term used to study the types of diseases. (b) Post positivism
(c) Acceptable level of knowledge in a field of study. (c) Deductive approach
(d) A type of interviewing technique. (d) Experimental approach
7. Which of the following is a function of theory? 11. Which of the following can be defined as the main
(a) Integrating and summarizing current knowledge attributes of research?
(b) Making predictions 1. Reliability
(c) Explaining phenomena 2. Validity
(d) All of the above 3. Objectivity

M02_MADAN 04_65901_C02.indd 42 23/12/22 7:15 PM


Research Aptitude 2.43

4. Subjectivity 2. The person must have the qualities of curiosity,


5. Intuition active imagination and intuition.
Codes: 3. The researcher should have the qualities of induc-
tive thinker for social research.
(a) 1, 2 and 3 4. The result of setting out a reasoned argument in
(b) 2, 3 and 4 steps is known as evaluation.
(c) 2, 3, 4 and 5 5. The researcher must have the mind set to generate
(d) 1, 2, 3, 4 and 5 new principles and theories.
12. Which of the following is defined as a systematic Codes:
method of evaluating statistical data based on the
results of several independent studies of the same (a) 1, 2 and 3 (b) 2, 3 and 4
problem? (c) 1, 2, 3, 4 and 5 (d) 1, 2, 3 and 4
(a) Factor analysis (b) Meta-analysis 18. What do you consider as the main aim of interdiscipli-
(c) Systematic analysis (d) Subjective analysis nary research?
13. In research, evidence that the same concept meas- (a) To bring out holistic approach to research.
ured in different ways yields similar results is called as (b) To reduce the emphasis of single subject in
research domain.
(a) divergent validity (c) To oversimplify the problem of research.
(b) discriminant validity (d) To create a new trend in research methodology.
(c) convergent validity
(d) face validity 19. The positivist researchers consider reality as
(a) Subjective (b) Objective
14. The conceptual framework in which a research is con- (c) Neutral (d) Insignificant
ducted is called a  [June 2019]
(a) Synopsis of research 20. Which of the following statements is true about theory?
(b) Research design (a) It explains phenomenon in simple manner.
(c) Research hypothesis (b) It explains the ‘how’ and ‘why’ questions.
(d) Research paradigm (c) It can be a well-developed explanatory system.
(d) All of the above
15. The rationalistic mode of knowledge accepts the rules
of 21. Given below are two statements
(a) Legality (b) Manipulation Statement I: Qualitative research is impressionistic
(c) Logic (d) Authority and subjective.
16. Match The Following: Statement II: Quantitative research has a higher
level of possibility of generalising the findings.

A S S E S S YO U R L E A R N I N G
Research In light of the above statements, choose the correct
Features Basic Points answer from the options given below:
(A) Reliability (i) entities, procedure, or (a) Both Statement I and II are true.
devices to measure the (b) Both Statement I and II are false.
dimensions that are (c) Statement I is true but Statement II is false.
used to measure (d) Statement I is false but Statement II is true.
(B) Validity (ii) constantly thinking 22. Which of the following point/s is/are significant for
about the research Research Methodology that is a wide term ?
process and its progress 1. theoretical perspectives or orientation to guide
research and logic of inquiry
(C) Intuition (iii) Linked with any
2. tools and techniques of data collection
personal bias may vitiate
3. methods of data analysis
research results.
4. only deductive methods
(D) Objectivity (iv) stable, consistent, and Codes:
dependable
(a) 1, 2 and 3 (b) 2, 3 and 4
Codes: (c) 1, 3 and 4 (d) 1, 2, 3 and 4
(a) (A)–(iv), (B)–(ii), (C)–(i), (D)–(iii) 23. The ‘feasibility studies’ are pieces of research done
(b) (A)–(iv), (B)–(i), (C)–(ii), (D)–(iii) before a main study to answer the question ‘can this
(c) (A)–(iii), (B)–(i), (C)–(ii), (D)–(iv) study be done?’ The feasibility of a research study
(d) (A)–(iv), (B)–(i), (C)–(iii), (D)–(ii) generally depends upon
17. Which of the following should be termed as the quali- 1. Cost factor
ties of a researcher ? 2. Time required to conduct research
1. The person must possess thinking and reasoning 3. The skill set of the researcher
ability. 4. The team spirit and work

M02_MADAN 04_65901_C02.indd 43 23/12/22 7:15 PM


2.44 Chapter 2

Codes: (a) Positivism


(a) 1, 2 and 3 (b) 2, 3 and 4 (b) Interpretivism
(c) 1, 3 and 4 (d) 1, 2, 3 and 4 (c) Qualitative
(d) None of the above
24. An empiricist believes that
(a) Natural science methods should not be applied to 28. Look at the following statements
social science research. 1. This method suggested by Karl Popper, is actually
(b) Social science methods cannot be applied in natu- a proposed description of scientific method.
ral sciences. 2. According to it, scientific inquiry proceeds by for-
(c) Knowledge is acquired through our sensory mulating a hypothesis in a form that could con-
perceptions. ceivably be falsified by a test on observable data.
(d) No knowledge is genuine. 3. A test that runs contrary to predictions of the
25. Look at the following statements hypothesis is taken as a falsification of the hypoth-
esis. Otherwise the hypothesis is corroborated.
1. These are socially constructed. 4. That the method which is common to all sciences,
2. They reflect understanding. natural and human.
3. The subjective and objective research methods are
acceptable. Which of the following concepts is being referred by
4. The understanding is contextual and there is the above statements?
reduced focus on universal aspects. (a) Hypothetico-deductive method
5. There are integrated activities. (b) Hypothetico-inductive method
Which of the following terms is being referred here? (c) Inductive method
(d) Descriptive method.
(a) Positivism (b) Hypothetico-deductivism
(c) Interpretivism (d) Constructivism 29. An important practical issue to consider while design-
ing a research project is
26. What is the position held by a positivist as far as (a) An interesting theoretical perspective.
acquiring knowledge is concerned? (b) Addition to knowledge of researcher only.
(a) A general positive attitude towards research. (c) Availability of time and other resources.
(b) Scientific research should be based on value-free, (d) It should be qualitative.
empirical observations.
(c) Exact knowledge can be acquired from society. 30. The time frame during which a research study
(d) None of the above explores a situation or problem is called as
(a) Reference period
27. The approach which is based on the assumption that (b) Age
social phenomena can be explained by observing (c) Longitudinal
A S S E S S YO U R L E A R N I N G

cause and effect is (d) Retrospective time

Types of Research
31. The two main approaches of a research are List-I List-II
(a) Data collection and data analysis
(b) Surveys and questionnaires (C) Philosophical method (iii) Present events
(c) Sampling and data collection (D) Experimental method (iv) Future action
(d) Qualitative and quantitative
32. Qualitative research is Codes:
(a) Without any specific purpose. (a) (A)–(i), (B)–(iii), (C)–(ii), (D)–(iv)
(b) Primarily concerned with in-depth exploration of (b) (A)–(i), (B)–(ii), (C)–(iii), (D)–(iv)
phenomena. (c) (A)–(i), (B)–(ii), (C)–(iii), (D)–(iv)
(c) Deals with the collection and analysis of numeri- (d) (A)–(ii), (B)–(iii), (C)–(i), (D)–(iv)
cal data. 34. Which of the following research specifically requires
(d) Dependent upon spiritual aspects only. objectivity to discover facts and causes from the data
33. Match List-I with List-II and choose the correct answer gathered for the ­purpose?
from the code given below. (a) Quantitative research
(b) Fundamental research
List-I List-II (c) Qualitative research
(d) Action research
(A) Historical method (i) Past events
35. There are two sets given below. Set-I specifies the type
(B) Survey method (ii) Vision of research, while Set-II indicates their characteris-

M02_MADAN 04_65901_C02.indd 44 23/12/22 7:15 PM


Research Aptitude 2.45

tics. Match the two and give your answer by selecting 39. In the method of naturalistic observation, there will
the appropriate code. [July 2018] be
(a) Haphazard behaviour as it naturally occurs.
Set-I Set-II (b) Setting up of controlled experiments by which
(Research Type) (Characteristics) they uncover causal elements in behaviour.
(A) Fundamental (i) Finding out the extent of (c) Set out to actively observe subjects in their natural
research perceived impact of an environments.
intervention (d) Interview subjects at different stages of life.
(B) Applied (ii) Developing an effective 40. Which of the following is a form of explanatory
research explanation through research in which the researcher develops a theoreti-
theory building cal model and empirically tests the model to deter-
mine how well the model fits the data?
(C) Action (iii) Improving an existing (a) Causal modelling
research situation through use of (b) Predictive research
interventions (c) Descriptive research
(D) Evaluative (iv) Exploring the possibility (d) Exploratory research
research of a theory for use in 41. Match the following two lists.
various situations
(v) Enriching technological List-I List-II
resources (A) Experimental (i) Criticism
Codes: (B) Historical (ii) Control
(a) (A)–(ii), (B)–(iv), (C)–(iii), (D)–(i) (C) Case study (iii) Interpretative
(b) (A)–(v), (B)–(iv), (C)–(iii), (D)–(ii)
(c) (A)–(i), (B)–(ii), (C)–(iii), (D)–(iv) (D) Ethnography (iv) Intensive
(d) (A)–(ii), (B)–(iii), (C)–(iv), (D)–(v) (v) Intuitive
36. Which of the following are features of the qualitative Codes:
research paradigm? [June 2020] (a) (A)–(ii), (B)–(iii), (C)–(iv), (D)–(v)
A. The research is concerned with understand- (b) (A)–(i), (B)–(ii), (C)–(v), (D)–(iii)
ing the social phenomena from the participant’s (c) (A)–(iii), (B)–(i), (C)–(iv), (D)–(v)
perspective. (d) (A)–(ii), (B)–(i), (C)–(iv), (D)–(iii)
B. It seeks to establish relationships, among vari-
ables and explains the causes of changes in meas- 42. Which of the following aspects are true in context of

A S S E S S YO U R L E A R N I N G
ured social facts. research in sciences, social sciences, and humanities?
C. It is conducted in actual settings as the direct source 1. The need of supervisor, topic, critical analysis,
of data and the researcher is the instrument. and patience.
D. It is concerned with the process rather than sim- 2. The triangulation method of research is essential
ply with outcomes or products. both in quantitative and qualitative researches.
E. It attempts to establish universal context-free 3. Random sampling is important and practical in
generalization. social sciences only.
Choose the correct answer from the options given 4. Manipulation is a necessary tool only in experi-
below: mental research that makes it different from
(a) A, B and C only descriptive research.
(b) B, C and D only Codes:
(c) A, C and D only (a) 1, 2 and 3
(d) C, D and E only (b) 2 and 4
(c) 2, 3 and 4
37. The research which is exploring new facts through
the study of the past is called (d) 1 and 4
(a) Philosophical research 43. Which one of the following is an important character-
(b) Historical research istic of the researchers involved in qualitative study?
(c) Mythological research (a) Non-partisan (b) Reflexivity
(d) Content analysis (c) Transparency (d) Enthusiasm
38. The scientific method can be used 44. Research can be classified as
(a) Only in physical sciences, such as physics and (a) Basic, applied, and action research.
chemistry. (b) Quantitative and qualitative research.
(b) Only in social sciences. (c) Philosophical, historical, survey, and experimen-
(c) Both in physical and social sciences. tal research.
(d) Only in qualitative kind of study. (d) All of the above

M02_MADAN 04_65901_C02.indd 45 23/12/22 7:15 PM


2.46 Chapter 2

45. The term associated with the theory and method of 56. Which of the following is classified in the category of
interpretation of human action in social sciences is developmental research?
(a) Theology (b) Hermeneutics (a) Philosophical research (b) Action research
(c) Ontology (d) Positivism (c) Descriptive research (d) All of the above
46. Which research approach is the most appropriate to 57. Action research is
establish a relationship that is causal in nature? (a) An applied research
(a) Causal-comparative (b) Experimental (b) A research carried out to solve immediate
(c) Correlational (d) Descriptive problems
47. Books and records are the primary sources of data in (c) A longitudinal research
(a) Historical research (d) Simulative research
(b) Participatory research 58. The term ‘phenomenology’ is associated with the pro-
(c) Clinical research cess of
(d) Laboratory research (a) Qualitative research (b) Analysis of variance
48. The type of research that tests hypothesis and theo- (c) Correlational study (d) Probability sampling
ries in order to explain how and why a phenomenon 59. Which of the following is not a longitudinal design?
operates as it does is (a) Panel
(a) Descriptive research (b) Predictive research (b) Cross-sectional
(c) Explanatory research (d) None of the above (c) Trend
49. The study in which investigators attempt to trace an (d) Both (a) and (c) are longitudinal designs
effect is known as 60. When a researcher starts with the dependent variable
(a) Survey research and moves backwards, it is called
(b) ‘Ex post facto’ research (a) Predictive research
(c) Historical research (b) Retrospective research
(d) Summative research (c) Exploratory research
50. Fundamental research reflects the ability to (d) Descriptive research
(a) Synthesize new ideas 61. The essence of the experimental method is
(b) Expound new principles (a) The correct calculation of Karl Pearson’s coeffi-
(c) Evaluate the existing material concerning cient of correlation.
research (b) Obtaining direct reports from subjects about their
(d) Study the existing literature regarding various subjective experience.
topics (c) Careful measurement and record keeping.
51. The strongest evidence for causality comes from which (d) Using control to identify cause-and-effect
A S S E S S YO U R L E A R N I N G

of the following research ­methods? connections.


(a) Experimental (b) Causal–comparative
62. The following is a qualitative research approach that
(c) Correlational (d) None of the above
involves observing variables in their natural envi-
52. One of the limitations of the case study is that ronments or habitats in order to arrive at objective
(a) There are few subjects for which it is applicable. research outcomes.
(b) There are no control groups. (a) Ethnographic research
(c) It requires a large and expensive sample size. (b) Case study research
(d) It can not be included in triangulation method. (c) Content analysis
53. Which of the following research method is termed as (d) Focused Interview
controlled observation? 63. Which of the following are common characteristics of
(a) Historical research experimental research?
(b) Philosophical research (a) It relies primarily on the collection of numerical
(c) Field experimentation data.
(d) All of the above (b) It can produce important knowledge about cause
54. Fieldwork-based research is classified as and effect.
(a) Empirical (b) Historical (c) It uses the deductive scientific method.
(c) Experimental (d) Biographical (d) All of the above
55. The research that applies the laws at the time of field 64. Which type of research is likely to provide the strong-
study to draw more and more clear ideas about the est evidence about the existence of cause-and-effect
problem is [December 2008] relationships?
(a) Applied research (b) Action research (a) Non-experimental research
(c) Experimental research (d) None of the above (b) Experimental research

M02_MADAN 04_65901_C02.indd 46 23/12/22 7:15 PM


Research Aptitude 2.47

(c) Historical research (a) Deductive method (b) Inductive method


(d) Descriptive research (c) Hypothesis method (d) Pattern method
65. Research in which the researcher uses the qualitative 71. Which scientific method is a bottom-up or generative
paradigm for one phase and the quantitative para- approach to research?
digm for another phase is known as (a) Deductive method (b) Inductive method
(a) Action research (c) Hypothesis method (d) Pattern method
(b) Basic research 72. The method of drawing conclusions based on the
(c) Quantitative research observation of each and every instance of a popula-
(d) Mixed method research tion is called the
66. Consider the following points (a) Scientific method (b) Deductive method
1. Existing theories are typically used only as point (c) Inductive method (d) Dialectic method
of departure for the analysis. 73. Which of the following is not a characteristic of a good
2. Theories are further developed by forming new theory or explanation?
concepts and relations. The contents of the new (a) It is parsimonious.
concepts are studied and illustrated. (b) It is testable.
3. Practical application of theory is illustrated by cases. (c) It is general enough to apply to different
Which of the following types of research is being situations.
referred in this kind of scenario? (d) All of the above.
(a) Quantitative Research 74. Which scientific method follows these steps, (i) obser-
(b) Qualitative Research vation/data, (ii) patterns, and (iii) theory?
(c) Mixed Research (a) Inductive (b) Deductive
(d) Experimental Research (c) Top-down (d) All of the above
67. The type of research typically conducted by teachers, 75. Which scientific method is a top-down or confirma-
counsellors and other professionals to answer ques- tory approach?
tions they have and to specifically help them solve (a) Deductive method (b) Inductive method
local problems is called (c) Hypothesis method (d) Pattern method
(a) Action research 76. The major characteristic of correlation analysis is to
(b) Basic research seek out [June 2019]
(c) Predictive research (a) Differences among variables
(d) Longitudinal research (b) Variations among variables
68. Consider the following statements in context of quan- (c) Association among variables

A S S E S S YO U R L E A R N I N G
titative research. (d) Regression among variables
1. The formulation of closed questions and the 77. Which of the following statements are true in context
answer categories to be prepared in advance. of hypothesis ?
2. The representativeness as per proportion of 1. A hypothesis is a tentative statement made that
population. needs logical and empirical confirmation.
3. The timing of analysis is after data collection. 2. A hypothesis can be formulated as a proposition
4. The standard statistical methods are frequently or set of propositions providing most probable
used. explanation for occurrence of some event or spec-
5. A-priori deducted theories are opeationalized and ified phenomenon.
tested on data. 3. A hypothesis must, therefore, be capable of being
Which of the above statements do apply? tested, either accepted or rejected.
4. A hypothesis should be stated in terms of a rela-
(a) 1, 2 and 5
tionship between the dependent and independ-
(b) 2, 3 and 4
ent variables.
(c) 1, 2, 3 and 5
(d) 1, 2, 3, 4 and 5 Codes:
69. Research that is done to examine the findings of some- (a) 1, 2 and 5
one else using the ‘same variables but different people’ (b) 2, 3 and 4
is called (c) 1, 2, 3 and 5
(a) Exploration (b) Hypothesis (d) 1, 2, 3, 4 and 5
(c) Replication (d) Empiricism 78. In the two sets given below, Set-I specifies the meth-
70. Which scientific method is a top-down or confirma- ods of research while Set-II describes the critical fea-
tory approach? ture associated with a method of research. Match the
two and select from the code to indicate your answer.

M02_MADAN 04_65901_C02.indd 47 23/12/22 7:15 PM


2.48 Chapter 2

84. Research study that takes place over a long period of


Set-I Set-II
time is termed as
(Research (critical features associated)
(a) Cross-sectional research
methods)
(b) Longitudinal research
(A) Experimental (i) Generalizing to the (c) Research methodology
method population from a large (d) Pure research
sample 85. The main difference between longitudinal and cross-
(B) Ex post facto (ii) Establishing the patterns sectional researches is in terms of
method of events which have (a) Frequency of data collection
happened, through primary (b) Primary versus secondary
and secondary sources (c) The qualification of the researcher
(d) None of the above
(C) Historical (iii) Studying shared
method behaviour patterns 86. Defining hypothesis is a useful way of approaching
research because
(D) Ethnographic (iv) Establishing cause and (a) It will impress the reader.
method effect through control (b) It allows the development of testable propositions.
(v) Probing into the casual (c) It allows for the development of indisputable
factors from the observed proof to be established in research findings.
effects (d) It looks suitably scientific.
87. The Government of India conducts census after
Codes:
every 10 years. The method of research used in this
(a) (A)–(iv), (B)–(v), (C)–(ii), (D)–(iii)
process is
(b) (A)–(i), (B)–(ii), (C)–(iii), (D)–(iv)
(a) Case study (b) Developmental
(c) (A)–(ii), (B)–(i), (C)–(iv), (D)–(iii)
(c) Survey (d) Experimental
(d) (A)–(ii), (B)–(iii), (C)–(iv), (D)–(v)
88. A nine-year-old is taller than seven-year-old ones.
79. A field experiment is one that takes place in This is an example of
(a) The real world (a) Vertical studies
(b) The laboratory (b) Cross-sectional studies
(c) Both in the real world and laboratory (c) Experimental studies
(d) Naturalistic environment (d) Case studies
80. Characteristics of the scientific method necessarily 89. The main difference between basic research and
include applied research lies in
A S S E S S YO U R L E A R N I N G

(a) Lab experiments only (a) Basic process (b) Sample size
(b) Controlled observation (c) Utility (d) All of the above
(c) Analysis formulation
90. Which type of method can be used in order to create
(d) All of the above
a real-world laboratory?
81. A correlational study determines (a) Correlational coefficients
(a) The relationship between independent and (b) Field experiment
dependent variable. (c) Case study
(b) Impact of the observer on the participant. (d) Random assignment
(c) Cause-and-effect relationship. 91. In a research study to learn the impact of the Inter-
(d) The relationship between two events. net surfing on exam performance, it was found that
82. A non-government organization conducted a study as the number of hours spent on Internet surfing
in a Gram Panchayat to see the impacts of campaign increases, exam performance deteriorates. This
approach on enrolment and retention of rural ele- study is an e­ xample of
mentary school children. This is an example of (a) Experimental method
(a) Descriptive study (b) Correlational research
(b) Field experiment (c) Case study
(c) Ex-post facto research (d) Descriptive Research
(d) Historical research 92. A researcher spent several years observing social
83. The classification of studies into exploratory, descrip- behaviour of people in their native habitat. The
tive, analytical, or predictive research is based on research method used here is
(a) Logic of the research (a) Case study
(b) Outcome of the research (b) Experimental method
(c) Process of the research (c) Correlational study
(d) Purpose of the research (d) Naturalistic observation

M02_MADAN 04_65901_C02.indd 48 23/12/22 7:15 PM


Research Aptitude 2.49

93. In a study of two variables, when one variable goes 98. In which of the following research studies, interpre-
up as another goes down in value is known as a tation and meaning get more attention than formu-
(a) Positive correlation lation of generalizations?
(b) No correlation 1. Historical studies
(c) Negative correlation 2. Survey studies
(d) Fluctuating correlation 3. Philosophical studies
94. Which of the following are the essential characteris- 4. Ethnographic studies
tics of the case study method? 5. Hypothetico–deductive studies
1. Negatively oriented 6. Ex post facto studies
2. Appreciation oriented Choose your answer from the options given below:
3. Particularistic
(a) 1, 2, and 3
4. Descriptive
5. Inductive (b) 4, 5, and 6
(a) 1, 2 and 3 (c) 2, 4, and 5
(b) 2, 3 and 4 (d) 1, 3, and 4
(c) 3, 4 and 5 99. Which of the following statements are correct in con-
(d) 1, 2 and 5 text of descriptive research?
95. Kindly look at the following question: (a) This describes the characteristics of the popula-
Why stressful living results in heart attacks? tion or phenomenon that is being studied.
In these types of questions, we need to look at rela- (b) This methodology focuses more on the “what” of
tionship between two aspects of a situation that are the research subject rather than the “why” of the
‘how’ and ‘why’. research subject.
Which kind of research helps in the above situation? (c) This should preferably be a qualitative research.
(a) Exploratory research (d) All of the above
(b) Descriptive study
100. Through which research method, the manipulation
(c) Ethnographic research
of an independent variable and its effect on depend-
(d) Explanatory Research
ent variable is examined with reference to a hypoth-
96. Look at the following statements in context of exper- esis under controlled conditions?
imental research? (a) Ex post facto research
1. The researchers randomly assign participants to (b) Descriptive research
groups. (c) Case study research
2. They provide control over extraneous variables (d) Experimental research
with an objective to isolate the effects of the inde-

A S S E S S YO U R L E A R N I N G
pendent variable on the results. 101. Commonly used in sciences such as sociology, psy-
3.  The treatment conditions may be physically chology, physics, chemistry, biology, and medicine,
manipulated. experimental research is a collection of research
4. There is statistical comparison of the groups. designs which make use of manipulation and con-
5. The experiment is designed to reduce the threats trolled testing in order to understand the
to internal validity and external validity. (a) Casual processes
(b) Non-causal process
Which ones of the above statements are applicable? (c) Only experimental group processes
(a) 1, 2, 3, and 4 (b) 2, 3, 4, and 5 (d) Only control group processes
(c) 1, 2, 3, and 4 (d) All of the above
97. In social sciences, it is a very difficult task to get the 102. Look at the following statements in context of exper-
most reliable results. Consider the following state- imental research?
ments in this context. 1. The researchers randomly assign participants to
1. The wording of questions groups.
2. The physical setting of interaction 2. They provide control over extraneous variables
3. The mood of respondent with an objective to isolate the effects of the inde-
4. The mood of the interviewer pendent variable on the results.
5. The nature of interaction 3.  The treatment conditions may be physically
6. The regression effect of an instrument manipulated.
Which of the above statements apply in the context? 4. There is statistical comparison of the groups.
(a) 1, 2, 3, 5, and 6 5.  The experiment is designed to reduce the
(b) 2, 3, and 5 threats to internal validity and external
(c) 1, 2, 3, 4, 5, and 6 validity.
(d) 1, 3, 4, and 5

M02_MADAN 04_65901_C02.indd 49 23/12/22 7:15 PM


2.50 Chapter 2

Which ones of the above statements are applicable? (a) Only 1 is true
(a) 1, 2, 3, and 4 (b) Only 2 is true
(b) 2, 3, 4, and 5 (c) Both 1 and 2 are true
(c) 1, 2, 3, and 4 (d) None of the above
(d) All of the above 108. Consider the following statements:
103. Consider the following statements: 1. This step is probably the most critical part of the
1. A patient’s belief that s/he is receiving treatment planning process.
can play an important role in her/his recovery 2. These stated research questions provide the basis
from an illness even if treatment is ineffective. for design and data analysis.
2. A placebo study involves two or three groups, 3. This step guides the researcher’s decision as to
depending on whether or not the researcher go for an experimental design or some other
wants to have a control group. orientation.
3.  If the researcher decides to have a control These three statements reflect the following.
group, the first group receives the treatment, (a) Literature survey
the second receives the placebo treatment and (b) Research design
the third – the control group – receives nothing. (c) Hypotheses
(d) Deciding about research topic
4. The decision about the group allocation can be
made through randomization. 109. Consider the following statements about an experi-
mental test.
Which of the above statements are right?
(a) 1, 2, and 3 (b) 2, 3, and 4 1. This type of design has two randomly assigned
(c) 1 and 3 (d) All of the above groups: an experimental group and a control group.
2. Neither group is pretested before the implemen-
104. The researcher manipulates two or more independ- tation of the treatment.
ent variables (factors) simultaneously to observe
their effects on the dependent variable. This design 3. The treatment is applied to the experimental group.
allows for the testing of two or more hypotheses in a Which kind of the test has been conducted in this case?
single project. This is called as (a) Pretest only design
(a) Randomized Block Design (b) Post-test only design
(b) Factorial Design (c) Pretest-Post-test only design
(d) One-group pret-est-post-test design
(c) Non-Randomized Design
110. Which of the following statements apply in case of a
(d) Quasi-experimental research
true experiment?
A S S E S S YO U R L E A R N I N G

105. Experimental research is used where (a) Control group and experimental group
(a) Time priority in a causal relationship (b) Researcher-manipulated variable
(b) consistency in a causal relationship (c) Random assignment
(c) magnitude of the correlation is great (d) All of the above
(d) All of the above 111. In quasi-experimental design is applied in situations
106. Consider the following statements: where
1. True experiments must have a control group. (a) the researcher has less control over the inde-
2. Control group is a group of research participants pendent variable than in the classical design.
that resemble the experimental group but do not (b) the classical design is much easier.
receive the experimental treatment. (c) both (a) and (b)
3. The control group provides a reliable baseline data (d) None of the above
to which you can compare the experimental results. 112. Consider the following statements:
Which of the above are the correct statements? 1. There are a large number of experimental groups.
(a) Only 1 and 2
2. This design is used when there are inherent dif-
(b) Only 2 and 3
ferences between subjects and possible differ-
(c) Only 1 and 3
ences in experimental conditions.
(d) All of the above
3. It is important to bring some homogeneity to
107. Consider the following statements:
each group.
1. The experimental group is the group of research
participants who receive the experimental Which type of design is being discussed in these
treatment. statements?
2. True experiments must have at least one control (a) Cross-over design
group and one experimental group, though it (b) Randomized block design
is possible to have more than one experimental (c) Factorial design
group. (d) Any type of block design

M02_MADAN 04_65901_C02.indd 50 23/12/22 7:15 PM


Research Aptitude 2.51

RESEARCH AS A Process
113. In the context of survey research, the following steps 121. Which of the following statements is ­correct?
are taken in certain order. Which of the following (a) Objectives should be pin-pointed.
options represent the correct order? (b) Objectives can be written in statement or ques-
1. Sampling tion forms.
2. Inference (c) Another word for problem is variable.
3. Data analysis (d) All of the above
4. Data collection 122. A satisfactory statistical quantitative method should
(a) 2, 3, 1, 4 (b) 1, 4, 3, 2 not possess which one of the following qualities?
(c) 3, 2, 4, 1 (d) 4, 1, 2, 3 (a) Appropriateness
114. Which of the following is not the requirement of a (b) Measurability
hypothesis? (c) Comparability
(a) Basis on facts. (d) Flexibility
(b) Being conceivable. 123. The accuracy of the research process depends
(c) Contradict the knowledge of nature. upon the
(d) Allow consequences to be deduced from it. (a) Unbiased attitude of the researchers
115. For a proposition to be true, it should have the (b) The sample size
following characteristics except which of the
­ (c) The research method adopted
­following? (d) All of the above
(a) It must be objective. 124. Who is regarded as the father of scientific social
(b) It must be in tune with accepted beliefs. surveys?
(c) It must be consistent. (a) Best (b) Booth
(d) It must be testable. (c) Darwin (d) Newton
116. The objective of a research can be written 125. A research plan
(a) Only in question form. (a) Should be detailed.
(b) Only in statement form. (b) Should be given to others for review and
(c) Both question and statement forms. comments.
(d) In hypothetical form. (c) Sets out the rationale for a research study.
117. sine qua non (essential element) of a good research is (d) All of the above
(a) A well-formulated hypothesis
126. Sources of researchable problems can include
(b) A good research supervisor
(a) Researchers’ own experiences as educators.

A S S E S S YO U R L E A R N I N G
(c) Adequate library
(b) Practical issues that require solutions.
(d) A well-formulated problem
(c) Theory and past research.
118. A good hypothesis should be (d) All of the above
(a) Precise, specific and consistent with most known
facts. 127. The introduction section of the research plan
(b) Formulated in such a way that it can be tested by (a) Gives an overview of prior relevant studies.
the data. (b) Contains a statement of the purpose of the study.
(c) Of limited scope and should not have global (c) Concludes with a statement of the research
significance. questions.
(d) All of the above (d) All of the above
119. Hypothesis cannot be stated in 128. A statement that predicts the cause-and-effect relation-
(a) Null and question form terms ship between variables is known as the
(b) Declarative terms (a) Null hypothesis
(c) General terms (b) Experimental hypothesis
(d) None of the above (c) Independent variable
(d) Dependent variable
120. Which of the following statements is true?
(a) In research, objectives can be worded in ques- 129. To be confident that a cause-and-effect relationship
tion form. exists, it is necessary to
(b) In research, objectives can be worded in state- (a) Engage in naturalistic observation.
ment form. (b) Develop a positive correlation.
(c) Objectives are to be stated in Chapter I of the (c) Perform a controlled experiment.
thesis. (d) Test for a negative correlation.
(d) All of the above

M02_MADAN 04_65901_C02.indd 51 23/12/22 7:15 PM


2.52 Chapter 2

130. Match List-I (interviews) with List-II (meaning) 136. In which of the following techniques of data col-
and select the correct answer from the code given lection, several participants, including facilitator,
below. emphazise on questioning a specific and defined
topic and interaction within the group and the joint
List-I List-II construction of meaning?
(Interviews) (Meanings) (a) Case study (b) Focus group
(A) Structured (i) Greater flexibility (c) Ethnography (d) Ideal Group
interviews approach 137. The process not needed in experimental research is
(a) Observation
(B) Unstructured (ii) Attention on
(b) Manipulation and replication
interviews questions to be
(c) Controlling
answered
(d) Reference collection
(C) Focused (iii) Individual life 138. Which type of study will be preferred by a researcher
interviews experience to estimate the degree of relationship between the
(D) Clinical (iv) Predetermined level of education and achievement motivation?
interviews question (a) Naturalistic (b) Inventory
(c) Correlational (d) Experimental
(v) Non-directive
139. Which scale is the simplest form of measurement?
Codes: (a) Nominal (b) Ordinal
(a) (A)–(iv), (B)–(i), (C)–(ii), (D)–(iii) (c) Interval (d) Ratio
(b) (A)–(ii), (B)–(iv), (C)–(i), (D)–(iii) 140. Which of the following is the correct order of
(c) (A)–(v), (B)–(iv), (C)–(iv), (D)–(i) Steven’s four levels of measurement?
(d) (A)–(i), (B)–(iii), (C)–(v), (D)–(iv) (a) Ordinal, nominal, ratio, and interval.
(b) Nominal, ordinal, interval, and ratio.
131. The first step of research is
(c) Interval, nominal, ordinal, and ratio.
(a) Selecting a problem
(d) Ratio, interval, nominal, and ordinal.
(b) Searching a problem
(c) Finding a problem 141. We use factor analysis
(d) Identifying a problem (a) To know the relationship between two variables.
(b) To test the hypothesis.
132. Which of the following statements is ­correct? (c) To know the difference between two variables.
(a) Objectives of research are stated in first chapter (d) To know the difference among many variables.
of the thesis.
142. The process not needed in experimental researches
(b) Researcher must possess analytical ability.
A S S E S S YO U R L E A R N I N G

is
(c) Variability is the source of problem.
(a) Observation (b) Manipulation
(d) All of the above
(c) Controlling (d) Content analysis
133. A research problem is feasible only when 143. Which of the following correlational values is the
(a) It has utility and relevance. strongest?
(b) It is researchable. (a) +0.10 (b) -0.95
(c) It is new and adds something to the knowledge. (c) +0.90 (d) -1.00
(d) All of the above
144. The correlation between intelligence test scores and
134. The first question that a researcher interested in the grades is
application of statistical techniques to his problem (a) Positive (b) Negative
has to ask is (c) Perfect (d) No correlation
(a) Whether the data could be quantified. 145. Which of the following figures reflect the highest
(b) Whether appropriate statistical techniques are value of Karl Pearson’s coefficient of correlation?
available. (a) +0.22 (b) +0.91
(c) Whether analysis of data would be possible. (c) -0.49 (d) -0.92
(d) Whether worthwhile inferences can be drawn.
146. A widely used format developed by Rensis Likert is
135. Which one of the following is a data collection
used for asking questions about
method?
(a) Attitude (b) Personality
(a) The opinion (b) Positivism
(c) Morale (d) Aptitude
(c) The case study (d) The interview

M02_MADAN 04_65901_C02.indd 52 23/12/22 7:15 PM


Research Aptitude 2.53

Research Characteristics
147. Objectivity in research implies (a) The incident is reported after a long period of
(a) Exact judgement of truth time from its occurrence.
(b) Findings consistent with reality (b) The author who is a source of information is
(c) Inter-researcher agreement biased and incompetent.
(d) Methodological sophistication (c) The researcher is not competent enough to draw
148. The extent to which an instrument measures a char- logical conclusions.
acteristic that cannot be directly observed but is (d) All of the above
assumed to exist is 154. The term used to describe when a research measures
(a) Face validity the variable or dimension it is supposed to measure is
(b) Construct validity (a) Validity (b) Reliability
(c) Criterion validity (c) Dependability (d) Suitability
(d) Content validity
155. Which of the following is not a characteristic of
149. Field study is related to research?
(a) Real-life situations (a) Research is systematic.
(b) Experimental situations (b) Research is not a process.
(c) Laboratory situations (c) Research is problem oriented.
(d) Subconscious reality (d) Research is not passive.
150. The verification that the method of measurement 156. Which of the following terms is closely related to
actually measures what it is expected to measure is generalization of outcome of research?
known as (a) External validity
(a) Content validity (b) Inference
(b) Face validity (c) Both (a) and (b)
(c) Construct validity (d) This can’t be ascertained
(d) Criterion validity
157. A research instrument giving inconsistent results has
151. The extent to which the results of a research study (a) Low validity (b) High validity
apply to situations beyond the study itself and the (c) Low reliability (d) High reliability
extent to which conclusions can be generalized is
(a) External validity 158. Authenticity of research finding is its
(b) Internal validity (a) Originality
(b) Validity
(c) Situation validity
(c) Objectivity

A S S E S S YO U R L E A R N I N G
(d) Unidentified validity (d) All of the above
152. The extent to which the design and data of a research 159. How can the objectivity of research be enhanced?
study allows the researcher to draw accurate conclu- (a) Through impartiality
sions about cause-and-effect and other relationships (b) Through reliability
within the data is (c) Through validity
(a) External validity (d) All of the above
(b) Internal validity
160. Manipulation is always a part of [December 2007]
(c) Situation validity (a) Historical research
(d) Unidentified validity (b) Fundamental research
153. The validity and reliability of a research will be at (c) Descriptive research
stake when (d) Experimental research

Formulation of Hypothesis
161. An educated guess about what is controlling some (b) While planning your research study.
behaviour is called (c) At the time of publishing the results.
(a) Experimental control (d) None of the above
(b) A hypothesis 163. To test null hypothesis, a researcher uses
(c) An experimental variable (a) t-test
(d) A theory (b) ANOVA
162. It is best to use the method of working multiple (c) c 2 (Chi-square test)
hypothesis (d) Factorial analysis
(a) During the final stages of research.

M02_MADAN 04_65901_C02.indd 53 23/12/22 7:15 PM


2.54 Chapter 2

164. Hypothesis cannot be stated in (c) Of limited scope and should not have global
(a) Declarative terms significance.
(b) Null hypothesis and question form terms (d) All of the above
(c) General terms 173. Statement I: A research hypothesis is a tentative
(d) Directional terms statement postulating a relationship between fac-
165. What type of research would be least likely to include tual and conceptual elements of the variables.
a research hypothesis? Statement II: A researcher sets up a null hypothesis
(a) Intervention research so that deduced consequences of a research hypoth-
(b) Associational research esis may be directly tested.
(c) Descriptive research
In light of the above statements, choose the correct
(d) Experimental research
answer from the options given below.
166. The basis on which the assumptions are formulated is (a) Both Statement I and II are true.
(a) Cultural background of the country (b) Both Statement I and II are false.
(b) Universities (c) Statement I is correct but Statement II is false.
(c) Specific characteristics of the castes (d) Statement I is incorrect but Statement II is
(d) All of the above true.
167. A null hypothesis is 174. Formulation of hypothesis may not be required in
(a) When there is no difference between the variables. (a) Survey method
(b) The same as research hypothesis. (b) Historical studies
(c) Subjective in nature. (c) Experimental studies
(d) When there is difference between the variables. (d) Normative studies
168. Research hypothesis are 175. An operational definition is
(a) A review of current research. (a) No relation to the underlying concept.
(b) Statements of predicted relationships between (b) An abstract, theoretical definition of a concept.
variables. (c) In terms of specific and empirical measures.
(c) Stated such that they can be confirmed or refuted. (d) None of the above
(d) Both (b) and (c)
176. What is the purpose of the conclusion in a research
169. Which of the following best describes the develop- report?
ment process for a research ­question? (a) It explains how concepts were operationally
(a) A broad question is made more specific as terms defined and measured.
are more clearly defined. (b) It summarizes the key findings in relation to the
(b) A broad question is made more specific in order research questions.
A S S E S S YO U R L E A R N I N G

to be more significant. (c) It contains a useful review of the relevant


(c) A specific question is broadened as terms are literature.
more clearly defined. (d) It outlines the methodological procedures that
(d) A specific question is broadened in order to be were employed.
more significant. 177. Which of the following is an advantage of stating
170. Good research questions are hypothesis?
(a) Clear, significant, and ethical. (a) It forces the researcher to think more deeply and
(b) Feasible, clear, significant, and ethical. specifically about the possible outcomes of a study.
(c) Feasible, clear, significant, and include a (b) It simplifies the study.
hypothesis. (c) It clarifies definitions.
(d) Feasible, clear, and ethical. (d) It reduces researcher bias.
178. Which of the following is an example of a directional
171 The essential characteristic of a researchable ques-
hypothesis?
tion is
(a) There will be a difference between the students’
(a) Question seems interesting to answer.
reading levels.
(b) Possibility of data collection that can be col- (b) There will be a difference between lecture and
lected in an attempt to answer a question. group instruction.
(c) Possibility of commercialization. (c) Group instruction is more effective than lecture
(d) A significant positive change in society. in the elementary classroom.
172. A good hypothesis should be (d) There will be an increase in learning.
(a) Formulated in such a way that it can be tested by 179. Formulation of hypothesis may not be necessary in
the data. (a) Survey studies
(b) Precise, specific and consistent with most known (b) Fact finding historical research
facts. (c) Experimental studies
(d) Quantitative Research

M02_MADAN 04_65901_C02.indd 54 23/12/22 7:15 PM


Research Aptitude 2.55

180. Which of the following is true in context of a 181. In mixed methods research, quantitative and qualita-
hypothesis? tive findings should be
(a) It is a tentative proposition. (a) Listed in the order of importance.
(b) The validity of hypothesis is unknown. (b) Contained in separate sections.
(c) It must be generalizable. (c) Integrated
(d) All of the above (d) Shown fully in appendices.

Research Variables
182. Attributes of objects, events or things which can be (c) Both (a) and (b)
measured are called (d) Neither (a) nor (b)
(a) Qualitative measure (b) Data 191. An example of a categorical variable is
(c) Variables (d) None of the above (a) Teacher’s hair colour.
183. A statistical technique used for large number of (b) Average time it takes a teacher to grade an
variables to establish whether there is a tendency of essay.
groups to be interrelated is (c) Distance a teacher has to travel from the office to
(a) Simple correlation her class.
(b) Multiple correlation (d) Time taken by a sports person to complete a circle
(c) Factor analysis
192. Which variables are those that a researcher chooses
(d) None of the above
to study in order to assess their possible effects on
184. When studying an active independent variable, an one or more other variables?
intervention or treatment given to group of partici- (a) Dependent
pants is called (b) Independent
(a) Experimental group (b) Control group (c) Extraneous
(c) Both (a) and (b) (d) Neither (a) or (b) (d) Confounding variable
185. Which of the following can best be described as a 193. A condition or characteristic that can take on differ-
categorical variable? ent values or categories is called
(a) Age (a) A constant
(b) Annual income (b) A variable
(c) Grade point average (c) A cause-and-effect relationship
(d) Religion

A S S E S S YO U R L E A R N I N G
(d) A descriptive relationship
186. Experimental studies are based on
(a) The manipulation of the variables 194. Which of the following includes examples of quanti-
(b) Conceptual parameters tative variables?
(c) Replication of research (a) Age, temperature, income and height
(d) Survey of literature (b) Grade point average, anxiety level and reading
performance
187. A manipulated independent variable is called? (c) Gender, religion and ethnic group
(a) Extraneous variable (d) Both (a) and (b)
(b) Intervening variable
(c) Subject variable 195. In which scale of measurement, classification, order
(d) Active variable and equality of units are ensured? [June 2020]
188. Control groups and experimental groups are exactly (a) Ordinal
the same except for (b) Nominal
(a) Dependent variable (c) Interval
(b) Independent variable (d) Ratio
(c) Extraneous variables 196. We use factorial analysis
(d) Replication variables (a) To know the relationship between two variables.
189. The behaviour that is measured during an experi- (b) To test the hypothesis.
ment is known as the (c) To know the difference between two variables.
(a) Confounding variable (d) To know the difference among many variables.
(b) Independent variable 197. A variable that is presumed to cause a change in
(c) Dependent variable another variable is called
(d) Control variable (a) A categorical variable
190. This type of longitudinal research studies the same (b) A dependent variable
individuals over an extended period of time. (c) An independent variable
(a) Trend study (d) An intervening variable
(b) Panel study

M02_MADAN 04_65901_C02.indd 55 23/12/22 7:15 PM


2.56 Chapter 2

198. Which of the following independent variables can- 203. An experiment is performed to test the effects of
not be manipulated in a research study? sleep deprivation on rote memory. In this experi-
(a) Gender ment, the dependent variable is
(b) Ethnicity (a) Number of hours subjects go without sleep.
(c) Intelligence and other traits (b) Rote memory scores.
(d) None of the above (c) Number of subjects deprived of sleep in the
199. In an experimental design, the dependent variable is experimental group.
the one (d) Correlation between hours of sleep and fatigue.
(a) In which any changes are observed 204. The variable manipulated by the researcher in an
(b) Which is not manipulated experiment is called the
(c) Both (a) and (b) (a) Response variable
(d) None of the above (b) Independent variable
200. This variable depends upon what is done to it by the (c) Dependent variable
independent variable. (d) Extraneous variable
(a) Extraneous (b) Dependent 205. Which of the following would not be an appropriate
(c) Manipulated (d) All of the above synonym for the dependent variable?
201. Extraneous variables are essentially (a) Outcome variable
(a) Independent variables (b) Response variable
(b) Dependent variables (c) Effected variable
(c) Independent or dependent (d) Experimental variable
(d) Neither independent nor dependent 206. An example of quantitative variable is the
202. These variables are those that are created by the (a) Date of birth
researcher and are typically found in experimen- (b) Highest educational qualification
tal studies. (c) Time taken to complete a task
(a) Extraneous (b) Manipulated (d) Postal code
(c) Dependent (d) None of the above

Sampling
A S S E S S YO U R L E A R N I N G

207. Generalized conclusion on the basis of a sample is (b) Stratified sampling


technically known as (c) Purposive sampling
(a) Data analysis and interpretation (d) Convenient sampling
(b) Parameter inference 212. The type of sampling where each person in popula-
(c) Statistical inference tion has equal chance of being selected is
(d) All of the above (a) Probability sampling
208. The process of selecting a subset of a population for (b) Non-probability sampling
a survey is known as (c) Judgement sampling
(a) Survey research (b) Representation (d) Convenience sampling
(c) Triangulation (d) Sampling 213. Here, some people have greater chance of being
209. Researchers ultimately want the answer to a research elected than other members of the population. It is
question to pertain to the (a) Probability sampling
(a) Sample (b) Non-probability sampling
(b) Accessible population (c) Quota sampling
(c) Target population (d) None of the above
(d) World 214. Which of the following variables cannot be expressed
in quantitative terms?
210. When a research problem is related to heterogene-
(a) Socio-economic status
ous population, the most suitable method is
(b) Marital status
(a) Cluster sampling (b) Stratified sampling
(c) Numerical aptitude
(c) Convenient sampling (d) Lottery method
(d) Professional attitude
211. An investigator wants to study the vocational aspira-
215. A representative sample is essential in
tions of visually challenged children in a wide geo-
(a) Survey method (b) Experimental method
graphical area. He should select his sample by using
(c) Case study (d) Clinical method
(a) Simple random sampling

M02_MADAN 04_65901_C02.indd 56 23/12/22 7:15 PM


Research Aptitude 2.57

216. Which one is known as non-probability sampling? (a) Simple random sampling
(a) Cluster sampling (b) Stratified random sampling
(b) Quota sampling (c) Cluster random sampling
(c) Systematic sampling (d) None of the above
(d) Stratified random sampling 227. Which of the following terms best describes data
217. While the statistical measure based upon entire pop- that were originally collected at an earlier time by a
ulation is called parameter, the measure based upon different person for a different purpose?
a sample is known as (a) Primary data
(a) Sample parameter (b) Inference (b) Secondary data
(c) Statistic (d) None of the above (c) Experimental data
218. A researcher selects a probability sample of 100 out (d) None of the above
of the total population. It is called 228. From the following identify those which are called
(a) A quota sample non-probability sampling procedures:
(b) A simple random sample (A) Systematic sampling
(c) A stratified random sample (B) Quota sampling
(d) A systematic sample (C) Starified sampling
219. A researcher divides the school students on the basis (D) Purposive sampling
of gender and then by using the random digit table, (E) Snowball sampling
he selects some of them from each group. This pro- Choose the correct answer from the options given below:
cess is called (a) (A), (B) and (C) only
(a) Stratified sampling (b) (B), (C) and (D) only
(b) Stratified random sampling (c) (B), (D) and (E) only
(c) Representative sampling (d) (C), (D) and (E) only
(d) None of the above 229. Which of the following is an example of a random
220. To ensure accuracy of a research, the sample should be sampling method?
(a) Taken randomly (a) Systematic sampling
(b) Fixed by quota (b) Convenience sampling
(c) Representative of the population (c) Purposive sampling
(d) Purposive (d) None of the above
221. A researcher can keep the sample size low if the popu- 230. Which of the following is not an example of a ran-
lation is dom sampling method?
(a) Heterogeneous (b) Inaccessible (a) Systematic sampling

A S S E S S YO U R L E A R N I N G
(c) Homogeneous (d) All of the above (b) Stratified random sampling
222. Which technique is generally followed when the (c) Simple random sampling
population is finite? (d) All of the above
(a) Area sampling technique 231. Which of the following is an example of a random
(b) Purposive sampling technique sampling method?
(c) Systematic sampling technique (a) Two-stage random sampling
(d) None of the above (b) Systematic sampling
223. Cluster sampling is used when (c) Convenience sampling
(a) Population is scattered and sample size is to be (d) Purposive sampling
kept large. 232. Which of the following is an example of a non-ran-
(b) Population is heterogeneous. dom sampling method?
(c) Long survey is needed. (a) Convenience sampling
(d) Both (a) and (c) (b) Stratified random sampling
224. A researcher divides his population into certain (c) Simple random
groups and fixes the size of the sample from each (d) Cluster random
group. This is called 233. The purpose of stratified random sampling is to
(a) Stratified sample (b) Quota sample make certain that
(c) Cluster sample (d) All of the above (a) Every member of the population has an equal
225. Which of the following is a non-probability sample? chance of being selected.
(a) Quota sample (b) Simple random sample (b) For proportionate representation from different
(c) Purposive sample (d) Both (a) and (c) categories.
(c) Prompt response from respondents.
226. If a researcher selected five schools at r­ andom and (d) None of the above
then interviewed each of the teachers in those five
schools, the researcher used

M02_MADAN 04_65901_C02.indd 57 23/12/22 7:15 PM


2.58 Chapter 2

234. A correlation coefficient is best characterized as 240. The standard error is an estimate of the standard
(a) A measure of the extent of the relationship deviation of a statistic. The standard error is a statis-
between two variables. tical measure of
(b) An index of the causal direction between an (a) The normal distribution of sample scores around
independent and dependent variable. the sample mean.
(c) An indication of the likelihood that an experi- (b) The extent to which a sample mean differs from
mental finding will be replicated by others. the population mean.
(d) A measure of the likelihood that observed differ- (c) Clustering of scores at each end of a survey scale.
ences may be attributed to chance. (d) Checking the accurate stratification of the
235. Responding to a substance like a sugar pill as if it was sample.
a drug is called 241. Which of the following statements refer to ‘data pro-
(a) The placebo effect cessing error’?
(b) An extraneous factor (a) Events related to the non response of sampling
(c) Variability process.
(d) None of the above (b) Error in faulty application of techniques of cod-
236. What is a cross-sectional design? ing and managing data.
(a) A study of one specific segment of customers. (c) Issues related with the implementation of the
(b) The research design that is free from any per- whole research process.
sonal bias. (d) The unavoidable discrepancy between the sam-
(c) The collection of data only once from different ple and the population.
respondents that belong to an institution. 242. In a research, the total population is small. The easy
(d) A comparison of two or more variables over a method of sampling is to number each element
long period of time. using separate slips of paper for each element. All
237. Consider the following two statements. the slips are put in a box. The researcher picks them
1. Categorical variables are measured on nominal out one by one without looking, until the number of
or ordinal measurement scales. slips selected equals the desired sample size. This
2. Continuous variables are measured on either an technique can termed as
interval or a ratio scale. (a) Snowball sampling method
3. Both categorical and continuous variables can be (b) Fishbowl sampling method
measured against any scale as per situation. (c) Stratified sampling method
(d) Cluster sampling method
Which of the above statements are correct?
(a) Only 1 and 2 243. A shift in response attitude in respondents between
two points during data collection is called
A S S E S S YO U R L E A R N I N G

(b) Only 2 and 3


(c) Only 1 and 3 (a) Reactive effect
(d) All of the above (b) Maturation effect
238. Which of the following statements refer to a sam- (c) Regression effect
pling frame? (d) Conditioning effect
(a) A comprehensive coverage of the various stages 244. From the list given below identify those which are
involved in designing a survey. called ‘Non-probability sampling procedures’:
(b) A summary view of all the main clusters of units 1. Simple random sampling
in a sample. 2. Dimensional sampling
(c) A list of all the units in the population from 3. Snowball sampling
which a sample will be selected.
4. Cluster sampling
(d) A table frame used to display tables of random
numbers. 5. Quota sampling
6. Stratified sampling
239. The multi-stage cluster sample is most helpful when
(a) There is wide geographical dispersion of Choose the correct option
population. (a) 1, 2, and 3
(b) The researcher has time and money constraints. (b) 2, 4, and 5
(c) The researcher would use a probability sample (c) 1, 3, and 6
in order to generalize the results. (d) 2, 3, and 5
(d) All of the above.

M02_MADAN 04_65901_C02.indd 58 23/12/22 7:15 PM


Research Aptitude 2.59

Research Ethics/Misc
245. Research is not considered ethical if it  (b) To sell the data at a later stage to recover the cost
 [December 2011] of research.
(a) Tries to prove a particular point. (c) To keep secrecy from other researchers.
(b) Does not ensure privacy and anonymity of the (d) None of the above
respondent. 254. The importance of measurement in quantitative
(c) Does not investigate the data scientifically. research is that
(d) Is not of a very high standard. (a) It allows us to delineate fine differences between
246. Ethical transgression is people or cases.
(a) Following ethical principles (b) It provides a consistent device or yardstick.
(b) Defying ethical principles (c) It allows for precise estimates of the degree of
(c) Defining ethics ­relationship between concepts.
(d) None of the above (d) All of the above
247. Which of the following ideas is not associated with 255. One of the preoccupations of quantitative research-
the stance of situation ethics? ers is with generalization, which is a sign of
(a) Anything goes (a) External validity (b) Internal reliability
(b) The end justifies the means (c) External reliability (d) Internal validity
(c) No choice 256. Population generalizability refers to
(d) All of the above (a) Conclusions researchers make about a random
sample.
248. Which of the following is a form of harm that might be
(b) Conclusions researchers make about informa-
suffered by research participants?
tion uncovered in research study.
(a) Physical injury
(c) The degree to which a sample represents the
(b) Stress and anxiety
­population of interest.
(c) Impaired development
(d) The degree to which results of a study can be
(d) All of the above
extended to other settings or conditions.
249. The main purpose of personal data being kept confi-
257. The degree to which results of a study can be
dential in research studies is
extended to other settings or conditions describes
(a) To hide from participants what has been written
(a) Population generalizability
about them.
(b) Conclusions researchers make about a random
(b) Apprehension of harming identification or dis-
sample.

#ntaugc-netimportantquestions
closure of personal information.
(c) Conclusions researchers make about information
(c) For access by government authorities.
uncovered in research study.
(d) To know more about private lives of participants.
(d) Ecological generalizability
250. Which method is most directly associated with lack
258. The research design is
of informed consent?
(a) A common method adopted by all researchers to
(a) In-depth interviewing (b) Content analysis
carry out research.
(c) Covert observation (d) Case study
(b) The final choice between using qualitative or
251. Which of the following is a form of harm that might quantitative methods.
be suffered by research participants? (c) Presentation of research findings.
(a) Psychological distress (d) A framework for every stage of the data collec-
(b) Physical injury tion and its analysis.
(c) Loss of self-esteem
259. Recognizing our own values in pursuit of research is
(d) All of the above
essential because
252. The act of publishing the same data and results in (a) It allows us to eliminate them.
more than one journal or p ­ ublication refers to which (b) It allows us to be honest with ourselves about
of the following professional issues? what may influence our research.
(a) Partial publication (c) At least we know what we can ignore.
(b) Duplicate publication (d) It enables us to persuade others of our values.
(c) Full publication
260. As the value of one variable is increasing, the value
(d) None of the above
of second variable is also increasing, then the
253. Why is it important that personal data about ­correlation coefficient will be
research participants be kept within secure confi- (a) Positive
dential records? (b) Negative
(a) To observe principle of research ethics. (c) Zero
(d) None of the above

M02_MADAN 04_65901_C02.indd 59 23/12/22 7:15 PM


2.60 Chapter 2

261. It is important that academics produce research that 270. Identify the correct sequence of steps in the research
can be widely used and valued by academic commu- process as identified by David Nachmias:
nity around the world. This is termed as A. Research design
(a) Research impact B. Data collection
(b) Citation impact C. Formulation of hypothesis
(c) Ethics impact D.Data analysis and generalisation
(d) None of the above E. Problem identification
262. The problem of researcher seeing only what they
Choose the most appropriate answer from the options
expect to see is called
given below:
(a) Researcher bias (b) Experimenter effect
(c) Leniency effect (d) Halo effect (a) A, B, C, D, E
(b) B, C, D, E, A
263. Which of the following is most appropriate to impart
(c) C, D, E, A, B
training for SPSS, a software package used for statis-
(d) E, C, A, B, D
tical analysis?
(a) Seminar (b) Conference 271. Match the following statements in context of ‘quanti-
(c) Workshop (d) Paper tative approach’ in research.
264. Which of the following is essentially an academic
meeting? Research
(a) Colloquium (b) Conference approach Basic Points
(c) Symposium (d) None of the above 1. Inferential A. operation of a
265. In which of the following is it required to cite sources approach numerical model that
in a bibliography at the end? represents the structure
(a) Conference (b) Colloquium of a dynamic process.
(c) Seminar (d) Paper 2. Experimental B. greater control is
266. Which of the following is most likely to make use of approach exercised over research
the Internet? environment through
(a) Web Conference (b) Seminar manipulations.
(c) Symposium (d) None of the above
3. Simulation C. database is
267. Which of the following is published in a journal? approach established through
(a) Article (b) Paper survey method and
(c) Both (a) and (b) (d) None of the above conclusion is drawn
#ntaugc-netimportantquestions

268. A researcher conducted three experiments with 100 about characteristics


subjects each following uniform design instead of or relationship of
one experiment with 300 subjects. This is known as variables.
(a) Replication (b) Manipulation
(c) Observation (d) Validation Codes:
269. In which of the following researches, attempts are (a) 1–C, 2–A, 3–B
made to establish context-free generalizations? (b) 1–B, 2–C, 3–A
(a) Phenomenology (c) 1–C, 2–B, 3–A
(b) Grounded theory (d) 1–A, 2–B, 3–C
(c) Symbolic interactionism
(d) Experimental Studies

M02_MADAN 04_65901_C02.indd 60 23/12/22 7:15 PM


Research Aptitude 2.61

Answer Keys
Basic Concepts
1. (a)  2. (d)  3. (c)   4. (d) 5. (a)  6. (c)  7. (d)  8. (c) 9. (d) 10. (b)
11. (d) 12. (b) 13. (c) 14. (b) 15. (c) 16. (b) 17. (c) 18. (a) 19. (b) 20. (d)
21. (a) 22. (d) 23. (d) 24. (c) 25. (c) 26. (b) 27. (a) 28. (a) 29. (c) 30. (a)
Types of Research
31. (d) 32. (b) 33. (a) 34. (a) 35. (a) 36. (c) 37. (b) 38. (c) 39. (c) 40. (a)
41.  (d) 42. (d) 43. (b) 44. (d) 45.  (b) 46. (b) 47. (a) 48. (c) 49. (b) 50. (b)
51. (a) 52. (b) 53. (c) 54. (a) 55. (a) 56. (d) 57. (b) 58. (a) 59. (b) 60. (b)
61. (d) 62. (a) 63. (d) 64. (b) 65. (d) 66. (b) 67. (a) 68. (d) 69. (c) 70. (a)
71. (b) 72. (b) 73. (d) 74. (a) 75. (a) 76. (c) 77. (d) 78. (a) 79. (a) 80. (b)
81. (d) 82. (b) 83. (d) 84. (b) 85. (a) 86. (b) 87. (c) 88. (b) 89. (c) 90. (b)
91. (b) 92. (d) 93. (c) 94. (c) 95. (d) 96. (d) 97. (c) 98. (d) 99. (d) 100. (d)
101. (a) 102. (d) 103. (d) 104. (b) 105. (d) 106. (d) 107. (c) 108. (c) 109. (b) 110. (d)
111. (c) 112. (b)
Research Process
113. (b) 114. (c) 115. (b) 116. (b) 117. (a) 118. (d) 119. (c) 120. (b) 121. (a) 122. (d)
123. (d) 124. (b) 125. (d) 126. (d) 127. (d) 128. (b) 129. (c) 130. (a) 131. (c) 132. (d)
133. (d) 134. (a) 135. (d) 136. (b) 137. (d) 138. (c) 139. (a) 140. (b) 141. (d) 142. (d)
143. (d) 144. (a) 145. (d) 146. (a)
Research Characteristics
147. (b) 148. (b) 149. (a) 150. (a) 151. (a) 152. (b) 153. (d) 154. (a) 155. (b) 156. (c)
157. (c) 158. (d) 159. (d) 160. (d)
Formulation of Hypothesis
161. (b) 162. (b) 163. (c) 164. (c) 165. (c) 166. (d) 167. (a) 168. (d) 169. (a) 170. (b)

#ntaugc-netimportantquestions
171. (b) 172. (a) 173. (d) 174. (b) 175. (c) 176. (b) 177. (a) 178. (c) 179. (b) 180. (d)
181. (c)
Research Variables
182. (c) 183. (c) 184. (a) 185. (d) 186. (a) 187. (d) 188. (b) 189. (c) 190. (c) 191. (a)
192. (b) 193. (b) 194. (d) 195. (c) 196. (d) 197. (c) 198. (d) 199. (c) 200. (b) 201. (a)
202. (b) 203. (b) 204. (b) 205. (d) 206. (c)
Sampling
207. (c) 208. (d) 209. (c) 210. (b) 211. (b) 212. (a) 213. (b) 214. (d) 215. (a) 216. (b)
217. (c) 218. (b) 219. (b) 220. (c) 221. (c) 222. (c) 223. (d) 224. (b) 225. (d) 226. (c)
227. (b) 228. (d) 229. (a) 260. (a) 231. (a) 232. (a) 233. (b) 234. (a) 235. (a) 236. (c)
237. (a) 238. (c) 239. (d) 240. (b) 241. (b) 242. (b) 243. (c) 244. (d)
Research Ethics/Misc
245. (b) 246. (b) 247. (d) 248. (d) 249. (b) 250. (c) 251. (d) 252. (b) 253. (a) 254. (d)
255. (a) 256. (c) 257. (d) 258. (d) 259. (b) 260. (a) 261. (b) 262. (a) 263. (c) 264. (a)
265. (d) 266. (a) 267. (c) 268. (d) 269. (d) 270. (d) 271. (c)

M02_MADAN 04_65901_C02.indd 61 23/12/22 7:15 PM


This page is intentionally left blank

M02_MADAN 04_65901_C02.indd 62 23/12/22 7:15 PM


CHAPTer

3 Comprehension

01 About Comprehension Passages

02 Solution Guidelines

03 Main Question Categories


LeArnInG
OBJeCTIVeS

04 Five Specimen Passages

05 Sixteen Practice Passages

M03_MADAN 04_65901_C03.indd 1 21/12/22 10:52 AM


3.2 Chapter 3

Comprehension Passage Main Question Categories


According to Wren and Martin, a comprehension ­exercise Comprehension passage questions can be classified into
can be defined as a passage upon which questions are the following categories:
set to test the students’ ability to understand the content
of the given text and to infer information and meanings Main T heme or Central Idea
from it. To put it in simple terms, reading comprehension
These type of questions are based on the passage as a
is the act of understanding what you are reading.
whole and are typical in nature as they reflect the author’s
It mainly requires two inputs from the reader, that
motive behind writing the passage.
is, vocabulary knowledge and text comprehension.
These questions will put someone to test by scrutiniz-
In order to understand a text, the reader must be able to
ing the ability of the reader in understanding the given
comprehend the vocabulary used in a piece of writing. A
paragraph.
comprehension test requires specific reading skills, the
ability to grasp the main ideas from the passage, link- Solution Approach
ing them, inferring, and drawing conclusions based on
Usually, the answers to such questions lie in the introduc-
a proper understanding of the passage. Eventually, by
tory or the concluding paragraphs. A quick reading of the
gaining a clear understanding, a candidate is supposed to
first or last or both paragraphs is helpful.
answer the questions given at the end of the passage.
For candidates, even though they have ample choice Examples:
amongst questions, reading comprehension is a must
1. Which of the following alternatives best narrate the
attempt as it does not require studying any specific topic
passage?
and some practice can help in answering the questions
2. Which of the following sentences reflect upon the
with confidence. Usually, each reading comprehension is
main idea of the passage?
followed by five to six questions in UGC Paper I Exam.
Paying attention to single words and phrases helps to
Solution Guidelines understand the relationship between the ideas within a
Here are some techniques that can be used to make com- paragraph.
prehension easier: 1. Cause and effect words—as a result, therefore
1. Identification of Key Ideas: There are always certain 2. Time words—meanwhile, before
key sentences that determine the flow of the ideas in 3. Contrast words—in contrast, conversely
a passage. To make comprehension simpler, these sen- 4. Addition words—also, in addition
tences are supposed to be identified during the first 5. Emphasis words—more important, remember
reading of the passage. The paragraph(s) should be
read quickly to get a general idea. It should be read Author’s O pinion and Attitude
again, a little slowly, to know the details. The key ideas Such questions are based on the author’s viewpoint, and
can also be underlined. the answers are not explicitly m
­ entioned. They have to be
2. Summarize the Paragraph: While reading the pas- derived from the key sentences that the author has used
sage, develop a habit of mentally summarizing each to express his thoughts and opinions. The questions are
paragraph and keep linking them as you proceed. framed to test one’s ability to judge the author’s attitude
This helps in understanding the overall idea of the or his knowledge on the subject by analyzing the content,
passage and, as a result, the candidate may find it eas- style, and phraseology used in the passage. The candidate
ier to answer the questions. Some readers may prefer needs to be empathetic with the author while answering
to study the questions again and return to the relevant such questions. Such a set of questions examines the can-
portions of the passage later. didate on many fronts.
3. Determine Sentence Links: Usually, the sentences
within a passage are directly or subtly linked. For this,
Solution Approach
practice helps a lot in identifying such links. However,
failing to practice would mean missing out the most One can look for the writing style and the sentences that
important points. determine the tone used by the author, which serve as a
4. Ask Questions: Whenever in doubt, the reader clue to the answers. If a particular part of the passage is in
should always ask questions to oneself. For ­example, question form, then try to understand the reasoning used
the following questions can be asked: why has the by the author to explain that part.
author mentioned this example? What is the purpose Alternatively, the reader could focus on the adverbs
of the second paragraph? This kind of reflection helps and adjectives used by the author to describe something.
in developing a deeper perspective about the main These words are indicative of the mood and intensity of
ideas. his thoughts.

M03_MADAN 04_65901_C03.indd 2 21/12/22 10:52 AM


Comprehension 3.3

Examples: These questions test our ability to move simple, known


1. What is the author’s tone in the second paragraph? words and their literal meanings to understand the lan-
2. According to the author, what can be a logical solu- guage usage and the subject matter of the passage. They
tion to the issue at hand? test our reasoning skills and the ability to relate to the
3. Which of the following ideas is the author most likely author’s ideas while reading the passage.
to agree with?
Solution Approach
Explicit or Direct Information A thorough second reading of that part of the passage
These are direct questions based on the names, dates, from where a phrase has been picked up helps us in cor-
figures, data, facts, or opinions mentioned in the pas- rectly understanding the context of the passage.
sage and are easily noticed during the first read itself. The
Examples:
answer options may also be direct sentences picked up
from the passage. Scan the passage to spot the keywords 1. In the context of this paragraph, what does, as good
for specific details. as it gets, mean?
Some specific detail questions are negative and are 2. From the options, find the word closest in meaning to
singled out by words such as not, except, most, and the given word as it has been used in the passage.
least. If you cannot find the correct answer for negative-
specific details, eliminate the choices given and choose Organization of the P assage
the one that remains as the correct response. It also These questions are based on the structure of the passage and
makes a lot of sense to read the questions first as it gives the literary techniques used by the author in expressing his
an idea of what to look for while reading the passage. arguments. Questions dealing with strengthening and weak-
Questions based on explicit information should be veri- ening arguments fall in this category.
fied by reading specific parts of the passage that contain These questions test our ability to analyze the structure
the answer. of the passage and identify sentences and paragraphs as
assumptions, arguments, solutions, conclusions, etc.
Examples:
1. Which one of the following options is true according Solution Approach
to the passage? A lot of reading can help a reader to acquire literary skills.
2. Which one of the following options is no longer a
motivation for youth today? Examples:
1. Which of these options best represent the structure of
I mplicit I nformation the passage?
The answers to these questions are not explicitly stated, 2. What is the fundamental premise on which the author
but are implied, and they are inferred/deducted from the has based his analysis?
passage. 3. What is the assumption made by the author when he
These questions test both our comprehension skills and says democracy will never fall?
our ability to relate to the author’s opinion.
Solution Approach
Analogous A rgument
A candidate is required to choose the option that is similar
Understanding the overall idea of the passage helps to
to or conforms to reasoning along the same lines as the
answer these questions. While reading the passage, the
arguments mentioned in the passage.
candidate should try to understand the arguments pre-
This question tests our comprehension and our ability
sented by the author.
to illustrate the analysis formed in our mind based on our
Examples: reading of the passage. It also tests our capacity to associ-
1. According to the passage, which of the following ate two separate ­illustrations that follow a single line of
options can be inferred? reasoning.
2. With which of the following arguments is the author
most likely to agree? Solution Approach
The key to answering such questions is to first under-
Language Expression Questions stand the basic arguments before looking at the options
These questions are based on specific words or phrases provided. Then, eliminate the options by comparing
mentioned in the passage. However, sometimes, there them to the line of reasoning in the ­question stem.
may be synonym- or antonym-based questions, which
test our understanding of a word in the context of the Examples:
passage. The aim of the reader should be to understand 1. Which of the following illustrations best represent the
the contextual meanings of the words with respect to arguments mentioned in the second paragraph?
the passage, rather than sticking to their dictionary 2. The phrase, to err is human, is applicable in which
meanings. one of the situations listed below?

M03_MADAN 04_65901_C03.indd 3 21/12/22 10:52 AM


3.4 Chapter 3

4. All of the following statements are true about boiling,


Specimen Passage 1 except which of the following?
The sugar maple is a hard maple tree. It can grow as tall (a) It enhances colour.
as 100 feet and as wide as four feet. The sugar maple is (b) It improves flavour.
commercially valued for its sap, which is used in making (c) It increases sugar content.
maple syrup. The two north-eastern states, Vermont and (d) It reduces the shelf-life of the syrup.
New York, are major producers of maple syrup. In Canada, This specifically detailed question is negative and we
Quebec’s annual syrup production surpasses 2.5 million can eliminate choices that are not applicable. Hence,
gallons. To make pure maple syrup, holes are made in the the correct answer is (d).
trunk of the tree at the end of winter or in early spring. The 5. What can be inferred about the production of maple
water-like sap seeps through the holes and runs through a syrup?
plastic spout that is put into the hole. Afterwards, the col- (a) It is simple, but time consuming.
lected sap is transferred into tubes that are hooked up to a (b) It is labour intensive.
tank kept in the sugar house. Then, the sap goes through a (c) Its processing demands complex equipment.
boiling process. Boiling enhances the flavour and adds col- (d) The higher the volume, the less predictable is the
our to the sap. Once the sugar content of the sap is about quality.
65–66 per cent, the sap is ready to be strained and mar-
keted. However, maple syrups found in supermarkets are This question tests our ability to infer meaning. The
usually not pure and have other additives. The colour of answer to the question is not directly stated in the
pure maple may range from golden honey to light brown. passage.
Between 35 to 50 gallons of sap is needed to produce 1 gal- Hence, the correct answer is (a). The production
lon of maple syrup. Also popular for the strength and finish ­technique is quite straightforward, but it takes time. The
of its wood, the sugar maple tree has been put to use in fur- producers have to depend on the natural flow of sap.
niture, interior woodwork, flooring, and crates. 6. The phrase, to be strained, could best be replaced by
which of the following statement?
Questions (a) To be tried (b) To be purified
1. What is the main purpose of the passage? (c) To be filtered (d) To be solidified
(a) To examine the economic viability of making This is a language/expression question. Hence, the
maple syrup. ­correct answer is (c).
(b) To list a number of major producers of maple
syrup.
(c) To provide information on the preparation of
maple syrup. Specimen Passage 2
(d) To discuss the use of maple wood in furniture and Now, India’s children have to receive at least eight years
other products. of education. The gnawing question is whether it will
This is a general inference question. Most of the pas- remain on paper or it will become a reality. One hardly
sage explains the technique used for making maple needs a reminder that this right is different from oth-
syrup. Hence, the correct answer is (c). ers enshrined in the constitution that the b ­ eneficiary—a
2. For which of the following words does the author six-year-old child—cannot demand it, nor can he or she
­provide a definition? fight a legal battle when the right is denied or violated.
(a) The sugar maple (b) A tank In all cases, it is the adult society that must act on behalf
(c) Additives (d) Furniture of the child. In another peculiarity, where a child’s right
to education (RTE) is denied, no compensation offered
This specific detailed question tests our understand-
later can be adequate or relevant. This is so because
ing of a definition. Only sugar maple is defined in the
childhood does not last long. If a legal battle fought on
first line of the passage. Other words have just been
behalf of a child is eventually won, it may be of little use
mentioned in the passage. Hence, the correct answer
to a boy or girl because the opportunity missed at school
is (a).
during childhood cannot serve the same purpose later
3. According to the passage, which of the following peri- in life. This may be painfully true for girls because our
ods is ideal for sapping? society permits them only a short childhood, if any at
(a) End of winter or early spring. all. The RTE has become a crucial point of law in India’s
(b) Beginning of winter or early spring. history when the ghastly practice of female infanticide
(c) End of winter or late spring. resurfaced in the form of foeticide. This is ‘symptomatic
(d) Throughout the year. of deeper turmoil’ in society, which is compounding the
This is again a specific question. Based on the infor- traditional obstacles to girls’ education. Tenacious prej-
mation in the passage, sapping takes place at the udice against intellectual potential of girls runs across
end of winter and in early spring. Hence, the correct our cultural diversity and the system of education has
answer is (a). not been able to address it.

M03_MADAN 04_65901_C03.indd 4 21/12/22 10:52 AM


Comprehension 3.5

Q uestions 4. The expression, symptomatic of deeper turmoil


reflects:
1. With reference to the passage, consider the following (a) Lack of proper education for girls in society.
statements: (b) Enactment of RTE.
(A) When children are denied education, the adult (c) Prevailing political environment in the country.
society does not act on behalf of them. (d) None of the above
(B) RTE as a law cannot be enforced in the country.
Explanation
Which of the statement(s) given above is/are
There is a mention of girl’s education in the ­second part
incorrect?
of the sentence. Hence, the correct option is (a).
(a) Only (A) (b) Both (A) and (B)
5. Which one of the following statements convey the key
(c) Only (B) (d) Either (A) or (B) message of the passage?
(a) India has declared that education is compulsory

Explanation
for children.
Statement (A) is incorrect. The enactment of RTE
(b) Adult society is not keen in implementing the RTE.
itself shows that adult society may act on behalf of (c) The RTE, particularly for a girl child, needs to be
children to ensure their education. safeguarded.
Statement (B) is incorrect as the passage does not (d) None of the above
indicate that the right cannot be enforced in the coun-

Explanation
try but rather mentions the shortfalls and difficulties
that may occur in the act of enforcing the law. Option (a) is incorrect as there is no mention of edu-

cation being made compulsory. Option (b) is also
Both A and B are incorrect statements. Hence,
incorrect, as there is no mention of adults not being
option (b) is the right answer.
keen in implementing the RTE; it simply mentions
2. According to the passage, what could be traditional that it is the adult society that must act on behalf of
obstacles to the education of girls? the child. The author is specifically concerned about
(A) Inability of the parents to fight a legal battle when girl child’s education. Hence, the correct option is (c).
the RTE is denied to their children.
(B) The traditional way of thinking about girls’ role in 6. Which one of the following statements convey the
society. inference of the passage?
(C) The prejudice against the intellectual potential of (a) The society has a tenacious prejudice against the
girls. intellectual potential of girls.
(D) Improper system of education. (b) Adults cannot be relied upon to fight on behalf of
Select the correct answer from the codes given below. children for their RTE.
(a) Only (A) and (B) (c) The legal fight to get education for children is
(b) Only (A), (C), and (D) ­protracted and prohibitive.
(c) Only (B), (C), and (D) (d) There is no sufficient substitute for education
(d) (A), (B), (C), and (D) received in childhood.

Explanation
Explanation
Statement (A) is not mentioned in the passage any- The option (a) is directly mentioned in the passage

where. The statements (B), (C), and (D) are men- and hence, it is not an inference. Option (c) has the
tioned as traditional obstacles at the end of paragraph. word ‘prohibitive’ that cannot be inferred from the
Hence, (c) is the correct answer. ­passage. Option (d) is supported by ‘if a legal b
­ attle …
if at all’, which points out the opportunity cost of a
3. Where a child’s RTE is denied, no compensation missed childhood, particularly for girls. Hence, the
offered later can be adequate or relevant, is reflected correct option is (d).
through the fact that:
(a) Childhood is short.
(b) Opportunity to learn missed during a specific Specimen Passage 3
childhood period may not be compensated later
The concept of creative society refers to the phase of
in life.
development of a society, in which a large number of
(c) Both (a) and (b) potential contradictions become articulate and active.
(d) None of the above This is most evident when oppressed social groups
Explanation are politically mobilized and demand their rights. The
Both statements are mentioned in the passage. upsurge of peasants and tribes, the movements for
Both (a) and (b) apply. Hence, the correct option regional autonomy and self-determination, the environ-
is (c). mental movements, and the women’s movements in the
developing countries are signs of emergence of a c­ reative

M03_MADAN 04_65901_C03.indd 5 21/12/22 10:52 AM


3.6 Chapter 3

society in contemporary times. The forms of social Which of the statement(s) given above is/are correct?

movements and their intensity may vary from country (a) Only (A) (b) Only (B)
to country and place to place within a country, but the (c) Both (A) and (B) (d) Neither (A) nor (B)
very presence of movements for social t­ransformations
in various spheres of a society indicates the emergence
Explanation
of a creative society in a country. Statements (A) and (B) invert the argument in the

passage. The paragraph does not mention having
Q uestions a variety of social movements is a necessary condi-
tion for a creative society in the entire paragraph,
1. What does the author imply by creative ­society? so statement (A) is incorrect. For statement (B),
(A) A society where diverse art forms and literary just the presence of potential contradictions and
writings seek incentive. conflicts is itself not a necessary condition for social
(B) A society where social inequalities are accepted as movement, but their ‘articulation and (being) active’
a norm. is also required. So statement (B) is also incorrect.
(C) A society where a large number of contradictions So the answer should be neither (A) nor (B). Hence,
are articulate. the correct option is (d).
(D) A society where the exploited and the oppressed 4. Which of the following are examples of different
groups grow conscious of their human rights and social movements?
upliftment. (A) Upsurge of peasants and tribes.
Select the correct answer using the codes given below: (B) Movements for regional autonomy and
(a) (A), (B), and (C) (b) Only (D) self-determination.
(c) Only (C) and (D) (d) Only (B) and (D) (C) The environmental movements.
(D) The women’s movements.

Explanation

Codes:
Statement (A) takes the literal meaning of creative
(a) (A), (B), and (C) (b) (B), (C), and (D)
­society, which is not appropriate in the given ­context (c) (A), (B), and (D) (d) All of the above
of the paragraph. Statement (B) contradicts the pas-
sage as there is talk about oppressed social groups
Explanation
getting politically mobilized and demanding rights. This is explicitly mentioned in the third line of the

Statement (C) is mentioned in the first sentence, passage. Hence, the correct option is (d).
where potential contradictions become ‘­ articulate’.
Statement (D) is mentioned in ‘this is most e­ vident … 5. Which of following can be described as the most
creative society in temporary times’. Hence, the cor- appropriate aim(s) for various social movements?
rect option is (c). (A) To achieve the status of a creative society.
(B) To achieve rights.
2. According to the passage, what are the manifestations (C) Social transformation
of social movements?
(A) Being aggressive
Codes:
(B) Involvement of the whole society. (a) (A) and (B) (b) (B) and (C)
(C) Quest for social equality and individual freedom. (c) (A) and (C) (d) Only (C)
(D) None of the above Explanation
Select the correct answer using the codes given below.
Statement (A) is not the explicit aim of any move-
(a) Only (A) and (B) (b) Only (C) ment. Statement (B) is mentioned in the second line
(c) Only (B) and (C) (d) (A), (B), and (C) of the paragraph. Social transformation has been
mentioned in the last line of the paragraph. Hence,
Explanation the correct option is (b).
Social movements do not need to be aggressive. 6. Which of following can be considered the most
Hence, (A) is incorrect. Statement (B) contradicts the ­suitable title for the passage?
passage since social groups get politically mobilized (a) Creative society and social movements
and demand their rights internally, not externally. (b) Social movement as the prerequisite for a creative
The entire passage supports statement (C). Hence, society
the correct option is (b). (c) Social movements
3. With reference to the passage, consider the following (d) None of the above
statements.
(A) To be a creative society, it is essential to have a
Explanation
variety of social movements. Option (a) seems to be the most appropriate option.

(B) To be a creative society, it is imperative to have Option (b) is not the answer: yes, social movements
potential contradictions and conflicts. indicate the emergence of creative society, but

M03_MADAN 04_65901_C03.indd 6 21/12/22 10:52 AM


Comprehension 3.7

nowhere is it mentioned in the paragraph that social Explanation


movement is the prerequisite for a creative society. Statement (a) is contradictory to the passage.
Option (c) is also not the answer as the paragraph is ­Statement (b) is an assumption implied from ‘­Spiritual
mainly about creative society. or any other … starvation and misery’. Hence, the cor-
rect option is (b).
3. Which of the following can be considered the main
Specimen Passage 4 theme of the paragraph?
A country under foreign domination seeks escape from (a) The state of mind of oppressed people.
the present in dreams of a vanished age and finds consola- (b) Starvation and misery.
tion in visions about the greatness of the past generations. (c) The growth of civilization.
That is a foolish and dangerous pastime in which many of (d) Body, mind, and spirit of people in general.
us indulge.
Explanation
An equally questionable practice for us in India is to The statements (b), (c), and (d) are just references in

imagine that we are still spiritually great, though we the passage. Hence, the correct option is (a).
have come down in the world in other respects. Spiritual
or any other such greatness cannot be found on lack of 4. According to the passage, the torture of the mind and
freedom and opportunity or on starvation and misery. spirit is caused:
Many western writers have encouraged the notion that (a) By the ruthlessness of foreign domination.
Indians are other worldly. I suppose that the poor and (b) By the desire to escape from foreign domination
unfortunate in every country become other worldly to and find consolation in visions of past greatness.
some extent, unless they become revolutionaries, for (c) By the desire to become either other worldly or
this world is evidently not meant for them. So are the revolutionary.
subjected people. (d) Due to lack of equilibrium between an external
As a man grows to maturity, he is not entirely life and an internal life.
engrossed in or satisfied with the external objective
Explanation
world. He also seeks some inner meaning and some
psychological and physical satisfaction. Therefore, Statement (d) is the correct option. It is mentioned

along with people and civilizations, they mature and in the last line of the third paragraph—this has been
grow as adults. Every civilization and every person rephrased in the specimen passage. Needs to be
exhibits these parallel streams of external and internal rephrased here also. ‘Where they meet or keep … and
lives. Where they meet or keep close to each other there crises arise that torture the mind and spirit’.
is an equilibrium and stability, and when they diverge, 5. As a person grows in maturity, she/he seeks:
conflicts and crisis arise that torture the mind and the (a) Psychological satisfaction only.
spirit. (b) Physical satisfaction only.
(c) Both psychological and physical satisfaction.
Q uestions (d) Neither (a) or (b).
1. The passage mentions that ‘this world is evidently not
meant for them’. It refers to people who:
Explanation
(a) Seek freedom from foreign domination This is mentioned explicitly in the last paragraph.

(b) Live in starvation and misery 6. Many western writers have encouraged the notion
(c) Become revolutionaries that Indians are other worldly. What can be the possi-
(d) All of the above ble meaning of ‘other worldly’ in the context of entire
passage?
Explanation
(a) They are still basking in past greatness and are
The pronoun, them, refers to the poor and unfor- not in touch with the realities of the present-day
tunate in every country. Hence, the correct option world.
is (b).
(b) Poor, unfortunate, and subject people.
2. Which of the following can be taken as the most valid
(c) Both (a) and (b)
assumption of the paragraph?
(a) A country under foreign domination cannot (d) Neither (a) nor (b)
indulge in spiritual pursuit.
Explanation
(b) Poverty is an impediment in suitable pursuit. The term ‘other worldly’ has been explicitly used in the

(c) Both (a) and (b) second paragraph for poor people. In common par-
(d) None of the above lance, it means the poor people that are cut-off from
main stream society. Hence, the correct option is (b).

M03_MADAN 04_65901_C03.indd 7 21/12/22 10:52 AM


3.8 Chapter 3

(c) In the short-term, some people may be harmed,


Specimen Passage 5 but in the long-term, everyone will benefit from
Ecosystems provide people with a variety of goods and ser- modifications in the earth’s ecosystem.
vices, such as food, clean water, clean air, flood control, soil (d) None of the above
stabilization, pollination, climate regulation, spiritual fulfil-
ment, and aesthetic enjoyment, to name just a few. Most of Explanation
these benefits are either irreplaceable or the technology nec- Only statement (a) is implied, (b) is irrelevant, and
essary to replace them is prohibitively expensive. For exam- the discussion in the passage does not support (c).
ple, potable freshwater can be extracted by desalinating sea Hence, the correct option is (a).
water, but only at great cost. 4. With reference to the passage, consider the following
The rapidly expanding human population has greatly statements:
modified the earth’s ecosystem to meet their increased (A) It is imperative to modify the earth’s ecosystem for
requirements of some goods and services, particularly the well being of mankind.
food, freshwater, timber, fibre, and fuel. These modifica- (B) Technology can never replace all the goods and
tions have contributed substantially to human well being ­services provided by the ecosystems.
and economic development. The benefits have not been
equally distributed. Some people have actually been Which of the following statement(s) given above is/
harmed by these changes. Moreover, s­ hort-term increases are correct?
in some ecosystems’ goods and services have come at the (a) Only (A)
cost of long-term degradation of others. For example, (b) Only (B)
efforts to increase the production of food and fibre have (c) Both (A) and (B)
decreased the ability of some ecosystems to provide clean (d) Neither (A) nor (B)
water, regulate flooding, and support biodiversity. Explanation
Statement (A) is incorrect because the passage is
Q uestions almost entirely about the positive and negative
1. Expanding human population has an adverse effect on: aspects of the modification of our ecosystem. State-
(A) Spiritual fulfilment ment (B) is mentioned clearly in the passage. Hence,
(B) Availability of potable freshwater. the correct option is (b).
(C) Employment 5. According to the passage, which of the following can
(D) Biodiversity be taken as the main reason for the modification of
the earth’s ecosystem?
Which of the statements given above are ­correct? (a) Technology
(a) (A), (B), and (C) (b) (B), (C), and (D) (b) Increasing population
(c) (B) and (D) (d) All of the above (c) Lack of an integrated approach
Explanation (d) All of the above
The last sentence indicates an adverse effect on the Explanation
availability of clean water and biodiversity. Hence, Though both technology and lack of integrated
the correct option is (c). approach may be applicable, both are long drawn
2. The passage mentions that ‘some people have actually inferences. Among the available choices, increasing
been harmed by these changes’. It indicates: population is clearly mentioned. Hence, the correct
option is (b).
(a) Inequitable distribution of benefits.
(b) Decrease in the ability of some ecosystems to pro- 6. Which of the following can be considered as the most
vide clean water, regulate flooding, and support suitable title for the passage?
biodiversity. (a) Modification of our ecosystem.
(c) Both (a) and (b) (b) Our natural resources.
(c) Harmful effect of the increase in human
(d) Neither (a) nor (b)
population.
Explanation (d) Human interference in our ecosystem.
Both (a) and (b) indicate inequitable distribution Explanation
of benefits and resources. Hence, the correct option Clearly, options (b), (c), and (d) may be relevant
is (c). to the discussion in the passage, but the passage is
3. Which of the following is correct in the context of the mainly about the causes and effect of changes in our
passage? ecosystem. Hence, the correct option is (a).
(a) The rapid expansion of population has adversely
affected some people.
(b) Sufficient efforts have not been made to increase
the production of food and fibre.

M03_MADAN 04_65901_C03.indd 8 21/12/22 10:52 AM


Comprehension 3.9

A s s e s s Yo u r L e a r n i n g

PrACTICe PASSAGeS
Passage 1 3. Who protected the school during the riot?
(a) Hindus (b) Local Muslims
Read the following passage and answer the questions
(c) Politicians (d) Christians
1–5.
4. Who is the teacher of Sanskrit?
Each day at Shantiniketan, the school starts with
Saraswati Vandana. When painting competitions are (a) Manisha Vakil
held in the school, images of Hindu gods and goddesses (b) Ranchodbhai Kiri
are the most common. Sanskrit is the favourite sub- (c) Husena Mansuri
ject of many students. Nothing is new about it except (d) Husena Khatoon
that the 1200 odd students studying in the Hindu-run
school are Muslims. In 1983, when Ranchhodbhai Kiri 5. What is the hope despite the communal riots?
started Shantiniketan in the all-Muslim Juhapura area of (a) Hindus and Muslims can live side by side
Ahmedabad in Gujarat, only 20 per cent of the students (b) Only Hindus can live
were Muslim, but when riots involving the Muslims of (c) Hindus and Muslims cannot live side by side
Juhapura and the Hindus of nearby Jivraj Park, Vejalpur,
(d) Only Muslims can live
affected the locality, Hindus started migrating. Today, all
the students are Muslims, and the school is an unparal- Passage 2
leled example of harmony. In 2002, when a section of the
Read the following passage and answer the questions
inflamed Muslims wanted the school closed, the parents
6–10.
of the students stood like a wall behind it. Shantiniketan’s
Some religious leaders have taught that man is made
principal said, ‘We never thought of moving the school
up of a body and a soul, but they have been silent about
out of the area because of the love and affection of the
intellect. Their followers try to feed the body to earth
local Muslims. Indeed, they value the high standard of
and save the soul from perdition after death, but they
education that we have set.’ Such is the reputation of the
neglected the claims of the mind. Bread for the body
school that some of the local Muslim strongmen accused
and virtue for the soul, these are regarded as indispen-
of involvement in communal riots are willing to protect

A S S E S S YO U R L E A R N I N G
sable requisites of human welfare. Nothing is said about
the school during riots. The parents of Shantiniketan’s
students believe that it is the best school when it comes knowledge and education. Thus, Jesus Christ spoke of
feeding the hungry, healing the sick, and converting the
to quality of teaching. A large number of students have
sinners, but he never taught the duty of teaching the
gone for both graduation and post-graduation studies.
ignorant and increasing scientific knowledge. Gautama
Significantly, the only Muslim teacher in the 40-member
Buddha also laid stress on morality, meditation, and
teaching staff, named Husena Mansuri, teaches Sanskrit.
asceticism, but he did not attach great importance to
In fact, she is so happy with the school that she recently
history, science, art, or literature. St. Ambrose depre-
declined the principalship of another Muslim-run school.
cated scientific studies and wrote, ‘To discuss the nature
Some of the students’ entries in a recent inter-school
and position of the earth does not help us in our hope
painting competition were truly moving. One drew a
for life to come.’ St. Basil said ‘It is not a matter of inter-
picture of Bharat Mata with a mosque and temple, while
est to us whether the earth is a sphere, a cylinder, or a
another portrayed a boy tying rakhi to his sister. Truly,
disc.’ Thomas Carlyle also followed the Christian tradi-
Shantiniketan is a beacon of hope that despite the provo-
tion and declared that he honoured only two kinds of
cations from both communities, Hindus and Muslims can
men and no third, that is, the manual labourer and the
live side by side with mutual respect.
religious teacher. He forgot the scientist, the scholar,
1. How does Shantiniketan school start the day? and the artist. The cynics of Greece despised education
(a) Prayer as well.
(b) National anthem 6. What have the religious teachers taught in the past?
(c) Saraswati Vandana
(a) That man is made up of body only.
(d) Puja
(b) That man is made up of soul only.
2. Which subject is most preferred by the students?
(a) English (b) Hindi (c) That man is made up of bubbles.
(c) Sanskrit (d) Gujarati (d) That man is made up of body and soul.

M03_MADAN 04_65901_C03.indd 9 21/12/22 10:52 AM


3.10 Chapter 3

7. According to the passage, what is food for the soul? In the context of India, at least two immediate factors
(a) Bread (b) Virtue increase the ratio mentioned above. The first of these
(c) Vice (d) Education is the rise in population level. By giving momentum
to expansion of population and the workforce, human
8. The following philosophers are mentioned in the resource development has achieved synergistic impor-
paragraph: tance. An increase in population is also a factor, but is not
(A) Jesus (B) Gautama Buddha the most important one that delineated environmental
(C) St. Ambrose (D) Thomas Carlyle decay in rural and urban areas. Second, as a large coun-
(E) St. Basil try, we cannot make an independent place for ourselves
Which of the following depicts the correct order as in the global system without developing appropriate abil-
they appear in the paragraph. ity for the development of our self-respect. In order to
(a) (A), (B), (C), (D), and (E) achieve this objective, the achievement of technical skills
(b) (A), (C), (D), (E), and (B) is a decisive step.
(c) (A), (B), (C), (E), and (D) So far, we have taken human resource development
(d) (B), (A), (C), (D), and (E) and technical and environmental issues as support-
9. Intellectual pursuits have been neglected because: ing factors of the main part of the plan. Along with the
(A) They are unnecessary and superfluous. expansion of quality of basic infrastructure and targets
(B) They make people dwarves. of production (tonnes of steel and kilowatt-hours of elec-
(C) They lead people to hell. tricity), other targets of capacity (kilometres), and other
targets (number of schools and students and number of
(a) Only (A) is correct
electrified villages), known techniques, full use of natural
(b) Only (B) is correct
resources, and maximum possible use of available finan-
(c) Only (C) is correct
cial resources have been emphasized upon.
(d) Only (A) and (B) are correct
11. According to this passage, what has been considered
10. The style of the passage is:
to be the most important by us out of the following?
(a) Narrative (b) Expository
(a) Basic facilities and increase in the number of
(c) Critical (d) Analytical
achieved targets.
Passage 3 (b) Ideal use of available natural resources.
(c) Maximum use of available financial resources.
Read the following passage and answer the ­questions
(d) All of the above
11–15.
The previous decade has reversed the presumptions about 12. According to the author of the passage, whose effect
development, and more than anything else, it has made out of the following is felt at the national level?
it difficult to decide what is in store for the next decade. (a) Expansion of workforce of high quality.
A S S E S S YO U R L E A R N I N G

However, there are some things about which one can (b) Lack of care and activism for the protection of
make claims with some confidence. environmental resources.
Firstly, education, health, and productive employment (c) Continuous decay of technical potentialities in
are the decisive factors for development and impartial- urbanized countries.
ity. We believe that all these are results of rapid economic (d) Emphasis on a slower pace of disarmament as
development, and to achieve these ends, development compared to disarmament in other nations.
only can generate resources. In the present form, it will 13. According to the author of the passage, which of the
be best to view these as a better reason than as a result ­following factors is of synergistic importance?
of development. In fact, in every case of successful devel- (a) Population growth
opment, the evaluation of previous reforms in education, (b) Workforce
technical skills, health, existence, and productive tasks is (c) Human resource development
included. (d) None of the above
Secondly, technical ability is a vital resource and 14. Which of the following areas has not been included
explains the high ratio of development in production and amongst various ‘targets’ mentioned in the passage?
trade as compared to ratios of development in more tradi- (a) Maximum use of financial resources
tional factors, such as natural resources or capital forma- (b) Electricity production
tion. There is no requisite capability in research. In fact, (c) Population growth
industrial momentum in a factory or farm is more impor- (d) Number of schools and children
tant than the presence of a research organization.
Thirdly, the essentially required environment also 15. Which of the following can be the most suitable title
cannot be ignored for a long time period, which is next of the passage?
only to the issue of disarmament in the list of interna- (a) Potential obstacles in economic development
tional issues. At the national level, there has been a defi- (b) Main factors of development
nite rise in ignorance towards the environment due to (c) Targets in development process
development. (d) Role of population growth in development

M03_MADAN 04_65901_C03.indd 10 21/12/22 10:52 AM


Comprehension 3.11

Passage 4 20. The author considers everyone to be a vehicle that


Read the following passage and answer the questions knows how to go forward:
16–20. (a) Without driving energy
The great Acharyas have said that everything discov- (b) With least consideration for others
ered has a great goal; surrender yourself to that goal (c) With no sense of direction
and act on it by drawing your inspiration from that goal (d) With no control on speed
whereby you will get a new column of energy. Do not
allow this energy to be dissipated in the futile memory Passage 5
of past regrets or failures, or excitement of the present, Read the following passage carefully and answer the
and bring that entire energy focused into activity, that ­questions 21–25.
is, the highest creative action in the world outside, The phrase ‘What is it like?’ stands for a fundamental
whereby the individual who is till now considered the thought process. How does one go about observing and
most inefficient, finds his way to the highest achieve- reporting on things and events that occupy the segments
ment and success. of earth space? Of all the infinite varieties of phenomena
This can be said very easily in a second. In order to train on the face of the earth, how does one decide what phe-
our mind to this attitude, considerable training is needed nomena to observe? There is no such thing as a complete
because we have already trained our mind wrongly to description of the earth or any part of it, as every micro-
such an extent that we have become perfect in imperfec- scopic point on the earth’s surface differs from every other
tions. Not knowing the art of action, we have mastered such point. Experience shows that the things observed
artists in doing wrong things, and the totality of activity are already familiar because they are like phenomenon
will bring the country to a wrong end indeed. that occur at home or because they resemble the abstract
If each individual is given a car to achieve an ideal images and models developed in the human mind.
socialistic pattern and nobody knows driving but starts How are abstract images formed? Humans alone,
driving, what would be the condition on road? Everybody amongst all other animals on the earth, possess lan-
has equal rights on the public roads. Then, each car will guage, and their words symbolize not only specific things,
necessarily dash against the other and there is bound to but also mental images of classes of things. People can
be a jumble. remember what they have seen or experienced because
There seems to be a very apt pattern of life that we are they attach a word symbol to them.
heading to. Every one of us is a vehicle. We know how During the long record of our efforts to gain more and
to go forward. The point intellect is very powerful, and more knowledge about the face of the earth as the human
everybody is driving, but nobody knows how to control habitat, there has been a continuing interplay between
the mental energy and direct it properly or guide it to the things and events. The direct observation through the
proper ­destination. senses is described as a percept, and the mental image is

A S S E S S YO U R L E A R N I N G
16. What is the effect of wrong training of our mind? described as a concept. Percepts are what some people
(a) Becoming perfect in all aspects of life. describe as reality, in contrast to mental images, which
(b) Becoming master artists. are theoretical, implying that they are not real.
(c) Taking the country to wrong destination. The relation of percept to concept is not as simple as
(d) Carrying on activities without knowing how to the definition implies. It is now quite clear that people of
­control mental energies. different cultures or even individuals in the same culture
17. The source of energy according to the author is: develop different mental images of reality and what they
perceive is a reflection of these preconceptions. The direct
(a) Highest creative action.
observation of things and events on the face of the earth
(b) Proper training of mind. is therefore, clearly a function of the mental images of the
(c) Inspiration from past events. mind of the observer that the whole idea of reality must
(d) Stimulation obtained from a set goal. be reconsidered.
18. The author’s main focus in the passage is: Concepts determine what the observer perceives,
(a) Finding out a worthy goal in life. yet concepts are derived from the generalizations of
(b) Regulation of energy in proper channels. previous percepts. What happens is that the educated
(c) Struggle for equal rights. observer is taught to accept a set of concepts and then,
(d) Car accidents due to lack of driving skills. he/she sharpens or changes these concepts during their
professional career. In any field of scholarship, profes-
19. The country may perish because of: sional opinion at one time determines what concepts
(a) Failures in past acts. and procedures are acceptable, and these form a kind
(b) Wrong deeds performed without proper of model of scholarly behaviour.
knowledge. 21. The problem raised in the passage reflects on:
(c) Completely surrendering to any one goal. (a) Thought process (b) Human behaviour
(d) Directing mental energy to the right destination. (c) Cultural perceptions (d) Professional opinion

M03_MADAN 04_65901_C03.indd 11 21/12/22 10:52 AM


3.12 Chapter 3

22. According to the passage, human beings have which It is perfectly true that Gandhi, functioning in a nation-
of the following in mind the most? alist plane, did not think in terms of the conflict of classes,
(a) Observation of things trying to compose their differences. However, the actions
(b) Preparation of mental images he indulged in and taught the people have inevitably
(c) Expression through language raised mass consciousness tremendously and made social
(d) To gain knowledge issues vital. Gandhi and the Congress must be judged by
the policies they pursued and the actions they indulged
23. Concept means: in. But behind this, personality counts and that colours
(a) A mental image those policies and activities. In case of very exceptional
(b) A reality person, like Gandhi, the question of personality becomes
(c) An idea expressed in language especially important in order to understand and appraise
(d) All of the above him. To us, he represented the spirit and honour of India,
24. The relation of percept to concept is: the yearning of her sorrowing millions to be rid of their
(a) Positive (b) Negative innumerable burdens. An insult to him by the British
(c) Reflective (d) Absolute Government or others was an insult to India and her
people.
25. In the passage, the earth is taken as:
(a) The globe (b) The human habitat 26. Which one of the following is true for the given
(c) A celestial body (d) A planet passage?
(a) The passage is a critique of Gandhi’s role in Indian
Passage 6 movement for independence.
Read the following passage carefully and answer the (b) The passage hails the role of Gandhi in India’s
­questions 26–30. ­freedom movement.
It should be remembered that the Nationalist Movement (c) The author is neutral on Gandhi’s role in India’s
in India, like all nationalist movements, was essentially a ­freedom movement.
bourgeois movement. It represented the natural histori- (d) It is an account of Indian National Congress’ sup-
cal stage of development, and to consider it or to criticize port to the working class movement.
it as a working class movement is wrong. Gandhi repre- 27. The change that the Gandhian movement brought
sented that movement, and the Indian masses in relation amongst the Indian masses was:
to that movement to a supreme degree, and he became (a) Physical (b) Cultural
the voice of Indian people to that extent. The main con- (c) Technological (d) Psychological
tribution of Gandhi to India and the Indian masses has 28. To consider the nationalist movement or to criticize
been through the powerful movements that he launched it as a working class movement was wrong because it
through the National Congress. Through nation-wide was a:
A S S E S S YO U R L E A R N I N G

action, he sought to mould the millions and largely suc- (a) Historical movement
ceeded in doing so. He changed them from a demoralized, (b) Voice of the Indian people
timid, and hopeless mass, bullied and crushed by every (c) Bourgeois movement
dominant interest and incapable of resistance, to people (d) Movement represented by Gandhi
with self-respect and self-reliance, resisting tyranny, and
capable of united action and sacrifice for a larger cause. 29. Gandhi played a revolutionary role in India because
Gandhi made people think of political and economic he could:
issues, and every village and every bazaar hummed with (a) Preach morality
arguments and debates on the new ideas and hopes that (b) Reach the hearts of Indians
filled the people. (c) See the conflict of classes
That was an amazing psychological change. The time (d) Lead the Indian National Congress
was ripe for it, and, of course, the circumstances and 30. Groups with advanced ideology functioned in air as
world conditions worked for this change. However, a great they did not fit in with:
leader was necessary to take the advantage of those cir- (a) Objective conditions of masses
cumstances and conditions. Gandhi was that leader, and (b) The Gandhian ideology
he released many bonds that imprisoned and disabled our
(c) The class consciousness of the people
minds, and none of us who experienced it can ever forget
that great feeling of release and exhilaration that came (d) The differences amongst masses
over the Indian people.
Passage 7
Gandhi has played a revolutionary role of greatest
importance in India because he knew how to make the Read the following passage carefully and answer the
most of the objective conditions and could reach the heart ­questions 31–35.
of the masses, whereas groups with a more advanced ide- Secularism is the very soul of Indian society and the
ology functioned largely in air because they did not fit in democracy. The feelings of co-existence, tolerance, co-
with those conditions and could, therefore, not evoke any operation and mutual respect between all its social and
substantial response from the masses. religious groups have been in the main stream of India

M03_MADAN 04_65901_C03.indd 12 21/12/22 10:52 AM


Comprehension 3.13

since ancient times. Hinduism itself was a religious con- (a) Unity in Diversity
glomeration of thousands of sects having distinct beliefs, (b) Hind Ki Chadar
rituals, customs and practices. Three nearly different (c) Diversity in Unity
religions branched out of it, namely, Buddhism, Jainism (d) Uniform society
and Sikhism having separate places of worship and holy
books. Islam is the second largest community of our 35. Which of the following personalities is christened as a
nation with a spectacular contribution. All the religions pivot to fought for secular spirit of India?
have added variety, colour and richness to native herit- (a) Jawahar Lal Nehru
age. Christianity had preceded Islam to India. St. Thomas, (b) Mahatma Gandhi
the twelve disciples of Christ arrived in India to preach his (c) Dr Bhim Rao Ambedkar
message when St. Peter was in Rome. (d) Sardar Vallabh Bhai Patel
Parsis came to India in eighth century to escape
from religious persecution in Iran. They brought Passage 8
Zoroastrianism. Jews sailed to India two thousand years Read the following passage carefully and answer the
ago to settle down in Mumbai, Pune, Kochi and Delhi. In ­questions 35–40.
all, India remained a shining example of unity in diversity, In India, co-education has not been a very long practice. It
nowhere else found in the world. started in India during pre-independence period. At that
Thus, our constitution makers declared our nation to be time, education movements started as a part of national
a secular state without any discrimination. Despite being movement. It was realised that co-education was a part
religious, they don’t harbour any ill will or intolerance of the modern education which reduced gender biases.
against other faiths. Even during the freedom struggle The modern education started in India in the early part
the leaders who led it made secularism a basic policy to of nineteenth century. Co-education is based on the feel-
rally all the people against the British. The secular polity ing of gender equality and skill development so that we
was fiercely strengthened by Mahatma Gandhi. Himself a can see women at top positions. The girls and boys look
deeply religious Hindu, he had great respect for all other for freedom to interact with each other so that a healthy
religions and faiths. practice can be set instead of being conservative, suspi-
To divide the Indians, the colonial rulers tried to cious, and ignorant. In our rural counterparts, we can still
destroy file spirit of secularism by aiding and abetting see separate schools for boys and girls. Now, co-education
communal forces. Some of our people terribly played in is being practiced in big towns and cities, in colleges, uni-
the hands of British. Our nation paid a terrible price for versities and other institutions. This situation is perceived
that lapse that got divided into two nations. Gandhi Ji sac- to be beneficial for complete personality development.
rificed to the bullets of a degenerated fanatic who had no In fact, Switzerland was the first nation to introduce
use for our secular legacy. It should teach us a lesson that co-education. Then, this phenomenon spread to other

A S S E S S YO U R L E A R N I N G
only secular spirit will keep India going. western nations. Now, it is a modern way of life.
This system of education is very cost effective as well. It
31. Which of the following tradition/s has/have been the reduces financial burden that is required with more sepa-
main tradition/s of India? rate buildings, teacher requirements. A developing nation
(a) Co-existence, tolerance, mutual respect and such as India, has always been under financial distress,
cooperation we always look for better returns on investment on educa-
(b) beliefs, rituals, customs and practices tion. This is mandatory as per our legislative commitment
(c) unity in diversity towards universal and compulsory education.
(d) All of the above A better understanding between boys and girls is ulti-
mately sign of our family and social progression. They
32. Which religions came to India from abroad? may know better the intricacies of relationships. The
(a) Islam, Christianity, Zoroastrians, Jews social media is also playing an important role in it. The
(b) Hindus, Sikhs, Buddhist, Jainism boys are not posed as dangerous creatures. The boys must
(c) Different sects over a period of time learn to treat girls in a respectful and civilized manner.
(d) None of the above The gender linked crime rate must come down. The soci-
ety can exert better social control and discipline as boys
33. What price India paid for its lapse in secularism?
behave decently in the presence of girls. Girls also come
(a) India got divided into two nations at the time of out of their social reservations. A healthy competition
independence. gets generated and they can explore their talent, skills
(b) Mahatma Gandhi sacrificed for the nation. and capabilities in the desired manner. This helps in the
(c) Both a and b refinement of society.
(d) None of the above
36 What was the main reason cited for development of
34. Which of the following concept has been used to show co-education system in India?
that India has successfully accommodated all impor- (a) Gender equality and skill development
tant religions? (b) This tallies with religious ordains

M03_MADAN 04_65901_C03.indd 13 21/12/22 10:52 AM


3.14 Chapter 3

(c) This is required for economic development The dissonance between what teachers say. or at least
(d) This is required for purely emotional feeling what their leaders say, and what they do takes many
forms and has several important consequences. For some,
37 Which of the following nations introduced co-education it lays the groundwork for the development of a cynical
for the first time in the world? outlook towards the admonitions of idealists and the
(a) Great Britain advocates of new and supposedly revolutionary practices.
(b) Switzerland This cynicism, which grows out of a prior sense of disil-
(c) Austria lusionment strikes many young teachers as they begin
(d) Russia to appreciate the unrealistic quality of several of the
expectations aroused during the period of their profes-
38. How boys are found to behave in the presence of girls? sional training. Teaching as actually experienced and as
(a) The boys are found to behave decently in presence described in textbooks and college courses often turns out
of girls. to be to quite different states of affairs. The result is that
(b) The violent incidents are bound to increase in the college instructors of education and other outsiders begin
society. to be looked upon with suspicion by many practitioners.
(c) The boys are likely to be more upset with issues as Even the testimony of fellow teachers may be viewed sus-
unemployment. piciously when it conflicts with the listener’s own experi-
(d) The boys will become more reserved in their ence in the classroom.
activities.
41. The use of stock phrases in educational discussions
39. As mentioned in the passage, why should boys and has tended to reveal:
girls study together and interact? (a) The linkage between thought and action
(a) It mainly started in India during independ- (b) The difference between saying and doing in class-
ence struggle as a part of the modern-education room contexts
feature. (c) The ceremonial importance of slogans
(b) It generates healthy respect for each other (d) The idealistic ring of talks
between boys and girls instead of suspicions.
(c) Expenses on education is reduced by co-educa- 42. How are the idealistic concepts related to the routine
tion as boys and girls share the same facilities and practices in education?
staff. (a) They intertwine theory and practice
(d) All of the above (b) They make classroom teaching innovative
(c) They make the business of education highly
40. What are the economic aspects of co-education in profitable
India? (d) They lead to the identification of gap between
A S S E S S YO U R L E A R N I N G

(a) The better skill development for the betterment of theory and practice
the society.
(b) The cost of setting up of schools can offers us 43. What is the sequel of lack of congruence between say-
better returns ing and doing as evident in teacher behaviour?
(c) The staff utilisation can offer us better returns (a) Opposition to revolutionary practices
(d) All of the above (b) Lays ground for frequent complaints
(c) Emergence of disillusionment among teachers
Passage 9 (d) Support to the admonition of idealists
Read the following passage carefully and answer the 44. Why do young teachers appreciate the unrealistic
­questions 41–45. [June 2021] quality of expectations?
The ceremonial use of slogans and catchwords in educa- (a) Because of the cynical attitude among teachers
tional discussions raises the suspicion of a tenuous linkage (b) Because of the testimonial given by fellow
between thought and action. As stock phrases multiply teachers
and the talk begins to take on an idealistic ring, the wary (c) Because of their professional training
listener might well begin to wonder whether the lip ser- (d) Because of their own experience in teaching
vice paid to these concepts is connected with what actu-
ally goes on in classrooms. It is difficult in this day and age 45. The passage analyses
to be opposed to democracy, creativity, and innovation (a) The linkage between language and discussions
in education, but how are these attractive words related (b) The schism between theory and actual practice of
to the more mundane business of teaching practice? The teaching
answer, of course, is that the two are often not related - a (c) The need to the use of high brow language in edu-
fact that accounts for one of the most frequently recurring cational discussions
complaints among today’s educators: the all-too-obvious (d) The importance of idealism in education
gap between theory (i.e., educational talk) on the one
hand and practice on the other.

M03_MADAN 04_65901_C03.indd 14 21/12/22 10:52 AM


Comprehension 3.15

Passage 10 49. The members of cooperative learning team should be


Read the following passage carefully and answer the given enough time to:
­questions 46–50. [June 2021] (a) reflect
How much time should you allot for group work? It (b) discuss
depends on task complexity, but you must make some (c) ask questions
more refined estimates as well. You need to determine (d) present new ideas
the time to devote to group work and time to devote to
all groups coming together to share their contributions. 50. The most appropriate caption for the passage will be:
This latter time may be used for group reports, a whole- (a) Guidelines for cooperative learning
class discussion, debriefing to relate the work experiences (b) Procedure for cooperative learning
of each group to the end product, or some combination (c) Precaution in the conduct of cooperative learning
of these tasks. Group work can easily get out of hand in (d) Limitations of cooperative learning
the excitement, controversy, and natural dialogue that
can come from passionate discussion. This possibility Passage 11
requires you to place limits on each stage of the coopera- Read the following passage carefully and answer the
tive learning activity, so one stage does not take time from ­questions 51–55.
another and leave the task disjointed and incomplete in According to a study published recently in the journal
your learners’ minds. Most time naturally will be devoted Nature Climate Change, while black carbon has a large
to the work of individual groups, during which the major effect on snow darkening and resultant melting of snow,
portion of the end product will be completed. Individual dust particles transported from as far as Saudi Arabia that
group work normally will consume 60% to 80% of the get deposited in the Western Himalayan Region (WHR)
time devoted to the cooperative learning activity. The have a large role to play in melting of snow, particularly at
remaining time must be divided among individual group higher elevations. Dust transported as elevated, aerosol
presentations and/or whole class discussion and debrief- layers get deposited at 1 – 5 km elevations, black carbon
ing that places the group work into the perspective of a emission is mostly a surface phenomenon and influences
single end product. If you plan both group reports and melting of snow from surface to about 3 km elevation.
whole class discussion for the same day, be aware that the Based on remote sensing data of spatial distribution of
discussion probably will get squeezed into a fraction of dust aerosol concentration over the Indian subcontinent
the time required to make it meaningful. To avoid this, the and dust induced snow albedo reduction over Himalayas
group discussion or debriefing for the following class day during the period 2011–2016 and simulations, it has
may be so scheduled so that class members have ample been inferred that the relative impacts of dust and black
time to reflect on their group reports and to pull together carbon vary with surface elevation of snow pack. This is in
their own thoughts about the collaborative process, which addition to snow-melt caused by warming due to climate

A S S E S S YO U R L E A R N I N G
may or may not have occurred as intended. Providing 15 change. Earlier studies have shown that the magnitude of
or 20 minutes at the beginning of class the next day is usu- snow mass decrease is about 1mm per year at 1 km eleva-
ally enough time for students to have acquired the proper tion, about 5 mm per year at 4.5 km elevation and about 3
distance to reflect meaningfully on their experiences of mm per year at 6 km elevation.
the day before-and to learn from them. Though black carbon has a larger snow albedo dark-
ening effect than dust due to a larger mass absorption
46. What is the main determinant of time allocation in efficiency, the study found that radiative effects of dust
cooperative learning? deposited on snow are comparable to black carbon in
(a) Time devoted to group work the WHR at higher elevations. This is mainly because the
(b) Time devoted to all groups deposition of the dust by mass is 100-1000 times more
(c) Time devoted to presentation of reports than black carbon. As the elevation increases, the influ-
(d) Complexity and the number of tasks involved ence of dust comes greater than black carbon and this
coincides with maximum intensity of snow melt reduc-
47. What can disrupt group work? tion seen at 3-5 km elevation. Between these two black
(a) Difficulty of task carbon mainly contributes to snow melt at lower eleva-
(b) A whole class discussion tion while dust is the major contributor for snow melt at
(c) Discussions which involve emotions higher elevation. Westerlies transport dust particles as
(d) Arguments on various points elevated aerosol layers at maximum intensities mostly
during the pre-monsoon period and this gets deposited at
48. A major chunk of time in cooperative learning is higher elevations in the WHR.
devoted to which of the following? Due to global warming, snow cover at lower elevations
(a) Individual group presentation in the Himalayas will occur less frequently or totally dis-
(b) Whole class discussion appear compared with snow cover at higher elevations.
(c) Individual group work The annual contribution of dust to snow melt will there-
(d) Debriefing fore likely increase in future as the black carbon effect at
lower elevation weakness with dwindling snow pack.

M03_MADAN 04_65901_C03.indd 15 21/12/22 10:52 AM


3.16 Chapter 3

51. Snow melt in Himalayas is on account of: The need for continuing and reinforcing value-oriented
(A) Dust deposition curriculum at the post graduate level and level of profes-
(B) Black carbon deposition sional studies may be considered even more important.
(C) Increase in albedo Most of the students trained at these levels are likely to
(D) Climate change become leaders of business and industry, technocrats,
(a) (A) and (B) only leaders of thought, teachers and educationists, research
(b) (A), (B), (C) and (D) workers, professionals, political leaders and social work-
(c) (A), (B) and (C) only ers etc. besides playing their normal roles as citizens and
(d) (A), (B) and (D) only family members like all other people. It appears that most
of them at the time of leaving the college after 3-6 years
52. Maximum rate of snow mass reduction per annum is of study, may have a poor idea of the values of freedom
at elevation of: and human dignity, the value of food, security and health
(a) 1 km (b) 3-5 km care for millions of poor and disadvantaged people, the
(c) 6 km (d) 2 km importance of correct public policies for managing soci-
eties and solving several problems, the importance of
53. Given below are two statements: One is labelled as proper human relations and personal life styles, etc. A
Assertion A and the other is labelled as Reason R: good number of them, however are now getting oppor-
Assertion A: Reduction in snow albedo is more in case tunities to acquire modern scientific and technical knowl-
of black carbon than dust edge of high standards particularly in some of our institu-
Reason R: Black carbon has higher mass absorption tions of excellence, comparable to the best institutions in
efficiency compared to dust. the developed countries. But the problem of inadequate
In the light of the above statements, choose the cor- access to this kind of modern knowledge for the poor and
rect answer from the options given below: disadvantaged groups in our society constitutes a serious
social and moral issue. Curricular reform and reconstruc-
(a) Both A and R are true and R is the correct explana- tion on proper lines should be the active concern of edu-
tion of A cationists and educational planners, because as pointed
(b) Both A and R are true but R is NOT the correct out in national policy document 1986, it can be a forceful
explanation of A tool for the cultivation of social and moral values. In our
(c) A is true but R is false culturally plural societies education should foster univer-
(d) A is false but R is true sal and eternal values, oriented towards the unity and
integration of our people. Such value education should
54. Given below are two statements: one is labelled as eliminate obscurantism religious fanaticism, violence
Assertion A and the other is labelled as Reason R: superstition and fatalism.
Assertion A: Radiative effects of dust deposited on
A S S E S S YO U R L E A R N I N G

snow are comparable to black carbon in the WHR at 56. In higher education, in the context of promoting
higher elevations. strong value orientation, which of the following state-
ment holds true?
Reason R: Both black carbon and dust darken the
(1) Provide opportunities to students to acquire scien-
snow cover.
tific knowledge of high standards.
In the light of the above statements, choose the cor- (2) A Policy for development of values may be
rect answer from the options given below: prepared.
(a) Both A and R are true and R is the correct explana- (3) Offer courses related to inter disciplinary contem-
tion of A porary socio-economic problems.
(b) Both A and R are true but R is NOT the correct (4) The universities should aim to enrich technologi-
explanation of A cal dimensions.
(c) A is true but R is false
57. The higher education system as of now emphasizes on
(d) A is false but R is true
(1) Utilitarian values
55. The passage clearly brings out the finding that: (3) Social and Political values
(a) Black carbon causes global warming (2) Human values
(b) Dust causes global warming (4) Spiritual values
(c) Dust modifies the radiative properties of snow
58. Value oriented curriculum at professional studies
cover to the same extent as black carbon
level is important
(d) Black carbon and dust are competing agents in
(1) To propagate the values further
melting of snow packs
(2) To have a glimpse of the issues of the poor and
Passage 12 disadvantaged
Read the following passage carefully and answer the (3) To balance other pedagogy, related deficiencies
­questions 56–60. [June 2021] (4) To train students as efficient resource agents

M03_MADAN 04_65901_C03.indd 16 21/12/22 10:52 AM


Comprehension 3.17

59. The most suitable caption for the passage may be: 60. In order to cultivate values, the higher education sys-
(a) Enforcing values in education tem should concentrate on:
(b) Importance of values (a) Plugging pedagogical deficiencies
(c) Value implementation (b) Integrating technology
(d) Value basis for tertiary education and society (c) Integrating technology with pedagogy
(d) Curricular reforms

A S S E S S YO U R L E A R N I N G

M03_MADAN 04_65901_C03.indd 17 21/12/22 10:52 AM


3.18 Chapter 3

Answer Keys
Passage 1
1. (c) 2. (c) 3. (b) 4. (c) 5. (a)
Passage 2
6. (d) 7. (b) 8. (c) 9. (a) 10. (a)
Passage 3
11. (d) 12. (b) 13. (c) 14. (c) 15. (b)
Passage 4
16. (d) 17. (d) 18. (b) 19. (b) 20. (c)
Passage 5
21. (a) 22. (b) 23. (a) 24. (c) 25. (b)
Passage 6
26. (b) 27. (d) 28. (b) 29. (b) 30. (a)
Passage 7
31. (d) 32. (a) 33. (c) 34. (a) 35. (b)
Passage 8
36. (a) 37. (b) 38. (a) 39. (d) 40. (d)
Passage 9
41. (b) 42. (d) 43. (c) 44. (a) 45. (b)
Passage 10
46. (d) 47. (c) 48. (c) 49. (a) 50. (c)
A S S E S S YO U R L E A R N I N G

Passage 11
51. (d) 52. (b) 53. (a) 54. (b) 55. (d)
Passage 12
56. (c) 57. (a) 58. (d) 59. (d) 60. (d)

M03_MADAN 04_65901_C03.indd 18 21/12/22 10:52 AM


CHaPTer

4 Communication

01 Communication: Meaning, Characteristics of Communication

02 Communication and its Types

03 Effective Communication:
learninG Verbal and Non-verbal,
Inter-cultural and Group
oBJeCTiVes Communications, Classroom
Communication

04 Barriers to Effective Communication

05 Mass-media and Society

M04_MADAN 04_65901_C04.indd 1 21/12/22 10:54 AM


4.2 Chapter 4

A sign is an ‘object’ or figure’ that represents a certain real-


Communication: Meaning, ity for those who interpret it. For example, the red colour
Types and Characteristics is a “stop” sign in traffic.
A symbol represents an ‘idea’ that must be perceived
The word ‘communication’ comes from the Latin word from the senses and that is linked to a convention accepted
communicare, meaning ‘to share’. This word literally by society. A red rose may indicate love and affection.
means “to make common.” Human beings are social ani- According to Vygotsky, symbols are learned within
mals, they have a set of needs that are dynamic. At the a social context, and they need to be internalized, and
most basic level, we need to communicate ourselves to then can be used to share meaning with others, they
fulfill those needs. At its highest level, communication function as mental tools to construct an understanding
means building strong, trusting relationships with people of the world.
whose perspectives are very different from your own. The study of relationships among words is called as
We have a compulsive urge to communicate with ‘syntactics’.
each other for the sake of becoming good citizens and
development. 3. Effective communication always includes feedback.
According to the Oxford Dictionary, ‘Communication 4. The modern age of communication is characterized by
is the transferring or conveying of meaning’. This is the speed, efficiency and the ability to transcend physical
exchange of thoughts, information or feelings by speech, or geographical limitations.
writing or other means of disseminating data. Almost 70% or more of our time is spent in commu-
According to Little, ‘Human communication is the nicating with others. No development is possible without
process by which information is passed between peo- communication. It is an important tool in our personality
ple by means of previously agreed symbols, in order to development also.
produce a desired response.’ The following aspects are Thus, we are going to explore all these facts in this
important. unit. The evolution of mankind is basically the story of our
1. We prioritize our senses in communication, and progress in communication methods.
through our bodily gestures, face expressions, eye In the communication process, the sender (encoder)
contact etc. Listening aspect is equally important in encodes the message and then using a medium/channel,
this world as all people aspire to be listened, may be sends it to the receiver (decoder) who decodes the mes-
for the sake of ‘feedback’. sage, and after processing the information, the receiver
2. Communication is a form of interaction that takes sends back appropriate feedback/reply using a medium/
place through signs and symbols also. channel.

3. A Channel (or
path), such as
2. A Sender sends Giving Information Phone/face-to-
a message face (talk/writing)
(giving or asking is used to transfer
CHANNEL the message.
for information).

1. The
REPLY TO
Communication SENDER MESSAGE
SENDER
process starts (ENCODING) RECEIVER
(DECODING)
with a sender.

CHANNEL

Receiving 5. The receiver


Information replies to the 4. The message
sender. is received by the
receiver.

Figure 4.1 Elements of Communication

M04_MADAN 04_65901_C04.indd 2 21/12/22 10:54 AM


Communication 4.3

aSSertive CoMMuniCation (b) Communicating our opinion and ideas clearly.


These days we tend to get aggressive very soon due to situ- Thus, being transparent and unbiased.
ations, stress etc. But the requirement is to become asser- (c) We need to strike a balance between our needs
tive, that is a healthier communication style. It involves and those of others.

(a) Expressing our views without disrespecting The details of steps of communication process have been
others or ourselves. given on page 18.

Concept Box
Theory of Communication model’ sees communication as a robotic and com-
It is helpful to examine communication and commu- puter-like situation.
nication theory through one of the following view- 4. Systemic: The individuals interact with each other
points: through symbols to create and interpret meanings.
The new messages are created via ‘through put’, and
1. Mechanistic: The perfect transaction of a message then interpreted and reinterpreted as they travel.
from the sender to the receiver. This information is mentioned in Shannon and
2. Psychological: The sending of a message to a Weaver Model also.
receiver and the emotions of the receiver upon inter- 5. Critical: A source of power and oppression of indi-
preting the message. viduals and social groups. Inspection of a particular
3. Social Constructionist (Symbolic Interactionist): theory on this level will provide a framework on the
It sees the product of the interactants sharing and nature of communication as seen within the con-
creating of meaning of message, while ‘transmission fines of that theory.

Feedback is missing in transmission model. This model is


Models of Communication also termed as pipeline or container model. The different
The different models help us in checking the flow transmission models have been discussed as follows.
of information over a period of time. They guide
us towards future as well. A ‘model’ analyses the
communication that takes a mechanistic perspective of Channel
‘human communication’. This tells us about the flow of
Sender Message Receiver
information. Communication is primarily a transmission
process. We need to understand relationship among its Channel
different variables so that it can be made more effective.
Alex Fish has proposed three models of communica-
tion—Transmission (or Linear Model), Interactive and Figure 4.2 Transmission Model
Transactional Models.
Aristotle Model
tranSMiSSion or linear Model This model dates back to 300 BC. This includes three
According to this model, communication has occurred communication elements– the speaker, the subject and
when a message has been sent and received, thus the audience. This model tries to make better and more
communication is through one way only. The three basic persuasive communicator. He gave the term ‘rhetoric’
elements are–channel (medium), sender and receiver. in this context that means ‘speaker’ that is intended to

Speaker Speech Audience Effect

Occasion

M04_MADAN 04_65901_C04.indd 3 21/12/22 10:54 AM


4.4 Chapter 4

persuade. Its main application is in public speaking, 3. Logos: This signifies logic so creating interests becomes
seminars, and lectures. The sender (public speaker, a challenge.
professor, etc.) passes on their message to the receiver
(the audience). Thus, the sender is the only active member The communication process have been five basic elements
in this model, whereas the audience is passive. Aristotle that are speaker, speech, occasion, target audience and effect.
used ‘discovery of the available means of persuasion’ for Lasswell’s Model
defining the whole art of persuasive communication.
Aristotle identified three elements that may help in Here, the communication is the transmission of a mes-
improving communication: sage with the effect as the result that is measurable as
well, with obvious change in behaviour of receiver. This
1. Ethos: This defines the credibility, authority and is dependent upon on five questions that have been put in
power of the speaker in a field of their choice. the diagram below. If any of these element change, their
2. Pathos: This connects the speaker with the audience ‘effects’ also changes.
through different emotions (anger, sadness, happi- The main components of this model are communicator,
ness, etc.) message, medium, audience (receiver) and effect.

This Lasswell model was represented by Michael Buhler

Situation WHO TO WHOM


depending on the
human sciences
Transmitter Receiver

BY WHAT
CHANNEL

Situation nearing SAYS WHAT WITH WHAT EFFECTS


the physical
sciences Message Influence reply
Stimulus

Shannon and Weaver Shannon and Weaver talked about the following con-
Claude Shannon and Warren Weaver in 1949 provided cepts, though in technical terms.
a visual mode of communication system in relation to 1. Entropy: Entropy is the degree of disorder or uncer-
electronic media. That was in context of ‘telegraphed tainty or unpredictability or randomness in a message;
communication. This model is referred to as engineering thus, negative entropy is the higher degree of order in
model or ‘mathematical model of communication’. They communication (that means less disorder). A highly
aimed to discover which channels are most effective for ‘organized message’ has low entropy. We can also say
communicating. This model was the first one to highlight that entropy means a process in which order deterio-
the role of ‘noise’ in communication, which can disrupt or rates with the passage of time. The growing entropy
alter a message between sender and receiver. (disorder) is the positive entropy. The more entropy
They talked about the three levels of issues in the com- leads to signalling errors. This helps us in choosing a
munication of information–technical (signals), semantic message, which message should be picked up, keep-
(interpretation of meaning) and influential (effectiveness). ing desired entropy in mind. The ratio of the actual to
Though all the explanation on process of communica- the maximum entropy is called relative entropy.
tion, it remained a linear, one-way communication model 2. Redundancy: This word basically means ‘overflow’
without any emphasis on feedback. or “more than necessary”. They convey highly

M04_MADAN 04_65901_C04.indd 4 21/12/22 10:54 AM


Communication 4.5

predictable (extra) information to the receiver. 3. Channel Capacity: The amount of information that can
Avoiding redundancy may help in better be transmitted per unit of time. Beyond this optimum
communication. High redundancy means that it level of information, communication will have some
requires almost no effort or less efforts to decode errors.
the messages. For example, instead of saying   This model is applied more in interpersonal com-
Twelve midnight, the word midnight is sufficient; munication than in group communication and mass
use ‘repeat’ instead of saying ‘repeat again’; use communication. Thus, the sender plays the primary
innovation instead of saying ‘new innovations’. role of sending messages, and the receiver plays the
Twelve, again and new words are not required and passive part. Feedback is taken as less important in
information should be concise. comparison to the messages sent by the sender.

Source Transmitter Receiver Destination


Message Signal Received Message
signal

Noise source

Figure 4.3 Shannon-weaver Information Model

Berlo ’s S-M-C-R model qualification), social system and culture (including cross-
Berlo tried to explain communication as S–R Model or cultural communication).
sender–receiver model, where sender stands for stimulus The key factors in communication are content
and receiver stands for response. This was later extended to (script), elements (speech itself, gestures, body lan-
Source, Message, Channel, and Receiver (S-M-C-R Model). guage, facial expressions), treatment (of message by the
This is a linear model of communication; there is no source itself), structure of message, and code (verbal
inclusion of feedback. There is no noise as well, and so no and nonverbal).
concept of barriers in communication. The hearing, sense of taste, sense of sight, smelling,
The communication depends upon communica- touching, all these linked with five sense organs are also
tion skills, attitude (to win), knowledge (depends upon important for conveying the messages.

Berlo ’s Model of Communication

S M C R
Source Message Channel Receiver

Communication Seeing Communication


skills Elements Structures skills

C C Hearing
Attitudes Attitudes
O O
N D Touching
Knowledge T E Knowledge
E
Smelling
Social N Social
system T system
Tasting
Culture Culture

Figure 4.4 S-M-C-R Model

M04_MADAN 04_65901_C04.indd 5 21/12/22 10:54 AM


4.6 Chapter 4

I nteractive Model of Communication


According to Schramm (1997), “The participants alter- Encoder Decoder
M
nate their positions as sender and receiver and generate
meaning by sending messages and receiving feedback Interpretor Interpretor
within physical and psychological contexts”. This model
M
incorporates feedback, which makes communication Decoder Encoder
a more interactive, two-way process. This model is also
known as ‘convergence model’. Field of experience is a Source Receiver
communication pattern. It is affected by alteration factors
such as culture, social, psychology, situation and channels
used. The overlapping field of experiences initiates con- This model is a circular model of communication, in which
versation and more conversation, that in turn, expands messages go in two directions.
the communicator’s field of experience. All these factors There are four principles in this model:
also affect the message interpretation. The Interaction 1. Communication is circular— The individuals involved
Model takes physical (environmental factors) and psy- change their roles as encoders and decoders.
chological (mental factors such as stress and anxiety and 2. Communication is equal and reciprocal— both par-
other emotional factors) factors into account. ties are equally engaged as encoders and decoders.
3. The message requires interpretation— the informa-
Channel tion needs to be properly interpreted to be understood.
Sender Message Receiver 4. There are three steps in the process of communica-
and and tion: Encoding, Decoding and Interpreting
Receiver Message Sender
Channel Katz’s Model
Robert Katz talked about the role of ‘gatekeepers’ or ‘inter-
Figure 4.5 Interactive Model of Communication mediaries’ or ‘censor groups’ in his work ‘Intermediary
Model of Communication’ in 1957. These gatekeepers
Osgood-Schramm Model have some ability to shape the organization through their
This model looks at reciprocal communication that shows selective sharing of information. He talked about tech-
how we have to encode, decode, and interpret informa- nical skills, human skills and conceptual skills in case
tion in real-time during a conversation. Charles Osgood of sharing of information. Gatekeepers are the persons
describes communication as a dynamic process and says working in different mass media, having the responsibil-
that a given communication event may begin with receiv- ity of deciding what should get printed, broadcasted or
ing stimuli. produced.

Speaker Gatekeeper Audience

An intermediary model
Figure 4.6 Katz’s Model

Schramm Model of Communication Weaver’s model to human communication and introduced


Wilbur Schramm (1964) talked about the role of ‘Field two concepts of encoder, decoder, redundancy, feedback
of Experience’ (type of orientation or attitudes) in his and noise into his model to explain the communication
communication model. Schramm adapted Shannon and process.

Field of experience Field of experience

Source Encoder Signal Decoder Destination

Figure 4.7 Schramm Communication Model

M04_MADAN 04_65901_C04.indd 6 21/12/22 10:54 AM


Communication 4.7

Newcomb’s Model (1953) object of the orientation (background, culture, and


Communication takes the shape of a triangle that has beliefs) of sender and receiver of messages. The role of
three points – sender, receiver and society. Thus, it tries to feedback is also significant.
explain the role of communication in a society or a social This model consists of nine crucial components—
relationship. According to it, communication maintains environment, sensory experience, source/Sender,
equilibrium within the social system and works like this. the object of the orientation of the source, receiver,
the object of the orientation of the receiver, feedback,
Westley and Maclean Model gatekeepers and opinion leaders.
This model is seen as an extension of Newcomb’s Model.
The messages in mass communication pass through George Gerbner’s Model of Communication (1956)
different check points called as ‘gatekeepers’ before they Here, the communication is seen as a transmission of mes-
are actually received by the audience. The communication sages. It relates the messages to ‘reality’ and thus enables
is influenced by environmental, cultural and personal us to approach the question of perception and meaning;
factors, its process starts with environmental factors, and further it sees the communication process as consisting of
not with the source or sender. The culture or society where two alternative dimensions—the perceptual or receptive
the speaker lives in and his space (public or private) play dimension and the communicating or means and control
an important role. Thus, this model takes into account the dimension.

Sender Message Treatment Channel Receiver

Figure 4.8 General Communication Model

Transactional Model of Communication 1. Cues: Here, the role of ‘cues’ is highlighted. Cues refers
This model was initially proposed by Watzlawick. Here, to the signs for doing something. There are public cues
the communication is integrated in our internal realities. (physical, environmental or artificial and natural or
The context, non-verbal communication and feedback man-made), private cues (objects of orientation which
have been added to transactional model. Here, the focus include senses of a person) and behavioural cues.
is on conceptualization of communication. Instead of These cues can be verbal as well as non-verbal. Jagged
participants being tagged as senders and receivers, the lines show us that availability of cues can be unlimited.
people in a communication encounter are considered to 2. Speech act: This refers to particular instance of com-
be communicators. The meaning lies is in people, not in munication in the model.
words, thus, it creates shared meaning. The communication 3. Filters: These are the realities of people engaged in
includes both content and relationship dynamic. communication. They can vary as per the cultures,
traditions, content of the message, etc.
nnel of feedba 4. Noise: This problem arises in communication flow
Field of Cha ck Field of and disturbs the message flow.
experience experience
5. Shared field experience: That kind of experience is
Channel
between sender and receiver.
Sender Receiver
and Message and The sending and receiving of messages happen simulta-
Receiver Sender neously between people. This model was further modified
Channel as General Transactional Model, thus shifting its focus
from being linear to dynamic and two way communica-
tion model. This model is taken as the most systematic
Ch ck model of communication.
a n n el f fe e d b a
o
Leagan’ s Model (1961)
Figure 4.9 Transactional Model
This model is specifically used in the ‘extension edu-
Some models of interactional communication have been cation’. The successful communication in it requires a
discussed below. skilled communicator sending a useful message through
a proper channel, effectively treated to an appropriate
Barnlund’s Transactional Model of Communication audience. The response is given as desired. These key
(1970) elements involved in his model are: communicator, mes-
This model says that communication exchange is recipro- sage or content, channels of communication, treatment of
cal. The following terms are important in this model. message, the audience and audience response.

M04_MADAN 04_65901_C04.indd 7 21/12/22 10:54 AM


4.8 Chapter 4

Helical Model of Communication Craig proposes seven different traditions as listed below.
Frank Dance proposed a communication model inspired 1. Rhetorical: The practical art of discourse
by a helix in 1967, known as helical model of communi- 2. Semiotic: The mediation by signs
cation. A helix is a three-dimensional spring-like curve in 3. Phenomenological: The experience of dialogue with
the shape of a cylinder or a cone. This model sees com- others
munication as a circular process that gets more and more 4. Cybernetic: The main flow of information
complex as communication occurs, which can be repre- 5. Socio-psychological: The interaction of individuals
sented by a helical spiral. The model is linear as well as 6. Socio-cultural: The production and reproduction of
circular combined, and disagrees with the concept of lin- the social order
earity and circularity individually. 7. Critical: A process in which all assumptions can be
challenged.

Transactional and I nteractional Models
Brown and Yule (1983) use two terms to describe the major
functions of language. The functions of language where the
transfer of information is involved is called ‘Transactional’,
and the function involved in expressing social relations
and personal attitudes is called ‘Interactional’.
1. A: There’s no message for you (Transactional)
B: Ok
2. A: How are you? (Interactional)
B: Fine, thank you.

Figure 4.10 Helical Model of Communication Stopover


1. Which of the following is an exclusive example of
Rogers and Kincaid Model (1981) non-verbal cue that includes the pitch, rate, volume
This model is called as ‘convergence model of communi- and use of pauses ? (December 2021)
cation’. This focus on the importance of information and (a) Linear Model of Communication
also on the manner in which information links individu- (b) Interactional Model
als together in social networks. Here, communication is (c) Non-verbal cues
defined as a process in which individuals create and share (d) Para-Language
information with one another in order to reach mutual The correct option is (d).
understanding.
2. Which of the following is Berlo’s Linear Model of
Becker’s Mosaic Model (1968) communication?
(a) S-M-R-C (b) S-M-C-R
Sam Becker proposed this communication model in “The
(c) S-R-M-C (d) S-R-C-M
Prospect of Rhetoric” in 1968. The model explains the
complexity of human communication. There is lot of The correct option is (b).
‘randomness’. There are immense number of fragments 3. If it is assumed that communication has no beginning
or bits of information on an immense number of topics or end, then it is termed as
in a mosaic. These bits are scattered over time and space (a) Mediation (b) Process
and modes of communication. This model is multi-lay- (c) Interaction (d) Transaction
ered. The correct option is (b).
1. Empty cells: Unavailable messages or sources 4. Which is ‘feedback’ in newspaper’s communication?
2. Vertical layers: Set of similar messages (a) Articles
3. Cells: Messages and sources (b) Editorials
4. Receivers go through the cells in loops every time. (c) Letters to the Editor
(d) News
Constitutive Metamodel The correct option is (c).
It is a communication model which proposes that commu- 5. Given below are two statements:
nication creates our social world. Communication creates
and produces our social world. This is a driving force in Statement I: Signs and symbols have internal rela-
our lives, in our relationships. Communication is the cen- tions to produce meanings
tral activity in our lives that creates all other social forces Statement II: This is possible because of a network of
in society. signs through such relations

M04_MADAN 04_65901_C04.indd 8 21/12/22 10:54 AM


Communication 4.9

In light of the above statements, choose the most


Classification on the Basis of
appropriate answer from the options given below
R elationship Element
(a) Both Statement I and II are true.
(b) Both Statement I and II are false. Intrapersonal Communication
(c) Statement I is true but Statement II is false. 1. It is communication within an individual almost all
(d) Statement I is false but Statement II is true. the times, including speaking to oneself, listening to
The correct option is (a). oneself and relating to oneself.
2. It includes individual reflection, meditation,
­contemplation and even praying to God.
3. We conceptualize and formulate our thoughts or ideas
before actually indulging in overt communication.
Mass Media Model Muttering the words such as ‘Oh my god’, ‘Oh no’ (when in
We have mass media in our syllabus. It has been discussed trouble), ‘Wow’ and ‘Thank god’ are a few common exam-
under communication types. Here we want to discuss ples of intrapersonal communication.
a communication model called the ‘Attention-gaining
Model” or “Attention Model”. It is an essential commu- Interpersonal Communication
nicative activity of mass media to attract and keep atten- 1. It is also termed as dyadic communication.
tion. Communication has three models – Transmission, 2. It is universal form of face-to-face routine communi-
Expression/Ritual and Attention-gaining. cation between two people, both– while sending and
receiving messages.
Mass Media Sender’s Receiver’s 3. It may be formal or informal, verbal or non-verbal.
Model Orientation Orientation 4. It takes place anywhere by means of words, sounds,
facial expression, gestures and postures.
Transmission Transfer of Cognitive 5. It is an effective communication situation because
meaning processing you receive immediate feedback.
6. Due to the proximity between the sender and the
Expression/Ritual Performance Shared receiver, interpersonal communication has an emo-
Experience tional appeal too. It can motivate, encourage and
coordinate work more effectively than any other form
Attention-gaining Display Spectatorship of communication.
7. The efficacy of interpersonal communication depends
very much on the mutual relationship between two
1. Transmission Model: This model is mostly linked partners in communication, their s­ tatus, roles and skills.
with institutional contexts, such as education, reli- 8. It has greater scope for grapevine.
gion and government. It is, however, not relevant to
most media activity. Interpersonal communication can be further subdivided
2. Ritual Model: The ritual model shares some elements into formal (meeting and conference) or informal (pri-
with the transmission model, but emphasises more on vate discussion with family members or friends) commu-
external interpretation by observers than the stated nication.
purpose of receivers and senders.
3. Attention gaining Model: This model is to attract Group Communication
audiences. Audiences view media as escapist and Group communication situations are quite common in
diversionary. Thus, it is in conflict with the above two day-to-day life. Our educational background, profession,
models. Media culture and practice have to develop economic status, religious affiliation, etc. distinguish us
close links with attention-holding goal. Attention and make us belong to one or more groups at the same
is measurable in the time allotted. The intensity of time. A group is a number of people with a common goal
involvement depends upon the type of contents. The that interact with one another to attain the goal that
mass communication market is mainly concerned recognize one another’s existence, and see themselves
with attention more than anything else. This is what as part of the group. An individual may belong to more
sells the this medium to the advertisers. than one group simultaneously. Since the large majority
of us do not live alone, we consequently live in groups –
all kinds of groups. Groups may be of two types:
Types of Communication 1. Primary Groups
2. Secondary Groups
We communicate with each other in different manners
that totally depends upon the message and context in According to C. H. Cooley, primary groups are com-
which it is being forwarded. The choice and style of posed of individuals with intimate, personal relations
channel also affects communication. and who interact face to face, figuratively and not

M04_MADAN 04_65901_C04.indd 9 21/12/22 10:54 AM


4.10 Chapter 4

literally. It is the degree of intimacy or social distance


rather than physical distance that determines the pri- Mass Media
mary group. The family is considered a primary group. Mass usually means a large number. Mass has both positive
The other examples of primary group are play groups, and negative meanings.
kinship groups, labour groups, clan, etc.
Secondary groups are those in which individuals have 1. In socialist tradition, mass has a positive meaning.
formal, impersonal and status relationships. All other It connotes the strength and solidarity of ordinary
groups which are not primary are considered secondary. working people when organised together for social
Groups may be statistical (demographic arrange- and political ends.
ments), societal (common consciousness), social (actual 2. In the negative sense, mass refers to the ignorant and
association) and associational groups (formal structure). unruly mass that implies a lack of culture, intelligence
Group communication is interpersonal communication. and even of rationality.

P Mass media is not a mere physical transmission of infor-


mation, but a complex, integrated, intermixed and interac-
P P tive system that has resulted in the seamless integration of
data, text, images and sound within a single digital infor-
mation environment, known as Multimedia. Mass media
is also widening its scope to include not only conventional
Group Leader = GL
P GL P paper-print media but also a variety of audio-visual and
Participants = P electronic media. Public relations and advertisements also
form parts of this field. Now mass media is regarded as the
fourth estate that shapes, influences and indirectly gov-
P P erns public affairs in a democratic set-up.
Masses are geographically distributed. With moderniza-
P
tion, the complexity of the society has increased further. A
crowd is a spontaneous collection of individuals. It is tem-
Figure 4.11 Group Communication porary and never reappears with the same composition.
Characteristics of a group in relation to an individual
Four Ways to represent Media
on the basis of membership are dependence, acceptance,
attraction, volition (voluntary basis), innate (by birth), There are four ways to look at audience in mass media:
pressure (confirm to certain standards), change and flex- 1. Media Reach: The owners and producers of the mass
ibility (no group is rigid and static) and leadership. media conceive the total population to whom their
The leader leads the group in two manners: communications can reach.
1. Task-oriented Roles: Initiation of discussion, giving 2. Media Access: Mass media may be available but the
and receiving information, elaboration and clarifica- capacity or willingness to use the media may not be
tion, orientation and summarization, tests of ­consensus there.
2. Maintenance Roles: Compromising, supporting and 3. Media Exposure: Only these individuals who actu-
encouraging, gate-keeping, standard-setting and testing. ally expose themselves to the media are the media
Leadership can be democratic, authoritarian or audience.
­Laissez-faire (creative and committed people). 4. Media Effects: Media effects individuals who have
been exposed to mass communication products and
The degree of formality governing the ‘jurisdiction’ of have undergone a change in their knowledge, opin-
the participants in group communication activity ­differs ions, attitude or behaviour.
according to the ‘context’.
Group communication is influenced by several Main Characteristics of Mass Media
­heterogeneous factors, such as age, sex, education and At a different level, mass communication has three char-
economic, social, linguistic, religious, national, regional acteristics.
and racial differences.
The careful balance of interaction in dyad no longer 1. The Audience is Large and Heterogeneous: such as
exists in group communication. According to Baker, the radio, newspapers and televisions.
best size in terms of total interaction and greatest effi- 2. The Source is an Institution or a Group of ­People:
ciency is between 5 to 7 members. Participation and shar- They may be in public sector or private sector.
ing of information is central to the functioning of a group. 3. Some Kind of Mechanism is Used to Reproduce
A group can function on the basis of committees (small Information:
groups for a specific purpose) or conferences (large group (a) It is also termed as ‘mediated communication’.
and more ­formal). Group communication is considered (b) It deals with mass audience such as large groups,
effective as it provides an opportunity for direct interac- state or nation.
tion among the members of the group and helps in bring- (c) Channels for mass communication are termed as
ing about changes in attitudes and beliefs. mass media.

M04_MADAN 04_65901_C04.indd 10 21/12/22 10:54 AM


Communication 4.11

Decision-makers

Mass Media

Treetops Tactics

Society Grassroots Tactics

Figure 4.12 Mass Media Model

(d) It uses mechanical devices that multiply mes- 1. Print: Books, pamphlets, newspapers, magazines, etc.
sages and convey information to a large number from late 15th century onwards. Galleys or ‘bound
of people simultaneously. proofs’ are used for promotional purposes.
The concept of mass media has become popular during 2. Recordings: Gramophone records, magnetic tapes,
20th century. Technology also helped as it allowed the cassettes, cartridges, CDs, and DVDs from the late
massive duplication of material, for example, nationwide 19th century onwards
radio networks, mass circulation newspapers and maga- 3. Cinema: Cinema in India began from 1900 onwards
zines. Physical duplication technologies, such as printing, 4. Radio: Radio came into existence since from 1910
record pressing and film duplication allowed the dupli- onwards
cation of books, newspapers and movies at low prices to
5. Television: Television began from 1950 onwards
huge audiences. Radio and television allowed the elec-
tronic duplication of information for the first time. 6. Internet: Use of internet started from 1990 onwards
Mass media is based on the economics of linear repli- 7. Mobile phones: Mobile use began from 2000
cation – a single work could make money proportionate onwards
to the number of copies sold. As sales volumes went up,
units costs came down, increasing profit margins further. The Internet and mobile phones are known popularly
Recently the Internet, social media, podcasting, blog- as ‘digital media’. The digitization of the news industry
ging, etc. have been added. The terms ‘public media’ has led to a compression of time and space, which led to
or ‘mainstream media’ carries very similar meaning. easy reach of distant news to the public within a blink of
‘Mainstream Media (MSM)’ includes those outlets which an eye. For example, we get to see news images of any
are in tandem with the prevailing direction of influence in ­demonstrations, riot, coups or even a war within minutes
the culture at large. of occurrance of these incidents in any part of the world.
MSM is also popular as ‘drive by media’ as Rush Electronic media and print media specifically include:
Limbaugh did this. It may make it difficult to decipher the 1. Broadcasting
difference between truth and untruth.
2. Various types of discs or tape
Again ‘mediated communication’ and ‘Mass media’ are
considered to be synonymous. The media is known as the 3. Film
fourth pillar of democracy. The audience is large and hetero- 4. Internet: Blogs and podcasts, etc.
geneous. The source is an institution or a group of people. 5. Publishing: Newspapers, books, magazines and
Under Article 19 of our constitution, everyone has the newspapers. Publication is important as a legal
right to freedom of opinion and expression; this right concept.
includes freedom to hold opinions without interference 6. Computer games
and to seek, receive and impart information and ideas
through any medium regardless of frontiers. Few Popular terms
Let’s look at a few popular terms related to mass media:
Evolution of Mass Media Gate Keepers: This refers to persons working in dif-
In India in the 19th and 20th century, a classification ferent mass media. They have the responsibility to
called the ‘seven mass media’ became popular as men- decide such as what should get printed, produced and
tioned below: broadcasted.

M04_MADAN 04_65901_C04.indd 11 21/12/22 10:54 AM


4.12 Chapter 4

Entropy: The tendency of communication system 1. It provides financial support to mass communication;
to move from a state of order to disorder or chaos. 2. Advertisements are usually mass based and hence all
Everything seems to be more chaotic when media is expertise required for mass communication are appli-
highly influential. cable here; and
New Wave: An experimental style of film-making, 3. Advertising is a pervasive part of modern culture, a
popularised first in France in the fifties; New wave medium to build up images of a society.
films are without any elaborate plot and chronological
continuity and is generally free from the atricalities, Impact of Mass Media on Society
but there is an originality with regard to treatment. Let’s look at the main impacts of Mass Media on society:
Prime Time: The time during which the media have 1. The mass media educates people about the world out-
their largest audience. side of their immediate context.
Yellow Journalism: This is a practice followed by the 2. It functions as an important accountability index. We
media that sensationalize a story. It is a type of jour- can discuss concepts such as advocacy, surveillance
nalism that does not report much of real news with here.
facts. It uses shocking headlines that catch people’s 3. Mass media promotes cultural diversity.
attention to sell more newspapers. It pushed Spain As per one’s perception, mass media can be used for
and USA into war in 1898. the following purposes:
Alternative Media: It is also a type of ‘mass media’.
1. Advocacy–for business and social issues such as
Alternative media use technology capable of reach-
advertising, marketing, propaganda, public relations,
ing many people, even if the audience is often smaller
and political communication.
than the mainstream.
2. Enrichment, socialization and education–such as
Participatory Media: The means of production are literature
widely available, and content creation is not based 3. Entertainment–acting, music, light reading, com-
on traditional editorial structures. Anyone with a cell puter games, etc.
phone camera and Internet access to participate can 4. Journalism–an activity of gathering, assessing, creat-
participate in the activity of journalism. This creates ing, and presenting news and information.
both new opportunities for information production 5. Public Relations–used to build positive image of an
and a more complex information environment. organization.
Sousveillance: It means the recording of an activity
This is performed by corporations, non profit organiza-
from the perspective of a participant in the activity.
tions, politicians, etc.
The participants possess small portable or wearable
recording devices that often stream continuous live
video on the Internet. Mass Media Functions by Harold Lasswell
Embedded Journalist: These journalists are inserted Harold Lasswell has considered three functions in context
into military units by the governments. of mass media in any society:
Multimedia: A complex, integrated, intermixed and 1. Surveillance: As an ‘informer’, ears, eyes and voice of
interactive system that has resulted in the seamless the audience and for shaping our thoughts, attitudes
integration of data, text, images and sound within a and actions. For instance, the advertisements make us
single digital information environment. aware about the new products.
Cliché refers to an expression that has been overused
Public Relations: This takes up the role to inform, to
to the extent that it loses its original meaning or nov-
resolve conflicts and/or to improve understanding
elty. A cliché may also refer to actions and events that
between persons. As demographic pressures have
are predictable because of some previous events. All
become more and more complex, communication
examples of cliché are expressions that were once
among groups and individuals has become charac-
new and fresh.
terised by a very complicated and involved process
2. Correlation: Developing public opinion, thus helping
that led to the development of ‘public relations’.
to create consensus in a society on key issues.
Advertising and Advertisements (ADS): 3. Admission of Social Inheritance: Present media have
Advertisements are forms of communication that assumed the functions of transmitting social inheritance,
involve creativity, persuasion and impact of the mes- which was done by parents, teachers and other elders.
sages carried by them. Advertisements specially prod- The media today provide main frames of reference to soci-
uct advertisements describe the features at the prod- ety. Urbanization, relative anonymity, social uprootings,
ucts or services, usefulness and also compare with and the transition from traditional social organizations
other products or services. These contain useful infor- like joint family, clan have increased the role of media as
mation and latest developments. transmitters of knowledge and values. Mass media have
Advertising is considered part of mass communica- today become essential to carry out certain functions of
tion for three reasons: socialization and transmission of social heritage. The

M04_MADAN 04_65901_C04.indd 12 21/12/22 10:54 AM


Communication 4.13

following are also listed as functions performed by mass relationships, and not mass media, had an enormous
media: influence on voters.
(a) Public opinion to create ‘cultivation of perceptions’. Two-step Flow: Many voters had limited exposure
(b) To set up political agenda that is called as ‘agenda to the mass media. The information they received
setting’. was mostly through other people, called “opinion
(c) Setting up link between the government and leaders”, who had first-hand access to mass media
people. information.
(d) Mass Media fills ‘Knowledge Gap’ also so as to Multi-step Flow: Multistep means opinion leaders
seek information. adopt the multi-directional influence–they interpret
the media messages for audiences. The role is not only
Main Theories of Mass Media ‘downwards’ but upwards also when they tell the gate-
This discussion has become important as NTA NET JRF keepers (editors of newspapers and so on) how to do
is asking many questions on theory as is being done in their job. Also, the influence is “sideways” when they
Unit 1 as well. shared insights with other opinion leaders. There are
1. Hypodermic or Bullet Theory: This theory reflects multiple relay points.
the fear or awe of mass media being used for massive 5. Sociological Theories of Mass Communication:
propaganda. It happened during World War II, when The sociological approach to communication theory
mass media was thought to have direct and power- is based on the assumption that there exists a defi-
ful influence upon audiences. The audiences may be nite relationship between mass communication and
manipulated by the media at will. The messages were social change (societal values, public beliefs and
thought to be like magic bullets that were shot directly opinions, etc.)
into the receiver. It assumes that receivers are passive Cultivation Theory: This theory, developed by George
and defenseless and take whatever is shot at them. Gerbner (1967), is based on the assumption that mass
2. Psychological or Individual Difference Theory: media have subtle effects on audiences who, unknow-
There is a psychological approach to understanding ingly, absorb the dominant symbols, images, and mes-
communication effects. Individual difference theory sages in the media. This is called as ‘cultivation of dom-
is among them. inant image pattern’. Powerful effects of mass media
Here, the different personality variables result in dif- act as moulds for the society.
ferent reactions to the same stimuli. In other words, an Agenda Setting Theory: Set up by Maxwell McCombs
individual’s psychological mechanism accounts for his and Donald L. Shaw (1972), the theory focuses on
reactions to media messages. Here, selective exposure about voting during elections. Media are more suc-
and selective perceptions are important. cessful in telling people “what is to think about” than
(a) Selective Exposure: Selective exposure occurs in telling them “what to think”.
when people tend to expose themselves selec- The Uses and Gratification Theory: This was set up
tively only to communications which are in gen- by Katz (1959). The “uses” approach assumes that
eral accordance with their established convic- audiences are active and willingly expose themselves
tions and avoid communications which seem to to media. The uses of mass media are dependent on
challenge their beliefs. the perception, selectivity, and previously held beliefs,
(b) Selective Perception: Once individuals have values, and interests of the people. The term “gratifica-
selectively exposed themselves to the messages tion” refers to the rewards and satisfaction experienced
in accordance with their preferences, they tend by audiences after using media.
to “read into” the message that suits their needs. Dependency Theory: This was proposed by Melvin
This process is called selective perception. De Fleur and Sandra Ball-Rokeach, in which they rec-
3. Cognitive Dissonance Theory: The psychological ognise various psychological and social factors that
conflict, which is the result of contradictory thoughts, prevent the media from exercising arbitrary control
is called cognitive dissonance. For example, a person over their audiences.
likes a Apple’s Ipad but he does not support the price Spiral of Silence Theory: The media publish opin-
of the product. Thus such every person may look for ions. The people modify or adjust their views accord-
balanced beliefs, thus wants to avoid cognitive disso- ingly to avoid being isolated. Here, free expression
nance. Therefore, we may always try to refrain from becomes automatically important. There are always
hearing any contrasting views. some people who don’t express themselves. These
If perceptual difference is due to our social group sta- people, with a suppressed point of view, are called as
tus, then it is called ‘standpoint theory’. ‘minorities’.
4. Personal Influence Theory: This theory is the out- Diffusion of Innovation Theory: This theory was
come of a classic study of the 1940 presidential elec- developed by Neil Gross and Bryce Ryan in 1943.
tions in USA by Paul E. Lazarsfeld and others in the This theory stresses the process by which different
book, The People’s Choice. The findings revealed that channels communicate new ideas to different people.
no voter seemed to have been directly influenced The various factors influence people’s thoughts and
by the mass media. It turned out that interpersonal actions about the original design or technology.

M04_MADAN 04_65901_C04.indd 13 21/12/22 10:54 AM


4.14 Chapter 4

Cultivation Theory: The media influences a person’s are deemed to be integrated into the sources of social
sense of reality. Most of us are not able to acquire power and authority. This theory gives a primacy to the
experience by the direct medium, for the simple rea- media as the cause and maintainer of mass society.
son that we cannot be present at every news that is 2. Political Economic Media Theory: This theory is
happening around the world. linked with Marxist theory, and the focus is on the
Confirmation bias Theory: All of us may perceive economic structure of the media rather than the ideo-
and interpret the world in a different manners. The logical content of the media.
reinforcing thoughts are accepted, while others are 3. Hegemony Theory: This theory concentrates less on
rejected. the economic system and more on the ideology itself.
6. Normative Theories of Mass Media: Normative Here, a greater degree of independence of ideology
theories explain how the media ‘ought to’ or can be from the economic base is emphasised.
‘expected to’ operate under the prevailing set of polit- 4. Culture and Semiotic Theory: The linguist Saussure
ico-economic circumstances. (1935) stressed on texts and its meaning in the light
Authoritarian Theory: The term was proposed by of the “host” culture. It is thus concerned with the
Siebert. It refers to an arrangement in which the press explanation of cultural as well as linguistic meaning.
is subordinated to state power and the interests of the Semiotics is the general science of signs, developed
ruling class. Press and other media should always be by the philosopher and logician Peirce (1931–35),
subordinate to established authority and should do which deals with how signs signify.
nothing to undermine it. This theory justifies advance Semiotic theory, thus, is not concerned with transmis-
censorship and punishment for deviation from exter- sion of message as is the case in process approach, but
nally set guidelines. In India, the press, which is with the derivation or transfer of meaning in commu-
free, lost its independence and freedom during the nication.
Emergency (1975-77). 5. Marshall McLuhan’s Impact of Media Theory:
Free Press Theory: This theory is also called as McLuhan linked major historical shifts, social trends,
“Libertarian Theory”. It was mainly promoted by psychological and sensory orientations on commu-
Siebert. It is based-on the fundamental right of an nication media and technologies. The central idea
individual to freedom of expression, which is regarded behind his saying “the medium is the message” is that
as the main legitimating principle for print media in the medium through which content is carried plays
liberal democracies. a vital role in its perception. The message delivered
Social Responsibility Theory: This is linked with through social media such as Facebook, Whatsapp
Commission on Freedom of Press (Hutchins 1947) in may be perceived differently when the same message
USA. The free market has failed to fulfill certain obliga- is conveyed through newspaper. He also gave the
tions, such as information, social and moral needs of concept of ‘global village’ that stated that masses can
the society. In fact, it has increased the power of a single unite to effect changes. He also gave the concept of
class. ‘dramatized society’.
Communist Media Theory: This theory is derived
mainly from the basic tenets of Marx and Engels. It Impact of Mass Media on Society
envisages media to be under the control of the working Denis McQuail (1983) proposed the relationship of mass
class, whose interest they are meant to serve. Private media and society as an interactive perspective, that mass
ownership of the press or other media is ruled out. The media are primarily molders of society as well as reflectors.
term ‘propaganda’ that means ‘agitation’ was given by The industrial and communication revolutions have
Georgy Plekhanob and later defined and popularized changed the face of our society. The impact of mass media
by Vladimir Lenin in 1902. has to be seen in the background of our tradition - and
Development Communication Theory: This theory value-bound social structure.
is mostly based upon the UNESCO Report (1980). Two social scientists, Bernard Beralson and Monis
Press has limited application in four fields and com- Janowitz, shared information about mass media in the
munication is used to carry out development tasks in following way:
line with nationally established policy. “The effects of communication are many and diverse.
Democratic-Participant Media Theory: This theory They may be short-termed or long-termed. They may be
favours three things – multiplicity of media, small- manifest or latent. They may be strong or weak. They may
ness of the scale of operation and a horizontality of derive from any number of aspects of the communica-
communication at all levels. It opposes uniform, cen- tion content. They may be considered as psychological or
tralized, high-cost, highly professionalized and state- political or economic or sociological. They may operate
controlled media. upon opinions, values, information levels, skills, taste, or
over behaviour.”
Few more important theories of Mass Media:
The growth of mass communication has made it pos-
1. Mass Society Theory: According to Denis McQuail, sible for us to get far more information today than before.
this theory emphasises the inter-dependence of institu- Information is indispensable in a complex, advanced
tions that exercise power in the society, and mass media society.

M04_MADAN 04_65901_C04.indd 14 21/12/22 10:54 AM


Communication 4.15

Information today is a commodity we are willing to pay Verbal Communication


for. The mass media today are not only entertaining the Verbal means the use of words in the communication pro-
masses, they are selling information as well. Information cess and in designing and formulating messages. In verbal
is power. Mass media sense our needs. Mass communica- communication, a message is transmitted verbally, i.e., by
tion is too essential to be ignored. We have print journal- making use of words, such as oral and written.
ism, radio, film, TV, video, cable and satellite TV. In verbal communication, remember the acronym KISS
India, towards the end of the 20th century, still largely – keep it short and simple.
remains an oral society. We spend more time communi- In order to deliver the right message, the communica-
cating interpersonally rather than through the channels tor must be empathetic. Verbal communication is further
of mass communication. The situation in the West is dif- divided into the following types:
ferent. People spend more time in consuming mass media
information. 1. Oral communication
Among the concerns faced by Indian society, there are 2. Written communication
five major ones are:
Oral Communication
1. Manipulation: This is actually the altering the
The spoken words are used in this communication.
meaning of recorded visuals and audio materials.
It includes face-to-face conversation, speech, telephonic
2. Privacy: This is actually the issue of linking privacy
conversation, video, radio, television and voice over the
issues with the public interest.
internet. In oral communication, the communication is
3. Security: Protection of secret government informa-
influenced by the following factors:
tion, private financial transactions and institutional
records in computer systems has become a matter of 1. Pitch and Volume: Pitch is the degree of highness or
concern. lowness of tone, and it depends upon the frequency
4. Democratic Process: The voting rights of voters are of sound waves. It is the key element in the teaching-
manipulated by using different means. learning process. The teacher’s voice and knowing
5. Isolation: While the communication revolution the correct language is the main input in speech. One
has the power to draw the global community closer, should be loud enough to be heard. When speaking,
simultaneously, it may actually result in the isolation one should remember to change the pitch in your voice
of individuals into small groups. This leads to the as a monotonous voice may become boring. Excitement
fragmentation of society. is indicated by a high pitch and anger by a low pitch.
Volume depends upon the proximity and number of
people and is measured in decibels.
Concept Box 2. Rate: It is the speed at which words are delivered.
One should maintain regularity when speaking,
Communication can also be categorized on the basis of
because a regular or rhythmic voice makes you sound
involvement of parties, such as
more confident. Irregular speech may show a sign of
1. Intrapersonal: Reading a newspaper, meditation, uncertainty. If a teacher talks at a slow pace, then the
introspection. students may become frustrated because the informa-
2. Interpersonal: Conversation with a colleague tion is not being given quickly enough.
3. Apersonal: Use of mass media, publicity, 3. Clarity in Speaking and Articulation: Proper pronun-
advertisement ciation and delivery of words contribute towards the
effectiveness of the message. It also depends upon clar-
ity of thought of the communicator. Pitch, volume, rate
Stopover and clarity are termed as components of paralanguage.
Which of the following is not the major concern for The spontaneity, being natural, easy understanding due
mass media in India? to ample choice of words, support by non-verbal com-
(a) Manipulation munication, physical availability of speakers are the main
(b) Privacy characteristics of oral communication. It helps in devel-
(c) Security oping a close relationship between the speaker and the
(d) Integration of society listener.
The correct option is (d). The limitations include its being a temporary communi-
cation as spoken words disappear into air, and temptation
to forget in comparison to written communication. The
CoMMuniCation t ypeS on the non-verbal communication supporting oral communica-
B aSiS oF ChannelS tion may not be understood by people from other cultures.
On the basis of channels, communication is of two types: Phonetics
1. Verbal communication We use sounds to speak and pronounce the words. The
2. Non-verbal communication words are made up of one or more sounds. We put these

M04_MADAN 04_65901_C04.indd 15 21/12/22 10:54 AM


4.16 Chapter 4

sounds together to pronounce words. Thus, phonetics is


the study of the sounds that we make when we speak. For Concept Box
example, pronouncing cat, c-a-t.
Mnemonics
Written Communication Mnemonics is the use of different strategies for learn-
In written communication, written signs and symbols, ing and recalling words, numbers, facts and so on in
both in printed or handwritten form, are used. Pictures, different contexts.
graphs, etc., are used to compliment the written text. The Mnemonics are memory devices that help learn-
communicator’s writing skills, style and knowledge of ers recall larger pieces of information, especially in
grammar affect the quality of message. In written com- the form of lists, such as characteristics, steps, stages,
munication, message can be transmitted through email, parts, phases, etc. A study conducted by Gerald R.
letter, report, memo, etc. Miller in 1967 found that mnemonics increased recall.
The organization needs to communicate with different Mnemonics can even be used to recall words or remem-
stakeholders, both in internal and external environments, ber numbers. For example, BRASS is an acronym
to meet its objectives. Thus, written communication can for how to shoot a rifle—Breath, Relax, Aim, Sight,
further have two dimensions, internal and external. Squeeze. They help in taking notes also.
The internal written communication is in the form of
memos, reports, bulletins, job descriptions, employee Non-verbal Communication
manuals and emails specifically for internal communica-
tion, i.e., within an organization. This is the transmission of messages or signals through
Emails, internet, websites (URLs), letters, propos- a nonverbal platform such as eye contact, facial
als, telegrams, faxes, postcards, contracts, advertise- expressions, gestures, posture, and body language. This
ments, brochures and news releases are used for external also includes the use of social cues, kinesics, distance
communication. and physical environments and appearance, sounds and
Even oral communication is used for both internal as touch also.
well external communication. Types of Non verbal Communication: They can be
Advantages of written communication are as follows: categorized in the manners below.
1. Messages can be edited and revised many times before 1. Facial Expression: These express feelings such as
being sent, so it minimizes the chances of error. happiness, sadness, anger, surprise, fear, etc. The
2. Written communication provides an automatic record smiling face, relaxed face, matching expressions with
for every message sent and can be saved for later study what we say).
or references. 2. Posture: These are positions of the body. They try to
3. A written message enables the receiver to fully show our confidence and feelings. Thus, keeping our
understand it and send appropriate feedback. It body relaxed, straight shoulders, sitting straight, keep
brings in the element of impersonality and more our hands by our sides while standing are different
objectivity. suggestions.
3. Gestures or Body Language: These bodily move-
Limitations of written communication are as follows: ments or gestures, especially hands or head, to express
1. Written communication may not provide prompt and an idea or meaning. This includes waving, pointing
spontaneous feedback. and using our hands when speaking. Raising a hand
2. Written communication may take more time. may mean asking a question, biting nails may show
3. Usually communication is a mix of both oral and writ- some anxiety are other examples. Avoid pointing at
ten formats. people with our finger, bending head while listening
or greeting are suggestions.
4. Touch: We communicate a great deal through our
taBle 4.1 Types of Verbal Communication touch, such as shaking hands and patting on the back
Oral Written in the reward.
5. Space: This is basically maintaining the physical
Small Group Conversation Letters/Memos distance between two people when speaking with
someone.
Telephone Telex 6. Eye contact: The way we look at someone can com-
Large Group Lectures Circulars municate things, such as interest or anger.
7. Paralanguage: This means how do we speak. The
Meetings Newsletters tone of our voice, speed and volume are important to
make a difference in the meaning we want to convey.
Radio Handbooks/Manuals Speaking too fast may show happiness, excitement or
Short Circuit Posters/Bulletin Board nervousness. Speaking too slow may show serious-
ness or sadness.

M04_MADAN 04_65901_C04.indd 16 21/12/22 10:54 AM


Communication 4.17

The non verbal communication can be divided into the Paralinguistic: When we speak, other people ‘read’ our
two categories – static and dynamic. They have been dis- voices in addition to listening to our words. They pay
cussed below: attention to our timing, pace, tone, loudness, pitch, vol-
ume rate, quality, intonation, inflection, vocalised pauses,
1. Static Features
etc.
Distance: The distance one stands from another fre-
quently conveys a non-verbal message. In some cul- Kinesics: There are many body signals that amplify mean-
tures it is a sign of attraction, while in others it may ing between communicators. These include gestures, eye
reflect status or the intensity of the exchange. contact, facial expressions, posture, locomotion and hap-
Orientation: People may present themselves in vari- tics. In USA, a long eye contact may be to show respect
ous ways, such as face-to-face, side-to-side, or even while in Japan, a long eye contact is better to be averted,
back-to-back. For example, cooperating people are it shows disrespect.
likely to sit side-by-side while competitors frequently
face one another.
Haptics: Haptics refers to how and what touch commu-
Physical Contact: Shaking hands, touching, holding,
nicates. Reaction to unsolicited touch va-ries from person
embracing, pushing, or patting on the back all convey
to person. This may vary according to culture. The exam-
messages. They reflect an element of intimacy or a
ples are gentle touch on the forearm, a two-handed hand-
feeling of (or lack of) attraction.
shake, or a slight kiss on the cheek.
Postures are also a part of it that has been discussed
right now.
Phonemics: Phonemics is a linguistic ability. A pho-
2. Dynamic Features: Facial expressions such as smile, neme is a unit of sound that distinguishes one word from
frown, raised eyebrow, yawn, and sneer all convey another in a particular language.
information. Facial expressions continually change
during interaction and are monitored constantly by Dress and Appearance: The meaning depends upon the
the recipient. There is evidence that the meaning of situation.
these expressions may be similar across cultures.
Gestures: For example, hand movement. Most people
Chromatics: Chromatics is communication through the
use hand movements regularly when talking. While
use of colour. Colours of clothing, products or gifts send
some gestures (e.g., a clenched fist) have universal
intended or unintended messages to the recipient of
meanings, most others are individually learned and
message.
idiosyncratic.
Looking: A major feature of social communication
is eye contact. It can convey emotion, signal when to Iconic: These systems are used to represent words or lin-
talk or finish, or aversion. The frequency of contact guistic concepts. We can take examples of different types
may suggest either interest or boredom. of ‘emoticons’ to represent the idea.

Concept Box
Visual Communication
Visual communication, as the name suggests, is com-
munication through visual aid. It is the communication
of ideas and information in forms that can be read or
looked at. Primarily associated with two dimensional
images, it includes signs, typography, drawing, graphic
design, illustration, color and electronic resources.

Eight Types of Non-verbal Communication:


Non-Verbal Communication is categorized into eight types
that have been discussed below, they increase loyalty and
rapport.
Codes: Codes in communications, an unvarying rule for
Proxemics: We can use personal and physical space to replacing a piece of information such as a letter, word, or
communicate many non-verbal messages including inti- phrase with an arbitrarily selected equivalent.
macy, affection, aggression or dominance.

M04_MADAN 04_65901_C04.indd 17 21/12/22 10:54 AM


4.18 Chapter 4

Single Strand: Here, each person communicates with the


Concept Box next in a single sequence.
A study by Mehrabian Cluster Networks: Here, a person gets information and
According to Mehrabian, ‘Words account for 7%, tone pass this information to their cluster network or keep the
of voice accounts for 38%, and our body language information to themselves. Each individual may pass on
accounts for 55% of communication.’ This has become the information to the next cluster network
the 7/38/55 rule. They are abbreviated as the 3 V’s, i.e., Probability Chain: Here, each individual ‘randomly’ tells
Verbal, Vocal and Visual. another individual the same piece of information.
Thus, in practice, it is always a mix of verbal as well
as non-verbal communication. ClaSSiFiCation on the BaSiS oF direCtion
In teaching, it is assumed that attention is the start-
Vertical Communication
ing point of desirable development change. Thus, atten-
tion is the starting point to arouse interest. Research has 1. This is basically formal communication.
shown that attention is sought by people in the follow- 2. This can be upwards (bottom–up) and downwards
ing proportion: (top–bottom).
Seeing – 87 %
Lateral or Horizontal Communication
Hearing – 7 %
Smell – 3.5 % 1. Communication with people at the same level in a
Touch - 1.5 % hierarchy of peers and colleagues is termed as lateral
Taste – 1.0 % communication.
2. This may combine both formal and informal
communications.
This is the most effective communication, as
ClaSSiFiCation BaSed on purpoSe and Style it is generally not stalled by a chain of command
These can also be taken as the main categories of commu- methods. This can help in building teams in an
nication. organization.
3. The amount of horizontal communication depends
Formal Communication upon the interdependence of different departments.
This refers to the flow of official information through Diagonal Communication
proper, predefined channels and routes. Such flow
is controlled and needs deliberate efforts for right 1. Diagonal communication is effective as hierarchical
communication. This needs a hierarchical structure and bindings are removed and there is a free flow of infor-
chain of command, that is usually top down, means from mation, cutting across positions or status.
senior to low level employees. Employees are bound to 2. It facilitates in building relationships and bonding
follow formal communication channels while performing between the superior and the subordinate.
their duties. Memos, intranet, meetings, conferences,
formal one-on-ones, bulletin boards, handouts, letters, CoMMuniCation netWorKS
presentations, speeches, notice boards, organizational Communication network shows all communication pat-
blogs, emails etc. are different examples. terns or relationships that may exist in the organization.
In specific example, in case an Assistant Professor has The various types of communication networks are repre-
to communicate with the college Principal, it is usually sented diagrammatically in Figure 4.13.
through the Head of Department (HoD).

Informal Communication
There is additional but not less important flow of infor-
mation in the company. The context is of informal group
conversations. This is free, multi-dimensional, diverse,
quicker, relational and natural. This is not bound by pre- Chain Wheel
defined channels and communication routes. This does
not have a paper trail.
The informal communication has the following ways.
Grapevine: This generally begins with employees
through social relations. This can be described as a casual
and unofficial communication system within the organi-
zation.
Circle All-channel
Gossip: Here, one person is at the epicenter of the wheel
who seeks and shares information with others. Figure 4.13 Types of Communication Networks

M04_MADAN 04_65901_C04.indd 18 21/12/22 10:54 AM


Communication 4.19

1. Chain Network: It follows a formal chain of com- 2. Behavioural flexibility


mand, as is the case with a typical bureaucratic orga- 3. Goal orientation
nization. Here, members communicate with each 4. Sociability and interest in other people
other in a pre-planned sequence. 5. Empathy-non-judgmental perspective
2. Wheel Network: It is also known as ‘STAR’ network. 6. Meta-communication skills
Here, information flows from one central member 7. Increase in cultural sensitivity
of the group to the rest of the members. Other group
Culture has its effect on verbal and non-verbal messages
members may not have to communicate with each
and, thirdly, by the influence it has on the historical set-
other to perform well and all communication is chan-
ting, relational setting and a person’s position within a
nelized through the supervisor, for example, one-to-
speech community.
one interactions of heads of departments (HoDs) with
the college principal, but little or no interaction of HoDs Power Distance: This relates to social inequality.
among themselves. It is not very effective in teams. Individualism versus collectivism relates to the degree of
3. Circle Network: Here, members communicate infor- integration and orientation of individuals. Femininity ver-
mally with adjoining members generally on the basis sus masculinity pertains to the division of roles between
of shared experiences, beliefs, areas of expertise, women and men. Uncertainty avoidance describes the
background, or office location. It may have a formal degree of tolerance for the unknown. Long-term ori-
leader as well, but interaction is still lateral. It works entation relates to trade-offs between long-term and
in an autonomous team. short-term gratification of needs. Finally, high versus low
4. All-channel Network: An all-channel network is context refers to the amount of information already con-
found in teams. Both the intensity and frequency of tained in the person or context versus the amount in the
interaction is high among members. Information coded, explicit and transmitted part of the message.
flows in all directions. There is no formal leader, and
communication may be started by any member. Ethnocentrism vs Cultural Relativism
Ethnocentrism: My culture’s the best in every way. It is
I ntercultural Communication the idea that one’s own culture is the main standard by
The credit for this term is often given to American anthro- which other cultures can be measured.
pologist Edward T. Hall, who used it for the first time in Cultural relativism: This posits “every culture is equally
his book The Silent Language in 1959. The book is some- valid, so you have no right to impose your culture’s values
times called “the field’s founding document”. on other cultures.” This concept covers a wide area of
Culture is the characteristics and knowledge of a par- human interactions, beliefs, values and practices.
ticular group of people, encompassing language, reli- Conservatives lean towards ethnocentrism, while lib-
gion, cuisine, social habits, music and arts. Language is erals lean towards cultural relativism. Most people dwell
an example of an important cultural component that is between these extremes. One needs to convert from being
linked to intercultural understanding. ethnocentric to ethnorelativistic.
Culture is a
1. Human creation Analytical Thinking Vs Holistic Thinking
2. Human part of the environment Analytical Thinking: Here, more attention is paid to
3. Non biological aspect of life objects in focus and specific details. Cause and effect rela-
tionships are more important. If you get good marks, they
Cultural is considered to be dynamic and considers inter- are a result of hard work.
action between values, beliefs and perceptions. Our brain Holistic Thinking: Eastern societies assume that indi-
is an open system, so we have free choice to respond. vidual parts cannot be fully understood unless they are
Intercultural communication is the study of communica- placed within interdependent relationships. If you get
tion between people whose “cultural perceptions and symbol good marks, they are a result of hard work, good environ-
systems are distinct enough” to alter their communication. ment, parental support etc. Holistic thinkers tend to be
Intercultural communication is important for the fol- dialectical thinkers as they accept grey areas, assuming
lowing reasons: that things constantly change.
1. Immigration and refugee patterns
2. Social contact High-context vs. Low Context Culture
3. International interaction – opening up of media High-context Culture: Members of high-context com-
Intercultural communication increases our competence. munication cultures, such as Eastern societies, rely on
There are few indicators of intercultural competence in their pre-existing knowledge of each other. The verbal
the form of motivation, knowledge, attitudes and skills. messages are perceived to be pointless, awkward, or even
Intercultural effective person is deceitful, without high context. Explicit, direct messages
are perceived to be either unnecessary or even threaten-
1. Tolerance of ambiguity – effect on the values, tradi- ing. This mainly happens in collectivist cultures. Being
tions, social and political relationships pleasant is important.

M04_MADAN 04_65901_C04.indd 19 21/12/22 10:54 AM


4.20 Chapter 4

Low Context Culture: In low-context communication processes it to top level for decision making. That
of Western cultures, the meaning is clearer when it is is called as the vertical and bottom-up approach.
expressed through explicit words. Direct, detailed and Similarly, in an authoritative manner, information
unambiguous words are required. In case of misunder- moves in top-bottom manner. Information flows in
standing, the sender of the message is often held respon- diagonal and horizontal manner.
sible as he was not able to construct a clear idea. 3. Effective Communication is a Two-way Process:
Communication is deemed to be effective when there
Self-enhancement vs . Self-effacement is some kind of feedback mechanism.
Self Enhancement: In individualistic, low-context com- 4. Communication is Always with a Context and a
munication cultures, society focuses on promoting indi- Relationship Aspect: Both in terms of (i) what you
viduals’ self-esteem and self-efficacy. Individuals are say and (ii) how you say the matter. Same message
direct and more expressive about their abilities and can be communicated authoritatively or politely.
accomplishments. For example, while asking for a glass of water, one
can say:
Self effacement: In collectivistic eastern cultures, such (a) Please bring me a glass of water
as Japan and China, the tendency is on self-criticism. (b) Give me a glass of water
Individuals hesitate while discussing their own abilities. (c) Can I have a glass of water?
It maintains group harmony and avoids offense. (d) Could you please give me a glass of water?
Even grammar is not always run by ‘set rules’ but by
Elaboration vs. Understating
emotions and feelings. Each message will have a dif-
Elaboration: The French, Arabs, Latin Americans, and ferent effect on the person you are talking to.
Africans prefer an elaborative communication style. This 5. Communication Exchanges are Based on
can also be called verbal exaggeration. This over assertion Symmetry and Complementary: Symmetry refers to
is practiced by Saudi Arabians. similarity and complimentary refers to the differences
Understanding: Understated communication style involves in characteristics. For example, two students who are
the extensive use of silence, pauses, and understatements very good in mathematics will have a good communi-
in conversations, as practiced by the Chinese. When con- cation relationship. This will be termed as symmetry.
flict arises, using silence as an initial reaction allows people On the other hand, suppose there are two students,
to calm down, exhibit emotional maturity and take time to one is good in Mathematics but poor in English and
identify better conflict management strategies. the other is good in English but poor in Mathematics.
The four barriers to intercultural communication include They may have good complementary communication
ethnocentrism, stereotyping, prejudice and discrimination. relationship with each other because they compen-
To deal with barriers, non-dominant group members sate each other’s weaknesses.
use one or more of three main strategies to get what they 6. Common Language: This helps developing better
want from dominant group members, such as assimila- understanding between the sender and the receiver
tion, accommodation or separation. of a message and thus, making communication more
The five ways to improve intercultural communication effective.
are as follows. 7. Meeting of Minds is Necessary: The receiver must
comprehend the intended meaning of the message
1. Pay attention to your own words and actions. that the sender wants him or her to understand.
2. Control your assumptions. 8. The Message Must have Substance: The gist of the
3. Engage in transpection—the process of empathizing message holds importance only until the receiver
across cultures. shows interest in the subject matter. For example,
4. Gain knowledge. any discussion about cricket will be well received by a
5. Gain experience. cricket fanatic.
The Internet offers a vehicle for searching common val- 9. Communication can Also be Conducted Through
ues, understandings and approaches to managing a world Gestures: Communication should not necessarily be
of different cultures. verbal or written. Nodding of heads, rolling of eyes,
movement of lips, etc., are some of the gestures used
to convey certain basic ideas.
Characteristics of Communication 10. Communication is of Different Types: It can be for-
mal or informal, verbal or non-verbal. The different
The nature of communication can be explained using the types have been discussed in detail in the ensuing
following characteristics: discussion.
1. Communication is Continuous: Communication 11. Communication Always has a Context:
is continuous, dynamic and action oriented process Communication always takes place within a context.
towards a desired goal. According to Bateson, ‘Without context, words and
2. Communication is All-pervasive: Lower levels actions do not carry any meaning at all’. At times, this
provide information to the middle level, that further context may not be obvious, and at times, it may stand

M04_MADAN 04_65901_C04.indd 20 21/12/22 10:54 AM


Communication 4.21

out boldly. The context of communication has four 5. Control


dimensions: 6. Cultural promotion
(a) Physical Context: It refers to the place where 7. Integration
communication is taking place, such as in public, 8. Communication skills are of four types, listening,
conference room, classroom, etc. It refers to the speaking, writing and reading.
ambience of the place as well.
(b) Social Context: Social context is mainly about Listening is termed as a good skill and hence, it is said
(i) the role and status relationship between the that one has to learn ‘how to listen so that others will
sender and the receiver of messages and (ii) the talk’. Listening is a whole range of other aspects of com-
norms and culture of the society. For example, munication.
the social context of friends attending a party
will be different from people attending a meet-
MaCro FunCtionS oF CoMMuniCation
ing in an organization or a doctor attending a As per changing exam pattern, in question statements,
patient. there is an increasing use of technical words. Thus, we
(c) Psychological Context: It refers to the environ- need to discuss some functions. The macro functions of
ment characterized by formality or informality, communication are listed below:
friendliness or unfriendliness. 1. Emotive Function: To communicate the inner state
(d) Temporal Context: It refers to the time factor in of mind and emotions. For example, ‘Oh My God!
communication, such as at what time of the day Oh no!’
the communication takes place. In the morning, we 2. Directive Function: This function seeks to affect the
usually talk about religious and spiritual aspects of behaviour of others, for example, ‘Fetch me a glass of
life, and during day time, it is usually about profes- water, please.’
sional aspects, and so on. 3. Phatic Function: This is to open the channel of
communication or checking that it is working. For
Concept Box example, ‘Hello, is it Ram?’ or ‘Can you hear me,
Mrs. Girdhar?’ This function establishes, acknowl-
Adjacency Pairs edges or reinforces social relations
One way in which meanings are communicated and 4. Poetic Function: This refers to the aesthetic func-
interpreted in conversation is through the use of what tion of language and focuses on ‘the message for
have been called adjacency pairs. Adjacency pairs are its own sake’. Thus, it is basically the essence of the
utterances produced by two successive speakers such message. For example, euphony (pleasantness of
that the second utterance is identified as related to sounds in speech and pronunciation) is an applica-
the first as an expected follow-up. Adjacency pair is tion of this function. This poetic function of language
described as the basic structural unit in conversation. can be used when one says ‘Asoka the Great’, instead
Consider the following examples of ‘the great Asoka’, which essentially has the same
(a) Greeting-Greeting meaning.
A: Hi 5. Referential Function: Referential function refers to
B: Hi any message that is constructed to convey information.
(b) Compliment-Acceptance 6. Metalinguistic Function: This function focuses
A: That’s a nice cap. attention on the code of language itself. When lan-
B: Thanks. guage is used to speak about language, for example,
in grammar or a dictionary, the metalingual function
has been employed. Metalinguistic activity may be
unconscious.
functions of Communication For example, ‘The use of both will or shall is correct in
modern usage.’
During recent times, communication by mass media and 7. Contextual Function: It facilitates the creation of a
through social media is the underlying force for changes particular kind of context. It sets the tone. For exam-
taking place in society. People communicate to learn ple, ‘Right! Let’s start the meeting now.’
what they need and want to cope with their physical and 8. Heuristic Function: Here, we use language to gain
social reality. Successful people are usually effective com- knowledge, learn and explore the environment.
municators. A child uses language to learn. This may be in the
The different objectives of communication are dis- form of questions and answers or the kind of run-
cussed as follows: ning commentary that frequently accompanies chil-
1. Information to keep oneself updated and for decision- dren’s play.
making In addition, there can be some more functions.
2. Education and instruction
3. Smooth functioning of groups 1. Denotation refers to the literal meaning of a word,
4. Motivation and morale the ‘dictionary definition’. For example, if you look up

M04_MADAN 04_65901_C04.indd 21 21/12/22 10:54 AM


4.22 Chapter 4

the word ‘snake’ in a dictionary, you will discover that Noise Decoding and
one of its denotative meanings is ‘any of numerous Formulation of interpretation
scaly, legless, sometimes venomous reptiles, having message Sender/
of message
a long, tapering, cylindrical body and found in most receiver
tropical and temperate regions’.
2. Connotation, on the other hand, refers to the asso-
ciations that are connected to a certain word or the Encoding Feedback/
emotional suggestions related to that word. The con- of message transmission
notative meanings of a word exist together with the
denotative meanings. The connotations for the word
‘snake’ could include evil or danger.
3. A simile is a comparison between two dissimilar
Transmission/ Encoding
objects using the words as or like to connect them.
feedback of message
For example, if you say, ‘my boyfriend is like a water-
melon in the summer’, you are creating a simile that
compares your boyfriend to a watermelon. If on the
other hand you are angry at him and say, ‘he’s like a
typhoon in the house’, then you are comparing your
Decoding and
boyfriend to a typhoon. Formulation of
interpretation
4. A metaphor is similar to a simile, except that a meta- Receiver/ message
of message
phor compares two dissimilar objects without using sender
the words as or like. If you write, ‘my boyfriend is an
angel’ or ‘my motorcycle is a bomb on wheels’, you are Noise
creating metaphors. Figure 4.14 Process of Effective Communication

Stopover
The information function of mass communication is
1. Formulation of message
described as 2. Message encoding
(a) Diffusion 3. Message transmission
(b) Publicity 4. Message decoding and interpretation
(c) Surveillance 5. Feedback and evaluation
(d) Diversion The process of an effective two-way communication is
The correct option is (c). shown in Figure 4.14.

Formulation of Message
Classroom Communication It is appropriate to recall the slogan, ‘Medium is the Message’
coined by Marshall McLuhan. This implies that the carrier
Process–Cole and Chan Model of the communication, in whatever physical form –human
voice, image or text—digitised– influences the message, the
The main objective of classroom teaching is transfer knowl-
sender, the audience and the effects of mass media, far more
edge, comprehension, application, analysis, synthesis etc.
significantly than it has ever been possible. This is more than
to name a few. Classroom teaching still dominates the
true with the advances in multimedia. technology. The suc-
scene of formal education system. This mainly deals with
cess of communication, therefore, depends on what we say
cognitive domain. It is an integral part of any teaching-
and how we say it.
learning process. Classroom communication is affected by
multiple factors pertaining to teachers, students, message, 1. All communication starts with an idea or a message
instructional methods and media and the learning envi- that is to be transmitted to the target audience (indi-
ronment. We need to understand the nature of this teach- vidual) with a motive to get a positive response. Com-
ing process so that it can be made more effective. We can municator (sender or encoder) is the one who initi-
improve the this sacred act through training and practice. ates the communication process.
All communication is based on symbols. It is a process that 2. A message is a set of verbal or non-verbal cues sent by
involves organizing, selecting and transmitting symbols a source. Messages can be spoken or written words,
in an appropriate manner to ensure that the receiver per- gestures, movements, etc. They cannot have a mean-
ceives in his/her thought process, the intended meaning of ing apart from the person involved in the sending and
the communicator. receiving process.
According to Cole and Chan, a typical process of class- 3. Effective communication depends on the communi-
room communication (and communication in general as cation skill, knowledge level and attitude of the com-
well) includes the following five distinct stages: municator and how he/she desires to affect his/her

M04_MADAN 04_65901_C04.indd 22 21/12/22 10:54 AM


Communication 4.23

receiver. Ability to think, organize thoughts quickly 2. If the receiver possesses the background information
and express himself/herself effectively are some of and is familiar with the form of language used, then
the attributes of a good communicator. it becomes easier for him/her to decode and compre-
4. Teachers concerned conceptualize ideas to be con- hend the message.
veyed to students by arranging them in coherent and 3. If the receiver does not have the ability to listen, read
meaningful sequence. In most instances, they will and think, he/she will not be able to receive and
draw on content from the curricula. Teachers inter- decode the messages in the manner the communica-
pret those ideas and determine the appropriate orga- tor wants him/her to.
nization of the content. 4. For effective communication, the receiver is the most
important link in the communication process.
Message E ncoding
1. The internal shaping, sorting and sifting of ideas for Feedback and Evaluation
clarification and organization is called message for- 1. Feedback is the response or acknowledgement of
mulation. Its purpose is to create a clear and meaning- the receiver to the communicator’s message. The
ful message. exchange is possible only if the receiver responds.
2. Encoding involves converting an idea into a form that Feedback is considered as motor function.
can be transmitted to receivers. 2. It involves the receiver reacting to the transmit-
3. The communicator not only translates his/her ted messages and conveying the response to the
­purpose (ideas, thoughts, or information) into a mes- sender.
sage but also decides on the medium to communicate 3. Most effective communicators encourage feedback
his/her planned message. and adjust the content and presentation of their mes-
4. The communicator must choose the media (speech, sages as per the feedback received.
writing, signalling, or gestures) that the receiver can 4. Teachers need to be aware of the subtle and direct
comprehend well. For instance, an illiterate receiver ways of students for conveying reactions to the mes-
may fail to understand a written message but can sages received. Skilled teachers can also control feed-
understand it well if told orally. back so as to avoid unnecessary interference in the
5. Teachers encode their ideas in different ways accord- ongoing communication process.
ing to the demands of various curricula. It is the 5. Even by fluttering eyelids, raising an eyebrow, mak-
teacher’s responsibility to use appropriate symbolic ing facial expressions, organizing a point and asking
forms for each subject and to teach students to use for explanation expresses a lot. Thus, the message is
these forms. shaped and reshaped by the communicator and the
receiver until the meaning becomes clear. In this way,
Message T ransmission both the participants in communication interact and
1. It is a critical stage in the communication pro- constantly exchange roles. In face-to-face communi-
cess and answers how a message is delivered. The cation, the receiver responds naturally, directly and
sender selects an appropriate channel or mode of immediately. This provides the communicator an
presentation. opportunity to improve and make his/her communi-
2. A ‘channel’ is the vehicle or means through which a cation effective.
message or stimulus is carried from the communica- 6. Therefore, feedback provides an opportunity to
tor to the receiver. There are various options available evaluate what is right or wrong about a particular
as channels, such as written, spoken, verbal, non-ver- communication. It helps to regulate the conversa-
bal, mass media, etc. tion among two or more individuals and also stim-
  For example, a teacher may use a film or physical ulates and reinforces an idea that is desired to be
demonstration to convey the same idea. Teachers are communicated.
usually able to use more than one channel in ways 7. An interactive model of communication (Cole and
that allow for an integrated presentation. Chan) has a specific application for classroom teach-
ing and learning. In most classroom interactions,
Message Decoding and I nterpretation
there are constant role changes and many times the
communication process is reversed.
1. Decoding is the interpretation of a message by the
receiver. Actually, the receiver looks for meaning in
the message that is common to both the receiver and Noise
the communicator. Noise is an interruption that can creep in at any point of
  The process of decoding by the receiver is not pas- time in the communication process and make it ineffec-
sive but active. Language does not have meaning; it tive. Environment is one major cause that interferes with
has the potential for meaning, and it is the decoder message reception, including noises from the roadside,
who is actively engaged in making meaning on the constant chattering of individuals, blaring loudspeaker,
basis of his/her background knowledge and the con- faulty transmission, etc. Noise can also occur in other
text of communication. forms, such as poor handwriting, heavy accent or soft

M04_MADAN 04_65901_C04.indd 23 21/12/22 10:54 AM


4.24 Chapter 4

speech, communication in a poorly lit room, etc. In fact,


these are barriers to effective communication. For smooth Principles of effective Classroom
and effective communication, it is necessary to eliminate Communication
or reduce noise as far as possible.
Noise can be mainly divided into the following Effectiveness of classroom communication depends on five
categories: major factors, namely teacher, student, message, instruc-
tional methods and media, and learning environment.
1. Physiological Noise: It is the distraction caused by The principles of effective classroom communication have
hunger, fatigue, headache, medication, etc. been discussed under the following four headings:
2. Physical Noise: It is the most direct form of interfer- 1. Principles for teachers
ence in our physical environment, for example, traffic 2. Principles for message design
horns and poor lighting. 3. Principles for selection of instructional methods and
3. Psychological Noise: It refers to the qualities in us media
that affect how we communicate and interpret mes- 4. Principles for creating conducive learning environment
sages to others. For instance, if a manager is preoc-
cupied with a very intense problem, then he/she prinCipleS For t eaCherS
may be inattentive in a meeting. Similarly, preju-
A teacher should have the following qualities.
dice and defensive attitude can also interfere with
communication. 1. Develop a Realistic Self-concept and a Perception
4. Semantic Noise: It occurs when words themselves About Surrounding: This requires analysis of one’s
are not mutually understood. Authors sometimes cre- strengths and weaknesses, acceptance of the reality
ate semantic noise by using jargon or avoidable tech- and efforts to realize one’s potential.
nical language. 2. Develop Proficiency in the Subject Matter: Desire to
acquire knowledge is a must for the teacher. In the
words of Mahatma Gandhi,
‘Live as if you were to die tomorrow
Concept Box Learn as if you were to live forever’.
Education as a Communication System A teacher should make every effort to remain
Tiffin and Rajasingham discuss education as a commu- updated in his/her subject area by independent read-
nication system that also shares the characteristics of an ing, participation in short-term courses, attending
open system: classes of proficient teachers/experts, discussing
subject-related issues and problems with other col-
1. Negative-entropy: Open systems resist the trend
leagues or seniors, etc. This helps in boosting the con-
towards disorder and more towards increased
fidence of the teacher.
organization. For this purpose, they depend on
3. Understand the Learners: A teacher should try to
feedback system and the environmental supra
collect as much information about the learners’ char-
system.
acteristics as he/she can. Information needs to be col-
2. Feedback: Feedback enables the system to take
lected about the previous knowledge, learning styles,
corrective steps to adjust its malfunctioning, if
cognitive styles, motivation and interests. A teacher
there is any.
should accept the fact that no two individuals are
3. Steady State: It is a condition of dynamic equi-
alike and thus cannot be treated in the same manner.
librium. Here, the system maintains its structure
4. Develop Effective Communication Skills (both
as stable, despite fluctuation in environmental
Verbal and Non-verbal): Communication skills can
conditions.
be acquired through training and practice. A teacher
4. Differentiation: Open systems adapt to the chang-
can record his/her own audio and analyse the same for
ing environment. They also move in the direction
identifying the strengths and weaknesses in the spo-
of higher level of organization or growth.
ken language. He/she should remain open to feedback
5. Equifinality: It is the ability of open systems to
from colleagues, superiors and students.
reach a given state or condition by several different
5. Knowledge About Pedagogy and Andragogy: This
paths.
enables the teacher to plan, organize, deliver and
6. Teleology: It is a characteristic of open system that
evaluate instructions based on sound principles of
indicates the purposefulness of the system and
teaching and learning, thereby promoting learning
works according to its objectives.
among students.
7. Hierarchy: Open systems are hierarchical, i.e.,
6. Adopt Flexible Approach: This helps in modifying
they are both independent framework consisting
instructions as per the needs of the students.
of a number of integral lower level sub-systems
7. Being Objective and Unbiased: A teacher should
and each one is a dependent member of the higher
treat students as equal and not give any preferences
level system.
to students on the basis of sex, caste or creed.

M04_MADAN 04_65901_C04.indd 24 21/12/22 10:54 AM


Communication 4.25

prinCipleS For MeSSage deSign prinCipleS For Creating ConduCive learning


The message should have the following characteristics: environMent
1. Clear and Specified Objectives: Objectives should 1. Classes Should not be Overcrowded: This is required
satisfy the criterion of being SMART. so that a teacher can pay individual attention to every
(a) Specific student.
(b) Measurable 2. Ensure Proper Seating Arrangement in the Class:
(c) Achievable This is required to maintain optimum distance between
(d) Realistic a teacher and students, ambiance and furniture of the
(e) Time framed classroom, for proper lighting and ventilation.
2. Relevant to the Objectives: Contents should be rel- 3. Build Rapport with the Students: The positive atti-
evant to cover all the objectives. tude is required from both sides in the classroom. The
3. Properly Sequenced: This is covered in Unit I under individuality of students must be maintained.
the maxims and principles of teaching. It should be 4. Encourage Cooperation and Healthy Competition
seen that the content is organized based on the fol- among students: This can be achieved by assigning
lowing parameters. group activities and pairing bright students with aver-
(a) Simple to complex age or poor oones.
(b) Easy to difficult 5. Feedback Mechanism: It should work both ways.
(c) Concrete to abstract 6. Encourage Experimentation: This is required to
(d) Known to unknown maintain innovations in the classroom.
(e) Observation to reasoning 7. Provide Reinforcement: A teacher should reinforce
4. Use Language Comprehensible to the Learners: the desired communication behaviour of learners
The language of the message should be simple and through praise, appreciation, rewards, social recogni-
comprehensible to the learners. Technical jargon tion and so on.
should be avoided. 8. Provide Non-threatening Environment: There
5. Use Appropriate Symbols: Symbols used in should not be a fear of punishment. Prior informa-
the message should be technically correct and tion regarding schedule of activities, tests, assign-
standardized. ments, etc. should be provided to learners in order
6. Include relevant exercises: The exercises selected to avoid any unnecessary anxiety on the part of the
should be of varying difficulty levels and involve the learners.
learners meaningfully.
7. Make Generous Use of Examples and Non-
examples: Examples from daily life and the world Concept Box
of work should be used to facilitate understanding of Paraphrasing
the applicability of content in different settings and Paraphrasing is a skill that is absolutely necessary for
thus, it ensures greater transferability of the learnt good listening. It means stating in your own words
materials. your understanding of what has just been said. It
prinCipleS For SeleCtion oF i nStruCtional gives the speaker the opportunity to find out what
message he/she is getting across to you. He/she can
M ethodS and Media then make any corrections needed. To begin para-
1. Select Relevant and Appropriate Methods and phrasing, you might start by saying, ‘What I hear you
Media: The methods and media should be relevant saying is ....’ or ‘It sounds like ....’ or ‘Let me see if I’m
to the objectives, content and context, and should understanding you ....’
be appropriate to the level of learners. Instructional
methods and teaching methods have been discussed Advantages of Paraphrasing
in Unit I. 1. It helps the students know that they are under-
2. Use Variety of Methods and Media: In order to cater stood by the instructor.
to the individual differences among learners and to 2. It prevents misunderstandings from occurring.
avoid monotony, a variety of methods and media 3. It helps to avoid impulsive or angry reactions.
should be used by the teacher. 4. It will prevent you from getting distracted easily.
3. Use Good Quality Media: The quality of media in 5. It helps to remember what was said frequently.
terms of its visibility, legibility, finish, colour and
printing should be judged before its use.
4. Integrate Media in Teaching–learning: Media
should not be used in isolation. For example, if a video
some important Terms
film is being used at the end of a lesson to reinforce 1. Synchronous Media: Media that takes place in real
what has been taught in the class, then it should either time, such as live television or radio and requires the
be followed by a discussion or some quick assessment audience to be present when the media is broadcasted
of the students’ understanding. or performed is called synchronous media.

M04_MADAN 04_65901_C04.indd 25 21/12/22 10:54 AM


4.26 Chapter 4

2. Asynchronous Media: Media that does not require 4. Legitimate Power: This power comes from formal
the audience to assemble at a specific time in order to titles, such as CEO, MD, parent, teacher, mentor,
use it is called asynchronous media. Examples of asyn- coach etc.
chronous media are printed materials or recorded 5. Reward Power: An employee is likely to respond
audio or video. favourably to orders and directions if he receives a
3. Time Shift: The recording of an audio or video event, tangible reward, such as a better job assignment or a
usually by the audience, to be watched later at a time pay raise.
other than when it was originally broadcast is called 6. Coercive Power: This power is the ability to pun-
time shift. Setting a VCR to record a favourite pro- ish someone for noncompliance with an order or
gram is an example of time-shifting. direction.
4. Surveillance: Primarily, the function of mass com-
munication is to provide information about the pro-
cesses, issues, events and other developments in Communication Barriers
the society. Communication barriers are interferences or obstacles
5. Convergence: It is the combination of computing, which limit the receiver’s understanding of the message.
telecommunications and media in a digital environ- Barriers may have the effect of entirely preventing com-
ment. Convergence and the changes that it is bring- munication, filtering part of it, or giving it an incorrect
ing are fundamentally changing many aspects of mass meaning.
media and communication. Barriers occur if any of the essential elements of com-
munication (sender, message, medium or receiver)
become faulty or defective.
Ten Commandments of As a matter of fact, no communication can break down;
Communication it merely changes. For instance, when you ignore some-
one, you communicate that you do not wish to talk.
Though we have studied them earlier, in order
to make communication effective, let us observe
some important aspects that make communication Stopover
­effective: Which of these is not a commandment of effective

communication?
1. Clarify ideas before communicating. (a) Clarity in language
2. Examine the true purpose of communication. (b) Listen poorly
3. Take the entire environment, physical and (c) Home communication skills
human, into consideration. (d) Adequate medium
4. When valuable, take advice from others while (b) is the right answer.
planning communication.
5. Be careful of the overtones and the basic content The correct option is (b).
of the message.
6. Use crisp language and be clear. Psychological Barriers
7. Follow-up on communication.
8. Communicate with the future as well as the pres- 1. Frame of Reference: Each person has a frame of
ent in mind. reference, a kind of a window to view the world,
9. Be a good listener. people, events and situations. A frame of reference
10. Exhibit congruency. is shaped by our cultural environment (norms, val-
ues and beliefs), childhood experiences and heredity.
All these factors are usually implicit. No two people
will have same frame of reference. Our frames of
Six Different T ypes of I nterpersonal Power reference change with passage of time as our needs
of C ommunication and experience also change. Here, it is important to
mention the concept of reference groups whom we
These do happen when a person is in leadership position
espouse as our own and try to follow in our routine
such as teacher.
behaviour.
1. Expert Power: Having expert knowledge about a sub- 2. Self-image: Self-image or self-concept is closely
ject that others value and do not possess themselves. related to frame of reference. People establish their
2. Referent Power: The ability to empathize with sub- points of view and interpret messages in accordance
ordinates gives a leader referent power in communi- with their self-concepts. They respond favourably
cation. Leaders can identify with how their subordi- those messages that reinforce their self-concept and
nates feel and think. reject those messages that are perceived to be threat-
3. Information Power: This power refers to the impor- ening to the self image.
tance of presenting information coherently and logi- 3. Stereotyping: It originates from the ego, and becomes
cally. This is linked with expert power. a barrier to communication when people react as if

M04_MADAN 04_65901_C04.indd 26 21/12/22 10:54 AM


Communication 4.27

Communication Barriers

Psychological Linguistic and Mechanical Organisational Physical


Barriers Cultural Barriers Barriers Barriers Barriers

Frame of reference Semantic barriers Language Size Size Competing stimulus


Self-image High/low culture Perpetual biases Distance Environmental stress
Stereotyping Cross cultural barrier Impersonal relationships Specialisation Subjective stress
Field of Experience Cultural differences Org Culture Medium unfamiliarity
Cognitive dissonance Rules & regulations
Defensiveness and Fear Power Structure
Selective Perception Complexity
Filtering Facilities
Lack of cooperation
Figure 4.15 Barriers of Communication

they already know the message coming from the also put a limit on the encoding process of an individual.
sender, or worse. However, senders and listeners Communication skills, knowledge of the topic, and person-
should continuously look for and address thinking, ality factors such as attitudes, values interests, and moti-
conclusions and actions based on stereotypes. vational needs are some of the traits or mental conditions
4. Field of Experience: This barrier occurs when that combine to limit, screen or filter what is encoded. They
the field of experience is not common to both - the also add to the quality of message.
sender as well as the receiver. If a teacher is teach-
ing Einstein’s equation to commerce students, he/she Linguistic and Cultural Barriers
will not get any response, but if it is taught to physics Language, both verbal or non-verbal, makes use of words.
students, his/her communication is likely to be more Words are mere symbols. Symbols can be comprehended
effective. differently by participants in ­communication. The com-
5. Cognitive Dissonance: Cognitive means thinking munication message might not use vocabulary that is
and dissonance means conflict. Thus, it means con- understood by the receiver. For example, excessive use of
flict in thinking. For example, although a smoker technical, financial, medical or psychological terms and
accepts the truthfulness of drug de-addiction in jargon.
advertisement messages by the Department of Social Most of the native languages are culture specific. When
Welfare, he is not able to react favourably. languages are distinct, communication is carried out
6. Defensiveness and Fear: This is closely related to the through translation, which increases the probability of
desire to maintain status quo, and a person always misunderstandings.
justifies his/her viewpoint or idea. Although languages are meant to improve understand-
Along with the feelings of nervousness, anxiety ing, different cultural contexts and languages can also
and tension, fear is the most restricting of all effects, prove to be a barrier to communication across different
resulting in narrow thinking, which selects and dis- cultures.
torts communication. However, a little fear and anxi-
1. Semantic Barriers: Semantic barriers occur when
ety can be turned into a source of energy and enhance
there is a disagreement about the words being used,
confidence as it motivates to perform better.
which is a result of individuals being from different
7. Selective Perception: It is also known as ‘I-know-it
cultures. Thus, disallowing the parties involved to
syndrome’. If somebody says, ‘It is a waste of time’, is
determine a common meaning of the words used. This
also exhibiting selective perception when one does
frequently occurs when the parties involved speak dif-
not apply full mind on communication situation.
ferent languages.
8. Filtering: A sender’s manipulation of information can
2. High-context and Low-context Cultures: High-
be seen as more favourable by the receiver.
context culture is the culture that relies heavily on
An individual’s particular psychological characteris- non-verbal and subtle situational cues to communi-
tics are basically termed as ‘conceptual filters’. They can cate. A lot of things are left unsaid, but it is already

M04_MADAN 04_65901_C04.indd 27 21/12/22 10:54 AM


4.28 Chapter 4

understood by the members that constitute the group. 3. Impersonal Relationships: Our perception is also
Asiatic societies such as India, Saudi Arabia and Japan based on past experiences with the communicator.
are prominent examples of high-context culture. The same communication from the highest authority
Low-context culture relies heavily on words to con- may be perceived differently than that from a subor-
vey meanings in a communication. A few words can dinate or a colleague.
communicate a complex message very effectively to 4. Cultural Differences: Effective communication
an in-group (in-group is one’s own culture) but less requires deciphering the basic motives, values,
effectively outside that group. In a low-context culture, assumptions and aspirations that operate across dif-
the communicator needs to be much more explicit. ferent cultures.
Western societies are usually low-context societies.
3. Cross-cultural Communication: Cross-cultural
communication is a field of study that analyse how Stopover
people from varying cultural contexts communicate Which of the following is termed as the first enemy of
in similar ways and also in different ways among communication?
themselves. How do they endeavour to communicate (a) Noise
across cultures? (b) Clarity
(c) Politeness
Stopover (d) Completeness
Which of the following denotes the problem arising The correct option is (a).
from expression?
(a) Cultural barriers
organizational BarrierS
(b) Semantic problems
(c) Wrong assumptions These barriers develop when an organization evolves.
They can be attributed to the following conditions:
(d) Selecting perception
The correct option is (b). 1. Size of the organization
2. Physical distance between employees
MeChaniCal BarrierS 3. Specialization of jobs and activities
4. Organizational culture, which impacts freedom
Mechanical barriers can exist in both interpersonal or
and trust
mass communication. There can be difficulty in recep-
tion, or some elements of the message may not reach the 5. Organizational rules and regulations
destination or both. It can be in the form of absence of 6. Power structure in the organization
communication facilities. 7. Complexity in organizational structure
Channel noise is a technical term used for such 8. Inadequate facilities and opportunities
mechanical barriers that includes any disturbance in 9. Lack of cooperation between seniors and subordinates
physical transmission of a message. Some examples of
mechanical barriers are disturbances in radio transmis- phySiCal BarrierS
sions, blurriness on TV screens, spreading of ink on a
newspaper, inaudibility in telephone devices, a barely The geographic distance between the sender and
readable point size or any kind of improper functioning receiver(s) can be taken as the most pertinent example of
of a device. They can also be associated with cultural or physical barrier in communication.
social issues, language, customs, beliefs, motives or sim-
ply illiteracy.
The various communication barriers at workplaces are Concept Box
as follows:
The 7 Cs provide a checklist for making sure that com-
1. Language: The same message can be interpreted munication in the form of teaching, instructions, meet-
differently by different people. Several factors affect ings, emails, conference calls, reports and presenta-
how an individual attributes meaning to particular tion is well constructed and clear so that the audience
words. understands the message.
2. Perpetual Biases: It prevents us from looking at real- According to the 7 Cs, communication needs to be:
ity in a truthful manner. The most common perceptual 1. Clear
biases are stereotyping, projection and self-fulfilling 2. Concise
prophecies. 3. Concrete
Stereotyping comes into play when we assume that 4. Correct
a person belonging to a certain group will display 5. Coherent
specific characteristics. However, the person may not 6. Complete
actually exhibit those characteristics. This is specifi- 7. Courteous
cally true in the case of traditional societies.

M04_MADAN 04_65901_C04.indd 28 21/12/22 10:54 AM


Communication 4.29

Four T ypes of Powers in communication 6. Control Emotions: Emotion can be in any form, such
These powers are following as anger, disgust and sadness. Emotions severely
cloud and distort meaning. The best thing is to defer
1. Expressing Yourself: from the very first moment we or postpone further communication until calmness
enter into this word. and coolness is established.
2. Listening and Responding 7. Watch for Non-verbal Cues or Body Language:
3. Regulating Attention and Intention: through emo- In case of oral communication, the sender should
tional intelligence also observe the actions of receiver and find whether they
4. Understanding and Responding to People in Their go along with the understanding. A supervisor should
“Style” watch the non-verbal cues or body language carefully.
Communication is generally easier over shorter distances Communications T hrough Emoticons
as more communication channels are available and lesser
or no technology is required. The ideal communication is Emoticon is the combination of “emotion” and “icon”.
face-to-face. They are visual representations of writers’ emotions. They
Although modern technology often helps to reduce are often taken as utterances in computer related commu-
the impact of physical barriers, the advantages and dis- nication that are interpreted by receivers. They are basi-
advantages of each communication channel should be cally graphic representations of facial expressions. They
understood so that an appropriate channel can be used to are produced by ASCII symbols (:-)) or by “pictograms,”
overcome the physical barriers. which are graphic symbols (), and now produced by Shift
There are four kinds of physical barriers: JIS art and Unicode art.
The first emoticon was used by Scott E. Fahlman in USA
1. Competing Stimulus: There can be some other con- in 1982. According to him, “smiley” face :-) and “frowny”
versation going on in the surrounding area or traffic face :-( could be used to identify jokes in computer sys-
noise in the background and so on. tems. They try to economize computer-mediated interac-
2. Environmental Stress: High temperature, poor ven- tion. They are akin to visual and body language. They are
tilation and so on. emotion markers.
3. Subjective Stress: Sleeplessness, ill health, effects of Emoji (from the Japanese e, “picture,” and moji, “char-
drugs, mood variations and so on. acter”) are a slightly more recent invention. They are dif-
4. Receiver’s Unfamiliarity with Medium: The use of a ferent from emoticons. Emoji are pictographs of faces,
medium with which the recipient is unfamiliar is also objects, and symbols.
a communication barrier. Emoticons function as “contextualization cues”, as
they supplement and organize in interpersonal relations
Stopover in written interaction. They serve three basic functions.
In which of these problems, is the actual message lost
1. Positive Attitude Function: specifically following
in the abundance of transmitted information? signatures
(a) Selecting perception 2. Humorous Function: when following utterances, they
(b) Over communication joke/irony markers.
(c) Under communication 3. Hedge Function: This is done in two ways:
(d) Filtering (a) Strengtheners: For expressive speech acts such
The correct option is (b). as thanks, greetings, etc.
(b) Softeners: For following directives such as requests,
corrections, etc.
O vercoming B arriers of Communication
To make communication more effective, supervisors/ We can use speech act and politeness theory for emoticons.
managers must try to attempt to remove the barriers. Now emoticons are used in ‘chat software’. They are identi-
fied as a typically ‘teenage phenomenon’. To some thinkers,
1. Regulate the flow of information. emoticons are superfluous and a waste of bandwidth. Their
2. Feedback: Acknowledgment of a message. excessive use may indicate may signal emotional instabil-
3. Human resources should use simple language. ity and a lack of control over one’s feelings. Thus, we need
4. Level of Knowledge: It is always advantageous to in-depth knowledge for the authentic use and communica-
speak in the subordinate’s language (level of knowl- tive functions of emoticons. Emoticon is used as semiotic
edge, his educational qualification and fluency of lan- resource in e-mail communication, it is used systematically
guage of the receiver). It is best to communicate in the to modify speech, to contextualize discourse and to organ-
mother tongue of the subordinate. ize social relationships.
5. Listen Carefully: Hearing is passive, whereas listen- The first study on emoticons was conducted in 1995 by
ing is an active and intellectual process. While listen- Rezabeck and Cochenour. Women used emoticons more
ing, one should stop talking, be patient, hold temper, often than men did, such as through Internet Relay Chat
ask questions, remove distractions and entirely con- (IRC). The majority of emoticons in corpus of Synchronous
centrate on what the speaker is saying. Instant Messaging (SIM) are produced by women.

M04_MADAN 04_65901_C04.indd 29 21/12/22 10:54 AM


4.30 Chapter 4

Emoticons modify the perception of ‘flaming’, so such (b) Illocutionary: These acts refer to the making of
messages should be taken less seriously. Thus, they rep- a request, offer, joke, promise, etc. in uttering a
resent a useful strategy for preventing unintentional out- sentence.
breaks of flaming. (c) Perlocutionary: These acts refers to the effect
Emoticons are linked with private e-mail, private chat that is brought to the audience.
(Instant Messaging), public chat (AOL chat; Internet There are following five main categories of speech
Relay Chat), and postings on public discussion forums. acts–Directives (requesting), Expressives (thanking),
The primary function of emoticons is not to convey emo- Representatives (asserting), Commissives (promising)
tion but rather to indicate an illocutionary force – that is and Declarations (appointing).
the intended effect of the utterance. Politeness Theory developed by Penelope Brown and
Stephen C. Levinson is defined as behaving in a socially
Speech act and politeness theory acceptable way with proper manners and etiquette.
There are two theories of linguistics - speech act theory
(a kind of social action called as speech act) and polite- Hedging: These are expressions with a “pragmatic
ness theory. We need to differentiate between two types value that modifies the speech act, not the logical
of utterances “constatives” and “performatives”. form of the utterance”. Moreover, they are referred
to as a “direct modification of propositional content”.
1. Constatives: These are descriptive statements which can Thus, hedges may be divided into “strengtheners”
be either true or false. For example, “It is hot in here”. emphasizing the propositional content and “soften-
2. Performatives: These are utterances which realize a ers” which soften the propositional content. They
social action–“I hereby pronounce you husband and “indicate something about the speaker’s commitment
wife”. toward what he/she is saying, and in so doing modify
There are three distinct acts of an utterance: the illocutionary force”.
(a) Locutionary: This is the act of making a meaning- To depict the categories of the speech acts, we use the capi-
ful utterance, a stretch of spoken language that is tal letters D(irectives), E(xpressives), C(ommissives) and
preceded by silence and followed by silence or a R(epresentatives) after the respective acts.
change of speaker that is also known as a locution
or an utterance act.

M04_MADAN 04_65901_C04.indd 30 21/12/22 10:54 AM


Communication 4.31

A s s e s s Yo u r L e a r n i n g

ConCePT anD naTure of CoMMuniCaTion


1. When we say that communication is intentional, it 7. If it is assumed that communication has no beginning
means that or end, then it is termed as
(a) The sender consciously intends to affect the (a) Mediation (b) Process
behaviour of the receiver of the message. (c) Interaction (d) Transaction
(b) Effective communication is a process of acting on 8. The essential components of communication are
information. (a) Source, message, interference, channel, receiver,
(c) Through speech communication, people make feedback and context.
sense of the world. (b) Sign, source, destination, interaction and
(d) None of the above correlation.
2. Which of the following statements are important in (c) Signs, symbols, understanding, communication
context of communication? and communicant.
1. The main objective is receiver understanding. (d) Symbols, understanding, purpose, ideas, opin-
2. Learning to communicate with others is key to ions, non-verbal and reaction.
establish rewarding relationships. 9. Which of the following statements are true?
3. A message can only be deemed effective when it is 1. Shannon and Weaver model of communication is
understood by others and produces the intended linear.
results. 2. Dyadic communication is the transfer of mes-
4. Communication is part of motivation function. sages from a person to another person and vice
5. Communication depends upon our basic needs. versa.
(a) 1, 2, 4 and 5 (b) 2, 3, 4 and 5 3. The study of relationships among words is called
(c) 1, 2, 3 and 4 (d) 1, 2, 3, 4 and 5 as syntactics.
4. Culture is a value free asset, according to Wilbur
3. Which of the following can be termed as the ‘context’ Schramm.
of communication?
(a) An interference with message reception. Codes:
(b) Effective communication. (a) 1, 2 and 3 (b) 2, 3 and 4

A S S E S S YO U R L E A R N I N G
(c) Verbal and non-verbal responses to messages. (c) 1, 2, 3 and 4 (d) 1, 3 and 4
(d) A physical and psychological environment for 10. The concept of global village that stated that masses
conversation. can unite to effect changes was given by
4. Which of the following is Berlo’s Linear Model of (a) Marshall Macluhan
communication? (b) Shannon and Weaver
(a) S-M-R-C (b) S-M-C-R (c) Aristotle
(c) S-R-M-C (d) S-R-C-M (d) Swami Vivekanand
5. Who among the following has stated that peo- 11. A hypothetical lost parent language from which actual
ple know each other and themselves only through languages are derived, is called as
communication? (a) Para-language (b) Derivative language
(a) George Herbert Mead (c) Metalanguage (d) Protolanguage
(b) Mahatma Gandhi 12. The major contributer to the theory of communica-
(c) Martin Luther King tive reason and communicative rationally was
(d) Henry Fayol (December 2021)
6. The ability to communicate effectively (a) Schulman (b) Allen Luke
(a) The education level only can make a contribution (c) Deng-Xiao-Ping (d) Habermas
towards communication. 13. When a group agrees to support and commit to the
(b) Communication is only a natural talent that can- decision of the group, they have reached
not be learned. (a) a census (b) a solution
(c) Communication has become absolutely technol- (c) a consensus (d) an analysis
ogy oriented.
(d) This skill can be acquired during different phases 14. Prospective taking in communication supports
of life. (December 2021)

M04_MADAN 04_65901_C04.indd 31 21/12/22 10:54 AM


4.32 Chapter 4

1. Behavioral rigidity 21. A high-context culture is one where


2. Openness (a) Much of the information is about the context or
3. Sharing of information the person.
4. Trust (b) Much of the information is spoken.
5. Secrecy (c) Most people use sign language.
Choose the correct answer from the option given below (d) Much of the information is unspoken.
(a) 1, 2 and 3 (b) 2, 3 and 4 22. Given below are two statements:
(c) 3, 4 and 5 (d) 1, 3, and 5 Statement I: In communication, language ambigu-
15. During the communication process, the message is ity takes place because of the incorrect use of words.
­converted to a symbolic form called Statement II: Informal time terms lead to different
(a) Decoding (b) Encoding interpretations.
(c) Deciphering (d) Expanding In light of the above statements, choose the most
16. One’s own description as a person is appropriate answer from the options given below:
(a) Self-concept (b) Self-perception (a) Both Statement I and Statement II are correct.
(c) Self-awareness (d) Self-respect (b) Both Statement I and Statement II are incorrect.
17. Which of the following terms describes the communi- (c) Statement I is correct but Statement II is
cation between two people that involves sending and incorrect.
receiving of messages? (d) Statement I is incorrect but Statement II is
(a) Decoding (b) Encoding correct.
(c) Transaction (d) Dyadic 23. Given below are two statements: One is labeled as
18. According to Shannon and Weaver Model, which of Assertion A and the other is labeled as Reason R
the following statements are true?  (December 2021)
1. Technical (signals), semantic (interpretation of Assertion (A): With the development of science
meaning) and influential (effectiveness). and technology, the forms and ways of communica-
2. The entropy and randomness are required for the tion have become more specialized and there is an
purpose of messages. increasing use of social media as the new methods
3. Their model was the first one to highlight the role of communication
of ‘noise’ in communication.
4. This communication is called as ‘telegraphed Reasons (R): In the socities of today, the communi-
communication. cation process is more complex and indirect because
of the use of modern gadgets in which the individu-
Codes: als do not interact face to face
A S S E S S YO U R L E A R N I N G

(a) 1, 2 and 3 (b) 1, 2, 3 and 4 In the light of the above statements. Choose the cor-
(c) 2, 3 and 4 (d) 3 and 4 rect answer from the options given below:
19. Being assertive in communication means the (a) Both (A) and (R) are true and (R) is the correct
following explanation of (A)
1. An assertive statement is frank and true expres- (b) Both (A) and (R) are true and (R) is the NOT
sion of their feelings. correct explanation of (A)
2. An assertive statement is being transparent and (c) (A) is true but (R) is false
unbiased. (d) (A) is false but (R) is true
3. An assertive statement means that we need to
24. Given below are two statements: One is labeled as
strike a balance between our needs and those of
Assertion (A) and the other is labeled as Reason (R):
others.
4. Assertive communication gives confidence to the Assertion (A): Agenda building is a collective com-
speaker. munication process in which media, government
5. Assertive communication means offering respect and the citizenry reciprocally influence one another.
to the other partners. Reasons (R): Agenda building presumes cognitive
codes: effects and inactive audience with less societal-level
effects.
(a) 2, 3, 4 and 5 (b) 3, 4 and 5
(c) 1, 2, 3, 4 and 5 (d) 2, 4 and 5 In the light of the above statements, choose the most
appropriate answer from the options given below:
20. A low-context culture is one where
(a) Most of the information is unspoken. (a) Both (A) and (R) are true and (R) is the correct
(b) Most of the information is explicitly stated in a explanation of (A)
­verbal message. (b) Both (A) and (R) are true but (R) is NOT the cor-
(c) Most of the information is apparent. rect explanation of (A)
(d) Most of the information is non-existent. (c) (A) is true but (R) is false
(d) (A) is false but (R) is true

M04_MADAN 04_65901_C04.indd 32 21/12/22 10:54 AM


Communication 4.33

25. Given below are two statements: 3. Sign is objective while symbol is subjective.
Statement I: Body language is the basis of deceptive 4. Words are symbols for thoughts,we use them
communication. according to the thoughts you associate with
Statement II: Non-verbal cues have their own shared those words
meanings. Codes:
In light of the above statements, choose the correct (a) 2, 3 and 4 (b) 1, 2 and 4
answer from the options given below: (c) 1, 2 and 3 (d) 1, 2, 3 and 4
(a) Both Statement I and Statement II are true. 31. Emoticons are
(b) Both Statement I and Statement II are false. (a) Emotional conference
(c) Statement I is true but Statement II is false. (b) Specifically expressive bodily gestures
(d) Statement I is false but Statement II is true. (c) Emotional queries
26. The primary channels used by individuals to commu- (d) Typed symbols that communicate facial
nicate with others are expressions
(a) Radio and television (b) E-mail 32. In the communication process, ‘to encode’ means to
(c) Tone of voice (d) Sight and sound (a) Translate ideas into a code
27. Match the following in context of Lasswell Model of (b) Interpret a code
Communication. (c) Block a pathway between the sender and receiver
of a message
Basic Aspects Meaning (d) Speak to large groups of people
1. Who A. Receiver 33. A person is more likely to use eye contact while
2. To whom B. Transmitter (a) Listening
(b) Uninterested being uninterested in communication
3. With what effect C. Stimulus message (c) Speaking
4. Says what D. Influrnce reply (d) Interpreting
Codes: 34. Which of the following statements are true in context
of feedback?
(a) 1–A, 2–B, 3–C, 4–D
(b) 1–B, 2–A, 3–C, 4–D 1. A situation in which the sender and the receiver
(c) 1–B, 2–A, 3–D, 4–C exchange information
(d) 1–A, 2–C, 3–B, 4–D 2. Listening is adversely affected by fast speed of
delivery of messages
28. The sequence of the stages of communication process
3. Listening is adversely affected by improper selec-
is (December 2021)

A S S E S S YO U R L E A R N I N G
tion and use of media
1. Level of acceptance
4. Listener’s verbal or non-verbal responses to a
2. Transmission of cognitive data
message.
3. Message Reception
4. Understanding Codes:
5. Reaction (a) 1, 2, 3 and 4
Choose the correct answer from the options given (b) 1, 2 and 4
below : (c) 2, 3 and 4
(d) 1, 2 and 3
(a) 1, 2, 3, 4 and 5
(b) 2, 3, 4, 5 and 1 35. Which of the following statements are true in context
(c) 3, 5, 4, 1 and 2 of classroom cybernetics?
(d) 2, 3, 4, 1 and 5 1. This means ‘to steer’, ‘to navigate’ or ‘to govern’ for
taking the system to desired goal.
29. Which of the three components are parts of the human
2. This means classroom teaching-learning process
communication process?
is a system and the goal is success of the learner
(a) Message, recording and feedback and learning process.
(b) Noise, feedback and jargon 3. This is constituted by Constructivism,
(c) Message, noise and feedback Conversation theory and a feedback system.
(d) Feedback, message and critiquing 4. This term was first defined by Norbert Wiener in
30. Which of the following statements are true in the con- 1948.
text of communication? codes:
1. A sign is an object or figure that represents a cer- (a) 1, 2, 3 and 4
tain reality for those who interpret it. (b) 1, 2 and 4
2. A symbol represents an idea that must be per- (c) 2, 3 and 4
ceived from the senses and that is linked to a con- (d) 1, 2 and 3
vention accepted by society.

M04_MADAN 04_65901_C04.indd 33 21/12/22 10:54 AM


4.34 Chapter 4

36. When we try to organize details in our minds, we are 45. An example of a communication channel is
seeking to (a) Noise
(a) Produce new ideas in our mind (b) Context
(b) Process complex information and then (c) Face-to-face conversation
categorize it (d) Feedback
(c) Categorize difficult and easy pieces of 46. The way one interprets information around oneself is
information (a) Always negative
(d) Process simple information and cat­egorize (b) Always positive
37. To decode a message is to (c) Related to one’s values, beliefs and past
(a) Evaluate a message experiences
(b) Translate ideas into code (d) Always mysterious
(c) Reject a message 47. Gatekeeping is related with
(d) Interpret a message (a) Intrapersonal communication
38. The process of filtering messages from source to receiver (b) Interpersonal communication
is called (c) Mass Communication
(a) Handout (d) Group communication
(b) Kinesics 48. Kinesics is related with
(c) Degeneration (a) Verbal communication
(d) Gatekeeping (b) Nonverbal communication
39. A message is a signal that serves as (c) Mass Communication
(a) Stimuli for a sender (d) Group communication
(b) Stimuli for a receiver 49. The term ‘rhetorics’ is a term related with
(c) Stimuli for a mass audience (a) Plato (b) Aristotle
(d) Noise reduction (c) Denis mcquaill (d) Marshal Mcluhan
40. .................. message have more or less same meaning 50. Listening
for the audience. (2021) (a) May be a complex process involving many steps
(a) Complex (b) Is always reflexive
(b) Connotative (c) A natural habit
(c) Denotative (d) A physiological process occurring in a spontane-
(d) Abstract ous manner
41. The subjective meaning of a word is its 51. The responding step of listening
(a) Denotative meaning (a) Is non-verbal
A S S E S S YO U R L E A R N I N G

(b) Indirect meaning (b) Depends upon human memory


(c) Antonym (c) Can be verbal or non-verbal
(d) Connotative meaning (d) Is always verbal
42. Which of the following can be defined as the functions 52. One of the most important communication skills is
of communication? (a) Active listening (b) Objective listening
1. Information function (c) Passive listening (d) Inactive listening
2. Command and instructive functions 53. A technique that might be used by an active listener is
3. Influence and persuasion function (motivational (a) Paraphrasing the speaker’s meaning
function) (b) Expressing concern
4. Integrative function (c) Explaining the speaker’s meaning
Codes: (d) Offering a viewpoint when in conversation
(a) 1, 2 and 3 (b) 2, 3 and 4 54. Effective listening includes
(c) 1, 2, 3 and 4 (d) 3 and 4 (a) Filtering out points of disagreement.
(b) Detailed analysis.
43. For which of the following word/s, the term
(c) Attaching subjective meaning to a message.
‘­
chronemics’ is used for the interpretation of
(d) Confirming one’s understanding of a message.
messages?
(a) Smell (b) Taste 55. A classification of body movements is called
(c) Time (d) All of the above (a) Emblems (b) Non-verbal
(c) Displays (d) Kinesics
44. In the communication process, a receiver
(a) Is a channel 56. Movements of the face that convey emotional mean-
ings are called
(b) Decodes a message
(a) Displays (b) Emblems
(c) Is the person who encodes an idea (c) Eye contact (d) None of the above
(d) Responsible for message interference

M04_MADAN 04_65901_C04.indd 34 21/12/22 10:54 AM


Communication 4.35

57. Touch is an important element in In the light of the above statements, choose the answer

(a) Business communication from the options given below:
(b) Interpersonal communication (a) Both (A) and (R) are true and (R) is the correct
(c) Intrapersonal communication explanation of (A)
(d) None of the above (b) Both (A) and (R) are true but (R) is NOT the cor-
58. Which of the following are parts of active listening? rect explanation of (A)
(a) Eye contact (b) Nodding (c) (A) is true but (R) is false
(c) Seeking clarification (d) All of the above (d) (A) is false but (R) is true
59. Evaluative listening is successful when we
67. Which of the following is not a successful
(a) Accurately distinguish stimuli in a message
communicator?
(b) Infer the meaning of a message
(c) Critically assess the accuracy of facts stated in a (a) One who presents material in a precise and
message clear way
(d) None of the above (b) One who is able to adopt himself/herself accord-
ing to the language of the communicatee
60. An effective communication does not require
(a) Change in speech pattern (c) One who knows a lot but is somewhat reserve in
(b) Appropriate gestures his/her attitude
(c) Mastery of content (d) One who sometimes becomes informal before the
(d) Handsome personality receiver and develops a rapport
61. One will be an effective communicator if one 68. The most important aspect of communication, i.e., lis-
(a) Is a humorous speaker tening can be improved by
(b) Has histrionic talents (a) Making the attention fully paid
(c) Is very clear about what one communicates (b) Making the communicated material novel, inter-
(d) Communicates in one’s mother tongue esting and need-based
62. Which of the following is considered unethical com- (c) Making voice effective and impressive
munication? (December 2021) (d) All of the above
(a) Use of alternative source of information 69. The process of communication is enhanced through
(b) Not reveling the source of information (a) Belongingness
(c) Non commercial publicity (b) Security and freedom to make choices
(d) Propaganda (c) Information of meeting and avoidance of pressure
63. Communication means ______ of information (d) All of the above
1. Suppression 70. Two-way communication becomes effective

A S S E S S YO U R L E A R N I N G
2. Exchange 1. Focusing on the eyes of the person speaking
3. Understanding 2. Making eye contact
4. Contextualization 3. Observing body language
5. Abstraction 4. Comments in social media
Choose the correct answer from the options given below: 5. Questions
6. Paraphrasing
(a) 1, 2 and 3 (b) 3, 4 and 5
(c) 1, 2 and 5 (d) 2, 3 and 4 Codes:
64. Which of the following steps would you consider first (a) 1, 2, 3 and 5 (b) 1, 2, 3, 4, 5 and 6
for an effective communication? (c) 2, 4, 5 and 6 (d) 3, 4, 5 and 6
(a) Select the channel of communication 71. Effective communication takes place when
(b) Plan the evaluation procedure
(a) The source is attractive and authoritarian
(c) Specify the objectives of communication
(d) Identify various media for communication (b) The message design incorporates audience
(c) Modern communication technologies are used
65. Which is ‘feedback’ in newspaper’s communication?
(d) Receivers are passive components
(a) Articles (b) Editorials
(c) Letters to the Editor (d) News 72. Given below are two statements:
66. Given below are two statements: One is labeled as Statement I: The art of Mass communications is much

Assertion (A) and the other is labelled as Reason (R): more simple than of face to face communication or any
Assertion (A): The positive feedback from the teacher type of communication.
is necessary to motivate students. Statement II: Each of us uses 3S (Sight, Sound and

Reasons (R): Continuous criticism of students is the Smell) and 2T (Touch and Taste) when we communi-
best method to make them realize positive outcomes cate with others.
in their pursuit of knowledge.

M04_MADAN 04_65901_C04.indd 35 21/12/22 10:54 AM


4.36 Chapter 4

In light of the above statements, choose the most 80. In communication, the language is
appropriate answer from the options given below: (a) The non-verbal code
(a) Both Statement I and Statement II are correct. (b) The verbal code
(b) Both Statement I and Statement II are incorrect. (c) The symbolic code
(d) The iconic code
(c) Statement I is correct but Statement II is incorrect.
(d) Statement I is incorrect but Statement II is correct. 81. Media that exist in an interconnected series of com-
munication points are referred to as
73. Which of the following is less important in the context (a) Networked media
of effective communication? (b) Connective media
(a) Total control over language (c) Nodal media
(b) Good vocabulary (d) Multimedia
(c) Attractive personality of the speaker 82. The main challenge of grapevine communication are:
(d) Total control on the content to be transacted A. Its formal nature
74. A good communicator is one who offers his/her B. Distortion
­audience C. Misunderstanding
(a) Plentiful of information D. Source anonymity
(b) A good amount of statistics E. Conformity
(c) Concise proof Choose the correct answer from the options given below:
(d) Repetition of facts
(a) A, B and C only (b) B, C and D only
75. Which of the following statements is correct? (c) C, D and E only (d) A, C and D only
(a) A communicator should have fine senses
83. Which of the following is an exclusive example of
(b) A communicator should have tolerance power Non- Verbal cue that include the pitch, rate , volume
(c) A communicator should be soft spoken and use of pauses? (December 2021)
(d) A communicator should have good personality (a) Linear Model of Communication
76. Which of the following can help the most in enhanc- (b) interactional Model
ing the effectiveness of active listening? (c) Non-Verbal Cues
(a) Developing apathy to the sender (d) Para Language
(b) Developing a system to minimize noise in the area 84. A position created to receive and respond to inquiries,
(c) Paying attention to the body language of the sender complaints, requests for policy clarification, or allega-
(d) Developing empathy with the sender tions of injustice by employees.
77. Marshall Mcluhan’s name is associated with the Through that position, employees can have their prob-
assertion: (December 2021) lems resolved quickly without going through lengthy
A S S E S S YO U R L E A R N I N G

(a) The message is the medium channels.


(b) The medium is the message
(a) Board of Directors
(c) The message determines the medium accurately
(b) Chief Executive Officer
(d) A message is a message
(c) Ombudsperson
78. Which of the following statements is not connected (d) National Judicial Commission
with communication?
85. Which of the following concept or model helps in the
(a) Medium is the message
process of giving and receiving feedback and it has four
(b) The world is an electronic cocoon
zones called as arena, facade, blind spot and unknown?
(c) Information is power
(a) Johari Window
(d) Telepathy is technological
(b) Aristotle Model
79. Postmodernism is associated with (c) Shannon and Weaver Model
(a) Newspapers (b) Magazines (d) Schramm Communication Model
(c) Radio (d) Television

Types of Communication
86.Writing in a personal diary or otherwise recording 87. Identify the sequence of human communication skills.
one’s thoughts and feelings are examples of A. Public speaking
(a) Mediated intrapersonal communication B. Group interaction
(b) Mediated interpersonal communication C. Relationship
(c) Mediated mass communication D. Feed forward
(d) None of the above F. Self-presentation

M04_MADAN 04_65901_C04.indd 36 21/12/22 10:54 AM


Communication 4.37

Choose the correct answer from the options given below: 96. Which of the following is also termed as mediated
(a) B, D, E, C, A communication?
(b) A, C, D, B, E (a) Intrapersonal communication
(c) C, E, A, D, B (b) Interpersonal communication
(d) E, C, B, A, D (c) Group communication
(d) Mass communication
88. Interpersonal communication occurs only when
(a) A person exchanges an idea with another one as a 97. Communication becomes circular when
unique individual (a) The decoder becomes an encoder
(b) A large number of people communicate with each (b) The feedback is absent
other at the same time (c) The source is credible
(c) Only friends talk (d) The channel is clear
(d) None of the above 98. Non-verbal communication includes
89. Peer-to-peer communication is (a) Delivering a speech
(a) Horizontal (b) Telephonic conversation
(b) Convoluted (c) Singing a song
(c) Non-transactional (d) Shaking hands
(d) Authoritarian 99. The following term deals with the interpretation of
90. Non-verbal messages are body language such as facial expressions and ges-
(a) Overestimated in importance tures or more formally, non-verbal behavior related to
(b) Attempts at manipulation and should be ignored movement, either of any part of the body or the body
(c) Generally irrelevant to overall message meaning as a whole. Body Language is technically known as
(d) Important for a listener to understand (a) Para-Language
(b) Kinesics
91. Intrapersonal communication helps one (c) Phonetics
(a) Learn about oneself (d) Grapevine
(b) Know what others are thinking
(c) Communicate with the general public 100. Which of the following is the technical term for the
(d) Become a talented public speaker voice cues that accompany spoken words. It is con-
cerned with the sound of the voice and the range of
92. More and more informal exchange within a group to meanings that people convey through their voices
resolve conflicts is an example of (December 2021) rather than the words they use.
(a) Horizontal communication (a) Para-Language (b) Kinesics
(b) Vertical communication (c) Phonetics (d) Grapevine
(c) Risk Communication

A S S E S S YO U R L E A R N I N G
(d) Personal communication 101. Leadership roles first emerge in which of the follow-
ing kinds of communication?
93. Communication between two or more people is called
(a) Intrapersonal communication
(a) Organizational communication
(b) Small group communication
(b) Interpersonal communication
(c) Face-to-face public communication
(c) Extrapersonal communication
(d) Media-like cell phones and instant messenger
(d) Intrapersonal communication
102. Four conditions influence the effectiveness of an
94. In interpersonal communication, ethics are
encoded message, the skills, attitudes, knowledge of
(a) Important
the sender and
(b) Communication barriers
(a) Social cultural system
(c) Ineffective
(b) Knowledge of the receiver
(d) None of the above
(c) Age of the sender
95. Which of the following are identified as verbal commu- (d) Environmental issues
nication skills?
103. Look at the following characteristics.
A. Use of aggressive language
1. It is beyond the control of the management.
B. Assertiveness
2. It is considered more reliable by the employees
C. Opening feedback channels
than the formal communication channels.
D. Taking credit for oneself
3. Employees mostly use it to serve their personal
E. Use of affirmative words
and social interests.
Choose the correct answer from the options given
Which type of communication group can be identi-
below:
fied by these ?
(a) A, B and C only (a) Formal group
(b) B, C and D only (b) Quality circles
(c) C, D and E only (c) Grapevine
(d) B, C and E only (d) External Network

M04_MADAN 04_65901_C04.indd 37 21/12/22 10:54 AM


4.38 Chapter 4

104. The lateral communication involves communication 111. Read the following statements
across chains of command. It facilitates coordina- 1. The manner in which a verbal statement is
tion among departments. It probably takes place presented, including its rhythm, breathiness,
because people prefer the informality of lateral com- hoarseness, or loudness.
munication to the formal downward and upward 2. It reflects psychological arousal, emotion, and
communication. Those actively involved in lateral mood.
communication are called 3. It may carry social information, as in a sarcastic,
(a) Man of The Match superior, or submissive manner of speaking.
(b) Boundary spanners The above statements indicate
(c) Hit The Wicket
(d) Wicket Keeper (a) Tone of voice (b) Emotional cues
(c) Both a and b (d) None of the above
105. The sequence of stage of communication process is
 (December 2021) 112. Which of the following are identified as verbal com-
1. Level of acceptance munication skills? (December 2021)
2. Transmission of cognitive data 1. use of aggressive language
3. Message reception 2. Assertiveness
4. Understanding 3. Opening feedback channels
5. Reaction 4. Taking credit for oneself
5. Use of affirmative words
Choose the correct answer from the option given
below Choose correct option below
(a) 1, 2, 3, 4, 5 (a) 1, 2 and 3 only (b) 2, 3 and 4 only
(b) 2, 3, 4, 5, 1 (c) 2, 3 and 5 only (d) 3, 4 and 5 only
(c) 3, 5, 4, 1, 2 113. Another term used for interpersonal communication
(d) 2, 3, 4, 1, 5 is
106. Given below are two statements: (a) Group communication
Statement I: Appropriate word choice is an essential (b) Face-to-face public communication
of effective oral communication. (c) Dyadic communication
Statement II: Facial expression and postures fall (d) Traditional communication
under visual communication. 114. Non-verbal message interpretation largely
In the light of the above statements, choose the cor- (a) Depends upon cultural context
rect answer from the options given below: (b) Depends upon physical context
(a) Both Statement I and Statement II are true. (c) Depends upon noise level
(d) Varies from person to person
A S S E S S YO U R L E A R N I N G

(b) Both Statement I and Statement II are false.


(c) Statement I is true but Statement II is false. 115. According to communication research findings,
(d) Statement I is false but Statement II is true. some amount of gossip is necessary for
107. The process of mass communication is not (a) Pro-social motivation
(a) Asymmetrical (b) Prevention of distortion
(b) Personal (c) Extending communication range
(c) Anonymous (d) Relocation of emotional affiliations
(d) Effected on a large scale 116. Audio-conferencing may be classified among which
108. Which of the communication has more emotional of the following types of communication?
appeal? (a) One-sided verbal
(a) Intrapersonal (b) Two-sided verbal
(b) Interpersonal (c) One-sided non-verbal
(c) Group (d) Two-sided non-verbal
(d) Mass 117. Recording a television programme on a VCR is an
109. Who proposed “The Agenda Setting theory”? example of
(a) Defer and Ballrockeach (a) Time-shifting
(b) Maxwell McComb and Donald Shaw (b) Content reference
(c) Joseph Klapper (c) Mechanical clarity
(d) Carl Howlan (d) Media synchronization
110. Which of the following model refers to gatekeeper 118. In describing elements of communication, person or
concept ? an event which provide you verbal or non-verbal use
(a) Newcombs to which someone can respond is known as
(b) Shannon and Weavers  (December 2021)
(c) Westleys and Leans (a) Source (b) Message
(d) George Gerbener (c) Channel (d) Communication Context

M04_MADAN 04_65901_C04.indd 38 21/12/22 10:54 AM


Communication 4.39

119. A negative reaction to a mediated communication is 126. Classroom communication involves 


described as  (November 2020)
(a) Flak A. Listening by learning
(b) Fragmented feedback B. Formal lecturing
(c) Passive response C. Informal discussions
(d) Non-conformity D. Less importance to feedback
120. In communication, chatting over the internet is E. Explaining
(a) Verbal communication Choose the correct answer from the options given below:
(b) Non-verbal communication (a) A, B, C only
(c) Parallel communication (b) B, C, D only
(d) Grapevine communication (c) B, C, E only
(d) C, D, E only
121. Match The Following
127. Given below are two statements: One is labelled as
Term Description Assertion (A) and the other is labelled as Reason (R):
 (November 2020)
1. Proxemics A. A personal space aspect of
body language Assertion (A): To be effective in the classroom, a
teachers should minimise the communication barriers.
2. Oculesics B. Role of eye contact in non
verbal communication Reasons (R): Since communication is a complex
process, regular feedback analysis is essential for this
3. Chronemics C. Use of time in nonverbal purpose.
Communication
In the light of the above statements, choose the most
4. Haptics D. The study of touching appropriate answer from the options given below:
(a) Both (A) and (R) are correct and (R) is the cor-
Codes: rect explanation of (A)
(a) 1–A, 2–C, 3–B, 4–D (b) Both (A) and (R) are correct but (R) is NOT the
(b) 1–A, 2–B, 3–C, 4–D correct explanation of (A)
(c) 1–A, 2–B, 3–D, 4–C (c) (A) is correct but (R) is not correct
(d) 1–D, 2–C, 3–B, 4–A (d) (A) is not correct but (R) is correct
122. Public communication tends to occur within a more 128. Match List I with List II. List I gives Modes of Com-
(a) Complex structure munication, while list II provides their description
(b) Political structure  (November 2020)
(c) Convenient structure

A S S E S S YO U R L E A R N I N G
(d) Formal structure List I List II
123. The information function of mass communication is (Modes of
Communication) (Description)
described as
(a) Diffusion (b) Publicity A. Publicity I. Reputation
(c) Surveillance (d) Diversion management
124. The main aims of interactive communication are B. Propaganda II. Paid message
1. Task coordination
2. Problem solving C. Advertising III. Non-commercial
3. Information sharing promotion
4. Conflict resolution D. Public relations IV. Communication with
5. Dominating other team members motives
Codes: Choose the correct answer from the options given

(a) 2, 3, 4 and 5 below:
(b) 1, 2, 3 and 4
(a) A-I, B-II, C-III, D-IV
(c) 3, 4 and 5
(b) A-II, B-III, C-IV, D-I
(d) 1, 2, 3, 4 and 5
(c) A-IV, B-I, C-III, D-II
125. Which of the following is essential in classroom com- (d) A–III, B-IV, C-II, D-I
munication? (November 2020)
129. Classroom communication is (November 2020)
(a) Frequent repetition of the message in different
A. Goal-oriented
ways
B. Style-centric
(b) Avoiding repetition of the message
C. Collaborative
(c) Interrupting the speaker frequently
D.Vertical
(d) Avoiding eye contact with students
E. Non-rhetorical

M04_MADAN 04_65901_C04.indd 39 21/12/22 10:54 AM


4.40 Chapter 4

Choose the correct answer from the options given D. Grapevine IV. It is the most common
below: Communication communication system
(a) A and B only (b) B and D only in an organization
(c) A and C only (d) D and E only
Choose the correct answer from the options given
130. Which of the following is essential in classroom com-
below:
munication? (November 2020)
(a) (A)–(I), (B)–(II), (C)–(IV), (D)–(III)
(a) Frequent repetition of the message in different
(b) (A)–(II), (B)–(I), (C)–(III), (D)–(IV)
ways
(c) (A)–(III), (B)–(IV), (C)–(I), (D)–(II)
(b) Avoiding repetition of the message
(d) (A)–(IV), (B)–(III), (C)–(II), (D)–(I)
(c) Interrupting the speaker frequently
(d) Avoiding eye contact with students 133. Identify the CORRECT sequence of categories of
grapevine communication
131. Classroom communication involves
A. Cluster chain
A. Listening by learning
B. Probability chain
B. Formal lecturing
C. Gossip chain
C. Informal discussions
D.Single strand chain
D.Less importance to feedback
E. Explaining Choose the correct answer from the options given
below:
Choose the correct answer from the options given
(a) A, B, C, D
below:
(b) B, C, D, A
(a) A, B, C only
(c) D, C, B, A
(b) B, C, D only
(d) C, D, A, B
(c) B, C, E only
(d) C, D, E only 134. Match List I with List II:
132. Match List I with List II: List I List II
List I List II Categories of
Communication Features
Types of
Communication Main Features A. Lateral I. Touch communication
A. Downward I. Grows primarily to B. Haptics II. Between equals
Communication meet organizational/ C. Semantics III. Interpretation of signs
member’s need for
communication D. Semiotics IV. Concerned with
A S S E S S YO U R L E A R N I N G

meanings
B. Upward II. It forms a useful link
Communication in decision making for Choose the correct answer from the options given

task coordination below:
(a) A-II, B-I, C-IV, D-III
C. Horizontal III. Provides management (b) A-III, B-II, C-I, D-IV
Communication with feedback about (c) A-IV, B-III, C-II, D-I
current issues and (d) A–I, B-IV, C-III, D-II
problems

Communication Barriers
135. Disturbances that interfere with the transmission, 137. Identify the barrier to communication from the list
receipt or feedback of a message are called given below: (December 2021)
(a) Feedback 1. Time and distance
(b) Feed forward 2. By passed instructions
(c) The channel 3. Encoding of thoughts and ideas
(d) Noise 4. Decoding of thoughts and Ideas
136. The most powerful barrier of communication in the 5. Logical sequence
class is Choose the correct answer from the options given below:
(a) Noise in the classroom (a) 3 and 4 only
(b) Confusion on the part of teacher (b) 2 and 3 only
(c) More outside disturbance in the classroom (c) 1 and 2 only
(d) Lack of teaching aids (d) 4 and 5 only

M04_MADAN 04_65901_C04.indd 40 21/12/22 10:54 AM


Communication 4.41

138. A disruption in the communication process is called (a) Generalizing (b) Consistency
(a) Transgression (b) Feedback (c) Stereotyping (d) Oversimplification
(c) Noise (d) Interaction 148. The goal of perception checking is
139. Which of the following is not true about the (a) Confirmation
grapevine? (b) Cultural sensitivity
(a) It is faster than formal communication network (c) To further explore the thoughts and feelings of
(b) It is less accurate than formal network others
(c) It is found in almost all organizations (d) Control
(d) It is preferred over formal communication 149. Which is a characteristic of prejudice?
140. Level C of the effectiveness of communication is (a) Generalized evaluation, specifically of out-
defined as group members
(a) Channel noise (b) Semantic noise (b) Negativity
(c) Psychological noise (d) Source noise (c) Biased perceptions
141. All of the following are barriers to effective commu- (d) All of the above
nication except 150. The main assumption of ‘primacy effect’ is
(a) Absence of noise (a) The most information is always one that
(b) Distortion of information comes first.
(c) Information overloads (b) The most important piece of information
(d) Cognitive dissonance comes last.
142. Informal transmission of information or filtered (c) The degree of importance depends upon the
information is called situation.
(a) Gossip (d) All pieces of information carry the same weight.
(b) Grapevine 151. In communication, a major barrier to reception of
(c) Vertical communication ­messages is
(d) Horizontal communication (a) Audience attitude
143. An example of physiological noise is (b) Audience knowledge
(a) Feeling hungry (c) Audience education
(b) A humming air conditioner unit (d) Audience income
(c) A lawn mower 152. Semantic noise in classroom communication can be
(d) A speaker using complex terms limited by avoiding the use of
144. A speaker using complex terms is an example of (a) Dialogues
(a) Physiological noise (b) Psychological noise (b) Cliched jargon

A S S E S S YO U R L E A R N I N G
(c) Semantic noise (d) Physical noise (c) Non verbal cues
(d) Multi-media
145. Which one of the following is an example of a tech-
nological barrier in communication? 153. Which of the following are useful in overcoming the
 (December 2021) communication barriers in a classroom?
(a) Physical location of communicators  (November 2020)
(b) Low bandwidth of Internet A. Identifying the level of redundancy
(c) Personal attitude towards technology B. Use of unfamiliar words
(d) Complexity of ideas C. Ignoring the students’ vocabulary
D.Fragmented sentences
146. A noise in the communication process E. Voice inflexion
(a) Causes listeners to listen to messages more F. Contextualizing the speech
carefully
(b) Interferes with a message Choose the correct answer from the options given
(c) Focuses wandering thoughts below:
(d) Enhances a message (a) Only A, B and C
(b) Only B, C and D
147. A fixed and categorized impression of a group of (c) Only C, D and F
people based on a predetermined set of qualities (d) Only A, E and F
is called

M04_MADAN 04_65901_C04.indd 41 21/12/22 10:54 AM


4.42 Chapter 4

Classroom Communication

154. A teacher will become an effective communicator if (c) Speaking with high authority
(a) She/he uses instructional aids (d) Telling what is useful to the listeners
(b) She/he helps students get meaning out of what 162. Given below are two statements
she/he teaches
(c) She/he asks question in between teaching Statement I: Body language is the basis of deceptive
(d) She/he helps students get correct answer to the communication.
questions on the topic Statement II: Non-verbal cues have their own
shared meanings.
155. The psychological aspects of the classroom are best
managed by In the light of the above statements, choose the correct
(a) The class teacher answer from the option given below.
(b) The subject teacher  (December 2021)
(c) The principal (a) Statement I is false but Statement II is true.
(d) The student themselves (b) Statement I is true but Statement II is false
(c) Both Statement I and Statement II are true
156. Books can be powerful source of communication, (d) Both Statement I and Statement II are false
provided
(a) The content is abstract 163. The facial expression of students relate to which
(b) The content is illustrative ­element of the communication process?
(c) The medium is Hindi (a) Message (b) Receiver
(d) The content is presented through good print (c) Channel (d) Sender
157. Which of the following statements is not correct? 164. Which group of communication aspects does not
(a) A good communicator cannot be a good teacher ­disrupt the communication process in the class?
(b) A good communicator has a good sense of (a) Reversing–evaluating–focussing
humour (b) Evaluating–focussing–illustrating
(c) A good communicator has wide reading (c) Evaluating–focussing–exaggerating
(d) A good communicator has command over (d) Focussing–illustrating–exaggerating
language 165. Which of the following will not hamper effective
158. Which of the following are the main constituents of communication in the classroom?
paralanguage? (a) A lengthy statement
1. Tone (b) An ambiguous statement
(c) A precise statement
A S S E S S YO U R L E A R N I N G

2. Pitch
3. Quality of voice (d) A statement that allows the listener to draw his/
4. Rate of speaking her own conclusions
(a) 1, 2 and 3 166. Which of the following cannot be a good way in pro-
(b) 2, 3 and 4 moting literacy among villagers?
(c) 1, 2 and 4 (a) Demonstration
(d) All of the above (b) Reading and writing
159. If a student raises a hand to speak in class, the person (c) Providing material on TV and film projector
is using (d) Large group discussion
(a) Modifier (b) Emblem 167. What are the most prominent elements in the
(c) Regulator (d) Display management of communication in an educational
160. Communication in the class fails because institution? (November 2020)
(a) The students are inattentive A. Sidetracking oppositional views
(b) The teacher is monotonous in delivering the B. Structuring work roles
message C. Commanding over colleagues and students
(c) The students have no interest in the lesson being D.Keep others informed
taught E. Guiding others wherever possible
(d) There is much noise in and around the Choose the correct answer from the options given
classroom below:
161. Which of the following will make communication (a) A, B and C only
more effective? (b) B, C and D only
(a) Cracking jokes in between (c) C, D and E only
(b) Using multi-sensory appeal (d) B, D and E only

M04_MADAN 04_65901_C04.indd 42 21/12/22 10:54 AM


Communication 4.43

Miscellaneous
168. Journal-like entries written by individuals and (c) William Francis
posted on the Internet that sometimes can generate (d) James gold
many online discussions are 177. A sender in communication process has very good
(a) Weblogs grammar. This competence is basically
(b) Simple posting (a) Phonemic (b) Semantic
(c) E-journals (c) Syntactic (d) Pragmatic
(d) None of the above
178. The communication among persons working in dif-
169. According to Mehrabian, the respective contribu- ferent levels who have no direct reporting relation-
tions of (i) words, (ii) tone of voice and (iii) body ship is called.
language in overall communication are (a) Interpersonal communication
(a) 38%, 7% and 55% (b) Horizontal Communication
(b) 7%, 38% and 55% (c) Diagonal communication
(c) 55%, 38% and 7 % (d) Upward communication
(d) None of the above
179. A location where the internet users can gain wireless
170. Paraphrasing is a skill that is absolutely necessary to access to the Internet is
effective (a) Extranet
(a) Listening (b) Intranet
(b) Hearing (c) Web conference
(c) Speaking (d) Wi-Fi hotspot
(d) None of the above
180. Which of the following statement(s) is/are true in
171. Marshal Mcluhan’s name is associated with the the context of paraphrasing?
assertion: (December 2021) 1. It is basically about stating in your own
(a) The message is the medium words, your understanding of what has just
(b) The medium is the message been said
(c) The Message determines the medium accurately 2. It gives speaker opportunity to find out what
(d) A message is a message message he/she is getting across to you
172. The transmission of culture from one generation to (a) Only 1 (b) Only 2
another is called (c) Both 1 and 2 (d) Neither 1 nor 2
(a) Acculturation 181. An example of asynchronous medium is
(b) Enculturation (a) Radio (b) Television

A S S E S S YO U R L E A R N I N G
(c) Interculturation (c) Film (d) Newspaper
(d) None of the above 182. In communication, connotative words are
173. Leakage cues refer to (a) Explicit (b) Abstract
(a) Facial expressions that people have misread. (c) Simple (d) Cultural
(b) One of the characteristic styles of facial 183. Who developed Wireless telegraphy ?
expressions. (a) Heinrich Hertz
(c) Unintended signs of how a person really feels. (b) Thomas Edison
(d) A technique employed by professional actors. (c) J C Maxwell
174. Who primarily contributed towards the develop- (d) Guglielmo Marconi
ment of www? 184. A message beneath a message is labelled as
(a) Steve Jobs (a) Embedded text (b) Internal text
(b) Tim Berners Lee (c) Intertext (d) Subtext
(c) Henry William Gates
(d) Michael Dell 185. In analogue mass communication, stories are
(a) Static (b) Dynamic
175. A person has a very advanced sense of what is (c) Interactive (d) Exploratory
socially appropriate and always knows what to say
in every social context. The person has which kind 186. The ability to understand, communicate with, moti-
of linguistic competence? vate, and support other people, both individually
(a) Phonemic (b) Cognitive and in groups, defines which of the following organ-
(c) Syntactic (d) Pragmatic izational skills?
(a) Hard skills
176. Comment is free; facts are sacred. Who made this (b) Soft skills
statement? (c) Conceptual skills
(a) CP Scott (d) Political skills
(b) Brian Roberts

M04_MADAN 04_65901_C04.indd 43 21/12/22 10:54 AM


4.44 Chapter 4

187. Match List I with List II: 1. Hedges are pragmatic values that modifies the
speech act, not the logical form of the utterance.
List I List II 2. Hedges are referred to as a ‘direct modification of
Concept of Power propositional content’.
Communication Description of feature 3. Hedges may be divided into “strengtheners” and
A. Legitimate Power I. Specialised knowledge “softners”.
4. Hedges indicate something about the speaker’s
B. Referent Power II. By virtue of position commitment toward what he/she is saying, and
C. Reward Power III. When others feel to be in so doing modify the illocutionary force.
like you Codes:
D. Expert Power IV. Control over what (a) 1, 2 and 3
others seek (b) 2, 3 and 4
(c) 1, 2, 3 and 4
Choose the correct answer from the options given
(d) 1, 3 and 4
below:
191. In speech-act theory, the following can be defined as
(a) A–I, B–II, C–III, D–IV
an act of making a meaningful utterance, a stretch
(b) A–III, B–IV, C–I, D–II
of spoken language that is preceded by silence and
(c) A–IV, B–I, C–II, D–III
followed by silence or a change of speaker ?
(d) A–II, B–III, C–IV, D–I
(a) Locutionary Act
188. Which of the following statements are true in the (b) Illocutionary act
context of Emoticons? (c) Perlocutionary Act
(1) Emoticons are basically visual representations (d) None of the above
of writers’ emotions.
192. Feedback reflects the reactions of the audience when:
(2) Emoticon is often taken as utterances in computer
related communication that are interpreted. (A) One knows the effects of communication.
(3) Emoticons are basically graphic representations (B) One knows the quality of content of the message.
of facial expressions and construction of the (C) One knows the future prospect of the object of
words such as “emotion” and “icon”. communication.
(4) Emoticons may be produced by ASCII symbols (D) One knows the financial implications of the
(:-)) or by “pictograms,” which are graphic communication.
symbols (). Choose the most appropriate answer from the options
Codes: given below:
A S S E S S YO U R L E A R N I N G

(a) 1, 2 and 3 (a) (A), (B), (D) only


(b) 2, 3 and 4 (b) (B), (D), (C) only
(c) 1, 2, 3 and 4 (c) (A), (C), (D) only
(d) 1, 3 and 4 (d) (A), (B), (C) only
189. Match The Following 193. The power which comes through connectedness of
one’s words matching one’s feelings, body and facial
Set 1 Set 2 expression in communication process is known as:
(Functions) (Description) (a) Conformity (b) Congruence
A. Positive Attitude 1. Expressive speech (c) Collaboration (d) Control
acts–‘strengtheners’, 194. Given below are two statements, one is labelled as
directives and softeners Assertion A and the other is labelled as Reason R
B. Humorous 2. Following utterances, Assertion A: Smiles and laughter should be avoided
Function they joke/irony markers when messages are serious.
C. Hedge Function 3. Specifically following Reason R: Non-verbal cues are always contextual
signatures like verbal communication.
In light of the above statements, choose the correct
Codes: answer from the options given below:
(a) A–1, B–2, C–3  (December 2021)
(b) A–3, B–2, C–1
(a) Both A and R are true and R is the correct expla-
(c) A–2, B–3, C–1
nation of A
(d) A–1, B–3, C–2
(b) Both A and R are true but R is NOT the correct
190. Which of the following statements are true in the explanation of A
context of ‘hedges’ in context of emoticons? (c) A is true but R is false
(d) A is false but R is true

M04_MADAN 04_65901_C04.indd 44 21/12/22 10:54 AM


Communication 4.45

Answer Keys
Concept and Nature of Communication
1. (a) 2. (d) 3. (d) 4. (b) 5. (a) 6. (d) 7. (b) 8. (a) 9. (c) 10. (a)
11. (d) 12. (d) 13. (c) 14. (b) 15. (b) 16. (a) 17. (d) 18. (b) 19. (c) 20. (b)
21. (d) 22. (a) 23. (b) 24. (c) 25. (d) 26. (d) 27. (c) 28. (d) 29. (c) 30. (d)
31. (d) 32. (a) 33. (a) 34. (a) 35. (a) 36. (b) 37. (d) 38. (d) 39. (b) 40. (c)
41. (d) 42. (c) 43. (c) 44. (b) 45. (c) 46. (c) 47. (c) 48. (b) 49. (b) 50. (a)
51. (c) 52. (a) 53. (a) 54. (d) 55. (d) 56. (a) 57. (b) 58. (d) 59. (d) 60. (d)
61. (c) 62. (d) 63. (d) 64. (c) 65. (c) 66. (c) 67. (c) 68. (d) 69. (d) 70. (b)
71. (b) 72. (d) 73. (c) 74. (a) 75. (a) 76. (d) 77. (b) 78. (d) 79. (d) 80. (b)
81. (a) 82. (b) 83. (d) 84. (c) 85. (a)
Types of Communication
86. (a) 87. (d) 88. (a) 89. (a) 90. (d) 91. (a) 92. (a) 93. (b) 94. (a) 95. (d)
96. (d) 97. (a) 98. (d) 99. (b) 100. (a) 101. (b) 102. (a) 103. (c) 104. (b) 105. (d)
106. (a) 107. (b) 108. (b) 109. (b) 110. (c) 111. (c) 112. (c) 113. (c) 114. (a) 115. (a)
116. (b) 117. (a) 118. (a) 119. (a) 120. (a) 121. (b) 122. (d) 123. (c) 124. (b) 125. (a)
126. (c) 127. (a) 128. (d) 129. (c) 130. (a) 131. (c) 132. (d) 133. (c) 134. (a)
Communication Barriers
135. (d) 136. (b) 137. (c) 138. (c) 139. (d) 140. (a) 141. (a) 142. (b) 143. (a) 144. (c)
145. (b) 146. (b) 147. (c) 148. (c) 149. (d) 150. (a) 151. (a) 152. (b) 153. (d)
Classroom Communication
154. (b) 155. (a) 156. (b) 157. (a) 158. (d) 159. (b) 160. (b) 161. (b) 162. (a) 163. (a)
 164. (d) 165. (c) 166. (d) 167. (d)
Miscellaneous
168. (a) 169. (b) 170. (a) 171. (b) 172. (b) 173. (c) 174. (a) 175. (d) 176. (a) 177. (c)
178. (d) 179. (d) 180. (c) 181. (d) 182. (d) 183. (d) 184. (d) 185. (a) 186. (b) 187. (d)
188. (c) 189. (b) 190. (c) 191. (a) 192. (d) 193. (b) 194. (a)

M04_MADAN 04_65901_C04.indd 45 21/12/22 10:54 AM


This page is intentionally left blank

M04_MADAN 04_65901_C04.indd 46 21/12/22 10:54 AM


ChAPtER Mathematical
5 Reasoning and
Aptitude

01 Concept of Reasoning

02 Types of Reasoning

03 Number Series, Letter Series,


LEARnIng Codes and Relationships

OBJECtIVES

04 Mathematical Aptitude (Fraction, Time


and Distance, Ratio, Proportion and
Percentage, Profit and Loss, Interest
and Discounting, Averages, etc.)

05 Solved Examples for Better Understanding of Concepts

M05_MADAN 04_65901_C05.indd 1 23/12/22 7:23 PM


5.2 Chapter 5

based on probabilities. Here, it is presumed that the


Reasoning most plausible conclusion is also the correct one.
Reasoning, one of the highest orders of thinking, is
the stepwise thinking and mental recognition of cause Logical
and effect relationships. It involves productive think- reasoning
ing in which insight and past experiences are required.
Reasoning is a factor of intelligence. It is a process in
which pre-knowledge, experiences, insight and under- Deductive Inductive Abductive
standing of relationship are used to solve problems. The reasoning reasoning reasoning
ability to reason is closely related to intelligence. It goes in
the direction given to the learner, that is, it is always goal- Syllogisms Non-verbal sequences
oriented. It is creative and reflective in nature. Reasoning Number sequences
ability develops gradually. It means that experiences are
also helpful in developing reasoning power along with Figure 5.1 Reasoning and its Types
intelligence. There may be more than one logic to draw
an inference—reasoning is multi-dimensional. Example:
Numerous philosophical mathematicians and psychol- Major premise: The container is filled with yellow peb-
ogists have given the following six steps for reasoning: bles.
Minor premise: Bobby has a yellow pebble in his hand.
Conclusion: The yellow pebble in Bobby’s hand was
taken out of the container.
Identification Defining the By abductive reasoning, the possibility that Bobby
of the problem problem took the yellow pebble from the container is reason-
able, though it is purely based on speculation. Anyone
could have given the yellow pebble to Bobby, or prob-
ably Bobby could have bought a yellow pebble at a retail
Verification/ Formation of store. Therefore, abducing that Bobby took the yellow
evaluation of hypothesis pebble, from the observation of ‘the yellow pebble–filled
hypothesis container’, may lead to a false conclusion. Unlike deduc-
tive and inductive reasoning, abductive reasoning is not
commonly used for psychometric testing.
Here, we can discuss other forms of reasoning as well.
Tabulating and Collection
systematizing of data Hypothetical Syllogism (Modus Ponens)
data A syllogism is simply a three-line argument that consists
of exactly two premises and a conclusion. A hypothetical
syllogism is a syllogism that includes at least one hypo-
thetical or conditional (if–then) premise. This is why
Types of Reasoning such type of deductive reasoning is also known as condi-
tional reasoning. This pattern of reasoning is also known
Aristotle gave an extended, systematic treatment of the
as modus ponens. The four varieties of modus ponens are
methods of human reasoning. The three methods were
as follows.
deductive, inductive and abductive reasonings (Fig. 5.1).
1. Deductive reasoning: Deductive reasoning is also Chain Argument
known as analytical reasoning as it deals with objects Chain arguments consist of three conditional statements
by looking at its component parts. Formal logic has that are linked together. Here is an example of a chain
been described as the science of deduction. The con- argument.
cept of syllogism has been explained in Chapter 6. If I do not appear in the exam, then I will fail in
2. Inductive reasoning: Inductive reasoning is also graduation.
known as ‘synthetic reasoning’ and deals with a class If I fail in graduation, then I will lose my time and
of objects by looking at the common properties of money.
each object in the class. The study of inductive rea- Therefore, if I do not appear in exam, I will lose my
soning is generally carried out within a field known time and money.
as ‘informal logic’ or ‘critical thinking’.
3. Abductive reasoning: Abductive reasoning is con- Modus Tollens
sidered as the third form of reasoning. It is somewhat These are sometimes called ‘denying the consequences’
similar to inductive reasoning. It takes its clues from because they consist of one conditional premise, a second
the term ‘guessing’, since conclusions drawn here are premise that denies (asserts to be false) the c­ onsequences

M05_MADAN 04_65901_C05.indd 2 23/12/22 7:23 PM


Mathematical Reasoning and Aptitude 5.3

of the conditional and a conclusion that denies to be the measurement (perhaps in conjunction with one or more
antecedent of the conditional. Here is an example of non-mathematical premises).
modus tollens argument. Twelve is greater than eight.
If we are in Manchester, then we are in Gujarat. Eight is greater than four.
We are not in Gujarat. Therefore, twelve is greater than four.
Therefore, we are not in Manchester.

Denying the Antecedent Argument Inductive Reasoning


In such arguments, the first premise denies (i.e. asserts to
Let us discuss the inductive reasoning in more detail.
be false) the antecedent of the conditional and a conclu-
The statement or proposition is based on general
sion denies the consequent of the conditional.
observations and experiences; such reasoning is called
Here is an example.
inductive reasoning. There is a strong contrast with
If we are in Chandigarh, then we are in the North.
deductive reasoning. Even in the strongest cases of
We are not in Chandigarh.
inductive reasoning, the truth of the premises does not
Therefore, we are not in the North.
guarantee the truth of the conclusion. Rather, the con-
We can notice that the premises in the above–men-
clusion of an inductive argument follows some degree
tioned examples are true and the conclusion is false.
of probability. There may be more information in the
The pattern of reasoning of this argument is not logically
conclusion of an inductive reasoning than that already
reliable.
contained in the premises. Thus, this method of reason-
Affirming the Consequent ing is applicative.
In inductive reasoning, a statement in one particu-
This pattern of reasoning is also faulty and affirms the lar case will be true in all other cases in the same serial
consequent. order. It may be applied generally to all such cases. Here,
For example, if we are on Venus, then we are in the one can formulate a generalized statement or principle
solar system. and conclusions on the basis of certain facts and specific
We are in the solar system. examples.
Therefore, we are on Venus. Once we have discussed inductive reasoning, we
Such pattern of reasoning has true premises and a false can now discuss six common patterns of inductive
conclusion; it is clear that affirming the consequent is not reasoning:
logically reliable.
1. Inductive generalization: It is an argument in which
Categorical Syllogism a generalization is claimed to be probably true based
They have been discussed in Unit 6 also. Here, the state- on the information about some members of a class. All
ments of the premises begin typically with ‘all’, ‘none’ inductive generalizations claim that their conclusions
or ‘some’, and the conclusion starts with ‘therefore’ or are probable rather than certain.
‘hence’. 2. Predictive argument: It is a statement about what
may or will happen in the future; here a prediction is
Argument from Definition defended with reasons. It is among the most common
In an argument from definition, the conclusion is pre- patterns of inductive reasoning. Here is an example: It
sented as being true by definition. has rained in Mumbai every June since weather records
Nand is a cardiologist. Therefore, Nand is a doctor. have been maintained. Therefore, it will probably rain
A straightforward relationship between the cardiolo- in Mumbai next June.
gist and the doctor (two elements) is observed here. The   Nothing in the future is absolutely certain; argu-
conclusion of valid deductive reasoning contains no more ments containing predictions are usually inductive.
information than the premise. 3. Argument from authority: The conclusion is sup-
ported by presumed authority or witness who has
Argument by Elimination said that the conclusion is true. We can never be abso-
lutely certain that a presumed authority or witness is
An argument by elimination seeks to logically rule out vari- accurate or reliable.
ous possibilities until only a single possibility remains. It is 4. Causal argument: One of the most basic, most com-
like attempting questions in an examination. mon and most important kinds of knowledge we seek
is the knowledge of cause and effect. A causal argu-
Argument Based on Mathematics ment asserts or denies that something is the cause of
The main aim of mathematics is to develop the reasoning something else.
power of humanity. 5. Statistical argument: A statistical argument rests
Here is an example. on statistical evidence, that is, evidence that some
Here, the conclusion is claimed to depend largely percentage of some group has some particular
or entirely on some mathematical calculation or characteristics.

M05_MADAN 04_65901_C05.indd 3 23/12/22 7:23 PM


5.4 Chapter 5

6. Argument from analogy: An analogy is a compari- Number Series


son of two or more things that are claimed to be alike
in some relevant respect. Prime Number Series
Example 1: 2, 3, 5, 7, 11, 13, 17, …
Towards the end, we can say that inductive reasoning is
informative because the conclusion of an inductive rea- (a) 15 (b) 17 (c) 18 (d) 19
soning contains more information than is already con-
tained in the premises. Solution: The given series is a prime number series. The
next prime number is 19.
Other Reasoning Types Therefore, (d) is the correct answer.
We are using verbal and non-verbal symbols to communi-
cate with others. There are five types of reasoning, which Example 2: 2, 5, 11, 17, 23, 31, …
are explained as follows: (a) 33 (b) 37 (c) 41 (d) 43
1. Verbal reasoning: Normally, we communicate with Solution: The prime numbers in this range are 2, 3, 5, 7,
others by language, and the language is a vehicle for 11, 13, 17, 19, 23, 29, 31, 37, 41, 43, … .
reasoning. So reasoning without language or words Prime numbers have been written alternatively. Thus,
and symbols is not possible. In verbal reasoning, we after 31, the prime numbers are 37, 41, … . Ignoring 37,
use linguistic symbols such as words. Some verbal the answer is 41.
reasoning ability tests are available.
2. Non-verbal reasoning: In competitive examinations, Therefore, (c) is the correct answer.
we observe that a part of the written paper consists of
reasoning, containing ome figures, graphs and draw- Difference Series
ings which can measure the non-verbal reasoning Example 3: 2, 5, 8, 11, 14, 17, …, 23, 26
ability of the contestant.
3. Reasoning as propositions: It is often difficult to (a) 19 (b) 21 (c) 20 (d) 18
determine whether a long and complex argument is
Solution: The difference between the numbers is 3
valid or invalid just by reading. The logical process of
(17 + 3 = 20).
analysis of the parts of whole arguments and symbol-
izing them to determine the validity of arguments is Therefore, (c) is the correct answer.
known as propositional reasoning.
  Here is an example. Example 4: 45, 38, …, 24, 17, 10, 3
  If a = 4, b = 7, a + c = b, then c = ? (a) 31 (b) 34 (c) 38 (d) 29
  4 + c = 7.
  Therefore, c = 3. Solution: The difference between the consecutive
4. Automated reasoning: It is basically an area of com- numbers is 7 (38 − 7 = 31).
puter science. It understands different aspects of rea- Therefore, (a) is the correct answer.
soning to allow the creation of computer software to
reason completely or almost completely automati- Multiplication Series
cally. Sometimes, it is usually considered a subfield
of artificial intelligence, but it also has strong con- Example 5: 2, 6, 18, 54, …, 486, 1458
nections to theoretical computer science and even (a) 152 (b) 182 (c) 162 (d) 108
philosophy.
5. Brain’s centre of reasoning: The left hemisphere Solution: The numbers are multiplied by 3 to get the next
is dominant for most people; it controls written and number (54 × 3 = 162).
spoken languages and mathematical calculations. Therefore, (c) is the correct answer.
The prefrontal cortex is the farthest forward area in
the brain and is an associated area of the frontal lobe. Example 6: 3, 12, 48, …, 768, 3072
The left hemisphere of the brain is said to be analyti-
cal, logical, mathematical and concerned with cause (a) 192 (b) 216 (c) 512 (d) 72
and effect related to scientific thinking.
Solution: The numbers are multiplied by 4 to get the next
number (48 × 4 = 192).
Series Completion Therefore, (a) is the correct answer.
A series may consist of a number series or a letter series.
There are several such series, such as finding the missing Division Series
numbers, replacing the wrong numbers, finding the miss-
ing letters and finding the wrong group of numbers or let- Example 7: 720, 120, 24, 6, …, 1
ters, to name a few. (a) 1 (b) 2 (c) 3 (d) 4

M05_MADAN 04_65901_C05.indd 4 23/12/22 7:23 PM


Mathematical Reasoning and Aptitude 5.5

Solution: 720 divided by 6 = 120 Alternatively: The series is 12 − 1, 22 − 2, 32 − 3, 42 − 4,


120 divided by 5 = 24 52 − 5, 62 − 6, 72 − 7, 82 − 8 and so on.
24 divided by 4 = 6 Therefore, (c) is the correct answer.
6 divided by 3 = 2 N 3 Series
2 divided by 2 = 1
Example 14: 1, 8, 27, 64, 125, 216, …
Therefore, (b) is the correct answer.
(a) 256 (b) 343 (c) 365 (d) 400
Example 8: 32, 48, 72, …, 162, 243
Solution: The series is 13, 23, 33, etc. The missing number
(a) 84 (b) 96 (c) 108 (d) 132 is 73 = 343.
Solution: Each number is being multiplied by 3/2 to get Therefore, (b) is the correct answer.
the next number.
N 3 + 1 Series
Therefore, (c) is the correct answer.
Example 15: 2, 9, 28, 65, 126, 217, 344, …
N 2 Series (a) 513 (b) 362 (c) 369 (d) 361
Example 9: 1, 4, 9, 16, 25, 36, …, 64
Solution: The series is 13 + 1, 23 + 1, 33 + 1 and so on.
(a) 42 (b) 44 (c) 45 (d) 49 Thus, the missing number is 83 + 1 = 513.
Solution: The series is squares of 1, 2, 3, 4 and so on. Therefore, (a) is the correct answer.
Therefore, (d) is the correct answer.
Letter Series
Example 10: 0, 4, 16, 36, 64, …, 144
In this type of problem, a series of letters of the English
(a) 100 (b) 84 (c) 96 (d) 120 alphabet will be given which follow a pattern or a
Solution: The series is squares of even numbers, such as sequence. The letter series mainly consists of skipping the
2, 4, 6, 8, 10 and 12. Hence, the answer is 102 = 100. letters.
To solve these types of problems, assign numbers
Therefore, (a) is the correct answer. 1–26 to the letters of English alphabet as shown below.
In some cases, it is useful to assign the numbers in a
N 2 − 1 Series reverse order.
Example 11: 0, 3, 8, 15, 24, 35, 48, 63, …
Concept of ‘EJOTY’
(a) 80 (b) 82
(c) 83 (d) None of the above Letters E J O T Y
Solution: The series is 12 − 1, 22 − 1, 32 − 1 and so on. The Position 5 10 15 20 25
next number is 92 − 1 = 80.
Therefore, (a) is the correct answer. The candidates can determine the relative positions of
various alphabets by just remembering the word ‘EJOTY’.
Alternative solution: The differences between the
Various types of letter series are given as ­follows:
numbers across the series are 3, 5, 7, 9, 11, 13, 15 and 17.
The next number is 63 + 17 = 80. • First alphabetical half: A to M → 1–13
• Second alphabetical half: N to Z → 14–26
N 2 + 1 Series
Tables 5.1 and 5.2 show both the forward and the
Example 12: 2, 5, 10, 17, 26, 37, …, 65 backward place values of the English alphabet. A very
(a) 50 (b) 48 (c) 49 (d) 51 important fact about the position of any letter is that the
sum of forward position and reverse position for any let-
Solution: The series is 12 + 1, 22 + 1, 32 + 1 and so on. The ter is always constant and equal to 27. For example, the
next number is 72 + 1 = 50. sum of both positions of H is (8 + 19 = 27), and for W is
Therefore, (a) is the correct answer. (23 + 4 = 27).

N 2 + N Series and N 2 − N Series Table 5.1 English Alphabet—Position Left to Right


Example 13: 0, 2, 6, 12, 20, 30, …, 56
A B C D E F G H I J K L M
(a) 36 (b) 40
(c) 42 (d) None of the above 1 2 3 4 5 6 7 8 9 10 11 12 13
N O P Q R S T U V W X Y Z
Solution: The series is 02 + 0, 12 + 1, 22 + 2, 32 + 3 and so
on. The missing number is 62 + 6 = 42. 14 15 16 17 18 19 20 21 22 23 24 25 26

M05_MADAN 04_65901_C05.indd 5 23/12/22 7:23 PM


5.6 Chapter 5

Table 5.2 English Alphabet—Position Right to Left Example 21: A, E, I, O, …


(a) Q (b) R (c) U (d) S
Z Y X W V U T S R Q P O N
1 2 3 4 5 6 7 8 9 10 11 12 13 Solution: These are all vowels.
M L K J I H G F E D C B A Therefore, (c) is the correct answer.
14 15 16 17 18 19 20 21 22 23 24 25 26 Example 22: A, D, I, P, …
(a) U (b) V (c) X (d) Y
Table 5.3 Series of Opposite English Alphabet
Solution: According to the alphabet series, the positions
A B C D E F G H I J K L M are 1, 4, 9, 16 and so on, which are (1)2, (2)2, (3)3, (4)2 and
so on. The next number is 52 = 25 and the corresponding
Z Y X W V U T S R Q P O N letter is Y.
One-Letter Series Therefore, (d) is the correct answer.

Example 16: A, C, E, G, …, K Example 23: D, F, H, I, J, L, …


(a) I (b) H (c) J (d) M (a) K (b) O (c) M (d) P
Solution: The series is A + 2 = C; C + 2 = E; E + 2 = G;
G + 2 = I; I + 2 = K. The missing letter is I. Solution: If the numbers are assigned, the series
becomes 4, 6, 8, 9, 10, 12 and so on, that is, composite
Therefore, (a) is the correct answer. number series. The next composite number is 14 and the
Example 17: A, B, D, G, …, P corresponding letter is N.
(a) K (b) L (c) M (d) N Therefore, (c) is the correct answer.

Solution: The series is +1, +2, +3, +4, +5 and so on. Example 24: A, Z, B, Y, C, X, D, …
A + 1 = B; B + 2 = D; D + 3 = G; G + 4 = K; K + 5 = P. (a) U (b) V (c) W (d) X
The missing letter is K.
Therefore, (a) is the correct answer. Solution: The sequence consists of two series, where
one is an ascending (A, B, C, D, etc.) and the other is a
Example 18: B, E, H, K, N, … descending (Z, Y, X, W, etc.) series.
(a) P (b) O (c) Q (d) R Therefore, (c) is the correct answer.
Solution: The series is +3. The missing letter is N + 3 = Q. Combined Two-Letter Series
Therefore, (c) is the correct answer. The first letters of the series follow one logic and the sec-
Alternative solution: Skip two letters to get the next ond letters follow another logic, and then they pair with
letter, that is, skip O and P after N to get Q. The missing each other.
letter is Q. Example 25: AM, BN, CO, DP, …, FR
Example 19: B, D, G, I, L, N, … (a) EQ (b) FT (c) GR (d) ER
(a) N (b) O (c) P (d) Q Solution: The first letters are A, B, C, D, E and F, and the
Solution: This series is formed by moving + 2 and +3 for second letters are M, N, O, P, Q and R.
each pair of letters starting from left. Thus, the missing Therefore, (a) is the correct answer.
letter is N + 3 = Q.
Example 26: AB, DE, GH, …, MN
Therefore, (d) is the correct answer.
(a) HI (b) JK
Alternative solution: Skip one and two letters (c) KL (d) KM
alternately to get the next l­ etter, that is, skip two letters O
and P after N to get Q. Solution: After every pair, one letter is skipped.
Therefore, (b) is the correct answer.
Example 20: B, C, E, G, K, …
(a) M (b) N (c) O (d) P Example 27: AA, CE, EI, GO, …
(a) IU (b) IQ (c) IR (d) IT
Solution: B = 2, …, K = 11 according to the alphabet
series (Table 5.1). Thus, the alphabet series have been Solution: The first letters of all pairs given in the question
constructed according to the prime numbers. The next follow a sequence of A + 2 = C, C + 2 = E and so on. The
prime number is 13 and the corresponding letter is M. second letters are vowels.
Therefore, (a) is the correct answer. Therefore, (a) is the correct answer.

M05_MADAN 04_65901_C05.indd 6 23/12/22 7:23 PM


Mathematical Reasoning and Aptitude 5.7

Three-Letter Series In these types of questions, the letters of the alpha-


This sequence consists of three letters in each term. The bet are exclusively used. These letters do not stand for
first letters follow one logic, the second letters follow themselves but are allotted some artificial values based
another logic and the third letters follow some other on some logical pattern or analogy. By applying those
logic (or the same logic in all the three cases). principles or observing the pattern involved, the candi-
dates are required to decode a coded word or encode a
Example 28: ABD, CDF, …, GHJ, IJL word. This can be further classified into the following
(a) EFH (b) IJL (c) HIJ (d) HIK categories.

Solution: The first letter of each triplet follows a sequence Simple Analogical Letter Coding
of A, C, E, G, I and so on. The second letter of each triplet These are also called arbitrary codes. There are two defi-
follows a sequence of B, D, F, H, J and so on, and the third nite principles or patterns involved. Codes are based on
letter forms a sequence of D, F, H, J, L and so on. the analogy of one example from which different codes
Therefore, (a) is the correct answer. are formed.

Example 29: CKZ, DLY, … , FNW, GOV Example 32: If ‘BELONGINGS’ is coded as ‘TABLESTESF’,
then how will you code ‘LINEN’?
(a) EMX (b) ENY
(c) ENX (d) None of the above (a) BTEAE (b) BTAEA
(c) BATEA (d) None of the above
Solution: The first letters form a series of C, D, E, F, G
and so on. The second letters form a series of K, L, M, N, Solution
O and so on. The third letters form a series of Z, Y, X, W, The coding is done as follows:
V and so on.
Therefore, (a) is the correct answer. Letters B E L O N G I N G S

Example 30: NAB, OEC, PIE, QOG, … ↓ ↓ ↓ ↓ ↓ ↓ ↓ ↓ ↓ ↓ ↓


(a) QPH (b) QUH (c) QUI (d) RUK Codes T A B L E S T E S F
Solution: The first letters form a series of N, O, P, Q, R
and so on. The second letters form a vowel series, and the Accordingly, the code for LINEN is BTEAE.
third letters form prime number series according to their Thus, the answer is (a).
number position.
Therefore, (d) is the correct answer. Letter Coding on Specific Pattern
First, the candidates are required to observe the specific
Example 31: ABC, CBA, DEF, …, GHI, IHG pattern involved and then proceed with encoding or
(a) JKL (b) FED (c) DFE (d) IJK decoding as the case may be.

Solution: The second term is the reverse order of the first Example 33: If ‘POSTED’ is coded as ‘DETSOP’, then how
term. In addition to the above-stated types, a number of will you code ‘SPEED’?
other types can also be identified. (a) DEEPS (b) DEESP
Therefore, (b) is the correct answer. (c) DESEP (d) SPEDE
Solution: By careful observation, we can say that the
Coding and Decoding letters have been written in the reverse order. Hence,
The codes are based on various principles or patterns such SPEED will be written DEEPS; therefore, (a) is the correct
that the message can be easily deciphered at the other end. answer.
They have become an almost regular feature of NET Paper Example 34: If ‘GREET’ is coded as ‘FQDDS’, then ‘CDDO’
I Exam to judge the candidates’ intelligence and mental stands for
abilities. They are required to encode and decode words
and sentences after observing the pattern and principles (a) EDDP (b) DEEP (c) PEED (d) EPED
involved. These questions can be broadly classified into Solution: Here, it is −1 for each letter of the word
alphabetical coding, numerical coding and mixed coding. ‘GREET’.
Alphabetical Coding
Word G R E E T
Remembering the positions of different letters is nec-
essary to solve any question on alphabetical series. The −1 −1 −1 −1 −1
English language contains 26 alphabets and their varied
positions are discussed in Tables 5.1−5.3. Code F Q D D S

M05_MADAN 04_65901_C05.indd 7 23/12/22 7:23 PM


5.8 Chapter 5

CDDO itself is a code. To find out what it stands for, we have There is no set of principles or patterns involved. The can-
to add +1 to the alphabets to get back the original word. didates are required to study the given examples before
getting started with the exercise.
Code C D D O
Example 38: If ‘SELDOM’ is coded as ‘1 2 4 3 6 5’, then
+1 +1 +1 +1 how will you code ‘MODE’?
Word D E E P (a) 3 6 2 1 (b) 6 2 3 1 (c) 5 6 3 2 (d) 6 2 1 3

(b) is the correct answer.


Solution

Example 35: If ‘PAT’ is coded as ‘QRBCUV’, then how will Word S E L D O M


you code ‘GRACE’?
↓ ↓ ↓ ↓ ↓ ↓
(a) HISTBCDEFG (b) HISTBCDEGF
(c) HISBTCDEFG (d) HISTBCEDFG Code 1 2 4 3 6 5
Solution: The pattern of coding is such that each letter On the basis of the analogical relationship established
has been allotted the value of two letters following the
between the letters and the numbers, we can code ‘MODE’
sequence, that is, A = BC, B = CD, C = DE, etc.
as 5632.
Hence, the word GRACE will be coded as ‘HISTBCDEFG’.
Therefore, option (a) is the correct answer. Therefore, (c) is the correct answer.

Example 36: If ‘EGHJKMKM’ is the code for ‘FILL’, then Coding with Specific Pattern
how will you decode ‘EGDFDFKM’? This is a pattern of coding that exhibits the natural cor-
(a) LEEF (b) FEEL (c) DEAL (d) REEL relation of Arabic numbers with alphabetical letters. For
instance, alphabets A to Z are assigned the numeric codes
Solution: Refer to alphabet series in Table 5.1. Starting from 1 to 26, where each letter gets the assignment in the
from the L.H.S. pair; E is the antecedent of F and G is the pattern as follows: A = 1, B = 2, C = 3, etc.
precedent of F. We get EG as code for F, HJ as code for I The sequence is classified as follows:
and so on.
1. Forward sequence: For example, A = 1, B = 2, …, Y =
Similarly, EGDFDFKM is the code for FEEL. F is coded
25, Z = 26
as EG with its precedent and antecedent letters. Similarly,
‘I’ is coded as HJ and L as KM. Hence, EGDFDFKM stands Example 39: If ‘PACE’ is code as 16−1−3−5, then how
for FEEL. will you code ‘RACE’?
Therefore, option (b) is the correct answer. (a) 18−1−3−5 (b) 16−1−3−5
(c) 16−3−5−1 (d) None of the above
Coding with Numerical Digits
By referring to Table 5.1, we can say that (a) is the
Here, the numerical values (not necessarily according to
correct answer.
rank of letters in the alphabet series) can be assigned to
letters. The values are allotted based on some specific pat- 2. Backward sequence: For example, Z = 1, Y = 2, …, B
tern that has to be discerned by the candidate in order to = 25, A = 26
solve the problem.
Example 40: If ‘GREAT’ is coded as 20–9–22–26−7, then
Example 37: If ‘TRAIN’ is coded as 23456, then how will how will you code ‘GATE’?
you code ‘RAIN’? (a) 20−26−7−22 (b) 20−26−6−22
(a) 3456 (b) 3546 (c) 2345 (d) 2456 (c) 26−20−7−22 (d) 26−20−22−7
Solution: By referring to Table 5.2, (a) is the correct answer.
3. Random sequence: The pattern can be established in
Word T R A I N alternative ways, but in every case, a set pattern has to
be discovered by careful examination of the example
Code 2 3 4 5 6 given in the question.
Example: Sometimes, A = 3, B = 4, Z = 28 might be
These values have been assigned arbitrarily. The question given, sometimes it might be given in the form such
can be solved on the basis of the relationship established. as A = 5, B = 7, C = 9 and so on.
For RAIN, the code is 3456, so (a) is the correct answer.
Example 41: If ‘FRANCE’ is coded as 10−22−5−18−7−9,
Analogical Coding with Numerical Digits then how will you code ‘INDIA’?
Here, the letters are assigned numerical values on the (a) 13−18−8−13−5
basis of the analogy of the example given in the question. (b) 13−17−6−12−4

M05_MADAN 04_65901_C05.indd 8 23/12/22 7:23 PM


Mathematical Reasoning and Aptitude 5.9

(c) 12−17−7−12−5 Coded Statements


(d) None of the above 1. In a certain code language, ‘do re me’ means ‘he is
Solution: The pattern of assignment can be read as given late’, ‘fa me la’ means ‘she is early’ and ‘so ti do’ means
in the following table: ‘he leaves soon’. Which word in that language means
‘late’?
A B C D E F G H I J (a) la (b) do
(c) me (d) re
5 6 7 8 9 10 11 12 13 14

Explanation
K L M N O P Q R S T do re me—he is late (i)
15 16 17 18 19 20 21 22 23 24 fa me la—she is early (ii)
so ti do—he leaves soon (iii)
U V W X Y Z Comparing (i) and (ii), we can say that ‘me’ stands
25 26 27 28 29 30 for ‘is’.
Comparing (i) and (iii), we can say that ‘do’ stands
Therefore, (a) is the correct answer. for ‘he’.
Substituting them in sentence (i), we can say that ‘re’
stands for ‘late”.
Mixed Coding (Letters + Digits) The following arrangement also helps in the
Mixed coding takes the pattern of coding with both the solution:
letters of the alphabet and the numerical assignment. The
candidates are required to study the analogy given in the he
do re me is late
question.
Example 42: If ‘A3T15R’ stands for ‘ACTOR’ and ‘D1T5’ me
fa la she is early
stands for ‘DATE’, then how will you code ‘ROTATE’?
(a) R15T1T5 (b) R16T1T5
(c) R15T1T6 (d) L15C1T7 so ti do he leaves soon

Explanation
Hence, the code for ‘late’ is ‘re’.
A 3 T 15 R (d) is the correct answer.
A C T O R 2. In a certain code, ‘jo ka ra’ means ‘go for walk’, ‘ma fo
ka’ means ‘do not walk’ and ‘sa ta jo’ means ‘good for
you’. What is the code for ‘go’?
D 1 T 5 (a) ra (b) ka
D A T E (c) ta (d) jo
Explanation
Similarly, (in inverse manner)
jo ka ra go for walk
R O T A T E
R 15 T 1 T 5 ma fo ka do not walk

Thus, (a) is the correct answer. sa ta jo good for you

Example 43: If ‘M I S S I O N’ is coded as ‘ ! ? ? ! ϕ $’ and Hence, the code for ‘go’ is ‘ra’.
‘LENS’ is coded as ‘@#$?’, then how will you code ‘LION’?
The correct answer is (a).
(a) @ ! ϕ $ (b) @ ! $ #
(c) ? ? # # (d) None of the above 3. If rain is water, water is road, road is cloud, cloud is
sky, sky is sea and sea is path, where do aeroplanes
Explanation fly?
(a) Road (b) Sea
LION = L + ION (c) Cloud (d) Water
Pick L from LENS and ION from MISSION. By careful Explanation
observation, we can identify that the code for L is @ and The aeroplanes fly in the ‘sky’ and the ‘sky’ is called
the code for ION is !ϕ  $. ‘sea’. Hence, the aeroplanes fly in the ‘sea’.
Hence, option (a) is the answer. Therefore, (b) is the correct answer.

M05_MADAN 04_65901_C05.indd 9 23/12/22 7:23 PM


5.10 Chapter 5

4. In a certain code, ‘247’ means ‘spread red carpet’, Choosing the O dd Pair of Words
‘256’ means ‘dust one carpet’ and ‘236’ means ‘one red In each of the following questions, four pairs of words are
carpet’. Which digit in that code means ‘dust’? given, out of which the words in four pairs bear a certain
(a) 2  (b) 3  (c) 5  (d) 6 common relationship. Choose the pair in which the words
Explanation are differently related.
In the first statement, the common code digit is ‘2’. In
the second statement, the common word is ‘carpet’. Example 46: Choose the pair in which the words are
Thus, ‘2’ means ‘carpet’. In the second and third state- differently related.
ments, the common code digit is ‘6’ and the common (a) Man : Crowd (b) Cow : Herd
word is ‘one’. Thus, ‘6’ means ‘one’. Therefore, in the (c) Sheep : Flock (d) Fish : Shoal
second statement, ‘5’ means ‘dust’.
Thus, the correct answer is (c). Explanation
5. In a certain code language, ‘Siberia is a cold place’ is writ- In all other pairs, the second word is a collective group of
ten as ‘a cold is place Siberia’. In the same code, ‘Water the first.
freezes to ice here’ is written as ‘freezes here ice to water’. Thus, the correct answer is (a).
How will ‘covers ten percent of earth’ be written?
(a) covers earth ten percent of Example 47: Choose the pair in which the words are
(b) covers earth of percent ten differently related.
(c) Earth covers ten percent of (a) Joule : Energy (b) Ampere : Current
(d) None of the above (c) Angle : Degree (d) Pascal : Pressure
Explanation Explanation
The first alphabet of the words is to be picked and In all other pairs, the first is a unit to measure the
then written as they appear in the alphabetic series. second.
The correct answer is (b). Thus, the correct answer is (c).

Classification Choosing the O dd N umeral


In each of the following questions, four numbers are
The questions based on classification are based on similarity given. Out of these, three are alike in a certain way
or dissimilarity between a number of items or objects. Some except one.
objects are grouped together on the basis of some common
characteristics. The candidate has to identify that charac- Example 48: Choose the number that is different from
teristic and separate out the object that does not belong to others in the group.
the group. This test is also known as ‘Odd Man Out’. (a) 139 (b) 177 (c) 144 (d) 183
Choosing the O dd Word Explanation
In these types of problems, some words belong to the real Number 144 is the only perfect square number in the
word. They have certain common features except for the group.
odd one.
Thus, the correct answer is (c).
Example 44: Choose the word that is least like the other Example 49: Choose the number that is different from
words in a group? others in the group.
(a) Calendar (b) Date (a) 127 (b) 345 (c) 361 (d) 514
(c) Day (d) Month
Explanation
Explanation
All other numbers except 361 are two more than the cube
All other words are parts of a ­calendar. of a certain number.
Thus, the correct answer is (a). Thus, the correct answer is (c).
Example 45: Choose the word that is least like the other Choosing the O dd N umeral Pair or Group
words in a group?
Choose the odd numeral pair or group in each of the fol-
(a) Peacock (b) Vulture lowing questions.
(c) Sparrow (d) Swan
Example 50: Choose the number pair or group that is
Explanation different from others.
Swan is the only water bird in the group. (a) 15 : 46 (b) 12 : 37
Thus, the correct answer is (d). (c) 9 : 28 (d) 8 : 33

M05_MADAN 04_65901_C05.indd 10 23/12/22 7:23 PM


Mathematical Reasoning and Aptitude 5.11

Explanation (a) Laces (b) Cobbler


In all other pairs, second number = (Ist number × 3) + 1 (c) Leather (d) Shoes
Thus, the correct answer is (d). Explanation
Example 51: Choose the number pair or group that is The first is made by the other.
different from others. Thus, the correct answer is (c).
(a) 3, 5 (b) 7, 2 (c) 6, 2 (d) 1, 7
Example 55: As ‘delicious’ is related to taste, ‘melodious’
Explanation is related to
In all other pairs, the sum of two numbers is 8. (a) Voice (b) Speak
Thus, the correct answer is (b). (c) Music (d) Highness

Choosing the O dd L etter Group Explanation


In each of the following questions, some groups of l­ etters Delicious represents good taste. Similarly, melodious
are given, all of them, except one, share a similarity. describes pleasant voice.
Thus, the correct answer is (a).
Example 52: Choose or find the odd letter group.
(a) BCD (b) NPR (c) KLM (d) RQP Completion of Analogous Pair
Explanation Example 56: Giant : Dwarf :: Genius : ?
All other groups contain three consecutive letters of the (a) Wicked (b) Gentle
alphabet series. (c) Idiot (d) Cunning
Thus, the correct answer is (b).
Explanation
Example 53: Choose the group of letters that is different As dwarf is the antonym of giant, idiot is the antonym of
from the others. genius.
(a) KLM (b) IJK (c) PQR (d) RST Thus, the correct answer is (c).
Explanation
Example 57: Fruit : Banana :: Mammal : ?
No other group contains a vowel.
(a) Cow (b) Snake
Thus, the correct answer is (b). (c) Fish (d) Sparrow

Analogy Explanation
As banana is a type of fruit, cow is a type of mammal.
The close meaning of analogy is correspondence. In the
questions based on analogy, a particular relationship is Thus, the correct answer is (a).
given and another similar relationship has to be identified
from the alternatives provided. Therefore, analogy tests Selecting the Right A nalogous Pair
are meant to test candidates for their overall knowledge, In these types of questions, a pair of word is given that is
power of reasoning and the ability to think concisely and followed by four pairs of words as alternatives. The can-
accurately. Questions on analogy test the ability of a can- didate is required to choose the pair in which the words
didate to understand the relationship between two given bear the same relationship to each other as in the case of
objects and apply the same relationship to find what was the words of the given pair.
asked in the question. Some c­ ommon relationships­are
given below, which will help you detect most analogies Example 58: Lamp : Darkness
better. (a) Fatigue : Exercise (b) Water : Thirst
Different types of analogy are given below.
(c) Medicine : Illness (d) Study : Classroom
Direct/Simple A nalogy Explanation
In these types of questions, the first two words have a Just as a lamp eliminates darkness, water quenches
definite relationship. According to the relationship, we thirst.
have to choose one word out of the given four alternatives
that have the same relationship with the third word as Thus, the correct answer is (b).
between the first two. Example 59: Weight : Kilograms
Example 54: Apparel is related to cloth in the same way (a) Axe : Grind (b) Ammeter : Current
as footwear is related to (c) Power : Ampere (d) Energy : Joule

M05_MADAN 04_65901_C05.indd 11 23/12/22 7:23 PM


5.12 Chapter 5

Explanation Solution: Clearly, the first, second, third and fourth


As weight can be measured in kilogram, joule is the unit letters of the first group are moved 11, 12, 13 and 14 steps
of measuring energy. forward, respectively, to obtain the corresponding letters
of the second group.
Thus, the correct answer is (d).

Choosing a Similar W ord A B C D


In this type of questions, a group of three or four words is +11 +12 +13 +14
given, followed by four other words as alternatives. The
L N P R
candidate is required to choose the alternative, which is
similar to the given words.
Similarly,
Example 60: Iron : Copper : Zinc
(a) Ceramic (b) Carbon (c) Nickel (d) Mercury F G H I
+11 +12 +13 +14
Solution: All are solid metals.
Q S U W
Therefore, (c) is the correct answer.
Example 61: Potato : Carrot : Radish Thus, the correct answer is (b).
(a) Tomato (b) Spinach
(c) Sesame (d) Groundnut
Number Based
Under this category, the following types of questions can
Solution: All of these crops/vegetables grow be asked:
underground.
1. Choosing a number related to a given number in the
Therefore, (d) is the correct answer. same manner as the two numbers of another given
pair are related to each other
Alphabet A nalogy 2. Choosing a similarly related pair as the given number
In this type of question, two groups of letters are related pair on the basis of the relationship between the num-
to each other in some way as given. The candidate is bers in each pair
required to find out this relationship and then choose 3. Choosing a number similar to a group of numbers
either a letter-group that is related in the same way to a on the basis of certain common properties that they
third group provided in the questions or a pair consisting possess
of similarly related letter-groups. 4. Choosing a number set similar to a given number set

Example 62: BEGK is related to ADFJ in the same way as Example 64: 9 : 14 :: 26 : ?
PSVM is related to …? (a) 2 (b) 13 (c) 15 (d) 31
(a) LOQT (b) ROUX (c) OTUZ (d) ORUL
Solution: The relationship is x : (x + 5).
Solution: Clearly, each letter of the first group in a pair Therefore, (d) is the correct answer.
is moved one step backward to obtain the corresponding
letter of the second group. Example 65: 8 : 28 :: 27 : ?
(a) 55 (b) 63 (c) 64 (d) 85
B E G K
Solution: The relationship is x : (3x + 4).
−1 −1 −1 −1
Therefore, (d) is the correct answer.
A D F J
Example 66: 42 : 56 :: 72 : ?
Similarly, (a) 81 (b) 90 (c) 96 (d) 100

P S V M Solution: The ratio between 42 and 56 is 3:4.


Similarly, 72 : 96 depicts the ratio 3:4.
−1 −1 −1 −1 Therefore, (c) is the correct answer.
Q R U L

Therefore, (d) is the correct answer. Blood Relations


The questions that are asked in this section depend on
Example 63: ABCD : LNPR :: FGHI : ? relation. The candidate should have a sound knowledge of
(a) SUPW (b) QSUW (c) QSVW (d) RSUW blood relations in order to solve the questions.

M05_MADAN 04_65901_C05.indd 12 23/12/22 7:23 PM


Mathematical Reasoning and Aptitude 5.13

To remember easily, the relations may be divided onto Suppose M is male and N is female. Some authors use
two sides as given in Table 5.4. the signs of + and − for indicating male and female. Cousin
is a common gender; it means that this relationship can be
Table 5.4  lood Relations of Paternal and
B used for both male and female.
Maternal Sides
Relations of paternal side Condition Sign

Father’s father Grandfather M is male +M

Father’s mother Grandmother N is female −N

Father’s brother Uncle M and N are married to each other M=N

Grandfather’s son Father or uncle P and Q are siblings P Q

Grandfather’s only son Father A is the child of B A

Father’s sister Aunt


Children of uncle Cousins B

Wife of uncle Aunt A has two children B and C A

Children of aunt Cousins


B C
Husband of aunt Uncle
Sister’s husband Brother-in-law Approach to Draw the F amily
Wife’s brother Brother-in-law R elations Diagram
Brother’s son Nephew To draw a family tree:
Brother’s wife Sister-in-law 1. First of all, identify the males and the females, and
then according to generation, put each member at the
Brother’s daughter Niece appropriate position in the tree.
Grandson’s or ­granddaughter’s Great 2. Draw the diagram with relationships among family
daughter granddaughter members using notations.
3. Once the diagram is filled, the candidate can answer
Mother’s or father’s son/daughter Brother/sister the given questions.
Son’s wife Daughter-in-law Example 67: X and Y are brothers. C and D are sisters.
Relations of maternal side X’s son is D’s brother. How is Y related to C?
(a) Uncle (b) Grandfather
Mother’s father Maternal (c) Father (d) None of the above
grandfather
Explanation
Mother’s mother Maternal
grandmother +X +Y

Mother’s brother Maternal uncle


Mother’s sister Aunt
Children of maternal uncle Cousins
Wife of maternal uncle Maternal aunt +Son −D −C
Others As per the statement and the family tree, X’s son, D and C
Children of same parents Siblings are siblings. Y is the uncle of C.
Thus, (a) is the correct answer.
Common term for husband and wife Spouse
Example 68: ‘A’ is the father of ‘C’, and ‘D’ is the son of ‘B’.
Developing a F amily Relationship Tree ‘E’ is the brother of ‘A’. If ‘C’ is the sister of ‘D’, then how is
‘B’ related to ‘E’?  [NET June 2007]
To develop a blood relation tree, some standard symbols
may be used to express the relationships among the fam- (a) Daughter (b) Husband
ily members. (c) Sister-in-law (d) Brother-in-law

M05_MADAN 04_65901_C05.indd 13 23/12/22 7:23 PM


5.14 Chapter 5

Explanation
Direction Sense
+E A=B There are four main directions: east, west, north and
south, shown as follows:
N

−C +D

Thus, B is the sister-in-law of E. (c) is the correct answer. W E

Example 69: E is the son of A; D is the son of B; E is


married to C; C is the daughter of B. How is D related to E?
 [NET June 2010]
S
(a) Brother (b) Uncle
(c) Father-in-law (d) Brother-in-law There are four cardinal directions, such as north-east
(NE), north-west (NW), south-east (SE) and south-west
Explanation (SW), shown as follows:
Family tree: NW N NE
A B
W E

SW S SE

Few Important Points


1. At the time of sunrise, if a person stands facing the
+E = −C +D east, then his/her shadow will be towards the west.
2. At the time of sunset, the shadow of an object is always
Hence, D is the brother-in-law of E. in the east.
Therefore, (d) is the correct answer. 3. If a person stands facing the north, at the time of sun-
rise, his/her shadow will be towards his/her left; and
Example 70: If B is the only child of C’s grandfather’s at the time of sunset, it will be towards his/her right.
only daughter, then how is C’s father related to B? 4. At 12 p.m., the rays of the sun are vertically down-
ward. Hence, there will be no shadow.
(a) Maternal uncle
(b) Father Example 71: Prakash walked 30 m westward, took a
(c) Paternal uncle left turn and walked 20 m. He again took a left turn and
walked 30 m. Then, he took a right turn and stopped. He
(d) Cannot be determined
is now facing
Explanation (a) South (b) North (c) East (d) West
30 m
C’s grandfather N

20 m W E
Only daughter Brother (C’s father)

30 m S

B C

As B is the only child, C cannot be the sister of B. On the Therefore, (a) is the correct answer.
other hand, B’s mother is the only daughter of her par-
ents, so she can have a brother. Hence, C’s father is the Example 72:
maternal uncle of B. If south-east direction becomes north, north-east direction
Thus, (a) is the correct answer. becomes west and so on, then what will west become?

M05_MADAN 04_65901_C05.indd 14 23/12/22 7:23 PM


Mathematical Reasoning and Aptitude 5.15

(a) North-west (b) North-east Here, Q is between G and S—this is 100%; whether right
(c) South-east (d) South-west or left, it requires more information. Therefore, the posi-
tion of Q cannot be determined with the available infor-
N
NW NE S W mation. Hence, (c) is the right option.
Therefore, (c) is the correct answer.
W E SE Example 75: Among M, N, T, R and D, each has a different
height. T is taller than D, but shorter than M. R is taller
than N, but shorter than D. Who among them is the tallest?
SW S SE E N
(a) R (b) M
Therefore, (c) is the correct answer. (c) D (d) None of the above
Example 73: Atul walks 20 m towards the south; turning Solution: M > T > D > R > N
left, he walks 30 m. Then, turning right, he walks 10 m Hence, M is the tallest.
and then, turning right, he walks 40 m. Then, turning
right, he walks 30 m and stops. In which direction is he Therefore, (b) is the correct answer.
standing with respect to his starting point? Example 76: A, B, C, D, E and F are sitting in a row facing
(a) East (b) West (c) North (d) South north. A is the neighbour of B and D. E is the neighbour of
C and F, and D is the neighbour of C. How many members
Explanation are there between A and E?
Starting point (a) Two (b) One (c) Three (d) Four
20 m Explanation
30 m The sitting arrangement is B A D C E F. Between A and E,
30 m there are two members D and C.
10 m
Thus, (a) is the correct answer.

40 m Mathematical Aptitude
Therefore, (c) is the correct answer.
Fractions
Basically, a fraction describes how a part of a group
Seating Arrangement relates to the whole group. Fractions represent complete
groups that have been fractured or broken apart in some
Example 74: Among six members P, Q, R, G, S and M way. They help us understand how those pieces fit into the
sitting along a circle facing the centre: original group.
  I. R is between G and P. When we look at a fraction, we look at the number that
represents the pieces (the fractured section) on the top of
II. M is between P and S.
the division line. Such a number on the top is called the
What is the position of Q? numerator. The number below the line represents how
(a) To the immediate left of G many total parts are in the group. This number is called
(b) To the immediate right of S the denominator. To easily tell these two parts of the frac-
(c) Cannot be determined tion apart, just remember that denominator and down
(d) None of the above both start with the letter d.
Part Numerator
Explanation Fraction = =
Whole Denominator
G P If there are 5 apples in a carton of 12 apples, then the frac-
5
tion of apples for the whole would be represented as .
R Q R M 12
There can be various types of fractions:
OR 1. Common fraction: A common fraction is a num-
ber written with a numerator and a denominator, in
5
P S G S which both are natural numbers, for example,
17 12
M Q and .
12

M05_MADAN 04_65901_C05.indd 15 23/12/22 7:23 PM


5.16 Chapter 5

2. Proper fraction: A proper fraction that is less than know English. How many students know both English
1 3 1
1 is known as proper fraction, such as and . and Hindi if students who know Hindi are th of total
2 4 8
A proper fraction has the same name as ratio. students in the class?
3. Mixed number fraction: It is basically a whole num-
ber plus a proper fraction. For example: 1 100 10 1
(a) (b) (c) (d)
24 24 12 4
1 1
2 = 2+ Explanation
3 3
3
4. Improper fractions: If we divide each whole unit th do not know English or Hindi.
4
into thirds, say, and keep counting them, then we will 1
3 4 5 Therefore, th know English or Hindi or both.
come to , , and so on. That is, we will come to 4
3 3 3 1 1
fractions that are equal to or greater than 1. We call th know English and th know Hindi.
those improper fractions. 6 8
1 1 1 1
How to convert an improper fraction to a mixed number Now, − − = −
or a whole number? 4 6 8 24
Thus, the correct answer is (a).
9 1
For example: =4
2 2 3
Example 78: of a group of children are girls. If there
5
Divide the numerator by the denominator. Write the are 24 girls, then how many children are there in the
quotient (4) and write the remainder (1) as the numera- group?
tor of the fraction, do not change the denominator.
Fractions can be added, subtracted, multiplied or (a) 32 (b) 36 (c) 40 (d) 42
divided also.
A fraction in which there is no common factor, except Solution
1, in its numerator and denominator is called a fraction in ?
the simplest or lowest form.
Fractions with same denominators are called like frac-
tions and if the denominators are different, then they are
called unlike fractions. 24 girls
• Fractions can be compared by converting them into 3 units = 24
like fractions and then arranging them in ascending
1 unit = 24 ÷ 3 = 8
or descending order.
• Addition (or subtraction) of like fractions is mostly 5 units = 5 × 8 = 40
done by adding (or subtracting) their numerators. There were 40 children in the group.
Addition (or subtraction) of unlike fractions can be
done by converting them into like fractions. Thus, (c) is the correct answer.
• Fractions with denominators 10,100, etc., can be Example 79: Shyam had 120 teddy bears in his retail
written in a form, using a decimal point called deci- 2
mal numbers or decimals. store. He sold of them at `12 each. How much did he
• Decimal numbers can be compared by using the idea 3
of place value; then they can be arranged in ascend- receive?
ing or descending order. (a) 800 (b) 840 (c) 960 (d) 920
• Decimals can be added (or subtracted) by writing
them with equal number of decimal places. Solution
• Many of our daily life problems can be solved by Step 1: The number of teddy bears sold =
converting different units of measurements, such as
money, length and weight, in the decimal form and 2 2 × 120
× 120 = = 80
then adding (or subtracting) them. 3 3
3 Step 2: The money received =
Example 77: In a class, th of the students do not
4 1 80 × 12 = 960
know either English or Hindi. But th of the students
6 Thus, (c) is the correct answer.

M05_MADAN 04_65901_C05.indd 16 23/12/22 7:23 PM


Mathematical Reasoning and Aptitude 5.17

Example 80: A fraction is divided by the reciprocal of 9 11


itself. It is then multiplied by the original fraction. What is Now let’s compare with .
13 17
25
the fraction if the answer obtained is 11 ? ad = 9 × 17 = 153
64 bc = 13 × 11 = 143
9 27 9 2 153 > 143
(a) (b) (c) (d)
4 8 8 3 9
∴ is largest.
Explanation 13
9 11 5
Let the fraction be A. So, the descending order is > >
13 17 8
According to the question 9 11 5
Answer: ; ;
A 25 13 17 8
× A = 11
1 64 1
A
Exampl e 82: Eiffel Tower’s th part is painted bronze.
1 5
729 9 × 9 × 9 th of the remaining part is painted maroon. Rest of the
A3 = = 4
64 4×4×4 Eiffel Tower has golden colour. If the height of this golden-
9 coloured part is 450 m, then what is Eiffel Tower’s height?
A=
4 (a) 2,250 m (b) 1,250 m
Thus, (a) is the correct answer. (c) 1,000 m (d) 750 m
Example 81: Which of the following is in descending Explanation
order? Let’s assume that length of Eiffel Tower is 100m.
5 9 11 5 11 9 1
(a) ; ; (b) ; ; th of 100 = 20 m is bronze.
8 13 17 8 17 13 5

9 11 5 11 9 5 1 1
(c) ; ; (d) ; ; th of the remaining = th of (100 – 20) = 20 m is maroon.
13 17 8 17 13 8 4 4
Rest = 100 – 20 – 20 = 60 m is golden.
Explanation
In such fractions, you can either divide directly or use the If golden part is 60m, then total height = 100 m
following way. If golden part is 1m, then total height = 100 / 60 m
If golden part is 450m, then total height = 100/60 × 450
Tip:
= 750 m
In two fractions a/b and c/d:
Therefore, the actual height of Eiffel tower is 750 m.
a c
• If ad > cb, then > .; ad < cb, (d) is the right answer.
b d
Time and Distance
a c
• If ad < cb, then < . If speed, time and distance are denoted by S, T and D,
b d
D D
a c respectively, then S = , , D = S × T and T =
• If ad = cb, then = . T S
b d
5
To convert from km/h to m/s, multiply by
18
5 9 5 11 18
First, let’s compare with and with . To convert m/s to km/h, multiply by
8 13 8 17 15
If the ratio of the speeds of A and B are a:b, then the
by cross multiplying ratio of the time taken by them to cover the same distance
5 × 13 = 65 5 × 17 = 85 1 1
is : or b:a.
8 × 9 = 72 11 × 8 = 88 a b
65 < 72 85 < 88 Suppose a man covers a certain distance at x kmph
5 9 5 11 and an equal distance at y kmph. Then, the average speed
∴ < ∴ <
8 13 8 17 2xy
during the whole journey = kmph (also covered
( x + y)
5
Thus, the smallest number is . under the topic of Averages).
8

M05_MADAN 04_65901_C05.indd 17 23/12/22 7:23 PM


5.18 Chapter 5

Example 83: A man covers 60 km in 4 hours. Find the The parts of distances are D/4, D/4 and D/2
speed.
⎛ D⎞ ⎛ D⎞ ⎛ D⎞
Distance 60 ⎜⎝ ⎟⎠ ⎜⎝ ⎟⎠ ⎜⎝ ⎟⎠
Solution: Speed = = = 15 kmph Total time = t 1 + t 2 + t 3 = 4 + 4 + 2
Time 4 20 10 80
Note: Depending on the answer choices, the kmph can D D D
5 = + +
also be converted into m/s by multiplying by . 4 × 20 4 × 10 2 × 80
18 D D D 7D
1 = + + =
Speed (m/s) = 15 × (5/18) = 4 m/s 80 40 160 160
6
D
Example 84: A man covers 20 km in 2½ hours. Find the ∴ Average speed = = 22.85 km/h
7D
distance covered in 9 hours.
160
D 20 km
Solution: Speed = = hours = 8 km/h Therefore, (a) is the correct answer.
T 2½
Distance covered in 9 hours = S × T = 8 × 9 = 72 km Example 88: An office boy increases his speed to 9/5
times of his original speed. By doing this he is able to
Example 85: A car completes a journey in 4 hours, the reach his office 30 minutes before the usual time. How
first half at a speed of 40 kmph and the second at 60 much time is usually taken by him?
kmph. Find the total distance covered. (a) 70.50 minutes (b) 54 minutes
(c) 66.67 minutes (d) 67.50 minutes
Solution: As the total journey is divided into equal parts,
the average speed can be calculated by the formula­2xy/ Solution
(x + y) = 2 × 40 × 60/(40+60) = 48 kmph.
The distance traveled is same in both the cases. Time T is
Distance = S × T = 48 × 4 = 192 km. being considered in minutes.
Example 86: A student walks from his house at a speed D=D
of 3 kmph and reaches the school 10 minutes late. If he
walks at a speed of 4 kmph, then he reaches the school 10 ⎛ 9S ⎞
S × T = ⎜ ⎟ (T − 30)
minutes earlier. What is the distance between his school ⎝ 5⎠
and his house? ⎛ 9ST − 270S ⎞
ST = ⎜ ⎟⎠
⎝ 5
Solution: Let the distance be x km.
5ST = 9ST − 270S
Difference between timings of reaching the school at
20 1 4ST = 270S
different speeds = 10 + 10 = 20 minutes or or hour.
60 3 4T = 270
According to question:
x x 1 270
− = T= = 67.5min
3 4 3 4

4 x – 3x 1 Hence, the correct answer is (d).


=
12 3 Example 89: Pankaj starts from home at a speed of
x = 4 km 30 km/h and reaches his school 20 minutes late. Then the
next day he increases his speed by 15 km/h but still gets
Example 87: Sidhu travels one-fourth of the total journey late by 8 minutes. How far is his school from his home?
at 20 km/h, one-fourth at 10 km/h and rest of his journey (a) 18 km (b) 35 km
at 80 km/h. Find the average speed per hour of Sidhu for
(c) 20 km (d) 27 km
the whole distance.
(a) 22.85 km/h (b) 25.15 km/h Solution: Let the distance be D.
(c) 50 km/h (d) 40 km/h With speed 30 km/h, he is 20 minutes late.
With speed 45 km/h, he is 8 minutes late.
Solution ∴ difference between two timings
D 12 1
∴ S = D/T = 20 − 8 = 12 minutes = hours= hours
60 5
S T
D
T=
Let the total distance be D km. S

M05_MADAN 04_65901_C05.indd 18 23/12/22 7:24 PM


Mathematical Reasoning and Aptitude 5.19

D D 12 360
Thus, − = Thus 100 =
30 45 60  Time
Solving it D = 18 km Time = 3.6 seconds
The correct answer is (a). Therefore, (a) is the correct answer.
Example 90: Mansi covers a distance at a speed of 24 Ratio and Proportion
km/h in 6 minutes. If she wants to cover the same distance
in 8 minutes, what should be her speed on per hour basis? Ratios can also be expressed as fractions. They represent
the basic relationship between two quantities. Proportions
(a) 18 km/h (b) 21 km/h are in comparison to the whole.
(c) 30 km/h (d) 15 km/h In a mixture of 20 l of milk and 30 l of water, the ratio
Solution: Distance is same. of milk and water is 2 : 3. This can be converted to fraction
2
∴ S1 × T1 = S2 × T2 of milk in the solution as 2 : 5 or th.
5
6 8 2 2
∴ 24 × = S2 × As seen, is nothing but × 100 = 40%.
60 60 5 5
(The minutes have been divided by 60 to convert them Example 93: What percentage of 180.50 is 36.1?
into hours) (a) 20 (b) 25
∴ S2 = 18 km/h (c) 20.50 (d) None of the above
Therefore, (a) is the correct answer. Solution: Let’s consider x% of 180.5 = 36.1.
Example 91: A man rows a boat at the speed of 12 km/h 36.1
in still water to some upstream point and back to the x% =
180.5
starting point. The speed of flow of river is 3 km/h. Find
his average speed (km/h) for the total journey. x = (36.1/180.5) × 100 = 20%
(a) 11 ¼ (b) 12 ¾ (c) 10 ½ (d) 9 ½ Example 94: A mixture contains milk and water in the
ratio of 8:3. On adding 3 litres of water, the ratio of milk
Solution: Speed of the man in still water = 12 km/h of water becomes 2:1. Find the quantities of milk and
Speed of the stream = 3 km/h water in the mixture.
Downstream speed = (12 + 3) = 15 km/h
(a) 20 and 9 (b) 9 and 24
Upstream speed = (12 – 3) = 9 km/h
Let the distance travelled be ‘d’ km. Then, average speed = (c) 12 and 15 (d) 24 and 12
Solution:
Total distance
Average speed= Let the original quantities of milk and water be 8a and 3a.
Total time After adding 3 L of water.
d+d 8a (3a + 3) = 2/1
= 8a = 6a + 6
d d
+ 2a = 6; a = 3
15 9 Hence, milk = 8 × 3 = 24 L and water = 3 × 3 + 3 = 12 L
2d This can be verified as present ratio = 24 L : 12 L = 2 : 1
=
9d + 15d Hence (d) is the answer.
15 × 9 Example 95: If 30 men do a piece of work in 27 days, in
2d ×(15× 9) 45 1 what time can 18 men do another piece of workthat is 3
= = km/h = 11 km/h
24d 4 4 times bigger?
Therefore, (a) is the correct answer. (a) 120 days (b) 135 days
(c) 150 days (d) 180 days
Example 92: The two trains of respective lengths 160 and
200 m travel at the speeds of 48 and 52 m/s, per second Solution:
respectively, in opposite direction to each other. What is We work on the principle of less men, more days and more
the total time taken by them to cross each other? work, more days.
(a) 3.6 seconds (b) 4.5 seconds Given that 30 men do 1 piece of work in 27 days
(c) 4.8 seconds (d) 5.4 seconds Then, 1 men can do 1 piece of work in 27 × 30 days
There 18 men can do 3 pieces (times bigger) work in (27
Solution: Relative speed = 48 + 52 = 100 m/s × 30 × 3)/18 days
Total distance covered by both the trains = 160 + 200 = 360 m 27 30 3
Distance the reqd. no. of days = =135 days
Speed= 18 1
Time Hence (b) is the right answer.

M05_MADAN 04_65901_C05.indd 19 23/12/22 7:24 PM


5.20 Chapter 5

Example 96: If 15 men or 24 women or 36 boys do a Solution:


piece of work in 12 days, working 8 hrs a day, how many 1
men must be associated with 12 women and 6 boys to Here 2 : 3 : 4 = 5 : 6 : 8
2
1 1 1
do another piece of work 2 times as great in 30 days Their proportional value = 5×1:6 × :8 × = 5:3:2
4 2 4
working 6 hrs a day? Their Total = 5 + 3 + 2 = 10
(a) 5 (b) 8 (c) 10 (d) 12 5
∴ the value of rupees = of `210 = `105
10
Solution:
We can make use of following reasoning. 3
The value of fifty-paise coins = of `210 = `63
(1) More days : less men. 10
(2) Less working hrs : more men. 2
(3) More work : more men. The value of 25-paise coins = of `210 = `42
10
Therefore, by the rule of three,
Therefore, there are 105 rupees, 126 fifty-paise coins and
168 twenty-five paise coin.
30 days : 12 days ⎤
⎥ Hence, (a) is the correct answer.
6 hrs : 8hrs ⎥ ::15men: the reqd. no. of men
⎥ Example 99: The contents of two vessels containing water
1 and milk are in the ratio 1:2 and 2:5 are mixed in the ratio
1work :2 works ⎥
4 ⎥⎦ 1:4. The resulting Mixture Will have water and milk in
15×12 × 8 × 2.25 the ratio.
∴the reqd. no. of men = = 18
30 × 6 ×1 (a) 23:64 (b) 31:68
(c) 28:75 (d) 31:74
Now, we have, 24 women = 15 men
Solution:
∴12 women =7.5men
Change the ratios into fractions.
And also, 36 boys = 15men
15 5 Water Milk
∴6 boys = = =2.5men
6 2 1 2
∴12 women + 6 boys = 7.5 + 2.5 = 10men Vessel I
3 3
So, 18 − 10 = 8men must be associated. 2 5
Vessel II
Thus, (b) is the correct answer. 7 7

Example 97: A certain sum of money is divided among A, 1 4


From Vessel I, is taken and from Vessel II, is taken.
B and C such that for each rupee A has 65 paise and C has 5 5
40 paise. If C’s share is `8, find the sum of money. Therefore, the ratio of water to milk in the new vessel
(a) `35 (b) `38 (c) 41 (d) 45
⎛ 1 1 2 4⎞ ⎛ 2 1 5 4⎞
= ⎜ × × × ⎟ :⎜ × + × ⎟
Solution: ⎝ 3 5 7 5⎠ ⎝ 3 5 7 5⎠
Here A : B : C = 100 : 65 : 40 = 20 : 13 : 8 ⎛ 1 8 ⎞ ⎛ 2 20 ⎞ 31 74
Their Total = 20 + 13 + 8 = 41 = ⎜ + ⎟ :⎜ + ⎟ = : = 31:74
⎝ 15 35 ⎠ ⎝ 15 35 ⎠ 105 105
8
As of the whole sum = `8 Hence, (d) is the correct answer.
41
Example 100: 465 coins consist of rupee, 50 paise and
8 41
∴ the whole sum of money = ` = `41 25 paise coins. Their values are in the ratio 5 : 3 : 1. Find
8 the number of each coin.
Hence, (c) is the correct answer.
(a) 155, 186, 124 (b) 135, 186, 124
Example 98: How many one rupee, fifty paise coins and (c) 155, 124, 186 (d) 155, 106, 124
twenty five paise coins are there if their numbers are
Solution:
1
proportional to 2 , 3 and 4. Their total worth is `210. The ratio of number of coins
2
100 100 100
(a) 105, 63 and 42 (b) 63, 42 and 105 = 5× :3× :1× = 5:6 :4
(c) 90, 63 and 42 (d) 75, 63 and 42 100 50 25

M05_MADAN 04_65901_C05.indd 20 23/12/22 7:24 PM


Mathematical Reasoning and Aptitude 5.21

465
∴ The number of one-rupee coins = × 5 = 155 If price increases, then consumption will come down as
5+ 6 + 4 we have to keep the total expenditure same.
465 100 × 100 = (100 – x) × (100 + 25)
The number of 50 P coins = × 6 = 186 x = 20%
5+ 6 + 4
Hence, (c) is the correct answer.
465
The number of 25 P coins = × 4 = 124 Alternatively, such problems can be solved by the follow-
5+ 6 + 4 ing methods also.
Hence, (a) is the correct answer. If the price of a commodity increases by r%, then the
reduction in consumption so as not to increase the expen-
Percentage Calculations diture, is
If it is said that India won 50% of the matches it played, ⎛ r ⎞
then neither does it mean that India won 50 matches nor ⎜⎝ ×100⎟ %
100 + r ⎠
does it mean that India won 0.5 matches. It means that
India played 100 matches and won 50 match. If the price of a commodity decreases by r%, then increase
Now assume that India won 40% of matches that it in consumption, so as not to decrease expenditure on this
played in 2015. We have to calculate how many matches item, is
it actually won, and for that, we ought to know the total
matches played, say, for example, 25 matches. Thus, if ⎡ r ⎤
⎢⎣ ×100 ⎥ %
India had played a total of 25 matches, then it would have 100_ r ⎦
40
won 40% of 25 = × 25 = 10 matches. Example 101: Express 12% as a fraction?
100
Percent in the simplest sense means per cent, that is, 2 3
(a) (b)
15 25 25
per hundred. For example, 15% = = 0.15.
100 4 7
1. To convert any per cent as a part or multiple of 1, (c) (d)
25 25
divide it by 100. Examples: 4% = 0.04, 40% = 0.4 and
400% = 4. Solution:
2. Conversely, to write any number in the percentage 12 3
form, multiply it with 100 and append a % ­symbol. 12% = =
Example: 2 = 200%, 0.2 = 20%, 0.02 = 2% and 0.002 100 25
= 0.2%. Hence, (b) is the correct answer.
As seen above, x% is nothing but a fraction with numera- Example 102: If two litres of water is evaporated on boiling
tor x and denominator 100. Identifying certain percent- from 8 litres of sugar solution containing 5% sugar, find
ages in the reduced form of the fractions can be very use- the percentage of sugar in the remaining solution?
ful. For example, whenever we have to calculate 20%,
2
20 1 3 (a) 6 % (b) 8%
we can calculate = . Similarly, the ratio can be 3
100 5 5 (c) 9% (d) 10%
60
expressed as , that is, 60%. Solution:
100
1 1 As sugar does not evaporate, its quantity will remain same
We assume that 1 is equal to 100%, then = 50%,
2 3 in the remaining solution, that is 8-2 = 6 litres of solution.
1 1 1 1 5% of 8 = x% of 6
= 33.33%, = 25%, = 20%, = 16.66%, x= 12.5%,
4 5 6 8 (5 × 8)
1 1 Thus, x =
1 6
= 8.33%, = 6.25%, = 5% and so on.
12 16 20 Thus,(a) = 6 2/3 % is the correct answer.
If we were to convert 3/8 to percentage, simply
­multiply its percentage equivalent of 1/8 by 3. Then, we Short cut formula
get an answer 3 × 12.5% = 37.5%. Suppose a quantity (such as price or salary) increases or
decreases first by x%, and then by y%. What is the net
If the price of a commodity increases by 25%, then by how
increase or decrease in the price?
much percentage, a household should decrease his con-
sumption so that his total expenditure remains the same?  xy 
Net increase = x + y +  . This formula is used
(a) 15% (b) 18 % (c) 20% (d) 30%  100 
Let both consumption and prices are 100 each. where ‘two’ quantities are to be multiplied as per formula
Total expenditure = price × consumption = 100 × 100

M05_MADAN 04_65901_C05.indd 21 23/12/22 7:24 PM


5.22 Chapter 5

or requirement of the question, and one or both of these Solution: In comparison questions, the quantity with
quantities may increase or decrease. which comparison is being done should be taken as
denominator (or base).
Example 103: The price of a commodity increases first
by 20% and then by 10%. What is the net increase in the Let the income of Y = `100
price? The income of X = `100 + 25 = `125
For sake of comparison, we have to make income of X as
Solution: Let original price = 100 equal to `100.
Price after first increase = 100 + 20 = 120 100 × 100
If income of X is 100, then income of Y = = `80
Price after second increase = 120 + (10% of 120) = 132 125
Net increase = 132 − 100 = 32% Thus, the difference = 100 – 80 = `20
Direct formula = 20 + 10 + [(20 × 10)/100] = 32% (a) is the correct answer.
Example 104: A retailer offers two successive discounts Alternatively, The difference can be calculated directly
of 20% and 30%. What is the net decrease in the price? with following formula.
r
Solution: Let’s consider original price = 100 Difference = (Here, r is the percentage
(100 + r) × 100
Price after first discount = 100 − 20 = 80 difference as asked in the question).
Price after second discount = 80 − (30% of 80) = 56
Net discount = Original price − Discount = 100 − 56 = 44 25
= = 20%
Direct formula = (−20) + (−30) + [(–20) × (–30)]/100] (100 + 25) × 100
= −50 + 6 = −44%. If there is mention of ‘less than’ in the question, then it can be
Negative sign shows the decrease. solved in the following manner to calculate the difference.
Example 105: The price of a commodity is first increased r
by 40% and then reduced by 20%. What is the net increase Difference =
(100 – r) × 100
or decrease in the price?
Example 109: In an exam, a student scored 50% of the
Solution: Let the original price = 100 maximum marks and yet failed by 15 marks. If he had
Price after increase = 100 + 40 = 140 scored 10% more than what he scored, then he would
Price after decrease = 140 − (20% of 140) = 112 have just managed to get the pass percentage. What is the
Net increase in price = 112 − 100 = 12% maximum marks for the paper?
Direct formula = 40 − 20 + [40 × (−20)]/100] = 12% Solution: Let maximum passing marks = 100
Example 106: A’s income is 70% of B’s income. B’s Actual marks obtained = 50
income is 50% of C’s income. If C’s income is `1,00,000, Had he scored 50 + 10% of 50, that is, 55 marks, then he
then what is A’s income? would have scored passing marks. In this situation, the dif-
ference between the actual and passing marks is 5.
⎛ 50 ⎞ Actual difference = 15
Solution: B’s income = ⎜ × 1,00,000 = 33.33%
`50,000
⎝ 100 ⎟⎠ 5% of maximum marks = 15
Maximum marks = 15 × 100/5 = 300
⎛ 70 ⎞
A’s income = ⎜ × 50,000 = `35,000 Example 110: In a basket of fruits, 60% are mangoes and
⎝ 100 ⎟⎠
remaining are apples. In that, 25% of the apples are green
and the rest are red. Of the mangoes, 80% are red and
Alternative method the rest of the mangoes are green. What percentage of the
A’s income = 70% of 50% of `1,00,000 = `35,000 green fruits are mangoes?
Example 107: The income of A is 60% of the income of Solution: Let us assume that the total number of fruits is
B. If the income of B is `15,000, then what is the income 100. Here, 60 are mangoes and 40 are apples.
of A?
Green apples = 25% of 40 = 10
(a) `8000 (b) `8500 Green mangoes = 20% of 60 = 12
(c) `9000 (d) `9500 Total number of green fruits = 10 + 12 = 22
Thus, required percentage = 12/22 × 100 = 54.5%
60
Solution: The income of A = = `9000 Example 111: If the milk to water ratio in a mixture is
100 × 15000
2:3, then what is the percentage of milk in the mixture?
Example 108: The income of X is 25% more than that of Solution: There are five parts in total, that is, two parts
income of Y. Calculate by how much percent, the income are of milk and three parts are of water.
of Y is less than that of income of X?
Percentage of milk in the mixture
(a) 18% (b) 20% (c) 25% (d) 28% = 2/5 × 100 = 40%

M05_MADAN 04_65901_C05.indd 22 23/12/22 7:24 PM


Mathematical Reasoning and Aptitude 5.23

Example 112: If two-third of residents in a housing ⎡ Loss × 100 ⎤


society own cars, and furthermore, one-half of car owners Loss% = ⎢ ⎥
own a Swift car, then what percentage of residents own ⎣ C.P. ⎦
Swift cars? 3. Selling price (S.P.)
Solution: The fraction of residents owning a Swift car ⎡ (100 + Profit %) ⎤
S.P. = ⎢ × C.P.⎥
= 1/2 of 2/3 = (1/2) × (2/3) = 1/3 ⎣ 100 ⎦
Converting it into percentage, (1/3) × 100 = 33.33%
4. Selling price (S.P.)
Example 113: Neeru’s expenditure and savings are in the ⎡ (100 − Loss%) ⎤
ratio 3:2. Her income increases by 10%. Her expenditure S.P. = ⎢ × C.P.⎥
⎣ 100 ⎦
increases by 12%. By what percentage does her savings
increase? 5. Cost price (C.P.)
⎡ 100 ⎤
Solution: Let Neeru’s income = 100 C.P. = ⎢ × S.P.⎥
⎣ (100 + Profit %) ⎦
Expenditure = [3/(2+3)] × 100 = 60
6. Cost price (C.P.)
Saving = 100 − 60 = 40
New income after 10% increase = 110 ⎡ 100 ⎤
C.P. = ⎢ × S.P.⎥
New expenditure after 12% increase ⎣ (100 − Loss%) ⎦
= 60 + 12% of 60 = 67.2%
New savings = 110 − 67.2 = 42.8 Example 115: The cost price of the goods for a shopkeeper
was X. He marked them at a 20% higher price than the
Percentage increase in savings
Cost Price. Finally he sold the goods at 30% discount. Did
= [(42.8–40)/40] × 100 = 2.8/40 × 100 = 7% he earn a profit or incur a loss? How much?
Percentage of a per cent = 20% of 30% will be nothing but (a) 5% Profit (b) 5.5% Profit
20 30 6 (c) 10% Profit (d) 16% Profit
× =
100 100 100
Solution:
6/100 is equal to 6%. Let CP = `100
Example 114: What is 20% of 30% of 40%? Marked Price = 20% more than CP
∴ MP = `120
⎛ 20 ⎞ ⎛ 30 ⎞ Discount = 30% on marked price
Solution: The value is= ⎜ × × 40% = 2.4%
⎝ 100 ⎟⎠ ⎜⎝ 100 ⎟⎠ ∴ SP=(100 – 30)% on MP
70
∴SP= ×120=RS.84
Profit and Loss 100
100-84
Whenever an article is sold or purchased, there are some Loss%= ×100=16%
commercial terms involved. Sometimes there is a profit, 100
sometimes a loss and in other cases neither of the two. Hence, the correct answer is (d).
This difference is calculated and then converted to per-
centage to make comparison easier. Example 116: One shopkeeper Keeper was offering two
Let’s have a look at the different formulas: successive discount of 15% and 10% on an item while the
second one is offering flat 25% discount on the same item.
Cost Price (C.P.): The price at which an article is pur-
From which shopkeeper should I buy the item?
chased by the seller.
(a) First Shopkeeper
Selling Price (S.P.): The price at which an article is sold. (b) Second Shopkeeper
Profit or Gain (P): If the difference between S.P. and C.P. (c) Any of them; both are same
is positive, then the amount is called profit or gain. (d) Data Insufficient

Loss: If the difference between C.P. and S.P. is positive, Explanation:


then the amount is called loss.
Tip:
1. Profit percentage (Profit %)
MULTIPLY
⎡ Profit × 100 ⎤ Single equivalent of 2 discounts =ADD −
Profit % = ⎢ ⎥ 100
⎣ C.P. ⎦
2. Loss percentage (Loss %) ADD = 15 +10 = 25

M05_MADAN 04_65901_C05.indd 23 23/12/22 7:24 PM


5.24 Chapter 5

MULTIPLY 15×10 Example 120: Romit sold his old TV and earned a profit
= = 1.5 of 10%. If he could have managed to sell it for `8100
100 100
more, then his profit would have been 37%. Find the price
Single equivalent = 25 – 1.5 = 23.5% at which he bought the TV.
The single equivalent discount offered by first shop-
keeper is 23.5%. (a) `30,000 (b) `41,000
Second shopkeeper is offering it at 25%, which makes (c) `44,500 (d) `55,000
the item cheaper for the buyer. Solution: First profit = 10%
Hence, (b) is the correct answer.
If Romit sells TV for `8100 more, then profit = 37%
Example 117: An umbrella was sold at a profit of 20%. S.P. = (100 + Profit%)% of C.P.
What is the selling price of the umbrella if the shopkeeper ∴ S.P.1 + 8100 = S.P.2
procured it at a cost of `180?
∴ (110)% of C.P. + 8100 = (137)% of C.P.
(a) 210 (b) 216 (c) 230 (d) 236 ∴ 27% of C.P. = 8100
Solution: Substituting values in the formula above, we get:
8100 × 100
(100 + 20)  ∴ C.P. = = `30,000
27
Selling price =   × 180
 100  Therefore, (a) is the correct answer.
Selling price = 12 × 18
Selling price = 216 Example 121: A man bought a horse for a certain sum
Therefore, the selling price of this umbrella is `216. and sold it at a loss of 8% on his outlay. If he had received
14
Example 118: An article is sold for `2400 at a profit of
`1800 more, then he would have gained 1 on his
25%. What would have been the actual profit or loss if it
outlay. What did the horse cost?
had been sold at `1800? 2%
Solution: Initially, let us find the cost price of the same. (a) `3500 (b) `5000
C.P. = 2400 × 100/125 = 1920. (c) `6000 (d) `8000
New selling price = `1800 ⇒ Loss = 1920 – 1800 = 120 Solution: C.P. = (Difference in S.P.) ÷ (% Difference in
∴ Loss percentage = 100 × 120/1920 = 6.25%. profit)
Example 119: Ram buys some paper wind fans at 4 per C.P. of the horse = 1800 × 100 / [14 1/2 – (–8)]
rupee. He then buys the same number of paper wind = 1800 × 100/22.5 ⇒ C.P. = 8000.
fans from another shop at 5 per rupee. He puts them all Therefore, (d) is the correct answer.
together and sells them at 4 per rupee. Will he make a
profit or incur a loss? How much? Example 122: A man purchases two pens for `740. He sells
one at 12% profit and the other at 8% loss. Then he neither
115 115 gains nor loses. Find the cost price of each pen (in `).
(a) Profit = (b) Loss =
9% 9% (a) 324, 416 (b) 296, 444
100 100 (c) 288, 452 (d) 365, 375
(c) Profit = (d) Loss =
9% 9% Solution: C.P. of two pens = 740.
1
Solution: C.P. of one wind fan in the first shop = ` Let C.P. of the first pen be x and C.P. of the second pen be y.
4
Since there is no profit and loss in the whole transac-
1 tion, so 12% of x = 8% of y.
C.P. of one wind fan in the second shop = `
5 ⇒ x:y = 2:3
On mixing these two wind fans, the cost price of these 2
Hence, the cost of first pen =   × 740 = `296, and that
1 1 9 3
two will be = + = ` 3
4 5 20 of the second pen =   × 740 = `444.
9 5
9 Therefore, (b) is the correct answer.
So, C.P. of one wind fan = 20 = `
2 40 Example 123: A merchant buys 30 kg of rice at `40/kg
Actual price at which one wind fan is sold after com- and another 20 kg of rice at `30/kg. He mixes them and
1 sells half of the mixture at `36/kg. At what price should he
bining = ` sell the remaining mixture to get an overall profit of 30%?
4
Solution: Total cost for the entire quantity of rice =
1 9 1
S.P. – C.P. = − = (30 × 40) + (20 × 30) = `1800.
4 40 40 If his profit is 30%, then sales value = 1800 × (100+30)
It is positive; so it is profit. = `2340.

M05_MADAN 04_65901_C05.indd 24 23/12/22 7:24 PM


Mathematical Reasoning and Aptitude 5.25

He sells 25 kg at `36/kg = `900. Therefore, to make the Principal


said amount of profit, he should sell the remaining 25 kg The money borrowed or lent out for a certain period is
of rice at `2340 – `900 = `1440. called the principal or the sum. It is denoted by P.
∴ The selling price of 1 kg of rice for the remaining 25
kg = 1440/25 = `57.6. Simple Interest Calculations
Interest is the extra money that the borrower pays for
Example 124: A dealer paid a car manufacturer `135000
using the lender’s money. Simple interest is denoted
for a car. What should be the selling price of the car if
by S.I.
after allowing a buyer 10% discount on the selling price,
Simple interest (S.I.) = P × R × T/100. Here, R and T are
he made a profit of 8% on his outlay?
basically rate of interest and time.
(a) `1,50,000 (b) `1,60,000 S.I
(c) `1,62,000 (d) `1,52,500 Principal amount is P = 100 ×
(R × T )
Solution: S.I.
T = 100 ×
Here, we have to make two kinds of assumptions. ( P × R)
Let CP of dealer = `100 S.I.
Profit = `8 R = 100 ×
SP = 100 + 8 = `108 (P × T)
Let us make other assumption to calculate SP after dis- Compound Interest
count of `10.
Now SP = `100 Here, we are exploring some simple questions on compound
Discount = `10 interest as per NTA-NET exam pattern. Compound interest
Net SP = `100 - Rs 10 = `90 is basically an amount paid not only on the principal amount
If Net SP is 90 then SP = `100 but also on whatever interest has been paid already. In other
If Net SP is 1 then SP = 100/90 ` words, interest amount is also added to the original principal
If Net SP is 108 then SP = 100/90 × 108 = `120 amount and that earns interest on the total investment.
As per question again Interest accumulated over one period is applied to
If CP is 100 then SP = `120 the principal before calculating the interest for the next
If CP is Rs 1,35,000 then SP = 120/100 × 1,35,000 = period. Typical intervals are quarterly (four times a year),
` 1,62,000 monthly, daily and on a regular basis.
Therefore, (c) is the correct answer. The formula for compound interest is given as follows:
Compound interest = A – P
Example 125: A man sells an article at a profit of 8%. If where A is the amount to be available at the end of
N
the cost price were 10% less and the selling price `18 less, ⎛ R ⎞
A = P ⎜1 + period.
then his profit would have been 15%. Find the cost price ⎝ 100 ⎟⎠
of the article.
(a) `430 (b) `450 Example 126: How much time will it take for an amount
(c) `220 (d) `400 of `900 to yield `81 as interest at 4.5% per annum of
simple interest?
Solution: Let C.P. of the article = 100 (a) 2 years (b) 3 years
Thus, old S.P. = `108 (c) 1 year (d) 4 years
New C.P. = 90
As the profit is 15%, so the new SP = 90 × (115/100) = Solution: P = `900
`103.5. S.I. = `81
The difference in the two S.P. = 108 – 103.5 = `4.5. T=?
If difference in S.P. is Rs 4.5, then C.P. = `100 R = 4.5%
If difference in S.P. is 18, then C.P. = (100/4.5) × 18 = `400. 100 × SI 100 × 81
Therefore, (d) is the correct answer. T= = = 2 years
PR 900 × 4.5
Example 127: Find S.I. on `6250 at 14% per annum for
Interest 146 days.
Simple Interest (a) `350 (b) `450
When we give some house for a rent, we get rent on it. (c) `550 (d) `650
Similarly, when we lend money to a borrower, the bor- Explanation
rower is to pay an extra amount of money to the lender. P = `6250, R = 14%
This extra money is known as the interest. If the interest
on a sum borrowed for a certain period is reckoned uni-  146  2
T=  = years.
formly, then it is called simple interest or the flat rate.  365  5

M05_MADAN 04_65901_C05.indd 25 23/12/22 7:24 PM


5.26 Chapter 5

6250 ×14 ×(2 /5)


S.I. = = `350 n
100 ⎛ 7 ⎞
Then, 30,000 ⎜ 1 + = 34347
34,347
Therefore, (a) is the correct answer. ⎝ 100 ⎟⎠

Example 128: A certain sum of money amounts to `1008 107 ⎞


n
34,347 11,449 ⎛ 107 ⎞
2

1 ⇒ ⎛⎜ = = =⎜
in 2 years and to `1164 in 3 years. Find the sum and the ⎝ 100 ⎟⎠ ⎟
30,000 10,000 ⎝ 100 ⎠
2
rate of interest. ∴ n = 2 years.
(a) 800, 14% (b) 800, 13% Example 132: Simple interest on a certain sum of money
(c) 800, 12% (d) 800, 19% for 3 years at 8% per annum is half the compound interest
on `4000 for 2 years at 10% per annum. The sum placed
Explanation on simple interest is
1
S.I. for 1 years = `(1164 – 1008) = `156 (a) `1550 (b) `1650 (c) `1750 (d) `2000
2
⎛ 2 ⎞ Explanation
S.I. for 2 years = ` ⎜ 156 × × 2⎟ = `208
⎝ 3 ⎠
⎡ ⎛ 10 ⎞
2

Therefore, principal = `(1008 – 208) = `800. C.I. = ` ⎢4000 × ⎜ 1 + ⎟ − 4000⎥
Now, P = 800, T = 2 and S.I. = 208. ⎢⎣ ⎝ 100 ⎠ ⎥⎦
(100 × S.I.) (100 × 208)  ⎡ 11 11 ⎤
Therefore, rate = =  = 13% = ` ⎢4000 × × − 4000⎥ = `840
(P × T )  (800 × 2)  10 10
⎣ ⎦
Therefore, (b) is the correct answer.
⎛ 420 × 100 ⎞
∴ Sum = ` ⎜ = `1750
Example 129: The simple interest on a certain sum of ⎝ 3 × 8 ⎟⎠
1
money for 2   years at 12% per annum is `40 less than
2 Therefore, (c) is the correct answer.
1
the simple interest on the same sum for 3   years at Example 133: The compound interest on a certain sum
2
10% per annum. Find the sum. of money for 2 years at 10% per annum is `525. The
(a) `600 (b) `666 (c) `780 (d) `800 simple interest on the same sum for double the time at
half the rate per cent per annum is
Explanation
(a) `400 (b) `500 (c) `600 (d) `800
Let the sum be `x. ( x × 10 × 7)   ( x × 12 × 5) 
Then we can write:   –  = 40. Explanation
 (100 × 2)   (100 × 2)  Let the sum (that is principal amount here) be `P.
This can be written as: 7x/20 – 3x/10 = 40.
Therefore, we have x = `800. ⎡ ⎛ 2

10 ⎞
Therefore, (d) is the correct answer. Then, ⎢ P ⎜ 1 + ⎟ − P ⎥ = 525
⎢⎣ ⎝ 100 ⎠ ⎦⎥
Example 130: A man took a loan from a bank at the rate
of 12% p.a. simple interest. After 3 years, he had to pay ⎡⎛ 11 ⎞ 2 ⎤
`5400 interest only for the period. The principal amount ⇒ P ⎢⎜ ⎟ − 1⎥ = 525
borrowed by him was ⎢⎣⎝ 10 ⎠ ⎦⎥
(a) `12,000 (b) `11,000 ⎛ 525 × 100 ⎞
(c) `14,000 (d) `15,000 ⇒ P =⎜ ⎟ = 2500
⎝ 21 ⎠
Explanation Thus, principal amount = `2500
(100 × 5400) 
Here, the principal = `   = `15,000.
 (12 × 3)  ⎛ 2500 × 5 × 4 ⎞
So, S.I. = `⎜ ⎟⎠ = `500
Thus, the correct option is (d). ⎝ 100
Example 131: The compound interest on `30,000 at 7% Therefore, (b) is the correct answer.
per annum is `4347. The period (in years) is
1 Example 134: An amount doubles itself in 15 years
(a) 2 (b) 2 (c) 3 (d) 4 What is the rate of interest?
2
Explanation (a) 7.85% (b) 9.41% (c) 6.66% (d) 4.21%
Amount = `(30,000 + 4347) = `34,347
Explanation
Let the time be n years.
Let the principle be P.

M05_MADAN 04_65901_C05.indd 26 23/12/22 7:24 PM


Mathematical Reasoning and Aptitude 5.27

As the amount doubles itself, the interest is P too. Total age = 15 × 30 = 450 years
So P = (P X R X 15) / 100. New total age = 16 × 31 = 496
⇒ R = 100/15 = 20/3% = 6.66% The new total age is higher because the age of the
Therefore, (c) is the correct answer. teacher is also included in it.
Age of teacher = 496 − 450 = 46 years
Example 135: A certain sum of money amounts to `2500
in a span of 5 years and further to `3000 in a span of Example 138: India scores 52 runs in the first 10 overs
7 years at a simple interest. The sum is in a 50 over one day cricket match against Pakistan. What
(a) `1800 (b) `2000 should be the run rate in the remaining 40 overs to reach
(c) `1400 (d) `1250 the target of 252 runs?
Explanation Solution: The score required in 40 overs = 252 − 52 =
2500 in the fifth year and 3000 in the seventh year. 200 runs
Therefore, in the gap of 2 years, `500 has increased. Average required run rate in 40 overs = 200/40 = 5
⇒ For a year, 500/2 = 250.
Thus, there is per year increase of `250; then in 5 years Example 139: The average monthly income of A and B is
`5050. The average monthly income of B and C is `6250
⇒ 250 × 5 = 1250 and the average monthly income of A and C is `5200. The
Hence, the initial amount must be 2500 – 1250 monthly income of A is
= `1250. (a) `3500 (b) `4000 (c) `4050 (d) `5000
Therefore, (d) is the correct answer.
Solution: Total income of A and B = (5050 × 2) = `10,100
Averages (i)
The arithmetic mean or simply ‘average’ or ‘mean’ of a Total income of B and C = (6250 × 2) = `12,500(ii)
group of values is the sum of the values divided by the
Total income of A and C = (5200 × 2) = `10,400(iii)
total number of values. It is one of the measures of central
Adding (i), (ii) and (iii), we get 2(A + B + C) = 33,000
tendencies. It is denoted by X, where X is the variable,
such as height, weight and score. Or A + B + C = 16,500
But B + C = 12,500 from Equation (ii)
Average of n observations = Sum of observations/
Hence, A = 16,500 − 12,500 = 4000
n = ∑x n
Therefore, (b) is the correct answer.
Combined average: N1X1 + N2 X2 + … + Nn Xn/(N1 + N2 +
… + N ), where N , N , …, N are the respective numbers Example 140: A person divides his total route of journey
n 1 2 n
into two equal parts. The speeds for both the parts are
of observations in different groups and X1, X2, …, Xn are
the averages. 40 km/h and 30 km/h. Find his average speed during the
Suppose a man covers a certain distance at x kmph and entire journey.
an equal distance at y kmph, or x and y are the speeds (a) 32 km/h (b) 34.3 km/h
in going to a station and coming back; then the average (c) 36.3 km/h (d) 38 km/h
speed during the whole journey is
Solution: If x and y and z are two speeds during the
2xy journey, we can use the following formula to calculate the
kmph
x+ y average speed =
Example 136: If a candidate scores 5, 15, 25, 10 and 2xy
15 marks in different subjects, then calculate the mean (x + y)
marks scored by the candidate. (2 × 40 × 30)
Average speed =
Solution: (40 + 30)
5 + 15 + 25 + 10 + 15 70 (2 × 40 × 30)
Mean marks = = = 14 = = 34.3 km/hr
5 5 70
Example 137: The average age of 30 boys in a class is 15 Hence, (b) is the correct answer.
years. If the age of the teacher is also included, then the
new average age becomes 16 years. What is the age of the Example 141: A train starts full of passengers. At the
teacher?
first station, it drops one-third of these and takes in 96
Solution: Here, average age = total age/number more. At the next, it drops one-half of the new total and
Total age = Average age × Number takes in 12 more. On reaching the next station, there are

M05_MADAN 04_65901_C05.indd 27 23/12/22 7:24 PM


5.28 Chapter 5

found to be 248 left. With how many passengers has the x x 5x


= + + hrs
train started with? (3 × 25) (4 × 30) (12 × 50)
(a) 464 (b) 520 (c) 564 (d) 680 18x 3x
= hrs = hrs
Solution: Let the train start with X passengers. 600 100
After dropping one-third and taking in 96 passengers, the 100 1
Then, average speed = (x/1) /(3x/100) = = 33 km/h
train has 3 3
X 2x 2x + 288 Thus, (b) is the correct answer.
X− + 96 = + 96 = Passengers
3 3 3 Example 144: The average salary of male employees in a
At the second station, the number of passengers firm was `6000 and that of females was `5600. The mean
salary of all the employees was `5800. What is the % of
2x + 288
= +12 female employees?
6
(a) 42% (b) 50% (c) 55% (d) 62%
2x + 288
Now, +12 = 248
6 Solution: Average salary of male employees = `6000
Or, 2x + 288 =1416 Average salary of female employees = `5600
∴   x = 564 Mean salary = `5800
Thus, (c) is the correct answer. By using allegation
(6000) (5600)
Example 142: There were 35 students in a hostel. If
(5800)
the number of students increases by 7, the expenses 200 200
of the mess increase by `42 per day while the average
expenditure per head diminished by `1. Find the original ∴ The ratio of male : female = 200:200 = 1:1
expenditure of the mess 1
Hence, % of female employees = ×100 = 50%
(a) 450 (b) 500 2
(c) 400 (d) 420 Therefore, option (b) is correct.
Solution: Example 145: Average present age of father, mother and
Suppose the average expenditure was `x daughter is 26 years. Average age of father and daughter
Total expenditure = 35x five years ago is 21 years. Find the age of mother after 10
According to question years
(35x + 42) (a) 26 yrs (b) 18 yrs (c) 32 yrs (d) 36 yrs
Average expenditure = =x–1
35 + 7
Solution: Total of ages of father, mother and daughter
35x + 42 = 42x – 42 = 26 × 3 = 78 yrs
x = 12
Thus, the original expenditure of the mess = 35 × 12 = `420 Total of ages of father and daughter = (21 × 2) + 10 = 52
Hence (d) is the correct answer. yrs
Age of mother = 78 – 52 = 26 years
Example 143: One third of a certain journey is covered Mother’s age after 10 yrs = 26 + 10 = 36 years
at the rate of 25 km/h, one fourth at the rate of 30 km/h, Thus, the correct answer is (d).
and the rest at 50 km/h. Find the average speed of whole Example 146: The average temperature for Wednesday,
journey. Thursday and Friday was 40°C. The average for Thursday,
1 Friday and Saturday was 41° C. If temperature on Saturday
(a) 34 km/h (b) 33 km/h
3 was 42° C, what was the temperature on Wednesday?
1 (a) 39° C (b) 44° C (c) 38° C (d) 41° C
(c) 35 km/h (d) 32 km/h
2 Solution:
Solution: Let the total journey be x km. Average temperature for Wednesday, Thursday and
x x Friday = 40° C
Then km travelled at the speed of 25 km/h and km Total temperature = 3 × 40 = 120° C
3 4
x x 5x Average temperature for Thursday, Friday and Saturday
at 30 km/hr and the rest of distance (x – – ) = was = 41° C
3 4 12
Total temperature = 41 × 3 = 123° C
travelled at the speed of 50 km/h.
Total time taken during the journey of x km Temperature on Saturday = 42° C

M05_MADAN 04_65901_C05.indd 28 23/12/22 7:24 PM


Mathematical Reasoning and Aptitude 5.29

Now, Counting of O dd Days


(Thursday + Friday + Saturday) – (Wednesday + 01 January 0001 is assumed to be a Monday.
Thursday + Friday) = 123 –120;
1 ordinary year = 365 days = 52 weeks + ‘1 day’. That 1
Saturday – Wednesday = 3
extra day is counted as odd day.
Wednesday = 42 – 3 = 39° C
1 leap year = 366 days = (52 weeks + 2 days)
1 leap year has 2 odd days.
Calendar Questions
100 years = 76 ordinary years + 24 leap years
In NET examination, usually one simple calendar ques- = (76 × 1 + 24 × 2) odd days = 124 odd days
tion is asked. We need to be familiar with a few concepts. = (17 weeks + 5 days).
It is necessary to know the concept of ‘odd days’ to It means 5 odd days.
solve calendar problems. We are supposed to find the day
of the week on a given date. Therefore, the number of odd days in 100 years is 5.
Number of odd days in 200 years = (5 × 2) = 10 days. If
Odd days: In a given period, the number of days more further divided by 7, then it gives us 3 odd days.
than the complete weeks is called odd days.
Number of odd days in 300 years = (5 × 3) = 15 days,
Leap year: A leap year has 366 days. Every year divisible which means ‘1 odd day’.
by 4 is a leap year, but not if it is a century. Only every Number of odd days in 400 years = (5 × 4 + 1) = 0 odd
fourth century year is a leap year. day
Examples: Each of the years, such as 1880, 1988, 2004, Similarly, each one of 800th year, 1200th year, 1600th
2012, 2016, 2020, 2024, is a leap year. year, 2000th year, 2400th year, and so on has 0 odd day.
Each of the years, such as 400, 800, 1200, 1600 and
2000, is a leap year, but 1700, 1800, 1900, and so on are Day of the W eek Related to O dd Days
not leap years. We usually count days from Sunday.
For example, if as per our calculation the number
of ‘odd days’ is 2, then it means that the answer will be
Ordinary year: The year that is not a leap year is called Tuesday.
an ordinary year. An ordinary year has 365 days.

Day Sunday Monday Tuesday Wednesday Thursday Friday Saturday


Number 0 1 2 3 4 5 6

Example 147: What was the day of the week on 1 January Although 1996 is a leap year, the day in question is
2001? [June 2009] in January, so there will be an increase of 1 day. Thus,
1 January 1996 is Monday.
(a) Friday (b) Tuesday
(c) Sunday (d) Wednesday Therefore, (b) is the correct answer.

Explanation Example 149: On 12 January 1980, it was a Saturday.


The day of week on 12 January 1979 was a
By the end of centuries 400, 800, 1200, 1600, 2000, 2400
and so on, there are no extra days left. It means that the (a) Thursday (b) Friday
last year 2000 was Sunday. (c) Saturday (d) Sunday
As per convention, the week starts with Monday and
hence, the next day after last day of year 2000, that is, Explanation
1 January 2001, is Monday. Although 1980 is a leap year, it would not have any impact
Therefore, (b) is the correct answer. on the calculation as the date in the question is in January
month (and before 28 February). Thus, it is assumed as a
Example 148: 1 January 1995 was a Sunday. What normal year. There is a gain of 1 day in the next year.
would be the day of the week on 1 January 1996? However, here, we will do backward calculation, and
 [December 2009] subtract 1 day from the year 1980. If it was Saturday, then
the same day in the previous year was to be Friday.
(a) Sunday (b) Monday
(c) Wednesday (d) Saturday Therefore, (b) is the correct answer.

Explanation Example 150: If 1 October is Monday, then 1 November


will be
There is an increase of 1 day (odd day) in the subsequent
year. In case of a leap year (if the day is after February), (a) Thursday (b) Friday
there will be an increase of 2 days. (c) Sunday (d) Monday

M05_MADAN 04_65901_C05.indd 29 23/12/22 7:24 PM


5.30 Chapter 5

Explanation
Clock Questions
October has 31 days. 1, 8, 15, 22 and 29 dates of October
will be Mondays. Thus, 1 November will be a Thursday. Angle (between minute hour and hour hand) = [(11/2) M
Therefore, (a) is the correct answer. – 30H)] where M shows minutes and H shows hours.

Example 151: If the first day of the month is Sunday, 155. We assume that the hours hand is closer to 12 than
then what date will be 3 days after the fourth Wednesday the minutes hand in a conventional clock. At which
in the month? [June 1998] of the following timings the angle between 10 am
and 11 am will the angle between the minutes hand
(a) 24 (b) 29 (c) 27 (d) 30 and hours hand be 3°
(a) 10:45 (b) 10:50 (c) 10:54 (d) 10:58
Explanation
First Wednesday of the month will be on 4th day of the Solution:
month, 2nd Wednesday will be on 11th, 3rd Wednesday Let us assume that 10 hrs × mins.
on 18th and 4th Wednesday on 25th. Now, 3 days after Let’s measure the angle from 12 in clockwise direction.
25th, which means 25 + 4 = 29th day of the month. Angle made by hours hand = 10(360°/12) + x(30°/60)
Therefore, (b) is the correct answer. = 10(30°) + × (1/2) = 300° + x/2...(1)
Angle made by minutes hand = x(360°/60)
Example 152: If the first day of the ordinary year (other = x(6°)…(2)
than the leap year) was Friday, then which was the last Given that hours hand is nearer to 12 than minutes
day of that year? hand, i.e., hours hand crossed minutes hand.
(a) Wednesday (b) Thursday Hence, (1) – (2) = 3°
(c) Friday (d) Sunday ⇒ 300°+ x/2 – 6x = 3°
⇒ –11x/2 = –297
Explanation ⇒ x = 54
An ordinary year has 365 days. Week starting with Friday Hence, the time will be 10:54.
will end in Thursday. Hence, the 364th day (end of com- Thus, the answer is option (c).
plete 52 weeks) will be Thursday. The 365th day will be 156. At which of the following timings the hands of the
Friday. Thus, the first and last days of an ordinary year clock will show the mirror image of 4:10?
are same. (a) 7:10 (b) 7:40 (c) 7:50 (d) 8:10
Therefore, (c) is the correct answer. Solution:
Example 153: It was Monday on 1 January 2007. What
was the day of the week on 1 January 2011?
(a) Sunday (b) Saturday
(c) Friday (d) Wednesday
4:10 7: 50
Explanation
It should be 7:50.
Odd days in 2007 = 1 (2007 is an ordinary year and we are So, the answer is option (c).
doing calculation from January 1)
Odd days in 2008 = 2 (2008 is a leap year) 157. A and C are running on a circular track with the
Odd days in 2009 = 1 (ordinary year) speeds of 20m/s and 24m/s. A is running in the
Odd days in 2010 = 1 (ordinary year) clockwise direction and C is running in the anti-
Thus, 1 January 2011 will be Monday plus 5 days, that clockwise direction. At how many distinct points
is, Saturday. these two will meet on the circular track?
(a) 5 (b) 6 (c) 11 (d) 10
Therefore, (b) is the correct answer.
Solution:
Example 154: If the 4th Saturday of a month is the 22nd
If two bodies are moving in opposite direction with
day, then what day is the 13th day of the month?
their speeds in the ratio a: b, the number is distinct
(a) Tuesday (b) Wednesday points at which they meet on a circular track = a + b.
(c) Thursday (d) Friday Here, the ratio = 20:24 ⇒ 5:6
Therefore, number of distinct points = 5+6 = 11
Explanation So, the answer is option (c).
The earlier 3 Saturdays are on 15th, 8th and 1st. If 15th is 158. What is the angle made between the hour and min-
Saturday, Thursday falls on 13th. Thus, 13th is Thursday. ute hands of a clock 30 minutes after they meet for
Therefore, (c) is the correct answer. the third time in a day?
(a) 150 degrees (b) 165 degrees
(c) 180 degrees (d) can’t be calculated

M05_MADAN 04_65901_C05.indd 30 23/12/22 7:24 PM


Mathematical Reasoning and Aptitude 5.31

Solution: 3. If one word belongs to a second group that further


In thirty minutes, the minute hand covers 180 belongs to the third group:
degrees and hour hand covers 15 degrees. So, angle
between the hands is 165 degrees. Cow, Mammals and Animal kingdom
So, the answer is option B.
159. For how many times in a day do the minute and hour
hands of a clock overlap?
(a) 23 (b) 24 (c) 22 (d) Any number
Solution: Mammals
In twelve hours, the hands overlap 11 times. So, in a
day, they overlap 22 times. Cow
So, the answer is option C.

Venn Diagrams
Animal kingdom
In NET Paper I examination, one or two questions are asked
about Venn diagrams almost every time. While the circles
or figures (also called sets) deal with individual data items,
the number shown in rectangle is the total number (rectan-
gle is also known as universal set). 4. If there is some relation between two items and these
There can be other shapes such as triangles and squares two items are completely related to a third item, then
to represent data. The following examples will help in better they will be shown as follows:
understanding on how to solve questions on Venn diagrams.
Men, Brother and Father

Venn Diagrams and Relationships


The main aim of this section is to test your ability about
the relation between some items of a group by diagrams.
In these questions, some figures of circles and some words
Father Brother
are given. You have to choose a figure that represents the
given words.
Some critical examples are as follows:
1. If all the words are of different groups, then they will Men
be shown by the diagram as follows:
Dog, Cow and Horse
Some brothers may be fathers and vice versa.
Similarly, some fathers may not be bothers and vice
Dog Cow Horse versa. But all the fathers and all the brothers belong
to the group men.
5. Two items are related to a third item to some extent but
All these three are animals but of different groups, not completely and the first two items are totally different:
and there is no relation between them. Hence, they Students, Boys and Girls
will be represented by three different circles.
2. If few words belong to a common group:
Cow, Horse and Mammals
Boys Students Girls

Mammals

The boys and girls are different genders while some


Cow Horse boys may be students. Similarly, among girls, some
may be students.
6. All the three items are related to one another but to
some extent and not completely:

M05_MADAN 04_65901_C05.indd 31 23/12/22 7:24 PM


5.32 Chapter 5

Boys, Students and Athletes Solution


Step I

Boys F H
Students

Athletes

Some boys may be students and vice versa. The eight students who play both hockey and football
Some boys may be athletes and vice versa. should go inside the intersection because they need to be
Some students may be athletes and vice versa. in both circles. The seven students who do not play either
7. Two items are related to each other completely and the of the two games should go outside because they should
third item is entirely different from the first two: not be in either of the circles.
Lions, Carnivorous and Cows Step II

F H

Lions
11 8 5

Carnivorous Cows 7
animals
There are 13 students who play hockey, so the numbers
in the hockey circle should add up to 13.
There are 8 students in the intersection, so there are 5
All lions are carnivorous but no cow is a lion or a carnivore. who play hockey but not football.
8. The first item is partially related to the second, but the Similarly, there are 19 students who play football.
third is entirely different from the first two: The students who play football but not hockey must be
Dogs, Flesh eaters and Cows 19 – 8 = 11.
Thus, the total number of students in the class = 11 + 8
+ 5 + 7 = 31.
There are 31 students in the class.
Flesh
Dogs Cows Example 161: In a class, there are 30 students.
eaters
(a) 21 students like Maths.
(b) 16 students like English.
Some dogs are flesh eaters but not all, whereas any (c) 6 students don’t like Maths or English.
dog or any flesh eater cannot be cow. How many students like both Maths and English?
  There can be many other situations too.
Solution
  Now, we discuss Venn diagram to solve mathemati-
cal problems. This can be used as an easier approach Step I
to solve problems, which can be practised by students
from non-mathematical background also. M E

Example 160: In a class, there are:


(a) 8 students who play football and hockey.
(b) 7 students who do not play football or hockey.
(c) 13 students who play hockey.
(d) 19 students who play football. 6
How many students are there in the class?

M05_MADAN 04_65901_C05.indd 32 23/12/22 7:24 PM


Mathematical Reasoning and Aptitude 5.33

The 6 students who do not like either of the two subjects Solution:
go outside because they should not be in either circles. We A. Yaman travels 600 m in 5 minutes, that means his
know that the total in the Maths circle needs to be 21, but we speed in m/s is
cannot put this in because we don’t know how many should Speed (in m/s) = 600 m / 300 seconds = 2 m/s
go inside the intersection (if they like both subjects) and Multiplying it by 18/5 will give us Km/hr
how many should go on the left (if they like only Maths). Speed (in Km/hr) = 2 x 18/5 = 7.2 km/hr
Step II B. Let’s assume that total journey = 1
Journey by train and bus = 2/15 + 9/20 = 35/60
   = 7/12
M E Journey on cycle = 1 – 7/12 = 5/12
Let’s verify this: 5/12 of 24 Kms = 10 Kms
21 in Thus, this statement is correct.
total like 3 C. Anita’s 8 day’s work = 1
Maths
Her 1 day’s work = 1/8
Her 2 day’s work = 1/8 × 2= 1/4
6
So, this statement is not correct.
Overall, (c) is the correct answer.
There are 30 students in the class, and if there are 6 stu-
dents outside the circles, then the other 3 sections must 163. The owner of a mobile phone charges his customer
add up to 24. 25% more than the cost price. If a customer paid
Furthermore, there are 21 students who like Maths, so 8,000/- for a mobile phone, then what was the cost
the middle and left sections must add up to 21. This leaves price of the mobile? [December 2021]
3 on the right as 24 − 21 = 3. (a) ` 5600 (b) ` 6000
(c) ` 6400 (d) ` 7000
Step III
Solution:
The owner sells the mobile at the profit of 25%. The
M E
customer pays ` 8000 for it.
Let the CP = ` x
8 13 3 SP = x + 25% of x = 1.25X
1.25x = ` 8000 (given)
X =8000/1.25
X = ` 6400
6 Thus, the CP = ` 6400
Correct answer is (c).
There are 16 students who like English. Hence, the 2 parts
164. Given the relationship between variables Y and X :
of the English circle should add up to 16 and so we can
Y = X2; if X changes by 6%. What is the percantage
find the number in the intersection by doing 16 − 3 = 13.
change in Y?
There are 21 students who like Maths. There are 8 (got by
(a) ~ 36% (b) ~ 12%
21 − 13) who like Maths but not English. If we check all the
(c) ~ 8% (d) ~ 6%
four facts given to us, then we can see that they are all true.
There are 13 students who like both Maths and English. Solution:
Y = X2
Mix Bag Questions Let the value of X = ` 100
Y = (100)2 = 10000
162. Which of the following statement/s is/are correct? New value of X = 100 + 6 = 106
 [December 2021] New value of Y = (106)2 = 11236
A. Yaman crosses a 600m long road in 5 minutes. His Thus, the difference in the value of Y = 11236 – 10000
speed is 7.2 km/hr = 1236
B. Vinod performs 2/15 of his total journey by train; Percentage difference = 1236 / 10000 × 100 = 12.36%
9/20 by bus and rest 10 km on cycle. His total = 12% apprx
journey is 24 km Short cut method = 6 + 6 + (6 × 6) / 100 = 12.36%
C. Anita can complete a work in 8 days. She can com- Correct Answer is (b)
plete 1/8 fraction of the work in two days.
Choose the correct answer from the options given below 165. A seller mixes 26 kg rice of a price of ` 20/kg, with
30 kg of different variety rice of a price of ` 36/kg.
(a) A only (b) B only
(c) A and B only (d) A and C only

M05_MADAN 04_65901_C05.indd 33 23/12/22 7:25 PM


5.34 Chapter 5

He sells both the mixed rices at ` 30/kg. What will A = 1/3 (B + C + D)


be the profit in percentage? [December 2021] 3A = B + C + D
Adding A on both sides
(a) 5% (b) 10% (c) 15% (d) 20% 3A + A = A + B + C + D
Solution: 4A = 80 (as per equation 1)
Total Input cost of 56 kg (26 + 30) of rice = ` (26 × 20 Thus, A = 80 / 4 = 20
+ 30 × 36) = ` (520 + 1080) = ` 1600 Alternative solution: The average of 4 terms is 20.
Then selling price of 56 kg rice =` 56 x 30 = ` 1680 Again, A represent the one third of sum of rest of
Profit = 1680 – 1600 = ` 80 three terms. This shows that A itself is equal to aver-
Profit percentage = 80/1600 × 100 = 5% age value. That average (of 20) is already given to us
in the question statement. Thus, the answer is (b).
166. A milk man purchases milk at ` X per litre and sells it
at ` 2x per litre. Still he mixes 2 litres of water with 169. X has one and a quarter times as many as Y, and Z has
every 6 litre of pure milk. What is his profit percent- one and a quarter times as many as X. Altogether,
age? [December 2021] they have 61. How many do Z, Y and X have, respec-
tively? [December 2021]
(a) 33.33 (b) 66.66 (a) 25, 16, 20
(c) 133.33 (d) 166.66 (b) 20, 16, 12
Solution: (c) 25, 20, 16
(d) 16, 20, 25
Let the total quantity = 6 L. And Price is ` 1 per L
Then total CP = 6 × 1 = ` 6 Solution:
Total quantity sold = 2 × 6 + 2 = 14 L X/Y = 5/4; that means X:Y = 5:4 OR
Total SP = 2 × 6 + 2 × 2 = ` 16 Y:X = 4:5            …….(i)
Profit = ` 16 – Rs 6 = ` 10 Z = 5/4 of X
Profit percentage = (16 – 6)/6 × 100 Z/X = 5/4 that means Z:X = 5:4 OR
        = 10/6 × 100 X:Z = 4 : 5           ………(ii)
        = 166.66% Now bringing (i) and (ii) together to put x, y and z to
Thus (d) is the answer. get their standard ratio.
167. The average monthly income of P and Q is ` 5,050/–. Y : X : Z
The avarage montly income of Q and R is ` 6,250/– 4 : 5
and that of P and R is ` 5,200. What will be the 4 : 5
monthly income of P? [December 2021] 4×4 : 5×4 : 5×5
(a) ` 3,000/– (b) ` 3,500/– 16 : 20 : 25
(c) ` 4,000/– (d) ` 4,500/– As per question: Z : Y : Z = 25 : 16 : 20
Solution: Thus, we get the ratio. As a matter of fact, their total
also come to be 61 (=25 + 16 + 20). So, no further
Let P, Q and R represent their respective monthly calculation is required to get their exact numbers. This
income. Then, we have: can be taken as their answer as well. (a) is the correct
Total income answer.
P + Q = 5050 × 2 = 10100
Q + R = 6250 × 2 = 12500 170. Consider the following statements:
P + R = 5200 × 2 = 10400  [December 2021]
Adding A. A single discount equivalent to three successive
2P + 2Q + 2R = 2(P + Q + R) = ` 10100 + 12500 + discounts of 10% 20% and 25% is 46%
10400 = ` 33000 B. A mobile is sold for ` 14,500 at a loss of 20%. The
P + Q + R = 33000 / 2 = ` 16500 cost price of the mobile is ` 18,225.
P + (12500) = 16500 C. If the loss is 1/3 of the selling price, the loss per-
P = ` 16500 – 12500 = ` 4000 centage is 28%
Thus (c) is the right answer.
Choose the correct answr from the options given below:
168. The average of 4 terms is 20, and the first term is (a) A and B only (b) B and C only
1/3rd of sum of the remaining 3 terms. What will be (c) A and C only (d) A only
the value of first term? [December 2021]
(a) 15 (b) 20 (c) 25 (d) 30 Solution:
Solution: Question Part A:
Let the terms are A, B, C and D The single discount equivalent to a series of 10%, 20%
Their total = A + B + C + D = 4 × 20 = 80 …..(1) and 25% is 46%

M05_MADAN 04_65901_C05.indd 34 23/12/22 7:25 PM


Mathematical Reasoning and Aptitude 5.35

First, we want to see the impact of discounts of 10% and 172. How many terms are there in the series
20%. Then third discount will be taken into account.  [December 2021]
The basic formula will be (X + Y – XY/100)
    = 10 +20 – (10 × 20/100)% 168, 175, 185, _ _ _ _, 266?

    = 30 – (200/100)% = (30 – 2)% = 28% (a) 13/13 (b) 14/14
Now, we want to see the impact of this calculated dis- (c) 15/15 (d) 16/16
count of 28% and 25% with same formula.
    = [28 + 25 – (28 × 25/100)]% Solution:
    = (53 – 7) % = 46% The first term in the series = 168
Thus, the statement A is true. Common difference = 175 – 168 = 182 – 175 = 7
Question Part B Let the number of terms in the series = n
The nth terms of the series = 266
CP of mobile = ` 18225, Loss = 20%, SP (given) = 14500 Therefore, 168 + (n – 1) 7 = 266
We can directly calculate SP. 7n = 266 – 168 + 7
SP = 18225 × (100 – 20)/100 = 18225 × 4/5 = ` 14580 7n = 105
Thus, calculated SP does not tally with the given SP, n = 15
so it’s a wrong statement. Thus, there are 15 terms given in the series.
Question Part C The total terms can be calculated manually also.
Let loss = x and selling price = 3x
Loss = CP – SP 173. Given below are two statements [December 2021]
X = CP – 3X Statement I: The compound interest on ` 280 for 18
CP = X + 3X = 4X months at 18% per annum is ` 44.3.
Now % age loss = 4X – 3X/ 4X × 100 = 25% Statement II: At 5.6% rate of simple interest, a
Hence, statement C is a false statement. certain sum will be doubled in 15 years.
Only statement A is correct. (d) is the correct answer.
In light of the above statements, choose the correct
171. In a class of 65 students, 27 speak Hindi, 21 speak answer from the options given below:
English and 17 speak Bangla. 7 speak only Eng- (a) Both Statement I and Statement II are true
lish and HIndi, 6 speak only Hindi and Bangla and (b) Both Statement I and Statement II are false
5 speak only English and Bangla. If 12 speak only (c) Statement I is true but Statement II is false
Hindi then how many speak all the three languages. (d) Statement I is false but Statement II is true
 [November 2020]
(a) 5 Solution:
(b) 2 Statement I:
(c) 4 Compound Interest = SI of First year + SI for 1/2 year
(d) 3 (on total amount of first year)
SI for first year = 10% of ` 280 = ` 28
Solution: Principal for next six months = 280 + 28 = ` 308
Students who speak all the three languages = SI for these six months = 308 × 5 × 1 /100 = 15.40
Students who can speak Hindi–(Students who can Thus, total interest = ` 28 + ` 15.40 = ` 43.40
speak only Hindi + Students who speak only English This question can be directly solved by compound
and Hindi + Students who speak only Hindi and interest formula.
Bangla)
= 27 – (12 + 7 + 6) There is difference of 10 paisa in calculated amount
= 27 – 25 and given amount of ` 43.3 in the statement. Thus, it
=2 is not correct.
Hence, (b) is the correct answer. Statement 2: As the amount is calculated on SI, that
The Venn diagram for the following question is given amount will remain the same through 15 years. Thus,
below. in 15 years, the principal amount of ` 100 will add
15 × 5.6 = ` 84. So, it is not doubling. This statement
English Hindi is also wrong.

7 7 12 174. A total of 324 coins of 20 paise and 25 paise make


2 a sum of ` 71. The number of 20 paise coins is
5 6  [December 2021]
(a) 144 (b) 124
4 (c) 200 (d) 125
Bangla

M05_MADAN 04_65901_C05.indd 35 23/12/22 7:25 PM


5.36 Chapter 5

Solution:
x x 18
Let the coins of 20p = X – =
Then the coins of 25p = 324 – X 4 5 60
Total value = X. 20p + (324 – X).25p
    = X.1/5 + (324 – X).1/4 = 71 5x – 4x 3
=
    = X/5 + 81 – X/4 = 71 20 10
X/5 – X/4 = 71-81 = –10
–X/20 = – 10 3
X = 10 × 20 = 200 ⇒x= × 20 ⇒ 6 km
10
Hence, the number of 20p coins is 200. The correct
answer is (c). Hence, option b is the correct answer.
175. The incomes of Ravi and Kavita are in the ratio 2:3
are their expenditures are in the ratio 3:5. If each 178. The LCM of two numbers is 2376 while their HCF
saves ` 100, then Ravi’s income is  is 33. If one of the numbers is 297, then the other
 [December 2021] number is
(a) ` 200 (b) ` 600 (a) 216 (b) 264
(c) ` 400 (d) ` 100 (c) 642 (d) 792

Solution: Solution:
Let the incomes of Ravi and Kavita are 2x and 3x. As there are two numbers, the question is going to be
Income = Saving + Expenditure easy.
Expenditure = Income – saving, the ratio of their LCM × HCF = 1st number × 2nd number
expenditures is 3 : 5 2376 × 33 = 297 × 2nd number
Thus, 2x – 100/3x – 100 = 3/5 Thus 2nd number = 2376 × 33 / 297 = 264
3(3x – 100) = 5 (2x – 100) 179. If Amit works 5 hours a day, he can complete the job
9x – 300 = 10x – 500 in 8 days. Bobby can complete the same job in 10
x = ` 200 days by working 6 hours a day. If they start working
Thus, income of Ravi = 2x = 2 × 200 = ` 400 together 4 hours a day, in how many days, they will
176. The new cost of a mobile which has been discounted complete the job?
by 15% is 3230. What was the cost of the mobile (a) 3 days (b) 6 days
originally? [December 2021] (c) 4 days (d) 8 days
(a) ` 3675 (b) ` 3650 Solution:
(c) ` 3600 (d) ` 3800 The total hours taken by Amit to complete the job =
Solution: 5 × 8 = 40 hours
Let original cost (OC) = 100 The total hours taken by Bobby to complete the job =
New cost = 100 – 15 = 85 10 × 6 = 60 hours
85% of OC = 3230 Thus, one hour job work of Anu and Bobby
OC = 3230 x 100 / 85 = Rs 3800 1 1 60 + 40 100 1
The correct answer is (d). ⇒ + = = =
40 60 2400 2400 24
177. Neeraj Chopra gets late by 9 minutes for his sports Hence, Amit and Bobby together can finish the job in
ground from his home if he rides his cycle at a speed 24 hours. Working 6 hours a day, the job will be com-
of 4 km/hour. If he rides at 5 km/hour, he arrives 9 pleted in 6 days.
min early. Find the distance between his home and
sports ground. 180. The income of A is 150% of the income of B and the
(a) 8 km (b) 6 km income of C is 120% of the income of A. If the total
(c) 7 km (d) 10 km income of A, B and C together is Rs. 86000, what is
Solution: B’s income?
(a) 20000 (b) 26000
Let’s look at the basic relationship of time and distance.
(c) 28000 (d) 25000
Distance = Time × Speed
Solution:
Distance Let’s assume that income of B is Rs. 100, then
Time =
Speed 120 × 150
A : B : C = 150 : 100 :
Let the distance be x km. 100
Time difference calculated as per formula =
Calculated difference of time = 15 : 10 : 18

M05_MADAN 04_65901_C05.indd 36 23/12/22 7:25 PM


Mathematical Reasoning and Aptitude 5.37

We can solve this question directly by this formula


10
Thus, share of B = × 86000 = ` 20000 Angle = [(11/2) M – 30 H] where M and H shows
43 minute and hours in the question.
So, our answer = [11/2 × 50 – 30 × 5]
181. Channel A can fill the storage tank in 6 hours, Chan-
nel B can fill the same in 10 hours and Channel C can         = 275 – 150 = 125 degrees
empty the tank in 5 hours. The first two Channels In case, we get answer in minus, even then the answer
are kept open for 3 hours in the beginning and then is to be taken in absolute value terms.
the third Channel is also opened. In what time is the
tank full? 184. What should be the angle in degrees between the
(a) 5 hours (b) 6 hours minute and hour hands of a clock at 8:30 ?
(c) 8 hours (d) 10 hours (a) 60 (b) 75
Solution: (c) 85 (d) 90
We take the LCM of 5, 6 and 10, and assume it to be
the total work, which is = 30 Solution:
Work done by A and B for 3 hours = (5 + 3) × 3 = 24 The answer by the direct formula
Balance of work = 30 – 24 = 6    = 11/2 x 30 - 30 x 8
When C is also opened for emptying the tank after    = 165 - 240 = - 75 degrees
3 hours, then A, B and C can complete the balance work Thus, minus sign is to be ignored. (b) is the correct
in [ 6 / (5 + 3 – 6)] = 3 hours answer.
Thus, total time taken to fill the tank = 3 + 3 = 6 hours.
Hence, option B is correct.
185. The difference between two numbers is 2500. If
182. The ratio of the ages of Anglika and her mother is larger number is divided by smaller number, the
1 : 4 and the ratio of the ages of Anglika’s mother quotient comes to be 8. The remainder comes to be
and her brother is 9 : 1. If Esha’s brother is 5 years 50. The smaller number is
younger than Anglika. What will be the age of Ang- (a) 250 (b) 280
lika’s mother after 4 years? (c) 350 (d) 400
(a) 36 years (b) 40 years
Solution:
(c) 45 years (d) 50 years
Let’s assume that numbers are ‘n’ and ‘2500 + n’
Solution: According To Question
Anglika : her mother = 1 : 4 2500 + n = 8n + 50
Her mother : Her brother = 9 : 1 8n – n = 2500 – 50
Anglika: Her mother : Her brother = 9 : 36 : 4 7n = 2450
Now, ATQ n = 2450 / 7 = 350
9x – 4x = 5 Hence, option C is the correct answer.
5x = 5
x=1 186. Rupesh borrowed ` 8300 from Shah at the rate of
Anglika mother age after 4 years = 36 × 1 + 4 = 40 12% per annum at simple interest for 3 years. He
years added some money to the borrowed sum. That
money was then lent to Sonia for 3 years at 14% per
183. What is the angle between two hands of a clock when annum interest. If Rupesh gains ` 939 in the whole
it is 50 minutes past 5 o’ clock? transaction. Find out what amount had he added
(a) 95° (b) 88° from his own pocket?
(c) 110° (d) 125° (a) ` 115 (b) ` 120
(c) ` 125 (d) ` 105
Solution:
Solution:
Let’s take the difference between two amounts of
interest.
(830 + x) × 14 × 3 830 × 12 × 3
– = 93.90
100 100
830 × 42 + 42x – 830 × 36 = 9390
42x + 830 × (42 – 36) = 9390

M05_MADAN 04_65901_C05.indd 37 23/12/22 7:25 PM


5.38 Chapter 5

4410 x 65
42x = 9390 – 4980 ⇒ x = = 105 ⇒ =
42 y 35
Thus, the amount added = ` 105 After 20 years, ratio will be
187. The sum of five numbers is 5200. The average of x + 20
the first two numbers is 800 and average of last two ⇒
numbers is 1400. Determine the third number? y + 20
(a) 660 (b) 800
(c) 1500 (d) 1200 65 + 20 85 17
⇒ = =
Solution: 35 + 20 55 11
The sum of first two numbers is 2 × 800 = 1600
Hence, option (B) is the correct answer.
The sum of last two numbers is 2 × 1400 = 2800
Thus, the middle third number = 5200 – (1600 + 2800) 189. A shopkeeper sold a shirt at ` 300 after giving 10%
   = 800 discount on labelled price. Had he not given the dis-
188. The sum of the present ages of a father and his son count, he would have earned a profit of 15% on the
is 100 years. 5 Years ago their ages was in the ratio cost price. What was the cost price of the shirt?
of 2 : 1. The ratio of the age of father and son after (a) ` 190 (b) ` 195
20 years will be (c) ` 200 (d) ` 198
(a) 15 : 13 (b) 17 : 11 Solution:
(c) 11 : 19 (d) 20 : 23
Let Cost Price = 100
Solution: Labelled Price = 100 + 15 = 115
Let us assume that present ages of father and son are Selling Price after discount = 115 – 10% of 115
x and y.     = 115 – 11.5 = 103.50
We have been given, x + y = 100 Thus 103.50 % of CP = 207
⇒ x = 100 – y ..............(i) CP = 207 × 100 / 103.50 = 200
Five years ago, their ages was in ratio of 2 : 1 190. There is a two digit number. The digits are inter-
x–5 2 changed and the number thus formed is added to
⇒ = the original number. The sum of two such numbers
y–5 1 must be divisible by
x – 5 = 2y – 10 (a) 11 (b) 9
2y – x – 5 = 0 (c) 5 (d) 3
x – 2y + 5 = 0.............(ii) Solution:
Putting the value given eqn. (i) in eqn. (ii) Let the two digits at ten’s place and unit place are x
100 – y – 2y + 5 = 0 and y.
100 – 3y + 55 = 0 Thus, the number is 10x + y.
105 = 3y After interchanging the digits, the number obtained
105 = 10y + x.
Thus y = = 35 ATQ,
3 The new number formed = 10x + y + 10y + x
   = 11x + 11y = 11 (x + y)
Now putting value of y in eqn. (i) This number is exactly divisible by 11.
x = 100 – 35 = 65 years Hence, option (A) is the correct answer.
Thus, the ratio of present ages of father and son

M05_MADAN 04_65901_C05.indd 38 23/12/22 7:25 PM


Mathematical Reasoning and Aptitude 5.39

A s s e s s Yo u r L e a r n i n g

nuMBER SERIES COMPLEtIOn


1. Find the missing number in the following series. 2/3, 4/7, ?, 11/21, 16/31
512, 256, 128, ?, 32, 16, 8 (a) 10/8 (b) 6/10 (c) 5/10 (d) 7/13
(a) 52 (b) 61 (c) 64 (d) 56 11. Find the wrong number in the following sequence.
2. Find the missing number in the following series. 125, 127, 130, 135, 142, 153, 165
2, 7, 17, 32, 52, 77, ? (a) 130 (b) 142 (c) 153 (d) 165
(a) 107 (b) 91 (c) 101 (d) 92 12. Find the missing number in the following series.
3. Find the missing number in the following series. 3, 10, 101, ?
15, 18, 24, ?, 45 (a) 10,101 (b) 10,201
(a) 27 (b) 30 (c) 33 (d) 36 (c) 10,202 (d) 11,012
4. Fill in the missing number in the following arrange- 13. Find the missing number in the following series.
ment based on some principle. 1, 1, 4, 8, 9, 27, 16, ?
(a) 32 (b) 64 (c) 81 (d) 256
8 6 4 14. Find the missing number in the following series.
3 ? 7 3, 5, 13, 43, 177, ?
(a) 891 (b) 713 (c) 885 (d) 899
14 16 18
15. Find the missing number in the following series.
(a) 4 (b) 5 (c) 6 (d) 8 3, 4, 7, 7, 13, 13, 21, 22, 31, 34, ?
(a) 41 (b) 43 (c) 45 (d) 47
5. Find the missing number in the following series.
8, 24, 12, ?, 18, 54 16. Find the missing number in the following series.
(a) 26 (b) 24 (c) 36 (d) 32 10, 24, 52, ?, 220, 44, 892
(a) 104 (b) 98 (c) 112 (d) 108
6. What is the number that comes next in the sequence?
12, 20, 100, ?, 8,900, 88,900, 8,88,900 17. Which of the following will not be a number of the
(a) 1,000 (b) 900 (c) 800 (d) 400 series
1, 8, 27, 64, 125, ?
7. Find the missing number in the following series.

A S S E S S YO U R L E A R N I N G
(a) 256 (b) 512 (c) 729 (d) 1,000
28, 33, 31, 36, ?, 39
(a) 30 (b) 32 (c) 33 (d) 34 18. Find the missing number in the following series.
13, 32, 24, 43, 35, ?, 46, 65, 57, 76
8. Find the missing number in the following series.
(a) 45 (b) 52 (c) 54 (d) 55
1, 3, 4, 8, 15, 27, ?
(a) 45 (b) 50 (c) 55 (d) 60 19. Find the missing number in the following series.
23, 33, 46, 62, 81, 103, ?
9. Find the missing number in the following series.
(a) 126 (b) 130 (c) 133 (d) 128
4, 16, 8, 64, ?, 256
(a) 16 (b) 24 (c) 32 (d) 20 20. Find the wrong number in this following sequence.
10, 26, 74, 218, 654, 1946
(a) 26 (b) 74 (c) 218 (d) 650
10. Find the missing number in the following series.

LEttER SERIES COMPLEtIOn


21. Find the missing letters in the following series. 24. Find the missing letters in the following series.
QAR, RAS, SAT, TAU, __________ JAK, KBL, LCM, MDN, __________
(a) UAV (b) TAS (c) UAT (d) TAT (a) OEP (b) NEO (c) MEN (d) PFQ
22. Find the missing letters in the following series. 25. Find the missing letters in the following series.
FTG, GTF, HTI, ITH, __________ AK, EO, IS, __________, QA, UE
(a) JTK (b) HTL (c) HTK (d) JTI (a) LV (b) MW (c) NX (d) LW
23. Find the missing letters in the following series. 26. Find the missing letters in the following series.
SCD, TEF, UGH, __________, WKL ELFT, GLHT, ILJT, __________, MLNT
(a) CMN (b) UJI (c) VIJ (d) IJT (a) OLPT (b) LLMT (c) KLMT (d) KLLT

M05_MADAN 04_65901_C05.indd 39 23/12/22 7:25 PM


5.40 Chapter 5

27. Find the missing letters in the following series. 29. Find the missing letters in the following series.
WE, SG, PJ, LN, __________ QAR, RAS, SAT, TAU, __________
(a) IS (b) SI (c) PT (d) QT (a) UAV (b) UAT
28. Find the missing letters and numbers in the following (c) TAS (d) TAT
series. 30. Find the next pair in the following sequence.
C2A, E5D, G8G, I11J, __________ C − 3, E − 6, G − 12, I − 24, K − 48, __________
(a) K14M (b) M14K (a) S − 48 (b) M − 96
(c) L14M (d) M14L (c) L − 96 (d) O − 48

Coding and Decoding


31. If ‘CERTAIN’ is coded as ‘XVIGZRM’ in a particular (a) LACANDER (b) CRIUCALR
code language, how will ‘MUNDANE’ be coded in that (c) CLANADER (d) None of the above
language?  43. In a code sign, ‘DRLAL’ is coded as 62014314. How is
(a) NFMWZMX (b) VMZWMFN ‘CAMEL’ coded?
(c) NFMWZMV (d) MIMXZMV (a) 5315714 (b) 35729310
32. If ‘EDUCATION’ is coded as NOITACUDE, then ‘RED (c) 5313613 (d) None of the above
FORT’ will be coded as 44. If ‘LIGHT’ is coded as ‘GILTH’, then find the code for
(a) TROFDER (b) FORTRED ‘RAINY’.
(c) TROFRED (d) FORTDER (a) IARYN (b) ARINY
33. In a certain code, ‘MOTHER’ is written as ‘OMHURF’. (c) NAIRY (d) RINAY
How will ‘ANSWER’ be written in that code? 45. If all the letters in the word ‘ARGUMENT’ are rear-
(a) NBWRRF (b) MAVSPE ranged in alphabetical order and substituted by the
(c) NBWTRD (d) NAWTRF letter immediately following it in English alphabet,
34. In a certain code, ‘COMPUTER’ is written as then what will be the new arrangement of letters?
‘­RFUVQNPC’. How is ‘PRINTER’ written in the same (a) BFHNOSUV (b) BFHONSWV
code? (c) BFHNOUSV (d) BFHNOQUV
(a) R F U O J S P (b) P F U O J S R 46. Which of the following pairs have the same relation-
(c) P S J O U F P (d) R S J O U F P ship as OFTEN : FOTNE?
35. BF is related to DH in the same way as PS is related to (a) HEART : TRAHE (b) OPENS : SNEOP
(a) SU (b) SV (c) RV (d) RU (c) RISKY : IRSYK (d) FIRST : IFRST
36. CLAIM : DNDMR :: CHARGE : ? 47. If water is called food, food is called tree, tree is called
A S S E S S YO U R L E A R N I N G

(a) DJDVLK (b) DIDWLL sky, and sky is called wall, then on which of the fol-
(c) DJCVMK (d) DIDWKL lowing a fruit grows?
37. COLD : FSQJ :: HEAT : ? (a) Water (b) Food (c) Sky (d) Tree
(a) XJFY (b) KIGZ 48. If red means blue, blue means green, green means
(c) KIFZ (d) YIGY orange, orange means pink, and pink means black,
38. In a certain code, ‘PAPER’ is written as ‘SCTGW’. How then what is the colour of clear sky?
is ‘MOTHER’ written in that code? (a) Orange (b) Green
(a) POXJJT (b) ORVLGW (c) Blue (d) None of the above
(c) PQXKJV (d) PQVJGT 49. In a certain code, ‘bi nie pie’ means ‘some good jokes’,
39. If HE = 13 and MOVER = 73, then BASIC = ? ‘nie bat lik’ means ‘some real stories’, and ‘pie lik tol’
(a) 55 (b) 34 (c) 50 (d) 49 means ‘many good stories’. Which word in that code
means ‘jokes’?
40. In a certain code, 15789 is written as XTZAL and 2346
(a) bi (b) nie
is written as NPSU. How is 235491 written in that
(c) pie (d) None of the above
code?
(a) NPTSL (b) NPTUL 50. If ‘table’ is called ‘chair’, ‘chair’ is called ‘cupboard’,
(c) NBTSL (d) PNTSL ‘cupboard’ is called ‘chalk’, ‘chalk’ is called ‘book’,
‘book’ is called ‘duster’, and ‘duster’ is called ‘table’,
41. If A stands for 5, B for 6, C for 7, D for 8 and so on,
then what does the teacher use to write on the
what do the following numbers stand for: 22, 25, 8,
blackboard?
22 and 5? 
(a) Book (b) Cupboard
(a) PRIYA (b) NEEMA
(c) Table (d) Duster
(c) MEENA (d) RUDRA
51. In a certain code language,
42. If ‘CALENDAR’ is coded as ‘CLANAEDR’, then the code
(A) ‘pit dar na’ means ‘you are good’.
for CIRCULAR is
(B) ‘dar tok pa’ means ‘good and bad’.

M05_MADAN 04_65901_C05.indd 40 23/12/22 7:25 PM


Mathematical Reasoning and Aptitude 5.41

(C) ‘tim na tok’ means ‘they are bad’. 54. In a certain code language, ‘526’ means ‘sky is blue’,
In that language, which word stands for ‘they’? ‘24’ means ‘blue colour’, and ‘436’ means ‘colour is
(a) na (b) tok (c) tim (d) pit fun’. Which of the following digits in that language
52. In a certain code language, ‘743’ means ‘mangoes are means ‘fun’?
good’, ‘657’ means ‘eat good food’, and ‘934’ means (a) 5 (b) 4
‘mangoes are ripe’. Which of the following digit in (c) 3 (d) 2
that code means ‘ripe’? 55. In a certain code language, ‘123’ means ‘hot filtered
(a) 9 (b) 4 coffee’, ‘356’ means ‘very hot day’, and ‘589’ means
(c) 5 (d) 7 ‘day and night’. Which of the following digits in that
53. In a certain code, ‘256’ means ‘you are good’, ‘637’ language means ‘very’?
means ‘we are bad’, and ‘358’ means ‘good and bad’. (a) 9 (b) 5
Which of the following digits in that code means ‘and’? (c) 8 (d) 6
(a) 2 (b) 5
(c) 8 (d) 3

Choose Odd Word


56. Choose the word that is least like the other words in 61. Choose the word that is least like the other words in
the group. the group.
(a) Moon (b) Sun (a) Lion (b) Cheetah
(c) Universe (d) Planets (c) Bear (d) Tiger
57. Choose the word that is least like the other words in 62. Choose the word that is least like the other words in
the group. the group.
(a) Chemistry (b) Geography (a) Sheet (b) Cot
(c) Zoology (d) Botany (c) Spain (d) Pillow
58. Choose the word that is least like the other words in 63. Find the odd word among the following:
the group. (a) Kiwi (b) Eagle
(a) Mechanic (b) Mason (c) Penguin (d) Ostrich
(c) Blacksmith (d) Architect 64. Find the odd word among the following:
59. Choose the word that is least like the other words in (a) Lake (b) Sea
the group. (c) River (d) Pool
(a) Sister (b) Friend 65. Find the odd word among the following:

A S S E S S YO U R L E A R N I N G
(c) Brother (d) Father (a) Arrow (b) Axe
60. Choose the word that is least like the other words in (c) Knife (d) Dagger
the group.
(a) Zinc (b) Aluminium
(c) Copper (d) Mercury

Odd Pairs
66. Find the odd pair of words. 70. Find the odd pair of words.
(a) Mason : Wall (b) Cobbler : Shoe (a) Room : House (b) Atom : Electron
(c) Farmer : Crop (d) Chef : Cook (c) Car : Engine (d) Milk : Water
67. Find the odd pair of words. 71. Find the odd number.
(a) Bottle : Wine (b) Cup : Tea (a) 13 (b) 53 (c) 63 (d) 23
(c) Pitcher : Water (d) Racket : Shuttle 72. Find the odd number.
68. Find the odd pair of words. (a) 51 (b) 144 (c) 64 (d) 121
(a) Lion : Roar (b) Snake : Hiss 73. Find the odd number.
(c) Frog : Bleat (d) Bees : Hum (a) 15 (b) 21 (c) 24 (d) 28
69. Find the odd pair of words. 74. Find the odd number.
(a) Daring : Timid (a) 324 (b) 244 (c) 136 (d) 352
(b) Beautiful : Pretty
75. Find the odd number.
(c) Clarity : Ambiguity
(a) 25 (b) 27 (c) 125 (d) 343
(d) Youth : Adult

M05_MADAN 04_65901_C05.indd 41 23/12/22 7:25 PM


5.42 Chapter 5

Choose the Odd Number Pair or Group


76. Find the odd number pair. 81. Find the odd number pair.
(a) 95 : 82 (b) 69 : 56 (a) 23 : 29 (b) 19 : 25
(c) 55 : 42 (d) 48 : 34 (c) 13 : 17 (d) 3 : 5
77. Find the odd number pair. 82. Find the odd number pair.
(a) 2 : 8 (b) 3 : 27 (a) 343 : 7 (b) 243 : 9
(c) 4 : 32 (d) 5 : 125 (c) 512 : 8 (d) 216 : 6
78. Find the odd number pair. 83. Find the odd number pair.
(a) 80 : 9 (b) 64 : 8 (a) 13 : 21 (b) 19 : 27
(c) 36 : 6 (d) 7 : 49 (c) 15 : 23 (d) 16 : 24
79. Find the odd number pair. 84. Find the odd number pair.
(a) 3 : 5 (b) 5 : 3 (c) 6 : 2 (d) 7 : 3 (a) 14 : 56 (b) 12 : 36
80. Find the odd number pair. (c) 23 : 92 (d) 15 : 35
(a) 1 : 0 (b) 3 : 8 85. Find the odd number pair.
(c) 6 : 35 (d) 7 : 50 (a) 5 : 26 (b) 6 : 37
(c) 7 : 49 (d) 8 : 65

Choose the Odd Letter Group


86. Find the odd letter group. 91. Find the odd letter group.
(a) ACE (b) PRT (a) BYX (b) LPO
(c) UWY (d) MNO (c) EVU (d) FUT
87. Find the odd letter group. 92. Find the odd letter group.
(a) RTW (b) QOM (a) CHM (b) HMR
(c) IKG (d) BDF (c) DIN (d) LPU
88. Find the odd letter group. 93. Find the odd letter group.
(a) KOM (b) LPN (a) XUW (b) DAC
(c) BFD (d) GLI (c) PMN (d) HEG
89. Find the odd letter group. 94. Find the odd letter group.
(a) BHE (b) DJG (a) RAT (b) SAT
A S S E S S YO U R L E A R N I N G

(c) SYV (d) JPM (c) CAT (d) MAT


90. Find the odd letter group. 95. Find the odd letter group.
(a) BCD (b) MNO (a) OTP (b) ABA
(c) KLM (d) PQR (c) SZX (d) UVB

Analogy
It is important to note that the analogy questions also 99. As ‘Pen’ is related to ‘Ink’, ‘Needle’ is related to
cover many aspects of ‘relationships’. Relationships have (a) Thread (b) Cloth
been specifically mentioned in NET Paper 1 syllabus. (c) Stitching (d) Art
100. ‘Chef’ is related to ‘Kitchen’ in the same way as
Direct or Simple A nalogy ‘­Scientist’ is related to
96. As ‘Boxing’ is to ‘Ring’, ‘Tennis’ is to (a) Medicine (b) Pharmacy
(a) Pool (b) Court (c) Laboratory (d) Chemist
(c) Arena (d) Ground 101. ‘Engineer’ is related to ‘Production’ in the same way
97. ‘Doctor’ is related to ‘Patient’ in the same way as as ‘Doctor’ is related to
‘­Consultant’ is related to (a) Cure (b) Hospital
(a) Customer (b) Accused (c) Body (d) Surgery
(c) Magistrate (d) Client 102. As ‘Design’ is related to a ‘Structure’, ‘Rhythm’ is
98. ‘Easiness’ is related to ‘Difficulty’ in the same way as related to
‘Comfort’ is related to (a) Music (b) Art
(a) Hardship (b) Rest (c) Kathak (d) Dance
(c) Poverty (d) Difficulty

M05_MADAN 04_65901_C05.indd 42 23/12/22 7:25 PM


Mathematical Reasoning and Aptitude 5.43

103. ‘Drama’ is related to ‘Director’ in the same way as 112. Man : Biography :: Nation : ?
‘­Magazine’ is related to (a) Democracy (b) Constitution
(a) Story (b) Editor (c) Geography (d) History
(c) Reader (d) Printer 113. Victory : Encouragement :: Failure : ?
104. ‘Demonstrator’ is related to ‘Laboratory’ in the same (a) Bad (b) Defeat
way as ‘Leader’ is related to (c) Anger (d) Frustration
(a) Podium (b) Assembly 114. Dilatory : Expeditious :: Direct : ?
(c) Parliament (d) State (a) Tortuous (b) Circumlocutory
105. ‘Author’ is related to ‘Book’ in the same way as (c) Straight (d) Curved
‘­Choreographer’ is related to 115. Foundation : Edifice :: Constitution : ?
(a) Drama (b) Dance (a) Government (b) State
(c) Masque (d) Opera (c) Nation (d) Cabinet
106. ‘Starvation’ is related to ‘Nutrition’ in the same way 116. Taxonomy : Classification :: Pedology : ?
as ‘Exhaustion’ is related to (a) Nature (b) Farming
(a) Energy (b) Bravery (c) Soil (d) Mountain
(c) Freshness (d) Courage
117. Monotony : Variety :: Crudeness : ?
107. ‘Science’ is related to ‘Laboratory’ in the same way as (a) Refinement (b) Raw
‘Astronomy’ is related to (c) Sobriety (d) Simplicity
(a) Observatory (b) Telescope
118. Interview : Job :: Armistice : ?
(c) Space (d) Station
(a) War (b) Treaty
108. ‘Bibliophile’ is related to ‘Books’ in the same way as (c) Amnesty (d) Agreement
‘Patriot’ is related to
119. Wizard : Witch :: Monk : ?
(a) Defence (b) Country
(a) Madam (b) Widow
(c) Mankind (d) Society
(c) Nun (d) Virgin
109. ‘Ancestor’ is related to ‘Descendant’ in the same way
120. Burglar : House :: Pirate : ?
as ‘Beautiful’ is related to
(a) Sea (b) Ship
(a) Gloomy (b) Handsome
(c) Sailor (d) Crew
(c) Girl (d) Ugly
121. Bird : Chirp :: Horse : ?
110. ‘Bee’ is related to ‘Hive’ in the same way as ‘Rat’ is
(a) Bray (b) Neigh
related to
(c) Race (d) Stable
(a) Burrow (b) Nest
(c) Hole (d) Stab 122. Insert : Extract :: Mighty : ?

A S S E S S YO U R L E A R N I N G
(a) Thin (b) Strong
Completing the Analogous Pair (c) Frail (d) Feeble
In each of the following questions, there is a certain rela- 123. Ornithologist : Birds :: Anthropologist : ?
tionship between two given words. On the left side of ::, (a) Plants (b) Animals
one word is given, whereas on the right side of ::, another (c) Mankind (d) Environment
word is to be found from the given alternatives having the 124. Oxygen : Burn :: Carbon dioxide : ?
same relation with this word as the words of the given (a) Isolate (b) Foam
pair. Now choose the correct alternatives for the questions (c) Extinguish (d) Explode
given below.
125. Seismograph : Earthquake :: Tachometer: ?
111. Terrible : Serene :: Roof : ? (a) Volcanoes (b) Resistance
(a) Door (b) Floor (c) Landslides (d) Strains
(c) Walls (d) Pillars

Blood Relations
126. If F is the brother of A, C is the daughter of A, K is the 128. A has three children. B is the brother of C, and C is
­sister of F, and G is the brother of C, then who is the the sister of D; E, who is the wife of A, is the mother
uncle of G? of D. There is only one daughter of the husband of E.
(a) F (b) K What is the relation between D and B?
(c) C (d) None of the above (a) B is brother of D
127. Ravi said to Seeta, ‘Your mother is the daughter of (b) B is not related to D
my grandmother.’ How are Ravi and Seeta related? (c) D is sister of B
(a) Uncle–niece (b) Father–daughter (d) None of the above
(c) Cousin (d) None of the above

M05_MADAN 04_65901_C05.indd 43 23/12/22 7:25 PM


5.44 Chapter 5

129. Dinesh said to Naveen, ‘The person playing this game (a) Uncle (b) Brother
is the youngest of the two brothers of the daughter (c) Cousin (d) None of the above
of my father’s wife.’ How is the person playing the 133. Vinita, who is the sister-in-law of Amit, is the
game related to Dinesh? ­daughter-in-law of Kamni. Deepak is the father of
(a) Cousin (b) Brother Sandy who is the only brother of Amit. How is Kaly-
(c) Son (d) Brother-in-law ani related to Ashok?
130. Pointing to a picture of a boy, Somesh said, ‘He is the (a) Mother-in-law (b) Aunt
son of the only son of my mother.’ How is Somesh (c) Wife (d) None of the above
related to that boy? 134. Pointing to a woman in a picture, Amit said, ‘Her
(a) Brother (b) Uncle granddaughter is the only daughter of my brother.’
(c) Cousin (d) Father How is the woman related to Amit?
131. Introducing a woman, Namrata said, ‘She is the (a) Sister (b) Grandmother
­daughter-in-law of the grandmother of my father’s (c) Mother-in-law (d) Mother
only son.’ How is the woman related to Namrata? 135. X and Y are siblings. C and D are wife and husband,
(a) Grandmother (b) Sister-in-law respectively; X is the only son of C. F is the sister of
(c) Sister (d) Mother D. How is Y related to F?
132. Pointing to a photograph Raveena says, ‘He is the (a) Niece (b) Nephew
son of the only son of my grandfather.’ How is the (c) Uncle (d) Cousin
man in the photograph related to Raveena?

Direction Sense
136. Raman travels a distance of 5 km in the south direc- 141. Kumar walked 5 m towards the north, took a left
tion. He turns to his right. After walking 3 km, he turn and walked for 10 m. He then took a right turn
turns to the left and walks 5 km. Now in which direc- and walked for 20 m, and again took a right turn and
tion is he from the starting place? walked 10 m. How far is he from the starting point?
(a) West (b) South (a) 20 m   (b) 15 m  (c) 25 m  (d) 30 m
(c) South-west (d) North-east
142. Shalini walked 15 m towards the south, took a right
137. Ravi left home and cycled 10 km towards the south. turn and walked 3 m. She took a right turn again and
He, then, turned right and cycled 5 km, and then walked 15 m before s­ topping. Which direction did
again turned right and cycled 10 km. After this, he Shalini face after ­stopping?
turned left and cycled 10 km. How many kilometres
(a) West (b) South
A S S E S S YO U R L E A R N I N G

will he have to cycle to reach his home straight?


(a) 15 (b) 20 (c) 25 (d) 30 (c) East (d) North
138. Chintan walked 15 km towards the west, and then, 143. Town D is 12 km towards the north of town A. Town
he turned left and walked for 20 km. Then, he moved C is 15 km towards the west of town D. Town B is
towards the east and walked 25 km, and finally, turn- 15 km towards the west of town A. How far and in
ing left, he covered 20 km. How far was he from his which direction is town B from town C?
starting point? (a) 15 km towards the north
(a) 20 km (b) 30 km (b) 12 km towards the south
(c) 10 km (d) None of the above (c) 3 km towards the south
139. Vijay starts walking towards the north. After walking (d) 12 km towards the south
some distance, he turns to his right. Then, walking 144. Read the following carefully and answer the ques-
for a while, he turns to his left and walks for 1 km, tion given below.
and then, he turns to his left again. In which direc- 1. Point B is 4  m towards the north of Point A.
tion is he moving now? 2. Point E is 8  m towards the east of Point B.
(a) North (b) South
3. Point C is 5  m towards the east of Point A.
(c) West (d) East
4. Point D is 9 m towards the west of Point C.
140. Rashi travels 20 km towards the north. Then, she
turns right and travels 30 km. Then, she turns right How far should one walk from Point A in order
and travels a distance of 35 km. Then, she turns left to reach Point D?
and walks 15 km. Finally, she turns left and walks 15 (a) 4 m (b) 9 m
km. In which direction and how many kilo metres is (c) 5 m (d) 14 m
she from the starting position? 145. Rahul started from Point A and travelled 8 km
(a) 5 km west (b) 30 km east towards the north to Point B; he then turned right
(c) 30 km west (d) 45 km east and travelled 7 km to Point C; and from Point C, he

M05_MADAN 04_65901_C05.indd 44 23/12/22 7:25 PM


Mathematical Reasoning and Aptitude 5.45

took the first right and drove 5 km to Point D. Then, 148. P, Q, R, S, T, U, V and W are sitting around a circle
he took another right and travelled 7 km to Point E, and are facing the centre.
and finally, he turned right and travelled for another 1. P is second to the right of T who is the neighbour
3 km to Point F. What is the distance between Points of R and V.
F and B? 2. S is not the neighbour of P.
(a) 1 km  (b) 2 km  (c) 3 km  (d) 4 km 3. V is the neighbour of U.
4. Q is not between S and W. W is not between U
Seating Arrangement and S.
146. A, P, R, X, S and Z are sitting in a row. S and Z are in Which two of the following are not neighbours?
the centre. A and P are at the ends. R is sitting to the (a) R and V (b) U and V
left of A. Who is to the right of P? (c) R and P (d) Q and W
(a) S (b) X 149. J, D L, H and F are travelling to a station; each one
(c) Z (d) None of the above reaches at a different time. L reaches only after J and
147. A, B, C, D and E are sitting on a bench. A is sitting D reaches only before F. Who among them is third to
next to B; C is sitting next to D, and D is not sitting reach?
with E who is on the left end of the bench. C is on the (a) F (b) L (c) D (d) H
second position from the right. A is to the right of B 150. P, Q, R, S and T are sitting in a straight line facing
and E. A and C are sitting together. In which position north. P sits next to S but not to T. Q is sitting next to
is A sitting? R, who sits on the extreme left corner. T does not sit
(a) Between B and D next to Q. Who sits to the left of S?
(b) Between B and C (a) P (b) Q (c) R (d) S
(c) Between E and D
(d) Between C and E

fractions
1
151. Which of the following has the largest value? 156. Ramu finishes part of a work in 1 hour. How much
3
32 0.320 1
(a) (b) part of the work will he finish in 2 hours?
0.05 50 5
13 11 7 9
3.2 3.2 (a) (b) (c) (d)
(c) (d) 14 15 15 14
0.05 50

A S S E S S YO U R L E A R N I N G
2 6 13
157. The ascending arrangements of , , is
3 3 7 21
152. of 3245 + 32% of 6250 − (?) = 103
2

5
6 2 13 13 2 6
(a) 58 (b) 60 (c) 62 (d) 65 (a) , , (b) , ,
7 3 21 21 3 7
2 6 13 2 2 6 13
153. 13 % of 3300 + 25% of 184 = 40% of ? , , , ,
3 (c) (d)
7 21 3 3 7 21
(a) 1242.5 (b) 1140.8
1
(c) 1325.3 (d) 1018.2 158. In a class of 40 students, of the total number of
5
154. The largest of the following is 2
students like to eat rice only, of the total number
1 5
(a) 0.0001 (b)
1000 of students like to eat chapati only, and the remain-
1 ing students like to eat both. What fraction of the
(c) (0.100)2 (d) ÷ 0.1 total number of students like to eat both?
10
(a) 8 (b) 16
(c) 20 (d) 24
1 1 1
155. Find the value of + + 159. Three planets take one revolution around the Sun
2 11 ⎛ 5 ⎞
4 3 ⎜ ⎟ in 200, 250 and 300 days, respectively, in their own
7 13 ⎝ 9 ⎠ orbits. When do they all come relatively to the same
(a) 172/75 (b) 164/75 position as at a certain point of time in their orbits?
(c) 181/90 (d) 175/77 (a) After 2800 days (b) After 3000 days
(c) After 3050 days (d) After 3200 days

M05_MADAN 04_65901_C05.indd 45 23/12/22 7:25 PM


5.46 Chapter 5

1 7 1
160. In the morning, a milkman filled 5 l of milk in his to Shadma he sold l; and to Jassi he gave 1 l.
2 3 8 2
can. He sold to Renu, Kamala and Renuka l each;
4 How much milk is left in the can?
7 5 3 6
(a) (b) (c) (d)
81 81 41 111

time and Distance


161. A person crosses a 1200 m long street in 10 minutes. 166. A man completes a journey in 10 hours. The first half
What is his speed in kmph? of the journey is covered at the speed of 21 km/h
(a) 7.2 kmph (b) 8 kmph and the second half at 24 km/h. The total distance
(c) 9 kmph (d) None of the above covered during the journey is
162. A person completes a journey of 48 km in 2 hours. (a) 220 km (b) 224 km
How much time will he take to cover a distance of (c) 230 km (d) 234 km
252 km? 167. A man on tour travels first 320 km at 64 kmph and
(a) 10 hours (b) 11 hours the rest at 80 kmph. The average speed for the entire
(c) 10½ hours (d) None of the above 640 km of the tour is
163. A train completed half a trip at 30 miles/h and the (a) 35.55 kmph (b) 36 kmph
other half at 60 miles/h. If the whole trip was of (c) 71.11 kmph (d) 71 kmph
20 miles, then how much time did the train take to 168. A person travelled a distance of 610 km in 9 hours.
complete the trip? He travelled the first phase at a speed of 40 kmph
(a) 90 minutes (b) 60 minutes and the rest at 90 kmph. The distance travelled dur-
(c) 45 minutes (d) 30 minutes ing the first phase is
164. A person performs half of his journey by train, one- (a) 140 km (b) 150 km
third by bus and the rest 5 km by auto rickshaw. Find (c) 160 km (d) 170 km
his total journey. 169. A train whose length is 320 m is running at a speed of
(a) 30 (b) 36 36 kmph. How much time will it take to pass a pole?
(c) 40 (d) 45 (a) 30 seconds (b) 32 seconds
165. Excluding stoppages, the speed of a bus is 54 kmph, (c) 36 seconds (d) 40 seconds
and including stoppages, it is 45 kmph. For how 170. A 280 m long train is moving at a speed of 80 kmph.
many minutes does the bus stop per hour? How much time will it take to pass a bridge that is
A S S E S S YO U R L E A R N I N G

(a) 9 (b) 10 120 m long?


(c) 12 (d) 20 (a) 30 seconds (b) 32 seconds
(c) 36 seconds (d) 40 seconds

Ratio, Proportion, Percentage and Discounting


1
171. The price of cooking oil has increased by 25%. The 175. A man spends `3500 and saves 12 % of his income.
percentage of reduction that a family should effect 2
His monthly income (in `) is
in the use of cooking oil so as not to increase the (a) 3937.50 (b) 4000
expenditure on this account is (c) 4250 (d) 4160
(a) 25% (b) 30% 176. The price of an item increased by 20% and then
(c) 20% (d) 15% decreased by 20%. The final price as compared to
172. In an organization, 40% of the employees are matric- the original price is
ulates, 50% of the remaining employees are gradu- (a) 20% less (b) 20% more
ates, and the remaining 180 are postgraduates. How (c) 4% more (d) 4% less
many employees are graduates? 177. X’s salary is half that of Y. If X got a 50% rise in his
(a) 360 (b) 240 ­salary and Y got a 25% rise in his salary, then the
(c) 300 (d) 180 percentage increase in combined salaries of both is
173. In a town, 96% of the population is 23,040. The total (a) 30% (b) 33.33%
population of the town is (c) 37.5% (d) 75%
(a) 32,256 (b) 24,000 178. A mixture of 40 l of milk and water contains 10%
(c) 24,936 (d) 25,640 water. How much water should be added to it so that
174. If 75% of the students in a school are boys and the water may be 20% in the new mixture?
number of girls is 420, then the number of boys is (a) 5 l (b) 4 l
(a) 1176  (b) 1350  (c) 1260  (d) 1125 (c) 6.5 l (d) 7.5 l

M05_MADAN 04_65901_C05.indd 46 23/12/22 7:25 PM


Mathematical Reasoning and Aptitude 5.47

179. A man spends 75% of his income. His income is 185. If the price of potatoes increased first by 20% and
increased by 20% and he increased his expenditure subsequently by 40%, then what is the final price per
by 10%. His savings increased by ­kilogram if the original price was `25 per kg?
(a) 10% (b) 25% (a) `40 (b) `42
(c) 37.5% (d) 50% (c) `45 (d) `48
180. If A’s income is 25% more than B’s, and B’s income 186. The radius of a circle has increased by 20%. By what
is 20% more than C’s, then by what per cent is A’s ­percentage does the circumference increase?
income more than C’s? (a) 20% (b) 40%
(a) 15% (b) 25% (c) 44% (d) 48%
(c) 33.5% (d) 50% 187. The radius of a circle has increased by 20%. By what
181. If the price of a television set is increased by 25%, ­percentage does its area increase?
then by what percentage should the new price be (a) 33% (b) 44%
reduced to bring the price back to the original level? (c) 55% (d) None of the above
(a) 15% (b) 20% 188. If the side of a square is increased by 25%, then its
(c) 25% (d) 30% area is increased by how much per cent?
182. A dealer marks his goods 20% above the cost price. (a) 25% (b) 50%
He then allows some discount on it and makes a (c) 62.5% (d) None of the above
profit of 8%. The rate of discount is 189. The length and breadth of a rectangle are 20 and
(a) 4% (b) 6% 10 cm, respectively. The length is increased by 10%
(c) 10% (d) 12% and the breadth is increased by 20%. What is the
183. As the price of mangoes decreased by 25%, I can new area of the rectangle?
­purchase four mangoes more for `60. What is the (a) 240 (b) 254
new price of one mango? (c) 264 (d) 280
(a) `5 (b) `4 190. The number of seats in an auditorium is increased by
(c) `3.75 (d) None of the above 25%. The price on a ticket is also increased by 12%.
184. If the length of a rectangle increases by 10%, then by What is the effect on the revenue collected?
what per cent should the breadth decrease to main- (a) 37 (b) 40
tain the same area? (c) 42 (d) None of the above
(a) 10 (b) 20
(c) 9.11 (d) 5

profit and loss

A S S E S S YO U R L E A R N I N G
191. The cost price of an article is `4800. It is to be sold 195. Arvind deals in carpets. He allows 4% discount on
at a profit of 6.25%. How much should be its selling the marked price. What price must be marked on a
price? carpet that cost `480 so as to make a profit of 10%?
(a) `5100 (b) `5150 (a) `528 (b) `550
(c) `5400 (d) `5500 (c) `580 (d) `600
192. A trader sells an article at a profit of 20%. Had he 196. A readymade producer manufactures both men’s
bought it at 10% more and sold it for `700 more, and women’s garments. The average profit is 5% of
he would have earned a profit of 25%. Find the cost the total sales. The profit on men’s garments is on an
price of the goods. average of 9% of the sales. The women’s garments
(a) `5000 (b) `4000 are 60% of the total output. The average profit per-
(c) `3800 (d) None of the above centage on women garments is
193. The cost of an article is `5000. The marked price is (a) 5% (b) 9%
`8000. What is the profit percentage for the seller if (c) 2.33% (d) 3.4%
he sells and offers a discount of 10% on the marked 197. The cost of an article is `500 and its original price was
price? marked up 40% by the shopkeeper. Then shopkeeper
(a) 36% (b) 38% sets a new price; he marks the cost price up by 80%
(c) 40% (d) 44% and then gives a discount of 20% to the customer.
194. The cost price of 100 pencils is equal to the selling How much more/less money will the shopkeeper
price of 60 pencils. The gain percentage or loss per- make now?
centage is (a) More price of `20
(a) 33 1/3% (b) 66 2/3% (b) Less price of `20
(c) 50% (d) 45% (c) More price of `80
(d) No extra profit

M05_MADAN 04_65901_C05.indd 47 23/12/22 7:25 PM


5.48 Chapter 5

198. The cost of an article reduces by 20%. This enables (a) 61,500 (b) 62,500
a person to buy 2 kg more articles for `300. Find the (c) 60,000 (d) 58,000
reduced and the original price per kg of rice. 200. After selling a cycle for `1680, a shopkeeper suffers
(a) `60 and `80 (b) `60 and `75 a loss of 16%. If he wants to earn 15% profit after
(c) `75 and `60 (d) `50 and `60 giving the discount of 8%, what will be the marked
199. Arjun makes a profit of 18% on cost price by selling a price?
bike for `59,000. The cost price of the bike increases (a) `2200 (b) `2450
by 5%. Arjun wants to make the same amount of (c) `2500 (d) `2600
profit. What will be the new profit percent on selling
price in rupees?

Interest
201. Yugpurush paid an interest of `11,400 after 9 years. was same as the second part which he invested at
For the first 2 years, the simple interest was 6%; for 20% for 3 years.
the next 3 years, it was 9%; and for the next 4 years, it (a) `0 (b) `150
was 14%. How much principal money was borrowed (c) `280 (d) `320
by him?
(a) `12,000 (b) `11,400 206. Rajinder divided `1301 among his sons Santa and
(c) `11,000 (d) `10,000 Banta. He asked them to invest the money at annual
4% compound rate of interest. It was observed that
202. Some amount becomes `900 in 3 years if calcu- each of his sons gets the same amount after 17 and
lated on compound interest, and the same amount 19 years, respectively. How money was given to
becomes `1000 in the fourth year. What was the Banta?
principal amount and rate of interest? (a) `705 (b) `615
(a) `695.80, 11.11% (c) `676 (d) `625
(b) `625, 12.50%
(c) `656.10, 11.11% 207. Nagarjuna invested some money at compound rate
(d) `686, 12.50% of interest. His amount gets doubled in 9 years. In
how many years will the amount become four times
203. `2000 is invested each on simple interest and com- itself?
pound interest. What will be the difference between (a) 13.5 years (b) 27 years
their amounts of simple interest and compound (c) 9 years (d) 18 years
interest if the rate of interest on each investment is
208. If at same rate of interest, in 2 years the simple inter-
A S S E S S YO U R L E A R N I N G

10% per annum and the time span is 3 years?


(a) `56 (b) `62 est is `40 and the compound interest is `41, then
what is the rate of interest?
(c) `70 (d) `76
(a) 4% (b) 5%
204. Manish gets `26,000 by making an investment of (c) 8% (d) None of the above
`20,000 in 5 years at some simple rate of interest.
209. If a sum of money invested at compound interest
Had the investment been made at another place
where rate of interest is 3% higher, how much becomes `800 in 3 years and `840 in 4 years, then
amount Manish would have made during the same what is the rate of interest per annum?
duration? (a) 5% (b) 6%
(a) `29,000 (b) `30,000 (c) 7% (d) None of the above
(c) `31,000 (d) `28,000 210. An amount if given on interest becomes four times
205. Navjot gets a salary of `8600. The salary was invested in 6 years. In how much time span will it become 64
by him in two parts. Find the difference between the times if the rate of interest remains unchanged?
two parts of his salary if in the first part he got some (a) 8 years (b) 16 year
simple interest at 15% per annum in 4 years, which (c) 18 years (d) 24 years

Averages
211. The average weight of eight persons increases by 2.5 212. The average weight of eight articles is 25 kg. If the
kg when a person weighing 60 kg is replaced by a average weight of three of them is 20 kg, then what
new ­person. What might be the weight of the new is the average weight of rest of the five articles?
person? (a) 26 kg (b) 27 kg
(a) 76 kg (b) 76.5 kg (c) 28 kg (d) None of the above
(c) 80 kg (d) Data inadequate

M05_MADAN 04_65901_C05.indd 48 23/12/22 7:25 PM


Mathematical Reasoning and Aptitude 5.49

213. The average speed of a taxi car from Town A to Town of 40 kmph. B starts at 9 a.m. and drives at a speed
B is 35 kmph and that during the return journey is of 50 kmph. If the destination is 350 km away from
42 kmph. What is the average speed of the whole the starting point, then how far will A be from the
journey? destination when B reaches there? 
(a) 35.5 kmph (b) 36 kmph (a) 20 km (b) 30 km
(c) 38.17 kmph (d) Cannot be determined (c) 50 km (d) 70 km
214. A car owner buys petrol at `8 and `10 for 2 succes- 217. A train runs for 2 hours at a speed of 35 kmph. It runs
sive years. What approximately is the average cost for 3½ hours at the speed of 60 kmph and then runs
per litre of petrol if he spends `4000 each year? for 2½ hours at the speed of 70 kmph. Find the aver-
(a) `9 (b) `9.90 age speed of the train. 
(c) `8.88 (d) None of the above (a) 50 kmph
215. A question is followed by two statements I and II (b) 55 kmph
that contain certain data. Tell by selecting one of the (c) 80 kmph
answer choices given whether the data given in the (d) 56.87 kmph
statements are sufficient for answering the question. 218. The average age of a husband and wife was 22 years
(a) If statement I alone is sufficient when they were married 5 years back. What is the
(b) If statement II alone is sufficient present average age of the family if they have a
(c) If both statements I and II taken together are 3-year-old child? 
sufficient (a) 19 years (b) 25 years
(d) If none of the statements is sufficient (c) 27 years (d) 28.5 years
Question: A horse ran 100 miles without stop- 219. A man’s monthly income is `1400. What should be
ping. What was the average speed in miles per his average monthly expenditure so that he is able to
hour? save `3600 in a year?
Statements (a) `1000 (b) `1100
I. The horse ran 20 miles/h for the first 50 miles. (c) `1150 (d) `1200
II. The entire journey starts from 8 p.m. on day 1 to
220. The average of A, B and C is 50. If D is 10, then what
4 a.m. the following day.
is the average of A, B, C and D?
216. A and B start from the same destination and take the (a) 15 (b) 40
same route. A starts at 8 a.m. and drives at a speed (c) 30 (d) 60

Calendar

A S S E S S YO U R L E A R N I N G
221. The year next to 1991 will have the same calendar as 226. If it is Sunday today, then what will be the day after
that of the year 1991. 60 days?
(a) 1992 (b) 1995 (a) Sunday (b) Thursday
(c) 1996 (d) 1997 (c) Tuesday (d) Friday
222. What day of the week was 28 May 2007? 227. If 22 April 2013 is Monday, then what was the day of
(a) Thursday (b) Friday the week on 22 April 2012?
(c) Saturday (d) Monday (a) Sunday (b) Saturday
223. What was the day of the week on 16 June 1999? (c) Tuesday (d) Wednesday
(a) Monday (b) Tuesday 228. What was the day of week on 1 April 2001?
(c) Wednesday (d) Thursday (a) Sunday (b) Saturday
224. What will be the day of the week on 16 August 2010? (c) Tuesday (d) Friday
(a) Sunday (b) Monday 229. The last day of a century cannot be
(c) Tuesday (d) Friday (a) Monday (b) Wednesday
225. It was Wednesday on 15 August 2012. What should (c) Tuesday (d) Friday
be the day on 15 November 2013? 230. 10 February 2005 was Thursday. What was the day
(a) Wednesday of the week on 8 February 2004?
(b) Thursday (a) Tuesday (b) Monday
(c) Friday (c) Sunday (d) Wednesday
(d) None of the above

M05_MADAN 04_65901_C05.indd 49 23/12/22 7:25 PM


5.50 Chapter 5

Venn Diagrams
231. Out of 40 students, 14 are taking English composi- 238. Which of the following diagrams indicates the best
tion and 29 are taking Chemistry. If five students are relation between teacher, parents and guardians?
in both classes, then how many students are in nei-
ther class? How many are in either class? (a) (b)
(a) 2 (b) 3
(c) 4 (d) 5
232. How many numbers are there between 1 and 100 (c) (d)
that are not divisible by 3 and 5?
(a) 47 (b) 43 239. Which of the following diagrams indicates the best
(c) 53 (d) 58 relation between professors, doctors and men?
233. In an examination, 52% of the candidates failed
in ­English, 42% in Mathematics and 17% in both.
(a) (b)
The number of those who passed in both the sub-
jects is
(a) 83% (b) 23%
(c) 64% (d) 55.5% (c) (d)
234. In a group of 40 people, 25 speak English and 20
speak both Hindi and English. All the people speak 240. Which of the following diagrams indicates the best
at least one of the two languages. How many people relation between mercury, zinc and metal?
speak Hindi?
(a) 15 (b) 20 (a) (b)
(c) 25 (d) 30
235. If 40% of the people read newspaper X, 50% read (c) (d)
­newspaper Y, and 10% read both the papers, then
what percentage of people read neither of the
newspaper? Questions 241−245: Study the diagram given below and
(a) 10% (b) 15% answer each of the following questions.
(c) 20% (d) 25%
236. Which of the following diagrams indicates the best
25 4 17 Artists
relation between a teacher, a writer and an artist?
A S S E S S YO U R L E A R N I N G

22 3
8 Players
(a) (b)
30 Doctors

(c) (d)
241. How many doctors are neither artists nor players?
237. Which of the following diagrams indicates the best (a) 17 (b) 5 (c) 10 (d) 30
relation between travellers, train and taxi? 242. How many doctors are both players and artists?
(a) 22 (b) 8 (c) 3 (d) 30
(a) (b) 243. How many artists are players?
(a) 5 (b) 8 (c) 25 (d) 16
244. How many players are neither artists nor doctors?
(c) (d) (a) 25 (b) 17 (c) 5 (d) 10
245. How many artists are neither players nor doctors?
(a) 10 (b) 17 (c) 30 (d) 15

M05_MADAN 04_65901_C05.indd 50 23/12/22 7:25 PM


Mathematical Reasoning and Aptitude 5.51

Answer Keys
Number Series Completion
1. (c) 2. (a) 3. (c) 4. (b) 5. (c) 6. (b) 7. (d) 8. (b) 9. (a) 10. (d)
11. (d) 12. (c) 13. (b) 14. (a) 15. (b) 16. (d) 17. (a) 18. (c) 19. (d) 20. (d)
Letter Series Completion
21. (a) 22. (a ) 23. (c) 24. (b) 25. (b) 26. (d) 27. (a) 28. (a) 29. (a) 30. (b)
Coding and Decoding
31. (c) 32. (a) 33. (d) 34. (a) 35. (d) 36. (a) 37. (c) 38. (c) 39. (b) 40. (a)
41. (d) 42. (b) 43. (a) 44. (a) 45. (a) 46. (c) 47. (c) 48. (b) 49. (a) 50. (a)
51. (c) 52. (a) 53. (c) 54. (c) 55. (d)
Choose Odd Word
56. (c) 57. (b) 58. (a) 59. (b) 60. (d) 61. (c) 62. (b) 63. (b) 64. (c) 65. (a)
Odd Pairs
66. (d) 67. (d) 68. (a) 69. (b) 70. (a) 71. (c) 72. (a) 73. (d) 74. (a) 75. (a)
Choose the Odd Number Pair or Group
76. (d) 77. (c) 78. (a) 79. (d) 80. (d) 81. (b) 82. (b) 83. (d) 84. (d) 85. (c)
Choose the Odd Letter Group
86. (d) 87. (a) 88. (d) 89. (d) 90. (b) 91. (b) 92. (d) 93. (c) 94. (d) 95. (b)

Analogy
Direct or Simple Analogy
96. (b) 97. (d) 98. (a) 99. (a) 100. (c) 101. (a) 102. (a) 103. (b) 104. (a) 105. (b)
106. (a) 107. (a) 108. (b) 109. (d) 110. (d)
Completing the Analogous Pair

A S S E S S YO U R L E A R N I N G
111. (b) 112. (d) 113. (d) 114. (b) 115. (c) 116. (c) 117. (a) 118. (b) 119. (c) 120. (b)
121. (b) 122. (d) 123. (b) 124. (c) 125. (d)
Blood Relations
126. (a) 127. (c) 128. (a) 129. (b) 130. (d) 131. (d) 132. (b) 133. (d) 134. (d) 135. (a)
Direction Sense
136. (c) 137. (a) 138. (c) 139. (c) 140. (d) 141. (c) 142. (d) 143. (d) 144. (a) 145. (b)
Seating Arrangement
146. (b) 147. (b) 148. (a) 149. (d) 150. (a)
Fractions
151. (a) 152. (c) 153. (a) 154. (d) 155. (a) 156. (b) 157. (b) 158. (d) 159. (b) 160. (a)
Time and Distance
161. (a) 162. (c) 163. (d) 164. (a) 165. (b) 166. (b) 167. (c) 168. (c) 169. (b) 170. (c)
Ratio, Proportion, Percentage and Discounting
171. (c) 172. (d) 173. (b) 174. (c) 175. (b) 176. (d) 177. (b) 178. (a) 179. (d) 180. (d)
181. (b) 182. (c) 183. (c) 184. (c) 185. (b) 186. (a) 187. (b) 188. (d) 189. (c) 190. (b)

M05_MADAN 04_65901_C05.indd 51 23/12/22 7:25 PM


5.52 Chapter 5

Profit and Loss


191. (a) 192. (b) 193. (d) 194. (b) 195. (b) 196. (c) 197. (a) 198. (b) 199. (a) 200. (c)
Interest
201. (a) 202. (c) 203. (b) 204. (a) 205. (a) 206. (d) 207. (d) 208. (b) 209. (a) 210. (c)
Averages
211. (c) 212. (c) 213. (c) 214. (c) 215. (b) 216. (b) 217. (d) 218. (a) 219. (b) 220. (b)
Calendar
221. (d) 222. (d) 223. (c) 224. (b) 225. (c) 226. (b) 227. (a) 228. (a) 229. (c) 230. (c)
Venn Diagrams
231. (a) 232. (c) 233. (b) 234. (a) 235. (c) 236. (a) 237. (d) 238. (b) 239. (c) 240. (b)
241. (a) 242. (c) 243. (c) 244. (a) 245. (c)
A S S E S S YO U R L E A R N I N G

M05_MADAN 04_65901_C05.indd 52 23/12/22 7:25 PM


Mathematical Reasoning and Aptitude 5.53

Solution
N umber Series Completion 16. (d): 10 × 2 + 4 = 24;
24 × 2 + 4 = 52;
1.(c): Each number is half of its preceding number. 52 × 2 + 4 = 108;
2.(a): +5, +10, +15, +20, +25, +30. and so on.
3.(c): +3, +6, +9, +12. 17. (a): The given series consists of cubes of natural num-
bers only. The number 256 is not the cube of any natu-
4.(b): In first row, it is −2; in second row, it is +2, and in
ral number.
third row, it is +2.
18. (c): The given sequence is a combination of two
5. (c): Multiply the first by 3 and divide the number so
series:
obtained by 2. This is to be done alternatively for all
First series: 13, 24, 35, 46, 57
other numbers.
Second series: 32, 43, ?, 65, 76
6. (b): ×10 − 100, ×10 − 100 and so on. The pattern in both series is +11. Hence, the miss-
7. (d): The pattern is +5, −2, +5, −2, …. Hence, the ing term = 43 + 11 = 54.
missing term = 36 − 2 = 34. 19. (d): 23 + 10 = 33, 33 + 13 = 46, 46 + 16 = 62 and so on.
8. (b): The sum of any three consecutive terms of the 20. (d): 10 × 3 − 4 = 26
series is given to the next term. Hence, missing num- 26 × 3 − 4 = 74
ber = 8 + 15 + 27 = 50. 74 × 3 − 4 = 218
9. (a): The second number is the square of the first num- 218 × 3 − 4 = 650
ber; the fourth number is the square of the third num- 650 × 3 − 4 = 1946
ber. Similarly, the sixth number should be the square
of the fifth number. Hence, the answer is 16, that is, L etter Series Completion
square root of 256. 21. (a): The third letter is repeated as the first letter of
10. (d): The numerator increases by addition of 2, 3, 4 and the next segment. The middle letter, A, remains static.
5. The denominator increases by addition of 4, 6, 8 The third letters are in alphabetical order beginning
and 10. with R.
11. (d): Prime numbers (numbers that are divisible by 1 22. (a): The middle letter T is fixed. The first and third let-
and themselves only) starting with 2 are being added ters are swapping their position in second and fourth
to each number to get the subsequent numbers. terms. The first letters are in alphabetical order: F, G, H,
Thus, the sequence of numbers can be read as 125, I and J. The missing segment begins with a new letter J.
125 + 2, 127 + 3, 130 + 5, 135 + 7, 142 + 11 and 153 + 23. (c): For first letter of each triplet, the series is STUVW.

A S S E S S YO U R L E A R N I N G
13. Hence, the last number should be 166. The remaining two letters of the series go like CD, EF,
12. (c): Each term in the series is obtained by adding 1 to GH, IJ, KL.
the square of the preceding term. 24. (b): This is an alternating series in alphabetical order.
Hence, the missing term = (101)2 + 1 = 10,202. The middle letters follow the order ABCDE. The first
13. (b): It is a question of mixed series. and third letters of the triplets are in alphabetical
(i) Numbers at 1st, 3rd, 5th, and 7th places are order beginning with J. The third letter is repeated as
squares of 1, 2, 3 and 4. a first letter in each subsequent three-letter segment.
(ii) Numbers at 2nd, 4th, 6th, and 8th places are 25. (b): If we look at the alphabetic series given in Table
cubes of 1, 2, 3, 4 and 5. Hence, the answer is 43 5.1, the difference between first letters of all pairs is 3.
= 64. Moreover, the difference between two letters in each
14. (a): The pattern is: pair is 10. Hence, MW is the answer.
3 × 1 + 2 = 5 26. (d): The second and fourth letters in the series, L and
5 × 2 + 3 = 13
T, are static. The first and third letters consist of an
13 × 3 + 4 = 43
alphabetical order beginning with the letter E.
43 × 4 + 5 = 177
177 × 5 + 6 = 891 27. (a):
Hence, (a) is the answer. First Letter Second Letter
15. (b): The given sequence is a combination of two W–4=S E+2=G
series: S–3=P G+3=J
(i) 3, 7, 13, 21, 31, ? and (ii) 4, 7, 13, 22, 34
The pattern in (i) is + 4, + 6, + 8, + 10, …. P–4=L J+4=N
The pattern in (ii) is + 3, + 6, + 9, + 12, …. L–3=I N+5=S
Hence, the missing term = 31 + 12 = 43.
Thus, (a) is the right answer.

M05_MADAN 04_65901_C05.indd 53 23/12/22 7:25 PM


5.54 Chapter 5

28. (a): This series is just like the above series. However, 38. (c): The letters at the odd positions are moved three,
the middle letter of each term is replaced by a number. four and five steps forward, whereas the letters at the
29. (a): The third letter is repeated as the first letter even positions are each moved two steps forward to
of the next segment. The middle letter, A, remains get the corresponding letters of the code.
static. The third letters are in alphabetical order 39. (b): Please refer to alphabetic series table and add the
beginning with R. number of alphabets in the given word.
30. (b): There is a gap of one letter in the successive 40. (a): The numbers are coded as follows:
terms. The numbers are doubled.
1 3 7 8 9
Coding and Decoding
X T Z A L
31. (c): Refer to Tables 5.1 and 5.2.
For example, C is ranked 3 in Table 5.1. The same 2 3 4 6
rank denotes ‘X’ in Table 5.2. Thus, X is coded for C.
Thus, CERTAIN has been coded as XVIGZRM. N P S U
Similarly, MUNDANE can be coded as NFMWZMV.
32. (a): Here, the code is the reverse of the given word. That is, 2 as N, 3 as P, 5 as T, 4 as S and 9 as L. Thus,
The answer is TROFDER, which is the reverse of RED 23549 is coded as NPTSL.
FORT. 41. (d)
33. (d)
A B C D E F G H I J K
M O T H E R
+1 +1 5 6 7 8 9 10 11 12 13 14 15

O M H U R F L M N O P Q R S T U
16 17 18 19 20 21 22 23 24 25
Similarly,
A N S W E R Now,
+1 +1 22 25 8 22 5
N A W T R F R U D R A
42. (b)
34. (a): The first and the last letters of the word swap
A S S E S S YO U R L E A R N I N G

their positions. All other letters are moved one place C A L A N D E R


and get placed in the reverse manner.
35. (d)
C L A N A E D R
36. (a)
C L A I M C I R C U L A R
+1 +2 +3 +4 +5
D N D M R
C R I U C A L R
C H A R G E
+1 +2 +3 +4 +5 +6 43. (a)
D J D V L K Letters D R L A L
37. (c) Position 4 18 12 1 12
Change +2 +2 +2 +2 +2
C +3 F
Code 6 20 14 3 14
O +4 S
L +5 Q Hence, the code for DRLAL is 62014314.

D +6 J Letters C A M E L
Similarly,
Position 3 1 13 5 12
H +3 K
Change +2 +2 +2 +2 +2
E +4 I
Code 5 3 15 7 14
A +5 F
T +6 Z Hence, the code for CAMEL is 5315714.

M05_MADAN 04_65901_C05.indd 54 23/12/22 7:25 PM


Mathematical Reasoning and Aptitude 5.55

44. (a) Hence, ‘4’ and ‘3’ are the codes for ‘mangoes’ and ‘are’,
L I G H T respectively. Thus, in the third statement, ‘9’ means
‘ripe’.
53. (c): In the first and third statements, the common
G I L T H code digit is ‘5’ and the common word is ‘good’. Hence,
‘5’ means ‘good’.
R A I N Y In the second and third statements, the common code
digit is ‘3’ and the common word is ‘bad’. Thus, ‘3’
means ‘bad’.
Thus, in the third statement, ‘8’ means ‘and’.
I A R Y N
54. (c): In the first and third statements, the common
45. (a): First, rearrange the letters of the word ‘ARGU- code digit is ‘6’ and the common word is ‘is’. Thus, ‘6’
MENT’ in alphabetical order and then substitute each means ‘is’.
letter by the letter immediately following it. In the second and third statements, the common code
digit is ‘4’ and the common word is ‘colour’. Thus, ‘4’
A E G M N R T U means ‘colour’.
+1 +1 +1 +1 +1 +1 +1 +1 Thus, in the statement, ‘3’ means ‘fun’.
B F H N O S U V 55. (d): In the first and second statements, the common
code digit is ‘3’ and the common word is ‘hot’. Thus,
46. (c): All words in the answer choices consist of five ‘3’ means ‘hot’.
letters. The first two and last two letters interchange In the second and third statements, the common code
their positions. digit is ‘5’, and the common word is ‘day’. Thus, ‘5’
O F T E N means ‘day’.
Thus, in the second statement, ‘6’ means ‘very’.

Choose O dd Word
F O T N E
56. (c): All, except the universe, form a part of the universe.
Similarly,
57. (b): All, except geography, are branches of science.
R I S K Y
58. (a): All, except a mechanic, help in building a house.
59. (b): All others denote blood relations.
I R S Y K 60. (d): Mercury is the only liquid metal in the group.

A S S E S S YO U R L E A R N I N G
61. (c): All, except the bear, belong to the cat family.
47. (c): The fruits grow on a ‘tree’, and ‘tree’ is called ‘sky’.
Hence, the fruits grow on the ‘sky’. 62. (b): All others are parts of bed spread.
48. (b): The colour of the sky is blue; here, blue is called 63. (b): All, except eagle, are flightless birds.
green. 64. (c): All, except river, contain stagnant water.
49. (a): In the first and second statements, the common 65. (a): All, except arrow, are used while holding hand.
code word is ‘nie’ and the common word is ‘some’.
Thus, ‘nie’ means ‘some’. Odd Pairs
In the first and third statements, the common code
66. (d): In all other pairs, second is prepared by the first.
word is ‘pie’ and the common word is ‘good’. Thus,
‘pie’ means ‘good’. Hence, ‘jokes’ is denoted by ‘bi’. 67. (d): In all other pairs, first is used to contain the
second.
50. (a): Teacher uses chalk to write on the blackboard.
Here, chalk has been called book. 68. (a): In all other pairs, second is the noise produced by
the first.
51. (c): In the first and third statements, the common
word is ‘na’ and the common word is ‘are’. Hence, ‘na’ 69. (b): In all other pairs, the two words are antonyms of
means ‘are’. each other.
In the second and third statements, the common code 70. (a): In all other pairs, second is a part of the first.
word is ‘tok’ and the common word is ‘bad’. Hence, 71. (c): Each of the numbers except 63 is a prime number.
‘tok’ means ‘bad’. Thus, in the third statement, ‘tim’
stands for ‘they’. 72. (a): Each of the numbers except 51 is a perfect square.
52. (a): In the first and third statements, the common 73. (d): Each of the numbers except 28 is divisible by 3.
code digits are ‘4’ and ‘3’, and the common words are 74. (a): Sum of the digits in each other number is 10.
‘mangoes’ and ‘are’. 75. (a): All other numbers are cubes of odd numbers.

M05_MADAN 04_65901_C05.indd 55 23/12/22 7:25 PM


5.56 Chapter 5

Choose the O dd Number Pair or G roup 100. (c): Second is the working place of the first.
76. (d): In all other pairs, first number is 13 more than the 101. (a): The prime job of the first is to do the second.
second. 102. (a): Second is made according to the first.
77. (c): In all other pairs, second number is the cube of 103. (b): First is prepared as per the directions of the
the first. second.
78. (a): In all other pairs, one number is the square of the 104. (a): Second is the place for the first to perform on.
other. 105. (b): First composes the second.
79. (d): In all other pairs, the sum of two numbers is 8. 106. (a): First denotes the lack of second.
80. (d): In all other pairs, the second number is 1 less 107. (a): Scientific experiments are conducted in a labo-
than the square of the first number. ratory. Similarly, astronomical observations are
81. (b): All other pairs consist of prime numbers only. made in an observatory.
82. (b): In all other pairs, first number is the cube of the 108. (b): Bibliophile is a lover of books. Similarly, patriot
second. is a lover of one’s country.
83. (d): All other pairs consist of odd numbers only. 109. (d): The words in each pair are antonyms of each other.
84. (d): In all other pairs, the second is the multiple of 110. (d): Second is the dwelling place of the first.
the first.
85. (c): In all other pairs, it is the square of the first Completing the Analogous Pair
term + 1. 111. (b): The words in each pair are antonyms of each
other.
Choose the O dd L etter Group
112. (d): Second contains the story of the first.
86. (d): All other groups contain alternate letters from 113. (d): First causes the second.
left or right.
114. (b): The words in each pair are antonyms of each
87. (a): All other groups contain alternate letters of the other.
alphabet.
115. (c): First forms the basis of the second.
88. (d): The first letter is moved two places forward to
obtain the third letter. Then, third letter is moved two 116. (c): Taxonomy is the science of classification. Simi-
positions forward to obtain the middle letter. larly, pedology deals with the study of soils.
89. (d): In all other groups, the third and second letters 117. (a): The words in each pair are antonyms of each
are three steps ahead of the first and third letters, other.
A S S E S S YO U R L E A R N I N G

respectively. 118. (b): First is followed by the second.


90. (b): Only (b) contains a vowel. 119. (c): Second is the feminine gender of the first.
91. (b): In all other groups, the first and second letters 120. (b): First robs the second.
occupy the same position in the alphabet from the 121. (b): Second is the sound produced by the first.
beginning and the end, respectively.
122. (d): The words in each pair are antonyms of each
The second letter is moved one step backward to other.
obtain the third letter.
123. (b): An ornithologist specializes in the study of birds.
92. (d): In all other groups, four intervening letters are Similarly, an anthropologist specializes in the study
skipped. of mankind.
93. (c): In all other groups, the second letter is moved two 124. (c): Oxygen supports burning, whereas carbon diox-
steps forward to obtain the third letter, which then is ide extinguishes fire.
moved one step forward to obtain the first.
125. (d): Seismograph measures the intensity of earth-
94. (d): All other groups end with AT. quakes. Tachometer measures strains.
95. (b): There is no repetition of any letter in any other
group. Blood Relations
Analogy 126. (a):

Direct or Simple A nalogy A +F −K

96. (b): Second word denotes the place where the sport
is held.
97. (d): First works for and earns from the second.
−C +G
98. (a): The words in each pair are antonyms of each pair.
99. (a): Second is required by the first to function. Hence, F is the uncle of G.

M05_MADAN 04_65901_C05.indd 56 23/12/22 7:25 PM


Mathematical Reasoning and Aptitude 5.57

127. (c): 132. (b): The man in the photograph is the son of the only
Grandmother
son of Raveena’s grandfather. Thus, the man is the
son of Raveena’s father. Hence, he is the brother of
Raveena.
Seeta’s mother 133. (d):
Kamni Deepak
Seeta Ravi

The maternal grandmother of Seeta and the grand-


mother of Ravi are the same person. Hence, Seeta Vinita Sandy Amit
and Ravi are cousins.
128. (a): Amit is the only brother of Sandy, and Vinita is the sis-
ter-in-law of Amit. Hence, Vinita is the wife of Sandy.
+A = − E
Kamni is the mother-in-law of Vinita. Kamni is the
mother of Amit.
134. (d):

+B −C +D Woman

Therefore, D is a boy because there is only one daugh- Brother (of Amit)
ter of E (or husband of E as given in the question
statement).
Here, + and – have been used for male and female, Grand daughter
respectively. = sign has been used to show husband–
wife relationship, and ↔ for sibling relationship. Hence, the woman in the picture is the mother of
Hence, B is the brother of D. Amit.
129. (b): 135. (a):
Father = Wife (Dinesh’s mother) −C = +D F

+X −Y

A S S E S S YO U R L E A R N I N G
Younger brother Daughter Elder brother As X is the only son of C (and D), Y is a female.
(playing game) (Dinesh) Looking at the family tree, we can say that Y is the
niece of F.
As per the question, the person playing the game is
different from Dinesh. The person playing the game Direction Sense
is the brother of Dinesh. 136. (c):
130. (d): Start point
Somesh’s mother 5 km

5 km 3 km
Only son (i.e. Somesh)

D
Son 137. (a):
131. (d): 138. (c):
Grandmother B 15 m A E
Daughter-in-law
20 m 20 m
Woman = Father

C 25 m D
Namrata

M05_MADAN 04_65901_C05.indd 57 23/12/22 7:25 PM


5.58 Chapter 5

139. (c): Fractions


E D
151. (a)
1 km
B 152. (c): 1947 + 2000 – 103 = (?)2
C 3947 – 103 = (?)2
A
3844 = (?)2
He is moving in the west direction now. ? = 62
140. (d): 2
153. (a): 13 % of 3300 + 25% of 184 = 40% of ?
B 30 km C 3
41 1
20 km × 3300 + × 184 = (41 × 11) + 46 = 40% of ?
300 4
F
2
35 km

15 km 451 + 46 = ×?
5

D 15 km E 5
? = 497 × = 1242.5
2
Required distance = AF
154. (d)
   = 30 + 15 = 45 km 1 1 1
155. (a): Given expression = + +
141. (c): ⎛ 30 ⎞ ⎛ 50 ⎞ ⎛ 5 ⎞
142. (d): ⎜⎝ ⎟⎠ ⎜⎝ ⎟⎠ ⎜⎝ ⎟⎠
7 13 9
143. (d): 7 13 9
144. (a): = + +
30 50 5
145. (b):
35 39 270 35 + 39 + 270 172
= + + = =
Seating Arrangement 150 150 150 150 75
146. (b): The seating arrangement is as follows: 35 39 270 35 + 39 + 270 172
= + + = =
150 150 150 150 75
1
P X S Z R A
A S S E S S YO U R L E A R N I N G

156. (b): The part of work finished by Ramu in 1 hour =


3
Therefore, the right of P is X. 1
So, the part of work finished by Ramu in 2 hours
147. (b): 5
1 1 11 1 11× 1 11
=2 × = × = =
5 3 5 3 5 × 3 15
E B A C D 157. (b)
Therefore, A is sitting in between B and C. Hint: The question can be solved by taking LCM.
148. (a): 158. (d)
Q 159. (b)
Hint: Take LCM of 200, 250 and 300.
W P
160. (a)
S R
Time and Distance
U T Distance 1200 m
161. (a): Speed = =
V Time 600s econds
= 2 m/s
149. (d): J > L and D > F.
Speed in kmph = 2 × (18/5)
Hence, J > L > H > D > F. = 7.2 kmph
Hence, H is the person to reach.
48
150. (a): 162. (c): Here, speed = = 24 kmph
North 2
R Q P S T 252
Time taken = = 10½ hours
24

M05_MADAN 04_65901_C05.indd 58 23/12/22 7:25 PM


Mathematical Reasoning and Aptitude 5.59

163. (d): Average speed = 2 × 30 × [60/(30+60)] 172. (d): Let total employees = 100
= 2 × 30 × (60/90) = 40 mph Matriculates = 40
Time taken by train to complete the trip = Distance/ Remaining = 100 − 40 = 60
20 Graduates = 50% of remaining 60 = 30
Speed = = ½ hour = 30 minutes Postgraduates = 100 − 40 − 30 = 30
40 Graduates = Postgraduates
164. (a): The distance covered by auto rickshaw Hence, graduates are also 180.
1 173. (b): 96% of total population = 23,040
= 1 − [(1/2)+(1/3)] = Hence, total population = 23,040 × 100/96 =
6
1 24,000
of total journey = 5 km
6 174. (c): Ratio between boys and girls = 75%:25% = 3:1
Total journey = 6 × 5 = 30 km Hence, number of boys = 3 × 420 = 1260
165. (b): Due to stoppages, it covers 9 km less. 175. (b): (100 – 12.5)%, that is, 87.5% of total income =
Time taken to cover 9 km = (9/54) × 60 minutes = 3500
10 minutes Total income = 3500 × 100/87.5 = `4000
(2 × 21 × 24)
166. (b): Average speed = = 22.4 kmph [+20 − 20 + ( − 20) × (+20)]
(21 + 24) 176. (d): = −4%.
100
Distance = Time × Speed = 22.4 × 10 = 224 km
177. (b): Let Y’s salary = 100; then X’s salary = 50
167. (c): As the distance in both ways is same, we can X’s new salary = 50 × 3/2 = 75
apply the formula (2xy/x)+ y. Y’s new salary = 100 × 5/4 = 125
(2 × 64 × 80) Combined old salary = 100 + 50 = 150
Average speed = = 71.11 kmph New combined salary = 75 + 125 = 200
(64 + 80)
Increase in combined salary = 50
168. (c): Let the distance travelled in the first phase be x km. Percentage increase = (50/150) × 100 = 33.33%
Then, distance travelled in the second phase = 178. (a): Let x water to be added to make water 20% of
(610 − x) km the new mixture.
Hence, x + (610 − x ) = 9 hours Water in the old mixture = 10% of 40 = 4 l
40 90 Water in the new mixture to make it 20% =
(4 + x) l
⇒ x = 160 km Now 20% of (40 + x) l = 4 + x
169. (b): Distance to be travelled = 320 m Solving equation, we get x = 5 l.

A S S E S S YO U R L E A R N I N G
Speed = 36 kmph = 36 × (5/18) = 10 m/s 179. (d): Let income = 100
320 New income = 120
Time taken = D/S = = 32 seconds
10 New expenditure = 75 + 10% of 75 = 82.5%
170. (c): Total distance to be travelled = 280 + 120 = 400 m New savings = 120 − 82.5 = 37.5
Speed should be converted into m/s. Old saving = 25
100 Percentage increase in savings
Speed = 40 kmph = 40 × (5/18) = m/s = (37.5 − 25)/25 × 100
9 = 12.5/25 × 100 = 50%
Time = D/S = (400/100)/9 = 36 seconds 180. (d): Let C’s income = 100
B’s income = 100 + 20 = 120
Ratio, P roportion, P ercentage and A’s income = 125% of 120 = 150
D iscounting Thus, A’s income is 50% higher than that of C.
171. (c): Let original price = 100 181. (b): Let original price = 100
Original quantity = 100 Increased price = 100 + 25 = 125
Total expenditure = 100 × 100 Now 125 is to be brought to 100; say it is multi-
New price = 100 + 25 = 125 plied by x.
New quantity = Q 125 × x = 100 4
As expenditure is to be kept same, x = 100/125 = that is equivalent to 80%.
5
100 × 100 = 125 × Q
Hence, the price should be reduced by 20%.
(100 × 100) 182. (c): Let cost price = 100
Q= = 80 Net selling price = 100 + 8 = 108
125
Now 120 – (x% of 120) = 108

% reduction in quantity = 100 − 80 = 20 x = 10

M05_MADAN 04_65901_C05.indd 59 23/12/22 7:25 PM


5.60 Chapter 5

183. (c): Discount on `60 = 25% of `60 = `15 Profit and Loss
Assume that the retailer offers a discount of `15
or four extra mangoes. 100 + 6.25 106.25
191. (a): S.P.SP = × 4800 = × 4800 = =`5100
New purchase price of four mangoes = `15 `5100 100 100
Purchase price of one mango = 15/4 = `3.75
184. (c): Suppose both length and breadth are 100. 192. (b): Let C.P. = `100 and S.P. = `120.
Hence, area = 100 × 100 New C.P. = `110
New length = 100 + 10 m = 110 m 110 × 125
As area is to be kept same, ∴∴S.P.
SP on 25% profit = == ``137.5
137.5
100
110 × new breadth = 100 × 100
New breadth = 100 × (100/110) = 1000/11 Difference of S.P.s = 137.5 – 120 = `17.5
Now take a ratio.
100
Decrease in breadth = 100 − (1000/11) = 17.5:100 :: 700:x
= 9.11% 11 100 × 700
As breadth was assumed to be 100, no additional ∴x = = ` 4000
17.5
calculation is required for percentage figure.
185. (b): Using the formula: x + y + xy/100, net percent- 193. (d): S.P. = 90% of 8000 = `7200
Profit = 7200 – 5000 = `2200
(20 × 40) Profit percentage = 2200/5000 × 100 = 44%
age increase = 20 + 40 + = 68%
100 194. (b): Let C.P. = S.P. = `100.
Increased price = `25 + 68% of 25 = 25 + 17 = `42 C.P. of 100 pencils = `100
Alternatively: Using the multiplier factor con- C.P. of 1 pencil = 100/100 = `1
⎛ 6⎞ ⎛ 7⎞ S.P. of 60 pencils = `100
cept, we get the final price as 25 × ⎜ ⎟ × ⎜ ⎟ =` 42 5
⎝ 5⎠ ⎝ 5⎠ S.P. of 1 pencil = `100/60 = `
3
= `42. 5 2
Profit = – 1 =
186. (a): Circumference of a circle = 2pr. As r increases by 3 3
20%, the circumference also increases by 20%. Profit percentage = (2/3)/1 × 100 = 66 2/3%
187. (b): Area of a circle = π  r2 195. (b): S.P. = 480 + 10% of 480 = `528
As r2 = r × r and as r increases by 20%, r2 increases Let marked price = M.
(20 × 20) Now 96% of M = 528, so M = `550.
by 20 + 20 + = 44% .
100 196. (c): The ratio of production between women and
A S S E S S YO U R L E A R N I N G

188. (d): Let side of square = 100 men is 60:40.


Area = 100 × 100 = 10,000 The profit on men’s garments = 9% of `40 = `3.6
To make profit of 5%, contribution of women’s
New side = 100 + 25 = 125
­garments = 5 – 3.6 = `1.4
New area = 125 × 125 = 15,625
Profit percentage on women = 1.4/60 × 100 =
Percentage increase = [(15,625 – 2.33%
10,000)/10,000] × 100 = 56.25
Alternatively = x + x + x2/100 = 25 + 25 + (25 × 197. (a): Old sales price = 500 + 40% of 500 = `700
25/100) = 56.25%  New marked sales price = 500 + 80% of 500 =
`900
11
189. (c): Increased length = 20 × = 22 cm Discounted price = 900 – 20% of 900 = `720
10 Thus, the new price is higher by `20.
120 6 198. (b): 20% reduction on `600 means benefit of `120 to
Increased breadth = 10 × = 10 × = 12 cm
100 5 a customer. That enables customer to buy 2 kg
extra article.
The new area = 22 × 12 = 264 cm 2
Thus, reduced selling price of 2 kg = `120 and of
As there is multiplication of two quantities in
1 kg = 120/2 = `60
the area formula, we can use the method a + b + If a value reduces by 20%, it means it should be
ab increased by 25% to reach at the original value
to calculate the percentage increase in area again.
100
also. Thus, original price = 60 + 25% of 60 = `75
xy 199. (a):  int: Original C.P. can be calculated first, that
H
190. (b): We can use the formula x + y + to calculate is, `50,000.
the net increase. 100
 Then C.P. increases by 5%. Same amount of
25 + 12 + (25 × 12)/100 = 40% profit can be added to it, and then percentage
calculated on it.

M05_MADAN 04_65901_C05.indd 60 23/12/22 7:26 PM


Mathematical Reasoning and Aptitude 5.61

1680 ∴ Interest = `840 – `800 = `40


200. (c): Cost Price = × 100 == ``2000
2000. For 1-year time period, simple intersest = com-
84 pound interest
115% P × R × T 800 × R × 1
Marked price = 2000 ×
== ``2500
2500. = = ` 40
92% ∴
100 100
∴ R = 5%
I nterest
210. (c): 4 times in 6 years.
P × 6 × 2 P × 9 × 3 P × 14 × 4 So 16 times in 12 years
11400 ==
201. (a): 11,400 + +
100 100 100 And 64 times in 18 years.
∴ P = `12,000 Averages
202. (c): Hint: First, calculate that `900 becomes `1000
in the fourth year. Calculate rate of interest by 211. (c): Total increase in weight = (8 × 2.5) kg = 20 kg
application of simple interest formula. Thereaf- Weight of new person = (60 + 20) kg = 80 kg
ter, principal amount can also be calculated. 212. (c): Total weight of 8 articles = 200 kg
203. (b) Total weight of 3 articles = 3 × 20 = 60 kg
204. (a): H int: First initial R can be calculated, and then Total weight of rest of 5 articles = 200 − 60
3% added to it. = 140 kg
205. (a): We can solve this question directly. Average weight of 5 articles = 140/5 = 28 kg
The simple interest amount is same in both cases. 213. (c): As the distance is not mentioned, L.C.M. of 35
15 × 4 = 20 × 3 and 42, that is, 210, can be assumed to be the total
206. (d): Let Banta get `M. distance.
∴ Santa gets `(1301 – M). Time taken from A to B = 210/35 = 6 hours
We know that both get the same amount. Time taken from B to A= 210/42 = 5 hours
For compound interest, total amount = Total time taken = 11 hours
n (210 + 210) 420
⎛ R ⎞ Average speed = = = 38.17 kmph
⎜⎝ 1+ ⎟ (6 + 5) 11
100 ⎠
Alternatively, by applying direct formula
17 19
⎛ 4 ⎞ ⎛ 4 ⎞ 2 xy
∴ (1301 − M ) ⎜ 1 + = M ⎜1 +
⎝ 100 ⎟⎠ ⎝ 100 ⎟⎠ =
x+ y

A S S E S S YO U R L E A R N I N G
2
⎛ 4 ⎞ = (2 × 35 × 42) / 35 + 42 = 38.17 kmph
∴ 1301 − M = M ⎜ 1 +
⎝ 100 ⎟⎠
214. (c): Petrol consumed during first year = 4000/8
676 M = 500
∴ 1301 − M =
625 Petrol consumed during second year = 4000/10
∴ 1301 × 625 – 625M = 676 M = 400
Total expenditure = 4000 × 2 = 8000
∴ M = `625 Total petrol consumed = 500 + 400 = 900
207. (d): Let amount be `person. Average cost per litre = 8000/900 = `8.88
It becomes double = 2A in 9 years. Note: The answer won’t be `9 as many have
In the next 9 years, it again doubles, that is, thought because calculation of average also
becomes 2 × 2A = 4A. depends on the quantity consumed. This ques-
So in 9 + 9 = 18 years, the amount becomes four tion can be solved with harmonic mean also.
times itself. 215. (b): First statement is not sufficient to answer the
208. (b): Simple interest for each year = 40/2 = `20 question because the speed of the second half of
Compound interest for the first year is `20 and the distance is not given here but statement II can
for the ­second year = 41 – 20 = `21 answer as information to calculate average speed =
Thus, `20 becomes `21; hence rate of interest distance/time is given in the statement.
350
can be calculated. 216. (b): Time taken by B to reach destination =
= 7 hours 50
P = `20, SI = `1, T = 1 year
So R = SI × 100/P × T = 1 × 100/20 × 1 = 5% If A has a head start of 1 hour, then the distance
travelled by A when B reaches the destination
209. (a): 4 years – 3 years = 1 year = 40 × (7 + 1) = 320 km
Thus, we can say that `800 becomes `840 in Hence, A has to still travel 30 km while B has
1 year. reached the destination.

M05_MADAN 04_65901_C05.indd 61 23/12/22 7:26 PM


5.62 Chapter 5

217. (d): Distance travelled in 2 hours = 35 × 2 = 70 km Odd days in 300 years = (5 × 3) = 1


Distance travelled in next 3½ hours = 60 × 3.5 98 years have 24 leap years + 74 ordinary years.
= 210 km Number of odd days in 98 years = (24 × 2 + 74) = 122
Distance travelled in next 2½ hours = 70 × 2.5 = 3 odd days.
= 175 km
Now come to the calculation of odd days in the
Total distance = 70 + 210 + 175 = 455 km
period from 1.1.1999 to 16.6.1999.
Average speed = Total distance/Total time taken
(70 + 210 + 175)

March
= = 455/8 = 56.87 kmph

Total
April

June
May
Feb
Jan
(2 + 3.5 + 2.5)
218. (a): The present total age of husband and wife = 2 ×
(22 + 5) = 54 years Odd 31 28 31 30 31 16 167
Total age of family (including child) = 54 + 3 = 57 days
years 57
Average age of family = = 19 years 167 days = 23 complete weeks + 6 odd days (this
3 result is obtained by dividing 167 days by 7)
3600 Total number of odd days = (0 + 1 + 3 + 6) = 10 ⇒ 3
219. (b): Monthly saving = = `300 odd days
12
Hence, monthly expenditure = income − saving = Thus, the day asked in the question is Wednesday.
1400 – 300 = `1100 224. (b): 16 August 2010 = (2000 + 9 years + period
220. (b): The total of a, b and c = 3 × 50 = 150 1.1.2010 to 16.8.2010)
d = 10 Odd days in 2000 years = 0
Total of a, b, c and d = 150 + 10 = 160 9 years = (2 leap years + 7 ordinary years) = (2 × 2 +
Average of a, b, c and d = 160/4 = 40 7 × 1) = 11 odd days
11 can be divided by 7 ⇒ 4 odd days
Calendar
221. (d): There are two conditions for 2 years having the

March
same calendar: both having same length in terms of

Total
April

June
May

July
Aug
Feb
Jan
the number of days and first day starting with same
day of the week.
The year 1991 has 365 days, that is, 1 odd day, and
Odd
the year 1992 has 366 days, that is, 2 odd days, while 31 28 31 30 31 16 31 16 228
days
A S S E S S YO U R L E A R N I N G

the year 1993 has 365 days, that is, 1 odd day. The
years 1994, 1995 and 1996 have 1 odd day each. The odd days can be calculated for each month sepa-
The sum of odd days so calculated from years 1991 rately and then added up. This can save some time
to 1996 = (1 + 2 + 1 + 1 + 1 + 1) = 7 odd days in calculation. Such as January has 3 odd days (31
Hence, the year 1997 will have the same calendar as divided by 7 gives remainder of 3 and so on).
that of the year 1991. Now, 228 days = 32 complete weeks + 3 extra days;
222. (d): 28 May 2007 = (2000 years + 6 years + period it means 3 odd days.
from 1.1.2007 to 28.5.2007) Total number of odd days = (0 + 0 + 4 + 4) = 8; it
Odd days in 2000 years = 0 means 1 odd day.
Odd days till 2006 = (5 ordinary years + 1 leap year) Thus, 16 August 2010 was Monday.
= (5 × 1 + 1 × 2) = 7 odd days 225. (c): From 15 August 2012 to 15 August 2013, there
is 1 extra day. In the rest of 16 days of August, there
Jan Feb March April May Total are 2 odd days.
31 28 31 30 28 148 In September, there are 2 odd days, and in October,
3 odd days. In the first 15 days of November, there
148 days = (21 weeks + 1 day) ⇒ 1 odd day will be 1 odd day.
Total number of odd days = (2000 years + 6 years + Thus, the total number of odd days is 1 + 2 + 2 + 3 + 1
period from 1.1.2007 to 28.5.2007) = 9; it means 2 extra (odd) days. Hence, 15 Novem-
= (0 + 7 + 1) = 8 odd days, that is, again 1 odd day. ber 2013 was Friday.
Hence, Monday is the answer. 226. (b): Each day of the week is repeated after 7 days.
223. (c): 16 June 1999 = (1900 years + 98 years + period Assuming that the first day is Sunday, 8, 15, 22, 29,
from 1.1.1999 to 16.6.1999) 36, 43, 50 and 57 days will be Sundays.
Hence, day 58 is Monday, 59 is Tuesday, and 60 is
Now first deal with 1998 complete years; then how
Wednesday.
many odd days are there in 1998 years?
Hence, day after 60 days will be Thursday.
Odd days in 1600 years = 0

M05_MADAN 04_65901_C05.indd 62 23/12/22 7:26 PM


Mathematical Reasoning and Aptitude 5.63

227. (a): The year 2012 is a leap year. Thus, it has 2 odd 232. (c): Let us draw a Venn diagram with the informa-
days. tion given in the question.
But as calculation is to be done from 22 April 2012 100
to 22 April 2013, so it has 1 odd day only. As the cal-
culation is to be done backwards, 22 April was 1 day
less. Thus, it was Sunday.
Numbers Numbers Numbers
Note: Had the question been about 22 April 2014, the divisible divisible divisible
answer would have been Tuesday. If the same ques- only by 3 by 3 and 5 only by 5
tion is framed for 22 April 2016, the answer would = 33 − 6 = 27 =6 = 20 − 6 = 14
have been Friday, as 2016 is a leap year.
228. (a): 1 April 2001 = (2000 years + period from
1.1.2001 to 1.4.2001) Numbers not divisible by 3 or 5 = 100 − (27 + 6 + 14) = 53
Odd days in 2000 years = 0
Now, calculate the odd days for the year 2001. Numbers divisible by 3 (3, 6, 9 and so on) = 33
Numbers divisible by 5 (5, 10, 15 and so on) = 20
Jan Feb March April Total Numbers which are divisible by both 3 and 5 (15, 30,
45, 60, 75 and 90) = 6
31 28 31 1 91 Numbers divisible by 3 or 5 or both 3 and 5 = 27 + 6 + 14
91 divided by 7 gives 0 as remainder, so, odd days = 47 (alternatively, 33 + 20 − 6 = 47)
can be counted as 0. Numbers divisible neither by 3 nor by 5 = 100 − 47 = 53
Total number of odd days = (0 + 0) = 0 233. (b): Number of candidates failed in English = 52%
On 1 April 2001, it was Sunday. Number of candidates failed in Mathematics = 42%
229. (c): 100 years contain 5 odd days. Number of candidates failed in both = 17%
Last day of the first century is Friday. Total failed = 52 + 42 – 17 = 77%
200 years contain (5 × 2) = 3 odd days. Passed in both the subjects = 100 − 77 = 23%
Last day of the second century is Wednesday. 234. (a):
300 years contain (5 × 3) = 15 odd days, that is,
equal to 1 odd day.
Last day of the third century is Monday.
400 years contain 0 odd days.
English only Both English Hindi only
Last day of the fourth century is Sunday. = 40 − (5 + 20)
= 25 − 20 = 5 and Hindi
This cycle is repeated. = 20 = 15
Last day of a century cannot be Tuesday or Thursday

A S S E S S YO U R L E A R N I N G
or Saturday.
230. (c): The year 2004 is a leap year. It has 2 odd days.
Hence, 10 February 2004, was Tuesday, and 8 Total = 40
February 2004 must be Sunday.
235. (c): People reading newspaper X or Y = 40 + 50 − 10 = 80
Venn Diagrams Hence, people reading neither X nor Y = 100 − 80 = 20
(Candidates can try to solve this question with Venn
231. (a) diagram also.)
Total = 40
236. (a): A teacher can be an author; an author can be an
­artist, and so on. Some person can be a teacher, an
author as well as an artist.
Both 237. (d): A traveller can travel by both taxi and train.
English 238. (b): Same explanation as in Question 236 offered in
Only English Only
composition Venn Diagram descriptions.
composition Chemistry
and
= 14 − 5 = 9 = 29 − 5 = 24 239. (c): Same explanation as in Question 236 offered in
Chemistry
=5
Venn Diagram descriptions.
240. (b): Mercury is also a metal, although it exists in liq-
uid state.
Neither English nor Chemistry = 40 − (9 + 5 + 24) = 2 241. (a)
242. (c)
Alternatively: Students taking either English compo-
243. (c)
sition or Chemistry = 14 + 29 – 5 = 38
Hence, students not taking any class = 40 − 38 = 2 244. (a)
245. (c)

M05_MADAN 04_65901_C05.indd 63 23/12/22 7:26 PM


This page is intentionally left blank

M05_MADAN 04_65901_C05.indd 64 23/12/22 7:26 PM


CHAPTER

6 Logical Reasoning

01 Understand the Structure of Arguments: Argument Forms,


Structure of Categorical Propositions, Mood, and Figure, Formal
and Informal Fallacies, uses of Language, Connotations and
Denotations of Terms, Classical Square of Opposition.

02 Evaluating and Distinguishing


Deductive and Inductive Reasoning.

03 Analogies
LEARnInG
ObJECTIVES 04 Venn Diagram: Simple and
Multiple use for Establishing
the Validity of Arguments.

05 Indian Logic: Means of Knowledge.

06 Pramanas: Pratyaksha (Perception),


Anumana (Inference), Upamana
(Comparison), Shabda (Verbal
Testimony), Arthapatti (Implication) and
Anupalabddhi (Non-apprehension).

07 Structure and Kinds of Anumana (Inference),


Vyapti (Invariable Relation), Hetvabhasas
(Fallacies of Inference)

M06_MADAN 07_65901_C06.indd 1 27/12/22 8:17 PM


6.2 Chapter 6

The word ‘because’ indicates that a premise or reason


Introduction is being provided to support a claim. The three main types
of claims are—Value, Policy, and Factual claims.
‘The whole of science is nothing more than a Examples of words and phrases that may signal a
refinement of everyday thinking’ premise are: as, due to, as indicated by, for the reason
 — Albert Einstein that, owing to, through, in the view of, as a result of, etc.
The word ‘therefore’ and its synonyms are clues that
Let’s start with an interesting example. There is a popular a conclusion, or claim, is being made. Other words are:
story that Newton was sitting under an apple tree, an apple accordingly, consequently, infer that, hence, prove that,
fell on his head, and he suddenly thought of the Universal conclude that etc.
Law of Gravitation. Apples were falling towards earth earlier The simple example:
also but that knowledge was implicit. What Newton did was The argument is, ‘The internet is a good invention.’
to make that part of knowledge as explicit. Knowledge makes Then, we support this contention with logical reasons,
the theories explicit. What makes reasoning important is such as ‘It is a source of endless information’, and ‘It is a
that not many problems are solved in life, we work for their hub of entertainment’, and so on. Hereby, we conclude
solution. But at the same time, what may be said about the argument by giving our verdict.
one theory, others may say it in a different manner, thus While evaluating an argument, one must consider both
progression takes in life. Logic is the way of life that trains a the forms as well as the content. We are discussing it in
person in certain methods that help in differentiating what the ensuing passages.
is right and what is wrong. Flow of knowledge is both the
ways which are called deductive and inductive. Arguments – Types of Reasoning
The two main types of reasoning involved in the disci-
Deductive–General Inductive–specific to pline of logic are deductive reasoning and inductive rea-
to specific general soning.
All Men are mortal Chanakya is a Man Saint Aristotle had emphasized the concept of deduc-
(general) (cause) tive reasoning. Immanuel Kant (1724-1804) rightly wrote
in the ‘Critique of Pure Reason’ that Aristotle’s theory
Chanakya is a Man (cause) Chanakya is mortal (effect) of logic completely accounted for the core of deductive
inference. For him certainty or being closer to the truth is
Chanakya is mortal All Men are mortal
attained only through deductive inference.
(effect) (general)
The modern period philosophers, particularly Francis
1. From general (rule) and cause, deduce the effect. Bacon and J S Mill, ‘lamented the powerlessness of deduc-
2. From cause and effect, induce the general (rule) tion’. They focused on ‘inductive reasoning’ to lead to new
3. From general and an effect, abduce the cause. knowledge. Deduction is simply the explication of what is
implied in the major premise and the conclusion cannot
Example: be more general than the major premise. Therefore, they
Most of the times, arguments have two or more premises. propose inductive inference to be the sole inference which
The immediate inference has a single premise that sup- can offer us to have new knowledge.
ports the conclusion. Here is an example:
Deductive Reasoning
Premise: No items on menu are vegetable dishes. Deductive reasoning is an inferential process that sup-
ports a conclusion with certainty.
Conclusion: Therefore, no vegetable dishes are
items on menu. Example from Deductive Reasoning:

Logical argument Premise 1: All men are mortal.


Evidence/ Premise 2: I am a man.
Premises
Accepted facts Conclusion: So I am mortal.
Inference
Statements
Here, conclusion is part of premises.
What is claimed to
Conclusion How many premises can we have to arrive at conclusions?
­follow from the evidence
There is no limit to it. In this example, it can be I, you, he,
Figure 6.1 she, Rohan, Sohan and so on. Even if we take one thou-
sand or one million such examples, there may always be
I dentifying A rguments next one. No totality is available. It is rather an impossible
There are some ‘signal words’ in our text to indicate an task. Usually we adopt N + 1 approach. Deductive is the
argument. ‘whole class’, nothing is excluded. It is basically termed

M06_MADAN 07_65901_C06.indd 2 27/12/22 8:17 PM


Logical Reasoning 6.3

Deductive Inductive

Theory Theory

Tentative
Hypothesis Hypothesis
Waterfall Hill climbing

Observation Pattern

Confirmation Observation

● Deductive reasoning works from the more ● Inductive reasoning works the other way,
general to the more specific. moving from specific observations to
● Sometimes this is informally called as ‘top- broader generalizations and theories.
down’ approach.
● Informally, we sometimes call this as ‘bot-
tom up’ approach.
● Conclusion follows logically from premises ● Conclusion is likely based on premises.
(available facts). ● Involves a degree of uncertainty.

Figure 6.2 Deductive vs. Inductive

as ‘deterministic’. It is considered more scientific. Formal 5. Deductive reasoning has three basic features – logi-
logic has been defined as the ‘science of deduction’. cal necessity, a priori and rational. Deductive pro-
Deductive inference is further categorized into vides a ‘priori logic’ (that is basically independent
immediate (where conclusion is drawn from a single of experience – a knowledge that is prior to sense
statement) and mediate (where conclusion is drawn experience).
from two statements, called syllogism). Syllogism has 6. Deductive is akin to analysis, that is separating any
been discussed further in this unit. Deductive is akin to material or abstract entity into its constituent elements.
analysis (separating any material or abstract entity into its 7. There are two dimensions in deductive reasoning –
constituent elements). valid (or invalid) and truth (or falsity) on the other
We need to look at the following points. hand. Validity is not a matter of degree, there are two
situations – Nothing comes in between. An invalid
1. Deductive reasoning is the process in which con- argument is a conclusion that does not come from the
clusions are drawn with ‘logical certainty’ from the premises.
given premises. Thus, conclusion is directly contained
in premises. It is taken as on a ‘conclusive ground’. The some of the methods to check ‘formal validity’ are
Deductive reasoning helps us to know what is ‘embed- these–‘six rules of valid categorical syllogism’, ‘Venn
ded’ or ‘entailed’ in the premises. Here, the premises diagram method’, ‘nine rules of inference’, ‘ten rules of
are complete, sufficient and proper. Conclusion does replacement’, ‘truth table method’, ‘shorter truth table
not go beyond premises. Thus, deductive offers very (reductio ad absurdum)’ etc.
less or no scope for innovation. The example of deductive reasoning:
2. We move from ‘general’ (theory) to ‘specific’ (specific Proposition I: All vegetables contain vitamins.
premises) in deductive reasoning. This is like deal-
ing with a formal system that is based on structure Proposition II: Carrot is a vegetable.
and form that’s why a deductive argument is called as Conclusion: Thus, carrot contains vitamins.
‘structural’ forms. That is require to make an argument
‘valid’. We should take a note that truth and falsity deal with the
3. Knowledge is not mere acquisition of new informa- ‘statements’ (be it individual premises and conclusion)
tion. It is the outcome of analytical (deeply thought), while validity and invalidity deal with arguments in their
critical and reflective attitude. whole form.
4. A deductive reasoning does not question the status of
premises, they may be actually true or not. The argu- vAlidity
mentator is ‘necessarily’ concerned with given set of Validity and Truth: A statement is assumed to be true
premises. when it agrees with reality. For example, all men are

M06_MADAN 07_65901_C06.indd 3 27/12/22 8:17 PM


6.4 Chapter 6

human beings. The above statement is true because men Invalid arguments (2, 5 and 7) may be interpreted in
are human beings. There are two types of truth: the similar manner. Argument 3 points towards the logi-
cal necessity in deductive logic. The false conclusions can
1. Material Truth: This is a matter of fact (truth). This
not be withdrawn from all true premises.
comes out after empirical query (verifiable by obser-
In deductive arguments, there are valid or invalid
vation or experience).
while propositions are true or false.
2. Logical Truth: This is outcome of the form of an
argument. This is linked with deductive reasoning
where the logician does not bother about the status
Soundness
of premises whether they are actually true of not. He Soundness is the attribute of a deductive argument that
is concerned what actually flows from those given denotes both the truth of its premises and its logical
premises. In deductive, the assumption about truth strength. A deductive argument is sound when:
of premises is assumed (the actual or material truth 1. It is valid.
does not matter). 2. It has all true premises.
Falsity as a statement is false it does not agree with the
For example:
realty. For example, Swami Vivekanand was not the social
reformer.
We will discuss them in details in the coming paragraphs. Premise 1: All cats are mammals.
Validity: An argument is valid when the conclusion is Premise 2: All mammals are animals.
drawn from premises. Thus, it is formal in nature. Conclusion: All cats are animals.
Validity is about strength of the inference, between the
premises and the conclusion. If all premises were true,
This argument is sound because:
then its conclusion must be true, by ‘necessity’. When the
form (or structural form) of an argument is correct, it is 1. It is valid as the premises support the conclusion by
called a valid argument. An argument is invalid, if it does necessity.
not have the correct form. There is nothing called as ‘less 2. All of the premises are actually true.
valid’, statements are either ‘valid’ or ‘invalid’.
We need to make the following assumptions: On the other hand, the example above used to dem-
onstrate validity (with dogs, dragons, and birds) is not
1. First, we need to assume that the premises are true, sound, because it does not have all (or any) true premises.
even if they are not. We assume that they are true. The form is still valid. We need to look at the figure also to
2. Then ask yourself, whether the conclusion would establish the whole picture.
need to be true, assuming that the premises are true.
For example, all men are honest. Inductive Reasoning
All honest are girls. We have discussed examples earlier. There is always a
Therefore, all men are girls. relationship of some doubt in inductive reasoning. We
This argument is valid as it is in right structural form, should be very clear on that aspect.
though ‘all men are girls’ is not a right statement. The Proposition I: Most vegetables contain vitamins.
combination of some statements will lead to valid
Proposition II: Carrot is a vegetable.
argument and false to invalid but this is not the case.
Thus, truth or validity may or may not coincide with each Conclusion: Therefore, a carrot contains vitamins.
other. According to deductive principle, knowledge is not Proposition I contains the word ‘most’ in inductive while
mere acquisition of new information, knowledge is the deductive contains the word ‘all’. Thus, deductive is
outcome of critical, reflective and analytical attitude— deterministic in nature while inductive is probabilistic.
eliminate ignorance (avidya) and become enlightened. Aristotle used the word ‘epagoge’ for induction and CS
Premises Conclusion Arguments Peirce used the word ‘ampliative’ for the same. There is
1. TRUE TRUE VALID possibility that the conclusion goes beyond the premises.
2. TRUE TRUE INVALID Thus, inductive arguments are innovative that work for
3. TRUE FALSE INVALID the solution of problems in the society.
4. FALSE TRUE VALID The study of inductive reasoning is generally carried out
5. FALSE TRUE INVALID in the field of ‘informal logic’ or ‘critical thinking’. As deduc-
6. FALSE FALSE VALID tive reasoning does not help much in the progression of
7. FALSE FALSE INVALID knowledge. Then Inductive reasoning comes into picture.
Inductive is more about social sciences. This cycle of knowl-
Valid arguments: 1, 4 and 6 are valid arguments. 1 edge moves on. Some claim that induction as a principle is
consists of true premise with true conclusion. 4 consists falsifiable because it is based on human observations.
of false premises with true conclusion. 6 says that false While inductive reasoning is not 100 per cent accurate
premise with false conclusion. all the time as a little doubt is always there in inductive

M06_MADAN 07_65901_C06.indd 4 27/12/22 8:17 PM


Logical Reasoning 6.5

reasoning. But still inductive reasoning a relatively ‘quick Inductively strong arguments may have:
way to make decisions. In most of situations in life, sav-
ing time is more important than being accurate and fact • True premises, true conclusion.
based always (as is the objective of deduction). • False premises, false conclusion.
The future examples may be highly uncertain. Inductive • False premises, true conclusion.
strength the attribute of inductive arguments. Inductively strong arguments cannot have:
1. In Inductive reasoning, we draw the most probable • True premises, false conclusion.
(or say ‘the best possible’) conclusion. These conclu-
sions are suggested by set of observations and expe- To summarize, a strong inductive argument is one
riential (gained through senses) statements. The where it is improbable for the conclusion to be false, given
observations never provide us the complete idea or that the premises are true. A weak inductive argument is
solution, so we use the world ‘probability’ with induc- one where the conclusion probably would not follow from
tive reasoning. the premises, if they were true.
2. We move from ‘specific (specific premises) to ‘theory’ Cogency
(general) in inductive reasoning.
3. Inductive arguments are tentative, probable and pro- Cogency is the attribute of an inductive argument that
visional. No empirical science, natural or social can denotes the truth of its premises and its logical strength.
try to explain the social phenomena with inductive An inductive argument is cogent when:
reasoning. 1. It is inductively strong, and
4. Inductive arguments are posteriori that means that 2. It has all true premises.
knowledge is acquired ‘after experience’. There is pos-
sibility of acquiring new knowledge, therefore, it is Here is an example:
innovative.
5. An inductive reasoning is tested by strong / weak and
cogency. The argumentator is concerned with ‘prob- Premise 1: Europa has an atmosphere containing
ably’ concerned with given set of premises. oxygen.
6. Inductive reasoning has three basic features – logical Premise 2: Oxygen is required for life.
probability and a post priori and some flexibility. We
get knowledge that is after sense experience. Conclusion: Thus, there may be life on Europa.
7. Inductive arguments are not evaluated as valid or
invalid, but true or false.
This argument is cogent because:
Comparison: Addition of premises may take the change
1. It is inductively strong (if the premises were true, then
the status from inductive to deductive. Both are not
the conclusion would probably be true), and
opposed or contradictory to each other, they are rather
2. The premises actually are true.
complimentary and supplementary to one another. They
differ mostly in their ‘beginning points’. According to On the other hand, the example above concerning
Hume, deductive is akin to ‘demonstrative’, and induc- peacocks, used to demonstrate inductive strength, is
tive is ‘probable’. JS Mills criticized the deductive as ‘syl- not cogent, because it does not have all true premises.
logism’. In inductive reasoning, challenge is how to get In summary, an inductive argument is one in which it
generality, necessity, normativity etc. is improbable that the conclusion is false given that the
Truth takes the value of ‘1’ and falsity the value of ‘0’. premises are true.
According to Immanuel Kant (1724–1804), Saint
Aristotle focused on the concept of deductive reason-
ing. To him, the certainty or being closer to the truth is
Good A rguments
attained only through deductive inference. The modern The important take-away from the information on the
period philosophers, particularly Francis Bacon and J attributes of both deductive and inductive arguments
S Mill, ‘lamented the powerlessness of deduction’. They is this:
focused on ‘inductive reasoning’ to lead to new knowl- A good argument proves its conclusion and has two key
edge. This helps in the solution of problems as well. features:
1. It is logically strong.
Premise 1: Most peacocks eat oats. 2. All of its premises are true.
Premise 2: This bird is a peacock. Logical Strength
Conclusion: T
 hus, probably this bird eats oat. Logical strength is the degree of support that the prem-
ises, if true, confer on the conclusion. This ­attribute
This argument is inductively strong because if all its applies to both deductive arguments (by virtue of valid-
premises were true, then it would be highly likely or prob- ity) and inductive arguments (by virtue of inductive
able that its conclusion would also be true. strength).

M06_MADAN 07_65901_C06.indd 5 27/12/22 8:18 PM


6.6 Chapter 6

1. A good deductive argument is not only valid, but is Logical reasoning is an essential part of human thought
also sound. process, but humans are not logical reasoners.
2. A good inductive argument is not only inductively Deductive method leads to acceptance or rejection of
strong, but is also cogent. decisions. Inductive method leads to tentative conclu-
sion that itself seems to be subjective. The inductive and
David Hume stated, ‘Will the Sun rise tomorrow?’ deductive theorizing are two complementary stages of
The laws of nature say this, but still not sure about constructing any theory, in a way that induction precedes
tomorrow. deduction.
Let us look at close examples of deductive and induc- According to Aristotle, the difference between deduc-
tive arguments: tive and inductive arguments does not lie in the words
Deductive Reasoning used within the arguments, but rather in the intentions of
Proposition I: All vegetables contain vitamins. the arguer. If we have no information about the intentions
Proposition II: Carrot is a vegetable. of the arguer, then we check for both. That is, we assess
Conclusion: Thus, carrot contains vitamins. the argument to see whether it is deductively valid and
whether it is inductively strong.
Inductive Reasoning
Proposition I: Most vegetables contain vitamins. 1. Priori and posteriori arguments: In context of New-
Proposition II: Carrot is a vegetable. ton’s example, or specific, let us look at ‘priori’ and
‘posteriori’ arguments.
Conclusion: Thus, carrot contains vitamins.
Priori argument is prior to sense experience, thus
‘priori’.
2. In contrast, a posteriori argument (enlightenment or
Stopover new enlightenment) is gained only after sense experi-
1. In a deductive argument, conclusion is: ence has already occurred.
(a) Summing up of the premises.
(b) Not necessarily based on premises. After Newton, many other scientific observations also
(c) Entailed by the premises. add to such kinds of relationships.
(d) Additional to the premises. Types of syllogism: On the basis of proposition, syl-
logism is of four types and they are as follows:
The correct option is (c).
2. Inductive reasoning is based on or presupposes: 1. Categorical: Here, the relationship between the sub-
(a) Uniformity of nature. ject and the predicate is without any condition.
(b) God created the world. Example: I. All trains are planes. II. All dogs are
(c) Unity of nature. animals.
(d) Laws of nature. Within syllogism, our focus is on categorical
syllogism.
The correct option is (a). All told, there are exactly 256 distinct forms of cat-
3. A deductive argument is valid if: egorical syllogism: four kinds of major premise mul-
(a) Premises are false and conclusion is true. tiplied by four kinds of minor premise multiplied by
(b) Premises are false and conclusion is also false. four kinds of conclusion multiplied by four relative
(c) Premises are true and conclusion is false. positions of the middle term.
(d) Premises are true and conclusion is also true. 2. Hypothetical: The relationship between the subject
The correct option is (d). and the predicate is asserted conditionally.
4. Structure of logical argument is based on: For example, if it rains, he will not attend.
(a) Formal validity 3. Disjunctive: In a disjunctive proposition, the asser-
(b) Material truth tion is of alteration.
(c) Linguistic expression Example: I. Either he is courageous or he is strong.
(d) Aptness of examples 4. Relational: Here, the relation between the various
terms is shown in an order:
The correct option is (a). Example: a > b > c > d; so a > d (conclusion).

Stopover Stopover
Which of the following is the characteristic feature of
Which of the following is an analogous statement?
an argument? [December 2019]  [2019]
(a) It is either valid or invalid. (a) Man is like God.
(b) It is neither valid nor invalid. (b) God is great.
(d) It is either true or false. (c) Gandhi is the father of the nation.
(d) It is neither true nor false. (d) Man is a rational being.
The correct option is (a). The correct option is (a).

M06_MADAN 07_65901_C06.indd 6 27/12/22 8:18 PM


M06_MADAN 07_65901_C06.indd 7
Logical Reasoning

Argument terminology Determine validity or strength:


If we assume the premises are true,
does the conclusion follow? Sound: Valid and
Sound
All premises ‘True’

Valid

Unsound
Deductive

Deductive: Conclusion
necessarily/certainly Invalid Unsound
follows from premises

Argument
Cogent: Strong and
Cogent
Argument: All premises ‘True’
• Collection of statements
Strong
(Premises) intended to
support or infer a claim
Uncogent
(Conclusion)
Inductive
• Each statement has a
truth value either ‘True’
or ‘False’ Inductive: Conclusion
follows from premises Weak Uncogent
with some probability
Figure 6.3 Argument Terminology
6.7

27/12/22 8:18 PM
6.8 Chapter 6

Proposition
Animals
A proposition is a sentence that makes a statement and
gives a relation between two or more terms. In logical rea-
soning, any statement is termed as a proposition. Cats
A proposition is assumed to be true and from which a
conclusion can be drawn. The statement, ‘All cats are lem-
ons’ is assumed to be true as a proposition (or premise),
but we all know that cats and l­ emons are entirely different
entities. Figure 6.4
Proposition consists of the following four parts: 2. Universal Negative (E): It denotes exclusion.
1. Quantifier: All, no, and some. They specify a quan- Form: No S is P.
tity. ‘All’ and ‘no’ are universal quantifiers, and ‘some’ Example: ‘No fishes are birds’ would be a universal
is a particular quantifier. negative.
2. Subject (S): About which something is being said. Distribution: Both subject and predicate. Here, an
3. Predicate (P): Something that affirms or denies the entire class of predicate term is denied to the entire
subject. class of the subject term.
4. Copula: Copula reflects relation between the subject
and predicate.
Fish Birds
Quantifier + Subject + Copula + Predicate

Examples: Figure 6.5


All bats are boys. Particular proposition: A particular proposition can also
Some players are doctors. be divided into two parts.
1. Particular Positive (I): It denotes ‘partial inclusion’.
Quality: Categorical propositions can have one of the two Form: Some S is P.
qualities, affirmative or negative, which have been clari- Example: Some men are foolish.
fied through ‘classification of proposition’. Distribution: Neither the subject nor the predicate.
In the example, the subject term, men, is used not for
Classification of Propositions all, but only for some men and similarly, the predicate
Propositions are basically of two types, universal and term, foolish, is affirmed for a part of subject class. So,
particular. Universal proposition is further divided into both are undistributed.
two parts.
Foolish
1. Universal Positive or Affirmative (A): It denotes
inclusion. Men
Form: All S is P, where S is the subject and P is the
predicate. Figure 6.6
Example: ‘All cats are animals’. It is basically about
2. Particular Negative (O): It denotes partial exclusion.
inclusion.
Form: Some S is not P, or not every S is P.
Distribution: It distributes the subject only. In the
Example: ‘Some birds are not carnivores.’
above statement, cats are distributed in animals.
Distribution: Only of the predicate.
  Predicate is not interchangeable with the subject
while maintaining the validity of a proposition. We Table 6.1 will help the candidates compare major
cannot say that all animals are cats. aspects of different forms of a proposition.

Table 6.1 Types and Main Characteristics of Propositions


Sign Statement form Examples Quantity Quality Distributed

A All S are P All politicians are liars. Universal Positive Only subject

E No S are P No politicians are liars. Universal Negative Both subject and predicate.

I Some S are P Some politicians are liars. Particular Positive Neither subject nor predicate.

O Some S are not P Some politicians are not liars. Particular Negative Only predicate

M06_MADAN 07_65901_C06.indd 8 27/12/22 8:18 PM


Logical Reasoning 6.9

A word of caution is required. Sometimes predicate carries them to draw a conclusion. For example, in a previous
negative force. But it does not make the quality of proposition NET examination, the statements (i) ‘only graduates are
negative. For example, ‘dishonest’, non-natural, etc. consti- eligible for this post’ and (ii) ‘most rickshaw pullers are
tute terms in their own right. They have nothing to do with graduates’ were given. These types of statements need to
the quality of proposition. Consider these two propositions: be converted to their logical form, i.e., quantifier + subject
1. Shakuni is dishonest. + copula + predicate, as discussed earlier. The rules of
2. Telepathy is a non-natural phenomenon. reduction can help in solving these types of problems.
These propositions are positive (affirmative) only. It 1. A-type Propositions: Statements starting with words
means that a proposition is negative only when a negative ‘each’, ‘every’, ‘any’, etc., are to be treated as A-type
word is a part of copula. However, quantity of proposition propositions (starting with all).
needs elaborate explanation.
Original sentence Logical form

Parts of Categorical Propositions Every man is liable to All men are persons who
commit error. are liable to commit
There are three parts of statements in categorical syllogism, mistakes.
the major premise, the minor premise, and a conclusion.
Each premise has one term in common with the conclusion. Each student All students are ­persons
­participated in the who ­participated in the
Parts Example event. event.
Major premise All humans are mortal.
Any one of the Indians All Indians are laborious.
Minor premise All Greeks are humans. is laborious.
Conclusion All Greeks are mortal. Only Indians are All students of this
students of this college. ­college are Indians.
1. Major Premise: Predicate of the conclusion is called
as the major term. The premise containing a major The honest alone are All successful persons
term is called a major premise. In the example, mortal successful. are honest.
is the major term.
 ote: Sentences with singular term or definite singular
N
2. Minor Premise: The subject of the conclusion is called
term without the sign of negation are also to be treated
minor term. The premise containing a minor term is
as A-type proposition. For example, ‘Ram is mortal’.
called a minor premise. In the example, Greeks is the
minor term. 2. E-type Propositions: Sentences with singular term
3. Middle Term: One term common in both the premises or definite singular term with the sign of negation are
is called middle term. It is not a part of conclusion. In to be treated as E-type propositions. Sentences begin-
the example, humans is the middle term. ning with the words like ‘no’, ‘never’, and ‘none’ are to
  For practical purpose, we can put the statements in be treated as E-type propositions.
the following form:   ‘Men are never perfect’ is ‘No men are perfect’ in its
logical form.
Statement 1:
3. I-type Propositions: Affirmative sentences with
A B words like ‘a few’, ‘certain’, ‘most’, and ‘many’ are to
B C be treated as I-type propositions.
Conclusion:
A C Sentence Logical form
According to our above discussion, A is the minor A few men are present. Some men are present.
term, C is the major term, and B is the ­middle term.
4. Conclusion: In conclusion statement, first term or Most of the students are Some students are
(subject) is the subject of the first proposition and laborious. laborious.
second term (or predicate) is the predicate of the sec-
ond proposition. Few men are not selfish. Some men are selfish.
Certain books are good. Some books are good.
Deductive Inference and Syllogism
Many Indians are Some Indians are
Conversion of Common Statements into their religious. religious.
Standard Logical Forms All students of my class, Some students of my
This step depends upon the requirement to do so. In except a few, are well class are well prepared.
logical reasoning or syllogism problems, the common prepared.
language sentences may have to be converted into their
logical standard form before we can apply logic rules on The poor may be happy. Some poor people are
happy.

M06_MADAN 07_65901_C06.indd 9 27/12/22 8:18 PM


6.10 Chapter 6

4. O-type Propositions: A negative sentence that begins are mortal. The immediate inference involves two and only
with a word like ‘every’, ‘any’, ‘each’, or ‘all’ is to be two terms: men and mortal. We have two methods–Eductions
treated as an O-type proposition. and ‘square of opposition’. First we are discussing Eductions
and then the ‘traditional square of opposition’.
Sentence Logical form
1. By Implication: If a given proposition is A type, then it
Every man is not rich. Some men are not rich. also implies that I-type conclusion must be true.

Certain books are not Some books are not Statement Implication of statement
readable. readable.
All chairs are tables Some chairs are tables
Most of the students are Some students are not (A-type). (I-type).
not rich. rich.
No chair is table Some chairs are not tables
Some men are not above Few men are above (E-type). (O-type).
temptation. temptation.
  Looking at the proposition again, when we say
5. Exclusive proposition: that ‘All chairs are tables’, it implies that ‘Some chairs
(a) In exclusive propositions, the subject is qualified (we are presently concerned with) are tables’. This is
with words such as ‘only’, ‘alone’, ‘none but’, or based on our knowledge that some is a part of all.
‘no one else but’.   Similarly, we can say that an E-type proposition also
(b) Here, the quantity is not explicitly stated. implies an O-type conclusion. If we say that ‘No chair
(c) They can be reduced to A, E, or I types by first is table’, we are absolutely sure that ‘Some chairs are
interchanging the subject and the predicate, and not tables’.
then replacing words like ‘only’ or ‘alone’ with The immediate inferencing by implication is quite
‘all’. similar to the concept of ­ sub-alternation also dis-
If the exception is definitely specified as in case of, ‘All cussed under Squares of Opposition.
metals except mercury are solid’, then the proposition is 2. By Conversion: First of all, let us be familiar with the
to be treated as universal (All non-mercury metals are few terms.
solid). In case the exception is indefinite, as in case of, Convertend: The original proposition.
‘All metals except one are solid’, the proposition is to be Converse: The new proposition.
treated as particular. Conversion: The process itself.
The nature of proposition depends upon the context The process consists of two steps. The first step is
as well. For example, ‘Students are present’ is reduced to, interchanging the subject and the predicate, the sub-
‘Some students are present’ (I type). ject will become the predicate, and predicate will
become the subject.
In certain cases, the predicates are qualified by words
The second step is to change the type of the given
such as ‘hardly’, ‘scarcely’, and ‘seldom’, but quantity is
proposition to the pattern given in Table 6.2.
not specified. Such propositions should be treated as par-
ticular negative. For example, ‘Businessmen are seldom These conversion rules are helpful, not only for immediate
honest’ is an irregular proposition. It is reduced to, ‘Some inference but also for mediate inference, depending on the
businessmen are not honest’. nature of the problem and answer choices. Thus, candi-
If such a proposition contains the sign of negation, dates are expected to learn the conversion rules by heart.
then this proposition is to be treated as an I-type proposi- Important Note: In NET examination, a question is fre-
tion. quently asked only about conversion.
For example, ‘Businessmen are not seldom honest’ is to
be reduced to ‘Some businessmen are honest’, which is an Table 6.2 Conversion Table
I type proposition. This is so because it involves a double
negation, which is equivalent to affirmation. Types of Statements Valid Conversion
As we discussed earlier, deductive inference problems Universal Positive (A) Only Particular Positive (I)
are basically of two types. All chairs are tables. Some tables are chairs.
1. Immediate inference Some chairs are tables.
2. Mediate inference Universal Negative (E) Universal Negative (E)
No chairs are tables. No tables are chairs.
I mmediate Inference Particular Positive (I) Only Particular Positive (I)
Here, the ‘conclusion’ is drawn only from ‘one given proposi- Some chairs are tables. Some tables are chairs.
tion’ in an argument. It means that there are two propositions
in total–one premise and a conclusion. For example, from “all Particular Negative (O) No conversion
men are mortal,” we can immediately deduce that some men Some chairs are not tables.

M06_MADAN 07_65901_C06.indd 10 27/12/22 8:18 PM


Logical Reasoning 6.11

Contrary Opposition
Stopover
What can be concluded from the given statement, ‘Some Every S is P No S is not P
men are honest’. Which of the following is true? If one is T the other must be F, but both can be F
(a) Some honest people are not men.
(b) All honest people are not men. A E
(c) Some honest people are men.

descend with truth, rise with falsity


(d) None of the above

descend with truth, rise with falsity


Solution: This statement is particular positive statement.
Hence, according to Table 6.3, it can be converted into

Subalternation

Subalternation
Particular Positive only.
Contradictory Opposition
Table 6.3  nswer Choices and Justification as per
A If one is F the other must be T,
Conversion Table If one is T the other must be F
Answer choices Justification
(a) Some honest Particular Negative, hence
people are not men. eliminated.
(b) No honest people Universal Negative, hence
are men. eliminated. I O
(c) Some honest Particular Positive, hence this If one is F the other must be T, but both can be T
people are men. is the correct answer.
Some S is P Some S is not P
(d) None of the above Not applicable
Sub-contrary Opposition
Thus, (c) is the correct answer.

1. The same is the case with E (Universal Negative)


and I (Particular Positive) that again differs in
‘Square of Opposition’–An Easy quantity and quality.
2. Two propositions are contradictories if one is
Approach denial or negation of the other. They can’t both be
This is also a part of immediate inference. We need to look true OR both be false.
at diagram 6.1 on page 6.2. Universal mean ‘All’ and ‘No’ 3. The contradictories exist in two forms – 1. A and
are given as subjects in the propositions. Affirmative means O 2. E and I
positive, it indicates presence. Negative means ‘No’ and A O E I
‘Not’ in the above propositions. Particulars means ‘some’.
We have tried to explain the contradictories, contrar- A All men are honest E No men are honest
ies, sub contraries and sub alterations. These have been
put in the diagram as well. A has relationship with E, I and O Some men are not honest I Some men are honest
O. E has relationship with A, I and O. So is the case with I
and O. Their network makes up the square of opposition. ‘X’ in the middle of square of oppostion represents
The sqaure of opposition tells us the relationship between contradictories, with opposite truth values. This can
quantiaty, quality or both. be easily remembered.
If we take a horizontal split in the middle across the square
of opposition, then A represents Universal Affirmative and E 2. Contrary Opposition
Universal Negative. Below that split, ‘I’ represents Particular Two universals (A and E) can’t be true together. If A is
Affirmative and O Particular Negative. given to be True, then E is False. If A is given to be false,
Similarly, assume a vertical line Affirmatives and then O will be doubtful. The vice versa is also applicable.
Negative across on left hand and right hand sides. A E
1. Contradictory Opposition A – All men are honest.
These categorical propositions differ from each other
E – No men are honest.
both in quantity and quality. For example, between A
type and O type, A is Universal Affirmative and O is Since God (or Truth) is assumed to be at the top of dia-
Particular Negative. Here, Universal and Particular grams. We assume that God is one, there can’t be two
differ in quantity. Also, Affirmative and Negative dif- Gods. In a similar manner, both of the contraries (A and
fer in Quality. E) can’t be true simultaneously.

M06_MADAN 07_65901_C06.indd 11 27/12/22 8:18 PM


6.12 Chapter 6

3. Sub-contrary Opposition 2. On the second level, there are F T F T. If E is True, then


Two particulars can’t be true together. If ‘I’ type is we will go to the second level, and see that A is F, I is
given to be True, ‘O’ type is doubtful. If ‘I’ is given to F and O is True. Thus, positions of T and F should be
be true, the other will be false. The vice versa is also ascertained from the diagonals.
applicable. It means that both won’t be true together. 3. If E is False, we should go to the third level where A is
Further, both terms have the same quantity, both D (doubtful), I is True and O is doubtful. If I is true we
‘I’ and ‘O’ types are particular (same quantity). They should look at third level as benchmark (and NOT to
differ on ‘quality’ terms only. For example, ‘I’ is of level 1).
affirmative quantity, then we know that ‘O’ type is of
negative quantity). 4. If A is False, then E and I are Doubtful. O is true in that
case.
I O Now look at the two recent questions.
Question 1 (June 2022 Exam)
Example:
If the statement ‘some plants are not carnivorous’ is
Some men are honest. given as false, then which of the following could be
Some men are not honest. immediately inferred from it ?
4. Sub-alternation A. ‘some pants are carnivorous’ is true.
When two propositions differ only in quantity (one B. ‘some plants are carnivorous’ is false.
is universal and other is particular), the opposition is C. ‘No plants are carnivorous’ is false
called sub alteration. D. ‘All plants are carnivorous’ is true.
(a) B, C and D
A I (b) A and D
E O (c) A and C
(d) A, C and D
Example Explanation:
Now look at the question statement – ‘Some plants
A All men are honest E No men are honest are not carnivorous’, that is basically O – type. This is
given to be false.
O Some men are not honest I Some men are honest A. ‘some pants are carnivorous’ denotes I-type –
given to be True in question.
B. ‘some plants are carnivorous’ again denotes I- type
Shortcut Method For ‘Square of – given to be False
O pposition’ thorugh T ruth T able C. No plants are carnivorous E-type – given to be
False
D. All plants are carnivorous denotes A-type – given
Proposition A E I O to be True
If O-type is false, then looking horizontally on Truth
Level 1 T F T F Table given above, we can say that
I-type is True – that denotes that option A is a part of
answer.
Level 2 F T F T
Similarly E as False, that is option C is a part of answer.
A as True, that is option D is also part of answer.
Level 3 D F T D
That means that A, C and D are part of answer. Thus
overall, (d) is the answer.
Level 4 F D D T Question 2 (June 2022 Exam)
If the statement ‘no birds are mammals’ is given as
There may 1-2 questions on true and false. Thus, we are true, what could be immediately inferred from it ?
providing a systematic diagramatic solution for this. The A. ‘all birds are mammals’ is false.
difficult relationships among A, E, I and O become easy B. ‘some birds are mammals’ is false.
with the main diagram. The T stands for Truth, F for False C. ‘Some birds are not mammals’ is false
and D for doubtful. D. ‘Some birds are not mammals’ is true.
1. In the first level, there is T F T F. Suppose, A is True, (a) A, B and C only
then E is false, I is True and O is False. (b) A, B and D

M06_MADAN 07_65901_C06.indd 12 27/12/22 8:18 PM


Logical Reasoning 6.13

(c) A and D 2. The obverse of ‘Some singers are males” is ‘Some


(d) A and C singers are not non-males.’
2. Contraposition
Explanation: Contraposition is a kind of immediate inference in
Now look at the question statement – ‘No birds are which from a given proposition we infer another
Mammals’ is Universal Negative that is E. As it is given proposition, having its subject the contrary of the
to be true, come to the second row where given predicate.
For example, the contrapositive of ‘All crows are birds’
A is False–it means that All Birds are Mammals is False. is ‘All non-birds are non-crows.’
I is False–it means that Some Birds are Mammals is False 3. Inversion
O is True–Some Birds are Not Mammals is True Inversion is a kind of immediate inference in which
from a given proposition we infer another proposi-
Thus, as per question A, B and D are part of answer. tion, having its subject the contradictory of the given
Thus, (b) is the correct option. subject.
For example, the inversion of ‘all men are mortal’ is
Stopover ‘Some not-men are not-mortal.’
1. By which of the following propositions, the proposi-
tion, ‘Some men are not honest’ is contradicted? Methods of Mediate I nference
(a) All men are honest. Mediate inferences (syllogisms) have three and only
(b) Some men are honest. three terms. There are basically two approaches to solve a
(c) No men are honest. syllogism problem, namely
(d) All of the above. 1. Analytical method
The answer is (a). 2. Venn diagrams
2. Two propositions that have the same subject and pred- The candidates may prefer Venn diagram method to
icate terms, but are different in quality are termed as: analytical method as they find it easier. In this book,
(a) Contradictory (b) Contrary there are many illustrations using both the methods.
(c) Subaltern (d) Subalternation As questions are frequently asked from the analytical
The answer is (a). method in NET examination, the candidates should
3. If two propositions are related in such a way that they be well versed with analytical method as well.
cannot be true together although they may be false Candidates are advised not to rely exclusively on Venn
together, then such a relationship is considered to be: diagrams as they can be ambiguous at times. As many ques-
 [2019] tions based on theory are expected, analytical method can
(a) Contrary (b) Contradictory reinforce our understanding about the concepts.
(c) Subaltern (d) Sub-contrary Analytical Method for Mediate
The answer is (a).
Inference Problems
4. ‘All students clear their examinations’ and ‘few stu-
dents did not clear their examination’. [2019] The basic steps to solve syllogism problems are
This is an instance of: 1. Alignment of statements
(a) Contradictories (b) Superaltern 2. Drawing conclusions
(c) Subaltern (d) Contraries
Depending on the nature of the problem, it can entail
The answer is (a). two additional steps also. Table 6.4 shows the steps
needed in analytical method for mediate inference.

Mediate Inference Table 6.4  teps in Analytical Method for Mediate


S
Inference Problems
In addition to conversion, let us look at other three cases
of mediate inference. These are also called as ‘Eductions’. Step I Alignment of the propositions—standard
The ‘square of opposition’ is also considered as a part of it format.
and has been discussed. Step II Draw conclusion by use of the table.
1. Obversion
Step III Check for immediate inferences.
Obversion is a kind of immediate inference in which
there is a change in the quality of the given proposi- Step IV Check for complementary pair if steps II and
tion, while its meaning remains unchanged. III fail.
Examples:
1. The obverse of ‘All tigers are insects” is ‘No tigers The requirement of steps I, III, and IV depends upon the
are non-insects’. question.

M06_MADAN 07_65901_C06.indd 13 27/12/22 8:18 PM


6.14 Chapter 6

Step I: Alignment of the Propositions As books is the common term, they are in the form A
It consists of two steps. Firstly, make sure that there are to B and C to B. The first statement does not require any
exactly three terms given in both the statements. In case, change. As the second statement is in particular positive
the number of terms is different, we need not go fur- (I-type), this can be changed to I-type only according to
ther, as there will be no conclusion. Secondly, we check conversion table given earlier. The second statement will
whether the propositions are in standard form or not. become, ‘Some books are stationery’.
For practical purposes, the following format can be Now, propositions are properly aligned, i.e., ‘Some
used as a standard. pens are books’ and ‘Some books are stationery’. We now
move to Step II.

Minor (or first) term A B


Example 3
Middle term (major or third) term B C Statements
1. No van is house.
A, B, and C used above denote the first, second, and third 2. All boxes are house.
term, just for quick representation of terms while solving
practical questions. Please note that this A (used for first Here, the common term, house, is the predicate in both
term) is different from A used for universal affirmative. propositions. Here, we have to alter the first proposition
As discussed earlier, in the conclusion statement, the and also change the order to bring it to the form A to B
first term (subject) is the subject of the first proposition and B to C.
and second term (predicate) is the predicate of the second
proposition. This fact becomes the basis for the alignment 1. All boxes are house.
of propositions. 2. No house is van.
In case, the problem is in the standard form, we can Now, the predicate of the first proposition is the subject
directly move to Step II. of the second statement.
If one or both propositions are not given in the stan-
dard format, align them by
Example 4
1. Converting the first or second statement or both Statements
2. Changing the order of premises
1. All boys are tigers.
We can make it clear through the following examples. 2. Some tigers are cowardly.
It is important to remind at this stage that sometimes the
words ‘mostly’, ‘generally’, ‘only’, and so on are mentioned in
one or both the statements. Initially, we convert them into Solution: Here, the middle term, tiger, is the predicate
logical form before doing their alignment, if required. This in first proposition and the subject of the second proposi-
has been discussed separately under ‘Converting common tion. No alignment is required here.
language statements into their logical Form’ on Page 6.9. After aligning the statements among themselves, we
can move to Step II.

Example 1
Statements Concept box
IEA Rule
1. Intelligent alone are laborious.
2. Most of the girls are smart. There can be confusion while aligning a pair of state-
ments, where the statement should be given prior-
ity in terms of conversion. For example, if there are
These statements should first be converted into logical two statements, A-type and I-type, which should be
forms according to the rules for logical form. converted so that our purpose of getting the standard
form is achieved? The IEA rule helps in such decision-
1. All smart are laborious. This is in the form B to C.
making.
2. Some girls are intelligent. This is in the form A to B.
If first statement given is of A-type and second is of
Just by changing their order, we can align them. After I-type, then as per IEA rule, the I-type statement should
alignment is done, we move to Step II. be given priority for conversion. Similarly, in case of
confusion between E-type and A-type, E-type should
be given priority over A-type.
Example 2
Statements
1. Some pens are books. Step II: Applying Syllogism Rules
2. Some stationery are books. After ensuring that propositions are in a standard
format, we apply syllogism rules to draw conclusions.

M06_MADAN 07_65901_C06.indd 14 27/12/22 8:18 PM


Logical Reasoning 6.15

Table 6.5 Rules of Syllogism


Proposition I (A to B) Proposition II (B to C) Conclusion Summarized form
Universal Positive (A) Universal Positive (A) A+A=A
Universal Positive (A)
Universal Negative (E) Universal Negative (E) A+E=E
Universal Positive (A) Particular Negative (O) E + A = O*
Universal Negative (E)
Particular Positive (I) Particular Negative (O) E + I = O*
Universal Positive (A) Particular Positive (I) I+A=I
Particular Positive (I)
Universal Negative (E) Particular Negative (O) I+E=O
*In this case, the flow is from C to A, and not from A to C, as in all other cases. (Please refer Table 6.5. A, B, and C stand
for first, middle, and second terms, respectively.)

After aligning the statements, as per our discussion in conclusion is the converse of the first statement, so (B)
Step I, the conclusion may be drawn by using Table 6.5, follows. Hence, (B) is the answer.
where the rules of syllogism are mentioned.
Question 2
No definite conclusion can be drawn for other combi-
Statements
nations like A + I or O + A, which have not been mentioned
1. All birds are men.
in the above table. In general, we can say that two nega-
2. All crows are birds.
tives (E + E, E + O, O + E, or O + O) do not lead to any
conclusion. Conclusions
Two particulars also do not lead to any conclusion. A. All crows are not men.
Statements: B. Some men are not crows.
  I. All chairs are tables. (A-type) Explanation
II. All tables are furniture (A-type) Step I: The middle term is birds. A close observation indi-
Conclusion: cates that the statements are in the form B to C and A to B.
All chairs are furniture. (A + A = A). After swapping, the statements will become ‘All crows are
birds’ and ‘All birds are men’.
Now, consider Example I as discussed in Step I.
Step II: The conclusion should be A + A = A (universal
1. Some pens are books. (I-type) positive). The conclusion is ‘All crows are men’. So (d) is
2. Some books are stationery. (I-type) the answer.
No conclusion as I + I = No conclusion. Question 3
Now, consider some examples from NET previous Statements
years’ exams. In each of the following questions (1–3), 1. All boats are boys.
two statements are followed by two conclusions, A and B. 2. All boys are lamps.
Assuming that the given statements are true even if they
are at variance with commonly known facts, pick up one Conclusions
of the following answer choices which you think is correct. A. All lamps are boats.
(a) If only conclusion A follows. B. All boats are lamps.
(b) If only conclusion B follows. Solution
(c) If both A and B follows. Step I: Statements are in the standard form, A to B and B
(d) If neither A nor B follows. to C. The common term, boys, is the predicate of the first
proposition and the subject of the second proposition. So
Question 1 no alignment is required.
Statements
1. Some doctors are fools. Step II: A + A ⇒ A
2. He is a doctor. The subject of the conclusion will be the subject of the
first statement, and the predicate of the conclusion will
Conclusions
be the predicate of the second statement. The common
A. He is a fool.
terms will disappear. So, the conclusion is ‘All boats are
B. Some fools are doctors.
lamps’. Thus, only conclusion 2 follows and (B) is the
Solution answer.
No conclusion can be drawn from the two particular Now solving the problem through Venn diagram
affirmative propositions. So (A) does not follow. Second solution.

M06_MADAN 07_65901_C06.indd 15 27/12/22 8:18 PM


6.16 Chapter 6

According to Statement I, ‘All boats are boys’. Choices


(a) I and II are valid.
Boys (b) II and III are valid.
(c) I and III are valid.
Boats (d) None of the above.
Solution
Figure 6.7 I + I = No conclusion
Two particulars don’t to any conclusion, but after immedi-
According to Statement I and Statement II, the Venn dia- ate inference, we find that (i) and (ii) are valid. So, option
gram looks as follows: (a) is the answer.
Lamps Step IV: Checking for Complementary Pair
Boys (If Required)
Boats Check for complementary pairs if Steps II and III fail. A
complimentary pair is a pair of contradictory statements,
and both cannot be true simultaneously.
We can call a pair as a complementary pair if:
Figure 6.8 1. The subject and predicate of both the sentences are
Looking at the Venn diagram, we can say that second con- the same.
clusion, ‘All boats are lamps’ is correct. 2. They are I + O or A + O or I + E type pairs, which have
been discussed below.
Question 4
Statements I + O type A + O type I + E type
1. All lemons are balls.
2. No bats are lemons. Some chairs are All chairs are Some chairs are
tables. tables. tables.
Conclusions
A. Some balls are not bats. Some chairs are Some chairs are No chair is a
B. Some bats are lemons. not tables. not tables. table.
Solution Note: Sometimes, the converse of the derived conclusions
By changing the order of the statements itself, we can is among answer choices.
align the sentences. The aligned pair is:
No bats are lemons. So (a) is the answer.
All lemons are balls.
E + A = O*. So, the conclusion is, ‘Some balls are not Concept box
bats’.
A Snapshot–Golden Rules of Syllogism
Note: In all the questions discussed previously, Step III and To sum up all the discussion, some golden rules have
Step IV are not required as per the answer choices. emerged to solve the syllogism problems. These are in
continuity with the earlier discussion.
Step III: Checking for Immediate Inferences
1. Every deduction should contain exactly three
(If Required)
terms.
We can check the conclusion (or even statements) for 2. The middle term (present in both the premises)
immediate inference as per answer choices. Usually, in must be distributed at least once.
this case, there are more than two conclusions. Even 3. If one of the premises is negative, then the con-
in case of two conclusion questions, we can go for clusion must be negative (will have the words ‘no’
this step. or ‘not’).
Let us discuss one comprehensive example. 4. If one of the premises is particular, then the conclu-
Statements sion must be particular (will have the words ‘some’,
1. Some tables are chairs. ‘few’, ‘many’, etc.).
2. Some chairs are furniture. 5. If both the premises are particular, then no conclu-
sion can be drawn from the given premises.
Conclusions 6. If both the premises are negative, then no conclu-
I. Some chairs are tables. sion can be drawn from the given premises.
II. Some furniture is chair. 7. A term that is not distributed in the premises can-
III. All tables are furniture. not be distributed in the conclusion.

M06_MADAN 07_65901_C06.indd 16 27/12/22 8:18 PM


Logical Reasoning 6.17

Examples of Mediate and Immediate Inference Relational A rguments


Question 1 In relationship arguments, both premises and their con-
Statements clusions are relational proposition. There are two char-
1. All movies are stories. acteristics of a relation—relation to itself and to ­others.
2. All stories are surprises. Deductive reasoning is also sometimes ­dependent on the
validity of relational arguments. In NET examination,
Conclusions questions have been asked on relational arguments.
A. All movies are surprises. These are quite easy to understand.
B. Some surprises are movies.
Symmetry
First, let us consider only the statements. The ­sentences
are already aligned. 1. Symmetrical relationship
Since A + A = A, the conclusion will be ‘All movies are sur- Example
prises’. Till this point, it is a question of mediate inference. A is equal to B.
If we convert this conclusion (immediate inference), So, B is equal to A—valid.
we get, ‘Some surprises are movies’. Hence, both the con- It is a ‘symmetrical relationship’.
clusions given in the question are true. 2. Asymmetrical relationship
Question 2 Example
Statements A is greater than B.
1. Some rooms are lamps. So, B is greater than A—invalid.
2. Some lamps are tubes. 3. Non-symmetrical relationship
Conclusions Example
A. Some rooms are tubes. A is the sister of B.
B. Some lamps are rooms. So, B is the sister of A—may or may not be valid.
B may be the brother of A.
We know that from a combination of I + I, no conclusion
can be drawn.
On converting the first statement, we get ‘Some lamps Transitivity
are rooms’, i.e., conclusion (B). 1. Transitive relation: It implies that a relation ­travels
Also, on converting the second statement, we get from A to C through B.
‘Some tubes are lamps’. This proposition is not given in Example
the conclusion part. So, in this example, conclusion (B) A is equal to B.
alone is true. B is equal to C.
Thus, we can see the importance of immediate infer- So, A is equal to C—valid.
ences in solving syllogism problems. In transitive relations, the premises are true and the
conclusion is also valid. ‘Younger to’, ‘precedes’, ‘suc-
Structure of A rguments: U se of languages
ceeds’, and ‘ancestor of’ are other examples of transi-
There are other perspectives or dimensions of structure tive relationships.
of arguments (relational arguments, symmetry, transitiv-
ity, reflexiveness, and connexity), squares of opposition 2. Intransitive relation: Here, relation does not travel
(contradictions, contraries, sub-contraries, and sub-alter- from A to C through B.
nations), definitions (stipulative, lexical, precising, oper- Example
ational, etc.), and other terms such as prejudices, facts, A is the father of B.
opinions, and advice that suggest more about the structure B is the father of C.
of arguments. The questions have been asked regularly in So, A is the father of C—invalid (false conclusion).
NET paper examination. The types of questions have been Relations such as ‘son of’ also fall in the category of
mentioned during the course of discussion as well as in intransitive relations.
practice questions theory. Candidates are expected to go 3. Non-transitive relation:
through these topics. There is one example taken from
Example
NET previous years’ paper.
A is an enemy of B.
Example B is an enemy of C.
In the expression, ‘Nothing is larger than itself’, the rela- So, A is an enemy of C—invalid or false conclusion.
tion ‘is larger than’ is: The relations such as ‘friend of’ and ‘neighbour of’ are
(a) Antisymmetric examples of non-transitive relationships.
(b) Asymmetric
(c) Intransitive
(d) Irreflexive

M06_MADAN 07_65901_C06.indd 17 27/12/22 8:18 PM


6.18 Chapter 6

Reflexiveness 5. Precising Definition: A definition developed to


The term reflexiveness is of following types. clarify a vague or ambiguous term. It is often used in
legal, scientific, or medical settings. For example, a
1. Reflexive relationship: It is between a term and virus is an infectious agent that causes small pox.
itself. Some examples are, ‘is equal to itself, ‘resem- 6. Persuasive Definition: A persuasive definition is any
bles itself’, ‘as old as’, and ‘as young as’. definition that attaches an emotive, positive, or derog-
2. Partial reflexiveness: It means establishing a rela- atory meaning to a term where it has none. This may
tionship with some other thing. Its examples are, ‘A be used as a rhetorical tool in a debate or discussion.
is as tall as B; B is as tall as C’. Hence, A is as tall as C. For example, someone against abortion may offer the
3. Irreflexive: This type of relationship cannot be held definition of ‘abortion’ as the murder of an innocent
between a term and itself. A is smaller (or greater) person during pregnancy. This definition carries a
than itself. A is west (or east) of itself and so on. negative connotation, as the term murder suggests
4. Non-reflexive: This may or may not be held between that abortion is wrongful killing and assumes that the
a term and itself. An example is, ‘A loves itself’. This aborted foetus is already a person. Such a definition
may or may not happen. is surely not appropriate in a fair debate on the moral
Connexity legitimacy of abortion, even though it might be useful
as a rhetorical tool.
This type of relationship is valid between any two terms.
7. Operational Definition: A definition that provides
For example, 3 is greater than 2, but less than 4.
a meaning to a term by specifying a measurement
Types of Definition procedure.
8. Functional Definition: A definition that specifies the
A definition is a comprehensive description of a concept
purpose or use of the items denoted by the term.
by means of known concepts expressed mainly through
verbal means. The purpose of a definition is as follows: 9. Ostensive Definition: A definition developed by
showing someone an object and attaching a word
1. To describe a concept at a given level of abstraction. to it.
2. To distinguish a concept from related concepts.   This type of definition is often used when a term is
3. To establish a relationship between the concept in difficult to define verbally, either because the words
question and the other concept in order to determine will not be understood (as with children and new
the position of the concept in the system. speakers of a language) or because of the nature of
4. To delimit a concept for the purpose of normative ter- the term (such as colours or sensations). For example,
minological work. defining red by pointing out red objects—apples, stop
The definition should be the starting point for select- signs, roses, etc.—is giving ostensive definition, as is
ing and analyzing the term. When selecting or seeking an naming.
appropriate term for a concept, it is necessary to start with 10. Analogous Definition: This definition has analogy;
a clear definition of the concept. For clarifying the concept, corresponding in some particular. A brain and a com-
its intension and its extension have to be determined. puter are analogous.
In NET examination, questions have frequently been   In biology, there is corresponding in function, but
asked on the definition of the following terms: of different origins and having evolved separately, as
the wings of birds and insects.
1. Intensional Definition: Specifying the properties or
features and also the meaning of a term. For exam- Connotative and Denotative Meanings
ple, in chemistry, water is defined as a compound of Words are not limited to one single meaning. Most words
hydrogen and oxygen, while in physics, it is defined have multiple meanings, which are either categorized as
as a liquid with freezing point of 0°C and boiling point denotative or connotative.
of 100°C.
2. Extensional Definition: Specifying the class mem- 1. The denotation of a word is its explicit definition as
bers of the term. For example, the planets of the solar listed in a dictionary. Let us consider the word ‘home’
system are Mercury, Venus, Earth, Mars, Jupiter, as an example. The denotative or literal meaning of
Saturn, Uranus, and Neptune. home is ‘a place where one lives; a residence’.
2. The expressiveness of a language, however, comes
3. Lexical Definition: It is also termed as reportive defi- from the other type of word meaning, a connotation
nition. Lexical definition is the dictionary meaning of or the association or set of associations that a word
a term, the common vocabulary of a given language, usually brings to mind. The connotative meaning of
for example, defining a book, chair and so on. home is a place of security, comfort, and family. The
4. Stipulative Definition: It is an arbitrary, specified quote ‘East or west, home is the best’ does not refer
definition. It is not used to explain the existing mean- to denotative meaning of home, but the emotions the
ing of a term, but to assign a new meaning to a term, word home evokes in most of us. By definition, syn-
whether or not it has already got a meaning. Some onyms have the same denotation or literal meaning,
examples are idioms and slangs used in English. but almost always have different connotations.

M06_MADAN 07_65901_C06.indd 18 27/12/22 8:18 PM


Logical Reasoning 6.19

Fact, O pinion, B elief , and Prejudice Belief


In these types of questions, a statement is given where the Unlike an opinion, a belief is a conviction based on cul-
candidate has to answer whether the statement is a fact, a tural or personal faith, morality, or values. Statements
prejudice, a belief, or just an opinion. such as ‘capital punishment is legalized murder’ are often
In the past, many questions have been based upon the called opinions because they express viewpoints, but are
understanding of these terms. not based on facts or evidence. They cannot be disap-
proved or even contested in a rational or logical manner.
Statement Since beliefs are inarguable, they cannot serve as the the-
English is an invaluable asset in international communi- sis of a formal argument.
cation.
Mark Analytical Reasoning
(a) If the statement is a fact. Analytical reasoning is considered to be the recent form
(b) If the statement is an advice. of logic in which almost all classical terms are covered.
(c) If the statement is an opinion. Here, a sentence, a group of sentences, or a short argu-
(d) If the statement is a prejudice. mentative paragraph covering concepts, facts, theories,
Similarly, there were statements such as: figures, and so on may be given and questions are asked
(i) Decline of the British Empire should have resulted in with regard to arguments, conclusion, inferences, impli-
the decline of English. cations, and so on. In dealing with these questions, the
(ii) Persons educated through a foreign language are sure methods generally used include inductive reasoning,
to be unpatriotic. deductive reasoning, quoting authorities and facts, find-
ings, and illustrations.
Facts
A fact is verifiable. We can determine whether it is true by Solution Approach
researching the evidence. The facts are as follows: 1. Underline the important assumptions in the case of a
1. Things known for certain to have happened. passage. Note the inferences that are both inductive
2. Things known for certain to be true. and deductive.
3. Things known for certain to exist. 2. Identify the supporting arguments.
This may involve numbers, dates, testimony and so on. 3. Note the premises of supporting argument.
For example, India became independent on 15 August 4. See the nature of questions.
1947. Facts provide crucial support for the assertion of
an argument. Nature of Questions
Based on the given information, the types of questions to
Opinion be asked include the following:
An opinion is a judgment based on facts, an honest
attempt to draw a reasonable conclusion from factual evi- 1. Assumptions and statements.
dence. Opinions are as follows: 2. Force of argument.
3. Assertion and reasoning.
1. Things believed to have happened.
2. Things believed to be true. 4. Statements (situation) and course of action.
3. Things believed to exist. Various concepts of analytical reasoning have been dis-
For example, we know that lakhs of people go without cussed below.
proper medical care in India, and so someone forms the
opinion that the country should institute national health Assumptions and Statements
insurance even though it would cost a few ­ thousand Assumptions are unstated or even unknown, but implied
crores of rupees. An opinion is potentially changeable, by the associated theory or argument. Thus, an assump-
depending on how the evidence is interpreted. Opinions tion can be termed as an implied premise. An assumption is
are debatable, but facts usually are not. defined as something which is assumed, supposed, or taken
for granted. In practical life, if something is to be conveyed,
Prejudice it is not put in words. Many things may not be said, but are
Another kind of assertion that has no place in serious taken for granted, which may be defined as an assumption.
argumentation is prejudice, a half-baked opinion based Implicit means hidden, and, therefore, implicit assump-
on insufficient or unexamined evidence (for example, tions are those assumptions that are hidden. A typical ques-
women are bad drivers). Unlike a belief, a prejudice is tion on implicit assumptions goes as follows:
testable—it can be contested and disapproved on the
basis of facts. We often form prejudices or accept them Directions (Questions 1–4): In each of the questions
from others, including family, friends, media, and so on, below, a statement is followed by two assumptions
without questioning their meaning or testing their truth. numbered as I and II. An assumption is something that

M06_MADAN 07_65901_C06.indd 19 27/12/22 8:18 PM


6.20 Chapter 6

is supposed or taken for granted. You have to consider Explanation


the statement and the following assumptions and decide The teacher warns his student in anticipation that he
which of the assumption is implicit in the statement. would stop troubling him. So, Assumption I is implicit.
The general nature of children cannot be derived from the
(a) Only assumption I is implicit.
statement. So, Assumption II is not implicit.
(b) Only assumption II is implicit.
(c) Both I and II are implicit. Example 4
(d) Neither I nor II is implicit. Statement
Of all the newspapers published in India, The Hindu has
Example 1 the largest number of readers.
Statement Assumptions
A to B – ‘In my opinion, you should undergo a training I. The volume of readership of all newspapers in India is
under an expert in order to be successful in your career’. known.
II. No newspaper in India other than The Hindu has a
Assumptions large readership.
I. B sought advice from A.
II. Experts are more competent to guide a person to be Explanation
successful in their career. It is on the basis of data that we can say The Hindu has the
largest number of readers. So, Assumption I is implied.
Explanation But, it is not possible to say that no other newspaper in
There are many instances in life when we get an advice India has a large readership. We need to define large read-
from a person without asking for it. Assumption I is not ership as well. So, Assumption II is not implicit.
definitely valid. Such decisions, as given in the statement, are taken
Only assumption II is implicit. Otherwise, A would not only after taking the existing vacancies into consider-
have advised B to get training from an expert. Therefore, ation. So, Assumption I is implicit, while II is not.
it is correct to assume that experienced people make bet-
ter guides. Force of Arguments
Argument: Earlier also we discussed about the valid-
Example 2 ity of arguments. An argument is a set of two or more
Statement premises leading to a conclusion. An argument can be
‘This multimedia CD-ROM offers you active help as you said to be valid if the premises, if true, definitely lead to
learn yoga without an instructor’ says a newspaper adver- a ­conclusion.
tisement. All scientists are intelligent people. Raman is a ­scientist.
So, Raman is an intelligent person (invalid).
Assumptions All scientists are genius. Raman is an intelligent per-
I. Everyone may not be able to get active help from a son. So, Raman is a scientist (invalid).
yoga instructor. The second argument is invalid as there is no premise
II. Aerobic exercises can be learnt with the help of a that states that all intelligent persons are scientists.
CD-ROM. Validity is the property of an argument.
Assertion and Reasoning
Explanation
In the above example, both I and II are valid. The multi- Introduction: Assertion and reasoning-type questions
media CD-ROM intends to teach yoga in the absence of an have one assertion (A) and one reason (R). We must first
instructor. This means that the absence of an ­instructor determine whether the statement is true. If statement is
is a distinct possibility for many people. Therefore, true, next we must determine whether the reason cor-
Assumption I is valid. The advertiser has come out with a rectly explains the assertion. There is one option for each
CD-ROM on yoga. It definitely implies that aerobic exer- possible outcome. These types of questions are followed
cises can be learnt by CD-ROMs. Hence, Assumption II is by four options.
also valid. (a) A is true but R is false.
Example 3 (b) A is false but R is true.
(c) Both A and R are true, and R is not the correct expla-
Statement
nation of A.
‘If you keep creating indiscipline in class, I will have
(d) Both A and R are true, and R is the correct explana-
to take a strict action against you’, a teacher warns his
tion of A.
­student.
A few examples have been discussed as given below.
Assumptions
I. With the warning, the student may stop creating Example 1
indiscipline in the class. Assertion (A): Most of the prominent places in ancient
II. All students are basically naughty. civilizations grew near rivers.

M06_MADAN 07_65901_C06.indd 20 27/12/22 8:18 PM


Logical Reasoning 6.21

Reason (R): Rivers provide water for irrigation and also (c) None of these follow
work as means of transportation. (d) All of these follow
Explanation: Here, we can use our basic general knowledge Explanation: Actions I and II are immediately required
or commonly known facts. We know that most of ancient as they are crucial in saving precious lives of the people.
civilizations grew near rivers, so, A is correct. In the exam- It may not be practicable for authorities to remove silt
ple, R is also simple and true. So, option (a) is the answer. and mud from the river body. So, only I and II ­follow, and
Example 2 hence, (a) is the answer.
Assertion (A): Tides indicate the regular and periodic rise Example 2
and fall in sea level. Indicate which of the following actions are the most
Reason (R): Tides are caused by the gravitational pull of appropriate in the situation given below.
the moon on sea level.
Situation: Two to three students in the class of a sincere
Explanation: In this case also, the concept of tides is the and devoted teacher frequently disturb him in the class
reason for their origin. Both A and R are true, so, option while teaching. He is fed up with them.
(c) is the answer.

Example 3 Courses of actions


(a) He tells the students of the class that he will not hold
Assertion (A): Mercury is the farthest planet from the sun. the classes if the disturbing students continue doing
Reason (R): Mercury is the smallest planet in the solar that.
system. (b) He suspends the disturbing students from attending
Explanation: Here, A is false as mercury is the closest to his class in the interest of the rest of the class.
sun. Hence, R is the correct option. (c) He talks to the disturbing students to find out what
makes them behave that way and what could become
Example 4 of them.
Assertion (A): Carbon monoxide when inhaled causes (d) He reports against them to the principal with the recom-
death. mendation to take strong action against them.
Reason (R): Carbon monoxide combines with haemoglobin. Explanation: (c) suggests long-term approach to deal
Explanation: The chemical composition of oxygen and car- with the issue of indiscipline in the class.
bon monoxide is the same. Carbon monoxide combines with
haemoglobin and reaches different parts of the body and
causes death. Hence, (a) is the correct answer.
Mood and Figure
Statements and Courses of Action As we have discussed earlier in this chapter, a categorical
Introduction: A course of action is a step or administrative syllogism meets the following strict qualifications to be in
decision to be taken for improvement, f­ollow-up, or a standard form.
further action in regard to a problem, policy, and so on. 1. It is an argument with two premises and one con-
On the basis of the information given in the statement clusion.
or situation, the candidate has to assume everything in 2. All three statements are categorical propositions.
the statement to be true, and then decide which of the
suggested courses of action logically follow for pursuing. 3. It contains exactly three different terms.
4. Each term is used exactly twice.
Example 1
Situation: The incessant rains that have been continu- We understand that a categorical syllogism has three
ing for the past several days have created the problem of statements – Major Term, Minor Term and Conclusion.
deluge, because the river bed is full of silt and mud. Each statement has two terms S and P. As discussed earlier
in Step I on page 6.11, we need to bring the statements
in the standard form. Take a note that premise 1 contains
Courses of actions
the major term, while premise 2 contains the minor term.
I. The people living close to the river should be trans-
Premise 1 is therefore called the major premise, while
ferred to a safer place.
premise 2 is called the minor premise. The standard form
II. People should be given information about the immi-
demands that the major premise (i.e., the one containing
nent danger through radio or television.
the major term) always be listed first.
III. Immediately after the reduction of water level of the
river, the silt and mud should be removed from the 1. All mammals are skinny organisms.
river body. 2. All cats are mammals.
(a) Only I and II follows. 3. Therefore, all cats (subject/minor) are skinny organ-
(b) Only II and III follows. isms (predicate/major).

M06_MADAN 07_65901_C06.indd 21 27/12/22 8:18 PM


6.22 Chapter 6

Standard form of a Syllogism

Major premise
1. Quantifier copula
(contains major term)

Minor premise
2. Quantifier copula
(contains major term)

3. Quantifier copula Conclusion

Minor Major
term term

We need to identify that there are three different terms 1. All mammals are skinny organisms.
in this argument. These are called as ‘major term’, ‘minor 2. All cats are mammals.
term’, and ‘middle term.’ The statement indicated by
‘therefore’ (or any such word) is called as conclusion. We are presenting a consolidated picture of few examples
Here, the conclusion has TWO of the three terms which which give an idea about the Subject, Predicate, Major,
are cats and skinny organisms. Mammals being Middle Minor and Middle Terms, and ultimately the “mood’.
Term will not appear anywhere in conclusion. 1. Major premise: All H are M. HAM
Now look at this arrangement. Minor premise: J is H. JAH
Major Premise: The predicate term of the conclusion Conclusion ∴ J is M. ∴ JAM
that is Major term also. That is represented by skinny 2. Major premise: All R are H. RAH
organism in the argument.
Minor premise: No R are L. REL
Minor Premise: The subject term of the conclusion Conclusion ∴ Some L are not H. ∴ LOH
that is a Minor Term. That is represented by ‘cats’ as
3. Major premise: No W is Y. WEY
given in the argument.
Minor premise: All Y is B. YAB
Middle Term: The term that does NOT appear in the Conclusion ∴ Some B is not W. ∴ BOW
conclusion (mammals)
Accordingly, we can list 64 moods. (At this stage, let
us not restrict ourselves to valid moods). There is no
Prepositions Term Preposition need to list all 64 moods. When we compute all possible
Type arrangements, we arrive at 64 moods. There are two
P1 All Mammals are skinny Major A important aspects. First, we have discovered a certain
organisms Term number of structures in which syllogistic arguments
can be constructed, and second, which we notice
P2 All cats are mammals. Minor A later, not all structures to which arguments subscribe
Term are valid. It is in this sense that the logical status of
C Therefore, all cats Mix A an argument is determined by the structure of that
(Subject / Minor) particular argument.
are skinny organisms
(Predicate /Major).
Figures of Syllogism
The Mood of this syllogism is AAA. Mood depends upon
the type of propositions (A, E, I, or O). The ‘mood’ of a Figure: The ‘figure’ of a syllogism is determined by the
syllogism is determined by the ‘quantity’ and ‘quality’ of position of ‘middle term’ as it appears both in the major
the three propositions. and in minor premises. There are four ‘Figure’ options, we
To do the ‘Figure’ task, we need to bring propositions need to pick up the correct option that depends on the
into proper standard form again. placement of ‘middle term’ in the premises statements.

M06_MADAN 07_65901_C06.indd 22 27/12/22 8:18 PM


Logical Reasoning 6.23

There is no role of ‘conclusion’, and obviously that of sub- 2. P M


ject and predicate as well. Major premise: All saints are pious SAP
S M
Figure 1 Figure 2 Figure 3 Figure 4 Minor premise: No criminals are pious CEP

M P P M M P P M
Conclusion: ∴ No criminals are saints CES
S P
S M S M M S M S

S P S P S P S P The knowledge of ‘figures’ help us to compute the total


number of possible moods. Mood is determined by the
quality and quantity of propositions. Since there are
We can draw lines through the middle terms in each of four figures in all, there is possibility of arranging 256
these four diagrams to create a collar-like shape, like categorical propositions. These are exactly what we
this: mean by moods. However, out of 256, 245 moods can be
shown to be invalid by applying the rules and corollaries.
Figure 1 Figure 2 Figure 3 Figure 4 So, we have only eleven ‘valid’ moods. Even this is not
sufficient to have a clear picture. There is no ‘figure’ in
M P P M M P P M which all eleven moods are valid. Within the framework
of traditional logic, in any given figure, only six moods are
S M S M M S M S valid in each sentence which are given below.
I. AAA, AAI, EAE, EAO, EIO, and AII
S P S P S P S P
II. AEE, AEO, EAE, EAO, EIO, and AOO
III. AAI, AII, IAI, EAO, EIO, and OAO
IV. AAI, IAI, AEE, AEO, EAO, and EIO
Now coming to a combined example of moods and figures. Moods are represented above in three ways.
All red are apples. 1. Moods in italics and bold form are called strength-
ened moods.
Some apples are green.
2. Moods in mere italics are called weakened moods.
Therefore, some greens are red. 3. All other moods are represented in normal form.
Now we need to look at the conclusion first, that indicates Strengthened Mood: This happens when the laws of syl-
that green are subject/minor and red are predicate/major. logism, permit two universal premises to yield logically
only one particular conclusion.
Sequence of Mood - A I I
Weakened Mood: This happens when we deduce par-
Therefore, apples are the middle term. ticular conclusion from two universal premises, even
when the laws of syllogism permit two universal premises
to yield logically only one universal conclusion.
P1 All reds are apples. Major Term A Pope John XXI devised a technique to remember the
method of reducing arguments from other figures to the
P2 Some apples are green. Minor Term I first figure. This technique is known as mnemonic verses.
C Some green (subject Conclusion I Dilemma: A ‘dilemma’ in logic means an argument that pre-
/ minor) are red sents an antagonist with a choice of two or more alternatives,
(predicate / major) each of which appears to contradict the original contention
and is inconclusive. Dilemma is a powerful instrument of
Thus, mood is A I I. Now looking at cross between apples, persuasion and also a devastating weapon in controversy.
we can say that it is ‘Figure 4’.

1. P M Formal and Informal Fallacies


Major premise: No soldiers are traitors SET
Fallacies are errors, but can be tricks of reasoning. Fallacy
M S is an error of reasoning if it occurs accidentally; it is a
Minor premise: All traitors are sinners TAS trick of reasoning if a speaker or writer uses it in order to
deceive or manipulate his audience.
Conclusion: ∴ Some sinners are not soldiers SOS A fallacy is ‘an argument or an apparent argument that
S P professes to be decisive of the matter at the issue, while in

M06_MADAN 07_65901_C06.indd 23 27/12/22 8:18 PM


6.24 Chapter 6

reality it is not’. Fallacies weaken arguments and in doing so, 3. Informal fallacies are more dependent on misuse of
weaken the overall strength of our paragraph or assignment. language and of evidence.
Usually, there are five common categories of fallacies,
as follows: Frequently, they may bring irrelevant information into an
argument or they are based on assumptions that, when
1. Using feelings examined, prove to be incorrect, but they may not always
2. Distracting from the argument. be easy to spot. Some moves are always fallacious, and
3. Misinformation others may be allowed on the basis of context.
4. Generalizations (to make a powerful statement).
Use of Ethos, Logos, and Pathos to Test
5. Irrelevant connections
Arguments for Fallacies
According to NET syllabus, fallacies are mainly of two To test an argument for fallacies is to focus on the con-
types, formal or informal. cepts of ethos, logos, and pathos.
Whatever its type, its use undercuts the validity and
soundness of any argument, and fallacious reasoning may • Ethos: For ethics, authority, and/or credibility.
damage the credibility of the originator of the message and • Logos: An appeal to logic.
plays with the emotions of the receiver. • Pathos: An appeal to emotion.
Ethos, logos, and pathos can be used to strengthen our
Formal F allacies argument or inappropriately manipulate an audience
Most formal fallacies are errors of logic, where the con- through the use of fallacies. Some fallacies may fit into
clusion is not supported by the premises, so it does not multiple categories. Thus, we can see that both formal
really follow the word ‘from’. Either the premises are and informal fallacies are errors of reasoning, and if
untrue or the argument is invalid. Given below is an speaker or writer relies on such fal­lacies, even uninten-
example of an invalid deductive argument. tionally, he/she undercuts their argument.

Premise: All black bugs are carnivores. Types of Informal Fallacies


Premise: All rats are carnivores. We shall now concentrate on different class of fallacies.
Conclusion: All rats are black bugs. These are also called non-logical fallacies because there
is no violation of any rule of inference as such. However,
Bugs are a subset of carnivores. Rats also are a subset of they are fallacious because in such arguments premises
carnivores. But these two subsets do not overlap, and this and conclusions are mutually irrelevant. They can also
fact makes the conclusion illogical. The argument is inva- be called as fallacies of irrelevance. Fallacy can also
lid, i.e., the relationship between the premises does not result due to ambiguity in language. Though the number
support the conclusion. of informal fallacies may run up to 300, we are discuss-
But then how do we recognize formal fallacies? ing only those which are important from examination
‘Rats are black bugs’ is instantaneously recognizable as point of view. If one particular fallacy is missing, it does
fallacious as it sounds illogical. However, that and other not guarantee the absence of other fallacies, they may
forms of poor logic play out on a daily basis, and they have exist.
real-world consequences. Below is an example of a falla- 1. Petitio Principii (Begging the Question): This is a
cious argument. very common fallacy. This is committed when in our
Premise: All Europeans are Christians. attempt to prove, we assume what has to be proved. It
simply says that something is proved on the basis of
Premise: All Russians are Christians.
itself. We start from a position and end our argument
Conclusion: All Russians are Europeans. by returning to the very same position. Petitio princi-
This argument fails on two levels: pii is sometimes defined as an immediate argument or
arguing in circle. It covers inductive principle.
1. The premises are untrue because although many For example,
Europeans and Russians are Christians, not all are. Murder is morally wrong.
2. The two ethnic groups are sets that do not overlap, but Abortion is murder.
the two groups are confused because they (largely) Therefore, abortion is morally wrong.
share one common quality. We mostly agree on the first premise, murder is mor-
ally wrong. The problem is in the second premise. Not
Informal Fallacies all individuals would agree that abortion is murder.
Informal fallacies take many forms. They are widespread However, as presented, the premise creates a pre-
in our routine lives. Informal fallacies develop when: sumption that it is valid in all cases.
2. Accident (Sweeping Generalization): This form
1. The relationship between premises and conclusion of fallacy has two types—direct fallacy and its con-
does not hold up. verse. The fallacy results due to inappropriate use
2. When premises are unsound. of generalization in both the cases. These fallacies

M06_MADAN 07_65901_C06.indd 24 27/12/22 8:18 PM


Logical Reasoning 6.25

are committed when the difference between normal 7. Argument ad Baculum: Here, of course, there is no
and special circumstances is ignored. Since it can be appeal, but threat. Again, Baculum is one method fol-
ignored in two ways, we have two types of fallacies. lowed by those who are after power or who, supported
When any norm, which applies to generalization, is by political authorities, try to enforce their ideology,
made applicable to any special case ignoring the dif- whether religious or social. Threat may be to life or
ference between them, then fallacy of direct accident property or position. Baculum has all the features of
is committed. totalitarian mindset and hence undemocratic.
  Sweeping Generalizhation can be explained with 8. Argument ad Ignorantiam: This is a commonplace
this example. Normally a sentence is read as ‘murder- fallacy committed in academic circles. When we
ers are to hanged, so all soldiers must be hanged. appeal to ignorance, we argue that the proposition
1. Murderers are to be hanged (normal circumstance). must be accepted unless someone can prove other-
2. so, all soldiers must be hanged (special wise. The argument rests not on any evidence, but on
circumstance). a lack of evidence. We are to believe the truth of the
argument because no one has disproven it.
  Here, the difference between ‘normal circumstance’   For example, God exists because reason has failed
and ‘special circumstance’ is ignored. to prove that God does not exist. The thesis is estab-
3. Converse Fallacy of Accident: It is related to earlier lished when its antithesis could not be established by
fallacy. When the norm, which applies to a special any opponent.
case, is blindly extended to general circumstances, 9. Argument ad Hominem: The arguments of ‘X’ are
the converse fallacy of accident is committed. good and supported by the good evidence. But at
  An enlightening example is the dialogue between the same time, the personality or character of ‘X’ has
Socrates and Polemarchus. When Polemarchus not been generally good. We shift our focus from the
argues that justice consists in repaying debt, Socrates premises to personality of the person putting the
promptly challenges him by demanding to know argument. This fallacy happens rarely in academic
whether justice consists in returning arms, bor- circles and mostly in political circles. Usually, rivalry
rowed from my friend, to him when I know that he is behind committing this fallacy.
has passed from sober state to disturbed state. If the Example 1: ‘Madan roots for an Indian football team.
answer had been ‘yes’, then fallacy of direct accident Clearly, he is unfit to be a police chief in a small city.’
would have been committed. If you argue in reverse
Example 2: ‘All people from Island are liars’
order then converse fallacy is committed.
10. Ignoratio Elenchi: This fallacy is quite different from
4. Argument ad Verecundiam: This type of fallacy is
other fallacies. In the case of former fallacies, the
committed when we choose irrelevant premise. It is
chosen premises are irrelevant. But in this case, we
irrelevant because the premise really does not provide
get some conclusion that is quite different from the
any support to the conclusion. This fallacy is commit-
expected or intended one. Instead of proving what is
ted when we try to get support from any person (usu-
intended, we prove something different.
ally famous and highly respected).
  ‘It is not the case of missing the bus, but it is a case
  Surely, from the logic point of view, what a person
of the bus missing the route’. It is a case of reasoning
says or does not say is irrelevant, more so when the per-
going ‘astray’.
son who is quoted is not an expert. This particular fal-
11. Complex question: Generally, complex question fig-
lacy, in most of the cases, describes those who indulge
ures prominently in legal field. Complex question is
in advertisement because in most of the advertise-
an example of clever way of manipulation in order to
ments, the models who bat for the advertising compa-
checkmate the accused in particular or opponent in
nies know nothing about the products. Yet, they speak
general. The question is framed in such a way that it
with authority, which is endorsed by others.
admits only two answers and no matter which answer
For example, ‘We have no evidence that the Rolls
is chosen, the accused walks into the trap. The ques-
Royce ever existed. They must have been so clever
tion is such that answers are hidden in it and hence it
they destroyed all the evidence.’
is impossible for anyone to construe any other answer
5. Argument ad Populum: This fallacy is committed when
to the question.
a speech appeals to emotion and stirs up love or hatred.
12. Strawman Argument: It is much easier to defeat
Generally, political speeches fall under this category.
your opponent’s argument when it is made of straw.
A classic example of this fallacy finds place in
The Strawman argument is aptly named after a
Shakespeare’s Julius Caser, when Mark Antony insti-
harmless, lifeless scarecrow. In the strawman argu-
gates the crowd to take revenge on Caesar’s killing. It
ment, someone attacks a position the opponent does
should be noted that, in such case the appeal is striking
not really hold.
and hence it is noticeable easily.
6. Argument ad Misericordiam: This is an appeal to Example 1: ‘The MP thinks we can solve all our eco-
pity. From Plato’s dialogues we understand that in logical problems by driving a Hybrid car.’
ancient Greece, the criminals followed this method Example 2: ‘Quite the contrary, the MP thinks the envi-
to escape punishment. It is doubtful whether this was ronment is such a wreck that no one’s car choice or
followed by one who was not guilty. driving habits would make the slightest difference.’

M06_MADAN 07_65901_C06.indd 25 27/12/22 8:18 PM


6.26 Chapter 6

13. Slippery Slope: This type of fallacy occurs when we 4. Composition: First fallacy consists in proceeding
assume one action will initiate a chain of events. It from parts to whole, whereas the second consists in
will result in an undesirable event at a later stage. proceeding from whole to parts. Generally, these fal-
We assume that final event is inevitable. If we are lacies are committed when the attributes are under
prone to slippery slope, we ignore the fact that there scrutiny. In the history of western philosophy we
are probably a number of other things that can hap- have a famous example of J. S. Mill, who consciously
pen between the initial event and the bottom of the committed the fallacy of composition. He said –
slope. every man desires his own happiness. All men desire
  For example, if sex education is taught in the col- the happiness of all.
lege, then students will enjoy sex. It will result in a 5. Division: Composition and division are recipro-
rash of unplanned pregnancies and sexually trans- cal fallacies. If the above-mentioned example is
mitted diseases. Students will be forced to drop out of reversed with a little modification, then it becomes
college. They will never have the chance to succeed division. No men desire the happiness of all. No man
in life. desires his own happiness. It is obvious that falla-
  It is very clear that learning about sex does not cies of ambiguity are due to wrong interpretation or
automatically mean that you will engage in sex. understanding, whereas logical fallacies are due to
Even more unlikely is the fact that merely learning wrong reasoning. Interpretation and reasoning are
about sex will force you to drop out of college. different.
14. Red Herring (Irrelevant Thesis): In Red herring, an
irrelevant topic (or fallacy) is introduced in an argu-
ment to divert the attention of listeners (or readers) Inductive Fallacy
from the original issue. This fallacy is often used in False cause: This fallacy consists in regarding an event
detective or suspense novels in literature. The listen- as a cause of given effect, when, in reality, it is not the
ers may be induced to make the false conclusions. cause. But how are we to know that the supposed cause
  For example, a teacher catches a student cheating is not the cause at all? The only way is to wait for the
during an exam. The student replies, ‘I know I have occurrence of effect, which does not follow the supposed
made a mistake. Kindly think of my parents. They cause. But suppose that it did not happen. Then, there
will feel very bad.’ The student uses a red herring in is no way of deciding against the supposed link between
his response. He tries to appeal to pity to distract his cause and effect.
teacher from the real issue. For example, a historian may claim that the cause
of India becoming independent is the World War II.
There is no way in which the sequence of events can be
Fallacies Due to Ambiguity repeated in future if this claim has to be tested. Hence,
Ambiguity is of three types, use of ambiguous words, proof in the strict sense of geometrical proof is impos-
ambiguous structure of sentence, and differing accent. sible in induction.
There is no rule in inductive logic. Hence, there is no
1. Equivocation: It may occur as a result of ambiguous
question of fallacy at all. With respect to inductive argu-
words. Good is one such ambiguous word. Consider
ments, it can only be remarked that whatever opposes an
this example. ‘Rama is good’. ‘Rama is a teacher’.
acceptable inductive argument is fallacious. Mere com-
Therefore, Rama must be a good teacher. It is one
mon sense or experience is enough to suggest what is
thing to be a good human being and something differ-
acceptable. Hence, without demanding logical proof, it is
ent to be a good teacher. This difference in the mean-
possible to decide what is acceptable and hence, what is
ing of the word ‘good’ is obliterated here. Hence, fal-
fallacious.
lacy of equivocation arises.
One advantage of knowing what fallacies are,
2. Amphiboly: It is due to the manner in which the
whether in strict sense or in loose sense, is that if we
words are combined and the hidden meaning that
know what is wrong, then we can correct mistakes or
such combination suggests. The way in which
we may refrain from making them, and this is the way
Socrates understood what the Oracle at Delphi said
knowledge grows.
and the way in which others understood the same
accounts for amphiboly. When the Oracle said that
Socrates is the wisest man in Greece, Socrates took
it to mean very differently. Stopover
3. Accent also can Lead to Fallacy: The premise ‘Everyone is going to the party. You should go too.’

emphasizes one aspect, while the conclusion empha- This inference commits which type of fallacy?[2019]
sizes another aspect. (a) Ad Populum
  For example, when Jesus in his sermon, advices his (b) Equivocation
disciples to ‘love their neighbour’, the advice could (c) Ad Verecundiam
have been misconstrued by placing emphasis on the (d) Ad Ignorantiam
word ‘your’, while, in reality, Jesus emphasized the
The correct option is (a).
word ‘neighbour’.

M06_MADAN 07_65901_C06.indd 26 27/12/22 8:18 PM


Logical Reasoning 6.27

In India, there are six orthodox schools of philosophy


Indian Logic: Means of knowledge that recognize the authority of Vedas as divine revela-
Knowledge is considered to be ‘luminous’ at the highest tion. Those who did not recognize this authority were
level, knowledge means to illuminate something. ‘Truth’ the Jains and Buddhists (both heterodox), and Charvaka
and ‘knowledge’ are similar in nature. (materialists).
After asking, ‘Can I know?’, the next question is obvi- There is much divergence of opinion among Indian phi-
ously ‘How do I know?’ or ‘What are the sources of knowl- losophers concerning the nature and scope of Pramana
edge?’ Epistemology is the study of the origin, nature, and (izek.k–source of knowledge).
limits of human knowledge. Indian philosophy divides itself into three main
Logic is the study of inference and argument. The logic periods:
and theory of knowledge of Indian systems are largely
coloured by their metaphysical tenets. Philosophy basically 1. Vedas or Vedic Period
deals with interpretation of man and nature. It is the analy- 2. Upanishad period and
sis, assessment, and exposition of the process of knowledge. 3. Later Vedic Period or Vedanta Vedic Age
In general, knowledge is gained through different 1. Vedic Period ( ): They originated in our
means. We have already discussed inductive, deductive, ancient system. They can be linked with ‘Rigveda’
and many aspects in Unit 1. Now we are learning experi- of very ancient era. The vedic Age (1500–500 BCE)
ential knowledge also that is important in Indian system. is the period between the Bronze Age and the early
Experiential Knowledge: Experiential (a posteriori) knowl- Iron Age. Veads belong to orthodox system because
edge is mainly of four types. they all have recognized the ultimate authority of the
1 Sensory Perception: This is the first and most Vedas. Vedas were assumed to be linked with influen-
obvious. Each of our five senses is like a door to the tial Brahmanical, which later met some resistance in
outside world; when we throw them open, we are the form of Jainism and Buddhism. It is also related
flooded with an endless variety of sights (visual to the Indo-Aryan tribe. The Vedic period defined the
function), sounds (auditory function), textures real nature of the soul or the philosophy about the
(tactual function), smells (olfactory function), and soul + cosmic principle (or Brahman) which repre-
tastes (gustatory function). This is closely linked with sented the ‘ultimate reality’. Its six different darshans
realist, empiricist, logical positivist and scientist. are also called ‘Shad Darshan’ ( ).
2. Introspection: directly experiencing our own mental    The post-Vedic period is a systematic period. Of
states. the systems of thought or philosophy, six became more
3. Memory: recording device that captures events that famous than the others. Gautama’s Nyaya, Kinnada’s
I experience more or less in the order that they occur. Vishishti, Kapila’s Samkhya Patanjali’s Yoga, Jaimini’s
4. Testimony: While it is convenient for us to trust the Purva-Mimamsa and Badarayana’s Uttara Mimamsa
testimony of others, the problem is that there is often or Vedanta. All of them accept the authority of the
a high likelihood of error. Vedas. He considered spiritual experience to be
greater than intellectual reason. Those who did not
The knowledge is first received through perception believe in the Vedas were Jains, Buddhists and mate-
(pratyakşa–izR;{k) or comparison (upamāna–mieku) or rialists. Vedic literature has developed in four phases,
words of sacred authority (shabda– ”kCn). later there was some separation.
Here, the aim is to study Indian logic by means of
knowledge. 1. Ved Samhita ( – code of conduct)
2. Brahmin ( )
3. Aranyaka ( )
Indian logic (Means of knowledge)
4. Upanishads ( )
The collection of mantras and hymns is called ‘Samhita’
( ) that has got four main types:
1. Rigveda ( ): It is considered to be the world’s
Orthodox (Aastik) Unorthodox first book. This is a song of praise.
Vedas (Naastik) Non vedas 2. Yajurveda ( ): Yajurveda contains prose and
poetry for the actual process of Yagya. Yajurveda is
1. Nyaya Philosophy 1. Buddhist primarily a prose text. The prose hymns uttered in the
2. Vaisheshika 2. Jainism Yagya are called Yajus.
3. Mimamsa/Purva Mimamsa 3. Charvaka (Materialists) 3. Samaveda ( ): This song-music is predominant.
4. Sankhya Philosophy It was sung by the ancient Aryans. It is the smallest in
5. Yoga Philosophy terms of size among the four Vedas, it has the highest
6. Uttara Mimamsa (Vedanta) prestige.
4. Atharva Veda ( ): This is mainly the Samhita
Figure 6.9 Indian Logic i.e. Mantra part, this Veda is also called Brahmaveda.

M06_MADAN 07_65901_C06.indd 27 27/12/22 8:18 PM


6.28 Chapter 6

Along with the praise of the gods, there are also mantras 5. Mimamsa philosophy does not accept God. The Samkhya
for medicine, science and philosophy. Due to his utterance, philosophy also creates controversy over God consider-
the nation progresses continuously without any distur- ing him to be imperfect.
bance. It is the earliest source of Sanatan Dharma, it has 10 6. Prakriti is also given priority in Sankhya philosophy.
mandalas, 1028 suktas (lwDr) and presently 10,600 man-
tras. ‘Manusamhita’ (euqlfa grk) is the most important text of The post-vedic period is systematic period, which saw
Indian practice. After the Vedas in India, the most recogni- the development of ‘orthodox systems’. Currently, we are
tion and the practice is of ‘Manusamhita’. It is also called starting with Charvaka system.
‘Manusmriti’ (euqLe`fr). It discusses all religious, social and
moral duties. The Vedas could not grow much mainly due Charvaka Materialist School’ s Views of
to the rituals of Brahmanism, but they sprouted under Knowledge ( )
the umbrella of the Aranyakas and developed a lot in the
Upanishads, which are briefly described further. Rishi Brihaspati was the founder of this school. Charvaka
is also called Lokayata, which is the Sanskrit word for
2. Upanishad period ( ): Upanishad means ‘worldly ones’ that is the meaning understood by common
that the student takes refuge in the Guru. Or get people.
the secrets of knowledge by sitting ‘near’ the Guru. As we discussed, ‘pratyaksha’ (izR;{k) is the only source
The composition of the Upanishads is believed to of valid knowledge. Only direct perception anubhava
be between 1000 and 800 BC. It could even be as ( ) is recognized. What we cannot perceive through
far back as 600 BC. During this period there was a senses must be treated as non-existent. They refute all
considerable development of Yagya culture in North other sources of knowledge—no mind, no consciousness,
India. In the Upanishads, Brahman and Atman are and then no soul. Only the physical body is real.
the central ideas. In this ‘Bhadranayak’ (Hknzuk;d) had There are four traditional elements—earth, water,
played the lead role. The total number of Upanishads fire, and air. The validity of inference is also rejected by
is considered to be 108, out of which twelve are the Charvakas. Inference is considered to be a mere leap into
most ancient and main. Of these ‘Bhagavad Gita the dark.
and Brahmasutra Upanishad’ Vedanta is the basis of We proceed from the known to the unknown, and
philosophy and ‘Maitri Upanishad’ Samkhya and Yoga there is no certainty in this, though some inferences may
philosophy are the basis of. turn out to be accidentally true. Induction is uncertain,
3. Later Vedic period or Vedanta ( and deduction is arguing in a circle. Deductive inference
): This era is between about 1400 BC to 600 BC. is vitiated by the fallacy of petition principia.
During this period, the society was divided into four Though we consider invariable association or Vyapti
varnas. In this period, Brahmins were given more (O;kfIr) as the nerve of all inferences, Charvakas challenges
priority because the responsibility of protecting the this and regards it as guess work. Perception does not approve
Vedas was considered to be ‘theirs’. People still feel this Vyapati. Inference and testimony do not approve it.
like this Theist and Atheist Knowledge (Orthodox and Charvaka reviews perception as valid and inference as
Unorthodox Knowledge). The Indian tradition of phi- invalid, as it is the result of inference. The creations, such
losophy is determined by two directions, one direc- as Kautilya’s Arthashastra (vFkZ”kkL=–Science of material
tion is called ‘astika’ ( ) and the other direction gain), are based on it as it is considered to be an hedonist
is called nastika ( ). opportunist approach.

The following elements are important, which help us in Orthodox Views of Knowledge
making a classification about the different philosophies.
The Nyaya and Vaisheshika schools are primarily analytic
1. A theist is the direction that accepts the supernatu- and are, therefore, more concerned with logic and episte-
ral and spiritual beings like God, soul, reincarnation, mology than ethics.
heaven-hell etc. in an intuitive manner. The atheist
never accepts these powers. Nyaya School
2. Only Charvak philosophy is such that does not accept Nyaya philosophy is very important as per syllabus. It was
any of the supernatural and spiritual authority. primarily given by sage ‘Gotama’ (different from Bhagwan
This recognizes only the supernatural and material Gautam Buddha). His creation is ‘Aksapada’. This
beings. school is also known as ‘tarakshastra’ (rdZ”kkL=–science
3. Buddhism and Jainism accept theories like reincarna- of reasoning), ‘pramanashastra’ (izek.k”kkL=–science of
tion and soul respectively, but not spiritual beings, so logic and epistemology), ‘hetuvidya’ (gsrqfo|k–science
these philosophies are also called atheists. of causes), ‘vadavidya’ (okn fo|k–science of debate),
4. Nyaya, Vaisheshika and Mimamsa philosophy have ‘anviksiki’ (v.kfof{kdh–science of critical study).
been considered realist under the astik philosophy, This school is associated with ‘atomistic pluralism’ and
because while accepting the spiritual and supernatu- logical realism, they actually means ‘liberation of the indi-
ral being, it also recognizes the planetary and material vidual self’. They are linked with material and spiritual
beings. as ultimate constituents of the word. We are concerned

M06_MADAN 07_65901_C06.indd 28 27/12/22 8:18 PM


Logical Reasoning 6.29

with both internal and external aspects of knowledge. We (Kku eheaklk) of Yoga Philosophy. It accepts Samkhya,
should be connected with expression of reality (arthab- psychology and metaphysics (lka[;, euksfoKku vkSj rRoeheaklk)
hava–vFkZHkko). The gain of knowledge is connected with but belongs to the ‘atheist school’ of thought. Yoga system
perception, inference, comparison and verbal testimony is the applied form of the ‘sankhya’ itself. It is more
which are part of syllabus also. systematic than Sankhya philosophy. Human beings
The objects of learning are self, body, sense organs, use these yoga methods for their purity by purifying,
sense objects, intellect, mind, and activity. controlling and ultimately eliminating the modifications
Knowledge is the knowledge of things, and it consti- of the mental system (var%dj.k ;k fpÙk). The method is
tutes the expression of reality (arthhubhava). Whatever called Aashtanga Yoga (vk’Vakx ;ksx) which includes the
its type, it is a natural response to the disposition present following exercises:
in human mind.
1. Yama (;e–self-control)
2. Niyam (fu;e–following certain principles)
Vaisheshika Philosophy ( ) 3. Aasnn (vklUu–adjacent-steady postures of the physical
Nyaya system is allied to the Vaisheshika systems, which body)
developed metaphysics and ontology. The Vaisheshika 4. Pranayama (izk.kk;ke–breathing control)
sutras are the oldest ones and were written by Kannada, 5. Pratyahara (izR;kgkj–removal of the senses and the mind
shortly before Gautama’s Nyaya Sutras (U;k; lw=). from the objects of enjoyment)
The word Vishesa (fo”ks’k) means particularity and 6. Dharana (/kkj.kk–mindfulness on a chosen object)
emphasizes the significance of individuals. It recognizes 7. Dhyaan (/;ku–meditation–controlled and sustained
three real objects of experience as substance, quality, attention to the object)
and activity. 8. Samadhi (lekf/k–the absorption of the mind in an object
There are three products of intellectual discrimination, –a man gets his own experience and realizes its true
which are generality, particularity, and combination. nature.
Like the Nyaya school, this school also acknowledges Pranayama means control of breathing, it has three main
perception, inference, comparison, and verbal testimony parts:
as the valid sources of knowledge.
1. Poorak (iwjd–breathing in)
Sankhya philosophy ( ) 2. Kumbhaka (dqEHkd–holding the breath inside)
The founder of Samkhya philosophy was the famous sage 3. Rechak (jspd–laxative–breathe out slowly)
Kapil. The Samkhya system is also called ‘dualistic’ ( ). Swami Vivekananda made a special contribution in this
It believes in Prakriti and Purushartha. field by composing ‘Raj Yoga’ on Patanjali Yoga Sutras.
The knowledge consists of three main causative
elements. Mimamsa Philosophy ( )
(a) one who knows the subject Mimamsa literally means ‘revered thought’ and was origi-
(b) known object nally applied to the interpretation of the Vedic rituals,
(c) process of knowledge which commanded highest reverence.
It is also very ancient; Mimamsa Sutra ( ) by
Samkhya accepts three pramanas—pratyksha, anumaan Jamini was written during 4th century BCE.
and shabda. (These are basically ‘yatharth gyan’). A cognition, which apprehends an object, cannot be
Nature is considered to be a fundamental and real sub- intrinsically invalid. Memory arises from the impression
stance. Human being (Purusha) as mere ‘consciousness’ of a priori cognition.
(chetna– ) is above all modifications and changes. It Kumarila defines that valid knowledge is free from
is a unique ideology, unaffected by ‘modifications’ in the causes and from defects and is not contradicted by subse-
mind. A man in bondage is freed in the form of intelli- quent knowledge.
gence by the efforts of nature. A valid cognition must fulfil four conditions.
There are two types of inference in Sankhya philosophy:
1. It must not arise from defective causes.
1. Veet ( ): This refers to the anumaan (inference) 2. It must be free from contradiction. It must be self-
which is dependent on the eternal ‘law sentence’ consistent and should not be set aside by subsequent
(shashvat vidhi). knowledge.
2. Aveet ( ): Here, the inference is dependent on 3. Novelty is an essential feature of knowledge agra­
the eternal negation (shashwat nishedh– ) hitgrahi (vxzfgrxzfg). Memory is excluded from valid
sentence. acknowledge.
4. It must truly represent the object.
Yoga Philosophy ( ) Here, all knowledge is valid by itself. It is not validated by
Maharishi Patanjali founded yoga philosophy in the any other knowledge. It is not due to any extraneous con-
2nd century BC. His Yoga Sutras are the first authentic ditions. A need for explanation is felt only when knowl-
texts of Yoga-philosophy. It is based on ‘Gyan Mimamsa’ edge fails.

M06_MADAN 07_65901_C06.indd 29 27/12/22 8:18 PM


6.30 Chapter 6

If a rope is mistaken for a snake, the knowledge of the


Jainism
snake is invalidated by the subsequent knowledge of the
rope.
Truth is normal, and error is abnormal. Belief is natural
and disbelief is an exception. Knowledge
According to bhadrayana ( )–(Uttara Mimamsa Immediate Mediate
and Vedanta), knowledge comes from the scriptures Aparoksa Paroksa
(Sruti) and other authorities (Smriti). Scripture refers to (vijks{k) (ijks{k)
the Vedas and Smriti to the Bhagavad Gita, Mahabharata,
and Laws of Manu.
According to Sankhya, both the validity pramanya Avadhi Manahparyâyâ Kevala Matti Shruta
( ) and the invalidity (Apramanya– ) of knowl- Figure 6.10 Knowledge as per Jainism
edge are self-evident. Whatever manifests itself at any
time has all along been hidden there.

O ther Heterodox Schools of Knowledge 1. Immediate Knowledge


(i) Avadhi ( ): This is the direct knowledge of
The word ‘Jain’ is derived from jina or jaina which means things even at a distance of space or time. It is
the ‘conqueror’. Jainism and Budhism don’t recognize called Avadhi or ‘limited’ because it functions
the authority of ‘vedas’, and are heterodox schools of within a particular area and up to a particular time.
philosophy. Jainism came to prominence in the 6th century It cannot go beyond spatial and temporal limits.
B.C., when Lord Mahavira propagated the religion. (ii) Manahparyâyâ ( ): This is the direct
There were 24 great teachers (tirthankaras), the last of knowledge of the thoughts of others (telepathy),
whom was Lord Mahavira. Tirthankaras attained all knowl- which is limited to space and time conditions.
edge and were liberated (that is called as Moksha). The first   Both Avadhi and Manahparyâyâ are called
Tirthankara was Rishabnatha. Kalpa Sutra (dYi lw=) is a extra-sensory form of knowledge, as the soul is
biography on numerous Tirthankaras with a primary inter- unaided by the senses or mind in obtaining direct
est in Vardhaman Mahavira and Parshvanatha. knowledge.
Lord Mahavira has given five Mahavrats (Great vows)– (iii)Kevala Jnana ( ): This is absolute, extraor-
truth, non violence, non stealing, non possession, extra- dinary, and infinite knowledge. It is not limited by
version/celibacy (lR;, vfgalk, vLrs;, vifjxzg vkSj czãp;Z). space, time, or object, and can be acquired only by
Three Jewels for Triratna are liberated souls.
1. Right Faith: (Samyakdarshana–lE;Dn”kZu),    These three kinds of immediate knowledge may
2. Right Knowledge: (Samyakjnana–lE;DKku), be called extra-ordinary and extra-sensory per-
3. Right Action: (Samyakcharita–lE;Dpfjr) ceptions. Avadhi is clairvoyance; Manahparyaya is
telepathy; and Kevala is omniscience.
Jainism is divided into two categories–Digambar (fnxEcj)
and Shwetambar (“osrkEcj). The second ones are more 2. Mediate Knowledge
liberal in their approach, towards women also. The The instruments of matti and shruta knowledge are
ultimate objective of Jainism is achieve Moksha (Ekks{k–the perception, inference, and authority.
ultimate liberation) through Kaivalya (dSoY;). (i) Matti ( ): This includes perceptual and infer-
ential knowledge. Pure perception in the sense of
Jainism mere sensation cannot be knowledge. Sensation
Jains have critically examined the valid sources of knowl- to become knowledge must be given meaning and
edge. Here, knowledge is of two kinds: arranged into order by thought.
1. Pramana refers to the knowledge of a thing as it is. (ii) Shruti ( ) means knowledge derived from
2. Naya ( ) or knowledge of a thing in its reflec- authority.
tion. It means the standpoint of thought from which
Shruta Jnana (Jqrk Kku) means knowledge derived from
we make a statement about a thing. All truth is rela-
figures of authority, creation of great sages, books etc.
tive to our standpoint. Partial knowledge of one of
Perceptual knowledge is ordinarily called as ‘imme-
the innumerable aspects of a thing is called ‘naya’.
diate’, thus admitted to be relatively so by Jainism.
Both Pramana and Naya are essential for the full and true Therefore, it is included in mediate knowledge. Pure per-
knowledge of a thing. ception in the sense of mere sensation cannot rank the title
Mediate knowledge is any knowledge which the of knowledge. It must be given meaning and arranged into
soul comes to have by the mediation of sense organs; order by conception or thought. Perceptual knowledge is,
any knowledge whose acquisition involves something therefore, regarded as mediate since it presupposes the
other than the soul itself. On the other hand, immedi- activity of thought. Mediate knowledge is divided into
ate knowledge is that which the soul obtains without the mati and shruta. Mati includes both perceptual and infer-
intervention of the sense organs. ential knowledge.

M06_MADAN 07_65901_C06.indd 30 27/12/22 8:18 PM


Logical Reasoning 6.31

According to Jaina epistemology, indirect knowledge is It can be achieved by following the Atthanga Magga (Eight
of five kinds: Fold Path).
1. Smrti (Le`fr –valid knowledge) Eight Fold Paths: These practices are linked with knowl-
2. Pratyabhijna (izR;fHkKk –recognition) edge, conduct, and meditative practices.
3. Tarka (rdZ –logic) 1. Right view
4. Anumana (vuqeku –inference) 2. Right intention
5. Agama (vxek –words of reliable people) 3. Right speech
4. Right action
Here, we can discuss two important aspects. 5. Right livelihood
1. Naya Vada: means a standpoint of thought from 6. Right mindfulness
which we make a statement about a thing. All truth is 7. Right effort
relative to our standpoints. Partial knowledge of one 8. Right concentration
of the innumerable aspects of a thing is called ‘Naya’. Five Precepts (Panchasheel Principles—iap”khy ds fl)kar).
2. Syad vada or Saptabhangi ( ): The disciples are needed to refrain from them.
is the most important part of Jaina logic. According 1. Violence
to this, we can know only some aspects of reality and 2. stealing
so all our judgements are relative. It is a theory of 3. sexual misconduct
the relativity of knowledge—what is true in my eyes, 4. lying or gossip
may be wrong in the eyes of others. 5. taking intoxicating substances e.g. drugs or drink
The process of practical perception in Jainism is given as In epistemological ideas also we can see the different
under: opinions among the four schools of Buddhism.
1. Avagrah ( ): distinguishing the type, horse, cow 1. Yogachara ( –Yoga Practice)
or man, without detailing the main features. 2. Madhyamika ( –middle path, also called as
2. iha ( ): inquiring about whereabouts, country, shoonyavaad ): This is called as
direction etc). Parmarthik element (charitable) of the world that
3. avaya ( ): correct identification says that origin of world is ‘Zero’.
4. Dharana ( ): recollecting the things popularized 3. Sautrantika ( –all is truth doctrine)
and keeping them in mind. 4. Vaibhasika ( –to get knowledge through two
means
Budhism ( ) (a) grehen ( ): to receive knowledge
Buddhism started in India almost 2,600 years ago with (b) adhyvsay ( ): to gain knowledge through
an objective of transforming a person. This is popular in hard work, diligence, persistence, assiduousness
South and South East Asia. etc.
This religion is based on teachings of Siddhartha Sautrantika says that the external objects are not known
Gautam, born in circa 563 BCE. He was born into royal through perception. According to Vaibhasika, the knowl-
family of Sakya clan who ruled from Kapilvastu, in edge of the external objects can also be gained through
Lumbini near Indo-Nepal Border. perception.
Gautama at the age of 29 left home and embraced a life- According to Vaibhasika, the inference of things exter-
style of asceticism, or extreme self-discipline. He attained nal to knowledge is self-contradictory. If all the external
‘enlightenment’ at the age of 49, at Bodhgaya in Bihar. He objects are inferred by their knowledge, then nothing can
delivered his first sermon at Sarnath. This event is known as be known by perception. In the absence of perception,
Dharma-Chakra-Pravartana (turning of the wheel of law). there can be no relation of concomitance between the
major and the minor premise, without which no inference
Tenets of Buddhism is possible. This is opposed to actual experience.
Buddha asked his followers to avoid indulgence in worldly The Vaibhasikas accept the presence of the external
pleasure. They were told to follow the practice of strict things and conceive them as subject to perception. To
abstinence and asceticism. them, by Pramana, only direct knowledge is possible. The
He ascribed instead the ‘Madhay Marg’ (middle path) Pramanas are of two types, namely Pratyaksa (percep-
which was to be followed. tion) and anumana (inferential). Both these Pramanas
Human life is set with sufferings, suffering is caused by are known as samyagjnana (right knowledge), and it is by
our desires and attachment, the truth of end of sufferings these that all the purusharthas are attained. Pratyaksa is
and truth of the path that leads to end of suffering. the knowledge that is without any imagination and error.
This knowledge is of four types:
Four noble truths: 1. Indriya Jnana ( ): Knowledge through senses.
Suffering (dukkha) is the essence of the world. 2. Mano Vigyan ( ): Sensual knowledge in the
Every suffering has a cause—Samudya. form of samanatara pratyaya after the knowledge
Suffering could be extinguished—Nirodha. through senses.

M06_MADAN 07_65901_C06.indd 31 27/12/22 8:18 PM


6.32 Chapter 6

3. Atma samvedana ( ): It is the manifesta- reform Hinduism. The languages spoken by the masses,
tion of chitta (fpRr), and its dharmas are like pleasure such as Prakrit and Pali, started getting prominence
and pain in their real form. over Sanskrit, a language which was limited to the
4. Yogic Jnana ( ): It is the ultimate knowledge priestly and aristocratic classes. The source of both the
of the things perceptible through various Pramanas. religions is vedic religion, and both are indebted to the
Upanishads.
Inference is of two types Buddhism is centred upon the life and teachings of
Gautama Buddha, whereas Jainism is centred on the life
1. Svartha (LokFkZ –for the self)
and teachings of Mahavira. Buddhism is a polytheistic reli-
2. Parartha (ijkFkZ –for others)
gion, and its main goal is to gain enlightenment. Jainism
In the former, the linga is inferential, i.e., in inference, if is also a polytheistic religion and its goals are based on
there is fire on the hill, the hill is linga and the fire is infer- non-violence and liberation of the soul.
ential. In it the linga remains in self-side (svapaksha–Lo%i{k),
Buddhism says that this life is suffering and the only way
just as the kitchen. The linga does not remain in the oppo-
to escape from this suffering is to dispel one’s cravings and
site side (vipaksa), e.g., a pool of water, etc.
ignorance by practising the Eightfold Path.
There are two sections of Budhism:
Jainism suggests respecting all living things and
1. Mahayana ( ): This Buddhism considered Gau- attaining liberation by avoiding and shedding of bad
tama Buddha to be a divine being who guided his karma, which is the cause of rebirths and all sufferings.
followers to attain nirvana (ultimate liberation from
desires and expectations in this birth - freedom from
cycles of birth and death). Pramana ( – Source of
2. Hinayana ( ): This sect of Buddhists consider Knowledge)
Gautama Buddha as an ordinary human being who
attained Nirvana. The general science of inference is logic, and its aim is to
make explicit the rules by which inferences are drawn.
There are three types of sects on the basis of principles. Inferences are rule-governed steps from one or more
1. Aneeshwarvaad ( ): This means ‘atheist’, propositions, known as premises, to another proposi-
they don’t trust the authority of God. tion, called conclusion. A deductive inference is one that
2. Anatmvaad ( ): What we call as ‘soul’ is actually is intended to be valid, whereas a valid inference is one
the integral flow of ‘consciouness’. Budhhism is unique in which the conclusion must be true if the premises are
to deny the existence of a soul. There is no unchanging, true. All other inferences are inductive.
permanent self, or essence in the phenomena. Our discussion is primarily based upon nyaya (U;k;) sys-
3. Kshanikvaad ( ): All universal things are tem. Vatsayana defines a Pramana as a source or means of
‘momentary’ in nature. valid knowledge. Gautama’s Nyaya Sutra defines perception
as an awareness that is
Tripitika ( —Three Baskets): This term is used in
many Buddhist scriptures. Much of the buddhist literature 1. produced from the connection between the sense
is written/discussed in ‘Pali”. The three pitakas are organ and object
2. not produced by words
1. Sutta Pitaka ( ): This is basket of discourse 3. not deviating from its object, i.e., it is always true
/discussion. It is the largest section for discussions. A 4. is of the nature of certainty.
sutta can be called a sutra in Sanskrit.
2. Vinaya Pitaka ( ): This is basket of discipline, There are four factors involved in any knowledge as
that is the set of hundreds of rules to be followed by listed below.
monks (fHk{kq ) and nuns in their monastic life so that 1. The subject who knows (Pramata–Áekrk)
they can lead a successful life in the sangha (la?k). 2. The object of knowledge (Prameya–Áes;)
   A monastery for Buddhist renunciates is known as 3. The means of valid knowledge (Pramana–Áek.k)
a Vihara (fogkj). 4. The resultant of valid knowledge (Prama–Áek)
3. Abhidhamma Pitaka ( ): This means
further or special teachings, is a systematic philo- Knowledge can be of two types:
sophical and sometimes “scientific” description of the 1. Prama ( –valid): This is also known as ‘presen-
nature of mind, matter and time. tative knowledge’ when the object of knowledge is
Theravada school Tripitaka is the only complete Tripitaka directly present to the knower. Four types of valid
preserved in Pali. knowledge includes perception, inference, compari-
son and sabda.
Sarvastivada is a near complete Tripitaka in Sanskrit, 2. Aprama ( –invalid): This is basically wrong appre-
Chinese and Tibetan. hension of an object. It includes memory (smriti), doubt
A small comparison between Buddhism and Jainism: (smasya), error (viparyaya) and hypothetical reasoning
Buddhism and Jainism movements were started to (tarka).

M06_MADAN 07_65901_C06.indd 32 27/12/22 8:18 PM


Logical Reasoning 6.33

Nyaya Epistemology

Valid Invalid

Anupalbdhi Arthapatti

Pratyaksha Anumana Upamana Shabda

Stages Type of Testimony

Indeterminate Determinate Vaidika Avaidika

Psychological Vyapti Induction

Swartha Prartha Kewalanyi Kevalanuitireki Airunya


vyitreki

Purvavat Shashavat Samayatra


prashta

Types

Extra
Ordinary
Ordinary

External Internal

Samanya Gyan Yogaza


Inner Cognitive Feelings Laksha Lakshna
sense

M06_MADAN 07_65901_C06.indd 33 27/12/22 8:18 PM


6.34 Chapter 6

Hence, pramana is the valid means of knowledge. It 1. Self comes into contact with mind (manas–Ekkul).
has four important means: 2. The manas with the senses.
1. Pratyaksa (ÁR;{k —Perception) 3. The senses with the object.
2. Anumana (vuqeku —Inference) The function of a sense organ with regard to its own object
3. Upamana (mieku —Comparison) is described in two ways, nature of contact and nature of
4. Shabda (“kCn —Verbal testimony) knowledge.
Different schools of knowledge accept or reject differ- Sense–object is also the instrumental cause of percep-
ent facets of these methods. tion, as it immediately gives rise to the perceptual knowl-
edge of that particular object.
1. All methods are accepted by Mimamsa. The modern school of Nyaya gives a new definition
2. Only perception, inference, and testimony by Samkhya of perception as it is direct or immediate cognition that
and Yoga. is not derived through the instrumentality of any other
3. Only perception and inference by Buddhism and cognition. It applies to all cases of perception, human or
Vaisheshika. divine. Even God’s omniscience has the highest degree of
4. Only perception by Charvaka. immediacy conceivable.
Perception is divided into the following two categories:
Pratyaksha ( –Perception ) Ordinary perception (Laukika): Knowledge results from
Pratyaksha is combination of Prati (Áfr) + aksha (v{k). the contact of the sense organs with the external objects.
It is basically what is before one’s eyes; ‘aksa’ means Extraordinary perception (Alaukika): it has three dis-
sense organ and ‘prati’ means the function of each sense tinctions,
organ. Perception is a valid form of knowledge produced
by the contact of an object with a sense organ. 1. Samanyalaksna ( perception of uni-
Perception in Gautama system is defined as the knowl- versals/classes): Universals are a distinct classes.
edge that originates from contact (sannikarsa–lfUu’d’kZ) of They represent all the particulars of the class being
a sense with its object that is determinate, unnamable and discussed. A man has all qualities of ‘manhood’ that
non-erratic. are common among all men.
The term avyapadeshya (vO;ins”;) is used for percep- 2. Jnana laksnana ( knowledge traits): If we
tion that is without any doubt or error. saw some object previously. On seeing the picture of the
According to Nyayikas, perception is the direct and object, we can recall important or some of its important
immediate cognition produced by the interaction between property. Suppose, on seeing the tree of ‘sandalwood
the object and sense organs. Perception needs four ele- (chandan), we can recall its strong fragrance.
ments - self, mind, sense organs and objects. 3. Yogaja ( Intuition): This is extraordinary per-
It is the first of the five means of knowledge or pramanas ception that is found in gurus and rishis. These are
that enable a person to have correct cognitions of the world. supernatural powers. They have power of meditation
Pratyaksha is of two kinds: that has intuitive and immediate perception of all
objects, past, present and future.
1. Anubhava ( ): Direct perception
2. Smriti ( ): Remembered perception According to later logicians, there are two kinds of verbal
testimony:
Perception is considered to be of two types.
1. Vaidika or Alukika : It is also known
1. Indiscriminate perception (nirvikalpaka– ): as divine or scripture. The vedas are spoken by God.
The object is perceived without its distinguishing fea- This testimony is divine and perfect.
tures such as colour, shape, size etc. This is also called 2. Laukika or secular : It is known as secular. The
as avykta (vO;Dr) that means that it can’t be explained human beings are not perfect. so only the words of trust
through our vocabulary. This perception is important worthy person can be considered as laukika.
to the followers of the Advaita (non-dualist) school of
Vedanta, as it allows the liberating perception of brah- The former relates to the words of God. The Vedas are
man (ultimate reality), which is without features. created by God and therefore, perfectly valid. The latter
2. Discriminate perception (savikalpaka– ): relates to the words of trustworthy people.
The distinguishing features are both observed and According to Nyayikas, since human beings are not per-
recognized. It has name, colour, shape etc. It gives fect, only the words of trustworthy people can be consid-
direct knowledge of the object. Examples are, it is a ered as Laukika Shabda.
smartphone. She is a woman. Anumana (Inference)
Knowledge arises by contact of sense organs (indriya– This word indicates knowledge after earlier knowledge
bfUnz;) with an object. Such contact is not the sole con- (anu—after, mana—knowledge). It is the second source
dition of perception, but it is its distinctive feature or of valid knowledge. Inference is defined as the knowledge
extraordinary cause (karana–dkj.k) of perception. The of an object (lingi) due to a previous knowledge of some
actual process is given below: sign or mark (linga).

M06_MADAN 07_65901_C06.indd 34 27/12/22 8:18 PM


Logical Reasoning 6.35

Gautama defines it as a specific form of knowledge Arthapatti ( –P resumption )


preceded by perception. The perception of the invariable
It is an independent source of knowledge. It is admitted
relation between the proban (linga) and the probandum
as a distinct pramana that cannot be brought under anu­
(lingi) is a previous perception of such a relation some-
mana or sabda.
where else. Again, the perception of the proban is invari-
It consists in the assumption of some unperceived fact in
ably related to probandum as it exists in the locus.
order to explain apparently inconsistent facts. Let us take an
Shabda (  Verbal T estimony) example of arthapati. Devadatta is alive and he is not pres-
ent in his house, and we presume that he is elsewhere. The
According to Nyaya Philosophy, Shabda is the fourth and essential element in presumption is that a certain fact like
last valid source of knowledge. Shabda literally means Devadatta’s ‘being alive’ and ‘not being present in his house’ is
verbal knowledge. The mere combination of words does unaccountable without presuming another fact such as being
not provide a valid knowledge. outside his house. In presumption, we proceed from the
All verbal statements are not valid. Hence, Gautama knowledge of something to be explained to the knowledge of
defines shabda pramana ('kCn izek.k) as the statement that which explains it. The means of presumption (karana) is
of a reliable person. In other words, verbal testimony the knowledge of the inner contradiction (anupatti) and its
is the communication from a trustworthy person—who result is the reconciliation of the contradiction (upapatti).
is a trustworthy person (apta) and why is assertion If Devadatta is fat and does not eat during the day, we
(upadesa) a testimony (prambna)? presume that he must be eating during night, otherwise
Analyzing the process of verbal testimony we find the the inconsistency between ‘being fat’ and ‘not eating dur-
following steps. ing day’ cannot be resolved.
First, there is the perception of the words of a sentence
uttered by a trustworthy person. Anupalabdhi ( –Non -apprehension
Second, there is the understanding of the meaning of Mimamsa)
words. This is called the karana (dkj.k) or the special cause
of the verbal knowledge. According to Kumarila Bhatta and others, non-appre-
The knowledge of words (padajnana–in Kku) leads to hension as the sixth independent source of knowledge
the knowledge of objects through the function (vyapaar– consists in the presentative knowledge of negative facts.
O;kikj) of recalling the meaning of words. In other words, negative facts are cognized by a special
Gautama and Vatsyayana stated in Nyaya school that instrument (karana) called non-apprehension.
verbal knowledge is of two kinds: Only positive facts are apprehended through positive
sources like perception, inference, etc, but negative facts
1. Drustartha (–’VkFkZ or –’V vFkZ): This one is relating to are apprehended through non-apprehension. For exam-
perceptible objects, which means the sensible object ple, the absence of a jar on the ground is apprehended
attainable in this world. through anuplabdhi.
2. Adrsta or Adrusthartha (v–’VkFkZ): A sabda that deals Kumarila argues that the concept of the emptiness of
with the imperceptible object is called as ‘adrsta’. For the container inevitably presupposes non-existence. He
example, honesty is the best policy. God is one. also refutes the Nyaya view that non-apprehension is the
same as perception or inference.
This is the division of words of the ordinary people and
Negation is never perceived, for there is no sense–
the seers.
object contact in it.
U pamana ( –Comparison)
Stopover
Upamana is the combination of ‘upa’ and ‘mana’. ‘Upa’
means similarity or ‘sadrusya’ and ‘mana’ means c­ ognition. The distinction between laukika and alaukika is

Thus, upamana is the knowledge derived from similarity. made with reference to which one of the following
It has been defined as the knowledge of relation between ­pramanas? [2019]
a person and its denotation. Upamana is the third source (a) Anumana (Inference)
of valid knowledge. (b) Upamana (Comparison)
For example, when we tell a city man that a wild cow is (c) Pratyaksa (Perception)
an animal like a cow, later on, when he sees a wild cow in (d) Sabda (Verbal testimony)
a forest, he recognizes it as a wild cow. Then, his knowl- The correct option is (c).
edge of the wild cow is the outcome of conjunction with
the knowledge of the cow. Hence, the ‘upamana’ is just the
knowledge of the relation between a name (here it is the Structure and kinds of Anumana
wild cow) and the object denoted by that name (the actual
wild cow seen in the forest). (Inference)
Mimansa treats Upamana as analogy. Buddhism accepts Knowledge that comes after perception is inferential or
comparison as an independent source of valid knowledge. relational, and it is called inference. Anumana, etymologi-
According to Mimansa, the following two schools have cally means ‘secondary proof’. The data for inference are
also been identified. derived from perception and verbal testimony.

M06_MADAN 07_65901_C06.indd 35 27/12/22 8:18 PM


6.36 Chapter 6

There are two main groups of inference: A vyapti may be of two types and they are as follows:
1. Vyapti ( ): It is when universal relation, such as 1. Samvyapti ( )
between fire and smoke, is known. 2. Asamvyapati ( )
2. Paksadharmata ( ): Fire is inferred on the
hill, where smoke is perceived in it. A vyâpti between terms of equal extension is called samavy­
âpti or equipollent concomitance, for example, ‘­nameable’
Inference is mediate and indirect, that is, arranged and ‘knowable’. Whatever is nameable is knowable and
through the medium of some mark which is called ‘hetu’. again whatever is knowable is nameable. Here, we can
This may be explained with the help of the typical exam- infer either of the terms from the other.
ple of inference, the presence of fire on the perception of On the other hand, a vyâpti between terms of unequal
smoke. When one sees smoke on distant hill, one remem- extension is called asamavyâpti. Fire is present in all cases
bers one’s experience of the universal concomitance where smoke is present, but the reverse is not true.
(Vyapti) between smoke and fire and concludes that there The Naiyayikas maintain that there are five ways or meth-
is fire on the distant hill. ods for the establishment of vyâpti. They are the following:
Thus, we can say that:
1. Anvaya or Agreement in Presence ( ): Vyâpti is
1. This hill has fire (pratijna–izfrKku). a relation of agreement in presence (anvaya) between
2. Because it has smoke (hetu–gsrq). two things.
3. Whatever has smoke has fire, for example, an oven 2. Vyatireka or Agreement in Absence ( ): The
(udaharana–mnkgj.k). hetu and the sâdhya should agree in being absent together.
4. This hill has smoke, which is invariably associated 3. Vyabhicaragraha ( ): We do not observe any
with fire (upanaya–miu;). contrary instance in which one of them is present and
5. Therefore, this hill has fire (nigamana–fuxeu).
the other is absent. That is, they must be related to each
The first, the pratijna, is the logical statement, which is to other.
be proved. The second is hetu or reason, which states the 4. Upâdhinirasa or Elimination of Condition
reason for the establishment of the proposition. The third ( ): Vyâpti is an unconditional relationship
is udaharana, which the universal concomitance together that is universal and necessary. An adventitious con-
with example. The fourth is upanaya or application of the dition may vitiate the natural and invariable relation
universal concomitance to the present case. The fifth is between hetu and sâdhya.
nigamana or conclusion drawn from the preceding propo- 5. Tarka or Hypothetical Reasoning ( ): Tarka is
sitions. These five members of Indian syllogism are called an indirect method to get the vyâpti. All the methods
Avayavas. mentioned above are direct methods. Ratiocination is
In the Aristotelian syllogism, the character which is the process of thinking about something in a logical
inferred (fire) is called sadhya; the mark on the strength way to establish the vyâpti.
of which the character is inferred is the hetu (smoke); the 6. Sâmânyalakaa Pratyaka ( ): Sâmân­
subject where the character is inferred is paksa (hill). The yalakaa pratyaka is an extraordinary perception. They
three terms sadbya, hetu and paksha (lk/;, gsrq vkSj i{k) maintain that when we perceive an individual case,
correspond to the major, the middle, and the minor terms. we also perceive all the actual and possible instances
of fire and smoke.
Linga Paramarsa: The Nyaya syllogism has five terms.
Among them, middle term works as a bridge between the Vyapti – Another classification of Inference
major and the minor terms. Therefore, the middle term Inference here is classified based on the nature of vyapti
has main responsibility to prove a syllogism valid or inva- between hetu (smoke) and sadhya (fire). Vyapti denotes a
lid. How a middle term is related to major term is linga-par­ correlation between two facts of which one is pervaded and
amarsha. There are five characteristics of a middle term. the other which pervades. E.g. Smoke is pervaded by fire
and fire pervades smoke. Vyapti is established based on its
Vyapati ( Invariable R elation) presence of both in all such events (wherever there is smoke
The word ‘vyâpti’ literally means ‘the state of pervasion’. there is fire) and the absence of both (wherever there is no
It implies a correlation between two facts, of which one is fire there is no smoke). The classification is based on the
pervaded (vyâpya), and the other pervades (vyâpaka). A relationship (causal uniformity or non-causal uniformity)
fact is said to pervade another when it always accompanies between the reason and what is inferred. There are three
the other. A fact is said to be pervaded by another when it types of inference.
is accompanied by the other. In the given example, smoke 1. Purvavat inference ( ): It is that in which
is pervaded by fire, since it is always accompanied by fire. we infer the unperceived effect from a perceived
But while all smoky objects are fiery, all fiery objects are cause. Example, we infer of future rain from the
not smoky, e. g., the red hot iron ball. Thus, vyâpti is a appearance of dark heavy clouds.
relation of invariable concomitance between middle term 2. Sesavat Inference ( ): It is that in which we
and the major term. Without the definite knowledge of infer the unperceived cause from a perceived effect.
such a relation, our inference of fire is impossible in spite Example, we infer of the past rain from swift muddy
of the perception of smoke. current of water in the river.

M06_MADAN 07_65901_C06.indd 36 27/12/22 8:18 PM


Logical Reasoning 6.37

3. Samanyatodrasta inference ( ): It is example, ‘the fact of being caused’ should not be used
that which we infer not based on causal relation but to prove the ‘eternality’ of sound (aviruddha).
on experience of uniformity. Example, on seeing the When any of the above features are violated, we get fal-
different positions of the moon at long intervals, we lacies. So they must be seen very carefully. The five kinds
infer that it moves although the motion might not of fallacies have been recognized with this approach.
have been perceived by us.
1. Asiddha ( ): This is the fallacy of the unproved
A comparison – Indian nyaya system and Western middle. The middle term must be present in the minor
­system.

Nyaya Syllogism – there are five propositions Western Example


1. Paksa – The Thesis OR The hill has fire Ram is mortal
Pratijna – Proposition
2. Hetu – the main reason Because it has smoke Because he is a man. f Inductive syllogism

3. Drshtanta – The corroboration Where there is smoke, there All men are mortal like my
is fire as in the kitchen grandfather
4. Upanaya – The application The hill is so Ram is also a man
fDeductive syllogism
5. Nigamana – The conclusion Therefore, the hill has fire. Therefore, Ram is a mortal

The first three propositions (1-3) form inductive syl- term (pakshadharmata). If it is not, it is unproved. It is
logism. The last three (3-5) form a deduction. The prop- of three kinds.
osition 3 is the conclusion for induction, and the major (a) Ashraya asiddha ( ): The minor term
premise for the deduction. is the locus of the middle term. If the minor term
When it denotes negative concomitance, it is said to is unreal, the middle term cannot be present in it.
be vyatireka vyapti. The example is opposite of above five For example, the sky-lotus is fragrant, because it
statements – The hill has no smoke; because there is no is a lotus, like the lotus of a lake.
fire; wherever there is no fire, there is no smoke; there is (b) Svarupa asiddha ( ): Here the minor
no fire in the hill; therefore, the hill has no smoke. term is not unreal. But the middle term cannot be
its very nature be present in the minor term. For
example, ‘sound is a quality, because it is visible’.
Hetvabhasa (Ancient Indian Here visibility cannot belong to sound which is
Fallacy System) audible.
(c) Vyapyatva asiddha ( ): Here, Vyapti
Hetvabhasa is a Sanskrit term which means fallacy in the is conditional (sopadhika). Thus, we can’t say that
English language. It means that in Indian logic system, ‘wherever there is fire there is smoke’. Fire smokes
a fallacy is termed as the Hetvabhasa. Here, the middle only when it is associated with the wet or watery
term appears to be a reason but is not the valid reason. All fuel. A red - hot iron ball or clear fire does not
fallacies in ancient Indian system are ‘material fallacies’. smoke. Hence ‘Association with wet fuel’ is a condi-
There are five features of a valid middle term which have tion necessary to the aforesaid vyapti. Being con-
been discussed below. ditioned, the middle term becomes fallacious if we
say that ‘the hill has smoke because it has fire’.
1. It must be present in the minor term (paksha-­dharmata).
For example, smoke must be present in the hill. 2. Savyabhichara or Anaikantika (
2. It must be present in all positive instances in which the ): This is fallacy of the irregular middle.
major term is present. For example, smoke must be pres- This is again of three types.
ent in the kitchen where fire exists (sapakshasattva). (a) Sadharana ( ): Here the middle term is
3. It must be absent in all negative instances in which too wide, it is taken as ‘mixed land’. It is taken
the major term is absent. For example, smoke must as present in sapaksha (positive) and vipaksha
be absent in the lake in which fire does not exist (negative) instances. It violates the rule that the
(vipaksha asattva). middle not be present in the negative instances
4. It must be non-incompatible with the minor term. (vipaksha asattva). For example, ‘the hill has fire
For example, it must not prove the coolness of fire because it is knowable’. Here ‘knowable’ is pres-
(abaadhita). ent in fiery as well as non – fiery objects.
5. It must be qualified by the absence of counteracting (b) Asadharana ( ): Here, the middle term
reasons which lead to a contradictory conclusion. For is too narrow. It is present only in ‘paksha’. It is

M06_MADAN 07_65901_C06.indd 37 27/12/22 8:18 PM


6.38 Chapter 6

present neither in ‘sapaksha’ not in ‘vipaksha’. It 4. Badhita ( ): Badhita means hindered or


violates the rule that the middle term should be prohibited. It is the non - inferentially contradicted
present in the sapaksha (sapaksha sattva). For middle. The middle term is contradicted by some
example, ‘sound is eternal, because it is audible’. other pramana and not by inference. It cannot prove
Here audibility belongs to sound only and is the major term which is disproved by another stronger
present nowhere else. source of valid knowledge. For example, ‘fire is cold,
(c) Anupasamhari ( ): Here the middle because it is a substance’. The middle term ‘substance’
term is non-exclusive. The minor term is all – is directly contradicted by perception.
inclusive and leaves nothing by way of sapaksha 5. Viruddha ( ): It is again the contradictory
or vipaksha. For example, ‘all things are non- middle. The middle term, instead of being pervaded
eternal, because they are knowable’. by the presence of the major term is pervaded by
3. Satpratipaksha ( ): Satpratipaksa consists the absence of the major term. Instead of proving
of two terms – sat and pratipak a. The middle term the existence of the major term in the minor term, it
is contradicted by another middle term. The reason proves its non-existence therein. For example, ‘sound
is counter - balanced by another reason. And both are is eternal, because it is produced’. Here, ‘produced’,
of equal force. For example, ‘sound is eternal, because instead of proving the eternality of sound, proves its
it is audible’ and ‘sound is non-eternal, because it is non-eternality.
produced’. Here ‘audible’ is counter – balanced by
‘produced’ and both are of equal force.

M06_MADAN 07_65901_C06.indd 38 27/12/22 8:18 PM


Logical Reasoning 6.39

A s s e s s Yo u r L e a r n i n g

THEORy QuESTIOnS
1. The process by which conclusion is arrived at on the Codes:
basis of other propositions is termed as (a) 1, 2 and 4
(a) Concept (b) Inference (b) 2, 3 and 4
(c) Connotation (d) Conference (c) 1, 3 and 4
2. Propositions that support the conclusion of an argu- (d) 1, 2 and 3
ment are called 10. Inductive reasoning is grounded on (June 2015)
(a) Inferences (b) Premises (a) Integrity of nature
(c) Concepts (d) None of the above (b) Unity of nature
3. That proposition which is affirmed on the basis of (c) Uniformity of nature
premises is called (d) Harmony of nature
(a) Major term (b) Concept 11. A cluster of propositions with a structure that exhibits
(c) Conclusion (d) Syllogism some inference is called a (December 2015)
4. When a group of prepositions, one preposition is (a) An inference
claimed to follow from the others, that group of (b) An argument
prepositions is called as (December 2014) (c) An explanation
(a) An argument (d) An invalid argument
(b) A valid argument 12. Which of the following statements are true?
(c) An explanation 1. In inductive reasoning, the conclusion is deterministic
(d) An invalid argument 2. Inductive arguments are properly characterized
5. Deductive proceeds from (November 2020) as strong and weak
(a) Particular to Universal 3. In logical reasoning, truth or falsehood is usually
(b) Universal to Particular propositions
(c) Particular to Particular 4. Validity or invalidity is always predicted by deduc-
(d) Universal to Universal tive arguments
6. With which of the following terms, deduction infer- 5. The falsehood of a valid deductive argument’s con-

A S S E S S YO U R L E A R N I N G
ence can be identified? clusion guarantees that the validity is uncertain
(a) Synthetic (b) Analytic Codes:
(c) Both (a) and (b) (d) None of the above (a) 2, 3, 4, and 5 (b) 1, 2 and 3
(c) 1, 2, 3, 4 and 5 (d) 3, 4 and 5
7. Which of the following can be defined as a group of
statements that have common conclusion? 13. Which one of the following is the characteristic fea-
(a) Proposition (b) Argument ture of an argument? [Dec 2019]
(c) Concept (d) Fallacy (a) It is either valid or invalid
(b) It is neither valid nor invalid
8. The premises provide conclusive grounds for conclu- (c) It is either true or false
sion in (d) It is neither true nor false
(a) Inductive reasoning
(b) Deductive reasoning 14. ‘All tigers are animals’ is an example of which type of
(c) Intuitive reasoning proposition? [Dec 2019]
(d) None of the above (a) Particular negative
(b) Particular affirmative
9. Which of the following statements are true ? (c) Universal negative
1. The defining feature of a valid deduction is its (d) Universal affirmative
certainty
2. Inductive arguments are typically synthetic 15. Which among the following terms are distributed in
3. The relationship between premises and con- the statement, ‘No dogs are reptiles’? [Dec 2019]
clusion in a deductive argument is basically of (a) Only subject term
implication–entailment (b) Only predicate term
4. Inductive and deductive always go in the same (c) Both subject and predicate terms
direction (d) Neither subject nor predicate term

M06_MADAN 07_65901_C06.indd 39 27/12/22 8:18 PM


6.40 Chapter 6

16. Consider the following statements with reference to 24. A deductive argument is sound if and only if it is
the proposition ‘Some girls are not students’.  (a) Valid and all its premises are true
 [Dec 2019] (b) Invalid and all its premises are true
  (i) Subject term students are distributed. (c) Is valid and one of the premise is false
(ii) Predicate term girls are undistributed. (d) Is valid and its conclusion is false
(iii)Predicate term students are distributed. 25. Lakshmana is a morally good person because
(iv) Subject term girls are undistributed. (a) He is religious. (b) He is educated.
Choose the correct option. (c) He is rich. (d) He is rational.
26. Look at the following statements
(a) Only (i) and (iii) are correct
1. In terms of nature, both of the premises and con-
(b) Only (ii) and (iii) are correct
clusion of an argument are propositions
(c) Only (i) and (iv) are correct
2. In syllogism, there are two premises and one
(d) Only (iii) and (iv) are correct
conclusion
17. The reasoning which would be helpful in finding new 3. In inductive reasoning, strong, weak, cogent and
knowledge of facts about the world is (June 2019) uncogent are crucial aspects
(a) Speculative 4. Inductive deals with the post-priori knowledge
(b) Deductive
Codes:
(c) Analogical
(d) Inductive (a) 1, 2, and 3 are correct
(b) 1, 2, 3 and 4 are correct
18. ‘Some students are sincere’—it is an example of which
(c) 3 and 4 are correct
proposition? [Dec 2019]
(d) 1, 2 and 4 are correct
(a) Universal affirmative
(b) Universal negation 27. Which of the following denotes a statement of rela-
(c) Particular affirmative tion between two terms?
(d) Particular negation (a) Proposition (b) Denotation
(c) Syllogism (d) None of the above
19. A fallacy is a
(a) True argument (b) False argument 28. Structure of a logical argument is based on
(c) Valid argument (d) Invalid argument (a) Formal validity
(b) Material truth
20. Inductive reasoning is based on
(c) Linguistic expression
1. Uniformity of nature
(d) Aptness of examples
2. Theory to observation approach
3. Aristotle was basically in favour of inductive 29. In a deductive argument, conclusion is
(a) Summing up of the premises
A S S E S S YO U R L E A R N I N G

reasoning
4. Observation to Theory approach (b) Not necessarily based on premises
(c) Entailed by the premises
Codes:
(d) Additional to the premises
(a) 1 and 4
30. Which of the following are correct statements.
(b) 1, 2, 3 and 4
1. Syllogistic reasoning is deductive in nature
(c) 3 and 4
2. Inductive is about critical analysis
(d) 2 and 3
3. Inductive knowledge is always innovative
21. If it is possible for the premises of a deductive argu- 4. Inductive and deductive are always same in terms
ment to be true and its conclusion to be false that of process
argument is: Codes:
(a) Valid (b) Invalid (a) 1, 2 and 3 (b) 2, 3 and 4
(c) Indescribable (d) Sound (c) 1, 3 and 4 (d) 2 and 3
22. Consider the following propositions 31. A syllogism must have
A is human and mortal. (a) Three terms (b) Four terms
B is human and mortal. (c) Six terms (d) Five terms
C is human and mortal.
32. The two kinds of propositions are
D is human and mortal.
(a) Connotative–denotative
Therefore, ‘All humans are mortal’ is an example of: (b) Categorical–conditional
(a) Deductive argument (b) Inductive argument (c) Both (a) and (b)
(c) Syllogistic argument (d) None of the above (d) None of the above
23. Which of the following describes a valid deductive 33. Which of the following denotes a relation between
argument with true premises? two terms?
(a) Sound (b) Unsound (a) Subject (b) Predicate
(c) Fallacious (d) Ambiguity (c) Object (d) Copula

M06_MADAN 07_65901_C06.indd 40 27/12/22 8:18 PM


Logical Reasoning 6.41

34. A deductive argument is valid if (a) All plastic are synthetic


(a) Premises are false and conclusion is true. (b) Some plastic are synthetic
(b) Premises are false and conclusion is also false. (c) Some plastic are not synthetic
(c) Premises are true and conclusion is false. (d) No plastics are synthetic
(d) Premises are true and conclusion is true. 41. The proposition ‘if you work hard you will succeed’ is
35. Which of the following statements are false? an example of
1. Inductive arguments always proceed from the (a) Categorical proposition
particular to the general. (b) Conditional proposition
2. A cogent argument must be inductively strong. (c) Negative proposition
3. A valid argument may have a false premise and a (d) Pre-emptive proposition
false conclusion. 42. A disjunctive proposition is a type of
4. An argument may legitimately be spoken of as (a) Conditional proposition
true or false.
(b) Unconditional proposition
Codes:
(a) 2, 3 and 4 (b) 1 and 3 (c) Categorical proposition
(c) 2 and 4 (d) 1 and 2 (d) Imperative proposition
36. Which of the following statements are correct? 43. Which of the following statements are true?
1. The sum total of the objects to which a term can be 1. Some arguments while not completely valid are
applied is called as its connotation almost valid.
2. Denotation is the same as ‘extension’ 2. A sound argument may be invalid.
3. The function of suggesting qualities possessed by 3. A cogent argument may have a probably false
the objects is known as connoting conclusion.
4. Denotation is the same as ‘intension’ 4. A statement may be true or false.
Codes: Codes:
(a) 1, 2, 3 and 4 (b) 2, 3 and 4
(c) 1 and 4 (d) 1, 2 and 3 (a) 1 and 2 (b) 1, 3 and 4
(c) 4 only (d) 3 and 4
37. ‘If it rains, then the drought will end. The drought
has ended. Therefore, it rained’. What kind of fallacy 44. Affirmative or negative is the classification of proposi-
does this commit? [Dec 2019] tions on the basis of
(a) Deductive fallacy (a) Quantity (b) Quality
(b) Inductive fallacy (c) Validity (d) Truth
(c) Abductive fallacy

A S S E S S YO U R L E A R N I N G
45. Universal or particular is the classification of the
(d) Informal fallacy propositions on the basis of
38. Which one of the following propositions is a c­ ontrary (a) Quantity (b) Quality
to ‘All poets are dreamers’?  [Dec 2019] (c) Validity (d) Truth
(a) Some poets are dreamers 46. A proposition in which the predicate refers to all indi-
(b) Some poets are not dreamers vidual objects denoted by the subject is called
(c) No poets are dreamers (a) Particular (b) Negative
(d) No dreamers are poets (c) Disjunctive (d) Universal
47. A proposition in which the predicate belongs only to a
39. The collection of all the objects to which a term may part of the denotation of the subject is called
correctly be applied is called: [2021] (a) Particular (b) Negative
(A) The intension of that term (c) Disjunctive (d) Universal
(B) Connotation of that term
(C) The extension of that term 48. Individual propositions are to be regarded as
(D) Denotation of that term (a) Universal (b) Particular
Choose the correct answer from the options given (c) Negative (d) None of the above
below: 49. The proposition ‘all men are mortal’ is an example of
(a) (A) and (B) only (a) Universal affirmative
(b) (B) and (C) only (b) Universal negative
(c) (A) and (D) only (c) Particular affirmative
(d) (C) and (D) only (d) Particular negative
40. Among the following, identify the two statements 50. The proposition ‘no men are perfect’ is an example of
which are in such a relation that the truth of one (a) Universal affirmative (b) Universal negative
implies the truth of the other, but not conversely. (c) Particular affirmative (d) Particular negative
 (December 2018)

M06_MADAN 07_65901_C06.indd 41 27/12/22 8:18 PM


6.42 Chapter 6

51. ‘Some flowers are green’ is an example of 62. Which of the following terms show the relationship
(a) Universal affirmative (b) Universal negative between two propositions having the same subject,
(c) Particular affirmative (d) Particular negative but differing in both quality and quantity?
52. The proposition ‘some Indians are not spiritual’ is an (a) Contrary opposition
example of (b) Contradictory opposition
(a) Universal affirmative (c) Sub-alternation
(b) Universal negative (d) Sub-contrary
(c) Particular affirmative 63. The relation between two particular propositions that
(d) Particular negative have the same subject and predicate, but differing in
53. When a term is used in its entire extent, referring to quality is
the objects denoted by the term, it is said to be (a) Contrary (b) Contradictory
(c) Sub-alternation (d) Sub-contrary
(a) Undistributed (b) Excluded
64. Which of the following shows that an opposition is the
(c) Distributed (d) Verified
relation between two propositions having the same
54. When a term refers only to a part of the class of things subject and predicate, but differing in quantity only?
denoted by the term, the term is said to be (a) Contrary (b) Contradictory
(a) Undistributed (b) Unexcluded (c) Sub-alternation (d) Sub-contrary
(c) Distributed (d) Verified
65. If one of the contraries is true, then the truth value of
55. Universal affirmative proposition distributes the other is
(a) Subject (a) True (b) Partially true
(b) Predicate (c) False (d) Neither true nor false
(c) Both subject and predicate
66. If one of the contraries is false, then the truth value of
(d) Neither subject nor predicate the other is
56. Universal negative proposition distributes (a) True (b) False
(a) Subject (c) Neither true nor false (d) Doubtful
(b) Predicate
(c) Both subject and predicate 67. If one of the contradictories is true, then the other must
(d) Neither subject nor predicate be
(a) True (b) False
57. Particular affirmative proposition distributes
(a) Subject (c) Doubtful (d) Neither true nor false
(b) Predicate 68. Both the contrary propositions cannot be
A S S E S S YO U R L E A R N I N G

(c) Both subject and predicate (a) True (b) False


(d) Neither subject nor predicate (c) True and false (d) Doubtful
58. Particular negative proposition distributes 69. Of the two sub-contraries, if one is false, the other is
(a) Subject necessarily
(b) Predicate (a) True (b) False
(c) Both the subject and the predicate (c) Doubtful (d) Neither true nor false
(d) Neither the subject nor the predicate
70. If one of the two sub-contraries is true, then the other
59. The process of passing directly from a single proposi- one is
tion to a conclusion is (a) True (b) False
(a) Immediate inference (b) Mediate inference
(c) Doubtful (d) Neither true nor false
(c) Definition (d) Classification
60. Which of the following asserts the agreement or 71. Between sub-alternations, if the universal is true,
­disagreement of a subject and predicate after having then the particular is
compared each with middle term? (a) True (b) False
(a) Immediate inference (c) Doubtful (d) Both true and false
(b) Mediate inference 72. If the particular proposition of a sub-alternation rela-
(c) Definition tion is false, then its corresponding universal proposi-
(d) Classification tion will be
61. Which of the following   shows the relationship (a) True (b) False
between two universal propositions having the same (c) Both true and false (d) Doubtful
subject, but differing in quality only? 73. If a particular proposition of a sub-alternation relation
(a) Contrary opposition is true, then the truth value of the universal proposi-
(b) Contradictory opposition tion is
(c) Sub-alternation (a) True (b) False
(d) Sub-contrary (c) Both true and false (d) Doubtful

M06_MADAN 07_65901_C06.indd 42 27/12/22 8:18 PM


Logical Reasoning 6.43

74. The proposition, ‘Chanakya is wise’ is an example of 86. No conclusion is possible from two
(a) Negative proposition (a) Universal premises (b) Affirmative premises
(b) Singular proposition (c) Negative premises (d) Categorical premises
(c) Emotive proposition
87. If one premise is particular, then the conclusion
(d) Invalid proposition
must be
75. The quantity of the proposition is determined by the (a) Universal (b) Negative
extension of the (c) Affirmative (d) Particular
(a) Subject
88. Which one of the following is not an argument?
(b) Predicate
(c) Both subject and predicate (a) If today is Tuesday, tomorrow will be Wednesday.
(d) Copula (b) Since today is Tuesday, tomorrow will be
Wednesday.
76. In a conditional proposition, the part which expresses
(c) Ram insulted me, so I punched him in the nose.
the condition by ‘if’ or its equivalent is
(a) Antecedent (b) Consequent (d) Ram is not at home, so he must have gone to town.
(c) Opposite (d) Meaning 89. Which one of the following statements is completely
77. Which of the following terms describe the syllogism? nonsensical?
(a) Mediate and deductive (a) He had been a bachelor, but he married recently.
(b) Immediate and deductive (b) He is a bachelor, but he married recently.
(c) Mediate and inductive (c) When he married, he was not a bachelor.
(d) Immediate inductive (d) When he was a bachelor, he was not married.
78. The major term is the 90. Venn diagram is a kind of diagram to
(a) Subject of the conclusion (a) Represent and assess the truth of elementary
(b) Copula inferences with the help of Boolean Algebra of
(c) Predicate of the conclusion classes.
(d) Predicate of the minor premise (b) Represent and assess the validity of elementary
inferences with the help of Boolean Algebra of
79. The minor term is the
classes.
(a) Subject of the conclusion
(b) Predicate of the conclusion (c) Represent but not assess the validity of elemen-
(c) Subject of the major premise tary inferences with the help of Boolean Algebra
(d) Predicate of the major premise of classes.
(d) Assess but not represent the validity of elemen-
80. That term which occurs in the premises and not in the tary inferences with the help of Boolean Algebra
conclusion is of classes.

A S S E S S YO U R L E A R N I N G
(a) Major term (b) Minor term
(c) Middle term (d) None of the above 91. ‘A is true because B is true; B is true because A is true.’
This type of argument is termed as
81. Which of the following performs the function of an
(a) Inductive argument
intermediary?
(a) Major term (b) Middle term (b) Deductive argument
(c) Minor term (d) Copula (c) Circular argument
(d) None of the above
82. How many times each term occurs in the syllogism?
(a) Once (b) Twice 92. Which of the following is an example of circular
(c) Thrice (d) Four times argument?
83. All Indians are hardworking. All Punjabis are Indians. (a) God created man in his image, and man created
Therefore, all Punjabis are hardworking. God in his own image.
The above argument is (b) God is the source of scripture and the scripture is
(a) Invalid  (b) False  (c) Valid  (d) True the source of our knowledge of God.
(c) Some of the Indians are great because India is great.
84. If one premise is negative, then the conclusion
must be (d) Rama is great because he is Rama.
(a) Negative 93. Determine the nature of the following definition:
(b) Positive ‘Poor’ means having an annual income of ` 1000.
(c) Both negative and positive (a) Persuasive (b) Precising
(d) Neither negative nor positive (c) Lexical (d) Stipulative
85. Combination of which of the following two types of 94. In the expression, ‘Nothing is larger than itself’, the
premises yields no valid conclusion? relation ‘is larger than’ is
(a) Universal (b) Particular (a) Antisymmetric (b) Asymmetrical
(c) Affirmative (d) Categorical (c) Intransitive (d) Irreflexive

M06_MADAN 07_65901_C06.indd 43 27/12/22 8:18 PM


6.44 Chapter 6

95. Determine the nature of the following definition, 104. Consider the following statements.
‘­Abortion means the ruthless murder of innocent 1. The sunset is beautiful.
beings’. 2. Mahatma Gandhi believed in non-violence.
(a) Lexical (b) Persuasive 3. Do not tell the world what you can do, just do it.
(c) Stipulative (d) Theoretical 4. Those who own a car are rich.
96. Which one of the following is not an argument? Which one of the following is correct?
(a) Abhimanyu does not eat in the day so he must Fact Opinion Advice Assumption
eat at night.
­ (a) 1 4 3 2
(b) If Abhimanyu is growing fat and if he does not (b) 2 3 4 1
eat during the day, he will be eating at night. (c) 4 2 1 3
(c) Abhimanyu eats in the night so he does not eat (d) 2 1 3 4
during the day. 105. Education and socio-economic development are
(d) Since Abhimanyu does not eat in the day, he (a) Related in a direct proportion.
must be eating in the night. (b) Related in an indirect proportion.
97. If P is true, Q is true. If P is false, Q is false. The rela- (c) Sometimes related and sometimes not related.
tion of this proposition is (d) Not related
(a) Independent (b) Equivalent 106. Bats are mammals because bats suckle their young,
(c) Subcontrary (d) Contradictory and
98. Two propositions with the same subject and predi- (a) All those who suckle their young are mammals.
cate terms, but different in quality are (b) All mammals suckle their young.
(a) Contradictory (c) Some of those who suckle their young are
(b) Contrary mammals.
(c) Subalternation (d) All of the above
(d) Subcontraries
107. ‘A single shelf of a good European library is worth
99. Which of the following statements are always true? the whole native literature of India and Arabia.’ This
1. A wooden table is a table. statement is
2. Now, it is raining or not raining. (a) Factual (b) Logical
3. The sun rises in the East every day. (c) Opinion (d) Reactionary
4. A chicken comes out of a hen’s egg.
Codes: 108. Consider the following statements:
(a) 1 and 3 (b) 1, 3 and 4 I think
(c)1 and 2 (d) 2 and 3 Thinking things exist
100. Persons educated through a foreign language are Therefore, I am (i.e., I exist as a thinking thing)
A S S E S S YO U R L E A R N I N G

sure to be unpatriotic. Mark the answer as follows: This argument is


(a) The statement is a fact. (a) Valid (b) Invalid
(b) The statement is an advice. (c) Doubtful (d) Cannot be said
(c) The statement is an opinion. 109. Which one of the following statements follow from
(d) The statement is a prejudice. the statement ‘only goods trains stop at this station’?
101. Statement: ‘The decline of British Empire should (a) Some goods trains stop at this station.
have resulted in the decline of English language.’ (b) All goods trains stop at this station.
Mark the answer as follows: (c) Some goods trains do not stop at this station.
(a) If the statement is a fact. (d) All trains that stop at this station are goods
(b) If the statement is an advice. trains.
(c) If the statement is an opinion. 110. Evaluate the following statements in terms of
(d) If the statement is a prejudice. whether each is a fact, opinion, prejudice, or advice.
102. If ‘X loves Y’, then what can be inferred about ‘Y loves 1. Women are not suitable for police service.
X’? It is 2. In a democracy, no party should be in power for
(a) True (b) False too long.
(c) May be true (d) None of the above 3. Have proper rest, even during the examination
103. ‘Honesty is the best policy’ because time.
(a) God rewards those who follow this maxim. 4. Obesity is a risk factor for coronary heart disease.
(b) It leads to recognition in the society. Which one of the following is correct?
(c) It facilitates cohesiveness in society. Opinion Prejudice Fact Advice
(d) It leads to material prosperity and spiritual (a) 2 3 4 1
awakening. (b) 2 1 4 3
(c) 3 1 4 2
(d) 2 4 3 1

M06_MADAN 07_65901_C06.indd 44 27/12/22 8:18 PM


Logical Reasoning 6.45

111. Information: NET test is conducted for postgraduates. 117. This worker is a graduate. No one else in the factory
Inferences: is a graduate.
(a) Kumar is a postgraduate, so he should have (a) All workers in the factory are graduates.
undertaken the test. (b) Everybody in the factory has a graduation
(b) Kamal is a graduate. She cannot take the NET certificate.
examination. (c) Some of the factory workers have higher
(c) All postgraduates can take the NET examination. qualification.
(d) No other agency can take the NET examination. (d) Only one worker in the factory is a graduate.
112. For a proposition to be true, it is necessary that it 118. All that glitters is not gold. [June 2003]
should have all the following characteristics EXCEPT (a) Many things that shine are other than gold.
(a) It must be objective. (b) Whatever shines is other than gold.
(b) It must be in tune with accepted beliefs. (c) Gold is not the only glittering substance.
(c) It must be consistent. (d) All of the above
(d) It must be testable. 119. All students are not geniuses. [June 2003]
113. Four inferences are drawn from the statement given (a) Many students are not genius.
below, which one can be the correct inference? (b) All geniuses are students.
 [December 1998] (c) No student is a genius.
Statement (d) None of the above
People in Australia generally speak English. 120. Which of the following statements say the same
Inferences thing? [December 2006]
(a) Rosy speaks English. So she is from Australia. 1. ‘I am a teacher’ (said by Arvind).
(b) Rahim knows English. That is why he is going to 2. ‘I am a teacher’ (said by Binod).
Australia next month. 3. My son is a teacher (said by Binod’s father).
(c) Rahul has been living in Australia, so he most 4. My brother is a teacher (said by Binod’s sister).
probably can speak English. 5. My brother is a teacher (said by Binod’s only
(d) Ramesh does not know English. He cannot talk sister).
to anyone if he goes to Australia. 6. My sole enemy is a teacher (said by Binod’s only
Directions (Questions 114–135): In each of the questions enemy).
below, a proposition is followed by four answer choices. Codes:
Match the proposition with one of the choices that seems (a) 1 and 2 (b) 2, 3, 4, and 5
to be the most justified or applicable. (c) 2 and 4 (d) 5 and 6
114. The spread of the Internet for higher education is 121. Which of the following are the correct ways of

A S S E S S YO U R L E A R N I N G
premised on [June 2000] ­arguing? [December 2006]
(a) Research and development are vital. 1. There can be no second husband without a sec-
(b) Browsing encourages critical thinking. ond wife.
(c) Easy management and dissemination of 2. Anil is a friend of Bob, Bob is a friend of Raj, hence
knowledge. Anil is a friend of Raj.
(d) India should be second to none in the world. 3. A is equal to B, B is equal to C, and hence, A is
equal to C.
115. World Trade Organization believes in intellectual
4. If everyone is a liar, then we cannot prove it.
property and liberalized trade regime.[June 2000]
Their belief is based on the assumption that Codes:
(a) Modern economy bestows equal benefits. (a) 3 and iv (b) 1, 3, and 4
(b) Knowledge-based economy and globalization (c) 2, 3, and 4 (d) 1, 2, 3, and 4
are reality. 122. Which of the following is an analogous statement?
(c) All countries are well suited for modern trade.  [December 2009]
(d) Rich countries will always help the poor countries. (a) Man is like God.
116. The essence and justification to beauty contests (b) God is great.
is that [June 2000] (c) Gandhi is the father of the nation.
(a) Women have standard features on which they (d) Man is a rational being.
can be rated.
123. ‘No men are mortal’ is contradictory to[June 2009]
(b) Beautiful women have a vision and role to play
(a) Some men are mortal.
in social upliftment.
(b) Some men are not mortal.
(c) They symbolize eternal freedom for women.
(c) All men are mortal.
(d) They provide an opportunity for beautiful
(d) No mortal is man.
women of the world to come together on one
platform.

M06_MADAN 07_65901_C06.indd 45 27/12/22 8:18 PM


6.46 Chapter 6

124. Certainty is [June 2010] (a) India Publishing House publishes textbooks only.
(a) An objective fact (b) Some textbooks written by Professor Bhardwaj
(b) Emotionally satisfying are published by publishers other than M/s
(c) Logical India Publishing House.
(d) Ontological (c) All the books published by M/s India Publishing
125. Inductive logic studies the way in which a premise House have been written by Professor Bhardwaj.
may [December 2011] (d) None of the above.
(a) Support and entail a conclusion. 133. All students in my class are bright. Mehtab is not
(b) Not support but entail a conclusion. bright.
(c) Neither support nor entail a conclusion. (a) Some students are not bright.
(d) Support a conclusion without entailing it. (b) Mehtab is not a student of my class.
126. Most dresses in that market are expensive, this (c) Mehtab must change the school.
means (d) No student is dull.
(a) There are no cheap dresses available in that 134. Which one of the following is not correct? A belief
market. becomes a scientific truth when it
(b) There are some cheap dresses also in that (a) Is established experimentally
market. (b) Is arrived logically
(c) Some dresses in that market are expensive. (c) Is accepted by many people
(d) None of the above (d) Can be replicated
127. Every library has books. 135. Child labour can best be eradicated if the following is
(a) Books are only in a library. done for the concerned children
(b) Libraries are meant for books only. (a) Schools are opened and free lunch is provided.
(c) No library is without books. (b) Employment is provided to parents and free
(d) Some libraries do not have readers. education is given to children.
(c) Appropriate laws are enacted and enforced.
128. The electronic media convinces its viewers that the (d) Employers of child labourers are punished and
likelihood of their becoming the victim of a violent
crime is extremely high; at the same time, by its children are sent to school.
very nature, it persuades viewers to passively accept 136. A reasoning where we start with a particular statement
whatever happens to them. and conclude with a universal statement is called a
(a) Exposure to electronic media promotes criminal (a) Deductive reasoning
behaviour. (b) Inductive reasoning
(b) The users of electronic media are more vulner- (c) Abnormal reasoning
able to become victims of violence than others. (d) Transcendental reasoning
A S S E S S YO U R L E A R N I N G

(c) Electronic media promotes a feeling of helpless 137. To be critical, thinking must be
vulnerability in viewers. (a) Practical (b) Socially relevant
(d) None of the above. (c) Individually satisfying (d) Analytical
129. This book can help because all good books help. 138. Consider the argument provided below:
(a) This is not a good book.  [Jun 2019]
(b) This is a good book.
‘Sound is impermanent because it is invisible’
(c) No good book helps.
(d) Some good books help. Identify the fallacy involved in the above argument
on the basis of Indian logic
130. Soldiers serve their country. (a) Fallacy of irrelevant reason
(a) Men mostly serve their country. (b) Fallacy of wrong assertion
(b) The persons who serve their country are soldiers. (c) Fallacy of trivial reason
(c) Some people who are soldiers serve their (d) Fallacy of contradictory reason
country.
(d) Women usually do not serve their country as 139. If ‘All men are mortal’ is given as True, then which of
they are not soldiers. the following options can be validly inferred from it?
 [2021]
131. To pass any competitive exam, one must work hard. A. ‘No men is mortal’ is False
(a) Getting good grades in exam needs hard work. B. ‘Some men are mortal’ is True
(b) All those who work hard, pass. C. ‘Some men are not mortal’ is True
(c) The candidates who work hard overcome anxi- D. ‘Some men are not mortal’ is False
ety in the competitive exam. E. ‘Some men are mortal’ is False
(d) Without hard work, one does not pass the com-
petitive exams. Choose the correct answer from the options given
below:
132. All the books written by Professor Bhardwaj are (a) A, B and D only (b) A, B and C only
textbooks. Some of his books are published by India
Publishing House. (c) A, B, C and D only (d) B, C, D and E only

M06_MADAN 07_65901_C06.indd 46 27/12/22 8:18 PM


Logical Reasoning 6.47

140. ________ messages have more or less same meaning 148. Look at the following statement.
for the audience. [2021] ‘My mother must be visiting, since there is a
(a) Abstract (b) Complex Mercedes parked outside’.
(c) Connotative (d) Denotative What kind of fallacy does it indicate?
141. Consider the following argument. Statement: (a) Fallacy of deviation
Some chairs are curtains. (b) Fallacy of contradiction
All curtains are bed sheets. (c) Fallacy of unestablishment
Conclusion: Some chairs are bed sheets. (d) Fallacy of rebuttal
What is the mood of the above proposition? 149. Look at the following statement
(a) IAI (b) IAA ‘Santa was not a kind man, since he gave his life for
(c) IIA (d) AII others.’
What kind of fallacy does it indicate?
142. ‘Everyone is going to the party. You should go too’.
(a) Fallacy of deviation
This inference commits which kind of fallacy?
(a) Ad populum (b) Equivocation (b) Fallacy of contradiction
(c) Ad verecundiam (d) Ad ignorantiam (c) Fallacy of unestablishment
(d) Fallacy of rebuttal
143. The inference ‘A mouse is an animal. Therefore, a
large mouse is a large animal’ commits which one of 150. In a Buddhist work, Katha— -vatthu, there is demon-
the following fallacies? stration of many inconsistencies between a propo-
(a) Straw man nent and an opponent. This title was written by
(b) Slippery slope (a) Kalidasa (b) Moggaliputta Tissa
(c) Equivocation (c) Varahmihir (d) Buddhyana
(d) Fallacy of composition 151. Look at the following statement
144. Humans and animals are both living, breathing I know that your mother is in town, since I saw a
beings. Humans have civil rights. Therefore, animals Rolls Royce parked outside your home.
have civil rights. What kind of fallacy does it indicate?
These statements indicate the following fallacy. (a) Fallacy of deviation
(a) False analogy (b) Fallacy of contradiction
(b) Ad hominem (c) Fallacy of unestablishment
(c) Red herring (d) Fallacy of rebuttal
(d) Complex question
152. When counterarguments of equal or greater force
145. If we teach sex education in school, then students will are put forth in support of an opposing conclusion,
have more sex. If students have more sex, we will have it is called a

A S S E S S YO U R L E A R N I N G
a rash of unplanned pregnancies and sexually trans- (a) Fallacy of deviation
mitted diseases. Students will be forced to drop out of (b) Fallacy of contradiction
school and will never have the chance to succeed in (c) Fallacy of counterbalanced
life. (d) Fallacy of rebuttal
These statements indicate which of the following
fallacy? 153. What is the number of ‘Moods’?
(a) False analogy (b) Ad hominem (a) 4 (b) 16
(c) Red herring (d) Ignoratio Elenchi (c) 64 (d) 256
146. People have been seeing ghosts for hundreds of 154. To test an argument for fallacies is to focus on the
years. No one has been able to prove definitively that ­concepts of
ghosts do not exist. Therefore, ghosts are real. (a) Ethos (b) Logos
These statements indicate which of the following (c) Pathos (d) All of the above
fallacy? 155. Consider the following statements in the context of
(a) Fallacy of ignorantiam ‘moods and figures’?
(b) Fallacy of Ad hominem 1. The ‘mood’ of a syllogism is determined by the
(c) Begging the question ‘quantity’ and ‘quality’ of the three propositions.
(d) Fallacy of Red herring 2. The ‘figure’ of a categorical syllogism is deter-
147. Murder is morally wrong. Abortion is murder. There- mined by ‘middle term’.
fore, abortion is morally wrong. 3. Figure and mood together determine the struc-
These statements indicate which of the following ture of syllogism.
fallacy? Codes:
(a) Argument ad Populum (a) 1 and 2 are correct
(b) Fallacy of slippery slope (b) 2 and 3 are correct
(c) Fallacy of strawman (c) 1 and 3 are correct
(d) petitio principii (d) 1, 2, and 3 are correct

M06_MADAN 07_65901_C06.indd 47 27/12/22 8:18 PM


6.48 Chapter 6

156. Identify the features of Aristotelian syllogism from 158. “The Daily News carried an article this morning
the following: [2020] about some local teenagers who were arrested on
(A) It is deductive-inductive and formal the charges of drug possession. Teenagers these
material. days are nothing but a bunch of junkies”.
(B) It is only deductive and formal. Which fallacy is committed in the above statement?
(C) The major and minor terms stand apart in the  [2021]
premises. (a) Red Herring
(D) It is verbalistic.
(b) Strawman
(E) Verbal form is not the essence of inference.
(c) Argument against the person (ad hominem)
Choose the correct answer from the options given
(d) Hasty Generalization
below:
(a) (A), (B) and (C) only 159. The inference ‘A mouse is an animal. Therefore, a
(b) (A), (C) and (E) only large mouse is a large animal’ commits which one of
(c) (B), (C) and (D) only the following fallacies? [Dec 2019]
(d) (A), (D) and (E) only (a) Straw man
(b) Slippery slope
157. Name the fallacy committed in the argument below:
 [2021] (c) Equivocation
(d) Fallacy of composition
“All men who understand women are potentially
perfect husbands. 160. ‘Mr X lives in a slum and is unemployed. Therefore,
Mr X deserves to be a minister’.
All potentially perfect husbands are men of infinite
Which kind of fallacy is committed in this argu-
patience.
ment?  [Dec 2019]
Therefore, some men of infinite patience are men (a) Fallacy of composition
who understand women.” (b) Ad misericordiam
(a) Exclusive premises (c) Fallacy of division
(b) Existential fallacy (d) Fallacy of accident
(c) Illicit major
(d) Undistributed middle

Practical Problems
Directions (Questions 161–175): In each of the ques- I. Some men are boys.
tions below, two statements are followed by the two con- II. All boys are fathers.
A S S E S S YO U R L E A R N I N G

clusions, I and II. Assuming that the given statements 164. Statements
are true, even if they are at variance with the commonly All teachers are girls.
known facts, pick one of the following answers that you No girl is dull.
think is correct. Conclusions
(a) If only conclusion I follows. I. No boy is a teacher.
(b) If only conclusion II follows. II. No teacher is dull.
(c) If both I and II follows.
(d) If neither I nor II follows. 165. Statements
All poets are inspiring.
161. Statements All artists are inspiring.
Some tables are golden. Conclusions
All teaks are tables. I. All artists are poets.
Conclusions II. Some inspiring persons are not artists.
I. Some teaks are golden.
II. Some golden are teak. 166. Statements
No tree is bottle.
162. Statements No bottle is unbreakable.
No man is a tiger. Conclusions
Karan is a man. I. No tree is unbreakable.
Conclusions II. Nothing unbreakable is a tree.
I. Karan is not a tiger. 167. Statements
II. All men are not Karan. All horses are donkeys.
163. Statements All donkeys are monkeys.
All boys are men. Conclusions
All men are fathers. I. All horses are monkeys.
Conclusions II. All monkeys are horses.

M06_MADAN 07_65901_C06.indd 48 27/12/22 8:18 PM


Logical Reasoning 6.49

168. Statements 176. Statements


No villagers own a car. All magic are women.
Shankar owns a car. All women are crazy.
Conclusions Conclusions
I. Shankar lives in a town. 1. All magic are crazy.
II. Shankar owns a car. 2. All crazy are magic.
169. Statements 3. Some crazy are magic.
Some teachers are women. 4. Some crazy are women.
No teacher is absent. Choices
Conclusions (a) Only I, III, and IV follows.
I. There are no male teachers. (b) Only II and III follows.
II. All women teachers are present. (c) All conclusions follow
170. Statements (d) None of the conclusions follow.
Some benches are chairs. 177. Statements
Hammer is a bench. All cats are mammals.
Conclusions No tigers are cats.
I. Some chairs are benches. Conclusions
II. Hammer is not a chair. (a) No tiger is a mammal.
171. Statements (b) No mammals are tigers.
All books are stones. (c) Cats are tigers.
All stones are papers. (d) None of the above
Conclusions 178. Statements
I. Some papers are books. Some boxes are round.
II. Some papers are stones. All rounds are spheres.
172. Statements Conclusions
Only graduates are eligible for this post.   I. Some boxes are spheres.
Most rickshaw-pullers are graduates.    II. Some spheres are boxes.
Conclusions III. Some spheres are round.
I. Some rickshaw-pullers are eligible for this post. IV. All spheres are round.
II. All those eligible for this post are graduates. Choices
173. Statements (a) I, II, and III follows.
All grapes are oranges. (b) II, III, and IV follows.
Some apples are not oranges. (c) I, III, and IV follows.

A S S E S S YO U R L E A R N I N G
Conclusions (d) I, II, and IV follows.
I. All apples are grapes. 179. Statements
II. Some apples are grapes. All books are clocks.
174. Statements Some clocks are chips.
All students are ambitious. Conclusions
All ambitious persons are hardworking.    I. Some clocks are books.
Conclusions II. No clocks are books.
I. All students are hardworking. III. Some books are chips.
II. All hardly working people are not ambitious. IV. No books are chips.
175. Statements Choices
All gardens are schools. (a) I and III follows.
All schools are colleges. (b) Only I follows.
Conclusions (c) Either I or II follows.
I. All gardens are colleges. (d) Either III or IV and I follows.
II. Some gardens are not colleges. 180. Statements
Directions (Questions 176–192): In each of the ques- Some tables are TVs.
tions below, there are two statements followed by three Some TVs are radios.
to four conclusions numbered as I, II, III, and IV. You Conclusions
have to take the given statements to be true even if they    I. Some tables are radios.
seem to be at variance with commonly known facts and II. Some radios are tables.
then decide which of the given conclusions logically fol- III. All radios are TVs.
low from the given statements. IV. All TVs are tables.

M06_MADAN 07_65901_C06.indd 49 27/12/22 8:18 PM


6.50 Chapter 6

Choices (c) Only either I or IV follows.


(a) Only I and III follows. (d) None follows
(b) Only II and IV follows. 185. Statements
(c) All follow All politicians are honest.
(d) None follows All honest are fair.
181. Statements Conclusions
Some rabbits are bears.   I. Some honest are politicians.
No goats are bears. II. No honest is politician.
Conclusions III. Some fair are politicians.
  I. Some rabbits are not goats.   IV. All fair are politicians.
  II. All rabbits are goats. Choices
III. Some goats are not rabbits. (a) None follows
  IV. All goats are rabbits. (b) Only I follows
Choices (c) Only I and II follows.
(a) Only I follows. (d) Only I and III follows.
(b) Only III follows. 186. Statements
(c) Either I or II follows. All the students passed the examination.
(d) Either III or IV and I follows. Some students are girls.
182. Statements Conclusions
No systems are desks.   I. Some boys passed the examination.
All desks are books. II. All the girls failed the examination.
Conclusions III. None of the boys passed the examination.
  I. Some systems are books.   IV. None of the girl students failed the examination.
  II. Some systems are not books. Choices
III. Some books are systems. (a) Only I and II follows.
  IV. Some books are not systems. (b) Only II and III follows.
Choices (c) Only I, II, and III follows.
(a) Only II follows. (d) None of the above
(b) Only IV follows. 187. Statement
(c) I and IV follows. Most of the Indian states existed before
(d) Either I or II and IV follows. independence.
Conclusions
183. Statements I. Some Indian states existed before independence.
A S S E S S YO U R L E A R N I N G

All branches are flowers. II. All Indian states did not exist before independence.
All flowers are trees. Choices
Conclusions (a) Only I is implied.
  I. All branches are trees. (b) Only II is implied.
  II. All trees are branches. (c) Both I and II are implied.
III. All flowers are branches. (d) Neither I nor II is implied.
  IV. Some trees are branches. 188. Proposition
Choices No teacher is on time for the class.
(a) Only I and IV follows. Conclusions
(b) Only II and III follows.
   I. No persons who are on time for their classes are
(c) All follow
teachers.
(d) None follows
  II. Some teachers are not late for their classes.
184. Statements III. Most teachers come to their classes on time.
Some bags are pockets.   IV. Few teachers come on time for their classes.
No pocket is a pouch.
189. Statement
Conclusions
Necessity is the mother of all inventions.
  I. No bag is a pouch.
Conclusions
  II. Some bags are not pouches.
  I. There can be no invention without there being a
III. Some pockets are bags.
mother.
  IV. No pocket is a bag.
II. Mother is a necessity.
Choices
Choices
(a) Only I and III follows.
(a) Only I is implied.
(b) Only II and III follows.
(b) Only II is implied.

M06_MADAN 07_65901_C06.indd 50 27/12/22 8:18 PM


Logical Reasoning 6.51

(c) Both I and II are implied. Choices


(d) Neither I nor II is implied. (a) Only inference I is correct.
190. Statement (b) Only inference II is correct.
Most teachers are hardworking. (c) Both inferences I and II are correct.
Conclusions (d) Neither inference I nor inference II is correct.
  I. Some teachers are hardworking. 192. Statements
II. Some teachers are not hardworking. All aeroplanes are trains.
Choices Some trains are chairs.
(a) Only I is implied. Conclusions
(b) Only II is implied.   I. Some aeroplanes are chairs.
(c) Both I and II are implied.   II. Some chairs are aeroplanes.
(d) Neither I nor II is implied. III. Some chairs are trains.
191. Statement   IV. Some trains are aeroplanes.
No man is infallible. Codes:
Conclusions (a) None follows
  I. All men are fallible. (b) Only I and II follow.
II. No infallible persons are men. (c) Only II and III follow.
(d) Only III and IV follow.

Miscellaneous
193. Which of the following statements are mutually 197. Proposition
inconsistent? No teacher is on time for the class.
1. Mostly poets are not egoistic. Conclusions
2. Mostly poets are humble. (a) No persons who are on time for their classes are
3. Some poets are egoistic. teachers.
4. Some poets are not non-egoistic. (b) Some teachers are not late for their classes.
(c) Most teachers come to their classes on time.
Codes:
(d) Few teachers come on time for their classes.
(a) 1 and 4 (b) 2 and 3
(c) 1 and 3 (d) 3 and 4 198. Which of the following statements are mutually

A S S E S S YO U R L E A R N I N G
contradictory?
194. Statement: If all men are mortal, and if Rama is a 1. All flowers are not fragrant.
man, Rama is also mortal. 2. Most flowers are not fragrant.
(a) The premise is true and the conclusion is true. 3. No flowers are fragrant.
(b) The premise is false and the conclusion 4. Most flowers are fragrant.
is false. Codes:
(c) The premise is false and the conclusion is true. (a) 1 and 2 (b) 1 and 3
(d) The premise is true and the conclusion is false. (c) 2 and 3 (d) 3 and 4
195. If ‘no politician is dishonest’ is false, then the state- Directions (Questions 199 and 200): In each of the
ment ‘some politicians are dishonest’ shall be questions below, two statements are given ­followed by
(a) True (b) False an inference. Mark (A) if the ­inference is definitely true,
(c) May be true (d) None of the above mark (B) if the inference is definitely false, mark (C) if
196. If the statement ‘some men are cruel’ is false, which the inference is probably false or true, and mark (D) if the
of the following statement/s is/are true? inference cannot be drawn.
1. All men are cruel. 199. Statements
2. No men are cruel. 1. Glass is brittle.
3. Some men are not cruel. 2. This substance is not brittle.
Codes: Inference
(a) 1 and 3 (b) 1 and 2 This substance is not glass.
(c) 2 and 3 (d) 3 only 200. Statements
1. Some intelligent people are happy.
2. Some intelligent people are rich.
Inference
Some people who are rich are happy.

M06_MADAN 07_65901_C06.indd 51 27/12/22 8:18 PM


6.52 Chapter 6

Indian Logic: Means of Knowledge


201. The ultimate objective of the Nyaya system is (c) Vaisheshika system
(a) To bring an end to human suffering, which (d) Jain system
results from ignorance of reality 210. The Nyaya theory of causation defines a cause as
(b) To make a person happy with all material an unconditional and invariable antecedent of an
wealth effect. Match the following sets of causes:
(c) To make a person happy in all possible manners
(d) All of the above Type of cause Meaning
202. Which of the following gurus wrote A. Inherent I. It helps in the
‘Vaisheshika­sutras’? production of a cause
(a) Kanada (b) Gautama B. Non-inherent II. The substance out
(c) Mahatma Buddh (d) Vatsayana cause of which an effect is
203. ‘Inference is considered to be a mere leap into produced
the dark’. C. Efficient cause III. The power that helps
Which of the following schools considers the above the material cause
statement? produce the effect
(a) Nyaya School (b) Mimamsa School
Codes:
(c) Charvaka School (d) None of the above (a) A-I, B-II, C-III (b) A-II, B-I, C-III
204. With which of the following did the Vedanta (c) A-II, B-III, C-I (d) A-III, B-I, C-II
schools have a special affiliation? 211. Nyaˉya epistemologists speak of cognition. Which
(a) The authority of shruti–that which is heard of the following are the components of such
(b) The Mimamsa–the questions after interpreting cognition?
the sacred texts 1. Jña
ˉna
(c) Both a and b 2. Buddhi
(d) None of the above 3. Upalabdhi
205. Mahatma Buddha teaches his Four Noble Truths. 4. Pratyaya
Which of the following statements does not apply? Codes:
(a) Dukkha–dissatisfaction with existence in the (a) 1 and 2 (b) 1, 2 and 3
phenomenal world (c) 1, 3 and 4 (d) All of the above
(b) The origination of dukkha in craving or desire
A S S E S S YO U R L E A R N I N G

(c) The cessation of dukkha 212. Which of the following statements are true in the
(d) The way leading to that cessation by following context of Vyapti?
Four fold Path 1. Vyapti is a relation between Hetu and Sadhya.
206. Which of the following is considered to be the most 2. Vyapti means co-presence, co-absence,
sophisticated natural theologies? co-presenceabsence of Hetu and Sadhya
(a) Nyaya (b) Buddhism 3. Vyapti (the invariable association of middle and
(c) Jainism (d) None of the above major terms) serves as the basis of Nyaya syllogism
Codes:
207. Upamana is basically
(a) Anumana (a) 1 and 2 (b) 2 and 3
(b) Testimony (c) 1 and 3 (d) 1, 2 and 3
(c) Comparison 213. Which of the following terms is used for the word
(d) Object of knowledge ‘probandum’?
208. Match the following three terms of Indian Logic (a) Sadhya (b) Vyapati
with Aristotelian logic. (c) Hetu (d) Anumana
Indian Logic Aristotelian Logic 214. Which of the following statements are true in the
A. Paksa I. Middle Term context of ‘anumana’?
B. Sadhya II. Major Term 1. According to Nyaya, it is the indirect source of
C. Linga III. Minor Term valid knowledge.
Codes: 2. Anumana should have three terms—hetu, sad-
(a) A-I, B-II, C-III (b) A-II, B-I, C-III hya, and paka
(c) A-III, B-II, C-I (d) A-I, B-III, C-II 3. Anumana has five propositions
209. In its metaphysics, Nyaya is allied to the Codes:
(a) Patanjali system (a) 1 and 2 (b) 2 and 3
(b) Buddha System (c) 1 and 3 (d) 1, 2 and 3

M06_MADAN 07_65901_C06.indd 52 27/12/22 8:18 PM


Logical Reasoning 6.53

215. Look at the following statement: 222. Nyaˉya syllogism employs a formal five-step argu-
‘One can infer the existence of fire after perceiving ment. Kindly match the items in List I with those
the smoke that has an uninterrupted connection in List II.
with the surface of the mountain’. List I List II
Which of the following characteristics of ‘anumana’
or inference emerge from the above statement? A. There is fire on the hill I. Pratijña
ˉ
1. It is indirect (paroksa). B. Because there is smoke on II. Hetu
2. It takes place through the medium of some mark the hill
which is called the ‘Hetu’. C. Wherever there is smoke, III. Uda
ˉharaa
ˉa
3. It is invariably connected with the ‘major term’ there is fire, like a kitchen
(Sadhya) hearth and unlike a lake
4. It is that cognition which presupposes some other D. This hill is likewise smoky IV. Upanaya
cognition
5. Anumana is a secondary proof E. Thus, there is fire on the hill V. Nigamana
Codes: Codes:
(a) 1, 2, 3, and 4 (b) 2, 3, and 4 (a) A-I, B-II, C-III, D-V, E-IV
(c) 1, 2, 3, and 5 (d) 1, 2, 3, 4, and 5 (b) A-II, B-I, C-III, D-IV, E-V
(c) A-I, B-II, C-III, D-IV, E-V
216. Which of the following should be considered as the
(d) A-I, B-III, C-II, D-IV, E-V
essential characteristic/s of ‘anumana’ (inference)?
1. Fire is inferred on the hill, where smoke is per- 223. Perception of a universal through an individual which
ceived in it, is called as Paksa̧ dharmata. instantiates, it is Nya
ˉya’s response to the problem of
2. When universal relation between fire and smoke (a) Induction
is known, it is called Vya ˉpti. (b) Deduction
(c) Both induction and deduction
3. Anumiti arises without the coexistence of the
(d) None of the above
Hetu and Sadhya.
Codes: 224. A reason appears to be real or appropriate but in fact
(a) Only 1 (b) Only 2 is not. With which of the following terms, the fallacy
is known as hetvabhasa?
(c) Both 1 and 2 (d) All of the above
(a) Minor Term (b) Middle Term
217. Which of the following is termed as the statement of (c) Major Term (d) All of the above terms
reason? 225. Which of the following should be considered as the

#ntaugc-netimportantquestions
(a) Hetu (b) Pratijana two fundamental requirements for inference?
(c) Upanaya (d) Nigamana 1. Awareness of pakşadharmata ˉ—the inferential
mark’s qualifying the locus of the inference
218. Which of the following statements does not apply in
2. Vyaˉpti—the sign’s invariable concomitance with
context of Anumana?
the target property or probandum
(a) Anumana is knowing something not by means 3. Hetva-bhasha
of contact between the senses and the objects of Codes:
the world (a) 1 and 2 (b) 2 and 3
(b) Anumana is not by observation (c) 1 and 3 (d) None of the above
(c) Anumana is not through the medium of a sign
226. A perception where we do not have the knowledge
(d) Anumana is inference for oneself or inference of an object characterized by any predicate but an
for others. apprehension of some unrelated elements is called
219. What is the number of propositions in Nyaya ­Theory as? [Jun 2019]
of perceptions? (a) Determinate
(a) three (b) four (b) Indeterminate perception
(c) five (d) six (c) Verbal testimony
(d) Implication
220. The oldest extant Nya ˉya text is the Nya
ˉya-su
ˉtra that
is attributed to great guru 227. Which of the following should be considered as the
(a) Patanjali (b) Gautama ­categories of metaphysical study?
(c) Vaˉtsyaˉyana (d) Udayana 1. Substance 2. Quality
3. Action 4. Universal
221. Tarka is basically
5. Individuator 6. Inherence
(a) Inference
7. Absence
(b) Conclusion
(a) 1, 2, 3, 5, and 7 (b) 1, 2, 3, 4, 5, and 6
(c) Hypothetical argument
(c) 3, 4, 5, 6, and 7 (d) All of the above
(d) Grounds of defeat

M06_MADAN 07_65901_C06.indd 53 27/12/22 8:18 PM


6.54 Chapter 6

228. Consider the following statements: 235. Anekantavada denotes the central theme of
1. A water buffalo looks something like a cow and (a) Buddhism (b) Jainism
such buffaloes are present at a certain place in (c) Islam (d) Hinduism
the countryside. 236. Consider the following statements.
2. When out in the countryside, he recognizes that 1. Knowledge is the state of soul itself.
the thing he is seeing is similar to a cow, and 2. 
The knowledge is of two types—the mediate
therefore is a water buffalo. (paroksa, acquired through sense organs) and the
These two statements endorse the concept of immediate (aparoksa, acquired without the inter-
(a) Anumana (b) Upamana vention of the sense organs).
(c) Sabada (d) Pratimana 3. 
Immediate knowledge is divided into Avadhi,
229. Which of the following statements are true in the Manahparyaya, and Kevala; and mediate knowl-
­context of Sabda edge into Mati and Shruta.
Which of the religious practice is being described
1. Sabda is thought of as a transmission of informa-
through the above statements?
tion or content.
(a) Hinduism
2. A person attains an accurate cognition through (b) Jainism
some prama ˉna token. (c) Buddhism
3. Sabda has the widest range of any source of (c) All of the above
knowledge, far outstripping what one may know
from personal perception, inference or analogy. 237. Which of the following concepts of Indian logic sys-
tem denotes circumstantial implication?
Codes:
(a) Upama–na (b) Artha–patti
(a) 1 and 2 (b) 2 and 3 (c) Anupalabdi (d) Pratyaks.a
(c) 1 and 3 (d) All of the above
238. According to some school/s, the proper means of
230. Match the qualities of the universals in List-I with knowledge must rely on these three pramanas:
their examples in List-II (a) Pratyaks.a — perception
List-I List-II (b) Anuma–na — inference
A. Substances I. Redness (c) Śabda — testimony or words of reliable experts
B. Motions II. Pot-hood The answer choices are
C. Qualities III. Contraction-hood (a) Sankhya
Codes: (b) Yoga
(a) A-I, B-II, C-III (b) A-II, B-III, C-I (c) Two sub-schools of Vedanta
#ntaugc-netimportantquestions

(c) A-III, B-II, C-I (d) A-II, B-I, C-III (d) All of the above
231. According to Kumarila Bhatta, anupalabdhi (say 239. The term ‘yogaja’ (intuitive) pertains to which of the
non-apprehension) as the sixth independent source following prama–n.as?
of knowledge consists in the presentative knowledge (a) Perception
of (b) Inference
(a) Negative facts (b) Positive facts (c) Verbal testimony
(c) Both (a) and (b) (d) Neither (a) or (b) (d) Comparison
232. Which of the following statements are true in con- 240. Which of the following schools accept only percep-
text of Arthapatti (Presumption)? tion as prama–n.a?
(a) It is an independent source of knowledge.
(b) It is admitted as a distinct pramana which can- (a) Charvaka school
not be brought under anumana or sabda. (b) Buddhhist school
(c) Both a and b (c) Jainism school
(d) Neither a nor b (d) Nyaya school
233. What is the total number of hetvabhasa or fallacies 241. Which of the following schools accept perception
of inference? and inference as prama–n.as?
(a) 4 (b) 5
(a) Buddhists and Vaiśes.ikas schools
(c) 6 (d) 8
(d) Nyaya school
234. ‘Milind Panho’ explaining the Buddhist doctrines in
the form of a dialogue between Milind and his teacher (c) Jainism school
Nagsena (the great Buddhist philosopher) is in: (d) All of the above
(a) Sanskrit 242. Which of the following accept only these two prama–n.
– –
(b) Pali Language as that are pratyaks.a-praman.a and paroks.a-praman.a?
(c) Hindi (a) Buddhists (b) Nyaya school
(d) Santhali (c) Jainism school (d) Vaiśes.ikas school

M06_MADAN 07_65901_C06.indd 54 27/12/22 8:18 PM


Logical Reasoning 6.55

243. The concept of Prama–n.a in our logic system is 248. Which of the following prama–n.as is used by classical
defined as a means of acquiring prama or certain, Indian school of logic to prove the existence of God?
correct, and true knowledge. (a) Artha–patti (postulation)
Which of the following have not been identified as (b) Upama–n.a (comparison)
the correct form of six Prama–n.a? (c) Pratyaksha (perception)
1. Pratyaks.a – perception (d) Anuma–n.a (inference)
2. Anuma–n.a – inference 249. Which one of the following is signified by Uda–harana
3. Upama–n.a – comparison and analogy of Anuma–n.a (inference) in Indian logic?
4. Artha–patti – implication (a) Statement of reason
5. Anupalabdhi – Apprehension (b) Proposition to be proved
6. Śabda – nonverbal testimony (c) Conclusion proved
Codes: (d) Universal proposition along with an instance
(a) 1 and 3 250. Devdatta is fat and he does not eat during the day.
(b) 2, 3, and 4 Therefore, Devdatta is eating during the night. The
(c) 5 and 6 above example in classical Indian school of logic is a
(d) 2, 4, and 6 case of:
244. Consider the following statements: (a) Comparison (b) Implication
1. Indriyarthasannikarsa—direct experience by (c) Perception (d) Verbal testimony
one’s sensory organ(s) with the object 251. Naiyayaikas have explained five members of syl-
2. Avyapadesya—non-verbal; correct perception is logism. If we take the example that the hill is fiery
not through hearsay because it has smoke, their constituents could be
3. Avyabhicara—does not wander; correct percep- illustrated in the following way:
tion does not change 1. Pratijna: The hill is fiery. (S is P)
4. Vyavasayatmaka—definite; correct perception 2. Hetu: Because it smokes. (S is M)
excludes judgments of doubt 3. Udaharana: Whatever smokes is fiery e.g. as is the
Which of the following are the correct explanations hearth. (M is P)
for Pratyaks.a that means perception? 4. Upanaya: So like the hearth kitchen, the hill is
(a) 1, 2, and 3 (b) 2, 3, and 4 smoky. (S is M)
(c) 1, 3, and 4 (d) All of the above 5. Nigamana: Therefore, the hill is fiery. (S is P)
245. Nyaya mentions four sources or Pramhas of knowl- Which of the above is incorrect?
edge—Perception (Pratyaksa), inference (Anumha), (a) Only 1
comparison (Upamiina), and the Vedic word (b) Only 2

#ntaugc-netimportantquestions
(Shabda). Nyaya argues that the outside world is (c) Only 3
known to us through (d) All of the above are correct.
(a) The senses 252. When the middle term is both positively and
(b) The mind ­negatively related to the major term, the inference is
(c) Both a and b called [2020]
(d) None of the above (a) Based on uniformity of co-existence
(b) Kevalanavayi
246. The distinction between ‘laukika’ and ‘alaukika’ is
(c) Kevala vyatireki
made with reference to which one of the following
(d) Anavaya vyatireki
prama–n.as?
(a) Anuma–n.a (Inference) 253. From the following identify those which illustrate
(b) Upama–n.a (Comparison) the fallacy of the unproved middle? [2020]
(c) Pratyaksha (Perception) (A) Ashrayasidha (B) Swarupasidha
(d) Śabda (Verbal testimony) (C) Vyapyatvasidha (D) Virudha
247. Identify the distinctive feature of traditional method (E) Badhita
of Indian education from the following list. Choose the correct answer from the options given
(a) ‘Direct perception of truth’—both as means and below:
ends (a) (A), (B) and (C) only
(b) Making everything on trust (b) (B), (C) and (D) only
(c) Evolving own ways of learning (c) (A), (D) and (E) only
(d) Following nyaya philosophy with deductive and (d) (A), (C) and (D) only
inductive process

M06_MADAN 07_65901_C06.indd 55 27/12/22 8:18 PM


6.56 Chapter 6

Answer Keys

Theory Questions
1. (b) 2. (b) 3. (c) 4. (a) 5. (b) 6. (b) 7. (a) 8. (b) 9. (d) 10. (c)
11. (b) 12. (a) 13. (a) 14. (d) 15. (c) 16. (d) 17. (d) 18. (c) 19. (b) 20. (a)
21. (b) 22. (b) 23. (a) 24. (a) 25. (d) 26. (b) 27. (a) 28. (a) 29. (c) 30. (a)
31. (a) 32. (b) 33. (d) 34. (d) 35. (c) 36. (a) 37. (a) 38. (c) 39. (d) 40. (a)
41. (b) 42. (a) 43. (d) 44. (b) 45. (a) 46. (d) 47. (a) 48. (a) 49. (a) 50. (b)
51. (c) 52. (d) 53. (c) 54. (a) 55. (a) 56. (c) 57. (d) 58. (b) 59. (a) 60. (b)
61. (a) 62. (b) 63. (d) 64. (c) 65. (c) 66. (d) 67. (b) 68. (a) 69. (a) 70. (c)
71. (a) 72. (b) 73. (d) 74. (b) 75. (a) 76. (a) 77. (a) 78. (c) 79. (a) 80. (c)
81. (b) 82. (b) 83. (c) 84. (a) 85. (b) 86. (c) 87. (d) 88. (a) 89. (b) 90. (b)
91. (c) 92. (b) 93. (b) 94. (d) 95. (b) 96. (b) 97. (c) 98. (d) 99. (b) 100. (d)
101. (c) 102. (c) 103. (c) 104. (d) 105. (a) 106. (a) 107. (d) 108. (a) 109. (d) 110. (b)
111. (c) 112. (b) 113. (c) 114. (c) 115. (b) 116. (a) 117. (d) 118. (d) 119. (a) 120. (b)
121. (a) 122. (a) 123. (b) 124. (c) 125. (d) 126. (c) 127. (c) 128. (c) 129. (b) 130. (c)
131. (d) 132. (b) 133. (b) 134. (c) 135. (b) 136. (b) 137. (d) 138. (a) 139. (a) 140. (c)
141. (a) 142. (a) 143. (c) 144. (a) 145. (c) 146. (a) 147. (b) 148. (a) 149. (b) 150. (b)
151. (c) 152. (c) 153 (c) 154. (a) 155. (d) 156. (c) 157. (b) 158. (d) 159. (c) 160. (b)
Practical Problems
161. (d) 162. (a) 163. (c) 164. (b) 165. (d) 166. (d) 167. (a) 168. (d) 169. (d) 170. (a)
171. (c) 172. (c) 173. (d) 174. (a) 175. (a) 176. (c) 177. (d) 178. (a) 179. (b) 180. (d)
181. (d) 182. (d) 183. (a) 184. (b) 185. (c) 186. (d) 187. (d) 188. (a) 189. (d) 190. (b)
191. (a) 192. (d)
Miscellaneous
193. (c) 194. (a) 195. (a) 196. (d) 197. (a) 198. (d) 199. (a) 200. (d)
Indian Logic: Means of Knowledge
#ntaugc-netimportantquestions

201. (a) 202. (a) 203. (c) 204. (c) 205. (d) 206. (a) 207. (c) 208. (c) 209. (c) 210. (b)
211. (d) 212. (d) 213. (a) 214. (d) 215. (d) 216. (c) 217. (a) 218. (c) 219. (c) 220. (b)
221. (c) 222. (c) 223. (a) 224. (b) 225. (a) 226. (b) 227. (d) 228. (b) 229. (d) 230. (b)
231. (a) 232. (c) 233. (b) 234. (b) 235. (b) 236. (b) 237. (b) 238. (d) 239. (a) 240. (a)
241. (d) 242. (c) 243. (c) 244. (d) 245. (c) 246. (c) 247. (a) 248. (d) 249. (d) 250. (b)
251. (d) 252. (d) 253. (a)

M06_MADAN 07_65901_C06.indd 56 27/12/22 8:18 PM


Logical Reasoning 6.57

HINTS AND Solutions


Practical Problems 164. (b): Three terms—Teacher, girls, and dull. Next, it is
in proper format. No alignment is required.
161. (d): Step I: First, we check that there are three terms, Now, A + E = E. That is, ‘No Teacher is dull’. Conclusion
tables, and teak. Here, table is the middle term. II follows. Boys is not mentioned as a term. So noth-
Then, we see whether it is in the form: A to B and ing can be said about conclusion I.
B to C. It is not; so alignment is required. For align- Venn diagram solution:
ment, we just need to change the order in this case.
After alignment: Girls
All teak are tables.
Some tables are golden. Teachers Dull
Step II: This is ‘A + I’. (Please refer to ‘syllogism rule
table)’. No conclusion can be drawn.
So, option (d) is the answer.
Venn diagram solution:
165. (d): There are three terms—poets, inspiring, and
Table Table artists.
Golden Or Teak
Golden All poets (A) are inspiring (B).
Teak All artists (C) are inspiring (B).
A to B.
Conclusion I: Some teak may or may not be golden. C to B.
There is ambiguity. Conclusion I does not follow. So alignment is required as it should be in the form
Conclusion II: Similarly, some golden may or may A to B and B to C.
not be teak. After converting second statement.
As there is ambiguity in both cases, none follows. All poets are inspiring (A type).
Thus, option (d) is the answer. Some inspiring persons are artists (I type). It contra-
162. (a): First check the number of terms. It is three. Now dicts second conclusion also.
look at alignment. A + I = No conclusion.
Now, the common term ‘man’ is not in the form A to So, option (d) is the answer.
B and B to C. So alignment is required. Venn diagram solution:
Align this pair as ‘Karan is a man’ and ‘No man is a
Tiger’. Inspiring

#ntaugc-netimportantquestions
Here, it is important to mention that statement
‘Karan is a Tiger’ is considered as A type. Artists
Poets
Now, A + E = E. So conclusion I, ‘Karan is not a Tiger’
follows.
Alternative solution:
Venn diagram solution: 166. (d): E + E = No conclusion. So, option (d) is the
answer.
Man 167. (a): A + A = A, only statement I follows, so option (a)
Tiger is the answer.
Karan
168. (d): Town is not mentioned in the statements. So
conclusion I cannot follow. The premise and con-
163. (c): There are three terms. No alignment is required
clusion cannot be the same. So, option (d) is the
as it is in the standard format.
answer.
A + A = A. The conclusion is ‘all boys are fathers’.
Conclusion II follows. 169. (d): If we look at conclusions directly, the term ‘male
By applying immediate reference to (converting) teacher’ in the first conclusion and ‘women teachers’
statement I, we can say that some men are boys. and ‘present’ in the second conclusion are not men-
Conclusion I also follows. Both I and II follows. tioned in the statements.
Venn diagram solution: So, option (d) is the answer. Here, it is important to
mention the syllogism propositions—the opposite of
Father ‘absent’ is not ‘present’.
Men 170. (a): The aligned pair is ‘Hammer is a bench’ and
‘Some benches are chairs’. There is no definite con-
Boys clusion for A + I-type pair. But conversion of state-
ment I implies conclusion I.
171. (c): The condition of three terms is satisfied. No
alignment is required.

M06_MADAN 07_65901_C06.indd 57 27/12/22 8:18 PM


6.58 Chapter 6

A + A = A, conclusion is ‘All books are papers’. The 177. (d): There are three terms. After alignment,
immediate inference of the conclusion is ‘Some Some mammals are cats.
papers are books’. So conclusion I follows. No cats are tigers.
The immediate inference of second statement is 178. (a): I + A = I. The conclusion is some boxes are
‘some papers are stones’; that is also conclusion II. spheres which is I. This can be converted to some
So, option (c) is the answer. spheres are boxes, which is II. Conclusion III follows
Venn diagram solution: after converting all rounds are spheres. So, option
(a) is the answer.
Papers Venn diagram solution:
Stones
Boxes
Book Round Sphere

179. (b): They are in desired format: A to B and B to C.


All books are clocks (A-type)
172. (c): This is a comprehensive question, covering all Some clocks are chips (I-type)
major aspects of syllogism. Please look at the solu- A + I = No conclusion.
tion carefully. Now, we can check first statement after immediate
We need to bring the statements to their logical form inference—Some clocks are books.
before checking them for alignment. Second statement after immediate inference—Some
All those eligible for the post are graduates. chips are clocks.
Some rickshaw-pullers are graduates. So only conclusion I follows. Thus, option (b) is the
There are three terms—eligible, graduates, and answer.
rickshaw-pullers. ‘Graduates’ is the common term. 180. (d): Since both the premises are of ‘particular’ type,
The statements are not in the form: A to B and A to B. no definite conclusion follows.
According to IEA, second statement should be given 181. (d): It has three terms.
priority for conversion: After alignment—‘no goats are bears’ and ‘some
‘Some graduates are rickshaw-pullers’. bears are rabbits’.
Now the statements are to be brought to A to B and E + I = O. So the conclusion is Some rabbits are not
B to C form. goats. Hence, I follows. III and IV also make a com-
‘All those eligible for the post are graduates’ (A-type) plementary pair.
A S S E S S YO U R L E A R N I N G

‘Some graduates are rickshaw-pullers’ (I-type) 182. (d): E + A = O*. Hence, the conclusion is ‘Some books
A + I = No conclusion. are not systems’. Again, I and II are complementary
173. (d): There are three terms—grapes, oranges, and pair.
apples. 183. (a): A + A = A, ‘All branches are trees’. Conclusion
A + O = No conclusion. I follows. The immediate inference of ‘All branches
We do not get any conclusion through immediate are trees’ is ‘Some trees are branches’, that is conclu-
inference (conversion) also. So, option (d) is the sion IV. So, option (a)’ is valid.
answer. 184. (b): Students can try to solve this question with help
174. (a): A + A = A, so conclusion I follows. of venn diagram.
In conclusion II, the term ‘hardly working’ is not 185. (c): Students can try to solve this question with help
mentioned in the statements. So it does not need of venn diagram.
any consideration.
186. (d): The opposites of ‘passed’ and ‘girls’ are not
175. (a): Since, both the premises are affirmative, the ‘failed’ and ‘boys’. Failed and boys are not mentioned
conclusion must be affirmative, conclusion II cannot in the statements. So, no conclusion is valid.
follow.
187. (d): The logical form of ‘Most of Indian states
176. (c): Only three terms (magic, women, and crazy). existed before independence’ is ‘Some Indian states
No alignment is required as they are in the standard existed before independence’. It is same as conclu-
format. sion I. Statement and conclusion cannot be the
Now A + A = A.‑ same. Conclusion II is also not implied. So, (d) is
Conclusion: All magic is crazy. So statement I is the answer.
­correct. Its immediate inference is ‘Some crazy are
magic’. It means the third statement is true. 188. (a): After converting statement I, we can say that ‘a’
The converse of second statement ‘All women are is true. ‘On time’ does not mean ‘not late’ in case of
crazy’ is ‘Some crazy are women’. So fourth conclu- propositions. Thus, options (c) and (d) are also not
sion is also true. Thus, option (c) is the answer. converse of (a).

M06_MADAN 07_65901_C06.indd 58 27/12/22 8:18 PM


Logical Reasoning 6.59

189. (d): The contextual meaning and logical meaning 195. (a): Please refer to ‘square of opposition’. If a state-
are different for both the terms. So, option (d) is the ment is false, its contradictory is taken to be true.
answer. The contradictory of ‘No politician is dishonest’
190. (b): ‘Most teachers are hardworking’ is ‘Some teachers (E-type) is ‘Some politicians are dishonest’ (I-type).
are hardworking’ in its logical form. Both statement So it is true.
and conclusion cannot be the same. So conclusion I 196. (d)
is not implied. However, conclusion II ‘Some teachers 197. (a): As there is a single proposition, we can apply
are not hardworking’ is complimentary of I, so only II immediate inference to draw conclusion. This is an
is implied. Thus, option (b) is the answer. E-type proposition with ‘teacher’ as subject and ‘(per-
191. (a): The immediate inference for ‘No man is infal- sons) on time for the class’ as the predicate, which
lible’ is ‘No infallible (persons) are men’. So con- can conclude only E-type. After conversion, ‘per-
clusion II is right. In logical form, the opposite of sons who are on time’ becomes subject, and teacher
infallible may not be fallible. So inference may not becomes predicate. So, option (a) is the answer.
apply. So, option (a) is the answer. 198. (d): According to squares of opposition: The logical
192. (d): Since the middle term ‘trains’ is not distributed form of ‘None of the flowers is fragrant’ is ‘No flower
even once in the premises, no definite conclusion is fragrant’ (i.e., E-type). It can be converted only to
follows. However, III is the converse of the second I-type, that is, statement IV (the logical form of ‘Most
premise, while IV is the converse of the first premise. flowers are fragrant’ is ‘Some flowers are fragrant’).
So, both of them hold. 199. (a)
Miscellaneous 200. (d): There are three terms. Both statements are
I-type, even after conversion for alignment purpose,
193. (c): The logical form of statement I is ‘some poets are
they will be of I type as shown below.
not egoistic’.
‘Some intelligent people are happy’, we get
The opposite of egoistic is not ‘humble’ or ‘not
Some who are happy are intelligent (I-type)—first
egoistic’.
statement.
So I and III are mutually inconsistent.
Some intelligent people are rich (I-type)—second
194. (a) statement.
I + I = No inference. So, option (d) is the answer.

A S S E S S YO U R L E A R N I N G

M06_MADAN 07_65901_C06.indd 59 27/12/22 8:18 PM


This page is intentionally left blank

M06_MADAN 07_65901_C06.indd 60 27/12/22 8:18 PM


CHapteR

7 Data Interpretation

01 Sources, Acquisition and Classification of Data

02 Quantitative and Qualitative Data

leaRnInG
03 Graphical Representation
oBJeCtIveS (Bar chart, Histograms,
Pie chart, Table chart and
Line chart)

04 Mapping of Data

05 Data Interpretation

06 Data and Governance

M07_MADAN 04_65901_C07.indd 1 21/12/22 11:26 AM


7.2 Chapter 7

Data Interpretation General Principles of Graphic


Representation
Data interpretation is one of the easiest chapters of NET
Some basic algebraic principles apply to all types of
Paper 1. It is basically about drawing conclusions and
graphic representation of data:
inferences from comprehensive data presented numeri-
cally in a tabular or graphical form by means of an illus- 1. There are two lines called coordinate axes, where the
tration, such as graphs, pie charts and so on. Therefore, vertical one is known as Y-axis and the ­horizontal one
the act of organizing and interpreting data to get mean- is called X-axis (Fig. 7.1).
ingful information is known as data interpretation. The 2. These two lines are perpendicular to each other.
important aspects of data sources, their acquisition and Where these two lines intersect each other is called
interpretation have been covered in Unit II on research point ‘O’ or origin.
aptitude. In this unit, the main focus is on solving practi- 3. On the X-axis, the distances right of the origin have
cal problems as per questions asked in the NET pattern. positive values and the distances left of the origin
Data interpretation needs some mathematical and sta- have negative values. On the Y-axis, distances above
tistical skills. It needs good knowledge of the ­concepts of the origin have positive values and those below the
percentage, ratio, proportion, average, etc. The other main origin have negative values.
input required is to practice a few problems.
Familiarity with graphical representation of data, such Y
as Venn diagrams, graphs, pie charts, histogram and poly-
gon, is helpful. Once the data is grasped well, questions
based on tables and graphs take little time.
Sometimes, data is presented in more than one table (X –ve, Y +ve) (X +ve, Y +ve)
or graph. The aim is to test not only the quantitative skills
but also relative, comparative and analytical abilities.
While the terms ‘data’ and ‘statistics’ are often used x x
interchangeably, in scholarly research there is an impor-
tant distinction between the two.
Data refers to individual pieces of factual information (X –ve, Y –ve) (X +ve, Y –ve)
recorded and used for the purpose of analysis. It is the raw
information from which statistics are created. Statistics
are the results of data analysis through its interpretation
and presentation. –Y
Data is usually collected for research purposes, and Figure 7.1 X and Y Coordinates
it is closely related with sampling as well. Data sources,
acquisition and classification have been taken care of in Graphical representation of data may have many for-
the second unit of research. Acquisition basically means mats. According to the NTA-NET syllabus, graphic repre-
collecting data and how it is collected with the help of sentation of data has been divided into bar chart, histo-
interviews, questionnaire and so on. Primary and sec- grams, pie chart, table chart and line chart. Here, we are
ondary sources, quantitative and qualitative data have discussing their brief description and some numerical
also been explained there. problems.

Bar Chart
Graphical Representation
It is also known as a column graph, a bar graph or a
Basically, it is a graphic representation of data. It is one of bar diagram. It is basically a pictorial representation of
the important ways of analysing numerical data. A graph data and is shown as rectangles spaced out with equal
is a kind of chart through which statistical data is repre- spaces between them and having equal width. Here,
sented in the form of lines or curves drawn across coor- the height (or length) of each bar corresponds to the
dinated points that are plotted on its surface. Graphs are frequency of a particular observation. We can draw bar
also easy to understand and are eye catching. graphs both vertically and horizontally depending on
They help us to study the cause and effect relationship whether we take the frequency along the vertical or
between two variables, to measure the extent of change in horizontal axes, respectively. We can take the following
one variable when another variable changes by a certain example.
amount. Bar charts are one of the easiest, graphically attractive
They enable us to study both the time series and the and, hence, most commonly used methods of presenting
frequency distribution. They give clear account and pre- all types of data. Given quantities can be compared by
cise picture of a problem. the height or length of a bar graph. A bar graph can have

M07_MADAN 04_65901_C07.indd 2 21/12/22 11:26 AM


Data Interpretation 7.3

either vertical or horizontal bars. The width of the bars is 07 00.09


largely inessential and is used only for clarity of presen-
00.08
tation. We can compare different quantities or the same 06
quantity at different times. 00.07
In bar graphs, the data is discrete. Presentation 05 0–5
00.06
of data in this form makes comparative evaluation 5–10
easier. 04 00.05

03 00.04 10–15
Sports Number of Participants 00.03
02 15–20
Cricket 15 00.02
01
Volleyball 25 00.01
00 00
Football 10
00 05 10 15 20
Total 50 Figure 7.3 Histogram

By taking sports along the x-axis and 5 as the common


representation on the y-axis, a bar chart can be repre-
sented as shown in Figure 7.2. f­ requency of the data values. The median and distribu-
tion of the data can be determined by a histogram. In
Participants addition, it can show any outliers or gaps in the data.
X
30
Distributions of a Histogram
25 1. A Normal Distribution: Here, points on one side of
the average are as likely to occur as on the other side
20 of the average (Fig. 7.4).
15

10

0 Y
Cricket Volleyball Football

Figure 7.2 Bar Chart of Above Table

Histograms Figure 7.4 Normal Distribution


As we could see, the bar chart graph depicts discrete
data, while histograms depict continuous data. The con- 2. A Bimodal Distribution: In bimodal distribution, the
tinuous data can take the shape of class intervals. Thus, data should be separated and analysed as separate
a histogram is a graphical representation with class normal distributions (Fig. 7.5).
intervals or attributes as the base and frequency as the
height (Fig. 7.3).
Histograms have bars without any spaces between
them and the rectangles need not be of equal width. It can
be understood with the following example.
Class intervals such as 0–5, 5–10, 10–15 and 15–20 are
continuous data. We may ignore the values on the right
vertical axis. Thus, for the class interval 0–5, the cor- Figure 7.5 Bimodal Distribution
responding frequency is 3. Again, for the class interval
5–10, the frequency is 7 and so on. 3. A Right-skewed Distribution: A right-skewed distri-
bution is also called a positively skewed distribution.
I mportance of Histograms In a right-skewed distribution, a larger number of
Histogram provides a visual representation of data dis- data values occur on the left side and a fewer number
tribution. It displays a large amount of data and the of data values on the right side (Fig. 7.6).

M07_MADAN 04_65901_C07.indd 3 21/12/22 11:26 AM


7.4 Chapter 7

In case the class intervals are not even in size, the type
of diagram given in Figure 7.9 emerges.

3
2.8
2.1

Frequency
2

1
0.5

0 15 25 40 60 100
Age Groups

Figure 7.9 Class Intervals


Figure 7.6 Right-Skewed Distribution

4. A left-skewed distribution: A left-skewed distribution Comparison of Bar Graphs and Histogram


is also called a negatively skewed distribution. A larger Bar graph
number of data values occur on the right side and a `500
fewer number of data values on the left side (Fig. 7.7).
`400
Gaps

`300

`200

`100

`0
USA India UK NZ Japan
Categories
Source: Tes.com
Figure 7.10 Bar Graph
Figure 7.7 Left-Skewed Distribution
Histogram
5. A random distribution: A random distribution lacks an
apparent pattern and has several peaks. Here, the dif- 50
ferent data properties are combined. Therefore, the data No gaps
should be separated and analysed separately (Fig. 7.8). 40

30

20

10

0
100 150 200 250 300 350
Number
ranges

Figure 7.11 Histogram


Figure 7.8 Random Distribution

M07_MADAN 04_65901_C07.indd 4 21/12/22 11:26 AM


Data Interpretation 7.5

Solved Examples on Simple Bar Graphs (Questions 1–20)


Directions: Study the bar graph given below and answer 4. The percentage increase in sales from 2011 to 2012
the questions 1–5. It consists of data on the sales of cel- was
lular phones during 2007–12. (a) 115% (b) 128%
48,000
(c) 122% (d) 118%
50,000
Sale of Cellular phones

40,000 40,000 Ans: (c) Increase in sales = 40000 – 18000 = 22000


40,000 Percentage increase in sales
30,000 25,000
30,000 = 22000/18000 × 100
18,000
20,000 = 122%
10,000
Hence, (c) is the correct answer.
0
2007 2008 2009 2010 2011 2012 5. What is the average sales figure of all the years?
Years (a) 32,000 units
(b) 33,500 units
1. The difference in the sale of cellular phones for the (c) 34,500 units
years 2007 and 2009 is
(d) 35,000 units
(a) 500 units 2,01,000
(b) 1000 units Ans: (b) Average = = 33,500 units
6
(c) 5000 units
(d) 18,000 units Directions: Study the bar graph given below and answer
Ans: (d) The difference = 48,000 – 30,000 questions 6–10. It consists of data on the number of
= 18,000 units students passed (in thousands) from two universities
between the years 2003 and 2008.
2. The two years between which the rate of change of
­cellular phones has been minimum are 70
(a) 2007 and 2008
(b) 2009 and 2010 60
(c) Both (a) and (b)
(d) 2011 and 2012 50
Ans: (c) Percentage changes over years: 40
8000
2007 and 08 = × 100 = 16.66% 30
48,000
10,000 20
2008 and 09 = × 100 = 25%
40,000 10
5000
2009 and 10 = × 100 = 16.66% 0
30,000 2003 2004 2005 2006 2007 2008
7000 University A University B
2010 and 11 = × 100 = 28%
25,000
6. What is the sum of students passing from University B
22,000 in 2003, 2005 and 2006 together?
2011 and 12 = × 100 = 122.22%
18,000 (a) 75,000 (b) 80,000
3. The sum of sales of cellular phones in the years 2009 (c) 88,000 (d) 90,000
and 2011 is equal to that in Ans: (b) Required number of students = (30 + 10 +
(a) 2007 40) thousands = 80,000
(b) 2008 7. What is the ratio of the number of students passed
(c) 2010 from University A in year 2007 and ­the number of stu-
(d) 2012 dents passed from University B in year 2004?
(a) 7 : 10 (b) 10 : 7
Ans: (a) Combined sales of 2009 and 2011 = 30,000
+ 18,000 = 48,000 (c) 4 : 5 (d) 5 : 4
It tallies with the sales figure of 2007. Ans: (b) Required ratio = 50 : 35 = 10 : 7

M07_MADAN 04_65901_C07.indd 5 21/12/22 11:26 AM


7.6 Chapter 7

8. The number of students from University B in the year ⎡ (35 − 25) ⎤


2008 is approximately what percentage of the total For 2017 = ⎢ × 100⎥ % = 40%
number of students passed from University A over the ⎣ 25 ⎦
years?
(a) 20 (b) 22 ⎡ (35 − 35) ⎤
For 2018 = ⎢ × 100⎥ % = 0%
(c) 28 (d) 30 ⎣ 35 ⎦
Ans: (b) Required percentage = 50/225 × 100 ⎡ (40 − 35) ⎤
= 22.22% ~ 22% approximately For 2019 = ⎢ × 100⎥ % = 14.29%
⎣ 35 ⎦
9. What is the ratio between the number of students
passed in the years 2007, 2008 and 2005 from Uni- ⎡ (50 − 40) ⎤
For 2020 = ⎢ × 100⎥ % = 25%
versity A?
⎣ 40 ⎦
(a) 3 : 5 : 5 (b) 5 : 3 : 3
(c) 5 : 3 : 2 (d) 5 : 2 : 2 Hence, the maximum percentage rise/fall in the
Ans: (b) Required ratio = 50 : 30 : 30 = 5 : 3 : 3 production of Company Y is for 2017.
10. What is the difference between the total number of 12. What is the ratio of the average production of Com-
students passed from both the universities together in pany X over the period 2018–20 to the average pro-
2007 and the total number of students passed from duction of Company Y over the same period?
both the universities together in 2005? (a) 1:1 (b) 15:17
(a) 70,000 (b) 80,000 (c) 23:25 (d) 27:29
(c) 85,000 (d) 90,000 Ans: (c) Average production of Company X over the
Ans: (a) Total number of students passed in 2007 period 2018–20
= 110 (in thousands)
Total number of students passed in 2005 = 40 ⎡1 ⎤ ⎛ 115 ⎞
= ⎢ × (25 + 50 + 40)⎥ = ⎜ ⎟ lakh tons
(in thousands) ⎣ 3 ⎦ ⎝ 3 ⎠
Required difference = 110 – 40 = 70 (in Average production of Company Y over the
thousands) period 2018–20
Directions: The bar graph given below shows the data of
the production of paper (in lakh tons) by three different ⎡1 ⎤ ⎛ 125 ⎞
= ⎢ × (35 + 40 + 50)⎥ = ⎜ ⎟ lakh tons
companies X, Y and Z over the years. ⎣3 ⎦ ⎝ 3 ⎠

Production of paper (in lakh tons) by three Companies ⎛ 115 ⎞


X, Y and Z over the years ⎜⎝ 3 ⎟⎠ 115 23
60 Thus, required ratio = = =
⎛ 125 ⎞ 125 25
⎜⎝ 3 ⎟⎠
50
Quantity (lakh tons)

40 13. The average production for 5 years was maximum for


which company?
30 (a) X (b) Y
(c) Z (d) Both X and Z
20
Ans: (d) Average production (in lakh tons) in 5 years
10 for the three companies is:
For Company X
0
2016 2017 2018 2019 2020 ⎡1 ⎤ 190
= ⎢ × (30 + 45 + 25 + 50 + 40)⎥ = = 38
Years ⎣ 5 ⎦ 5
X Y Z
For Company Y
11. For which of the following years, the percentage rise/ ⎡1 ⎤ 185
fall in production from the previous year is the maxi- = ⎢ × (25 + 35 + 35 + 40 + 50)⎥ = = 37
⎣5 ⎦ 5
mum for Company Y?
(a) 2017 (b) 2018 For Company Z
(c) 2019 (d) 2020
⎡1 ⎤ 190
= ⎢ × (35 + 40 + 45 + 35 + 35)⎥ = = 38
Ans: (a) Percentage change (rise/fall) in the produc- ⎣ 5 ⎦ 5
tion of Company Y in comparison to the previ-
ous year for different years is: Thus, average production of five 5 is maximum
for both the Companies X and Z.

M07_MADAN 04_65901_C07.indd 6 21/12/22 11:26 AM


Data Interpretation 7.7

14. In which year was the percentage of production 16. How many companies have shown production below
of ­Company Z to the production of Company Y the their average production in 2012–2013, but have
maximum? showed above the average production in 2013–14?
(a) 2016 (b) 2017 (a) One (b) Two
(c) 2018 (d) 2019 (c) Three (d) Four
Ans: (a) The percentages of production of Company Ans: (c) Average sales of company:
Z to the production of Company Z for various (6 + 14 + 21)
years are: Honda = = 13.66
3
⎛ 35 ⎞ (12 + 18 + 18)
For 2016 = ⎜ × 100⎟ % = 140% GM = = 16
⎝ 25 ⎠ 3
⎛ 40 ⎞ (5 + 9 + 15)
Maruti = = 9.66
For 2017 = ⎜ × 100⎟ % = 114.29% 3
⎝ 35 ⎠
(16 + 9 + 12)
Hindustan Motors = = 12.33
⎛ 45 ⎞ 3
For 2018 = ⎜ × 100⎟ % = 128.57%
⎝ 35 ⎠ (8 + 14 + 7)
Hyundai = = 9.66
3
⎛ 35 ⎞
For 2019 = ⎜ × 100⎟ % = 87.5% 17. The ratio of Hindustan Motors production in 2013–
⎝ 40 ⎠ 2014 to Honda’s production in 2012–2013 is
(a) 0.66 (b) 1.5
⎛ 35 ⎞ (c) 2 (d) None of these
For 2020 = ⎜ × 100⎟ % = 70%
⎝ 50 ⎠ Ans: (b) The required ratio is (9/6) = 1.5.
Clearly, this percentage is highest for 2016. 18. For how many companies has there been no decrease
in production in any year from the previous year?
15. What is the percentage increase in the production of
(a) One (b) Two
Company Y from 2016 to 2019?
(c) Three (d) Four
(a) 30% (b) 45% Ans: (c) By visual inspection, we can say that Honda,
(c) 50% (d) 60% GM and Maruti have not shown a decrease.
Ans: (d) Percentage increase in the production of 19. Which of the following companies has shown consist-
Company Y from 2016 to 2019 ent growth during the period ?
⎡ (40 − 25) ⎤ (a) Maruti (b) Honda
= ⎢ × 100⎥ % (c) Hindustan Motors (d) Both (a) and (c)
⎣ 25 ⎦ Ans: (d) By simple observation, we can say that (d) is
⎡ 15 ⎤ the right answer.
= ⎢ × 100⎥ %
⎣ 25 ⎦ 20. Which of the following companies have shown high-
est and lowest growth rates?
= 60% (a) Honda and Hyundai (b) Honda and Maruti
Directions: The following chart shows the production of (c) GM and Hyundai (d) GM and Honda
cars in thousands. Ans: (b) By simple observation, we can say that (b) is
the right answer.
Production of cars for 2012–2015 period from the
selected manufacturers Circle G raphs (or P ie Charts)
25
Circle graphs are used to show how various sectors are in the
whole. They are popularly called pie charts. Circle graphs
20
usually give the percentage that each sector receives.
In such representation, the total quantity in question
15 is distributed over a total angle of 360°. While using pie
chart to find the ratios of various sectors, there is no need
10 to find the amounts each sector received and then the
ratio of the amounts. Find the ratio of the percentages,
5 which is much easier.
If some percentage figure is to be converted into degrees
of angles extended at the centre, then multiply it by 3.6. For
0
Honda GM Maruti Hindustan Hyundai example, if some category is 20%, in degrees equivalent, it
Motors is 72°, that is, (20 × 3.6). Conversely, if degree data is to be
2012–2013 2013–2014 2014–2015 converted into per cent figure, then divide it by 3.6.

M07_MADAN 04_65901_C07.indd 7 21/12/22 11:26 AM


7.8 Chapter 7

Solved Examples on Pie Charts (Questions 21–30)


Directions: Study the pie chart given below and answer (b) Total of miscellaneous and cultural per-
Ans: 
Questions 21–25. It gives the breakup of expenses of an centages = 15 + 5 = 20%. Hence, angle extended
educational institute. by them at the centre = 20/100 × 360 = 72°.

Publicity Directions: Study the pie chart given below and answer
4% Questions 26–30. It gives the breakup of expenses
incurred in publishing a book.
Expenditure Incurred in Publishing a Book
Staff
Maintenance
salaries
20%
38%

Transportation Binding Royalty


10% 20% 15%

Sports Promotion cost


Transportation 10%
8% cost 10%
Cultural Miscellaneous
events 15% Printing cost
5% 20%
Paper cost
25%
21. If total expenditure during a year is `65 lakhs, then
what are the expenses on cultural events (in lakhs)?
(a) 1.67 (b) 2.65
(c) 3.25 (d) 4.25
Ans: (c) 5% of `65 lakhs = `3.25 lakhs 26. If the publisher has to pay `61,200 as the printing cost
22. What is the approximate angle extended by staff sala- for production of a certain quantity of books, then
ries at the centre (in degrees)? what will be the amount of royalty to be paid for these
(a) 132 (b) 137 books?
(c) 142 (d) 145 (a) `22,950 (b) `45,000
Ans: (b) Angle extended by staff salaries at the cen- (c) `45,900 (d) `48,900
tre = 38 × 3.6 = 137° Ans: (c) The ratio between printing cost and royalty
23. The total of which of the following heads is equal to = 20 : 15 or 4 : 3.
­salary expenses? Let royalty be denoted by R.
(a) Miscellaneous, transportation and cultural events
Then 4 : 3 = 61,200 : R
(b) Maintenance, transportation and sports
R × 4 = 61,200 × 3
(c) Maintenance, publicity and miscellaneous
R = (61,200 × 3)/4 = `45,900
(d) None of the above
27. If the cost price of the book is `150, then what is the
Ans: (b) Maintenance + transportation + sports =
combined printing and binding cost for a single copy
38%
of the book?
24. If the teachers’ salaries are increased by 20%, then (a) `60 (b) `75
what will be the new angle extended at the centre (in (c) `80 (d) `72
degrees)?
(a) 45.6 (b) 43.6 Ans: (a) Total printing and binding cost = 20 + 20
= 40%
(c) 56 (d) Cannot be determined
Printing and binding cost = 40% of cost price
Ans: (d) No separate data is given for teachers’ sal- = 40% of 150 = `60
ary. Hence, it cannot be determined. So, new
angle extended may not be determined. 28. What is the central angle of the sector corresponding
25. What is the angle extended at the centre by a combi- to the expenditure incurred on royalty?
nation of miscellaneous and cultural events expenses (a) 48° (b) 54°
at the centre (in degrees)? (c) 60° (d) 72°
(a) 65 (b) 72 (b) Central angle corresponding to royalty
Ans: 
(c) 78 (d) 84 = (15% of 360)° = 15/100 × 360 = 54°

M07_MADAN 04_65901_C07.indd 8 21/12/22 11:26 AM


Data Interpretation 7.9

29. What is the difference between promotion and Purpose


­printing cost if we assume that total cost is `200? The purpose of tabulation is to present data in such a way
(a) `10 (b) `20 that it becomes more meaningful and can be easily under-
(c) `30 (d) `24 stood by a common person. However, in case of voluminous
data, it may require closer reading than graphs of charts
Ans: (b) Difference in percentage terms = 20 – 10 and hence, it is difficult and time consuming to interpret.
= 10%
Then, 10% of `200 = `20 Essential Parts of a T able
30. If 1000 copies are published and the transportation A statistical table is divided into the following eight parts:
cost on them amounts to `15,000, then what should 1. Title of the table: A title is a heading at the top of the
be the selling price per book so that the publisher can table describing its contents. It mainly reflects upon
earn a profit of 20%? the nature of the data, where the data is, what time
(a) `150 (b) `160 period the data covers and how the data is classified.
(c) `180 (d) `200 2. Caption: The headings for various columns and
rows are called column captions and row captions,
Ans: (c) To calculate the selling price of a single respectively.
book, we need to total the cost. Let us calculate 3. Box head: The portion of the table containing t­ he col-
that first. umn caption is called the box head.
Transportation cost per book = 15,000/1000 4. Stub: The portion of the table containing row c­ aption
= `15 is called stub.
As transportation cost is 10% of total cost, thus 5. Body of the table: The body of the table contains the
`15 = 10% of total cost statistical data that has to be presented in different
rows and columns.
Total cost of printing a copy = 15 × 100/10 6. Prefatory notes or head notes: Prefatory notes
= `150 appear between the title and the body of the table and
Now selling price = total cost + profit are enclosed in brackets. They may throw some light
= 150 + 20% of 150 = 150 + 30 = `180 on the units of measurements.
7. Footnote: A footnote is always given at the bottom of
Tables the table but above the source note. It is a statement
about something which is not clear from the headings,
A table is a systematic arrangement of data into vertical title, stubs, captions and so on.
columns and horizontal rows. The process of arranging 8. Source note: A source note is placed immediately
data into rows and columns is called tabulation. below the table but after the footnote. It refers to the
source from where information has been taken.

Solved Questions on Tables and Simple Line Graphs (Questions 31–55)


Directions: Study the table given below and answer Questions 31–35. (The table consists of marks obtained by candi-
dates in different subjects. The numbers in the brackets give the maximum marks in each subject.)
Marks Obtained in Different Subjects by Various Candidates
Subjects (Maximum Marks)
Computer
Students Maths Chemistry Physics Geography History
Science
(150) (130) (120) (100) (100) (100)
Ayush 90 50 90 60 70 80
Aman 100 80 80 40 80 70
Sajal 90 60 70 70 90 70
Rohit 80 65 80 80 60 60
Muskan 80 65 85 95 50 90
Tanvi 70 75 65 85 40 60
Tarun 65 35 50 77 80 80

M07_MADAN 04_65901_C07.indd 9 21/12/22 11:26 AM


7.10 Chapter 7

31. What is the average of marks obtained by Aman for all Ans: (b) Average marks in Physics =
subjects?
(a) 72 ⎡ 90 + 80 + 70 + 80 + 85 + 65 + 50 ⎤ ⎡ 90 + 80
⎡ 520 ⎤ + 70 + 80 + 85 + 65 + 50 ⎤ = ⎡ 520 ⎤
(b) 75 ⎢ ⎥ =⎢ ⎢ ⎥ ⎥ ⎢ 7 ⎥
⎣ 7 ⎦ ⎣⎣ 7 ⎦ 7 ⎦ ⎣ ⎦
(c) 80
= 74.28 = 74 . 28
(d) 85
Ans: (b): Average marks 34. Who among the following has obtained the highest
aggregate marks?
⎡ 100 + 80 + 80 + 40 + 80 + 70 ⎤ ⎡ 450 ⎤ (a) Ayush (b) Aman
=⎢ ⎥ =⎢ ⎥ = 75 (c) Sajal (d) Muskan
⎣ 6 ⎦ ⎣ 6 ⎦
Ans: (d) Aggregate marks: Ayush, 440; Aman, 450;
32. What is the approximate percentage of marks Sajal, 450; and Muskan, 465
obtained by Ayush? 35. Who among the following has secured the highest
(a) 62.8 ­percentage marks in Physics, Chemistry and Maths?
(b) 72.5 (a) Ayush
(c) 75 (b) Aman
(d) 74 (c) Rohit
(d) Muskan
Ans: (a): Percentage marks
Ans: (b) As the denominator is same for the calcu-
⎡ 90 + 50 + 90 + 60 + 70 + 80 ⎤ lation of percentage of marks in all the cases,
=⎢ × 100⎥ there is no need to calculate the percentage fig-
⎣ 150 + 130 + 120 + 100 + 100 + 100 ⎦
ures. Simply, the aggregate marks of three sub-
⎡ 440 ⎤ jects will provide the answer:
=⎢ × 100⎥ = 62.8%
⎣ 700 ⎦  
Ayush, 230; Aman, 260; Rohit, 225; and
Muskan, 230.
33. What is the average of marks obtained by all candi- Directions: Study the table given below and answer
dates in Physics? Questions 36–40. (The table consists of the number of can-
(a) 62.85 (b) 74.28 didates who appeared and qualified in a competitive exam-
(c) 78.52 (d) None of the above ination from different states from 2007 to 2011.)

Number of Candidates Who Appeared and Qualified in a Competitive Examination


Year
State 2007 2008 2009 2010 2011
App. Qual. App. Qual. App. Qual. App. Qual. App. Qual.
M 5200 780 7800 1170 8000 1200 8800 1320 9000 1350
N 7500 750 8500 850 8600 860 9200 920 8800 880
P 6400 960 8800 1100 9000 900 9200 920 10,000 1100
Q 8100 850 9000 1350 9200 1480 10,000 1200 10,000 1280
R 7800 1560 7600 760 9800 700 (78010,000
+1170 +12001250
+132011,200
+1350) 1650
⎡ ⎤
=⎢ ×100 ⎥
⎣ (5200 + 7800 + 8000 + 8800 + 9000) ⎦
36. What is the percentage of candidates ­who qualified ⎡ 4900 ⎤
­during 2007 for all states combined? =⎢ ×100 ⎥ = 14%
(a) 12% ⎣ 35,000 ⎦
(b) 14%
37. What is the average percentage of candidates who
(c) 15%
qualified from State M for all the years?
(d) None of the above
(a) 12%
Ans: (b) Percentage of candidates qualified = (b) 14%
780 + 750 + 960 + 850 + 1560 (c) 15%
5820 = × 100 (d) 16%
5200 + 7500 + 6400 + 8100 + 7800

M07_MADAN 04_65901_C07.indd 10 21/12/22 11:26 AM


Data Interpretation 7.11

Ans: (c) Percentage of candidates ­qualified (a) 702 (b) 852


(c) 862 (d) 902
= ⎡Number of candidates
(780 +1170 +1200 +1320qualified
+1350)
⎡ (780
× +1170
100 ⎤ +1200 +1320 +1350)
Ans: (b) ⎤
= ⎢Number of candidates appeared =⎢ ×100 ⎥ ×100 ⎥
⎣ (5200 + 7800 + 8000 + 8800 + 9000
⎣ (5200) + 7800
⎦ + 8000 + 8800 + 9000) ⎦
⎡ 750 + 850 + 860 + 920 + 880 ⎤
⎡780 4900
+ 1170 +⎤ 1200 + 1320 ⎡+ 4900
1350 ×100 ⎤ = 14% Average = ⎢
=== ⎢⎡ (780 +…)×100 ⎥ =
×100 ⎥

14% = ⎢ × 100 ⎥ ⎣ 5 ⎥⎦
35,000
⎣⎢⎣5200
(5200++…
7800
) ⎦+ 8000 + 8800 ⎣+35,000
9000 ⎦
⎦ ⎡ 4260 ⎤
=⎢ = 852
=⎢
⎡ 5820 ⎤
×100 ⎥ = 15% ⎣ 5 ⎥⎦
⎣ 38,800 ⎦
40. What is the percentage increase in candidates who
If we look at all the figures for individual years, appeared between 2007 and 2011?
we can see that the result has been consistent at (a) 35% (b) 40%
15% for all these years. (c) 45% (d) 50%
38. What is the average number of candidates who Ans: (b) Total number of candidates in 2007
appeared from State Q during the given years? = 35,000 (as calculated in Question 36)
(a) 8660 Total number of candidates in 2011
(b) 9260 = 9000 + 8800 + 10,000 + 10,000 + 11,200
(c) 9560 = 49,000
(d) 9660
Increase = 49,000 – 35,000 = 14,000
Ans: (b) Average number of candidates appeared
Percentage increase = 14,000/35,000 × 100
⎡ (8100 + 9000 + 9200 +10,000 +10,000) ⎤ = 40%
=⎢ ⎥
⎣ 5 ⎦
Directions for Questions 41–45: Presented below is the
⎡ 46,300 ⎤ percentage distribution of households by household size
=⎢ = 9260
⎣ 5 ⎥⎦ and the average sizes of household in:
39. What is the average number of candidates who quali- • All Indian rural and urban areas
fied from State N during all these years? • Urban areas classified as per population size

20
19
18
16
17
15
14
13
12
Percentage

11
10
9 Urban
8
7 Rural
6
5
4
3
2
1
0
1 2 3 4 5 6 7 8 9 10 11 12 13 14 15
Household size

M07_MADAN 04_65901_C07.indd 11 21/12/22 11:26 AM


7.12 Chapter 7

(c) In urban areas, the average size of the household


Distribution of Average Size of is the least for towns.
All India People Household
(d) In urban areas, there are 460 persons on an aver-
Rural – 5.08 age per 100 households.
Ans: (b)
Urban – 4.60
Explanation: Only statement (b) can be inferred
Distribution of Below 15,000 4.75 from the given graphs.
urban
15,000–50,000 4.50 Directions: Study the following graph carefully and
answer Questions 46–50 given below it. It depicts profits
50,001 to above 4.70
earned by a company during various years. Profit earned
in lakhs
41. In rural areas, which of the following sizes of the (Profit = Revenue − Expenditure)
households is the highest in number?
(a) 15 (b) 2 40
(c) 3 (d) 4 35
Ans: (d) 30
Explanation: From the graph, we see that the line for 25

Profit
rural peaks at about 4–5. 20
Hence, the best answer is 4. 15
42. In urban areas, among the households of different 10
sizes, what is the percentage of households of size 5 5
or less?
0
(a) 13 (b) 72 2005 2006 2007 2008 2009
(c) 36 (d) 87
46. What is the average profit earned by the company
Ans: (b)
over the years?
Explanation: The total number of households ≤5 is (a) `26 lakhs
17 + 14 + 12 + 15 + 14 = 72. (b) `28 lakhs
(c) `30 lakhs
43. In rural areas, per 100 households, there were 31 (d) `32 lakhs
households of size
(a) 3 or above Ans: (c) Total profit earned over 5 years = (25 + 35 +
(b) 3, 4 and 5 22.5 + 30 + 37.5) = 150 lakhs
Average profit = 150/5 = `30 lakhs
(c) 3 or less
(d) 4 or less 47. If the expenditure of the company in 2009 was
`28 lakhs, then what was the revenue of the company
Ans: (c) in that year?
Explanation: Total up the values for rural area for the (a) `65.5 lakhs
size 3 or less to get the answer. (b) `72.5 lakhs
(c) `75 lakhs
44. As the urban population of towns increases, the aver- (d) None of the above
age household size
Ans: (a) Revenue in 2009 = Profit + Expenditure
(a) Increases = `65.5 lakhs
(b) Decreases 48. What is the approximate percentage increase in the
(c) Remains constant profit of the company in 2008 in comparison to the
(d) Fluctuates previous year?
Ans: (d) (a) 28 (b) 30
Explanation: As the population size increases, we see (c) 36 (d) 40
that household sizes are 4.75, 4.50 and 4.70. Hence, Ans: (b) Percentage increase = (30 − 23)/23 × 100
the values are fluctuating. = 30.43 ∼ 30%
45. Which of the following statements is true? 49. What is the ratio of profit earned by the company in
(a) On the average, there are more persons per family 2005 to the profit earned in 2009?
in urban areas than in rural areas. (a) 1 : 3 (b) 2 : 3
(b) In rural areas, 35% of the households are of the (c) 3 : 5 (d) 1 : 2
size 7 and above. Ans: (b) The required ratio = 25 : 37.5 = 2 : 3

M07_MADAN 04_65901_C07.indd 12 21/12/22 11:26 AM


Data Interpretation 7.13

50. If the revenue of the company in 2007 was `45 lakhs, Ans: (d) The differences between the productions of
then what was the expenditure of the company in Companies X and Y in various years are as fol-
that year? lows (all figures in thousands):
(a) `20.5 lakhs For 2007 (139 − 119) = 20
(b) `22.5 lakhs For 2008 (120 − 99) = 21
(c) `24.5 lakhs For 2009 (141 − 100) = 41
(d) `25.5 lakhs For 2010 (128 − 78) = 50
For 2011 (120 − 107) = 13
Ans: (b) Expenditure in 2007 = 45 − 22.5 = `22.5 For 2012 (159 − 148) = 11
lakhs Hence, the maximum difference was in 2010.
Directions: Study the following graph carefully and 55. The production of Company Y in 2010 was approxi-
answer Questions 51–55. The graph consists of data mately what percentage of the production of Com-
about the number of vehicles manufactured by two com- pany X in the same year?
panies over the years. (a) 173 (b) 164
(c) 132 (d) 97
Number of Vehicles Manufactured by Two
Companies Over the Years Ans: (b) As the comparison is with Company X, its
production figure will appear as denominator.
No. of vehicles (thousands)

160 159 Required percentage = (1,28,000/78,000) × 100


139 141 148
140 120
= 164%
128 120
120
119 107
100
80
99 100 Data Mapping
78
60 Data mapping is the process of mapping data fields from a
40 source file to their related target fields. The accessibility to
2007 2008 2009 2010 2011 2012 required data can make some organization more success-
Years X Y ful. Somehow, data is easier to use when it is visualized.
Visual data helps people understand how different con-
51. What is the difference between the number of vehi- cepts originate and their relation with each other. Data
cles manufactured by Company Y in 2010 and 2011? mapping helps in all these. For example, ‘Name’, ‘Email’
(a) 50,000 (b) 42,000 and ‘Phone’ fields from an Excel source are mapped to the
(c) 33,000 (d) 21,000 relevant fields in a delimited file, which is our destination.
Ans: (d) Difference = 1,28,000 − 1,07,000 = 21,000 Data mapping helps by providing organizations with
52. What is the difference between the total production of procedure links to show how certain tasks are to be uti-
the two companies in the given years? lized. Forty per cent of our nerve fibres that are linked
(a) 19,000 to the brain are in the retina. Data mapping helps us see
(b) 22,000 what makes different pieces of data useful and helpful.
(c) 26,000 Customer trends can be traced in the real time. The
(d) 28,000 causes of trends and past data numbers can be analysed
and other calculations of information and variables can
Ans: (c): Total production of Company X from 2007 be done. We can also use data mapping software to com-
to 2012 (in thousands) = 119 + 99 + 141 + 78 + pare our date with that of competitors. This should make
120 + 159 = 716 it easier for your business to grow when chosen right.
Total production of Company Y from 2007 to It also works by establishing larger maps. Sales-force of
2012 (in thousands) = 139 + 120 + 100 + 128 + any organization has a particularly strong data mapping
107 + 148 = 742 software program that can be put to use. This helps in real
Difference = (742 − 716) thousands = 26,000 time also. We can get connected to a cloud network to get
53. What is the average numbers of vehicles manufac- information in real time.
tured by Company X over the given period? Data mapping works for all businesses. For example,
(a) 1,19,333 (b) 1,17,166 if we were in the retail sector, then we can use data map-
(c) 1,12,778 (d) 1,11,223 ping to calculate how discount sales can influence the
Ans: (b) Average number of vehicles manufactured overall sales totals in a business. Similarly, financing and
by Company X = (119 + 99 + 141 + 78 + 107 + investment decisions can also be made.
159)/6 = 1,17.166 (in thousands) = 1,17,166 Data may be internal or external, but if it is getting
54. In which of the following years was the difference more dispersed and voluminous, then data leverage is
between the productions of Companies X and Y the important and actionable insights should be developed.
maximum? There are arrays of data points to collect information.
(a) 2007 (b) 2008 Their language may be quite different. We can develop
(c) 2009 (d) 2010 separate data models.

M07_MADAN 04_65901_C07.indd 13 21/12/22 11:26 AM


7.14 Chapter 7

Data mapping tasks vary in complexity, depending on so it needs to enter the analysis process with haphazard
the hierarchy or disparity between the structure of the ordering. Data analysis is usually subjective and, thus,
source and of the target. Every application, on-premise or the goals of interpretation may vary from one business to
on cloud basis, uses metadata to explain data fields. another. Basically, there are two main types of ­analysis,
Microsoft SharePoint, InetSoft Style Intelligence quantitative and qualitative.
and IBM Cognos Business Intelligence help us to review A good decision should be made regarding scales of
information by generating simple charts and graphs. measurement. The varying scales include the following:
Depending on the number and schema of the data
• Nominal scale: It consists of non-numeric categories
sources, database mappings can have a varying degree of
that cannot be ranked or compared quantitatively.
complexity.
­Variables are exclusive and exhaustive.
In general, data mapping helps with the following
• Ordinal scale: It consists of categories that are exclu-
activities.
sive and exhaustive but with a logical order. Quality
ratings and agreement ratings are examples of ordi-
Data Integration nal scales (i.e. good, very good, fair, etc., or agree,
It includes data mapping tools to cover differences in the strongly agree, disagree, etc.).
schemas of data source and destination, allowing busi- • Interval: It is a measurement scale where data is
nesses to consolidate information from different data grouped into categories with orderly and equal dis-
points ­easily. tances between the categories. There is always an
arbitrary zero point.
Data Migration • Ratio: It contains features of all three mentioned
It is moving data from one database to another. Here, above.
using a code-free data mapping solution that can auto- When interpreting data, an analyst must try to discern
mate the process is important to migrate data to the des- the differences between correlation, causation and coin-
tination successfully. cidences, etc., in addition to some other factors. In this
part, we will look at the two main methods of inter-
Data Warehousing pretation of data with a qualitative and a quantitative
Data mapping in a data warehouse is the process of creat- analysis.
ing a connection between the source and the target tables
or attributes. Qualitative Data I nterpretation
Narrative data is mostly collected by employing a wide
Data T ransformation variety of person-to-person techniques. It is basically
It is essential to break information silos and draw insights. described as ‘categorical’. The description is not through
Data mapping is the first step in data transformation. numerical values or patterns but through descriptive
context or text. These techniques include the following
Data Mapping T echniques parameters:
Although an essential step in any data management pro- • Observations: Here, behaviour patterns may be the
cess, data mapping can be complex and time-­consuming. amount of type and time spent in an activity and com-
Based on the level of automation, data mapping tech- munication used.
niques can be divided into two types: • Documents: Here, different types of documentation
resources can be coded and divided based on the type
1. Manual Data Mapping: Although hand-coded, man- of material they contain.
ual data mapping process offers unlimited flexibility. • Interviews: They are described as the best collection
2. Semi-automated Data Mapping: Schema mapping method for narrative data. Enquiry responses can
is often classified as a semi-automated data mapping be grouped by theme, topic or category. The inter-
technique. The process involves identifying two data view approach helps in highly focused data segmen-
objects that are semantically related and then build- tation.
ing mappings between them.
A person to person data collection technique can lead to
three basic principles, notice things, collect things and
Data Interpretation think about things. Qualitative data much open to inter-
The interpretation of data assigns a meaning to the infor- pretation must be ‘coded’ so as to facilitate the grouping
mation analysed and determines its signification and and labelling of data into identifiable themes.
implications. It refers to the implementation of processes
through which data is reviewed for the purpose of arriv- Quantitative Data Interpretation
ing at an informed conclusion. The keyword in quantitative is ‘numerical’. It is a set of pro-
Keeping in view its importance, data mapping should cesses by which numerical data is analysed. It involves the
be done properly. Data is obtained from multiple sources, use of statistical modelling, such as standard ­deviation,

M07_MADAN 04_65901_C07.indd 14 21/12/22 11:26 AM


Data Interpretation 7.15

mean and median. Let’s quickly review the most common ­ henomenon with its correlation. When two actions
p
statistical terms that are as follows: occurred together, one caused the other. This is not
accurate as actions can occur together. The remedy is
• Mean: A mean represents a numerical average for a to attempt to eliminate the variable you believe to be
set of responses. causing the phenomenon.
• Standard Deviation: It reveals the distribution of 2. Confirmation Bias: It occurs when we have a theory
the responses around the mean, the degree of con- or hypothesis in mind, but are intent on discover-
sistency within the responses and then insight into ing only data patterns that provide support, while
data sets. rejecting those that do not. This pitfall is often
• Frequency Distribution: This is a measurement of based on subjective desires. Thus, always remem-
gauging the rate of a response appearance within ber to try to disprove a hypothesis rather than try to
a data set. It is extremely keen in determining the prove it.
degree of consensus among data points. 3. Irrelevant Data: As large data is no longer centrally
It entails correlation tests between two or more variables. stored and as it continues to be analysed at the
The different processes can be used together or sepa- speed of thought, it is inevitable that analysts will
rately, and comparisons can be made to ultimately arrive focus on data that is irrelevant to the problem they
at a conclusion. Other signature interpretation processes are trying to correct. The remedy is to proactively
of quantitative data include the following: and clearly frame any data analysis variables and
key performance indicators prior to engaging in a
• Regression analysis data review.
• Cohort analysis
• Predictive and prescriptive analysis Keeping in view all these aspects, we need to be careful
about the following factors:
I mportance of Data I nterpretation
1. Collect your data and make it as clean as possible.
The purpose of collection and interpretation is to acquire 2. We need to be careful about the type of analysis to
useful and usable information and to make the most perform, be it qualitative or quantitative, and apply
informed decisions possible. Data interpretation includes the methods, respectively, to each. We have already
the following characteristics: discussed qualitative and quantitative aspects.
• Data identification and explanation 3. We may need to take a step back and think about data
• Comparing and contrasting of data from various perspectives and what it means for vari-
• Identification of data outliers ous participants or actors of the project.
4. We need to reflect on our own thinking and ­reasoning,
• Future predictions
such as correlation versus causation, subjective bias,
There are some common issues with data interpretation: false information and inaccurate data.
1. Informed Decision Making: Data analysis should
include identification, thesis development and data
collection followed by data communication. Data and Governance
2. Anticipating needs with Trends Identification: Data governance is a requirement in today’s fast-moving
Data insights provide knowledge, and knowledge is and highly competitive enterprise environment. Now that
power. organizations have the opportunity to c­apture massive
3. Cost Efficiency: Proper implementation of data anal- amounts of diverse internal and external data, they need
ysis processes can provide businesses with profound a discipline to maximize their value, manage risks and
cost advantages within their industries. reduce costs.
4. Clear Foresight: Companies that collect and ­analyse Data governance is a collection of processes, roles,
their data gain better knowledge about themselves, policies, standards and metrics that ensures the effective
their processes and performance. and efficient use of information in enabling an organiza-
tion to achieve its goals. It defines who can take what
In addition, there are certain problems with data inter-
action, on what data, in what situations and using what
pretation.
methods.
It is usually said that ‘big data equals big trouble’ where Data governance ensures that roles related to data are
some ‘pitfalls’ do exist and can occur when analysing data, clearly defined, and that responsibility and accountabil-
especially at the speed of thought. Let’s identify three of the ity are agreed upon across the enterprise. A well-planned
most common data misinterpretation risks and shed some data governance framework covers strategic, tactical and
light on how they can be avoided: operational roles and responsibilities.
1. Correlation mistaken for causation: It is the While crafting data and governance strategy, we need
tendency of data analysts to mix the cause of a to be careful.

M07_MADAN 04_65901_C07.indd 15 21/12/22 11:26 AM


7.16 Chapter 7

Data governance is not data management: Data man- • Consistent Compliance: Data governance provides
agement refers to the management of the full data lifecy- a platform for meeting the demands of government
cle needs of an organization. Data governance is the core regulations.
component of data management, including data ware- • Improved Data Management: It brings a human
housing. dimension into an otherwise highly automated and
data-driven world.
1. Data Governance is not Master Data Management:
Master data management focuses on identifying an We can use technology as the enabler for the same.
organization’s key entities and then improving the Open source and cloud are the basic strategies for data
quality of this data. governance tools. iPaaS is also closely linked with them.
2. Data Governance is not Data Stewardship: Data These tools also help us achieve the following:
stewards take care of data assets, making certain that
1. Capture and understand our data
the actual data is consistent with the data governance
2. Improve the quality of our data
plan, linked with other data assets and in control in
3. Managing Data: With metadata-driven ETL and ELT
terms of data quality, compliance or security.
and data integration applications
4. Controlling our data
Benefits of Data Governance 5. Document our data
6. Empower the people that know the data best: To
An effective data governance strategy provides many ben- contribute to the data stewardship
efits to an organization, including the following: 7. Protect sensitive data
• There is a common understanding of data. We need to understand that data governance is not
• There is improved quality of data, such as data
optional.
accuracy, completeness and consistency.
The implementation known as a ‘data lake’ necessar-
• Data map is available.
ily requires processes that allow you to keep the data you
• Holistic View: Data governance provides a 360° view
need in a way that eliminates technical b­ arriers and gives
of each ­customer and other business entities, basi-
new capabilities to process that data.
cally ‘a single ­version of the truth’.

M07_MADAN 04_65901_C07.indd 16 21/12/22 11:26 AM


Data Interpretation 7.17

A s s e s s Yo u r L e a r n i n g

Directions: Study the table given below and answer ques- 6. The number of students who appeared from School
tions 1–5. The table shows the populations of three states E in 2004 is approximately what percentage of the
over the years 2002–2007. total number of students who appeared from all the
schools together in that year?
Population (in lakhs) of three states over the years (a) 16% (b) 18%
(c) 20% (d) 25%
Year
7. What is the average number of students who appeared
State 2002 2003 2004 2005 2006 2007 from School B for all the years?
A 4.5 4.8 5.2 5.4 5.8 6.2 (a) 676 (b) 787
(c) 808 (d) 818
B 3.2 3.6 3.4 3.8 4.1 4.4 8. The number of students who appeared in 2006 from
C 5.6 5.5 5.8 6.3 6.6 6.9 School A is what per cent of the total number of stu-
dents who appeared from School A for all the years
1. What is the average population of State B for all the together?
years together (in lakhs)? (a) 25.25 (b) 21.33
(a) 3.5 (b) 3.6 (c) 22.45 (d) 23.45
(c) 3.75 (d) 3.8 9. What is the ratio between the total number of stu-
2. What is the percentage increase in population of State dents who appeared in 2004 and 2005 from Schools
A between 2002 and 2003? C and D, respectively?
(a) 5 (b) 5.25 (a) 84 : 79 (b) 79 : 84
(c) 6.67 (d) 7.5 (c) 84 : 89 (d) 89 : 84
3. What was the difference between combined popu- 10. What is the average number of students who appeared
lations of all the three states for the years 2004 and from the given schools in 2007?
2005? (a) 825 (b) 836 (c) 845 (d) 863
(a) 90,000 Directions: Study the following table carefully and
(b) 1,00,000 answer questions 11–15. It consists of data on the gradu-
(c) 1,10,000 ates and postgraduates living in various towns.

A S S E S S YO U R L E A R N I N G
(d) None of the above
4. What was the average population of all the three Towns Graduates Postgraduates
states in 2006 (in lakhs)? A 10,200 8000
(a) 5.5 (b) 5.8
(c) 6.1 (d) 6.3 B 25,250 18,000
5. What is the ratio between combined populations of all C 15,150 10,500
the three states in 2004 and 2005?
(a) 155 : 144 (b) 144 : 155 D 20,200 16,250
(c) 144 : 165 (d) 165 : 144 E 24,000 20,000
Directions: Study the following table carefully and F 16,500 18,450
answer questions 6–10. It consists of data on the num-
ber of candidates who appeared from five schools in the 11. What is the difference between the number of gradu-
board exams from 2004 to 2008. ates and the number of postgraduates in town C?
(a) 4500 (b) 4600
Schools
(c) 4650 (d) 4560
Year A B C D E 12. What is the average number of postgraduates in all
2004 650 760 820 800 780 the towns together?
(a) 15,000 (b) 15,500
2005 700 740 860 780 740 (c) 16,250 (d) 15,200
2006 800 820 940 750 730 13. What is the ratio of the number of graduates from
towns A and B together to the number of postgradu-
2007 750 880 920 840 790 ates from towns A and E together?
2008 850 840 900 860 770 (a) 709 : 580 (b) 709 : 560
(c) 560 : 709 (d) None of the above

M07_MADAN 04_65901_C07.indd 17 21/12/22 11:26 AM


7.18 Chapter 7

14. What is the total number of graduates and postgradu- Directions: Study the following bar graph carefully and
ates in towns A, D and F together? answer questions 21–25. It consists of data on student
(a) 85,500 enrolment in different states.
(b) 88,600 120
(c) 89,600 110
(d) 90,600 100
90
15. The number of graduates in town F is approximately 80
what percentage of the number of postgraduates in Student 70
60
the same town? enrolment 50
(a) 84 (b) 89 in 40
30
(c) 92 (d) 95 thousands 20
10
Directions: Study the following bar graph carefully 0
State State State State State State
and answer questions 16–20. It consists of data on
A B C D E F
exports and imports of IT and electronics industry over 2010 2011
a period.
Exports and Imports of
21. What is the ratio of the total enrolment of State B for
IT and Electronics Industry (in billion `)
years 2010 and 2011 to the total enrolment of State D
for both the years combined?
90 (a) 7 : 9 (b) 4 : 5
80 (c) 3 : 5 (d) 2 : 3
70 22. What is the percentage of total enrolment of State C
60 in comparison to enrolment of State E for years 2010
50 and 2011?
40 (a) 73 (b) 126
30 (c) 137 (d) 145
20
23. By what per cent is the enrolment in the year 2011
10 higher than that of 2010 for all the states combined?
2004 2005 2006 2007 2008 2009 (a) 12 (b) 15
Exports Imports (c) 18 (d) 20
24. Which of the following states has shown the highest
16. For how many years, the exports are at least 10% increase in student enrolment from years 2010 to
A S S E S S YO U R L E A R N I N G

higher than the imports? 2011?


(a) 1 (b) 2 (a) A (b) C
(c) 3 (d) 4 (c) E (d) F
17. In which year have the exports shown the highest 25. What is the average enrolment for all the states for the
growth in percentage terms in comparison to the pre- year 2010?
ceding year? (a) 80 (b) 85
(a) 2005 (b) 2006 (c) 90 (d) 95
(c) 2007 (d) 2008
Directions: The bar graph given below shows the percent-
18. What are the average exports (in billion `) for the age distribution of the total expenditures of a company
period 2004–2009? under various expense heads during 2003. Study it and
(a) 60.35 answer questions 26–30.
(b) 65.83
(c) 70.20 Breakup of Total Expenditure of a Company
(d) 75.36 25
20 20
Per cent spent

19. By how much percentage are the total exports higher 20 17.5
than total imports from 2004 to 2009? 15
15
(a) 4.15 12.5 10
10
(b) 5.33 5
(c) 6.33 5
(d) 7.58 0
Advertisement
Infrastructure

Interest on
Transport

Taxes

Salary
R&D

20. What is the percentage increase in imports between


loans

the years 2004 and 2009?


(a) 110 (b) 114
(c) 125 (d) 135

M07_MADAN 04_65901_C07.indd 18 21/12/22 11:26 AM


Data Interpretation 7.19

26. The total amount of expenditure of the company is 33. If the funds contributed by FIIs is `33,000 crore, then
how many times the expenditure on research and the funds contributed towards subscription of Bharat
development? Bonds by NRIs is
(a) 27 (b) 20 (a) `10,000 crore (b) `11,000 crore
(c) 18 (d) 8 (c) `12,000 crore (d) `12,500 crore
27. If the expenditure on advertisement is 2.10 crore, 34. If total subscription is `55,000, then the difference
then the difference between the expenditure on trans- between funds invested by banks and by public is
port and on taxes is (a) `550 crore (b) `1100 crore
(a) `1.25 crore (c) `1050 crore (d) `1650 crore
(b) `95 lakhs 35. If the difference between funds contributed by pub-
(c) `65 lakhs lic and by banks on one hand and by pension funds
(d) `35 lakhs on the other is `5000 crore, then the total funds sub-
28. What is the ratio of the total expenditure on infra- scribed for Bharat Bonds are
structure and transport to the total expenditure on (a) `25,000 crore
taxes and interest on loans? (b) `50,000 crore
(a) 5:4 (b) 8:7 (c) `55,000 crore
(c) 9:7 (d) 13:11 (d) `1,10,000 crore
29. If the interest on loans amounted to `2.45 crore, then
the total amount of expenditure on advertisement, Directions: Study the following pie chart carefully and
taxes and research and development is answer questions 36–40. It consists of data on tourist
(a) `7 crore (b) `5.4 crore arrival from different countries.
(c) `4.2 crore (d) `3 crore
Total Tourist Traffic = 20 lakhs
30. The expenditure on the interest on loans is by what
per cent more than the expenditure on transport? UK
(a) 5% (b) 10% 10%
(c) 20% (d) 40% USA
Others
20% 40%
Directions: Study the following pie chart carefully and
answer questions 31–35. It consists of data about sub- Japan
scription from different sources for Bharat Bonds issued 30%
by the Government of India.
Subscription Generated for 36. The difference between tourist numbers from the USA

A S S E S S YO U R L E A R N I N G
Bharat Bonds and Japan is
(a) 2 lakhs (b) 3 lakhs
NRIs
(c) 4 lakhs (d) 5 lakhs
11% FII
33% 37. The angle extended at the centre by sector of tourists
from the USA is
Companies (a) 108° (b) 118°
34%
Pension (c) 144° (d) 165°
funds 38. If the number of tourists from the UK doubles up
16% while the total remains the same, then the new angle
Banks extended by tourists from the UK will be
2% Public (a) 60° (b) 72°
4% (c) 90° (d) 120°
31. If the investments by NRIs is `8000 crore, then the 39. If the total number of tourists doubles while the abso-
combined investment of companies and FIIs into lute number of tourists from ‘others’ remains the
Bharat Bonds is same, then the new angle extended by ‘others’ at the
(a) `48,726 crore centre will be
(b) `48,000 crore (a) 36° (b) 72°
(c) `50,827 crore (c) 108° (d) None of the above
(d) Insufficient information 40. If the tourist traffic from the USA shows a growth of
32. If the total investment is `55,000 crore, then the com- 50% while the total number of tourists remains the
bined investment by pension funds and public is same, then the new percentage from the USA is
(a) `10,000 crore (b) `10,200 crore (a) 40% (b) 45%
(c) `10,500 crore (d) `11,000 crore (c) 50% (d) None of the above

M07_MADAN 04_65901_C07.indd 19 21/12/22 11:27 AM


7.20 Chapter 7

Directions: Study the following table carefully and answer 46. What is the respective ratio of the total number of
questions 41–45. It consists of breakup of expenses of a girls enrolled in painting in Institutes A and C together
company over different years. to those enrolled in stitching in Institutes D and E
together?
Items of Expenditure (`, in lakhs) (a) 14:23 (b) 16:23
(c) 18:23 (d) 8:12
Year Salary Fuel and Bonus Interest Taxes
Transport on Loans 47. The number of girls enrolled in stitching in Institute
B forms approximately what percentage of the total
1998 288 98 3.00 23.4 83 number of girls enrolled in stitching in all the insti-
1999 342 112 2.52 32.5 108 tutes put together?
(a) 19 (b) 21
2000 324 101 3.84 41.6 74 (c) 23 (d) 25
2001 336 133 3.68 36.4 88 48. What is the respective ratio of the total number of
2002 420 142 3.96 49.4 98 girls enrolled in painting and stitching from all the
institutes put together?
(a) 11:12 (b) 12:11
41. What is the average amount of interest per year which (c) 11:14 (d) 12:17
the company had to pay during this period?
(a) `32.43 lakhs 49. The number of girls enrolled in dancing in Institute
(b) `33.72 lakhs A forms what percentage of the total number of girls
(c) `34.18 lakhs enrolled in all the vocational courses together in that
(d) `36.66 lakhs institute?
(a) 20.7 (b) 25.5
42. The total amount of bonus paid by the company dur- (c) 28.2 (d) 29.5
ing the given period is approximately what percent-
age of the total amount of salary paid during this 50. What is the total number of girls enrolled in painting
period? from all the institutes together?
(a) 0.1% (b) 0.5% (a) 1050
(c) 1% (d) 1.25% (b) 1100
(c) 1150
43. Total expenditure on all these items in 1998 was (d) 1200
approximately what percentage of the total expendi-
ture in 2002? Directions: Study the following line graph carefully and
(a) 62% (b) 66% answer questions 51–55. It consists of profit data of a
(c) 69% (d) 71% company for 2003 and 2004.
A S S E S S YO U R L E A R N I N G

44. The total expenditure of the company over these


Profit (`, in crores) Earned by Companies During
items during year 2000 is
2003 and 2004 (Profit = Income − Expenditure)
(a) `544.44 lakhs (b) `501.11 lakhs
(c) `446.46 lakhs (d) `478.87 lakhs 90
80
45. The ratio between the total expenditure on taxes for
all the years and the total expenditure on fuel and 70
transport for all the years is approximately 60
2003
(a) 4:7 (b) 10:13 50 2004
(c) 15:18 (d) 5:8 40
30
Directions: Study the following bar graph carefully and
20
answer questions 46–50. It consists of data on student
10
enrolment in different vocational courses in A, B, C, D and
0
E institutes. A B C D E F G

450 51. What is the ratio between the profits earned by Com-
400
350
pany A in 2004 and Company B in 2003?
300 (a) 4:3 (b) 3:2
Painting (c) 3:4 (d) 1:1
250
200 Stitching 52. What is the difference between the total profit earned
150 Dancing by Companies E, F and G together in 2003 and 2004
100 (`, in crores)?
50 (a) 70 (b) 72
0 (c) 78 (d) 80
A B C D E

M07_MADAN 04_65901_C07.indd 20 21/12/22 11:27 AM


Data Interpretation 7.21

53. What is the ratio between the profit earned by Com- 57. What is the difference in graduate and postgraduate
pany C in 2003 and 2004 together and the profit courses in College A?
earned by Company E in the same 2 years mentioned (a) 1400 (b) 1600
above? (c) 1800 (d) 2000
(a) 11:9 (b) 10:11 58. By what percentage are admissions in graduate
(c) 9:11 (d) None of the above courses higher than postgraduate courses in case of
54. What is the approximate average profit earned by all College F?
the companies in year 2003 (`, in crores)? (a) 31.25% (b) 25.50%
(a) 53 (b) 58 (c) 33.33% (d) 35.50%
(c) 62 (d) 68 59. What is the difference between the highest admis-
55. The profit earned by Company B in 2004 is what per sions in graduate and the highest admissions in post-
cent of the profit earned by the same company in graduate courses in any combination of the colleges?
2003? (a) 1200 (b) 1500
(a) 60% (b) 75% (c) 1600 (d) 2000
(c) 125% (d) 133.33% 60. By what per cent are postgraduate admissions lower
Directions: Study the following pie charts carefully and than graduate admissions in case of College D?
answer questions 56–60. It consists of data on admission (a) 45% (b) 55%
(c) 70% (d) 80%
in graduate and postgraduate courses in different institu-
tions. Directions: The bar graph given below shows the foreign
exchange reserves of a country (in million USD) from
Postgraduate Admissions 2011–12 to 2018–19.
(Percentage Breakup)
Foreign Exchange Reserves of a Country (in Million USD)

F, 16 A, 20 6000
5040
5000 4320
3720 3360
4000
E, 14 2640 2520 3120 3120
3000
B, 18 2000
1000
D, 10
0

A S S E S S YO U R L E A R N I N G
C, 22
2

9
-1

-1

-1

-1

-1

-1

-1

-1
11

12

13

14

15

16

17

18
20

20

20

20

20

20

20

20
Total Admissions = 20,000

Graduate Admissions 61. The ratio of the number of years in which the foreign
(Percentage Breakup) exchange reserves are above the average reserves to
those in which the reserves are below the average
F, 14 reserves is
A, 18 (a) 2:6 (b) 3:4
(c) 3:5 (d) 4:4
E, 16
62. The foreign exchange reserves in 2017–18 were how
many times of those in 2014–15?
B, 20 (a) 0.7 (b) 1.2
(c) 1.4 (d) 1.5
D, 12
63. For which year has the percentage increase in foreign
C, 20 exchange reserves over the previous year been the
highest?
Total admissions = 30,000 (a) 2012–13 (b) 2013–14
(c) 2014–15 (d) 2016–17
56. What are the total admissions in College B for both 64. The foreign exchange reserves in 2016–17 were
graduate and postgraduate courses? approximately what percentage of the average for-
(a) 9600 (b) 9800 eign exchange reserves over the period under review?
(c) 10,200 (d) 10,500 (a) 95% (b) 110%
(c) 115% (d) 125%

M07_MADAN 04_65901_C07.indd 21 21/12/22 11:27 AM


7.22 Chapter 7

65. What was the percentage increase in the foreign Codes:


exchange reserves in 2017–18 over those in 2013–14? (a) A-I, B-II, C-III, D-IV
(a) 100% (b) 150% (b) A-I, B-III, C-II, D-IV
(c) 200% (d) 250% (c) A-IV, B-III, C-II, D-I
(d) A-IV, B-II, C-III, D-I
Directions: Study the bar graph given below and answer
73. The National Sample Survey (NSS) came into being
questions 66–70.
to collect information through sample surveys on a
Online Travel Industry Sales Turnover variety of socioeconomic aspects in the year
(`, in crores) (a) 1950 (b) 1952
37,900 (c) 1956 (d) 1962
40,000
35,000 74. The Indian Statistical Institute is located in
30,000 (a) New Delhi (b) Kolkata
25,300
25,000 (c) Mumbai (d) Hyderabad
20,000 Sales
10,500 15,000 turnover 75. How many divisions are there in the Central Statisti-
15,000
6300 cal Organization?
10,000
5000 (a) 2 (b) 3
0 (c) 4 (d) 5
2007 2008 2009 2010 2011
76. The National Statistical Commission was set up in the
year
66. Which year witnessed the maximum growth in sales (a) 2000 (b) 2003
turnover in absolute terms? (c) 2005 (d) 2008
(a) 2008 (b) 2009
(c) 2010 (d) 2011 77. Which of the following methods is best suited to show
on map the types of crops grown in a region?
67. Which year witnessed the maximum growth in sales (a) Choropleth
turnover in percentage terms? (b) Chorochromatic
(a) 2008 (b) 2009 (c) Choroschematic
(c) 2010 (d) 2011 (d) Isopleth
68. What is the average sales turnover during all the years 78. Which of the following sources of data is not based on
(`, in crores)? primary data collection?
(a) 17,000 (b) 18,000 (a) Census of India
(c) 19,000 (d) None of the above (b) National Sample Survey
69. What is the approximate percentage growth between (c) Statistical Abstracts of India
A S S E S S YO U R L E A R N I N G

2007 and 2011? (d) National Family Health Survey


(a) 400% (b) 500% 79. Which of the following is not a source of data?
(c) 525% (d) 600% (a) Administrative records
70. How many years witnessed more than aver­ age (b) Population census
growth? (c) GIS
(a) 1 (b) 2 (d) Sample survey
(c) 3 (d) None of the above 80. Which of the following is the oldest archival source of
71. Census is conducted after every data in India?
(a) 5 years (b) 10 years (a) National Sample Survey
(c) 11 years (d) 15 years (b) Agricultural statistics
72. Match List-I with List-II. (c) Census
(d) Vital statistics
List-I (Event) List-II (Year)
A Statistical abstract of British I 1868
India
B Agricultural statistics of II 1886
British India
C First census III 1881
D Central Statistical IV 1952
Organization

M07_MADAN 04_65901_C07.indd 22 21/12/22 11:27 AM


Data Interpretation 7.23

Answer Keys
1. (c) 2. (c) 3. (c) 4. (a) 5. (b) 6. (c) 7. (c) 8. (b) 9. (a) 10. (b)
11. (c) 12. (d) 13. (b) 14. (c) 15. (b) 16. (b) 17. (a) 18. (b) 19. (b) 20. (b)
21. (a) 22. (c) 23. (b) 24. (a) 25. (a) 26. (b) 27. (d) 28. (d) 29. (c) 30. (a)
31. (a) 32. (d) 33. (b) 34. (b) 35. (b) 36. (a) 37. (c) 38. (b) 39. (a) 40. (c)
41. (d) 42. (c) 43. (c) 44. (a) 45. (b) 46. (b) 47. (b) 48. (a) 49. (a) 50. (b)
51. (d) 52. (d) 53. (a) 54. (a) 55. (b) 56. (a) 57. (a) 58. (a) 59. (c) 60. (d)
61. (c) 62. (d) 63. (a) 64. (d) 65. (a) 66. (d) 67. (d) 68. (c) 69. (b) 70. (b)
71. (b) 72. (a) 73. (a) 74. (b) 75. (d) 76. (c) 77. (a) 78. (c) 79. (d) 80. (b)

A S S E S S YO U R L E A R N I N G

M07_MADAN 04_65901_C07.indd 23 21/12/22 11:27 AM


7.24 Chapter 7

Solution
1. (c): Required average Total number of postgraduates from A and E = 8000
+ 20,000 = 28,000
⎡ 3 . 2 + 3 . 6 + 3 . 4 + 3 . 8 + 4 .1 + 4 .4 ⎤ Required ratio = 35,450:28,000 = 709:560
=⎢ ⎥⎦
⎣ 6 14. (c): Total number of graduates in A, D and F = 46,900
⎡ 22.5 ⎤ Total number of postgraduates in A, D and F = 42,700
=⎢ ⎥ = 3.75 lakhs Required total = 46,900 + 42,700 = 89,600
⎣ 6 ⎦
15. (b): Required percentage = 16,500/18,450 × 100
2. (c): Required percentage = (4.8 – 4.5)/4.5 × 100 = 89.43 ~ 89%
= 0.3/4.5 × 100 = 6.67% 16. (b): As the comparison is with imports, the imports
3. (c): Combined population in 2004 = 5.2 + 3.4 + 5.8 will appear as the denominator.
= 14.4 lakhs The formula to calculate the percentage figure for
Combined population in 2005 = 5.4 + 3.8 + 6.3 higher exports with regard to imports is the following:
= 15.5 lakhs Difference/imports × 100
Difference = 15.5 − 14.4 = 1.1 lakhs = 1,10,000 In 2004, exports = 40 > 35
4. (a): Average population in 2006 = (5.8 + 4.1 + 6.6)/3 Required percentage figure = 5/35 × 100 ∼ 14%
= 16.5/3 = 5.5 lakhs Similarly, for the year 2005, required percentage
= [(65 − 60)/60] × 100 = 8.33%
5. (b): Total population in 2004 = 5.2 + 3.4 + 5.8
For 2006, imports > exports, so no calculation is
= 14.4 lakhs
required.
Total population in 2005 = 5.4 + 3.8 + 6.3 = 15.5 lakhs
For 2007, 5/50 × 100 = 10%
Required ratio = 14.4:15.5 = 144:155
For 2008, 5/80 × 100 = 6.25%
6. (c): Total candidates who appeared in 2004 = 3810 For 2009, 5/75 × 100 = 6.66%
Percentage of students from School E = 780/3810 Thus, for years 2004 and 2007, the exports are at
× 100 = 20.47 ∼ 20% least 10% higher than imports.
7. (c): Required average = (760 + 740 + 820 + 880 17. (a): The required formula = increase/exports in pre-
+ 840)/5 = 4040/5 = 808 ceding year × 100
8. (b): Total number of students who appeared from In 2005, percentage increase in exports = (65 − 40)/
School A during 2004–2008 = (650 + 700 + 800 + 750 40 × 100 = 25/40 × 100 = 62.5%
+ 850) = 3750 In 2006, percentage increase in exports = 5/65 × 100
A S S E S S YO U R L E A R N I N G

Required percentage = 800/3750 × 100 = 21.33% = 7.69%


9. (a): Required ratio = (820 + 860):(800 + 780) In 2007, exports have actually declined over the pre-
= 1680:1580 = 84:79 ceding year, so no calculation is required.
10. (b): Required average In 2008, percentage increase in exports = [(85 − 55)/
55] × 100 = 54.54%
⎡ (750 + 880 + 920 + 840 + 790 ⎤ ⎡ 4180 ⎤ In 2009, exports declined in compar­ison to 2008;
=⎢ ⎥⎦ = ⎢⎣ 5 ⎥⎦
⎣ 5 again no calculation is required.
= 836 18. (b): Average export = (40 + 65 + 70 + 55 + 85 + 80)/6
= 395/6 = `65.83 billion
11. (c): Number of graduates in C = 15,150 19. (b): Total exports = `395 billion
Number of postgraduates in C = 10,500 Total imports = `375 billion
Difference = 15,150 − 10,500 = 4650 Difference = `20 billion
12. (d): Required average Percentage difference = 20/375 × 100 = 5.33%
As the comparison is with imports, imports will
⎡ (8, 000 + 18, 000 + 10, 500 ⎤ appear as denominator.
⎢ ⎥
+16, 250 + 20, 000 + 18, 450) ⎥ 20. (b): The imports have increased from 35 in 2004 to
=⎢
⎢⎣ 6 ⎥⎦ 75 in 2009. Hence, percentage increase in imports
should be calculated by taking 2004 as the base
⎡ 91, 200 ⎤
=⎢ ⎥ = 15, 200 year.
⎣ 6 ⎦ Required percentage = [(75 − 35)/35] × 100 = 114.28
∼ 114%
13. (b): Total number of graduates from A and B = 10,200 21. (a): The required ratio is
+ 25,250 = 35,450 (65 + 75)/(85 + 95) = 140/180 = 7:9

M07_MADAN 04_65901_C07.indd 24 21/12/22 11:27 AM


Data Interpretation 7.25

22. (c): As the comparison is with State E, it will appear as 36. (a): Difference = 40 – 30 = 10
denominator. Now, 10% of 20 lakhs = 2 lakhs
The required ratio = [(95 + 110)/(70 + 80)] × 100 Hence, option (a) is the answer.
= 136.66% ∼ 137% 37. (c): 40/100 × 360 = 144°
23. (b): Total enrolments for the year 2010 = 80 + 75 + 95 38. (b): Old number = (10/100) × 20 = 2 lakhs
+ 85 + 75 + 70 = 480
New number = 2 × 2 = 4 lakhs
Total enrolments for the year 2011 = 105 + 65 + 110
New angle extended at centre = 4/20 × 360 = 72°
+ 95 + 95 + 80 = 550
Percentage difference = [(550 − 480)/480] × 100 39. (a): Old number from ‘others’ = 20% of 20 lakhs
= 14.58% ∼ 15% = 4 lakhs
New total of tourists = 20 × 2 = 40 lakhs
24. (a): For State A, percentage increase = [(105 – 80)/
80] × 100 = 31.25% So, new angle extended at centre by ‘others’ = 4/40
× 360 = 36°
For State C, percentage increase = [(110 − 95)/95]
× 100 = 15.78% 40. (c): Old number of tourists from the USA = 40% of
For State E, percentage increase = [(95 − 75)/75] 20 lakhs = 8 lakhs
× 100 = 26.66% Increase of 50% = 50% of 8 lakhs = 4 lakhs
For State F, percentage increase = [(80 − 70)/70] New traffic figure from the USA = 8 + 4 = 12 lakhs
× 100 = 14.28% New total number from all countries = 20 + 4
25. (a): Average enrolments for 2010 = 480/6 = 80 = 24 lakhs
26. (b): Expenditure on R&D is 5% of the total Revised percentage of tourist traffic from the USA
expenditure. = 12/24 × 100 = 50%
It means that it is 1/20th of the total expenditure. 41. (d): Average interest =
27. (d): Advertisement expenditure is 15% of the total
⎡ 23.4 + 32.5 + 41.6 + 36.4 + 49.4 ⎤
expenditure. = `⎢ ⎥ lakhs
Given, 15% of total expenditure = 2.10 crore ⎣ 5 ⎦
Hence, total expenditure = 2.10 × 100/15 = 14 crore ⎡ 183.3 ⎤
Transport cost = 12.5% of 14 crore = 1.75 crore = `⎢ ⎥ lakhs = `36.66 lakhs
⎣ 5 ⎦
Taxes = 10% of 14 crore = 1.4 crore
So, the difference between transport and taxes = 1.75 42. (c): Required percentage
− 1.40 = 0.35 crore = 35 lakhs
⎡ (3.00 + 2.52 + 3.84 + 3.68 + 3.96) ⎤
28. (d): Total of infrastructure and trans­ port:total of =⎢ × 100⎥ %
⎣ (288 + 342 + 324 + 336 + 420 ) ⎦

A S S E S S YO U R L E A R N I N G
taxes and interest on loans
= (20 + 12.5) : (10 + 17.5) ⎡ 17 ⎤
=⎢ × 100⎥ % = 1%
= 32.5 : 27.5 ⎣ 1710 ⎦
= 13 : 11
43. (c): Required percentage
29. (c): As interest on loan is 10% of total expenses,
17.5% of total expenditure = `2.45 crore ⎡ (288 + 98 + 3.00 + 23.4 + 83) ⎤
=⎢ × 100⎥ %
Total expenditure = (2.45 × 100)/17.5 = 14 crore ⎣ (420 + 142 + 3.96 + 49.4 + 98) ⎦
Advertisement + taxes + R&D = 15 + 10 + 5 = 30% ⎡ 4955 .4 ⎤
30% of `14 crore = `4.2 crore =⎢ × 100⎥ % = 69.45%
⎣ 713.36 ⎦
30. (a): Difference = 17.5 – 12.5 = 5%
31. (a): Combined investment by companies and FIIs 44. (a): Total expenditure during 2000
= 67/11 × 8000 = `48,726 crore = `(324 + 101 + 3.84 + 41.6 + 74) lakhs
32. (d): (16 + 4)/100 × 55,000 = `11,000 crore = `544.44 lakhs
33. (b): FIIs contribute 33% and NRIs contribute 11% of 45. (b): Required ratio
the total funds. ⎡ (83 + 108 + 74 + 88 + 98) ⎤
Hence, funds contributed by NRIs = 11/33 × 33,000 =⎢ ⎥
⎣ (98 + 112 + 101 + 133 + 142) ⎦
= `11,000 crore
⎡ 451 ⎤ 10
34. (b): Percentage difference = 4 – 2 = 2% =⎢ ⎥= = 10 : 13
Given that 2% of `55,000 crore = (2/100) × 55,000 ⎣ 586 ⎦ 13
= `1100 crore 46. (b): Total number of girls enrolled in painting in A
35. (b): Difference in percentage terms = 16 – (2 + 4) = 10% and C = 250 + 150 = 400
10% of total funds = `5000 crore Total number of girls enrolled in stitching in D and E
Hence, total funds = 5000/10% = 5000 × 100/10 = 250 + 325 = 575
= `50,000 crore Required ratio = 400:575 = 16:23

M07_MADAN 04_65901_C07.indd 25 21/12/22 11:27 AM


7.26 Chapter 7

47. (b): Total number of girls enrolled in stitching in all Admission in postgraduate courses in College C
institutes = 325 + 250 + 50 + 250 + 325 = 1200 = 22% of 20,000 = 4400
Number of girls enrolled in stitching in B = 250 Admission in graduate courses in College C = 20% of
Required percentage = 250/1200 × 100 = 21% 30,000 = 6000
48. (a): Total girls enrolled in painting = 250 + 225 + 150 Difference = 6000 − 4400 = 1600
+ 175 + 300 = 1100 60. (d): Admission in postgraduate courses in College D
Total enrolment in stitching course = 1200 (calcu- = 10% of 20,000 = 2000
lated in the earlier question) Admission in graduate courses in College C = 12% of
Total enrolment in dancing = 150 + 200 + 75 + 400 30,000 = 3600
+ 350 = 1175 Difference = 3600 − 2000 = 1600
Required ratio = 1100:1200 = 11:12 As the comparison is with admissions in graduate
49. (a): Total enrolments of girls in A = 250 + 325 + 150 courses, 3600 will be taken as the denominator.
= 725 Percentage difference = 1600/3600 × 100 = 44.5%
Number of girls enrolled in dancing in A = 150 ∼ 45%
Required percentage = 150/725 × 100 = 20.69 61. (c): Average Foreign Exchange Reserves (FER) over
∼ 20.7% the given period = 3480 million USD
50. (b): Solution is done as in Question 48. (a): The country had reserves above 3480 million USD
51. (d): Profit earned by Company A in 2004 = `40 crore during the years 2012–13, 2016–17 and 2017–18, that
Profit earned by Company B in 2003 = `40 crore is, for 3 years.
Required ratio = 40 : 40 = 1 : 1 (b): They are below 3480 million USD during the years
2011–12, 2013–14, 2014–15, 2015–16 and 2018–19,
52. (d): Total profits earned by Companies E, F and G in that is, for 5 years. Hence, the required ratio is 3 : 5.
2003 = 50 + 80 + 60 = `190 crore
62. (d): Required ratio = 5040/3360 = 1.5
Total profit earned by Companies E, F and G in 2004
= 40 + 20 + 50 = `110 crore 63. (a): There is an increase in foreign exchange reserves
Required difference = 190 − 110 = `80 crore during the years 2012–13, 2014–15, 2016–17 and
2017–18 as compared to the previous year (as shown
53. (a): The profit earned by Company C in 2003 and by bar graph).
2004 = 50 + 60 = `110 crore
The percentage increases in reserves during these
Profit earned by Company E in 2003 and 2004 years compared to previous years are as follows:
= 40 + 50 = `90 crore For 2012–13:
Required ratio = 110 : 90 = 11 : 9
54. (a): Required average = (20 + 40 + 50 + 70 + 50 + 80 ⎡ (3720 − 2640) ⎤
=⎢ ×100 ⎥ % = 40.91%
A S S E S S YO U R L E A R N I N G

+ 60)/7 = 370/7 = 52.86 ∼ `53 crore ⎣ 2640 ⎦


55. (b): Profit earned by Company B in 2004 = `30 crore
For 2014–15:
Profit earned by Company B in 2003 = `40 crore
Required ratio = 30/40 × 100 = 75% ⎡ (3360 − 2520) ⎤
=⎢ ×100 ⎥ % = 33.33%
56. (a): Postgraduate admissions in College B = 18% of ⎣ 2520 ⎦
20,000 = 3600
Graduate admissions in College B = 20% of 30,000 For 2016–17:
= 6000
Total admissions = 3600 + 6000 = 9600 ⎡ (4320 − 3120) ⎤
=⎢ × 100 ⎥ % = 38.46%
57. (a): Admissions in postgraduate courses in College A ⎣ 3120 ⎦
= 20% of 20,000 = 4000 For 2017–18:
Admissions in graduate course = 18% of 30,000
= 5400 ⎡ (5040 − 4320) ⎤
=⎢ × 100 ⎥ % = 16.67%
Required difference = 5400 − 4000 = 1400 ⎣ 4320 ⎦
58. (a): Admissions in postgraduate courses in College F
= 16% of 20,000 = 3200 Thus, the percentage increase over previous years is
Admissions in graduate courses in College F = 14% of highest for 2012–13.
30,000 = 4200 64. (d): Average FER over the given period
Difference = 4200 − 3200 = 1000 = (2640 + 3720 + 2520 + 3360 + 3120 + 4320 + 5040
Percentage difference = 1000/3200 × 100 = 31.25% + 3120)/8
59. (c): The highest admissions were in College C for = 3480 million USD
graduate as well postgraduate college. FER in 2016–17 = 4320 million USD

M07_MADAN 04_65901_C07.indd 26 21/12/22 11:27 AM


Data Interpretation 7.27

According to the question, let’s assume FER for Percentage growth in 2009 = (4500/ 10,500) × 100
2016–17 = x% of average FER ∼ 43%
Thus, 4320 = x% of 3480 Percentage growth in 2010 = (10,300/15,000) × 100
x% = 4320/3480 × 100 = 124.1% ∼ 125% ∼ 69%
Percentage growth in 2011 = (12,600/25,300) × 100
65. (a): As the FER doubled from 2520 in 1993–94 to
∼ 50%
5040 in 2017–18, there is an increase of 100%.
Alternatively, it can be calculated very easily. 68. (c): Total turnover = 6300 + 10,500 + 15,000 + 25,300
Increase = 5040 − 2520 = 2520 + 37,900 = 95,000 crore
Increase in percentage = (2520/2520) × 100 = 100% Average = 95,000/5 = 19,000 crore
66. (d): Growth in 2008 = 10,500 – 6300 = 4200 crore 69. (b): Increase between 2007 and 2011 = 37,900 – 6300
Growth in 2009 = 15,000 – 10,500 = 4500 crore = 31,600 crore
Growth in 2010 = 25,300 − 15,000 = 10,300 crore Percentage growth = 31,600/6300 × 100 = 500%
Growth in 2011 = 37,900 – 25,300 = 12,600 crore 70. (b): Years 2010 and 2011 witnessed higher than aver-
67. (d): Percentage growth in 2008 = (4200/6300) × 100 age sales turnover figures.
= 66.66%

A S S E S S YO U R L E A R N I N G

M07_MADAN 04_65901_C07.indd 27 21/12/22 11:27 AM


This page is intentionally left blank

M07_MADAN 04_65901_C07.indd 28 21/12/22 11:27 AM


CHaPTER Information and
8 Communication
Technology (ICT)

01 Meaning of Information and


Communication Technology

02 ICT: General Abbreviations and


Terminology

LEaRNING
03 Basics of Internet,
oBJECTIVEs Intranet, Email, Audio and
Video conferencing

04 Digital Initiatives in Higher Education

05 ICT and Governance

M08_MADAN 04_65901_C08.indd 1 23/12/22 7:38 PM


8.2 Chapter 8

Table 8.1  asic Rationales for Introducing IT in


B
Introduction Education
Information and communication technology (ICT) is a
very important topic. We can see that during pandemic Rationale Basis
COVID19 situation also, that made us to look to digital Social Keeping in view the role that
education in a bigger manner. We have a large population, technology now plays in society,
and in year or so we are going to overtake China to become students need to learn technology
number one. The knowledge expansion but limited time
Vocational Now, most jobs require technological
span has compelled us to look at our digital journey more
orientation
seriously. We need to achieve our objectives of access,
equity and quality in education with the help of ICT. In Catalytic To enhance teaching effectiveness with
this unit, we are exploring all the aspects. the help of technology
Pedagogical To make use of technology in
ICT Basics and Its Link with enhancing learning, flexibility and
efficiency in the course of delivery
Education
UNESCO defines ICT as ‘the scientific, technological and
engineering disciplines and the management technique In terms of demography, almost half of India’s population
used to handle transmit information with men and is below 25 years of age, 70% people live in rural areas,
machines’. the literacy rate is around 60% and people speak 15
ICT includes both old and new tools. Old ICT tools major languages. The demand for education outstrips
mainly include radio, TV and telephone. New ICT tools the conventional system’s ability to provide it, leaving no
mainly include computers, satellite, wireless technology. alternative but the use of technology in education. Urban–
These different tools are now able to work together and rural divide (sometimes termed digital divide) still exists
combine to form our networked world. in terms of access, equity and resources.
These conventional and more familiar technologies are
referred under the collective heading of analogue media, Objectives of Using ICT in Higher Education
while the newer computer and internet-based technolo-
gies are called the digital media. The differentiation Now ICT has become a more permanent feature of our
between the old ICT and the new ICT is subjective to some education. The main objectives of using ICT are as follows:
extent. 1. Improving access to the system through online
Education has been identified as one of the 12 main ­education
services under General Agreement on Trade in Services 2. Improving the quality of teaching, especially across
(GATS), which needs to be opened up for free flow remote locations
of trade between countries. Knowledge is expected to 3. Increasing transparency and strengthening the
become a tradable commodity and it will be essential ­systems, processes and compliance norms in higher
for Indian educators to keep pace with the change or education institutes
else perish in the face of competition from multinational 4. Measuring the students’ learning participation and
forces in all fields of education and learning, including effectiveness
adult learning. 5. Analysing students’ behaviour to maximize their
According to Cross and Adam, the four basic ratio- involvement, optimizing retention and improving
nales behind introducing ICT in education are given in placements
Table 8.1. 6. Analysing students’ performance, placement, applica-
According to our main objectives, ICT in education has tion volume, website analytics and social media met-
three main parts: rics for brand audit
1. ICT Education: This refers to the creation of trained
manpower to meet the IT needs of knowledge ­society,
Strengths and L imitations of ICTs
in terms of both software and hardware. Strengths of ICT
2. ICT-supported Education: This is sometimes termed Some of the strengths of ICTs are as follows:
multimedia education. Nowadays, many distance
education universities and institutions use ICT to sup- 1. Individualization of Learning: Due to ‘individual-
plement printed study materials. Here, ICT mainly ization’ of human nature, and complications in soci-
includes broadcast media such as radio and TV pro- ety, we need ‘customization’ of education. We need
grammes, audio and video tapes. Multimedia con- self paced learning.
tents such as lessons are offered on CDs. 2. Interactivity: Through interactivity feature, the learner
3. ICT-enabled Education: In this case, a compre- can relate to the content more effectively, go forward
hensive educational programme is purely delivered and backward in the content and start at any point,
through ICTs. depending on prior knowledge and experience. It is

M08_MADAN 04_65901_C08.indd 2 23/12/22 7:38 PM


Information and Communication Technology (ICT) 8.3

not always necessary that the learner should move in a Synchronous and A synchronous Media
sequential manner. The ICT media can be divided into two categories.
3. Distance and Climate Insensitive: Teaching and
learning could be taken out of the conventional edu- (a) S
 ynchronous media: This requires all participants
cation system of schools and colleges. This can help in to be together at the same time even when they
overcoming geo-climatic conditions also. are at different locations. The examples are online
4. More Eeconomical, Higher Speed of Delivery and chat, instant messaging and video conferencing.
Wider Reach: This happens with increase in the (b) Asynchronous media: This allows participants
number of learners. in the learning process to be at different times
5. Multiple Teaching Functions and Diverse and at different places. The online learning
Audiences: Using ICTs, especially computer-and resources used to support asynchronous learning
internet-based technologies, can be useful in ‘drills include email, electronic mailing lists, threaded
and practices’ to help diagnose and solve problems conferencing systems, online discussion boards,
for accessing information and knowledge about vari- wikis, blogs, etc.
ous related themes.
6. Uniform quality: With effective ‘ICT’ tools, the same
quality of education can be delivered to the rich and Stopover
the poor, the urban and the rural equally at the same
low cost. Thus, ICT has become a great equalizer. Which of the following communication technologies

employ only asynchronous communication?
(1) Video conferencing
Limitations of ICT (2) Email
Some of the limitations depend on the use of ICTs: (3) Forums
(4) Instant messaging
1. High Infrastructure and Start-up Costs: It costs Choose the correct option.
money to build ICT systems and to maintain them. The (a) (1) and (3)
cost of hardware and software can be very high. (b) (2) and (4)
2. Little Attention Towards Individual Differences in (c) (2) and (3)
Order to Achieve Economies of Scale: In order to (d) (1) and (4)
reduce the cost, the same contents may be delivered
The correct option is (c).
to all. Thus, the basic learning goal of ‘individual dif-
ferences’ may be ignored. That may create a ‘digital
divide’ also where one resourceful section of society
may be benefited more. The others are deprived of Major ICT Learning Categories
such benefits. In view of ICT, education can be classified into three main
3. Accessibility Issue: Timing of broadcast, electric- categories as follows:
ity supply, socio-cultural ­factors, poverty, illiteracy,
1. E-learning
time constraints, mobility and relevance are the key
2. Blended learning
factors.
3. Distance learning
4. ICT is Basically a Delivery System: A medium is dif-
ferent from the content. ICTs are essentially meant In addition, there are face-to-face, self-paced and
only to deliver content and are not expected to initi- online collaborative learning under major ICT learning
ate major attitudinal or behavioural changes. categories.
5. Difficulty in Performance Evaluation: Learning
from ICTs is usually multidimensional and with E-learning or Electronic Learning
long-term perspective. Thus, it may take longer time This happens with Advanced Learning Technologies (ALT).
to assess performance in comparison to classroom These tools enhance human learning with advanced net-
assessment which is spontaneous. worked and multimedia technologies. Distance education
6. Continuous Training Requirement: As technologies provided the base for e-learning’s development. E-learning
change, there is a continuous need to train the train- can be ‘on demand’. It overcomes timing, attendance and
ers, which may sometimes be resisted by them. We travel difficulties and also allows higher participation and
need to increase their expertise. greater interaction.
7. Call for Attitudinal Change to Understanding of
Teaching and Learning: ICT is basically a commu- Blended Learning
nication tool rather than face-to-face conversation. It is a combination of multiple approaches to learning.
Thus, there is possibility of increase of transactional It is usually used to define a situation where different
distance that we discussed in Unit 3. The potential delivery methods are combined together to deliver a
threat of plagiarism is high as students can copy particular course. These methods may include a mixture
information rather than learn and develop their own of face-to-face learning, self-paced learning and online
skills. classrooms.

M08_MADAN 04_65901_C08.indd 3 23/12/22 7:38 PM


8.4 Chapter 8

Face-to-Face Learning The Education Ministry has also taken measures such
It refers to learning that occurs in a traditional classroom as ‘Cashless Campus’ and Digital Financial Literacy of
setting where a faculty member ­delivers instructions to a community by students. Our National Education Policy
group of learners. This could include ­lectures, workshops, 2020 is targeting 50 per cent Gross Enrollment. Ratio
presentations, tutoring, conferences and so on. (GER) by 2035. The physical expansion of facilities
requires both infrastructural and human resource
Self-paced Learning limitations. The use of technology becomes imperative
It provides the flexibility to learn according to the avail- due to the factors such as being affordable, higher quality,
ability of learners’ own time and pace. It occurs in a inclusiveness, employability, skilling the unskilled (under
variety of ways, such as reading specific chapters from a Skill India Mission) etc.
­textbook, studying the course material presented through Now we can discuss various initiatives started by gov-
web-based or CD-based courses, attending pre-recorded ernment and other agencies to democratize the opportu-
classes or sessions, reading articles referred by the fac- nities of quality education:
ulty members, working on assignments and projects, and
1. National Mission on Education through Informa-
searching on and browsing the internet.
tion and Communication Technology (NMEICT):
Online Collaborative Learning The NMEICT is a ‘centrally sponsored scheme’ (CSS)
to realize the potential of ICT in teaching learning pro-
It involves interaction between the learners and the
cess for the benefit of all the ­learners in higher edu-
faculty members through the web. This interaction can
cation institutions anytime and anywhere. Content
occur synchronous and asynchronous interactions.
generation and connectivity along with provision for
Distance Learning access devices for institutions and learners are the
main objectives. Now many universities have been pro-
Here the students work on their own at home or at office
vided 1 Gbps connectivity and more than 14,000 col-
and communicate with the faculty and other students
leges have also been provided virtual private network
through email, electronic forums, video conferencing,
(VPN) connectivity.
chat rooms, instant messaging and other forms of
  A-VIEW software has been developed under the
computer-based communication. It is also known as
NMEICT for teacher training. This could become the
open learning. World Wide Web (internet), computer
basis for successful implementation of the proposed
based training (CBT), creation of ‘virtual classrooms’
National Mission on Teachers.
can help in the process.
  Under the N-list program of the Information and
The creation of ‘digital libraries’ are also created
Library Network (INFLIBNET) under the NMEICT,
with the help of ICTs where the students, teachers and
lakhs of e-books and thousands of high-quality paid
professionals can access research material and course
e-journals have been made available to colleges and
material from any place at any time. Such facilities
universities with a view to inculcate research cul-
allow the networking of academics and researchers,
ture in teachers and students. The model needs to be
hence such sharing leads to quality enhancement in
scaled up for maximizing coverage and for productive
teaching and learning.
usage of the available resources.
2. National Programme on Technology Enhanced
Stopover Learning (NPTEL): NPTEL is an initiative by seven
Read the following two statements: Indian Institutes of Technology (IIT Bombay, Delhi,
Guwahati, Kanpur, Kharagpur, Madras and Roorkee)
I: Information and Communication Technology and Indian Institute of Science (IISc) for creating
(ICT) is considered a subset of Information Tech- course contents in engineering and science. NPTEL
nology (IT). provides E-learning through online Web and Video
II: The ‘right to use’ a piece of software is termed courses in Engineering, Science and Humanities
copyright. streams. NOC stands for NPTEL Online Certification.
Which of the above statement(s) is/are correct? The main benefits of participating in an online
(a) Both I and II course under NPTEL are:
(b) Neither I nor II
(c) II only
(a) Students: credit transfer and better resume
(d) I only
(b) Faculty: Refresher courses, AICTE recognized
FDP courses
The correct option is (b).
(c) Working professionals: For upskilling and
reskilling
Digital and O ther Initiatives in Higher
3. SWAYAM: It is an indigenous (Made in India) IT
Education MOOCs platform for providing best quality education
We have already discussed MOOC, SWAYAM, SWAYAM that can be accessed by anyone, anytime and anywhere
PRABHA, National Digital Library (NDL) etc. in Unit 1. using the IT system. It was launched by the Government

M08_MADAN 04_65901_C08.indd 4 23/12/22 7:38 PM


Information and Communication Technology (ICT) 8.5

of India to achieve the three cardinal principles of edu- 9. e-Yantra: This is a robotics outreach program funded
cation—access, equity and quality. (This topic is dealt by the Ministry of Education and hosted at IIT
with in detail in Chapter 1.) Bombay. The main objective is to harness the talent of
4. SWAYAM PRABHA—the 34 educational direct- young engineers to solve problems using technology
to-home (DTH) channels: The SWAYAM PRABHA across a variety of domains such as: agriculture, man-
has been conceived as the project for telecasting high- ufacturing, defense, home, smart-city maintenance
quality educational programmes through 34 DTH and service industries.
channels on 24×7 basis. Every day, there will be new   e-Yantra creates projects that aims to train teachers.
content of at least 4 hours which would be repeated The Ministry of Education sees the core skills devel-
six times a day, allowing students to choose the time oped by the IITB. All the projects and code are avail-
of their convenience. Earlier, the number of channels able on the e-Yantra website www.eyantra.org as open
was 32. source content.
5. National Digital Library (NDL): A project titled 10. Campus Connectivity: Establishment of 1 Gbps con-
‘Development of National Digital Library of India, nectivity to universities and 512 kbps broadband con-
Towards Building a National Asset’ has been sanctioned nectivity to 20 colleges has been provisioned under
to IIT, Kharagpur under the NMEICT by the Ministry the NMEICT.
of Education. A Repository integrates contents from   On guidelines from ‘Digital India’ initiative of the
different Indian Institutional Repositories. There PMO, the Ministry of Education has decided that
are more than 60 types of learning resources are university campuses (having 1 Gbps bandwidth) shall
available–books, articles, manuscripts, video lectures, be made Wi-Fi-enabled campuses. All IITs, IIMs, and
thesis, etc. NDL is designed to hold content of any NITs have established Wi-Fi campuses. The process
language and provides interface support for 10 most of laying the optical fibre and provision of Wi-Fi in
widely used Indian languages. central universities is currently underway.
  There is development of design and development of 11. Talk to a Teacher: Talk to a Teacher, developed by
‘OAI-PMH’ Server for Metadata Harvesting, Indexed, IIT Bombay, is an initiative of the National Mission
etc. on Education through ICT. It has been funded by the
6. National Academic Depository (NAD): It is an ini- Ministry of Education to provide free access to a selected
tiative of the Ministry of Education to facilitate digital few graduate and postgraduate courses, taught at IIT
issuance, storage, access and verification of academic Bombay largely by distinguished faculty members and
awards issued by academic institutions. NAD is a scholars. It uses A-View collaboration tool developed
unique, innovative and progressive initiative under by Amrita University for providing virtual classrooms
‘Digital India’ theme towards achieving digital enable- to the faculty across the country. These courses can be
ment of the education records. It aspires to make the viewed absolutely free of charge at lower bandwidths
vision of Digital Academic Certificates a reality for on a personal computer (PC)/laptop that has a head-
every Indian. phone and internet connection.
7. e-Shodh Sindhu: This is again sponsored by UGC- 12. e-Acharya: e-Acharya is an ‘integrated e-content’
INFONET Digital Library Consortium, NLIST and portal developed under NME-ICT. The portal provides
INDEST-AICTE Consortium. This will provide current facility to search and browse the learners all learning
as well as archival access to more than 10,000 core materials includes audio, video, textual materials, etc.
and peer-reviewed journals, and also bibliographic, through a single interface. The portal cover quality
citations and factual databases in different disciplines learning resources from top institutions in the country
from a large number of publishers and aggregators in eight subject categories viz Agriculture Science,
to its member institutions including centrally-funded Biological Science etc. Contents are mostly provided by
technical institutions, universities and colleges that are the NCERT.
covered under 12(B) and 2(f) Sections of the UGC Act. 13. e-Kalpa and e-Yantra: It is combined project of
INFLIBNET, Gandhinagar is the implementing agency. Minsitry of Education and NMEICT initiative. It creates
8. Virtual labs: The physical distances and lack of other Digital-Learning Environment for Design in India.
resources make us unable to perform experiments,
(a) Digital online content for learning design with
especially when they involve sophisticated instru-
e-learning programs on design
ments. Good teachers are always a scarce resource.
(b) Digital Design Resource Database, including the
Web-based and video-based courses address the issue
craft sector
of teaching to some extent. VLs project addresses the
(c) Social networking for higher learning with col-
following:
laborative learning space for design

(a) Access to online labs to those engineering col- (d) Design inputs for products of National the
leges that lack these lab facilities. Mission in Education through ICT

(b) Access to online labs as a complementary facility 14. Free and Open Source Software for Education
to those colleges that already have labs. (FOSSEE): This project sanctioned to IIT Bombay has
Training and skill-set augmentation through
(c)  been promoting the use of open source software in
workshops and on-site/ online training. educational institutions (http://fossee.in). It does this

M08_MADAN 04_65901_C08.indd 5 23/12/22 7:38 PM


8.6 Chapter 8

through instructional material, such as spoken tutori- 24. Private Players: An increasing number of private
als, documentation, such as textbook companions, players, such as Hughes Global Education, Manipal
awareness programmes, such as conferences, training Education Group, Centum Learning, UEI Global and
workshops and internships. The Textbook Companion Shiv Nadar University, on are offering online educa-
(TBC) is a collection of code for solved examples of tion courses in association with leading central and
standard textbooks. Scilab and Python TBCs are also on state universities that have good ICT infrastructure.
the cloud. 25. Digitization of Books (e-text books): This helps
15. e-VIDWAN: ‘The INFLIBNET’ Centre took the initia- in the creation of a digital repository of books that
tive called ‘VIDWAN: Expert Database and National creates a digital learning environment for students.
Researcher Network’ with financial support from The digital version of the books embedded with
the NMEICT. The main objective is deal with many text and pictures along with video, simulations and
research and knowledge issues in a collaborative man- visualizations helps students learn the concepts in
ner by working with scientists, faculty and research an interactive way. NMEICT has plans to generate
scientists. new online course contents for UG, PG and doctoral
16. Central Cloud Infrastructure: The MHRD under education.
the NMEICT has awarded a project to IIT Delhi to set 26. Content Delivery Using IT/ICT: Higher educa-
up a robust 24×7-backed data centre, and the activi- tion is purely a content-driven play, in which educa-
ties have been put up at the National Informatics tional content is delivered through innovative use of
Centre (NIC)/National Knowledge Network (NKN) ICT. There is an increased trend in higher education
Data Centre; the cloud is called ‘Baadal’. The IIT institutes to render content through radio, TV and
Delhi cloud is hosting e-content and video content of satellite.
e-Acharya. 27. Open Education Resources: Many Indian universi-
17. Lekhika 2007: It was developed by the Centre for the ties are contemplating technology-enabled free access
Development of Advanced Computing (C-DAC) under to educational resources. The All India Council for
India’s IT Ministry and Israel’s FTK Technologies. The Technical Education-Indian National Digital Library
objective of this project is to spread computer literacy in Engineering and Technology (AICTE-INDEST) is
to the masses in India who do not know English. a consortium set up by the Ministry of Education to
18. Campus Wide Networking: This can be linked with enhance greater access and generate annual savings
https://ernet.in/. The main objective is to integrate in access of bibliographic databases. The UGC has also
ICT into the learning environment. The focus is to launched its Digital Library Consortium to provide
improve the quality of education and laying a good access to peer-reviewed journals and bibliographic
foundation at the higher learning institution and databases covering subjects such as arts, humanities,
encourage the institution and its affiliated colleges technology and sciences. The AICTE-INDEST and
to share the resources, knowledge content, promo- Digital Library Consortium can be called e-journal
tion and implementation of e-governance, faculty consortia.
development and exchange of skills. 28. Virtual Technical University (VTU): The National
19. National Knowledge Network (NKN) and Connected Mission on Education through ICT is working hard to
Digital: An initiative has been undertaken to cover establish a VTU to impart training to UG/PG students
1000 institutions besides providing digital campuses, along with new teachers. It focuses on science, technol-
video-conference classrooms, wireless hotspots and ogy, management and other related areas.
laptops/desktops to all students of professional/sci- 29. Gyan Darshan: Launched in 2000, Gyan Darshan
ence courses and Wi-Fi connectivity in hostels. is a joint effort of IGNOU and the IITs. It is a bouquet
20. Centre for Distance Engineering Education of channels that broadcasts educational programmes
Program (CDEEP): The CDEEP is an emulated class- for schoolkids, university students and adults. The
room interaction programme that uses real-time courses are contributed by IGNOU, UGC Consortium
interactive satellite technology. This was launched by for Educational Communication (UGC-CEC), IITs and
IIT-Bombay. so on. CEC is an inter-university centre of the UGC.
21. EDUSAT: The launch of EDUSAT brought satel- 30. Gyan Vani: It is a bouquet of frequency modulation
lite connectivity to large parts of rural India. Indira (FM) radio channels that broadcasts programmes
Gandhi National Open University (IGNOU) is lever- contributed by institutions such as IGNOU and the
aging satellite, TV and internet technologies to offer IITs. Under the UGC Countrywide Classroom initia-
online courses. tive, education programmes are telecast on Gyan
22. IIMs: IIM-C, IIM-B, IIM-K, XLRI and other management Darshan and Doordarshan’s National Channel (DD1)
institutes have started offering courses (in association every day.
with private players such as Hughes Global Education, 31. e-Gyankosh: It is a knowledge repository launched by
Reliance, NIIT) after the Distance Education Council IGNOU in 2005 which aims at storing and preserving
(DEC) allowed them to do so in 2007. digital learning resources. Almost 95% of IGNOU’s
23. Brihaspati: This open source e-learning platform has printed material has been digitized and uploaded on
been developed by IIT-Kanpur. the repository.

M08_MADAN 04_65901_C08.indd 6 23/12/22 7:38 PM


Information and Communication Technology (ICT) 8.7

32. Education and Research Network (ERNET): It is pro- The key challenges in our higher education system are as
moted by the Department of Information Technology. follows:
It provides communication infrastructure and services
1. Lack of desired level of knowledge and technology
to academic research institutions in India. It has under-
readiness
taken networking projects, such as AICTE-Net, Indian
2. Poor implementation
Council of Agricultural Research (ICAR)-Net and UGC-
3. Linguistic barriers
Infonet to provide internet and intranet facilities.
33. Sakshat Portal: Launched in 2006, Sakshat is a one-
stop education portal for addressing all the educa- Stopover
tion and learning-related needs of students, scholars, 1. Virtual education refers to imparting instructions in a
teachers and lifelong learners. It has been developed learning environment where the teacher and the stu-
at IGNOU. The portal was developed by the NIC and dents are separated by
provides links to vast knowledge resources, educa- (a) Time only
tional news, examination alerts, sample papers and (b) Space only
other useful links that are available on the web. It has (c) Both (a) and (b)
an in-built repository of educational resources and (d) None of the above
online testing facility.
34. GRID GARUDA: It is India’s first national grid bring- The correct option is (c).
ing together the academic, scientific and research 2. Which of the following can be termed the digital
communities for developing their data and other library for education?
applications. It is connected with the NKN. (a) Swayam
35. Shruti-Drishti: It is basically created for the visually (b) MOOC
impaired women’s empowerment (VIWE). (c) NPTEL
36. Consortium for Educational Communi­ cation: (d) Sakshat
It has been tasked with the creation of e-content for 87 The correct option is (c).
undergraduate courses. The UGC has cleared a proposal
3. Consider the following statements:
to publish e-content for 77 postgraduate courses (PGC).
I: The initiative is a collaboration of the scientific,
37. DIKSHA: This platform offers engaging learning
engineering and academic community to carry
material, relevant to the prescribed school curriculum,
out research and experimentation on a nation-
to teachers, students and parents. We can download
wide grid of computational nodes.
the DIKSHA app and scan QR codes in our textbooks
II: It is a mass storage that aims to provide the
for easy access to all of our lessons. This can be linked
distributed data and compute intensive high-
with NCERT also, called as Diksha-NCERT.
performance computing solutions for the 21st
38. E-Pathshala: This has been developed by NCERT
century.
for showcasing and disseminating all educational
III: 
The Department of Information Technology has
e-resources including textbooks, audio, video, peri-
funded the Center for Development of Advanced
odicals and a variety of other print and non-print
Computing to deploy the nationwide computational
materials through website and mobile app.
grid.
39. Samagra Shiksha, a PRABANDH System: This is
IV: The availability of this efficient, high-speed
meant to leverage technology to enhance efficiency
(multi-gigabit) network makes the backbone of
and manage the implementation of newly launched
National Knowledge Network.
Centrally Sponsored Integrated Scheme for School
Education. This has many programmes for girl edu- Which of the following programmes is being referred
cation. It wants to bridge gender and social category through these statements?
gaps at school level. (a) SWAYAM
40. Vidya Lakshmi: This is a first of its kind portal for (b) GRID GARUDA
students seeking Education Loan. This portal has (c) MOOC
been developed under the guidance of Department of (d) NPTEL
Financial Services (Ministry of Finance), Department The correct option is (b).
of Higher Education (Ministry of Education) and
Indian Banks Association (IBA).
41. Education and Research Network (ERNET): This Social Learning–Web 1.0, Web 2.0 and Web 3.0
is an autonomous scientific society of the MeITY. The technological learning of society helps in making
ERNET has made an important contribution to the better citizens and its economy do better. The
emergence of networking in the country. This practi- understanding of Web 1.0, Web 2.0 and Web 3.0 gives us
cally brought the Internet to India. This has built up the idea how technological flows have taken flow in the
national capabilities in the area of net-working, espe- past two to three decades.
cially in protocol software engineering. It is the first Web 1.0: This refers to the first stage of the World Wide
internet service in India. Web evolution. The following were the main features:

M08_MADAN 04_65901_C08.indd 7 23/12/22 7:38 PM


8.8 Chapter 8

1. Static pages Under Web 3.0, we can speak about ‘metaverse’


2. Content served from the server’s file system technology in future. Metaverse forms a bridge
3. Pages built using Server Side Includes or Common between the real and virtual worlds. The metaverse
Gateway Interface (CGI) is a concept of a persistent, online, 3D universe that
4. Frames and Tables used to position and align the ele- combines multiple different virtual spaces. To them,
ments on a page Internet is the only reality.
Web 2.0: This refers to world wide website that high- The Main Differences between Web 1.0, Web 2.0 and
lights user-generated content, usability and interoper- Web 3.0:
ability for end users. It is also called participative social
web. The web browser technologies are used in Web Web 1.0 Web 2.0 Web 3.0
2.0 development and they include AJAX and JavaScript
frameworks. Mostly read-only Wildly Portable and
With time, Learning Management Systems (LMS) read-write personal
such as Blackboard, Sakai, Moodle or Web Course Tools Company focus Community Individual focus
(WebCT) have become course-centred and teacher- focus
driven. Mobile phones have become more integrated with
learning. Home pages Blogs/Wikis Live-streams/
Five major features of Web 2.0 are as follows: Waves
1. Free sorting of information, with permission to Owning content Sharing Consolidating
retrieve and classify the information collectively content content
2. Dynamic contents and a participative society.
3. Information flows by means of evaluation and online Web forms Web Smart
commenting applications applications
4. Developed APIs to allow self-usage
Directories Tagging User behaviour
5. Shift to wider variety of users
The eight major tools are–Podcasting, Blogging, Tagging, Page views Cost per click User
Curating with RSS, Social bookmarking, Social network- engagement
ing, Social media, Web content voting. Banner Interactive Behavioural
Web 3.0: This refers to the evolution of web utilization
Advertising Advertising Advertising
and interaction which includes altering the web into a
database. It enables the upgradation of back end of the Britannica online Wikipedia The Semantic
web. In this, data is not owned but instead shared, where Web
services show different views for the same web/the same
data. HTML/portals XML/RSS RDF/RDFS/OWL
The following are the unique features for Web 3.0.
1. Semantic Web: It helps to create, share and connect Stopover
content through search and analysis based on the
capability to comprehend the meaning of words. 1. Consider the following statements
2. Artificial Intelligence: They become more intelligent Assertion (A): The Semantic Web is a collaborative
to fulfil the requirements of users. movement led by international standards body the
3. 3D graphics: Museum guides, computer games, World Wide Web Consortium.
ecommerce, geospatial contexts, etc. Reason (R): The Semantic Web provides a com-
4. Connectivity: The user experience evolves to another mon framework that allows data to be shared and
level of connectivity that leverages all the available reused across application, enterprise, and community
information. boundaries.
5. Ubiquity: Content is accessible by multiple applica- (a) A is correct and R is the correct explanation.
tions; every device is connected to the web; the ser- (b) A is correct but R is not the correct explanation.
vices can be used everywhere. (c) A is not correct but R is the correct statement.
6. Blockchain and Metaverse: There have been a ques- (d) Both A and R are incorrect.
tion on blockchain in NET JRF exam. Under Web 3.0,
we all know about bitcoin, a cryptocurrency that is The correct answer is (a).
fungible. That is based on blockchain. Linked with 2. Which of the following is a characteristic of Web 2.0
it is the Non-Fungible Tokens (NFTs) that represents applications?
a unique asset like a piece of art, digital content or (a) Multiple users schedule their time to use Web 2.0
media. Apart from bitcoin, the other examples of applications one by one.
cryptocurrencies are ethereum, hyperledger, doge- (b) Web 2.0 applications are focused on the ability of
coin, shiba inu. people to collaborate and share information online.

M08_MADAN 04_65901_C08.indd 8 23/12/22 7:38 PM


Information and Communication Technology (ICT) 8.9

(c) Web 2.0 applications provide users with content According to Function
rather than facilitating users to create it. Computers are of the following types according to their
(d) Web 2.0 applications use only static pages. function.
The correct option is (b).
Analogue Computers
Analogue computers are those in which data varies con-
Basic Computer Terms tinuously, that is, the movement of data is continuous.
These are generally meant to measure physical variables,
A computer is a programmable machine. The two prin- such as voltage, pressure, temperature and speed. These
cipal characteristics of a computer are: It responds to are mainly used for communication and broadcast trans-
a specific set of instructions in a well-defined manner mission.
and it can execute a pre-recorded list of instructions If instead of computer types, we take the basis of ‘sig-
(a ­program). nals’, the examples are thermometer, old radio, mega-
Modern computers are electronic and digital. The phone, old landline telephone sets, old stereo systems.
actual machinery—wires, transistors and circuits —is When we increase the sound of stereo system, that
called hardware; the instructions and data are called increases slowly and gradually. They were mostly used
software. before 1995.
The characteristics of computers that have made
them so powerful and universally useful are speed, Digital Computers
accuracy, diligence, versatility and storage capacity.
The hardware and software components have been Digital computers are those computers in which data flows
discussed after the types of computers. in a discrete form. These are high-speed ­programmable
electronic devices that perform mathematical calcula-
Classification of Computers tions, compare values and store the results. These use
Computers are classified according to (i) ­ generation, binary number system in which there are only two dig-
(ii) function, and (iii) size and configuration (Figure 8.1). its, 0 and 1 (each is called a bit). The digital computer
is designed using digital circuits in which there are two
According to Generation levels for an input or output signal. These two levels are
Generation refers to the time period during which a com- known as logic 0 and logic 1. Digital computers provide
puter was developed. The different generations of com- more accurate and faster results. The origin of this can be
puter are given in Table 8.2. traced to the ‘Morse code’.
Digital computers are better suited for solving complex
problems in science, engineering and technology. Hence,
they are increasingly used in the field of design, research
Classification of computers and data processing.
Based on their purpose, digital computers can be fur-
ther classified as follows:
Function Size and Generation 1. General-purpose computers
­configuration 2. Special-purpose computers
General-purpose computers are used for any type of appli-
First cations. Most computers are general-purpose computers.
Analogue Super
generation A special-purpose computer is one that is built for a spe-
cific application.
Digital Mainframe Second
generation Hybrid Computers
A hybrid computer combines the desirable features of
Hybrid Mini frame
Third analogue and digital computers. It combines the speed
generation of analogue computers and the accuracy of digital com-
Workstations puters. Now, analogue-to-digital and digital-to-analogue
converters are used for transforming data into suitable
Fourth form for either type of computation.
generation For example, in a hospital’s intensive care unit
Micro
(ICU), analogue devices might measure a patient’s
Fifth temperature and blood pressure. These analogue mea-
generation surements may then be converted into numbers and
supplied to digital components in the system for better
Figure 8.1 Types of Computers According monitoring. Hybrid computers are mainly used for spe-
to Operations cialized tasks.

M08_MADAN 04_65901_C08.indd 9 23/12/22 7:38 PM


8.10 Chapter 8

Table 8.2 Computer Generation: Comparison

Generation Device Hardware Features Characteristics System Names

First • Vacuum tubes • Support machine language • ENIAC


(1942–59) • Punch cards only • EDVAC
• Very costly • TBM 701
• Generate lot of heat
• Huge size
• Consumed lot of electricity

Second • Transistors • Batch operating system • Honeywell 400


(1959–65) • Magnetic tapes • Faster, smaller and reliable • CDC 1604
than previous generation • IBM 7030
• Costly

Third • ICs • Time sharing OS • IBM 360/370


(1965–75) • Large-capacity disk and • Faster, smaller, reliable • CDC 6600
magnetic tapes and cheaper • PDP 8/11
• Easier to update

Fourth • ICs with VLSI technology • Multiprocessing and GUI OS • Apple II


(1975–88) • Semiconductor memory • Object-oriented programs • VAX 9000
• Magnetic tapes and floppy • Small, affordable, easy • CRAY 1/2
as portable to use
• Easier to update

Fifth • ICs with ULSI technology • Powerful, cheaper, reliable, • IBM


(1988 to present) • Large-capacity hard disk easy to use, portable • Pentium
with RAID support • Rapid software • PARAM
• Optical disks as portable development possible
read-only storage media
• Powerful servers, internet,
cluster computing

According to Size and Configuration 8000 was the first super computer in the serirs and
There are four different types of computers which are PARAM Pravega is the latest one developed in January,
classified based on their size and configuration. These are 2022.
discussed in the subsequent paragraphs. Mainframe Computers
Supercomputers Mainframe computers can also process data at very high
speeds (in million instructions per second, shortly termed
They are mostly used for applications that require inten-
‘MIPS’). They are large-sized, powerful, multi-user
sive numerical computations, such as stock analysis,
­computers that can support concurrent programs. They
weather forecasting, nuclear energy research, electronic
can accommodate more than 1000 workstations simul-
design and analysing geological data. They can process
taneously. Normally, they are used in banking, airlines,
billions of instructions per second. The best known super-
railways and so on for their applications.
computer manufacturer is Cray Research. Some of the
The technique that allows many people at the termi-
companies that produce supercomputers are Cray, IBM
nals to access the same computer concurrently is called
and HP. PARAM is India’s first supercomputer.
time sharing.
PARAM Supercomputers: PARAM is a series of super-
computers designed and assembled by the Centre for Minicomputers
Development of Advanced Computing (C-DAC) under Minicomputers have lesser speed and storage capacity in
Ministry of Electronics and Information Technology comparison to mainframe computers. Hence, their per-
(MeITY). PARAM means “supreme” in the Sanskrit lan- formance is also less than that of mainframes. They are
guage, whilst also creating an acronym for “PARAllel mid-sized multiprocessing computers. They can perform
Machine. The C-DAC is headquartered in Pune. PARAM several actions at the same time and can support 4–200

M08_MADAN 04_65901_C08.indd 10 23/12/22 7:38 PM


Information and Communication Technology (ICT) 8.11

users simultaneously. Some of the features of main- of a computer system can be classified into CPU and
frames are not available in minicomputers. peripherals.
In recent years, the distinction between minicomputers
and small mainframes has blurred. Often the distinction Central Processing Unit (CPU)
depends on how the manufacturer wants to market his/ CPU is a microprocessor that controls the execution of
her machine. program instructions (microprocessor is programmable
while integrated circuits or ICs perform predetermined
Workstations tasks only). The CPU performs the following functions:
Workstations are powerful, single-user computers. They 1. Arithmetic operations
have the capacity to store and process large quantities of 2. Logical operations
data, but they can be used only by one person at a time. 3. Input/output operations
They are typically linked together to form a computer 4. Data manipulations
network called a local area network (LAN), in which
several people, such as staff in an office, can communi- Main CPU Components
cate with each other and share electronic files and data. To carry out the functions as mentioned earlier, the CPU
In terms of computing power, workstations lie between has the following components (Figure 8.2):
personal computers and minicomputers. Workstations
commonly support applications that require relatively 1. Arithmetic Logic Unit (ALU): It performs various
high-quality graphics capabilities and a lot of memory, calculations, such as addition, subtraction, division,
such as desktop publishing, software development and multiplication and comparison.
engineering applications. 2. Control Unit (CU): It coordinates the operation
A workstation is similar to a personal computer but is of hardware, the flow and execution of data and
more powerful and often comes with a higher quality instructions that are fed into memory or main storage
monitor. through CPU.
3. Memory Unit (main storage): It holds data instruc-
Microcomputers tions, intermediate results and final results that are
A microcomputer is also called a PC. It is a small and ready for output. The data and instructions are passed
relatively inexpensive computer, commonly consist- from the main store into ALU or to and from the storage
ing of a display screen, a keyboard, a central process- device under the control of CU.
ing unit (CPU), one or more disk drives and a printer,   CPU Scheduling: This is required to complete job on
with limited storage and based on a microprocessor. It is time. This CPU process allows one process to use the
designed for an individual user. CPU while other process is on standby mode (delayed)
The invention of microprocessor (single-chip CPU) due to unavailability of any resources such as I / O etc.
gave birth to much cheaper microcomputers. They are Thus, we are able to make full use of the CPU. This
further classified into the following processors: makes the system more efficient, faster, and fairer. This
can be done on the basis of ‘First In First Out (FIFO)’,
1. Desktop computers ‘Last In First Out’ (LIFO), ‘Shortest Job First’ (SJF). In
2. Laptop computers Round Robin concept, the algorithm uses ‘time sharing’.
3. Handheld computers (PDAs)   Now, computers hold megabytes or even gigabytes
of data. A megabyte is a unit of 1 ­million bytes, a giga-
Computer Hardware byte is 1 billion bytes and a terabyte is 1 trillion bytes.
It is the physical equipment required to create, use, If a computer has a memory of 64 megabytes, then it
manipulate and store electronic data. The hardware can hold 64 million bytes of information.

Central processing unit

Arithmetic
logic unit
Input unit Control unit Output unit

Main
memory

Secondary
storage

Figure 8.2 Main CPU Components

M08_MADAN 04_65901_C08.indd 11 23/12/22 7:38 PM


8.12 Chapter 8

Set of Registers Storage Devices


Within  a CPU,  there are a number of high-speed, special- Computer data storage, often known as memory, refers
purpose memory units called registers. These registers to the computer components and recording media which
carry out critical functions in the execution of programs. retain the digital data. Memory refers to the temporary
They are used for storing small information. internal storage areas within a computer. The term mem-
ory is usually used as a shorthand for physical memory
Motherboard and refers to the actual chips ­capable of holding data.
The computer’s main circuit board contains the CPU, the Some computers also use virtual memory, which expands
memory and expansion slots for additional circuit boards physical memory onto a hard drive.
known as adapters or cards.
Buses Size of the Memory
They are a set of parallel wires for connecting the CPU of a Memory is the part of a computer which stores informa-
computer to all other input–output devices. The data can tion. A computer has different types of memories that bal-
be transmitted in two directions, from and to the CPU. ance the functions of speed and cost (Figure 8.3):
1. Cache memory: It is the volatile computer memory
Peripheral Devices which is closest to the CPU memory. The access time
A peripheral device is any device that can be connected of data is less than that of main memory. All the
to a computer. This term includes monitors, keyboards, recent instructions are stored into the cache memory.
mouse, webcams, drawing pads, joystick, modems, print- It is the fastest memory that provides high-speed data
ers, scanners, interactive whiteboards, drive wheel and access to a computer microprocessor. The capacity of
so on. cache memory is too low in comparison to memory
Random Access Memory (RAM) and hard disk. Now
I nput Devices microprocessors contain the cache memory on the
Any device that is capable of inputting information into chip itself, earlier that was not the case.
a computer system, for example, keyboard, microphone, 2. Random Access Memory (RAM): It is also termed
mouse, scanner and webcam, is known as an input the main memory. It is the temporary memory (vola-
device. tile) that allows information to be stored randomly
and accessed quickly and directly (without the need
O utput Devices to go through intervening data).
The output devices are basically meant for anything that   A computer can both write data into and read data
comes out of a computer after being processed. They from RAM. Every time a user turns on his/her com-
include the monitor, headphone, printers, speakers, plot- puter, a set of operating instructions is copied from
ters and visual display units (VDUs). the hard disk into RAM. These instructions, which

Bytes Internal
CPU
Registers Memory
Volatile
Cache
kB/MB Memory
Faster (SRAM)
access Primary
GB Main Memory (RAM)
Higher (DRAM) Memory
capacity and
economical GB USB/Flash Memory
(EEPROM) Secondary
Storage
TB
Magnetic Disk/Hard Disk
Non-volatile

PB/EB Tertiary
Magnetic Tapes/Tape drives
Storage

Figure 8.3 Memory Hierarchy

M08_MADAN 04_65901_C08.indd 12 23/12/22 7:38 PM


Information and Communication Technology (ICT) 8.13

help control the basic computer functions, remain in (a) Primary Storage: It consists of RAM and ROM. In
RAM until the computer is turned off. As soon as the primary memory, data is not stored permanently but
power is turned off, whatever data is present in the stored temporarily. After accessing the i­nformation,
RAM disappears. the data is erased from the memory of the computer.
3. Read-Only Memory (ROM): It is the permanent Such types of data are considered short-term memory.
memory of a computer. A set of instructions is built (b) Secondary Storage: This ‘auxiliary’ storage is the slowest
into the computer by the manufacturer and cannot be and the cheapest form of memory. It cannot be processed
accessed or changed by the user. It permits the user directly by the CPU and must be copied first into primary
to only read data. Computers always contain a small storage (RAM). The main secondary memory devices
amount of ROM that holds instructions for starting up are magnetic disks, such as hard drives and floppy disks,
the computer. optical disks such as CDs and CD-ROMs, and magnetic
4. Programmable Read-Only Memory (PROM): tapes, and they were actually the first forms of second-
A PROM is a memory chip in which you can store a ary memory. In computer disk storage, any sector is a
program. Once the PROM has been used, you cannot subdivision of a track on a magnetic disk or optical disk.
wipe it clean and use it to store something else. Like Each sector stores a fixed amount of user-accessible data,
ROMs, PROMs are non-volatile. traditionally consisting of 512 bytes for hard drives and
5. Erasable Programmable Read-Only Memory 2048 bytes for CD-ROMs and DVD-ROMs.
(EPROM): An EPROM is a special type of PROM that Memory can be of following types as well:
can be erased by exposing it to ultraviolet light.
6. Electrically Erasable Programmable Read-only 1. Volatile memory is the type of computer memory that
Memory (EEPROM): An EEPROM is a special type of is temporary in nature. It stores the data inside it only
PROM that can be erased by exposing it to an electri- until the power is supplied.
cal charge. It is also termed flash memory. 2. Non-volatile memory is the type of computer memory
that is permanent in nature. The data stored in such
kind of memory remains there even after the system
Stopover is turned off.
Which of the following correctly lists computer mem-
The storage devices can be divided into two
ory types from highest to lowest speed? categories:
(a) Secondary Storage; Main Memory (RAM); Cache
Memory; CPU Registers   (i) Internal: They reside within a computer. Hard disk is
(b) 
CPU Registers; Cache Memory; Secondary an example of a magnetic disk in which the computer
Storage; Main Memory (RAM) data can be stored. They usually hold more data and
(c) 
CPU Registers; Cache Memory; Main Memory are conventionally faster.
(RAM); Secondary Storage (ii) External: They are portable and are usually used as
(d) 
Cache Memory; CPU Registers; Main Memory removable storage devices. USB flash drive (com-
(RAM); Secondary Storage monly termed pen drive), CD, digital video disk
The correct option is (c). (DVD) and Blu-ray Disc are certain examples.
USB flash drives look like a small flat pen. They may have
large storage capacity and can be plugged into any com-
Memory is also classified as (a) primary storage and puter with a USB socket. Flash drives are also commonly
(b) secondary storage(Figure 8.4). referred to as pen drives or memory sticks.

MEMORY
MEMORY

Primary (Volatile Memory) Secondary (Non-volatile memory)

HDD
FDD
ROM RAM
DVD
PROM
Pen drive
EPROM
EEPROM SRAM DRAM

Figure 8.4 Types of Memory

M08_MADAN 04_65901_C08.indd 13 23/12/22 7:38 PM


8.14 Chapter 8

Hard Device Disk and Solid State Drive (HDDs and When software is stored in a hardware that cannot be
SDDs): As buyers we look for optimum utilization of easily modified (such as BIOS, ROM in a PC), it is called a
resources. Both SSDs and HDDs are storage devices, we firmware, which indicates that it falls into an area some-
invented them long time ago, but their ‘pricing’ aspect led where between the software and the hardware.
to late adoption of SSD. Technologically SSD is preferred
over HDD. System Software
HDD: HDD consists of a series of platters covered by a fer- System software provides the basic functions for com-
romagnetic coating. The system is basically ‘mechanical’, puter usage and helps to run the computer hardware and
the hard disk is the slowest and most ‘fragile’ component system.
of any computer. It is ‘more economical’ in comparison to It includes a combination of devices, drivers, oper-
SSD. It is best for storing extra data, such as movies, pho- ating systems (OSs), servers and utilities. It manages
tos and documents. the hardware devices. It is also responsible for resource
allocation. The OS and utility software are the two
SSDs: are newer types of disks that store information on major categories of system software.
‘flash memory’, which consists of individual memory cells
storing bits that are instantly accessible by the control- Types of System Software
ler. SDD is faster, it may have shorter lifespan, it is more
expensive, it is shock resistant. SSD is best for storing 1. Compilers: They convert the source code (written
operating systems, gaming apps and frequently used files. in a programming language) into another computer
language called object code. The examples are ALGOL
Booting: This is the process of starting a computer. This compilers, BASIC compilers, C++ compilers and
may be started by hardware (such as by pressing a but- FORTRAN compilers.
ton) or simply by a software command. When a system is 2. Operating System (OS): An OS controls all other
switched on, a CPU has no software in its main memory, resources of a computer system. It can be 16-bit,
so some processes must load software into memory before 32-bit or 64-bit.
execution. This may be done by hardware / firmware in
CPU. This may also be done by a separate processor in the The main categories of operating system are as follows:
computer system. (a) Unix/BSD: Unix system V, AIX, HP-UX, Solaris, IRIX,
Rebooting: This is basically restarting a computer that BSD Distros
can be done in two ways. The ‘hard’ rebooting is done (b) GNU/Linux: Linux, Google Chrome OS
after electrical power to the CPU is switched from off to (c) Windows: 2000, XP, Vista, 7, CE
on. The “soft” means where the power is not cut. (d) Mac OS: Mac OS X
(e) Embedded and real-time OS
Stopover (f) Experimental: Oberon/Bluebottle, Amoeba, Plan 9
(Bell Labs)
1. The process of copying files to a CD-ROM is known as
(a) Burning Windows was developed in 1985 as an add-on to
(b) Zipping MS-DOS in response to the growing interest in graphical
(c) Digitizing user interfaces (GUIs). Mac OS was the previous OS that
(d) Ripping used a GUI. The most recent client version of Windows
The correct option is (a). after Vista is Windows 10.
2. Consider the following statements: Linux was developed by Linus Torvalds. It is a freely
1. Use of flash memory to deliver superior perfor- available multi-tasking and multi-user OS. Since the
mance and durability very beginning, Linux was placed under General Public
2. Without moving parts, are more durable, run License (GPL).
cooler and use less energy Utility Software
3. Can be thought of as large USB drives
It is a system software designed to help, analyse, monitor,
Which of the technology is being referred in this case? configure and optimize settings and maintain the com-
(a) Hard Disk Drive puter. Skins that customize the media player’s looks and
(b) Solid State Drive DVD burners are examples of utility software.
(c) Non-flash Drive Some utility software that is frequently used is classi-
(d) None of the above fied as follows:
The correct option is (b).
1. Antivirus Software: Antivirus is a protective s­ oftware
designed to defend a computer against malicious soft-
Computer Software ware (malware).
Software is a set of programs, procedures, algorithms and 2. Desktop Widgets: Widgets are virtual tools that pro-
associated documents that provide instructions to a com- vide single-purpose vehicles showing the latest news,
puter by ordering what to do, when to do and how to do. time, calendar, dictionary, map, calculator and so on.

M08_MADAN 04_65901_C08.indd 14 23/12/22 7:38 PM


Information and Communication Technology (ICT) 8.15

3. Backup Software: The backup software are backup readable. For example, assembly language state-
files, cleaning up hard disk and shredding files. ments such as MOV and ADD are more recognizable
Examples are Windows Backup and Restore Centre than sequences of 0’s and 1’s.
and Net Backup by Symantec. 3. High-level Language (HLL): HLL are what most
programmers use. Languages such as C++ and Java
Application Software are HLLs. One advantage of HLLs is that they are
It helps the user perform singular or multiple related very ­readable. The statements in these languages
tasks. It can be further classified into the following: are in English. Programs written in HLLs are trans-
lated into assembly language or machine language
1. Productivity Software: Examples are word proces- by a compiler. Assembly ­language programs are
sors, spread sheets, schedulers, DBMS, desktop pub- translated into machine language by a program
lishing, ERP, Mozilla Firefox called an assembler.
2. Collaborative software: These are also referred to
as groupware software. The examples are Facebook Every CPU has its own unique machine language.
application and Etherpad. Programs must be rewritten or recompiled to run on dif-
ferent types of computers.
Software can be either proprietary (also called closed) First-and second-generation computers are identified
or open. Proprietary software is privately owned and as ‘low-level languages’ and third-fourth-and fifth-gener-
controlled. The specifications of such software are not ation computers as ‘HLL’.
divulged in order to prevent other companies from dupli-
cating it.
The users usually prefer using open software, which is Program
publicly accessible. Anyone can create add-on products A program is a set of instructions for performing a par-
for it because one can understand how it was designed. ticular task. These instructions are just like English words.
Freeware is a software that can be copied and used with- The computer interprets the instructions as 1’s and 0’s. A
out payment to the author(s), although there may be program can be written in assembly language as well as in
some restrictions on distribution. HLL. This written program is called the source program.
The source program is to be ­converted into machine lan-
guage, which is called an object program. A translator is
Stopover required for such translation.
Which of the following is an open source software?

(a) MS Word Language Processors
(b) Windows 1. Assembler: This language processor converts the
(c) Mozilla Firefox program written in assembly language into machine
(d) Acrobat Reader language.
The correct option is (c). 2. Interpreter: This language processor converts a HLL
program into machine language by converting it line-
by-line. If there is any error in any line, then it reports
Levels of P rogramming Languages it at the same time, and program execution cannot
These are the rules, conventions and specific commands resume until the error is rectified.
that are used to write a computer program. Most programs 3. Compiler: It also converts the HLL program into
must be converted into machine language or binary code machine language. It converts the entire HLL pro-
so that the instructions can be performed on a specific gram in one go and reports all the errors of the pro-
computer platform. gram along with the line numbers. After all the errors
are removed, the program is recompiled and after
1. Machine Language: This is the first generation that, the compiler is not needed in the memory as the
and also the only language that is understood by a object program is available.
computer. Each statement in a machine language
program is a sequence of bits. Each bit may be set to 0
or 1. While easily understood by ­computers, machine
languages are the most difficult for humans to put to How Does a Computer Process
use. They are entirely compiled by binary numbers. Information?
Programmers, therefore, use either a high-level
programming language or an assembly language. When data is entered into a computer, the numbers or
2. Assembly Language: This is known as second- words we understand are translated into a binary number
generation language. It is also called as ‘symbolic system (Figure 8.5). Binary language is the language of
machine code’. It contains the same instructions as computers.
machine language, but the instructions and variables Everything you type, input, output, send, retrieve,
have names instead of just numbers. The advantage draw and so on, in the end, is converted to the computer’s
of assembly language is that its instructions are native language, that is binary.

M08_MADAN 04_65901_C08.indd 15 23/12/22 7:38 PM


8.16 Chapter 8

Binary N umber System The binary system uses only two symbols, 0 and 1, to
Formally, we all know that base-10 system is a positional represent values.
system. Computers are built from transistors and an individual
In decimal notation, we write a number as a string of transistor can only be ON or OFF (two options).
symbols, where each symbol is one of these 10 digits, and Thus, the values in a binary system, such as 0’s and
to interpret a decimal number, we multiply each digit by 1’s, are called binary digits or bits. Computers use base 2
the power of 10 associated with that digit’s position. because they can recognize only two values, 1 or 0.

Given a decimal number, convert it to binary

Decimal number Binary number

Given a binary number, convert it to decimal

Figure 8.5

For example, consider a decimal number ‘6249’. The unit position of this number is illustrated as shown in Figure 8.6.

6 2 4 9 = 6 ⋅ 103 + 2 ⋅ 102 + 4 ⋅ 101 + 9 ⋅ 100

103 102 101 100


position position position position
(thousand’s position) (hundred’s position) (ten’s position) (one’s position)

Figure 8.6 Conversion of Binary into Decimal and Vice versa

We can consider binomial number 11012 and see how this can changed into decimal number (Figure 8.7).

1 1 0 1 = 1 ⋅ 23 + 1 ⋅ 22 + 0 ⋅ 21 + 1 ⋅ 20 = 1310

23 22 21 20
position position position position
(eight’s position) (four’s position) (two’s position) (one’s position)

Figure 8.7 Conversion of Binary into Decimal and Vice versa

Example 1 We are solving this first question in a detailed manner.


Find the decimal value of 1110012 Step 1: Perform the successive MOD operation by 2
for the given decimal number 294 and note down the
Binary number 1 1 1 0 0 1 remainder (either 0 or 1) for each operation. The last
Power of 2: 25
2 4
23
22 21
20 remainder is the most significant bit (MSB) and the
first remainder is the least significant bit (LSB).
We can put the solution in the following form also:
1110012 = 1 × 25 + 1 × 24 + 1 × 23 + 0 × 22 + 0 × 21 + 294/2 = 147: Remainder is 0 → LSB
1 × 20 = 5710 147/2 = 73: Remainder is 1
73/2 = 36: Remainder is 1
Example 2 36/2 = 18: Remainder is 0
Convert the decimal number 29410 into its binary equiv- 18/2 = 9: Remainder is 0
alent. 9/2 = 4: Remainder is 1

M08_MADAN 04_65901_C08.indd 16 23/12/22 7:38 PM


Information and Communication Technology (ICT) 8.17

4/2 = 2: Remainder is 0 4. Hexadecimal Number System: Base of this number


1/2 = 0: Remainder is 1 → MSB system is 16, that is, 0 1 2 3 4 5 6 7 8 9 A B C D E F.
Step 2: Write the remainders from MSB to LSB, providing
the equivalent binary number: Decimal Number Binary Number
29410 = 101001102 0 0000
Example 3 1 0001
Convert 17410 to its binary equivalent.
2 0010
Division Quotient Remainder 3 0011
174/2 87 0 4 0100
87/2 43 1 5 0101
43/2 21 1
6 0110
21/2 10 1
7 0111
10/2 5 0
5/2 2 1 8 1000

2/2 1 0 9 1001
1/2 0 1 10 1010
11 1011
So, 17410 = 101011102 12 1100
Example 4
13 1101
Convert 1310 to its binary equivalent.
14 1110
Division by 2 Quotient Remainder
15 1111
13/2 6 1
6/2 3 0 Binary Digit (Bit)
3/2 1 1 It is a digit within the binary number system. A bit is the
1/2 0 1 smallest unit of information held in a computer. In order
to make the bits useful, they are combined into bytes of
Therefore, 1310 = 11012 information.

Byte
Stopover It is a combination of bits that represents one character.
If the binary equivalent of the decimal number 48 is A byte is usually composed of 8 bits.
110000, then the binary equivalent of the decimal
number 51 is given by American Standard Code for Information
(a) 110011 Interexchange (ASCII)
(b) 110010 It assigns a specific pattern of (ASCII) to each char-
(c) 110001 acter as described earlier. Another code that may be
(d) 110100 found, especially in IBM brand mainframe computers,
The correct option is (a). is Extended Binary Coded Decimal Interchange Code
Hint: Here the binary digits of 3 can be directly added (EBCDIC).
to that of 48.
Extended Binary Coded Decimal
Interchange Code
N umber Systems in a nutshell
The important point to remember about these codes is that
1. Binary Number System: It has only base 2, that is, 0 their main value is to store information so that it is read-
and 1. able by other computers. By using ASCII or EBCDIC, it is
2. Octal Number System: Base of octal is 8, that is, 0, 1, possible for people to retrieve and use someone else’s data
2, 3, 4, 5, 6, 7. using a different type of hardware or software. The main
3. Decimal Number System: Base of decimal is 10, that disadvantage of using ASCII is that formatting or other
is, 0 1 2 3 4 5 6 7 8 9. special qualities of computer-ized information may be lost.

M08_MADAN 04_65901_C08.indd 17 23/12/22 7:38 PM


8.18 Chapter 8

A Ready Reckoner of Conversion Table


Traditional Units Decimal for Comparison
Name Symbol Binary Equal to Name IEC Decimal
Kilobyte kB 210 1024 B Kilobit kbit 103
Megabyte MB 220 1024 kB Megabit Mbit 106
Gigabyte GB 230 1024 MB Gigabit Gbit 109
Terabyte TB 240 1024 GB Terabit Tbit 1012
Petabyte PB 250 1024 TB Petabit Pbit 1015
Exabyte EB 260 1024 PB Exabit Ebit 1018
Zettabyte ZB 270 1024 EB Zettabit Zbit 1021
Zotabyte YB 280 1024 ZB Zotabit Ybit 1024

in some larger network (e.g. the internet) instead of


Computer Networks ­physical wires.
1. Network: A network is a system of interconnected
computers. There are a number of types of computer Intranet
networks. Intranet is a private network. It is created only for the
2. Local Area Networks (LANs): LANs use cable to con- employees of one company to improve their commu-
nect a number of computers within the same location nication. An intranet is an ideal way to communicate
or in close proximity. in a secure environment. It is a collection of resources
3. Wide Area Networks (WANs): WANs use telephone to which only internal users have access. Intranets are
lines or other telecommunication devices to link com- often separated from the internet by using a firewall.
puters in widely separated locations. They help to overcome some of the limitations of exist-
4. Wireless Local Area Network (WLAN): WLAN links ing LANs and WANs.
two or more devices using some wireless distribution An intranet operates on a common technology. While
method and provides connection through an access an intranet allows all kinds of internet connections,
point to a wider internet. from Telnet to File Transfer Protocol (FTP) to Gopher, it
5. Personal Area Network (PAN): PAN is a computer is die web that is most commonly used on an intranet.
network used for communication among computers The web and its browsers offer a common interface that
and different IT devices close to one person. is comfortable and well known to most of an intranet’s
6. Metropolitan Area Network (MAN): MAN is a net- intended users.
work of computers located at different sites within a Intranet as technology enabler brings the resources
large fixed area, such as a city. to the users in a specific domain of interest. The ways to
7. Campus Area Network (CAN): CAN is a network of authenticate users are as follows:
multiple interconnected LANs in a limited geographi-
cal area. It is smaller than a WAN or MAN. 1. Server permissions-based authentication
8. Storage Area Network (SAN): SAN is known as 2. Database-based authentication
server area network. 3. Firewall
9. Internet: The internet is a system that links existing 4. IP-based authentication or some combination of the
networks into a worldwide network. above
10. Intranet: It is a private network inside a company or 5. Database(s)—Access, SQL Server, MySQL
an educational organization and used over its LAN. 6. XML
This can be taken as akin to local internet and it is 7. Hypertext Markup Language (HTML) or XHTML-
under a single administrative set-up. based forms
11. Extranet: It is a technology that permits users of an 8. Web scripting (ASP, PHP, ColdFusion)
organization’s intranet to enter portions of another
organization’s intranet in order to conduct business A basic knowledge of computers is very useful here.
transactions or collaborate on joint projects. Basic intranet is usually made up of a simple shared folder
12. Virtual Private Network (VPN): It is a computer net- connected to a network. All files in the folder will be vis-
work in which some of the links between the nodes ible to other intranet members. It depends on their per-
are carried out by open connections or virtual circuits mission levels also.

M08_MADAN 04_65901_C08.indd 18 23/12/22 7:38 PM


Information and Communication Technology (ICT) 8.19

Intranets offer several facilities that aid knowledge 1. Simple Virus: This virus targets the program file,
sharing and they are as follows: when we run that program then it gets executed itself.
This virus automatically attaches itself to other pro-
1. Easy to access and use: The use of WWW browsers
grams.
give a low-cost and easy-to-use interface to informa-
2. Encrypted Virus: An encrypted virus is a computer
tion and applications.
malware that has become a serious threat to global
2. Universal access to information: The intranet mar- businesses over the past half-decade. An encrypted
ket has expanded to smaller businesses as a result virus is defined as a computer virus/malware that
of less cost. is capable of encrypting its payload in order to
3. Person-to-person interaction: It is mainly through make its detection difficult, i.e. can create a state of
electronic mail and computer conferencing. suspicion.
4. Informal networks: Publishing information and mak- 3. Polymorphic Virus: The evolution of this virus
ing contact is quick and informal on an intranet. infects the ‘sign in’ of the virus, making it more diffi-
An extranet is conceptually the same thing, but tech- cult for the antivirus to detect the virus. This virus can
nically, it is used when employees are located in differ- only be detected by special codes written specifically
ent places and cannot access the same local server. for polymorphic viruses.
4. Macro Virus: It is a mini-program in its name that
runs inside other programs. These virus scripts are
specially written to infect computers and can spread
Concept Box rapidly.
5. Trojan Horse: A Trojan horse resides in a disguised
Topology: The physical topology of a network refers or embedded form in a common software program.
to the configuration of cables, computers and other With that program installed, it gets installed on your
peripherals or simply as arrangement of computers. computer as well. It is also destructive. It is not actu-
Here, the workstation or node refers to computer sys- ally a virus, as it cannot replicate itself, but there is a
tems which are in the network. The topology can be possibility that a Trojan horse may have a virus pro-
classified as follows: gram hidden in it.
1. Bus Topology: Here, every computer is connected 6. Bombs: It is very similar to Trojan horse. It also has
to a main (single) cable, such as a bus. a feature that this logic bomb can include a timing
2. Star Topology: There is a hub or switch box to device, thus it will go off only on a particular date and
which all workstations are directly connected. time.
3. Ring Topology: Here, every device has exactly two 7. Worms: The worms themselves are a kind of copycat,
neighbours. it can spread to many computers connected in a net-
4. Mesh Topology: Every computer is connected to work. You need software with antivirus and firewall
another, so messages can take any path. to protect your system from worms.
8. Boot Sector Computer Virus: It damages the BOOT
5. Tree Topology: It is a combination of two or more
record on digital storage equipment, which includes
star networks.
the computer’s hard disk and all peripheral disks used
on them, including DVDs, CDs, and floppy disks in
general. These viruses can be loaded onto the com-
puter at any time, but they become active after the
Computer Virus BOOT record has been used by the computer. Once
A computer virus is a malicious software program that is also activated, it is a very difficult task to remove them. At
called as ‘malware’. This computer code or program is capa- present the redundancy of floppy disks has reduced
ble of affecting computer data in an adverse manner. It may the presence of boot sector viruses.
destroy computer programs. The virus has the tendency to 9. Master Boot Record Computer Virus: These
make its duplicate copies at a very fast pace. The virus spreads viruses are special variants of boot sector viruses, in
across every folder and damage the data of our computer sys- that they mainly affect the master boot record of the
tem. The infected computer programs may include data files, computer.
or even the “boot” sector of the hard drive. The virus is mostly 10. File Infector Computer Virus: File Infector Viruses
detected by seeing through the functionality of our computer exist on files that are known as executable files, such
system. The computer affected by a virus does not take com- as extensions and .com. Most of these viruses attach
mand properly. themselves to files, and download those files to other
users’ computers.
types Of virUs wOrms 11. Macro-computer virus: These viruses exist on mac-
This computer program replicates itself at a swift pace. ros, which are data files, and are generally associated
But it is not exactly like a computer virus. It is self-con- with MS Office programs. Because they target the
tained and hence does not need to be part of another pro- software rather than the system, macro viruses can
gram to propagate itself. infect any operating system.

M08_MADAN 04_65901_C08.indd 19 23/12/22 7:38 PM


8.20 Chapter 8

The Entry of Virus in System: The virus may affect our Basic Mechanism for I nternet W orking
system in the following ways. Internet uses the standard Transmission Control Protocol/
1. Through downloading files from the Internet. Internet Protocol (TCP/IP).
2. Through removable media or drives. We need the following two items for sending and
3. Through pen drive. receiving information:
4. Through mail attachments. 1. Address for communication
5. Through unpatched software and services. 2. A secure means for moving data in the form of elec-
6. Through unprotected or poor administrator pass- tronic signals
words.
We need protocols for sending and receiving files.
There are basically two protocols for the purpose—TCP
I mpact of Virus and IP.
1. It disrupts the normal functionality of respective com- Role Played by TCP in Internet: TCP divides the data
puter system. into data packets for the purpose of sending and receiving
2. It modifies configuration setting of the system data. In addition, the rules for reassembling data and
3. It disrupts system network use. damage-free delivery are also specified.
4. It destroys the confidential data.
5. It disrupts our computer network resources. Role of IP: The IP puts destination—address­ing infor-
mation on such packets. The address can be typically
The antivirus software in our system can easily check written as user@host.domain. The user is the name of
programs and files on a system for virus signatures. the internet account holder.
The name must be the same which is used by the user
Virus Preventive Measures while logging in. The hosts are individual machines at a
1. Through installation of an effective antivirus software. particular location. Resources of the host machine are
2. Patching up the operating system and client software. normally shared and used by any internet user.
3. We need to put highly secured Passwords. Every host and router on the internet has an IP address.
4. The use of firewalls also helps. An IP address consists of four sets of numbers that are
separated by dots. These addresses are organized from
The Internet Basics left to right.
Internet is defined as an information superhighway, to Three points are important regarding IP address.
access information over the web. 1. IP address is a unique set of four numbers (such as
• The concept of internet was originated in 1969. 110.22.33.114) which identifies a computer location.
• The concept of internet was generated in Advanced We need to see that all parts are a number from 0 to
Research Project Agency Network (ARPANET) that 255. The lowest IP address is 0.0.0.0 and the highest
was developed by the U.S. Department of Defense. is 255.255.255.255.
Then its main objective was to provide communication 2. A special computer Domain Name Server (DNS) is
among the government bodies. used to give name to the IP address so that user can
locate a computer by a name.
3. A portion of a number separated by a dot is known as
Benefits an octet. Thus, IP addresses have four octets that are
Internet is the ‘network of networks’. It covers almost equal to 32 bits. Computers translate word addresses
every aspect of life. or person-understandable addresses into numeric
1. Easy communication with the people sitting at remote equivalents.
locations, through social media platforms such as Face-
book, Instagram, Twitter, WhatsApp, Pinterest etc. IPv4, IPv6 and IPv12
2. Surf for any kind of information, and also for The designers of TCP/IP defined an IP address as a 32-bit
entertainment. number, a system still in use today known as Internet
3. Accessing many services such as banking, matrimo- Protocol Version 4, or IPv4. Later, due to the widespread
nial services, online shopping, online ticket booking, development of the Internet and the resulting lack of
online bill payment, data sharing and email. available addresses, a new identification system (IPv6)
4. Internet provides the concept of electronic commerce was developed in 1995 using 128 bits for the address.
(e-commerce). It allows the business deals to be The main function of a network protocol is to transfer
conducted data from one computer to another in a packet network
5. For conducting research. such as the Internet. IPv12 can be called the ‘protocol of
6. Work From Home during pandemics such as covid19. the future’.
7. Mapping services such as Street Views (that may be User Datagram Protocol (UDP) can also be used in
allowed in India in year 2022 itself ) on electronic place of TCP. IP and UDP can also be called connection-
systems. less protocols.

M08_MADAN 04_65901_C08.indd 20 23/12/22 7:38 PM


Information and Communication Technology (ICT) 8.21

Smart Phone

iPad
I.S.P. Ph
on
Internet Ca e Wi-Fi
ble

Router

Wi-Fi Wired

COMPUTER
Web Server
Website 1
Website 2 Webpage 1 Web Browser
Website 3 Webpage 2
Website 4 Webpage 3
etc. Webpage 4
etc.

Figure 8.8 Internet Basic Elements

The network number is assigned by Network 7. Telnet: It is a protocol that allows a computer to con­
Information Centre to avoid the possibility of any conflict. nect with a remote host computer on the inter­net. The
The web servers placed over internet are registered to use of Telnet has diminished as the web’s HTTP has
a domain. One web server can serve multiple virtual web become a dominant protocol.
servers, where each virtual server is mapped or registered 8. Hypertext Markup Language (HTML): It is a for­
to different or unique names. matting language which is used to establish the appear-
All the servers/systems on the network communicate ance of a web page. Thus, it helps in cre­ating pages on
with each other through a particular path, which is called the WWW. HTML also enables us to insert hypertext
a gateway. links within one web page or to other pages.
Other Important Internet Protocols Open Systems Interconnection (OSI): This model
In addition to TCP/IP which are the essence of internet, describes the seven layers that computer systems use to
the following protocols are also very important. communicate over a network. This is basically the first
standard model for network communications, adopted by
1. Hypertext Transfer Protocol (HTTP): It defines the all major computer and telecommunication companies in
format for communication between web browsers the early 1980s.
(web clients) and web servers.
2. Internet Message Access Protocol (IMAP): It Who Governs the I nternet ?
defines the format for communication between email The Internet was developed by Bob Kahn and Vint Cerf in
servers and clients. the 1970s. Packet switching was thought of as a better and
3. Secure Sockets Layer (SSL): It is a protocol devel­ faster method to transfer data than the hardware solution
oped by Netscape for transmitting private docu­ments to the problem. Internet has no president or chief operat-
through the internet. ing officer, and it is governed by a number of authorities.
4. File Transfer Protocol (FTP): It is a set of guidelines or The ultimate authority of internet rests with the Internet
standards that establish the format in which files can Society (ISOC), a voluntary membership organization
be transmitted from one computer to another. with an objective to promote global interchange of infor-
5. Trivial File Transfer Protocol (TFTP): This pro­ mation. Another authority is a group of invited volunteers
tocol allows transferring of files between the net­work called the Internet Architecture Board (IAB). The IAB sets
devices. standard and provides internet addresses. The Internet
6. Simple Mail Transfer Protocol (SMTP): Network- Engineering Task Force (IETF) discusses the technical
based email was initially exchanged on the ARPANET and operational problems in internet.
in extensions to the FTP, but it is now carried by the No one pays to use the internet, but everyone pays for
SMTP. its part.

M08_MADAN 04_65901_C08.indd 21 23/12/22 7:38 PM


8.22 Chapter 8

Main Applications of Internet 3. Digital subscriber line (DSL): It works over plain old
When the internet started, it had five main applications telephone service (POTS), that is, copper telephone
and they are listed as follows: lines. DSL has two types:
(i) Asymmetric digital subscriber line (ADSL) pro-
1. Emails 2. News vides different bandwidths in the upstream and
3. Remote login 4. File transfer downstream directions.
5. Research (ii)  Symmetric digital subscriber line (SDSL) pro-
Hardware Requirements for the Internet vides the same bandwidth in both the directions.
4. Satellite connection: It is akin to receiving TV ­signals
1. Modem (Modulator and Demodulator): A modem directly from the satellite.
is a device or program which converts computer data 5. Bluetooth: Bluetooth is an open wireless technology
to a signal that can be transmitted over a telephone for data exchange over short distances. It is a proto-
line. It can also reconvert a signal coming into a com- col that permits wireless exchange of information
puter through a telephone line so that it can be under- between computers, cell phones and other electronic
stood by the computer. Modems are used to connect devices within a radius of about 30 ft.
computers with the internet. 6. Broadband: Broadband refers to various high-capac-
2. Hub: A hub is a place of convergence where data arrives ity transmission technologies to transmit data, voice,
from one or more directions and is forwarded out in one and video across long distances and at high speeds.
or more directions. A hub is a common connection point The main mediums to transmit data are coaxial cables,
for networked computers and other devices. Hubs are fiber optic cables, and radio waves. Broadband is
used to connect devices in a LAN. always connected and removes the need for dial-up.
3. Bridge: A bridge is a network device that connects two In September 2021, Telecom Regulatory Authority of
or more networks, such as one LAN to another LAN India (TRAI) has proposed that a minimum of 2Mbps
that uses the same protocol (Ethernet or token ring). speed is allowed in broadband connections in India.
4. Router: This is actually a ‘networking layer system’   Fibre optic internet is also a broadband connection
that is used to manage and forward data packets to that can reach upto speeds of 940 mbps. This uses fibre
computer networks. Data packets are sent through optic cable. It has low lag time.
the e-mail and web pages. The packet’s destination or 7. WiFi: This is basically Wireless fidelity. It is also known
target IP address is determined by a router. A router as high-frequency wireless local area network (LAN).
is linked to two or more data networks from different They transmit and receive data at a local level. Many
IP addresses. A router sets the shortest path for the hotels, educational institutions, railways and airports
data packets to travel from source to destination. offer Wi-Fi access to people using laptops. Wireless
5. Gateway: A gateway is essentially a system used to access points support different IEEE standards. This
communicate between networks with different pro- can be in Wireless LAN, MAN, PAN, WAN etc. WiMax is
tocols and are responsible for converting one proto- also wireless technology that uses spectrum to deliver
col into another. A gateway is a node in a computer connection to network. WiMax is used to provide inter-
network that serves as an entrance to another net- net services such as Mobile Data and ‘hotspots’.
work. The hardware examples are connector, cables, 8. Leased Line: A leased line is a dedicated data connec-
adopters, circuits, switches etc. tion with a fixed bandwidth. This enables oragnisa-
tions to connect to internet in a secure, reliable, and
I nternet Service Provider efficient manner. They will have maximum download
A company that provides a subscription service to enable capacity and discipline. They are symmetrical, uncon-
the user or organization to access the internet is an ISP. tended (can not be shared), and they are point to point.
An ISP has a network of computers permanently linked
to the internet. Dedicated lines are also provided by some
ISPs. ISPs also give you an email address and space on the Other Important Internet -related T erms
WWW for setting up one’s own website. World Wide Web (WWW or W3)
Main Internet Connection Options WWW is a hypertext system that uses the Internet in order
To be specific, the following technological options are to access specific locations and content stored there. It is
available to us for accessing the internet: basically between clients (browsers) and servers, HTTP
and HTTPS (Hypertext Transport Protocols) control the
1. Dial-up connection: It uses an analogue telephone transmission of data through accessing locations. It is
line for establishing a temporary communication. It actually a subset of Internet. WWW and Internet are not
converts a computer’s digital signals into analogue sig- synonyms.
nals. The Integrated Services Digital Network (ISDN) Tim Berners-Lee is credited with the invention of the
is used for this purpose that is without a modem. It is World Wide Web. While working with CERN, he became
termed wired ether. interested in a new technology called ‘hypertext’. He can
2. Cable TV connections: Households with cable TVs be credited with three key technologies, HTML, URL, and
have the option of cable modem internet access. HTTP which are fundamental the web today. Nikola Tesla

M08_MADAN 04_65901_C08.indd 22 23/12/22 7:38 PM


Information and Communication Technology (ICT) 8.23

theorized a “world wireless system” over one hundred There are geographic names as well:
years ago. The concept of the World Wide Web, a global
“network of networks,” stretches back to the 1930s. The Country Domain Name
public service of WWW started in 1993.
WWW consists of documents called web pages that Australia .au
contain information on a particular topic. A web page China .cn
might also contain one or more links that point to other
web pages. A website is a location on the WWW. Germany .de

Web Browsers India .in


A web browser takes us anywhere on the internet. It acts Japan .jp
as an interface between the user and inner working of
United Kingdom .uk
internet (specifically WWW). It retrieves information
from other parts of the web. That information is dis- United States .us
played on our desktop or mobile device. The informa-
tion is transferred using the Hypertext Transfer Protocol
(HTTP). Thus, transmission of text, images and video is Domain N ame Server (DNS)
done on the web. Some popular browsers are enlisted as A DNS is a directory of domain names that align with IP
follows: addresses. When we try to connect to a website with a
1. Internet Explorer domain name such as puchd.ac.in (Punjab University,
2. Mozilla Firefox 3.5 Chandigarh), a request is first made to a name server to
3. Opera resolve this name into an IP address, which is then used to
4. Safari 4.0 locate the system where the website is stored. Then we can
establish a connection with it.
Cross-platform Browsers
Internet Protocol A ddress
They can operate on more than one platform, such
as Windows, Mac and Linux. SeaMonkey, Flock and It is the unique numerical address of a computer on
K-Meleon are different examples of cross-platform the internet and it is expressed as four sets of numbers
browsers. A browser can be (maximum three digits each) separated by dots, for
example, 150.237.176.24. Computers on the internet
(a) graphical or are nearly always referred to by more memorable domain
(b) text based names, which are mapped onto their IP addresses by special
The main examples of graphical browsers are Microsoft internet computers known as name servers.
Edge, Netscape Navigator, Mosaic and Opera. Lynx can be
taken as an example of a text-based browser. The services Home Page
of Microsoft Internet Explorer ended on June 15, 2022. The main page of a website is its home page. Typically, the
Bookmarks: The web browser can also store a list of home page serves as an index or table of contents to other
favourite sites, also called as bookmarks, to permit the documents stored at the site (i.e. the address). A home page
users to jump directly to the site they wish to see instead is similar to the title page and table of contents in a book. It
of having to enter its URL every time. identifies the site and contains links to other pages at the site
which is called Microsoft Explorer.
Domain Name Hyperlink
It is the unique name that identifies an internet site or An element in a hypertext document is highlighted by
website. Domain names always have two or more parts means of underlining or with the use of a different colour.
separated by dots. The part on the left is the most specific When the highlighted element is clicked, the user is con-
and the part on the right is the most general. A provided nected to another element in the same document or
machine may have more than one domain name but a another document.
given domain name points to only one machine.
Domain names are the alphabetical names which are Firewall
used to refer to computers on the internet. The suffix indi- A firewall is a software package that sits between the com-
cates what type of an organization is hosting the site. There puter and the internet connection, keeping an eye on the
are six main categories and they are as follows: traffic going to and fro. It also warns the user in case some-
1. .com: Commercial institutions or organization thing suspicious appears. Any computer is vulnerable when
2. .edu: Educational institutions connected to the internet.
3. .gov: Government sites
4. .mil: Military sites Search Engines and Meta Search Engines
5. .net: Gateways and administrative hosts This programme searches documents for specified
6. .org: Private organizations keywords and returns a list of documents where the

M08_MADAN 04_65901_C08.indd 23 23/12/22 7:38 PM


8.24 Chapter 8

keywords were found. Google (linked with chrome display the information server. These programs are
browser), Yahoo, Microsoft Bing, Baidu (Chinese origin), called
Yandex (Russian origin), DuckDuckGo, ask.com, AoL. (a) Internet Explorer
com, Internet Archive are the popular search engines. (b) HTML
(c) FTP
Meta Search Engine: This is a search tool that sends user (d) Web browsers
requests to several other search engines and/or databases
and aggregates the results into a single list or displays The correct option is (d).
them according to their source.
Wide Area Information Server (WAIS) is an internet
system in which specialized subject database are created at
multiple server locations, their tracks are kept. Email–Electronic Mail
The examples of meta search engines are All4one and
MetaCrawler. It is the transmission of text-based messages among net-
worked computers. Email is one of the earliest and most
basic messaging resources on the internet, and in many
Stopover ways, it still acts as the lowest common denominator for
computer communications (Figure 8.9).
1. The code for a web page is written using
The features of an email are as follows:
(a) Hypertext Markup language
(b) Perl 1. It is faster and more secure than conventional mail
(c) FTP that requires less physical effort to edit and send a let-
(d) URL ter of communication.
The correct option is (a). 2. The internet is free; it does not require the attention
of both parties at the same time.
2. Each computer connected to the internet must
3. It provides a time-stamped proof of an interaction.
(a) be an IBM PC
Also, many email services (such as Yahoo) collate
(b) be internet compatible
the conversation on the same subject into single
(c) have a modern connection
threads.
(d) have a unique IP address
4. It is easy to archive for future recall. Most of the email
The correct option is (d). services provide search facility through emails. An
3. The web standard allows programmers on many dif- email can be edited and rephrased as much as desired
ferent computer platforms to dispersed format and before sending it to the recipient(s).

From : Alex Diaz alex@tutorialspoint.com Wed Jun 22 11:00 EDT 2022


Date : Wed, 22 Jun 2022 11:00:46 EDT 2022-0400 (EDT)
E-mail
To : shiller@tutorialspoint.com
header
Subject : bean dip
Cc : wong@tutorialspoint.com
Bcc :

Greeting Hi Guys,

Someone accidently finished off the black bean dip last night. Can one
of you pick up another case of it on your way home? I think luke is on his bike
today, so you might have to Tak.
Text
--Alex
Body

******************************************************************************************

Alex T. Diaz office: (401)347 - 2345


332, Toast Lane fax : (401)347 - 0013
Signature East Providence,
Rhode Island 05893
******************************************************************************************
Figure 8.9 Sample email

M08_MADAN 04_65901_C08.indd 24 23/12/22 7:38 PM


Information and Communication Technology (ICT) 8.25

5. It is easy to send the same piece of information to 3. Subject: It contains a brief summary of the contents
several people simultaneously, such as circulation of of the message.
memos, agendas and minutes, or to disseminate edu- 4. Date: It contains the local time and date when the
cational material. message was originally sent.
An email address has the following three parts. 5. CC: It stands for carbon copy. It contains the email
address(es) of those who will receive a copy of the
Username
message in addition to the receiver(s) mentioned in
The first part of an email address is the username. This is
the ‘To’ field.
the unique name that you or your ISP selects. This can be
6. BCC (Blind Carbon Copy): Those people who are
your real name or a nickname.
able to receive the message, but the sender does not
@ Symbol want to inform about them to others.
An ‘at’, or ‘@’, symbol is the second part of an email 7. Message-ID: It shows the number assigned to the
address. This fits in between the username and the message by the mail program at the host machine.
domain of your email address. 8. A series of received: These are the lines, showing
Domain details of the systems through which the email has
The last part of an email address is the domain, which can passed (useful for troubleshooting if the mail bounces
be broken down into two portions: the mail server and the back).
top-level domain (TLD). 9. A reply-to: It gives the preferred address for replies
(usually, but not always, the same as the sender’s
Types of Email Services address).
The email services are usually categorized into the Blank spaces are not allowed in an email address. Also, an
­following three parts: email address is not case-sensitive.
1. Free Web-based Email Services are the commonly Body: The body of an email contains the message itself,
used email accounts accessible through web browser mostly in text form. An account may be configured to
(such as Internet Explorer, Firefox), and generally use automatically assign a signature (of the user) at the end.
HTTP. Almost all the major services provide secure login Signature is the text appearing at the end of the body by
using Hypertext Transfer Protocol Secure (HTTPS), default in each message. Normally, it is the name of the
which can be identified in the URL line while signing sender and other contact details.
into the account (https://gmail.com or https://mail. Features
yahoo.com).
2. Priced Web-based Email Services are free web-based 1. Email is based on push technology, that is, email is
email service providers, such as Yahoo! and Hotmail, that delivered to the recipients so they do not have to work
offer premium accounts on payment basis. The major to get it and just need to open their inbox to access the
advantages of more secured transaction of communica- email.
tion services are personalized email address, better spam 2. Most email clients offer to create Multipurpose
filtration, increased storage space, etc. Internet Mail Extensions (MIME) and HTML emails
with colourful fonts, graphics and links.
3. Private Email Services are used by the institutions
3. The email account can be set to remind an upcoming
having their own dedicated mail server and offer mail
event.
account for free to their staff and other members of
4. The accounts offer facility for spellcheck while writ-
the institution, for example, abc@ugc.ac.in.
ing an email.
Mail Merge: This is a useful tool that allows us to produce 5. Any kind of document including multimedia objects
multiple letters, labels, envelopes, name tags, and more can be sent through an email as attachments. However,
using information stored in a list, database, or spread- the maximum size of the object which can be attached
sheet. This is used on Word document. or received is fixed and set by the service providers.
6. An email collates threads of communication on a
Structure and F eatures of Email single subject. Therefore, it sometimes acts as time-
Internet email messages consist of two major sections, the stamped proof of communication. Gmail of Google
header and the body. offers this service.
7. A message can be saved or printed along with all com-
Header: The header of an email is structured into various munication details.
fields, such as summary, sender, receiver and other infor- 8. A list of contacts (along with other details, such as phone
mation about the email. It can be easily distinguished from number, fax) can be created within an account with an
the body of the email. Various fields included within the
ease to recall any nickname associated with each email
header are as follows:
address. Hence, the user only has to enter one word for
1. From: It contains the email address and, optionally, an email, instead of the full address.
the name of the sender. 9. Web-based email programs usually have a virus scan
2. To: It contains the email address(es) and, optionally, the function that scans attachments before they are sent
name(s) of the receiver(s) who receives the message. along with the main email.

M08_MADAN 04_65901_C08.indd 25 23/12/22 7:38 PM


8.26 Chapter 8

10. One common feature of all email programs is the use of Messaging
folders. These folders include an inbox, drafts folder, Messaging is a method of communication between two
sent items folder and deleted messages folder. The people or organizations. It could be done using the power
users can also create other folders to sort their email of internet or through cell phones. There are two types of
better. Filters are also included in email programs to messaging and they are explained as follows:
define certain words or phrases that the program will
look for in a message. The programs will then delete 1. Asynchronous Messaging: It is a method of communi-
the message, forward it to a specified address or put it cation between programs in which a program places a
in a particular folder. message on a message queue and leaves. It really does
not matter how the message will be delivered. It is the
Functioning of Email Systems delivery agent or the kind of infrastructure that ensures
1. Protocols: A protocol can be defined as a set of rules the delivery of message, even if the recipient is offline,
to perform a specific task. For example, IMAP is used for example, delivery of emails.
by the client that is used to read email, like Firebird, 2. Synchronous Messaging: In this kind of communica-
Outlook Express and Apple Mail (also known as Access tion, both the sender and the receiver have to be in con-
Client). Then there are TCP/IP, SMTP, HTTP, etc. nection while transferring the message, for example,
2. Delivery agent: The following parts have been telephonic conversation. Hence, a program places a
mentioned here: message in a message queue and then waits for a reply
to its message before continuing further.
Mail Transfer Agent Mail: Transfer Agent is a
(a) 
piece of software that transfers messages or mails Instant Messaging
from one host or machine to other. It is often
referred to as mail server. It is a form of real-time communication between two or
(b) Mail Delivery Agent: This is a computer soft- more people based on typed text or using audio or video.
ware used by the Mail Transfer Agent to deliver The message is conveyed via devices connected over a net-
email to a particular user’s mailbox. work. Most IM messages provide these features.
Access Client: It is an agent acting as a cli-
(c)  1. Instant messages/chat: These involve sending and
ent towards an email server to access an email receiving text/notes with an online friend.
account. It is a kind of application software. Some 2. Chat rooms: These are a common platform where
of the examples of Access Client are Outlook two or more than two people can communicate.
Express, Outlook or Thunderbird. 3. Files/web links/videos/images: These can be
shared over a network.
Starting an Email Account
4. Talk: Instead of a phone, the internet can be used to actu-
Starting an email account includes setting up an account ally talk with friends. Examples: Google+, Hangouts.
option also. Most web-based mail services offer four 5. Mobile capabilities: Instant messages can be sent to
standard email folders, the ‘Inbox’, ‘Sent’, ‘Drafts’ and mobile/cell phones.
‘Trash’. Each of them is described next.
Inbox: It enlists all emails received from other email Unified Messaging
accounts. It highlights the newly received/unread It is a combination of different media into one channel. A user
mails. can access information from different media using a single
device. Normally, unified messaging is common in mobile
Sent: Copies of messages sent are put into the ‘Sent’ communication, where voice, text and fax can be accessed
folder, provided that the mail account is set to save all the using one mailbox. It provides the power to reach people
sent messages. almost anywhere at any time and the flexibility to allow peo-
Drafts: It is a place for storing unfinished messages. If ple to control when they can be reached.
the writing of a message is not yet finished and needs to The major issues with messaging are spamming, pri-
be stopped in between, clicking the ‘Save’ button puts vacy and security. Spamming is a collection of unwanted
the ­message into the ‘Drafts’ folder, from where it can be mails which includes threats, promotional mails and so
accessed later on. on. These messages are called spam. There are two kinds
Trash: It stores an email that is deleted from other folders. of spams, mail lists and individual mails.
The messages are not truly deleted until they are deleted
from this folder. Widgets
A message can be opened by clicking its ‘subject’. Once Widgets are small cards that display dynamic content from
the mail gets opened, there are various options available your favorite apps and services on our Windows desktop.
at the top to delete, print or forward a message. The mes- They appear on the widgets board, where you can discover,
sages in ‘Trash’ folder that are more than few days old will pin, unpin, arrange, resize, and customize widgets to
automatically be deleted. reflect our interests. The widgets board is optimized to
Messaging can synchronous and asynchronous as we show relevant widgets and personalized content based on
discussed earlier. our usage. They are an offering during Web 2.0.

M08_MADAN 04_65901_C08.indd 26 23/12/22 7:38 PM


Information and Communication Technology (ICT) 8.27

E-mail hacking: Hacking is gaining of unauthorized At present, five types of e-commerce can be summa-
access to data in a system or computer. This can be done in rized here basically:
any of the following ways:
1. Business-to-Consumer (B2C) e-commerce
1. E-mail Spamming: This may happen whenever a 2. Business-to-Business (B2B) e-commerce
commercial organization sends mail to the e-mail 3. Consumer-to-Consumer (C2C) e-commerce
account holders without any request to advertise its 4. Peer-to-Peer (P2P) e-commerce
products and services. These are also called ‘junk 5. Mobile Commerce (M-commerce)
mail’. These are also called ‘unsolicited bulk emails’.
2. Virus: An e-mail file contains malicious script, when appLicatiOns Of e-cOmmerce
we run such file on our computer, there is a possibility E-commerce has various forms of applications:
of loss of important data.
3. Phishing: Phishing is a type of online-in scam that 1. Electronic Payments: This is the best form of payment
targets consumers by sending an e-mail that appears at present. Electronic payments can be done through
to be from a well-known source (ISP, bank, mortgage various ways, such as through electronic credit cards,
company etc). It asks the consumer to provide per- electronic cash, smart cards, electronic fund transfer
sonal identifying information. Its main purpose is to (EFT) and e-wallets and purchasing cards.
‘steal’ sensitive information such as user name, pass- 2. Banking Gateway: E-commerce plays a vital role in
word and credit card details, etc. This mail contains the banking sector for inter-bank transactions and
links that direct the e-mail user to a website that looks building a separate gateway for the unified banking
like the original website. gateway.
4. Splag: Splag is a ‘malintent’—that is harming others. 3. E-governance: Now, the Government of India
It means that a spammer ‘keep spamming’ someone has initiated total online transactions for tax pay-
until there is a lot of mess in his computer and the ment, phone bill payment, loan EMI payment from
computer crashes. banks, etc.

Stopover
Concept Box
1. Which of the following is a correct format of email
address? BHIM
(a) name@website@info Bharat Interface for Money (BHIM) is an initiative to
(b) name@website.info enable fast, secure, reliable cashless payments through
(c) www.nameofebsite.com your mobile phone. BHIM is interoperable with other
(d) name.website.com Unified Payments Interface (UPI) applications and
The correct option is (b). bank accounts for quick money transfers online. It
2. Which of the following is not a search engine? was developed by the National Payment Corporation
(a) Bing of India (NPCI) as a part of the Digital India initiative.
(b) Google BHIM is the only online payment app that you ought
(c) Yahoo to have.
(d) Windows
The correct option is (d). Security in Electronic Payment
3. Which of the following is a type of online chat that The security requirements needed for conducting e-com-
offers real-time text transmission over the internet? merce transactions are as follows:
(a) Asynchronous messaging
(b) Scheduled messaging 1. Authentication for both the parties
(c) Instant messaging 2. Integrity for unaltered transactions
(d) All of the above 3. Non-repudiation for unjustified denial of placing
The correct option is (c). orders
4. Privacy of identity to be secured
5. Safety for providing credit card number in the
internet
E-CoMMERCE
Electronic commerce (e-commerce) has changed the mObiLe a pps
lifestyle of society. With the help of e-commerce, it is To a common person, a mobile app is a function that
possible to buy, sell and exchange products, services simplifies some work just by a click of a button on his/
and information through computer networks, primarily her mobile. For some tech person, mobile apps are a
through the internet. Though the definition of computer-generated program or software applications
e-commerce is quite debatable, it is still very much useful which are built to run on various mobile devices, such as
for both individuals and corporates. iPhones, smartphones and tablets.

M08_MADAN 04_65901_C08.indd 27 23/12/22 7:38 PM


8.28 Chapter 8

Apps can be broadly classified into the following: security of the call. Call organizers provide the necessary
information to participants through emails to provide back-
1. Web ground data or perhaps to review documents in the call.
2. Mobile apps
Skype for Business includes the audio conferencing
Mobile apps can further be divided into native and feature for just this situation. People call into Skype for
hybrid apps. business meetings using a phone, instead of using the
Skype for Business app on a mobile device or PC. The
W eb Apps organizers need to set up audio conferencing for people
When an application is accessed using a web browser over who plan to schedule or lead meetings.
a network like an internet, it is termed web app. Web apps The following can be termed the advantages of audio
are functional and interactive. They need not to be down- conferencing:
loaded like mobile apps. They are loaded on browsers such
1. Audio conferencing is a cost-effective communication
as Chrome or Firefox. They do not consume memory or stor-
tool.
age space on the user’s device. People can easily write, using
cross-platform, standard web technologies such as HTML, 2. Audio conferencing is easily accessible.
CSS or JavaScript, a web app. 3. Audio conferencing can save you a lot of time.
A web app can be used on any mobile device that has a
Disadvantages of Audio Conferencing
web browser, be it iOS, Android or Windows.
On the flipside, device level features such as push The following are the three obvious disadvantages of
notification, work offline and load on the home screen audio conferencing and its possible solutions
are not supported. 1. The communication is only verbal.
Mobile Apps 2. Audio conferencing cannot keep you focused on the
meeting.
They are of two types:
3. The quality of a conference call is not reliable.
1. Native App: The native apps are created, designed
and coded for specific platforms, such as iOS, Android Audio conferencing has brought a lot of benefits to peo-
and Window phones. They are much more complex if ple, especially in the business world. When the technology
compared to web apps. Hence, one native app created grows to a certain degree, it comes across its bottleneck.
for one OS will not run on the other. Video-conferencing
  If development of native apps for iOS, Android and
It is a computer-based communications system that
Windows is required, then we need to create separate
allows a group of computer users at different locations to
apps. Unlike iOS devices, Android devices have the ‘back
conduct a ‘virtual conference’. Here, the participants can
button’ and hence, they do not need separate user inter-
see and hear each other as if they were in the same room
face element for this.
participating in a real conference.
  Native apps are released and distributed through an Video conferencing can occur between two partici-
‘App Store’. The user gets access to hundreds of apps pants in a peer-to-peer call or between multiple partici-
at a single location, which makes it easier to install pants via a video conferencing bridge, sometimes called
them. Each of these platforms has its own publication a multi-point control unit (MCU). Here, the bridge can be
procedure. located within a company network and can be made avail-
2. Hybrid Apps: As per the name itself, they are an amal- able from a service provider that can be on a subscription
gamation of native and web apps. They get installed or metered basis or free of cost. It may depend on the
as native app and feel like web apps. intended use and service model. Video conferencing also
  They are built on JavaScript, HTML or CSS and includes an audio channel and may include a document
run on a simplified browser within the app, termed or screen-sharing capability. It can also exist on a com-
web view. pany’s internal network, although when external partici-
pants join, they generally connect over the internet using
Audio Conferencing a specialized type of firewall.
A computer-based communications system allows a group Video conferencing can be divided into three types:
of computer users at different locations to conduct a ‘vir-
1. Hardware-based video conferencing
tual conference’. Here, the participants hear one another
2. Software-based video conferencing
as though they are in the same room participating in a real
3. Hybrid conferencing
conference. The audio conferencing systems do not allow
the participants to see one another. Video conferencing system consists of end points (including
In audio conferencing, multiple callers are allowed to main peripheral equipment for end points, such as cameras,
join a conversation by dialling into an audio conferencing microphones and other A/V processing equipment),
bridge. The participants are supplied with an access num- MCU (equals to a server) and network connection. When
ber, a conference ID and possibly a secure PIN number to a participant talks to the screen before us, our voice and
uniquely identify the participant. This also improves the image are going into the software in analogue form. The

M08_MADAN 04_65901_C08.indd 28 23/12/22 7:38 PM


Information and Communication Technology (ICT) 8.29

server will transform it to digital form when accepting the The ‘SMART Governance’ is basically about the
message we sent through the software and then transforms following:
it back into analogue form to the receiving end. This is the
1. Simple: There are no elaborate procedures, no paper
situation for point-to-point video call.
work and no need for frequenting government offices.
For multi-point video conference, it will be much more
2. Moral: The system is cleaned up of corruption and
complicated. With the help of scalable video coding tech-
other unethical practices.
nology, the server can send each participant a set of video
3. Accountable: Since all the information is in public
streams, so that every receiving end in different locations
domain, the government’s accountability enhances.
can receive the optimized voices and images.
4. Responsiveness: Through the use of ICT, the govern-
Teleconferencing ment can be in regular touch with the masses and get
It refers to a ‘virtual’ conference with participants in dif- their feedback.
ferent locations, via either telephone (audio conferenc- 5. Transparency: The information which was shrouded
ing) or video (video conferencing). The UNHCR uses a in secrecy within dark file chambers will become
number of teleconferencing tools, including Skype for available to all on the click of a mouse.
Business (Microsoft Lync) and Cisco WebEx. The following are the main components of e-governance:
ICT and Governance 1. Government–Citizen Interface
2. Government–Government Interface
In 1954, W. Howard Gammon wrote an e-government
3. Government–Business Interface
research paper on the use of ICT for providing good gov-
4. Government–Employee Interface
ernance. The internet, SMS and d ­ ifferent mobile apps help
5. Government–Society Interface
people to access i­nformation quickly. The data access has
become very cheap due to cost competitiveness among The rationale behind all is to achieve the basic princi-
companies. Now they are being used in education, banking ple of democracy, information sharing and greater citizen
services, railway and other g­ overnance issues. Government participation.
has started utilization of internet to serve common people The benefits of ICT-based governance are as follows:
through e-governance.
Among researchers, there is an issue on the correct use 1. Automation of Administrative Processes
of the two terms. While e-governance is the use of ICT to 2. Workforce Reduction
support in the administration or management of govern- 3. Better Service Delivery—through integration
ment, e-government is the use of ICT to provide services 4. Technical and Supportive Role
in maintaining government operations correctly. 5. Transparency
If governance is done through the use of ICT, it is said 6. Economic Development
to be e-governance. Through e-governance, the broader 7. Social Development
objective of paper-free and hassle-free government offices 8. Innovative Role
is achieved (Figure 8.10). 9. Change in the Administrative Culture

1. Achieving Visibility: The general public comes to The challenges faced in implementing E-governance
know about that organ of government. Most of the are as follows:
sites maintained by state provide certain basic infor- 1. Lack of Resources
mation and profile of that state. 2. Lack of Infrastructure
2. Online Access to Information in Public Domain: 3. Social Issues in the Regulation of Cyberspace
Knowledge of laws, rules and regulation reports of 4. Digital Divide
various commissions, etc., is being provided by gov- 5. Lack of Relevant Information in Local Languages
ernment on line now. 6. Building E-governance Capacity
3. Completing Transaction Online: Filing of tax returns, 7. Security Issues in Cyberspace
driving licences, passport, etc., is done online now. 8. Reluctant Bureaucracy
Thus, the ultimate objective of ‘good governance’ is
achieved. Basis Conventional ICT-based
Governance Governance
Manual Nature Secretive Transparent
Power structure Hierarchical Horizontal/
networked
ICT
Government Citizens Response Passive and Pro-active and quick
slow
Manual
Communication One way Direct/immediate
Emphasis Compliance Achieving targets
Figure 8.10 ICT’s Value in E-Governance

M08_MADAN 04_65901_C08.indd 29 23/12/22 7:38 PM


8.30 Chapter 8

In the recent past, the Government of India has taken Adware: Adware is a software that may have been installed
a number of initiatives to implement e-governance. The on your computer by a remote computer, that is, through
e-sampark centres have been set up in Chandigarh. The the web.
Information Technology Act, 2000, provides the necessary
legal framework for e-governance. Though the Act deals AJAX (Combination of Asynchronous JavaScript and
with a variety of issues related to cyber world, one of its XML): It is a web programming tool (or rather a set of
objectives is to promote e-governance by providing legal tools) which makes it possible to create interactive web
recognition to online transactions with the government. applications. AJAX is a programming tool that is used
The government has set up NIC under the Department extensively in what are known as Web 2.0 applications.
of Information Technology as the specialized body to pro- Analogue: ‘Something that corresponds to something
vide the network backbone and e-governance support to else.’ For example, in the context of equipment used for
the central government, state governments, UT adminis- recording and playing back sound, analogue refers to the
trations, districts and other government bodies. Almost all way in which the sound is recorded and reproduced.
the government departments now maintain web presence.
The following is the list of certain projects undertaken Anchor: In the context of HTML, an anchor the coding
at the central level: system used for creating web pages. An anchor is the main
target of a hyperlink.
1. India Image (Government of India Portal)
2. Agricultural Marketing Information Network Animation: The display of a sequence of images in a com-
(AGMARKNET) puter program or on a web page to give the impression of
3. Central Passport System movement.
4. Community Information Centres (CICs)
Application Programming Interface (API): The word
5. Computeried Rural Information Systems Project (CRISP)
Application refers to any software (available on Operating
The e-courts have been set up in judiciary. The state System)with a distinct function. Interface can be thought
government has started its own projects. of as a contract of service between two applications. This
Some state governments projects are the following: contract defines how the two communicate with each
other using requests and responses.
1. Bhoomi
2. e-Seva (electronic Seva) Apache: It is the most popular web server software on the
3. CARD in Andhra Pradesh World Wide Web. Apache mainly runs on UNIX systems.
4. FRIENDS
5. Gyandoot Applet: It is a small program written in the Java program-
6. Vidya Vahini ming language and embedded in a web page. When you
7. Lok Mitra use your browser to access a web page, an applet may run
8. SETU inside the web page, as it were to perform an interactive
9. Jan Mitra animation and make a calculation or carry out another
10. Aarakshi simple task.
App: Application.
Archie: It allows a user to search files in FTP sites. It regu-
Glossary of ICT and the Internet larly monitors hundreds of FTP sites and updates a data-
Terms base (called an Archie server) on software, documents
and data files available for downloading. By clicking on
Absolute Link: It is used by web authors. In an HTML a list of the Archie server, it will take the user to another
document, a relative link indicates the location of a file computer system where relevant files are stored. The
relative to the document, whereas an absolute link talks Archie server may allow users to continue their searches
about the full URL. for files until they locate what they actually need.
Acceptable Use Policy (AUP): It is a set of rules that sets Archive: It is used to describe the documents or files that
standards and defines the ways in which ICT facilities can are not immediately needed but would not be completely
and cannot be used in an institution. discarded. An archive may be stored in an external hard
disk, such as CD-ROM, DVD or other storage devices.
Accessibility: Everyone should have access to the ser-
vices provided by ICT, for example, computer programs, Advanced Streaming Format (ASF): Microsoft’s own
email and the World Wide Web, regardless of any visual, file format that stores both audio and video information
auditory or other physical impairment they might have. and is specially designed to run over the internet.
Address Book: Usually, it is supplied as part of your e Attachment: It is a term used in connection with an email.
mail software. An address book, in this sense, is used to An attachment can be a file of almost any kind, such as a
keep a record of all the email addresses of people whom document file, an image file, a sound file or a video clip
you may wish to contact by email. that you can add, that is, attach to an e mail.

M08_MADAN 04_65901_C08.indd 30 23/12/22 7:38 PM


Information and Communication Technology (ICT) 8.31

Authoring Package/authoring Program/authoring Non Fungible Tokens (NFTs): They are created using
Tool: These terms specify the content-free software pack- the same type of programming used for cryptocurrencies.
ages that allow a teacher to develop interactive learning These are based on blockchain technology but can not be
and teaching materials without having to hold detailed traded like bitcoin or ethereum.
knowledge about computer programming languages.
These terms may also be applied to software packages Blog: A blog (short form for web log) is an online diary in
used for creating web pages, for example, FrontPage or which an individual records and publishes his/her thoughts
Dreamweaver. on one or more subjects. It can contain news items, short
essays, annotated links, documents, graphics and multime-
Automatic Speech Recognition (ASR): A branch of dia. These posts are usually in a reverse chronological order
human language technologies that helps in automatic and often take the form of a journal or diary. Blogger refers
processing of human speech. to someone who blogs, that is, who regularly writes blogs.
Avatar: It is a graphical representation of a real ­person; Bookmark: It is a facility within a browser that enables
it is used as MUVE or MMORPG, a kind of ‘virtual world’. a person to keep a record of web pages visited and that
may be visited again. Bookmarks are stored in a subdirec-
AVI: It refers to Audio Video Interleave. tory of the Windows directory on computer. In Internet
Backup or back up: Used as a verb; to back up means to Explorer, bookmarks are known as Favorites.
copy a file or folder from your computer to another storage Boot: It is a verb used for starting up a computer by load-
device. ing the operating system into memory.
Bandwidth: It is the amount of data that can be sent Bot: It is short for robot; we can look at ‘crawler’, as well.
from one computer to another through a particular con-
nection in a certain amount of time. For example, it can Branching: It is the process of interrupting a sequence of
be connected through a computer to the internet and instructions in a computer program so as to go to a differ-
vice versa. The more the bandwidth, the faster the access ent point.
to information. Bandwidth is usually measured in kbps Bug: Not a nasty insect but a logical fault in a computer
or Mbps. program that causes it to malfunction.
Baud: It is a unit of measurement at which data can be Bulletin Board: A type of forum on the internet or an
transferred (i.e. the baud rate), for example, over a tel- intranet, where users can post messages by email or
ephone line through a modem or from a computer to an WWW for other users to read and respond to. BBS stands
external device, such as a printer. Baud is rarely used for Bulletin Board Systems.
nowadays, as transfer rates are normally expressed in
kbps or Mbps. Burn: When data is written on a CD, this process is often
referred to as ‘burning a CD’. This system is seldom used
BGAN: They are short for ‘Broadband Global Area these days.
Network’ and are portable terminals which provide inter-
net connectivity and voice communications in remote Cache: It contains the information stored by a web
locations. browser on the hard disk, so that we do not have to down-
load the same material repeatedly from a remote com-
BIOS: Basic input/output system. puter. The cache is normally stored under Windows in a
Bitmap (BMP): It is a computer graphic or image com- folder called Temporary Internet Files.
posed of thousands of individual dots or pixels, each pixel CALI: Computer Assisted Language Instruction.
being stored as a number.
Camcorder: A portable video camera, capable of record-
Blackboard: It is a commercial virtual learning environ- ing live motion video for later replay through a videocas-
ment (VLE) package. sette recorder (VCR), DVD player or computer.
Blockchain: Blockchain was introduced in 1991 by Card: An electronic circuit board, usually one which can
Stuart Haber and W. Scott Stornetta. It was introduced be slotted into your computer in order to fulfil a special-
to public in 2008 by Satoshi Nakamoto. Crypto currency ized function. Examples are sound card and video card.
that is supported by Blockchain that is actually a ‘Group
of blocks (chunks)’ that holds group of information. The Cascading Style Sheets (CSS): CSS are a feature of
information is stored in ‘digital format’ that is used in HTML that enables a range of styles for headers, body
transactions in business network. They are used crypto- text, bullet points, links, etc.
currency systems, such as Bitcoin, meme crypto, dogecoin Case Sensitivity: It is used to describe how a computer
(linked with the richest person Elon Musk). It maintains program, for example, a browser, interprets upper-and-
a ‘secure’ and decentralized record of transactions. It is lower case letters.
also used for decentralized finance (DeFi) applications,
non-fungible tokens (NFTs), and smart contracts. Closely Cathode Ray Tube (CRT): An older type of computer
linked with it is NFT. display screen or monitor, in which beams of high-voltage

M08_MADAN 04_65901_C08.indd 31 23/12/22 7:38 PM


8.32 Chapter 8

electrons are fired at a screen causing thousands of red, CMC: Computer-mediated communication.
green and blue (RGB) dots to glow in different combina-
CMY: Cyan Magenta Yellow, the scheme used in colour
tions and intensities. It produces full-colour image dis-
printing.
played on the screen.
CBT: Computer-based training. CODEC: COmpressor/DECompressor or COder/
DECoder. A CODEC is a software that is used to compress
Compact Disc Read-only Memory (CD-ROM): This is a or decompress a digital audio or video file.
pre-pressed optical compact disc that contains data (audio,
text, sound, pictures etc). Computers can read—but not Collaborative Writing: A process that involves the
write or erase on them (read-only memory). During the creation and editing documents using Web 2.0 tools
1990s, CD-ROMs were popularly used to distribute soft- designed for use by multiple authors, for example, Google
ware and data for computers and fifth generation video Documents or Zoho Writer.
game consoles. They have been replaced with Memory Colour Depth: The number of colours that can be dis-
Flash such as Flash Memory cards, Integrated Chips (ICs) played at any one time on a computer display screen.
and cloud computing.
Combination Drive: A disk drive that is capable of read-
CELL: Computer enhanced language learning. ing and writing CD-ROMs, audio CDs and DVDs.
CERN: European Organization for Nuclear Research in Compatibility: It is basically the setting between hard-
Geneva. It is the largest particle physics laboratory. It is ware and/or software so that they can work together.
also the birthplace of the World Wide Web, which was
invented there by Tim Berners-Lee. Compression: A technique which reduces the amount of
space required to store data.
Computer Graphics: This deals with generating images
with the aid of computers. Now, it has become a core Concordance Program: It operates on a body of texts (a
technology in digital photography, film, video games, cell corpus) and is commonly used for compiling glossaries
phone and computer displays etc. and dictionaries.
CGI Script: Common Gateway Interface. A term used by Condenser Microphone: This type of microphone is
web authors, it processes data from an HTML form. probably the best type to use in multimedia CALL pro-
grams as it provides a stronger signal when the learner is
Character User Interface (CUI): CUI is a user interface
recording his/her own voice.
where the user interacts with the computer solely through
the keyboard. He requires a command to perform any task. Content and Language Integrated Learning (CLIL): If a
CUI is the precursor of GUI and was utilized in most of the foreign language is to be used.
early computers. Most computers use GUI rather than CUI.
Content-free: Used to describe a computer program which
Chat Room: A synchronous, mainly text-based commu- is supplied as an ‘empty shell’, that is, without content, such
nication facility, offering a web-based environment in as texts, images, audio recordings or video recordings.
which people either drop into or arrange to meet and chat
at specific times. Continuing Professional Development (CPD): It can
take the form of seminars, research, training courses, etc.
Chunking: This is the method of presenting information
which splits concepts into small pieces or chunks so that Cookie: It is a piece of information stored in a user’s com-
they can be understood faster and easier. puter by a web browser when the user visits a website for
the first time. Websites use cookies to recognize users
Client: A computer that receives services from another who have previously visited them. The next time the user
computer. We can refer to LAN and VAN in the chapter. visits that site, the information in the cookie is sent back
Clipart or Clip Art: A collection of image files that can to the site so that the site can tailor what it presents to
be embedded or inserted into web pages, word-processed the user, for example, tastes in music or shopping habits.
documents, PowerPoint presentations, etc. Copyright: New technologies have raised all kinds of new
Clipboard: A temporary storage area in a computer’s issues relating to copyright mainly because it has become
­memory. so easy to copy materials from a variety of digital sources.
Clock Speed: The speed of a computer’s CPU that is nor- Course Management System (CMS): It is a type of virtual
mally expressed in megahertz (1 million cycles/s) or giga- learning environment (VLE), for example, Moodle.
hertz (1000 MHz). Crash: It basically refers to ‘hang’ (stopping) of software
Cloze Procedure: It was invented by Wilson Taylor. It was and hardware during their working. Their was a ‘google
originally conceived as a tool for measuring the readabil- hangout’ that was a cross platform instant messaging
ity of a text or a learner’s reading comprehension level. It service that was originally a feature of google+ (that also
derives itself from the gestalt psychology term ‘closure’, closed). In ‘frozen screen’, the keyboard and/or mouse go
whereby people tend to complete a familiar but incom- dead with the result that nothing can be typed and the
plete pattern by ‘closing’ the gaps. cursor cannot be moved around the screen.

M08_MADAN 04_65901_C08.indd 32 23/12/22 7:38 PM


Information and Communication Technology (ICT) 8.33

Crawler: This searches the web for new links, new content numeric characters that IBM developed for its larger
and changes in order to keep the search engine results up operating systems.
to date. It may also be called a bot (short form for robot)
Executable: This describes a program which has been
or ­spider. Crawlers within the search engines perform a
converted (compiled) into binary machine code. If you
useful indexing function, but there are also crawlers or
double-click an executable program name in Windows
bots that have more sinister motives, such as gathering
Explorer, then it will immediately execute itself, that is,
addresses to be targeted by spammers.
run. Executables usually have the extension .exe or .com.
Data-driven Learning (DDL): It was pioneered by
Extension: In computer jargon, an extension is an optional
Tim Johns, whereby learners of a foreign language
addition. Usually, it consists of a dot plus three or four let-
gain insights into the language that they are learning
ters to the name of a file. The extension to the filename
by using concordance programs to locate authentic
helps the computer (and the user) recognize what type of
examples of language in use.
file it is and what it may contain, for example, .doc is a
Debug: To test a program and remove all bugs. Permanent Word document file, .exe is a computer program, .jpg or
bugs that defy eradication are often referred to ironically .jpeg is a picture file and .htm or .html is a web page file.
as ‘features’.
Favorites: A facility within the Internet Explorer browser
Default: A setting or value automatically assigned to a that allows to keep a record of web pages that you have
computer program or device in the absence of a choice visited and may like to visit again. These are also known as
made by the user. bookmarks.
Defrag: This term is used for maintenance of systems, it Feedback: Feedback is an automatic response from a
reduces the fragmentation of system. This organizes the computer, which may take the form of text, image, audio,
data or contents of mass storage device (such as storage video or any combination of these, to a learner’s input.
devices). Finger: This command allows the display of the contents
Desktop Publishing (DTP): An application for laying out of the files that are associated with a particular user iden-
text, graphics and pictures in order to produce a profes- tifier at a particular internet site.
sional-looking publication. Firewall: A firewall is a software package that sits between
Diacritic: When added to a letter, it gives it a special your computer and your internet connection, keeping an
phonetic value. This indicates a particular pronunciation eye on the traffic going to and fro. Otherwise hackers can
of the words. use them to their benefits.
Digital: It is ‘based on numbers’. The modern com- Firewire: It allows a person to transfer video recordings
puter is a typical example of digital technology, so are from one device to another in a fast manner, that is, from
CD-ROMs, DVD-ROMs, audio CDs and video DVDs, on a camcorder to a computer, using a special cable.
which numbers are coded as a string of tiny pits pressed Firmware: It is a software that has been written to a
into a p
­ lastic disc. (read-only memory (ROM) chip by the manufacturers.
Directory: A group of files and subdirectories grouped Flame: When we use discussion list, forum or blog, Flame
together for organizational purposes. The term is used is used to describe a language that is rude, sarcastic or
synonymously with folder. condescending.
Discussion list: An electronic discussion list, also known Flash Drive: A portable storage device. Its storage capac-
as a forum, is a way of sharing emails with the members ity is impressive.
of a group of people with a common interest.
Fuzzy Matching: A matching technique which is used in
Dithering: This technique of combining dots of primary programs when allowances have to be made for inaccura-
colours is used to give the appearance of an intermediate cies in spelling on the part of the learner.
colour.
Gap Filler: It consists of two parts: (i) a teacher’s program
Dpi: Dots per inch. which allows him/her to input a text, and then specify
DVD: Digital Video Disc or Digital Versatile Disc. words, parts of words or phrases that are to disappear, and
(ii) a student’s program which enables the learner to inter-
Dynamic Microphone: It is used when a learner has to act with the computer by filling in the gaps.
record his/her own voice.
Geek: A term to describe someone obsessed with comput-
Encryption: A system of coding that helps prevent access ers and who uses them at every opportunity in his/her
to private information on computer networks or on free time, mainly for ‘social’ purposes.
the web.
Generic Software/application: They may be used in
Extended Binary Coded Decimal Interchange Code many areas, but are not specifically for use in a specific
(EBCDIC): This is a binary code for alphabetic and subject area, for example, a word processor (Word),

M08_MADAN 04_65901_C08.indd 33 23/12/22 7:38 PM


8.34 Chapter 8

spreadsheet package (Excel), presentation software Integrated Learning System (ILS): It is a computer-
(PowerPoint) and database package (Access). driven system of learning in which the content is pre-
sented in tutorial format and which monitors and records
GIF: Graphic Interchange Format. It is a file format used the progress of the learner.
for storing simple graphics.
Intelligent CALL (ICALL): It mimics human intelligence.
Gopher: This is an application-layer protocol that pro-
vides the ability to extract and view Web documents Interactive Whiteboard (IWB): It is a touch-sensitive
stored on remote Web servers. Gopher was conceived in projection screen that allows the teacher to control a com-
1991 as one of the Internet’s first data/file access proto- puter directly by touching the screen, that is, whiteboard,
cols to run on top of a TCP/IP network. This is equivalent rather than using a keyboard or mouse.
to FTP.
Interface: It is a connection between two systems. It can
This is linked with ‘Veronica’ that stands for Very Easy be hardware or software. It may take the form of a plug,
Rodent Oriented Netwide Index to computer archives. It cable or socket or all the three.
helps in the searching for the text.
iPod: It is the name of a portable (mobile) media player
Graphical user Interface (GUI): It consists of graphi- designed and marketed by Apple.
cal elements known as icons and allows the user to run
Java: It is a programming language invented by Sun
programs and also to carry out other operations. Icon is a
Microsystems that is specifically designed for writing
small symbol or picture used in a GUI.
programs that can be downloaded to one’s computer
Hacker: A person who spends time to gain access to through the internet and immediately executed. Java is a
information stored on other people’s computers all programming language designed for programs or applets
around the world. Some hackers are harmless and some used through the internet.
harm others.
JPEG (Joint Photographic Experts Group): It is a
HF (High Frequency): ‘High frequency’ is the desig- format for storing complex graphics in a compressed
nated term for the range of radio waves between 3 and form.
30 MHz.
LCD: Liquid crystal display is a type of flat panel computer
Host: It refers to a computer that provides services to display screen.
other computers that are linked to it through a local net-
Learning Object: It is capable of being reused in a vari-
work or through the internet.
ety of applications and may be described as a Reusable
Hub: A common connection point for networked comput- Learning Object (RLO).
ers and other devices. Hubs are used to connect devices in
Learning Platform: A term used to describe the software
a local area network (LAN).
and systems that are used to deliver e-learning. Sometimes
Hypermedia: It is an extension of hypertext that inte- it is virtual learning environment (VLE) and sometimes
grates audio, video and graphics with text (like multi- managed learning environment (MLE).
media). Hypermedia is an extension to what is known as
Leased Line: Also known as a private circuit, it is a dedicated
hypertext. Information bits are stored in the form of sig-
communications link between two sites. It is separate from
nals in hypermedia. Internet is the best example of use
the public telephone network and reserved exclusively for
of hypermedia. It is the ability to open new Web pages by
the use of the owner, usually at a fixed tariff regardless of
clicking text links on a Web browser.
usage levels.
Hypertext: It is a technology that links text in one part of
Link Rot: It describes the tendency of hypertext links
a document with its related text in other part of the docu-
from one website to another to die as other sites cease to
ment or in other documents. A user can quickly find the
exist or remove or restructure their web pages.
related text by clicking on the appropriate keyword, key
phrase, icon or button. Linux: A Unix-type operating system, which is similar to
Windows and the Apple Mac operating system.
Integrated Chips (ICs): This IC is a set of electronic
circuits on one small flat piece (or “chip”) of semiconductor Lurker: It is mainly used in connection with a ­discussion
material, that is usually made up of silicon. list, forum or blog. This term describes someone who pre-
fers to read other people’s messages rather than posting
Interactive Whiteboard (IWB): It is a touch-sensitive his/her own views.
projection screen that allows the teacher to control a com-
puter directly by touching the screen, that is, the white- Machine-assisted Translation (MAT): It assists us in the
board, rather than using a keyboard or a mouse. process of translating natural language.
Internet Relay Chat (IRC): This service allows each par- Mashup: A mashup is a web page that brings together
ticipant’s contribution to be displayed on the screens to all data from two or more web services and combines it into
others taking part in the conversation. a new application with added functionality.

M08_MADAN 04_65901_C08.indd 34 23/12/22 7:38 PM


Information and Communication Technology (ICT) 8.35

Maze: The maze is divided into action mazes and text Ning: A platform that enables you to create your own
mazes that have been used by language teachers for many social network.
years for reading and comprehension activities and to
Online Learning: The use of the internet to follow a
stimulate conversation in the classroom.
course that usually results in the award of a diploma or
Menu: A list of options from which a computer user makes certificate.
a selection in order to determine the course of events in a Open Source: It is used to describe a software that is pro-
program. Menu bar helps in it. vided free of charge along with the original source code
Microblogging: It is an approach to blogging in which used to create it, so that anyone can modify, improve and
very short texts are posted and contains snippets of infor- work in ways that reflect one’s own preferences. Moodle is
mation about events, websites and other sources. a typical example of an open source software.
MicroSD: This is smaller variant of Secure Digital (SD). Open and Integrated Learning System (OILS): A vari-
It is used in certain cellphones, PDAs, and smaller lighter ant of Integrated Learning System. The word Open means
devices. An adopter is to be used for its working. extra dimension.
MLAT: Modern Language Aptitude Testing. Optical Character Recognition: Optical character recog-
nition (OCR) software is used in conjunction with a scan-
MLE: Managed learning environment. ner to convert printed text into a digital format.
Moblog: A contraction of mobile and blog. Pathname: The pathname of a file on a computer specifies
Moodle: Moodle is an open source software, which means exactly its position on disc and it consists of at least three
it is free to download, use, modify and even distribute. parts: (i) drive letter, (ii) directory and (iii) filename, for
example, c:\windows\user.exe.
MOO: Multi-User Domain Object Oriented.
PDA: Personal digital assistant.
MP3: A file format for storing high-quality audio files.
PDF: It is Portable Document Format, a file type created
MP4: There are two basic types, namely, MP4 Advanced by Adobe that allows fully formatted documents to be
Audio Codin (AAC g) and MP4 Advanced Video Coding transmitted across the internet and viewed on any com-
(AVC). puter that has Adobe Acrobat Reader software.
MPEG or MPG: Motion Picture Expert Group. Personal Digital Assistant (PDA): It is a handheld device
that combines computing, audio communication, brows-
Multimedia: The integration of two or more types of ing and networking features and serves as an organizer
information (text, images, audio, video, animation, etc.) for personal information.
in a single application.
Phishing: An illicit attempt to trick individuals into handing
Multitasking: This is the execution of more than one pro- over personal, confidential information. Phishing requests
gram, apparently at the same time, on a computer. often seem legitimate. For example, they carry an official-
Narrowband: A term used to describe a slow-speed con- looking logo of a known service like VISA and may be perpe-
nection to the internet, normally via a modem and less trated by phone, email or the internet.
than or equal to 64 kbps. Pixel: A Pixel is a contraction of picture element. Every
Natural Language Processing (NLP): A general term online graphic consists of a bunch of tiny coloured squares
used to describe the use of computers to process informa- working together to form an image. In a way, online graphics
tion expressed in natural human languages. emulate a painting technique called ‘Pointillism’.
Navigation: This describes the process of finding your Platform: Often used as an alternative term for a com-
way, that is, navigating, around a series of menus within a puter system, including both the hardware and the soft-
computer program or finding your way around the World ware. Platform-independent software means that the
Wide Web by means of a browser. software can be run on any computer.

Netbook: A netbook is a small, lightweight computer that is Pointing Device: It is a device which allows to control the
smaller than a laptop computer, with a long battery life and position of the cursor on a computer screen by physical
ideal for travelling with. Netbook computers have built-in manipulation of the device in different directions.
Wi-Fi and are optimized for browsing the web and email. Podcast: The term podcast takes its name from a combi-
Netiquette: Etiquette on the internet is a code of behaviour nation of iPod and broadcasting. Podcasts can simply be
for people communicating by email via the internet. downloaded to a computer and played using a standard
media player program.
Netizen: It is a term used to describe someone who uses
Pop-up: It is a small window that appears within a pro-
network resources.
gram or over a web page to deliver additional informa-
Netscape: An early web browser, which first appeared in tion. Pop-ups on the web can be annoying as they are
1994, shortly after the World Wide Web went public. often used for unwanted advertising material.

M08_MADAN 04_65901_C08.indd 35 23/12/22 7:38 PM


8.36 Chapter 8

Portable Network Graphics (PNG): This is a graphics Scroll: To move up and down or from side to side through
format that is specifically designed for use on WWW. PNG a document or a window to view or access all of its
compresses images without any loss of quality, even for ­contents.
high resolution images. This is assumed to be better than
Search Engine: A search facility provided at a number of
JPG.
sites on the World Wide Web.
Portal: It is a web page, website or service that acts as a link Semantic Web: The Semantic Web allows the user to
or entrance to other websites on the internet. Typically, a search the web in a more sophisticated way.
portal includes an annotated catalogue of websites and
may also include a search engine, email facilities, a forum Server: A computer which provides services to other com-
and other services. puters, which are known as clients.
Presentation, Practice and Production (PPP): A long- Server: It is a computer which provides services to other
established approach to language teaching, consisting of computers and is known as clients.
three main phases: (i) presentation phase, (ii) practice Setup Program: A program that enables the user to set
phase and (iii) production phase. up a program.
Public Domain: It is a material that is copyright free, Shockwave Player: Software developed by Adobe that
whose copyright has expired or which cannot be copy- helps in containing interactive multimedia materials to be
righted. Many people think that because something is on played on the web.
the web, it must be in the public domain. This is not so. A
work is in the public domain only if it is explicitly stated Skimming: This is the process of reading only main ideas
to be so. within a document to get an overall impression of the con-
tent. It is applied when we are confronted with a lot to
PVP: Portable Video Player. read within a limited time.
QR Code: Quick Response code. It is a two-dimensional Silicon chip: An encased piece of extremely pure ­silicon
barcode that can store a variety of different types of infor- on to which electronic circuits are etched.
mation, such as a text, a website URL, a telephone num-
ber, an SMS message and an email address. Simultaneous Peripheral Operations Online (SPOOL):
With this, the output is not directly sent to the printer. It
Relative Link: A term used mainly by web authors. In an is first transferred to an intermediary storage medium
HTML document, a relative link indicates the location of such as a disk file. The output can then be stored in
a file relative to the document, whereas an absolute link separate files and printed at a later stage depending on
specifies the full URL. the availability of time and storage.
Repurpose: To reuse content in a different way from that Smartphone: These include mobile phones that primarily
which was originally intended. run on the ‘Android’ and ‘iOS’ (of Apple) operating systems.
Response Analysis: A feature of CALL programs whereby They include any open operating system that has a software
the computer attempts to diagnose the nature of errors development kit available to developers where native
the learner makes and to branch to remedial exercises. APIs are used to write applications. They have cameras
for recording, many video and audio functions. They run
Rip: To extract or copy data from one format to another. various applications. Example is Apple’s iPhone.
Root Directory: The topmost directory in the directory hier- Smiley: In email messages, a facial expression con-
archy, from which all other directories are descended. structed sideways (for the lateral-minded) with standard
RSS: It is a development in the internet technology that characters is called a smiley. It is also referred to as an
enables the users to subscribe to websites that change or emoticon (emotions with icons).
add content regularly, for example, news sites (such as the Social Media: It is a term used to describe a variety of
BBC) and sites containing blogs, Nings, podcasts and Wikis. Web 2.0 applications that enable people to share images,
RSS makes use of software that presents new ­additions. audio recordings and video recordings through the web
and also initiate discussions about them.
RTF: Rich Text Format is an alternative way of storing a
document created by a word processor. Social Networking: It is a term applied to a type of web-
site where people can seek other people who share their
Sampling: This term refers to taking the value of a wave-
interests, find out what is going on in their areas of inter-
form (e.g. a sound wave or video signal).
est and share information with one another.
Scanning: This is a reading technique that is used when Spam: These are unsolicited email advertisements, which
you want to find specific information quickly. We quickly are the internet equivalent of junk mail.
move our eyes through the text to locate specific word or
a phrase or ideas. It’s similar to looking up the dictionary Spambot: A spambot program is designed to collect email
for a specific word or looking for information in reference addresses from the internet in order to build mailing lists
book. for sending spam.

M08_MADAN 04_65901_C08.indd 36 23/12/22 7:38 PM


Information and Communication Technology (ICT) 8.37

Splog: The splog site creator (i.e. a splogger) begins by remote locations, using satellite connections. They are
finding a subject that attracts a lot of visitors. Then, the similar to BGAN in many respects.
splogger sets up a blog that plagiarizes content from other
TIFF or TIF: Tag Image File Format.
sites dealing with this subject. Splogs may consist of hun-
dreds of blogs with plagiarized content, containing mul- Toolbar: A toolbar is a kind of menu bar. It is mostly
tiple links to selected websites. located at the top of a computer screen that contains icons
Spyware: It is a term that may be used synonymously for the most commonly used commands in an application,
with adware but implies more sinister motives on the part for example, in a word processor or browser.
of the person who has dumped it onto your computer, Total cloze: An activity in which a complete text is
for example, with a view to steal private information reduced to sets of blanks.
such as bank account numbers, credit card numbers and
passwords. Touchscreen: A display screen which enables a com-
puter system to react to the touch of a finger, for example,
SVGA: Super Video Graphics Adaptor. This is used to con- smartphones and tablet computers.
trol the output on a computer display screen.
Traditional Knowledge Digital Library (TKDL): Set up in
Tablet Computer: A tablet computer is a compact por- 2001, this is an Indian digital knowledge repository of the
table computer that makes use of a touchscreen instead of traditional knowledge, especially about medicinal plants
a keyboard for typing and running applications. Apple’s and formulations used in Indian systems of medicine.
iPad is a typical example of a tablet computer.
Trainspotter: A colloquial term that is often used to
Tagging: This has become very common in last few describe someone who is fascinated by the technology of
years as a result of widespread use of social media. This computers but not particularly interested in their appli-
helps in sharing images, audio recordings, video record- cations.
ings, website references and so on. They are also used in
HTML, to define how the on-screen text is rendered by Trojan: Usually, malicious programs that install them-
the browser. selves or run surreptitiously on a victim’s machine.
Tags and Attributes: An HTML tag is indicated by open- Troll: A troll intentionally posts derogatory or provocative
ing () brackets. Each tag contains various attributes, messages in an online community such as a discussion list,
depending on the tag used. forum and blog to bait other users into responding.
Tandem Learning (Buddy Learning): It is a form of Unicode: The Unicode Worldwide Character Standard is
learning in which two language learners pair up in order a character coding system designed to support the inter-
to learn each other’s language. This may take place face- change, processing and display of the written texts of the
to-face or through the internet and includes using virtual diverse languages of the modern world.
worlds, such as second life. Universal Serial Bus (USB): It is a means of connecting a
Task-based Learning (TBL): An approach to learning in wide range of devices, such as digital cameras, camcord-
which the learner acquires knowledge of the subject that ers, iPods, mobile phones, scanners and printers, through
is being studied by focusing on a specified task. a cable to a computer.
TELL: Technology-Enhanced Language Learning. Unix: An operating system widely used on large com-
puter systems in institutions on which many web servers
Telnet: A program which allows you to log in to a remote are hosted. A PC version of Unix is called Linux. It has
host computer and carry out the same commands as if you become popular as an alternative to Windows.
were using a terminal at the host site.
URL: Uniform Resource Locator. It is also known as a web
Text File: It is a data file consisting entirely of printable address. A URL contains the location of a resource on the
ASCII characters, that is, plain unformatted text. Text internet.
files often have a .txt extension after the filename (e.g.,
readme.txt) and their contents can be viewed using Vector Graphic: It is a method of creating graphic
programs such as Windows Notepad. This file can also be images on a computer. It tells to draw lines in particular
used for ‘authoring packages’ such as Fun with Text. positions.
Thumbnail: This is a small picture (miniature version) Very High Frequency (VHF): ‘VHF’ is the designated
that links to a larger image. This is a graphic. Thumbnail term for the range of radio waves between 30 and 300
graphics are hyperlinked to larger versions of the graphic. MHz. VHF radio covers short distances, extendable
After we master the art of resizing and cropping images, through VHF repeaters.
we are ready to use thumbnails. This is specifically used in
Video Memory: The dynamic memory available for the
YouTube.com.
computer’s display screen. The greater the amount of
Thuraya IP+: These are portable terminals which pro- memory, the greater is the possible colour depth and reso-
vide internet connectivity and voice communications in lution of the display.

M08_MADAN 04_65901_C08.indd 37 23/12/22 7:38 PM


8.38 Chapter 8

Video Conferencing: A computer-based communication Webmail: A facility for creating, sending and receiving
system that allows a group of computer users at messages through the internet.
different locations to conduct a virtual conference in
WebQuest: A webQuest is a task-oriented activity. Here,
which the participants can see and hear one another
the learner draws on material from different websites in
as if they were in the same room participating in a real
order to achieve a specific goal.
conference.
Webserver: A computer or a software package running
Virtual Learning Environment (VLE): A VLE is a web-
on a computer that delivers, that is, server, web pages to
based package designed to help teachers create online
its clients.
courses, together with facilities for teacher–learner com-
munication and peer-to-peer communication. Wiki: It is a website or a similar online resource which
allows anyone to set up a resource in which the content
Virtual Private Network (VPN): A VPN allows remote can be created collectively. It allows anyone who views
offices or users to enjoy secure access to their organiza- the Wiki to add or to edit the existing content. Wiki also
tion’s network using the internet or other public telecom- refers to the software used to create such website.
munications systems.
Wikipedia: It is the best known example of a Wiki. It is a
Virtual Reality: The simulation of an environment by collaboratively written encyclopaedia.
presentation of 3D moving images and associated sounds,
giving the user the impression of being able to move Windows: The name of a range of several different graph-
around with the simulated environment. ical user interface (GUI) operating systems produced by
the Microsoft Corporation.
Very-small-aperture Terminal (VSAT): Typically, 1.8–
3.8 m in diameter, a VSAT is a fixed satellite communica- Wireless Application Protocol (WAP): WAP is basically
tion system or earth station with an antenna that accesses a technical standard for accessing information over a
satellites to provide internet connectivity in remote mobile wireless network. A WAP browser is a web browser
­locations. for mobile devices such as mobile phones that uses this
protocol.
Vlogs (or video blogs): Vlogs are akin to blogs, these are
simply the video versions of blogs. We share video content Wizard: This software guides us about step-by-step
in vlogs and educate people accordingly. through a complex task. For example, configuring a printer
to output data in a special format.
Vodcast: It is the short form for video podcast. This pod-
cast incorporates video as well as audio features. WMA: Windows Media Audio.
Voice Over Internet Protocol (VoIP): It is an audio WorldCALL: It is basically the worldwide umbrella asso-
communication using the internet instead of telephones. ciation of CALL associations (http://www.worldcall.org). It
It is another name for internet telephony. Skype and helps the countries that are currently not derserving in the
Ventrilo are the examples of VoIP. applications of ICT.

VRML: Virtual Reality Mark-up Language allows the dis- World Wide Web Consortium (W3C): It is an interna-
play of 3D images. tional non-profit organization which acts as a resource
centre for the WWW and is active in setting technical
VSAT Calling: The UNHCR maintains a global VSAT standards.
network that allows cost-effective voice communication
between offices from HQ to the field as well as between eXtensible Markup Language (XML): This specification
field offices. is as per demand from the WWW Consortium (W3C). It
allows web designers to create their own language for
W3C: It is the abbreviation for World Wide Web displaying documents on the web.
Consortium, an international non-profit organization
that acts as a resource centre for the World Wide Web, and Zip: ZIP is an archive file format that supports lossless data
is active in setting technical standards. compression. A ZIP file may contain one or more files or
directories that may have been compressed. It was invented
Website: It is an area on the WWW where an organization in 1989. Examples are WinZip or WinRar. Zipped files are
or individual stores a collection of pages of material such recognized by the extension .zip or .rar (for files created by
as the web pages. The pages are usually interlinked with WinRar) and they have to be unzipped before they can be
one another and with other websites. Every website has a used, again using proprietary programs.
unique web address or URL.
Zip Drive: A type of disc drive that accepts portable zip
Web Whacking: This involves saving entire websites for disks. Zip drives themselves are also portable and can be
offline use. connected to almost any computer.

M08_MADAN 04_65901_C08.indd 38 23/12/22 7:38 PM


Information and Communication Technology (ICT) 8.39

A s s e s s Yo u r L e a r n i n g

ICT BasICs
1. ICT stands for strengthen inclusion, and improve education admin-
(a) Information and communication technology istration and governance.
(b) Information-controlled technology (a) Both Assertion (A) and Reason (R) are correct
(c) Information-capable technology statements, and Reason (R) is the correct expla-
(d) None of the above nation of the Assertion (A).
(b) Both Assertion (A) and Reason (R) are correct
2. Which of the following is the appropriate definition
statements, but Reason (R) is not the correct
for information technology?
explanation of the Assertion (A).
(a) Information technology refers to the use of hard-
(c) Assertion (A) is correct, but Reason (R) is incor-
ware and software for processing information.
rect statement.
(b) Information technology refers to the use of hardware
(d) Assertion (A) is incorrect, but Reason (R) is cor-
and software for distribution of useful information.
rect statement.
(c) Information technology refers to the use of hard-
ware and software for storage, retrieval, process- 7. Assertion (A): In this everchanging technologically
ing and distributing information of many kinds. advanced world, educators must cope with the inven-
(d) Information technology refers to the use of prin- tions and advancements to cater to the students of
ciples of physical sciences and social sciences for this generation.
processing of information of many kinds. Reason (R): With the help of ICT, individuals can
3. Which of the following is also termed multimedia generate awareness amongst students apart from
education? augmenting their knowledge level in different aspects.
(a) ICT-supported education (a) If both Assertion and Reason are correct and
(b) ICT-enabled education Reason is the correct explanation of Assertion.
(c) ICT education (b) If both Assertion and Reason are correct, but
(d) None of the above Reason is not the correct explanation of Assertion.
(c) If Assertion is correct but Reason is incorrect.
4. Read the following two statements. [Jan 2017] (d) If Assertion is incorrect but Reason is correct.
I: Information and Communication Technology
8. NPTEL stands for

A S S E S S YO U R L E A R N I N G
(ICT) is considered as a subset of Information
(a) National Programme on Technology Enhanced
Technology (IT).
Learning
II: The ‘right to use’ a piece of software is termed as
(b) National Programme on Technology Enabled
copyright.
Learning
Which of the above mentioned statement(s) is/are (c) National Programme on Technology Enhanced
correct? Lessons
(a) Both I and II (b) Neither I nor II (d) National Programme on Technology Embedded
(c) II only (d) I only Learning
5. As per UNESCO, which of the following can be taken 9. The ERNET stands for
as the sequence of ‘understanding ICT in Education’? (a) Engineering and Research Network
(a) Policy Understanding, Policy Application, Policy (b) External and Regulated Network
Innovation (c) Educational and Research Network
(b) Policy Application, Policy Understanding, Policy (d) None of the above
Innovation 10. At which of the following institutions is ERNET used
(c) Policy Innovation, Policy Application, Policy to connect computers?
Understanding (a) IISCs
(d) Policy Innovation, Policy Understanding, Policy (b) IITs
Application (c) National Centre for Software Technology
6. Assertion (A): Information and Communication (d) All of the above
Technology (ICT) can complement, enrich and trans- 11.Which of the following technologies is mostly used by
form education for the betterment of society. a teacher in virtual education?
Reason (R): There are many ways technology (a) Course management applications
can facilitate universal access to education, bridge (b) Multimedia resources
learning divides, support the development of teach- (c) The internet
ers, enhance the quality and relevance of learning, (d) Video conferencing

M08_MADAN 04_65901_C08.indd 39 23/12/22 7:38 PM


8.40 Chapter 8

12. Which of the following statements are correct? 19. Which of the following statements are true?
1. The combination of computing, telecommunica- 1. Sending and receiving messages/signals occurring
tion and media in a digital atmosphere is referred at the same time is denoted by synchronous.
to as convergence. 2. Video conferencing is an example of synchronous
2. A dialogue between a human being and a com- technologies.
puter program that occurs simultaneously in vari- 3. Education through CD is asynchronous.
ous forms is described as ‘interactivity’. 4. Digital divide is the term used in the context of
3. The attitude is counted as the most important differentiation in the use of IT/ICT in developed
component of ICT. and developing nations, and also in urban and
(a) 1, 2 and 3 (b) 2 and 3 rural India.
(c) 1 and 2 (d) 1 and 3 (a) 1, 2, 3 and 4 (b) 2, 3 and 4
13. Which of the following open source e-learning plat- (c) 1, 3 and 4 (d) 3 and 4
forms has been developed by IIT Kanpur? 20. Which of the following statements are true in the con-
(a) e-Gyan (b) e-Saraswati text of hypermedia database?
(c) Brihaspati (d) None of the above 1. Information bits are stored in the form of signals
14. Recording a television programme on a VCR is an 2. Internet is the best example of use of hypermedia
example of 3. Hypermedia is an extension to what is known as
(a) Time shifting (b) Content reference hypertext.
(c) Mechanical clarity (d) Media synchronization 4. It is the ability to open new Web pages by clicking
text links on a Web browser.
15. Which of the following is/are the main challenge/s in
ICT adoption in Indian universities? (a) 2, 3 and 4 (b) 1, 2, 3 and 4
(a) Lack of technological readiness (c) 1, 3 and 4 (d) 3 and 4
(b) Poor implementation of ICT initiatives 21. Communication bandwidth that has the highest
(c) Linguistic barrier capacity and used by microwave, cable and fibre
(d) All of the above optics lines is known as
16. Given below are two statements, one is labelled as (a) Hyperlink (b) Broadband
Assertion (A) and the other is labelled as Reason (R): (c) Bus width (d) Carrier wave
[2020] 22. Symbols A–F are used in which one of the following?
Assertion (A): Machine Learning requires good qual-  [Dec 2014]
ity and sufficient data to train and test the algorithm. (a) Binary number system
Reason (R): For correct classification, good quality (b) Decimal number system
data which is free from noise, and sufficient data is
A S S E S S YO U R L E A R N I N G

(c) Hexadecimal number system


required for training and testing of the algorithm. (d) Octal number system
In light of the above statements, choose the most
23. The concept of connect intelligence is derived from
appropriate answer from the options given below:
(a) Virtual reality
(a) Both A and R are correct and R is the correct
(b) Fuzzy logic
explanation of A
(c) Bluetooth technology
(b) Both A and R are correct but R is NOT the correct
(d) Value-added networks
explanation of A
(c) A is correct but R is not correct 24. The function of mass communication of applying
(d) A is not correct but R is correct ­information regarding the processes, issues, events
and societal developments is known as
17. TKDL stands for
(a) Traditional Knack Digital Library (a) Content supply (b) Surveillance
(b) Traditional Knowledge Digital Library (c) Gratification (d) Correlation
(c) Transfer Knowledge Desktop Literature 25. Which of the following statements are true in the con-
(d) Transfer Knowledge Digital Library text of ICT?
18. Statement 1: Asynchronous communication means 1. Online learning
‘out of sync’ that is ‘not in real-time’. 2. Learning through the use of EDUSAT
3. Web-based learning
Statement 1: The examples of asynchronous com-
4. Asynchronous and synchronous media
munication are Discussion Forums, Blogs, e-Groups,
(a) 1, 2 and 3 (b) 2 , 3 and 4
Wikipedia (Knowledge base), Google (search engine)
(c) 1, 2, 3 and 4 (d) 1, 3 and 4
and Mobile SMS, MMS and Podcasting.
(a) Both Statement 1 and Statement 2 are correct. 26. Information that is a combination of graphics, text,
(b) Both Statement 1 and Statement 2 are not correct. sound, video and animation is called
(c) Statement 1 is correct but Statement 2 is not correct. (a) Multiprogramme (b) Multifacet
(d) Statement 1 is not correct but Statement 2 is correct. (c) Multimedia (d) Multiprocess

M08_MADAN 04_65901_C08.indd 40 23/12/22 7:38 PM


Information and Communication Technology (ICT) 8.41

27. Which of the following institutions launched a knowl- 36. Which of the following statements are true in context
edge repository e-Gyankosh in 2005 that aims at stor- of Web 3.0 that is taken as the latest phase in social
ing and preserving digital learning resources? learning?
(a) IIT Kanpur 1. This is freer for users, from a behavioural perspective.
(b) IGNOU 2. There is higher individual focus, and user
(c) Allahabad University engagement.
(d) Delhi University 3. There is more of customization
4. Blockchains and NFTs are connected with Web 3.0.
28. The institution promoted by the Department of IT to
(a) 1, 2, 3 and 4 (b) 2, 3 and 4
provide communication infrastructure and services to
(c) 1, 2 and 4 (d) 1, 2 and 3
academic research institutions in India is
(a) INFLIBNET (b) UGC 37. Which of the following services can help students to
(c) ERNET (d) None of the above access computer files from remote locations through
mobile phones?
29. The bouquet of FM radio channels which broadcast (a) Facebook (b) Renren
programmes contributed by institutions such as (c) Soonr (d) Twitter
IGNOU and IITs is
(a) Gyan Vani (b) Gyan Darshan 38. Satellite communication works through
(c) EDUSAT (d) None of the above (a) Transponder (b) Radar
(c) TV (d) Fibre optics cable
30. Which of the following institutions has been working
in the direction to develop a Virtual Technical Univer- 39. Assertion (A): The leading agencies should strengthen
sity in India? their leading role in Artificial Intelligence (AI) in
(a) IGNOU (b) UGC education, as a global laboratory of ideas, standard
(c) NMCEIT (d) AICTE setter, policy advisor and capacity builder.
31. The joint initiative of the IITs and IISc to provide Reason (R): The deployment of Artificial Intelligence
­e-learning through online web and video courses in (AI) technologies in education should be purposed to
many streams, specifically engineering in the country, enhance human capacities and to protect human rights
by providing free online courseware is for effective human-machine collaboration in life,
(a) National Programme on Technology Enhanced learning and work, and for sustainable development.
Learning (a) Both Assertion (A) and Reason (R) are correct
(b) AICTENET statements, and Reason (R) is the correct
(c) NMCEIT explanation of the Assertion (A).
(d) None of the above (b) Both Assertion (A) and Reason (R) are correct
statements, but Reason (R) is not the correct

A S S E S S YO U R L E A R N I N G
32. The terms gif, jpg, bmp, png are used as extensions for
files which store [December 2014] explanation of the Assertion (A).
(a) audio data (b) image data (c) Assertion (A) is correct, but Reason (R) is incorrect
(c) video data (d) text data statement.
33. ‘A-VIEW’, the software that has been developed under (d) Assertion (A) is incorrect, but Reason (R) is
the NMEICT, is basically developed for correct statement.
(a) Teacher’s training (b) Technical training 40. Consider the following statements
(c) Students’ training (d) All of the above Statement 1: Non Fungible Tokens (NFTs) are unique
34. Web 2.0 tools have made traditional learning more cryptographic tokens that exist on a blockchain and
social and personalized. Which of the following can cannot be replicated.
be referred to as Web 2.0 tools? Statement 2. Non Fungible Tokens (NFTs) can
(a) Blogs and Wikis represent real-world items such as artwork and
(b) Podcasts and mashups real estate. ‘Tokenizing’ these real-world tangible
(c) Social networking communities assets makes buying, selling, and trading them more
(d) All of the above efficient while reducing the probability of fraud.
35. Which of the following is a characteristic of Web 2.0 (a) Both Assertion (A) and Reason (R) are correct
applications? [January 2017] statements, and Reason (R) is the correct
(a) Multiple users schedule their time to use Web 2.0 explanation of the Assertion (A).
applications one by one. (b) Both Assertion (A) and Reason (R) are correct
(b) Web 2.0 applications are focused on the ability statements, but Reason (R) is not the correct
for people to collaborate and share information explanation of the Assertion (A).
online. (c) Assertion (A) is correct, but Reason (R) is incorrect
(c) Web 2.0 applications provide users with content statement.
rather than facilitating users to create it. (d) Assertion (A) is incorrect, but Reason (R) is
(d) Web 2.0 applications use only static pages. correct statement.

M08_MADAN 04_65901_C08.indd 41 23/12/22 7:38 PM


8.42 Chapter 8

Computer Terms
41. A computer consists of 48. Select the option that includes the file formats suitable
(a) A central processing unit for distributing sound files across the internet.
(b) A memory  [Dec 2019]
(c) Input and output unit (a) wmv, mp4, wma, mp3
(d) All of the above (b) avi, midi, wav, mp3
42. A typical modern computer uses (c) avi, mov, wmv, mp3
(a) LSI chips (d) midi, wav, wma, mp3
(b) Vacuum tubes 49. Group of instructions to direct the functioning of a
(c) Valves ­computer is called
(d) All of the above (a) Storage (b) Memory
43. Which of the following is/are the correct statement/s? (c) Program (d) Byte
(a) Computers can be used for diagnosing the diffi- 50. The main component of first-generation computers
culty of a student in learning a subject. was
(b) Psychological testing can be done with the help of (a) Transistors
a computer, provided that a software is available. (b) Vacuum tubes and valves
(c) A set of instructions is called a programme. (c) Integrated circuits
(d) All of the above (d) None of the above
44. Which of the following statements is not correct? 51. FORTRAN is the acronym for
(a) Computer is capable of processing only digital (a) File translation
signal. (b) Format translation
(b) Computer is capable of analysing both quantita- (c) Formula translation
tive and qualitative data. (d) Floppy translation
(c) Appropriate software is required for processing
52. In analogue computer
the data.
(a) Input is first converted to digital form
(d) Computer is capable of processing digital as well
(b) Input is never converted to digital form
as analogue signals.
(c) Output is displayed in digital form
45. Which of the following statements are correct? (d) All of the above
1. Virus improves the speed of processing informa-
tion in the computer. 53. A hybrid computer
2. Virus is a part of software. (a) Resembles a digital computer
(b) Resembles an analogue computer
A S S E S S YO U R L E A R N I N G

3. Computer is an electrical machine.


4. Computer can think on its own due to AI. (c) Resembles both a digital and an analogue computer
5. Computer can hold data for any length of time. (d) None of the above
(a) 2, 3, 4 and 5 (b) 1, 2, 3, 4 and 5 54. In the latest generation of computers, the instruc­tions
(c) 3, 4 and 5 (d) 1, 3 and 5 are executed
46. Which of the following is the appropriate definition of (a) Parallelly only
a computer? [December 2006] (b) Sequentially only
(a) Computer is a machine that can process (c) Both sequentially and parallelly
information. (d) All of the above
(b) Computer is an electronic device that can store, 55. The data storage hierarchy consists of  (November
retrieve and quickly process both quantitative and 2017)
qualitative data accurately. (a) Bytes, bits, fields, records, files and databases.
(c) Computer is an electronic device that can store, (b) Bits, bytes, fields, records, files and databases.
retrieve and quickly process only quantitative (c) Bits, bytes, records, fields, files and databases.
data. (d) Bits, bytes, fields, files, records and databases.
(d) Computer is an electronic device that can 56. Select the option that shows the storage devices in
store, retrieve and quickly process only quali- order of capacity from lowest to highest. [Jun 2019]
tative data. (a) CD-ROM, DVD-ROM, Blu-ray
(b) Blu-ray, CD- ROM, DVD-ROM
47. Which one the following pairs least matches in respect (c) CD-ROM, Blu-ray, DVD-ROM
of computer? (June 2019) (d) CD-ROM, DVD-ROM, Blu-ray
(a) 1 Giga Byte : (1024)8 bits
(b) CRT : Cathode Ray Tube 57. The hexadecimal number system consists of the
symbols
(c) ROM : Rapid Online Memory
(a) 0–7 (b) 0–9, A–F
(d) CPU : Central Processing Unit (c) 0–7, A–F (d) None of the above

M08_MADAN 04_65901_C08.indd 42 23/12/22 7:38 PM


Information and Communication Technology (ICT) 8.43

58. The binary equivalent of (−15)10 is (two’s complement 2. The size of computers was very large in first gen-
system is used) eration due to vacuum tubes and punch cards.
(a) 11110001 (b) 11110000 3. Microprocessors as switching devices are for
(c) 10001111 (d) None of the above fourth generation of computers.
59. 1 GB is equal to 4. Integrated chips were used in third generation
(a) 230 bits (b) 230 bytes computers.
(c) 220 bits (d) 220 bytes (a) 2, 3 and 4 (b) 3 and 4
(c) 1, 2 and 3 (d) 1, 2, 3 and 4
60. Which of the following statements are true?
1. All computers, big, medium or small, must have 68. UNIVAC is
ALU, Control Unit and Primary Storage. (a) Universal automatic computer
2. A byte consists of eight bits. (b) Universal array computer
3. A nibble consists of four bits. (c) Unique automatic computer
4. A bit is the smallest unit of computer memory. (d) Unvalued automatic computer
5. Bit is a binary digit meaning that it can take the 69. Which of the following programming languages is
value of either 1 or 0. widely used in computer science, engineering and
(a) 1, 2 and 4 (b) 3, 4 and 5 also business?
(c) 1, 2, 3 , 4 and 5 (d) 2, 3 and 4 (a) COBOL (b) FORTRAN
61. Which of the following is flash memory? (c) PASCAL (d) LISP
(a) RAM (b) EEPROM 70. Which of the following can be termed a the first super
(c) PROM (d) EPROM computer of India?
62. Which of the following is/are example/s of operating (a) PARAM 8000 (b) PARAM Yuva
systems? (c) PARAM Padma (d) PARAM Brahma
   I. Unix/BSD 71. The following fastest super computer was installed
II. GNU/Linux under National Supercomputing Mission (NSM) at
III. Windows the Indian Institute of Science, Bengaluru in January
IV. Mac OS: Mac OS X 2022. It runs on CentOS 7.x, has 4 petabytes of stor-
(a) I, II and III (b) II, III and IV age and a peak computing power of 3.3 Petaflops.
(c) I, III and IV (d) All of the above (a) PARAM Pravega (b) PARAM Sanganak
63. Statement 1: The system software designed to help in (c) PARAM Shivay (d) PARAM Siddhi-AI
analyzing, monitoring, configuring, optimizing set- 72. PARAM is a series of supercomputers designed and
tings and maintaining the computer is known as util- assembled by the Centre for Development of Advanced
ity software. Computing (C-DAC) under Ministry of Electronics

A S S E S S YO U R L E A R N I N G
Statement 2: The general term used for the software and Information Technology (MeITY). PARAM
that can be copied and used without payment to the means “supreme” in the Sanskrit language, whilst
author(s), although there may be some restrictions also creating an acronym for “PARAllel Machine. The
on distribution, is called as Freeware. C-DAC is headquartered in
Codes: (a) Pune
(b) Bengaluru
(a) Both statement 1 and statement 2 are true (c) Chennai
(b) Both statements 1 and 2 are false (d) SAS Nagar, Mohali
(c) Statements 1 is true and statement 2 is false
(d) Statements 1 is false and statement 2 is true 73. Normally, these computers are used in banking,
­airlines, railways, etc., for their applications. They can
64. The silicon chips used for data processing are called accommodate more than 1000 workstations simulta-
(a) RAM chips (b) ROM chips neously and can process data at a very high speed and
(c) Microprocessors (d) PROM chips can support concurrent programmes. These are
(a) Mainframe computers
65. The metal disks that are permanently housed in (b) Microcomputers
sealed and contamination-free containers are called (c) Workstations
(a) Hard disks (b) Floppy disks (d) Supercomputers
(c) Winchester disks (d) Flexible disks
74. Which of the following is used for manufacturing
66. All modern computers operate on chips?
(a) Information (b) Floppies (a) Bus (b) Control unit
(c) Data (d) Word (c) Semiconductors (d) (a) and (b) only
67. Which of the following statements are correct? 75. Which of the following correctly lists the types of
1. Instructions and memory address are represented the computer memory from highest to lowest speed?
by binary codes.  [Jan 2017]

M08_MADAN 04_65901_C08.indd 43 23/12/22 7:38 PM


8.44 Chapter 8

(i) Secondary storage (a) Both statements 1 and 2 are right


(ii) Main memory (RAM) (b) Statement 1 and Statement 2 are incorrect.
(iii)Cache memory (c) Statement 1 is correct but statement 2 is incorrect.
(iv) CPU registers (d) Statement 1 is incorrect but statement 2 is correct.
Codes: 83. ALU stands for
(a) (i), (ii), (iii), (iv) (a) Arithmetic logic unit
(b) (iv), (iii), (i), (ii) (b) Array logic unit
(c) (iv), (iii), (ii), (i) (c) Application logic unit
(d) (iii), (iv), (ii), (i) (d) None of the above
76. Which of the following can be taken as the exact 84. The ALU of a computer responds to the commands
sequence for the flow of data in a computer? ­coming from
(a) Cache – CPU – Memory (a) Primary memory (b) Control section
(b) Cache – Memory – CPU (c) External memory (d) Cache memory
(c) CPU – Cache – Memory 85. It contains buttons and menus that provide quick
(d) CPU – Memory – Cache access to commonly used commands. It is a
(a) Menu bar (b) Toolbar
77. Pick the correct statements (c) Window (d) None of the above
1. The source programme is written in high language.
2. The set of computer programs that manage the 86. Which of the following statements are correct?
hardware/software of a computer is called oper- 1. The type of keys Ctrl, Shift and Alt are termed as
ating system. Modifier Keys.
3. A software that converts a program from assem- 2. ALU, Logical operations, Input output operations
bly language to machine language is called a and Data Manipulation are functions of CPU.
assembler. 3. The instructions for starting the computer are set
4. A software which converts a high-level language up on Read-only memory chip.
program to machine language in one go is called (a) 2 and 3
compiler. (b) 1, 2 and 3
(a) 1, 3 and 4 (c) 1 and 3
(b) 2, 3 and 4 (d). 1 and 2
(c) 1, 2, 3 and 4 87. The ALU of a computer normally contains a number of
(d) 1, 2 and 3 high-speed storage elements called
78. A computer program that translates one program (a) Semiconductor memory
instructions at a time into machine language is called (b) Registers
A S S E S S YO U R L E A R N I N G

a/an (c) Hard disks


(a) Interpreter (b) CPU (d) Magnetic disk
(c) Compiler (d) Simulator 88. A factor which would strongly influence a business
79. A small or intelligent device is so called because it ­person to adopt a computer is its
­contains within it a (a) Accuracy (b) Reliability
(a) Computer (b) Microcomputer (c) Speed (d) All of the above
(c) Programmable (d) Sensor 89. The keyboard of a computer is encoded in
80. Which of the following statements are true? (a) Baudot code (b) ASCII code
1. Hard disk memory has the highest capacity. (c) BCDIC code (d) EBCDIC code
2. Central processing unit performs read or write 90. EBCDIC stands for
operations directly with RAM. (a) Extended Binary-Coded Decimal Interchange
3. An algorithm is basically the logical flow of a Code
program. (b) Extended Bit Code Decimal Interchange Code
(a) 1 and 2 (c) Extended Bit Case Decimal Interchange Code
(b) 1, 2 and 3 (d) Extended Binary Case Decimal Interchange
(c) 2 and 3 Code
(d) 1 and 3 91. EBCDIC can code up to how many different characters?
81. A collection of 8 bits is called (a) 256 (b) 16
(a) Byte (b) Word (c) 32 (d) 64
(c) File (d) Folder 92. If the binary equivalent of the decimal number 53 is
82. Which of the following statement/s is/are true? 110101, then the binary equivalent of the decimal
Statement 1: CPU is the brain of a computer. number 50 is given by
Statement 2: CPU consist of control unit , logic unit (a) 110011 (b) 110010
and primary storage. (c) 110001 (d) 110100

M08_MADAN 04_65901_C08.indd 44 23/12/22 7:38 PM


Information and Communication Technology (ICT) 8.45

93. ASCII stands for Statement 2:The Internal Memory or Primary Mem-
(a) American Stable Code for International ory comprises of main memory, cache memory and
Interchange CPU registers which are directly accessible by the
(b) American Standard Case for Institutional processor.
Interchange (a) Both Statement 1 and Statement 2 are correct.
(c) American Standard Code for Infor­ mation (b) Both Statement 1 and Statement 2 are not
Interchange correct.
(d) American Standard Code for Inter­ change (c) Statement 1 is correct but Statement 2 is not
Information correct.
94. CD ROM stands for (d) Statement 1 is not correct but Statement 2 is
(a) Computer Disk Read-only Memory correct.
(b) Compact Disk Read-over Memory 103. Which of the following statements are true in con-
(c) Compact Disk Read-only Memory text of memory hierarchy?
(d) Computer Disk Read-over Memory 1. As we move from top to bottom in the hierarchy,
95. SIMM is the capacity increases.
(a) Serial in Memory Module 2. As we move from top to bottom in the hierarchy,
(b) Serial Input Memory Module the access time increases.
(c) Synchronous In Memory Module 3. One of the most significant ways to increase sys-
(d) Synchronous Input Memory Module tem performance is minimizing how far down
the memory hierarchy one has to go to manipu-
96. Which of the following is not an example of primary late data.
memory? 4. As we move from bottom to top in the hierarchy,
(a) RAM (b) ROM the cost per bit increases that means that Internal
(c) Cache memory (d) Magnetic tape Memory is costlier than External Memory.
97. Which of the following statements are true? (a) 1, 2 and 3
Statement 1: Data can be saved on backing storage (b) 2, 3 and 4
medium known as Computer Disk rewritable. (c) 1, 2, 3 and 4
Statement 2: Process of copying files to a CD-ROM is (d) 2 and 4
known as Burning. 104. The act of retrieving existing data from the memory
(a) Both statement are wrong is called
(b) Both statements are right (a) Read-out (b) Read from
(c) Statement 1 is right but statement 2 is wrong (c) Read (d) All of the above
(d) Statement 1 is wrong but statement 2 is right. 105. RAM is used as short memory as it is

A S S E S S YO U R L E A R N I N G
98. RAM means (a) Expensive (b) Small capacity
(a) Random access memory (c) Programmable (d) Volatile
(b) Rigid cccess memory 106. The computer memory used for temporary storage
(c) Rapid access memory of data and program is called
(d) Revolving access memory (a) ROM (b) RAM
99. Microprocessing is made for (c) EROM (d) EPROM
(a) Computer (b) Digital system 107. The memory which is programmed at the time of its
(c) Calculator (d) Electronic goods manufacturing is
100. Which is the largest unit of storage among the (a) ROM (b) RAM
­following? (c) PROM (d) EPROM
(a) Terabyte (b) Megabyte 108. Which of the following is a secondary mem­ ory
(c) Kilobyte (d) Gigabyte device?
101. In comparison to secondary storage, primary (a) CPU (b) ALU
storage is (c) Floppy diskette (d) None of the above
(a) Slower and less expensive 109. The most important advantage of a video disk is
(b) Faster and more expensive (a) Compactness (b) Potential capacity
(c) Faster and less expensive (c) Durability (d) Cost-effectiveness
(d) Slower and more expensive 110. Floppy disks which are made from flexible plastic
102. Which of the following statements is / are true? material are also called
Statement 1:The External Memory or Second- (a) Hard disks (b) High-density disks
ary Memory comprises of Magnetic Disk, Opti- (c) Diskettes (d) Templates
cal Disk, Magnetic Tape i.e. peripheral storage 111. What is the latest write-once optical storage media?
devices which are accessible by the processor via (a) Digital paper (b) Magneto-optical disk
I/O Module. (c) WORM disk (d) CD-ROM disk

M08_MADAN 04_65901_C08.indd 45 23/12/22 7:38 PM


8.46 Chapter 8

112. Regarding a VDU, which of the following statements Choose the correct answer from the options given
is more correct? below:
(a) It is an output device. (a) (C), (A), (D), (B)
(b) It is an input device. (b) (C), (A), (B), (D)
(c) It is a peripheral device. (c) (A), (C), (D), (B)
(d) It is a hardware item. (d) (A), (C), (B), (D)
113. The two main memory types are 122. Which of the following statements are true in con-
(a) Primary and secondary text of memory hierarchy?
(b) Random and sequential 1. Memories are made up of registers.
(c) ROM and RAM 2. Memory locations are identified using Address.
(d) All of the above 3. The total number of bits a memory can store is its
114. Which of the following will happen when data is capacity.
entered into a memory location? 4. A storage element is called a Cell.
(a) It will add to the content of the location. 5. The data in a memory are stored and retrieved
(b) It will change the address of the memory by the process called writing and reading
location. respectively.
(c) It will erase the previous content. (a) 1, 2, 3 and 4
(d) It will not be fruitful if there is already some data (b) 2, 3 and 4
at the location. (c) 1, 2, 3, 4 and 5
115. A storage area used to store data to compensate for (d) 1, 2 and 4
the difference in speed at which the different units 123. Which of the following statements are true about
can handle data is RAM?
(a) Memory (b) Buffer 1. RAM is also called read-write memory or the main
(c) Accumulator (d) Address memory or the primary memory.
116. Which of the following memories allow simultane- 2. The programs and data that the CPU requires
ous read and write operations? during the execution of a program are stored in
(a) ROM (b) RAM RAM.
(c) EPROM (d) None of the above 3. RAM is a volatile memory as the data is lost when
the power is turned off.
117. Which of the following memories has the shortest 4. RAM is classified into two types- SRAM and DRAM
access times?
(a) Cache memory (a) 1, 2 and 3 (b) 2, 3 and 4
(b) Magnetic bubble memory (c) 1, 2, 3 and 4 (d) 2 and 4
A S S E S S YO U R L E A R N I N G

(c) Magnetic core memory 124. Which of the following statements are true about
(d) RAM ROM?
118. To locate a data item for storage is 1. ROM stores the most crucial information that is
(a) Field (b) Feed essential to operate the system such as the pro-
(c) Database (d) Fetch gram required to boot the computer.
2. ROM is used in embedded systems where the pro-
119. The magnetic storage chip used to provide non-vol- gramming does not need any change.
atile direct access storage of data and that has no 3. ROM is not volatile as it always retains its data.
moving parts is known as 4. ROM is used in calculators and peripheral devices.
(a) Magnetic core memory 5. ROM is classified into four types which are MROM,
(b) Magnetic tape memory PROM, EPROM, and EEPROM.
(c) Magnetic disk memory (a) 1, 2, 3 and 4
(d) Magnetic bubble memory (b) 2, 3 and 4
120. OCR stands for (c) 1, 2 and 4
(a) Operational Character Reader (d) 1, 2, 3, 4 and 5
(b) Optical Character Reader 125. Which of the following statements are true in con-
(c) Only Character Reader text of cache memory?
(d) None of the above 1. Cache memory is used to reduce the average time
121. Identify the correct order of the following computer to access data from the Main memory.
storage capacities ranked from largest to smallest 2. Cache memory is costlier than main memory or
capacity. [2021] disk memory but economical than CPU registers.
(A) 100 GBytes 3. Cache memory is an extremely fast memory type
(B) 10 000 000 000 Bytes that acts as a buffer between RAM and CPU.
(C) 1 TBytes (a) 1, 2 and 3 (b) 2 and 3
(D) 1000000 KBytes (c) 1 and 2 (d) 1 and 3

M08_MADAN 04_65901_C08.indd 46 23/12/22 7:38 PM


Information and Communication Technology (ICT) 8.47

126. Consider the following statements 134. MSI stands for


Statement 1: The memory devices used for primary (a) Medium-Scale Integrated Circuits
memory or main memory or internal memory are (b) Medium-System Integrated Circuits
semiconductor memories. (c) Medium-Scale Intelligent Circuit
Statement 2: The secondary memory or auxiliary (d) Medium-System Intelligent Circuit
devices are magnetic and optical memories. 135. WAN stands for
(a) Both Statement 1 and Statement 2 are correct. (a) WAP Area Network
(b) Both Statement 1 and Statement 2 are not (b) Wide Area Network
correct. (c) Wide Array Net
(c) Statement 1 is correct but Statement 2 is not (d) Wireless Area Network
correct. 136. Choose the correct options
(d) Statement 1 is not correct but Statement 2 is 1. MICR stands for Magnetic Ink Character Reader.
correct. 2. OCR is used for directly image printed text.
127. Which of the following statements are true? 3. Out put quality of printer is measured by dot per
1. The primary memory is more economical than inch.
the secondary memory. 4. Laser printer is very commonly used for desktop
2. The memory that is capable of storing high vol- publishing.
ume data is referred to secondary memory. Options
3. Secondary memory is slower than the primary (a) 1, 2, and 4 (b) 1, 3, and 4
memory. (c) 1, 2, 3, and 4 (d) 1, 2, and 3
(a) 1, 2 and 3 (b) 2 and 3 137. An output device that uses words or messages
(c) 1 and 2 (d) 1 and 3 recorded on a magnetic medium to produce audio
128. Which of the following stores the program to initially response is
boot the computer and only allows Reading? (a) Magnetic tape
(a) RAM (b) Voice response unit
(b) ROM (c) Voice recognition unit
(c) RAM and ROM (d) Voice band
(c) Peripheral memory 138. An error in the software or hardware is called a bug.
129. Which of the following is called as the flash memory? What is the alternative computer jargon for it?
(a) Static RAM (a) Leech (b) Squid
(b) Dynamic RAM (c) Slug (d) Glitch
(c) EEPROM 139. Errors in computer program are called

A S S E S S YO U R L E A R N I N G
(d) CD – ROM (a) Follies (b) Mistakes
130. Which of the following is a way to access secondary (c) Bugs (d) Spam
memory? 140. Modern computers are very reliable but they are not
(a) Random access memory (a) Fast (b) Powerful
(b) Action method (c) Infallible (d) Cheap
(c) Transfer method 141. Personal computers use a number of chips mounted
(d) Density method on a main circuit board. What is the common name
131. A CD-RW disk for such boards?
(a) Has faster access than other disks (a) Daughter board (b) Motherboard
(b) Is a type of optical disk (c) Father board (d) Dashboard
(c) Can be written only once 142. What is meant by a dedicated computer?
(d) Can be erased and rewritten (a) Which is used by one person only
132. EEPROM stands for (b) Which is assigned one and only one task
(a) Electrically Erasable Programmable Read-only (c) Which uses one kind of software
Memory (d) Which is meant for application software
(b) Easily Erasable Programmable Read-only 143. The system unit of a personal computer typically
Memory ­contains all of the following except
(c) Electronic Erasable Programmable Read-only (a) Microprocessor (b) Disk controller
Memory (c) Serial interface (d) Modem
(d) None of the above 144. Programs designed to perform specific tasks are
133. VGA denotes known as
(a) Video Graphics Array (a) System software
(b) Visual Graphics Array (b) Application software
(c) Volatile Graphics Array (c) Utility programs
(d) Video Graphics Adapter (d) Operating system

M08_MADAN 04_65901_C08.indd 47 23/12/22 7:38 PM


8.48 Chapter 8

145. The time during which a job is processed by the (a) Dynamic disk exchange
­computer is (b) Dodgy data exchange
(a) Delay time (b) Real time (c) Dogmatic data exchange
(c) Execution time (d) Down time (d) Dynamic data exchange
146. An approach that permits the computer to work on 157. An application program that helps the user to change
­several programs instead of one is any number and immediately see the result of that
(a) Online thesaurus change is
(b) Multiprogramming (a) Desktop publishing program
(c) Overlapped processing (b) Database
(d) Outline processor (c) Spreadsheet
147. The term gigabyte refers to (d) All of the above
(a) 1024 bytes (b) 1024 kilobytes 158. CAD stands for
(c) 1024 megabytes (d) 1024 gigabytes (a) Computer-Aided Design
148. The device that provides information which is sent (b) Computer Algorithm for Design
to the CPU is termed (c) Computer Application in Design
(a) Input (b) Output (d) All of the above
(c) CPU (d) Memory 159. Which of the following is required when more than
149. The type of media with varying capacities that is one person use a central computer at the same time?
used in the storage subsystem in a microcomputer is (a) Terminal (b) Light pen
(a) Memory or video (c) Digitizer (d) Mouse
(b) Magnetic or optical 160. Which of the following is used only for data entry
(c) Optical or memory and storage, and never for processing?
(d) Video or magnetic (a) Mouse
150. Which is considered a direct entry input device? (b) Dumb terminal
(a) Optical scanner (b) Mouse and digitizer (c) Microcomputer
(c) Light pen (d) All of the above (d) Dedicated data entry system
151. The computer code for interchange of information 161. Which of the following will be required to produce
between terminals is high-quality graphics?
(a) ASCII (b) BCD (a) RGB monitor (b) Plotter
(c) EBCDIC (d) All of the above (c) Inkjet printer (d) Laser printer
152. On the keyboard of a computer, each character has 162. Magnetic tapes can serve as
A S S E S S YO U R L E A R N I N G

an ASCII value which stands for (a) Secondary storage media


(a) American Stock Code for Information (b) Output media
Interchange (c) Input media
(b) American Standard Code for Infor­ mation (d) All of the above
Interchange
(c) African Standard Code for Information Interchange 163. If in a computer, 16 bits are used to specify address in
(d) Adaptable Standard Code for Infor­ mation a RAM, then the number of addresses will be
Interchange (a) 216 (b) 65,536
(c) 64k (d) Any of the above
153. Which part of the CPU performs calculations and
makes decisions? 164. The two major types of computer chips are
(a) Arithmetic logic unit (a) External memory chip
(b) Alternating logic unit (b) Primary memory chip
(c) Alternate local unit (c) Microprocessor chip
(d) American logic unit (d) Both (b) and (c)
165. What is the responsibility of the logical unit in the
154. Dpi stands for
CPU of a computer?
(a) Dots per inch (b) Digits per unit
(a) To produce result
(c) Dots pixel inch (d) Diagrams per inch
(b) To compare numbers
155. The process of laying out a document with texts, (c) To control the flow of information
­graphics, headlines and photographs is involved in (d) To do mathematical works
(a) Deck top publishing 166. The secondary storage devices can only store data
(b) Desktop printing but they cannot perform
(c) Desktop publishing (a) Arithmetic operations
(d) Deck top printing (b) Logical operation
156. The transfer of data from one application to another (c) Fetch operations
line is known as (d) Either of the above

M08_MADAN 04_65901_C08.indd 48 23/12/22 7:38 PM


Information and Communication Technology (ICT) 8.49

167. Which of the printers used in conjunction with 177. Multimedia is basically a/an
­computers uses dry ink powder? (a) Animation feature
(a) Daisy wheel printer (b) Line printer (b) Programming language
(c) Laser printer (d) Thermal printer (c) Technology
168. Which of the following produces the best quality (d) All of the above
graphics reproduction? 178. What do you need to put your web pages on the
(a) Laser printer (b) Inkjet printer WWW?
(c) Plotter (d) Dot matrix printer (a) A connection to the internet
169. A 32-bit microprocessor has a word length equal to (b) A web browser
(a) 2 bytes (b) 32 bytes (c) A web server
(c) 4 bytes (d) 8 bytes (d) All of the above
170. An error in computer data is called 179. Which is the largest unit of storage among the
(a) Chip (b) Bug following?
(c) CPU (d) Storage device (a) Terabyte (b) Megabyte
(c) Kilobyte (d) Gigabyte
171. A set of information that defines the status of
resources allocated to a process is 180. Which of the following is not a linear data structure?
(a) Process control (b) ALU (a) Array (b) Binary tree
(c) Register unit (d) Process description (c) Queue (d) Stack
172. Any method for controlling access to or use of mem- 181. Which of the following is not a network device?
ory is known as (a) Router (b) Switch
(a) Memory map (c) Hub (d) CPU
(b) Memory protection 182. Which of the following represents the binary equiva-
(c) Memory management lent of the decimal number 23?
(d) Memory instruction (a) 01011 (b) 10111
173. The accounting software Tally was developed by (c) 10011 (d) None of the above
(a) HCL (b) TCS 183. Computers on the internet are identified by
(c) Infosys (d) Wipro (a) Email address (b) Street address
174. The file extension of Excel spreadsheet is (c) IP address (d) None of the above
(a) .doc (b) .txt 184. An example of asynchronous medium is
(c) .xls (d) .ppt (a) Radio (b) Television
175. The XML stands for (c) Film (d) Newspaper

A S S E S S YO U R L E A R N I N G
(a) Extensible Markup Language 185. A message beneath a message is labelled as
(b) External Makeup Language (a) Embedded text (b) Internal text
(c) External Markup Logic (c) Intertext (d) Subtext
(d) External Magnitude Language 186. In analogue mass communication, stories are
176. The standard size of IP address is (a) Static (b) Dynamic
(a) 16 bits (b) 32 bits (c) Interactive (d) Exploratory
(c) 48 bits (d) 64 bits

The Internet and Email


187. What is the ‘Internet’? 189. In web search, finding a large number of documents
(a) A hardware networking technique with very little relevant information is termed
(b) World Wide Web (a) Poor recall
(c) Computers that are all linked together in a (b) Web crawl
network (c) Poor precision rate
(d) None of the above (d) Poor web response
188. Line access and avoidance of collision are main func- 190. Use of an ordinary telephone as an internet appli-
tions of ance is called
(a) CPU (a) Voice net (b) Voice telephone
(b) Monitor (c) Voice line (d) Voice portal
(c) Network protocols 191. Using websites to pour out one’s grievances is called
(d) Wide Area Networks (a) Cyberventing (b) Cyber ranting
(c) Web hate (d) Web plea

M08_MADAN 04_65901_C08.indd 49 23/12/22 7:38 PM


8.50 Chapter 8

192. Which of the following is the appropriate format of 202. Which of the following identifies a specific web page
URL of email? and its computer on the web?
(a) www_mail.com (b) www@mail.com (a) Website (b) Website address
(c) WWW@mail.com (d) www.mail.com (c) URL (d) Domain name
193. Which of the following statements is correct? 203. Which of the following terms applies to all the web
(a) Modem is a software. pages for Amazon.com?
(b) Modem helps in stabilizing the voltage. (a) Top-level domain (b) Website
(c) Modem is an operating system. (c) Website address (d) Web domain
(d) Modem converts analogue signals into digital 204. Given below are two statements—one is labelled as
and vice versa. Assertion (A) and the other is labelled as Reason
194. The term WWW represents (R). [Dec 2019]
(a) Who What And Where Assertion (A): The internet is a collection of inter-
(b) Weird Wide Web connected computer networks that are linked by
(c) Word Wide Web transmission medium such as copper wires, fiber-
(d) World Wide Web optic cables, wireless connections, etc.
195. HTML is basically used to design Reason (R): World Wide Web is a collection of inter-
(a) Web page (b) Website connected documents.
(c) Graphics (d) Tables and frames From the above two statements, choose the correct
answer.
196. Which of the following protocols manages the assem- (a) Both (A) and (R) are true and (R) is the correct
bling of a message or file into smaller packets that explanation of (A)
are transmitted over the internet and received by a (b) Both (A) and (R) are true but (R) is not the cor-
layer that reassembles the packets into the original rect explanation of (A)
message, and also handles the address part of each (c) (A) is true but (R) is false
packet so that it gets to the right destination? (d) (A) is false but (R) is true
(a) HTTP (b) TCP/IP
(c) FTP (d) All of the above 205. Given below are two statements—one is labelled as
Assertion (A) and the other is labelled as Reason
197. A set of guidelines or standards that establish the (R). [Dec 2019]
­format in which files can be transmitted from one
Assertion (A): The internet is a collection of inter-
computer to another is
connected computer networks that are linked by
(a) TCP (b) IP
transmission medium such as copper wires, fiber-
(c) FTP (d) All of the above
optic cables, wireless connections, etc.
A S S E S S YO U R L E A R N I N G

198. The protocol used specifically for exchange of Reason (R): World Wide Web is a collection of inter-
emails is connected documents.
(a) SMTP (b) TCP
(c) IP (d) None of the above From the above two statements, choose the correct
answer.
199. SMTP stands for (a) Both (A) and (R) are true and (R) is the correct
(a) Simplest Mail Transfer Protocol explanation of (A)
(b) Simple Mail Transfer Protocol (b) Both (A) and (R) are true but (R) is not the cor-
(c) Sample Mail Transfer Protocol rect explanation of (A)
(d) Separate Mail Transfer Protocol (c) (A) is true but (R) is false
200. Given the following email fields, which of the (d) (A) is false but (R) is true
email addresses will ‘swami’ be able to see when he 206. Look at the following statements.
receives the message? [July 2018] Statement 1: The language used for creating Web
To ... ram@test.com pages is HTML (Hyper Text Markup Language).
Cc ... raj@test.com; ravi@test.com Statement 2: All web pages are linked with each
Bcc ... swami@test.com; rama@test.com other with the help of Hyperlink and hypermedia.
(a) Statement 1 and statement 2 are true
(a) ram@test.com
(b) Statements 1 and 2 are false
(b) ram@test.com; raj@test.com; ravi@test.com
(c) Statement 1 is true and statement 2 is false
(c) ram@test.com; rama@test.com (d) Statement 1 is false and statement 2 is true
(d) ram@test.com; rama@test.com; raj@test.com;
ravi@test.com 207. Which of the following is ‘web server’?
(a) Apache
201. A word in a web page that when clicked opens (b) yahoo
another ­document is (c) Google Chrome
(a) Anchor (b) URL (d) Baidu
(c) Hyperlink (d) Reference

M08_MADAN 04_65901_C08.indd 50 23/12/22 7:38 PM


Information and Communication Technology (ICT) 8.51

208. Firewalls are used to protect a communication net- 220. Bit stands for
work against (a) Binary information term
(a) Unauthorized attacks (b) Virus attacks (b) Binary tree
(c) Data-driven attacks (d) Fire attacks (c) Binary digit
209. Which of the following ISP is free of cost? (d) Bivariate theory
(a) CompuServe (b) AOL 221. Which of the following is not a linear data structure?
(c) NetZero (d) None of the above (a) Array (b) Binary tree
210. Which of the following is virtual reality website? (c) Queue (d) CPU
(a) First Life (b) Second Life 222. A compiler is used to convert the following to object
(c) Gyan Life (d) None of the above code which can be executed.
211. Which of the following refers to an educational (a) High-level language
­website that allows its members to add or change its (b) Low-level language
information? (c) Assembly language
(a) Educational or .edu (d) Natural language
(b) Social networking site 223. Which of the following is not a network device?
(c) Wiki (a) Router (b) Switch
(d) Web 2.0 (c) Hub (d) CPU
212. If you want to create a broadcast that can be down- 224. What is the full form of HTTP?
loaded from the internet, then what would you create? (a) Hypertext Transmission Protocol
(a) iBroadcast (b) eShow (b) Hypertext Transfer Protocol
(c) Podcast (d) iCast (c) Hyperterminal Transport Protocol
213. Video transmission over the internet that looks like (d) Hyperterminal Transfer Protocol
delayed livecasting is called 225. The protocol dealing with transfer of hypertext
(a) Virtual video (b) Direct broadcast between two or more computers is
(c) Video shift (d) Real-time video (a) HTML (b) FTP
214. It is a very high-speed data transmission line that (c) HTTP (d) TCP
provides networking facilities to relatively small 226. IRC stands for
but high-speed internet service providers (includ- (a) Internet-Related Chat
ing commercial, educational, military, government (b) Internet Relay Chat
establishments) all around the world. It is known (c) Internet-Related Content
as (d) None of the above
(a) Pathway 227. Which of the following are types of internet

A S S E S S YO U R L E A R N I N G
(b) Backbone connections?
(c) Communication system (a) SLIP (b) DIAS
(d) Routers (c) PPP (d) All of the above
215. What type of telecommunication hardware allows 228. Internet’s initial development was supported by
you to access the web? (a) ARPANET (b) Bill Rogers
(a) Browser (b) Modem (c) Bill Gates (d) Microsoft
(c) FTP (d) IRC
229. The Internet Explorer was invented and released
216. VoIP technology converts voice calls from by
(a) Analogue to digital (b) Digital to analogue (a) Microsoft (b) WIPRO
(c) It depends (d) None of the above (c) Sun Microsystems (d) Infosys
217. What is the domain originally meant for non-profita-
ble organizations? 230. Gopher protocol is used to
(a) .gov (b) .org (a) Distributing documents through the internet
(c) .net (d) .in (b) Searching documents through the internet
(c) Retrieving documents through the internet
218. Which of the following is an internet connection (d) All of the above
option?
(a) Dial-up connection 231. World Wide Web was proposed by
(b) Digital subscriber line (a) Bill Gates (b) ARPANET
(c) Broadband (c) Tim Berners-Lee (d) Bill Rogers
(d) All of the above 232. What are the uses of the internet?
219. Which of the following is not related to information (a) Communication
security on the internet? (b) Information retrieval
(a) Data encryption (b) Watermarking (c) Presentation of information
(c) Data hiding (d) Information retrieval (d) All of the above

M08_MADAN 04_65901_C08.indd 51 23/12/22 7:38 PM


8.52 Chapter 8

233. .edu domain is used for (c) Hypertext Markup Language


(a) Educational institutions (d) HoT MeaL
(b) The internet infrastructure and service providers 246. A machine that links two networks using two differ-
(c) International organizations ent protocols is known as
(d) None of the above (a) Gateway (b) Protocol way
234. What is the full form of URL? (c) E-way (d) Gate
(a) Uniform Resource Library 247. Email is an abbreviation of
(b) Uniform Resource Locator (a) Electronic mail (b) Extra mail
(c) United Resource Library (c) Enhanced mail (d) None of the above
(d) United Resource Locators 248. ISP is an acronym of
235. The main advantages of an email is/are (a) Internal Service Provider
(a) Speed (b) Cost (b) Internet Service Procedure
(c) Record keeping (d) All of the above (c) Internet Service Provider
236. WWW provides standardized access to (d) Instant Service Protocol
(a) Gopher (b) Telnet 249. Web server is a software for
(c) FTP (d) All of the above (a) Analysing web traffic
237. Which of the following is not an internet browser? (b) Serving web page on user’s request
(a) Netscape Navigator (c) Crawling the web
(b) Chrome (d) Unix operating system
(c) Drupal 250. FTP stands for
(d) Opera (a) Full Text Processing
238. What is the World Wide Web? (b) File Transfer Program
(a) A computer game (c) File Transfer Process
(b) A software program (d) File Transfer Protocol
(c) Part of the internet that enables information 251. Telnet is
­sharing through interconnected pages (a) A telephone network
(d) Another name for the internet (b) A teller network
239. Which of the following is a search engine? (c) Standard Internet Protocol for remote login
(a) Macromedia Flash (d) Dialup program
(b) Google 252. LAN is the acronym for
(c) Netscape (a) Local Area Network
(d) Librarians’ Index to the Internet (b) Logical Area Network
A S S E S S YO U R L E A R N I N G

240. What is a URL? (c) Last Area Network


(a) A computer software program (d) None of the above
(b) A type of UFO 253. The term used to connect a number of computers
(c) Address of a document or page on the WWW. within the same location or in close proximity is
(d) An acronym for unlimited resources for learning (a) LAN (b) WAN
241. http://www.classzone.com is an example of what? (c) PAN (d) All of the above
(a) URL (b) Access code 254. Ethernet is a family of protocols used in
(c) Directory (d) Server (a) LAN (b) Internet
242. What is a browser? (c) Extranet (d) None of the above
(a) An HTML system 255. MAN is a network of computers located at different
(b) A server sites within a large fixed area, such as a city. It stands
(c) Application software for accessing and viewing for
web pages (a) Metropolitan Area Network
(d) None of the above (b) Manual Area Network
243. A chat program that allows people to communicate (c) Marginal Area Network
through the internet in real time is known as (d) Maximum Area Network
(a) Distant messaging (b) Instant messaging 256. A computer network in which some of the links
(c) Chatting room (d) All of the above between the nodes are carried by open connections
244. The first page of a website is called the or virtual circuits in some larger network, such as an
(a) Home page (b) Index internet, instead of physical wires is a
(c) Menu bar (d) None of the above (a) Virtual private network
245. HTML is an acronym of (b) Virtual public network
(a) High Tech Markup Language (c) Virtual private net
(b) How To Markup Language (d) Virtual public net

M08_MADAN 04_65901_C08.indd 52 23/12/22 7:38 PM


Information and Communication Technology (ICT) 8.53

257. The type of computer networking accessible to 267. Network-based email was initially exchan­ged on the
computers that are not physically part of an organi- (a) ARPANET (b) SMTP
zation’s own private network but are also not acces- (c) TCP/IP (d) None of the above
sible to the general public is 268. The type of digital telephone service used for trans-
(a) Internet (b) Intranet ferring large chunks of data to and from the internet
(c) Extranet (d) None of the above without a modem is
258. The name of the protocol that supports linking from (a) ISDN (b) MSDN
one web page to another page is (c) TSDN (d) None of the above
(a) HTML (b) IP 269. The term/protocol used for open wireless technol-
(c) HTTP (d) FTP ogy for data exchange over short distances is
259. The first part of a complete URL that is required to (a) ISDN (b) Bluetooth
access the web resource is (c) Wi-Fi (d) None of the above
(a) Address (b) Name 270. ISDN stands for
(c) Location (d) Protocol (a) Integrated Services Digital Network
260. An address for a specific document found on the web (b) Intelligent Services Digital Network
is termed (c) Individual Services Digital Network
(a) HTTP (b) URL (d) Image Services Digital Network
(c) ISP (d) None of the above 271. Wired Ethernet is basically
261. Which of the following organizes the web into (a) ADSL (b) ISDN
categories? (c) Broadband (d) Internet
(a) Search engine (b) Encyclopaedia 272. The software that permits a user with a click of mouse
(c) Archive (d) Directory to locate, display and download text, video, audio
262. A large number of computers in a wide geographical and graphics stored in a host computer on the web is a
area can be efficiently connected using (a) Web browser (b) Domain
(a) Twisted pair lines (c) URL (d) None of the above
(b) Coaxial cables 273. Which of the following are web browsers?
(c) Communications satellites    I. Internet Explorer
(d) All of the above II. Mozilla Firefox
263. A computer network allows sharing of resources. III. Opera
The software resources exist in the form of files of IV. Safari
data, which needs to be moved between two specific (a) I, II and III (b) II, III and IV
computer systems for the purpose of sharing (c) 1, III and IV (d) All of the above

A S S E S S YO U R L E A R N I N G
(a) Address for communication 274. Which of the following domain name extension is
(b) A secure means for moving data in the form of used for gateways and administrative hosts?
electronic signals (a) .gov (b) .mil
(c) Both (a) and (b) (c) .net (d) .org
(d) Either (a) or (b) 275. .com, .org and .net are popular as
264. The hardware and software combinations that con- (a) Top-level domains
nect devices running different native protocols are (b) Tax-level domains
known as (c) Terrific-level domains
(a) Protocols (b) Models (d) None of the above
(c) Gateways (d) Ports 276. The unique numerical address of a computer on
265. USB stands for the internet (expressed as four sets of numbers and
(a) United Serial Bus maximum three digits each, separated by dots, such
(b) Universal Serial By-Pass as 150.237.176.24) is termed
(c) Universal Serial Bus (a) Internet Protocol address
(d) Universal System Bus (b) URL
(c) ISP
266. Which of the following denotes the internet hard- (d) None of the above
ware requirements?
   I. Modem 277. The term used for a search tool that sends user
  II. Hub requests to several other search engines and/or
III. Bridge databases and aggregates the results into a single
IV. Router list or displays them according to their source is
V. Gateway (a) Metasearch engine
(a) I, II and III (b) I, III, IV and V (b) Google
(c) I, IV and V (d) I, II, III, IV and V (c) Moodle
(d) None of the above

M08_MADAN 04_65901_C08.indd 53 23/12/22 7:38 PM


8.54 Chapter 8

278. An internet email message consists of 288. A message with replies in a newsgroup is often
   I. Message envelope called a
II. Message header (a) List (b) Thread
III. Message body (c) Comment (d) Post
(a) I and II 289. S/MIME in the internet technology stands for
(b) II and III (a) Secure Multipurpose Internet Mail Extension
(c) I and III (b) Secure Multimedia Internet Mail Extension
(d) All of the above (c) Simple Multipurpose Internet Mail Extension
279. Which of the following symbols is in email addresses (d) Simple Multipurpose Internet Mail Extension
to separate the username from the ISP? 290. A small text file that a web server stores on a user
(a) $ (b) @ hard drive when the user visits certain websites is
(c) % (d) * called
280. The size of the IPv4 is (a) Cookie (b) History
(a) 16 bits (b) 32 bits (c) Logjam (d) None of the above
(c) 64 bits (d) 128 bits 291. Which of the following software allows users to surf
the internet?
281. Which of the following can be used to send emails to
(a) Search engine (b) Internet service provider
a large group at one time?
(c) Multimedia (d) Browser
(a) LISTSERV (b) Group
(c) Groupware (d) Mail server 292. Twitter is a/an
(a) Program
282. Which of the following lines gives an overview of (b) Micro bloging site
­message while sending an email? (c) (a) and (b) both
(a) To (b) Subject (d) None of these
(c) Contents (d) CC
293. Given below are two statements: [2021]
283. Unsolicited commercial email is usually referred
Statement I : Search Engines are websites that help a
to as
user to search for information on the worldwide web
(a) Junk (b) Hoaxes
(www).
(c) Spam (d) Hypertext
Statement II: For any search term, Search Engines
284. Which of the following is used by news servers to provide the exact same results to all the users.
­distribute documents to readers? In the light of the above statements, choose the
(a) Network News Transfer Protocol (NNTP) corrrect answer from the options given below:
(b) Hypertext Transfer Protocol
(a) Both Statement I and Statement II are true
A S S E S S YO U R L E A R N I N G

(c) File Transfer Protocol


(d) None of the above (b) Both Statement I and Statement II are false
(c) Statement I is true but Statement II is false
285. To search FTP archives in a file, the tool used is (d) Statement I is false but Statement II is true
(a) Gophe (b) Jughead
(c) Archie (d) Jalopy 294. In which year, was community radio started in India?
(a) 2001 (b) 2002
286. Which of the following is often referred to as ‘the CB
(c) 2003 (d) 2004
of the internet’?
(a) IRC (b) FTP 295. The file extension of MS Window sound file is
(c) Email (d) HTTP (a) .m (b) .c
(c) .wav (d) None of the above
287. Which of the following monitors postings and
enforces the site’s rules in context of discussion 296. RFID is an acronym for [Dec 2018]
groups? (a) Roaming Frequency Identification
(a) Judge (b) Sysop (b) Radio Frequency Identification
(c) Narrator (d) Censor (c) Runtime Frequency Identification
(d) Remote Frequency Identification

M08_MADAN 04_65901_C08.indd 54 23/12/22 7:38 PM


Information and Communication Technology (ICT) 8.55

Answer Keys

ICT Basics
1. (a) 2. (c) 3. (a) 4. (b) 5. (a) 6. (a) 7. (a) 8. (a) 9. (c) 10. (d)
11. (c) 12. (c) 13. (c) 14. (a) 15. (d) 16. (a) 17. (b) 18. (a) 19. (a) 20. (b)
21. (b) 22. (c) 23. (d) 24. (b) 25. (c) 26. (c) 27. (b) 28. (c) 29. (a) 30. (d)
31. (a) 32. (b) 33. (a) 34. (d) 35. (b) 36. (a) 37. (b) 38. (a) 39. (a) 40. (a)
Computer Terms
41. (d) 42. (a) 43. (d) 44. (d) 45. (b) 46. (b) 47. (c) 48. (d) 49. (c) 50. (b)
51. (c) 52. (b) 53. (c) 54. (c) 55. (b) 56. (a) 57. (b) 58. (a) 59. (b) 60. (c)
61. (b) 62. (d) 63. (a) 64. (c) 65. (a) 66. (c) 67. (d) 68. (a) 69. (c) 70. (a)
71. (a) 72. (a) 73. (a) 74. (c) 75. (c) 76. (c) 77. (c) 78. (a) 79. (d) 80. (b)
81. (a) 82. (a) 83. (a) 84. (b) 85. (b) 86. (b) 87. (b) 88. (d) 89. (b) 90. (d)
91. (a) 92. (b) 93. (c) 94. (c) 95. (a) 96. (d) 97. (b) 98. (a) 99. (a) 100. (a)
101. (b) 102. (a) 103. (c) 104. (d) 105. (d) 106. (b) 107. (a) 108. (c) 109. (b) 110. (c)
111. (d) 112. (c) 113. (c) 114. (c) 115. (b) 116. (b) 117. (a) 118. (d) 119. (d) 120. (b)
121. (b) 122. (c) 123. (c) 124. (d) 125. (a) 126. (a) 127. (b) 128. (b) 129. (c) 130. (a)
131. (d) 132. (a) 133. (a) 134. (a) 135. (b) 136. (c) 137. (a) 138. (d) 139. (c) 140. (c)
141. (b) 142. (b) 143. (d) 144. (b) 145. (c) 146. (c) 147. (c) 148. (a) 149. (b) 150. (d)
151. (a) 152. (b) 153. (a) 154. (a) 155. (c) 156. (d) 157. (c) 158. (a) 159. (a) 160. (b)
161. (b) 162. (d) 163. (b) 164. (d) 165. (b) 166. (d) 167. (c) 168. (c) 169. (c) 170. (b)
171. (d) 172. (b) 173. (b) 174. (c) 175. (a) 176. (b) 177. (c) 178. (d) 179. (a) 180. (b)
181. (d) 182. (b) 183. (c) 184. (d) 185. (d) 186. (a)
The Internet and Email
187. (c) 188. (c) 189. (a) 190. (c) 191. (a) 192. (b) 193. (d) 194. (d) 195. (a) 196. (b)
197. (c) 198. (a) 199. (b) 200. (b) 201. (c) 202. (d) 203. (b) 204. (b) 205. (b) 206. (a)
207. (a) 208. (a) 209. (c) 210. (b) 211. (c) 212. (c) 213. (d) 214. (b) 215. (b) 216. (a)
217. (b) 218. (d) 219. (d) 220. (b) 221. (b) 222. (a) 223. (d) 224. (b) 225. (c) 226. (b)
227. (d) 228. (a) 229. (a) 230. (d) 231. (d) 232. (d) 233. (a) 234. (b) 235. (d) 236. (d)

A S S E S S YO U R L E A R N I N G
237. (c) 238. (c) 239. (b) 240. (c) 241. (a) 242. (c) 243. (b) 244. (a) 245. (c) 246. (a)
247. (a) 248. (c) 249. (b) 250. (d) 251. (a) 252. (a) 253. (a) 254. (a) 255. (a) 256. (a)
257. (b) 258. (c) 259. (d) 260. (b) 261. (d) 262. (c) 263. (c) 264. (c) 265. (c) 266. (d)
267. (a) 268. (a) 269. (b) 270. (a) 271. (b) 272. (a) 273. (d) 274. (c) 275. (a) 276. (a)
277. (a) 278. (d) 279. (b) 280. (b) 281. (c) 282. (b) 283. (c) 284. (a) 285. (b) 286. (a)
287. (b) 288. (b) 289. (a) 290. (a) 291. (d) 292. (b) 293. (c) 294. (b) 295. (c) 296. (b)

M08_MADAN 04_65901_C08.indd 55 23/12/22 7:38 PM


This page is intentionally left blank

M08_MADAN 04_65901_C08.indd 56 23/12/22 7:38 PM


CHAPTER People,
9 Development and
Environment

01 The Concept of People

02 Development and Environment: Millennium


Development and Sustainable Development Goals

03 Human and Environment Interaction:


Anthropogenic Activities and Their
Impacts on Environment

04 Environmental issues: Local,


LEARNING Regional and Global; Air Pollution,
Water Pollution, Soil Pollution,
OBJECTIvES Noise Pollution, Waste (Solid,
Liquid, Biomedical, Hazardous,
Electronic), Climate Change and
its Socio-economic and Political
Dimensions

05 Impact of Pollutants on Human Health

06 Natural and Energy Resources:


Solar, Wind, Soil, Hydro, Geothermal,
Biomass, Nuclear and Forests

07 Natural Hazards and Disasters: Mitigation Strategies


Environmental Protection Act (1986), National Action Plan on
Climate Change, International agreements/efforts—Montreal
08 Protocol, Rio Summit, Convention on Biodiversity, Kyoto Protocol,
Paris Agreement, International Solar Alliance

M09_MADAN 07_65901_C09.indd 1 23/12/22 8:00 PM


9.2 Chapter 9

Ecology is the scientific study of the relations that liv-


People and Environment ing organisms have with respect to one another, their nat-
“Mother Earth” is a common expression in a number of coun- ural environments and ecosystems.
tries and regions, Population is a near-permanent group of It is present at three levels, which are as follows:
interbreeding individuals of a species found in space or geo- 1. The individual organism (how individuals are affected
graphical area at a particular point in time. It is called a local by the environment and how they, in turn, affect the
population or deme. Metapopulation is a complex of local environment)
populations connected by dispersing individuals. 2. The population
The main factors affecting population are as follows: 3. The community
1. Natality (birth rate)
2. Mortality (death rate) Ecology is defined as the study of ecosystems.
3. Population dispersal (emigration, immigration and Ecological Footprint: Ecological footprint measures
migration) human consumption of natural resources in comparison
4. Age distribution (pre-reproductive, reproductive and to the earth’s ecological capacity to regenerate them.
post-reproductive) Calculation of ecological footprint takes into account our
5. Population growth rate consumption habits at both (i) micro (individual) level
6. Carrying resources on earth such as food, water, and (ii) macro (area or nation) level. The objective of
space etc. are expressed through carrying capacity. its calculation is also to educate people about the need
Population density is the number of individuals divided to change our consumption behaviour to make it more
by space (such as per square kilometre). sustainable. Ecological footprint is measured in global
The term environment is derived from the French hectares (gha).
word environner. It means ‘to surround’. According to
the Environment (Protection) Act, 1986, environment SpecieS and community
includes all the physical and biological surroundings The organisms in this world can be divided into different
of an organism and their interactions. Environment is species; just as human beings are a species, so are roses
defined as the sum of water, air and land, and the inter- and neem trees.
relationships that exist among them and with human A species is a set of organisms that resemble each other
beings, other living organisms and materials. in certain features. The members of a species living together
The flora, fauna, microorganisms and the man-made and interacting with each other are called a population.
structures in our surroundings have a bidirectional interac- The members of a population live within a given area.
tion with us, either directly or indirectly. The totality of all
these components and their interactions constitute the envi- Autoceology and Synecology: Autecology is the study
ronment. Air, water and land constitute our environment of individual organisms / species, while synecology is the
and influence us directly. We too exert an influence on our study of group of organisms of different species.
environment due to overuse or overexploitation of resources Species Diversity
or due to discharge of pollutants in air, water and land.
In the context of relation between people and environ- • It is an important attribute of the biotic community,
ment, the following concepts have developed: which is determined by the total number of species
and their relative abundance.
1. Environmental Determinism: It means that envi- • Greater species diversity indicates a higher number of
ronment has been determined by culture and nature. niches and greater stability of the community.
It developed in nineteenth-century Europe, a belief
that environment determines how a culture develops. Keystone Species
For example, white European nations believed that • It is a species that has a significant and dispropor-
people from warmer climates were lazier because tionately large influence on community structure and
they did not have to work as hard to survive. characteristics.
2. Environmental Possibilism: It developed in the first • It has often considerably low abundance and biomass
half of the twentieth century. It is the belief that the as compared to dominant species.
environment puts limits on people, but it does not deter-
mine how they will behave. This can be further linked Critical Link Species
with ‘anthropocene’ age and ‘anthropogenic’ activities • These are species that play an important role in sup-
also where man plays greater role for the development porting network species as pollinators, dispersal
of self and society. These have been discussed further. agents, absorption or circulation of nutrients, etc.
Mycorrhizal fungi help vascular plants in obtaining
inorganic nutrients from soil and organic residues.
Ecology
The term ecology was coined by Ernst Haeckel in 1869. Community
Ecology deals with the study of organisms in their natural A community is an assemblage of all the interacting popula-
home. tions of different species in a geographical area. It is a com-

M09_MADAN 07_65901_C09.indd 2 23/12/22 8:00 PM


People, Development and Environment 9.3

plex interacting network of plants, animals and microorgan- Each level in the transfer of energy is a trophic level.
isms. Each population has a defined role in the community. Organisms at each level use energy in cellular respira-
tion and heat loss and store the rest.
Stopover Food chains are generally found to be interlinked and
Population consists of: interwoven as a network and hence known as a food
(a) Organisms in a species that are capable of repro- web. There are several options of eating and being
ducing among themselves eaten in a food web. Hence, these are more stable.
(b) Different species that can interbreed 2. There is unidirectional flow of energy. It flows from the
(c) Collective members of a species living and inter- sun and then after capture by primary producers (green
acting across the biosphere plants), it flows through the food chain or the food web.
(d) Some organisms that may not belong to any species 3. Materials (nutrients) in an ecosystem move in a cyclic
The correct option is (c). manner. The cycling of nutrients takes place between
biotic and abiotic (water, light, wind, soil, humidity,
minerals and gases) components.
Ecosystem 4. Every ecosystem functions to produce and sustain
Ecosystem was defined by Arthur Tansley in 1935. some primary production (plant biomass) and sec-
Ecosystem is a self-regulating community of living organ- ondary production (animal biomass).
isms (populations of species), interacting with each other 5. Every ecosystem regulates and maintains itself. This
and their non-living, physical environment, for example, self-regulation or control system is known as cyber-
forest ecosystem and ocean ecosystem (Fig. 9.1). Even a netic system.
clump of bushes can be an ecosystem. All ecosystems on Ecotone
the planet are interconnected and interdependent, and
An ecotone is the transitional area of vegetation between
together, they make up the biosphere.
two different plant communities, such as forest and grass-
There is also an exchange of matter and energy with the
land. The influence of the two bordering communities on
physical environment. In an open ecosystem, there is free
each other is known as the edge effect.
exchange of energy and matter with the outside world. In
a closed ecosystem, there is no or very limited exchange. Ecological Succession: In ecological succession, ecosystems
Ecosystems show large variations in their size, struc- tend to change over a period due to seasonal environmental
ture, composition and so on. However, all the ecosystems changes. The changes may be man made (anthropogenic) or
are characterized by certain basic structural and func- natural. Some changes may take decades.
tional features which are common. There can be differ-
ent types of ecosystems, such as forest ecosystem, marine trophic LeveLS and ecoLogicaL pyramidS
ecosystem and desert ecosystem. The concept of ecological pyramid was developed by
The composition and organization of communities and Charles Elton. All ecological pyramids begin with produc-
physical components decide the structure of an ecosys- ers like plants at the bottom and proceed through various
tem. Thus, ecosystems have basically two types of compo- trophic levels such as herbivores (consume plants) and car-
nents, namely, biotic and abiotic. nivores (consume herbivores) (Fig. 9.2).
Ecological pyramids show how energy flows through
FunctionS oF ecoSyStem ecosystems. The base of the pyramid is composed of
Every ecosystem performs the following important species called autotrophs, the primary producers of the
functions: ecosystem. All other organisms in the ecosystem are
consumers called heterotrophs, which either directly
1. It has different food chains and food webs. Food chain
is the sequence of eating and being eaten.
A food chain is the simplest path that energy takes Solar energy
through an ecosystem. Energy enters from the sun.

ers
consum
Tertiary arnivores)
a rg e c
(l
Grass Grasshopper
Hawk Frog Snake mers
consu s)
Grass Grass Hawk Hawk ndary
Seco ll carnivo
re
(sma

ary )
Prim erbivores
Grasshopper Frog Snake Grass Hawk cons
ume rs (h

Grasshopper Grasshopper
Frog Snake
Frog Snake ucers
Prod t)
(plan

Figure 9.1 Ecosystem Figure 9.2 Ecological Pyramid

M09_MADAN 07_65901_C09.indd 3 23/12/22 8:00 PM


9.4 Chapter 9

2. Mesotrophic: ‘With intermediate nutrient


concentration’
3. Eutrophic: ‘High in nutrients’ (they result in
Tertiary supra-optimum growth of plants)
consumers 4. Nypertrophic: ‘Very high in nutrients’
10 kcal
Dystrophic means water bodies rich in undecom-
posed organic matter, for example, bogs and marshy
Secondary lakes.
consumers Algal blooms are very low in the oligotrophic level.
100 kcal Energy ‘lost’
Oligotrophic Mesotrophic Eutrophic Hypereutrophic
Primary
consumers
1000 kcal
Producers
10,000 kcal Energy transferred

Figure 9.3 Energy Pyramid

or indirectly depend on the primary producers for food


energy.
There are three types of ecological pyramids and they
are as follows: Biosphere
There are different types of ecosystems around us, which
1. Pyramid of Energy: In this pyramid, at each trophic involve living organisms and non-living things. If we com-
level only a small proportion of energy (approxi- bine all the ecosystems present on the earth, it is called a
mately 10%) is transferred to the next level. This is biosphere.
called ‘10% rule’. This pyramid describes the over-
all nature of the ecosystem. The energy pyramid is Biomes
always upright and vertical as shown in Figure 9.3. The terrestrial portion of the biosphere is divided into
The higher the organism is on the trophic pyramid, biomes. They usually have distinct climates and life forms
the lower the amount of available energy. For exam- adapted to that climate. Deserts, grasslands, tropical for-
ple, plants and other autotrophs (primary produc- ests and rain forests are the main examples of biomes.
ers) convert only a fraction of the enormous amount
of solar energy they have access to into food energy. Habitat
2. Pyramid of Numbers: This depicts the relation in terms The area or natural environment in which an organism or
of the number of producers, herbivores and carnivores population normally lives is called a habitat (Fig. 9.4).
at their successive trophic levels. They can be of three A habitat is made up of two types of factors:
types—upright, partly upright and inverted.
3. Pyramid of Biomass: In this pyramid, there is a grad- 1. Physical factors such as soil, moisture, temperature
ual decrease in the biomass from the producers to the and light
higher trophic levels. They show the mass and can be 2. Biotic factors such as availability of food and preda-
upright and inverted. tors
Cohabitat is when organisms of different species live
Concept Box together.
Eutrophication describes the biological effects of an Microhabitat is when particular organism(s) live in
increase in the concentration of nutrients. These ‘nutri- a small and specific part(s) of a habitat. A habitat is not
ents’ are essential for growth. Eutrophication is mostly always a geographical area.
caused by increase in the availability of nitrogen and
phosphorus in the form of nitrate and phosphate in soil Stopover
and water. Permafrost is the defining characteristic of which
Phytoplankton communities in water columns biome?
reflect growth pattern. There can be some alterations (a) Taiga
in concentrations of plant nutrients that may create (b) Tundra
different trophic levels mentioned as follows: (c) Grassland
1. Oligotrophic: ‘Low in nutrients’ (oxygen-rich bot- (d) Desert
tom supports cold water fish such as trout) The correct option is (b).

M09_MADAN 07_65901_C09.indd 4 23/12/22 8:00 PM


People, Development and Environment 9.5

The part of Biosphere


Biosphere the earth
that contains
all ecosystems
Large region
Biome with same
plant life and
climate
Community Hawk, snake, bison, prairie dog,
Eco- and its grass, stream, rocks, air
system non-living
surroundings
Populations Hawk, snake, bison, prairie dog, grass
Community that live
together in
a defined area
Group of
Population organisms of
one type that live
in the same area
Bison herd
Individual
Organism
living thing

Bison
Figure 9.4 Habitat

Four Spheres of Earth Mesosphere


The area near the surface of the earth can be divided into The mesosphere starts just above the stratosphere and
four interconnected geospheres: extends to 85 km high. Meteors burn up in this layer.
1. Lithosphere (soil): It is solid, inorganic and composed Thermosphere
of minerals.
2. Hydrosphere (water): It consists of oceans, rivers, The thermosphere starts just above the mesosphere and
lakes and moisture. Oceans consist of 97% of the total extends to 600 km high. Aurora and satellites occur in
water and fresh water is only 3%. this layer.
3. Biosphere (life): It consists of all living organisms. Life is
present 3 m below the ground and up to 30 m above the Ionosphere
ground. It is present in the top 200 m of oceans and seas. The ionosphere is an abundant layer of electrons and
4. Atmosphere (air): The atmosphere can be described ionized atoms and molecules that stretches from about
as a layer of gases surrounding the earth. These gasses 48 km above the surface to the edge of space at about
are oxygen, carbon dioxide, nitrogen, argon and other 965 km, overlapping into the mesosphere and thermo-
gases. Another component of atmosphere which is of sphere. This dynamic region grows and shrinks based
great importance is the water vapour. on solar conditions and divides further into the sub-
regions: D, E and F, based on what wavelength of solar
Atmospheric Layers radiation is absorbed. The ­ionosphere is a critical link in
Troposphere the chain of sun–earth interactions. This region is what
makes radio communications possible.
The troposphere starts at the earth’s surface and extends
8–14.5 km high. This part of the atmosphere is the densest.
Exosphere
Almost all weather is in this region.
This is the upper limit of our atmosphere. It extends from
Stratosphere the top of the thermosphere up to 10,000 km.
The stratosphere starts just above the troposphere and This layer also absorbs heat that is reflected back from
extends to 50 km high. The ozone layer, which absorbs and the ground in a process called the greenhouse effect.
scatters the solar ultraviolet (UV) radiation, is in this layer. ‘Shooting stars’ also occur in this layer.

M09_MADAN 07_65901_C09.indd 5 04/01/23 4:52 PM


9.6 Chapter 9

2. The idea of limitations imposed by technology and


social factors
Sustainable development presupposes the existence of
space and time.
The concept of human development was largely devel-
oped by the United Nations Development Programme in
the 1990s.
Human development is defined as a process of enlarg-
ing people’s choices. If these choices are available, other
opportunities will become accessible.
The term ‘human development’ refers to the process of
widening people’s choices and ensuring well-being. Thus,
human development has two sides as follows:
1. Formation of human capabilities, such as improved
health, knowledge and skill
2. To make use of acquired capabilities for productive
Figure 9.5 Layers of Atmosphere purposes, leisure and being active in cultural, social
and political affairs
The major elements in the concept of human develop-
Stopover ment are as follows:
In which of the following layers of the atmosphere is 1. Productivity: Full participation in the process of
ozone (O3) gas present? income generation and employment
(a) Stratosphere 2. Equity: Equal opportunities for people
(b) Mesosphere 3. Sustainability: Physical, human and environment
(c) Troposphere capital for present and future generations
(d) None of the above 4. Empowerment: Full participation of people in the
The correct option is (a). decisions and processes that shape their lives

un conceptS oF human deveLopment–hdi


Human Development and We get the basic idea of HDI from the given image. The
Environment HDI for the countries of the world is scored and ranked
on a scale of 0 to 1.0, with 1.0 being the highest human
Human development refers to the biological and psycholog- development. HDI is divided into four levels:
ical changes that occur in human beings between birth and
the end of adolescent period as an individual progresses 1. Very High Human Development (0.8-1.0)
from dependency to increasing autonomy. The key princi- 2. High Human Development (0.7-0.79)
ple of sustainable development underlying all others is the 3. Moderate Human Development (0.55-.70) and
integration of environmental, social and economic concerns 4. Low Human Development (below 0.55)
into all aspects of decision making. The first HDI was published in 1990.
There are four areas in which we grow: On the basis of information released on December
1. Physical 2020, India is ranked 131st out of 189 countries in the
2. Psychological and cognitive United Nations Human Development Index (UN-HDI),
3. Social and emotional which is two places less than last year, although this
4. Sexuality and gender identity year India’s The HDI value has increased from 0.642 to
0.645. Norway topped the HDI index followed by Ireland,
We use different methods, such as longitudinal method,
Switzerland, Hong Kong and Iceland. Somalia ranked
cross-sectional method, sequential and time lag method,
lowest.
for development purpose. There are always some obsta-
The Life expectancy of Indians at birth in 2019 was
cles to apply the research methods because human beings
69.7 years while Bangladesh has a life expectancy of 72.6
as samples are not perfect representatives. There are many
years and Pakistan 67.3 years.
variations within human beings.
The United Nations Development Program (UNDP)
Sustainable development is development that meets
for the first time introduced a new metric to reflect each
the needs of the present without compromising the ability
country’s per capita carbon emissions and the impact of
of future generations to meet their own needs. It contains
its physical footprint, the amount of fossil fuels, metals
two key concepts:
and other resources needed to make up the consumable
1. The concept of needs material. This gives us a clear assessment.

M09_MADAN 07_65901_C09.indd 6 23/12/22 8:01 PM


People, Development and Environment 9.7

Figure 9.6 Human Development Index

I nequality -adjusted Human Development Multidimensional Poverty Index (MPI)


Index (IHDI) The MPI is a very versatile methodology that can be readily
The difference between the IHDI and the HDI is the human adjusted to incorporate alternative ­indicators, cut-offs and
development cost of inequality, also termed loss to human weights that might be appropriate in regional, national or
development due to inequality. The IHDI informs poli- subnational contexts. There are currently two broad cat-
cies towards inequality reduction and it leads to a better egories of MPI measures–(a) Global MPI and (b) Regional
understanding of inequalities across population and their or National MPIs.
contribution to the overall human development cost.
Millennium Development and Sustainable
G ender Development Index (GDI) Development Goals
The GDI measures gender gaps in human development Today, people, nations and economies are more connected
achievements by accounting for disparities between than ever, and so are the global development issues we
women and men in three basic dimensions of human are facing. These issues span borders and straddle social,
development, health, knowledge and living standards, economic and environmental realms.
using the same component indicators as in the HDI. It is Some important aspects are as follows:
the ratio of the HDIs calculated separately for females and 1. 1798: Known for his work on population growth,
males using the same methodology as in the HDI. Thomas Robert Malthus argued that, left unchecked,
a population will outgrow its resources. There are two
G ender Inequality Index (GII) ways to ‘check’ a population:
The GII is an inequality index. It shows the loss in poten- (a) preventive checks, such as the moral restraint of
tial human development due to disparity between female postponing marriage
and male achievements in three dimensions: (b) positive checks, such as famine, disease and
warfare
1. Reproductive health
2. 1968: Garrett James Hardin warned of the dangers
2. Empowerment
of human overpopulation. He wrote ‘The Tragedy
3. Economic status
of Commons’—it is about the damage that innocent
Overall, the GII reflects how women are disadvantaged in actions by individuals can inflict on environment.
these dimensions. 3. Club of Rome: It is a global think-tank for welfare of
The GII ranges between 0 and 1. Higher GII values indi- masses—linked to sustainable development.
cate higher inequalities and thus higher loss to human 4. 1972: Club of Rome published ‘Limits to Growth’ .
development. There is no country with perfect gender 5. 1972: UN Conference on the Human Env­ironment,
equality. All countries suffer some loss in achievements in also called Stockholm Conference, was conducted.
key aspects of human development when gender inequal- 6. 1983: The UN created the Brundtland Commission.
ity is taken into account. The GII is similar in method to 7. 1984: Brundtland Commission report was published.
the IHDI. 8. 1987: Our Common Future was published.

M09_MADAN 07_65901_C09.indd 7 23/12/22 8:01 PM


9.8 Chapter 9

9. During Rio Earth Summit in 1992, Agenda 21 was A comparison of MDGS and SDGs: After the comple-
adopted as a comprehensive plan of action to build a tion of MDGs, we needed to progress to the next step that
global part for sustainable development to improve were called Sustainable Development Goals. The number
human lives and protect the environment. of goals in MDGs are 8, the number of targets is 21 and
10. The Millennium Development Goals were started in number of indicators is 60. The same for SDGs are 17,
2000 which focused on three areas with the intent to 169 and 232 respectively. The main differences between
increase living standards - Human capital, infrastruc- MDGs and SDGs have been mentioned below.
ture and human rights. They are collectively called as
1. Zero Goals: The target in the MDGs was to reach half-
‘Goal Eight’, under the name of ‘Global Partnership for
way, but the SDGs have been designed for completing
Development’. There were 21 targets and 60 measurable
the job. The goals are supposed to achieve zero level,
‘health indicators’ and ‘economic indicators’.
for examples, zero hunger or zero preventable child
11. There was reaffirmation of Sustainable Development
deaths.
and Plan of Implementation in Johannesburg declara-
2. Universal Goals: The strategy employed in case of
tion at World Summit in 2002.
MDGs was more of ‘rich donors aiding recipients’ nature.
12. The motto of ‘The Future We Want’ was adopted at UN
Many nations such as China, India, Brazil and African
Conference on Sustainable Development (Rio+20) in
progressed. The problems of inequalities also existed in
Rio de Janeiro, Brazil, in June 2012.
the developed nations. Thus, we should not focus only
13. In 2013, the General Assembly set up a 3 ­ 0-member
on rising inequalities between rich and poor countries
open working group to develop a proposal on the
only. The SDGs cover the targets for all nations.
Sustainable Development Goals (SDGs) that led to UN’s
3. Being Comprehensive and Inclusive: The SDGs
General Assembly’s post-2015 development agenda.
have better coverage than the MDGs with respect to
14. 2015 was a landmark year for multilateralism and
gender and people’s participation at all levels. The
international policy shaping, with the adoption of
MDGs had been formulated in a top-down manner,
several major agreements:
while the SDGs have been prepared through much
(a) Sendai Framework for Disaster Risk Reduction better consultations among nations. There is segre-
(March 2015) gation of goals in SDGs. For example, Hunger and
(b) Addis Ababa Action Agenda on Financing for Poverty have been put as two separate goals in SDGs.
Development (July 2015) They were put together in MDGs.
(c) Transforming our world (the 2030 Agenda for    The goals under 4, 5 and 6 can be linked with Goal 6
Sustainable Development with its 17 SDGs was of SDGs. Similarly Goals 7 to 15 on SDGs can be linked
adopted at the UN Sustainable Development with Goal 7 of MDG. We can see to these while discussing.
Summit in New York on September 2015) 4. Funding: The MDGs depended upon the aid from
(d) Paris Agreement on Climate Change (December rich developed nations. But SDGs focus on ‘sustain-
2015) ability’ aspect. SDGs also look for inclusive economic
development and revenue generation capabilities so
that nations can deal with their social challenges in a
Millennium Development and better manner.
Sustainable Development Goals 5. Peace Building: Peace is an essential element in a
society to prosper and grow socially as well as eco-
(MDGS and SDGs) nomically. The SDGs have peace as a goal, whereas
MDGs are eight goals that were to be achieved by the year MDGs have not focused upon it.
2015. They were announced in September 2000 to fight 6. Data Revolution: Monitoring, evaluation and
against poverty, hunger, disease, illiteracy, environmental accountability are better in case of SDGs in com-
degradation and discrimination against women. MDGs parison to MDGs for income, age, race, ethnicity and
focused on three areas to increase living standards - migratory status, disability, geographic location and
Human capital, infrastructure and human rights. These other characteristics relevant in national contexts.
‘MDG Eight’ were the following. 7. Quality Education: MDGs were confined to achieve
universal primary education. SDGs are concerned
1. Eradicate poverty and hunger with quality of education and higher aspects. There
2. Achieve universal primary education are better indicators to address the issues.
3. Promote gender equality and empower women
4. Reduce child mortality Nature of Sustainable Development
5. Improve maternal health Goals (SDGs)
6. Combat HIV/AIDS, malaria and other diseases
Sustainable development means that the needs of the
7. Ensure environmental sustainability
present without compromising the ability of future
8. Develop a global partnership for development.
generations to meet their own needs. Thus, it built
At the end of MDGs, we shifted our goals towards SDGs. on ‘The Future We Want’ resolution. SDG is officially
We need to go for a comparison so that we can understand known as ‘Transforming our world: the 2030 Agenda for
the shift mechanism better though basics are same. Sustainable Development.’

M09_MADAN 07_65901_C09.indd 8 23/12/22 8:01 PM


People, Development and Environment 9.9

According to UNDP ‘SDGs also known as the Global in April 2022, with extremely poor people have been
Goals, were adopted by the UN in 2015 as a universal call halved in number.
to action to end poverty, protect the planet, and ensure (a) The limit for deciding poverty is below $1.9 on
that by 2030 all people enjoy peace and prosperity. the Purchasing Power Parity.
We are living in a ‘anthropocene age’ that can be called
(b) The Tendulkar Committee was set up in 2009 and
as ‘human driven age’ that puts extreme pressure on
C Rangarajan Committee in 2012 for defining
resources and natural cycles of earth. Examples of this
poverty limits.
include submerging of islands and mass migrations.
SDG 2: Zero Hunger – End hunger, achieve food security,
India’s approach - India’s philosophy is based ‘Vasudhaiva
improve nutrition and promote sustainable agriculture
Katumbhkam’ (olq/kSo dqVEq cde~)–whole of society, it aims to
engage all the stakeholders, and ‘leave no one behind’. Legal entitlements: 40% of child deaths occur due to
National Institution for Transforming India (NITI under-nutrition in India. So, domestic food production, its
Aayog) prepares a National Review Report that is imports and its access to population is important. There
submitted to UN. It has been called the “Decade for Action”. are food security measures (legal entitlements) under
India is the first nation to formulate an ‘SDG India Index’, National Food Security Act (NFSA), 2013. Antyodaya
and has developed a robust ‘SDG Localisation Model’. SDG Anna Yojana (AAY) for is meant for eradication of hunger.
India Index provides us the overall progress of our nation
on the social, economic and environmental status of the Nutrition Security: These are through Pradhan Mantri
nation. It covers 13 out of 17 SDGs (leaving out Goals 12, Matru Vandana Yojana and the Scheme for Adolescent Girls.
13, 14 and 17). There are 62 National Indicators. Agricultural Productivity: Doubling the income of farm-
The goals are reflected in social, economic and politi- ers through improved crop productivity, livestock produc-
cal agenda of government. They are linked with ‘Directive tivity, increased cropping intensity, crop diversification,
Principles of State Policy’ as well as various other policies better prices; and non-farm occupations. Government has
and budgets. also started ` 500 cr Accelerator Funds for Agri startups in
Thus, in the following SDG goals, we have taken exam- October 2022.
ples from Indian perspective.
Monitoring: The nutritional requirements of women,
SDG 1: No Poverty–End poverty in all its forms every­ their stunted growth and anaemia can be monitored
where by 2030: Bringing the level of poverty to ‘Zero’ is the in situations such as adaptive climate and sustainable
most daunting task. agriculture.
Govt has taken a multi-pronged strategy to remove pov-
erty from vulnerable sections of the society by developing Global Hunger Index Report 2022 placed India at 107th posi-
infrastructure, services, assets, skills, protective measures, tion among 121 nations, though India rejected this report.
etc. SDG 3: Good Health and Well-being – Ensure healthy
lives and promote well being for all at all ages
Social Protection and Safety Nets: The example is India is the largest provider of vaccines and drugs as
Mahatma Gandhi National Rural Employment Guarantee we could see during COVID19 situation. India started
Act (MNREGA) that allows for 100 days of work in a year National Health Policy, 2017.
per family.
Important Govt Schemes: For Universal and Affordable
The goals of Sustainable Development are access to Healthcare, government has started ‘Ayushman Bharat’
basic services. We need access to nutrition, healthcare, (National Health Protection Scheme) in 2018 that is an
drinking water, sanitation, affordable housing, electricity, inclusive health care without any cost that covers poor
clean cooking, roads and utilities. The financial inclusion sections of the society.
is also included in it that provides banking, credit, Pradhan Mantri Jan Aarogya Yojana is cashless health-
insurance and pension. Government is providing schemes care scheme to provide benefits to 100 million vulnerable
such as Pradhan Mantri Jan-Dhan Yojana; and Pradhan families. Government provides INR 5 lakhs per family per
Mantri Gareeb Kalyan Yojana. year in such conditions.
Strengthening Livelihood Opportunities and Skilling Doctor Patient Ratio: Government aims to set this ratio
Ecosystem: Agriculture provides employment to almost to 1:1000. The current such ratio is 1:1456. Government is
half of our population. Schemes such as Pradhan Mantri opening a number of AIIMS, medical colleges, and primary
Fasal Bima Yojana (Prime Minister Crop Insurance health centres so as to improve the basic infrastructure.
Scheme) and Pradhan Mantri Krishi Sinchayee Yojana In context of Maternal Mortality Ratio (MMR),
(Prime Minister Irrigation Scheme) help in this. We need Government wants to reach a target of 70 by 2030. As
to develop ‘ecosystem’ for more productive employment. per data available in March 2022–there are 7 states
We need continuous monitoring for rapid urbanisation, achieving this target of 70 which are led by Kerala (30),
gender equality, education, employment, etc. Maharashtra (38), Telangana (56).
India could pull the last 10% of its population out of India has seen a decline in all communicable and
extreme poverty well within last one decade. India is no non-communicable diseases like polio (zero patients),
longer a ‘poster boy of poverty’ as per World Bank findings Malaria, Tuberculosis and Lymphatic Filariasis.

M09_MADAN 07_65901_C09.indd 9 23/12/22 8:01 PM


9.10 Chapter 9

SDG 4: Quality Education—Ensure inclusive equitable a campaign ‘Jal Shakti Abhiyan’ for water conservation,
quality education and promote lifelong learning rain water harvesting and renovation of water bodies.
opportunities for all Central Water Commission has designed multi pronged
‘Leaving no one behind’ is the main agenda. Human strategy to deal with ground and surface water issues.
capital is required so that all children are able to get sec- Under National Perspective Plan, 30 river links (16
ondary education. under Peninsular Component and 14 under Himalayan
For early education, there are schemes such as National Component) have been identified. Ken-Betwa Link proj-
Early Childhood Care and Education Policy (2013), ect is the first such project to have been implemented.
Anganwadi Centres and Integrated Child Development Swachh Bharat Mission aims at constructing 109
Services Scheme. million household and community toilets for ‘sanitation
Education was made a Fundamental Right under The and hygiene for all’.
Right of Children to Free and Compulsory Education
(RTE) Act, 2009 for children up to 14 years. SDG 7: Affordable and Clean Energy—Ensure access
The government scheme Pradhan Mantri Kaushal to affordable, reliable, sustainable and modern energy
Vikas Yojana is meant for better livelihood. for all
New Education Policy, 2020 aims to achieve ‘Functional This aims at universal access to electricity and clean
Literacy and Numeracy Goals’ by 2026–27. The key aspect cooking fuel. Government wants to enhance energy
areas are NEP Child, NEP Teacher and NEP School. Some efficiency through Bureau of Energy Efficiency (BEEE)
of the important points are as follows. that was launched on March 1, 2002 under Energy
Conservation Act, 2001. Our energy mix is well diversified
(a) States have been asked to ensure 100% Gross
now, with inclusion of more of solar, wind, hydro and
Enrollment Ratio (GER) by 2030.
waste-to-energy conversion. Our renewable energy
(b) There should be introduction of School Preparation
installed capacity has almost doubled in last one decade.
Module (Vidya Pravesh) for every Grade 1 child
There is Pradhan Mantri Sahaj Bijli Har Ghar Yojana –
from 2022–23.
Saubhagya that aims at providing electricity to each house.
(c) Early Childhood Care and Education, and a bilin-
There are concepts such as Clean Energy-Cooking
gual teaching approach was suggested as soon as
Energy, Concept of Sustainable Development Energy
possible.
Efficiency. Government played an important role in
National Intellectual Property Awareness Mission indi- designing and implementing International Solar Alliance
cates a knowledge based economy. (ISA) with France.
SDG 5: Gender Equality – Achieve gender equality and SDG 8: Decent Work and Economic Growth – Promote
empower all women and girls sustained, inclusive and sustainable economic growth,
MNREGA (2005) specifies that women must comprise full and productive employment and decent work for all
of 33 per cent of the beneficiaries of gainful employment. India has the third largest entrepreneurship ecosystem
Deen Dayal Antyodaya Yojana (National Rural in the world due to the following:
Livelihoods Mission), specifically for building up women
skills. Self-Help Groups (with specific help from National 1. Funding Small Enterprises: ‘Start-up India’ schemes.
Bank on Agriculture and Rural Development (NABARD) Micro Units Development and Refinance Scheme
are also important for women. In ‘Pradhan Mantri Jan- (MUDRA) scheme provides financial support to small
Dhan Yojana’ 54% of those accounts are of women. and micro enterprise sector.
The ‘Beti Bachao Beti Padhao’ (Save and Educate Girl 2. Structural Reforms: The examples are ‘The
Child) in 2015 for better women education and check the Insolvency and Bankruptcy Code’, ‘Goods and
drop out rate of girls from education system. Services Tax Act’ etc. The main objective is to make
Govt has launched 75 Digital Banking Units in October our economy more competitive. Our Ease of Doing
2022 to enhance financial inclusion. Business rankings have improved.
SDG 6: Clean Water and Sanitation – Ensure availability SDG 9: Industry, Innovation and Infrastructure—
and sustainable management of water and sanitation Build resilient infrastructure, promote inclusive and
for all sustainable industrialisation; and foster innovation:
India has 4 per cent of the world’s fresh water India is a very vast nation. India has launched the
resources, while we have to support 17 per cent of world’s ‘Bharatmala’ Pariyojna that intends to build roads
population. Many children under 5 years of age die due through corridors and feeder routes. These corridors
to water-borne diseases. All villages, Gram Panchayats, are efficient in improving national and international
Districts, States and UTs declared themselves ‘open- connectivity. Govt is setting up many sector wise
defecation free’ (ODF) by 2 October 2019, that coincides regulators for better development.
with the 150th birth anniversary of Mahatma Gandhi.
Government was able to construct over 100 million toilets SDG 10: Reduced Inequities–reduce inequality within
in rural India by that date. and among countries: India needs to remove inequality in
A unified Ministry of Jal Shakti is working for Safe and all dimensions in areas of income, gender, social, political
Affordable Drinking Water for all. Government started and economic through MNREGA, Aadhar etc. Such schemes

M09_MADAN 07_65901_C09.indd 10 23/12/22 8:01 PM


People, Development and Environment 9.11

are linked with our banking system. We need to decrease efforts on wetlands, water bodies, protecting wildlife,
dropout rates in schools. Concepts such as Gross Enrolment combating desertification and conserving biodiversity.
Ration (GER), Gender Disparity Index (GPI) help in the
SDG 16: Peace, Justice and Strong Institutions
process. Then, programmes such as Beti Bachao, Beti
India is working towards reducing violence, access to
Padhao; Support to Training and Employment Programme
justice, accountable and transparent institutions, and
for Women (STEP), ‘Deendayal Disabled Rehabilitation
also for inclusive societies (which includes all sections of
Scheme for persons with disabilities’ help us a lot.
society) for sustainable development.
Indian banking system has helped in the creation of
Self-Help Groups (SHGs) for Financial Inclusion. SDG 17: Partnerships for the Goals
India has developed global and local partnerships
SDG 11: Sustainable Cities and Communities–Make (for cooperative federalism) to strengthen the means of
cities and human settlements inclusive, safe, resilient implementation and revitalisation for sustainable devel-
and sustainable opment. The domestic resource mobilisation, improving
Government promotes inclusive and sustainable urban- public expenditure, promoting entrepreneurship and pri-
isation that also calls for participatory, integrated and sus- vate sector, strengthening the south-south cooperation,
tainable human settlement. coalition-based approach are crucial.
Pradhan Mantri Awaas Yojana (urban housing),
National Urban Transport, Smart Cities Mission help us a SDG Scorecard: The 2022 SDG Index is topped by
lot. The schemes such as Swachh Survekshan propel cities Finland, followed by three Nordic countries–Denmark,
towards ‘Round The Clock Action’ so that we are able to Sweden and Norway.
achieve ‘Sampurn Swachhata’. India, with a rank of 121 and at a score of 66 out of 100
stands behind Bhutan, Sri Lanka, Nepal and Bangladesh.
SDG 12: Responsible Consumption and Production India stands better than Pakistan that is on 125th rank.
—Ensure sustainable consumption and production Among states, Kerala ranks first that is followed by
patterns Tamil Nadu and Himachal Pradesh, both at second posi-
This covers resource use efficiency, green employment, tion. The third position is shared by Goa, Karnataka,
equitable access to basic services and better quality of life Andhra Pradesh and Uttarakhand. Bihar is at the bottom
are integral to sustainable consumption and production. of the list at a rank of 16 with a score of 52.
The relevant examples for India are Sustainable Among UTs, Chandigarh ranks first, followed by Delhi,
Food Systems, Soil Health Cards, National Mission on Lakshadweep and Puducherry for the second place and
Sustainable Agriculture, National Resource Efficiency the Andaman and Nicobar Islands for the third position.
Policy, LEED (Leadership in Energy and Environmental India faces major challenges in achieving 11 of the
Design), Green Rating for Habitat Assessment (GRIHA), 17 SDGs. The SDG 8 is the most challenging for us.
and Energy Conservation Building Code (ECBC).
For ‘waste management’, India has adopted a sustain-
able development framework based on ‘precaution’ and Stopover
‘polluter pays’ principles (discussed later in this unit).
Which of the following are priority areas in relation to

SDG 13: Climate Action – Take urgent action to combat the Sustainable Development Goals?
climate change and its impacts (a) No poverty
As an active participant in international efforts, India (b) Zero hunger
needs less of total carbon emissions (India is at rank (c) Reducing urbanization
three now) and justified per capita emissions to keep (d) Peace, justice and strong institutions
world temperature increase below 2 degrees centigrade. Choose the correct answer from the options given
We need to do better on disaster resilience, green tech- below:
nologies and meeting energy needs. At Glasgow meet in (a) (i), (ii), (iii)
2021, we committed to take renewable energy consump- (b) (i), (iii), (iv)
tion to 50% of the total energy. (c) (ii), (iii), (iv)
(d) (i), (ii), (iv)
SDG 14–Life below Water – Conserve and sustainably The correct option is (d).
use the oceans, seas and marine resources for
sustainable development:
India regularly monitors possible pollution by oil, sea cargo,
sewage and garbage, as well as air pollution by ships etc. Human and Environment Interaction:
SDG 15: Life on Land: We want to protect, restore and A nthropogenic A ctivities and their Impacts
promote sustainable use of terrestrial ecosystems, man- on E nvironment
age forests, ensure diversification of life etc. There is need There are basically two types of activities:
for better conservation, development and management
of forests. Afforestation can be done through natural and 1. Natural: Occur in the natural manner
artificial regeneration. The government is also making 2. Anthropogenic: Basically man-made activities

M09_MADAN 07_65901_C09.indd 11 23/12/22 8:01 PM


9.12 Chapter 9

Anthropogenic: Basically man-made activities. The present is also present in tobacco smoke, natural gas and
age can be termed as anthropocene age. kerosene.
2. Benzene is a solvent used in petrol, ink, oil, paint, plastic
1. The main objective is to improve human living con- and rubber. Trichloroethylene is used in metal degreas-
ditions with human efforts. Once they reach a point, ers, dry cleaning solvents, inks, paints, lacquers, var-
they can start causing harm to the human life. nishes and adhesives. Ozone is produced from copying
2. The related energy consumption and technical evolu- machines. Fumes are produced from cleaning solvents.
tion cause pollution. 3. The use of air conditioning system may lead to growth of
The life on this planet emerged 3000 million years ago in disease causing bacteria in air ducts and filters.
the form of single cell organisms. With evolution of life, 4. The inhalation of asbestos can cause fibrotic lung dis-
their was increase in oxygen (and CO2) levels. Ozone levels ease (asbestosis), changes in the lining of the chest
also increased that acts as protective layer against solar UV cavity (pleura) and mesothelioma.
rays. Increase in the levels carbon dioxide and other gases 5. Cockroach droppings may cause allergic asthma.
and pollutants is the result of anthropogenic activities.
Industrialization and urbanization were the major causes
Effects of Anthropogenic Mining Activities
for increase in pollution. The changes were spatial (local on E nvironment
to global) and temporal (short term to long term). The environmental effects of mining depend on factors
such as ore quality, mining procedures, local hydrologi-
The Main Impacts on T raditional T ype of cal conditions and development stage of resource.
Agriculture The mining is mainly of two types—land dredging
The major impacts on agriculture are deforestation, soil (digging a hole in land) and river dredging.
erosion, depletion of nutrients, etc. There have been both River dredging involves moving along a river on a plat-
positive and negative effects on environment in order to form or boat.
increase production. Mining is the main cause of deforestation. The harm-
ful chemicals, such as cyanide, mercury or methyl mer-
The main problems are: cury, may be used in the extraction of minerals.
1. Micronutrient imbalance—in the form nitrogen, The following damages may also be caused by
phosphorus and potassium. mining (Fig. 9.7):
2. Nitrate pollution—leaching deep into the soil and • Devegetation and Defacing of Landscape can occur.
contaminating groundwater; when the concentration • There may be subsidence of land tilting of build­
exceeds 25 mg/L, they cause blue baby syndrome or ings, example, cracks in houses, etc.
methaemoglobinaemia. • Groundwater Contamination: Sulphur is the
3. Eutrophication—excessive use of N and P fertilizers main element that gets converted into sulphuric
in agriculture, leading to algal blooms.
acid, which makes the water acidic. Some heavy
There can be pesticide-related problems, which include her- metals also get leached into the ground­water and
bicides, insecticides, fungicides, biocides, etc. Pesticides can contaminate it by posing health hazards.
also be classified as inorganic, synthetic or biological (biope- • Surface Water Pollution: Acidic water is detrimen-
sticides), although the distinction can sometimes blur. tal to many forms of aquatic life. Sometimes, radioac-
Waterlogging (over-irrigation of croplands) and salin- tive substances such as uranium also contaminate the
ity (increased concentration of soluble salts in the soil) water bodies.
are other major issues. • Air pollution: It has been discussed separately.
• Occupational Health Hazards: Miners may suffer
I mpact of Housing-Related A nthropogenic from asbestosis, ­silicosis, black lung disease, etc.
A ctivities on Environment
High-tech buildings reduce the overall cost. The excessive Effects of T ransportation A ctivities on
use of building materials, such as solvents, finishes and E nvironment
cleansers, for maintenance and protection may cause ‘sick Gases: The activities of the transport industry release sev-
building syndrome’. The mining and manufacturing of eral million tons of gases each year into the atmosphere.
metals may be carcinogenic (cancer causing). These include lead (Pb), carbon monoxide (CO), ­carbon
Some insulating materials made from chlorofluorocar- dioxide (CO2), methane (CH4), nitrogen oxides (NOx),
bon (CFCs) and their safe recovery add to global warming. nitrous oxide (N2O), chloro-fluoro carbons (CFCs) per-
The indoor air pollution is a major source of pub- fluorocarbons (PFCs), heavy metals (zinc, chrome, cop-
lic exposure to air pollutants and has potential to cause per and cadmium) and particulate matters (ash, dust).
chronic health problems.
1. Nitrous oxide depletes the stratospheric ozone (O3)
1. Formaldehyde (specifically in modern buildings): layer, which naturally screens the earth’s surface from
Pressed wood products use adhesives that contain UV radiation.
urea-formaldehyde (UF) resins. Formaldehyde 2. CO, CO2 and CH4 participate in greenhouse effect.

M09_MADAN 07_65901_C09.indd 12 23/12/22 8:01 PM


People, Development and Environment 9.13

The severe cold


regions such as the
Arctic Ocean areas Shifting of climatic
More clashing Tornadoes and
warm up to become patterns changes the wind
of warm and cold hurricane-type storms
more habitable directions and patterns
fronts to cause more will increase in inten-
violent weather sity and damage

More warmed water


circulation results in more
Global Retention of more
rains; rainfall patterns also
heat in atmosphere re-
change. warming impact
sulting in more winds

The high-temperature
The intensity of flash regions lose soil mois-
floods will increase ture and it may cause
decrease in agricultural
productivity

Low regions of the earth,


e.g. India, Indonesia, Florida
Melting of glaciers Higher sea levels
in the USA, Belgium, get
more and more rains

Figure 9.7 Impact of Anthropogenic Activities

CO, NO2, SO2 and NOx and O2 cause acid rain. Acid Water Freshwater Inland and marine
precipitation affects construction, reduces agricultural shortage water pollution
crop yields and causes forest decline.
Flora and fauna Deforestation Loss of genetic
Noise: Long-term exposure to noise levels above 75 dB
and fuelwood diversity and loss
­seriously hampers human hearing and affects both physi-
shortage of cropland
cal and psychological well-being.
The issues may emanate from the local level, regional
Environmental Issues level and global level. The 1972 UN World Conference
Ideally, there should be common focus, ‘a common future’ on the Human Environment held at Stockholm also
or ‘global sharing’ concept. The structural inequalities in highlighted the differences in the points of view of the
the global economic system are responsible for the hin- developed and the developing countries regarding
drance in realizing the issue of sustainable development development and environment. The same kind of issues
in totality. emerged in the Rio Summit as well. The issues can be
In case we look at the aspects of developing and developed divided into three levels listed as follows:
nations, the situation given in Table 9.1 may emerge. 1. Local Issues: These relate to issues, such as water
shortage in certain areas, soil erosion of some local
Table 9.1  nvironmental Concerns of the
E forests, different pollution level in the same city and
Developed and Developing Countries landslides. They need to be tackled at the local level.
2. Regional Issues: Some regional issues have been
Environmental Developing Developed mentioned as follows:
Aspect Countries Countries (a) Desertification and droughts
Air Air pollution in Air pollution, (b) Floods and soil erosion
major cities global warming, (c) Rise in sea level, beach erosion, saltwater intru-
ozone depletion sion resulting in increased salinity, floods and
flood damages, threats to coastal wetlands and
Land and soil Soil erosion, Soil loss, solid threats to agriculture
desertification waste disposal (d) Deforestation
and loss of and changing 3. Global Issues: The global issues that have to be
arable land land use addressed are as follows:

M09_MADAN 07_65901_C09.indd 13 23/12/22 8:01 PM


9.14 Chapter 9

(a) Climate Change and Global Warming: These Air Pollution


include increasing drought and desertification, crop Greenhouse gas (GHG) concept has been discussed later.
failures, melting of the polar ice caps, coastal flood-
ing, displacement of major vegetation regimes,
coral mortality, change in ocean behaviour, natural Earth’s
disasters, infectious diseases, degradation of eco- Atmosphere Percentage Greenhouse Gas
systems, scarcity of food supply and rise in sea level. Nitrogen 78.08 No
(b) North–South Divide: This ‘North’ world has just
over 20% of the world population but consumes Oxygen 20.95 No
80% of the world’s energy. On the other hand, the
Water 0–4 Yes
‘South’ comprising the developing nations of the
world is still struggling to provide the basic needs, Argon 0.93 No
such as food, water, shelter, clothing, basic educa-
tion and health for its population. Carbon dioxide 0.039 Yes
Biodiversity: This includes loss of diversity of life
(c)  Neon 0.0018 No
including both flora and fauna.
Helium 0.0005 No
G eopolitics of Climate Change
Methane 0.00017 Yes
Developed Nations: Industrially and historically, they
are the biggest emitters of CO2. They are also part of Hydrogen 0.00005 No
‘Common But Differentiated Principle’ (CBDP); they need
to provide technological access to developing for their Nitrous oxide 0.00003 Yes
economic development in a sustainable manner. Ozone 0.000004 Yes
Developing Nations: It is blamed that these nations are
developing economically and technologically but still burn World Health Organization defines air pollution as the
fuel inefficiently. Nations such as China, India and Brazil presence of materials in the air that are harmful to the living
have developed during recent times, and so their contri- beings once they cross their threshold concentration levels.
bution to emission levels is also very high. They are also The foreign bodies, gases and so on act as air pollutants. Air
accused for the same. The UN Environment Programme pollution is the most extensive and worst form of pollution
in 2002 highlighted Asian brown cloud (later named as (Table 9.2).
Atmospheric Brown Cloud due to shift patterns); it has Types of Air Pollution
many smog-like features. There has been an increase in
population, shifts in rainfall patterns, etc. There are two types of air pollutants, primary and sec-
If we specifically focus on India, Green India Mission ondary.
has taken up the following projects. Primary Pollutants
1. Green Highway Policy: 1,40,000-km-long ‘tree line’ Primary pollutants enter the atmosphere directly from the
along both sides of national highways—1% of the source. Some important primary pollutants are as ­follows:
project cost to be earmarked for plantations
2. Namami Gange: Plantation along rivers 1. Suspended particulate matter (SPM)
3. Compensatory Afforestation Fund Management 2. Oxides of carbon
and Planning Authority (CAMPA): Allocation of $6 3. Hydrocarbons (methane)
billion to states 4. Sulphur oxides (SOx)
4. Provisions under REDD (Reducing Emissions from 5. Nitrogen oxides (NOx)
Deforestation and Forest Degradation) Plus, National 6. CFCs
Agroforestry Policy (NAP), Joint Forest Management, etc. 7. Lead
5. Finance Commission incentives for creation of Secondary Pollutants
carbon sinks: The Union Government attaches 7.5% Secondary pollutants are not directly emitted from sources.
weightage attached by the Union Government to These pollutants are formed as a result of chemical reac-
‘area under forests’ to states. tions between the primary pollutants and certain atmos-
pheric constituents in the presence of sunlight. Sulphates,
Pollution and its Types nitrates and organic particles can be transported over large
distances, such as hundreds and even thousands of kilome-
An undesirable change in the physical, chemical and
tres. Some important secondary pollutants are as follows:
biological characteristics of the environment, especially
air, water and land, that may adversely affect human 1. Sulphur Trioxide: It is a compound formed when
population and the wildlife, industrial processes and sulphur dioxide reacts with oxygen. It combines with
cultural assets (building and monuments) is called water to form sulphuric acid.
pollution. The agents that pollute the environment or 2. Smog Formation: Smog is an odd combination of
cause pollution are called pollutants. smoke and fog. The effect of smog is maximum just

M09_MADAN 07_65901_C09.indd 14 23/12/22 8:01 PM


People, Development and Environment 9.15

Table 9.2 Major Air Pollution–Related Chemical Substances


Pollutants Source Pathological Effect on Human Beings
Ozone Vehicular exhaust—this “bad” ozone is not Lung function—aggravation of asthma,
(ground level) emitted directly into air; it is created by chemical emphysema and chronic bronchitis
reactions between nitrogen oxides and volatile
organic compounds in the presence of sunlight
Lead Leaded petrol (used as anti-knocking agent) Central nervous system, interference with
development of RBCs
Sulphur dioxide Thermal power plant and industries Severe respiratory problem, reduces
exchange of gases from lung surface
Nitrogen oxides Thermal power plant Heart and lung problems, bronchitis,
asthma and also carcinogenic issues
Carbon monoxide Incomplete combustion of fossil fuel, wood Reduces oxygen-carrying capacity of blood,
stoves and cigarette smoke breathing problems
Hydrogen sulphide Oil refineries and chemical industries Nausea, irritation of eyes and throat
Hydrogen cyanide Blast furnace, fumigation and chemical industry Headache, dry throat, indistinct vision and
dysfunction of nerve cells
Ammonia Fertilizer industry, dye-making, bleaching and Acidification of water bodies at a high level
explosives
Phosgene Chemical and dye-making industry Pulmonary oedema
Volatile organic Vaporize easily and some examples are gasoline, Smog formation
compounds benzene
Arsenic Arsenic soldering Damages red blood cells and kidneys, and
causes jaundice
Suspended Incinerators and basically manufacturing Emphysema, respiratory system problems—
Particulate Matter processes asthma, chronic bronchitis
(SPM)*
Some of the pollutants fall under categories of both air and water pollutants, such as heavy metal cadmium.
* SPM includes soot, smoke, dust, fibres, pesticides, metals (such as mercury, lead and copper), dust mites and pollen.

before sunrise as smog particles are entrapped in cold


air and are unable to rise. Two types of smog are prev-
alent as per records: Air Pollution and Aerosols
(a) Sulphurous or London Smog: London was • Aerosols: They are stable suspensions of solid or
affected from this smog long time ago in 1952, liquid particles in air. Aerosols affect the weather
hence this kind of smog was named as London conditions by blocking solar radiations. Deposition
smog. It is also termed ‘reducing smog’ as its of aerosols on leaves affects the process of photo-
mixture of components is chemically reducing synthesis.
in nature. This is due to the presence of sulphur • Mist: It refers to aerosols consisting of liquid drop-
dioxide in air. It is more prevalent during the lets.
morning hours of winter season when the rela- • Dust: It refers to aerosols consisting of solid parti-
tive humidity is high and air near the ground is cles.
also cooler. London smog causes throat irritation • Fume: It refers to aerosols consisting of hot vapours
and difficulty in breathing. of metals.
(b) Photochemical or Los Angeles Smog: This type • Smoke is also an aerosol, which is a mixture of liq-
of smog is caused by the presence of oxides of uid and solid particles as a result of burning.
nitrogen in the atmosphere, formed as a result of • Plume: It is a geometrical form of smoke.
vehicular exhaust. It is formed due to chemical • Smog is the mixture of smoke and fog (­discussed
reactions involving ozone, nitrogen oxide, hydro- separately).
carbons and peroxyacetyl nitrate (PAN) in the

M09_MADAN 07_65901_C09.indd 15 23/12/22 8:01 PM


9.16 Chapter 9

presence of sunlight. This phenomenon mainly (h) Lead


occurs during warm sunny days as sunlight is PM1 particles are 1 μm or less in diameter. They are
required to carry out photochemical reaction 35–50% of PM2.5 in number. They penetrate deep
in seasons when the sky is clear. Photochemical into the lungs and bloodstream. They cause cardio-
smog consists of brown hazy fumes. It irritates vascular diseases.
the eyes and lungs, and causes cracking of rub-   The unit of measurements in case of pollutants
ber and extensive damage to plant life. mentioned above is microgram per cubic metre except
(c) Ground-level Ozone: Tropospheric or ground- in the case of CO, for which it is milligram.
level ozone is formed from photochemical reac-   The AQI is classified along with one of the follow-
tion between two major classes of air pollutants, ing six categories:
such as volatile organic compounds (VOCs) and (a) Good (0–50)
nitrogen oxides. (b) Satisfactory (51–100)
3. Acid Rain: Acid rain is caused by a chemical reaction (c) Moderately polluted (101–200)
that begins when compounds such as sulphur dioxide (d) Poor (201–300)
and nitrogen oxides are released into the air. These sub- (e) Very poor (301–400)
stances can rise very high into the atmosphere, where (f) Severe (401–500)
they mix and react with water, oxygen and other chem- The formulation of the index was an initiative under
icals to form more acidic pollutants known as acid rain. Swachh Bharat Mission (Cleanliness Mission) based
Effects of Acid Rain on the recommendations of IIT Kanpur and the expert
group formed in this regard.
1. Acid rain has many ecological effects, but none is 2. National Ambient Air Quality Standards: These ambi-
greater than its impact on lakes, streams, wetlands ent air quality standards are specified separately in India
and other aquatic environments. It makes water for around 12 pollutants, including the 8 that constitute
acidic and causes it to absorb aluminium. the NAQI. The additional four pollutants are arsenic,
2. Acid rain also damages forests, especially those at nickel, benzene and benzopyrene. PM2.5 is particularly
higher elevations. It erodes the soil of essential nutri- dangerous and can cause adverse health effects owing
ents and releases aluminium in the soil, which makes to its greater penetrability into the human respiratory
it hard for trees to take up water. system and eventual accumulation in human organs
3. Leaves, trees and plants are less able to withstand and blood. PM concentrations are higher in winter sea-
the effects of acid rain combined with other environ- son and are lower during monsoon months.
mental stressors such as cold temperatures, insects   SO2 levels are within the prescribed National
and disease. The pollutants may also inhibit the trees’ Ambient Air Quality Standards in residential areas of
ability to reproduce. all the cities. Decreasing trend may be due to various
G overnment I nitiatives to C ontrol interventions that have taken place in recent years,
such as reduction of sulphur in diesel, use of cleaner
Air Pollution fuel such as compressed natural gas (CNG) in Delhi
The following are the major government initiatives to and implementation of Bharat Stage III emission
monitor air pollution: norms. In addition, there has been a change in the use
of domestic fuel from coal to LPG, which may have
1. National Air Quality Monitoring Programme contributed to reduction in ambient levels of SO2.
2. National Ambient Air Quality Standards   NO2 levels are within the prescribed National
3. System of Air Quality and Weather Forecasting Ambient Air Quality Standards in residential areas
1. National Air Quality Monitoring Pro­gramme: The of most cities. Measures to reduce NO2 include ban-
Central Pollution Control Board is executing a nation- ning of old vehicles, better traffic management, etc.
wide programme of ambient air quality monitor- Despite an increase in the number of vehicles, CO
ing known as National Air Quality Monitoring Pro- levels have reduced during the last few years. The
gramme (NAMP). decrease may be attributed to measures such as con-
  The National Air Quality Index (NAQI) was launched version of diesel-run three-wheelers to CNG.
on 17 October 2014 to disseminate information on air 3. System of Air Quality and Weather Forecasting:
quality in an easily understandable form for the general The Ministry of Earth Sciences (MoES) has intro-
public. The measurement of air quality is based on the duced a major national initiative, ‘System of Air Qual-
pollutants mentioned as follows: ity and Weather Forecasting and Research’ known as
(a) PM10—particulate matter (size less than 10 µm) ‘SAFAR’, for greater metropolitan cities of India, to
(b) PM2.5—particulate matter (size less than 2.5 μm) provide location-specific information on air quality
(c) NO2 in near real time and its forecast 1–3 days in advance
(d) SO2 for the first time in India. It has been combined with
(e) CO the early warning system on weather parameters. The
(f) Ozone SAFAR system is developed by the Indian Institute of
(g) Ammonia Tropical Meteorology, Pune.

M09_MADAN 07_65901_C09.indd 16 23/12/22 8:01 PM


People, Development and Environment 9.17

WHO has its own standards. Across the globe, most of • The policy is modelled on the pattern of Euro norms.
the emissions that reach the atmosphere come from coal • Emission norms were introduced such as Bharat I, II,
(43%) followed by oil (33%). III, IV, V and VI in a phased manner, though Bharat V
skipped later.
Indoor Air Pollution • With effect from February 2000, lead has been phased
The major reasons for indoor air pollution are inefficient out of automobile fuel.
burning of inferior fuels during cooking or heating, such
as cow dung, agricultural residue, and coal and fuelwood, Advantages of BS-VI Norms
along with poor ventilation systems inside the house • The central government mandated that vehicle
(Fig. 9.8). The problem aggravates during winters when makers must manufacture, sell and register only BS-VI
the doors and windows of the houses, especially in rural (BS6) vehicles from April 1, 2020.
areas, are kept shut. The smokeless chulhas were intro- • In BS-VI fuel, the volume of PM 2.5 ranges from
duced as a solution to the problem of indoor air pollution. 20 to 40 μg/m3, whereas in BS-IV fuel it is up to 120
However, they could not make much impact. μg/m3.
• BS-VI fuel will bring down sulphur content by five
Bharat Stage (BS) Norms times from the current BS-IV levels. It has 10 ppm of
• Bharat stage (BS) emission standards are laid down sulphur as against 50 ppm in BS-IV.
by the government to regulate the output of air pol- • Sulphur in the fuel contributes to fine particulate matter
lutants from internal combustion engine and spark- emissions. High sulphur content in the fuel also leads to
ignition engine equipment, including motor vehicles. corrosion and wear of the automobile engine.
National Auto Fuel policy was announced offi- • With BS-VI fuel, for every 1 km, a car will emit 80% less
cially in 2003. The road map for the implementation particulate matter and nearly 70% less nitrogen oxide.
of the BS norms was laid out until 2010. • Air pollutants in BS-VI fuel are much less as compared
• The Auto Fuel Policy was suggested by the expert to in BS-IV fuel.
committee headed by Dr R. A. Mashelkar.
• There were recommendations on auto fuel’s quality and Main Methods of Controlling Gaseous Pollutants
vehicular emission norms applicable to whole of India. 1. Combustion: This technique is used for controlling
• It recommended introduction of low-sulphur diesel, those air pollutants that are in the form of organic
unleaded petrol and low-benzene gasoline in a phased gases or vapours. Here, the organic air pollutants are
manner. subjected to flame combustion technique (catalytic
• It recommended promotion of alternative fuels such combustion). The organic pollutants are converted
as CNG. into less harmful products and water vapour.
2. Absorption: Absorption is a process in which a sub-
stance penetrates another substance such as scrub-
bers. The gaseous pollutants are passed through
absorbing material such as scrubbers. These scrub-
bers contain a liquid absorbent. This liquid absorbent
removes the pollutants present in gaseous effluents.
Indoor Thus, the air coming into scrubber is free from pollut-
Air Quality ants and it is discharged into atmosphere.
3. Adsorption: Adsorption is a process in which a sub-
90% of our lives 2– More pollution stance sticks to the surface of another substance
spent indoors 5x indoors than (called absorbent). The gaseous effluents are passed
outdoors through porous solid absorbent kept in a container.
The gaseous pollutants stick to the surface of the
porous material and clean air passes through. The
organic and inorganic constituents of gaseous efflu-
Common Indoor Air Pollutants
ents are trapped at the interface of the solid adsorbent
Airborne Indoor Household Ozone Carbon and physical and physical adsorbent.
particles formaldehyde odours and from outdoor dioxide
from diesel from building gases air (ground- from people Mechanical Methods: Some other methods to control
exhaust, materials, from activities level ozone exhaling and particulate air pollutants are as follows:
dust, smoke furniture, such as is harmful to cooking
and other cooking and painting, breathe)
1. Fabric Filters: In this technique, gaseous emission
sources smoking cooking and containing dust, soot and fly ash is passed through
smoking porous fabric filters made of fabric (cloth-woven or
filled fabric). The particles of pollutants get trapped
in this fabric and are collected in the filter and
the gases free from the pollutant particles are dis-
Figure 9.8 Indoor Air Quality charged.

M09_MADAN 07_65901_C09.indd 17 23/12/22 8:01 PM


9.18 Chapter 9

For your Information

Dust, Trucks, Two-Wheelers


Beat Cars as Delhi Air Killers
Road dust is the top contributor to the high level of participate matter, also called particle
pollution or PM, in Delhi followed by vehicular emissions, cooking and industry or power plants,
finds a study by IIT Kanpur. Trucks are the worst polluters among vehicles. Here is a snapshot.
WHAT’S PUSHING VEHICLES POLLUTING* THE MOST ...
UP DELHI’S PM
2.5 LEVELS .... 18%
Two-wheelers
22% ¤
Power plant/large
Industry 14–15%
Passenger cars
22% ♦
Domestic cooking
24–25%
25–36%* Trucks
Vehicles *PM 2.5

35% POLLUTING CAUSED


Road dust BY SECONDARY
PARTICULATE MATTER ...
¤ Excluding road dust
Figures may not add up to 100% as
they are average of range estimates
♦ 5–10% through wood/biomass-
60% 40%
Power plants, Vehicular
based cooking excluding road dust cooking, all emissions
* Average 25%, 35–36% in peak other sources
time and in certain areas

Figure 9.9 Urban Pollution Origins

2. Mechanical Devices: There are many mechanical 3. Following measures can reduce industrial pollution:
devices that clean the air of pollutants either (i) due installation of tall chimneys and installation of
to gravity in which the particles settle down by gravi- devices that do not allow pollutants to be released in
tational force or (ii) by sudden change in the direc- the environment so easily, such as filters, electrostatic
tion of gas flow in which particles separate out due to precipitators and scrubbers.
greater momentum.
3. Electrostatic precipitators: In this technique, a Oxygen Depletion in Water Bodies
gas or air stream containing aerosols in the form High levels of organic waste increase the rate of decom-
of dust, mist or fumes is passed between the two position by bacteria, which use oxygen for this process.
electrodes of the electrostatic precipitator. During This causes a drop in dissolved oxygen in water. In other
this process, the aerosol particles get precipitated words, the biochemical oxygen demand (BOD) of water
on the electrodes. increases. A high BOD indicates a low level of dissolved
The following methods also help: oxygen in water. This destruction leads to the destruction
of sensitive organisms, such as phytoplankton, molluscs
1. Domestic Measures: At domestic level, burning and fish.
of wood and dung cakes can be replaced by use of Chemical oxygen demand (COD) is the amount of
cleaner fuel and biogas (formed by the decomposition oxygen required to break down the organic material via
of animal and plant wastes in a biogas plant). ‘oxidation’.
2. Transportation Measures: Pooling of transport or
use of public transport, use of unleaded petrol and
CNG, regular tuning and servicing of the engines, Biomagnification
and switching off the engine at red lights or when not The accumulation of a toxic chemical in the bodies
in use. of organisms as we move from producers to primary

M09_MADAN 07_65901_C09.indd 18 23/12/22 8:01 PM


People, Development and Environment 9.19

consumers to secondary consumers, etc., is called


Stopover
biomagnification. It occurs when a chemical becomes
more and more concentrated as we move up the 1. The common features between NO2, SO2, CO and SPM
food chain. This is specifically true in case of non- is that all of them
biodegradable chemicals or pollutants. The classic (a) Are classified as primary pollutants
example is DDT. It is first eaten by planktons, then by (b) Are classified as secondary pollutants
small fish and then by big fish. The fish are eaten by (c) Have equal role in causing acid rain
birds and so on. At each level, its concentration goes (d) Are greenhouse gases
up. The consumption of DDT by birds causes thinning The correct option is (a).
of their eggs, which rupture prematurely during their
warming and babies die a premature death. The process 2. Sulphur dioxide is mainly released from
of biomagnification is also called bioaccumulation. (a) Cars and trucks
(b) Photochemical reactions
Bioaccumulation (c) Factories
(d) None of the above
This is the gradual accumulation of substances, such as
pesticides or other chemicals, in an organism. For example, The correct option is (a).
car emission chemicals build up in birds and other animals.
Similarly, the mercury is built up in fish.
W ater Pollution and its Causes
Eutrophication
Water is an important resource. Water is the basis of life and
The inorganic nutrients in the run-off from agricultural it makes up to 60–95% of the total weight of any function-
fields reaching a water body increase the nutrient content ing living cell.
of the water body. These nutrients causes the profuse Water is the most scarce and the most valuable
growth of algae (algal bloom) in it. This growth eventually commodity available to us. Earth is proudly called as
causes the death of small fish and organisms in them. ‘Water Planet’. The 68.7% of the fresh water on earth is
found in icecaps and glaciers, 30.1% is found in ground
Methane (CH4) water. The fresh water is only 1.2%. This 1.2% can be
Methane is a hydrocarbon that is a primary component of further split into ground ice and permafrost (69.7%),
natural gas. Methane is emitted from anthropogenic and lakes (20.9%) and rest into soil moisture, rivers, living
natural sources. creatures etc. Thus, over more than 99 % earth’s water
Anthropogenic (human-influenced) emission sources is unusable by humans and many other forms of life. It
include landfills, oil and natural gas systems, agri- is amazing to see that water supports all terrestrial, as
cultural activities (such as rice cultivation), coal min- well as aquatic, life on our planet is actually so scarce.
ing (coal bed methane), combustion of stationary and (Fig 9.10).
mobile, wastewater treatment, and some industrial pro- The amount of water is not diminishing, but the
cesses. Methanogens are bacteria in animals that breaks demand for it is steadily increasing. In addition, the
down plant based organic matter in their digestive sys- amount of water that is clean and drinkable is steadily
tem such as cows and thus they produce methane. The decreasing because of pollution (Fig. 9.11).
melting of permafrost regions in arctic oceans also helps Waterfall falling near a river’s ‘headwaters’ (the source
in release of methane. The release of methane gas itself of river–origins) flows downhill due to gravity through a
is exacerbated by global warming, that again adds to ‘watershed’.
global warming. Urban areas, obviously, have a greater need for water
Methane is the second most abundant anthropogenic beyond the basics for drinking and sanitation. But over-
GHG after carbon dioxide. Methane is 25 times more population in undeveloped countries means that many
potent in comparison to carbon dioxide in trapping heat people do not even get the basics.
in atmosphere. The methane levels have more than dou- The sector-wise breakup of water withdrawal in India
bled in last two decades. is as follows:
Methane is thus a powerful greenhouse gas, and it is 1. Agriculture and livestock—91%
short-lived compared to carbon dioxide. If we are able 2. Municipalities—7%
to achieve substantial reductions in methane emissions, 3. Industry—2%
this can help in reduction of global warming in future.
China, USA, Russia, India, Brazil, Indonesia are the Pollution of water affects drinking water, lakes, rivers and
major methane emitters. oceans all over the world.
The presence of methane gas on other planetary bod- Water pollution is the presence of harmful sub-
ies as our moon, Saturn’s moon, Mars etc. may indicate stances in a water body and thus, it makes water unfit for
microbial life or some volcanic activity there. intended use.

M09_MADAN 07_65901_C09.indd 19 23/12/22 8:01 PM


9.20 Chapter 9

Distribution of Earth’s Water


Surface water
Saline and other
ground- freshwater Atmosphere water
Freshwater 2.5% 1.3%
water 0.22%
0.93% Biological water
0.22%
Saline
lakes
0.07% Groundwater Rivers 0.46%
30.1% Lakes Swamps and
20.1% marshes
2.53%
Soil
moisture
3.52%

Oceans
96.5%
Glaciers
and
ice caps Ice and
68.6% snow
73.1%

Total global Freshwater Surface water and


water other freshwater
Figure 9.10 Water Bodies on Earth
Source: Igor Shiklomanov’s Chapter “World Fresh water resources” in Peter H. Gleick (1993), Water in Crisis: A Guide to world’s
Fresh Water resources.

Types of Water Pollution


According to the sources of pollution, there are two types Water Flow
of water pollution,

1. Point Source Pollution


The discharge of harmful substances by specific sources
into a water body leads to point source pollution. Sediments Acid water
Industrial wastewater and hot water from ­thermal power Crop dusting
from mine
plants cause point source pollution.
The major sources of this type of water pollution are Industrial
as follows: wastes

1. Industrial Wastewater: Both small and large indus- Barnyard


trial units produce wastewater, which has a variety wastes
of organic and inorganic pollutants. This is the major Oil pollution
Fertilizer
reason for river pollution in India. runout
2. Hot Water: Industries such as thermal power plants
and oil refineries use water as coolant. When this
water is discharged into the water body, its temper-
ature is higher by 15°C. The warmer temperature Sewage treatment plant Nuclear reactor
decreases the solubility of oxygen and increases the
metabolism of fish. This changes the ecological bal-
ance of the river. Figure 9.11 Water Pollution Due to Human Activities

M09_MADAN 07_65901_C09.indd 20 23/12/22 8:01 PM


People, Development and Environment 9.21

2. Non-point Source Pollution restrict the amount of oxygen that reaches the brain,
The discharge of pollutants into a water body from large causing blue baby syndrome. Haemoglobin in blood
areas leads to non-point source pollution. Construction carries oxygen from the lungs or gills to the rest of
run off and acid rain cause non-point pollution. the body. Normally 1–2% of a person’s haemoglo-
Non-point source pollution is caused by the following bin is methaemoglobin. A higher percentage due to
pollutants: nitrates or nitrites can cause health problems known
as methaemoglobinaemia.
1. Municipal Wastewater: It is mainly from domestic 5. Chromium: Chromium is a mineral that aids in the
sources such as kitchen and toilet. This is the major body’s ability to use insulin to convert carbohydrates
reason for river pollution in India. to energy. It is used in many industries also.
2. Surface Run-off: The practices followed in agricul-   Naturally occurring trivalent chromium is essential
ture affect groundwater quality. Intensive cultiva- for good health.
tion causes fertilizers and pesticides to seep into   Hexavalent chromium does not occur naturally but
the groundwater, in a process known as leaching. is produced by certain industries.
Irrigation run-off from agricultural fields causes high   It is the most toxic form of chromium. It causes lung
nitrate content in groundwater. The problem is aggra- cancer on continuous exposure.
vated if industries are located in that area.   Breathing chromium dust is the main route for
3. Oil Spills: An oil spill is the accidental discharge of exposure to it.
petroleum into oceans or estuaries, leading to the pol-   Plants also absorb chromium and it can be passed
lution of the marine ­ecosystem. Oil spills are caused on to those who eat the plants.
due to capsized oil tankers or offshore oil mining and   Even contact with such contaminated soil can result
oil explorations. in exposure to chromium.
6. Arsenic: Earlier it was used as an insecticide, roden-
Effect of Water Pollution on Human Life ticide, for wood preservation and for medical prepa-
Water contamination due to domestic sewage containing ration. It has many industrial applications. Different
pathogens, such as viruses, bacteria, parasitic protozoa fungi and microorganisms convert arsenic to dimethyl
and worms, can cause diseases, such as jaundice, chol- arsenic in water, which gets detected in natural water,
era, typhoid and amoebiasis. This type of contamination bird egg shells, sea shells and human urine. Arsenic
renders water unfit for drinking, bathing, swimming and poisoning through water can cause damage to the
even irrigation. liver, nervous system disorders, vascular disease, skin
cancer (dermatitis) and bronchitis.
Contamination of Water by Heavy Metals 7. Cadmium: Cadmium compounds are mainly used in
Heavy metals are commonly defined as those having a rechargeable nickel–cadmium batteries. Cadmium is
specific density of more than 5 g/cm3. The main threats also used in making fusible alloys, electroplating and
to human health from heavy metals are associated as control rods in nuclear reactors.
with exposure to lead, cadmium, mercury and arsenic   Cigarette smoking is a major source of cadmium
(a ­metalloid). Heavy metal contamination of water bod- exposure. In non-smokers, food is the most impor-
ies and groundwater due to industrial wastewater affects tant source of cadmium exposure. It damages the
health in a number of ways. Industrial effluents contain- heart, liver, lungs and reproductive organs, and also
ing lead, fluorides, nitrates and arsenic pose a grave dan- causes kidney damage. The itai-itai disease in Japan
ger to human beings. was due to cadmium pollution, where it caused bone
1. Mercury: Mercury compounds in wastewater are defects and fractures.
converted by bacterial action into extremely toxic 8. Cyanide: It is used in extraction of gold and silver
methyl mercury. Fish accumulate this poison in their metals, metal painting and pesticides. Its consump-
bodies, and the consumption of such fish can cause tion leads to nausea and death.
numbness of limbs, lips and tongue, deafness, blurring 9. Manganese: It is abundant in nature. Higher concen-
of vision and mental derangement. This syndrome is tration of manganese causes cramps, tremors, halluci-
called Minamata disease since it was first noticed in nations, manganic pneumonia and renal degeneration.
Japan in the 1950s, where people developed it after 10. Iron: The excessive presence of iron in human body
consuming fish from the Minamata Bay. It can also can aggravate thalassaemia that is basically a genetic
cause gingivitis. disorder. This has an adverse impact on red blood cor-
2. Pesticides: Organophosphates and carbonates present puscle (RBC) count and haemoglobin. Water bodies
in pesticides that get washed off into water bodies dam- also become foul due to abundance of iron-oxidizing
age the nervous system and can cause cancer. microbes.
3. Fluoride: Excess fluoride can cause yellowing of
teeth and damage to the spinal cord. Effect of Water Pollution on Marine Life
4. Nitrates: Drinking water contaminated with Marine oil spill is also a type of water pollutant and has
nitrates can prove fatal, especially to infants feed- direct impact on marine life. It is the accidental release of
ing on formula milk made with this water. Nitrates petroleum products into the ocean or coastal waters.

M09_MADAN 07_65901_C09.indd 21 23/12/22 8:01 PM


9.22 Chapter 9

1. Tanker Spills: Even small amounts of oil spread Soil and Causes for its Pollution
across large areas of water prevent oxygen in the air Soil is a thin covering over the land consisting of a mix-
from dissolving in water, thus making it difficult for ture of minerals, organic ­materials (carbon compounds,
organisms to breathe. generally derived from organisms), living organisms, air
2. Oil Coating: Oil coating results in poisoning of and water.
marine birds such as seagull. The oil coating reduces Mature soil is arranged into a series of zones called soil
their body temperature and makes it impossible for horizons (Table 9.3).
them to survive the cold temperature of the ocean. The following factors affect the formation of soil:

W ater A vailability in I ndia 1. Parent materials


2. Topography
According to Indian Council for Agriculture Research 3. Climate
(ICAR) the per capita annual water availability has 4. Organisms
declined to 1,508 cubic meter in 2014 from 5,177 cubic 5. Time
meter in 1951.
The per capita availability of water is estimated to Different types of soils vary in the following ways:
decline further to 1,465 cubic meter by 2025 and 1,235
1. Clay: Very fine particles
cubic meter by 2050. If it declines further to around
2. Silt: Fine particles
1,000-1,100 cubic meter, then India could be declared as
3. Sand: Medium-sized particles
water-stressed country,
4. Gravel: Coarse particles
Namami Gange: We can name ‘Namami Gange’ scheme
The following are the different causes of soil
here that was started in 2014. (Sikkim was given the sta-
pollution:
tus of ‘Fully Organic State’ in the year 2016). There is
scheme on interlinking of rivers to ensure equitable dis- 1. Industrial Waste: This includes heavy metals and
tribution of water. toxic chemicals.
2. Municipal and Medical Wastes: Some wastes are
Ramsar Wetlands in I ndia non-biodegradable.
A Ramsar wetland is a wetland placed under protec- 3. Radioactive Wastes also Cause Soil Pollution.
tion due to its international and ecological significance. 4. Agrochemicals: These include pesticides, weedicides
The policies governing the protection of the wetlands and excess inorganic fertilizers.
were discussed during the Ramsar Convention held on 5. Opencast Mining: Digging the earth’s surface for
February 2, 1971 in Iran under the auspices of UNESCO. extraction of mineral ores degrades the top soil of
India adds 10 more wetlands designated as Ramsar sites earth.
to make total 64 sites covering an area of 12,50,361 ha 6. e-Waste: Used computers, mobile phones, TV, etc.,
in the country in August 2022. The 10 new sites include: simply dumped into landfills. Since these are of toxic
Six (6) sites in Tamil Nadu and One (1) each in Goa, nature, they affect the quality of soil. The toxins may
Karnataka, Madhya Pradesh and Odisha. leach from landfills and also spoil the groundwater.
Two Lakshadweep islands, Thundi and Kadamat gets 7. Pesticides and Fertilizers: Temperature, light and
‘Blue Flag’ certification due to better management by carbon dioxide levels affect photosynthesis. Farmers
Govt. use fertilizers, pesticides and biological control

Stopover Table 9.3 Layers of Soil


Assertion (A): Methaemoglobinaemia is a condition

in which blood is not able to carry and deliver enough Layer Description
oxygen to the body. O-Horizon Freshly fallen leaves, twigs, animal waste,
Reason (R): Consuming drinking water with high
fungi and so on.
nitrate level may cause methaemoglobinaemia.
A-Horizon Partially decomposed organic matter
Choose the correct answer from the options given
(humus) and some inorganic mineral
below: particles.
(a) Both (A) and (R) are true and (R) is the correct
explanation of (A). B-Horizon It is also called subsoil, where it has less
(b) Both (A) and (R) are true but (R) is not the correct organic matter and fewer organisms than
explanation of (A). A-horizon soil.
(c) (A) is true, but (R) is false. C-Horizon Helps to determine the pH of soil and
(d) (A) is false, but (R) is true. determines the soil’s rate of water
The correct option is (a). absorption and retention.

M09_MADAN 07_65901_C09.indd 22 23/12/22 8:01 PM


People, Development and Environment 9.23

to increase crop yields. Excessive use of fertilizers Red Soil


reduces the population of soil-borne organisms, the • Rich in iron and hence, it is red in colour
crumb structure of the soil and productivity of the • formed as a result of breakdown of igneous and meta-
soil. This can cause problem of waterlogging. The plant morphic rocks
roots cannot respire due to excess water in soil profile. • Found in areas of India with low rainfall, such as in
Nitrogen is lost from waterlogged soils due to leaching Madhya Pradesh, South Karnataka, Maharashtra and
and denitrification (degassing). Denitrification leads to Rajasthan
the gaseous loss of nitrous oxide (N2O) into the atmo- • Suitable for crops such as red gram, groundnut and cas-
sphere, which is the major GHG, and adds to the phe- tor seed
nomenon of global warming.
8. Other Pollutants: Many air pollutants (acid rain) Laterite Soil
and water pollutants ultimately become a part of soil • Formed from a mixture of clay and red soil and also
pollution. due to leaching process
• Rich in minerals, such as aluminium and iron
Soil Degradation • Found in hot and wet tropical areas
Ideally, it may take thousands of years to make an inch • Has very low fertility and becomes hard when
of good-quality soil that consists of silicates, organic exposed to air, so it is used as a building material
material, humus, clay, etc. Soil needs a dynamic eco- • suitable for crops such as coffee, coconut and
system. For example, nitrogen is the major gas in the cashew
­atmosphere; it is absorbed in the form of nitrates, which
are carried out by nitrifying bacteria present in the soil. Regur Soil
Soil exchanges gases with the environment. Here, the • It is also known as black soil or cotton soil and found
soil breaks down the organic wastes and recycles the in the Deccan trap.
nutrients back to the plants. • It is rich in nutrients, such as calcium, potassium and
We can look at the following points: magnesium, but has poor nitrogen content.
• It is appropriate for growing crops, such as cotton,
1. Soil ecosystem is disturbed by deforestation. tobacco, oil seeds and maize. Other varieties of soil
2. The use of heavy machinery results in soil compaction are desert soil (coarse or sandy texture) and moun-
or pressing, which reduces the porosity of soil and tain soil, formed from deposition of organic matter
also its water-holding capacity. from woodlands and forests.
3. Sewage water is used to irrigate the fields, or s­ ewage
sledge is used as a fertilizer, which increases heavy
metal content in the soil. Noise Pollution
4. With intensification of agriculture as a result of Green Noise pollution may be defined as environmental noise
Revolution, the same type of crop is raised again and that causes physiological or psychological damage if the
again, which deprives the soil of a particular type of volume is high or exposure is prolonged. Noise is defined
nutrient. To prevent this, crop rotation should be fol- as unwanted sound, where it is an irritant and a source of
lowed as a regular process. stress. The hair cells in the ear are damaged to an extent
  High-yielding varieties of food grains demand use that cannot be repaired or replaced. The intensity or
of more water and fertilizers. loudness of sound is felt in the form of pressure waves and
5. Use of water with high salt content to raise crops may affects our eardrums. Just like any other form of pollution,
result in high salinity of soil. Salt makes the layer at noise pollution too has serious impact on the working of
the top impermeable, which does not allow water to our vital organs. The noise pollution can impact the body
seep into the soil and thus, it results in the problem of in terms of elevated blood pressure, reduced cognitive
waterlogging. (thinking) working and chronic stress.
6. Excessive use of pesticides and fertilizers to increase Sound is measured in decibel (dB), named after
land productivity also degrades the quality of soil and Graham Bell, who also invented the telephone.
ultimately these fertilizers and pesticides enter our 1. Noise is measured not on a linear scale but a logarith-
ecological system. mic scale.
7. The overexploitation of groundwater results in the fall 2. The threshold of hearing is assigned a sound level of
of the water table and ultimately in desertification. 0 decibel, that corresponds to an intensity of 1. 10–12
Watts/m2.
Alluvial Soil 3. A sound that is 10 times more intense is assigned a
• Formed as a result of flooding of plain areas, especially sound level of 10 dB.
in lower courses of rivers 4. A change from 40 to 80 dB represents a 10,000-fold
• Basically sedimentary rocks, and are very fertile increase in loudness.
• Usually lack humus and nitrogen, but rich in potassium 5. A modified scale decibel-A takes into account the
• Suitable to grow paddy, sugarcane and so on pitch as well. Noise pollution is measured in decibels
Example: Indo-Gangetic Plains. in 1 pico watt per m2.

M09_MADAN 07_65901_C09.indd 23 23/12/22 8:01 PM


9.24 Chapter 9

Sound power is measured in watts (joules per second). Noise affects the heart rate, peripheral circulation and
As per Environment Protection Rules, 1999, the permit- breathing patterns. Persistent noisy environment can
ted noise level is 125 dB. cause irritability, headache and sleeplessness by decreas-
ing productivity.
Permissible Limitations of Noise in India
The Central Pollution Control Board (CPCB) has laid down Radioactive Pollution
the permissible noise levels in India for different areas. Radiation is defined as the transmission of energy in the
Noise pollution rules have defined the acceptable level of form of waves through space or a material medium. It is
noise in different zones for both daytime and night time. of two kinds, namely, ionizing and non-ionizing. Ionizing
1. In industrial areas, the permissible limit is 75 dB for radiation or high-energy radiation such as X-rays or
daytime and 70 dB at night. gamma rays can alter DNA and can be harmful.
2. In commercial areas, it is 65 dB and 55 dB, while in Non-ionizing radiation is low-energy radiation as emit-
residential areas it is 55 dB and 45 dB during daytime ted by mobile phones or radio towers and tends to gener-
and night, respectively. ate heat.
3. Additionally, state governments have declared ‘silent Radiation can be natural or can arise from human
zones’ which includes areas that lie within 100 meters activities. Most radiation exposure is from natural
of the premises of schools, colleges, hospitals and sources, such as rocks, earth’s crust and cosmic, among
courts. The permissible noise limit in this zone is 50 other sources. Radon is the most prominent example of
dB during the day and 40 dB during the night. natural radiation. Human activities typically account for
up to 20% of radiation exposure on an average.
Effects of Noise Pollution on Human Health
Radiation particularly associated with nuclear medi-
WHO has included noise as one of the most hazardous cine and the use of nuclear energy, along with X-rays, is
factors that affect living conditions in crowded cities. ‘ionizing’ radiation, which means that the radiation has
sufficient energy to interact with matter, especially the
Hearing Loss
human body, and produce ions.
The intensity, frequency and duration of noise have a
proportionate impact on our body. The threshold of Effects of Radioactive Pollution
human hearing is 0 dB. Persistent exposure to intensity
Some of the UV radiations from the sun are considered as
of noise in the range of 71–85 dB or even below can cause
ionizing radiation and provide a starting point while consid-
­permanent loss of hearing. When noise level reaches
ering its effects. UV from sunlight is important in producing
around 130 dB, it can even cause physical pain.
vitamin D in humans, but too much exposure causes sun-
Masking burn and, potentially, skin cancer. The skin tissue gets dam-
aged and the damage to DNA (through mutation) could not
It is the inability to hear important environmental cues
be repaired properly and hence, over time, cancer develops
and animal signals.
and could be fatal. The depletion of ozone layer may increase
our exposure to UV rays and thus, it causes skin cancer.
Table 9.4 Permissible Sound Levels
Genetic abnormalities occur in children of parents who
Area Day (dB) Night (dB) had significant exposure to radiation.
Industrial 75 65
Stopover
Commercial 65 55
Which of the statement/s about peroxyacyl nitrates

Residential 50 45 (PAN) is/are true?
(a) They are secondary pollutants
Silence zones 50 40 (b) They are produced when hydrocarbon radical
reacts with nitrogen oxide
Table 9.5 Typical Average Decibel Levels (c) They cause respiratory diseases in human beings
(d) All of the above
Source dBA The correct option is (d).
Threshold of hearing 0
Quiet whisper 30
Normal conversation 60 Environmental Waste
Loud singing 75 The Directive Principles of State Policy (Article 47) in
the Indian Constitution asks for the protection of envi-
Automobile 80 ronment by the state. The Ministry of Environment and
Forests (MoEF) works for the implementation of India’s
Jet plane 130
Hazardous Wastes Rules.

M09_MADAN 07_65901_C09.indd 24 23/12/22 8:01 PM


People, Development and Environment 9.25

According to the Environment Protection Act, 1990, solid waste management in India. Hence, the casual atti-
waste is defined as ‘any substance which constitutes a tude towards waste management should change.
scrap material, or an effluent or other unwanted surplus Now, we can look at various options to deal with waste
substance arising from application of any process’. management.
With rapid urbanization, industrialization and an
explosion in population in India, waste management will Solid Waste Management
be a key challenge for state governments and local munic-
ipal bodies in the twenty-first century. After the collection of municipal waste from households,
The waste is usually of the following types: there are three ways of disposal by municipal authorities:

1. Biodegradable Waste: It is degraded through micro- 1. Composting Units: It is the cycle of sustainable
bial activity. The main examples are food residue and nutrient reuse by turning waste into valuable organic
human excreta. input. It helps to improve soil vitality, root growth and
2. Non-biodegradable Waste: It does not degrade. The soil moisture retention. The main objective is to col-
main examples are petroleum, plastic and glasses. lect only organic waste.
3. Biomedical: Usually, the leftovers from medicine, 2. Biomethanation produces biogas.
such as needle, syringe and body parts, are counted 3. Heat energy is recovered in the form of dry fuels from
as biomedical wastes. combustible fractions.
4. e-Waste: Computer parts, batteries and CFL bulbs are The composting units can further be categorized into
some of the main examples. the following:
Sources of Waste 1. Aerobic Composting: The bacterial conversion of
organics present in solids under the presence of air
1. Domestic Waste: Polythene, bottles, food, cotton, etc.
under hot and moist conditions is called composting,
2. Industrial Waste: It originates from industrial activi-
where the final product is called compost (humus)
ties and is divided into the following:
that is used as fertilizer, and is non-odorous and free
(a) Food Processing: Organic wastes, pathogens
of pathogens. The waste volume can be reduced to
(b) Paper Industry: Chlorine, sulphur dioxide,
50–85%. The composting methods may use either
methyl mercaptan
manual or mechanical means.
(c) Textile Industry: From boiling and processing of
2. Vermicomposting: It is basically the joint action of
fibres
earthworms and aerobic microorganisms. The worm
(d) Petroleum: Inorganic sulphur, hydrocarbons,
cast is a fine, odourless and granular product. This
organic acids, etc.
product can be used as a biofertilizer in agriculture.
(e) Chemical: Phosphorus, fluorine, silica, etc.
3. Anaerobic Digestion: If the organic waste is buried
(f) Metal: Copper, lead, chromium, cadmium
in pits under partially anaerobic conditions, then it
(g) Cement: Particulate matter, dust
will be acted upon by anaerobic microorganisms. The
(h) Nuclear Reactor: Radioactive wastes such as
methane and carbon dioxide are released and the
plutonium
leftover organic residue is good manure. It is slower
(i) Agricultural Waste: Fertilizer, crop residue, pes-
than aerobic composting and it occurs naturally in
ticides, fumigants
landfills. It may lead to energy recovery through bio-
(j) Radioactive Waste: X-ray machines, nuclear
gas generation that has 55–60% methane and can be
plants, laboratories, etc.
used directly as a fuel for power generation.
(k) Municipal Waste: Waste produced by public
offices, parks, shops, etc. This is not that attractive in India due to high moisture and
organic content and low calorific value of the wastes. The
Solid W aste Lucknow biomethanation plant in 1990 failed because it
was designed to handle only wet segregated waste but
Solid waste is basically a solid or semi-solid domestic waste,
had to cope with mixed waste.
sanitary waste, commercial waste, institutional waste, etc.
There is a need to provide appropriate incentives and
‘Swachh Bharat Abhiyan’ (Clean India Mission) was
regulatory framework needs to be provided.
started on 2 October 2014 to deal with issues related
There are other schemes as well to deal with solid
to waste management, cleanliness and sanitation on a
wastes and they are discussed as follows:
national level.
Only 68% of the garbage generated in the country is col- 1. Incineration: Incineration process can reduce the orig-
lected, of which 28% is treated by municipal authorities. inal volume of combustible solid waste by 80–90%.
Untapped waste can generate more than 30,000 of (a) Incineration involves the combustion of organic
TPD of combustible waste. The amount of waste that is substances contained in waste materials.
generated, if collected and treated well, can be effectively (b) Incineration can be described as ‘thermal treat-
used to generate energy. ment’.
The World Health Organization says that 22 types of (c) Incineration of waste materials converts the waste
diseases can be prevented or controlled by improving into ash, flue gas and heat.

M09_MADAN 07_65901_C09.indd 25 23/12/22 8:01 PM


9.26 Chapter 9

2. Pyrolysis: Here, solid is converted into liquid state The Kasturirangan Report by erstwhile Planning
and liquid is converted into gas. These products Commission highlights the need for an integrated
of treatment can then be used for production of approach that means 5 R’s:
energy.
1. Reduce
3. Landfill: It is burying off the waste in vacant loca-
2. Reuse
tions around the cities that should be covered with
3. Recover
soil to prevent contamination. Suitable trees should
4. Recycle
be planted to hold the soil (of shallow roots). Landfill
5. Remanufacture
may result in the release of poisonous gases, secretion
of toxic liquid and destruction of vegetation. It emphasizes setting up of two approaches for waste
4. Methanogenesisor: It is biomethanation, that is, management:
the formation of methane by microbes known as
methanogens. The main objective is to produce an 1. Centralized: For incineration, gasification, pyrolysis
improved solid fuel or pellets from solid waste. 2. Decentralized: For biomethanation, vermicomposting
5. Recycling is done for plastic, paper, glass, rubber, fer-
rous and non-ferrous metals.
6. Rag Picking: It is mostly done in formal sector. It is Concept Box
manual process and cannot be scaled up so easily. Effects of Plastic Waste
7. Leachate: A major problem arising from landfills
• The plastics, right from their manufacture to their
is the discharge of leachate that moves into the sur-
disposal, are a major problem to the environment.
rounding soil, groundwater or surface water and
• They can lead to reproductive problems.
could lead to severe pollution problems.
• The land becomes ugly and unhygienic.
8. Sensitization of citizens as well as government
• Plastics can choke drains.
authorities, community participation and involvement
• They deteriorate soil fertility. They remain in the
of NGOs. Littering should be prohibited.
soil for years. They disturb the soil microbe activity.
9. Bioremediation: It is the use of living organisms,
• Dioxin (a persistent organic pollutant) is a highly
primarily microorganisms, to degrade environmen-
carcinogenic and toxic by-product of the manufac-
tal contaminants into less toxic forms. For example,
turing process. This chemical is believed to be passed
Pseudomonas bacterium can decompose synthetic
on through breast milk to the nursing infant. PVC
pesticides. Here, the pollutants can be treated on
releases dioxin and also furan into the atmosphere.
site and thus, exposure risks are reduced for per-
• Stockholm Convention on Persistent Organic
sonnel.
Pollutants (2004) is an international environmen-
tal treaty.
Segregation and Community Participation are the key
factors. Plastic bags have been banned in a number of
big cities. Liquid WaSte
Waste management can be done in two ways: The liquid wastes are wastewater, fats, oils or grease, used
oil, liquids, solids, gases or sludges and hazardous house-
1. Waste reduction
hold liquids that are hazardous or potentially harmful to
2. Recycling
human health or the environment.
The Government of India had notified the Municipal The composition of liquid waste depends on its source.
Solid Waste (Management and Handling) Rules in 2000, The three main sources are following:
thereby making it mandatory for all urban local bodies
1. Residential: Domestic sewage is called black water
in the country to engage in collection, segregation, sec-
that contains excreta and grey matter.
ondary storage in covered bins, transportation in covered
2. Commercial: Fats and oil from restaurants and cafes
vehicles, processing through composting or waste-to-
may be removed using a grease trap.
energy technologies and disposal of rejects in engineered/
3. Industrial Areas: The composition of industrial
sanitary landfills.
wastewater depends on the type of industry, the raw
Some of the major issues concerning solid waste man-
materials used and the processes undertaken. The
agement are as follows:
wastewaters from facilities that make food products
1. Absence of Segregation of waste at source will not be harmful to humans, but those from other
2. Lack of Financing industries may contain a variety of chemical com-
3. Lack of Technical Expertise and appropriate institu- pounds, some of which may be hazardous (and there-
tional arrangement fore potentially harmful). Industrial wastewaters
4. Lack of Willingness which contain hazardous substances must be treated,
and the substances removed before the wastewater is
The indifference of citizens, lack of community partici- discharged to the environment.
pation and sewage management plan are some impor-
tant issues. Storm water is also a source of liquid waste.

M09_MADAN 07_65901_C09.indd 26 23/12/22 8:01 PM


People, Development and Environment 9.27

Characteristics of Liquid Wastes excreta. Ammonia is broken down in the environment


Liquid wastes can be described according to their physi- by natural processes and on release into a river, it is con-
cal, chemical and biological characteristics. verted by the action of bacteria to nitrate (NO3), which is
less harmful. Other examples of inorganic chemicals in
Physical Characteristics of Liquid Wastes ­wastewaters are chloride (from salt), phosphates (from
1. Solids: There may be settleable solids (at bottom of con- chemical fertilizers and human wastes) and metal com-
tainer) or suspended solids (carried along in flow of liq- pounds (from mining operations).
uids). They can be measured by filtering out. Biological Characteristics of Liquid Wastes
2. Temperature: Wastewaters are generally warmer
than the ambient temperature. Liquid wastes contain many different types of bacte-
3. Odour: Wastewaters can have an odour, usually due ria and other microorganisms originating from human
to generation of gases as a result of biodegradation in wastes and other sources. Many of these bacteria are
the wastewater. Biodegradation is the breaking down beneficial and are responsible for the biodegradation
(decomposition) of organic substances by bacteria of organic components of the wastes; others may be
and other microorganisms. pathogenic.
In the context of quantity of organic matter in a liq-
uid waste, we can refer to Biological Oxygen Demand
Chemical Characteristics of Liquid W astes (BOD) that is an important measure of its polluting
Organic Matter potential.
We have already discussed biochemical oxygen demand We can use the following methods to deal with the liq-
(BOD). BOD tests may take up to 5 days. Chemical oxygen uid wastes.
demand (COD) test may take up only 2 hours. COD tends
to give higher results than BOD because the chemical pro- Methods to Deal with Liquid W aste
cess can oxidize more material than the biological process.
Treatment of Sewage
Inorganic Material The sewage can be treated by a modern technique involv-
Wastewater also contains inorganic chemicals. For exam- ing three steps—primary treatment, secondary treatment
ple, ammonia (NH3) is present in human and animal and tertiary treatment (Fig. 9.12).

Wastewater Treatment Methods

Physical Chemical Biological

Aerobic Anaerobic

Precipitation Adsorption Disinfection

Screening Mixing Flocculation Floatation Filtration Sedimentation Gas Transfer

Figure 9.12 Wastewater Treatment Methods

M09_MADAN 07_65901_C09.indd 27 23/12/22 8:01 PM


9.28 Chapter 9

Primary treatment: The main objective is to separate the 7. Gas Transfer: This is required in water treatment
solids. The primary treatment involves physical removal plants to dissolve chlorine gas or ozone. This can also
of particles from the sewage through filtration and sedi- be used to remove unwanted volatile chemicals such
mentation. The neutralized sewage may still contains as carbon tetrachloride, chloroform, bromoform etc.
many pathogens and organic matter. This treatment can from water.
be termed as ‘mechanical treatment’.
Chemical Methods
Secondary Treatment: These neutralized effluents are 1. Precipitation: This is considered to be the most
passed through a reactor called sludge. In this reactor, effective method for the removal of heavy metals
the anaerobic bacteria degrade the biodegradable mate- from wastewater. It is widely employed in industries
rial into neutralized effluents. In this process, the foul since it is relatively inexpensive and easy to oper-
odour and methane are released and the sewage is con- ate. The traditional chemical precipitation processes
verted into clean water. This water is sent to aeration include the use of hydroxide precipitation and sulfide
tanks where air and bacteria are added to it. This process precipitation.
is called biological or secondary treatment. Thus, we can 2. Adsorption: A solid is used in this process to remove
see that it is a biological treatment. Primary treatment is a soluble substance from the water. Adsorption is
physical treatment. applied for the removal of dissolved impurities. The
Tertiary Treatment: This treatment is a disinfecting pro- most common process is application of activated car-
cess where final traces of disinfecting bacteria and other bon for removal of organic substances.
dissolved organic solids are removed. Then, the chlorina- 3. Disinfection: Once the water is filtered, water treat-
tion, evaporation and exchange absorption methods are ment plants may add one or more chemical disin-
employed to obtain clean water. fectants (such as chlorine, chloramine, or chlorine
dioxide) to kill any remaining parasites, bacteria, or
viruses.
Water Treatment Methods–Another Category
In this categorization, we use physical, chemical and Biological Treatment
biological methods, we have focused on their definitions 1. Aerobic treatment: This treatment of wastewater
only. means use of oxygen that breaks down organic com-
pounds. Oxygen is continuously mixed into the waste-
Physical Methods water or sewage by a mechanical aeration device,
1. Screening: This is a wastewater pre-treatment. It such as an air blower or compressor.
intends to prevent coarse solids, such as plastics, rags 2. Anaerobic treatment: This is a proven and energy-
and other trash, from entering a sewage system or efficient method for treating industrial wastewater.
treatment plant. The bacteria (biomass) are used in sewage treatment.
2. Coagulation/Mixing: Following screening, the They reduce the volume of sludge. The methane gas is
coagulant must be added to the raw water and perfectly produced in the process.
distributed into the liquid, this is known as coagulation.
Such uniformity of chemical treatment is reached Polychlorinated Biphenyls (PCBs)
through rapid agitation or mixing. Coagulation results PCBs are heavy, syrupy hydrocarbons. They are non con-
from adding salts or iron or aluminum to the water. ductors of electricity, water-insoluble, and are extremely
3. Flocculation: Flocculation allows for the destabilized stable in high temperature conditions. They are used to
particles to agglomerate into larger particles that can manufacture many electrical components.
be removed by gravity through sedimentation or they PCBs were banned in 1979, after being linked with can-
may float to the surface (flotation). A floc is created, cer and developmental problems in humans. They have
that’s why this step gets the name of flocculation. This an affinity for fat, so they can stay longer to accumulate
can then be more easily removed from the liquid. in living organisms. They become more dangerous while
4. Flotation: This is a separation technique that employs moving up the food chain.
the use of gas bubbles as a transport medium. They occur in warehouses, landfills, and even rivers,
Suspended particulate matter (SPM) that is hydro- uncontrolled or abandoned hazardous wastes still
phobic (such as oil or fats that lack affinity for water) remained. A Superfund Program was created by the
attaches to the bubbles. They move away from gravity, Environmental Protection Agency to control them.
in a direction towards the aqueous solution surface.
5. Filtration: Flirtation is the process in which solid Biomedical W aste Management (BMW)
particles in a liquid are removed by the use of a filter It is the waste produced during diagnosis, treatment or
medium (having different pore sizes) that allows the immunization of human or animal research activities,
fluid to pass through while retaining the solid particles. pertaining thereto or the production or testing of biologi-
6. Sedimentation: This is a physical water treatment cal or health camps.
process using gravity to remove suspended solids The three basic principles of such good practice are
from water. 3Rs, reduce, recycle and reuse. It aims at avoiding the

M09_MADAN 07_65901_C09.indd 28 23/12/22 8:01 PM


People, Development and Environment 9.29

generation of waste or recovering as much waste as pos- 3. Grey goods—Computers, printers, fax machines,
sible rather than disposing. Hence, the waste should be scanners etc.
tackled at the source rather than ‘end-of-pipe approach’.
The e-Waste (Management) Amendment rules were
Usually 10–25% of BMW is hazardous, and the remain-
enacted in 2018.
ing 75–95% is non-hazardous.
The composition of electronic waste is well diversified.
WHO came with a ‘The Blue Book’ in 1999 that sug-
It falls under ‘hazardous’ and ‘non-hazardous’ catego-
gested newer methods for safe disposal of BMW. The book
ries. Producers and consumers of electronic goods have
was revised in 2014.
a responsibility under the E-waste (Management and
There are three international agreements and conven-
Handling) Rules, 2011, to ensure proper disposal. Now
tions which are particularly pertinent in biomedical waste
the E-waste (Management) Rules, 2016, provide several
management:
options to manufacturers, such as collection of a refund-
1. Basel Convention on Hazardous Waste: The most able deposit and paying for the return of goods to meet
inclusive global environmental treaty on hazardous the requirements of law.
and other wastes Global E-Waste Monitor was set up in 2017 by UN
2. Stockholm Convention (2006): This convention is University. 82% of our e-waste comes through personal
on persistent organic pollutants (POPs). These chem- devices. Only 1.5% of e-waste generated in India gets
icals are formed by medical waste incinerators and recycled.
other combustion processes. The vast majority of illegal e-waste ends up in landfills,
3. Minamata Convention on Mercury (2014): ­Phasing incinerators and ill-equipped recycling facilities, which
out of certain medical equipment in healthcare ser- have been described as ‘toxic time bombs’.
vices, including mercury-containing medical items, There is a short life span of electronic products. The
such as thermometers and blood pressure devices availability of choices, affordability of products, chang-
ing pace of life, rapid urbanization and increased pur-
Biomedical Waste (Management & Handling) Rules, chasing capacity of the middle class have all contributed
1998, notified under the Environment (Protection) Act, to the growth of the electrical and consumer durable
1986, require segregation according to colour code and industry. The most potent risks of electronic wastes in
treatment and disposal. India are the following:
Hazardous Waste 1. Environmental: Toxic metals, such as lead, cadmium,
mercury, arsenic, chromium, PCBs and CFC, can cause
It means any waste which by reason of any of its physi-
soil, pollution and air pollution in the form of fumes
cal, chemical, reactive, toxic, flammable, explosive or cor-
due to burning (dioxins and furans).
rosive characteristics causes danger or is likely to cause
2. Health Concerns: For general population as well as
danger to health or environment, whether alone or when
well as for those who handle it.
in contact with other wastes or substances.
3. Electronic waste often ends up in landfills in India.
Hazardous substances mostly contain the following
contaminants: There are potential assets that must be recovered, for
example, aluminium, copper, platinum, gold, silver and
1. Components of Electronic Waste: Cadmium and palladium.
lead and PVC sheathing on cables Electronic waste rules were notified by the Ministry
2. Household Chemicals: Bleach, oven cleaners, tur- of Environment Friendliness in 2011 for proper manage-
pentine and paints ment and handling. The concept of Extended Producers
3. Products Incorporating Nanoparticles: Zinc and Responsibility (EPR) has been enshrined in these rules.
titanium oxide in sunscreen, cosmetics, skin gel, etc. Electronic waste recycling can be undertaken only in
4. Commercial and Industrial Waste Stream: facilities authorized and registered with State Pollution
Chemicals and heavy metal Control Boards/Pollution Control Committee (PCC). The
5. Construction and Demolition Waste Stream: Asbestos wastes generated are required to be sold to a registered or
6. Outside those waste streams, biosolids, particularly authorized recycler or re-processor having environmen-
sewage sludge tally sound facilities.
There is comprehensive legal and regulatory framework in However, there are some limitations of electronic waste
place in India to deal with such issues, which include a lack rules. There are no take-back targets for manufacturers
of financial resources, a shortage of staff, a lack of stand- and hence, there is no clear responsibility. There are no
ardized protocols and a lack of legal authority. guidelines to set up an electronic waste collection system.
The current law does not provide for any plan to rehabili-
Electronic Waste tate those involved in informal recycling.
The electronic goods are classified under three major Among the eight largest e-waste-generating states,
heads Maharashtra ranks first followed by Tamil Nadu (second),
Andhra Pradesh (third), Uttar Pradesh (fourth), Delhi
1. White goods—household appliances (fifth), Gujarat (sixth), Karnataka (seventh) and West
2. Brown goods—TVs, camcorders Bengal (eighth).

M09_MADAN 07_65901_C09.indd 29 23/12/22 8:01 PM


9.30 Chapter 9

Table 9.6 Electronic Waste Elements


Materials Recycling (%) Location Effects
Lead 5 Acid battery, CRT Kidney failure, central and peripheral
nervous systems, damage to the
reproductive systems;
Cadmium 0 Battery, CRT, housing Long-term cumulative poison, Bone
disease;
Mercury 0 Batteries, switches, Chronic damage to brain, liver
housing damage, damage to the central and
peripheral nervous systems as well as
the foetus;
Chromium VI 0 Decorative hardener, DNA damage, lung cancer;
corrosion protection
agents
Plastic 20 Computer mouldings, Generates dioxins and furans.
cablings
Source: https://www.ncbi.nlm.nih.gov/

Plastic Waste Management Rules, 2016 Climate Change and Global


To increase the thickness of plastic carry bags from 40
to 50 micron and stipulation of 50 micron thickness for Warming
plastic sheets is likely to increase the cost by about 20 Climate change is the most important issue of the twenty-
%. Hence, the tendency to provide free carry bags will first century with potential direct adverse impact on global
come down and collection by the waste-pickers also economy and civilization (Fig. 9.13). It is a long-term
increase to some extent. The options on reuse of plastic change in the average weather patterns. It may occur over a
in various applications namely, road construction, period of time, which may range from a decade to millions
waste to oil, waste to energy will enhance the recycling of years. Climate change may be limited to a specific region
of plastic. or may occur across the whole world.
Palaeoclimatology is the study of climatic changes
Ban on Single Use of Plastic, 2022: India has banned taken on a scale of the entire history of the earth. It makes
manufacture, import, stocking, distribution, sale and use of data from ice sheets, tree rings, sediments, corals,
use of identified single use plastic items, which have low shells and rocks.
utility and high littering potential, all across the country Global warming is the progressive increase in the aver-
from July 1, 2022. age temperature of the earth’s near-surface air and oceans
The carry bags made of virgin or recycled plastic, shall during the last few decades and its likely continuation in
not be less than 75 microns in thickness till 31st December future as well. This is mainly due to the emission of heat-
2022 and after that 120 micron. retaining GHGs into the atmosphere, which results from
Extended Producer Responsibility (EPR): This is the human activities, such as burning of fossil fuel and defor-
responsibility of a producer for the environmentally sound estation. One of its main effects is a shift in the global
management of the product until the end of its life. weather patterns, referred to as climate change. Here, it is
important to mention that global warming is closely asso-
ciated with climate change and both terms may be used
Stopover interchangeably.
Match the following in the context of waste
Causes of Climate Change
management: They can be divided into two types of factors, natural and
(a) Incineration: Uncontrolled combustion process anthropogenic.
(b) Autoclaving: Low-heat thermal process
(c) Microwaving: Radiation frequency Natural Causes
(d) Shredding: Cutting into smaller blocks and The main natural factors are as follows:
disinfection
The correct option is (a). 1. Continental Drift: Almost all continents on the globe
appear to fit with each other like the parts of a jigsaw

M09_MADAN 07_65901_C09.indd 30 23/12/22 8:01 PM


People, Development and Environment 9.31

Socio-economic Systems Environmental Systems


• Population growth • Loss of Habitat and Biodiversity
EFFECTS
• Globalization • Natural resource degradation
• Economic conditions • Land-use changes
• Chemical-based industry GHG • Loss of environmental quality
• Commercial exploitation emission • Loss of ecosystem services
• Energy intensiveness • Loss of assimilative capacity
Natural charge Climate
Increased in
change
hazards and
Natural hazards extreme events
Disaster impacts
Increased demand for DISASTER
(damages and
relief and recovery losses)

Increased in
vulnerability of land
and community

Figure 9.13 Causes of Climate Change

puzzle. Continents were formed when parts of a large fossil fuels and, thus, contribute to global warming and
landmass called Pangaea began drifting apart gradu- climate change. All our gadgets run on electricity, which
ally around 200 million years ago. There are many is generated mainly from thermal power plants that fur-
similarities between plant and animal fossils and ther run on fossil fuels (coal).
rocks on the two continents.
2. Volcanoes: Large volumes of SO2, water vapour, dust Effects of Climate Change and G lobal
and ash are thrown out into the atmosphere during
volcanic eruption. SO2 can reach the upper levels of W arming
atmosphere (called stratosphere) where it combines Cyclonic Storms
with water to form tiny droplets of sulphuric acid.
Both the intensity and the frequency of tropical storms
These small droplets and dust particles reflect sunlight
have increased in the past decade. They are caused by
and partially block the incoming rays of the sun, lead-
evaporation of water from oceans. Coriolis effect causes
ing to cooling in the lower levels of atmosphere (tro-
the storms to spin and a hurricane is declared when this
posphere). Winds in the stratosphere carry the aero-
spinning mass of storms attains a wind speed greater than
sols rapidly around the globe in either an easterly or
119 km/h.
a westerly direction. This gives some idea of cooling,
An ice storm is a particular weather event in which pre-
which is brought about for a few years after a major
cipitation occurs as ice due to atmospheric conditions.
volcanic eruption.
3. Earth’s Tilt: The earth makes one revolution around
Loss of Biodiversity
the sun in 365 days. It is tilted at an angle of 23.5°
to the perpendicular plane of its orbit, which causes The most dramatic impact is the loss of habitat for m
­ illions
seasonal variations. Furthermore, the earth’s orbit is of species.
somewhat elliptical, which means that the distance 1. Seventy per cent of earth’s land animals and plants live
between the earth and the sun varies during the in forests, and many cannot survive the loss of their
course of a year. natural habitat. Deforestation results in the decline of
4. Ocean Currents: Oceans cover about 71% of the earth biodiversity and in the extinction of many species.
and absorb about twice as much of the sun’s radia- 2. Forests support biodiversity, providing habitat for
tion as the atmosphere. Ocean currents transfer vast wildlife. Moreover, forests foster medicinal conser-
amounts of heat across the planet, which causes tem- vation. With forest biotopes being an irreplaceable
perature difference and climatic changes. source of new drugs (such as taxol), deforestation can
destroy genetic variations (such as crop resistance)
Anthropogenic Causes (Human Causes) irretrievably.
Beyond a point, human activities and consumption 3. It was only during Earth Summit in 1992 that these
styles are considered as human interference in nature. figures came out, which equates to 50,000 species
Urbanization and industrialization have been powered by a year.

M09_MADAN 07_65901_C09.indd 31 23/12/22 8:01 PM


9.32 Chapter 9

Sea-level Rise and Small Islands 3. Emission of CO2 and Methane: More CO2 and meth-
The role of oceans in global warming is very complex. They ane are emitted from nature. Soils, forests, peat, seas,
serve as a sink for CO2, taking up much that would oth- organic deposits in permafrost and methane clath-
erwise remain in the atmosphere, but increased levels of rates all emit some amount of CO2 and methane. As
CO2 have led to the acidification of oceans. Furthermore, the environment warms, natural emissions increase.
as the ocean temperature rises, their ability to absorb Tipping point is a small amount of warming that may
excess CO2 decreases. Global warming is projected to set off unstoppable and irreversible changes. The best
have a number of effects on the oceans. The ongoing example is the melting of ice caps. Once the temperature
effects include rising sea levels due to thermal expansion goes up by certain degrees, all ice caps may melt (even
and melting of ­glaciers and ice sheets, the warming of the though complete melting of ice at the Arctic and Antarctic
ocean surface leading to increased temperature stratifica- may take thousands of years). The tipping point in many
tion. Other possible effects include large-scale changes in scientists’ view is a 2°C rise in temperature. The European
oceanic circulation. Union has adopted that as the maximum limit that man-
Bleaching of Coral Reefs kind can risk. Beyond that point, there is a possibility of
runaway climate change.
Coral reefs are a collection of biological communi-
ties forming one of the most diverse ecosystems of the Key Developments in the Context of Climate
world (termed rain forests of the oceans). Corals are
important for a variety of reasons, and some of these Change and G lobal Warming
are as follows: 1. 1824: Jean Baptiste Joseph Fourier was the first to
describe that without the presence of GHGs, the earth
1. They provide habitat for a variety of organisms. would have been cooler by 33°C. When radiations
2. They prevent erosion of soil on beaches. from the sun enter the earth’s atmosphere, they are
3. They function as a carbon sink (help in absorption of of short wavelength, and when emitted by the earth,
CO2). they are of longer wavelength. GHGs do not allow
They are found in shallow coastal areas of tropical and these radiations to escape into the outer atmosphere
sub-tropical regions, where light can penetrate for syn- and hence, they are reflected back to the earth. This
thesis of food. They feed on small fish and live in colo- causes the heating of the earth.
nies, where each coral is called a polyp. They enjoy a 2. 1896: Svante Arrhenius claimed that fossil fuel com-
symbiotic relationship with algae. The increase in sea bustion may eventually result in enhanced global
temperature, salinity of water, increased UV radiation warming. As much as 25% of CO2 emissions are nat-
and so on will result in decreased photosynthesis activ- urally absorbed by the oceans, and another 25% are
ity, and this in turn leads to loss of algae. Ultimately, the absorbed by the biosphere, such as trees, plants and
reefs become dead and lose their colour. This is called soil. It is evident that 50% of CO2 emissions are not
coral bleaching. absorbed by nature and accumulate in the atmosphere.
Melting of polar ice and migration of fish are other 3. 1950: The World Meteorological Organization
effects of climate change on marine life. (WMO) was set up in Geneva to promote interna-
tional exchange of weather reports and other weather-
Runaway Climate Changes and T ipping Point related services.
Runaway climate change is what happens when global 4. 1950s and 1960s: Aerosol pollution called smog
warming becomes self-sustaining and beyond the con- became a serious local problem in many cities, caus-
trol of human beings. This may upset the normal sys- ing dimming and fall in temperature to such an extent
tem of checks and balances that keep the climate in that many scientists talked about the return of the Ice
equilibrium. Age. The phenomenon of London Smog in 1954 is a
A global warming spiral kicks in if the following pos- prominent example.
sibilities occur: 5. 1958: Keeling curve is a graph that has been show-
ing the variations in the concentration of atmospheric
1. The Environment Absorbs less CO2: About 50% of our CO2. It is based on continuous measurements taken at
current emissions are absorbed by the ­environment, that the Mauna Loa Observatory in Hawaii.
is, roughly half of that by the oceans and the other half by 6. Roger Randall Dougan Revelle suggested that the
the plants on land. The uptake of CO2 by the environment earth’s oceans would absorb excess CO2 generated by
may already be in decline. humanity at a much slower rate, thereby contributing
2. Reflection of Sunlight Drops: As snow covers in the to the greenhouse effect and global warming. Revelle
form of glaciers are retreating (meaning that they factor is a measure of resistance to the absorption of
are shrinking in terms of geographical extent), dark atmospheric CO2 by ocean surface layer due to differ-
grounds and darker water are exposed, which absorb ent factors.
less sunlight and this has caused further increase in 7. 1972: UN Conference on Human Environment (also
global warming. called as Stockholm Conference) was held.

M09_MADAN 07_65901_C09.indd 32 23/12/22 8:01 PM


People, Development and Environment 9.33

Concept Box
Greenhouse Gases
‘Climate change is real. There is strong evidence that significant global warm-
ing is occurring’.
The greenhouse effect is a natural phenomenon. Without the greenhouse
effect, the earth would have been much colder at –18°C. With it, the aver-
age temperature of our earth is 14°C. Greenhouse gases act like a blanket for Sunlight
the earth. This blanket is found in the troposphere layer of the atmosphere.
Greenhouse gases are very picky about which kind of radiation they will
absorb. Some heat
The whole process begins when the shortwave solar radiations reach the escapes
earth and heat it up. The earth then radiates long-wave or infrared radiation into space
back into the space. This infrared radiation heats the atmosphere and some Greenhouse
of it is trapped by the greenhouse gases. These radiations come back to the gases trap
earth. Thus, our atmosphere warms up in the process (Fig. 9.14). some heat
As a result of anthropogenic activities, the amount of greenhouse gases
has only increased since the 1850s, and resulted in an increase in tempera-
Atmosphere
ture that is harmful to us.
The Global Warming Potential (GWP) was developed to allow compari-
sons of the global warming impacts of different gases. Specifically, it is a
Earth’s surface
measure of how much energy the emissions of 1 ton of a gas will absorb over
a given period of time, relative to the emissions of 1 ton of carbon dioxide
(CO2). The larger the GWP, the more a given gas warms the earth compared Figure 9.14 Green House Effect
to CO2 over that time period. The GWP tentatively depends on the mass of
a gas.

Global Warming
Gas Source Share of Global Emissions Potential
Carbon dioxide Fossil fuels, deforestation, Total—76.7%, with 56.6% 1
degradation of soils, land from fossil fuel use
clearing
Methane Agricultural activities, energy 14.3% 21
production, waste
Nitrous oxide Agricultural activities—fertilizer 7.9% 310
use, fossil fuel combustion
Hydrofluorocarbons Used as replacements for 1.1% 140–11,700
ozone-depleting substances
Perfluorocarbons 6500–9200
Sulphur hexafluoride Used in some industrial Very less 23,900
processes and in electric
equipment
Nitrogen trifluoride During production of electronic Very less 17,200
components, fluorinated
compounds, high- energy lasers

M09_MADAN 07_65901_C09.indd 33 23/12/22 8:01 PM


9.34 Chapter 9

Table 9.7 CO2 Emissions by Nations


Country Percentage Share in CO2 Emissions per
Global Annual Emissions Capita (tons/person)
World 100 4.9
China 28.6 7.1
United States 15.1 16.4
European Union 10.9 7.4
India 5.7 1.6
Russia 5.1 12.4
Japan 3.8 10.4
Source: Press Information Bureau (PIB).

Fossil fuel
supply Waste
Buildings 5% 3%
8% Power supply
21%

Transport
13%

Industry
Agriculture 19%
14%

Forestry
17%
Sources of GHG Emissions

8. 1975: Wallace S. Broecker was the first to use the 12. 1992: The United Nations Framework Convention
term ‘global warming’. on Climate Change (UNFCCC) was set up in 1992
9. 1979: The first World Climate Conference took place during Rio Summit. It is an international environ-
in Geneva. mental treaty produced at the UN Conference on
10. 1983: The UN’s World Commission on Environment Environment and Development (UNCED; also known
and Development was set up to discuss sustainable as the Earth Summit) held in Rio de Janeiro (Brazil)
development. It was later named the Brundtland in June 1992. Its objective is to stabilize GHG concen-
Commission. trations in the atmosphere at a level that would pre-
11. 1988: The Intergovernmental Panel on Climate vent dangerous anthropogenic interference with cli-
Change (IPCC) was set by the WMO and UNEP to mate system. The UNFCCC was opened for signature
assess the risk of human-induced climatic changes in May 1992; it entered into force on 21 March 1994.
and its impact, and to suggest alternatives solutions. Agenda 21 is an action plan of the UN related to sus-
It comes with ‘Assessment Reports’ (ARs). The IPCC tainable development and was an outcome of the
shared the Nobel Peace Prize 2007 with the for- Earth Summit. It is a comprehensive blueprint of the
mer US Vice President Al Gore who also wrote ‘An actions that need to be taken globally, nationally and
Inconvenient Truth’ about climatic changes and global locally by organizations of the UN, governments and
warming. major groups in every area in which humans directly
affect the environment.

M09_MADAN 07_65901_C09.indd 34 23/12/22 8:01 PM


People, Development and Environment 9.35

Note: Carbon Footprint is judged with energy emissions Intensive A gricultural Practices
from energy sector (68%), agriculture (19.6%), indus- To meet the food requirements of large population, more
trial process (6%), Land Use Change (3.8%) and Forestry and more land has been brought under cultivation.
(1.9%).
Global warming is now 400 parts per million (ppm) in Culture of Consumerism
comparison to 280 ppm in pre industrial era. Forests are
major carbon sinks (almost 45%). Excessive demand leads to a mad scramble for resources
and conflicts.

Stopover Non -equitable Distribution of Resources


1. Sea level arises primarily as a result of The raw material for finished goods is available in
(a) Heavy rainfall underdeveloped or developing nations. To earn foreign
(b) Melting of glaciers exchange and taxes, the governments allow the exploita-
(c) Submarine volcanism tion of resources, however, without a long-term approach
(d) Seafloor spreading to replenish them or mitigate the after -effects.
The correct option is (a). Natural resources around us provide a variety of
sources of energy. During the Stone Age, it was wood.
2. Which of the following gases has the highest global During the Iron Age, we had coal. In the modern age, we
warming potential? have petroleum and natural gas. In the near future, solar
(a) Carbon dioxide and geothermal energy may dominate the scene.
(b) Methane Good sources of energy should have the following
(c) Chlorofluorocarbon qualities:
(d) Sulphfur hexafluoride
The correct option is (d). • Optimum heat production per unit of volume/mass
used
• Easy to transport
• Least polluting
Our Natural Resources • Cost-effective
In the beginning of chapter, we discussed about environ-
mental determinism and environmental possibilism. They Our Energy Resources
reflect how the energy pattern has shifted.
Energy is the capacity to do work. It is the basic require-
Rapid I ndustrialization, Urbanization and ment for a living being, machine and matter to move, func-
I ncrease in Population tion or perform any kind of work. The whole development
of civilization is based on the availability of energy. Energy
All our gadgets run on electricity, generated mainly from is present in different forms and it has been further modi-
thermal power plants, which further run on fossil fuels fied from time to time to suit the requirements of mankind.
(coal). Manufacturing industries are primarily located The common forms of energy are as follows:
in urban areas, which create jobs, and people move from
rural areas to the cities over the years. This process is con- 1. Mechanical energy of a body is the energy it possesses
tinuing even today. by virtue of its motion or its position. When a body
During the twentieth century, world population is in motion, it possesses kinetic energy. Potential
increased by 4.5 times, from 1.5 to 7.6 billion, but urban energy is the energy possessed by a body due to its
population grew 13 times from 225 million to 3.4 bil- position. For example, energy stored in a compressed
lion, or 47% of the total population. By 2030, the figures spring is an example of potential energy.
are likely to increase to 4.9 billion or 60%. Urban areas 2. Thermal energy is the energy a substance or system
cover just 3–4% of the world’s land surface and accom- has in relation to its temperature, that is, the energy
modate half of the world’s population but consume of moving or vibrating molecules.
around 80% of the global energy supply and, thus, emit 3. Chemical energy is stored in the form of molecular
the bulk of GHGs. bonds.
Transport vehicles also run mainly on petrol or d ­ iesel, 4. Nuclear energy is the mass converted into energy.
both of which are fossil fuels. Out of consumer cult, plas- 5. There can be a few other forms of energy, such as radi-
tics, timber and other natural resources are being used in ant energy or light energy.
a big way.
Law of Conservation of Energy
Availability of T echnology The total amount of energy in the universe remains con-
Increase in sophistication of technology enables quick stant. It changes from one form to another. For example,
and efficient extraction of natural resources. For example, when water is stored in a dam at a height, it possesses
rates of deforestation have increased greatly due to elec- potential energy. However, when water falls through the
tric saws. turbine, it possesses kinetic energy.

M09_MADAN 07_65901_C09.indd 35 23/12/22 8:01 PM


9.36 Chapter 9

Energy Scenario in India The main disadvantages of using fossil fuels are in
Electricity Act was enacted in the year 2003. Its objective terms of harm to the environment:
is to introduce competition so as to protect consumer’s 1. Global Warming: The SO2 and CO2 produced during
interests and provide power for all. the burning of fossil fuels contribute towards global
The contribution of different sectors in installed warming and acid rains.
generation capacity as per June, 2022 data–Central Sector 2. Health Hazards: Fly ash and other particulate matter
(24.6%), State Sector (26%) and private sector (49.5%). cause health hazards, such as asthma and tuberculosis.
As per September 2022 report, the contribution of 3. Oil Spills: They are a threat to marine life and our
different sources in energy production is divided into ecosystem.
three parts:
Eventually, in the future, fossil-based resources will be
Fossil Fuels (basically thermal–total 57.8%): expensive to harvest, and humanity will need to shift its
Coal–50 reliance to other sources of energy.
Lignite–1.6 As fossil fuels are non-renewable and thus finite, they
Gas–6.1 will eventually run out of stock, or they will become too
Diesel–0.1 expensive or too environmentally damaging to retrieve. In
Non-Fossil Fuels (42.2%) contrast, many types of renewable energy resources, such
Hydro–11.5 as wind and solar energy, are constantly replenished and
Wind–10.2 will never run out. At present, the main energy sources
Solar–14.9 used by humans are non-renewable fossil fuels as they
Bio Mass/Cogen–2.5 meet 80% of our energy needs.
Waste to energy–0.1
Small Hydropower–1.2 Petroleum Oil
Nuclear Energy–1.7% Oil occurs in rock formations in the earth, which before
Overall, India is the third-largest producer and getting processed in refineries is called crude oil, a mixture
second-largest consumer of electricity worldwide, with of hydrocarbons. It is processed by fractional distillation
an installed power capacity of 401.01 GW as of April 30, and transported to points of consumption.
2022. 100% FDI allowed in the power sector has boosted The petroleum exploitation impact environmental in
FDI inflow in this sector. a bad manner. The extraction, refining and burning of
petroleum fuels all release large quantities of greenhouse
Primary Energy and Secondary Energy gases, so petroleum is one of the major contributors to
climate change.
Primary energy form is directly found in nature, such as
coal and sunlight. This energy can be renewable or non- Ethanol
renewable. When primary energy form is converted into Ethanol (ethyl alcohol) is one of a group of chemical
some convenient form of energy, it is known as secondary compounds (alcohols). It contains a hydroxyl group
energy. For example, coal or sunlight is converted into (OH) bonded to a carbon atom. Ethanol is produced
electrical energy, which can be consumed in homes or through the fermentation of agricultural products such
industry. as sugarcane, corn, and manioc, among others. Most
ethanol is produced from sugarcane, mainly in Brazil. In
N on -renewable Energy Sources USA, ethanol is made from corn.
A non-renewable resource is a natural resource that can- Ethanol is used as a high-octane fuel in vehicles. Currently
not be reproduced, grown, generated or used on a scale 10% of ethanol is allowed in petrol, the government may
which can sustain its consumption rate. However, once it enhance it to 20%, depending upon availability conditions
is depleted, there is no more available for future needs. of ethanol. Govt has set the target of year 2025 to do the
Resources that are consumed much faster than nature can same. This may require some changes in the engine also.
create them are also considered as non-renewable. They
basically consist of fossil fuels. Coal
Coal is a black or brown carbonaceous sedimentary rock
Fossil Fuels formed by combustion of partially decomposed plant material.
Fossil fuels consist of oil and coal. They are preferred It takes millions of years to form from decayed plants. The pro-
for the following main reasons: cess of formation of coal is termed coalification.
Coal provides 30.3% of global primary energy needs and
1. They have a high calorific value.
generates 42% of the world’s electricity. In India, 70–80%
2. The technology is available to exploit these resources.
of electricity is produced by burning coal. The amount of
The market is well developed for trading of the fossil energy in coal is expressed in British Thermal Units per
fuels. Our conventional infrastructure and transport sys- pound (a BTU is the amount of heat required to raise the
tems, which are fitted with combustion engines, remain temperature of 1 lb of water by 1°F). Higher the carbon
prominent throughout the globe. content in coal, higher is its calorific value and its quality.

M09_MADAN 07_65901_C09.indd 36 23/12/22 8:01 PM


People, Development and Environment 9.37

The grading of coal is done on the basis of the carbon margins. Gas hydrate resources in India are estimated at
contents: 1,894 trillion cubic meters and these deposits occur in
Western, Eastern and Andaman offshore areas.
1. Peat: Peat is not a typical variety of coal. The ­carbon
content is 50–60%. Under conditions of temperature Compressed Natural Gas and Piped Natural Gas
and pressure, it converts into lignite, bituminous [CNG (transport) and PNG (domestic)]
and subsequently anthracite. The carbon contents
increase in subsequent stages. CNG is one of the cleanest and most environment friendly
2. Lignite or Brown Coal: It contains 70% carbon and fuel as compared to other fuels used by automotive
is found in geologically young mines and is the lowest vehicles. The level of vehicular emissions is significantly
rank of coal with a high moisture content. lower in case of CNG when compared to liquid fuels like
3. Bituminous Coal: It is also termed soft coal and it Diesel and Petrol. Natural gas when compressed becomes
contains around 80% carbon. CNG to use as fuel in automobiles.
4. Anthracite coal: The burning of anthracite coal gives The same gas is also piped to household kitchens and
very little smoke. industries for cooking and other purposes, then called as
PNG. Both are refined natural gas with methane being the
When bituminous coal is heated at extremely high primary constituent.
temperature, the residual matter is called coke. Different Their molecular structure is compact that prevents
types of coal have different uses. Steam coal, also known the formation of Ozone (O3) in the troposphere. CNG is
as thermal coal, is mainly used in power generation. lighter than and narrow range of ignition. It is cheaper
Coking coal, known as metallurgical coal, is mainly used than conventional fuel.
in steel production. It is composed primarily of carbon
and hydrocarbons. Its extraction causes environmental Renewable Energy Resources
hazards. Air pollution (mainly SPM) due to burning of Most renewable energy comes either directly or indirectly
coal causes respiratory problems. from the sun. Sunlight, or solar energy, can be used directly
Coal bed Methane (CBM): This is a unconventional for heating and lighting homes and other buildings, for
source of natural gas. This is now considered as an generating electricity and for hot water heating, solar
alternative source for augmenting India’s energy resource cooling and a variety of commercial and industrial uses.
that is the fifth largest proven coal reserves in the world. The sun’s heat also drives the winds, whose energy is cap-
Thus, there are significant prospects for exploration and tured with wind turbines. Then, the winds and the sun’s heat
exploitation of CBM. cause the water to evaporate. When this water vapour turns
Thermal power stations produce waste in the form into rain or snow and flows downhill into rivers or streams,
of fly ash. Large dumps are required to dump this waste its energy can be captured using hydroelectric power.s
material. Burning coal causes smog, soot, acid rain, global According to government sources, 60% of India’s
warming and toxic air emissions. total power capacity will come from renewable sources
Coal is the most abundantly available fossil fuel, which at by 2030. This ambitious target will help India offer the
the current rate of consumption may last up to 200 years. global community a 35 % reduction in the GHG emission
China, the USA, India, Australia and Indonesia are the five intensity of its economy below 2005 levels by 2030 as
major coal producers in the world. Coal deposits in India part of its Intended Nationally Determined Contributions
belong to the Gondwana age. Nearly three-fourths of coal (INDCs) under the Paris Agreement.
deposits are situated in Damodar Valley. As per Ministry of New and Renewable Energy (MNRE)
An unconventional form of gas formed during coalifi- Report 2021, the share of wind energy is 40.8% in energy
cation process and found on the internal surfaces of coal production from renewable energy sources.
is called ‘coal bed methane’. MNRE has set a target of achieving 127 GW by RE. That
can be split into 114 from Solar, 67 from wind and rest
Natural Gas from others. There is a plan to set up 5000 bio gas plants
in India by 2023. There is plan to increase RE electricity
1. Natural gas mainly comprises methane, butane,
supply to 175 GW and 450 GW by 2030. 49% of electricity
ethane and propane, and it has a very high calorific
supply will be generated by RE by the year 2040.
value.
The achievement of SDGs has been linked with RE
2. Some of the organic material was changed by heat
energy production. Five SDGs are highly linked (with a
and pressure into oil and coal, while natural gas was
score of 3 out of 3) with RE production and three SDGs are
trapped within the earth’s crust.
moderately associated (with a score of 2 out of 3). Four
3. It was formed from decaying plants and animals, mil-
SDGs are least linked and five SDGs are not concerned.
lions of years ago.
Natural gas hydrates: They occur naturally in ice like Solar Energy
combination of natural gas and water found in oceans Solar energy is the ultimate source of energy for almost all
and polar regions. Their reserves are found to be far larger living organisms. It is the heat and light energy produced
than the volume of all known conventional gas resources. as a result of nuclear fusion and fission reactions taking
They occur in marine sediments on continental shelf place inside the sun (Fig. 9.15).

M09_MADAN 07_65901_C09.indd 37 23/12/22 8:01 PM


9.38 Chapter 9

5. Silicon used in the production of solar photovoltaic


(SPV) cells is a pollutant.
Grid
6. Hybrid solar cell battery is made up of three
electrodes–the first electrode is the mesh solar panel
(solar cells), the second is of a thin sheet of porous
carbon and third one is a sheet of lithium metal.
Battery National Solar Mission: The National Solar Mission is
AC one of the eight missions set up by the National Action
(optional)
Meter and Inverter Plan on Climate Change (NAPCC) that was released on
Fuse box (DC to AC) 30 June 2008.
DC
The government stepped up India’s solar power capac-
ity target under the Jawaharlal Nehru National Solar
Mission (JNNSM) by five times, reaching 1,00,000 MW
by 2022. The new targets were approved in June 2015.
The 100-GW target itself is divided into two parts:
Home appliances
1. Forty gigawatts through rooftop
Figure 9.15 Typical Grid-Connected PV Solar System 2. Sixty gigawatts through large- and medium-scale
grid-connected solar power projects
The producers produce food by photosynthesis. Energy With this ambitious target, India will become one of
from the sun is responsible for all the weather phenomena the largest green energy producers in the world, surpass-
in nature, such as the wind, storms, rain and sea waves. ing several developed countries.
Now, scientists are devising methods to make optimum The following are the main factors for increase in pop-
use of solar energy in routine lives in the form of solar ularity of solar power:
cooker, water heater and solar cells, which can be used in
multiple devices. Many advanced nations have come up 1. Improvements in technology
with concepts such as energy-efficient green buildings. 2. Scaling up production capacities
A solar power system is a set-up that generates electric- 3. Cost competitiveness
ity by utilizing the solar energy system. A typical solar sys-
tem consists of the following: Wind Energy
Wind energy possesses some kinetic energy due to its high
1. Solar panels (which absorb sunlight) speed. It can produce mechanical or electrical energy by
2. Inverter (which converts DC into AC) using windmills. Wind is a result of solar energy, as heat-
3. Mounting structure (which hold the panels in place) ing of land results in the movement of air. Wind energy
4. Batteries (to store the extra power generated) has been used for hundreds of years for sailing, grinding
5. Grid box grains and irrigation. Wind energy systems convert kinetic
6. Balance of systems (wires, nuts, etc.) energy of winds to other forms of energy or to generate
A solar system comes in various sizes such as 1, 3, 5, electric power. Windmills for water pumping have been
and 10 kW. installed in many countries, particularly in the rural areas.
To generate electricity on a large scale, a number of
Advantages windmills are set up over a large area called a wind energy
1. Solar energy is a readily available, inexhaustible, farm. Such areas need a wind speed of 15 kmph.
clean, uninterrupted and continuous source of energy.
2. Solar devices can be installed in remote, inaccessible Advantages
areas, such as small villages in interior regions, for- 1. Inexhaustible source of energy
ests, deserts, mountains, off-shore platforms and 2. No pollution and emission of GHGs
remote oceanic islands. 3. Possibility of large-scale production
3. It is possible to produce solar energy in large quanti- 4. Scope of direct use as mechanical energy
ties across many regions in the world, especially tropi- 5. Can use land around wind turbines for other pur-
cal regions. poses, for example, farming
Limitations Limitations
1. It can be produced in tropical and sub-tropical areas 1. Storage is expensive during peak production time.
only and that too in specific seasons. 2. Winds are uncertain and unpredictable.
2. Technology is still expensive and involves a high 3. The visual aesthetic impact is not good.
installation cost. 4. Large open areas are required for setting up wind farms.
3. It is difficult to store and run heavy machines. 5. Noise pollution occurs.
4. Solar panels consume land, as power generation per 6. There is a possible threat to wildlife.
square unit is low. 7. Maintenance cost is high.

M09_MADAN 07_65901_C09.indd 38 23/12/22 8:01 PM


People, Development and Environment 9.39

in the form of dams. Potential energy is converted into


More Data about Renewable Energy kinetic energy.
1. According to MNRE data, Rajasthan, Karnataka, 2. It is the second largest source of electricity.
Gujarat, Tamil Nadu and Andhra Pradesh are the 3. It is heavily dependent on rainfall and melting of
five top states in solar power generation in 2021. snow in the mountainous regions.
2. China, USA, Japan, India and Germany are the top 4. It entails heavy investment for construction of dams,
five nations for solar power generation in the year but per unit cost of electricity is low.
2021.
3. Located in Jodhpur, Rajasthan, Bhadla Solar Park Risk factors include the following:
is the world’s largest solar park located in India. 1. As a huge water body is created, the release
Pavagada Solar Park (Karnataka), Kurnool Ultra during heavy rainfall may cause floods and loss of
Mega Solar Park (Andhra Pradesh), NP Kunta biodiversity.
(Andhra Pradesh) and Rewa Ultra Mega Solar 2. Dams impede the migration of fish along the river. The
(Madhya Pradesh) follow as other major four solar silt pile-up may threaten the structure and decrease
power plants in India. They will generate mini- the life of the dam.
mum of 10 MW of power.
4. A floating solar power plant is going to be built in
Geothermal Energy
Madhya Pradesh’s Khandwa.
5. Over 800 biomass power and bagasse / non- Geothermal energy is thermal energy, which is generated
bagasse cogeneration projects aggregating to through the natural hot springs.
10205.61 MW capacity have been installed in the The core of the earth is very hot and so it is possible
country for feeding power to the grid. to make use of this geothermal energy. These are areas
where water and steam gush out in the form of hot springs
and geysers, which may be used to run turbines to pro-
duce electricity.
Gujarat, Rajasthan, Maharashtra, Tamil Nadu and Madhya
The various methods used to produce geothermal
Pradesh are the five top states in terms of Wind Power
energy are the following:
Potential. India ranked fourth behind China, the US and
Germany in wind power. India ranked third globally for 1. Dry steam power plant
total renewable power capacity additions with 15.4 GW 2. Flash cycle plant
in 2021, following only China (136 GW) and the US (43 3. Binary cycle power plants
GW).
The Netherlands is called ‘the land of windmills’. Presently in India geothermal energy installed capacity
is mostly experimental; however, the potential capacity is
more than 10,000 MW.
There are seven geothermal provinces in India: the
Himalayas, Sohana, West coast, Cambay, Son–Narmada–
Major Renewable Energy Institutes in India Tapi (SONATA), Godavari and Mahanadi.
1. National Institute of Solar Energy, Gurgaon
2. National Institute of Wind Energy, Chennai
Biomass
3. SSS National Institute for Renewable Energy,
Kapurthala (Punjab) Organic matter that makes up the plants is known as bio-
4. Indian Renewable Energy Development Agency, mass. Biomass is derived from sources such as b­ y-products
New Delhi of the timber industry, agricultural crops, forest waste,
5. Solar Energy Corporation of India, New Delhi household waste and municipal waste dumps. It can be
used to produce electricity, transportation fuels or chemi-
cals. The use of biomass for any of these purposes is called
In 2014, the scenario was Fossil Fuels were 69.5%, Hydro bioenergy.
was 16.1%, Nuclear was 1.9% and Renewable was 12.5%. Combustion, Gasification, Anaerobic Digestion and
Thus, we can see that gap been coal and solar has been Pyrolysis are the main techniques used for production of
narrowing over a period of time. energy from biomass.
The contribution of solar and wind energy in select
Indian states in their total energy production dur- Advantages
ing 2020 –21 is given as: Karnataka (29%), Rajasthan
1. It is a renewable, cost-effective and less polluting
(20%), Tamil Nadu (18%), Andhra Pradesh (16%) and
source of energy.
Gujarat (14%).
2. It provides manure for agriculture and gardens.
3. There is tremendous potential to generate biogas
Hydroelectric Power energy.
1. It is electricity generated using the force of running 4. Growing biomass crops use carbon dioxide and pro-
water falling from a height. The water may be stored duce oxygen.

M09_MADAN 07_65901_C09.indd 39 23/12/22 8:01 PM


9.40 Chapter 9

Limitations 3. The leftover matter called slurry is used as manure in


1. Initial cost of construction of biogas plant is high. agriculture fields.
2. Continuous supply of biomass is required to generate Biomass fuels used in India account for about one-third
biomass energy. of the total fuel used in the country, being the most impor-
3. Storage and transportation is difficult. tant fuel used in over 90% of the rural households and
4. Many food crops such as corn and wheat are being about 15% of the urban households.
diverted to make ethanol, which may lead to high Biomass can be used in briquette form, which is used
food inflation. directly as fuel instead of coal in the traditional chulhas
and furnaces. Alternatively, gasifiers convert solid fuel
Biofuel into a more convenient-to-use gaseous form of fuel called
Bio-fuels are produced from animal waste, algae, and producer gas.
industrial and agricultural waste. Biofuels are renewable
liquid or gaseous fuels made from living organisms or the Hydrogen
wastes that they produce. Bioethanol and biodiesel are the 1. It is the most abundant element on the earth and has
two main types of biofuel that are currently ­commercially the highest calorific value.
produced. This oil is extracted and mixed with diesel and 2. It does not occur naturally as a gas.
is used as fuel. Bioethanol is produced from sugar beet, 3. As it is highly reactive, it combines with other ­elements
sugarcane and corn. The biofuels are divided into two cat- such as oxygen to form water.
egories. 4. Once separated from other elements, hydrogen can
be burned as a fuel or converted into electricity.
First-generation or Traditional Biofuels 5. As it burns completely, it does not result in atmo-
spheric pollution and in greenhouse effect.
Another source of biofuels is oil extracted from the seeds
of plants such as Jatropha and Pongamia, which have good Fuel Cells
calorific value. They are less sustainable as they have an
Fuel cells use hydrogen as a fuel. They convert the chemi-
adverse impact on the supply of food for the human popu-
cal energy of a fuel directly and efficiently into electricity
lation and, hence, are less preferred. Food prices increase
and heat. Thus, they are electrochemical devices. There
as a result.
is no combustion as in the case of fossil fuels. Hydrogen
or a mixture of compounds containing hydrogen is used
Second- and Third-generation Biofuels as a fuel. It consists of two electrodes, with electrolyte in
They are generated from non-food crops. Mi­crobes play a between. Oxygen passes over one electrode and hydrogen
key role in the development of these biofuels. They are more over the other and they react electrochemically to gener-
sustainable than first-generation biofuels, as they produce ate electricity, water and heat.
higher yields, reduce GHG production and do not compete The fuel cells have been used in space flights as well.
with crops grown for food. Example is oil extracted from Electric vehicles using fuel cells for their energy require-
seeds of plants, such as Jatropha and Pongamia, which ments are the best options available to dramatically
have good calorific value. reduce urban air pollution. Not all renewable energy
In the beginning of 2013, it was recommended that resources come from the sun.
5% ethanol blending is mandatory for petrol, whereas A hybrid car is a petrol- and electricity-driven vehicle–a
in 2007, it was recommended to be as low as 10% by the car starting with petrol engine and switching to electric
Group of Ministers. motor at low speed.
Shale Gas
Renewable Power Capacity (Ministry of
Shale gas refers to natural gas that is trapped within shale Non-Renewable Energy) as of 31 March 2018
formations. Shales are fine-grained sedimentary rocks
that can be rich sources of petroleum and natural gas.
Over the past decade, the combination of horizontal drill- Total Installed 2022 Target
ing and hydraulic fracturing has allowed access to large Source Capacity (MW) (MW)
volumes of shale gas that were previously uneconomical Wind power 34,046 60,000
to produce. The production of natural gas from shale for-
mations has rejuvenated the natural gas industry in the Solar power 21,651 1,00,000
United States. Biomass power 8701 10,000
Biogas Waste-to-power 138
1. Bacterial action is introduced in digesters with sewage
Small hydropower 4486 5000
of human beings and animals (animal dung).
2. The decomposition of sewage produces methane, Total 69,022 1,75,000
which is used for cooking and fuel.
Source: Ministry of Non-renewable Energy.

M09_MADAN 07_65901_C09.indd 40 23/12/22 8:01 PM


People, Development and Environment 9.41

N uclear Energy 2. Storage of nuclear wastes can lead to disastrous


Earlier, we discussed about the law of conservation of effects if not disposed or stored in a proper manner.
energy in which energy can be changed from one form 3. Though uranium stock seems to be abundant, it may
into another. The source of nuclear energy is the mass exhaust one day. That will make nuclear power plants
of the nucleus and energy generated during a nuclear obsolete.
reaction is due to conversion of mass into energy 4. Radioactive minerals are unevenly distributed around
(Einstein’s theory). The energy produced is given by e = the world and are found in limited quantities.
mc2, where m is the mass and c is the speed of light. This 5. Starting a nuclear plant requires huge capital invest-
equation was developed by Einstein. There are two ways ment and advanced technology.
to obtain nuclear energy: 6. There have been instances of proliferation of nuclear
technology.
1. Nuclear Fission Reaction: The nucleus of a heavy
radioactive element, such as uranium, plutonium or The three major nuclear accidents were as follows:
thorium, splits up into smaller nuclei when bombarded 1. Three Miles (1979, USA)
with low-energy neutrons. A huge amount of heat is 2. Chernobyl (1987, Ukraine)
generated in this process. At this point, carbon dioxide 3. Fukushima (2011, Japan–caused by a tsunami)
in gas form is pumped into the reactors with uranium,
removing heat from the system. The gas turns very hot Germany declared that it would close all its power plants.
and this heat is used to convert water into steam. The France produces maximum percentage of its electricity
steam created from this process drives the turbines, from nuclear power.
which in turn drive the generators that produce nuclear
energy. Nuclear Fuels–Indian Scenario
2. Nuclear Fusion Reaction: It involves the combina- There are 22 operational nuclear power reactors in India
tion or fusion of two light elements, such as hydro- with total installed capacity of 6780 MW as per informa-
gen, to form a heavier element, resulting in the tion available in October 2021.
release of uncontrollable energy inside it. Thus, it India has more of Thorium reserves rather than
cannot be used to generate electricity and cannot be Uranium reserves. So, such kind of technology itself is a
commercialized as is the case with fission reaction. challenge. Thorium-232 itself does not split and release
Sun’s energy is generated by nuclear fusion reaction. energy.
The heat and light that we receive from the sun is India’s three stage nuclear power generation had
all due to the continuous fusion reactions going on been generated to use India’s vast thorium-232 reserves.
inside it. Thorium, somehow, can not be used as a fuel in natural
The nuclear power reactor that creates all these reac- state, it is needed to be converted into ‘fissile’ state after
tions is controlled through rods of boron, known as con- some reactions.
trol rods. These boron rods absorb the neutrons. The rods 1. During first stage, Pressurised Heavy Water Reactors
are lowered into the reactor to absorb neutrons and slow will be used to produce energy from natural uranium,
down the process of fission. To generate more power, the and Plutonium (Pu) - 239 will also be produced.
rods are raised again to allow even more neutrons to crash 2. During second stage, indigenous Fast Breeder Reac-
into the atoms of uranium. tor Technology (FBRT) will be used to produce more
Advantages Plutonium.
3. During third stage, Plutonium obtained from second
1. Nuclear energy is more cost-effective when compared stage will be used to produce more energy with its
to coal. reaction with Thorium-232. During process, another
2. It does not use as much fuel in the process. fissile material U-233 will also be produced.
3. It produces less waste and does not produce c­ arbon
dioxide or smoke. Thus, nuclear energy does not con-
tribute to environmental hazards or greenhouse effect.
4. Nuclear power stations are usually very compact Stopover
when compared to the thermal stations. Although the 1. In which of the following fuel sources are, the tech-
initial capital cost of building a nuclear plant is high, nologies such as horizontal drilling and hydraulic
the maintenance and running costs are relatively low. fracturing used?
(a) Coal bed methane
Limitations (b) Shale gas
Creating nuclear energy is a complex chemical process (c) Biogas
that can be very dangerous. (d) All of the above
1. There is a great deal of radiation danger associated The correct option is (b).
with nuclear energy. It is capable of causing genetic 2. Which of the following is considered better for dump-
disorders. ing of nuclear waste?

M09_MADAN 07_65901_C09.indd 41 23/12/22 8:01 PM


9.42 Chapter 9

(a) Salt mines farming on narrow flat steps cut one after another across
(b) Landfills the slope or terrace is an age old practice. There are many
(c) Forests medicinal plants, which should better be raised in hilly
(d) Oceans areas.
The correct option is (a). This has become a very chal-
lenging task. Wildfires
3. Which state in India ranks first in renewable energy Not all deforestation is intentional. Some are caused by a
capacity? combination of human and natural factors, such as wild-
(a) Maharashtra fires and subsequent overgrazing, which may prevent the
(b) Gujarat growth of young trees. However, in the last decade, the
(c) Madhya Pradesh intensity and frequency of wildfires has increased due to
(d) Tamil Nadu global warming.
The correct option is (a).
Negative Effects of Deforestation on the
4. Which of the following nuclear accidents can be
termed the latest one in our history? E nvironment
(a) Fukushima Daiichi, Japan Globalization is viewed as another root cause of defor-
(b) Chernobyl, Ukraine estation. The degradation of the forest ­ecosystems has
(c) Three Mile Island, Middletown, Pennsylvania, USA been traced to economic incentives that make forest
(d) None of the above conversion appear more profitable than forest conser-
The correct option is (a). vation. The forest cover, which helps in absorbing GHG,
has shrunk from 4.7 billion ha in 1949 to 714.9 million
Hint: Happened in 2011, while (b) and (c) are the ha in 2015.
years 1986 and 1978.
Climate Change
Less trees absorb less carbon dioxide that results in higher
Our Forest Resources release of GHGs and thus global warming.
Deforestation is clearing the earth’s forests on a mas- Plants help in removing carbon dioxide from the
sive scale, resulting in damage to the quality of land. The atmosphere during the process of photosynthesis and
world’s rain forests will completely vanish in about hun- release oxygen back into the atmosphere during normal
dred years at the current rate of deforestation. respiration. Actively growing trees help better in this
situation. The decay and burning of wood release much
of this stored carbon back into atmosphere.
Expansion of Agriculture Forests are effective as carbon sinks or biodiversity
The major direct cause of deforestation is agriculture, reserves.
with subsistence farming responsible for 48% of defor-
estation, commercial agriculture for 32% and fuelwood Reduced Moisture Content in the Environment
for 5%.
Forest soils are moist, but without protection from sun-
blocking tree cover, they quickly dry out. Trees also help
Shifting Cultivation perpetuate the water cycle by returning water vapour
Shifting cultivation is practiced in Assam and Madagascar back into the atmosphere. Without trees to fill these
(Indian Ocean) for subsistence farming. roles, many former forest lands can quickly become bar-
Shifting cultivation has occurred due to poor fertility ren deserts.
of soil. In this cultivation, a small patch of tropical forests
is cleared, and vegetation is destroyed and burned. Crops Effects of Deforestation on Water Cycle
are grown as long as the soil is productive, after which Deforestation changes the quantity of water on surface, in
the cultivation is abandoned and cultivations move on to soil or in the atmosphere. This in turn changes the erosion
a fresh patch of land. rates and the availability of water, either for ecosystem
Timber Harvesting functions or for human services.
Logging (for the world’s wood and paper products) is Decrease in Water Precipitation
responsible for 14% of deforestation. Countries such as Trees extract groundwater through their roots and release
Myanmar, Malaysia, Indonesia, Brazil, Argentina and it into the atmosphere. When trees are removed, there will
many African countries are examples. not be any evaporation, resulting in a much drier climate.
Thus, trees help in perpetuating the water cycle by return-
Extension of Cultivation on Hill Slopes ing water vapour back into the atmosphere. Without
This is termed contour farming. Although agriculture has trees, it would result in reduced rainfall and many former
always been concentrated on planes and floors of valleys, forest lands would quickly become barren deserts. Forest

M09_MADAN 07_65901_C09.indd 42 23/12/22 8:01 PM


People, Development and Environment 9.43

cover brings down the temperature of the area, which is India has deposits of high-grade iron ore, that is, haem-
crucial for rainfall. atite and magnetite. It is the second largest producer of
iron ore after Brazil. The mineral is found mainly in
Soil Erosion Jharkhand, Odisha, Chhattisgarh, Madhya Pradesh, Goa,
Maharashtra and Karnataka. In Goa, there are open cast
Tree roots bind soil together, and if the soil is sufficiently iron ore mines, which are mechanized.
shallow, they act to keep the soil in place by binding with
the underlying bedrock. Tree removal on steep slopes Bauxite
with shallow soil increases the risk of landslides, which Bauxite is an aluminium ore. Aluminium is used in manu-
threatens the people living nearby. The quicker transport facturing of aeroplanes, electrical gadgets and so on. India
of surface water results in flash floods and more local- is the third largest producer of bauxite in the world. The
ized floods than those which would occur with the forest major bauxite-producing areas are Jharkhand, Odisha,
cover. Chhattisgarh, Madhya Pradesh, Gujarat, Maharashtra
Deforestation generally increases the rate of soil ero- and Tamil Nadu.
sion by increasing the amount of run-off and reducing
the protection of soil from tree litter. Yellow River is an Mica
example. Its yellow colour is caused by the downstream India is the largest producer and exporter of mica in the
carriage of loess and causes flooding in its lower reaches world. Mica deposits mainly occur in Bihar, Jharkhand,
(hence, the river’s nickname China’s sorrow). Andhra Pradesh and Rajasthan. It is used in manufactur-
ing electrical fittings.
Desertification: Desertification and deforestation are
linked closely. Desertification is a systemic phenomenon Copper
resulting from excessive felling of trees. It is the degra- It is a ductile metal with very high thermal and electri-
dation of land in any dryland. Dryland ecosystems are cal conductivity. Pure copper is soft and malleable.
fragile and can rarely sustain the increased pressures that It is mainly produced in Rajasthan, Madhya Pradesh,
result from intense population growth. Jharkhand, Karnataka and Andhra Pradesh.
The following factors also matter:
Manganese
1. There may be over-cultivation of desert lands. It
causes nutrients in the soil to be depleted faster. India is the third largest producer of manganese after
Improper irrigation practices result in salinated soils Russia and South Africa. India’s manganese deposits lie
and depletion of aquifers. in Maharashtra, Madhya Pradesh, Chhattisgarh, Odisha,
2. Vegetation plays a major role in determining the bio- Karnataka and Andhra Pradesh.
logical composition of soil. The soil erosion and water
Limestone
run-off decrease with increased vegetation cover.
3. Forests cause precipitation and maintain humidity of an Limestone with high silica content is used in the manufac-
area. Air humidity not only results in less penetration of ture of white cement. Major limestone-producing states
solar heat to ground during the day but also checks heat in India are Bihar, Jharkhand, Odisha, Madhya Pradesh,
escape to outer atmosphere during the night. Thus, for- Chhattisgarh, Rajasthan, Gujarat and Tamil Nadu.
ests tend to provide seasonal stability to the terrain.
4. As a result of deforestation, there is higher wind Gold
velocity, which increases the rate of soil erosion. Kolar in Karnataka has deposits of gold in India. These
mines are among the deepest in the world, which makes
N atural Resources—Our Mineral A ssets mining of this ore a very expensive process.
India is rich in metallic minerals of ferrous groups, such
as iron, manganese, chromite and titanium. However, Salt
petroleum and some non-ferrous minerals, such as cop- It is obtained from seas, lakes and rocks. India is one of
per, lead, zinc, tin and graphite, are not adequate. the world’s leading producers and exporters of salt.
Iron
Coal
Iron ores have been categorized into three grades,
Coal is used as a raw material in the chemical and fer-
namely (i) haematite, also known as red ore, which
tilizer industries. It is found in two regions, namely, in
contains up to 68% iron, (ii) magnetite (60%, known as
(i) Gondwana and (ii) extra-peninsular areas, such as
black ore) and (iii) lignite, containing between 35% and
Assam and other north-east states. There are high lignite
50% iron.
reserves in Tamil Nadu.
Pure iron is soft, but it is hardened due to metallurgi-
cal processes. A certain proportion of carbon is required
in ferrous to make it steel. Chromite is essential for the Petroleum
manufacture of stainless steel and high-temperature Petroleum exists in anticlines and fault traps. In India, it is
alloys. found in sedimentary rocks.

M09_MADAN 07_65901_C09.indd 43 23/12/22 8:01 PM


9.44 Chapter 9

Uranium and Thorium making it impossible to predict. Seismology is the study


Uranium is embedded in igneous and metamorphic rocks of earthquakes and seismic waves that move through and
in Bihar (Jaduguda in Singhbhum district of Bihar), around the earth.
Rajasthan and Andhra Pradesh.
The USA, Australia and India have particularly large Causes of Earthquake
reserves of thorium. In India, thorium is found in mona-
Earth’s crust is not a single piece but consists of portions
zite sands across the coasts of Kerala.
called plates, which vary in size from a few hundreds to
thousands of kilometres. According to the theory of plate
tectonics, when these plates contact each other, stress
Natural Hazards and Mitigation arises in the crust. The plates may pull away from each
other, push against each other or slide sideways. If plates
A hazard may be defined as a dangerous condition or get locked together, they are unable to move. It results in
event, which threatens or has the potential for causing stress in areas around the plate boundaries, called faults.
injury to life or damage to property or the environment. When this reaches a maximum point, the fault rupture
The main classification has been given here. Then few generates vibration called seismic waves, which radiates
natural hazards have been discussed. in all directions from the focus (Figures 9.17 and 9.18). To
put simply, the focus can also be defined as underground
origin of an earthquake.
National Hazard Main Events
Geophysical • Earthquake Seismic Waves
• Volcano
• Tsunami
Hydrological • Flood Seismic
• Landslides Waves
• Wave Action
Meteorological • C yclone, Storm Surge, Body Waves Surface Waves
– Hazard caused by Tornado, Convective Storm,
short-lived, micro- Extra-tropical Storm, Wind
to meso-scale • Cold Wave, Derecho
extreme weather • Extreme Temperature, Fog, Longitudinal Transverse Rayleigh Love
and atmospheric Frost, Freeze, Hail, Heat wave (P Waves) (S Waves) Waves Waves
conditions • Lightning, Heavy Rain
• Sand Storm, Dust Storm Figure 9.16 Earthquake: Causes
• Snow, Ice, Winter Storm,
Blizzard
Climatological •  rought
D These are waves of energy that travel through the earth’s
– Unusual, • Extreme hot/cold conditions layers and there are two main types of seismic waves
extreme weather • Forest/Wildfire Fires (Fig. 9.18):
conditions • Glacial Lake Outburst 1. Body waves
– Intra-seasonal • Subsidence 2. Surface waves
to multi-decadal
(long-term)
climate variability
Biological • E pidemics: Viral, bacterial,
Surface
– Exposure to parasitic, fungal or prion
waves
germs and toxic infections
substances • Insect infestations
• Animal stampedes Body
waves
The human society is also vulnerable to Chemical,
Biological, Radiological and Nuclear (CBRN) disasters.
Earth

Earthquake
It is the sudden shaking of the earth’s crust. The impact of
an earthquake is sudden and there is hardly any warning, Figure 9.17 Body Waves and Surface Waves

M09_MADAN 07_65901_C09.indd 44 23/12/22 8:01 PM


People, Development and Environment 9.45

Earth’s surface 2. Modified Mercalli Scale: It expresses the intensity


of an earthquake’s effect on people, structure and the
P-wave earth’s surface on a scale from I to XII.
Into earth
“front”
Earthquake S-wave
“front”
The Bureau of Indian Standards, based on the past
seismic history, grouped the country into four seis-
mic zones, viz, Zone II, III, IV and V. Of these, zone V
Figure 9.18  (Primary) Waves and S (Secondary)
P is the most seismically active region, while zone II is
Waves the least.
The Modified Mercalli (MM) intensity, which
Body Waves measures the impact of the earthquakes on the surface
These are called body waves as they pass through the of the earth, broadly associated with various zones, is
body of the earth, and they are further categorized as fol- as follows:
lows (Fig. 9.18): Seismic Zone Intensity MM scale
1. Primary or compressional waves, also popularly II (low-intensity zone) VI (or less)
known as P waves III (moderate-intensity zone) VII
2. Secondary or transverse waves, also popularly known IV (severe-intensity zone) VIII
as S waves V (very-severe–intensity zone) IX (and above)

Primary waves travel faster than secondary waves. Thus, Zone V comprises the entire north-eastern India,
they are the first to reach the seismograph stations. parts of Jammu and Kashmir, Himachal Pradesh,
Uttaranchal, Rann of Kutch in Gujarat, parts of North
Surface Waves Bihar, and Andaman and Nicobar Islands.
They travel along the earth’s crust and vibrate the ground Zone IV covers the remaining parts of Jammu and
horizontally and vertically. They are more dangerous Kashmir and Himachal Pradesh, union territory of
than body waves and can destroy buildings and highways Delhi, Sikkim, northern parts of Uttar Pradesh, Bihar
which come in their path. Their amplitude decreases with and West Bengal, parts of Gujarat and small portions
increasing depth into the earth. These are further catego- of Maharashtra near the west coast, and Rajasthan.
rized as (i) Love waves and (ii) Rayleigh waves. Zone III comprises Kerala, Goa, Lakshadweep
Love waves move the ground from side to side. islands, remaining parts of Uttar Pradesh, Gujarat
Confined to the surface of the crust, Love waves produce and West Bengal, and parts of Punjab, Rajasthan,
an entirely horizontal motion. Madhya Pradesh, Bihar, Jharkhand, Chhattisgarh,
Rayleigh moves the ground up and down and side-to- Maharashtra, Orissa, Andhra Pradesh, Tamil Nadu
side. Love waves are faster than Rayleigh waves. and Karnataka.
Earthquakes can be shallow (less than 60 km), medium Zone II covers the remaining parts of the country.
(60–300 km) and deep (300–600 km). Shallow-focus
earthquakes are the most damaging because of their prox-
imity to the surface. Mitigation of Earthquakes
Planning
Measurement of Intensity of Earthquakes
The Bureau of Indian Standards has published certain
There are mainly two techniques available to measure
building codes and guidelines for safe construction of
the intensity of earthquakes, and they are as follows:
buildings against earthquakes. Before the buildings are
1. Richter Scale: An earthquake’s magnitude or constructed, the ­building plans have to be checked by the
amount of energy released is determined by the use ­municipality, according to the rules laid down by the law.
of a seismograph, which is an instrument that con-
tinuously records ground vibrations. An earthquake Public Education
with a magnitude 7.5 on the Richter scale releases 30 Educating the public on causes and characteristics of an
times more energy than the one with 6.5 magnitude earthquake and preparedness measures helps to a certain
and will also have a shaking amplitude of 10 times extent in mitigating the natural disaster.
higher. Similarly, an earthquake that measures 5.0
on the Richter scale has a shaking amplitude 10 times Engineered Structures
larger than one that measures 4.0 and corresponds Buildings need to be designed and constructed as per
to a 30 times larger release of energy. An earthquake the laws to withstand ground shaking. Architectural and
of magnitude 3 is the smallest that is normally felt by engineering inputs need to be put together to improve
humans. building design and construction practices. The soil type
needs to be analysed before construction.

M09_MADAN 07_65901_C09.indd 45 23/12/22 8:01 PM


9.46 Chapter 9

Stopover Crater Main vent


Earthquakes are produced during

(a) Plastic failure within the mantle Secondary
(b) Brittle failure during faulting vent
(c) Mushrooming during folding
(d) None of the above
The correct option is (b).

Tsunami Magma chamber


The term tsunami has been derived from a Japanese term Figure 9.20 Structure of a Volcano
Tsu meaning harbour and nami meaning waves.
A tsunami is a series of ocean waves with very long
wavelengths (typically hundreds of kilometres) caused by
large-scale disturbances of the ocean, such as the follow- nuclear power plant reactors and that resulted in mas-
ing (Fig. 9.19): sive radioactive pollution. The tsunami that originated
1. Earthquakes in the Indian Ocean caused massive damage in Indonesia
2. Landslides and India.
3. Volcanic eruptions
4. Explosions Volcanoes
5. Meteorites A volcano is an opening or rupture in the earth’s surface or
crust (Fig. 9.20). Pyroclastic flows happen basically when
These disturbances can either be from below (e.g. extremely hot gases, ash, sulphur, steam and other mate-
underwater earthquakes with large vertical displace- rials of more than 1000°C rapidly flow down the flank of a
ments and submarine landslides) or from above (e.g. volcano (more than 700 km/h) during an eruption.
meteorite impacts). Volcanoes are generally found where tectonic plates
Tsunamis can have wavelengths ranging from 10 to diverge or converge. Subduction zones are places where
500 km and wave periods of up to an hour. As a result of two plates, usually an oceanic plate and a continental plate,
their long wavelengths, tsunamis act as shallow-water collide; this is the case with earthquakes as well. Volcanoes
waves. A wave becomes a shallow-water wave when the tend to exist along the edges of tectonic plates, which are
wavelength is very large compared to the water depth. massive rock slabs that make up the earth’s surface. About
Tsunamis may reach a maximum vertical height onshore 90% of all volcanoes exist within the ‘Ring of Fire’ along the
above sea level, often called a run-up height, of tens of edges of the Pacific Ocean.
metres. Eruption of volcanoes away from the subduction zone
Tsunamis can result in massive destruction when they is referred to as mantle plumes, also known as ‘hotspots’.
arrive onshore, and they can cause severe coastal erosion, These are columns taking out magma from the deep inte-
which is the wearing away of coastal land or beaches. The rior of the earth.
power of water can wash away vegetation, making it hard Erupting volcanoes can pose many hazards. Volcanic
to establish the shoreline. As tsunamis are triggered by ash can be a threat to aircraft, especially jet aircrafts.
sudden events, there may be little time to warn coastal Large eruptions can affect temperature as ash and
residents of their arrival. droplets of sulphuric acid obscure the sun and cool the
This fact became evident in March 2011 when Japan earth’s lower atmosphere or troposphere.
was struck by a large tsunami that was triggered by a
9.0 magnitude earthquake, which damaged several Cyclone
Cyclone is a region of low atmospheric pressure sur-
rounded by high atmospheric pressure, resulting in swirl-
2. Large waves ing atmospheric disturbances, accompanied by powerful
form and winds blowing in an anticlockwise direction in the north-
move at up to 800 ern hemisphere and in a clockwise direction in the south-
km/h (500 mph) ern hemisphere. It occurs mainly in the tropical and tem-
perate regions of the world. Cyclones are called by various
names in different parts of the world. Cyclones in India
are moderate in nature. Some of the general features of a
3. Waves slow in shallower
coastal area, but increase in height typical cyclone are as follows:
1. Earthquake changes shape of 1. Strong winds
sea floor causing displacement of water
2. Exceptional rain
Figure 9.19 Tsunami: Causes 3. Storm surge

M09_MADAN 07_65901_C09.indd 46 23/12/22 8:01 PM


People, Development and Environment 9.47

Cyclones are called by different names: usually submerged. There are different types of floods,
namely, flash flood, riverine flood and urban flood.
1. Typhoons and Hurricanes: Caribbean and Gulf of
Mexico Flash Floods
2. Tropical Cyclones: India These are rapid inland floods due to intense rainfall. A flash
3. Willie: Australia flood describes sudden flooding within a short duration. In
4. Tornadoes: South America sloped terrains, the water flows rapidly with a high destruc-
Damage by Cyclones tion potential. In flat terrains, the rainwater cannot infiltrate
into the ground or run off (due to small slope) as quickly as
India faces more cyclones on its eastern coast (Bay of it falls. Flash floods are typically associated with thunder-
Bengal) than on western coast due to direction of wind, storms. A flash flood can occur at virtually any place.
more rains and water. The state of Odisha suffers the most
due to cyclones. There are five categories of cyclones. Causes of Flash Floods
Under Category 5, cyclone speed can go up to 252 km/h. 1. Heavy rainfall in a short span of time
They cause extensive damage to infrastructure and 2. Heavy siltation of the river bed, which reduces the
buildings. There will be heavy rains and flooding. There water-carrying capacity of the rivers or stream
can be outbreak of malaria, diarrhoea, etc. 3. Blockage in the drains, which leads to flooding of
Mitigation of Cyclones the area
4. Landslides blocking the flow of the stream
Green belt plantation along the coastal line in a scientific, 5. Construction of dams and reservoirs
interweaving pattern can reduce the effects of a hazard. 6. In areas prone to cyclone, strong winds accompanied
Providing a cover through green belt sustains less dam- by heavy downpour along with storm surge
age. Forests act as a wide buffer zone against strong
winds and flash floods. The roots of the plants and trees The understanding of the concept of catabatic (also
keep the soil intact and prevent erosion and slow run-off termed katabatic) and anabatic winds can help ­further
to prevent or lessen flooding. to understand the phenomenon of flash floods. The term
catabatic wind is used for downslope winds flowing from
high elevations of mountains, plateaus and hills, down
their slopes to the valleys or planes below. This happens
during the night time. The concept of anabatic winds is
Cyclones in India
just opposite to that of catabatic winds; they flow up the
The WMO maintains rotating lists of names which hill, specifically during the daytime. The catabatic winds
are appropriate for each tropical cyclone basin. If a cause the occurrence of flash floods.
cyclone is particularly deadly or costly, then its name
is retired and replaced by another one. Cloudbursts
Major Cyclones A cloudburst is an extreme amount of precipitation
sometimes with hail and thunder, which normally lasts
• Sitrang: 2022 • Gaja: 2018 no longer than a few minutes but is capable of creating
• Amphan: 2020 • Ockhi: 2017 flood conditions. Colloquially, the term cloudburst may
(super cyclone) • Vardah: 2016 be used to describe any sudden heavy, brief and usually
• Nisarga: 2020 • Hudhud: 2014 unforecasted rainfall.
• Gati: 2020 • Phailin: 2013 Heavy rainfall, flash floods, landslides and cloudbursts
• Nivar: 2020 • Helen: 2013 in June 2013 caused havoc in Uttarakhand, causing death
• Burevi: 2020 • Nilam: 2012 of thousands of people.
• Fani: 2019 • Phyan: 2009
The depressions ARAB-01,BOB-02,ARB-03, BOB-03 Drought
arose in Arabian sea in 2020. Drought is either the absence of rainfall or its deficiency
from its normal pattern in a region for an extended period
Odisha Cyclone in 1999 had been the strongest one. of time, leading to general suffering in the society. It is an
Vayu, Hikaa, Kyarr, Maha, Bulbul and Pawan have interplay between the demand that people place on natu-
been other cyclones in the sequence in year 2019. The ral supply of water and the natural event that provides
four of them—Vayu, Hikka, Kyarr and Maha—originated water in a given geographical region. The state of Kerala,
in the Arabian Sea that is the maximum number over which receives more than 3000 mm of rainfall every year,
many decades. was declared drought affected in the past as it was insuffi-
cient to have two good crops. The more the imbalance in
supply, the higher is the drought. The following explains
Flood this general definition further:
Flood is a state of high water level along a river channel 1. It is the slow onset of disaster and it is difficult to
or on the coast that leads to inundation of land that is not demarcate the time of its onset and the end.

M09_MADAN 07_65901_C09.indd 47 23/12/22 8:01 PM


9.48 Chapter 9

2. It is any unusual dry period that results in a shortage 3. Intense rainfall: Intense rainfalls, heavy melting of
of useful water. snow in the hilly terrains
Although drought is basically caused by rainfall deficit, 4. Human excavation: Mining, deforestation and
which is a meteorological phenomenon, it manifests into dif- irrigation
ferent spheres because of various vulnerability factors asso- 5. Earthquake
ciated with it. Some of these factors are induced by humans. 6. Volcanic eruption
Although drought is a natural disaster, its effects are
made worst in developing countries due to overpopulation,
nationaL diSaSter management
overgrazing, deforestation, soil erosion, excessive use of
ground water and surface water for growing crops and loss
of biodiversity. The four types of droughts are as follows: Pre- Disaster
1. Meteorological Drought: It is simply the absence or Disaster
deficit of rainfall. It is the least severe form of drought
and is identified by sunny days and hot weather. Response
Preparedness
2. Hydrological Drought: It leads to reduction of nat-
ural stream flows or groundwater levels plus stored
water supplies. The main impact is on the water Post-
resource systems. Mitigation: Disaster
3. Agricultural Drought: This form of drought occurs Prevention
when moisture level in the soil is insufficient to & Risk
maintain average crop yields. The initial conse- Reduction Recovery
quences are reduced seasonal output of crops and
other related production. An extreme agricultural Figure 9.21 National Disaster Management
drought can lead to a famine, which is prolonged
shortage of food in a restricted region, causing wide- National Disaster Management Act, 2005
spread disease and death from starvation.
The overall coordination of disaster management vests with
4. Socio­economic Drought: It correlates the supply
the Ministry of Home Affairs (MHA). The Cabinet Committee
and demand of goods and services with the three
on Security (CCS—with PM as its member) and the National
above-mentioned types of drought.
Crisis Management Committee (NCMC) are the key commit-
W iLdFire tees involved in the top-level decision-making with regard to
disaster management. The NDMA is the lead agency respon-
A wildfire is any uncontrolled fire in combustible vegeta- sible for the preparation of DM plans and the execution of
tion that occurs in the countryside or a wilderness area. It DM functions at the national level (Fig. 9.22).
differs from other fires by its extensive size, the speed at The High Power Committee on Disaster Management
which it can spread out from its original source, its poten- in 2001 suggested the following levels for disaster
tial to change directions unexpectedly and its ability to management:
jump gaps, such as roads, rivers and fire breaks.
1. Level L0: The period of normalcy, it should be uti-
LandSLideS lized for disaster risk reduction.
The term landslide includes all varieties of mass movement of 2. Level L1: The level of disaster that can be managed
hill slopes and can be defined as the downward and outward within the capabilities and resources at the district
movement of slope-forming materials composed of rocks, level. However, the state authorities will remain in
soils, artificial fills or a combination of all these materials readiness to provide assistance if needed.
along surfaces of separation by falling, sliding and flowing, 3. Level L2: This signifies the disaster situations
either slowly or quickly from one place to another. Although that require assistance and active mobilization of
landslides are primarily associated with mountainous ter- resources at the state level and deployment of state
rains, these can also occur in areas where activities such -level agencies for disaster management. The central
as surface excavation for highways, buildings and open-pit agencies must remain vigilant for immediate deploy-
mines take place. Landslides take place in conjunction with ment if required by the state.
earthquakes, floods and volcanoes. At times, prolonged rain- 4. Level L3: This corresponds to a nearly catastrophic
fall causing landslides may block the flow of a river for quite situation or a very large-scale disaster that over-
some time. The formation of river blocks can cause havoc to whelms the state and district authorities.
the settlements downstream on their bursting.
Mitigation and adaptation
Causes of Landslides • Mitigation: addresses the root cause of climate
1. Geologically Weak Material: Weakness in the com- change (emissions)
position and structure of rock or soil • Adaptation: addresses the impacts of climate
2. Erosion: Erosion of slope top due to cutting down of change
vegetation and construction of roads

M09_MADAN 07_65901_C09.indd 48 23/12/22 8:01 PM


People, Development and Environment 9.49

Overall Coordination:
Ministry of Home Affairs
Designated
Nodal Ministries
(Disaster-
Top-Level Decision-Making specific) National Disaster
Management
Authority
National (NDMA)
Cabinet
Crisis
Committee
Management
on
Committee
Security
(NCMC) National
(CCS)
Executive
Committee
(NEC)

Armed Forces National National


and Institute of Disaster
State Central Armed Disaster Response
Governments/ Police Forces Management Force
Union (CAPF) (NIDM) (NDRF)
Territories
Figure 9.22 National Disaster Management Institutional Mechanism

significantly decrease the losses of life, livelihoods,


Greenhouse gas concentrations and assets—economic, physical, social, cultural and
environmental—by maximizing the ability to cope
with disasters at all levels of administration as well as
Long-term among all communities’.
Effect • As mentioned in Sendai Framework, it has five the-
Climate change
matic areas for actions:
– Understanding risk
– Inter-agency coordination
– Investing in DRR—structural measures
Impacts – Investing in DRR—non-structural measures
Short- – Capacity development
term
Responses Effect Environment Protection Act (1986)
India is one of the few countries in the world where
‘environment’ finds an explicit mention in the constitu-
Mitigation Adaptation tion itself. Though it is not guaranteed as the status of a
fundamental right, it has been recognized as an integral
component of the ‘right to life’ by the higher judiciary in
The NDMP was enacted in the year 2016. It has been judicial decisions.
aligned broadly with the goals and priorities set out in
1. Factories Act, 1948: This Act aims at providing infor-
the Sendai Framework for Disaster Risk Reduction. The
mation on hazardous processes taking place inside
NDMP is a dynamic document in the sense that it will
the factory to its workers, local residents and govern-
be periodically improved keeping up with the emerging
ment officials.
global best practices and knowledge bases in disaster
2. Insecticides Act, 1968: It aims to regulate import,
management.
manufacture, sale, transport, distribution and the
• The vision of the plan is to ‘make India disaster resil- use of insecticides to prevent risk to human and
ient, achieve substantial disaster risk reduction, and animal life.

M09_MADAN 07_65901_C09.indd 49 23/12/22 8:01 PM


9.50 Chapter 9

3. Water (Prevention and Control of Pollution) Act, • The main emphasis is given to ‘environment’, defined
1974: This act defines what water pollution is and to include water, air and land, and the inter-rela-
determines its penalties. tionships which exist among water, air and land and
4. Air (Prosecution and Control of Pollution) Act, human beings and other living creatures, plants,
1981: This act controls and regulates emissions from microorganisms and property.
automobiles and industrial plants. • It defines environmental pollution as ‘hazardous
5. The Forest Conservation Act, 1980: It mainly pro- substances’ including any substance or preparation,
hibits the state governments from declaring any which may cause harm to human beings, other living
reserve forest as non-reserve without approval of the creatures, plants, microorganisms, property or the
central authority. environment.
6. The Wildlife Protection Act, 1972: It aims to provide
The main provisions of this act are given as follows:
necessary protection against serious threat to wildlife
(both animals and birds) by the expansion and advance- 1. The Act empowers the centre to ‘take all such mea-
ment of agriculture, industry and urbanization. sures as it deems necessary’.
2. By virtue of this Act, the central government has armed
The Indian Board of Wildlife was set up in 1952.
itself with considerable powers such as coordination of
Various projects have been launched for the protection of
action by state, planning and execution of nationwide
endangered species, such as lions (1972), tigers (1973),
programmes, laying down environmental quality
crocodiles (1974) and brown-antlered deer (1981). India
standards, restrictions on the location of India’s and
became a party to the Convention of International Trade in
authority to issue direct orders.
Endangered Species of Fauna and Flora (CITES) in 1976.
3. The Act explicitly prohibits discharges of environ-
India also started a national component of the United
mental pollutants in excess of prescribed regulatory
Nations Educational, Scientific and Cultural Organization’s
standards.
(UNESCO) Man and Biosphere (MAB) programme in 1971.
4. There is also a specific prohibition against handling
The Central Pollution Control Board was set up in 1974
hazardous substances except those in compliance
as an implementing agency of the Water Act. Later, it also
with regulatory procedures and standards.
took on the implementation of the Air Act of 1981. The
5. The Act provides provision for penalties. For each fail-
CPCB is a statutory body attached to the MoEF.
ure or contravention, the punishment includes a prison
In 1980, a separate Department of Environment was
term for up to 5 years or fine up to `1 lakh, or both.
constituted with a mandate to plan, promote and coor-
6. The Act imposed an additional fine of up to `5000 for
dinate programmes relating to the environment. A full-
every day of continuing violation.
fledged MoEF was established in 1985 to oversee the
7. If a failure or contravention occurs for more than
environmental protection measures at the national level.
1 year, the offender may be punished with imprison-
The National Environment Appellate Authority (NEAA)
ment, which may be extended to 7 years.
was set up in 1997.
8. Section 19 provides that any person, in addition to
A new institutional set-up is envisaged for conserva-
authorized government officials, may file a complaint
tion issues under the Biodiversity Act of 2002, compris-
with a court alleging an offence under the Act.
ing a National Biodiversity Authority, State Biodiversity
9. This ‘Citizens’ Suit’ provision requires that the per-
Boards and Biodiversity Management Committees.
son has to give a notice of not less than 60 days
• The genesis of the Environment (Protection) Act, of the alleged offence of pollution to the central
1986, is in Article 48A (Directive Principle of State ­government.
Policy) and Article 51A(g) (Fundamental Duty) of the
The Genetic Engineering Appraisal Committee (GEAC)
Indian Constitution.
is the apex body constituted in the MoEF under ‘Rules
• There is a directive given to the state as one of the
for Manufacture, Use, Import, Export and Storage of
Directive Principles of State Policy regarding the
Hazardous Microorganisms/Genetically Engineered
protection and improvement of the environment.
Organisms or Cells, 1989’, under the EPA 1986. The dif-
Article 48A states ‘The State shall endeavour to protect and ferent genetic engineering committees are also set up
improve the environment and to safeguard the f­orests and under the Act. Presently they are six in number.
wildlife of the country’.
• The Environment Protection Act of 1986 (EPA) Stopover
came into force soon after the Bhopal Gas Tragedy 1. The Genetic Engineering Appraisal Committee is con-
and is considered an umbrella legislation as it filled stituted under the
many lacunae in the existing legislations. (a) Food Safety and Standards Act, 2006
• The EPA is also an umbrella act because it provides (b) Geographical Indications of Goods (Registration
the framework to the central government to make and Protection) Act, 1999
the coordination between different states as well (c) Environment (Protection) Act, 1986
as the central authorities using different acts, such as (d) Wildlife (Protection) Act, 1972
Water Act. The correct option is (c).

M09_MADAN 07_65901_C09.indd 50 23/12/22 8:01 PM


People, Development and Environment 9.51

2. Which of the following gases are considered for the NMEEE has four initiatives:
calculation of ‘Air Quality Index’?
• Perform, Achieve and Trade (PAT)
1. Carbon dioxide
• Market Transformation for Energy Efficiency (MTEE)
2. Carbon monoxide
• Energy Efficiency Financing Platform (EEFP)
3. Nitrogen dioxide
• Framework for Energy Efficient Economic Develop-
4. Sulphur dioxide
ment (FEEED)
5. Methane
Codes:
(a) 1, 2 and 3 National Mission on Sustainable Habitat
(b) 2, 3 and 4 (NMSH)
(c) 1, 4 and 5
(d) 1, 2, 3, 4 and 5 This was approved in June 2010. The key deliverables of
the Mission include:
The correct option is (b).
• The development of standards for robust development
strategies and addressing climate change related con-
India’s National Action Plan On cerns
• The preparation of city development plans that
Climate Change (NAPCC) address adaptation and mitigation concerns
• The development of plans which take care of long-
National Action Plan on Climate Change was launched in
term, energy efficient and cost effective transport
2008 by the Prime Minister’s Council on Climate Change.
planning
The following are the main principles of the NAPCC:
• Capacity building for undertaking activities relevant
1. The use of technologies for inclusive and sustainable to the Mission
development.
2. Inclusion of Local governments, public–private part- National Water Mission (NWM)
nerships and civil society action.
3. Better understanding of climate change, adapta- This mission helps to conserve water, minimize wastage
tion and mitigation, energy efficiency and natural and ensure more equitable distribution both across and
resource conservation. within states. We need to have assessment of the water
4. Linked with national growth. related issues. The Mission wants to increase the water
5. Devising efficient and cost-effective strategies for use efficiency by 20 per cent through regulatory mecha-
demand-side management. nisms with differential entitlements and pricing. The
6. Welcoming international cooperation approach. water needs of urban areas are to be taken care of through
recycling of waste water. New technologies should be
The eight missions have been discussed first. Thereafter adopted such as for low temperature desalination tech-
National Mission on Seabuckthorn, National Mission nologies that allow for the use of ocean water.
on Bio-energy, National Mission on Hydrogen, National
Mission on Ammonia have been discussed. National Mission for sustaining the
N ational Solar Mission (NSM) Himalayan Eco -system (NMSHE)
This was started in January 2010, with the objective This particular mission sets the goal to prevent melting
of establishing India as a global leader in solar energy, of the Himalayan glaciers and to protect biodiversity in
by creating the policy conditions for solar technology the Himalayan region. There should be Wildlife conser-
diffusion across the country as quickly as possible. vation and protection. We need four capacities - Human
The initial target of NSM was to install 20 GW solar power and knowledge capacities, Institutional capacities, Policy
by 2022, that was later upscaled to 100 GW in early 2015. based governance, and self-learning.
The Mission adopts a three-phase approach, Phase 1
(up to 2012–13), Phase 2 (2013–17) and Phase 3 (2017–
National Mission for a Green I ndia
22). India wants to set up an enabling environment for This mission was made a Centrally Sponsored Scheme.
solar technology penetration in the country both at a cen- Greening is meant to enhance ecosystem services such as
tralized and decentralized level. carbon sequestration (carbon sinks) and storage (in forests
and other ecosystems), hydrological services and biodiver-
N ational Mission for Enhanced E nergy sity; as well as other provisioning services such as fuel, fod-
der, small timber and non- timber forest products (NTFPs).
E fficiency (NMEEE)
This mission was started in 2011. National Mission for Sustainable
This aims to strengthen the market for energy effi-
ciency by creating useful regulatory and policy regime Agriculture (NMSA)
and has envisaged fostering innovative and sustainable NMSA was implemented in the year 2014-15. This
business models to the energy efficiency sector. intends to make agriculture more productive, sustainable,

M09_MADAN 07_65901_C09.indd 51 23/12/22 8:01 PM


9.52 Chapter 9

remunerative and climate resilient. Government wants to and agro-industrial residue can potentially generate
promote location specific integrated /composite farming 25,000 MW of power in India.
systems; soil and moisture conservation measures; com-
prehensive soil health management; efficient water man- National Hydrogen Mission and Ammonia
agement practices and mainstreaming rain-fed technolo-
gies. There are practices such as Mission
This mission was started in August, 2021. The main objec-
1. Farm Water Management (FWM) tive is to make India self-reliant in energy production by
2. Per Drop More Crop (PDMC) that is component of the year 2047 (when India completes 100 years of inde-
Pradhan Mantri Krishi Sinchayee Yojana (PMKSY) pendence).
3. Soil Health Management (SHM) that aims at promot- Green hydrogen can be made by splitting water mol-
ing Integrated Nutrient Management (INM) through ecules into oxygen and water. India has set a target of 5
judicious use of chemical fertilizers including second- million tonnes of green hydrogen by 2030. Hydrogen and
ary and micro nutrients, along with organic manures Ammonia are envisaged to be the future fuels of India to
and bio-fertilizers. replace fossil fuels.
Pure hydrogen and hydrogen mixed with natural gas
N ational Mission on Strategic Knowledge (HYTHANE) have been used effectively to power automo-
biles. Hydrogen can be obtained from water, natural gas,
for C limate C hange (NMSKCC)
coal, sewage, coal gas etc.
This mission seeks to build a vibrant and dynamic Hydrogen has high energy content. This can be used
knowledge system that would inform and support either directly in IC engines or through fuel cells for
national action for responding effectively to the objective production of motive power and electricity. There is no
of ecologically sustainable development. pollutant gas emission such as CO2 in Hydrogen powered
The additional missions have been discussed below. vehicles, this will reduce global warming trends.
The one limitation of hydrogen is that it has negative
N ational Mission on Sea Buckthorn net energy. It means it takes more energy to produce than
The sea buckthorn plant is popularly known as Leh it contains.
Berries, as well as ‘Wonder Plant’ and ‘Ladakh Gold’. It has Green Ammonia is a chemical that is used mainly
the following properties: in the manufacture of nitrogenous fertilizers, ‘carbon-
neutral’ fertilizer products, thus food value chain is
1. It has numerous medicinal and nutritional properties, decarbonised. This has the potential as a future climate-
and it also helps in soil conservation and nitrogen neutral shipping fuel. This can be put to other uses too,
­fixation. such as to run engines.
2. It is hardy, drought resistant and tolerant to extreme The green ammonia can be made by using hydrogen
temperatures, from –43ºC to +40ºC. from water electrolysis and nitrogen separated from the
3. The plant has an extensive root system, which can fix air. Then they are fed into the Haber process.
atmospheric nitrogen, making it ideal for controlling Green ammonia production helps in making renew-
soil erosion and preventing desertification. able energy sources such as hydro-electric, solar power or
The MoEF and DRDO have launched a major national wind turbines.
initiative for sea buckthorn cultivation in the high-­
altitude, cold desert ecosystems. The initiative is one
of many conservation measures for fragile high-altitude Indian Network on Climate
­ecosystems. Change Assessment and NATCOM
This initiative is a part of Sub-Mission on Cold Desert
Ecosystems under the Green India Mission, which is a It was launched by the Ministry of Environment and
part of the NAPCC. Forest in an effort to promote domestic research on cli-
mate change. The reports are prepared by the Indian
N ational Bio-Energy Mission Network for Climate Change Assessment (INCCA). India’s
The union government is in the process of preparing a NATCOM has been initiated in 2002 that is funded by the
National Bio-Energy Mission to boost power generation Global Environment Facility (GEF).
from biomass, a renewable energy source abundantly
available in India.
This national mission will aim at improving energy
Montreal Protocol
efficiency in traditional biomass-consuming industries This protocol was finalised in 1987. This agreement
that seek to develop a bio-energy city project and provide protects the stratospheric ozone layer by phasing out
logistics support to biomass processing units. the production and consumption of ozone depleting
It also proposes a GIS-based National Biomass substances (ODS). This protocol called for an action to
Resource Atlas to map potential biomass regions in the phase out the production and consumption of ODS, such
country. According to the estimates, biomass from agro as CFCs and halons.

M09_MADAN 07_65901_C09.indd 52 23/12/22 8:01 PM


People, Development and Environment 9.53

The stratospheric ozone layer filters out harmful UV Conferences of the Parties
radiation. This radiation can increase prevalence of skin Since the UNFCCC entered into force, parties have been
cancer and cataracts; reduced agricultural productivity meeting annually in COPs to assess the progress in dealing
and also cause the disruption of marine ecosystems. with climatic changes. The meeting is held every year from
Kigali Agreement: Kigali is the capital of Rwanda. In the last week of November to the first week of December.
2016, there was an amendment to Montreal Protocol.
The Kigali Amendment aims for the phase-down of hydro- Convention on Biological Diversity
fluorocarbons (HFCs) by cutting their production and Biodiversity is the variety of plant and animal life that
consumption. Given their zero impact on the depletion of exists in a particular habitat or world, that includes
the ozone layer, HFCs are currently used as replacements animals, plants, fungi, and even microorganisms like
of hydrochlorofluorocarbons (HCFCs) and chlorofluoro- bacteria that make up our natural world. Each of these
carbons (CFCs), however they are powerful greenhouse species and organisms work together in ecosystems, like
gases. This their phaseout can result in reducing global an intricate web, to maintain balance and support life.
warming. The goal is to achieve over 80% reduction in UN Environment Protection conducted a meeting of Ad
HFC consumption by 2047. The impact of the amendment Hoc Meeting of Experts on biodiversity in 1988. Further
will avoid up to 0.5 °C increase in global temperature by changes were made during Nairobi Conference in 1992
the end of the century. which was a part of sustainable development.
The overall objective of CBD is to ensure sustainable
Rio Summit future, thus it has great concern for human beings. CBD
covers biodiversity at all levels: ecosystems, species and
Climate change is real. The UNCED started a conference genetic resources.
Earth Summit that was held at Rio de Janeiro, Brazil, CBD was opened for signature in 1992 at the Earth
in the year 1992. The main objective was to reconcile Summit in Rio de Janeiro, and entering into force in
worldwide economic development with protection of December 1993.
the environment. This Rio Summit was the largest gath- The convention establishes three main goals:
ering of world leaders. There were many treaties and
other documents signed at the c­ onference: 1. Conservation of biological diversity
2. Sustainable use of its components
1. The Convention on Biological Diversity (CBD) is a 3. Fair and equitable sharing of the benefits from the use
binding treaty that requires nations to take invento- of genetic resources
ries of their plants and wild animals and protect their
endangered species. The first CBD was conducted in 1994 in Bahamas.
2. The UNFCCC or Global Warming Convention is a
binding treaty that requires nations to reduce their Biological diversity is often understood in terms of the
emission of carbon dioxide, methane and other wide variety of plants, animals and microorganisms. So
GHG, though the treaty had not set any binding tar- far, about 1.75 million species have been identified, which
gets for emission reductions. Such targets were set in are mostly small creatures such as insects. Scientists
Kyoto Protocol (1997). The Paris Agreement on cli- reckon that there are actually about 13 million species,
mate change issues was signed in 2015 (Fig. 9.23). though estimates range from 3 million to 100 million.
3. The Declaration on Environment and Development, Biodiversity also includes genetic differences within
or Rio Declaration, laid down 27 broad, non- each species. It includes chromosomes, genes and DNA.
binding principles for environmentally sound devel- 1. Cartagena Protocol on Biosafety to the Convention
opment. Agenda 21 outlined the global strategies for on Biological Diversity is an international treaty
cleaning up the environment and encouraging envi- governing the movements of living modified organ-
ronmentally sound development. isms (LMOs) resulting from modern biotechnology
4. There was non-binding Statement of Principles on from one country to another. It was adopted on 29
Forests for tropical region. January 2000 as a supplementary agreement to the
There were some disputes during the Earth Summit CBD and entered into force on 11 September 2003.
between the wealthy industrialized nations of the North 2. Nagoya Protocol: The protocol was adopted in 2010
and the poorer developing countries of the South. in Nagoya, Japan. This protocol aims at sharing the
In general, the countries of the South were reluc- benefits arising from the utilization of genetic resources
tant to hamper their economic growth with the envi- in a fair and equitable way, including appropriate
ronmental restrictions urged upon them by the North access to genetic resources and appropriate transfer of
unless they received increased Northern financial aid, relevant technologies.
which they claimed would help make environmentally The Aichi Biodiversity Targets Task Force (ABTTF)
sound growth possible. Every year a meeting is held in was established to provide a platform for agencies and
the form of Conference of the Parties (COP). organizations to coordinate their activities in support
Benchmarking is the setting up of emission reduction of implementation of the Strategic Plan for Biodiversity
commitment as measured against a particular base year. 2011–20.

M09_MADAN 07_65901_C09.indd 53 23/12/22 8:01 PM


9.54 Chapter 9

Zero Extinction, Birdlife, CBD, Convention on Migratory Non-annexure and observer nations.
Species, CITES, Food and Agriculture Organization, Global We need to keep in mind the following three
Environment Facility, The Nature Conservancy, Ramsar mechanisms:
Convention, International Union for Conservation of Nature
1. Clean Development Mechanism (CDM)
and Natural Resources (IUCN) and UNEP are the different
2. Emission Trading (cap and trade) (emissions trading
agencies working with the ABTTF.
allows nations to sell unused emission units to coun-
CBD is now held once in 2 years. The 2018 UN
tries that have exceeded their targets).
Biodiversity Conference was held at Sharm El-Sheikh,
3. Joint Implementation (JI)
Egypt. There was a meeting of CBD in Hyderabad in 2012.
There was face-to-face meeting for COP 15 in Kunming, In market mechanism, carbon trading and carbon credits.
China in April-May 2022. Under the Kyoto Protocol, there are two commitment
periods:
Kyoto Protocol 1. 2008–12
The Kyoto Protocol was concluded in 1997 during the third 2. 2013–20
COP, Japan. The main goal was to fight global warming
by reducing GHG concentrations in the atmosphere to ‘a
level that would prevent dangerous anthropogenic inter- Some important COPs
ference with the climate system’. It established the legally COP 1 - Berlin (Germany) in 1995: First meeting commit-
binding obligations for developed countries to reduce their ment for subsidiary for scientific and technological advice.
GHG emissions. The Kyoto Protocol aimed to cut emis-
sions of GHG across the developed world by about 5% by COP 3 - Kyoto Japan in 1997 - Kyoto Protocol annexe 1
2012 compared with 1990 levels. India ratified the Kyoto for binding commitments from developed nations.
Protocol in 2002.
COP 8 - New Delhi in 2002 - this called for transfer of
The Kyoto Protocol came into force in February 2005.
technologies to developing nations to minimize the
The USA never ratified the Kyoto Protocol.
impact of climate change on developing nations.
Common But Differentiated Responsibilities (CBDR):
COP 11 - Montreal (Canada) in 2005 - ‘Montreal Action
The major feature of this protocol is that it sets binding
Plan’ - to extend the life of Kyoto Protocol beyond 2012.
targets on industrialised nations including the European
Union. Protocol is based on the principle of common COP 13 - Bali (Indonesia) in 2007: This talked about
but differentiated responsibilities (CBDR). The issue in 2007 talked about financial mechanism, going beyond
is common for all nations, but it puts the obligation to the existing Global Environmental Facility.
reduce current emissions on developed countries on the
basis that they are historically responsible for the cur- COP 14 - Poznan (Poland) in 2008 to launch the
rent levels of GHG in the atmosphere. ‘Adaptation Fund’ under the Kyoto Protocol.
CBDR divides countries into two categories: COP 15 - Copenhagen (Denmark) in 2009 included the
1. The biggest polluting developed nations such as United goal of ‘limiting the maximum global average tempera-
States, United Kingdom, France, Japan and Russia that ture increase to no more than 2 degrees Celsius above
have a record of polluting the earth (these countries preindustrial levels, subject to a review in 2015’.
must accept certain binding limits on GHG emissions; COP 16 - Cancún (Mexico) COP in 2010 - this asked to
under CBDR, they can help in it) lower that maximum of 2 degree centigrade to 1.5 degrees
2. Developing nations such as China, India and Brazil in the near future’. It established ‘Green Climate Fund’.
that have been polluting since 1950s
COP 17 - Durban (Denmark) in 2015 - progressed for
Depending on CBDR, the countries were divided into creation of Green Climate Fund (GCF) of 100 billion dol-
Annexe I Nations: developed nations and economies lars to help poor countries to adopt to climate change.
in transition such as Australia USA, UK, France etc. The COP 18 - Doha Commitments in 2012: The Kyoto
overall aim of targeted reduction of 8% in emission gases Protocol was extended till 2020.
was redistributed among such 43 nations.
COP 19 was held in Warsaw (Poland) in 2013, COP 20 in
Annexe II Nations: Annex II is a subset of Annex I. They Lima (Peru) in 2014.
are required to provide financial and technical support
to the EITs (economies in transition) and developing COP 21 - Paris (France) - 2015 - this COP resulted in
countries to assist them in reducing their greenhouse gas Paris agreement that has been discussed separately.
emissions. Intended Nationally Determined Contribution (INDCs)
were declared subsequently by different nations.
Non-Annexe II Nations: These nations don’t have any
binding commitments but must take care of carbon emis- COP 22 was held in Marrakech (Morocco) in 2016, COP
sions. Examples are developing nations such as China, 23 in Bonn (Germany) in 2017 and COP 24 in Katowice
India, Indonesia etc. (Poland) in 2018.

M09_MADAN 07_65901_C09.indd 54 23/12/22 8:01 PM


People, Development and Environment 9.55

COP 25 was held in Chile with logistical support from 3. The reduction of total projected carbon emissions by
Spain in 2019. one billion tonnes from now to 2030.
4. The reduction of the carbon intensity of the economy
COP 26 was held in Glasgow, Scotland (UK) in November by 45 per cent by 2030, over 2005 levels.
2021. The new Indian commitments have been discussed 5. To achieve the target of net zero emissions by 2070.
separately.
COP 27: The meeting was held in Sharm El-Sheikh in Carbon T rading
November 2022. This COP expresses support to Glasgow’s The Kyoto Protocol suggested two ways to reduce pollu-
COP efforts. There was emergence of ‘Loss and Damage’ tion. This shortfall was to be measured in units of carbon
concept to see the compensation of the most vulnerable credits. One carbon unit is equal to one ton of carbon.
countries for their losses due to climate-related disasters. The first of these methods was that rich countries invest
‘Loss and Damage’ refers to impacts of climate change that money in the clean development mechanism or buy car-
cannot be avoided either by mitigation (cutting green- bon credits from the market. This means that if compa-
house gas emissions) or adaptation (modifying practices nies in developed countries cannot reduce greenhouse
to buffer against climate change impacts). The benefits of gas emissions on their own, then buy carbon credits from
adaptation accrues at locally and not at a global level. The developing countries such as India and China.
developing nations demand that at least 50% of funds Carbon credits (or carbon offsets) can be acquired
should be directed towards adaptation process. through afforestation, renewable energy, CO2 sequestra-
Few General Updates: tion, methane capture, buying from an exchange (carbon
credits trading), etc.
1. IPCC reports and Climate Vulnerability Index (CVI)
identify India as one of the world’s HOTSPOT. Carbon Sink
2. IPCC says that 32-132 millions people to move to extreme
poverty in the next decade due to climate action. A carbon sink is a natural or artificial storage process in
3. Sixth Assessment Report of the IPCC talks about which carbon is stored in the form of compounds of car-
prompt action. bon for an indefinite period of time.
4. Towards mitigation, India has added 100 GW of The process by which carbon is separated from atmo-
renewable energy (excluding hydro) by 2021. The spheric CO2 is called carbon sequestration. The importance
total target by 2030 is 450 GW. of carbon sinks has increased after the Kyoto Protocol.
5. An additional 3 million hectares have been added Carbon sinks are divided into two categories:
over last ten years. By 2030, India aims at 26 million 1. Natural Carbon Sink
hectares. There will be additional carbon sink of 2.5 In the process of photosynthesis by plants, atmo-
to 3 billion tonnes. spheric CO2 is converted into carbohydrate and CO2 is
6. India will produce 5 million tonnes of ‘Green Hydrogen’. also absorbed by the oceans.
7. FAME (Faster Adoption and Manufacturing of Hybrid 2. Artificial Carbon Sink
and Electric Vehicles) was started in India in 2019. 1. Landfills   2. Carbon Capture and Storage
8. Govt aims to adopt 100% electrification of Indian
Railways by 2024 and NET Zero by 2030. Carbon Sequestration
9. There will blending of 20% Ethanol in petrol (EBP)
by 2025. Carbon sequestration is a method of capturing atmospheric
10. PanchAmrit in India talks about electrification using carbon dioxide (CO2) and storing it for a long time. This
RE energy sources, Production Linked Incentive method has the following aspects:
schemes etc. 1. Taking CO2 directly from emission sources such as
11. IPCC says that between 2030-50, developing nations factory chimneys and storing them underground.
need between $127 and $295 billion annually for 2. Taking up and storing carbon directly from the
adaptation. atmosphere.
12. There is talk about additional 3 degree centigrades by 3. Naturally occurring carbon cycle.
end of this century. India needs $1 trillion to $10 tril-
lion till 2070 for NET ZERO emissions.
PM-KUSUM (Pradhan Mantri–Kisan Urja Suraksha Stopover
evm Utthan Mahabhiyaan): This was launched in 2019. 1. By which year does India hope to reach Net Zero,
The main objective is to obtain 40% energy consumption according to its commitments at the COP 26 held at
from renewable energy sources. Glasgow in 2021?
(a) 2030
India’s Commitments at 26th COP Meet in Glasgow (b) 2050
1. To get 500 GW Non-fossil energy capacity by 2030. (c) 2070
2. 50 % of its energy requirements from renewable (d) 2100
energy by 2030. The correct answer is (c).

M09_MADAN 07_65901_C09.indd 55 23/12/22 8:01 PM


9.56 Chapter 9

2. Which of the following statements regarding ‘Green to combat climate change. The Kyoto Protocol (1997)
Climate Fund’ is/are correct? was another major international commitment under the
1. It is intended to assist the developing countries UNFCCC.
in adaptation and mitigation practices to counter
climate change. Intended Nationally Determined
2. It is founded under the aegis of UNEP, OECD, Contributions (INDCs)
Asian Development Bank and World Bank
The Paris Agreement requires all parties to put forward
Select the correct answer using the code given below. their best efforts ‘voluntarily’ through ‘INDCs’ and also to
(a) 1 only strengthen these efforts in the years ahead.
(b) 2 only
(c) Both 1 and 2 India’s Intended Nationally Determined
(d) Neither 1 nor 2 Contribution: At a Glance
The correct option is (a). The INDC proposals are on the following:
• (a) Sustainable Lifestyles, (b) Cleaner economic
Paris Agreement development, (c) Reducing emission intensity of
The framework under Paris Agreement is given as under. gross domestic product (GDP), (d) Increasing the
1. Technology framework share of non–fossil fuel–based electricity, (e) Enhanc-
2. Capacity-building framework ing carbon sink (forest), (f) Adaptation, (g) Mobiliz-
3. Transparency framework ing finance, (h) Technology transfer and capacity
Some of the salient points of the INDC are as follows:
The Kyoto Protocol is set to expire in 2020. The Paris
Agreement deals with what should be done in this decade • To put forward and further propagate a healthy and
and coming decade. The text of the agreement includes sustainable way of living based on traditions and val-
a provision requiring developed countries to send $100 ues of conservation and moderation
billion annually to their developing counterparts begin- • To adopt a climate-friendly and a cleaner path than
ning in 2020. That figure will be a ‘floor’ that is expected the one followed hitherto by others at corresponding
to increase with time. level of economic development
On 5 October 2016, the threshold for entry into force • To reduce the emissions intensity of its GDP by
of the Paris Agreement was achieved. 33–35% by 2030 from the 2005 level
During earlier deals for climate change, there were • To achieve about 40% cumulative electric power
­targets set by the UNFCCC that the COP was not able to installed capacity from non–fossil fuel–based energy
adhere to. So a new mechanism was set in place to deal resources by 2030, with the help of transfer of tech-
with such situation. nology and low-cost international finance, including
In 2015, the Paris Agreement was signed as an inter- Green Climate Fund
national agreement with an aim to reduce global GHG • To create an additional carbon sink of 2.5–3 billion
and thus, it deals with the climate change. There are tons of CO2 equivalent through additional forest and
29 articles in the agreement. The main aim of the Paris tree cover by 2030
Agreement is to keep the global temperature rise this • To better adapt to climate change by enhancing
century well below 2°C above the pre-industrial level. investments in development programmes in sec-
There have to be efforts to limit the temperature increase tors vulnerable to climate change, particularly
even further to 1.5°C. In French, the Paris Agreement is agriculture, water resources, Himalayan region,
known as L’accord de Paris. coastal regions, health and disaster management
It talks about limiting the amount of GHG emitted • To mobilize domestic and new and additional funds
by human activity to the same levels that trees, soil and from developed countries to implement the above
oceans can absorb naturally, beginning at some point mitigation and adaptation actions in view of the
between 2050 and 2100. It also mentions the need to resource required and the resource gap
review each country’s contribution to cutting emissions • To build capacities, create domestic framework and
every 5 years so they scale up to the challenge. international architecture for quick diffusion of
The rich nations should help poorer nations by pro- cutting-edge climate technology in India and for joint
viding ‘climate finance’ to adapt to climate change and collaborative R&D for such future technologies
switch to renewable energy.
The Paris Agreement has a ‘bottom-up’ structure. The International Solar Alliance (ISA)
agreement is binding in some elements such as reporting At COP 21, as part of its INDC, India had committed to
requirements, while leaving other aspects of the deal such achieving 40% of its installed electricity capacity from non-
as the setting of emissions targets for any individual coun- fossil energy sources by 2030. The country has achieved
try as non-binding. this target in November 2021 itself.
The Paris Agreement comes under the broad umbrella The ISA consists of 121 countries. It consists many of
of the UNFCCC. The UNFCCC is a convention held in 1992 those countries that are located between the Tropic of

M09_MADAN 07_65901_C09.indd 56 23/12/22 8:01 PM


People, Development and Environment 9.57

TARGETS FOR PARIS AGREEMENT

India China USA EU (28)


Emission Emission intensity of Absolute Absolute
intensity of GDP GDP 60–65% below emissions emissions
33–35% below 2005 levels by 2030. 26–28% 40%
2005 levels by Peak emissions below below
2030, power around 2030. Non- 2005 1990
capacity to be fossil fuel to be 20% levels by levels by
40% non-fossil of primary energy 2025 2030
fuel-based consumption by 2030

Figure 9.23 Targets for Paris Agreement

Cancer and the Tropic of Capricorn. These countries are


Stopover
located at the shortest distance from the sun, and that is
why solar energy is available in these quantities through- In which of the following cities has the headquarter of
out the year. The ISA has set a target of 1 TW of solar the International Solar Alliance been set up?
energy by 2030. (a) Gurugram
The French Development Agency will allocate (b) New Delhi
€300 million to developing solar energy over the next (c) Bengaluru
5 years in order to finance the initial projects. (d) Chennai
The following are the main objectives of the ISA: The correct option is (a).
1. The purpose of the ISA is to bring such countries
(which are located between the Tropics of Cancer and Natural Resources—Biodiversity
Capricorn) on a platform that supports clean energy,
sustainable environment, clean public transport and Biodiversity is the number of living organisms (both
clean climate. plants and animals) present in an ecosystem. Any loss
2. This alliance wants to overcome the obstacles in the in species in the food chain means breaking a link in the
way of promoting solar energy. chain, which in turn affects all those who benefit from
3. The ISA will promote the development and use of the chain. The diversity can be divided as follows:
solar energy in order to provide energy security to 1. Genetic Diversity: It refers to variation of genes in
present and future generations. Its goal is to generate species that are from a single population.
1 TW (1000 GW) of solar power by 2030. 2. Species Diversity: It is the most basic way to keep an
4. The establishment of the ISA will motivate other account of biodiversity as it includes all forms of life,
countries of the world to increase the production from single-cell organisms, such as amoeba and virus,
and consumption of solar energy for sustainable to multicellular organisms, such as plants and animals.
development. 3. Ecosystem Diversity: It differentiates between dif-
ferent habitats, ecological processes and ecosystems
i ndia ’ S contribution in i nternationaL SoLar in which the species exist. This can be a forest ecosys-
aLLiance tem, marine ecosystem, desert ecosystem and so on.
4. Endemic Species: Endemic species are likely to
The ISA has its secretariat in India. It will increase the
develop on biologically isolated areas such as islands.
stature of India at the international level. Our stated
Due to their geographical isolation, endemics can
objective is ‘India aims to produce 100 gigawatt solar
easily become endangered or extinct if their habitat
energy (which will be one­tenth of ISA’s target) by
changes, not only due to human actions but also due
2022’. India will produce 175 GW of electricity from
to the introduction of new organisms. The opposite of
renewable sources.
endemic species is cosmopolitan species.
The Indian Renewable Energy Development Agency
(IREDA) and Solar Energy Corporation of India (SECI) India is one of the richest countries in the world in terms
announced contribution of US$ 1 million each to the ISA of biodiversity. Due to anthropogenic activities, many spe-
corpus fund. cies have lost their habitat and even become extinct. The
India has kept a target of 450 GW of power genera- IUCN has categorized wild flora and fauna into eight cate-
tion capacity would come from renewables like solar and gories (known as the Red List). This includes the following:
wind by 2030. Besides 60 GW would come from hydro- 1. Extinct
electric power. 2. Extinct in wild

M09_MADAN 07_65901_C09.indd 57 23/12/22 8:01 PM


9.58 Chapter 9

3. Critically endangered conServation oF biodiverSity


4. Endangered The process of conservation can be divided into two types:
5. Vulnerable
6. Lower-risk 1. In Situ Conservation: When conservation is attempted
7. Data-deficient at the natural habitat of the species, by creating
8. Not evaluated national parks, sanctuaries and biosphere reserves, it
is called in situ conservation.
Schedule I of Wildlife (Protection) Act, 1972, defines 2. Ex Situ Conservation: This is done in case of com-
endangered species in a formal manner. So far, 38 species plete degradation of natural habitat. The endan-
of birds, 18 of amphibians and reptiles, and 81 of mam- gered species is kept under total human supervision,
mals have been labelled as endangered. There are 47 criti- such as in zoos, botanical gardens and seed banks.
cally endangered species in India. Manas National Park has been declared a world heri-
bioLogicaL hotSpotS in i ndia
tage site. The other world heritage sites (natural) in
India are Kaziranga National Park (Assam), Manas
A biodiversity hotspot is a biogeographical region with a Wildlife Sanctuary (Assam), Keoladeo National Park
significant reservoir of biodiversity that is under threat (Rajasthan), Sundarbans National Park (West Bengal)
from humans. India is home to 7.31% species of fauna and Nanda Devi National Park (Uttar Pradesh). These
and 10.78% species of flora. Among the 34 hotspots world heritage sites are recognized by UNESCO.
of the world, 2 are located in India and then extend- There are 27 tiger reserves in India under Project
ing to the neighbouring countries. These are Eastern Tiger. Other prominent reserves are Bandipur
Himalayas and Western Ghats (and Sri Lanka). Eastern (Karnataka), Corbett (Uttaranchal), Kanha (Madhya
Himalayas is home to 163 globally threatened spe- Pradesh), Ranthambore and Sariska (Rajasthan) and
cies including Asian elephants, one-horned rhinoceros Sundarbans (West Bengal).
(Rhinoceros unicornis), wild water buffalos and panthers
and tigers. Earlier, Eastern Himalayas was clubbed with
Indo-Burma biodiversity hotspot. The Agasthyamalai bioSphere reServeS
Hills in Western Ghats is home to the highest level of Biosphere reserves protect larger areas of natural habitat in
plant biodiversity. comparison to national parks or animal sanctuaries. They

Concept Box
concept oF t hreatened SpecieS
Threatened species (T)
These species are likely to become extinct if immediate steps are not taken to ensure that they have proper food,
proper habitat, and protection from predators and exotic species so that they are able to realize their biotic potential.
A record of threatened species of plants and animals is maintained by the International Union for Conservation
of Nature and Natural Resources (IUCN), Morges, Switzerland. It is called the Red Data Book. For conservation
purpose, the following four criteria have been used for this categorization:
1. Distribution: Present, past, continuous or discontinuous distribution, area and degree of decline, if available
2. Population: Decline in population in course of time
3. Natural habitat: Abundance and quality
4. Importance: Potential value and biology of the species (the IUCN has identified four categories of threatened
species)
Endangered species (E)
These are threatened species or taxa that are in danger of extinction if the current causal factors continue to operate.
Examples: Lion-tailed macaque (Macaca silenus) and Asiatic wild ass (Asinus hemionus Khur).
Vulnerable species (V)
Vulnerable species or taxa have sufficient population at present, but at the same time, they deplete fast (hence,
depleted species), so they are likely to enter the category of endangered species if the factors bringing about deple-
tion are allowed to continue.
Examples: Golden langur (Presbytis geei) and leopard cat (Felis bengalensis).
Rare species (R)
The populations of species or taxa are small, either localized or thinly scattered.
Examples: Hawaiian monk seal (Monachus schauinslandi) and slow loris (Nycticebus coucang).

M09_MADAN 07_65901_C09.indd 58 23/12/22 8:01 PM


People, Development and Environment 9.59

Table 9.8 List of Biosphere Reserves of India as per Man and Biosphere (MAB) List
S. No. Year Name State Type Key Fauna
1 2008 Great Rann of Kutch Gujarat Desert Indian wild ass
2 1989 Gulf of Mannar Tamil Nadu Coasts Dugong or sea cow
3 1989 Sundarbans West Bengal Gangetic Delta Royal Bengal tiger
4 2009 Cold Desert Himachal Pradesh Western Himalayas Snow leopard
5 1988 Nanda Devi Uttarakhand Western Himalayas NA
6 1986 Nilgiri Biosphere Tamil Nadu, Kerala and Western Ghats Nilgiri tahr, lion-tailed
Reserve Karnataka macaque
7 1998 Dihang-Dibang Arunachal Pradesh Eastern Himalaya NA
8 1999 Pachmarhi Madhya Pradesh Semi-arid Giant squirrel, flying
Biosphere Reserve squirrel
9 2010 Seshachalam Hills Andhra Pradesh Eastern Ghats NA
10 1994 Simlipal Odisha Deccan Peninsula Gaur, Royal Bengal
tiger, sild elephant
11 2005 Achanakmar- Madhya Pradesh, Maikal Hills NA
Amarkantak Chhattisgarh
12 1989 Manas Assam East Himalayas Golden langur, red panda
13 2000 Khangchendzonga Sikkim East Himalayas Snow leopard, red panda
14 2001 Agasthyamalai Kerala, Tamil Nadu Western Ghats Nilgiri tahr, elephants
Biosphere Reserve
15 1989 Great Nicobar Andaman and Nicobar Islands Saltwater crocodile
Biosphere Reserve Islands
16 1988 Nokrek Meghalaya East Himalayas Red panda
17 1997 Dibru-Saikhowa Assam East Himalayas Golden langur
18 2011 Panna Madhya Pradesh Ken river Tiger, chital, chinkara,
sambhar and sloth bear
1. Twelve (given in bold) of the eighteen biosphere reserves given above are a part of the World Network of Biosphere Reserves,
based on the UNESCO Man and the Biosphere (MAB) Programme list.
2. Agasthyamala Biosphere Reserve has been added in World Network of Biosphere Reserves (UNESCO’s MAB) in March 2016.
Source: wikipedia.com.

are the areas of t­errestrial and coastal ecosystems that As on 1 April 2016, the total number of biosphere
promote the conservation of biodiversity with its sustainable reserves in India is 18. Ten of these are a part of UNESCO’s
use. UNESCO’s MAB was launched in 1971. There are MAB Programme list (Table 9.9).
over 500 biosphere reserves across the globe in more Biosphere reserves are large areas of biodiversity where
than 100 countries. National parks, wildlife sanctuaries, flora and fauna are protected. They roughly correspond to
conservation reserves and community reserves are the four IUCN Category V Protected areas. The Indian government
types of protected areas under the Wildlife Protection Act, has established 18 Biosphere Reserves of India, which
1972. Biosphere reserves are considered akin to national protect larger areas of natural habitat (as compared to a
parks and they are usually larger than national parks. The national park or wildlife sanctuary).
primary criteria for the selection of biosphere reserves are Biosphere reserves of India often include one or more
effective protection and minimally disturbed core area. The national parks or sanctuaries, along with buffer zones that
secondary criteria include the following: are open to some economic uses. Protection is granted not
only to the flora and fauna of the protected region but
1. Having rare and endangered species also to the human communities who inhabit these regions
2. Diversity of soil and microclimatic conditions and their ways of life.
3. Preservation of tribal and rural life

M09_MADAN 07_65901_C09.indd 59 23/12/22 8:01 PM


9.60 Chapter 9

Concept Box
Important Legislations and Terms Relating to Environment, Pollution, Forests and Wildlife
1. National Green Tribunal (NGT): The tribunal was set on 18 October 2010 under the NGT Act, 2010, for effec-
tive and expeditious disposal of cases related to environmental protection and conservation of forests and other
natural resources. It is a specialized body equipped with the necessary expertise to handle environmental dis-
putes involving multidisciplinary issues.
2. Environment Impact Assessment (EIA): It is a widely recognized study to assess the environmental impact of
development projects. It is basically a cost–benefit analysis in the context of the environment. Environmental risk
assessment (ERA) is a fact-finding stage, in which EIA indicates the potential hazards. In India, the Department
of Environment and National Council of Environmental Planning (NCEP) have adopted a preliminary procedure
to prepare EIA.
3. Precautionary Principle: The precautionary principle is a moral and political principle, which aims to prevent
any action that can cause damage to the public or society at large. For example, the Supreme Court of India
ordered the shifting of industries outside Delhi a few years back. In the Earth Summit held at Rio de Janeiro in
1992, a precautionary approach was codified for the first time at a global level, to protect the environment in the
form of Principle 15 of Agenda 21. It emphasizes that every state should apply the principle according to its capa-
bilities, and lack of full scientific knowledge should not be used as a reason for not taking action. This is necessary
to check the possibility of any irreversible damage to the environment. There are two popular terms associated
with precautionary principle:
(a) Polluter Pays Principle (PPP): This idea first originated in the Organization for Economic Cooperation and
Development, in which pollution control costs are to be financed by the polluter alone.
(b) Beneficiary Pays Principle (BPP): It suggests that funding for environmental improvement should be
obtained from its beneficiaries. This method of financing would generate larger revenue for the government
as the rich are willing and capable of paying more for environmental enrichment. Imposing carbon tax is an
example.
4. ISO 14000: After the success of ISO 9000 Quality Management System, which focuses on continual improve-
ment of the processes and quality of goods and services, the International Organization for Standardization
(ISO) introduced ISO 14000 series of Environmental Management System in 1996. It specifically deals with
environmental aspects of processes in products and services.
Few Important International Conventions on Environment and Biodiversity
Convention on International Trade in Endangered Species of Wild Fauna and Flora (CITES), 1973: The objec-
tive is to control international commercial trade in endangered species or products derived from them.
Carbon space refers to the amount of carbon that can be released into the atmosphere by 2100 so that the rise in
global temperature can be capped at 2°C. It is set at 1000 gigatons of carbon dioxide equivalent (GtCO2eq).
Basel Convention, 1989: The main aim is to minimize transboundary movement of hazardous wastes.
Nagoya Protocol: It is a supplementary agreement to the Convention on Biological Diversity. The Nagoya Protocol
on Access and Benefit Sharing (ABS) was adopted on 29 October 2010 in Nagoya, Japan.
Stockholm Convention: It was adopted in 2001 and enforced in 2004. It deals with reducing and eliminating the
production and use of persistent organic pollutants.
Ramsar Convention: It was signed in the Iranian city of Ramsar in 1971 and enforced in 1975. It deals with conser-
vation and use of wetlands.
A Snap Shot of Main Environmental Movements in India
Chipko Movement: It was started in 1973 in Chamoli District of Uttarakhand. It entailed embracing trees on seeing
an axe coming near the tree to cut it. Its main leader was Shri Sunderlal Bahuguna.
Appiko Movement: It was started in 1983 in the villages of Western Ghats in Uttar Kannada region of Karnataka.
The main purpose was to prevent commercial felling of trees. It became a symbol for people’s power for their rights
of natural resources with regard to the state.
Tehri Dam Movement: This movement was spearheaded by Baba Amte against the submergence of land by con-
struction of dams and population displacement among other environmental concerns. It started in the 1970s and
continued until a decade ago.
Save Silent Valley Movement: It was started in Palakkad district of Kerala in 1973 to save the Silent Valley Reserve
Forest from being flooded by a hydroelectric project. The valley was declared as Silent Valley National Park in 1985.

M09_MADAN 07_65901_C09.indd 60 23/12/22 8:01 PM


People, Development and Environment 9.61

Narmada Bachao Andolan: It was launched to protect the population from the adverse effects of Narmada Valley
Project, that is two megaprojects, Sardar Sarovar Project and Narmada Sagar Project in Madhya Pradesh. This inter-
state project involving Madhya Pradesh, Maharashtra and Gujarat has been constructed on the Narmada River. Its
main leader is Medha Patkar. The main issues involved were related to displacement of population and submerging
of forest land.
Taj Trapezium Zone: Sulphur dioxide gas released by Mathura Oil Refinery and other industries (combined with
oxygen and moisture to form sulphuric acid) caused extensive damage to the Taj marbles. It corroded the marble and
formed fungus, also referred to as ‘marble cancer’.
Key Institutions for Environment in India
1. Bombay Natural History Society, Mumbai
2. World Wide Fund for Nature-India, New Delhi
3. Centre for Science and Environment, New Delhi
4. CPR Environmental Education Centre, Chennai
5. Centre for Environment Education (CEE), Ahmedabad
6. The Botanical Survey of India (BSI), Kolkata
7. Zoological Survey of India (ZSI), Kolkata
8. National Environment Engineering Research Institute, Nagpur

Important Days Linked with Environment

Dates Important Days Dates Important Days


30 January World Leprosy Day 27 June International Diabetes Day
2 February World Wetlands Day 11 July World Population Day
25 February World Sustainable Energy Day 28 July World Nature Conservation Day
20 March World Sparrow Day 29 July World Tiger Day
21 March World Forestry Day 6 August Hiroshima Day
22 March World Water Day 10 August World Biofuel Day
22 April World Earth Day 3 October World Nature Day
25 April World Malaria Day 4 October World Animal Day
31 May World No Tobacco Day 2 December World Pollution Prevention Day
5 June World Environment Day 14 December World Energy Day
8 June World Oceans Day 29 December International Biodiversity Day

M09_MADAN 07_65901_C09.indd 61 23/12/22 8:01 PM


9.62 Chapter 9

A s s e s s Yo u r L e a r n i n g
CONCEPT OF PEOPLE, MDG, SDG, HuMAN-ENvIRONMENT INTERACTION
1. A population consists of a group of organisms Statement 2: Synecology is the study of group of
(a) of the same species living in a specific geographic organisms of different species which are associated
area that have the potential to interbreed. together as a unit in form of a community.
(b) residing in a particular geographic area. (a) Both statement 1 and 2 are true
(c) living in a specific geographic locale. (b) Only statement 1 is true
(d) of the same species. (c) Only statement 2 is true
2. The conservation of environment requires (d) Neither statement 1 nor statement 2 is true
(a) Maintenance of biodiversity 7. Which of the following are the main features of Earth
(b) Maintenance of gaseous and material cycles Hour?
(c) Maintenance of ecological order and natural balance 1. This is organized by World Wildlife Fund.
(d) All of the above 2. This intends to diversify people to take action on
3. Assertion (A): Microenvironment refers to the imme- environmental issues and protect the planet.
diate local surrounding of the organism and macro 3. This was started as the lights were switched off for
environment refers to all the physical and biotic con- a fixed duration in Sydney, Australia in 2007.
ditions that surround the organism externally. (a) 1 and 2 are correct
(b) 1 and 3 are correct
Reason (R): Environment refers to both abiotic (phys-
(c) 2 and 3 are correct
ical or non-living) and biotic (living) environment.
(d) All of the above
Which of the following is the correct answer?
8. Assertion (A): In a given area, the maximum population
(a) Both (A) and (R) are true and (R) is the correct size of the species that the environment can sustain is
explanation of (A). called the carrying capacity.
(b) Both (A) and (R) are true but (R) is not the correct
explanation of (A). Reason (R): As a population grows in an area, a
(c) (A) is true, but (R) is false. population may experience the effects of increased
(d) (A) is false, but (R) is true. densities.
Which of the following is the correct answer?
4. Statement 1: In the study of man–environment inter-
A S S E S S YO U R L E A R N I N G

action, the statement of Miss Semple that ‘the humans (a) Both (A) and (R) are true and (R) is the correct
are slowly the product of their environment’ is a fact. explanation of (A).
(b) Both (A) and (R) are true but (R) is not the correct
Statement 2: A habitat is a particular area inhabited explanation of (A).
by plants and animals. (c) (A) is true, but (R) is false.
Which of the above statements are true? (d) (A) is false, but (R) is true.
(a) Both statements 1 and 2 are true 9. Assertion (A): As a result of biomagnification, the
(b) Only statement 1 is true secondary consumers will be the most toxic.
(c) Only statement 2 is true
Reason (R): The contaminants in biomagnification
(d) Neither statement 1 nor statement 2 is true.
may be heavy metals such as mercury, arsenic, pesti-
5. Statement I: The concept of Environmental cides such as polychlorinated biphenyls and DDT.
Determinism says that environment is determined by
Which of the following is the correct answer?
culture and nature.
(a) Both (A) and (R) are true and (R) is the correct
Statement II: Environmental possibilism believes
explanation of (A).
that the environment puts limits on people, but it
(b) Both (A) and (R) are true but (R) is not the correct
does not determine how they will behave. Thus, it
explanation of (A).
can be linked with ‘anthropocene’ age and ‘anthropo-
(c) (A) is true, but (R) is false.
genic’ activities.
(d) (A) is false, but (R) is true.
(a) Both Statements 1 and 2 are true
(b) Only statement 1 is true 10. Assertion (A): ‘Ecosystem processes’ are basically
(c) Only statement 2 is true the transfers of energy and materials from one pool
(d) Neither statement 1 nor statement 2 is true to another which are known to ‘take place at a wide
range of scales’.
6. Statement 1: Autecology that is also known as popu-
lation ecology, is the study of individual organism or Reason (R): An ecosystem consists of all the organ-
individual species. isms and the abiotic pools linked with physical envi-

M09_MADAN 07_65901_C09.indd 62 23/12/22 8:01 PM


People, Development and Environment 9.63

ronment with which they interact. The biotic and Statement II: Pyramid of biomass is inverted since

abiotic components are linked together through primary producers make up less biomass than pri-
nutrient cycles and energy flows. mary consumers. The inverted pyramid of biomass is
Choose the correct answer from the options given below: seen in a lake or pond ecosystem, as the phytoplank-
ton is less as compared to the small herbivorous fish.
(a) Both (A) and (R) are true and (R) is the correct
explanation of (A). Which of the above statements are true?
(b) Both (A) and (R) are true but (R) is not the correct (a) Statements 1 and 2 are true
explanation of (A). (b) Only statement 1 is true
(c) (A) is true, but (R) is false. (c) Only statement 2 is true
(d) (A) is false, but (R) is true. (d) Both statements are false.
11. Assertion (A): Autotrophic plants are considered as 15. Statement I: Bioaccumulation takes place in a single
producers in a food chain using light, water, carbon organism over the span of its life, resulting in a higher
dioxide, or other chemicals. concentration in older individuals.
Reason (R): Autotrophs make use of energy from the Statement II: Biomagnification takes place as chemi-

sun to convert water from the soil and carbon dioxide cals transfer from lower trophic levels to higher tro-
from the air into a nutrient called glucose. phic levels within a food web, resulting in a higher
Which of the following is the correct answer? concentration at higher trophic levels.
(a) Both (A) and (R) are true and (R) is the correct Which of the above statements are true?
explanation of (A).
(b) Both (A) and (R) are true but (R) is not the correct (a) Statements 1 and 2 are true
explanation of (A). (b) Only statement 1 is true
(c) (A) is true, but (R) is false. (c) Only statement 2 is true
(d) (A) is false, but (R) is true. (d) Both statements are false.
12. Assertion (A): In the analysis of man–environment 16. Assertion (A): Cloudy nights are warmer compared
relationship, the man has to watch and assess the situ- to clear nights (without clouds) during winters.
ation and then go ahead with resource utilization Reason (R): Clouds permit cold waves from the sky
Reason (R): This mainly includes anthropogenic reaching the earth
activities which mean increased concentration of CO2 Which of the following is the correct answer?
in the air and thus includes burning of fossil fuels, (a) Both (A) and (R) are true and (R) is the correct
deforestation, land-use changes, livestock produc- explanation of (A).
tion, waste management, industrial processes, etc. (b) Both (A) and (R) are true but (R) is not the correct

A S S E S S YO U R L E A R N I N G
Which of the following statements is correct? explanation of (A).
(c) (A) is true, but (R) is false.
(a) Both (A) and (R) are true and (R) is the correct
(d) (A) is false, but (R) is true.
explanation of (A).
(b) Both (A) and (R) are true but (R) is not the correct 17. Assertion (A): The heat is trapped close to Earth’s
explanation of (A). surface by mostly natural occurring gases such as
(c) (A) is true, but (R) is false. carbon dioxide, methane, nitrous oxide, and water
(d) (A) is false, but (R) is true. vapors and synthetic fluorinated gases.
13. Match the following: Reason (R): These gases make a sort of blanket
around the Earth, keeping the planet warmer than it
List I List II would be without them.
A. Troposphere I. Dust particles Which of the following is the correct answer?
(a) Both (A) and (R) are true and (R) is the correct
B. Stratosphere II. Ozone layer explanation of (A).
C. Ionosphere III. Meteors (b) Both (A) and (R) are true but (R) is not the correct
explanation of (A).
D. Exosphere IV. Aurora (c) (A) is true, but (R) is false.
(d) (A) is false, but (R) is true.
Codes:
18. Statement 1: Carbon sink is a process or mechanism
(a) A–I, B–II, C–III, D–IV that removes carbon dioxide from the atmosphere.
(b) A–II, B–I, C–III, D–IV
Statement 2: Oceans are the natural resources are
(c) A–IV, B–II, C–III, D–I
the main carbon sinks they absorb up to half of the
(d) None of the above
carbon dioxide due to Phytoplankton.
14. Statement I: Pyramid of energy is always upright.
This is because during energy transfer between adja- Statement 3: The largest sink (pollutant receptor) of

cent trophic levels, energy is lost in the form of heat. the planet is hydrosphere.

M09_MADAN 07_65901_C09.indd 63 23/12/22 8:01 PM


9.64 Chapter 9

(a) Statements 1, 2 and 3 are true (d) Maximizing the present-day benefits through
(b) Only statements 1 and 2 are true increased resource consumption.
(c) Only statements 2 and 3 are true 24. Assertion (A): The Sustainable Development goals
(d) Only statements 1 and 3 are true are ‘more universal’ in nature in comparison to Mil-
19. Identify the gas from the statements given below: lennium Development Goals.
1. A powerful greenhouse gas is released that accel- Reason (R): The achievement of goals under Sustain-
erates the phenomenon of climate change. able Development Goals is no more the case of ‘rich
2. Wetlands are the largest natural source of that donors aiding recipients’.
atmospheric gas in the world. (a) Both (A) and (R) are true and (R) is the correct
(a) Carbon dioxide (b) Methane explanation of (A).
(c) Water vapours (d) Nitrous oxide (b) Both (A) and (R) are true but (R) is not the correct
explanation of (A).
20. Assertion (A): Ocean acidification and negative
(c) (A) is true, but (R) is false.
impact on corals, algae, shellfish is caused due to
(d) (A) is false, but (R) is true.
excess of carbon dioxide in sea water.
Reason (R): The natural carbon sinks have a maxi- 25. Assertion (A): The goals are supposed to achieve
mum limit of carbon dioxide absorption. zero level, for examples, zero hunger or zero prevent-
able child deaths.
Which of the following is the correct answer?
Reason (R): The target in the MDGs was to reach
(a) Both (A) and (R) are true and (R) is the correct
halfway, but the SDGs have been designed for com-
explanation of (A).
pleting the job.
(b) Both (A) and (R) are true but (R) is not the correct
explanation of (A). (a) Both (A) and (R) are true and (R) is the correct
(c) (A) is true, but (R) is false. explanation of (A).
(d) (A) is false, but (R) is true. (b) Both (A) and (R) are true but (R) is not the correct
explanation of (A).
21. What is the definition of sustainable development?
(c) (A) is true, but (R) is false.
(a) The growth that satisfies current demands with- (d) (A) is false, but (R) is true.
out jeopardising future generations’ ability to ful-
26. Statement I: Sustainable development is aimed at
fil their own needs.
ensuring quality of life to present and future generations.
(b) Conserve mineral wealth and explore alternative
energy sources while decreasing pollution and Statement II: ‘Leave no one behind’ and ‘Endeav-
environmental impact. our to reach the furthest behind first’ are the pledges
(c) It is the process of creating land and building proj- (agenda) for ‘Sustainable Goals’.
A S S E S S YO U R L E A R N I N G

ects in such a way that they have a lower environ- Which of the above statements are true?
mental effect by enabling them to produce fuel- (a) Both statements 1 and 2 are true
efficient self-sufficiency patterns. (b) Only statement 1 is true
(d) All of the above (c) Only statement 2 is true
22. Statement I: The area that matches the most under (d) Both statements are false
‘Sustainable Development Goals’ and ‘Millennium 27. Which of the following UN commission is responsi-
Development Goals’ is environment. ble for reviewing progress in the implementation of
Statement II: The agenda for sustainable develop- Agenda 21 and the Rio Declaration on Environment
ment was settled in the year 2012 in UN Conference and Development?
through ‘The Future We Want’. (a) United Nation Disarmament Commission
Which of the above statements are true? (b) United Nations Statistical Commission
(a) Statements 1 and 2 are true (c) United Nations Commission on Sustainable
(b) Only statement 1 is true Development
(c) Only statement 2 is true (d) United Nations Commission on Human Rights
(d) Both statements are false. 28. NITI Aayog has released the 2019 Sustainable
23. Sustainable development will NOT aim at: Development Goals—India Index. The index
evaluates the progress of states and union territories
(a) Social economic development which optimizes
on social, economic and environmental parameters.
the economic and societal benefits available in
the present, without spoiling the likely potential Which of the following states has received the first
for similar benefits in the future. rank in 2022?
(b) Reasonable and equitable distributed level of eco- (a) Himachal Pradesh
nomic well-being that can be perpetuated continually. (b) Goa
(c) Development that meets the need of the present (c) Jammu and Kashmir
without compromising the ability of future gen- (d) Kerala
eration to meet their own needs.

M09_MADAN 07_65901_C09.indd 64 23/12/22 8:01 PM


People, Development and Environment 9.65

29. Which of the following statements are true in context that are in terms of zone and their chemical character.
of reports by UNEP in March 2022?
1. India is now behind all south Asian nations except List I List II
Pakistan. (Zone) (Chemical Character)
2. Among states Kerala, Tamil Nadu (along with A. Atmosphere I. Inert gases
Himachal Pradesh) and Goa are the three states
in SDG category. B. Biosphere II. Salt, freshwater, snow
3. Among Union Territories, Chandigarh was ranked and ice
first, followed by Delhi (along with Lakshadweep C. Hydrosphere III. Organic substances
and Puducherry) for the second and Andaman
and Nicobar Islands with third place. D. Lithosphere IV. Light silicates
4. SDG India Index provides us the overall progress
of our nation on the social, economic and envi- Codes:
ronmental status of the nation. (a) A–II, B–III, C–I, D–IV
Codes: (b) A–I, B–III, C–II, D–IV
(a) 1, 2 and 3 (c) A–II, B–I, C–III, D–IV
(b) 2, 3 and 4 (d) A–IV, B–II, C–III, D–I
(c) 1 and 4 34. The earth is often struck by solar eruptions, which
(d) 1, 2, 3 and 4 comprise energetic particles that are hurled away
30. As per data available in November 2022, which of the from the sun into space. Which of the following state-
following are the correct statements in Maternal Mor- ments reflect the effect on the earth?
tality Rate (MMR)? 1. When these eruptions interact with the magnetic field,
1. MMR refers to deaths of pregnant women or they cause beautiful auroras.
within 42 days of termination of pregnancy 2. They break down radio communication and power
depending upon certain factors. It is calculated supplies.
on per lakh live births cases. 3. They do not affect life support system on the earth.
2. MMR has declined from to 97 during 2018-20,
Codes:
from 103 in 2017-2019 and 130 in 2014-2016.
3. Kerala is the top-ranking state with 19. (a) 1 only
4. West Bengal is the least ranking state with 195. (b) 1 and 2 only
Codes: (c) 2 and 3 only
(d) 3 only
(a) 1, 2 and 3

A S S E S S YO U R L E A R N I N G
(b) 2, 3 and 4 35. Statement 1: Beginning from the surface of the earth,
(c) 1 and 4 the layers of atmosphere are troposphere, strato-
(d) 1, 2, 3 and 4 sphere, ionosphere, exosphere.
31. PM-KUSUM Scheme is linked with which of the fol- Statement 2: Troposphere is the most dynamic layer
lowing Ministry of Government of India? of the atmosphere in terms of climatic and weather
(a) Ministry of Women and Child Development conditions.
(b) Ministry of Education Statement 3: The ozone gas is present in the strato-
(c) Ministry of New and Renewable Energy sphere only.
(d) Ministry of Home Affairs Which of the following statements are true?
32. Assertion (A): According to IIT-Bombay studies–The
smog holes are fog burn-off taking place in the middle (a) Only statements 1 and 2 are true
of morning hours. The temperature within cities is (b) Only statements 2 and 3 are true
warmer than surrounding rural areas. (c) Only statements 1 and 3 are true
(d) Statements 1, 2 and 3 are true.
Reason (R): The holes as big as cities are the result of
urban island effect. 36. Which of the following statement is true in context of
an ecosystem ?
Which of the following is the correct answer?
1. The transition area between two biomes is called
(a) Both (A) and (R) are true and (R) is the correct an ecotone.
explanation of (A). 2. The species which are indigenous to an area are
(b) Both (A) and (R) are true but (R) is not the correct known as endemic species.
explanation of (A). 3. The species which are restricted to the transition
(c) (A) is true, but (R) is false. area of ecotone and don’t move into either of the
(d) (A) is false, but (R) is true. biomes are called as edge species.
33. The structure of the earth’s system affects the environ- 4. The species which play a significant role in the func-
ment in a very complex manner. Match List I with List II tioning of an ecosystem are called keystone species.

M09_MADAN 07_65901_C09.indd 65 23/12/22 8:01 PM


9.66 Chapter 9

(a) 2, 3 and 4 41. Match List-I with List-II.


(b) 1, 2, 3 and 4
(c) 3 and 4 List-I List-II
(d) 1, 3 and 4 (Day) (Context)
37. Which of the following fluorinated gases has been
A. World Earth Day 1. August 10
added as the seventh greenhouse gas by the United
Nations Framework Convention on Climate Change? B. World Population Day 2. June 5
(a) Hydrofluorocarbons
(b) Perfluorocarbons C. World Environment Day 3. July 11
(c) Sulphur hexafluoride D. World Biofuel Day 4. April 22
(d) Nitrogen trifluoride
38. Consider the following statements in context of com- Codes:
parison of Coal Bed Methane and shale gas?
(a) A – 4, B – 2, C – 3, D – 1
1. Coal bed methane is the PURE methane gas (b) A – 1, B – 3, C – 2, D – 4
extracted from coal streams, while shale gas is a (c) A – 4, B – 3, C – 2, D – 1
mixture of propane and butane only that can be (d) A – 4, B – 1 C – 2, D – 3
extracted from fine grained sedimentary rocks.
2. In India, abundant coal bed methane sources exist 42. Which of the following statements is true in context of
but, so far, no shale gas has been found. wetlands?
Which of the statements given above is/are correct? 1. A wetland is a distinct ecosystem that is flooded
by water, either permanently or seasonally, where
(a) Both 1 and 2 are correct oxygen-free processes prevail. A wetland has
(b) 1 is correct while 2 is incorrect the characteristic vegetation of aquatic plants,
(c) 1 is incorrect while 2 is correct adapted to the unique hydric soil.
(d) Neither 1 nor 2 is correct. 2. February 2 is celebrated as International Wetlands
39. Which of the following statements are true in context Day as the Ramsar Convention was signed on
of Coalbed Methane (CBM)? February 2, 1971.
1. CBM is an unconventional source of natural gas 3. India adds 11 more wetlands to the list of Ramsar
that is now considered as an alternative source for sites to making a total of 100 such sites in India.
augmenting India’s energy resource. 4. Wetlands are the largest natural source of methane.
2. India has the fifth largest proven coal reserves in This produces 78% of natural methane emissions.
the world and thus holds significant prospects for (a) 1, 2 and 3 (b) 2, 3 and 4
exploration and exploitation of CBM. (c) 1, 2 and 4 (d) 2 and 3
A S S E S S YO U R L E A R N I N G

3. The Gondwana sediments of eastern India host 43. Which of the following statements is true/ are in con-
the bulk of India’s coal reserves and all the cur- text of climate change and global warming?
rent CBM producing blocks. 1. An increase in sea level due to melting of glaciers
(a) Only statements 1 and 2 are true and polar ice would indicate a warming in the
(b) Only statements 2 and 3 are true average global temperature.
(c) Only statements 1 and 3 are true 2. The smog that had occurred in London in
(d) Statements 1, 2 and 3 are true December 1952 was called as ‘sulfurous’ smog.
40. The Emissions Gap Report 2022 finds that the world 3. Mauna Loa in Hawaii in USA is famous for continuous
must cut emissions by the following percentage to monitoring of atmospheric CO2 levels since 1957.
avoid global catastrophe. (a) 1 and 2 (b) 2 and 3
(a) 10% (b) 20% (c) 1, 2 and 3 (d) 1 and 3
(c) 25% (d) 45%

Pollution, Pollutants and Waste Management


44. The balance in oxygen level is maintained by (c) Suspended Particulate Matter
(a) Photosynthesis and cellular respiration (d) Sulphur dioxide
(b) Industrial emissions of gases 46. Identify the rank of nations which emit highest car-
(c) Release of ozone in upper atmosphere bon dioxide pollution in an increasing order.
(d) Photochemical smog (a) India, China, USA
45. Which of the following is a non-vehicular pollutant? (b) Japan, India, China
(a) Carbon Monoxide (c) India, USA, China
(b) Chlorofluorocarbon (d) USA, India, China

M09_MADAN 07_65901_C09.indd 66 23/12/22 8:01 PM


People, Development and Environment 9.67

47. Palau and Qatar are the nations with highest PER 52. Match List-I with List-II for chemical gases and their
CAPITA carbon dioxide emissions. Identify the list impact.
of such nations from the following in an increasing
order. List-I List-II
(a) USA, Russia, Japan, China and India A. Carbon 1. Produced from automobile
(b) India, China, Japan, Russia and USA dioxide engines and their reaction
(c) China, USA, Japan, Russia and India makes photochemical smog.
(d) India, China, USA, Japan and Russia
B. Sulphur 2. Dangerous to human beings at
48. The rank of Greenhouse Gas emissions at a global dioxide high concentrations. They are
level in the decreasing order is responsible for acid rain.
(a) Carbon dioxide, Methane, Sulphur dioxide and
Flourinated gases C. Sulphur 3. Burning of sulphur containing
(b) Flourinated gases, Carbon dioxide, Nitrous oxide, oxide compounds of fossil fuels in
Methane vehicles.
(c) Carbon dioxide, Methane, Nitrous oxide and D. Nitrogen 4. Fossil fuel burning and
Flourinated gases oxide deforestation.
(d) Methane, Nitrous oxide, Carbon dioxide and
Flourinated gases
Codes:
49. Which of the following shows the distribution of
greenhouse gas emissions in India sector wise? (a) A – 1, B – 3, C -2, D – 4
(b) A – 4, B – 2, C -3, D – 1
(c) A – 4, B – 3, C -2, D – 1
List-I List-II (d) A – 1, B – 2, C - 3, D – 4
(Sector) (%age) 53. Which of the following statement(s) is/are true about
the methane as a green house gas ?
A. Electricity /Heat 1. 9
(a) Methane is the second most abundant anthropo-
B. Agriculture 2. 21 genic GHG after carbon dioxide
(b) Methane accounts for about 20 percent of global
C. Manufacturing/construction 3. 17
emissions.
D. Transportation 4. 37 (c) Methane is 25 times more potent then carbon
dioxide at trapping heat in the atmosphere
(d) All of the above
(a) A – 1, B – 2, C -3, D – 4

A S S E S S YO U R L E A R N I N G
54. Which of the following greenhouse gases is entirely
(b) A – 4, B – 2, C -3, D – 1 anthropogenic in origin?
(c) A – 4, B – 2, C -3, D – 1
(d) A – 2, B – 4, C -3, D – 1 (a) Water vapour
(b) Carbon dioxide
50. The main difference between primary air pollutants (c) Chlorofluorocarbons
and secondary air pollutants is (d) Methane
(a) Primary pollutants are released directly into 55. Normally, temperature decreases with increase in ele-
the atmosphere, while the secondary pollut- vation, that is called as normal lapse rate. In cities, suf-
ants are formed by chemical reactions in the fering from smog such as Delhi, the situation may be
atmosphere. reversed and the normal lapse rate is inverted. Thus, a
(b) The best example of primary air pollutants is temperature inversion is a condition when the air
Carbon dioxide released from burning of coal.
(c) The best example of secondary air pollutant is (a) Near the ground is lighter
Ozone produced in photochemical smog. (b) Near the ground is cooler than air at the higher
(d) All of the above altitudes
(c) Near the ground is hotter than that at higher
51. Which of the following statements is/are true about CO2? altitudes
(a) CO2 is the most common greenhouse gas that (d) None of the above
accounts for about 76% of greenhouse gases.
56. The most relevant effect of particulate matter in air on
(b) CO2 is basically a trace gas.
human health is [November 2020]
(c) 90% of CO2 comes from the burning of fossil fuels,
(a) Impaired blood formation
namely coal, oil, and natural gas.
(b) Aggravation of respiratory disease
(d) All of the above (c) Chest tightness
(d) Headache and rise in blood pressure level

M09_MADAN 07_65901_C09.indd 67 23/12/22 8:01 PM


9.68 Chapter 9

57. Cigarette smoking is considered to be more pollut- 63. Statement 1: The nitrogen oxides, sulphur oxides
ant than air pollution. Which of the following are the and carbon oxides are primary air pollutants that con-
main factors? tribute to acid deposition.
(a) Cigarette smoking causes cardiovascular disease, Statement 2: Atmospheric pollutants are mainly
cancer, chronic lung disease, diabetes etc. present in Troposphere and lower stratosphere
(b) Cigarette smoking pollutes the system with nico-
tine, nickel, cadmium, heavy metals, plastics and Statement 3: Persistent pollutants remain in environ-
other pollutants. ment for a long time such as pesticides, plastics and
(c) People who do not smoke but are exposed to nuclear waste while non persistent pollutants break
second hand smoke at home or at work increase into simple products such as garbage.
their risk of developing heart disease by 25–30%. (a) Only statements 1 and 2 are true
Second hand smoke is already contaminated. (b) Only statements 2 and 3 are true
(d) All of the above (c) Only statements 1 and 3 are true
58. The most dominant source of benzene emissions in (d) Statement 1, 2 and 3 are true
ambient air is [December 2019] 64. Match the following:
(a) Cement industry
(b) Cigarettes List I List II
(c) Car exhausts (Metal) (Effects)
(d) Paints and varnish A. Chromium I. Thalassaemia
59. An organization or entity receives one carbon credit if
B. Arsenic II. Itai-itai
it is able to avoid emission of
(a) One ton of carbon dioxide or its equivalent C. Cadmium III. Dermatitis
(b) One ton of any greenhouse gas
(c) One ton of hydrocarbon emissions D. Iron IV. Carcinogen
(d) One ton of carbon monoxide Codes:
60. Match List-I with List-II. [October 2020] (a) A–IV, B–III, C–II, D–I
List-I List-II (b) A–IV, B–II, C–III, D–I
(Emitted (Environmental Impact) (c) A–I, B–III, C–II, D–IV
Pollutants) (d) A–I, B–II, C–III, D–IV
65. Statement 1: Industries generating hazardous waste
A. Carbon dioxide I. Formation of acid rain are classified as red.
B. Carbon monoxide II. Toxic and are Statement 2: Intake of lead that is also known anti
A S S E S S YO U R L E A R N I N G

carcinogenic knocking agent (tetraethyl) may primarily cause


damage to the brain.
C. Nitrogen oxide III. Toxic and can cause
respiratory diseases Statement 3: The chemical responsible for blue baby
syndrome, that is caused by restricted supply of oxy-
D. Benzene and IV. Contribution to global gen to the brain, is linked to nitrates.
hydrocarbons warming as a GHG (a) Only statements 1 and 2 are true
Codes: (b) Only statements 2 and 3 are true
(a) A – IV, B – III, C – I, D- II (c) Only statements 1 and 3 are true
(b) A – IV, B – II, C – I, D- III (d) Statement 1, 2 and 3 are true
(c) A – IV, B – II, C – III, D- I 66. Arrange List II in proper sequence so as to match it
(d) A – I, B – II, C – III, D- IV with List I and choose the correct answer from the
61. Using coal, natural gas or oil for electricity, heat or code given below.  [December 2004]
transportation releases CO2 into the atmosphere. The List I List II
daily CO2 emissions make up the (Activity) (Noise Level [dB])
(a) Carbon footprint
A. Hearing I. 30
(b) Carbon dating
(c) Carbon credit B. Whispering II. 1
(d) None of the above
C. Interference with sleep III. 60
62. With the absorption and decomposition of CO2 in
ocean water beyond desired level, there will be D. Normal talk IV. 30–50
(a) Decrease in temperature
(b) Increase in salinity Codes:
(c) Growth of phytoplanktons (a) A–I, B–II, C–III, D–IV
(d) Rise in sea level (b) A–II, B–I, C–IV, D–III

M09_MADAN 07_65901_C09.indd 68 23/12/22 8:01 PM


People, Development and Environment 9.69

(c) A–IV, B–II, C–III, D–I 74. The small amount of warming that may set off unstop-
(d) A–III, B–I, C–II, D–IV pable and irreversible changes is termed
67. Global warming during winter becomes more pro- (a) Tipping point (b) Inflection point
nounced at the (c) Turning point (d) None of the above
(a) Equator (b) Poles 75. Which of the following is not considered a greenhouse
(c) Tropic of Cancer (d) Tropic of Capricorn gas?
(a) Methane
68. Statement I: The main pollutant of the Indian coastal (b) Chlorofluorocarbons (CFCs)
water is industrial effluents. (c) Hydrogen
Statement II: Tamil Nadu coastal belt has drinking (d) Ozone
water problems due to seepage of sea water. 76. The atmosphere is mainly heated by the
Which of the above statements are true? (a) Short wave solar radiation
(a) Statements 1 and 2 are true (b) Long wave terrestrial radiation
(b) Only statement 1 is true (c) Reflected solar radiation
(c) Only statement 2 is true (d) Scattered solar radiation
(d) Both statements are false 77. Statement I: An aerosol is a suspension of fine solid
69. Arrange List II in proper sequence so as to match it particles or liquid droplets in air or another gas.
with List I and choose the correct answer from the Statement II: Aerosols can be natural or anthropogenic.
code given below. Which of the above statements are true?
List I List II (a) Both statements 1 and 2 are true
(Water Quality) (pH Value) (b) Only statement 1 is true
(c) Only statement 2 is true
A. Neutral I. 5 (d) Both statements are false
B. Moderately acidic II. 7 78. Which of the following features are true about Per-
oxyacyl Nitrate (PAN)?
C. Alkaline III. 4 1. PANs are secondary pollutants
D. Injurious IV. 8 2. PANs are produced when hydrocarbon radical
reacts with nitrogen oxide
Codes: 3. PANs cause respiratory diseases in human beings
(a) A–II, B–III, C–I, D–IV 4. PAN causes irritation to the eyes
(b) A–I, B–III, C–I, D–IV Codes:
(c) A–II, B–I, C–IV, D–III

A S S E S S YO U R L E A R N I N G
(a) 1, 2, 3 and 4 are true
(d) A–III, B–I, C–II, D–IV
(b) 2 and 3 are true
70. Bog is a wetland that receives water from (c) 1 and 3 are true
(a) Nearby water bodies (b) Melting (d) Only 1 is true
(c) Rainfall only (d) Sea only 79. In case of fuel, what will be the major difference
71. Which of the following creatures is termed as the rain between BS-IV and BS-VI?
forest of ocean? (a) Sulphur (b) Mercury
(a) Algae (b) Blue whale (c) Methane (d) Carbon
(c) Coral reef (d) Mangrove 80. Statement 1: The aquatic organisms depend on the
72. Which of the following creatures is responsible for the oxygen present in water or dissolved oxygen (DO) for
phenomena of ‘red tide’? their respiratory needs that mainly depends upon on
(a) Coral reef (b) Algae water temperature, the quantity of sediment etc.
(c) Red drum (d) Crocodile Statement 2: Biological Oxygen Demand (BOD) is the
73. Permafrost melt is the main reason for release of a amount of oxygen which is consumed by bacteria while
greenhouse methane gas. In which part of the world decomposing organic matter under aerobic conditions.
is it melting the most? Statement 3: Chemical Oxygen Demand (COD) is the
(a) Himalayan region amount of oxygen required for the chemical oxidation
(b) Arctic region of total organic matter in water.
(c) Antarctic region
Which of the above statements are true?
(d) British Columbia
(a) Only statements 1 and 2 are true
(b) Only statement 2 and 3 are true
(d) Only statement 1 and 3 are true
(a) All the statements 1, 2 and 3 are true

M09_MADAN 07_65901_C09.indd 69 23/12/22 8:01 PM


9.70 Chapter 9

81. Match List I with List II. 85. Which of the following methods can be used to
increase the supply of usable, good-quality water?
List I List II (a) Chlorination (b) Metallurgy
(Disease) (Cause) (c) Cloud seeding (d) None of the above
86. Ozone is considered as a secondary air pollutant
A. Osteoporosis I. Fluorine
because it
B. Leukaemia II. Sr-90 (a) Is in the upper layers of the atmosphere
C. Lung cancer III. Benzopyrene (b) Is formed by chemical reactions in atmosphere
(c) Is produced in industry
Codes: (d) Has a secondary role in causing pollution
(a) A–I, B–III, C–II (b) A–III, B–I, C–II 87. Which of the following statements are true in context
(c) A–I, B–II, C–III (d) A–III, B–II, C–I of photochemical smog?
1. The photochemical smog is formed from reactions
82. Which of the following statements is / are true? occurring between secondary air pollutants and
(a) The decibel (dB) is the unit used to measure the sunlight.
intensity of a sound. 2. The oxides of nitrogen are the main pollutants
(b) The smallest audible sound (near total silence) is released by burning of diesel fuel that results in
0 dB. A sound 10 times more powerful is 10 dB photochemical smog.
but a sound 100 times more powerful than 0 dB is 3. All hydrocarbons, except methane contribute to
20 dB, and 1,000 times more powerful is 30 dB. the production of Photochemical smog.
(c) Any sound above 85 dB can cause hearing loss, 4. This smog is typically worse in the summer as there is
and the loss is related both to the power of the more solar energy to drive photochemical reactions.
sound as well as the length of exposure. (a) 1, 2 and 3 (b) 2, 3 and 4
(d) All of the above statements are true. (c) 1, 2, 3 and 4 (d) 3 and 4
83. Consider the following statements are true in context 88. Which of the following statements are true?
of ozone gas:
1. Chloro Flouoro Carbons release free radical of 1. Carbon dioxide and methane are similar in terms
halogens which reacts with ozone. This results in of contribution to global warming.
depletion of ozone layer and cause health prob- 2. Burning fossil fuels at high temperatures is the
lems, food and water security. main source of pollutants of nitrogen oxides, sul-
2. Surface ozone is formed in transport sector phur oxides and carbon oxides.
when heat and sunlight cause chemical reactions 3. Petrol is mainly responsible for release of lead into
A S S E S S YO U R L E A R N I N G

between oxides of nitrogen (NOX) and Volatile the environment.


Organic Compounds (VOC). (a) 1 ad 2 (b) 2 and 3
3. The ozone produced in transport sector is bad and (c) 1 and 3 (d) 1, 2 and 3
stratospheric is protective in nature. 89. Assertion (A): Sulphur dioxide is a major pollutant
4. Many adverse effects of high UV exposure have causing acid rain. Removal of sulphur from coal can
been avoided thanks to the Montreal Protocol. help in reducing air pollution.
Codes: Reason (R): Air pollution is an increasing problem in
(a) Statements 2, 3 and 4 are correct. developing countries due to burning of more coal to
(b) Statements 1, 3 and 4 are correct. produce electricity, changing lifestyles, unavailability
(c) Statements 1, 2 and 3 are correct. of environment-friendly technologies.
(d) Statements 1, 2, 3 and 4 are correct. Choose the correct answer from the options given
84. Assertion (A): Methaemoglobinaemia is a condition below:
in which blood is not able to carry and deliver enough (a) Both (A) and (R) are true and (R) is the correct
oxygen to the body. (June 2019) explanation of (A).
Reason (R): Consuming drinking water with high (b) Both (A) and (R) are true but (R) is not the correct
nitrate level may cause methaemoglobinaemia. explanation of (A).
Choose the correct answer from the options given (c) (A) is true, but (R) is false.
below: (d) (A) is false, but (R) is true.
(a) Both (A) and (R) are true and (R) is the correct 90. Match List-I with List-II.
explanation of (A).
List-I List-II
(b) Both (A) and (R) are true but (R) is not the correct
Emission of gas Impact on health
explanation of (A).
(c) (A) is true, but (R) is false. A. Carbon 1. Endocrine and immune
(d) (A) is false, but (R) is true. monoxide system

M09_MADAN 07_65901_C09.indd 70 23/12/22 8:01 PM


People, Development and Environment 9.71

B. Nitrogen 2. Pulmonary oedema carbon emissions is


oxide (a) Global warming
(b) Ecological balance
C. Pesticides 3. Affects oxygen carrying (c) Greenhouse effect
capacity to different parts (d) Desertification
of body. 95. Which of the following are the reasons/factors for
D. Phosgene 4. Heart problems, respiratory exposure to benzene pollution?
irritation, bronchitis 1. Automobile exhaust
asthma, carcinogenic issues 2. Tobacco smoke
3. Wood burning
Codes: 4. Using varnished wooden furniture
5. Using products made of polyurethane
(a) A – 3, B – 4, C – 1, D – 2
(b) A – 2, B – 4, C – 1, D – 3 Select the correct answer using the code given below:
(c) A – 3, B – 1, C – 4, D – 2 (a) 1, 2 and 3 only
(d) A – 4, B – 3, C – 2, D – 1 (b) 2 and 4 only
91. Match List-I with List-II. (c) 1, 2, 3, 4 and 5
(d) 1, 3 and 4 only
List-I List-II 96. Assertion (A): Eutrophication is basically flow of
Emission of gas Impact on health nutrient material from land into water bodies. These
A. Ammonia 1. Central Nervous System, materials run off from our farms and cities into rivers,
RBC development lakes, and coastal waters through acid rains and air
born pollutants.
B. Sulphur 2. Severe respiratory Reason (R): These nutrient sources are called non-
dioxide problems, reduce exchange point because they involve widely dispersed activities.
of gases from lung surface They are difficult to be measured and thus regulated
C. Arsenic 3. Damages RBCs and and they vary with the seasons and the weather.
kidneys; causes jaundice (a) Both (A) and (R) are true and (R) is the correct
explanation of (A).
D. Lead 4. Acidification of Water (b) Both (A) and (R) are true but (R) is not the correct
bodies at a high level explanation of (A).
(c) (A) is true, but (R) is false.
Codes: (d) (A) is false, but (R) is true.

A S S E S S YO U R L E A R N I N G
(a) A – 2, B – 4, C – 1, D – 3 97. Which of the following statement/s is/are true?
(b) A – 3, B – 1, C – 4, D – 2
(c) A – 4, B – 2, C – 3, D – 1 Statement 1: Coarse (bigger) particles, called PM10,
(d) A – 4, B – 3, C – 2, D – 1 can irritate your eyes, nose, and throat. Dust from
roads, farms, dry riverbeds, construction sites, and
92. Consider the following statements: mines are types of PM10.
1. Decibels is the unit of measurement of noise pollution.
Statement II: Fine (smaller) particles, called PM2.5,
2. A change from 40 to 80 dB is exactly double in are more dangerous because they can get into the
loudness. deep parts of your lungs — or even into your blood.
3. Zero decibel is the ‘threshold point’ for hearing.
(a) Both statements I and II are true.
4. A jet plane is one of the highest decibel sources of (b) Statement I is true.
noise. (c) Statement II is true.
Which of the above statements are true? (d) Neither statement I nor statement II is true.
(a) 1, 2 and 3 98. Given below are two statements. One is labelled as
(b) 2 and 3 Assertion (a) and the other is labelled as Reason (R).
(c) 1, 3 and 4  [November 2020]
(d) 1, 2, 3 and 4 Assertion (A): Carbon monoxide (CO) is a serious
93. With which of following kinds of pollution is the term asphyxiant; even a short exposure may have fatal
‘green muffler’ associated? health issues.
(a) Water pollution Reason (R): Haemoglobin present in the blood
(b) Air pollution has greater affinity towards carbon monoxide than
(c) Nuclear pollution oxygen.
(d) Noise pollution
(a) Both (A) and (R) are true and (R) is the correct
94. A natural phenomenon that becomes harmful due to explanation of (A).

M09_MADAN 07_65901_C09.indd 71 23/12/22 8:01 PM


9.72 Chapter 9

(b) Both (A) and (R) are true but (R) is not the correct 103. Match List-I with List-II in context of water diseases.
explanation of (A).
(c) (A) is true, but (R) is false. List I List II
(d) (A) is false, but (R) is true. A. Cholera (due 1. Spreads by the faecal-
99. Which of the following statements is NOT true? to bacteria oral route, jaundice and
(a) Water suitable for drinking is called as potable called as ‘vibrio chances of acute liver
water. cholerae’) failure
(b) All sources of water such as the oceans, lakes,
river and underground water together constitute B. Amoebiasis 2. Large intestine and
the hydrosphere. (Traveller’s liver. symptoms include
(c) Of the total fresh water, 69% is available in gla- Diarrhoea) diarrhoea with blood
ciers, 30% underground and less than 1% is and mucus.
located in lakes and rivers. C. Hepatitis A 3. Small intestinal disease
(d) On the planet Earth, the fresh water available - symptoms include
for use amounts is around 4% of the total water diarrhoea and vomiting
found.
100. Statement 1: Radioactive substances is the example Codes:
of non-biodegradable waste that has the potential of (a) A – 1, B – 2, C – 3
polluting the earth to dangerous levels of toxicity if (b) A – 2, B – 1, C – 3
not handled properly. (c) A – 3, B – 2, C –1
Statement 2: Oil spill is an example of non-point (d) A – 3, B – 1, C – 2
source of water pollution. 104. Which of the following statements is not a method of
Codes: purifying water?
(a) Both statements 1 and 2 are true. (a) Reverse osmosis
(b) Only statement 1 is true (b) UV radiation
(c) Only statement 2 is true (c) Distillation
(d) Neither 1 nor 2 is true. (d) Evaporation from a water body
101. Assertion (A): Water logging and soil salinity are 105. Freshwater achieves its greatest density at
the problems that come up when the irrigation is (a) – 4°C (b) 0°C
done without proper drainage of water. (c) 4°C (d) – 2.5°C
Reason (R): Improper drainage leads to water log- 106. Statement 1: The presence of high algal content in
ging in the soil. The salt gets deposited as thin crust on water indicates that the water is acidic.
A S S E S S YO U R L E A R N I N G

the soil surface and starts collecting at the roots of the Statement 2: Cyanobacteria has been accepted as
plants. This salinity affects the growth of the plants. a major indicator of eutrophication in freshwater
(a) Both (a) and (R) are true and (R) is the correct as their blooms are common in waters affected by
explanation of (A). nutrient concentration.
(b) Both (a) and (R) are true but (R) is not the cor- Which of the above statement is/are true?
rect explanation of (A).
(c) (a) is true, but (R) is false. (a) Both statements 1 and 2 are true
(d) (a) is false, but (R) is true. (b) Only statement 1 is true
(c) Only statement 2 is true
102. Assertion (A): Hygroscopic and combined water (d) Neither statement 1 nor statement 2 is true
are of no use to plants.
107. Assertion (A): Fluorosis is a condition that results
Reason (R): Some water in the soil forms an in white or brown speckles on your teeth.
extremely thin tightly held film around the soil par-
ticles. It is called hygroscopic water. In the soil, a Reason (R): Fluorosis is caused by overexposure to
small portion of soil water is chemically bound with fluoride in the early years of life, when your perma-
soil materials. It is called combined water. nent teeth are developing and fluoride concentra-
tion in water exceeds 1.5 mg/l or so.
(a) Both (A) and (R) are true and (R) is the correct
explanation of (A). (a) Both (A) and (R) are true and (R) is the correct
(b) Both (A) and (R) are true but (R) is not the cor- explanation of (A).
rect explanation of (A). (b) Both (A) and (R) are true but (R) is not the correct
(c) (A) is true, but (R) is false. explanation of (A).
(d) (A) is false, but (R) is true. (c) (A) is true, but (R) is false.
(d) (A) is false, but (R) is true.
108. The safe permissible limits of sulphate and chloride
in domestic water supplies, respectively, are

M09_MADAN 07_65901_C09.indd 72 23/12/22 8:01 PM


People, Development and Environment 9.73

(a) 400 mg/l and 1000 mg/l 115. Petrol engines release gaseous oxides of
(b) 200 mg/l and 800 mg/l (a) Sulphur (b) Nitrogen
(c) 800 mg/l and 400 mg/l (c) Phosphorous (d) Carbon
(d) 500 mg/l and 500 mg/l 116. The Agent Orange is covered under the Stockholm
109. Which of the following statements are true in con- Convention. This toxic was sprayed by the US forces
text of acid rain? during Vietnam War to destroy the forest cover.
(a) Acid rain is formed by the combination of chemical Which of the following chemicals is being referred to
air pollutants with atmospheric water droplets. in the above statement?
(b) Acid rain once seeped into ground can dissolve (a) Dioxin (b) Asbestos
nutrients such as magnesium and calcium, that (c) Benzene (d) All of the above
are required by trees to stay healthy. The release
117. The best extinguisher for flammable materials that
of Aluminium into the soil makes it difficult for
takes away the oxygen element of the fire triangle is
trees to take up water.
(c) The normal rain has the pH value between 5.0 (a) Water (b) SO2
and 5.5. The acid rain takes this value to the lower (c) CO2 (d) CO
values of 4 or even less. There is no soil alkalinity. 118. Which of the following is not one of the prime health
(d) All of the above risks associated with greater UV radiation through the
110. Assertion (A): India is water stressed nation as this atmosphere due to depletion of stratospheric ozone?
inhabits 17.1% of the World Population but water (a) Increased skin cancer
availability is only 4% of world water. (b) Reduced immune system
Reason (R): The increased water demand in the cit- (c) Increased liver cancer
ies can be better met by adoption of conservation (d) Damage to eyes
measures. 119. Unburnt carbon particles cause
(a) Both (A) and (R) are true and (R) is the correct (a) Cardiac problem
explanation of (A). (b) Respiratory problems
(b) Both (A) and (R) are true but (R) is not the correct (c) Throat problems
explanation of (A). (d) Skin infection
(c) (A) is true, but (R) is false. 120. Statement 1: Maximum soot is released from ther-
(d) (A) is false, but (R) is true. mal power plants.
111. What is the name of substance whose accumulation Statement 2: The plume rise in a coal-based power
in pelicans of Lake Michigan led to the formation of plant depends on buoyancy, atmospheric stability
thin shells of their eggs? and momentum of exhaust gases.

A S S E S S YO U R L E A R N I N G
(a) CFC (b) PAN
(c) DDT (d) PAC (a) Only statement 1 is true
(b) Only statement 2 is true
112. Assertion (A): In a lake polluted with pesticides, the (c) Neither 1 nor 2 is true
big fish will contain the maximum amounts of pesti- (d) Statements 1 and 2 are true
cides as a result of biomagnification.
121. Which of the following is not one of the major environ-
Reason (R): Biomagnification is the process in which ment problems resulting from human interference in
a harmful chemical enters the food chain and gets the nitrogen cycle?
concentrated at each level in the food chain. (a) Global warming due to release of nitrous oxide
(a) Both (A) and (R) are true and (R) is the correct (b) Acid rain
explanation of (A). (c) Eutrophication
(b) Both (A) and (R) are true but (R) is not the correct (d) Ozone depletion
explanation of (A).
(c) (A) is true, but (R) is false. 122. Which of the following statements are correct in the
(d) (A) is false, but (R) is true. context of carbon monoxide emissions?
1. They are mainly due to incomplete combustion of fuel.
113. The use of which of the following fuels can help in 2. They are more evident in petrol engine.
the reduction of air pollution? 3. Long-term exposure can cause nausea.
(a) Petrol (b) Diesel Codes
(c) CNG (d) Coal
(a) 1 and 2 only (b) 1, 2 and 3
114. Which of the following organisms is the main indica- (c) 2 and 3 only (d) 1 and 3 only
tor for sulphur dioxide and nitrogen pollution?
123. Which of the following body parts is most likely to be
(a) Blue green algae affected by nuclear radiation during the early stages
(b) Lichens of exposure?
(c) E. coli (a) Skin (b) Bones
(d) None of the above (c) Bone marrow (d) None of the above

M09_MADAN 07_65901_C09.indd 73 23/12/22 8:01 PM


9.74 Chapter 9

124. Which of the following terms is used to describe 131. Silicosis is caused in the
the phenomenon of removing carbon dioxide from (a) Textile industry (b) Sugar industry
nature by carbon sinks (natural or artificial)? (c) Stone crushers (d) All of the above
(a) Decomposition (b) Biosequestration
(c) Emission trading (d) None of the above 132. Sewage consists primarily of
1. Water and also contains less than one part of solid
125. Which of the following terms is used to reflect the matter per thousand parts of water.
potential of a greenhouse gas to cause global warming? 2. The solids, can be in the form of either dissolved
(a) Warming potent solids or suspended solids.
(b) Warming potential 3. Normal mixture of solid and gas
(c) Global warming potential 4. Gaseous waste
(d) None of the above
Codes:
126. Which of the following statements are true in the
context of particulate matter? (a) 1, 2 and 3
(b) 2 and 3
1. It is basically a mist, more prevalent during winter (c) 1 and 2
weather. (d) 3 and 4
2. It is more prevalent in diesel engine vehicles.
3. Long-term exposure can harm respiratory track 133. Sustainable waste management is achieved through
and lung functions. A. Training people for required technologies
B. Development of Institutional Framework
Codes: C. Online purchasing
(a) 1 and 2 (b) 1, 2 and 3 D.Formulating laws, policies and rules
(c) 2 and 3 (d) 1 and 3 E. Developing community-based partnerships
127. Match List I with List II. Choose the correct answer from the options given below
(a) C and D
List I List II (b) A, B and E
(Act) (Year) (c) B, C, D and E
A . Water (Prevention and I. 1974 (d) A, B, D and E
Control of Pollution) 134. The duration for which usable compost generation is
B. Air (Prosecution and Control II. 1981 3 to 6 months?
of Pollution) (a) Vermicomposting
C. Wildlife Protection Act III. 1972 (b) Semi-automatic waste converter
(c) Pit composting
A S S E S S YO U R L E A R N I N G

D. Environment Protection Act IV. 1986 (d) Automatic waste converter


Codes: 135. High hazard waste may contain [December 2020]
(a) A–I, B–II, C–III, D–IV (a) Pathogens
(b) A–I, B–III, C–II, D–IV (b) Radioactive waste
(c) A–IV, B–II, C–III, D–I (c) Non reactive
(d) A–IV, B–III, C–II, D–I (d) Corrosive
128. Which of the following elements is deposited into 136. Municipal Solid Waste is so linked with our routine
aquatic and terrestrial ecosystem when used in life, it mainly consists of
intensive agriculture practices? (i) Households waste
(ii) Sanitation residue
(a) Nitrogen (b) Phosphorus
(iii)Waste from Street
(c) Sulphur (d) None of these
(iv) Construction linked material and debris
129. Discharge of industrial wastewater causes all of the
following except Choose the correct answer from the code below.
(a) Depletion of dissolved oxygen (a) (i),(ii) and (iii)
(b) Destruction of aquatic life (b) (i) and (ii)
(c) Change in climate (c) (i), (iii) and (iv)
(d) Impairment of biological activity (d) (i), (ii), (iii) and (iv)
130. Which of the following is not recommended for man- 137. Landfill Gas is a natural byproduct of the decomposi-
agement of plastic waste? tion of organic material in landfills. This gas consists of
(a) Incineration (a) Almost 50 percent methane as the primary
(b) Deep burial component
(c) Autoclave/hydroclave (b) Almost 50 percent carbon dioxide
(d) All of the above (c) A small amount of non-methane organic compounds
(d) All of the above

M09_MADAN 07_65901_C09.indd 74 23/12/22 8:01 PM


People, Development and Environment 9.75

138. Which of the following are true about floatation (c) 1 and 3
method for waste management? (d) 1 and 2
A. This is a water treatment process that clarifies 144. Match the columns.
wastewaters or other waters by the removal of
suspended matter such as oil or solids. Treatment of Brief description
B. The removal is achieved by dissolving air in the liquid waste
water or wastewater under pressure and then A. Primary 1. Sludge–the use of
releasing the air at atmospheric pressure in a Treatment anaerobic bacteria to
flotation tank basin. degrade the biodegradable
C. Flotation is a separation technique that employs
the use of gas bubbles as a transport medium. B. Secondary 2. Disinfecting process for
(a) A, B and C (b) B and C Treatment removal of bacteria and any
(c) A and C (d) A and B dissolved organic solids
139. Which of the following termed is used for waste C. Tertiary 3. To separate the solids
management by oxidation, burning or thermal treat- Treatment
ment of the waste?
(a) Floatation (b) Sledging Codes:
(c) Composting (d) Incineration (a) A – 3, B – 2, C – 1
140. Biogas is a mixture of methane, CO2 and small quan- (b) A – 3, B – 1, C – 2
tities of other gases produced by anaerobic digestion (c) A – 2, B – 3, C – 1
of organic matter in an oxygen-free environment. (d) A – 1, B – 2, C –3
This is called as 145. Vermicomposting is a method of composting that
(a) Pelletization involves
(b) Bio Methanation
(a) Silkworms (b) Earthworms
(c) Pyrolysis
(c) Bacteria (d) None of these
(d) Leachate
146. Which of the following pairs of health problem and
141. This is a waste to energy recovery method. This is the
its causing pollutant is not correctly matched?
process of chemically decomposing organic materi-
als such as plastics and rubber at elevated tempera- (a) Nervous system—adrin
ture in the absence of oxygen. Which of the following (b) Neurological disorders—pesticides
terms is being referred here? (c) Loss of consciousness—oxides of nitrogen
(d) Skin cancer—PAN
(a) Pelletization

A S S E S S YO U R L E A R N I N G
(b) Bio Methanation 147. What are the three main components of the water
(c) Pyrolysis cycle?
(d) Leachate (a) Evaporation, sublimation, solidification
142. Assertion (A): The best way to dispose plant waste (b) Evaporation, condensation, precipitation
is composting (c) Rain, snow, sleet
(d) Liquid, solid, gas
Reason (R): The process of converting wet waste
into manure is called as composting. 148. Solid waste treatment by pyrolysis refers to
(a) Both (A) and (R) are true and (R) is the correct (a) Heating in the absence of air
explanation of (A). (b) No heating
(b) Both (A) and (R) are true but (R) is not the correct (c) Heating in the presence of air
explanation of (A). (d) Treating with chemicals before heating
(c) (A) is true, but (R) is false. 149. Which of the following results in bioaccumulation
(d) (A) is false, but (R) is true. and contamination of food chains?
143. Which of the following statements are true in con- (a) Pesticides
text of sludge in waste-water treatment? (b) Polychlorinated biphenyls
1. Sludge management is one of the easiest tasks of (c) PAN
wastewater treatment plants. (d) All of the above
2. There is high water content and poor dewaterabil- 150. Pollutants in the soil can be broken by
ity in sludge management. microorganisms.
3. There are usually strict regulations for sludge
reuse or disposal. This process is called
Codes: (a) Probiotics
(b) Bioremediation
(a) 1, 2 and 3 (c) Bioaugmentation
(b) 2 and 3 (d) None of the above

M09_MADAN 07_65901_C09.indd 75 23/12/22 8:01 PM


9.76 Chapter 9

151. Match the following: Codes:


List I List II (a) A – 1, B – 2, C – 4, D – 3
(Disease) (Causes) (b) A – 4, B – 1, C – 2, D – 3
(c) A – 1, B – 4, C – 2, D – 3
A. Black foot I. Arsenic (d) A – 1, B – 4, C – 3, D – 2
B. Pulmonary oedema II. Nitrogen oxides 157. Stockholm Convention is a global treaty to protect
environment and human health from
C. Hay fever III. Allergy (a) Greenhouse gases
Codes: (b) Persistent organic pollutants
(c) Hospital-acquired Infections
(a) A–II, B–I, C–III (d) None of the above
(b) A–I, B–II, C–III
(c) A–III, B-II, C–I 158. The pesticide directly affecting the nervous system is
(d) A–II, B–III, C–I (a) DDT
152. Which of the following is considered as the common (b) Aldrin
indicator organism of water pollution? (c) Organic phosphates
(d) None of the above
(a) Coral reefs
(b) Plasmodium 159. The process of dumping solid waste in a scientifically
(c) Escherichia coli designated land area is called
(d) None of the above (a) Dumping (b) Waste disposal
153. The process of conversion of solid waste into organic (c) Sanitary landfill (d) None of the above
fertilizers by making use of microorganisms, such as 160. Consider the following statements:
bacteria and fungi, is called 1. The main source of water pollution is sewage
(a) Disposal water.
(b) Remediation 2. Microorganisms ultimately causes oxygen deple-
(c) Composting tion in water bodies.
(d) None of the above 3. Cholera is a water-borne disease.
154. Which of the following statement(s) is/are correct in Which of the above are true?
context of artificial rain? (a) 1 and 2
(a) Artificial rain is called as cloud seeding. (b) 2 and 3
(b) Artificial rain precipitates the clouds. (c) 1 and 3
(c) The most common chemicals used for artificial
A S S E S S YO U R L E A R N I N G

(d) 1, 2 and 3
rain are silver iodide, potassium iodide and dry ice.
(d) All the above 161. Assertion (A): Animal dung is biodegradable
155. Which of the following pairs regarding typical com- Reason (R): The best solution to get rid of non-bio-
position of hospitals wastes is incorrect? degradable waste such as Nylon is recycling.
(a) Both (A) and (R) are true and (R) is the correct
Type of waste %age explanation of (A).
(a) Plastics 9–12 (b) Both (A) and (R) are true but (R) is not the correct
explanation of (A).
(b) Metals 1–2 (c) (A) is true, but (R) is false.
(d) (A) is false, but (R) is true.
(c) Ceramic 8–10
162. Biodegradable bags are the bags that are capable of
(d) Biodegradable 35–40 being decomposed by bacteria or other living organ-
isms. They are basically made of
156. Match List-I with List-II in context of medical waste. (a) Normal plastic
List-I (Types) List-II (Examples) (b) Synthetic fibres
(c) Starch
A. Infectious 1. body parts, medical (d) Petroleum
devices 163. With reference to Neem tree, consider the following
B. Pathological 2. needles and syringes statements:
1. Neem oil can be used as a pesticide to control
C. Cutter-driller 3. radioactive materials
the proliferation of some species of insects and
D. Miscellaneous 4. diagnostic samples, mites
blood, chemicals 2. Neem seeds are used in the manufacture of biofu-
els and hospital detergents

M09_MADAN 07_65901_C09.indd 76 23/12/22 8:01 PM


People, Development and Environment 9.77

3. Neem oil has applications in the pharmaceutical 164. The United Nations World Water Development
industry Report is UN-Water’s flagship report on water and
Which of the statements given above is/are correct? sanitation issues. Which of the following is the UN
theme for the World Water Day 2022 that is cele-
(a) 1 and 2 only brated on March 22?
(b) 3 only
(c) 1 and 3 only (a) Leaving No One Behind
(d) 1, 2 and 3 (b) Better Water, Better Jobs
(c) Water and Climate Change
(d) Groundwater - Making the Invisible Visible

Exploitation of Natural and Energy Resources


165. Which of the following statements are true? (a) Decaying trees release carbon dioxide.
Statement 1: The major source of energy in a food (b) Living trees remove carbon dioxide from the
chain originates from the sun. atmosphere via photosynthesis.
(c) Burning wood releases carbon dioxide into the
Statement 2: Solar energy is created by nuclear atmosphere.
fusion that takes place in the sun. Fusion occurs (d) All of the above
when protons of hydrogen atoms violently collide in
the sun’s core and fuse to create a helium atom. 171. Assertion (A): Deforestation during the recent
Statement 3: The infrared, visible, and UV waves that decade has led to climate change, soil erosion, land
reach the Earth take part in a process of warming the slide, loss of biodiversity.
planet and making life possible is called as ‘green- Reason (R): The maximum loss of forest lands in
house effect.’ India is caused by agriculture due to increasing pop-
(a) Statement 1, 2 and 3 are true ulations, increased requirement of food production.
(b) Only statements 1 and 2 are true (a) Both (A) and (R) are true and (R) is the correct
(c) Only statements 2 and 3 are true explanation of (A).
(d) Only statements 1 and 3 are true (b) Both (A) and (R) are true but (R) is not the correct
166. The energy source that eventually runs out of stock explanation of (A).
is known as (c) (A) is true, but (R) is false.
(d) (A) is false, but (R) is true.
(a) Renewable resource
(b) Non-renewable resource 172. The tallest trees in the world are found in the

A S S E S S YO U R L E A R N I N G
(c) Primary resource (a) Equatorial region
(d) Secondary resource (b) Temperate region
167. What is the total number of states and UTs in India (c) Monsoon region
which have more than 33% geographical area under (d) Mediterranean region
forest cover?
173. Which of the following statements are correct in
(a) 17 (b) 20 context of forest cover as per Forest Survey of India
(c) 11 (d) 7? Report released in 2022?
168. Which of the following is considered as the best 1. The total forest cover in India is 21.71% of the
quality of coal? total geographical area. India has added 1,540
(a) Peat (b) Lignite square km of forest cover from 2019 to 2021.
(c) Bituminous (d) Anthracite 2. The three top states for most forest cover areas
169. Which of the following statement(s) is/are true as are Madhya Pradesh, Arunachal Pradesh and
per September 2022 report, the contribution of dif- Chhattisgarh.
ferent sources in energy production? 3. The three top states/UTs with the most forest
cover percentage of their geographical area are
(a) The sector wise distribution of fossil fuels used in
Lakshadweep, Mizoram and Andaman & Nicobar
energy production is in the order of coal, lignite,
Islands.
gas and diesel.
4. The three states with an increase in their areas
(b) The sector wise distribution of non-fossil fuels
from 2019 to 2021 are Andhra Pradesh, Telangana
used in energy production is in the order of hydro,
and Odisha.
wind, solar and biomass.
(a) 2, 3 and 4
(c) The energy production by nuclear sector is 1.7%.
(b) 1, 2 and 3
(d) All of the above
(c) 1, 2 and 4
170. Deforestation leads to an increase in atmospheric (d) 1, 2, 3 and 4
carbon dioxide because

M09_MADAN 07_65901_C09.indd 77 23/12/22 8:01 PM


9.78 Chapter 9

174. Match the following: 180. Assertion (A): India despite having large amount of
coal reserves, have to import coal from Indonesia,
List I List II Australia, Russia, South Africa, etc., which account
(Type of Coal) (Carbon Content) for over 80 percent of India’s coal imports.
A. Peat I. 50–60% Reason (R): A large part of Indian coal reserve is sim-
ilar to Gondwana coal. It is of low calorific value and
B. Lignite (brown) II. 70% high ash content.
C. Bituminous III. 80% Choose the correct answer from the options given
below:
D. Anthracite IV. 90–95%
(a) Both (A) and (R) are true and (R) is the correct
Codes: explanation of (A).
(b) Both (A) and (R) are true but (R) is not the correct
(a) A–I, B–II, C–III, D–IV explanation of (A).
(b) A–I, B–III, C–II, D–IV (c) (A) is true, but (R) is false.
(c) A–IV, B–II, C–III, D–I (d) (A) is false, but (R) is true.
(d) A–IV, B–III, C–II, D–I
181. Which of the following is the top state in producing
175. As per the government aims, there will blending of solar energy as per data available in January 2022
following percentage in Ethanol in petrol (EBP) by whose total capacity increased by 10,000 MW?
2025
(a) Gujarat
(a) 10% (b) 20% (b) Rajasthan
(c) 25% (d) 30% (c) Karnataka
176. Consider the following statements for renewable (d) Tamil Naidu
energy. 182. Why the solar energy has become popular source of
(a) Towards mitigation measures, India has added energy in India during recent times?
100 GW of renewable energy that excludes hydro (a) Some states in India have continuous solar
power. radiation.
(b) The renewed and increased total target for renew- (b) The per unit power cost in India has come down as
able energy of India by 2030 is 500 GW. the investment cost has also come down.
(c) By 2030, India wants to achieve target of 40% of (c) The government provides financial and physical
total energy by renewable energy. help to the solar power producers.
(d) All of the above (d) All of the above
177. As per reports from Ministry of New and Renewable
A S S E S S YO U R L E A R N I N G

183. Consider the following statements in context of wind


Energy in September 2022, which of the following energy.
statements are correct in context of India? (a) The country currently has the fourth highest wind
1.India is the world’s third largest producer of renew- installed capacity in the world with total installed
able energy. capacity of 39.25 GW (as on 31st March 2021).
2. At COP–21 in Paris in 2015, India committed to a (b) The Government, through National Institute of
40% share of power generation from non-fossil Wind Energy, has installed over 800 wind-moni-
fuel sources. We have achieved this target a dec- toring stations all over country.
ade ahead of the 2030 timeline. (c) As per data available made available by Ministry
3. India’s massive UJALA LED bulb campaign is reduc- of New and Renewable Energy, Tamil Nadu,
ing emissions by 40 million tonnes annually. Gujarat, Karnataka, Maharashtra and Rajasthan
4. Over 800 biomass power and bagasse/non-bagasse are the top five states in wind energy production.
cogeneration projects have been installed in India (d) All of the above
for feeding power to the grid. 184. Thermal power generation in India is carried out by
Codes: burning
(a) 1, 2, 3 and 4 (a) Natural gas (b) Coal
(b) 2, 3 and 4 (c) Oil (d) All of the above
(c) 1, 3 and 4
(d) 1 and 4 185. Assertion (A): Calorific value is basically about fuel
efficiency. The fuel efficiency is measured by calo-
178. Bitumen is obtained from rific value.
(a) Forests and plants (b) Kerosene oil Reason (R): Calorific value is the amount of heat
(c) Crude oil (d) Underground mines energy present in food or fuel and which is deter-
179. The largest soil group of India is mined by the complete combustion of specified quan-
(a) Red soil (b) Black soil tity at constant pressure and in normal conditions.
(c) Sandy soil (d) Mountain soil The unit of calorific value is kilojoule per kilogram.

M09_MADAN 07_65901_C09.indd 78 23/12/22 8:01 PM


People, Development and Environment 9.79

(a) Both (A) and (R) are true and (R) is the correct 194. Regur soil refers to
explanation of (A). (a) Black cotton soil (b) Laterite soil
(b) Both (A) and (R) are true but (R) is not the correct (c) Desert soil (d) Alluvial soil
explanation of (A).
195. The soil’s fertility can be increased by growing more
(c) (A) is true, but (R) is false.
(d) (A) is false, but (R) is true. (a) Food grains
(b) Leguminous plants
186. Which of the following causes the least pollution
(c) Fibre crops
when burnt?
(d) None of the above
(a) Petrol (b) Diesel
196. Which of the following soils is very hard to cultivate?
(c) Coal (d) Natural gas
(a) Alluvial soil (b) Red soil
187. With the help of photosynthesis, plants convert solar
(c) Cotton soil (d) Sandy soil
energy into
197. Which of the following statements is true about lat-
(a) Chemical energy (b) Mechanical energy
erite soils?
(c) Kinetic energy (d) Nuclear energy
(a) They are formed as a result of leaching.
188. Which of the following statements about the biogas
(b) They are rich in minerals such as aluminium and
is true?
iron.
1. Biogas is a renewable gas. (c) They are found in hot and wet tropical areas.
2. Biogas is produced as a result of anaerobic breakdown (d) All of the above
and fermentation of raw materials such as agricul-
198. Which of the following layers of soil determines its
tural waste, manure, municipal waste, plant mate-
pH value and also its rate of water absorption and
rial, sewage, green waste and food waste.
3. The main constituent of biogas is methane, car- retention?
bon dioxide and hydrogen sulphide. (a) O-horizon (b) A-horizon
(a) 1 and 3 (c) B-horizon (d) C-horizon
(b) 1 and 2 199. Which of the following is measured by the
(c) 2 and 3 porometer?
(d) 1, 2 and 3 (a) Soil fertility (b) Soil salinity
189. The biggest freshwater lake in India is (c) Soil acidity (d) All of the above
(a) Wular Lake (b) Sukhna Lake 200. Which of the following states has forests rich in
(c) Dal Lake (d) Loktak Lake sandalwood?
190. What is the correct order where you would find (a) Andhra Pradesh

A S S E S S YO U R L E A R N I N G
stream deposits starting at the headwaters and going (b) Karnataka
towards the mouth across the course of the river? (c) Kerala
(a) Gravel, sand, silt (d) Madhya Pradesh
(b) Sand, silt, gravel 201. Soils in the Mahanadi Delta are less fertile than those
(c) Silt, sand, grave in the Godavari Delta because of
(d) None of the above
(a) Erosion of top soil by annual floods
191. The majority of earth’s water is in the form of (b) Inundation of land by sea water
(a) Oceans (c) Traditional agriculture practices
(b) Atmosphere (d) The derivation of alluvial soil from red soil
(c) Freshwater lakes and glaciers hinterland
(d) Rivers 202. In India, oil is mostly found in
192. The largest thorium reserves in the world are in (a) Anticlines and fault traps
(a) India (b) USA (b) Sedimentary rocks
(c) Australia (d) None of the above (c) Igneous rocks
193. Which of the following statements is not true? (d) None of the above
(a) Uranium is the most widely used fuel by nuclear 203. CNG stands for
power plants for nuclear fission. Nuclear power (a) Compressed natural gasoline
plants use a certain type of uranium—U-235—as (b) Compressed natural gas
fuel because its atoms are easily split apart. (c) Compressed nitrogen gas
(b) The nuclear fuel used in the fast breeder reactor is (d) Calibrated natural gas
thorium. 204. Which of the following are saltwater wetlands?
(c) Thorium is much more abundant in nature than
uranium. (a) Marsh lands (b) Bogs
(d) Thorium is not safer and efficient to mine than (c) Fish ponds (d) Estuaries
uranium.

M09_MADAN 07_65901_C09.indd 79 23/12/22 8:01 PM


9.80 Chapter 9

205. Statement 1: Hydroelectric power is a renewable (a) Wetland


energy source that is produced with the help of (b) Enteric digestion
dams. (c) Fuel gases
Statement 2: Hydroelectric power does not contrib- (d) Permafrost
ute to global warming. 214. Which of the following statements are true in con-
(a) Both statements 1 and 2 are true. text of methane?
(b) Only statement 1 is true. 1. Methane is a hydrocarbon that is a primary gas as
(c) Only statement 2 is true. well as a greenhouse gas.
(d) Neither statement 1 nor statement 2 is true. 2. Methane is emitted from a variety of anthropo-
206. On which principle does a hydroelectric power plant genic and natural sources.
work? 3. Methane is 25 times less potent as carbon dioxide
at trapping heat in the atmosphere.
(a) Law of conservation of energy (a) 1 and 2
(b) The conversion of potential energy into kinetic (b) 1 and 3
energy (c) 1, 2 and 3
(c) The conversion of mechanical energy into electri- (d) 2 and 3
cal energy
(d) All of the above 215. Which of the following is a renewable source of
energy?
207. Biodiesel is produced in India presently from
(a) Uranium (b) Petroleum
(a) Calotropis (b) Catharanthus (c) Coal (d) Biomass
(c) Jatropha (d) Delonix
216. Which of the following elements is used in the mak-
208. For harnessing ocean thermal energy, the tempera- ing of solar cells?
ture difference between water at the surface and
water at depths up to 2 km should be more than (a) Platinum (b) Carbon
(c) Silicon (d) Silver
(a) 5°C (b) 10°C
(c) 15°C (d) 20°C 217. The metal used in a solar panel is
209. Which of the following is not a biomass energy (a) Gold (b) Copper
source? (c) Silver (d) Nickel
(a) Wood (b) Gobar gas 218. Which of the following is a renewable resource?
(c) Nuclear energy (d) Biogas (a) Natural gas (b) Petroleum
210. The production of nuclear energy (c) Groundwater (d) Coal
219. Which of the chemical substances released into the
A S S E S S YO U R L E A R N I N G

(a) Follows Einstein’s principle of conversion of mass


into energy environment while burning of fossil fuels can lead to
(b) Is not ultimately derived from the sun’s energy acid rain?
(c) Both (a) and (b) (a) Oxides of sulphur (b) Oxides of carbon
(d) None of the above (c) Oxides of nitrogen (d) All of the above
211. Which of the following is the list of nations in 220. Which of the following is not a biomass source?
decreasing order in harnessing wind energy? (a) Gobar gas (b) Coal
(a) USA, Germany, India, Japan and Spain (c) Wood (d) Nuclear energy
(b) China, USA, Germany, India and Spain 221. The energy which is not derived from the sun is
(c) USA, Germany, China, India and Spain
(a) Biomass (b) Fossil fuels
(d) China, USA, Denmark, Germany and India
(c) Nuclear energy (d) Geothermal energy
212. Which of the following statements are true about
222. Harmful radiation emitted by the sun is
shale gas?
(a) Visible (b) Infrared
(a) Shale gas is basically entrapped in sedimentary
(c) Ultraviolet (d) Radio waves
rocks.
(b) The technology of hydraulic fracturing or fracking 223. Fuel formed under the earth’s surface by the decom-
is used in shale gas exploitation. position of organic matter is called
(c) USA is the major producer of shale gas. (a) Fossil fuel
(d) All of the above (b) Inorganic fuel
213. The largest natural sources for methane can be iden- (c) Biogas
tified as which emit the gas from microorganisms (d) None of the above
(bacteria) that decompose organic materials in the
absence of oxygen. The source is

M09_MADAN 07_65901_C09.indd 80 23/12/22 8:01 PM


People, Development and Environment 9.81

224. The main constituent of LPG is 229. A solar cell converts


(a) Methane (b) Butane (a) Heat energy into electrical energy
(c) Hydrogen (d) Propane (b) Solar energy into electrical energy
225. The main constituent of Compressed Natural Gas is (c) Heat energy into light energy
(d) Solar energy into light energy
(a) Methane (b) Butane
(c) Ethane (d) Propane 230. Which of the following sources of energy makes use
of floating generators for its exploitation?
226. Which of the following is not a renewable source of
energy? (a) Tidal energy (b) Wave energy
(c) Wind energy (d) OTEC power plant
(a) Solar energy (b) Natural gas
(c) Wind energy (d) Ocean tidal energy 231. Which of the following terms is used for the mol-
ten material mixed with gases in the mantle of the
227. Exposure to which of the following radiations can earth?
cause skin problem?
(a) Litho (b) Lava
(a) Infrared (b) Ultraviolet (c) Geyser (d) Magma
(c) Gamma rays (d) None of the above
232. The production of electricity from waste material is
228. Which of following gases has the highest calorific
called
value?
(a) Pyrolysis (b) Landfill
(a) Butane (b) Methane
(c) Dumping (d) None of the above
(c) Ethane (d) Hydrogen

Natural Disasters and Their Mitigation


233. Which of the following is not an example of natural 236. Each increase of one unit on the scale represents a
disaster? 10-fold increase in the magnitude of an earthquake
(a) Tsunami (b) Heat waves on Richter scale. In other words, numbers on the
(c) Nuclear accident (d) Epidemic Richter scale are proportional to the common (base
234. Which of the following statement/s is/are true in 10) logarithms of maximum wave amplitudes. If we
context of earthquake and tsunami? compare energy level of two earthquakes measuring
4.0 and 5.0, how many more times of energy will be
1. The earthquake is mainly caused due to collision
released by the earthquake measuring 5.0 in com-
of tectonic plates
parison to an earthquake measuring 4.0?
2. The immediate energy source for earthquakes is

A S S E S S YO U R L E A R N I N G
stored elastic energy in bent rock (a) 10 times
3. Tsunami means series of waves. It is basically a (b) 100 times
strong earthquake or landslide in the sea. (c) 1.25 times
4. Japan is known for its frequent earthquakes (d) 31 times
(a) 1, 2 and 3 237. An earthquake is rated as ‘major’ if its magnitude in
(b) 2, 3 and 4 Richter Scale is in the range of [December 2018]
(c) 1, 2, 3 and 4 (a) 4.0–4.9 (b) 6.0–6.9
(d) 1, 2 and 4 (c) 7.0–7.9 (d) 5.0–5.9
235. Which of the following statements are true in con- 238. Which of the following statements are true?
text of an earthquake? 1. The three seismograph stations are needed to locate
1. The amount of ground displacement in an earth- the epicentre of an earthquake.
quake is called as the slip. 2. The ninety percent of earthquakes occurring at plate
2. The point where a movement that triggers an boundaries.
earthquake is called as Focus. 3. Body waves consist of P and S waves
3. The sequence that correctly lists the different 4. In general, the most destructive earthquake waves are
arrivals from first to last is P waves ... S waves ... the surface waves
Surface waves codes:
Codes: (a) 2, 3 and 4
(a) 2 and 3 (b) 1, 2 and 3
(b) 1 and 2 (c) 1, 2, 3 and 4
(c) 1, 2 and 3 (d) 1, 3 and 4
(d) 1 and 3

M09_MADAN 07_65901_C09.indd 81 23/12/22 8:01 PM


9.82 Chapter 9

239. El Niño and La Niña are responsible for producing (a) Primary waves
(a) Changes of opposite direction in global (b) Secondary waves
temperature (c) Surface waves
(b) Precipitation patterns (d) None of the above
(c) Both (a) and (b) 245. Which of the following regions is in the way of high-
(d) None of the above risk zone of earthquakes?
240. Flood is a natural hazard that affects the Indian peo- (a) Central Indian Highland
ple in a big way. The flood is (b) Coastal region
(c) Himalayan region
(a) The building up of large quantities of water (d) Indian desert
(b) Any obstruction in the water flow
(c) Rise in groundwater level 246. The most recurring natural hazard in India is
(d) None of the above (a) Earthquakes (b) Floods
(c) Landslides (d) Volcanoes
241. Which of the following statements are true in con-
text of volcanic eruptions? 247. For which of the following phenomenon is solar
energy primarily responsible?
1. There is explosion or emission of lava, ashes and (a) Precipitation
toxic gases in volcanic eruptions. (b) Wind
2. There is extremely high temperature and pressure (c) Erosion of earth materials
that causes the rock to melt and become liquid (d) All of the above
rock or magma. When a large body of magma has
formed, it rises through the denser rock layers 248. Which of the following can serve as a reliable safety
toward Earth’s surface. hedge against coastal calamities?
3. Pacific Ocean on earth is known as ‘Ring of Fire’ (a) Coral reefs
as it has the earth’s most active volcanoes. (b) Mangroves
4. The most significant volcanic eruptions have been (c) Both (a) and (b)
felt in the form of change in weather. (d) None of the above
codes: 249. The term used in the context of landslides and basi-
cally the downslope movement of materials under
(a) 2, 3 and 4 the influence of gravity is
(b) 1, 2 and 3 (a) Mass wasting
(c) 1, 2, 3 and 4 (b) Mass tracking
(d) 1, 3 and 4 (c) Mass transfer
242. Which of the following tropical cyclone can be (d) None of the above
A S S E S S YO U R L E A R N I N G

termed as the super cyclonic storm that originated 250. The point at which a fault first ruptures in the earth
in area of West Bengal, Odisha, Bangladesh, Sri during earthquake is called
Lanka, Bhutan in May 2020 and caught the speed of (a) Hypocentre
240 Kms per hour? (b) Epicentre
(a) Nisarga (b) Gati (c) Mouth
(c) Amphan (d) Nivar (d) None of the above
243. Match List I with List II. 251. A flash flood differs from a normal flood
(a) By sudden increase in the level of water
List I List II (b) As it is likely to cause much more damage
A. Rio Summit 1 1997 (c) Both (a) and (b)
(d) None of the above
B. Johannesburg Earth Summit 2 1972
252. Which of the following is an effective protection
C. Kyoto Protocol 3 1992 against cyclones and tsunamis?
(a) Shrimp farms
D. Stockholm Conference 4 2002 (b) Building walls
(c) Mangrove forests
Codes: (d) None of the above
A B C D
(a) 3 4 1 2 253. Tidal energy is also a potential renewable energy
(b) 1 2 3 4 resource. Which of the following is the main cause
(c) 4 3 2 1 for the formation of tidal waves?
(d) 2 1 4 3 (a) Gravitational pull of the moon
(b) Gravitational pull of the sun
244. The earthquake waves that have transverse move- (c) Gravitational pull of the sun and the moon
ments are known as (d) None of the above

M09_MADAN 07_65901_C09.indd 82 23/12/22 8:01 PM


People, Development and Environment 9.83

254. Which of the following is a method to prevent flood? Codes:


(a) Levees (a) A–I, B–II, C–III, D–IV
(b) Winding streams (b) A–II, B–I, C–III, D–IV
(c) Efficient sewage systems (c) A–IV, B–II, C–III, D–I
(d) All of the above (d) None of the above
255. Richter scale measures 257. What should you do to prepare for a hurricane?
(a) The number of deaths (a) Prepare a safety route.
(b) The size of its epicentre (b) Arrange for emergency food and water.
(c) The energy released by an earthquake (c) Both (a) and (b)
(d) The body wave count (d) None of the above
256. The almost identical basic phenomenon is known by 258. The main cause(s) that tsunami waves lose some of
different names across countries in the world. its energy as they get closer to the shore is
Match the following: (a) There is lesser depth/room for the tsunami
(b) Buildings on the beach
List I List II
(c) Friction and turbulence from the beach
(Phenomenon) (Country)
(d) None of the above
A. Cyclone I. India 259. The main effect of volcanic eruptions is
B. Hurricane II. USA (a) Flying of ash contents
(b) Mud slides
C. Typhoon III. China (c) Low temperatures on the earth’s surface
D. Willy willy IV. Australia (d) All of the above

Biodiversity and Miscellaneous


260. Biodiversity is described as 264. The Great Indian Bustard bird is found in
(a) The range of different species in an environment (a) Thar Desert of Rajasthan
(b) The seasonal and daily changes in an environment (b) Coastal regions of India
(c) The way species differ from one another (c) Malabar coast
(d) The influence of physical factors on an (d) Delta regions
environment 265. Turpentine oil used in the manufacture of medicines
261. How an organism is suited to live in a particular is obtained from

A S S E S S YO U R L E A R N I N G
place is called (a) Acacia (b) Chir pin
(a) Competition (b) Adaptation (c) Sunflower (d) None of the above
(c) Addition (d) Participation 266. The main characteristic of biodiversity hotspots is/are
262. Which of the following is termed the Tiger State? (a) Threat from human beings
(a) Rajasthan (b) Biogeographical region with a specified percent-
(b) Gujarat age of endemic species
(c) Madhya Pradesh (c) Both (a) and (b)
(d) Jammu and Kashmir (d) Neither (a) nor (b)
263. Match the following: 267. The Sagarmatha National Park has been established
to preserve the ecosystem of which mountain peak?
List I List II (a) Kanchenjunga (b) Mount Everest
(National Parks) (States) (c) Annapurna (d) Dholavira
A. Periyar I. Orissa 268. The species which are especially likely to develop on
biologically isolated areas such as islands are called
B. Nandankanan II. Kerala
(a) Endemic species
C. Corbett National Park III. Rajasthan (b) Extinct
(c) Wild
D. Sariska Tiger Reserve IV. Uttarakhand
(d) None of the above
Codes: 269. The great Indian rhino has its natural home in
(a) A–II, B–III, C–IV, D–III (a) Kaziranga National Park
(b) A–I, B–II, C–IV, D–III (b) Corbett National Park
(c) A–III, B–II, C–I, D–IV (c) Sunderbans
(d) A–I, B–II, C–III, D–IV (d) Kanha National Park

M09_MADAN 07_65901_C09.indd 83 23/12/22 8:01 PM


9.84 Chapter 9

270. The loss of an entire animal species is referred to as D. Nanda Devi National IV. Uttar
(a) Distinction Park Pradesh
(b) Annihilation
(c) Genocide Codes:
(d) Extinction (a) A–I, B–II, C–III, D–IV
271. Match the following: (b) A–I, B–III, C–II, D–IV
(c) A–IV, B–II, C–III, D–I
List I List II (d) A–IV, B–III, C–II, D–I
A. Ozone depletion I. Basel Convention 278. MAB stands for
(a) Man and Biome
B. CO2 reduction II. Kyoto Protocol
(b) Man and Biodiversity
C. Sustainable III. Rio Summit (c) Man and Biosphere
development (d) Man and Biosciences
D. Hazardous waste IV. Montreal Protocol 279. How many biological hotspots are located in India?
(a) 1 (b) 2
Codes: (c) 3 (d) 4
(a) A–IV, B–II, C–III, D–I 280. IUCN stands for
(b) A–IV, B–III, C–II, D–I (a) International Union for Conservation of Nature
(c) A–I, B–II, C–III, D–IV and Natural Resources
(d) A–I, B–III, C–II, D–IV (b) Indian Union for Conservation of Nature and
272. The Lion Tail Macaque is endemic to Natural Resources
(a) Andaman and Nicobar Islands (c) Integrated Union for Conservation of Nature and
(b) Lakshadweep Natural Resources
(c) Nilgiris (d) None of the above
(d) Arunachal Pradesh 281. Eighty per cent of the crocodiles in the world are
273. Which of the following states has maximum of man- found in India. In which of the following states is the
grove cover in India? number of crocodile sanctuaries the highest?
(a) Goa (a) Goa (b) Odisha
(b) West Bengal (c) Andhra Pradesh (d) Tamil Nadu
(c) Orissa 282. Launched in 2005, Indian Rhino Vision 2020 is an
(d) Andaman and Nicobar Islands ambitious effort to attain a wild population of at
A S S E S S YO U R L E A R N I N G

274. Dachigam Sanctuary is associated with conservation least 3000 greater one-horned rhinos.
of Which of the following states in India has the maxi-
(a) Hangul (b) Tiger mum number of ‘one-horned rhinos’?
(c) Lion (d) Rhinoceros (a) Mizoram (b) Assam
275. The most efficient method of biodegradable urban (c) West Bengal (d) Arunachal Pradesh
solid waste management is 283. What does mean “Agenda 21”?
(a) Landfills (b) Pelletization (a) It’s an agreement between 20 developing coun-
(c) Gasification (d) Composting tries of the world on climate change.
276. Wild water buffalo or Asian Buffalo or Indian Buffalo (b) It’s a free trade agreement between 7 developed
is mostly found in countries of the world.
(c) Agenda 21 is a non-binding, voluntarily imple-
(a) Eastern Himalayas (b) Western Ghats mented action plan of the United Nations with
(c) Aravali Hills (d) Satpura Range regard to sustainable development.
277. Match the following: (d) None of the above
284. Minamata Convention on Mercury is a global treaty
List I List II
to protect human health and the environment from
(Sanctuary) (State)
the adverse effects of mercury and its compounds. In
A. Kaziranga National Park I. Assam which of the following years was it signed?
B. Keolado National Park II. Rajasthan (a) 2006
(b) 2013
C. Sunderbans National III. West Bengal (c) 2016
Park (d) 2019

M09_MADAN 07_65901_C09.indd 84 23/12/22 8:01 PM


People, Development and Environment 9.85

285. Which of the following regions has the greatest (a) Corbett National Park
biodiversity? (b) Hemis National Park
(a) Tropical rain forests (c) Silent Valley National Park
(b) Arctic region (d) Dachigam National Park
(c) Sub-tropical region 287. The endangered largest living lemur Idri idri is an
(d) None of the above inhabitant of
286. Which national park is situated at the highest alti- (a) Nepal (b) Pakistan
tude in the country? (c) Madagascar (d) USA

Miscellaneous Questions
288. Which of the following is/are the collaborative pro- 294. Consider the following statements in context of
gram of Reducing Emissions from Deforestation and organic farming.
Forest Degradation (UN-REDD)? 1. Organic food is supposed to be better for human
(a) Food and Agricultural Organization health as it is grown without the use of artificial
(b) United Nations Development Programme fertilizers and pesticides.
(c) United Nations Environment Programme 2. Fossil fertilizers are used instead of chemical fer-
(d) All of the above tilizers and organic fertilizers are used instead of
289. The Indian government has established around 18 insecticides during organic farming.
biosphere reserves in India. Ten of 18 biosphere 3. Sikkim is recognised as the “world’s first organic
reserves are a part of the World Network of Bio- state” by the World Book of Records London.
sphere Reserves, based on the UNESCO’s Man and (a) 1 and 2
the Biosphere (MAB) Programme list. The latest was (b) 2 and 3
added in the year 2016. Which one was that? (c) 1 and 3
(a) Agasthyamala Biosphere Reserve (d) 1, 2 and 3
(b) Seshachalam Hills 295. Which of the following are the main outcomes of Rio
(c) Achanakmar-Amarkantak Biosphere Reserve Summit that was held in 1992?
(d) Great Nicobar 1. Rio Declaration encouraged people to Think
290. Which of the following organizations has categorized global, Act local
wild flora and fauna into eight categories (known as 2. Agenda 21 is a non-binding action plan of the
Red List), such as extinct, critically endangered and United Nations with regard to sustainable
endangered? development.

A S S E S S YO U R L E A R N I N G
(a) International Union for Conservation of Nature 3. The Earth Summit led to the establishment of
and Natural Resources the Convention on Biological Diversity, and
(b) UNESCO the United Nations Framework Convention on
(c) Conference on Biodiversity Climate Change (UNFCCC).
(d) Kyoto Protocol (a) 1 and 2
291. Which of the following objectives of REDD+? (b) 2 and 3
(a) Carbon stocks (c) 1 and 3
(b) Causes of forest degradation (d) 1, 2 and 3
(c) Major carbon sinks 296. Which of the following organizations has promoted
(d) All of the above Reducing Emissions from Deforestation and Forest
292. In which year did India announce a National Action Degradation (REDD+)?
Plan on Climate Change (NAPCC)? (a) International Union for Conservation of Nature
(a) 2008 (b) 2005 (b) United Nations
(c) 2003 (d) 1998 (c) World Bank
293. Which of the following is/are the main policy mea- (d) International Monetary Fund
sures of Hydrogen Mission/Ammonia Mission? 297. The Government of India submitted its Intended
(a) Green hydrogen and green ammonia are envis- Nationally Determined Contribution (INDC) targets
aged to be the future fuels to replace fossil fuels. to United Nations Framework Convention on Cli-
(b) This renewable energy (RE) measure will be the mate Change.
basic ingredient in making green hydrogen to Which of the following statements is true in this
help international commitments for clean energy. context?
(c) India aims to become self-reliant in energy by
2047 with help from Hydrogen Mission. 1. To reduce the emissions intensity of its GDP by
(d) All of the above 33–35% by 2030 from the 2005 level.
2. To achieve about 40% cumulative electric power

M09_MADAN 07_65901_C09.indd 85 23/12/22 8:01 PM


9.86 Chapter 9

installed capacity from non–fossil fuel–based (c) India has promised to cut its emissions to net zero
energy resources by 2030. by 2070.
3. To create an additional carbon sink of 2.5–3 bil- (d) All of the above
lion tons of CO2 equivalent through additional 302. Which of the following statements are true in context
forest and tree cover by 2030. of COP27 meeting held at Sharm el-Sheikh (Egypt)
Codes: in November 2022?
(a) 1 and 2 only (b) 1 and 3 only (a) The concept of ‘loss and damage’ from climate-
(c) 1, 2 and 3 (d) 2 and 3 only induced disasters was accepted that is impor-
tant for small island states and other vulnerable
298. In which of the following launched an initiative nations.
‘Clean Air, Better Life’ in the year 2016? (b) Mitigation was discussed to keep global tempera-
(a) Niti Ayog and CII tures below 2 degrees Celsius.
(b) Niti Ayog and UNEP (c) India wanted to include a commitment to phase
(c) Central pollution control board down all fossil fuels (and not just coal).
(d) UNESCO and WWF (d) All of the above
299. Which of the following statement/s is/are true in 303. Government wants to achieve the target of 100%
context of Mission Bhagirath? electrification of Indian Railways and its NET Zero
(a) Mission Bhagirath is to interlink Bhagirathi River target by the following years, respectively.
with Krishna River. (a) 2024 and 2030
(b) Mission Bhagirath is a safe drinking water for (b) 2030 and 2070
every household. Under the Telangana Drinking (c) both by 2047
Water Supply Project, a mammoth 1.30 lakh km (d) both by 2030
stretch of pipelines would be laid to quench the 304. Global Methane Pledge that is an US-EU led effort to
thirst of Telangana towns and villages apart from cut methane emissions by 2030 was announced dur-
providing water for the industrial needs. ing Glasgow 26 COP Meet. The 100 nations signing
(c) Both a and b the pledge want to cut methane emissions by
(d) Neither a nor b (a) 20%
300. Which of the following statements is/are true (b) 30%
(a) The world’s largest producer of rare earth metals, (c) 45%
which are used mainly in electronics industry, is (d) 100%
China. 305. Which of the following statements about Central
(b) Taiwan makes 65% of the world’s semiconductors Bank Digital Currency that can be linked to SDGs are
and almost 90% of the advanced chips.
A S S E S S YO U R L E A R N I N G

true?
(c) Afghanistan has an estimated 1.4 million tonnes 1. December 1, 2022 onwards, e `-R will be the digi-
of rare earth minerals including lithium (used in tal legal tender currency.
batteries), uranium (used for nuclear fuel). 2. It will be in the same denominations as that of
(d) All of the above current paper currency and coins.
301. Which of the following statements are linked with 3. The transactions will be person to person and per-
Net Zero Target which were stated by India at COP- son to merchant with the help of QR codes.
26 meeting at Glasgow? 4. The first phase is to be implemented by SBI, ICICI
(a) Net Zero is referred to as carbon neutrality. At this Bank, Yes Bank and IDFC Bank in select cities.
stage country’s emissions are compensated by the codes:
absorption and removal of greenhouse gases from
the atmosphere. (a) 1, 2 and 3
(b) The absorption of the emissions can be increased (b) 2, 3 and 4
by creating more carbon sinks, carbon capture (c) 1 and 4
and afforestation. (d) 1, 2, 3 and 4

M09_MADAN 07_65901_C09.indd 86 23/12/22 8:01 PM


People, Development and Environment 9.87

Answer Keys

Concept of People, MDG, SDG, Human-Environment Interaction


1. (a) 2. (d) 3. (a) 4. (a) 5. (a) 6. (a) 7. (b) 8. (a) 9. (a) 10. (a)
11. (a) 12. (a) 13. (a) 14. (a) 15. (a) 16. (c) 17. (a) 18. (a) 19. (b) 20. (a)
21. (d) 22. (a) 23. (d) 24. (a) 25. (a) 26. (a) 27. (c) 28. (d) 29. (d) 30. (a)
31. (c) 32. (a) 33. (a) 34. (b) 35. (a) 36. (b) 37. (d) 38. (d) 39. (d) 40. (d)
41. (c) 42. (c) 43. (c)
Pollution, Pollutants and Waste Management
44. (a) 45. (b) 46. (c) 47. (b) 48. (c) 49. (c) 50. (d) 51. (d) 52. (c) 53. (d)
54. (c) 55. (b) 56. (b) 57. (d) 58. (c) 59. (a) 60. (a) 61. (a) 62. (c) 63. (d)
64. (a) 65. (d) 66. (a) 67. (d) 68. (a) 69. (c) 70. (a) 71. (c) 72. (b) 73. (b)
74. (a) 75. (c) 76. (b) 77. (a) 78. (a) 79. (a) 80. (d) 81. (c) 82. (d) 83. (d)
84. (a) 85. (c) 86. (b) 87. (c) 88. (d) 89. (a) 90. (a) 91. (c) 92. (c) 93. (d)
94. (c) 95. (a) 96. (a) 97. (a) 98. (a) 99. (d) 100. (a) 101. (a) 102. (a) 103. (c)
104. (b) 105. (c) 106. (a) 107. (a) 108. (a) 109. (d) 110. (a) 111. (c) 112. (a) 113. (c)
114. (b) 115. (b) 116. (a) 117. (c) 118. (c) 119. (b) 120. (d) 121. (d) 122. (b) 123. (c)
124. (b) 125. (c) 126. (b) 127. (a) 128. (a) 129. (c) 130. (a) 131. (c) 132. (c) 133. (d)
134. (c) 135. (a) 136. (d) 137. (d) 138. (a) 139. (d) 140. (b) 141. (c) 142. (a) 143. (b)
144. (b) 145. (b) 146. (d) 147. (b) 148. (a) 149. (a) 150. (b) 151. (b) 152. (c) 153. (c)
154. (a) 155. (d) 156. (c) 157. (b) 158. (b) 159. (b) 160. (d) 161. (b) 162. (c) 163. (c)
164. (d)
Exploitation of Natural and Energy Resources
165. (a) 166. (b) 167. (a) 168. (d) 169. (d) 170. (d) 171. (b) 172. (a) 173. (d) 174. (a)
175. (b) 176. (d) 177. (a) 178. (c) 179. (a) 180. (a) 181. (b) 182. (d) 183. (d) 184. (b)
185. (a) 186. (a) 187. (a) 188. (d) 189. (a) 190. (a) 191. (a) 192. (a) 193. (d) 194. (a)
195. (b) 196. (b) 197. (d) 198. (d) 199. (b) 200. (b) 201. (a) 202. (b) 203. (b) 204. (d)
205. (a) 206. (d) 207. (c) 208. (d) 209. (c) 210. (c) 211. (b) 212. (d) 213. (a) 214. (a)

A S S E S S YO U R L E A R N I N G
215. (a) 216. (d) 217. (d) 218. (c) 219. (d) 220. (d) 221. (d) 222. (c) 223. (a) 224. (b)
225. (a) 226. (b) 227. (b) 228. (d) 229. (d) 230. (b) 231. (d) 232. (a)
Natural Disasters and Their Mitigation
233. (c) 234. (c) 235. (c) 236. (d) 237. (c) 238. (c) 239. (c) 240. (a) 241. (c) 242. (c)
243. (a) 244. (b) 245. (c) 246. (d) 247. (b) 248. (b) 249. (a) 250. (a) 251. (c) 252. (c)
253. (c) 254. (d) 255. (c) 256. (a) 257. (c) 258. (a) 259. (a)
Biodiversity
260. (a) 261. (b) 262. (c) 263. (a) 264. (a) 265. (b) 266. (c) 267. (b) 268. (a) 269. (a)
270. (d) 271. (a) 272. (c) 273. (b) 274. (d) 275. (a) 276. (a) 277. (a) 278. (b) 279. (a)
280. (c) 281. (b) 282. (c) 283. (b) 284. (b) 285. (a) 286. (b) 287. (c)
Miscellaneous Questions
288. (d) 289. (a) 290. (a) 291. (d) 292. (a) 293. (d) 294. (d) 295. (d) 296. (a) 297. (c)
298. (a) 299. (c) 300. (d) 301. (d) 302. (d) 303. (a) 304. (b) 305. (d)

M09_MADAN 07_65901_C09.indd 87 23/12/22 8:01 PM


This page is intentionally left blank

M09_MADAN 07_65901_C09.indd 88 23/12/22 8:01 PM


CHAPTER
Higher Education
10 System

01 Institutions of Higher Learning and Education in


Ancient India

02 Evolution of Higher Learning and


Research in Post-Independent India

LEARNING
03 Oriental, Conventional and
OBJECTIVES Non-conventional Learning
Programmes in India

04 Professional, Technical and Skill-Based


Education

05 Value Education and Environmental Education

06 Policies, Governance and Administration

M10_MADAN 07_65901_C10.indd 1 23/12/22 8:05 PM


10.2 Chapter 10

Education is the manifestation of the perfection


Concept Box
already in man.
— Swami Vivekananda 3300 years of Indian Philosophy: 1500 BCe upto
1800 Ce
(a) Vedas and Upanishads (osn vkSj mifu’kn): bce 1500–
Introduction to Higher Education 500: Vedic period (oSfnd dky)
In Chapter 1, we discussed that education is fundamental (b) Jainism Buddhism, Bhagavad Gita, the Manu
for achieving full human potential, developing an equita- Smriti (tSu /keZ, ckS) /keZ, Hkxon xhrk, euq Le`fr) 600–200
ble and just society, and promoting national development. ce:, the rise of the orthodox darshanas during Epic
We discussed about all important aspects of education. period
On higher education, we stand at third position (c) Nagarjuna and the rise of Mahayana Buddhism
after USA and China. We have highly favourable demo- (ukxktqZu vkSj egk;ku ckS) /keZ dk mn;) — 200 ce, Sutra
graphic dividend ratio that belongs to 15–59 years group. period (lw= le; vof/k)
Demographic dividend, as defined by the India has 62.5% (d) Shankaracharya scholastic period and the rise of
of its population in the age group of 15-59 years which is Vedanta (“kadjkpk;Z “kSf{kd dky vkSj osnkar dk mn;): 600 ce
ever increasing and will be at the peak around 2036 when (e) Rise of other Vedantic schools: Visishtadvaita,
it will reach approximately 65%. Dvaita (vU; osnakfrd fo|ky;ksa dk mn;, fof”k’Vk}Sr, }Sr
United Nations Population Fund (UNFPA), is “the eco- vkfn): post-900 ce
nomic growth potential that can result from shifts in a
population’s age structure, mainly when the share of the
working-age population (15 to 64) is larger than the non- Sutra literature followed the Vedic literature. It
working-age share of the population (14 and younger, happened between 600 bc and 200 bce. There was perhaps
and 65 and older)”. greater need of social conduct into existence. Patanjali’s
The rich heritage of ancient and eternal Indian knowl- Yoga, Gautama’s Nyaya (U;k;), and Mimamsa Shastras
edge and thought has been a guiding light for New (eheaklk “kkL=) happened during this period. Sutra literature
Education Policy, 2020. The pursuit of knowledge (jnan can be identified with the Upanishad period. The Sutra
- Kku), wisdom (pragyaa - izKk), vkSj truth (satya - lR;) was period was identical to that of the Upanishad period.
always considered in Indian thought and philosophy as The word Veda is derived from the word ‘Vid’ which means
the highest human goal. ‘knowledge’. Veda is supposed to be boundless because
We are starting our discussion with ancient education knowledge is boundless. The education in the ancient period
in this chapter. was influenced by religious, political or economic factors.
The division of education can be shown in Fig. 10.2.
i nstitutions oF higher learning and The total configuration of ideals, practices and conduct
is called dharma (righteous path).
education in ancient india The knowledge was divided into two streams:
1. Para vidya ( ): The higher knowledge and the
We owe a lot to Indians who taught us how to spiritual wisdom. The supreme objective of any indi-
count without which no worthwhile scientific dis- vidual duty was to achieve his/her expansion into the
covery could have made. Absolute God, a spark of the Divine. Education was to
— Albert Einstein help in this objective.
2. Apara vidya ( ): It deals with lower knowl-
edge and the secular sciences. We gain knowledge
‘Education during the Vedic age was a journey from mortality about physical, materialistic and comfortable life
to immortality, from chaos to spiritual bliss.’ India has a rich here.
and glorious history of pursuit of excellence in education.
The history of education in India is nearly 5000 years old.
Ancient scriptures known as Vedas formed the strong objectiVes oF education in a ncient india
foundations of Indian civilization. It was during this Prof. M. Achyutan mentioned the following three objec-
period that the famed citadels of learning, the Nalanda tives of education:
and the Vikramshila Universities, thrived, attracting seek- 1. The acquisition of knowledge
ers of knowledge from far and wide. It was wholesome 2. Inculcation of social and religious duties
personality arrangement. 3. Promotion of character
In ancient education, there are a variety of authorita-
tive schools of thought. The explanations depend on her- In ancient system, ‘self realisation’ and moksha (eks{k -
meneutic (the way of analysing). salvation of life) were the ultimate objectives of education.
As knowledge is objective and impersonal, so is the Vedic It means liberation from the cycle of death and rebirth
education. So Vedic education is essentially secular in nature. (samsara - lalkj). For this, attaining such a high level of
Let us have a look at the chronology. knowledge is required.

M10_MADAN 07_65901_C10.indd 2 23/12/22 8:05 PM


Higher Education System 10.3

Elementary Secondary Tertiary


education education education

Higher Vocational
education education and training

General Professional Traditional New and emeging

Science Engineering ITI/ITC/Polytechnic Travel and Tourism, Aviation


Major fields Commerce Medicine courses Hospitality, Retail
of study Arts Architecture It/Computer training

Figure 10.1

Other objectives of education have been stated as follows: 2. Education was identified as a process, leading to one’s
inner progression and self-fulfilment.
1. Education was a matter of individual concern. The
3. The teachers would implement the techniques, pro-
focus was on personality development of the students,
cedures and approaches so that contents could be
by taking into account inner and outer perspectives.
understood by the students.
4. The progression means training of his or her mind as
the instrument of acquiring knowledge. This knowl-
Indian Sources of Knowledge edge would improve the creative abilities.
5. The thinking principle was estimated higher than the
subject of thinking. Thus, the primary subject of edu-
cation was the mind itself.
6. Learners identified their duty with devotion to the
Darshan ideal of (Latin expression that means the highest
Vedas Smritis Puranas Gita or ultimate good) of mankind. Human soul was the
Shastras
• Poorv material world.
Mimansa
• RigVeda • Nyay
• YajurVeda Bhagwatam
• Vaisheshik
• SamVeda • Sankhya Vedangas
• AtharvVeda • Yog
• Uttar
Mimansa Itihas
(Brahma • Ramayan
Sutra) • Mahabharat Vyakarna: Nirukta:
Sanskrit Grammar Dictionary
Sanctions of
Vedas
• Sanhita Writings of Writings of
• Brahman Acharyas, Rasik, Bhakt,
Shiksha:
• Aranyak Jagadgurus Saints Chhanda:
How to pronounce
Upanishads Poetic Stanzas
Vedic mantras

Upved
Vedanga • Arthved
• Dhanurved Jyotishya: Kalpa Sutra:
• Gandharvaved Astrology, Types which give
• Ayurved Astronomy concise form of Vedic
religion
Figure 10.2 Indian Sources of Knowledge Figure 10.3 Parts of Vedangas

M10_MADAN 07_65901_C10.indd 3 23/12/22 8:05 PM


10.4 Chapter 10

7. Learners achieved Chitta Vritti Nirodh (fpÙk o`fÙk fujks/k%), 2. Remembered texts or Smriti ( ) (what is remem-
which is the control of mental activities connected bered): The Bhagavat Gita, 18 Puranas, Arthasastra,
with the concrete world. Kama Sutra, Tantras and many others come under the
8. The doctrine of action (deZ) occupies a very significant Smrti Category.
place.
9. Ancient Indian literature refers to vocational education Education Process ( )
also—there were 64 professions or arts which include Two stages are important in the education process:
weaving, dyeing, spinning, art of tanning leather,
manufacture of boats and chariots, the art of training 1. Vidyarambha ( ): Education would start at the
elephants and horses, the art of making jewels and so age of 5.
on. Young men used to work as apprentices under a 2. Upanayana ( ): Formal education ceremony, also
trainer for a number of years and gained expertise in called sacred thread ceremony. In the new home of
their respective professions. Education was free and Guru, a learner had a second birth and was called
provided with boarding and lodging by the trainer. Dvija (f+}t). Dvija means second birth. One birth is
10. The word ‘rta’ in Vedic education has two meanings: by the father and mother, and the other birth is by
natural order and moral order. the spiritual master and Vedic knowledge. It would
11. Design of curriculum (what to teach) was very start at the age between 8 and 12 years. A learner
important. would be called ‘Brahmacharin’. (czãpkfju - celibacy). A
Brahmacharin after finishing his education is eligible
to become a Grihasta (x`gLFk - a householder). ‘Brahm
Sangh (czã la?k)’ was the opportunity for students to
acquire higher knowledge. The boys practiced this cer-
emony at different ages according to their castes.
3. The students had to study in the Gurukul (xq#dqy) for
about 12 years. After that they were allowed to go home,
and the Guru gave them final instructions. This ritual
was called the Samavartan Sanskar (lekorZu laLdkj).
The society and state did not interfere at all or much with
payment of fees. There was concept of working hard by
learners in the form of ‘dignity of labour’. There was concept
of offering some acknowledgement, gift or thanks at the end
of education; that is called ‘gurudakshina’ (xq# nf{k.kk).
Figure 10.4 Ashrama and Sages Performing Yagya As per another perspective, the process of education
involved three basic stages:
Relation of Knowledge with Life 1. Sravana ( ): It refers to acquiring knowledge of
‘shrutis’ by listening.
1. Books were not used as sources of learning. There was
2. Manana ( ): Pupils think, analyse themselves
oral transmission of knowledge, and listening to teachers
about what they heard, assimilate the lessons taught
and meditation during Vedic education. At a later stage,
by their teacher and make their own inferences.
some text education and wider subjects were included.
3. Nididhyasana ( ): comprehension of truth
Thus, acquiring academic knowledge, values, cultures,
and applying it into real life. It is a kind of reflective
norms, morals, principles and ethics was important.
stage.
2. The students would acquire practical knowledge of
the world and society. An attempt was made to make Vedas
the students capable of experiencing the Supreme
Truth themselves. Vedas are perhaps the oldest books written to guide the
3. In the acquisition of education, students were required to mankind.
give up all the material wealth and comforts. Each book has four major kinds of text—the Samhitas
4. During the process of education, students used to stay (lafgrk–mantras and benedictions), the Aranyakas (vj.;d–
in gurukulas and ashrams (Fig. 10.4). They were gen- text on rituals, ceremonies, sacrifices and symbolic sac-
erally located on the banks of the rivers or lakes so life rifices), the Brahmanas (czãkul–commentaries on rituals,
was usually tough. ceremonies and sacrifices) and the Upanishads (texts dis-
5. The students would work for self-purpose and apply cussing meditation, philosophy and spiritual knowledge).
skills and knowledge for the society. The Vedas are written in Vedic Sanskrit.
The Vedic system refers to Vedas. It comprises the
We need to look at the following basic classification: following:
1. Revealed texts or Shruti ( ) (what is heard): 1. Six Vedangas (N% osnkax)
Four Vedas and 108 Upanishads come under the Surti 2. Six darshanas (N% n”kZu)
category. 3. Purana (iqjk.k–bfrgkl)

M10_MADAN 07_65901_C10.indd 4 23/12/22 8:05 PM


Higher Education System 10.5

4. Tarkshastra (rdZ “kkL=) involvement was more in reciting hymns which were
5. Upnishad (mifu’kn) necessary for the Yajna (sacrifice) or other ritualistic
operations. Women sages were called Rishikas. The
Upanishads, Smritis and Puranas all acknowledge the
scholarly women could be named Maitreyi and Gargi.
superiority of Vedas. The Vedas are four in number:
1. Rig Veda ( –the book of mantra): It is the earliest This phase marks another transition from the spirit
work of all Indo-European languages. It is in the praise of Brahmanas to the spirit of the Upanishads. While the
of God. It comprises ‘Plain Living’ and ‘High Thinking’. Brahmanas are called Karma Kanda (deZ dk.M–karma is
Gayatri mantram touches the highest point of knowl- another word for yajna) because of total emphasis on rituals,
edge and sustains human souls to this day. The Rig Veda the Upanishads came to be known as Jñãna Kanda because
is a collection of 1017 hymns, though thousands remain here knowledge becomes primary. Since the shift involved
unexplored. It mentions women Rais that is called Brah- total change in attitude, it needed a link. This phase is called
manavadinis to denote equality between the sexes in the Aranyanka (vj.;ad) because it became relevant when men
field of knowledge. In Rig Veda, a teacher is called rishi. retired to forests due to old age. Why should old people
2. Sama Veda ( –the book of chant): The Sama retire to forests? Surely, this is a provoking question. Forest
Veda is a collection of verses from the Rig Veda for was the dwelling place for tribes. If old people retired to for-
liturgical purposes. Liturgical is the participation of ests, then it must be due to strong affinity to the place of
people in the work of God. origin, which prompted them to choose so.
3. Yajur Veda ( –the book of ritual): It is the col- Hence, proper study of philosophy begins only from
lection of prose mantras or yajurs (sacred incanta- the Upanishads. The Aranyakas constitute a phase in the
tions), though the duty of chanting the hymns on the life of an individual.
occasion of sacrifice was mainly undertaken by the Subjects in Vedic Education
Hotri, the first order of priesthood.
Both Sama Veda and Yajur Veda have Gayatri Mantra It might take 12 years for a student to develop expertise in
in them. one Veda and thereafter, it would be 12, 20, 36 years and
4. Atharva Veda ( –the book of spell): This Veda so on. A graduate was called snatka (Lukrd–graduate)and
is thoroughly secular in character containing a vivid the graduation ceremony was called smavartna (lekorZu).
description of various arts and sciences. Let us have a look at some other subjects:

Rig Veda is also called Rig Veda Samhitaa (samhitaa 1. Itihas (history)
means collection). The institutionalization of rituals 2. Anviksiki [vkUohf{kdh–the science of inquiry/logic–it has
brought into effect two major changes; two types–atma (vkRek–soul) and hetu (gsrq–theory of
Firstly, it brought into existence the gods, with their reasons)].
respective powers. 3. Mimamsa (interpretation)
Secondly, it gave rise to a priestly class. 4. Shilpashastra (f”kYi”kkL=–architecture)
During initial stages of education, memorization of 5. Arthashastra (vFkZ”kkL=–polity)
texts such as Vedas and dharamsastras would take place. 6. Varta (okrkZ–agriculture, trade, commerce, animal
At a later stage, it would be grammar, logic, metaphysics, husbandry)
etc. There were some allied subjects also. 7. Dhanurvidya (/kuqfoZ|k–archery)
Vedic education also included sutras and brahmanism. 8. Krida (ØhM+k physical education—games, recreational
The basic streams were the following: activities)
9. Vyayamaprakara (O;k;ke izdkj–exercises)
1. Pure oral teaching—purely Vedic 10. Dhanurvidya (archery for acquiring martial skills)
2. Thinking —‘chintan’ (fparu) 11. Yogasadhana (;ksxlk/kuk–training the mind and body)
The Brahman Sangh was the set-up through which meri- 12. Shastrartha (“kkL=kFkZ–learned debates)
torious students were given chance to explore their mind. Buddhist Education
The primary subject of education was the mind itself.
Buddha used story telling to explain his point. Non-Vedic philosophy schools reject the authority of the
Upanishads, that is, Jains, Buddhists and Charvakas.
These are the nãstika schools.
About Women Education Buddhism rejected the subjects leading to enrichment
There was a high standard of learning for women also. In of worldly life and stressed the renunciatory aspect of
house, they might learn music and dancing. They had to education.
undergo the Upanayana ceremony.
1. Lord Buddha realized the need of educating devotees
There were two types of educated women:
on a massive scale. Buddhist education started 2600
1. Sadyodwahas ( ): They were people who years ago. It was collective in the sense that it was
pursued studies till their marriages. not based on caste system. The Brahmanism followed
2. Brahmavadinis ( ): They were people who did monocratic policy. Vinaya is the ethical code of the
not marry and pursued studies throughout their lives. Buddha, the rules for monks and the moral advice to
Women used to learn Vedas and Vedangas. Their lay people (Dhamma Vinaya–/kEe fou;).

M10_MADAN 07_65901_C10.indd 5 23/12/22 8:05 PM


10.6 Chapter 10

2. Lord Buddha delivered his first sermon in the village Eightfold Path Principle ( ): The path
of Sarnath, near the city of Benares in Uttar Pradesh. consists of various interconnected activities related to
This event is known as turning of the Wheel of Law knowledge, conduct and meditative practices:
(Dharma-Chakra-Pravartana /keZ pØ ifjorZu). 1. Right view 5. Right livelihood
3. Lord Buddha asked his followers to avoid the worldly
2. Right intention 6. Right mindfulness
extremes of indulgence. He suggested ‘middle path’
(Madhyam Marg) to follow. Everyone was responsible 3. Right speech 7. Right effort
for his/her own happiness in life, stressing on the indi- 4. Right action 8. Right concentration
vidualistic component of Buddhism. The concept of ‘beg- The essence of Buddhism is the attainment of enlighten-
ging’ by disciples was to take out ego from their minds. ment. There is no supreme god or deity in Buddhism.
They were forbidden to accept gold, silver or anything The ultimate goal of Buddha’s teaching was the attain-
precious; it was only for routine things, food, etc. ment of nibbana which was not a place but an experience,
4. A child would start his education at the age of 8 years and could be attained in this life.
after the Prabrajya or Pabbajja ceremony that was Buddha established a code of conduct called five pre-
open to all castes. cepts (pancasil) and suggested to refrain from them:
5. As Buddhism was based on democratic principles, it
• Violence
expanded very fast and many monasteries and viha-
ras were established. At a later stage, many of these • Stealing
monasteries had converted into full-fledged uni- • Sexual misconduct
versities of international importance. The prospec- • Lying or gossip
tive Bhikkhus (Buddhist monks) and Bhikkhunis or • Taking intoxicating substances, for example, drugs or
Upasikas (Buddhist nuns) were the main students. drinks
6. The monks were organized into the Sangha for better Buddhist Councils
management.
7. Its contribution to the art and architecture of India In total, four major Buddhist councils were conducted.
was notable. The stupas at Sanchi, Bharhut and Gaya The First Council (izFke ifj"kn] laxhfr ;k egklHkk) was held
are wonderful pieces of architecture. in 483 bce in the Sattapani cave at Rajgriha. The teach-
8. In the Vedic age the student was given education up ings of Buddha were divided into three Pitakas:
to 25 years of age and after that he was permitted 1. (a)  Vinaya Pitaka ( ): Rules of conduct
to go home and lead the life; in the Buddhist system and discipline applicable to the monastic life of
after having received education the student never the monks and nuns
came back to his parents place for leading the life of (b) Sutta Pitaka ( ): The main teaching or
a householder. He remained a monk for good and cut Dhamma of Buddha (it is divided into five Nikayas
off his worldly relationships forever. or collections)
9. The Buddha’s teaching was oral; his teachings were (c) Abhidamma Pitaka ( ): A philosoph-
written down around 25 bce in Pali. ical analysis and systematization of the teaching
10. Other important Buddhist texts include Divyavadana, and the scholarly activity of the monks
Dipavamsa, Mahavamsa, Milind Panha, etc. 2. The Second Council was conducted in Vaishali.
11. The subjects mentioned in the Jatakas are medicine, 3. The Third Council was conducted in Pataliputra
Vashikaran Kam-Tantra, Tantra-Mantra, archery, under the patronage of Ashoka.
elephant taming, hunting, giving life to the dead, the 4. The Fourth Council was conducted at Kundalvana,
knowledge of the voices of various animals, search of Kashmir.
truth and Nirvana. Buddhism was divided into two sects, namely, Mahayan
12. Some students who could not get mental satisfaction and Hinayana.
even through higher studies used to go to the isolated 1. Mahayana ( –Great Vehicle): It believes in
place of some monk and spent their lives in search of the higher order and idol worship of Buddha. It soon
truth and Nirvana. spread in northern India and then to China, Korea,
13. Gradually gaining spiritual knowledge, they became Tibet and Japan. Zen is also a school of Mahayana.
ascetics in their future life. 2. Hinayana ( –Lesser Vehicle): Also known as
14. Lord Buddha suggested four noble truths (ariya- Abandoned Vehicle or Defective Vehicle. It believes in
sachchani–vfj;k lPpkfu) and Eightfold Path (astangika the original teaching of Buddha or doctrine of elders.
marg–v’Vkafxdk ekxZ). Theravada is a Hinayana sect.
Four noble truths in Buddhism:
In fact, Buddhism is a way of ‘soft diplomacy’ that was
1. Dukkha ( ): Suffering is the essence of the liberal than Vedic approach. It promoted the spread of
world. Indian culture to other parts of Asia.
2. Samudya ( ): Every suffering has a cause. Effective revitalization of the Nalanda University proj-
3. Nirodha ( ): Suffering could be extinguished. ect and encouragement of Buddhist studies in well-estab-
4. Atthanga Magga ( ): It can be achieved by lished universities will bring international community at
following the Eight Fold Path. a common platform.

M10_MADAN 07_65901_C10.indd 6 23/12/22 8:05 PM


Higher Education System 10.7

jainism Ujjain and Nasik developed. The Nalanda University was


The art of writing was known very well in India. In Jainism, known all over Asia for its high standards of scholarship.
works such as Samavaya Sutra (leo; lw=) and Pragnapara The subjects taught included Vedanta, philosophy, study
Sutra (izkx.kijk lw=) with reference to 18 different scripts of the Puranas, epics, grammar, logic, astronomy, phi-
are available. The former refer to 64 types of scripts and losophy, medicine and so forth. Sanskrit, the court lan-
the latter to about a dozen types of scripts. Chinese travel- guage, was the medium of instruction.
ler Hiuen Tsang says that children began by learning the The Jains used Sanskrit literature such as ‘Adipurana’
alphabets. Then they began the study of five subjects— (vkfniqjk.k) and ‘Yashatilaka’ (;'kfryd) for educational
grammar, arts and crafts, medicine, logic and philosophy. purposes in the earlier phase. To enable the individuals to
There are basically three essential aspects in Jainism obtain recognition of education, the medium was changed
to Prakrit and other regional languages such as Tamil and
1. Anekantavada: This can be termed as ‘non–absolutism’. Kannada. Books in the Jain and Buddhist libraries were
Here, we assume that the objects have infinite modes written on palm leaves that were tied together and were
of existence, so only Kevalins can comprehend all known as granthas (xzaFk).
aspects of knowledge. Gradually, Jainism and Buddhism lost royal patronage
2. Syadavada: This is a theory of conditioned predication. and their monasteries started declining as centres of edu-
The objects and situations have a conditional point of cation and learning. The ‘mathas’ supported by Brahmins
view, practically all of our judgements are conditional, were institutes equivalent to Jain and Buddhist monaster-
they depend upon the situation. ies. The functions and tasks implemented in the ‘mathas’
3. Nayavada: This has partial viewpoints which focus for educational purposes were like those of the ashrams.
on describing reality from different points of view.
Post-Gupta Period
Concept Box During the reign of Harsha, art and education gained prom-
inence. He encouraged education at all levels. Education
Languages Used was provided in temples and monasteries. For the acqui-
1. Sanskrit during Vedic education sition of higher education, the universities of Taxila,
2. Prakrit during Jain education Ujjain, Gaya and Nalanda gained prominence. In Nalanda,
3. Pali during Buddhist education Hiuen Tsang spent several years studying Buddhist scrip-
tures. The head of Nalanda University was Shilabhadra, a
There were no caste restrictions during Buddhist and
renowned scholar. In the seventh and eighth centuries, the
Jain periods. That also impacted choice of language.
colleges attached to the temples emerged as new centres
Sanskrit was used all the times.
of learning. They provided Brahmanical education and the
medium of instruction was Sanskrit. Entry to these temple
Education during Mauryan Period colleges was open only for the upper castes.
During the Mauryan and the post-Mauryan periods,
Knowledge
society progressed a lot. There was emergence of urban
life. The guilds of the merchants began to render an Knowledge was imparted orally. The different methods
imperative contribution in making provision of education. used were as follows:
The new areas of education were in metallurgy, mining, 1. Memorization
carpentry, weaving and dyeing. There was focus on 2. Critical analysis
architecture and astronomy. There was development 3. Critical introspection
of medical sciences. Sushruta was famous for surgery. 4. Story telling: Method mainly used by Lord Buddha
Charaka Samhita (pjd lafgrk/lfEgrk) written by Charaka mainly to explain his doctrines
was an accurate and comprehensive work on medicines. 5. Question and answer method: For further probe
Samhita is also used after Rigveda, Samveda, Yajurveda into the discussion
and Atharvaveda (_Xosn, lkeosn, ;tqosZn vkSj vFkoZosn). 6. Hands-on method: For practical and professional
courses such as medical science
Education during Gupta Period 7. Seminars: Through debates and discussions
There was development of Jain and Buddhist systems of
education. The learning was imparted orally and later
literary texts came to be put into practice for acquisition of Stopover
education. The monasteries had libraries, where important
texts could be found. Students from other countries, such as 1. Where and when was the Second Buddhist Council
China and South-East Asia, came to the Buddhist monasteries held?
for education. The maintenance of the monasteries was (a) Pataliputra in 250 bce
normally done by kings and the rich mercantile class. (b) Sri Lanka in the first century bce
Fa-Hien (Qk-fg;ku), a Chinese Buddhist monk, also (c) Vaishali in 383 bce
spent several years in the monastery at Pataliputra, (d) Mandalay in 1871 ce
studying Buddhist religious books. Varanasi, Mathura, The correct option is (c).

M10_MADAN 07_65901_C10.indd 7 23/12/22 8:05 PM


10.8 Chapter 10

2. We can know about the early Vedic period from 7. Agraharas ( ): They were settlements of Brahmins
(a) Archaeological excavations in ­villages where they used to teach.
(b) The Rig Veda 8. Mathas ( ): They were mainly for residing and
(c) Jatak Katha receiving religious and secular instructions. These
(d) Contemporary culture mathas belonged to both Shaiva and Vaishnava
The correct option is (b). sects and were normally attached to some temple
associations.
9. Brahmapuri ( ): It was a settlement of learned
Types of Teachers Brahmins in towns and cities or in any selected area
• Acharya ( ): A teacher to teach Vedas without for education purpose.
charging fee from the students 10. Vihara ( ): It was a Buddhist monastery where all
• Upadhyaya ( ): To earn his livelihood and Buddhist preaching and philosophy were taught.
taught only a portion of the Veda or Vedangas
• Charakas ( ): Wandering scholars to visit the Main Educational Institutions of Higher
nation for higher knowledge, usually regarded as pos- Education during A ncient India
sible source of knowledge by Satapatha Brahmana
(Hiuen Tsang gained the knowledge this way) Ancient Universities ( )
• Guru ( ): Used to lead a grihastha life by imparting The universities of ancient India have a prouder history
education and by maintaining his family than that of their counterparts in the ancient Western
• Yaujanasatika ( ): Famous for their pro- world. At least one of them, viz., Taxila, flourished several
found scholarship, students from distant places would centuries before the universities of Alexandria, Athens
visit them to seek education and Constantinople.
• Shikshika ( ): Instruction in arts such as dancing
1. Taxila University ( )
Educational Institutions • Taxila (Takshashila) was probably the first uni-
versity in the world. It flourished from 600 bce to
1. Gurukul ( ): This was the house of the teacher 500 ce, in the kingdom of Gandhar. Takshashila
who was a settled householder. was thus the intellectual capital of India, a cen-
2. Parishads ( ): Here, the students usually settled tral university that exercised suzerainty over the
for higher education; Parishads originally used to world of letters in India.
have 3 Brahmins. The number gradually increased, • It was not a properly centralized university.
and a Parishad even consisted of 20 Brahmins who • The university was popular for Brahmanical and
were well versed in philosophy, theology and law. Buddhist education. Taxila is also described in some
Sangam was also such a Parishad during the first detail in the Buddhist Jãtaka tales (trkdk dFkk,a).
century ce in Tamilnadu; here some works were • The university taught 68 subjects such as Vedas,
submitted for criticism also. These gatherings were languages, grammar, philosophy, medicine,
patronized by kings. surgery, archery, politics, warfare, astronomy,
3. Goshthi ( ): This conference was a national gath- accounts, commerce, documentation, music,
ering summoned by a great king in which represen- dance and other performing arts, futurology, the
tatives of various schools were invited to meet and occult and mystical sciences, and complex math-
exchange their views. ematical calculations.
4. Ashramas ( ): This hermitage was another cen- • The minimum entry age was 16 years. It attracted
tre where students from distant and different parts students from Babylon, Greece, Syria and China.
of the country flocked together for learning around • There was no exam system.
famous sages and saints, for example, the ashrama • Taxila was influenced by the Greek culture to an
of Bharadwaj at Prayag. A full-fledged ashram a is extent.
described as consisting of several departments such • This university was well known for its specialized
as the following: training on the subject of Tantra (Tantrism).
1. Agnisthana (vfXuLFkku): for fire worship and prayers • A wide range of subjects were taught by experi-
2. Brahmasthana (czãLFkku): for Vedas enced masters: Vedas, language, grammar, phi-
3. Vishnusthana (fo".kqLFkku): for teaching Raja Niti, losophy, medicine, surgery, archery, politics,
Arthaniti, Vartta, etc. warfare, astronomy, astrology, accounts, com-
5. Vidyapeetha ( ): This was an educational insti- merce, futurology, documentation, occult, music,
tution for spiritual aspects started by the great Acharya. dance, etc.
Sri Shankara started such institutions at Sringeri,
Kanchi, Dwarka, Puri and Badri. The following were its main scholars:
6. Ghathikas ( ): Here, both the teachers and (a) Kautilya ( ): The author of ‘Arthashastra’
the pupils met and discussed. The cultured scholars (vFkZ'kkL=) on statecraft, economic policy and
would meet, discuss and clash also. In South India, military strategy, written in Sanskrit (Kautilya
colleges called as ghathikas emerged from temples. is also identified as Vishnugupta and Chanakya;

M10_MADAN 07_65901_C10.indd 8 23/12/22 8:05 PM


Higher Education System 10.9

he was the teacher and guardian of Emperor • Nalanda made unique contributions to the evolu-
Chandragupta Maurya) tion, expansion and refinement of Indian culture.
(b) Panini ( ): Grammarian for Sanskrit • 
Bakhtiyar Khilji put the university towards
(c) Charak ( ): Name associated with Charaka destruction by the end of the twelfth century ce.
Samhita, which was on ancient Indian medicine
that is ayurveda
(d) Vishnu Sharma ( ): Author and compiler
of the Panchtantra
2. Nalanda University ( )
A historian writes, ‘The University of Nalanda was the
educational center of international moral comparable
in the universalism of its thought, the wide range of
its studies, the international character of its commu-
nity to the greatest universities of modern time like
Oxford, Cambridge, Paris and Harvard.’
•  The ancient Nalanda University was located
near Rajgriha in Bihar; it has been the birth
place of Sariputta, a favourite disciple of Lord
Buddha, who was closely linked with Mahayana
(Fig. 10.5). Figure 10.5 Ancient Nalanda University
• The university was a Buddhist centre of learning
(Mahayana Vehicle) from 427 to 1197 ce. Its 3. Valabhi University ( )
campus was 1 mile in length and 0.5 mile in width. •  Vallabhi University was an important center of
• Sheelbhadra ('khyHknz) was the kulpati or chancellor Buddhist learning and championed the cause of
of the university. He had studied all Sutras and Hinayana Buddhism between 600 ce and 1400 ce.
Shastras books. Chinese travellers Hiuen Tsang and Itsing who
• There was Dwar Pandi, a teacher who was the visited this university during the seventh century
in-charge for admission to the university. describes it as a great centre of learning—as
• The university was famous for 8 big halls named glorious as Nalanda.
smgrhma (lexze) and the 300 study chambers. • This university was not just a centre of religious
• It is stated that at one time there were 10,000 education but other secular subjects such as
monks staying at Nalanda. Hien Tsang wrote in Arthashastra (economics), Niti Shastra (law) and
his diary that 10,000 students and 200 profes- Chikitsa Sastra (medicine) were also taught here.
sors were at the Nalanda University. Hiuen Tsang • Though this university championed the cause of
came here in the seventh century ce. Hinayana Buddhism, Brahminical sciences were
• This university was also renowned for its cosmo- also taught here.
politan and catholic character. It was famous for • Valabhi was running in good financial position
its faculty of Logic. till 755 ce but some portions were destroyed due
• The admission criteria were tough. The entrance to Arab invasion. It still continued till the twelfth
examination was very difficult and the pass rate century.
was 3 out of every 10 students. • Gunamati and Sthirmati (xq.kefr vkSj fLFkjefr) were the
• The minimum age limit was 20 years for admis- two famous scholars graduating from this university.
sion into the university. • In September 2017, the Government of India had
• Many facilities were being offered free of cost. plans to revive this university.
• There were three methods of teaching, namely, 4. Vikramshila University ( –
(i) verbal and explanatory, (ii)  lectures and 800–1023 ce)
(iii) d
 ebates and discussions. • Vikramshila Vihara or University (Buddhist mon-
• It has boasted a massive library called Dharma astery) was set up by Emperor Dharampala of Pala
Gunj or Mountain of Knowledge. It had nine sto- dynasty in the eighth century era in Bhagalpur
reys. The library had three departments known as Northern on the banks of the river Ganges.
‘Ratna Sagar’, Ratnodavi and Ratnayanjak. •  The buildings were well planned and
• The university had a towering observatory called accommodative.
the Ambudharaavlehi for astronomical research. • The university was famous for religious teach-
• The university progressed a lot during Gupta ings. It actually represented Mahayana form of
dynasty. Buddhism but important branches of Hinduism
• In year 2010, a new Nalanda University was set were also taught. The courses taught in Hindu
up in Bihar as a Central University with help from institutions covered subjects such as the 14 Vidyas,
Japan, China, Thailand, Laos, Singapore and the 18 Silpas and the 64 arts which embrace all
Australia by various manners. the knowledge necessary for a householder.

M10_MADAN 07_65901_C10.indd 9 23/12/22 8:05 PM


10.10 Chapter 10

• The admission was allowed to only those who   Famous poets Vidyapati, who had written in Hindi,
wanted to become monks. and Jaideo, a prominent poet of Sanskrit literature,
• Vikramashila appears to have had a more clearly were born here.
delineated hierarchy than other mahaviharas, as   From the twelfth to the fifteenth century, besides
follows: literature and fine arts, scientific subjects were also
– Abbot (Adhyaks.a– v/;{k) taught there.
– Six gate protectors or gate scholars (Dvārapāla • There was a Nyaya Shastra and Tarka Shastra.
or Dvārapan.d.ita }kjiky ;k }kj-iafMr), one each • Gangesha Upadhyaya founded a school of New
for the Eastern, Western, First Central, Second Logic (Nvaya Nyaya–uO; U;k;).
Central, Northern and Southern Gates • An epoch-making work named Tattva Chintamani
– Great scholars (Mahapandita–egkiafMr) (rÙo fparkef.k) had been written.
– Scholars (Pan.d.ita), roughly 108 in number Mithila produced a number of other scholars and lit-
– Professors or teachers (Upādhyāya or Āchārya), erary celebrities.
roughly 160 in number including pan.d.its Even up to Emperor Akbar, it continued to flourish as
– Resident monks (bhiks.u–fHk{kq), roughly 1000 in an important centre of education and culture.
number 8. Nadia University ( ): Situated at the
• The university was famous for its Tibet connection. confluence of Ganga and Jalangi rivers in Bengal, it was
• The university was later organized into six col- formerly called Navadweep. Education in Nadia Univer-
leges. The central building was called the Vigyan sity was imparted at three centres, namely, Navadweep,
Bhawan. Shantipur and Gopaalpura.
•  Mahasthavir was the highest authority of the • The lyrics of Gita Govind (xhr xksfoan) by Jaideva
university, being known as the Kulpati of the reverberated here.
Gurukula. • A school of logic owed its existence to Raghunatha
• The main subjects of study were Vyakaran, Logic, Shiromani.
Philosophy, Tantra Shastra and Karamkanda. • Learning and efficiency in discussions was consid-
• Tantra Shastra (ra= 'kkL=–Tantric Buddhism) gained ered to be an essential qualification of a teacher of
prominence – at a later stage. this university.
• Degrees were conferred on the graduates and 9. Ujjain University ( ): It was famous
post-graduates at the time of samavartana (con- for its secular learning, including mathematics and
vocation) by the rulers of Bengal. astronomy.
• The university was destroyed by Bakhtiyar Khilji 10. Salotgi ( ): It was an important centre of learn-
in 1203 ce. ing in Karnataka. It had 27 hostels for its students
• The Archaeological Survey of India (ASI) has who hailed from different provinces. This college was
chosen the ancient Vikramshila University in richly endowed in 945 ce by Narayana, the minister
Bhagalpur to conserve and develop on interna- of Krishna III, with the revenues of houses, land and
tional lines. levies on marriages and other ceremonies.
5. Odantapuri University ( ): This 11. Ennayiram ( ) in Tamil Nadu provided free
university had been established long before the Kings boarding and tuition to 340 students. Other impor-
of Pala dynasty came into power in Magadha. tant centre of learning in South India were Sringeri
  Odantapuri could not attain that level of fame and Kanchi.
and repute which either Nalanda or Vikramshila had
Note: The universities at Takshashila, Nalanda, Valabhi,
accomplished.
Vikramshila, Odantapuri and Jagaddala developed in con-
  Nearly 1000 monks and students resided and
nection with the viharas (mainly Buddhism). The univer-
received education there. The university attracted
sities at Benaras, Navadeep and Kanchi developed in con-
students from Tibet also.
nection with temples and became centres of community
6. Jagaddala University ( ): Pal King,
life in the places where they were situated.
Raja Ram Pal of Bengal constructed a monastery and
named it Jagaddala. It remained a centre of Buddhist
education for about 100 years. It was again destroyed Few of our ancient scholars
during the invasion in 1203 ce. 1. Maharishi Charaka ( ): Living time between
  In Jagaddala, there were many scholars notable ad 150-200–100 bc)–editor of the medical treatise
for their knowledge. The books were translated in Charaka Samhita (pjd lafgrk) that was composed by
Tibetan language. Agnivesa (vfXuos'k) and later edited by Charaka (pjd).
7. Mithila University ( ): In the This is part with Brhat-Trayi. Charaka was a contem-
Upanishadic age, Mithila became a prominent seat of porary of Kanishka
Brahmanical education. It was named Videha (fonsg). 2. Sushruta ( ): Father of Surgery / Plastic Surgery.
  It continued with its glory from Raja Janak up to His work is Sushruta Samhita (lqJr q lafgrk)
the Buddhist period. Later on, this place produced 3. Vagbhata ( ): He composed Ashtanga Hridayam
devotees of Lord Krishna. Samhita (v"VkX³ân;lafgrk).

M10_MADAN 07_65901_C10.indd 10 23/12/22 8:05 PM


Higher Education System 10.11

4. Āryabha a ( ): Āryabha a belonged to Stopover


Patliputra, (5th century AD during Gupta Era). He
was an Indian mathematician and astronomer, and Which of the following universities is being referred

the author of the Maha-Siddhanta. His great work on through the following statements?
Explanation of lunar eclipse and solar eclipse, rotation (1) It was perhaps the oldest university in the world.
of Earth on its axis, reflection of light by moon. He (2) There was no exam system.
invented zero. He referred to Algebra as Bijaganitam. (3) It has some impact of Greece culture.
He calculated the most accurate value of pi. He also (4) The students will enter the university at the age
worked on solar systems and calculated the length of 16.
of the solar year to 365.8586805 days. In his book Identify the university.
‘Aryabhattiyam’ (vk;ZHkÍh;e), he worked on suryasid- (a) Vikramshila University
hanta (lw;Zfl)akr), chakra yantra (pØ ;a=–disk instru- (b) Nalanda University
ment), Gola yantra (xksy ;a=–type of armillery sphere) (c) Ujjain University
and shadow instruments. Aryabhatta deduced that (d) Takshashila University
earth is a rotating sphere. Lalla (yYyk) followed the The correct option is (d).
tradition of Aryabhata. He was well known because of
twelve instruments which he brought into practice.
5. Varahamihira ( ): Born in 505 in Ujjain. Decline of A ncient Education
He is the Indian philosopher, astronomer, and With continuous Islamic invasions, many education
mathematician who wrote the Pancha-siddhantika centres were destroyed. Vedic system of education moved
(iap fl)kfUrdk–Five Treatises), a collection of Greek, to South. It was under the patronage of Vijayanagara
Egyptian, Roman, and Indian astronomy. rulers that the Vedic savants Sayana and Madhava wrote
6. Bhaskaracharya ( ): The leading math- commentaries on the Vedas.
ematician of the 12th century, who wrote the first The Muslim ruler elite promoted urban education in
work with full and systematic use of the decimal num- terms of libraries and literary societies. They founded
ber system. He found gravitational theory 500 years primary schools maktab (edrc) in which students
before Isaac Newton. learned reading, writing and basic Islamic prayers, and
6. Panini ( ): Dated between 6th and 4th century secondary schools (madrasas) to teach and train for
BCE, Panini was a great Sanskrit philologist, gram- advanced language skills. Often attached to mosques,
marian. He made several discoveries in the field of Islamic schools were open to the poor but were gender
phonetics, phonology and morphology. segregated, often only for boys. Muslim girls of affluent
7. Pingala ( ): A great mathematician associated families studied at home.
with binary numbers (similar to current Morse code). During Mughal era, Persian was the court language,
This indicates his deep understanding of arithme- and elite boys could attend Persian schools to learn liter-
tic. He offered ideas about Fibonacci numbers and ature, history, ethics, law, administration and court pro-
Pascal’s triangle. tocol. More intimate settings for the spread of ideas were
8. Brahmagupta ( 598–668 AD): He wrote on math- the retreats (khanqah - [kudkg) of famous Sufis (Muslims
ematics and astronomy in Brahamasphutasiddhanta who professed mystic doctrines). Sanskrit academies
(czãLQqVfl)akr) in 628 AD where he discussed general continued to teach young male Brahmans literature and
quadratic, square, square roots, cubes and cube roots of law; apprenticeship and commercial schools taught boys
an integer, fractions etc. the skills needed for business. Education for girls was an
9. Bhaskara-I ( ): A 7th century Indian exception rather than a rule.
mathematician–the first to write numbers in the Hindu
decimal system with a circle for the zero, and who gave Policies Perspective of Modern
a unique and remarkable rational approximation of the
sine function in his commentary on Aryabhata’s work. Education
10. Vachaspati Misra ( –ad 840): Vachaspati
Let us look at the basic perspective of higher education in
anticipated solid (co-ordinate) geometry eight centu-
India.
ries before Descartes (ad 1644)
Keeping in view the nature and scope, there are varia-
11. Chakrapani Datta ( , from Bengal): He
tions between the definitions of an education policy. An
wrote Charaka Cheturanana, Sushruta Sahasranayana
education policy may be defined as follows:
and Mahamahopadhyaya (pjdk prqjkuu, lqJr q k vkSj
lgL=u;ue egkegksik/;k;). He made contributions towards 1. A process through which any society handles an edu-
Sanskrit grammar, Nyaya QylQk. He compiled shabad- cational problem, which includes a society’s expressed
chandrika ('kCnpfUnzdk) dictionary. His annotations on intentions and official enactments as well as its con-
Gauatama's Nyāya Sūtras are noteworthy. sistent patterns of activity and inactivity in the area of
12. Nāgārjuna ( ): He was an Indian metallurgist education
and alchemist. He knew about extraction of mercury Social problem → Government action → Impact
and iron. on society

M10_MADAN 07_65901_C10.indd 11 23/12/22 8:05 PM


10.12 Chapter 10

Entrepreneurship Specialization

Productivity
JOBS
PRIVATE

Economic
Growth
Increasing Spending
Sustained
Higher
Income
Education Tax Revenues Growth
Poverty
Reduction

PUBLIC
R&D SOCIAL
FDI DEVELOPMENT
GOVERNANCE SAFETY

2. The process by which governments (society) translate The British also wanted to understand the local cus-
their educational vision into programmes and activi- toms and laws well.
ties to deliver outcomes, that is, desired changes in
1. Warren Hastings established the Calcutta Madrassa
the real world
in 1781 for the teaching of Muslim law.
3. An explicit or implicit single decision or group of deci-
2. In 1791, a Sanskrit College was started in Varanasi by
sions that may set out directives for guiding future
Jonathan Duncan for the study of Hindu philosophy
decisions, initiate or retard action, or guide imple-
and law system.
mentation of previous decisions
3. The Charter Act, 1813 was the first step towards edu-
Policy, therefore, is a course of action rather than the cation being made an objective of the government.
everyday decisions or actions, perceived by analysts. There was some split in the government about the
A framework of educational policy analysis involves a nature of education, whether it should be traditional
process in which various stakeholders analyse, generate, or modern.
implement, assess and redesign policies. This is referred 4. The oldest college was set up in Calcutta in 1817.
to as educational policy cycle. It is defined as a tool used Later in 2010, it was converted into a university that
in the analysis of a policy item development. It involves is called as Presidency University.
the following steps: 5. The oldest university that is still in operation is Senate
of Serampore College. It was set up in 1818. It got the
1. Problem identification university status in 1829.
2. Policy formation to deal with the identified problem 6. The present system of higher education can be
3. Decision-making targeting the identified problem dated back to Mountstuart Elphinstone’s minutes
4. Implementation, that is, the realization of an applica- of 1823 which stressed on the need for establishing
tion, or execution of the plan schools for teaching English and European science
5. Analysis and evaluation of a policy to continue or ter- subjects.
minate 7. Lord Macaulay in his minutes in 1835 advocated
“efforts to make natives of the country thoroughly
Evolution of Higher Learning and Research good English scholars”.
in P ost -I ndependence I ndia 8. In 1835, under Lord William Bentick, it was decided
Modern education began in India under the British rule. to introduce English as the medium of instructions in
The three basic agents of modern education in India India. The start was to be made among from upper
were as follows: and middle classes students. Ultimately that policy
was to trickle down to the masses. That was called as
1. British rule ‘Infiltration Theory’. The main objective was to create
2. Christian missionaries a class of Indians who Indian in colour and blood but
3. Indian intellectuals and reformers were to English in taste and affiliation.
The company wanted some educated Indians who could 9. In 1835, Elphinstone College (Bombay) and Calcutta
assist them in the administration of the land. Medical College were founded.

M10_MADAN 07_65901_C10.indd 12 23/12/22 8:05 PM


Higher Education System 10.13

10. Sir Charles Wood’s Dispatch of 1854 that is also called 20. Wardha Scheme of education (1937) Nai Talim or
as “Magna Carta of English Education in India’ rec- basic education, at the recommendation of Mahatma
ommended to create a properly articulated scheme of Gandhi.
education from the primary school to the university. 21. Sargent Report (1944) also known as ‘Scheme of
It sought to encourage indigenous education and also Post war Educational Development in India’ recom-
the creation of coherent policy of education. mended setting up of University Grant Commission.
11. Subsequently, the universities were set up in 1857. It has been discussed further also as it set precedence
(a) University of Calcutta. for setting up of University Grant Committee.
(b) University of Bombay (now in Mumbai).   The education system under British rule set up
(c) University of Madras. the modern education in India. After independence,
12. Indian Education Commission, also called as Hunter government of India changed the focus to meet the
Commission (1882–83) recommended the segrega- expectations of people. They were still considered as
tion of education into primary education and higher a basis for post-independence education system.
education.
13. In 1902, Indian Universities Commission was set up Stopover
under Sir Thomas Raleigh to enquire into conditions
and prospects of setting up of universities in India. As Universities in the Presidency towns in India were

a result of its recommendations, Indian Universities established in
Act was passed in 1904. (a) 1857 (b) 1858
14. In 1905, National Council of Education was set up (c) 1900 (d) 1909
by Swadeshi nationalist leaders. Jadavpur University The correct option is (a).
was set also result of this effort. Sri Rabindranath
Tagore set up Shanti Niketan institutions in Bengal. Modern Education after Independence
In 1921, Vishwa Bharti University by set by Tagore.
The Government of India took several initiatives to
That central university is an ‘Institution of National
improve and promote higher education in the country
Importance’ now. It is the only university where Prime
after independence.
Minister is the Chancellor.
15. In 1913, there was a resolution on education policy. 1. Government of India Act, 1935: This act did not
In 1917, Calcutta University Commission, that is also lead to any significant change, except that it put an
called as Sadler Commission, suggested the separa- end to the inherently defective system of diarchy. For
tion of intermediate education from degree colleges. the first time, a constitutional distinction was made
Sadler Commission suggested to provide large amount between the powers of the Centre (Federation, as it
of autonomy to the teaching bodies of the universities. was known in the 1935 Act) and those of the prov-
16. The Morley-Minto Reforms had been introduced in inces. Two lists of subjects were drawn up in this Act;
1919. In respect of education, it meant the transfer one enumerated central powers and the second the
of the constitutional authority to manage education, provincial powers.
not only to the provinces but to Indian ministers as 2. Sargent Report: Its official name was Central
well. Some subjects were reserved, and some were Advisory Board of Education on Post-War Educational
transferred. This system was part of ‘diarchy (or Development in India. It was set up in 1944. This was
dyarchy) system. the first attempt to formulate a national system of
17. This commission had been the precursor to the education in India. It recommended the formation
10 + 2 + 3 system and setting up of ‘Central Advisory of a University Grants Committee, which was formed
Board of Education’ (CABE). CABE is the oldest and in 1945 to oversee the work of the three central uni-
the most important advisory body of Government of versities of Aligarh, Banaras and Delhi. In 1947, the
India in education was first established in 1920 and Committee was entrusted with the responsibility of
dissolved in 1923 as a measure of economy. It was dealing with all the then existing universities. It rec-
revived in 1935 and has been in existence ever since. ommended that the University Grants Committee be
  The government of India act made education as reconstituted on the general model of the University
provincial subject. Grants Commission of the United Kingdom with
18. The Inter-University Board (later known as the a full-time Chairman and other members to be
Association of Indian Universities) was established appointed from among educationists of repute.
in1925 to promote university activities, by sharing 3. University Education Commission: This was
information and cooperation in the field of education, appointed in 1948 under the Chairmanship of Dr S.
culture, sports and allied areas. It was represented by Radhakrishnan. He made an interim recommendation
the vice-chancellors. to the government wherein he suggested that education
19. Hartog Commission (1929) focused on quality and be made a concurrent subject while the Constitution
standards of education. Sapru Committee (1934) was under formation. This recommendation was not
focused upon unemployment issue. Abbot Wood accepted by the Constituent Assembly. However, a com-
report (1937) recommended English as a medium of promise was worked out; certain entries were inserted
instruction at university level. which gave far-reaching powers to the Centre.

M10_MADAN 07_65901_C10.indd 13 23/12/22 8:05 PM


10.14 Chapter 10

(a) Entry 63 dealt with the control of institutions 8. Education Subject in Concurrent List (1976)
such as the National Library, Calcutta, and uni- India has a federal set up and education is the con-
versities (now often called ‘central universities’) current responsibility of both the centre as well as
such as Banaras, Aligarh and Delhi. the states. Post-independence, education (includ-
(b) Entry 64 dealt with institutions of scientific and ing university education) was the responsibility of
technical education—INI (IIT). the states, while the centre was given the function of
(c) Entry 65 dealt with agencies and institutions for coordination and determination of standards. How-
professional, vocational or technical training or ever, in 1976, through Entry 25 (42nd Constitution
scientific or technical institutions could also be Amendment) in the concurrent list of the constitution
established by the Centre. of India. The centre was also given the responsibilities
(d) Entry 66 dealt with various issues regarding along with the states for all levels of education.
higher education—specifically it is about coordi- 9. In 1978, there was publishing of ‘Development of
nation and determination of standards in institu- Higher Education: A Policy Framework approach.
tions for higher education or research or scien- 10. The government set up the ‘National Commission on
tific and technical institutions. Teachers (higher education) in 1984.
11. During the years the UGC has implemented the deci-
Here, it is worth noting that UGC, AICTE, Medical sions such as for setting up study centres or cells
Council of India (that became National Medical for women studies in 1986 and ‘Career Orientation
Commission in 2019), Nursing Council of ICAR, Pharmacy to Education (Vocationalisation of Education)’ in
Council of India, Institute of Architects, Institute of 1994–95.
Accountancy, Bar Council of India, etc., were established 12. National Education Policy 1986 and Point of Action
under Entry 66 perspective. 1992
1. In 1952, the Union Government decided that all The main objective of the National Policy of Education
cases pertaining to the allocation of grants-in-aid of 1986 and Programme of Action, 1992 was to estab-
from public funds to the central universities and lish a national system of education implies that all stu-
other universities and institutions of higher learning dents irrespective of castes; creed, sex, and religion have
might be referred to the UGC. Consequently, the UGC access to education of a comparable quality. There was
was formally inaugurated by late Shri Maulana Abul due emphasis on primary and secondary education.
Kalam Azad, the then Minister of Education, Natural This policy came after when the education was
Resources and Scientific Research on 28 December entered into concurrent list in the year 1976 through
1953. 42nd constitutional amendment. Most of our classic
2. The UGC, however, was formally established only in government schemes such as Sarva Shiksha Abhiyan,
November 1956 as a statutory body of the Government Mid Day Meal Scheme, Navodaya Vidyalayas (NVS
of India through an Act of Parliament for the coordina- schools), Kendriya Vidyalayas (KV schools) and use of
tion, determination and maintenance of standards of IT in education were started under the NEP of 1986.
university education in India. The NPE emphasized on enhancing and promoting
3. Mudaliar Commission (1952–1953) the vocationalisation of education adult, education,
It is also popular as the Secondary Education education for the mentally and physically challenged
Commission. It recommended introducing a three- persons, non-formal education, open universities
year secondary and a four-year higher education sys- and distance learning, rural university, early child-
tem. It also advocated the setting up of multipurpose care and education. Delinking degrees from job was
schools and vocational training institutes. also one of the basic objectives of National Policy of
4. Committee on Emotional Integration (1961) Education 1986.
It was set up under the chairmanship of Dr 13. Main points in Point of Action, 1992
Sampurnanand to study the role of educational pro- • Guidelines given to new colleges for affiliation
grammes for the youth, in order to strengthen the • Committee set up to look into management pattern
process of emotional integration. of universities
5. Students should remember that in 1963, there was a • Steps initiated to conduct qualifying exam to
decision by the Gujarat University to make Gujarati recruit university teachers
the sole medium of instruction and examination. But • The procedure to conduct qualifying exam (such as
it was taken back after a court decision. NET / SET/JRF) was adopted.
6. Kothari Commission recommendations (1964–66) • There was provision to set up management pattern
have been discussed later in the chapter. Following of universities
that, the NPE was announced in 1968. • Redesigning of courses
7. The National Council for Teacher Education • UGC approved schemes for Academic Staff Colleges
(NCTE): It was set up in 1973 by a government reso- • Performance appraisal for teachers finalized
lution as a national expert body to advise central and • State Councils for higher education finalization
state governments on all matters pertaining to teacher • Development of autonomous colleges
education. The Council was made a statutory body by • Establishment of ‘State Councils of Higher Educa-
an Act of Parliament in 1993. tion’ (SCHEs)

M10_MADAN 07_65901_C10.indd 14 23/12/22 8:05 PM


Higher Education System 10.15

14. The origin of NUEPA (earlier known as National Dr Anil Kakodkar Committee
Institute of Educational Planning and Administration It was constituted to recommend strategies to improve
[NIEPA]) is associated with the UNESCO’s Regional technical education in the country. It recommended
Center for Educational Planners and Administration 2% budget in every institution to be earmarked for
started in 1960–61 for taking care of educational research.
needs of South Asia.
15. The focus of the NPE, 1986, was on development of K. B. Pawar Committee
human resources. Constituted by the UGC, the Committee recommended
four models of public–private partnership (PPP) in higher
I mportant Committees’ Reports education.
Gnanam Committee (1993)
It recommended flexibility and autonomy for ensur-
Education Commission
ing academic excellence and asked for restricting the The Education Commission is also named as Kothari
unchecked growth of deemed universities. It emphasized commission. Daulat Singh Kothari, popularly known as
the need for a National Commission on higher education D. S. Kothari, was an outstanding scientist and a great
and research to regulate the quality of education and to educationist. The report of this commission is called as
encourage research in university system. ‘Educational and National Development’ report.
The four main themes of the commission were:
Sam Pitroda Committee
1. Increase in Productivity.
It was established in 2007. It is also popularly known as the 2. Promoting social and National Integration.
National Knowledge Commission (NKC). It ­recommended 3. Educational and Modernization.
the restructuring of curricula to meet the demand for mul- 4. Developing social, moral and spiritual values.
tidisciplinary professionals and c­riteria-based resource
allocation to ensure maintenance of standards and stra- The main recommendations have been given in the table.
tegic preferences to promote excellence in higher educa- ‘Free and compulsory education’ for children between
tion. It supported the entry of foreign universities and also 6 to 14 years under the spirit of Article 45 of constitution
favoured reducing the burden of affiliation of colleges on as per Directive Principles of State Policy (DPSP).
universities. The NKC recommended increasing the num- The establishment of Indian Education Service (IES).
ber of universities to 1500 by 2015. The commission recommended that 6% of the national
income should be spent on education.
Yashpal Committee Work-experience and national service should become
It suggested scrapping of all higher education, regulatory integral part of education.
or monitoring bodies and the creation of a super-regu- With a view to accelerate the growth of national econ-
lator, that is, a seven-member Commission for Higher omy, research, science and education should receive high
Education and Research (CHER). State Higher Education priority.
Councils would form the second tier of the system. A small number of ‘cluster of centres’ to achieve high-
It also recommended that the deemed university sta- est possible standards in research and training were to be
tus be abandoned and that all deserving deemed varsi- set up.
ties be either converted into full-fledged universities or Most of the recommendations of Kothari Commission
scrapped. The Committee stressed the need for more were reflected that has become the part of Education
attention to undergraduate programmes and a multidis- Policy of 1968.
ciplinary approach to learning. It also strongly recom- Three Languages System: The National Policy of
mended reducing the burden of affiliation of colleges on Education suggested for the study of “Hindi, English, and
the universities and a GRE-like test be evolved for univer- modern Indian language (preferably one of the southern
sity education. languages) in the Hindi speaking states and Hindi, English,
The recommendations of the Yashpal Committee and and the Regional language in the non-Hindi speaking
the NKC emanated from the realization that the fragmen- States”. Tamil Nadu does not follow the three-language
tation of various fields of knowledge in higher education formula prescribed by the first education policy in 1968.
led to inadequate growth of interdisciplinary learning.
Committees for Financing of Education
Sharma Committee
The following committees were set up for the financing of
Set up under Prof. M. M. Sharma, it deliberated on the education:
development of science and technology education in
India. The Committee suggested the establishment of 1. Punnayya Committee Report, 1992–93
the Indian Institutes of Science Education and Research 2. Anandkrishnan Committee, 1999
(IISER). It also recommended the expansion of techni- 3. Mahmood-ur Rahman Committee to formulate
cal education, assuring quality and providing access and revised fee structure in the central and deemed uni-
affordability for technical education. The Committee also versities, 2000
recommended that `500 crore be spent on research in 4. Swaminathan Committee on resource mobilization by
basic sciences every year by the UGC. technical and professional institutions, AICTE, 1991

M10_MADAN 07_65901_C10.indd 15 23/12/22 8:05 PM


10.16 Chapter 10

Teacher Education (NCTE) in consultation with


New Education Policy, 2020 (NEP) NCERT.
NEP is based on Draft National Education Policy, 2019. 10. By 2030, the minimum degree qualification for teach-
Dr. K. Kasturirangan was the Chairperson for the purpose. ing will be a 4-year integrated B.Ed. degree.
This policy intends to facilitate an inclusive, participatory 11. No child loses any opportunity to learn and excel
and holistic approach. There is a progressive shift towards because of the circumstances of birth or back-
a more scientific approach to education. ground. There will be special focus on Socially and
The key objective is to develop cognitive development Economically Disadvantaged Groups(SEDGs) which
as well as social and physical awareness stages of chil- include gender, socio-cultural, and geographical
dren. The global education development agenda reflected identities and disabilities.
in the Goal 4 (SDG4) of the 2030 Agenda for Sustainable 12. Recruitment of teachers will be through robust, trans-
Development, adopted by India in 2015 - seeks to “ensure parent processes. Promotions will be merit-based,
inclusive and equitable quality education and promote with a mechanism for multi-source periodic perfor-
lifelong learning opportunities for all” by 2030. The mance appraisals and available progression paths
NEP 2020 aims at making “India a global knowledge to become educational administrators or teacher
superpower”. educators.
NEP is third education policy after 1968 and 1986. A common National Professional Standards for
The name of MHRD has been changed to the Ministry of Teachers (NPST) will be developed by the National
Education. This policy can be divided into four parts: Council for Teacher Education by 2022, in consulta-
tion with NCERT, SCERTs, teachers and expert orga-
1. School education nizations from across levels and regions.
2. Higher education 13. The schools will be organized into complexes or clus-
3. ‘Other Key Areas of Focus’ such as adult education, ters which will be the basic unit of governance and
promoting Indian languages and online education ensure availability of all resources including infra-
4. ‘Making it Happen’ that discusses about policy’s structure, academic libraries and a strong profes-
implementation sional teacher community.
14. There will be clear, separate systems for policy mak-
School Education ing, regulation, operations and academic matters.
1. The objectives is the universalization of education States/UTs will set up independent State School
from preschool to secondary level with 100% GER Standards Authority (SSSA). SSSA, will be used
by 2030. About 2 crores out of school children will be extensively for public oversight and accountability.
brought back into main stream under NEP 2020. 15. The SCERT will develop a School Quality Assessment
2. The current 10 + 2 system to be replaced by a new and Accreditation Framework (SQAAF) through con-
5 + 3 + 3 + 4 curricular structure corresponding to sultations with all stakeholders.
ages 3-8, 8-11, 11-14, and 14-18 years respectively.
3. The government shall constitute a ‘Gender-Inclusion Higher Education
Fund’ to provide equitable and quality education to all 1. Holistic Undergraduate education with a flexible cur-
girls and transgender students. riculum can be of 3 or 4 years. There will be multiple
4. Class 10 and 12 board examinations to be made exit options and appropriate certification within this
easier, to test core competencies rather than memo- period. For example, Certificate after 1 year, Advanced
rised facts, with all students allowed to take the exam Diploma after 2 years, Bachelor’s Degree after 3 years
twice. and Bachelor’s with Research after 4 years.
5. There will be ‘National Mission on Foundational 2. All the courses at undergraduate, postgraduate
Literacy and Numeracy by Ministry of Education’. and PhD level will now be interdisciplinary. M. Phil
There should be no rigid separation between academic courses will be discontinued.
streams, extracurricular, vocational streams in schools. 2. Academic Bank of Credits to be established to facili-
6. Vocational Education will start from Class 6 with tate Transfer of Credits.
Internships. 3. Multidisciplinary Education and Research Universities
7. Teaching up to at least Grade 5 to be in mother (MERUs), at par with IITs, IIMs, to be set up as mod-
tongue/regional language. No language will be els of best multidisciplinary education of global stan-
imposed on any student. Sanskrit to be offered at all dards in the country.
levels of school and higher education as an option for 4. The National Research Foundation will be created as an
students, including in the three-language formula. apex body for fostering a strong research culture and
8. Assessment reforms with 360 degrees Holistic building research capacity across higher education.
Progress Card, tracking Student Progress for achiev- 5. Higher Education Commission of India (HECI) will be
ing Learning Outcomes. set up as a single umbrella body for the entire higher
9. A new and comprehensive National Curriculum education, excluding medical and legal education.
Framework for Teacher Education (NCFTE) 2021, Public and private higher education institutions will
will be formulated by the National Council for be governed by the same set of norms for regulation,

M10_MADAN 07_65901_C10.indd 16 23/12/22 8:05 PM


Higher Education System 10.17

accreditation and academic standards. HECI will be 9. All professional education will be an integral part of
have four independent verticals. the higher education system.
(a) National Higher Education Regulatory Council Stand-alone technical universities, health science uni-
(NHERC) for regulation versities, legal and agricultural universities etc will
(b) General Education Council (GEC) for standard aim to become multi-disciplinary institutions.
setting 10. Centre and the States will work together to increase
(c) Higher Education Grants Council (HEGC) for the public investment in Education sector to reach
funding 6% of GDP at the earliest. According to our Finance
(d) National Accreditation Council (NAC) for Minister, the expenditure on education as a percent-
accreditation. age of GDP was 2.8 % during 2019–20, and 3.1%
(e) HECI will function through faceless intervention (estimated during 2021–22).
through technology, & will have powers to penal-
Other Changes:
ise Higher Education Institutions which do not
conform to norms and standards. 1. An autonomous body, the National Educational the
6. There is plan to phase out affiliation of colleges system Free Exchange Technology Forum (NETF), will be
in 15 years. There will be a stage-wise mechanism for created to provide a platform for of ideas on the use
granting graded autonomy to the colleges in lieu of that. of technology to enhance learning, assessment, plan-
7. GER in higher education to be raised to 50% by 2035. ning, administration.
3.5 crore seats to be added in higher education. The 2. Nationals Assessment Centre- ‘PARAKH’ (Performance
current GER in higher education is 26.3%. Assessment, Review, and Analysis of Knowledge for
8. A new and comprehensive National Curriculum Holistic Development) has been created to assess the
Framework for Teacher Education, NCFTE 2021, students. It also paves the way for foreign universities
will be formulated by the NCTE in consultation with to set up campuses in India.
NCERT (already in the process). 3. NEP emphasizes setting up of Gender Inclusion Fund
By 2030, the minimum degree qualification for teach- (GIF), Special Education Zones (SEZs) for disadvan-
ing will be a 4-year integrated B.Ed. degree. taged regions and groups.
Stringent action will be taken against substandard 4. In ‘Virtual Labs’ existing e-learning platforms such as
stand-alone Teacher Education Institutions (TEIs). DIKSHA, SWAYAM and SWAYAMPRABHA will also be

Existing Academic Structure New Academic Structure


New pedagogical and curricular structure
of school education (5+3+3+4): 3 years in
2 Years Anganwadi/pre-school and 12 years in
(Age 16-10) 4 Years School
Secondary

(Class 9 to 12)
(Age 14-18)

• Secondary Stage (4) multidisciplinary


study, greater critical thinking, flexibility
3 Years and student choice of subjects
Middle

(Class 6 to 8)
10 Years (Age 11-14)
• Middle Stage (3) experiential learning
(Age 6-16) in the sciences, mathematics, arts,
Foundational

3 Years social sciences, and humanities


(Class 3 to 5)
(Age 8-11)
• Preparatory Stage (3) play, discovery,
and activity-based and interactive
2 years classroom learning
(Class 1 & 2)
Preparatory

(Ages 6-8)
3 years • Foundational stage (5) multilevel,
(Anganwadi/ play/activity-based learning
pre-school/
Balvatika)
(Ages 3-6)

Figure 10.6 Transforming Curricular and Pedagogical Structure


Source: National Education Policy 2020, Ministry of Education, Govt. of India.

M10_MADAN 07_65901_C10.indd 17 23/12/22 8:05 PM


10.18 Chapter 10

leveraged for creating virtual labs so that all students 2. Certificate after 1 year, Advanced Diploma after
have equal access to quality practical and hands-on 2 years, Bachelor’s Degree after 3 years and
experiment-based learning experience. Bachelor’s with Research after 4 years.
5. National Institute for Pali, Persian and Prakrit, Indian 3. HECI will be set up as a single overarching
Institute of Translation and Interpretation should be umbrella body the for entire higher education,
set up. It also aims to increase the public investment excluding medical and legal education.
in the Education sector to reach 6% of GDP at the 4. A National Mission for Mentoring will be estab-
earliest. lished, with a large pool of outstanding senior/
retired faculty.
Currently, India spends around 4.6% of its total GDP Which of the above statements are true in context of
on education though the target set in 1968 and 1986 poli- New Education Policy, 2020?
cies was 6%. Codes:
We need to consider these constitutional aspects: (a) 1, 2 and 3 (b) 2, 3 and 4
1. Part IV of Indian Constitution, Article 45 and Article (c) 1, 3 and 4 (d) All of the above
39 (f) of Directive Principles of State Policy (DPSP), The correct option is (d).
has a provision for state-funded as well as equitable 5. What is the likely time span during which colleges
and accessible education. will be given graded autonomy to give degrees?
2. The 42nd Amendment to the Constitution in 1976 (a) 10 years (b) 12 years
that we discussed earlier. (c) 15 years (d) 20 years
3. The 86th Amendment in 2002 made education an The correct option is (c).
enforceable right under Article 21-A.
4. Right to Education (RTE) Act, 2009 aims to provide 6. Which of the following has been the main objective of
primary education to all children aged 6 to 14 years new education policy 2020?
and enforces education as a Fundamental Right. (a) Localization of education
5. NEP mandates 25% reservation for disadvantaged (b) Top rated global universities to be facilitated to
sections of the society where disadvantaged groups. come to India and top Indian Institutions to be
encouraged to go global
(c) Universalization of Education
(d) Both b and c
Stopover
The correct option is (d).
1. According to National Education Policy 2020, by how 7. Higher Education Commission of India (HECI) will be
much percent the Gross Enrolment Ratio in higher set up as a single umbrella body for the entire higher
education to be raised by 2035? education, excluding medical and legal education.
(a) 25% (b) 30% Public and private higher education institutions will
(c) 40% (d) 50% be governed by the same set of norms for regulation,
The correct option is (d). accreditation and academic standards.
2. Under the chairmanship of which of the follow- Which of the following independent verticals will be
ing personalities Committee for Evolution of the looked after by HECI?
‘New Education Policy’ submitted its report in May 1. National Higher Education Regulatory Council
2016? (NHERC) for regulation
(a) Late Shri T.S.R Subramanian 2. General Education Council (GEC) for standard
(b) Dr K. Kasturirangan setting
(c) Rina Roy 3. Higher Education Grants Council (HEGC) for funding
(d) Shri Sanjay Dhotre 4. National Accreditation Council (NAC) for
accreditation.
The correct option is (b). Codes:
3. In NEP 2020, the current 10+2 system to be replaced (a) 1, 2 and 3
by a new curricular structure. What is the new cur- (b) 2, 3 and 4
ricular structure? (c) 1, 3 and 4
(a) 3 + 4 + 4 + 5 (d) All of the above
(b) 5 + 3 + 3 + 4 The correct option is (d).
(c) 4 + 3 + 3 + 5
8. Which of the following is/are the crucial points for
(d) 5 + 4 + 3 + 3
New Education Policy, 2020?
The correct option is (b). (a) Academic Bank of Credits to be established to
4. Consider the following statements. facilitate Transfer of Credits
1. Multiple Entry and Exit Points with appropriate (b) Multidisciplinary Education and Research
certification UG education can be of 3 or 4 years Universities (MERUs), at par with IITs, IIMs, to be
with multiple exit options and appropriate certifi- set up as models of best multidisciplinary educa-
cation within this period. tion of global standards in the country.

M10_MADAN 07_65901_C10.indd 18 23/12/22 8:05 PM


Higher Education System 10.19

(c) The National Research Foundation will be created 3. Cost comparison


as an apex body for fostering a strong research 4. Location restriction
culture and building research capacity across
higher education. Orthodox Education
(d) All of the above Knowledge was passed on orally from one generation to
The correct option is (d). another as per orthodox education. Even now education
9. Consider the following points is being imparted in an orthodox manner.
(a) New Policy promotes Multilingualism in both Earlier in this chapter, we discussed about three steps:
schools and higher education. 1. ‘Sravana’ or ‘shrutis’
(b) National Institute for Pali, Persian and Prakrit , 2. ‘Manana’
Indian Institute of Translation and Interpretation 3. ‘Nidhyasana’
to be set up.
(c) Teaching up to at least Grade 5 to be in mother C. Rajgopalachari had said, ‘If there is honesty in India
tongue/ regional language. No language will be today, any hospitality, any charity any aversion to evil, any
imposed on any student. love to be good, it is due to whatever remains of the old
(d) All of the above faith and the old culture.’ Tolerance, truth, Ahimsa, peace
The correct option is (d). and non-aggression are the hallmark of Indian culture.
With a rational mind, raising it from ignorance, one can
10. Which of the following formal initiative is being understand the greatness of Vedic literature.
introduced in new education policy for assessment of Our ancient education system was mostly based on
learners? orthodox system.
(a) Assessment reforms with 360 degree Holistic
Progress Card. Conventional versus N on -Conventional
(b) Assessment reforms with Absolute Benchmarking
(c) Assessment reforms with Complete Rating Education
(d) Assessment reforms with complete success We can try to understand the concepts of conventional
The correct option is (a). and non-conventional educations by differentiating
between them.
Non-conventional education basically deals with the
distance education and also with the concepts such as
Orthodox, Conventional and online education. In this chapter, distance education has
Non-conventional Education been dealt with separately.

There are basically four factors that help us to decide Regulatory and Policy Framework
which system to opt for and they are listed as follows: Structure of Higher Education in India
1. Length of the programme Now again we shift to higher learning in post-inde-
2. Technical access pendence India. Education is in the Concurrent List,

Conventional Teaching Non-conventional Teaching


It is basically passive learning, in which the child listens Active learning, in which students move freely, choosing
to the teacher and follows directions from the teacher their own work and the pace at which it is being done.
who sets the pace for instructions Teacher, if any, may just be a facilitator
Students provided with knowledge, skills and experience General development of students and mastering of
learning modules
Core competence—examples, facts, arguments and text, Holistic development as it includes generalization of
a dedicated time and effort, the purpose is to master law, theory, rule and concept. Cognitive, social and
defined skills and develop socially psychological development of mind
There is constant peer contact There is limited peer contact
Group, individual, whole class Teamwork and collective way of training
Verbal, visual and practical limited contact time Problem statement, partial search and heuristic/
brainstorming methods
Control and assessment by teacher Self-control and self-assessment
Subject–object relations Cooperation and collaboration

M10_MADAN 07_65901_C10.indd 19 23/12/22 8:05 PM


10.20 Chapter 10

where both the central and the state governments can


legislate. Apex-level Bodies
There are eight apex-level bodies (regulatory bodies/
regulatory FrameWork oF higher education research councils) under the Department of Higher
in i ndia Education, which are responsible for higher education
While the Centre coordinates and determines the stand- in India. These bodies can be broadly divided into
ards in higher and technical education, school educa- two categories: (i) regulatory bodies and (ii) research
tion is primarily the responsibility of the state. The key councils.
policy-making agencies for higher education are as fol-
lows (Fig. 10.7): regulatory bodies
1. Central Government: It lays down the NPE. It pro- There are three regulatory bodies—the UGC, All India
vides grants to the UGC and establishes central Council for Technical Education (AICTE) and Council
universities/institutions of national importance in of Architecture (COA)—to regulate higher education in
the country. It is also responsible for declaring an India.
educational institution as ‘Deemed-to-be university’
on the recommendations of the UGC.
2. State Government: Many states have also set up state Concept Box
councils and advisory boards to provide guidelines 1. The word ‘university’ is derived from the Latin
for proper functioning of higher education institu- word Universitas that means ‘a whole’.
tions in the states. State councils for higher education 2. University means a specialized association between
coordinate the roles of government, universities and students and teachers.
apex regulatory agencies in higher education within 3. Universities are the seats of higher learning from
the state. where the society gets its leaders in science, arts
3. CABE was set up for coordination and cooperation and various other fields of national life. University
between the union and the states in the field of educa- education aims at providing knowledge and wis-
tion, including policy-making. dom for developing personality.
4. The functions of the university mainly include the
following:
(a) Providing instruction
(b) Conducting research and postgraduate studies
Education as part of Concurrent (c) Giving affiliation and extension to the colleges
List provides equal power to the under it
central and State In India, university means a university established
governments in regulation
or incorporated by or under a central act, a provincial
act or a state act and includes any such institution as
may be recognized by the UGC in accordance with the
Central State regulations made under this Act.
government government Universities have degree-granting powers and are
responsible for conducting examinations.
They have autonomy in matters of fees and curricu-
MHRD and lum design. They also have affiliating powers for col-
other leges within a particular geographical region.
ministries Degree-granting colleges have autonomy in admis-
sions. However, they have to follow the fee, examina-
tion and curriculum standards of the university they
Regulatory
Departments/ are affiliated to.
bodies/
Accreditation councils of
professional
bodies: higher/
councils
NAAC, NBA technical uniVersity grants commission (ugc)
e.g.
education
UGC, AICTE) The UGC is a quasi-independent body. It was set up
to discharge the responsibility of coordinating and
maintaining standards in the fields of higher education.
More specifically, the UGC takes charge of the general
Higher education institutions higher education in Arts, Sciences, Commerce and
professional education which are under the multi-faculty
Figure 10.7 Higher Education Regulatory universities. The UGC governs universities in India and
Framework came into existence on 28 December 1953. It became a

M10_MADAN 07_65901_C10.indd 20 23/12/22 8:05 PM


Higher Education System 10.21

statutory organization established by an act of Parliament receive central assistance (UGC grant) under Section 12
in 1956. (B)** of UGC Act, 1956 as per approved pattern of assis-
1. According to Section 12 of the UGC Act, the main tance under various schemes.
function of the UGC is coordination, determination Central Universities
and maintenance of standards in universities.
2. It also disburses funds within the university educa- A central university or a union university in India is
tion system. Most importantly, it acts only as a recom- established by the Act of Parliament and is under the
mendatory body since it does not have any power to purview of the Department of Higher Education in the
establish or derecognize any university. Ministry of Education (MoE). In general, universities in
3. The UGC consists of the Chairman, Vice-Chairman India are recognized by the UGC, which draws its power
and 10 other members appointed by the central gov- from the University Grants Commission Act, 1956.
ernment. Secretary is the Executive Head. The UGC 1. There are 54 central universities under the purview
functions from New Delhi as well as its six regional of the MoE. Out of them, 16 new central universities
offices located in Bengaluru, Bhopal, Guwahati, were established in 2009 by an Act of Parliament,
Hyderabad, Kolkata and Pune. namely, Central Universities Act, 2009.
4. The UGC also implements various schemes aimed 2. IGNOU, New Delhi, is funded directly by MoE.
at improving the quality of higher education, such 3. The President of India is the Visitor of all central uni-
as Universities with Potential for Excellence (UPE), versities. In that capacity, he/she nominates some
Colleges with Potential for Excellence (CPE), Centre members to important committees of the university for
with Potential for Excellence and a Particular Area their effective functioning. He/she also exercises pow-
(CPEPA), Special Assistance Programme (SAP) and ers in various legal matters and relevant amendments.
Basic Scientific Research (BSR).
5. Sir Shanti Swaroop Bhatnagar was the first Chairman The following universities are not under purview of
of the UGC. Dr M Jagadesh Kumar is the current the UGC:
Chairman of the UGC. 1. Central Agricultural University, Imphal, Manipur
Updation: UGC, AICTE and National Council of Teacher 2. Indira Gandhi National Open University, New Delhi
Education (NCTE that was set up in 1995 under NCTE 3. Indian Maritime University, Chennai
Act, 1993) are to be merged to form Higher Education 4. Nalanda University, Rajgir, District Nalanda, Bihar
Commission of India (HECI). This should work for 5. South Asian University, Akbar Bhawan Campus in
autonomy and multidisciplinary approach to education. Chanakyapuri, New Delhi
This will be in synchronisation with New Education 6. Rajiv Gandhi National Aviation University, Rae Bareli,
Policy (NEP-2020). All the courses approved by UGC, Uttar Pradesh
AICTE and NCTE shall be mapped for their equiva- 7. Rani LakshmiBhai Central Agricultural University,
lence under National Higher Education Qualification NH-75, Near Pahuj Dam, Gwalior Road, Jhansi (Uttar
Framework (NHEQF) which would make academic Pradesh)
mobility smoother, both vertically and laterally. This will 8. Dr. Rajendra Prasad Central Agricultural University,
give multidisciplinary education and research a boost. Pusa, Samastipur, Bihar
9. National Sports University, Koutruk, Manipur
Categorization of Universities
Universities can be set up only through legislation or the The Indian National Defence University (INDU) is a
deemed route. The main constituents of universities or uni- proposed university for defence sector by the Government
versity-level institutions at present are listed in Table 10.1. of India to be established at Binola in Gurgaon, Haryana.
The principal proposal was accepted by the Union Cabinet
Table 10.1 in May 2010. The 66% of students will be drawn from the
armed forces and the rest from other government agen-
Universities Total Universities cies, police and civilians.
Number under 12 (B)
State Universities
State universities 460 267
A university established or incorporated by a Provincial
Deemed-to-be universities 128 50 Act or by a State Act is called a state university.
Central universities 54 54 The state universities are included in the List of 12(B)
of UGC Act, 1956, and are eligible for central assistance.
Private universities 430 25 Although the development of state universities is the
Total 1072 396 primary concern of state governments, development
grants, including grants under special schemes, are pro-
Source: ugc.ac.in as on November 25, 2022. vided to all eligible state universities. Such grants facili-
tate the creation, augmentation and upgradation of
UGC provides financial assistance to eligible colleges infrastructural facilities that are not normally available
which are included under Section 2(f)* and declared fit to from the state government or other sources of funds.

M10_MADAN 07_65901_C10.indd 21 23/12/22 8:05 PM


10.22 Chapter 10

State Name of Central University


Andhra Pradesh Central University of Andhra Pradesh, Anantapuram
Central Tribal University of Andhra Pradesh, Vijayanagram
The National Sanskrit University, Tirupati
Assam University, Silchar
Assam
Tezpur University, Sonitpur
Arunachal Pradesh Rajiv Gandhi University, Itanagar
Central University of South Bihar, Gaya
Dr. Rajendra Prasad Central Agricultural University, Samastipur
Bihar
Mahatma Gandhi Central University, East Champaran
Nalanda University, Rajgir
Chhattisgarh Guru Ghasidas Vishwavidyalaya, Bilaspur, Chhattisgarh
Indira Gandhi National Open University
Jamia Millia Islamia
Jawaharlal Nehru University
Delhi South Asian University
University of Delhi
The Central Sanskrit University
Shri Lal Bahadur Shastri National Sanskrit University
Gujarat Central University of Gujarat, Gandhinagar
Haryana Central University of Haryana, Mahendergarh
Himachal Pradesh Central University of Himachal Pradesh, Dharamashala
Central University of Kashmir, Srinagar
Jammu and Kashmir
Central University of Jammu, Jammu
Jharkhand Central University of Jharkhand, Ranchi
Karnataka Central University of Karnataka, Gulbarga
Kerala Central University of Kerala
Dr. Harisingh Gour Vishwavidyalaya, Sagar
Madhya Pradesh
The Indira Gandhi National Tribal University, Amarkantak
Maharashtra Mahatma Gandhi Antarrashtriya Hindi Vishwavidyalaya, Wardha
Central Agricultural University, Imphal
Manipur Manipur University, Imphal
National Sports University, Koutruk
Mizoram Mizoram University
Meghalaya North Eastern Hill University, NEHU Campus, Shillong
Nagaland Nagaland University
Odisha Central University of Odisha, Koraput
Pondicherry Pondicherry University
Punjab Central University of Punjab, Bathinda
Rajasthan Central University of Rajasthan, Ajmer
Sikkim Sikkim University, Gangtok, Sikkim
Central University of Tamil Nadu, Thiruvarur
Tamil Nadu
Indian Maritime University, Chennai
English and Foreign Languages University, Osmania University Campus, Hyderabad
Telangana Maulana Azad National Urdu University, Hyderabad
University of Hyderabad, Hyderabad
(Continued)

M10_MADAN 07_65901_C10.indd 22 23/12/22 8:05 PM


Higher Education System 10.23

State Name of Central University


Tripura Tripura University, Agartala
Uttarakhand Hemwati Nandan Bahuguna Garhwal University, Srinagar, Garhwal
Aligarh Muslim University, Aligarh
Babasaheb Bhimrao Ambedkar University, Lucknow
Banaras Hindu University, Varanasi
Uttar Pradesh
University of Allahabad, Allahabad
Rajiv Gandhi National Aviation University, Rae Bareli
Rani Lakshmi Bai Central Agricultural University, Jhansi
West Bengal Visva-Bharati, Shantiniketan

State universities dominate university education in 3. Shri Lal Bahadur Shastri Rashtriya Sanskrit
India as they account for almost half the universities and Vidyapeetha, New Delhi
also for 84% of total enrolment. 4. Rashtriya Sanskrit Vidyapeetha, Tirupati
5. National Dairy Research Institute, Karnal
Private Universities 6. Tata Institute of Fundamental Research, Mumbai
A university established through the state or central act Kindly take note of the following points:
by a sponsoring body, namely, a society registered under
the Societies Registration Act, 1860, or any other corre- 1. The top ranking states in terms of the total num-
sponding law for the time being in force, in a state or a ber of universities are Rajasthan, Uttar Pradesh and
public trust or a company registered under Section 25 of Tamil Nadu.
the Companies Act, 1956, is called a private ­university. 2. The 11th Five-Year Plan envisaged the establishment of
Private universities are competent to award degrees 14 world class central universities (renamed as innova-
as specified by the UGC under Section 22 of the UGC Act tive universities aiming at world-class standards).
with the approval of the statutory councils, wherever 3. The P. N. Tandon Committee in 2009 suggested black-
required, through their main campus. listing 44 deemed universities, saying that they lacked
The first private university, set up in 1995, was the the required quality.
Sikkim Manipal University of Health, Medical and 4. In 2015, the UGC asked 10 deemed universities includ-
Technological Science, Gangtok. ing BITS Pilani to shut their off-campus centres.
5. In February 2016, the UGC amended its regulation
Deemed-to-be University allowing private deemed universities to have up to
A deemed-to-be university, commonly known as a six off-campuses. This ceiling would not apply in case
deemed university, refers to a high-performing institu- of government-established and managed deemed
tion, as declared by the central government under Section universities.
3 of the UGC Act, 1956. The central universities, deemed-to-be universi-
Deemed universities can be approved only by an exec- ties are largely funded by the federal government
utive order after the UGC’s recommendation. Although and the funds are channelled through the UGC. The
they enjoy all the powers of a university, they do not have institutions of national importance and the National
the right to affiliate colleges. Open University are directly funded by the Central
government.
1. The Indian Institute of Science (IISc), Bengaluru, and
Indian Agricultural Research Institute, Delhi, were
the first two institutes to be granted a deemed status. Stopover
The IISc was granted the status in 1958, though it was In which of the following states has the 50th central

set up in the year 1908. university been set up?
2. The Manipal Academy of Higher Education (MAHE)
(a) Tripura (b) Andhra Pradesh
was the first private institution to be declared a
deemed university in 1976. (c) Telangana (d) Goa
The correct option is (c).
The following institutions of higher learning are few
prominent examples of a deemed-to-be university:
1. National University of Educational Planning and Meta University
Administration (NUEPA), New Delhi During the 12th Plan, the UGC initiated a concept of Meta
2. Rashtriya Sanskrit Sansthan, New Delhi University.
Meta Universities are basically

M10_MADAN 07_65901_C10.indd 23 23/12/22 8:05 PM


10.24 Chapter 10

1. Second-generation universities
Concept Box 2. Free from physical boundary conditions and able to
operate in virtual space
Rashtriya Uchchatar Shiksha Abhiyan (RUSA)
3. Taking advantage of the innovation and flexibility
RUSA was launched in 2013. It has been given approval possible in some domains
for its continuity by the Cabinet Committee on Economic
Affairs as Centrally Sponsored Scheme from April 2017 The University of Delhi and Jamia Millia Islamia are
to March 2020. It aims at providing strategic funding to the first two universities of India to have adopted the con-
eligible state higher educational institutions. cept of Meta University. A 2-year joint degree programme
‘Master of Mathematics Education’ (equivalent to M.Sc.
Funding Mathematics Education) started from 2015 session.
• The central funding to the states and UTs would be
norm based and outcome dependent. It is in the ratio cluster innoVation centre (cic)
of 60:40 for general category states, 90:10 for spe- The CIC is a Government of India – funded institute estab-
cial category states and 100% for union territories. lished under the aegis of the University of Delhi. It was
• The funding would flow to the State Higher Edu- founded in 2011 and introduced innovation as a credit-
cation Councils before reaching the identified based course for the first time in India.
institutions.
• The funding to states would be done on the basis of other higher leVel institutions
critical appraisal of State Higher Education Plans. Inter-University Centres (IUCs)
It would take into account each state’s strategy to
The UGC has established autonomous IUCs within the
address issues of equity, access and excellence in
university system with an objective to provide common,
higher education.
advanced, centralized facilities and services for univer-
Objectives sities, in order to offer the best expertise in each field to
teachers and researchers across the country.
• RUSA would create new universities through At present, there are seven IUCs functioning within
upgradation of existing autonomous colleges and the university system under Clause 12(ccc) of the UGC
conversion of colleges in a cluster. Act:
• It would create new model degree colleges and
new professional colleges, and provide infrastruc- 1. Inter-University Accelerator Centre (IUAC), New
tural support to universities and colleges. Delhi—the first one to be established in 1994
• The faculty needs and improvement programmes 2. Inter-University Centre for Astronomy and Astro-
are part of RUSA. Physics (IUCAA), Pune
• A separate component to synergize vocational edu- 3. UGC-DAE Consortium for Scientific Research (UGC-
cation such as polytechnics with higher education DAECSR), Indore
has also been included in RUSA. 4. Information and Library Network (INFLIBNET),
• RUSA also supports reforming, restructuring and Ahmedabad
building capacity of institutions in participating 5. Consortium for Educational Communication (CEC),
states. New Delhi
6. National Assessment and Accreditation Council
(NAAC), Bengaluru
7. Inter-University Centre for Teacher Education,
Kakinada

INI Acts—Year of Declaration Association of Indian Universities (AIU)


• The AIU is a forum of administrators and academi-
All India Institute of Medical Sciences (amendment)
cians of member universities to exchange views and
Act, 2012
discuss matters of common concern.
Institutes of Technology Act, 1961 • The idea originated during the Vice Chancellors’ Con-
National Institutes of Technology, 2007 ference at Shimla in 1924 that was convened by Lord
Reading. It got its present name in 1973.
Rajiv Gandhi Institute of Petroleum Technology Act, 2007 • The members include traditional universities, open uni-
Indian Institutes of Information Technology Act, 2014 versities, professional universities, Institutes of National
Importance (INI) and deemed-to-be universities.
School of Planning and Architecture Act, 2014 • There is a provision of granting associate membership
National Institute of Design Act, 2014 to universities of neighbouring countries.
• It brings out many useful publications such as Univer-
Footwear Design and Development Institute Act, 2017 sities Handbook, research papers and a weekly journal
Indian Institute of Management Act, 2017 titled University News.

M10_MADAN 07_65901_C10.indd 24 23/12/22 8:05 PM


Higher Education System 10.25

Institutes of National Importance (INIs) Institutes of Eminence (IOE)


As the name indicates, such institutions enjoy the special India lacks in international rankings. There are different
status of national importance. reasons for that. Over a period, the government has done
INI status is conferred to a public institution of Indian many efforts to supplement the gap.
higher education by an Act of Parliament. The Institutes of Eminence scheme comes under the
INI receive special recognition and funding from the Ministry of Education to project Indian institutes to get
Government of India. global recognition. The selected institutes will enjoy com-
INIs can be sub-grouped under different specialized plete academic and administrative autonomy. The selec-
sectors, namely, agricultural sciences, design, engineer- tion shall be made through challenge method mode by
ing, food processing, general, information technology, the Empowered Expert Committee (EEC) constituted for
language studies, management, medicine, pharmacy, sci- the purpose.
ence and engineering, sciences and youth affairs. The ranking of institutions has been done by the UGC
The most of such INIs are the AIIMS, IITs, IIMs, IIITs as per the recommendations of QS-2020 World Rankings.
and NITs which have been discussed further also.
Objectives
Indian Institutes of Science Education and Research
(IISERs); They are a group of premier public research • Excellence and innovation
institutions in India. The institutes were established by the • Specialization (distinctive contributions)
Government of India through the Ministry of Education. • Global rating
They provide collegiate education in basic sciences coupled • Quality teaching and research
with research at the undergraduate level. The institutes
were formally established by the Parliament of India Criteria
through an amended act in 2010. Seven IISERs have been Eligibility: Only higher education institutions currently
established as IISERS at Pune, Kolkata, Mohali, Bhopal, placed in the top 500 of global rankings or top 50 of the
Thiruvananthapuram, Tirupati, and Berhampur. All National Institutional Ranking Framework (NIRF) are eli-
IISERs were declared as Institutes of National Importance gible to apply for the eminence tag.
by the Parliament of India in 2012. The private IOE can also come up as greenfield ven-
More examples: tures, provided that the sponsoring organisation submits
a convincing perspective plan for 15 years. Private institu-
1. All India Institutes of Medical Sciences, New Delhi tions will not get any financial support, but they will be
2. Indian Institute of Technology, Mumbai entitled for more autonomy as a special category Deemed
3. Indian Institute of Management, Ahmedabad University.
4. Indian Institute of Food Processing Technology, The greenfield institutions would get a period of 3
Thanjavur years to establish and operationalize the institution.
5. Indian Statistical Institute, Kolkata The UGC has nominated 20 institutions (10 public and
10 private institutions) for IOE in August 2019. The initial
Let us have a look at the number of popular categories:
rankings have 15 institutions each from public and pri-
• IISER: 7 vate sector.
• AIIMS: 7 The top three declared IOE from the public sector are
• IITs: 23 the following:
• NITs: 31 1. IIT Bombay (INI)
• IIMs: 20 2. IIT Delhi (INI)
• National Institute of Pharmaceutical Education and
3. IISC, Bengaluru (deemed university)
Research (NIPER): 7
• School of Planning and Architecture (SPA): 3 The top three declared IOE from the private sector are
• Indian Institute of Engineering and Technology (IIET): 1 the following:
• National Institute of Design: 1
1. BITS Pilani, Rajasthan
There are some other institutions also. The total num- 2. MAHE
ber of institutions is 141. 3. Jio Institute (Reliance Foundation, Maharashtra)—
The IIITs are autonomous institutions offering tech- greenfield project (yet to be started)
nical education focused on the information technology
and communication studies. They have gained popularity Nominations have been made on the recommenda-
among students and are considered among the top col- tions of the EEC headed by N. Gopalaswami.
leges only after IITs and NITs. Implications
However, there are a total of 25 IIITs in India, 5 of
which are listed as the INI while the others are set up on • Autonomy: Institutes with IOE tag will be given
the PPP model. greater autonomy and freedom to decide fees, course
durations and governance structures to enable them
to vault of the top global institutions. They would be

M10_MADAN 07_65901_C10.indd 25 23/12/22 8:05 PM


10.26 Chapter 10

able to admit foreign students and recruit faculty from


abroad. Concept Box
• Grant: The public institutions under IOE tag will
Classical Languages
receive a government grant of `1000 crore, while the
private institutions will not get any funding under the Sanskrit, Tamil, Telugu, Kannada, Malayalam and
scheme. Odia are the six classical languages in India. Tamil was
the first language to be assigned the status of classical
language in 2004. Odia was the last one to be assigned
Stopover the status in February 2014, but it is the first language
Which of the following statements applies in the con- from the Indo-Aryan linguistic group to be assigned
text of institutions selected as Institute of Eminence? the status.
The Sahitya Academy’s Expert Committee gave the
1. The institute enjoys complete academic and
following four criteria for a classical language:
administrative autonomy.
2. The ranking of institutions has been done by the 1. There should be high antiquity of early texts/
UGC as per the recommendations of QS-2020 recorded history of over 1500–2000 years.
World Rankings. 2. There should be a body of ancient literature/texts
3. IIT Bombay is the first such institute from public that is considered a valuable heritage by genera-
sector. tions of speakers.
4. The private institute from greenfield category will 3. The literary tradition should be original and not
also get the financing benefit. borrowed from another speech community.
4. The classical language and literature should be dis-
Codes:
tinct from the modern and there may also be a dis-
(a) 1, 2 and 3 (b) 2, 3 and 4
continuity between the classical language and its
(c) 1, 3 and 4 (d) 1, 2, 3 and 4
later forms of offshoots.
The correct option is (a).
The proposals are made by the Ministry of Culture.
Once a language is declared classical, it gets financial
Research Councils assistance for setting up a centre of excellence for the
1. Indian Council of Social Science Research (ICSSR), study of that language and also opens up an avenue for
New Delhi two major awards for scholars of eminence. Besides,
2. Indian Council of Philosophical Research (ICPR), the UGC can be requested to create to begin with at
New Delhi least in central universities a certain number of profes-
3. Centre for Studies in Civilizations, Project of History of sional chairs for classical languages for scholars of emi-
Indian Science, Philosophy and Culture (PHISPC) nence in that language.
4. Indian Council of Historical Research (ICHR), The fathers of the Constitution conferred Sanskrit
Guwahati special status by Article 351 as it was the primary
5. National Council of Rural Institutes (NCRI), Hyderabad source language for many languages including Hindi.
Five languages in the world, namely, Chinese, Sanskrit,
Arabic, Greek and Latin, have been assigned the status of
Language universities classical languages.
India has six language universities, out of which three
are deemed-to-be universities and three are central
universities. The deemed-to-be universities are for the
education
promotion of Sanskrit and the three central universities Subordinate Offices Under the Bureau of
are, one each, for the promotion of English and foreign Language Education
language, Hindi and Urdu.
The UGC is providing funds to the following language 1. Central Hindi Directorate, New Delhi, was set up
universities: in the year 1960 to develop Hindi as a link language
throughout India, in pursuance of Article 351 of the
1. Shri Lal Bahadur Shastri Rashtriya Sanskrit
Constitution of India. Its regional offices are located
Vidyapeetha, New Delhi
in Chennai, Kolkata, Hyderabad and Guwahati.
2. Rashtriya Sanskrit Vidyapeetha, Tirupati
2. Commission for Scientific and Technical
3. English and Foreign Languages University, Hyderabad
Terminology, New Delhi: It was constituted to evolve
4. Mahatma Gandhi Antarrashtriya Hindi
and define scientific and technical terms in Hindi and
Vishwavidyalaya, Wardha
in all Indian languages.
5. Maulana Azad National Urdu University, Hyderabad
3. Central Institute of Indian Languages (CIIL),
6. Rashtriya Sanskrit Sansthan, New Delhi
Mysore, was set up in 1969 to help in evolving and
FeW imPortant o FFices or a gencies in higher implementing the language policy of the Government
of India and to coordinate the development of Indian

M10_MADAN 07_65901_C10.indd 26 23/12/22 8:05 PM


Higher Education System 10.27

languages by ­conducting research in areas of lan- is linked to funding year-wise a number of institutions
guage analysis, language pedagogy, language tech- accredited.
nology and language use in the society. It is important to note that although accreditation is vol-
4. Regional Language Centres (RLC) located at untary in India, some states, such as Karnataka and Tamil
Bhubaneswar, Pune, Mysore, Patiala, Guwahati, Nadu, have made it mandatory, especially for professional
Solan and Lucknow work for the implementation of institutions. Despite this, only very few institutions are
the three-language formula of the government and accredited. In fact, only 36% of engineering and 10% of
for the preparation of instructional materials. management programmes have been accredited by the NBA.
5. National Testing Service (NTS) was approved by the The government is in the process of creating a single
MHRD in 2006–07 and implemented by the Centre independent body to regulate various aspects of higher
of Testing and Evaluation (CT and E) under the CIIL, education. The same should be done at the earliest.
Mysore. However, due care needs to be taken to ensure that it gets
6. Linguistic Data Consortium for Indian Languages adequate independence and autonomy.
(LDC-IL): A central sector scheme was implemented by
the CIIL, Mysore, from the financial year 2007–2008.
7. National Translation Mission (NTM): On the basis Non-conventional Education
of recommendations of the NKC, the MHRD set up
the NTM with the main objective of functioning as a
Open and Distance Learning
clearinghouse for all translation activities, both theo- ‘Distance education’ is basically part of ‘non-conventional’
retical and practical, in as many Indian languages as education that has been discussed earlier.
possible. The CIIL, Mysore, is the nodal organization According to the MHRD, Open and Distance Learning
for operation of the scheme. (ODL) system is a system wherein teachers and learners
8. National Book Trust was established in 1957 with the need not necessarily be present at either same place or
objective of promoting a culture of reading in the soci- same time and is flexible in regard to modalities and tim-
ety by publishing good literature at affordable price in ing of teaching and learning as also the admission criteria
all major Indian languages, including English, and by without compromising necessary quality considerations.
undertaking book promotion activities, such as orga- Open learning is a philosophy. The lesser the restric-
nization of seminars, workshops, book fairs and book tions, the higher the degree of openness. The distance
exhibitions in India and abroad. education is the mode used for translating it into reality.
Both are complementary to each other.
Distance learning refers to all the teaching and learn-
Accreditation in Higher Education ing arrangements in which the learner and the teacher
Higher education sector ensures quality of the educa- are separated by space and time.
tional process with the help of accreditation agencies Transaction of the curriculum is effected by means of
established for the purpose. self-study learning materials.
Technology also helps in it. Communication between
National Assessment and Accreditation Council the institution, teacher and learners is mainly through
electronic media (telephone, interactive radio counselling,
NAAC is an autonomous body established in 1994 by the
teleconferencing, video conferencing, chat sessions, email,
UGC and has its headquarters in Bengaluru. It was estab-
website, etc.) and also through postal correspondence and
lished as per recommen­dations of the NPE (1986).
limited face-to-face contact sessions held at study centres.
The prime function of NAAC is to assess and accredit
The major objectives of the DE system are as follows:
institutions of higher learning, universities and col-
leges or their departments, schools, institutions, pro- 1. To democratize higher education to large segments of
grammes, etc. the population
It regularly publishes manuals and promotion materi- 2. To provide an innovative system of university-level
als for assessment and accreditation. education
3. To provide an opportunity for upgradation of skills
N ational Board of Accreditation (NBA) and qualifications
Set up in 1994, the NBA is an autonomous body estab- 4. To develop education as a lifelong activity to enable
lished by the AICTE to conduct periodical evaluation of people to update their knowledge
technical courses offered in India. It has the authority to
recognize or derecognize institutions or programmes. India has one of the largest DE systems in the world
The accreditation process is not linked to funding. after China.
There are the following types of institutions offering DE:
Accreditation Board (AB) 1. National Open University
The AB was set up by the Indian Council of Agricultural 2. State Open Universities
Research (ICAR) in 1996 with a mandate to accredit 3. Distance Education Institutions (DEIs) at:
­agricultural institutions. Accreditation done by the AB is (a) Institutions of national importance
generally valid for a period between 5 and 10 years and (b) Central universities

M10_MADAN 07_65901_C10.indd 27 23/12/22 8:05 PM


10.28 Chapter 10

(c) State universities coordinator of the Open and Distance Education system
(d) Deemed-to-be universities in the country. The distance Education Council (DEC)
(e) State Private Universities was set up by IGNOU in 1991 as a statutory mechanism
4. DEIs at Stand-alone Institutions under the IGNOU Act. It became operational in February
(a) Professional associations 1992.
In August 2010, the MHRD constituted a committee
(b) Government institutions
under the chairmanship of Prof. Madhava Menon in the
(c) Private institutions context of regulation of standards of education imparted
through distance mode. The MHRD accepted the rec-
historical deVeloPments in distance ommendations of Madhava Menon Committee for the
education in india creation of a new regulatory body for ODL system, that
In 1962, the Delhi University started the ‘School of is, the Distance Education Council of India (DECI). The
Correspondence Courses and Continuing Education’. Committee also recommended to shift the DEC of IGNOU
Subsequently, the Education Commission (1964–66) to the UGC. It was done in 2012.
under the chairmanship of Dr D. S. Kothari also perceived
correspondence education as an answer to the increasing Stopover
pressure of numbers as well as the growing financial pres-
sures on the universities. Which of the following is the biggest university in India?
(a) Indira Gandhi National Open University, New Delhi
• The 1970s saw the growth and spread of the corre- (b) Jawaharlal Nehru University, New Delhi
spondence education system in India through corre- (c) Dr. B.R. Ambedkar University Open University,
spondence courses. Hyderabad
• With increased growth and popularity, the govern- (d) Nalanda Open University, Patna
ment introduced Open University System (OUS) in The correct option is (a).
the 1980s. The MHRD in its NPE, 1986, gave promi-
nence to an OU system as a means to ‘augment oppor-
tunities for higher education and as an instrument of
democratizing education’. Thus, the vision of OUS is Concept Box
different from that of conventional universities.
• Dr. B.R. Ambedkar Open University was set up in Sakshat
Hyderabad in 1982. This one-stop education portal was launched on 30
October 2006 to facilitate lifelong learning for stu-
i ndira gandhi national o Pen uniVersity dents, teachers and employees and for those in pur-
suit of knowledge free of cost. The content develop-
IGNOU is a central university located in New Delhi, India.
ment task for Sakshat was looked after by the Content
IGNOU was founded to serve the Indian population by
Advisory Committee (CAC). The National Knowledge
means of distance and open education, providing quality
Network (NKN) interconnects all universities, librar-
higher education opportunities to all segments of society.
ies, laboratories, hospitals and agricultural institutions
It has a large number of programmes, ranging from purely
for sharing data and computing resources across the
academic to technical, professional and vocational at vari-
country over a high-speed information network having
ous levels.
gigabyte capabilities.
IGNOU makes use of the information and communica-
tion technologies (ICTs) extensively for imparting educa-
tion. In addition to self-instructional printed materials,
the university utilizes audio/video programme tapes, state o Pen uniVersities
teleconferencing, Gyan Vani (FM Radio), Gyan Darshan Presently, there are 13 State Open Universities in India,
(educational TV channels) and computer networks for which are single-mode institutions. This means that they
imparting instructions. It also has ‛One Stop Education provide education only in the distance mode. These uni-
Portal SAKSHAT’. versities cater to people who are unable to pursue regular
With active enrolment of over 4 million students, courses due to various reasons. The list of total 14 univer-
IGNOU is the largest university in the world. sities is as follows:
IGNOU hosts the Secretariats of the SAARC Consortium 1. Indira Gandhi Open University, Delhi
on Open and Distance Learning (SACODiL) and the 2. Dr. B.R. Ambedkar Open University, Hyderabad
Global Mega Universities Network (GMUNET), initially 3. Vardhman Mahaveer Open University, Kota, Rajasthan
supported by UNESCO. 4. Nalanda Open University, Patna, Bihar
IGNOU has started a decentralization process by set- 5. Yashwantrao Chavan Maharashtra Open University,
ting up five zones: north, south, east, west and north-east. Nashik, Maharashtra
The 10% of the budget is spent on the north-east states. 6. Madhya Pradesh Bhoj Open University, Bhopal,
Ultimately, IGNOU entered into a dual role—work- Madhya Pradesh
ing as an Open University and also as a promoter and 7. Dr. Babasaheb Ambedkar Open University,
Ahmedabad, Gujarat

M10_MADAN 07_65901_C10.indd 28 23/12/22 8:05 PM


Higher Education System 10.29

8. Karnataka State Open University, Mysore, Karnataka The professional regulatory bodies grant approval for
9. Netaji Subhas Open University, Kolkata establishment of institutes and determine standards for
10. U.P. Rajarshi Tandon Open University, Allahabad the same. Some of the specialized professional bodies are
11. Tamil Nadu Open University, Chennai, Tamil Nadu as follows:
12. Pt. Sunderlal Sharma Open University, Bilaspur,
1. Medical Council of India
Chhattisgarh
2. Dental Council of India
13. Uttarakhand Open University, Haldwani, Uttarakhand
3. India Nursing Council
14. Krishna Kanta Handique State Open University,
4. Council of Architecture
Guwahati
5. Bar Council of India
6. Pharmacy Council of India
7. Indian Council of Agricultural Research (ICAR)
Professional, Technical and Skill 8. Rehabilitation Council of India
development Education in India 9. Central Council of Homeopathy
10. Central Council of Indian Medicine
There is a huge demand for professional, technical and 11. Veterinary Council of India
skill development education in the modern age of science
and technology. The new role for educational institutions Thus, a significant part of the Indian higher education
found it necessary to redefine its goal mainly related to system is regulated. However, there are certain areas that
economic development and to ensure a place for India in are not.
the community of prosperous nations.
Technical Education
Concept Box Technical education imparts knowledge of a specific trade,
craft or profession. It can meet the increasing demands
Commonwealth of Learning (COL) of expanding society and it is multiplying demands and
The COL is an intergovernmental organization estab- development.
lished by the Commonwealth countries in 1988 to To train our students/professionals in response to the
encourage development and sharing of open learn- need of the time, our education system must be reor-
ing and distance education knowledge, resources and ganized to give it the necessary practical and technical
technologies. bias. The Government of India is also very keen for uni-
The COL was hosted in Canada by the Government versal recognition of our education system, and because
of Canada. The major voluntary contributors currently of that, all universities are trying hard to get the NBA
are Canada, India, New Zealand, Nigeria, South Africa accreditation.
and the United Kingdom, where each are entitled to We find that in a zeal to cope with the advancement
seats on the COL’s Board of Governors. The following of science and technology, our technical education system
Indian organizations are partners of the COL for differ- has become primarily skill-oriented and almost deficient
ent purposes: or casual in the education in human values.
1. Indira Gandhi National Open University
2. National Institute of Open Schooling technical education scenario in i ndia
3. National Assessment and Accreditation Council In India, technical education covers programmes in engi-
neering, technology, management, architecture, town
planning, pharmacy, applied arts and crafts, and hotel
management and catering technology.
Profession
1. The first engineering college was established in Uttar
A profession is a vocation founded upon specialized Pradesh in 1847 for training of civil engineers at Roor-
educational training, the purpose of which is to supply kee. It conferred diplomas that were considered to be
counsel and service to others, for a direct and definite equivalent to degrees.
compensation, wholly apart from expectation of other 2. Three engineering colleges were opened by about
business gain. 1856 in three presidencies, namely, Calcutta, Bombay
A profession arises when any trade or occupation and Madras.
transforms itself through ‘the development of formal 3. In Bengal, the leaders of the Swadeshi Movement tried
qualifications based on education, apprenticeship, and to start many institutions. However, only the College of
examinations, the emergence of regulatory bodies with Engineering and Technology at Jadavpur survived.
powers to admit and discipline members, and some 4. Many technical courses were started at the University
degree of monopoly rights.’ of Banaras with great efforts put by Pandit Madan
The need for professional education has only increased Mohan Malviya (1917).
with changing needs of society and economy.

M10_MADAN 07_65901_C10.indd 29 23/12/22 8:05 PM


10.30 Chapter 10

5. Many other courses were also started at the Bengal 1. IIT Kharagpur (1951)
Engineering College at Shibpur in the 1930s. 2. IIT Bombay (set up in 1958 with help from the USSR
6. A number of engineering colleges were established and UNESCO)
since 15 August 1947. It was due to the realization 3. IIT Madras (1959)
that India had to become a great industrial country 4. IIT Delhi (1963)
and would require a large number of engineers than 5. IIT Guwahati (1994)
could be supplied by the older institutions.
Eight new IITs in Patna, Jodhpur, Bhubaneswar,
Hyderabad, Gandhinagar, Indore, Mandi and Ropar were
All India Council for T echnical Education set up in 2008.
• 1943: The Technical Education Committee of the According to the National Plan of Science and
CABE was constituted. Technology, five Centres of Advanced Study and Research
• 1944: The Sergeant Report was prepared. have been set up:
• 1945: The AICTE was formed.
• 1986: As stipulated in the National Policy of Educa- 1. Energy studies: IIT Delhi
tion (1986), the AICTE was vested with the following: 2. Material science: IIT Kanpur
• Statutory authority for planning, formulation and 3. Cryogenic engineering: Kharagpur
maintenance of norms and standards 4. Ocean engineering: IIT Madras
• Quality assurance through accreditation 5. Resource engineering: IIT Bombay
• Funding in priority areas, monitoring and evaluation
• Maintaining parity of certification and awards Indian Institutes of Management (IIM)
• The management of technical education in the The IIMs are a group of 20 public, autonomous institutes of
country management education and research in India. They primar-
• 1987: The AICTE was setup. The purview of the ily offer postgraduate, doctoral and executive education
AICTE (the Council) covers programmes of technical programmes. The establishment of IIMs was initiated by
education including training and research in the fol- Jawaharlal Nehru, the first Prime Minister of India, based
lowing at different levels: on the recommendation of the Planning Commission.
1. Engineering The following is the list of the first six IIMs:
2. Technology
3. Architecture • IIM Calcutta (with support from the MIT Sloan Insti-
4. Town planning tute of Management)—1961
5. Management • IIM Ahmedabad (with support from the Harvard
6. Pharmacy Business School)—1961
7. Applied arts and crafts • IIM Bengaluru—1973
8. Hotel management and catering technology, etc. • IIM Lucknow—1984
• The council discharges its functions through an exec- • IIM Kozhikode—1996
utive committee. • IIM Indore—1996
• The AICTE is headquartered in New Delhi and has In the second and third generations of IIMs, six insti-
regional offices at 10 places—Mumbai, Chennai, tutes each were opened.
Bengaluru, Bhopal, Chandigarh, Kanpur, Kolkata, As we discussed, they have been set up under different
Hyderabad, Guwahati, and Thiruvanthapuram. acts of the Parliament. The Lok Sabha passed the IIM Bill
2017 to make IIMs the INI. It allows IIMs greater autonomy
Council of A rchitecture and empowers them to grant degrees instead of diplomas.
The COA was constituted by the Government of India
under the provisions of the Architects Act, 1972. The Act Indian Institute of Science, Bengaluru
provides for registration of architects and matters con- The IISc, Bengaluru, was started in 1909 through the pio-
nected therewith. neering vision of J. N. Tata.
Ministry of Education F unded Institutions • The Institute has been engaged in higher learning
and advanced research in the fields of science and
- funded Institutions engineering.
Technical education system in the country is broadly • The IISc was the first deemed university in India.
classified into three categories: (i) central government– • Five more institutions devoted to science, education
funded institutions, (ii) state-funded institutions and and research have been set up as Indian Institutes
(iii) self-financed institutions. The centrally funded of Science Education and Research (IISER) broadly
institution of technical and science education are as on the pattern of the IISc at Kolkata, Pune, Mohali,
follows. ­Bhopal and Thiruvananthapuram.

I ndian Institute of T echnology (IIT)


The five initial IITs are the following:

M10_MADAN 07_65901_C10.indd 30 23/12/22 8:05 PM


Higher Education System 10.31

National Institutes of Technology The main objectives of skill development in India are the
• On the recommendations of the erstwhile Planning following:
Commission in 1955, eight Regional Engineering Col- 1. To enhance the employability of youth
leges (RECs; two each in east, west, north and south) 2. To maintain their competitiveness through provisions
were set up in early 1960s. of multi-entry multi-exit learning opportunities and
• Gradually, the number of colleges increased to 17. vertical mobility
• In 2003, RECs were rechristened as NITs and taken 3. To fill the gap between educated and employable
over as fully funded institutes of the central govern- 4. To reduce the dropout rate at the secondary level
ment. They were granted a deemed university status.
Presently, the country faces a demand–supply mis-
• Over a period, the total number of NITs has increased
match, as the economy needs more ‘skilled’ workforce
to 30.
than that is available. We are well aware of the fact that
Indian Institute of Information Technology skills acquired during education are seldom applicable in
industry.
IIITs were specifically set up to meet the manpower
The Ministry of Skill Development and Entrepre­
requirements of the IT sector. The central government
neurship was created in 2014 to coordinate all skill devel-
initially established four IIITs at Allahabad, Gwalior,
opment efforts across the country.
Jabalpur and Kanchipuram. These institutions are meant
National Skills Qualifications Framework (NSQF):
to provide undergraduate as well as postgraduate educa-
The Government of India launched the National
tion. Presently, the total number of IIITs is 25.
Vocational Education Qualification Framework (NVEQF)
List of IIITs which was later on assimilated into the NSQF.
Sector Skill Councils (SSCs): UGC launched this pro-
1. IIIT, Allahabad
gramme in collaboration with Ministry of Education. This
2. Atal Bihari Vajpayee IIIT, Gwalior
involved Qualification Packs (QPs), National Occupational
3. Pandit Dwarka Prasad Mishra IIIT (Design and
Standards (NOSs) and assessment mechanisms in the
Manufacturing), Jabalpur
respective domains of trainees as per needs of the industry.
4. IIIT (Design and Manufacturing), Kanchipuram
National Institute of Technical Teachers’ Training B.Voc. Degree Programme
and Research (NITTTRs) The UGC launched another scheme of B.Voc. degree pro-
Four NITTTRs located at Bhopal, Chandigarh, Chennai, gramme to expand the scope of vocational education and
and Kolkata were established in the mid-1960s for the also to provide vertical mobility to the students admitted
training of polytechnic teachers to undertake activities in into Community Colleges for diploma programmes to a
the areas of education, planning and management. degree programme in the universities and colleges.
Kaushal: Further ‘Deen Dayal Upadhyay Centres
Externally Aided Projects in Technical Education for Knowledge Acquisition and Upgradation of Skilled
Human Abilities and Livelihood (KAUSHAL)’ was also
1. Technical Education Quality Improvement Pro- incorporated.
gramme (TEQIP): It was launched by the MHRD in Kaushal Kendras formulate courses at postgraduate
2002 to upscale and support the ongoing efforts in level keeping in mind the need of
improving the quality of technical education.
2. Technician Education Project-III: It was launched 1. Industry in specialized areas
with the help of the World Bank for the upgradation 2. Instructional design, curriculum design and con-
of polytechnics in the country. tents in the areas of skills development
3. Pedagogy, assessment for skills development educa-
Skill Development in I ndia tion and training
Introduction skills and knowledge are the driving forces 4. Trained faculty in the areas of skill development
of economic growth and social development in any coun- 5. Entrepreneurship
try. India, at present, is recognized as one of the younger Since the above three provisions serve a common purpose,
nations in the world with over 50% of the population they have been merged into a single scheme for providing
under the age of 30 years. The census projection report skill-based education under the National Qualification
shows that the proportion of population in the work- Framework.
ing age group (15–59 years) is likely to increase from Skill India Initiative: This initiative aims to skill 40
approximately 58% in 2001 to more than 64% by 2021. crore people by 2022 through its ‘Skill India’ programme.
It is estimated that by about 2025, India will have 25% Various initiatives under this campaign are the
of the world’s total workforce. Thus, we need to take following:
optimum benefit of 'demographic dividend'. Our train-
ing covers only 10% of the population that is very low in • National Skill Development Mission
comparison to 60-70% of developed nations. • National Policy for Skill Development and Entrepre-
neurship, 2015

M10_MADAN 07_65901_C10.indd 31 23/12/22 8:05 PM


10.32 Chapter 10

• Pradhan Mantri Kaushal Vikas Yojana (PMKVY)— potentially called the CIHEC was established to promote
under the Ministry of Skill Development and and facilitate industry–higher education collaboration.
Entrepreneurship The Indian Institute of Entrepreneurship (IIE): This
• Skill Loan Scheme institute was established in the year 1993 in Guwahati.
• Rural India Skill This is an autonomous national institute with an aim to
undertake training, research and consultancy activities in
Global Initiative for Academic Network (GIAN): GIAN
small and micro-enterprises focusing on entrepreneurship
aims at tapping the talent pool of scientists and entre-
development.
preneurs, internationally, to encourage their engage-
National Institute for Entrepreneurship and Small
ment with the institutes of higher education in India so
Business Development (NIESBUD): It is a society under
as to augment the country’s existing academic resources,
the Ministry of Micro, Small and Medium Enterprises
accelerate the pace of quality reform and elevate India’s
engaged in training, consultancy, research and publica-
scientific and technological capacity to global excellence.
tion, in order to promote entrepreneurship. The insti-
It is proposed to initiate the programme under collabora-
tute has been financially self-sufficient since 2007–08. It
tion with various countries.
is operating from an integrated campus in Noida, Uttar
Skills Assessment Matrix for Vocational
Pradesh.
Advancement of Youth (SAMVAY): SAMVAY is basi-
Community colleges: Skill development has been
cally a credit framework. It allows vertical and lat-
an integral part of education system in many nations
eral mobility within vocational education system and
such as USA, Germany, Canada, UK, Japan, China, etc.
between the current education systems. The strength
In India, idea of starting ‘community colleges’ started
of this framework is the seamless integration of pursuit
in 2012 many nations such as USA, Germany, Canada,
of academic knowledge and practical vocational skills.
UK, Japan, China, etc. At some places skill development
Efforts like these will improve the employability of our
starts at the school level while at other places, it starts
educated youth.
at the college level. More known among them are the
Unnat Bharat Abhiyan: Under this programme, the
community colleges. These colleges provide education
IIT, IISER and NIT adopt villages and develop appropriate
above the secondary level and below the degree level
rural technologies for sustainable development through
with appropriate skills keeping in view the needs of local
peoples’ participation. This will enable processes that
industry and community, thereby leading to gainful
connect institutes of higher education with local commu-
employment and option for horizontal and vertical
nities. Focus is on water management, organic farming,
mobility. While vertical mobility facilities movement
renewable energy, frugal technology, infrastructure and
to higher education and research, horizontal mobility
livelihood.
creates opportunity for entry into the employment
Ishan Uday—Scholarship Scheme for Students of
market with appropriate skills. There is also flexibility for
North–east Region: The UGC has launched a Special
further skill development.
Scholarship Scheme for students of the north-east region,
Skill India Mission—Recent Developments: In
Ishan Uday, from the academic session 2014–15.
2015, Prime Minister launched the ‘Skill India Mission’,
Ishan Vikas—Academic Exposure for North-Eastern
which was in accordance with his vision to help
Students: The programme has been launched with a
India become ‘Atmanirbhar’ (self-reliant). State Skill
plan to bring selected college and school students from
Development Missions and District Skill Committees
the north-eastern states into close contact with IITs, NITs
and successfully implement the Pradhan Mantri Kaushal
and IISERs during their vacation periods for academic
Vikas Yojana were envisaged for seven North East states
exposure.
in April 2022. The Technology Information, Forecasting
Saakshar Bharat—Adult Literacy and Pradhan
and Assessment Council (TIFAC) launched SAKSHAM
Mantri Jan Dhan Yojna: Centrally Sponsored Scheme
(Shramik Shakti Manch), a work portal for mapping
of Adult Education and Skill Development has a special
skills of ‘Shramiks’ (labour) in February, 2022. A pro-
focus on underprivileged groups. Four hundred and ten
gramme called as ‘Specified Skilled Worker’ (SSW) was
districts are covered under the programme.
launched in collaboration with Japan.
PRAGATI—Scholarships for Girl Child for Technical
India envisages to become a five trillion dollars econ-
Education: PRAGATI aims at providing encouragement
omy by 2025. We want to increase our exports to one tril-
and support to girl child to pursue technical education.
lion dollars by 2030.
Swami Vivekananda Scholarship for Single Girl
Child: The UGC has introduced the Swami Vivekananda
Scholarship for Single Girl Child for research in social sci-
ences with an aim to compensate direct costs of higher Value Education
education especially for such girls who happen to be the From a broader perspective, the aim of value education is
only girl child in their family. linked with the fundamental question of what education
Council for Industry and Higher Education itself is meant for:
Collaboration (CIHEC): This relates to creating linkages
between the industry and the academia. A nodal agency 1. Individual’s Perspective: To enable students to
achieve personal fulfilment for success in life and work

M10_MADAN 07_65901_C10.indd 32 23/12/22 8:05 PM


Higher Education System 10.33

2. Societal Perspective: To prepare the youth to contrib- Globalization


ute to the society, nation and world Societies have become less and less mono-­ cultural.
Plato wrote in The Republic, ‘The mark of an educated per- Consequently, the pluricultural environment in which we
son is the willingness to use one’s knowledge and skills to live now is more complex and multiple with different cul-
solve the problems of society.’ tures developing.
Diversity
Key Developments in Value Education
Universality and diversity may seem incompatible, but
1. National Commission of Secondary Education
both have to coexist in a democratic and diverse society
(1952–53) emphasized character-building as the
such as ours.
defining goal of education.
2. University Education Commission (1962) noted, ‘If Inclusivity
we exclude spiritual training in our institutions, we
A healthy, happy society is one in which all its members
would be untrue to our whole historical development.’
feel included.
3. Education Commission of 1964–66 put the spotlight
on education and national development. Agreeing Environment
with the Sri Prakasa Committee Report, 1959, it rec-
The reckless exploitation of e­nvironment, depletion of
ommended direct moral instruction for which one or
ozone layer, global warming, industrial pollution, deforest-
two periods a week should be set aside in the school
ation and soil erosion are few problems faced by humanity.
time table.
4. NPE (1986) advocated turning education into a Exploding World of Science and Technology
forceful tool for the cultivation of social and moral
How science and technology are used is a question of
values. Education should foster universal and eternal
values. Application of science and technology in a more
values, oriented towards the unity and integration of
humane and rationale way is related to moral and ethical
our people.
responsibility.
5. NPE—Programme of Action (1992) tried to inte-
grate the various components of value education Mass and Social Media
into the curriculum at all stages of school education,
A major ubiquitous aspect of contemporary society is the
including the secondary stage.
intrusion of mass media into day-to-day life of all societies.
6. Chavan’s Committee Report (1999) provided impe-
The values and attitudes that get transmitted are rather con-
tus to resume work on value orientation of education.
trary to the values desired by the family, society or school.
7. The National Curriculum Framework for School
Education (2000) advanced a plea to integrate value All India Survey on Higher Education (AISHE)
education into the curriculum.
To portray the status of higher education in the country,
8. National Curriculum Framework (2005) articu-
the MHRD has endeavoured to conduct an annual web-
lates the need to reaffirm our commitment to the
based AISHE since 2010–11. The survey covers all the
concept of equality amidst diversity, mutual interde-
institutions in the country engaged in imparting of higher
pendence of humans to promote values that foster
education. Data is being collected on several parameters
peace, humaneness and tolerance in a multicultural
such as teachers, student enrolment, programmes, exam-
society. The NCF, 2005, particularly emphasizes edu-
ination results, education finance and infrastructure.
cation for peace as one of the national and global
Indicators of educational development such as Institution
concerns.
Density, Gross Enrolment Ratio, Pupil–Teacher Ratio
(PTR), Gender Parity Index and Per Student Expenditure
Main I ssues behind the Need for I mparting
are also calculated from the data collected through the
Value Education AISHE. These are useful in making informed policy deci-
Tradition versus Modernity sions and research for the development of education sector.
Developing societies, such as India, face a conflict AISHE Report (2020–21)
between tradition and modernization. It is important to • Institutions are categorized into three broad catego-
make young learners develop attitudes where they do not ries; university, college and stand-alone institutions.
see everything in tradition as bad or everything in mod- • A total of 962 universities, 38,179 colleges and 9190
ernization as good. There are many things positive about stand-alone institutions have responded during the
our traditional culture, which need to be appreciated and survey. A total of 298 universities are affiliating, that
understood, such as tolerance of dissonance, harmony is, having colleges.
rather than control over environment, collectivism and • Sixteen of the universities are exclusively for
self-definition in a social context emphasizing modesty, women—three in Rajasthan, two in Tamil Nadu and
cooperation, duty, acceptance and so on. one in many others.
• In addition to 1 Central Open University, 14 State
Open Universities and 1 State Private Open Univer-

M10_MADAN 07_65901_C10.indd 33 23/12/22 8:05 PM


10.34 Chapter 10

Concept Box
National Institutional Ranking Framework (NIRF)
The NIRF has been approved by the MHRD and was launched on 29 September 2015. The first list was released in
April 2016. This framework outlines a methodology to rank institutions across the country. The methodology draws
from the overall recommendations and broad understanding arrived at by the Core Committee set up by the MHRD
(now Ministry of Education), to identify the broad parameters for ranking various universities and institutions.
There are various agencies such as Times Higher Education World University Rankings (THER) and the
Quacquarelli Symonds (QS) ranking agencies to address various ranking parameters. Arizona State University, USA is
the top university in QS, with USA and UK are the most represented nations. India has 15 such institutuions in the QS
rankings, with IITB, IITD and IIS at the top three. India tried to supplement the gap with the introduction of the NIRF.

The following five parameters (weightages in parentheses) were broadly considered:


1. Teaching and learning (0.30)
2. Research and professional practices (0.30)
3. Graduation outcomes (0.20)
4. Outreach and inclusivity (0.10)
5. Perception (0.10)
The institutions were ranked by the National Bureau of Accreditation (NBA). All institutions were judged based
on self-disclosure of information.
Although the ranking frameworks are similar, the exact methodologies are domain specific. Ranking methods
have been worked out for the following categories:
India Ranking 2022: Top 100 in Overall
Name State Rank
Indian Institute of Technology Madras, Chennai Tamil Nadu 1
Indian Institute of Science, Bengaluru Karnataka 2
Indian Institute of Technology Bombay, Mumbai Maharashtra 3

India Ranking 2022: Top 3 Universities


Name State Rank
Indian Institute of Science, Bengaluru Karnataka 1
Jawaharlal Nehru University, New Delhi Delhi 2
Jamia Milia Islamia, New Delhi Delhi 3

India Ranking 2022: Top 3 in Engineering


Name State Rank
Indian Institute of Technology Madras, Chennai Tamil Nadu 1
Indian Institute of Technology Delhi, New Delhi Delhi 2
Indian Institute of Technology Bombay, Mumbai Maharashtra 3

India Ranking 2022: Top 3 Institutions (Management)


Name State Rank
Indian Institute of Management, Ahmedabad Gujarat 1
Indian Institute of Management, Bengaluru Karnataka 2
Indian Institute of Management, Kolkata West Bengal 3

(Continued)

M10_MADAN 07_65901_C10.indd 34 23/12/22 8:05 PM


Higher Education System 10.35

India Ranking 2022: Top 5 Colleges


Name State Rank
Miranda House, Delhi Delhi 1
Hindu College, Delhi Delhi 2
Presidency College, Chennai Tamil Nadu 3
Loyola College, Chennai Tamil Nadu 4
Lady Shri Ram College for Women, New Delhi Delhi 5

sity, there are 110 dual-mode universities, which offer level, maximum students are enrolled in Social Sci-
education through distance mode also and the maxi- ence stream and Management comes at number two.
mum, 13, of them are located in Tamil Nadu. • The share of Ph.D. students is highest in State Pub-
• Uttar Pradesh comes at number one with the high- lic University (34.3%) followed by the INI, Deemed
est student enrolment followed by Maharashtra and University-Private and State Private University.
Tamil Nadu. • The highest share of foreign students comes from
• The maximum universities are from general, techni- Nepal (26.88%) followed by Afghanistan, Bangla-
cal and agriculture & allied. There are 13 Sanskrit and desh, Sudan and Bhutan.
9 language universities. • At the all-India level, there are merely 73 female
• The top eight states in terms of highest number of teachers per 100 male teachers.
colleges are Uttar Pradesh, Maharashtra, Karnataka, • The PTR in universities and colleges is 29 if regular mode
Rajasthan, Haryana, Tamil Nadu, Gujarat and Mad- of enrolment is considered, whereas PTR for universities
hya Pradesh. and its constituent units is 18 for the regular mode.
• Bengaluru Urban district tops in terms of the number • At the post graduate level, MA pass number of stu-
of colleges with 880 colleges followed by Jaipur with dents is maximum followed by M.Sc. and MBA.
566 colleges. • At the Ph.D. level, the maximum number of student
• College density, that is, the number of colleges per out-turn is in Science stream followed by Engineering
lakh eligible population (population in the age group and Technology. On the other hand, at the PG level, the
18–23 years), varies from 7 in Bihar to 53 in Karna- maximum student out-turn is observed in Social Sci-
taka as compared to all India average of 28. ence and Management stream comes at number two.
• Sixty-one per cent of colleges are located in rural
areas. Eleven per cent of colleges are exclusively for
Stopover
females.
• Only 2.5% of colleges run Ph.D. programme and 34.9% Which of the following can be termed the district for

of colleges run post graduate level programmes. maximum number of colleges in India?
• Females constitute 48.6% of the total enrolment. The (a) Jaipur (b) Bengaluru Urban
Gross Enrolment Ratio (GER) in higher education in (c) New Delhi (d) Chennai
India is 26.3%, which is calculated for the age group The correct option is (b).
of 18–23 years. The GER for the male population is
26.3% and for females, it is 26.4%. For Scheduled
Castes, it is 23% and for Scheduled Tribes, it is 17.2% Key Issues Facing Spread of Higher Education
as compared to the national GER of 26.3%. in India
• Distance enrolment constitutes about 10.62% of the The National Policy on Higher Education translated
total enrolment. the vision of the Radhakrishnan Commission and the
• Maximum numbers of students are enrolled in B.A. pro- Kothari Commission into an actionable policy by setting
gramme followed by B.Sc. and B.Com. programmes. five main goals for higher education, which are enumer-
• At the undergraduate level, the highest number ated as follows:
(35.9%) of students are enrolled in Arts/Humanities/
Social Sciences courses followed by Science (16.5%), 1. Access
Engineering and Technology (13.5%) and Commerce 2. Equity—fair access to the poor and the socially disad-
(14.1%). vantaged groups to higher education
• At the Ph.D. level, the maximum students are enrolled 3. Quality and excellence
in Science stream that is followed by Engineering and 4. Relevance
Technology. On the other hand, at the post graduate 5. Value-based education

M10_MADAN 07_65901_C10.indd 35 23/12/22 8:05 PM


10.36 Chapter 10

As per the UGC guidelines, student–teacher ratio should Organizational Structure of U niversities
be 30:1. However, in some of the states, the ratio is as Ancient education in India has been based on Vedanta,
high as 100:1. Buddhism and Jainism, our university system was also
Despite having one of the largest higher education sys- at its peak according to that time, due to which India has
tems in the world, few Indian institutions have earned also been called ‘Jagat Guru’. The Government of India
global distinction. There is no Indian institute among the wants to keep at least some part of the ancient education
world’s top 200. system in the present education system.
The present university system was originally inherited
G overnance and Administration from the British. We can look at the following points for
Governance signifies a change in the meaning of gov- the sake of recall.
ernment, referring to a new process of governing; or a 1. Many institutions such as Calcutta Madrasa (1781),
changed condition of ordered rule; or the new method by Asiatic Society of Bengal (1784), Banaras Sanskrit
which society is governed. College (1791), Fort William College (1800), Ser-
Governance can be defined in the following ways: ampore College (1818) etc. were established.
• As the minimal state 2. Indian education was declared one of the duties of the
• As corporate governance state by the British Parliament under Charter Act of
• As the new public management 1813.
• As ‘good governance’ 3. Then British education was given priority by ‘Minute
• As international interdependence of Macaulay’ (1835).
• As a socio-cybernetic system 4. Three universities were established in Bombay,
• As networks Calcutta and Madras in the year 1857 as per sugges-
tions by the Charles Wood Report. They were based
As governance has been put under higher education, we mainly on the University of London, which was our
are discussing some constitutional aspects (as the minimal role model. The University of Allahabad was estab-
state) linked with higher education minimal aspects. lished in the year 1887, focused more on gradu-
In the following paragraphs, we will focus on some ate studies and as a peripheral adjunct to centres of
constitutional aspects of governance. advanced university education. The British govern-
ment used the university system as an instrument of
Governance of Higher Education ‘cultural colonization’.
5. British control over the Indian education system con-
System tinued until the Government of India Act 1935. This
act transferred more power to the provincial politi-
Earlier we discussed about UGC, universities, institutions
cians and the ‘Indianisation’ of education started.
of national importance, NAAC, NBA etc. They are linked
with the governance aspect as well. Good governance After Independence Scene: These happenings are in
basically focuses on the process of governance, which is continuity with before independence scenario. The goal
dependent on a loop of dialogue between various formal of higher education was not achieved till independence.
and informal institutions. We have already described Till that point, higher education was meant for elite sec-
the policies and politics in Indian education under the tions of society. Many changes were inevitable to fulfil the
recommendations of various committees, history, new aspirations of Indian public after independence.
education policy. There is a need to influence policies and Many new universities were set up for the sake of
decisions related to public life through good governance. expansion, many diverse courses were also added in
The success of governance depends on the restructur- professional studies—IITs, BITS-Pilani, regional engineering
ing of the government with a social objective, the revi- colleges and IIMs were set up as highly rated educational
talization of the non-government sectors. Political will, institutions. National Institutes of Technology (NITs),
regulatory and organizational flexibility are needed. We Indian Institutes of Science and Indian Institutes of Science
need to critically examine whether policies are being Education (IISc) and Research (IISERs) were developed.
implemented efficiently and honestly by government The languages and humanities got importance until
agencies and organizations which are responsible for per- the 1980s. In 1990s and 2000s new IITs and IIMs were
forming assigned tasks. Good governance requires mean- set up. Despite all the expansion of higher education, no
ingful cooperation of various elements of civil society and significant change was observed in the organizational
a multi-pronged strategy. structure of the universities which most of the Indian
State, private sector and NGOs, especially community universities followed the principle of affiliation. They
based organizations should cooperate and coordinate intended to provide efficient governance for their
with each other to make good governance possible. Good constituent and affiliated colleges. Thus, administrative
governance helps in development, it can bring change in bodies, otherwise known as statutory bodies, have also
the society. It has a minimum state, a socio-cybernetic been constituted on similar principles. We will discuss
systems, self-organized networks and some corporate them in the following manner. In general, we can discuss
governance. two types of universities.

M10_MADAN 07_65901_C10.indd 36 23/12/22 8:05 PM


Higher Education System 10.37

1. Unitary Universities: The examples are Banaras 2. Non-recurring Grants: These grants are normally
Hindu University, Hyderabad University, Pondicherry used for buildings, equipment etc. In general, land for
University. The teaching and research is conducted in a a university is given by the State government.
single campus (sometimes with subsidiary campuses) 3. Development Grants: These are meant for the new
and there is a provision for both undergraduate and schemes sponsored by the federal/state governments
postgraduate teaching, as also for research. and a university generally gets these grants on a
2. Affiliating Universities: This is mostly with state matching basis for various activities by the UGC.
universities. They have central campus on which there
The Central government provides grants to universi-
are departments or schools that impart postgraduate
ties and colleges for general education, and are for the
instruction and conduct research. They have a
development purposes, which are channelised through
variable number of colleges affiliated to them and
the UGC. When the onus of providing grants shifts on to
these may be distributed over a number of districts, in
states, they may start facing trouble.
accordance with the jurisdiction of the university.
The majority of Indian Universities are funded by Statutory Bodies of Universities
Central or State governments. Some of the deemed
universities are sponsored by private trusts. These Every Indian university is governed by its various statutory
include Birla Institute of Technology and Science, Pilani, bodies. In the case of Central Universities, the Central
Thapar University, Patiala etc. In few states, private Government is represented by the nominees of the Central
universities have been set up under Acts of Legislatures Government and in the case of State Universities, the State
such as Sikkim-Manipal University of Health, Media Government is represented by persons nominated by the State
and Technological Sciences in Sikkim and the Amity Government in the Governing Bodies of these Universities.
University in Chattisgarh. The following bodies control Indian universities:

Colleges: The bulk of undergraduate teaching is done in 1. Court/Senate


colleges. They are of two types—the constituent colleges 2. Executive Council/Syndicate/Board of Management
and the affiliated colleges. 3. Academic Council
4. Board of Studies
1. Constituent Colleges (or Conducted Colleges): 5. Finance Committee
These are established and managed by the University. 6. Boards or Committees
2. Affiliated Colleges: They are set-up and managed,
outside the university campus, either by the Now, we shall discuss each of these bodies, their constitution
government or by educational trusts. The power of and functions as well as the powers vested by them for the
granting affiliation to a college usually vests with the effective functioning of the university system.
universities and this power is generally exercised in 1. University Court/Senate
consultation with the concerned government. These are the supreme governing bodies of the universities.
Source of Funding: The income of a university in India, is In case of Central Universities. It is known as Court, while
derived from the following sources: in State Universities it is known as Senate. Such a body
does not exist in the case of IGNOU. In general, each
1. Grants from the Central Government Court/Senate of universities has about 100 or so members.
2. Grants from the State Governments It consists of some ex-officio members, representatives
3. Contributions from students’ fees of various professions/industries, principals of affiliated
4. Endowments, donations, etc. colleges, students, non-teaching staff, alumni and some
5. Financial support from different specialised agencies teachers, nominated members.
The main functions are as follows.
Types of Grants 1. This supreme statutory body university has the power
It is always interesting to look into the types of grants to take all the major decisions.
received by higher education institutions in the country. 2. It controls the budget of the University, considers the
The State government grant to the universities can be annual report and passes resolutions thereon.
grouped into three different heads: 3. It reviews the Acts of the University and examines the
role of the principal bodies of the University.
1. Maintenance/Block Grants: These grants are meant 4. It amends and repeals Acts as and when necessary.
for the routine working of the institutions. This 5. It passes ordinances to establish new colleges and
head includes the salaries of the teaching and non- maintain both new and old ones.
teaching staff. These grants are determined by the 6. It also supervises the establishment of teaching and
state governments for a specified period, they usually administrative posts, and is responsible for awarding
increase as per changes in the costs. The quantum of degrees.
grants is based on the net deficit of the university, 7. The Court/Senate may delegate its power to any
depending upon demand for approved and rise in authority or any body of the University as it may
costs. deem fit.

M10_MADAN 07_65901_C10.indd 37 23/12/22 8:05 PM


10.38 Chapter 10

2. Executive Council/Syndicate/Board of Management Quorum at a meeting of the Board of Studies shall be


In central universities, the principal governing body is 5 members present in person.
known as the Executive Council; In state universities 5. Finance Committee
it is called Syndicate/Executive Council or Board of The Vice-Chancellor of the University is the ‘Ex-officio
Management. Chairman of this Committee’. Generally, the members
1. All the principal office bearers of the University are of this committee include the Visitor/Vice Chancellor,
ex-officio members of this body. It has representatives Government and the Executive Council/ Syndicate/
from both the principals and faculty of alumni, as well Management Board nominees.
as government representatives. Some prominent per- The Finance Officer of the University is ordinarily the
sons from the field of education are also nominated to Member Secretary of the Finance Committee, the Finance
the council. Officer of the University in the capacity of Member
2. Manages and regulates the accounts, investments, Secretary of the Finance Committee prepares the annual
property, business of the University and all other accounts and places the same before the Finance Committee
related administrative matters. for consideration and comments, and thereafter to the
3. It passes ordinances for constituent colleges, and Executive Council. Submits for approval of Senate/Board of
these councils/syndicates/boards of management Management. The Finance Officer exercises general control
finance, fund, oversee the effective management of over the funds of the University and advises the Finance
the respective colleges. Committee and the officials of the University on its financial
4. It also controls and maintains the Publications Divi- policies. He performs those functions which are assigned to
sion and the University Press. They also have the him by the Executive Council/Syndicate.
opportunity to hear and redress the grievances of the Subject to the control of the Executive Council/
employees of the University. Providing opportunities Syndicate, the Finance Officer protects and manages the
for their entertainment also comes under their scope. assets, and oversees the investments, including trusts and
endowed assets, to further any purpose of the University.
This body also has the power to delegate to the Committee The officer also sees that the limit fixed by the Finance
the right to consider certain matters of the University for Committee on recurring and non-recurring expenditure
the effective and efficient management of the University. for a year does not exceed and all money is spent on
the purposes for which they were originally granted or
3. Academic Council allotted.
All senior officers of the University including the Dean and
Head of the Department of Studies/ School are ex-officio 6. Boards or Committees
members of an Academic Council. Representation The above statutory bodies of the University may appoint
for ex-officio membership is generally based on the boards or committees to deliberate on certain issues.
principle of rotation. In some universities, students also These boards or committees may have members from
get representation in this body. If a university has its the respective statutory bodies or from outside. The main
constituent affiliated colleges, the principals of these function of such a committee is to deliberate and make
colleges are also representatives of this body. The main recommendations on the matter entrusted to it, subject
responsibility of the academic council of a university is to to subsequent confirmation by the body appointing it.
make important decisions on the programs and courses to It is clear from the above discussion that the statutory
be offered to the students. It also includes maintenance bodies of the University have been entrusted with a broad
of standards in instruction, examination, evaluation etc. framework of responsibilities for its smooth functioning.
But there are some instances where these bodies exceed
4. Board of Studies the limits of authority assigned to them and do not take
Universities have study boards in various subjects. They any action in respect of subjects which fall within their
mainly consist of heads of various departments of the purview.
university, heads of departments of affiliated colleges
and teachers and representatives of co-opted members.
In some universities students are also nominated as Organization Structure of U niversities
members of the study boards of the faculties/schools. Its employees are responsible for the smooth functioning
The Board of Studies coordinates the teaching and of any organization. They are an important link between
research work in the departments, appoints committees the authorities of the society and the institution. In the
to organize teaching and research work in the subjects; context of a university, it is even more encouraging to see
Allows for courses of study, and takes an active part in the hierarchy created by these functionaries. The follow-
examinations and evaluation processes. ing sections form the formal pyramid of power structure
It promotes the research work of the Faculty/Schools within a university system.
and forwards these reports on research to the Academic
Council. It also acts on subjects referred to by the Vice- 1 Chancellor of University: Normally, every university
Chancellor, Executive Council/Syndicate or Academic in India has a chancellor. In the state universities that
Council from time to time. is incorporated by an act of the legislative assembly,

M10_MADAN 07_65901_C10.indd 38 23/12/22 8:05 PM


Higher Education System 10.39

the governor of the state acts as the chancellor. In and how the decisions taken thereunder are
Central Universities, the President of India is known implemented by the officers of the University.
as the Visitor of the University. S/He appoints the (d) The VC has to convene the meetings of the statu-
Chancellor of such Universities. The Chancellors of tory bodies. He may, without being a member,
Central Universities, whether elected or nominated attend and address any meeting of the University
by the Central Government, are usually persons of Body or Committee.
eminence or academic excellence. (e) Being the Head of Research and Ph.D., he has
There are two types of functions of Vice to ensure the standard and priority areas of the
Chancellors–statutory and non-statutory research efforts undertaken by the University.
(f) He presides over the committee constituted for
(a) Statutory: The statutory functions are normally the selection of teachers and nominates the mem-
laid down in the Act of the University. One of the bers of the expert committees for the selection of
statutory functions of the Chancellor is to pre- other academic and administrative personnel of
side over the meetings of the Court/Senate of the the University.
University. S/He is vested with the power to nom-
inate senior officers. He is required to appoint the The task of the VC, as one might actually imagine, is not
Vice-Chancellor of the University, and to appoint so simple. He is the chief executive officer and principal
some of the members of the Court/Senate, the academic head of a university. He has to play his role
Executive Council, the Academic Council, the efficiently. He has to take constant guidance not only from
Teachers’ Selection Committee and the members the university authorities but also from the teachers and
of the University. other concerned agencies outside the university.
(b) Non-statutory functions: These develop from The tenure of the VC is normally five years in central
the culture and tradition of the country-society. universities and three years in state universities. They can
The Chancellor is empowered to intervene in mat- also be reappointed in state universities.
ters of general administration of the Universities
as well as in academic matters, such as in offer- 3. Pro Vice Chancellor–PVC
ing courses, evaluation procedures, holidays and PVC/Rector is the second executive and administrative
other matters relating to the University, and also officer in a university. In central universities, the PVC/
matters relating to supervision etc. Rector is appointed by the Executive Council on the
recommendation of the VC and is selected from amongst
2 Vice Chancellor: Vice Chancellor is the linchpin the senior professors of the University. He exercises such
of a university on all academic matters including powers and performs such duties as may be prescribed by
control and management of libraries, laboratories, the laws of the University. In state universities, PVCs are
workshops, etc. He is the keeper of the conscience of appointed from a panel of names recommended by the VC.
the University, as noted in the University Education The office of the PVC/Rector is interrupted or co-terminus
Commission Report 1948–49, and so has been said with that of the VC. It should not exceed five years.
by the Kothari Commission (1964–68) that he should The PVC/Rector is expected to share the responsibil-
be committed to the scholarship and truth of the ity of the VC in such areas as the VC deems fit. He also
University. needed. He is the key figure in the entire serves in the absence of the VC, when he is away from the
university system. The success of the University Headquarters on official business, or is on leave. Therefore,
depends to a large extent on them, how effectively the main function of the PVC/Rector is a kind of partnership
and efficiently they discharge their responsibilities. with the VC to make the entire operation of the University
Here in the following statements the main functions system more effective.
of the VC of a University are listed.
(a) Selection of the VC to the important statutory 4. Registrar
bodies of the University like Senate/Court, Registrar has traditionally been the permanent administra-
Executive Council/Syndicate/Management Board, tive head of the university. This stability benefits the Vice-
Academic Council, Planning Board, Finance Chancellor with a fixed tenure. The term of the Registrar
Committee, Selection Committee etc. has been fixed at five years.
The Registrar is the ex-officio Secretary of the
As committee chair, she presides over the meet- Court/Senate and Executive Council/Syndicate of the
ings, controls the proceedings and continuously University. He is the custodian of the records, seal/seal of
monitors the discussions at the meetings. the University and all other properties of the University
(b) As the executive head of a university, the VC is which the Executive Council/Syndicate entrusts for his
responsible for the implementation of the deci- care. He has to sign all the contracts and agreements
sions taken by various statutory bodies. In this made on behalf of the University.
context, the ability of the VC to run the University Registrar should maintain correspondence with other
efficiently and effectively is truly reflected. statutory bodies. He organizes the meetings of all the bod-
(c) It is the duty of the VC to ensure that the provi- ies as per the advice of the Vice-Chancellor. The Registrar
sions of the Act, statutes, ordinances, regulations, is also responsible for the preparation of annual reports

M10_MADAN 07_65901_C10.indd 39 23/12/22 8:05 PM


10.40 Chapter 10

on the activities of the University and other miscellaneous A Gyanam was the chairman of this committee, in this
subjects. report important recommendations were made on various
The functions and powers of the Registrar discussed aspects of university administration.
above are by no means exhaustive. It has been observed Deemed to be university level was also an effort
that the more the Vice-Chancellor and the Registrar come towards giving autonomy the following steps are neces-
together, the greater is the power of the university system. sary to grant autonomy to the university.
5. Dean of Faculty 1. Government control in universities should be reduced,
The Dean is the head of the School’s Faculty of Studies. so as to strengthen the university’s capacity for auton-
In most universities in India, a faculty dean is appointed omy and accountability and make academic decisions
by the vice-chancellor from amongst the professors of the based on merit.
school, for a period of two years on the basis of seniority. 2. New methods and procedures of financial regula-
The Dean is responsible for the conduct and standards tions should be devised and direct interference of
of teaching and research in its faculty schools. He coordi- finance department in financial management of
nates and supervises these through the heads of depart- universities should be prevented, which may also be
ments. It is his responsibility to arrange the examination counterproductive.
of the University in respect of the students of his Faculty/ 3. Since colleges are the nutritional source of universities,
School as per the instructions given by the Academic there are more than 1000 universities in India, and
Council. He performs such other academic duties as may there are more than 40,000 colleges, there is a great
be assigned by the Vice-Chancellor, the Executive Council, need for better coordination in their functioning and
the Syndicate or the Academic Council from time to time. activities. The participation of the teaching faculty
should be ensured through a democratic process.
6. Head of Department 4. Complete transparency should be maintained in the
The head of a department or centre is appointed by the working of the executive/academic bodies of the
Vice-Chancellor, on a rotation basis, for a period specified Universities and other Governing Councils. There is
in the statutes. He may be from amongst the professors an urgent need to review the University Acts in various
or readers of the department. He organizes and presides states and revise them in the light of new requirements
over the meetings of the Department under the general and challenges being faced by the universities. New
supervision of the Dean. He organizes teaching and technologies of information and communication
research work in the department, and is in charge of should be used to achieve administrative efficiency.
maintaining discipline in the classroom, laboratories, etc. 5. Higher education should be developed as a basic
He assigns the necessary duties to the teachers of the framework for the social and economic development
department for the proper functioning of the depart- of the country.
ment. He plans, executes and monitors the working of his 6. Students’ participation in the field of university/col-
department in the field of teaching, research and exten- lege administration should be encouraged.
sion activities. 7. Political interference in the appointment of teachers
7. Other functionaries and administrators of the University should be com-
In this category, we have included all other office pletely stopped.
bearers of the University, who are responsible for both
administrative and academic subjects.
The most important among such office bearers are the NITI Aayog
controller of examination, finance officer, deputy reg- The NITI Aayog is a policy think-tank of government that
istrar and assistant registrar. Since the examination is replaces the Planning Commission and aims to involve
related to the university administration the states in economic policy-making in India. It will be
An important and confidential matter, the position providing strategic and technical advice to the central and
of Controller of Examinations is very important to the the state governments. The government had announced
system. The credibility of the University rests to a large the formation of NITI Aayog on 1 January 2015. It has the
extent on the successful conduct of examinations and following levels:
hence the role of Controller of Examinations is of utmost
importance. 1. Prime Minister of India will be the Chairperson.
Another important officer, who is usually appointed on 2. Governing Council comprises the Chief Ministers
deputation from the Government, is the Finance Officer of all the states and Lieutenant Governors of union
of the University. He is generally trained in accounts and territories.
advises the university authority on all financial matters. 3. Regional Councils will be formed to address specific
issues and contingencies impacting more than one
state or a region. These will be formed for a specified
Autonomy in Higher E ducation System tenure.
A UGC committee submitted a report titled ‘Towards 4. Experts, specialists and practitioners with relevant
a New Educational Management’ in January 1990 on domain knowledge as special invitees will be nomi-
alternative models of university management. Professor nated by the Prime Minister.

M10_MADAN 07_65901_C10.indd 40 23/12/22 8:05 PM


Higher Education System 10.41

3. Section 2(j): ‘Right to Information’ means the right


Right to Information (RTI) Act, 2005 to information accessible under this Act which is held
Participation, transparency, legitimacy and responsive- by or under the control of any public authority and
ness form the pillars of good governance. The concept of includes the right to (i) inspection of work, documents
good governance was applied in India through the pass- and records; (ii) taking notes, extracts or certified copies
ing of the RTI Act, 2005. of documents or records; (iii) taking certified samples of
According to Thomas Jefferson, information is the material; and (iv) obtaining information in the form of
currency of democracy. diskettes, floppies, tapes, video cassettes or in any other
Article 19(1) of the Indian Constitution specifies that electronic mode or through printouts where such infor-
the Right to Information (RTI) is a part of the fundamen- mation is stored in a computer or in any other device.
tal rights. ‘Public authority’ means any authority or body or
It says that every citizen has freedom of speech and institution of self-government established or consti-
expression. It was said that in the Indian democracy, people tuted by government or non-government funded by
are the masters which give them the right to know about government.
the functioning of the government. The RTI Act provides Section 4 of the RTI Act requires suo motu disclosure
machinery for exercising this fundamental right. of information by each public authority. However,
The motivation lies in international developments such such disclosures have remained less than satisfactory.
as the Universal Declaration of Human Rights (1948) Section 8(1) mentions exemptions against furnishing
and International Covenant on Civil and Political Rights information under the RTI Act.
(1966). Section 8(2) provides for disclosure of information
As per the RTI Act, 2005, every citizen has the right to exempted under the Official Secrets Act, 1923, if larger
receive a timely response from the government for any public interest is served.
information that is sought by him/her with respect to the The Act also provides for appointment of Information
functioning of the government. Commissioners at central and state levels. Public
The main objectives of RTI are as follows: authorities have designated some of its officers as Public
1. To empower the citizens Information Officers. They are responsible to give informa-
2. To make government systems more transparent and tion to a person who seeks information under the RTI Act.
accountable • Time period: In normal course, information to an
3. The prevent and eliminate corruption applicant should be supplied within 30 days from
4. To make democracy work better the receipt of application by the public authority.
5. For better vigilance on the instruments of governance • If information sought concerns the life or liberty of
An RTI portal has been created by the Ministry of a person, it shall be supplied within 48 hours.
Personnel, Public Grievances and Pensions. It acts as a • The RTI Act, 2005, did not create a new bureaucracy
gateway for obtaining information on the details of first for implementing the law. Instead, it tasked and man-
Appellate Authorities, Principle Information Officers, etc., dated officials in every office to change their attitude
in a computerized form that could be easily accessed. and duty from one of secrecy to one of sharing and
A written or electronic request must be placed by a openness. It strengthens participatory democracy and
person seeking information under the RTI Act. No rea- ushering in people-centred governance.
son shall be sought from the person placing the request • RTI empowers the poor and the weaker sections of
except the contact details in case the authorities would society to demand and get information about public
have to contact him/her. However, the authorities are not policies and actions, thereby leading to their welfare.
obligated to provide any such information that is received • The Official Secrets Act, 1923, provides that any gov-
under confidence by a foreign government, cabinet ernment official can mark a document as confidential
papers, information forbidden to be shared by the court of so as to prevent its publication.
law and information that could potentially hurt the sover-
eignty and integrity of India.
Lokpal and Lokayukta
• The Lokpal and Lokayukta Act, 2013, provided for
Features of the Act the establishment of Lokpal for the Union and Lokay-
1. Section 1(2): It extends to the whole of India. ukta for states.
2. Section 2(f): ‘Information’ means any material in any • The main objective is to prevent corruption.
form, including records, documents, memos, emails, • These institutions are statutory bodies without any
opinions, advices, press releases, circulars, orders, log- constitutional status.
books, contracts, reports, papers, samples, models and • They perform the function of an ‘ombudsman’ and
data material held in any electronic form and informa- inquire into allegations of corruption against certain
tion relating to any private body which can be accessed public functionaries and for related matters.
by a public authority under any other law for the time • The term Lokpal and Lokayukta were coined by Dr L.
being in force. M. Singhvi.

M10_MADAN 07_65901_C10.indd 41 23/12/22 8:05 PM


10.42 Chapter 10

• In 1809, the institution of ombudsman was inaugu- • Chief Executive Officer: Appointed by the Prime
rated officially in Sweden. Minister for a fixed tenure, in rank of Secretary to the
• Great Britain adopted the institution of the Government of India
ombudsman in 1967 under the Whyatt Report of • Special invitees: Experts and specialists with domain
1961. It became the first large nation in the demo- knowledge nominated by the Prime Minister
cratic world to have such a system.
• In 1966, Guyana became the first developing nation
to adopt the concept of the ombudsman. NITI Aayog Hubs
• The concept of constitutional ombudsman was first
• Team India Hub acts as an interface between states
proposed by the then law minister Ashok Kumar
and the Centre.
Sen in the Parliament in the early 1960s.
• Knowledge and Innovation Hub builds the think-
• In 1966, the First Administrative Reforms Commis-
tank acumen of the NITI Aayog.
sion recommended the setting up of two independ-
• Three planned documents:
ent authorities—at the central and state levels.
1. Three-year action agenda
• Starting 1968 till 2011, eight attempts were made to
2. Seven-year medium-term strategy paper
pass the Bill, but all met with failure.
3. Fifteen-year vision document
• In 2005, the Second Administrative Reforms Com-
mission chaired by Veerappa Moily recommended
that the office of Lokpal should be established with-
Objectives
out delay. • Cooperative federalism and decentralization with
• ‘India Against Corruption’ movement led by Anna more involvement of states (India is a diversified
Hazare put pressure on the United Progressive Alli- country and its states are in various phases of
ance (UPA) government at the Centre and resulted in economic development along with their own
the passing of the Lokpal and Lokayuktas Bill, 2013, strengths and weaknesses; in this context, a ‘one-
in both the Houses of ‘Parliament. It received assent size-fits-all’ approach to economic planning is
from President on 1 January 2014 and came into obsolete; it cannot make India competitive in today’s
force on 16 January 2014. global economy)
• To formulate credible plans at the village level and
aggregate these progressively at higher levels of
N ational I nstitution for T ransforming India government
• Planning has been in Indian psyche as our leaders came • To ensure national interests in economic strategy and
under influence of the socialist clime of the erstwhile policy
USSR. The Planning Commission served as the plan- • To provide advice and encourage partnerships
ning vehicle for close to six decades with a focus on con- between key stakeholders and national and interna-
trol and command approach. tional like-minded think-tanks, as well as educational
• The Planning Commission was replaced by a new insti- and policy research institutions
tution—NITI Aayog—on 1 January 2015 with emphasis • To create a knowledge, innovation and entrepreneur-
on ‘bottom-up’ approach to envisage the vision of Maxi- ial support system
mum Governance, Minimum Government, echoing the • To maintain a state-of-the-art resource centre, be a
spirit of ‘cooperative federalism’. repository of research on good governance and best
• The 65-year-old Planning Commission had become practices in sustainable and equitable development
a redundant organization. It was important in a com-
mand economy structure. The Future
• Planning should be decentralized.
Administration • Bureaucratic inertia needs to be shaken.
• Chairperson: Prime Minister • The NITI Aayog is basically an ‘agent of change’;
• Vice-Chairperson: To be appointed by the Prime it could contribute to the government’s agenda of
Minister improving governance and implementing innovative
• Governing Council: Chief Ministers of all states and measures for better delivery of public services.
Lieutenant Governors of union territories • The NITI Aayog continues to be representative of
• Regional Council: To address specific regional issues, efficient, transparent, innovative and accountable
comprising Chief Ministers and Lieutenant Governors governance system in the country with distinguished
chaired by the Prime Minister or his/her nominee work ethics.
• Ad hoc membership: Two members in ex officio
capacity from leading research institutions on a rota- NITI Aayog ’ s I ndia Innovation Index
tional basis Karnataka has bagged the top rank in NITI Aayog’s India
• Ex officio membership: Maximum four from the Innovation Index, 2022, which determines innovation
Union Council of Ministers to be nominated by the capacities and ecosystems at the sub-national level.
Prime Minister Manipur secured the lead in the Northeast and Hill States

M10_MADAN 07_65901_C10.indd 42 23/12/22 8:05 PM


Higher Education System 10.43

category, while Chandigarh was the top performer in the Good Governance Day is observed on the birth anni-
Union Territories and City States category. India wants versary of former Prime Minister Shri Atal Bihari Vajpayee
to be named among the top 25 nations in the Global (25 December). It was observed for the first time in 2014.
Innovation Index. This can be achieved through the The Index takes 10 sectors into consideration for GGI
following measures. rankings—agriculture and allied, commerce and indus-
tries, human resource development, public health, public
1. Increasing Gross Domestic Expenditure on R&D
infrastructure and utilities, economic governance, social
(GDERD)
welfare and development, judicial and public security,
2. Promoting private sector participation in R&D
environment and citizen-centric governance. Performance
3. Closing the gap between industry demand and what
in these sectors is measured on the basis of 50 indicators.
the country produces through its education systems.
Tamil Nadu topped the GGI among 18 big states in the
I nnovation Index country as per the Index in December 2019.
The table lists the score and ranking of 17 major States Topping the class
Rank State
in the third edition of NITI Aayog's India Innovation Among 18 big states in the coun-
Index. Karnataka ranked first followed by Telangana 1. Tamil Nadu try, Tamil Nadu put up a good
and Haryana while Chhattisgarh, Odisha and Bihar were all-round show, lagging only in
ranked at the bottom of the index. 2. Maharashtra
‘commerce and industries.’
3. Karnataka
State Score Rank
4. Chhattisgarh Parameter-wise Ranking
Karnataka 18.01 1 among Big States
5. Andhra
Telangana 17.66 2 Pradesh Public infrastructure 1
Haryana 16.35 3 6. Gujarat Judicial and public 1
security
Maharashtra 16.06 4 7. Haryana
Public health 2
Tamil Nadu 15.69 5 8. Kerala
Environment 3
9. Madhya
State Score Rank Pradesh Human resource 5
development
M.P. 12.74 13 10. West Bengal
Economic governance 5
Gujarat 12.41 14 11. Telangana
Social welfare and 7
Bihar 11.58 15 12. Rajasthan development
Odisha 11.42 16 13. Punjab Agriculture and allied 9
Chhattisgarh 10.97 17 14. Odisha Commerce and industries 14
Source: TheHindu.com dated July 21, 2022. 15. Bihar
*Ranking on citizen-centric
16. Goa ­governance was not available.
G ood G overnance I ndex (GGI) 17. Uttar Pradesh
The GGI was released by the Union Ministry of Personnel, 18. Jharkhand
Public Grievances and Pensions.

M10_MADAN 07_65901_C10.indd 43 23/12/22 8:05 PM


10.44 Chapter 10

A s s e s s Yo u r L e a r n i n g

ANCIENT EduCATION
1. Given below are two statements, one is labelled as 4. Which of the following statements are correct in con-
Assertion (A) and other is labelled as Reason (R). text of our education system?
Assertion (A): According to Mark Twain - India is the 1. The knowledge was considered sacred, and no fee
cradle of the human race, the birthplace of human was charged. All members of the society donated
speech, the mother of history, the grandmother of or contributed certain things in one form or the
legend, and the great grandmother of tradition. Our other to the education system.
most valuable and most instructive materials in the 2. Learners identified their duties and devotion
history of man are treasured up in India only. towards the ideal of ‘sommum bonum’ that indi-
cates the highest values and priorities. The human
Reason (R): Rishis chanted the pious mantras for soul was assumed as the material world.
peace and progress in society, vedas also called as 3. Learners achieved ‘chitta vritti nirodha’ which is
shrutis created the ecosystem that was carried for- the control of mental activities connected with
ward to ancient universities, with the result that India the concrete world.
had created a great heritage and society.
4. There was no relation of knowledge with life.
In the context of above two statements, choose the cor- 5. The word ‘rta’ in vedic education has the dual
rect option. meanings of natural order and moral order.
(a) Both Assertion (A) and Reason (R) are the codes:
true and Reason (R) is a correct explanation of (a) 2, 3 and 4
Assertion (A). (b) 3, 4 and 5
(b) Both Assertion (A) and Reason (R) are the true (c) 1, 2, 3 and 5
but Reason (R) is not a correct explanation of (d) 1, 2, 3, 4 and 5
Assertion (A).
(c) Assertion (A) is true and Reason (R) is false. 5. If we look at the six main subject areas such as pho-
(d) Assertion (A) is false and Reason (R) is true. netics, ritualistic knowledge, grammar, exegetics,
2. Which of the following statements are true in context metrics and astronomy, these are called
of ancient education system in India? (a) Vedangas (b) Darshanas
A S S E S S YO U R L E A R N I N G

1. During and after Rigveda period, our ancient edu- (c) Pitakas (d) Upanishads
cation system evolved for the holistic develop- 6. The ‘sutra’ period during ancient education was
ment of the individual. assumed to be equal to
2. The values of humility, truthfulness, discipline, (a) Vedic period
self-reliance and respect for all creations were (b) Upnishad period
created during ancient education system. (c) Bhakti period
3. The teaching and learning during ancient educa- (d) Poorva Mimansa
tion followed the path towards self, family and
society. 7. The question is linked with the three basic three
stages of education during ancient era.
codes:
Match List I with List II.
(a) 1 and 2
(b) 2 and 3 Stage - List 1 Brief description – List II
(c) 1, 2 and 3
(d) 1 and 3 A. Sravana 1. Comprehension of truth
and applying into real
3. In which of the following periods can Indian philoso-
life
phy be ordered in a sequence?
(a) Vedic period, Upanishadic period and post-Vedic B. Manana 2. Thinking, analysing
period and assimilation of the
(b) Upanishadic period, Vedic period and post-Vedic texts heard
period C. Nididhyasana 3. Acquiring knowledge of
(c) Vedic period, post-Vedic period and Upanishadic ‘shrutis’ by listening
period
(d) No such history of sequencing is available.

M10_MADAN 07_65901_C10.indd 44 23/12/22 8:05 PM


Higher Education System 10.45

Codes: C. Sadyodwahas 3. Women education, till


(a) A – 2, B – 1, C – 3 they get married
(b) A – 3, B -2, C – 1
(c) A – 1, B -2, C - 3 D. Brahmvadinis 4. Women studying
(d) A – 3, B – 1, C – 2 throughout life –
8. Match List I with List II and select the correct answer women sages were
from the codes given below. called as rishikas.

Type of Veda - List 1 Brief description – List II Codes:


A. Rig veda 1. The book of spell (a) A – 1, B – 2, C – 3, D – 4
– roughly secular in (b) A – 4, B – 3, C – 2, D – 1
character, that is linked (c) A – 4, B – 3, C – 1, D – 2
with various arts and (d) A – 2, B – 4, C – 3, D – 1
sciences 11. Which of the following is/are true in context of ancient
B. Sama veda 2. The book of ritual education?
– collection of prose (a) Karam Kanda – with focus of rituals, mainly be
mantras or yajurs – Brahmins
mainly done by hotris (b) Jnana Kanda – with focus on knowledge
who is assumed to (c) Aranyanka – a link between the Brahmanas and
be the first order of the Upanishads, meant for better understanding
priesthood. of vedic literature.
(d) All of the above
C. Yajurveda 3. The book of chant 12. Which among the following is considered to be the
– collection of verses – oldest Veda?
for liturgical purpose (a) Sam Veda
that is participation of (b) Yajur Veda
people in the work of (c) Rig Veda
God. (d) Atharva Veda
D. Atharava veda 4. The book of mantra 13. Which is known as book of melodies and chants?
– praise of God, and (a) Rigveda
meant for plain living (b) Samveda
and high thinking (c) Atharvaveda

A S S E S S YO U R L E A R N I N G
Codes: (d) Yajurveda
(a) A – 1, B – 2, C – 3, D - 4 14. Which of the following statements are true in context
(b) A – 4, B – 3, C – 2, D – 1 of ancient education system in India?
(c) A – 4, B – 3, C – 1, D -2 1. The Jataka tales and the records of Chinese schol-
(d) A – 2, B -4, C – 3, D -1 ars Xuan Zang and I-Qing showed us that kings and
9. Ayurveda, which deals with indigenous system of society took active interest in promoting education.
medicine, could be traced to [October 2020] 2. The universities at Takshashila, Nalanda, Valabhi,
(a) Rigveda Vikramshila, Odantapuri and Jagaddala devel-
(b) Yajurveda oped in connection with the viharas.
(c) Atharvaveda 3. The universities at Benaras, Navadeep and Kanchi
(d) Samaveda developed in connection with temples and
became centres of community life in the places
10. The stages of education in ancient system were for-
where they were situated.
mally systematised.
Codes:
Match List I with List II.
(a) 1 and 2
Stages of Education (b) 2 and 3
in Ancient India – Brief description – List II (c) 1, 2 and 3
List I (d) 1 and 3
A. Vidyarambha 1. The initial stage of 15. Which of the following aspects are important in context
starting the education of pupils who studied at ancient Takshashila (Taxila)
University?
B. Upanayana 2. Formal education – with 1. Panini, the legendary Indian grammarian and
sacred thread ceremony, who authored Ashtadhyayi
called as dvija that 2. Jivaka, one of the most renowned physicians in
means second birth. ancient India

M10_MADAN 07_65901_C10.indd 45 23/12/22 8:05 PM


10.46 Chapter 10

3. Chanakya, also known as Kautilya, a skilled expo- 3. This university supported the Vajrayana sect thrived
nent of statecraft and authored Arthashastra here.
Codes: 4. This university was famous for Tantric teachings.
(a) 1 and 2 Codes:
(b) 2 and 3 (a) 1, 2 and 3
(c) 1, 2 and 3 (b) 2 and 3
(d) 1 and 3 (c) 1 and 3
16. Which of the following are the important aspects of (d) 1, 2, 3 and 4
the ancient Nalanda University? 21. Which of the following are the important aspects of
1. This university was a centre of learning from the Valabhi university?
5th century CE to 12th century CE. 1. This university was located in Bihar state.
2. This university is located in present day Rajgir, 2. This university was famous for Hinayana Buddhism.
Bihar, India, Nalanda. 3. Heun Tsang, a Chinese scholar, paid a visit to this
3. This university has declared as the World Heritage university.
site by UNESCO. Codes:
4. The new Nalanda University is envisaged as a cen-
tre of inter-civilisational dialogue. (a) 1 and 2
(b) 2 and 3
Codes: (c) 1, 2 and 3
(a) 1, 2 and 3 (d) 1 and 3
(b) 2 and 3 22. Which among the following schools of Hindu Philoso-
(c) 1, 2, 3 and 4 phy is considered as anti-ascetic and anti-mysticist?
(d) 1 and 3 (a) Vaisheshika
17. Consider the following statements in context of (b) Nyaya
library of the university. Identify the university. (c) Mimansa
1. The library of the university was known as Dharma (d) Samkhya
Gunj (Mountain of Truth) or Dharmagañja 23. According to scholar Sukumar Dutt, a vihara or uni-
(Treasury of Truth), that was intimately linked versity was linked with Abbot (Adhyakṣa), six gate
with the Buddhist knowledge. protectors (Dvārapāla or Dvārapaṇḍita), Great Schol-
2. The collection at this library collection comprised ars (Mahapaṇḍita), Great Teachers (Upādhyāya or
of hundreds of thousands of volumes. Āchārya) and Resident monks (bhikṣu) to name a few.
3. The library had three main buildings as high as The university was
nine stories tall, Ratnasagara (Sea of Jewels), (a) Vikramashila University
A S S E S S YO U R L E A R N I N G

Ratnodadhi (Ocean of Jewel) (b) Vallabhi University


(a) Vikramshila University (c) Benaras University
(b) Nalanda University (d) Nalanda University
(c) Odantapuri University
(d) Kanchi University 24. What are the ‘tripitakas’?
(a) The three signs of life
18. Which of the following university was visited by the (b) Triratnas of Mahavira
Chinese scholars I-Qing and Xuan Zang in the 7th (c) Collection of the preachings of Buddha
century CE? (d) The place of birth of Mahatma Budh
(a) Nalanda university
(b) Takshashilla University 25. Which of the following educational institution was set
(c) Benaras University up by Kalinga rulers?
(d) Kanchi University (a) Pushpagiri Vihara and Lalitagiri in Odisha
(b) Nalanda University in Bihar
19. At which of the following universities, Xuan Zang (c) Takshshila University in Pakistan
studied Yogashastra? (d) Vallabhi University in Gujarat
(a) Nalanda university
(b) Takshashilla University 26. Which of the following university was founded by
(c) Benaras University Kumargupta (also called as Shakraditya) in the 5th
(d) Kanchi University century A.D?
(a) Vikramshila University
20. Which of the following are the important aspects of (b) Nalanda University
Vikramshila university? (c) Kanchi University
1. This university was established by the King (d) Benaras University
Dharmapala of the Pala dynasty.
2. The university was manly based on Buddhist 27. Which of the following modes of admission was prev-
teachings. alent in Nalanda University in ancient times?

M10_MADAN 07_65901_C10.indd 46 23/12/22 8:05 PM


Higher Education System 10.47

(a) Entrance examination Which among the above were Buddhist Mahaviharas
(b) Interview or Viharas?
(c) Good academic credentials (a) Only 1 and 2
(d) Peer discussion (b) Only 2 and 3
28. How many students were studying in the Nalanda (c) Only 1 and 3
University as mentioned by Hiuen Tsang? (d) 1, 2 and 3
(a) 8,000 (b) 10,000 36. Assertion (A): During the Maurya empire, the Indian
(c) 12,000 (d) 15,000 culture and way of life were deeply influenced by
29. Which of the following Chinese travellers found a Buddhism.
flourishing Nalanda University as a Buddhist centre of Reason (R): Buddhism appealed to people of lower
learning which, by then, had a worldwide reputation, castes because it emphasized individuals’ path to
and had an entrance system for getting admission? enlightenment and salvation, which could be attained
(a) Hiuen-Tsang in this life.
(b) Fi-Hien In the context of above two statements, choose the
(c) I-Tsing correct option.
(d) None of the above
(a) Both Assertion (A) and Reason (R) are the true and
30. Which of the following statements is true about Tak- Reason (R) is a correct explanation of Assertion
shshila (Taxila) University? (A).
1. The university was a centre of its own kind which is (b) Both Assertion (A) and Reason (R) are the true but
frequently referred to as a seat of higher learning in Reason (R) is not a correct explanation of Assertion
Buddhist literature. (A).
2. The university was a Brahmanic intellectual cen- (c) Assertion (A) is true and Reason (R) is false.
tre as well. (d) Assertion (A) is false and Reason (R) is true.
3. The university is now located in Pakistan; it was
within the capital of Gandhara kingdom. 37. Set I and Set II have been mentioned in the context of
4. It was in the native land of Panini. Buddhist Education.
Codes: Noble Truth - Set I Brief Explanation - Set II
(a) 1, 2 and 3 (b) 2, 3 and 4
(c) 1, 3 and 4 (d) All of the above A. Dukkha 1. It can be achieved by
following the Eight Fold
31. What Nalanda University was to east India, which of Path
the following universities was to west India?
(a) Takshashila (b) Vallabhi B. Samudya 2. Suffering could be

A S S E S S YO U R L E A R N I N G
(c) Odontopuri (d) Maithili extinguished.
32. Which of the following universities was famous for its C. Nirodha 3. Every suffering has a
specialized training on the subject of Tantric Buddhism? cause
(a) Vikramashila University D. Atthanga 4. Suffering is the essence
(b) Nalanda University Magga of the world.
(c) Takshshila University
(d) Odontopuri University
Codes:
33. The education at which of the following universities
was spread over three places of Nawadweep, Shan- (a) A – 1, B -3, C -2, D -4
tipur and Gopalpura? (b) A – 4, B -3, C -1, D -2
(a) Ujjain University (c) A – 4, B -3, C -2, D -1
(b) Nadia University (d) A – 1, B -2, C -3, D -4
(c) Jagaddala University 38. Which of the following aspects are true in context of
(d) Vikramshila University Buddhist Education?
34. Many books of which of the following universities 1. This education focused on three major points, dis-
were translated into Tibetan language? cipline, meditation and wisdom, with wisdom as
(a) Ujjain University the goal through discipline and meditation.
(b) Nadia University 2. The three about clear idea of life are Tipitaka
(c) Jagaddala University which are further categorised as Sutta pitaka,
(d) Vikramshila University Vinaya pitaka and Abhidhamma pitaka.
3. During initial stages of a learner, the medium of
35. Consider the following: education was mother tongue, later it were Pali
1. Vikramshila and Prakrit. Sanskrit were included in the system
2. Odantapuri at a later stage.
3. Sompapuri 4. This education could never get the support of kings.

M10_MADAN 07_65901_C10.indd 47 23/12/22 8:05 PM


10.48 Chapter 10

Codes: Statement II: Shvetambaras literally means ‘white


(a) 2, 3 and 4 clad’ which assert that complete nudity is not impor-
(b) 3 and 4 tant for salvation.
(c) 1, 2, 3 and 4 (a) Both statements 1 and II are true.
(d) 1, 2 and 3 (b) Only statement I is true.
(c) Only statement II is true.
39. Which of the following statements are true? (d) Neither statement 1 nor statement II is true.
1. Buddha means the enlightened one.
2. The first Buddhist council was held at Sattapani 44. Statement I: Mostly present in the South India, an
cave Rajgriha. Agrahara was a bigger institution, a whole settlement
3. Gautam Buddha attained Mahaparinirvana at of learned Brahmins, with its own powers of govern-
Kushinagar. ment and was maintained by generous donations
from the society.
Codes:
Statement II: In South of India, a Ghatika was a centre
(a) 1 and 2 (b) 2 and 3 of learning including religion and was small in size.
(c) 1, 2 and 3 (d) 1 and 3 (a) Only Statement I is true
40. Ashoka called the third buddhist council at which of (b) Only Statement II is true
the following places? (c) Both Statements I and II are true
(a) Magadha (b) Pataliputra (d) Neither Statement I nor Statement II is true
(c) Burma (d) Therevada 45. Which of the following terms is used for Jain Temples?
41. Match the SET I with SET II in context of Jain (a) Basadis (b) Viharas
Philosophy (c) Matha (d) Tols
Jain Philosophy Brief description - Set II 46. Match the following:
Tenets - Set I 1. Bengal A. Tols
2. Western India B.Pathshalas
A. Anekantavada 1. This is partial viewpoints 3. Bihar C. Chatuspadis
that means the system of
describing reality from Set 1 – Name of Set II – Name of
different points of view. Region Ancient Institution
B. Syadavada 2. The objects and situations A. Bengal 1. Tols
a conditional point of
view – all judgements are
B. Western India 2. Pathshalas
conditional, they depend
A S S E S S YO U R L E A R N I N G

upon the situation. C. Bihar 3. Chatuspadis


C. Nayavada 3. Non – absolutism, the Codes:
objects have infinite
modes of existence, (a) A - 1, B - 2, C - 3 (b) A - 2, B - 1, C - 3
so only kevalins can (c) A - 3, B - 2, C - 1 (d) A - 1, B - 3, C – 2
comprehend all aspects of 47. Which of the following did not recognize the Vedas?
knowledge. (a) Buddhists (b) Jains
(c) Materialists (d) All of the above
Codes: 48. Prakrit languages are related to Sanskrit but it dif-
(a) A – 2, B – 1, C – 3 fers from Sanskrit in many ways. When the Prakrit
(b) A – 3, B – 2, C – 1 languages were formalized by literary use, their vari-
(c) A – 1, B – 2, C – 3 ations from the conventional Vedic languages came to
(d) A – 3, B – 1, C – 2 be known as
42. Samyag Darshana (Right faith), Samyag Jnana (Right (a) Apabhramsha (b) Pali
knowledge) and Samyag Charitra (Right conduct) are (c) Persian (d) Hindi
called as triratna. These are the three pillars of 49. Consider the following statements:
(a) Budhhism ‘It means “going out”. The students after being
(b) Jainism admitted to a monastery had to renounce all their
(c) Vedic Education worldly and family relationships. After admission into
(d) Upanishad “Sangh”, they could remain as a monk. The age limit
43. Consider the following statements in context of Jain fi xed for the ceremony was 8 years’.
system. Which ceremony is being referred through above
Statements I: Diagambaras literally means ‘sky clad’ statements?
which emphasize on nudity, as it is the absolute pre- (a) Pabbajja (b) Upasampada
requisite to attain salvation. (c) Upanayana (d) Janeu

M10_MADAN 07_65901_C10.indd 48 23/12/22 8:05 PM


Higher Education System 10.49

50. Which of the following languages is termed the Codes:


most suitable language for the computer software as (a) 1—A, 2—C, 3—B, 4—D
reported in Forbes magazine? (b) 1—D, 2—B, 3—C, 4—A
(a) English (b) Japanese (c) 1—D, 2—C, 3—B, 4—A
(c) Sanskrit (d) Roman (d) 1—D, 2—C, 3—A, 4—B
51. India invented the number system. Who among the 56. Kindly look at the following forms of ancient education:
following invented the ‘zero’?
1. Para vidya: Supreme or highest knowledge for
(a) Aryabhatta (b) Bhaskaracharya
self-realization or knowledge of the supreme self
(c) Budhayana (d) Sushrutha
2. Apara vidya: The lowest knowledge of the four
52. Which of the following nations is the main source Vedas, six Vedangas, etc.
nation of algebra, trigonometry and calculus?
(a) Only 1 is correct.
(a) India (b) Rome
(b) Only 2 is correct.
(c) USA (d) China
(c) Both 1 and 2 are correct.
53. 8. In which of the following specific languages are the (d) Both 1 and 2 are incorrect.
canonical texts such as Tipitaka of Theravada Bud-
57. Identify the university with the help from following
dhism maintained?
statements.
(a) Hindi (b) Pali
(c) Sanskrit (d) Ardhamagadhi 1. One of the largest international universities of India.
2. It was made responsible for the administrative
54. Match the following:
affairs of Nalanda University, specifically under
1. Acharya A. Teaching as a profession for livelihood
Pala Kings.
2. Charakas B. Teachers famous for their profound
3. This was one of the largest Buddhist universities.
scholarship
4. Tantarism was the major branch of learning there.
3. Yaujanasatika C. Wandering scholars in quest for
5. This university is known to us through multiple
higher knowledge
Tibetan sources.
4. Upadhyaya D. No charging fee from the pupils
(a) Taxila University
Codes: (b) Vaishali University
(a) 1—D, 2—C, 3—B, 4—A (c) Vikramshila University
(b) 1—A, 2—C, 3—B, 4—D (d) Odontapuri University
(c) 1—D, 2—B, 3—C, 4—A 58. Which among the following is considered to be the
(d) 1—D, 2—C, 3—A, 4—B official law book of the Guptas?
55. Match the following: (a) Manusmriti
1. Gurukul   A. Buddhist monastery (b) Parashara Smriti

A S S E S S YO U R L E A R N I N G
2. Parishads   B. A national gathering (c) Yajnavalkya Smriti
3. Goshti   C. Discussions and discourses (d) Vyasa Smriti
4. Vihara   D. Teacher was a settled householder

Basics of Modern Education, Institutions of Higher Education

59. Match Set 1 with Set II in context of steps taken by D. Saddler 4. The education being
British Government in context of spread of Education University made an objective of
sector. Commission, British government - Rs
1917 - 1919 one lakh were allocated
Step Taken Set – I Brief description Set – II for the purpose.
A. Charter Act, 1. To look into the
1813 performance of Codes:
Calcutta University (a) A – 1, B – 3, C – 2, D – 4
B. The English 2. Vernacular education, (b) A – 4, B – 2, C – 3, D – 1
Education Act, to set up three (c) A – 4, B – 3, C – 1, D – 2
1835 universities in (d) A – 4, B – 3, C – 2, D – 1
Presidency towns. 60. It is said that the present higher education system
has its origin in Mountstuart Elphinstone’s minutes,
C. Wood’s 3. Macaulay’s minutes -
which stressed on the need for establishing schools
Dispatch, 1854 to establish the need
for teaching English and European sciences. This
to impart English
reported was presented in the year
education to Indian
(a) 1823 (b) 1835
‘natives’.
(c) 1858 (d) 1861

M10_MADAN 07_65901_C10.indd 49 23/12/22 8:05 PM


10.50 Chapter 10

61. Inter-University Board was set up in 1925 to promote 70. Which of the following modality of higher education
university activities, by sharing information and is the example of evolution in the post-independence
cooperation in the field of education, culture, sports India? [October 2020]
and allied areas. Now, it is known as (a) Teacher education
(a) Association of Indian Universities (b) Technical education
(b) Central Advisory Board of Education (c) Legal Education
(c) Central Board of Secondary Education (d) Distance Education
(d) None of the above 71. Tertiary education includes
62. Regarding Wood’s Dispatch, which of the following (a) Primary and secondary education
statements are true? [2018] (b) Higher education
(c) Vocational education and training
1. Grants-in-Aid system was introduced. (d) Both (b)and (c)
2. Establishment of universities was recommended.
3. English as a medium of instruction at all levels of 72. The main governing body at the tertiary level of edu-
education was recommended. cation in India is
(a) NCERT (b) CBSE
Select the correct answer using the code given below: (c) AICTE (d) UGC
(a) 1 and 2 only 73. The government established the University Grants
(b) 2 and 3 only Commission by an Act of Parliament in the year
(c) 1 and 3 only (a) 1980 (b) 1948
(d) 1, 2 and 3 (c) 1950 (d) 1956
63. Which of the following is a precursor to 10 + 2 + 3? 74. Which of the following is the regulator of higher
(a) Saddler Commission education?
(b) Wood’s Dispatch (a) NCERT (b) AICTE
(c) Kothari Commission (c) UGC (d) Ministry of Education
(d) None of the above
75. Given below are two statements, one is labelled as
64. Which of the following committee’s recommended Assertion (A) and other is labelled as Reason (R).
setting up UGC and is also known as the University
Education Commission? Assertion (A): For General Development Assistance,
(a) Radhakrishnan Commission the assistance will be provided to only Government
(b) Mudaliar Commission and Government aided private colleges included under
(c) Wardha Commission Section 2(f) and declared eligible to receive central
(d) Kothari Commission assistance under Section 12B of the UGC Act, 1956.
Reason (R): The development of colleges is pro-
A S S E S S YO U R L E A R N I N G

65. Which of the following documents is termed the


Magna Carta of English education in India? foundly influenced by three objectives which are
(a) Charles Wood’s Dispatch Access, Equity and Expansion.
(b) Macaulay minutes In the context of above two statements, choose the
(c) Mountstuart Elphinstone’s minutes correct option.
(d) Wardha system (a) Both Assertion (A) and Reason (R) are the
true and Reason (R) is a correct explanation of
66. In which of the following years were Presidency Uni-
Assertion (A).
versities of Calcutta, Bombay and Madras set up fol-
(b) Both Assertion (A) and Reason (R) are the true
lowing the recommendations of Wood’s Dispatch?
but Reason (R) is not a correct explanation of
(a) 1854 (b) 1857
Assertion (A).
(c) 1858 (d) 1861
(c) Assertion (A) is true and Reason (R) is false.
67. The University of Allahabad was founded in the year (d) Assertion (A) is false and Reason (R) is true.
(a) 1887 (b) 1901
76. Given below are two statements, one is labelled as
(c) 1905 (d) 1911
Assertion (A) and other is labelled as Reason (R).
68. Which of the following institutions was set up in 1945
Assertion (A): In 1952, the Union Government decided
to look after the functioning of three central universi-
that all cases pertaining to the allocation of grants-in-
ties of Aligarh, Banaras and Delhi?
aid from public funds to the Central Universities and
(a) University Grants Commission
other Universities and Institutions of higher learning
(b) University Grants Committee
might be referred to the University Grants Commission.
(c) Inter-University Board
(d) Association of Indian Universities Reason (R): There was University Education Com-
mission (1948) under the Chairmanship of Dr. S
69. The Report of the Central Advisory Board of Educa-
Radhakrishnan to suggest for the present and future
tion on Post-War Educational Development in India is
needs of the education sector. It recommended that
also termed
the University Grants Committee be reconstituted on
(a) Sargent Report (b) Nehru Report
the general model of the University Grants Commis-
(c) Wardha Report (d) None of the above
sion of the United Kingdom.

M10_MADAN 07_65901_C10.indd 50 23/12/22 8:05 PM


Higher Education System 10.51

In the context of above two statements, choose the 84. Which of the following are indicators of external
correct option. accountability for an institution of higher Learning
(a) Both Assertion (A) and Reason (R) are the  [October 2020]
true and Reason (R) is a correct explanation of (A) Provision for professional development of teachers.
Assertion (A). (B) Utilization pattern of library and technical resources.
(b) Both Assertion (A) and Reason (R) are the true (C) Relevance of courses to the societal needs
but Reason (R) is not a correct explanation of (D) Performance of students in public examinations
Assertion (A). (E) Employment pattern of pass-out students.
(c) Assertion (A) is true and Reason (R) is false. Choose the correct answer from the options given below
(d) Assertion (A) is false and Reason (R) is true.
(a) A, B, C only
77. Which of the following statements are true in context (b) B, C, D only
of University Grants Committee? (c) C, D, E only
1. This committee was formed in 1945 to oversee the (d) D, E, A only
work of the three Central Universities of Aligarh, 85. The UGC has the unique distinction of being the only
Banaras and Delhi. grant-giving agency in the country which has been
2. In 1947, the Committee was entrusted with the vested with the two responsibilities of providing funds
responsibility of dealing with all the then existing and that of coordination, determination and mainte-
Universities. nance of standards in institutions of higher education.
3. In 1952, the grants-in-aid function to was referred
to University Grants Commission and this institu- Which of the following points are included under
tion was formally inaugurated in 1953. UGC mandate:
4. UGC became a statutory based organisation by an 1. Promoting and coordinating university education.
Act of Parliament in 1956. 2. Determining and maintaining standards of teach-
ing, examination and research in universities.
Codes: 3. Framing regulations on minimum standards of
(a) 1, 2 and 3 education.
(b) 2 and 3 4. Monitoring developments in the field of collegiate
(c) 1, 3 and 4 and university education; disbursing grants to the
(d) 1, 2, 3 and 4 universities and colleges.
78. The idea of Four Pillars of Education was suggested by 5. Serving as a vital link between the Union and state
(a) UNICEF (b) UGC governments and institutions of higher learning.
(c) NCTE (d) UNESCO 6. Advising the Central and State governments on
the measures necessary for improvement of uni-
79. The name of the plan to increase enrolment in the

A S S E S S YO U R L E A R N I N G
versity education.
higher education institutions is
(a) 1, 2, 3 and 4
(a) Rashtriya Uchchatar Shiksha Abhiyan (RUSA)
(b) 2, 3, 4 and 6
(b) Sarva Shiksha Abhiyan
(c) 1, 2, 3, 4 and 5
(c) Both (a) and (b)
(d) All of the above
(d) None of the above
86. Which section of the UGC Act provides for the promo-
80. Which of the following agencies put forward the con-
tion and coordination of university education and for
cept of proactive university?
the maintenance of standards of teaching, examina-
(a) UNDP (b) UNESCO
tion and research?
(c) AICTE (d) UGC
(a) Section 12 (b) Section 15
81. ‘Destiny of India is being shaped in her classroom.’ (c) Section 25 (d) Section 28
This is stated in
87. Which of the following commission’s report is titled
(a) National Policy on Education (1986)
Education and National Development report?
(b) National Knowledge Commission (2005)
(a) Radhakrishnan Commission
(c) Education Commission (1964–66)
(b) Kothari Commission
(d) University Education Commission (1948–49)
(c) Mudaliar Commission
82. CHEER stands for (d) None of the above
(a) Children Enrichment Education through Radio
88. National Committee on 10 + 2 + 3 education struc-
(b) Child Health Education Electronic Recording
ture set up in 1972 was headed by
(c) Children for Engineers and Energy Requirement
(a) Dr P. D. Shukla (b) Dr D. S. Kothari
(d) None of the above
(c) Dr Radhakrishnan (d) None of the above
83. Educational TV was first introduced in India in the
89. The following commissions were set up by the gov-
year
ernment or its agencies after independence. Arrange
(a) 1961 (b) 1959
the following in chronological order (in terms of their
(c) 1968 (d) 1965
occurrence):

M10_MADAN 07_65901_C10.indd 51 23/12/22 8:05 PM


10.52 Chapter 10

I. University Education Commission Codes:


II. Secondary Education Commission (a) Conventional teaching
III. Education Commission (b) Non-conventional teaching
Codes: (c) Oriental teaching
(a) III, II and I (b) II, I and III (d) Ancient teaching
(c) I, II and III (d) I, III and II 95. Chairman of UGC Committee appointed in 1969 for
90. Consider the following statements. the purpose of administrative legislation of the uni-
1. India has a federal set up and education is the con- versities was
current responsibility of both the centre as well as (a) Dr D. S. Kothari (b) Dr P. B. Gajendragadkar
the states. (c) Prof. Yashpal (d) None of the above
2. After independence, the education (including 96. Which of the following personalities who headed
university education) was the responsibility of Knowledge Commission and headed the committee-
the states, while the centre was given the function recommended setting up of 1500 universities so as
of coordination and determination of standards. to achieve the target gross enrolment of 30%?
3. Through 42nd amendment In 1976, the education (a) Dr D. S. Kothari (b) Sam Pitroda
was put in the concurrent list of the constitution (c) Prof. Yashpal (d) None of the above
where the centre was given the responsibilities 97. Which of the following percentage figures was
along with the states for all levels of education. recommended by both Kothari Commission and
Codes: National Policy that should be spent on education
(a) 2 and 3 (1986)?
(b) 1, 2 and 3 (a) 4% of GDP (b) 5% of GDP
(c) 1 and 3 (c) 6% of GDP (d) None of the above
(d) 1 and 2 98. Which of the following state has the maximum num-
91. The committee set up by the Ministry of Human ber of central universities?
Resource Development (now Ministry of Education) (a) Uttar Pradesh
which recommended autonomy of IITs was headed by (b) Bihar
(a) Prof. Anil Kakodkar (b) Prof. Yashpal (c) Telangana
(c) Sam Pitroda (d) None of the above (d) Rajasthan
92. A committee was set up in 1990 to review NPE (1986). 99. Which of the following institutions or organizations
Its report titled ‘Towards an Enlightened and Humane publishes the Universities Handbook?
Society’ stated that system of higher education (a) University Grants Commission
encourages memorization of facts and regurgitation (b) Association of Indian Universities
A S S E S S YO U R L E A R N I N G

rather than creativity. The head of the committee was (c) Inter-University Centres
(a) Acharya Ramamurti (b) Prof. Yashpal (d) Central Advisory Board of Education
(c) Prof. M. M. Joshi (d) None of the above 100. In which year was the Association of Indian Universi-
93. Which of the following committees recommended the ties (AIU) originally set up?
setting up of National Commission for Higher Educa- (a) 1925 (b) 1945
tion and Research (NCHER) for prescribed standards (c) 1953 (d) None of the above
of academic quality and defining policies for advance- 101. Which of the following authorities is not empowered
ment of knowledge in higher educational institutions? to bring a university into existence?
(a) Sam Pitroda Committee (a) State government (b) Central government
(b) Prof. Yashpal Committee (c) UGC (d) None of the above
(c) M. M. Sharma Committee 102. How many universities were set up under the Cen-
(d) Gnanam Committee tral Universities Act, 2009?
94. Consider the following statements. (a) 12 (b) 16
1. Active learning, in which students move freely, (c) 40 (d) 43
choosing their own work and the pace at which 103. The term open learning represents approaches that
it is being done. Teacher, if any, may just be a focus on [November 2021]
facilitator. A. Opening access to education and training provision
2. There is Holistic development as it includes gen- B. Freeing learners from the constraints of time
eralization of law, theory, rule and concept. and place
Cognitive, social and psychological development C. Learning and evaluation without a specified
of mind. curriculum
3. There should be a problem statement, partial D. Offering flexible learning opportunities to indi-
search and heuristic/ brainstorming methods viduals and groups of learners
Which type of teaching is being reflected in the above E. Making students free from any educational loads
statements.

M10_MADAN 07_65901_C10.indd 52 23/12/22 8:05 PM


Higher Education System 10.53

Choose the correct answer from the options given 112. Institutes of National Importance is an institution in
below: higher education that serves as a pivotal player in
(a) A, B and C only developing highly skilled personnel within the spec-
(b) A B and D only ified region of the country or state. This status can be
(c) A, B and E only conferred on them by
(d) B, Cand D only (a) University Grants Commission
(b) An Act of Parliament
104. Universities having central campus for imparting
(c) All Indian Council for Technical Education
education are called
(d) All of the above
(a) Central universities
(b) Deemed universities 113. Commonwealth of Learning (CoL) is the only official
(c) Residential universities Commonwealth agency located outside Britain. It is
(d) Open universities located in
(a) New Delhi, India
105. An Institution established by Act of Parliament and
(b) Vancouver, Canada
funded by the Government of India and that include
(c) Sydney, Australia
all the IITs, NITs and AIIMs institutes, is called as
(d) Islamabad, Pakistan
(a) Institution of National Importance
(b) Deemed university 114. Which of the following measures have been sug-
(c) Meta university gested by UGC for capacity-building in cluster insti-
(d) Central university tutions of higher education for faculty so that they
can teach, train and research in multi-disciplinary
106. Which of the following are ‘deemed to be university?
academic programmes?
 [November 2021]
A. NCERT (a) Annual Refresher Programme in Teaching (ARPIT)
B. NIEPA (b) Learning assessment tools
C. Jamia Hamdard (c) both a and b
D.Jamia Millia Islamia (d) None of the above
E. IIT 115. EHEI stands for
Choose the correct answer from the options given (a) Equity in Higher Education Institutions
below: (b) Equity in Higher Education Index
(c) Equality in Higher Education Index
(a) A and B only
(d) Equity in Hidden Education Income
(b) B and C only
(c) C and D only 116. API stands for
(d) D and E only (a) Academic Parameter Indicator

A S S E S S YO U R L E A R N I N G
(b) Academic Performance Indicator
107. Which of the following agencies provides funding to
(c) Academic Paid Instalment
Indira Gandhi National Open University?
(d) None of the above
(a) University Grants Commission
(b) Ministry of Education 117. National Literacy Mission was launched with the
(c) Both (a) and (b) approval of the Cabinet as an independent and
(d) None of the above autonomous wing of the Ministry of Education (then
MHRD) in the year
108. The university that telecasts interactive educational
(a) 1996 (b) 1988
programmes through its own channel is
(c) 1999 (d) 2000
(a) Dr B. R. Ambedkar Open University, Hyderabad
(b) IGNOU 118. The main aim of National Council for Teacher Edu-
(c) University of Pune cation is
(d) Annamalai University (a) To open college of education
(b) To promote research in education
109. Which of the following pairs of authorities are at the
(c) To maintain standards in colleges of education
top of hierarchy in the case of a central university?
(d) To provide grant to colleges of education
(a) Visitors and Chancellor
(b) Visitors and Vice Chancellor 119. National Council of Educational Research and Train-
(c) Chancellor and Vice Chancellor ing was established in
(d) Vice President and Vice Chancellor (a) 1961 (b) 1962
(c) 1963 (d) 1964
110. The state with most universities is
(a) Tamil Nadu (b) Andhra Pradesh 120. Which of the following organizations set up the
(c) Rajasthan (d) Uttar Pradesh National Assessment and Accreditation Council?
(a) AICTE
111. The state with most deemed universities is
(b) UGC
(a) Tamil Nadu (b) Andhra Pradesh
(c) Ministry of Education
(c) Maharashtra (d) Karnataka
(d) All of the above

M10_MADAN 07_65901_C10.indd 53 23/12/22 8:05 PM


10.54 Chapter 10

121. National Assessment and Accreditation Council (NAAC) 129. In which of the following years was ‘The National
is headquartered in Policy of Education’ revised and a ‘Programme of
(a) New Delhi (b) Mumbai Action’ also attached to it?
(c) Hyderabad (d) Bengaluru (a) 1968 (b) 1976
122. NUEPA is mainly concerned with (c) 1986 (d) 1992
(a) Educational supervision 130. Which of the following days is celebrated as National
(b) Educational unity Education Day?
(c) Educational planning (a) 5 September (b) 2 October
(d) Educational evaluation (c) 11 November (d) 14 November
123. Which of the following statements are correct about 131. In order to achieve the 12th Five-Year Plan’s Inclu-
a Central university? [Jun 2014] sive and Qualitative Expansion of Higher Education,
1. Central university is established under an Act of the number of proposed women universities is
Parliament. (a) 10 (b) 15
2. The President of India acts as the visitor of the (c) 20 (d) 30
University. 132. Apart from few short-term programmes, academic
3. The President has the power to nominate some staff colleges set up for teachers’ training basically offer
members to the Executive Committee or the (a) Subject-specific refresher courses
Board of Management of the university. (b) General orientation programmes
4. The President occasionally presides over the (c) Both (a) and (b)
meetings of the Executive Committee or court. (d) None of the above
Select the correct answer from the codes given
below. 133. Which of the following institutes is/are funded by
central government?
Codes: (a) National Institute of Foundry and Forge
(a) 1, 2, and 4 Technology, Ranchi
(b) 1, 3, and 4 (b) North Eastern Regional Institute of Science and
(c) 1, 2 and 3 Technology, Itanagar
(d) 2, 3 and 4 (c) Sant Longowal Institute of Engineering and
124. The number of representatives of central government Technology, Longowal, Punjab
in UGC is (d) All of the above
(a) 9 (b) 2 134. The first committee to be constituted after inde-
(c) 6 (d) 3 pendence by the Government of India on Women’s
125. Which of the following agencies or institutions set up education was
A S S E S S YO U R L E A R N I N G

the Accreditation Board (AB)? (a) Dr. Radhakrishnan Commission


(a) Indian Council of Agricultural Research (b) The Mudaliar Commission
(b) All India Council for Technical Education (c) The Shri Sri Prakasa Committee
(c) University Grants Commission (d) Dr. Durgabai Deshmukh Committee
(d) Ministry of Education 135. Which article of the Constitution provides for the
126. Higher Education institutions go through a rigorous rights of minorities to establish and administer the
process to validate that they meet stringent educa- institutions of their choice?
tional standards. This is known as (a) Article 21(1) (b) Article 30(1)
(a) Accreditation (c) Article 32 (d) None of the above
(b) Evaluation 136. In which of the following years was National Com-
(c) Assessment mission for Minorities Educational Institutions
(d) Test Marketing (NCMI) set up?
127. The National Board of Accreditation (NBA) was set (a) 2001 (b) 2003
up in the year 1987 by (c) 2004 (d) 2005
(a) AICTE 137. Which of the following is a specialized agency of UN
(b) UGC to deal with copyright and other intellectual prop-
(c) Ministry of Education erty rights?
(d) IGNOU (a) WIPO (b) GATS
128. Which of the following types of institutions come (c) GATT (d) None of the above
under the ambit of National Board of Accreditation 138. Which of the following universities has been assigned
(NBA)? [October 2020] the status of central university in 2013?
(a) Teacher education institutions (a) Panjab University
(b) Technical institutions (b) Osmania University
(c) Agriculture institutions (c) Nalanda University
(d) Medical institutions (d) Indira Gandhi National Tribal University

M10_MADAN 07_65901_C10.indd 54 23/12/22 8:05 PM


Higher Education System 10.55

139. Rashtriya Sanskrit Vidyapeetha is located in 145. Assertion (A): Oriental education became impor-
(a) Tirupati (b) Chennai tant to evolve a method by which our ancient knowl-
(c) Hyderabad (d) Jaipur edge and system can be maintained, revived and
140. Which of the following are the basic guidelines for preserved.
declaring a language as ‘Classical’? Reason (R): The term orient mostly refers to eastern
1. High antiquity of its early texts/recorded history cultures where the parents, indigenous places of wor-
over a period of 1500-2000 years; ship, and local pathshalas play an important role.
2. A body of ancient literature/texts, which is con- In the context of above two statements, choose the
sidered a valuable heritage by generations of correct option.
speakers;
3. The literary tradition be original and not bor- (a) Both Assertion (A) and Reason (R) are the
rowed from another speech community; true and Reason (R) is a correct explanation of
4. The classical language and literature being dis- Assertion (A).
tinct from modern, there may also be a discon- (b) Both Assertion (A) and Reason (R) are the true
tinuity between the classical language and its but Reason (R) is not a correct explanation of
later forms or its offshoots Assertion (A).
(a) 2, 3 and 4 (c) Assertion (A) is true and Reason (R) is false.
(b) 1, 2, 3 and 4 (d) Assertion (A) is false and Reason (R) is true.
(c) 1, 3 and 4 146. Match List I with List II and select the correct answer
(d) 1, 2 and 3 from the codes given below.
141. Which of the following languages was assigned the
status of classical language in 2013? List I List II
(a) Sanskrit (b) Tamil A. National Council of Rural I. New Delhi
(c) Hindi (d) Malayalam Institutes
142. Which of the following institutions have been given B. Indian Council of H
­ istorical II. Hyderabad
the Central status under the Central Sanskrit Uni- Research
versities Bill, 2019?
(a) Rashtriya Sanskrit Sansthan, New Delhi C. Indian Council of III. Shimla
(b) Shri Lal Bahadur Shastri Rashtriya Sanskrit Philosophical Research
Vidyapeeth, New Delhi D. Indian Institute of IV. Bengaluru
(c) Rashtriya Sanskrit Vidyapeeth in Tirupati. Advanced Study
(d) All of the above
Codes:

A S S E S S YO U R L E A R N I N G
143. Which of the following statements are correct about
classical status of languages in India? (a) A—II, B—IV, C—I and D—III
1. Currently there are six languages that enjoy the (b) A—II, B—IV, C—III and D—I
‘Classical’ status in India, with Tamil being the (c) A—II, B—III, C—IV and D—I
first one in 2004 and Odia being the last one in (d) A—II, B—I, C—IV and D—III
2014.
2. All the Classical Languages are listed in the 147. Match List I with List II [November 2021]
Eighth Schedule of the Constitution. List I List II
3. The Ministry of Culture provides the guidelines
regarding Classical languages. A. NIEPAI I. Promotion of quality in
4. Sanskrit is not the part of the classical languages technical education
list. B. NAAC II. To promote research in social
Codes: sciences
(a) 2, 3 and 4 (b) 1, 2, 3 and 4 C. AICTE III. Planning and management
(c) 1, 3 and 4 (d) 1, 2 and 3 of education
144. Choose the oriental learning institutes in India from D. ICSSR IV. Accreditation of higher
the following: [December 2019] educational institutions
A. Asiatic society
B. Madras Sanskrit College Choose the correct answer from the options given
C. Tezpur University below:
D.Mythic Society
(a) A – III, B -IV, C – II, D - I
Choose the correct option: (b) A – IV, B – III, C – II, D -I
(a) A, B and D only (b) A, B and C only (c) A – III, B- IV, C -I, D -II
(c) A, C and D only (d) B, C and D only (d) A – IV, B – II, C -I, D -III

M10_MADAN 07_65901_C10.indd 55 23/12/22 8:05 PM


10.56 Chapter 10

148. The term open learning represents approaches that (a) Distance Education Council (DEC)
focus on [November 2021] (b) National Council for Teacher Education (NCTE)
A. Opening access to education and training provision (c) National Council of Educational Research and
B. Freeing learners from the constraints of time and Training (NCERT)
place (d) National Assessment and Accreditation Council
C. Learning and evaluation without a specified (NAAC)
curriculum 156. Centre for Cellular and Molecular Biology is located in
D. Offering flexible learning opportunities to indi- (a) Hyderabad (b) Chennai
viduals and groups of learners (c) Bengaluru (d) Ahmedabad
E. Making students free from any educational loads 157. Which of the following regulatory bodies does not
Choose the correct answer from the options given enjoy statutory status as a professional body?
below: (a) Bar Council of India
(a) A, B and C only (b) A B and D only (b) All India Council for Technical Education
(c) A, B and E only (d) B, Cand D only (c) Medical Council of India
(d) None of the above
149. CBCS is [November 2021]
(a) Criteria Based Choice System 158. Under which of the following constitutional amend-
(b) Choice Based Credit System ments was education transferred to the Concurrent List?
(c) Criteria Based Creditable Scores (a) 42nd (b) 73rd
(d) Credit Based Choice Scores (c) 74
th
(d) 101st
150. CBCS provides an opportunity for students to choose 159. UGC gets its funding from
courses from: [November 2021] (a) Central government only
(b) State governments
A. Core courses (c) Both central and state governments
B. Environmental education (d) None of the above
C. Elective courses
D.Computer courses 160. Which of the following universities in India has the
E. Ability enhancement courses Prime Minister as its Chancellor?
(a) Indira Gandhi National Open University
Choose the correct answer from the options given (b) Visva-Bharati University
below: (c) Indian Maritime University
(a) A, B and C (b) B, C and D (d) Indian Defence University
(c) C, D and E (d) A, C and E 161. Faculty Improvement Programme of UGC aims at
#ntaugc-netimportantquestions

151. In CBCS, Ability Enhancement Courses are of two (a) Appointing additional faculty
kinds. They are:  [November 2021] (b) Making faculty research oriented
A. Ability Enhancement Compulsory Courses (c) Improving physical infrastructure
B. Skill Enhancement Compulsory Courses (d) None of the above
C. Ability Enhancement Elective Courses 162. Mahatma Gandhi Antarrashtriya Hindi Vishwavidy-
D.Skill Enhancement Elective Courses alaya is located at
E. Skill Enhancement Courses (a) New Delhi (b) Jaipur
Choose the correct answer from the options given below (c) Porbandar (d) Wardha
(a) A and B only (b) A and C only 163. CABE was set up in the year
(c) A and D only (d) A and E only (a) 1925 (b) 1857
(c) 1956 (d) 1992
152. Which of the following project(s) is/are funded by
external agencies? 164. IGNOU was set up in the year
(a) Technical Education Quality Improvement (a) 1985 (b) 1988
Programme (c) 1991 (d) 1992
(b) Technician Education Project 165. In June 2013, Distance Education Council was taken
(c) Colombo Plan Staff College, Manila over by newly established Distance Education Bureau
(d) All of the above to govern the distance education programmes in
153. The South Asian University is located at India. Distance Education Bureau was established by
(a) New Delhi (b) Jaipur (a) UGC (b) Ministry of Education
(c) Kathmandu (d) Islamabad (c) AICTE (d) IGNOU
154. The Vikram Sarabhai Space Research Centre is 166. There was conflict of interest between UGC and
located at IGNOU. According to court order, IGNOU cannot
(a) Thiruvananthapuram (b) Sriharikota guide other universities for running their Open and
(c) Pune (d) Bengaluru Distance Learning courses through its
(a) Distance Education Council
155. Which of the following Councils has been disbanded
(b) Distance Education Bureau
in 2013?

M10_MADAN 07_65901_C10.indd 56 23/12/22 8:05 PM


Higher Education System 10.57

(c) Distance Education Branch (c) Harvard Business School


(d) None of the above (d) Delhi University
167. The Ministry of Human Resource Development, that 175. Which of the following is a private university? 
existed before Ministry of Education was created in  [November 2021]
the year (a) Anna University
(a) 1972 (b) 1980 (b) BITS Pilani
(c) 1985 (d) 1991 (c) Burdwan University
168. Which of the following personalities was the first (d) IIIT
Chairman of UGC? 176. Universities with NAAC ‘A’ Grade and 10 years of
(a) Dr Shanti Swaroop Bhatnagar standing can get financial support from UGC for
(b) Dr Manmohan Singh innovative teaching or educational programmes;
(c) Shri Humayun Kabir these universities are known as
(d) Shri C. D. Deshmukh (a) Meta Universities
169. According to University Grants Commission (Mini- (b) Innovation universities
mum Standards and Procedure for Award of Ph.D. (c) Central universities
Degree) Regulations, 2022, which of the following (d) None of the above
statements apply in the context of PhD degree? 177. In which of the following countries did the Hum-
1. The students ideally with four years of degree boldtian Concept of University Education originate?
course will be allowed to do PhD. (a) USA (b) Poland
2. Ph.D. programme shall be for a minimum dura- (c) France (d) Germany
tion of two years excluding course work, and a 178. The major concept underlying distance education is
maximum of six years. (a) Continuing education
3. Extension beyond the above limits will not be (b) Simplicity
beyond more than the two years. (c) Cost-effectiveness
4. Women candidates may be allowed a relaxa- (d) Objectivity
tion of two years for a Ph.D. in the maximum 179. The course content of a professional degree empha-
duration. sizes on
(a) 1, 2, 3 and 4 (b) 1, 2 and 3 (a) Acquiring skills and practical analysis
(c) 2, 3 and 4 (d) 2 and 3 (b) Theory and research
170. The first open university in India was (c) Building concepts
(a) Andhra Pradesh Open University (d) None of the above

#ntaugc-netimportantquestions
(b) Indira Gandhi National Open University 180. The number of universities at the time of independ-
(c) Delhi University ence Was
(d) None of the above (a) 15 (b) 20
171. The first university in India to start correspondence (c) 28 (d) 35
course was 181. Networking of libraries through electronic media is
(a) Andhra Pradesh Open University called
(b) IGNOU (a) Inflibnet (b) Libinfnet
(c) Delhi University (c) The internet (d) HTML
(d) None of the above 182. The first virtual university of India came up in
172. The National Knowledge Commission, a high-level (a) Andhra Pradesh (b) Maharashtra
advisory body to the Prime Minister of India, with (c) Uttar Pradesh (d) Tamil Nadu
the objective of transforming India into a knowledge 183. Match List I with List II and select the correct answer
society was set up in the year from the codes given below.
(a) 2005 (b) 2006
(c) 2007 (d) 2008 List I List II
173. The concept of a countrywide network for higher A. Vikram Sarabhai I. Thiruvananthapuram
education that would allow students the flexibility Space Research
to design their own curriculum and combine sub- Centre
jects of their choice is basically of
(a) Innovation university B. Centre for Cellular II. Hyderabad
(b) Meta university and Molecular
(c) Central university Biology
(d) Deemed university C. Indian Space III. Bengaluru
174. The concept of Meta University was first pro- Research
pounded by Organization
(a) Massachusetts Institute of Technology D. The Energy and IV. New Delhi
(b) Jawaharlal Nehru University Resources Institute

M10_MADAN 07_65901_C10.indd 57 23/12/22 8:05 PM


10.58 Chapter 10

Codes: (b) Jadavpur University (Kolkata), Jawaharlal


(a) A—I, B—II, C—III and D—IV Nehru University (New Delhi), Jamia Millia
(b) A—I, B—III, C—II and D—IV Islamia, (New Delhi)
(c) A—III, B—II, C—I and D—IV (c) Jawaharlal Nehru University (New Delhi),
(d) A—III, B—I, C—II and D—IV Indian Institute of Science (Bengaluru),
Jadavpur University (Kolkata)
184. In which year was the NBA set up? (d) Indian Institute of Science (Bengaluru), Jamia
(a) 1991 (b) 1994 Millia Islamia, (New Delhi), Jadavpur University
(c) 2001 (d) 2005 (Kolkata)
185. What does NIRF stand for? [November 2021] 190. Which is the ranking of the top research institute in
(a) National Institute of Ranking Federation NIRF – 2022 list?
(b) National Institute of Ranking Framework (a) Indian Institute of Technology Madras, Chennai
(c) National institutional Ranking Framework (b) Indian Institute of Science, Bengaluru
(d) National Institutional Ranking Federation (c) Indian Institute of Technology Bombay, Mumbai
186. Which of the following statements are true in context (d) Indian Institute of Technology Delhi, New Delhi
of National Institutional Ranking Framework (NIRF)? 191. Which is the ranking of the top management insti-
1. NIRF has five parameters - Teaching, learning tute in NIRF – 2022 list?
& Resources, Research & Professional Practice, (a) Indian Institute of Management Ahmedabad
Graduation Outcome, Outreach & Inclusivity (b) Indian Institute of Management Bangalore
And Perception. (c) Indian Institute of Management Calcutta
2. NIRF was approved by the MHRD and was in (d) Indian Institute of Technology, Delhi
September 2015.
3. The first results were published in 2016 where 192. India has been attracting only a fraction of interna-
ranking is a six monthly affair. tional students because of [November 2021]
4. Accreditation gives absolute grade, ranking is A. Lack of residential accommodation for foreign
relative to the other institutions similarly placed students
in context of NIRF. B. Difficulties in recognition of international
qualification
Codes: C. Rigid admission process
(a) 2, 3 and 4 D. Lack of enthusiasm among teachers
(b) 1, 2, 3 and 4 E. Lack of enthusiasm among students
(c) 1, 2 and 4 Choose the correct answer from the options given
(d) 1, 2 and 3 below
187. Which of the institutions has shown the top rank in
A S S E S S YO U R L E A R N I N G

(a) A and B only


the category of ‘overall’ institutions in NIRF – 2022 (b) B and C only
list? (c) Cand D only
(a) Indian Institute of Technology Madras, Chennai (d) D and E only
(b) Indian Institute of Science, Bengaluru
(c) Indian Institute of Technology Bombay, Mumbai 193. Aligarh Muslim University was set up in the year
(d) Indian Institute of Technology Delhi, New Delhi (a) 1857 (b) 1875
(c) 1905 (d) 1916
187. Which is the ranking of the three top colleges in a
sequence in the category of ‘colleges’ in NIRF – 2022 194. SITE means
list? (a) Satellite Instructional Television Experiment
(a) Miranda House (Delhi), Hindu College (Delhi), (b) Satellite Informational Television Experiment
Presidency College (Chennai) (c) Satellite Instructional Telecom Experiment
(b) Miranda House (Delhi), Hindu College (Delhi), (d) Satellite Informational Telecom Experiment
Loyola College (Chennai) 195. Name of the research station on Antarctica set up by
(c) Hindu College (Delhi), Miranda House (Delhi), India is
Presidency College (Chennai) (a) Dakshin Gangotri
(d) Hindu College (Delhi), Loyola College (b) Dakshin Ganga
(Chennai), Presidency College (Chennai) (c) Dakshin Godawari
189. Which is the ranking of the three top universities in (d) None of the above
a sequence in the category of ‘universities’ in NIRF 196. The foundation training of selected IAS officers is
– 2022 list? conducted at
(a) Indian Institute of Science (Bengaluru), (a) LBS National Academy of Public Administration,
Jawaharlal Nehru University (New Delhi), Mussoorie
Jamia Millia Islamia, (New Delhi) (b) MG Institute of Public Administration, Chandigarh
(c) UPSC, New Delhi
(d) All state capitals

M10_MADAN 07_65901_C10.indd 58 23/12/22 8:05 PM


Higher Education System 10.59

197. The main objective of ‘National Mission on Teachers (c) Inter-University Accelerator Centre (formerly
and Training’ is to improve Nuclear Science Centre)
(a) Students’ enrolment (d) None of the above
(b) Quality of teaching 205. Which of the following was the first private institu-
(c) Women education tion to be granted the status of a deemed university
(d) All of the above in 1976?
198. PACER is the acronym for (a) Manipal Academy of Higher Education
(a) Professionals Action Committee for Educational (b) Symbiosis Institute, Pune
Reforms (c) Thapar University, Patiala
(b) Professionals Accreditation Committee for (d) Amity University, Noida
Educational Reforms 206. Match List I with List II and select the correct answer
(c) Professionals Action Committee for Educational from the codes given below.
Research
(d) None of the above List I List II
199. Mahila Samakhya was born out of the New Edu-
cation Policy of 1986, which stressed the need for A. Chinmaya ­International I. Delhi
an intervention to create gender equality through Foundation
women’s education. It was launched in the year B. The Oriental ­Institute II. Baroda
(a) 1989 (b) 1992 C. Academy of S
­ anskrit III. Melkote
(c) 1996 (d) 1998 Research (Karnataka)
200. Who was the chairman of University Education
Commission set up in 1948? D. Rashtriya Veda Vidya IV. Ernakulum
(a) Dr S. Radhakrishnan Pratishthan
(b) Dr D. S. Kothari
(c) Maulana Abul Kalam Azad Codes:
(d) C. D. Deshmukh (a) A—IV, B—II, C—III and D—I
201. University Education Commission recommended the (b) A—II, B—IV, C—III and D—I
setting up of UGC on the lines of University Grants (c) A—II, B—III, C—IV and D—I
Commission of (d) A—II, B—I, C—IV and D—III
(a) USA 207. Match List I with List II [November 2021]
(b) Great Britain
(c) Germany List I List II

A S S E S S YO U R L E A R N I N G
(d) USSR A. Indian Institute of
I. Noida, UP
202. Which of the following agencies or institutions pro- Advanced Studies
posed introduction of India Education Index (IEI) B. Indian Biological Sciences
for ranking institutes based on academic, research, II. Shimla,HP
and Research Institute
performance and other parameters?
(a) AICTE C. Indian Institute of Social
III. Bhopal, MP
(b) NAAC Sciences
(c) UGC Indian Institute of IV. Lucknow, UP
(d) IIT Delhi D. Sugarcane Research
203. The largest university in the world in terms of stu-
dent enrolment is Choose the correct answer from the options given
(a) Beijing University, Beijing, China below:
(b) Dr B. R. Ambedkar University Open University, (a) A – II, B -I, C – III, D - IV
Hyderabad (b) A – III, B – IV, C – II, D -I
(c) Indira Gandhi National Open University, New (c) A – IV, B- III, C -II, D -I
Delhi (d) A – I, B – IV, C -II, D -III
(d) None of the above 208. Through which of the following constitutional
204. The first Inter-University Centre which was estab- amendments was Article 21A inserted in the Consti-
lished by UGC in 1984 is tution to provide for free and compulsory education
(a) Inter-University Accelerator Centre (Nuclear to all children of age group between 6 and 14 years
Science Centre) as a fundamental right?
(b) Inter-University Centre for Astronomy and (a) 73rd (b) 74th
Astro-Physics (IUCAA) (c) 86th (d) None of the above

M10_MADAN 07_65901_C10.indd 59 23/12/22 8:05 PM


10.60 Chapter 10

209. Match List I with List II and select the correct answer 217. The Right of Children to Free and Compulsory Edu-
from the codes given below. cation Act, also popular as Right to Education Act
(RTE Act), was enacted in the year
List I List II (a) 2008 (b) 2009
(Institutions) (Locations) (c) 2010 (d) 2011
A. National Law Institute I. Shimla 218. Right to Education Act (RTE) makes it mandatory to
provide free and compulsory education for children
B. Indian Institute of age group between
II. Bhopal
Advanced Studies (a) 6 and 14 years
C. National Judicial Academy III. Hyderabad (b) 6 and 12 years
D. National Savings Institute IV. Nagpur (c) 8 and 14 years
(d) None of the above
Codes: 219. The Right to Education Act came into force in the year
(a) A—III, B—II, C—IV and D—I (a) 2008 (b) 2009
(b) A—I, B—II, C—III and D—IV (c) 2010 (d) None of the above
(c) A—IV, B—III, C—I and D—II 220. Which of the following articles of Constitution states
(d) A—III, B—I, C—II and D—IV that the offi cial language of the union shall be Hindi
210. Indian Institute of Mass Communication is located in in Devanagari script?
(a) Ahmedabad (a) 343(1) (b) 356
(b) New Delhi (c) 75 (d) 351
(c) Hyderabad 221. The first language to have been assigned the status
(d) Jaipur of classical language in 2004 is
211. Indian National Defence University (INDU), India’s (a) Tamil (b) Sanskrit
first defence university, which was inaugurated on (c) Kannada (d) Malayalam
May 2013, is proposed to be located at 222. Odia has been assigned the status of classical lan-
(a) Rohtak guage in February 2014. With this, the number of
(b) Pune classical languages in India is
(c) Hyderabad (a) Three (b) Four
(d) Gurgaon (c) Five (d) Six
212. The Ministry of Education includes 223. Ninety-third Constitution Amendment Bill seeks
(a) Department of Elementary Education and (a) To grant statehood to Uttaranchal
Literacy (b) To make elementary education compulsory
A S S E S S YO U R L E A R N I N G

(b) Department of Secondary Education and Higher (c) To make army service mandatory
Education (d) None of the above
(c) Department of Women and Child Development
(d) All of the above 224. Which of the following parts of the Indian Constitution
enjoins upon the state to provide free and compulsory
213. The language which enjoys special status in Article education for children up to 14 years of age?
351 of the Constitution of India as the primary (a) Directive Principles of State Policy
source language for the development of the official (b) Fundamental rights
standard of Hindi is (c) Fundamental Duties
(a) Sanskrit (b) Tamil (d) Special provisions related to certain classes
(c) Bhojpuri (d) Maithili
225. Which statutory body of a University has the power
214. Which of the following is the official language(s) as to accord formal approval to the programmes and
per Indian Constitution? courses of studies? [October 2020]
(a) Hindi (a) Senate
(b) English (b) Syndicate
(c) Hindi with English as an additional language (c) Academic Council
(d) No official language (d) Board of Studies
215. The national language of India is 226. The education imported through institutions of
(a) Hindi higher learning in India such as ‘Sanskrit Vidyapith’
(b) English is an example of [October 2020]
(c) Both English and Hindi (a) Non-conventional learning Programmes
(d) There is no national language (b) Value Education Programmes
216. Which of the following High Courts gave the ruling (c) Oriental learning Programmes
that India has no national language? (d) Professional education programmes
(a) Mumbai (b) Gujarat
(c) Lucknow (d) Patna

M10_MADAN 07_65901_C10.indd 60 23/12/22 8:05 PM


Higher Education System 10.61

227. List I gives stages of development of values while List In the light of the above statements, choose the most
II their outcomes [October 2020] appropriate answer from the options given below
Match List I with List II (a) Statement I is incorrect, but Statement II is correct
(b) Both Statement l and Statement l are correct
List I (c) Both Statement I and Statement II are incorrect
(Stage of development List II (Outcomes) (d) Statement I is correct, but Statement II is incorrect
of values
229. Statement I : The vice-chancellor of a University is an
A. Stage I I. Value Collection executive head who is vested with administrative as
through invitation well as academic responsibilities.
B. Stage II II. Inner evaluation Statement II: For a charismatic leadership attribute in
a vice chancellor both transactional as well as trans-
C. Stage III III. Value consolidation formational leadership competencies are needed.
D. Stage IV IV. Value clarification In the light of the above statements, choose the cor-
rect answer from the options given below:
Choose the correct answer from the options given (a) Both Statement I and Statement I are true
below. (b) Both Statement I and Statement I are false
(a) A – II, B - III, C – I, D - IV (c) Statement I is correct but Statement II is false
(b) A – I, B – II, C – IV, D -III (d) Statement I is incorrect but Statement I is true
(c) A – IV, B- I, C -III, D - II 230 Global Citizenship Education promotes
(d) A – III, B – IV, C -II, D -I  [November 2021]
228. Given below are two statements [November 2021] (a) Common international laws for global citizenship
Statement I: Policy action and implementation plans (b) Issues pertaining to citizenship across countries
require sound database systems at the regional level involving VISA
only. (c) International public relations
(d) More peaceful, tolerant, inclusive, secure and sus-
Statement II: The statistical system should ensure its
tainable societies
impeccability with data architecture, security, quality,
cleaning integration.

More Questions on New Education Policy


231. Given below are two statements, one is labelled as 2. This will replace 10+2 School Curricula with

A S S E S S YO U R L E A R N I N G
Assertion (A) and other is labelled as Reason (R). 5+3+3+4 curricular structure.
Assertion (A): The National Education Policy, 2020 3. This policy aims to raise Gross Enrolment Ratio
envisions an education system rooted in Indian ethos in higher education from current 26.3% to 50 %
that contributes directly to transforming India, that by 2035.
is Bharat, sustainably into an equitable and vibrant 4. This policy is likely to increase the public invest-
knowledge society, by providing high-quality educa- ment in Education sector to 6% of GDP from cur-
tion to all, and thereby making India a global knowl- rent 4.6 percent.
edge superpower. (a) 1, 2 and 3
(b) 2 and 3
Reason (R): The implementation of previous poli-
(c) Both 1 and 2
cies on education has focused largely on issues of
(d) All of the above
access and equity.
233. To translate this particular vision of NEP-2020, under
In the context of above two statements, choose the
the ‘Atmanirbhar Bharat’ programme, which of the
correct option.
following statements are linked to?
(a) Both Assertion (A) and Reason (R) are the
true and Reason (R) is a correct explanation of 1. National Initiative for Proficiency
Assertion (A). 2. NIPUN—Bharat
(b) Both Assertion (A) and Reason (R) are the true 3. The learning needs of nearly five crore children in
but Reason (R) is not a correct explanation of the age group of 3 to 11 years are to be achieved.
Assertion (A). 4. The mission is to take a holistic approach that
(c) Assertion (A) is true and Reason (R) is false. includes all stakeholders.
(d) Assertion (A) is false and Reason (R) is true. Codes:
232. Which of the following statement/s is/are correct in (a) 1, 2 and 3
context of New Education Policy (NEP), 2020? (b) 2 and 3
1. The recommendations of this NEP were made by (c) Both 1 and 2
former ISRO chief K Kasturirangan. (d) All of the above

M10_MADAN 07_65901_C10.indd 61 23/12/22 8:05 PM


10.62 Chapter 10

234. By which of the following year, the minimum degree 242. Which of the following statements of New Education
qualification for teaching is going to be a 4-year inte- Policy, 2020 are to be implemented for universalisa-
grated B. Ed degree? tion of education from pre-school to secondary level?
(a) 2021 (b) 2025 (a) 100 per cent Gross Enrolment Ratio in school
(c) 2028 (d) 2030 education by 2030
235. Which of the following agency is likely to be created (b) 50% Gross Enrolment Ratio in higher education
as an apex body for fostering a strong research cul- to 50 per cent by 2025.
ture and building research capacity across higher (c) Both a and b
education as per NEP 2020? (d) None of the above
(a) National Commission Foundation 243. A per NEP 2020, National Assessment Centre, PAR-
(b) HigherEducation Agency AKH will be set up as a standard-setting body. What
(c) National Research Foundation does letter ‘R’ stand for in PARAKH?
(d) Higher Education Commission (a) Revise
236. Which of the following agencies aims to use artifi- (b) Rational
cial Intelligence to make learning more personalized (c) Review
and customized as per the learner’s requirement (d) Revival
that even proposes to create national alliance with 244. What does letter ‘P’ stand for in NPST that finds men-
EdTech companies for a better learning experience? tion in NEP 2020?
(a) National Testing Agency (a) Primary
(b) National Testing Portal (b) Professional
(c) National Educational Alliance for Technology (c) Public
(d) National Educational Association for Technology (d) Probation
237. Which of the following agency regarding multiple 245. Which of the following number of new seats will be
entry and exit options in UG stream as per New Edu- added to Higher education institutions?
cation Policy, 2020 is not correctly matched? (a) 2.1 Crore
(a) Certificate after 1 year (b) 2.5 Crore
(b) Advanced Diploma after 2 years (c) 3.2 Crore
(c) Bachelor’s Degree after 3 years (d) 3.5 Crore
(d) Bachelor’s with Research after 5 years 246. Higher Education Commission of India (HECI) that
238 As per NEP 2020, teaching upto at least what grade will act as a single overarching umbrella body the for
to be in mother tongue/ regional language? entire higher education does not include which edu-
(a) Grade 3 cation sectors?
A S S E S S YO U R L E A R N I N G

(b) Grade 5 (a) Medical and Legal education


(c) Grade 8 (b) Engineering and Management
(d) Grade 10 (c) Legal education and Engineering
239. National Curricular and Pedagogical Framework for (d) Management and Medical
Early Childhood Care and Education (NCPFECCE) 247. In how many years affiliation of colleges is to be
for children up to the age of 8 will be developed by phased out?
which organisation? (a) 10
(a) NITI Ayog (b) 12
(b) NCERT (c) 15
(c) AICTE (d) 20
(d) NCTE 248. Which of the following is the full form of NMM?
240. National Committee for the Integration of Voca- (a) National Mission for Mentoring
tional Education (NCIVE) will be set as per NEW (b) National Maritime Museum
Education Policy, 2020. Vocational education will (c) New Multidisciplinary Mission
start inschools from the which grade that will also (d) Net Market Makers
include internships? 249. In order to transform the regulatory system of
(a) Grade 3 Higher Education, the NEP proposes to set up HECI
(b) Grade 4 (Higher Education Commission of India). Which of
(c) Grade 5 the following are verticals of HECI?
(d) Grade 6  [November 2021]
241. Which of the following agency will develop National A. NHERC
Curricular Framework for School Education (NCFSE) B. NAS
2020–21? C. HEGC
(a) NCERT (b) AICTE D.NCIVE
(c) IITs and NITs (d) NCTE E. NAC

M10_MADAN 07_65901_C10.indd 62 23/12/22 8:05 PM


Higher Education System 10.63

Choose the correct answer from the options given synchronisation with New Education Policy
below: (NEP-2020).
(a) A, B and C only 3. Under HECI, all the courses approved by UGC,
(b) B, C and E only AICTE and NCTE shall be mapped for their
(c) A, C and E only equivalence under National Higher Education
(d) C, D and E only Qualification Framework (NHEQF) which
250. As per UGC sources, which of the following state- would make academic mobility smoother, both
ments are correct in context of Higher Education vertically and laterally.
Commission of India (HECI)? Codes:
1. UGC, AICTE and National Council of Teacher (a) 1 and 3
Education are to be merged to form HECI. (b) 2 and 3
2. HECI will work for autonomy and multidisci- (c) 1 and 2
plinary approach to education. This will be in (d) 1, 2 and 3

A S S E S S YO U R L E A R N I N G

M10_MADAN 07_65901_C10.indd 63 23/12/22 8:05 PM


10.64 Chapter 10

Answer Keys

Ancient Education
1. (a) 2. (c) 3. (b) 4. (c) 5. (a) 6. (b) 7. (b) 8. (b) 9. (c) 10. (a)
11. (d) 12. (c) 13. (b) 14. (c) 15. (c) 16. (c) 17. (b) 18. (a) 19. (a) 20. (d)
21. (b) 22. (c) 23. (a) 24. (c) 25. (a) 26. (b) 27. (a) 28. (b) 29. (a) 30. (d)
31. (b) 32. (a) 33. (b) 34. (c) 35. (d) 36. (a) 37. (c) 38. (d) 39. (c) 40. (b)
41. (b) 42. (b) 43. (a) 44. (c) 45. (a) 46. (a) 47. (d) 48. (a) 49. (a) 50. (c)
51. (a) 52. (a) 53. (b) 54. (a) 55. (d) 56. (c) 57. (c) 58. (c)
Basics of Modern Education, Institutions of Higher Education
59. (d) 60. (a) 61. (a) 62. (a) 63. (a) 64. (a) 65. (a) 66. (b) 67. (a) 68. (b)
69. (a) 70. (d) 71. (d) 72. (d) 73. (d) 74. (c) 75. (a) 76. (a) 77. (d) 78. (d)
79. (a) 80. (b) 81. (c) 82. (a) 83. (c) 84. (c) 85. (d) 86. (a) 87. (b) 88. (a)
89. (c) 90. (b) 91. (a) 92. (a) 93. (b) 94. (b) 95. (b) 96. (b) 97. (c) 98. (a)
99. (b) 100. (a) 101. (c) 102. (b) 103. (b) 104. (b) 105. (a) 106. (b) 107. (b) 108. (b)
109. (a) 110. (c) 111. (a) 112. (b) 113. (b) 114. (c) 115. (b) 116. (b) 117. (b) 118. (c)
119. (a) 120. (b) 121. (d) 122. (c) 123. (c) 124. (b) 125. (a) 126. (a) 127. (a) 128. (b)
129. (d) 130. (c) 131. (c) 132. (c) 133. (d) 134. (d) 135. (b) 136. (c) 137. (a) 138. (c)
139. (a) 140. (b) 141. (d) 142. (d) 143. (d) 144. (a) 145. (a) 146. (a) 147. (c) 148. (b)
149. (b) 150. (d) 151. (d) 152. (b) 153. (a) 154. (a) 155. (a) 156. (a) 157. (d) 158. (a)
159. (a) 160. (b) 161. (b) 162. (d) 163. (a) 164. (a) 165. (a) 166. (a) 167. (c) 168. (a)
169. (a) 170. (a) 171. (c) 172. (a) 173. (a) 174. (a) 175. (b) 176. (b) 177. (d) 178. (a)
179. (a) 180. (b) 181. (a) 182. (b) 183. (a) 184. (b) 185. (c) 186. (c) 187. (a) 188. (a)
189. (a) 190. (b) 191. (a) 192. (a) 193. (b) 194. (a) 195. (a) 196. (a) 197. (b) 198. (a)
199. (a) 200. (a) 201. (b) 202. (b) 203. (c) 204. (c) 205. (a) 206. (a) 207. (a) 208. (c)
209. (d) 210. (b) 211. (d) 212. (d) 213. (a) 214. (c) 215. (d) 216. (b) 217. (b) 218. (a)
219. (c) 220. (d) 221. (a) 222. (d) 223. (b) 224. (a) 225. (c) 226. (c) 227. (b) 228. (a)
229. (a) 230. (d)
More Questions on New Education Policy
A S S E S S YO U R L E A R N I N G

231. (a) 232. (d) 233. (d) 234. (d) 235. (c) 236. (c) 237. (d) 238. (b) 239. (b) 240. (d)
241. (a) 242. (c) 243. (c) 244. (b) 245. (d) 246. (a) 247. (c) 248. (d) 249. (c) 250. (d)

M10_MADAN 07_65901_C10.indd 64 23/12/22 8:05 PM


MOCK TEST

1. Consider the following statements in the context of Select your answer from the options given below:
curriculum? (1) (a)–(i); (b)–(iii); (c)–(ii); (d)–(iv)
Assertion (A): Curriculum is a complete plan for (2) (a)–(ii); (b)–(iv); (c)–(i); (d)–(iii)
implementation of educational aims. (3) (a)–(iii); (b)–(i); (c)–(ii); (d)–(iv)
Reason (R): Curriculum is the sum total of curricu- (4) (a)–(iii); (b)–(ii); (c)–(iv); (d)–(i)
lum core and syllabus. 5. In which of Collaborative Learning Approaches,
Codes: skills such as generation of ideas and creativity are
(a) Both (A) and (R) are correct and (R) is the correct developed?
explanation of (A). (a) Brainstorming
(b) Both (A) and (R) are correct, but (R) is not the (b) Task group
correct explanation of (A). (c) Inquiry group
(c) (A) is correct and (R) is false. (d) Tutorial group
(d) (A) is false but (R) is correct. 6. The researcher manipulates two or more independent
2. Which of the following learning objectives are true in variables (factors) simultaneously to observe their
the context of constructivist perspective? effects on the dependent variable. This design allows
(a) A concept has to be an integral whole and for the testing of two or more hypotheses in a single
thus, cannot be divided into different levels of project. This is called as:
learning. (a) Randomized Block Design
(b) Learning is a divergent process that occurs (b) Factorial Design
through various exposures. (c) Non-Randomized Design
(c) Learning objectives are contextual in nature. (d) Quasi-experimental Research
(d) Learning is a multidimensional process. 7. Consider the following statements:
Codes: 1. This step is probably the most critical part of the
(a) Only 1, 2, and 3 planning process.
(b) Only 2, 3, and 4 2. These stated research questions provide the basis
(c) Only 1, 3, and 4 for design and data analysis.
(d) All of the above. 3. This step guides the researcher’s decision as to
3. The ultimate objective of any teaching-learning process go for an experimental design or some other
is: orientation.
(a) To develop a flexible curriculum setting system. These three statements reflect which of the
(b) To help learners learn to become autonomous following?
learners. (a) Literature survey
(c) To equip learners with degrees and diplomas. (b) Research design
(d) To help learners find any job in the organizations. (c) Hypotheses
4. Below given are two columns. Column–A lists meth- (d) Deciding about research topic.
ods of teaching and Column–B lists the focus of a 8. During research process, which of the following
method. type/s of error/s is/are associated with biasedness?
(a) Sampling Error.
Column–A Column–B (b) Non-sampling Error.
MOCK TEST
(a) Lecture method (i) Dynamic, proactive (c) Both (a) and (b).
method of teaching (d) Cannot be ascertained at all.
(b) Demonstration (ii) Experience-based 9. The important pre-requisites of a research in sciences,
method learning social sciences, and humanities are:
(c) Heuristic method (iii) Imparting large (a) Laboratory skills, records, supervisor, and topic.
amount of knowledge (b) Supervisor, topic, critical analysis, and patience.
(d) Differentiated (iv) The student has to (c) Archives, supervisor, topic, and flexibility in
instruction solve own problem by thinking.
unaided efforts (d) Topic, supervisor, good temperament, and pre-
conceived notions.

Z01_MADAN 07_65901_Mock Test.indd 1 23/12/22 4:38 PM


M.2 MOCK TEST

10. The final result of a research study will be more accu- some reality hosts will, one day, make the great leap
rate if the sample drawn is: into nationwide politics, perhaps even running for
(a) Taken randomly Presidency.
(b) Fixed by quota 11. According to the passage which of the following state-
(c) Representative of population ments is/are false?
(d) Purposive (a) The party chief of Yucheng participated in one of
Read the following passage carefully and answer the shows with a slipped disc.
the following questions: (b) The traditional occupation of the people of Shanxi
China does not have free elections. It has reality tel- province is coal mining.
evision instead. The latest such show even has the (c) Presidential elections in China are held through
flavour of a political contest: the competitors are all reality television.
high-ranking officials. It has been a big hit. Codes:
Since May the programme ‘Sights of Shanxi’ has (a) Only (A)and( B)
been airing live every Friday on a channel in the north- (b) Only (B) and (C)
ern province of that time. In the show, local cities bid (c) Only (A) and (C)
to play host to a tourism development conference. (d) All (A), (B), and(C)
The contestants have to tell four judges why there 12. According to the author, what is/are the reason(s) for
city is such a great place, in front of a studio audi- launching such a TV show on Shanxi?
ence of 100 people and a panel of experts. The judges (A) The government intends to ensure that its officials
grill the contestants who advance or fall according to are on their toes all the time.
votes cast by the audience in the studio, the judges, (B) The government intends to curb corruption in
and internet users. Shanxi province.
What makes this special is that for the first time (C) The government intends to develop the region’s
local bigwigs are not just speaking in public, which struggling economy.
is rare enough, but competing against one another Codes:
and being judged by ordinary folk, which is unheard (a) Only A and B
of. One tourism official from Shanxi told a newspa- (b) Only B and C
per that ‘in the past all we had to do was hand in a (c) Only A and C
report, but because it is on TV, the process now had (d) A, B, and C
to be taken more ­seriously.’ 13. According to the author, how have the bigwigs of
Top officials have to be involved. Of the eleven con- Shanxi reacted to this TV show?
testants, three are municipal communist party bosses, (a) They have refused to appear on the show.
five are city mayors, and three are vice mayors. (b) They have taken this challenge head-on.
For most of them, it has been their first experience (c) They have staged protests, as such a show gives a
to live cameras. They have taken to it like natural. The lot of power to the masses.
mayor of Yangquan city learnt a bit of English to spice (d) They have punished the judges by grilling them
up his bid. ‘Seeing is believing’, he said. ‘The open and on Live cameras.
inclusive Yangquan people welcome you to come.’ 14. Which of the following is a suitable title for the
The party chief of Yucheng suffered a slipped disc, passage?
but soldiered on, rehearsing her speech flat on her (a) Sights of Shanghai.
back. The Deputy Mayor of Linfen handed out virtual (b) The communist style of functioning.
reality glasses to the judges as part of his pitch and it (c) Tourism in Shanxi.
worked, he won his round. (d) A Development strategy for Shanxi.
Viewers love seeing judges take officials to task, 15. Which of the following is the central idea of passage?
‘you would do better to tell us just one or two things (a) The style of Chinese Governance has changed
instead of so many that we forget them’ said one over the past few years.
MOCK TEST

judge. ‘What did you mean by your slogan?’ asked (b) Use of innovative governance to promote and
another. ‘I did not get it.’ develop tourism in the Shanxi province.
(c) The Shanxi province is currently in a very bad
So far 8.3 million people have voted online. The
shape economically.
government itself is partly responsible for the show’s
(d) ‘Sights of Shanxi’ is the first reality show on televi-
success and has good reason to be encouraging the
sion in China.
people. Shanxi’s economy has been struggling;
tourism is its favoured way of diversifying away 16. Which of the following terms is closely
from its traditional coal mining business. In other related to feedback?
words, appealing to an audience actually helps its (a) Brainstorming (b) Heuristics
broader aims. Perhaps, the idea will catch on and (c) Cybernetics (d) None of the above.

Z01_MADAN 07_65901_Mock Test.indd 2 23/12/22 4:38 PM


MOCK TEST M.3

17. Which of the following can be termed as the ‘context’ (c) 9.09 per cent
of communication? (d) No decrease or increase in percentage terms.
(a) An interference with message reception. 26. Statements:
(b) Effective communication. I. Some mangoes are not red.
(c) Verbal and non-verbal responses to messages. II. All red are raw.
(d) A physical and psychological environment for III. Some raw are mangoes.
conversation. Conclusions:
18. A person is more likely to use eye contact while: I. Some mangoes are not raw.
(a) Listening II. Some red are not mangoes.
(b) Uninterested in communication. III. All raw are red.
(c) Speaking (a) Only Conclusion I follows
(d) Interpreting (b) Only Conclusions II and III follow
19. Which of the following can help the most in enhanc- (c) Only Conclusions I and III follow
ing the effectiveness of active listening? (d) No conclusion follows
(a) Developing apathy with the sender. 27. Amongst the following propositions, two are related
(b) Developing a system to minimize the noise in the in such a way that they cannot both be true, but
area. can be false. Select the code that states those two
(c) Paying attention to the body language of the propositions.
sender. Propositions:
(d) Developing empathy with the communicator. (a) Every student is attentive.
20. The main assumption of ‘primacy effect’ in the process (b) Some students are attentive.
of communication is: (c) Students are never attentive.
(a) The most important piece of information is always (d) Some students are not attentive.
that comes first. Codes:
(b) The most important piece of information comes in (a) (i) and (ii)
the last. (b) (i) and (iii)
(c) The degree of importance depends upon the (c) (ii) and (iii)
situation. (d) (iii) and (iv)
(d) All pieces of information carry the same weight. 28. Which of the following statements are correct?
21. Complete the series: (a) When the form of an argument is correct, it is
139 135 128 116 97? called a valid argument.
(a) 63 (b) 66 (b) While evaluating an argument, one must consider
(c) 69 (d) 80 both the form and content.
22. A sum of money is to be divided amongst P, Q, and R (c) A sound argument is an argument where the form
in the ratio 2:3:7. If the total share of P and Q together is valid and the content part is also true.
is ` 1500 less than that of R, what is the share of P? (d) Good inductive arguments are said to be induc-
(a) ` 1500 (b) ` 1750 tively strong.
(c) ` 2000 (d) ` 2500 (a) 1, 2, and 3 (b) 2, 3 and 4,
(c) 1, 3, and 4 (d) All of the above.
23. In a certain code language, ‘TOASTER’ is written as 29. Consider the following statement:
‘VQCRRCP’ and ‘RODENTS’ is written as ‘TQFDLRQ’. ‘I made low grades on my first tests in the subjects of
How will ‘PHANTOM’ be written in that code Math and Hindi. I must really be dumb.’
language?
(a) RJCMVQO (b) NFYORMK Which kind of fallacy is being committed in this
(c) RJCMRMK (d) NFYOVQO statement?
(a) Fallacy of converse accident.
24. P and Q are sisters, and R and S are brothers. P’s (b) Fallacy of ad populum.
daughter is R’s sister. What is Q’s relation to S? (c) The fallacy of false cause.
MOCK TEST
(a) Mother (d) The fallacy of ad ignorantiam.
(b) Grandmother
(c) Sister 30. Which of the following system of logic that forms one
(d) Aunt of the six principal schools of Hindu philosophy, and
in a wider sense it denotes Buddhist and Jaina logic as
25. When water is frozen to ice, its volume increases by well?
10 per cent. What is the decrease in percentage terms (a) Nyaya school
when it melts to water? (b) Mimamsa school
(a) 10.11 per cent (c) Yoga school
(b) 4 per cent (d) Sankhya school

Z01_MADAN 07_65901_Mock Test.indd 3 23/12/22 4:38 PM


M.4 MOCK TEST

Direction for Questions 31–35: Study the following table 39. A small picture that represents either a program or a
to answer the questions. shortcut on a computer screen is:
(a) A pointer (b) A logo
Item of Interest (c) A graphics (d) An icon
Expenditure/ Fuel and on
Year Salary Transport Bonus Loans Taxes 40. ADSL is a data communication technology that ena-
bles faster data transmission over telephone lines.
1998 288 98 3.00 23.4 83 ADSL stands for:
1999 342 112 2.52 32.5 108 (a) Accelerated Digital Subscriber Line.
(b) Accelerated Digitized Subscriber Line.
2000 324 101 3.84 41.6 74
(c) Analogous Digital Subscriber Line.
2001 336 133 3.38 36.4 88 (d) Asymmetric Digital Subscriber Line.
2002 420 142 3.96 49.4 98 41. The two most common types of biofuels presently in
use are:
31. The ratio between the total expenditure on taxes for all (a) Ethanol and biodiesel.
the years and the total expenditure on Fuel and Trans- (b) Ethanol and biopetrol.
port for all the years, respectively, is approximately: (c) Methanol and biodiesel.
(a) 4:7 (b) 10:13 (d) Methanol and biopetrol.
(c) 15:18 (d) 5:8 42. Which of the following is not a raw material for the
32. The total expenditure of the Company over these production of biogas?
items during the year 2000 is: (a) Water hyacinth (b) Coal
(a) ` 544.44 Lakhs (b) ` 501.11 Lakhs (c) Algae (d) Grass
(c) ` 446.46 Lakhs (d) ` 478.87 Lakhs 43. Which of the following sources primarily causes Mer-
33. What is the average amount of interest per year which cury pollution?
the Company had to pay during this period? (a) CFL lamps (b) Aviation sector
(a) ` 32.43 lakhs (b) ` 33.72 lakhs (c) Polymers (d) Diesel engines
(c) ` 34.18 lakhs (d) ` 36.66 lakhs 44. The meaning of ‘Carbon Footprint’ is described by the
34. Total expenditure on all these items in 1998 was amount of:
approximately what per cent of the total expenditure (a) Carbon dioxide released into the atmosphere as a
in 2002? result of the activities of a particular individual,
(a) 62 per cent (b) 66 per cent organization, or community.
(c) 69 per cent (d) 71 per cent (b) Greenhouse gases emitted by industries contrib-
uting to global warming.
35. The total amount of bonus paid by the Company dur-
(c) Carbon emissions released by the burning of jet
ing the given period is approximately what per cent of
fuel.
the total amount of salary paid during this period?
(d) Increase in the carbon content of the atmosphere
(a) 0.1 per cent (b) 0.5 per cent
due to the felling of trees.
(c) 1 per cent (d) 1.25 per cent
45. Union Health Ministry launched National Strategic
36. A chat program that allows people to communicate
Plan for Malaria Elimination under which malaria
over the Internet in real time is called as:
will be ultimately eliminated from the country by the
(a) Distant Messaging
year:
(b) Instant Messaging
(a) 2030 (b) 2022
(c) Chatting room
(c) 2032 (d) 2025
(d) All of the above.
46. ‘Good Governance Index’ was released by the Minis-
37. Which of the following organizations have signed a
try of Personnel, Public Grievances & Pensions, on the
pact in February 2018 to integrate a course on ‘Digital
occasion of Good Governance Day on 25 December
Citizenship and Safety’ in Information and Technol-
2019. Which of the following states have achieved the
MOCK TEST

ogy curriculum?
first rank?
(a) NCERT and Google.
(a) Punjab (b) Kerala
(b) UGC and Google.
(c) Tamil Nadu (d) Goa
(c) NCERT and Microsoft.
(d) UGC and Microsoft. 47. Which of the following statements are true about
RUSA?
38. Which of the following dignitaries is the author of
(a) RUSA is a Centrally Sponsored Scheme launched
title ‘Exam Warriors’?
in 2013.
(a) Narender Modi (b) Ram Nath Kovind
(b) The aim of RUSA was to increase enrolment in
(c) Pranab Mukherji (d) Shashi Tharoor
higher education by 30 per cent.

Z01_MADAN 07_65901_Mock Test.indd 4 23/12/22 4:38 PM


MOCK TEST M.5

(c) RUSA primarily provides strategic funding to eli- (b) NIRF – Ranking of higher educational institutions
gible State higher educational institutions. across the world.
(d) All of the above. (c) GIAN – To attract the best foreign academics to
48. The first university in India to start correspondence Indian Universities of Excellence.
course was: (d) SWAYAM – Web portal to make MOOCs on all
(a) Andhra Pradesh Open University kinds of subjects.
(b) IGNOU 50. Which of the following is not provided in the
(c) Delhi University constitution?
(d) None of the above. (a) Election Commission
49. Which of the following statements are not matched (b) Finance Commission
correctly in the case of higher education? (c) NITI Ayog
(a) IMPRINT INDIA – Pan IIT and IISc joint initiative (d) University Grant Commission
for research to solve major engineering and tech-
nology challenges in select domains in India.

MOCK TEST

Answer Keys
1. (a)  2. (d)   3. (b)   4. (a) 5. (a)  6. (b)  7. (c)  8. (c) 9. (b) 10. (c)
11. (a) 12. (c) 13. (b) 14. (d) 15. (b) 16. (c) 17. (d) 18. (a) 19. (d) 20. (a)
21. (c) 22. (a) 23. (c) 24. (d) 25. (c) 26. (d) 27. (b)  28. (d) 29. (a) 30. (a)
31. (b) 32. (a) 33. (d) 34. (c) 35. (c) 36. (b) 37. (a) 38. (a) 39. (d) 40. (d)
41. (a) 42. (b) 43. (a) 44. (a) 45. (a) 46. (c) 47. (d) 48. (c) 49. (b) 50. (c)

Z01_MADAN 07_65901_Mock Test.indd 5 23/12/22 4:38 PM


M.6 MOCK TEST

Solution
5. (a): Alex Osborn developed the original approach 24. (d):
of brainstorming. He published it in his 1953 book, P and Q are sisters.
‘Applied Imagination.’ Since then, researchers have R and S are sons of P.
made many improvements to his original technique. Therefore, Q is aunt of S.
Brainstorming combines a relaxed, informal approach 25. (c): Required percentage = (10 /110) × 100 = 100/11
to problem solving with lateral thinking. It encour- = 9.09 per cent
ages people to come up with thoughts and ideas that 26. (d):
can, at first, seem a bit crazy. Some of these ideas can First statement is particular negative (O-Type)
be crafted into original, creative solutions to a prob- Second Statement is Universal Affirmative (A –Type)
lem, while others can spark even more ideas. Third Statement is Particular Affirmative (I-Type)
8. (c): Biasedness is associated with sampling and non- Now,
sampling errors. All red are raw
9. (b): These skills are generalized ones.
10. (c): The final results depend upon the analysis of data Some raw are mangoes
that depends upon the quality of sampling. For accu- A + I = No conclusion
racy of results, samples should be representative of So (d) is the right answer.
population. 27. (b): We need to take help of ‘Square of opposition’.
16. (c): Cybernetics is closely related to the concept of As per definition in question statement, we need to
automatic control and also with physiology, particu- look at ‘contrary statements’, which should be univer-
larly the physiology of the nervous system. A ‘control- sal positive (A) and universal negative (E). Thus, (b) is
ler’ might be the human brain, which receives signals the right answer.
from a ‘monitor’ (the eyes) regarding the distance 31. (b): R  equired Ratio = (83 + 108 + 74 + 88 + 98)/(98
between a reaching hand and an object to be picked + 112 + 101 + 133 + 142)
up. The information sent by the monitor to the con- = 451/586 ≈ 1/1.3 = 10/13
troller is called feedback, and on the basis of this feed- 33. (d): Average amount of interest paid by the company
back the controller might issue instructions to bring during the given period
the observed behaviour (the reaching of the hand) = ` [(23.4 + 32.5 + 41.6 + 36.4 + 49.4)/5] lakhs
closer to the desired behaviour (the picking up of = ` (183.3/5) lakhs = ` 36.66 lakhs.
the object). It is used in computer sciences and other 34. (c): Required percentage
areas as well.
= [(288 + 98 + 3.00 + 23.4 + 83)/(420 + 142 +
21. (c): 139 – 4 = 135 3.96 + 49.4 + 98)] × 100 per cent
135 – 7 (=4 + 3) = 128 = (495.4/713.36 × 100) per cent ≈ 69.45 per cent
128 – 12 (=7 + 5) = 116
116 – 19 (=12 +7) = 97 35. (c): Required percentage
97 – 28 (=19+9) = 69 = [(3.00 + 2.52 + 3.84 + 3.68 + 3.96)/(288 + 342
22. (a): + 324 + 336 + 420)] × 100 per cent
Assume that desired ratio is 2x, 3x, and 7x. = (17/1710 × 100) per cent = 1 per cent
According to question: 40. (d): ADSL is a type of Digital Subscriber Line technol-
7x – (2x + 3x) = ` 1500 ogy that enables faster data transmission over copper
Thus, x = ` 750 and share of P = 2 × 750 = ` 1500 telephone lines rather than a conventional voiceband
23. (c): Modem can provide.
41. (a): Ethanol is blended with petrol that is also called
T O A S T E R blended petrol. Ethanol and biodiesel represent the
+2 +2 +2 –1 –2 –2 –2 first generation of biofuel technology.
MOCK TEST

V Q C R R C P 43 (a): CFLs, like all fluorescent lamps, contain mercury


as a vapor inside the glass. Mercury (Hg) is a heavy
R O D E N T S metal that exists in several forms. Human contact
+2 +2 +2 –1 –2 –2 –2 with mercury, in any form, can produce toxic effects
if the dose is high enough.
T Q F D L R Q
50. (c): NITI Ayog was set up on 1 January 2015. It
Therefore,
replaced Planning Commission that was an extra con-
stitutional body that was not defined in the Constitu-
P H A N T O M tion of India.
+2 +2 +2 –1 –2 –2 –2
R J C M R M K

Z01_MADAN 07_65901_Mock Test.indd 6 23/12/22 4:38 PM


Previous Years’ Papers

Z02_MADAN 07_65901_Z02.indd 1 23/12/22 4:39 PM


This page is intentionally left blank

Z02_MADAN 07_65901_Z02.indd 2 23/12/22 4:39 PM


NTA-UGC NET/JRF Paper 1 December 2019

INSTRUCTIONS To NTA-NET ASPIRANTS


Time Allowed: 1 hour  50 × 2 = 100 Marks

1. This paper consists of Fifty (50) objective type questions of Two (2) marks each.
2. All questions are compulsory.
3. Each item has four alternative responses marked (a), (b), (c), and (d). You have to darken the circle as indicated below
on the correct response against each item.
Example: a b c d where (c) is the correct response.
4. Your responses to the items are to be indicated in the OMR Sheet given at the end of the book.
5. Read instructions provided with each question carefully.
6. There are no negative marks for incorrect answers.

Data Interpretation 3. In which of the following years was the total produc-
The following table presents the data on the number of tion of all the three types of vehicles 60,000?
vehicles of three different types (A, B, and C) produced (a) 2013
by a company during seven different years, that is, from (b) 2014
2012 to 2018. Based on the data in the table, answer the (c) 2015
questions from 1 to 5. (d) 2016
Year-wise Production of Three Types of Vehicles 4. The number of A type vehicles produced in the year
2014 was what per cent of the number of C type vehi-
Vehicle cles produced in the year 2016?
(a) ~33.33 per cent (b) ~66.67 per cent
A Type B Type C Type (c) 50 per cent (d) 15 per cent
Year
5. What has been the percentage increase of B Type vehi-
cles in the year 2017 over 2016 ?
2012 5000 20,000 15,000 (a) 10 per cent (b) 5 per cent
2013 15,000 10,000 22,500 (c) 20 per cent (d) 25 per cent
2014 10,000 15,000 30,000 6. Choose the person who has control over self, mode of
communication, and method of delivery in classroom
2015 17,500 15,000 23,000 from the following:
2016 25,000 20,000 15,000 (a) Student (b) Teacher
(c) Audience (d) Visitor
2017 13,000 25,000 20,000
7. Given below are two statements—one is labelled as
2018 30,000 35,000 25,000 Assertion (A) and the other is labelled as Reason (R).
1. In how many years was the production of A type vehi- Assertion (A): Skill development is a key feature in
every individual’s life.
cles less than its average production over the given
years? Reason (R): An individual, who wants to get
employed, needs to possess good education only.
(a) 2 (b) 4
(c) 3 (d) 1 In the context of the above statements, choose the
correct option.
2. What was the average number of B type vehicles pro- (a) Both (A) and (R) are true, and (R) is the ­correct
duced by the company over the years? explanation of (A).
(a) 20,000 (b) Both (A) and (R) are true, but (R) is not the cor-
(b) 25,000 rect explanation of (A).
(c) 15,000 (c) (A) is true but (R) is false.
(d) 30,000 (d) (A) is false but (R) is true.

Z02_MADAN 07_65901_Z02.indd 3 23/12/22 4:39 PM


Z.4 NTA-UGC NET/JRF Paper 1 December 2019

8. Which amongst the following is not the outcome of 14. Which amongst the following factors does not
grounded theory research? ­contribute to assessment bias?
(a) Concepts (b) Narratives (a) When language of the test and the tester is
(c) Categories (d) Hypotheses ­different from the languages of the students.
9. Given below are two statements—one is labelled as (b) Answers that support middle class values.
Assertion (A) and the other is labelled as Reason (R). (c) Its assessment procedures are flexible and diverse
Assertion (A): Government of India established to make disadvantaged students comfortable.
higher educational statutory bodies to ensure the (d) Objective test for assessing abstract reasoning of
quality of education. the student.
Reason (R): There are some educational institutions 15. Choose two factors that do NOT affect the ­academic
in India that do not provide quality education. performance of students adversely.
In the context of the above statements, choose the   (i) Low self-efficacy belief.
correct option. (ii) Belief in God.
(a) Both (A) and (R) are true, and (R) is the correct (iii)Indifferent attitude towards politics.
explanation of (A). (iv) Teacher’s low expectation from students.
(b) Both (A) and (R) are true, but (R) is not the cor- Choose the correct option.
rect explanation of (A). (a) (i) and (ii) only (b) (ii) and (iii) only
(c) (A) is true but (R) is false. (c) (iii) and (iv) only (d) (ii) and (iv) only
(d) (A) is false but (R) is true. 16. By selling 70 pencils for `90, a person makes a loss
10. Match List I with List II. of 25 per cent. How many pencils should be sold for
`234 to gain a profit of 30 per cent?
List I List II (a) 95 (b) 100
(Digital Initative) (Purpose) (c) 105 (d) 110
(a) UMANG   (i) G2B services 17. Which of the following sequence represents the cor-
rect order of soil particles arranged in terms of parti-
(b) BHIM (ii) Learning platform cle size from the smallest to the largest?
(c) eBiz (iii) Unified Payment Interface (a) Gravel, Silt, Clay, and Sand.
(b) Silt, Clay, Gravel, and Sand.
(d) SWAYAM (iv) Single point of access to (c) Clay, Silt, Sand, and Gravel.
all government services. (d) Silt, Clay, Sand, and Gravel.
18. Given below is a question followed by three
Choose the correct option.
­statements. Identify the statements that are necessary
(a) (a)-(ii), (b)-(i), (c)-(iii), (d)-(iv)
to answer the question.
(b) (a)-(iii), (b)-(ii), (c)-(iv), (d)-(i)
(c) (a)-(iv), (b)-(iii), (c)-(i), (d)-(ii) Question: What is the percentage profit gained by a
vendor when he sells an article?
(d) (a)-(i), (b)-(iv), (c)-(ii), (d)-(iii)
Statements:
11. Identify the correct statements from the following.   (i) He gives a 10 per cent discount on the marked
  (i) Sampling-related error is subsumed under the price.
category non-sampling error. (ii) He would have earned 25 per cent profit if he did
(ii) Sampling error does not occur in multistage clus- not give any discount.
ter sampling. (iii)The cost price of the article is `8000.
(iii)Inaccurate sampling frame and non-response are Choose the correct option.
examples of sampling-related error. (a) (i) and (ii) only
(iv) There is an inverse relation between heterogene- (b) (ii) and (iii) only
ity of population and sampling error. (c) (i) and (iii) only
Choose the correct option. (d) (i), (ii) and (iii)
(a) (i) and (iii) only (b) (i) and (iv) only
(c) (ii) and (iv) only (d) (ii) and (iii) only 19. Devdatta is fat and he does not eat during the day.
Therefore, Devdatta is eating during the night.
12. How many national parks and wildlife sanctuaries
The above example in classical Indian school of logic
were there in India till March 2011? is a case of:
(a) 8 and 421 only. (b) 10 and 75 only. (a) Comparison (b) Implication
(c) 75 and 421 only. (d) 102 and 515 only. (c) Perception (d) Verbal testimony
13. Which amongst the following is NOT a component of
20. In Social Sciences, a central issue is the question of
effective teaching?
whether the social world can be studied using the
(a) Questioning to check for the understanding of
methods of natural sciences. This statement refers to
students.
the issue of:
(b) Providing students with feedback.
(a) Ontology (b) Axiology
(c) Being flexible about how long it takes to learn.
(c) Epistemology (d) Etymology
(d) Reactive mode of student’s behaviour.

Z02_MADAN 07_65901_Z02.indd 4 23/12/22 4:39 PM


NTA-UGC NET/JRF Paper 1 December 2019 Z.5

21. Out of a total of 10 students, the average weight 27. Match List I with List II.
of nine students is 60 kg. If the weight of the 10th
­student is nine kg more than the average weight of all List I List II
the 10 students, then the weight of the 10th ­student (a) Ontology   (i) Ideographic
would be:
(a) 61 kg (b) 69 kg (b) Epistemology (ii) Realism
(c) 70 kg (d) 71 kg
(c) Methodology (iii) Survey
22. Which of the following sets of letters would complete
the letters series, when they are placed sequentially? (d) Method (iv) Positivism
bb ___ b ___ ab ___ cbba ___ bcbb ___
(a) a c b a c (b) a c b b b Choose the correct option.
(c) c b b b a (d) a b b b c (a) (a)-(ii), (b)-(iv), (c)-(iii), (d)-(i)
(b) (a)-(ii), (b)-(i), (c)-(iii), (d)-(iv)
23. Identify the factors that do not contribute to the effec- (c) (a)-(iv), (b)-(ii), (c)-(i), (d)-(iii)
tiveness of teaching from the options given below. (d) (a)-(ii), (b)-(iv), (c)-(i), (d)-(iii)
  (i) Socioeconomic background of teacher.
(ii) Teacher’s skill in pleasing the students. 28. Consider the following statements with reference to
(iii)Teacher’s subject knowledge. the proposition ‘Some girls are not students’.
(iv) Teacher’s personal contact with students.   (i) Subject term students are distributed.
(ii) Predicate term girls are undistributed.
Choose the correct option.
(iii)Predicate term students are distributed.
(a) (i), (iii), and (iv)
(iv) Subject term girls are undistributed.
(b) (ii), (iii), and (iv)
(c) (i), (ii), and (iii) Choose the correct option.
(d) (i), (ii), and (iv) (a) Only (i) and (iii) are correct.
(b) Only (ii) and (iii) are correct.
24. Of the following options, select the advantage of feed- (c) Only (i) and (iv) are correct.
back in the process of communication. (d) Only (iii) and (iv) are correct.
(a) It is beneficial in understanding of the subject
matter. 29. Match List I with List II.
(b) It diagnoses the defects in receiver. List I List II
(c) It clarifies the communication. (Soil Order) (Description)
(d) It explores the defects in receiver.
25. Given below are two statements—one is labelled as (a) Histosols   (i) Soil order that includes soils
Assertion (A) and the other is labelled as Reason (R). formed on volcanic ash
Assertion (A): A blog is a form of social media. (b) Andisols (ii) Soil order of cold regions
Reason (R): A blog allows anyone to communicate to including soils underlain by
a public audience. permafrost
In the light of the above statements, choose the cor-
rect option. (c) Gelisols (iii) Soil order consisting of soils of
(a) Both (A) and (R) are true, and (R) is the correct dry climates
explanation of (A). (d) Aridisols (iv) Soil order consisting of soils
(b) Both (A) and (R) are true, and (R) is not the cor- with a thick upper layer of
rect explanation of (A). organic matter
(c) (A) is true but (R) is false.
(d) (A) is false but (R) is true. Choose the correct option.
26. Which of the following statements are true about a (a) (a)-(iv), (b)-(i), (c)-(ii), (d)-(iii)
fuel cell? (b) (a)-(ii), (b)-(iv), (c)-(i), (d)-(iii)
  (i) It is a device that uses electrochemical ­reactions (c) (a)-(i), (b)-(ii), (c)-(iii),(d)-(iv)
to produce electric current. (d) (a)-(i), (b)-(ii), (c)-(iv), (d)-(iii)
(ii) It consists of a positive electrode and a n
­ egative 30. From the following, select the alternative features
electrode separated by an electrolyte. which make open universities as non-traditional
(iii)Just like batteries, it needs to be recharged with universities.
an electric current.
  (i) Innovative method of teaching and learning.
Choose the correct option from those given below.
(ii) Innovative method of admission, curriculum, and
(a) (i) and (ii) only
evaluation.
(b) (ii) and (iii) only
(c) (i) and (iii) only (iii)Modern communication techniques.
(d) (i), (ii), and (iii) (iv) Teacher’s friendly approaches.

Z02_MADAN 07_65901_Z02.indd 5 23/12/22 4:39 PM


Z.6 NTA-UGC NET/JRF Paper 1 December 2019

Choose the correct option. 36. Which of the following was not a Millennium Devel-
(a) (ii), (iii), and (iv) only opment Goals (MDG)?
(b) (i), (ii), and (iii) only (a) Eradicate extreme poverty and hunger.
(c) (i), (iii), and (iv) only (b) Improve maternal health.
(d) (i), (ii), and (iv) only (c) Ensure healthy lives and promote well-being for
31. Select the option that lists components for a cyber- all at all ages.
space address (URL) in the correct order from begin- (d) Ensure environmental sustainability.
ning to end. 37. Regarding e-governance, which of the following
(a) Protocol, domain name of the host, and path or file. statements is/are correct?
(b) Path or file, protocol, and domain name of the   (i) Decreasing transparency is an ICT enabled advan-
host. tage of e-governance.
(c) Protocol, path or file, and domain name of the host. (ii) E-governance is related to the implementation of
(d) Domain name of the host, protocol, and path ICT in the government processes and functions.
or file. (iii)Common Service Centres (CSCs) help in provid-
ing and using e-governance related services.
32. Match List I with List II. Choose the correct option.
List I List II (a) (i), (ii), and (iii)
(Historic Places) (Discipline) (b) (ii) and (iii) only
(c) (i) and (iii) only
(a) Taxila   (i) Astronomy (d) (i) and (ii) only
(b) Ujjain (ii) Buddhism 38. Which one of the following is signified by Udāharan.a
of Anumāna (inference) in Indian logic?
(c) Sarnath (iii) Art, Architecture and (a) Statement of reason.
Painting (b) Proposition to be proved.
(d) Ajanta (iv) Medicine (c) Conclusion proved.
(d) Universal proposition along with an instance.
Choose the correct option. 39. Which one of the following propositions is c­ ontrary to
(a) (a)-(i), (b)-(ii), (c)-(iii), (d)-(iv) ‘All poets are dreamers’?
(b) (a)-(ii), (b)-(i), (c)-(iv), (d)-(iii) (a) Some poets are dreamers.
(c) (a)-(iii), (b)-(iv), (c)-(ii), (d)-(i) (b) Some poets are not dreamers.
(d) (a)-(iv), (b)-(i), (c)-(ii), (d)-(iii) (c) No poets are dreamers.
(d) No dreamers are poets.
33. In what ratio must a vendor mix two varieties of sugar
40. Which amongst the following is NOT related to quali-
worth `60 per kg and `78 per kg so that, by selling the
tative research?
mixture at `76.8 per kg, the vendor gains 20 per cent?
(a) Thematic analysis (b) Case study
(a) 7:4 (b) 7:2 (c) Discourse analysis (d) Survey method
(c) 5:3 (d) 5:2
41. As per the classical square of opposition, if ‘A’
34. Arrange the steps of research in the sequence in which ­proposition is given as true, then which one of the fol-
they are presented in the thesis. lowing is correct?
(a) Discussion, review of literature, result, c­ onceptual (a) ‘E’ proposition is false, ‘I’ proposition is true, and
framework, and problem. ‘O’ proposition is false.
(b) Review of literature, problem, conceptual frame- (b) ‘E’ proposition is true, ‘I’ proposition is true, and
work, result, and discussion. ‘O’ proposition is true.
(c) Problem, review of literature, conceptual frame- (c) ‘E’ proposition is false, ‘I’ proposition is false, and
work, result, and discussion. ‘O’ proposition is true.
(d) Problem, review of literature, conceptual frame- (d) ‘E’ proposition is true, ‘I’ proposition is false, and
work, discussion, and result. ‘O’ proposition is false.
35. Which of the following statements is/are correct? 42. Select the period in which India became a centre of
(i) Software is loaded into secondary storage from higher learning.
RAM and then it is executed by the CPU. (a) Gupta period (b) Buddha period
(ii) A search engine is a software system that is (c) Mughal period (d) British period
designed to search for information on the World
Wide Web. 43. Given below are two statements—one is labelled
Choose the correct option. as Assertion (A) and the other is labelled as
(a) Only (i) Reason (R).
(b) Only (ii) Assertion (A): Through online teaching, a large
(c) Both (i) and (ii) number of students can be taught by a very compe-
tent faculty.
(d) Neither (i) nor (ii)

Z02_MADAN 07_65901_Z02.indd 6 23/12/22 4:39 PM


NTA-UGC NET/JRF Paper 1 December 2019 Z.7

Reason (R): Online teaching helps students in devel- understanding, and knowledge that can help all the peo-
oping critical thinking more than the offline teaching ple of the world, including the most disadvantaged mem-
can do. bers of the world population. A comprehensive rejection
In the light of the above statements, choose the cor- of globalization can thus be powerfully counterproduc-
rect option. tive. There is a strong need to separate out the different
(a) Both (A) and (R) are true, and (R) is the correct questions that appear merged in the rhetoric of the anti-
explanation of (A). globalization protests. The globalization of knowledge
(b) Both (A) and (R) are true, but (R) is not the cor- deserves particularly high profile recognition, despite all
rect explanation of (A). the good things that can be rightly said about the impor-
(c) (A) is true but (R) is false. tance of ‘local knowledge’.
(d) (A) is false but (R) is true. Globalization is often seen in both journalistic dis-
44. Match List I with List II. cussions and in remarkably many academic writings, as
a process of Westernization. Indeed, some who take an
List I List II upbeat view of the phenomenon even sees it as a contri-
(Values) (Distinguishing Functions) bution of Western civilisation to the world.
(a) Sacrifice   (i) Working in a stipulated 46. According to the passage, which one of the following
time is not a well-defined concept?
(a) Multiculturalism (b) Identity
(b) Sincerity (ii) Confinement of individual (c) Globalization (d) Local knowledge
mind on action 47. As per the passage, a wholesale rejection of
(c) Self-control (iii) Showing love and affection ­globalization would result in affecting:
  (i) Global businesses
(d) Altruism (iv) Helping without selfish (ii) Movement of ideas.
motives (iii)Local knowledge systems.
Choose the correct option. Choose the correct answer from the options given
below.
(a) (a)-(ii), (b)-(i), (c)-(iv), (d)-(iii)
(a) (i) and (ii) only (b) (i) and (iii) only
(b) (a)-(iv), (b)-(i), (c)-(ii), (d)-(iii)
(c) (iii) and (ii) only (d) (i), (ii), and (iii)
(c) (a)-(iii), (b)-(iv), (c)-(i), (d)-(ii)
(d) (a)-(i), (b)-(iii), (c)-(ii), (d)-(iv) 48. According to the passage, which one of the following
is counterproductive?
45. Which of the following groups has the components of (a) Comprehensive support to globalization.
paralanguage communication? (b) Wholesale rejection of globalization.
(a) Voice, emphasis, and impression. (c) Comprehensive rejection of anti-globalization
(b) Social space, voice, and impression. protests.
(c) Voice, social space, and expression. (d) Recognition of local knowledge systems.
(d) Emphasis, social space, and voice.
49. The attempt of the author in the passage is:
Comprehension: (a) Unconditioned advocacy of globalization.
Read the following passage and answer the questions (b) Unconditioned rejection of globalization.
from 46 to 50. (c) Unconditioned rejection of local knowledge
Even though globalization is one of the most d
­ iscussed systems.
topics in the contemporary world, it is not altogether a (d) Unbiased evaluation of globalization.
well-defined concept. A multitude of global interactions 50. According to the passage, globalization is often per-
are put under the broad heading of globalization, vary- ceived by media and academia as:
ing from the expansion of cultural and scientific influ- (a) Supporting local knowledge systems.
ences across borders to the enlargement of economic (b) Detrimental to local knowledge systems.
and business relations throughout the world. A whole- (c) A process of Westernization.
sale rejection of globalization would not only go against (d) A process of facilitating global business.
global business, it would also cut out movements of ideas,

Answer Keys
1. (b) 2. (a) 3. (d) 4. (b) 5. (d) 6. (b) 7. (c) 8. (b) 9. (a) 10. (c)
11. (a) 12. (d) 13. (d) 14. (c) 15. (b) 16. (c) 17. (c) 18. (a) 19. (b) 20. (c)
21. (c) 22. (c) 23. (d) 24. (c) 25. (a) 26. (a) 27. (d) 28. (d) 29. (a) 30. (b)
31. (a) 32. (d) 33. (b) 34. (c) 35. (b) 36. (c) 37. (b) 38. (d) 39. (c) 40. (d)
41. (a) 42. (a) 43. (c) 44. (b) 45. (a) 46. (c) 47. (a) 48. (b) 49. (d) 50. (c)

Z02_MADAN 07_65901_Z02.indd 7 23/12/22 4:39 PM


Z.8 NTA-UGC NET/JRF Paper 1 December 2019

Solution
1. (b): Average production of A Type over the given year Narrative research is a separate type of qualitative
115500 research. It entails exploring the life of an individual.

= =16500 It is about studying one or more individuals. It pri-
7 marily uses interviews and documents.
In 2012, 2013, 2014, and 2017 production is less than 9. (a): Here, we can take an example of UGC that is a
average. statutory organization established by an Act of Par-
2. (a): Average number of B Type vehicles produced over liament in 1956 for the coordination, determination,
the year and maintenance of standard of university education.
= (20000 + 10000 + 15000 + 15000 They provide funding also.
+ 20000 + 25000 + 35000) / 7 The institutions have not been able to provide quality
education because of lack of resources and supervi-
140000
= = 20000 sion as their sheer number is very high. Government
7 has set up other institutions for their grading.
3. (d): Total production in 2016 10. (c): UMANG (Unified Mobile Application for New-age
= 25000 + 20000 + 15000 = 60000 Governance) is envisaged to make e-governance. It is
4. (b): Percentage of production of A Type over B Type developed by the Ministry of Electronics and Informa-
tion Technology (MeitY) and National e-Governance
10000 Division (NeGD) to drive Mobile Governance in India.

= × 100 = 66.67 per cent
15000 UMANG provides a single platform for all Indian citi-
5. (d): Percentage increase in 2017 zens to access pan-India e-Gov services ranging from
Central to Local Government bodies and other citi-
= (25000 – 20000) / 20000 × 100 = 25 per cent
zen-centric services.
6. (b): Teacher has control over self, mode of communi-
BHIM (BHARAT INTERFACE FOR MONEY) is an app
cation, and method of delivery in classroom. A teacher
that lets you make simple, easy, and quick payment
needs to be organized in classroom. They should have
transactions using unified payment Interface. You
control over mode of communication and method of
can make instant bank to bank payment and pay and
delivery to make students learn the concepts. They
collect money using just mobile number and virtual
should transmit genuine commitment.
payment address. National Payments Corporation of
7. (c): The Assertion (A) is true to settle his or her career in India (NPCI) is an umbrella organization for operat-
any job, profession, or any mode of entrepreneurship. ing retail payments and settlement systems in India
In Reason (R), we need to focus on the word ‘only’. eBiz: eBiz is being implemented by Infosys Technolo-
Education is required for job or entrepreneurship. That gies under the guidance and aegis of Department of
makes it false. Industrial Policy and Promotion, Ministry of Com-
8. (b): Grounded theory is a systematic methodology in merce, and Industry Govt. of India. The focus of eBiz
the social sciences involving the construction of theo- is to improve the business environment in the country
ries through methodical gathering and analysis of data. by reducing unnecessary delays in regulatory pro-
This research methodology uses inductive reasoning. cesses required to start and run business.
Research SWAYAM: MHRD has started a major and new initia-
question tive project called ‘Study Webs of Active Learning for
Young Aspiring Minds’ (SWAYAM). It is an integrated
platform and portal for online courses. This covers
Theoretical all higher education subjects and skill sector courses.
sampling The objective is to ensure that every student in our
country has access to the best quality higher educa-
Theoretical Data tion at the affordable cost.
saturation? collection
11. (a): We do not collect data from every member of pop-
ulation to save time and money. The negative impact is
Coding
sampling errors. They happen in any kind of research.
process In (i), we need to focus on the words ‘related’ and
‘subsumed’. Non-sampling error is ‘catch-all’ term.
(ii) Sampling error do occur in every kind of sampling
Constant
whether single stage or multistage.
comparision
(iii) Inaccurate sampling frame and non-response
should be right and ‘motivational’.
Grounded (iv) Rather, the more homogeneous is the population,
theory
less will be the sampling error.

Z02_MADAN 07_65901_Z02.indd 8 23/12/22 4:39 PM


NTA-UGC NET/JRF Paper 1 December 2019 Z.9

12. (d): For future questions, we should try to understand (ii) Belief in God – It is one’s personal issue.

the differences between ‘National Park’ and ‘Wildlife (iii) We may not be interested in politics, but it is

Sanctuary’. part of democracy. So some interest in politics
is desirable.
National Park Wildlife Sanctuary
(iv) Teacher’s low expectation from students is not

A national park is a Wildlife sanctuary is good for the confidence of a teacher himself or
more restricted area as not a very restricted herself.
compared to wildlife area. It is open to
sanctuary. Random access general public. 16. (c): Let SP of 70 pencils = ` 90
to people is not allowed. According to Question:
Tickets or permissions are Tickets or permissions 90 = CP × (100 – 25) / 100
required to enter into a are not required to visit CP = ` 120
national park. a wildlife sanctuary. In 2nd situation, for a SP of ` 234 and profit of 30 per
A national park has a A wildlife sanctuary cent, the CP is to be found first.
definite boundary. does not have a specific CP = 234 × 100 / 130 = ` 180
boundary. If number of pencils for ` 120 is 70, then for CP of
General public cannot General public cannot ` 180, the number of pencils should be one and half
use the park for collecting use the park for times, that is, 70 × 1.5 = 105
any materials like fruits, collecting any materials 17. (c): The size of soil particles gives texture to the soil.
firewood. like fruits, firewood. It also determines the amount of air and moisture that
Example: Jim Corbett Example: Krishna exist in the soil. They can be arranged in the following
national park (Gujarat) wildlife sanctuary manner, in terms of size.
(Andhra Pradesh) 1. Clay: Clay has the smallest particles. A particle
measures less than 0.002 mm. It feels sticky when
Source: meritnation.com wet. It absorbs water well and it is nutrient rich.
13. (d): Reactive mode of student’s behaviour is not a 2. Silt: Silt particles measures 0.06 to 0.002 mm.
component of effective teaching as reaction does take Silt’s particles are larger than clay particles, but
place during extreme situations. smaller than sand particles.
14. (c): Assessment bias unfairly penalizes students on 3. Sand: Sand particles measures 2.0 mm to
the basis of race, ethnicity, socioeconomic status, reli- 0.06 mm. Sand is the largest type of soil particle.
gion, etc. It allows quick drainage and plenty of airflow.
Here, we need to work on one assumption that every 4. Gravel: Gravel is a loose aggregation of rock frag-
learner is unique and some flexible mechanism may ments. Gravel is classified by particle size range
help. and includes size classes from granule to boulder
15. (b): size fragments.
(i) The concept of self-efficacy was given by Albert 18. (a): Let Marked price = ` 100
Bandura. Self-efficacy reflects confidence in the
ability to exert control over one’s own motivation, Discount = 10 per cent
behaviour, and social environment. Self-efficacy Then SP = ` 90
means looking into one’s potential. If no discount is offered then profit is 25 per cent. It
means SP is ` 100.
Experience
Thus CP = 100 × 100 / (100 + 25) = ` 80
We have to calculate only ‘percentage’ profit, so we do
not need the third statement.
Vicarious
experience 19. (b): Let us look at the complete example.
Self-efficacy
Behaviour and 1. Fat Devadatta does not eat during day time.
Performance 2. But, he does not lose weight.
Social
persuasion
These two contradictory statements are reconciled
with arthāpatti pramāa that he must be eating during
night. Arthāpatti pramāa is a Sanskrit term meaning
implication. Thus (b) is the right answer.
Physiological
feedback Self-efficacy impacts 20. (c):
behaviour and performance (a) Epistemology is the study of nature and scope of
knowledge and justified belief.

Z02_MADAN 07_65901_Z02.indd 9 23/12/22 4:39 PM


Z.10 NTA-UGC NET/JRF Paper 1 December 2019

(b) Ontology is part of metaphysics, a branch of phi- Feedback


losophy that looks at the very nature of things,
their being, cause, or identity.
(c) Axiology is the branch of philosophy that con- Sender Encoding Message Decoding Receiver
siders the nature of value and what kinds of
things have value. Axios means value.
(d) Etymology: The study of the origin of words and Noise
the way in which their meanings have changed
throughout history.
25. (a): Blogs as social media include vlogs, wall post-
21. (c): Let the avg. weight of 10 students = x ings, emails, instant messaging, music-sharing, crowd
Then 10x = [(9 × 60 )] + (x + 9) sourcing, voice over IP, etc.
10x = 540 + x + 9 Blogs is an individualized approach to communicate
9x = 549 to public audience.
x = 61 So both ‘A’ and ‘R’ are true and R is the correct expla-
nation of (A).
Weight of the 10th student = x +9
26. (a): A fuel cell is an electrochemical cell that con-
= 61 + 9 = 70 kg. verts the chemical energy of a fuel (often hydrogen)
22. (c): The series and an oxidizing agent (often oxygen) into electric-
bbc| bba |bbc|bba|bbc|bba ity through a pair of redox reactions. Fuel cells can
The pattern bbc, bba is repeated. produce electricity continuously for as long as fuel
and oxygen are supplied. Every fuel cell has two elec-
23. (d): We need to focus on objective criteria as teaching trodes called, the anode and cathode.
is both art and science. Teachers are also trained for
the purpose. 27. (d): Ontological realism is a term best applied to theo-
ries that are realist regarding what there is.
Thus, subject knowledge matters.
Conceptual pragmatism is related to a philosopher
24. (c): Feedback is essential in communication so as to and logician Clarence Irving Lewis. The ‘epistemol-
know whether the recipient has understood the mes- ogy’ of conceptual ‘pragmatism’ was first formulated
sage in the same terms as intended by the sender and in a book ‘Mind and the World Order: Outline of a
whether he agrees to that message or not. Theory of Knowledge’ in 1929.
Feedback is essential for ‘effective communication’. An idiographic method focusses on individual cases
or events.

28. (d): The review of ‘categorical propositions’ can help.

Type Form Quantity Quality Distribution


Subject Predicate
A All S is P Universal Affirmative Distributed Undistributed
E No S is P Universal Negative Distributed Distributed
I Some S is P Particular Affirmative Undistributed Undistributed
O Some S is not P Particular Negative Undistributed Distributed

We need to look at statement – ‘Some girls are not stu- 31. (a):
dents’. This statement goes with Type ‘O’. 1. Protocol: Protocol is set of rules, with TCP/IP being
Here, the ‘girls’ is a subject as we are discussing her as the most important one. The protocol declares how
a person or thing, she does something. your web browser should communicate with a
Predicate means which tells something about the web serve when sending or fetching a web page or
subject. document.
29. (a): Here, we need to recall some definitions of ‘soil’. 2. Domain name: A domain name is a unique refer-
ence that identifies a web site on the internet, for
30. (b): Here, non-traditional education means ‘Open and example doepud.co.uk
Distance Learning’ (ODL). It always focus on ‘innova-
tive methods’ to reach out to prospective learners.

Z02_MADAN 07_65901_Z02.indd 10 23/12/22 4:39 PM


NTA-UGC NET/JRF Paper 1 December 2019 Z.11

3. Path: The path typically refers to a file or directory 37. (b): (i) Increasing transparency is one of the main
on the web server, for example Directory file. Php. advantages of e-governance. We usually talk about
‘Transparent Government’. RTI, good governance are
Domain name also parts of it.
38. (d): Anumana means the method by which knowl-
Path edge is derived from other knowledge. It is a univer-
sal proposition. Udāharan means instance as given in
https://deoput.co.uk/blog/anatomy-of-a-url option (d).
39. (c):
Every S is P No S is P
Protocol A Contraries E

32. (d)
33. (b): Sale price of 1 kg of the mixture = ` 76.8
Gain = 20 per cent Sub-alterns Contradictories Sub-alterns
100
C.P. of 1 kg of the mixture = × ` 76.8
100 + 20

= `64 I Sub-contraries O
Applying the rule of allegation
Some S is P Some S is not P
Cost of 1 kg sugar of 1st variety Cost of 1 kg sugar of 2nd

variety ‘All poets are dreamers’ is ‘A-Type’.
60 78
‘No poets are dreamers’ is ‘E-Type’.
Contraries are pair of propositions in which both can-
not be true, but both can be false. ‘A’ and ‘E’ are con-
Mean Price trary statements.
64
40. (d): Quantitative research focusses more on the abil-
ity to complete statistical analysis. With quantitative
14 4 studies, each respondent is asked to respond to the
same questions. Surveys and questionnaires are the
Thus, 14: 4 OR 7:2 is required answer.
most common techniques for collecting quantitative
34. (c): This is direct question of sequence of steps. data.
35. (b): (i) The sequence is loading of software from sec- 41. (a): We need to look at ‘square of opposition’ diagram
ondary storage into RAM and finally executed by CPU in question 39 to get the answer.
(registers). 42. (a): During Gupta period, the focus was to patronize
(ii) Give direct definition of search engine. the centers of higher education at Nalanda, Takshila,
36. (C): The Eight millennium development Goals are: Ujjain, Vikramshila and Vallabhi, etc. These centres
attracted scholars from all parts of India and from
1. To eradicate extreme poverty and hunger.
several foreign countries.
2. To achieve universal primary education.
43. (c): Assertion addresses the issues of ‘access to
3. To promote gender equality and empower women. education’.
4. To reduce child mortality. In Reason, it is difficult to assess the difference
5. To improve maternal health. between online and offline for this.
6. To combat HIV/AIDS, malaria, and other diseases. 44. (b): This is a direct question of words and their mean-
7. To ensure environmental sustainability ings. They probably fall under the topic of languages.
8. To develop a global partnership for development. 45. (a): Paralanguage is the area of non-verbal com-
munication that emphasizes on body language and
Thus ‘ensure healthy lives and promote well-being voice nuances as means of expressing thoughts and
for all at all ages’ was not a ‘Millennium Develop- feelings.
ment Goal’.

Z02_MADAN 07_65901_Z02.indd 11 23/12/22 4:39 PM


N TA - U G C N E T / J R F P a p e r 1 2 0 2 0
1. Match List I with List II. E. Evaluation involving judgment using internal or
external standards.
List 1 List II Choose the correct answer from the options given
Research concepts Description below:
(A) Hypothesis    (I) It is statistical test (a) A, B and C only
that does not make (b) B, C and D only
assumptions about (c) C, D and E only
the parameters (d) B, D and E only
4. A teacher uses a question-answer session to ensure
(B) Sample   (II) Device using
desired learning outcomes in his/her classroom. In
which data are
this process, he/she offers the following type of com-
collected in
ment to a few answers given by a student:
research
“Yes, you are right, good’
(C) Research tool (III) A subset drawn This will be considered as an example of
from a larger set to (a) Positive feedback
represent it (b) Negative feedback
(D) Non-parametric (IV) A tentative (c) Confirmatory feedback
tests statement (d) Corrective feedback
indicating 5. Match List I with List II.
relationship
between two or List 1
more than two Behavioural
variables teaching List II
competencies Description
(A) Instructing    (I) Optimizing
Choose the correct answer from the options given ‘on task’ and
below: minimizing ‘off
(a) A -IV, B - III, C - II, D - I task’ activities
(b) A - I, B - II, C - III, D - IV
(c) A - II, E - I, C - IV, D - III (B) Communicating   (II) Watching groups,
(d) A - III, B - IV, C - I, D - II behaviors and
2. In quantitative research paradigm, which of the fol- pace etc. and
lowing sampling methods are given preference? reacting quickly
A. Simple random sampling (C) Managing (III) Using methods
B. Stratified sampling and activities by
C. Quota sampling the teacher to
D. Snowball sampling promote learning
E. Systematic sampling (D) Monitoring (IV) Transmitting
Choose the correct answer from the options given thoughts and
below: ideas verbally and
(a) A, B and C only non-verbally
(b) A, B and E only
(c) B, C and D only Choose the correct answer from the options given
(d) C, D and E only below:
3. In the cognitive domain of teaching-learning objec- (a) A - I, B - II, C - III, D- IV
tives, which will be categorized as a higher type of (b) A - IV, B - III, C - II, D - I
learning outcomes? (c) A - III, B - IV, C - I, D - II
A. Knowledge and comprehension (d) A - II, B - I, C - IV, D - III
B. Analysis of conceptual elements 6. Given below are two statements, one is labelled as
C. Application of acquired knowledge and skills Assertion (A) and the other is labelled as Reason (R).
D. Synthesis involving the creative organization of Assertion (A): Evaluation is said to be formative
ideas when the intention is to identify scope and potential
for improvement of teaching-learning system.

Z03_MADAN 07_65901_2020_Paper 1 LAW.indd 12 23/12/22 5:11 PM


NTA-UGC NET/JRF Paper 1 2020 Z.13

Reason (R): The form of such evaluation is informal


(a) Both A and R are correct and R is the correct
and has to take place during instruction/teaching. explanation of A
In light of the above statements, choose the correct
(b) Both A and R are correct but R is NOT the correct
answer from the options given below explanation of A
(a) Both A and R are true and R is the correct explana- (c) A is correct but R is not correct
tion of A (d) A is not correct but R is correct
(b) Both A and R are true but R is NOT the correct
explanation of A Comprehension Passage (for question numbers 11–15)
(c) A is true but R is false
(d) A is false but R is true Much is still unknown about human behavior. Unanswered
questions remain and further research is necessary.
7. In which research method manipulation of ‘independ- Knowledge about motivation, leadership, behavior, and
ent variable’ and control of ‘extraneous variables’ are change will continue to be of great concern to practi-
neither possible nor desirable? tioners of management for several reasons: It can help
(a) Experimental method improve the effective leadership of human resources; it
(b) Ex post facto method can help in preventing resistance to change, restriction to
(c) Historical method output, and personnel disputes; and often it can lead to a
(d) Descriptive survey method more productive organization. Our intention has been to
8. Identify those statements which describe the charac- provide a conceptual framework that may be useful to you
teristic features of qualitative research paradigm in applying the conclusions of the behavior sciences. The
A. Researchers tend to analyze their data inductively value that a framework of this kind has is not in changing
B. The substantive research hypothesis is tested via one’s knowledge, but in changing one’s behavior in work-
Null hypothesis ing with people.
C. Data take the form of words or pictures We have discussed three basic competencies in influenc-
D. Actual settings are the direct source of data ing: diagnosing - being able to understand and interpret
E. It assumes that there are social facts with a single the situation you are attempting to influence; adapting -
objective reality. Choose the correct answer from being able to adapt your behavior and the resources you
the options given below: control to the contingencies of the situation; and commu-
(a) A, B and C only nicating - being able to put the message in such a way that
(b) A, B and E only people can easily understand and accept it. Each of these
(c) B, C and E only competencies is different and requires a different devel-
(d) A, C and D only opmental approach. For example, diagnosing is cogni-
tive or of the mind in nature and requires thinking skills;
9. Given below are two statements adapting is behavioral in nature and requires behavioral
Statement I: Maximum performance tests are practice; and communicating is process-oriented and
designed to assess the upper limits of the examinees’ requires learning and interrelating the key steps in the
knowledge and ability. process. Because these three competencies require differ-
Statement II: Typical response tests are designed ent knowledge and skills, how do we continue the process
to measure specific behaviors and characteristics of that we started with
examinees. The key to starting the process of changing behav-
In light of the above statements, choose the most ior is sharing what you have learned with other people
appropriate answer from the options given below: in your own organization. Two things occur when peo-
(a) Both Statement I and Statement II are true ple who work together all have a common language.
(b) Both Statement I and Statement II are false First, they are able to give each other feedback and
(c) Statement I is correct but Statement II is false help in a very rational, unemotional way that effects
(d) Statement I is incorrect but Statement II is true behavior.
Second, when followers start to realize that if their
10. Given below are two statements, one is labelled as manager is using situational leadership, it is not the man-
Assertion (A) and the other is labelled as Reason (R). ager, but their behavior, that determines the leadership
Assertion (A): A cross break is a numerical tabular style to be used with them.
presentation of data usually in frequency or percent- 11. What is the value outcome of applying a theoretical
age form in which variables are cross-partitioned to framework of behavioral science?
study relations between them. (a) Changes in one’s own knowledge
Reason (R): The categories are set up according to (b) Change in one’s behavior while working with
the research hypothesis. others
In light of the above statements, choose the most (c) Not understandable human behavior
appropriate answer from the options given below (d) Emergence of value-loaded framework

Z03_MADAN 07_65901_2020_Paper 1 LAW.indd 13 23/12/22 5:11 PM


Z.14 NTA-UGC NET/JRF Paper 1 2020

12. What prompts a change in a person’s behavior? In light of the above statements, choose the most
(a) Leadership style appropriate answer from the options given below:
(b) Situational support to the managers (a) Both Statement I and Statement II are true
(c) Sharing of learning outcomes with others in the (b) Both Statement I and Statement II are false
organization (c) Statement I is true but Statement II is false
(d) Segregation of competencies (d) Statement I is false but Statement II is true
13. The inferences that can be drawn from the passage 19. Match List I with List II.
are
A. Common language among people in an organiza- List I
tion will ensure unbiased feedback Type of List II
B. People are known for fluctuating behavior communication Characteristics
C. People’s behavior influences the leader (A) Vertical    (I) Feedback oriented
D. Emotions and human behavior are separate and
(B) Horizontal   (II) Inter-personal
easily explicable. Choose the correct answer from
the options given below: (C) Circular (III) Top-down
(a) A and B only (D) Transactional (IV) Rumour-oriented
(b) B and C only
(c) C and D only Choose the correct answer from the options given
(d) A and C only below:
14. Which of the following prohibits resistance to change? (a) A - I, B - II, C - III, D - IV
(a) Knowledge of leader behavior (b) A - II, B - III, C - IV, D - I
(b) Removing restriction on output (c) A - III, B - IV, C - I, D - II
(c) Personnel disputes (d) A - IV, B - I, C - II, D - III
(d) Non-productive organization 20. In the following letter series, the next term is
15. Each of the basic competencies needs BDF, EGI, HJL, …
(a) Exclusivity (a) KMO
(b) Situational contingency (b) JMP
(c) Inter-relation with others (c) ILO
(d) A different approach in acquiring it (d) KNO
16. Which of the following can be considered as psycho- 21. Given below are two statements:
logical barriers to effective communication? Statement I: Different communities exist independ-
A. Descriptive message ent of one another but jointly somehow, they give an
B. Inattentiveness idea of communitarian life of a society.
C. Too much reliance on the written word
D. Limited retention Statement II: Earth exists either through a blind
E. Logical organization chance or through an inner necessity or through an
F. Flow of thought external cause.
Choose the correct answer from the options given In light of the above statements, choose the most
below: appropriate answer from the options given below:
(a) A, B and C only (a) Both Statement I and Statement II are true
(b) B, C and D only (b) Both Statement I and Statement II are false
(c) C, D and E only (c) Statement I is true but Statement II is false
(d) D, E and F only (d) Statement I is false but Statement II is true
17. A motorist travels to a place 150 kms away at an aver- 22. Given below are two statements, one is labelled as
age speed of 50 km / hour and returns at a speed of 30 Assertion (A) and the other is labelled as Reason (R):
km / hour. His average speed for the whole journey is Assertion (A): Human touch in communication ena-
(a) 35 km/hour bles students to become participatory.
(b) 37.5 km/hour
Reason (R): In a technology-driven classroom envi-
(c) 40 km/hour
ronment, the communicative role of a teacher is
(d) 42.5 km/hour
secondary.
18. Given below are two statements:
In light of the above statements, choose the correct
Statement I: Persuasive communication by a teacher answer from the options given below
in the classroom is intended to make his/her version (a) Both A and R are true and R is the correct explana-
of information acceptable to students. tion of A
Statement II: If it happens at the institutional level, (b) Both A and R are true but R is NOT the correct
the purpose behind it is image building. explanation of A

Z03_MADAN 07_65901_2020_Paper 1 LAW.indd 14 23/12/22 5:11 PM


NTA-UGC NET/JRF Paper 1 2020 Z.15

(c) A is true but R is false In light of the above statements, choose the most
(d) A is false but R is true appropriate answer from the options given below:
23. Deduction proceeds from (a) Both Statement I and Statement II are true
(a) Particular to universal (b) Both Statement I and Statement II are false
(b) Universal to particular (c) Statement I is true but Statement II is false
(c) Particular to particular (d) Statement I is false but Statement II is true
(d) Universal to universal 30. Given below are two statements:
24. When communication meets the aims and objectives Statement I: All knowledge is a recollection. It is true
of classroom teaching, it becomes on a certain account.
(a) Extra-ordinary Statement II: All men by nature, desire understanding.
(b) Personal
(c) Functional In light of the above statements, choose the most
(d) Obligatory appropriate answer from the options given below:
(a) Both Statement I and Statement II are true
25. A man is 30 years older than his son. Five years ago, (b) Both Statement I and Statement II are false
his age was 6 times the age of his son. The age of the (c) Statement I is true but Statement II is false
son is (d) Statement I is false but Statement II is true
(a) 10 years
(b) 11 years Comprehension Passage (for question numbers
(c) 12 years 31–35)
(d) 15 years
Study the data given in the table and answer the ques-

26. Match List I and List II. tions that follow
List 1 List II The following table shows the monthly reporting of

(Concepts) (Schools) patients for different diseases to three hospitals in a city:
(A) Nirvana    (I) Sankhya Disease/ Hospital Hospital Hospital
(B) Nihshreyasa   (II) Jainism Hospital A B C

(C) Apavarga (III) Upanishads Heart Attack 320 240 360

(D) Moksha (IV) Buddhism Blood Sugar 650 1350 1950


Blood Pressure 700 1400 2100
Choose the correct answer from the options given Kidney 210 420 630
below: Problems
(a) A - IV, B - III, C - II, D - I
(b) A - I, B - II, C - IV, D - III
31. What is the average number of patients for the least
(c) A - II, B - I, C - II, D - IV
reported disease?
(d) A - II B - IV, C - II, D - I
(a) ~ 415.2 (b) ~ 325.4
27. A sum of money earning compound interest annu- (c) ~ 391.7 (d) ~ 306.6
ally doubles itself after 4 years. What is the rate of
32. What is the percentage increase in the total number of
interest?
patients from hospital A to hospital C?
(a) ~14%
(a) ~ 120 % (b) ~ 168.96
(b) ~19%
(c) ~ 141 % (d) ~ 145 %
(c) ~18%
(d) ~15% 33. What is the percentage of number of patients report-
ing for blood sugar compared to the total patients in
28. What is the next term in the following series?
all three hospitals?
4, 12, 28, 52, _____
(a) ~ 25.90% (b) ~ 38.24 %
(a) 76
(c) ~ 39.42 % (d) ~ 35.12 %
(b) 84
(c) 98 34 What is the percentage increase in the number of
(d) 104 patients reporting heart attacks from hospital A to
hospital C?
29. Given below are two statements:
(a) 12.5% (b) 13.2%
Statement I: When I look deep into myself, I always (c) 11.9% (d) 9.2 %
stumble with one perception or another. I can never
35. For patients reporting for kidney problems, what is
catch myself.
the ratio of the number of patients reporting at hospi-
Statement II: Self is the logical presupposition of all tal A and those reporting at hospital C?
knowledge. Therefore, the concepts like substance (a) 2:3 (b) 3:4
and person cannot be applied to it. (c) 1:3 (d) 1:5

Z03_MADAN 07_65901_2020_Paper 1 LAW.indd 15 23/12/22 5:11 PM


Z.16 NTA-UGC NET/JRF Paper 1 2020

36. Match List I with List II. 40. Match List I with List II.
List 1 List 1
Categories of Environmental
environmental List II Protocol/
education Examples of those Agreement/Action List II
objectives objectives Plan Provisions
(A) Environmental    (I) To help social groups (A) Montreal    (I) Integrated water
Awareness and individuals Protocol resource management
identify and solve (B) Paris Agreement   (II) Emissions trading
environmental
problems (C) Climate Action (III) Phasing out ozone-
Plan depleting substances
(B) Environmental   (II) To help social groups
Knowledge and individuals (D) Kyoto Protocol (IV) Intended Nationally
acquire a set of values Determined
for the environment Contributions
(C) Environmental (III) To help social groups
Attitude and individuals Choose the correct answer from the options given
acquire a sensitivity to below:
the total environment (a) A - III, B - II, C- I, D - IV
(b) A - II, B - III, C- I, D - IV
(D) Environmental (IV) To help social groups (c) A - II, B - IV, C - III, D - I
Skill and individuals gain a (d) A - III, B - IV, C - I, D - II
variety of experience 41. The Chinese scholars, l-Qing and Xuan Zang visited
in environment ancient Nalanda University during
and its associated (a) 5th century ce
problems (b) 7th century ce
(c) 10th century ce
Choose the correct answer from the options given (d) 12th century ce
below:
42. Using a website to pour out one’s grievances is called
(a) A - II, B - I, C - III, D - IV
(a) Cyber venting
(b) A - I, B - III, C - IV, D - II
(b) Web hate
(c) A - IV, B - II, C - I, D - III
(c) Web anger
(d) A - III, B - IV, C - II, D - I
(d) Cyber abuse
37. Noise pollution is measured in decibels (dB) with ref-
43. Speed of Internet is measured in
erence to a standard sound intensity of the following
(a) GHz
magnitude
(b) Gbps
(a) 1 pico watt per m2
(b) 1 milliwatt per m2 (c) GB
(c) 1 nanowatt per m2 (d) dpi
(d) 1 microwatt per m2 44. Which of the following correctly lists computer mem-
38. In which year, the Union Government decided that all ory types from lowest to highest speed?
cases pertaining to the allocation of grants-in-aid from (a) Main Memory, Cache Memory, Secondary Storage
public funds to the Central Universities and other uni- (b) Cache Memory, Secondary Storage, Main Memory
versities and institutions of higher learning might be (c) Secondary Storage, Main Memory, Cache Memory
referred to the University Grants Commission? (d) Secondary Stage, Cache Memory, Main Memory
(a) 1952 45. What is the full form of the abbreviation GIF?
(b) 1953 (a) Graphics Interchange Format
(c) 1954 (b) Graphics Interchange File
(d) 1956 (c) Global Interchange Format
39. In a polluted urban area, which one of the following (d) Graphics Input Format
has the highest concentration (ppv) in photochemical 46. Given below are two statements, one is labelled as
smog? Assertion (A) and the other is labelled as Reason (R):
(a) Ozone Assertion (A): Machine Learning requires good qual-
(b) PAN ity and sufficient data to train and test the algorithm.
(c) Hydrocarbons (without methane) Reason (R): For correct classification, good quality
(d) Carbon monoxide data which is free from noise, and sufficient data is
required for training and testing of the algorithm.

Z03_MADAN 07_65901_2020_Paper 1 LAW.indd 16 23/12/22 5:11 PM


NTA-UGC NET/JRF Paper 1 2020 Z.17

In light of the above statements, choose the most


(a) Both Statement I and Statement II are true
appropriate answer from the options given below: (b) Both Statement I and Statement II are false
(a) Both A and R are correct and R is the correct (c) Statement I is correct but Statement II is false
explanation of A (d) Statement I is incorrect but Statement II is true
(b) Both A and R are correct but R is NOT the correct 49. According to the International Commission on Educa-
explanation of A tion for the Twenty-first Century headed by Jacques
(c) A is correct but R is not correct Delors, which of the following main tensions will be
(d) A is not correct but R is correct witnessed during the 21st century;
47. Given below are two statements, one is labelled as A. The tension between the global and the local
Assertion (A) and the other is labelled as Reason (R): B. The tension between tradition and modernity
C. The tension between the spiritual and the material
Assertion (A): Lakes are considered to be more sus- D. The tension between the poor and the rich
ceptible to pollution than rivers E. The tension between the developed and the devel-
Reason (R): Water in lakes may take decade(s) to be oping societies
replaced Choose the correct answer from the options given
In light of the above statements, choose the most below:
appropriate answer from the options given below (a) A, B and C only
(a) Both A and R are correct and R is the correct (b) B, C and D only
explanation of A (c) C, D and E only
(b) Both A and R are correct but R is NOT the correct (d) A, D and E only
explanation of A 50. Thermal reactors produce energy by fission of the fol-
(c) A is correct but R is not correct lowing nuclear fuels
(d) A is not correct but R is correct A. Uranium-235 (235U)
48. Given below are two statements B. Uranium-233 (233U)
C. Uranium-238 (238U)
Statement I: Universities are established by Central/ D. Thorium-232 (232Th)
State Act under 2(f) of the UGC Act, 1956. E. Plutonium-239 (239Pu)
Statement II: Deemed to be University is declared Choose the correct answer from the options given
by a Notification of the Central Government, on the below:
advice of the UGC, under section 3 of the UGC Act, (a) A, B, C and E only
1956. (b) A, B, C, D and E
In light of the above statements, choose the most (c) A, B, D and E only
appropriate answer from the options given below: (d) A, B and E only

Answer Keys
1. (a) 2. (b) 3. (d) 4. (a) 5. (c) 6. (a) 7. (b) 8. (d) 9. (a) 10. (a)
11. (b) 12. (c) 13. (d) 14. (a) 15. (d) 16. (b) 17. (b) 18. (a) 19. (c) 20. (a)
21. (a) 22. (c) 23. (b) 24. (c) 25. (b) 26. (a) 27. (b) 28. (b) 29. (a) 30. (a)
31. (d) 32. (b) 33. (b) 34. (a) 35. (c) 36. (d) 37. (a) 38. (a) 39. (a) 40. (d)
41. (b) 42. (a) 43. (b) 44. (c) 45. (a) 46. (a) 47. (a) 48. (a) 49. (a) 50. (d)

Z03_MADAN 07_65901_2020_Paper 1 LAW.indd 17 23/12/22 5:11 PM


Z.18 NTA-UGC NET/JRF Paper 1 2020

Solution
1. (a) with a gauge of where their level of understanding
These concepts are important in research process. is at the current moment, and enable the teacher to
(a) Hypothesis testing in statistics is a way for the adjust accordingly to meet the emerging needs of the
researcher to test the results of a survey or experi- class.
ment to see, whether we can have meaningful 7. (b)
results.
In Ex post facto study (or say after-the-fact research),
(b) A sample refers to a smaller, manageable version
the investigation starts after the fact has occurred
of a larger group. It is a subset containing the
without interference from the researcher. Here, the
characteristics of a larger population. Samples
groups with qualities that already exist are compared
are used in statistical testing when population
on some dependent variable. The assignment of sub-
sizes are too large for the test to include all pos-
jects to different groups is based on whichever vari-
sible members or observations.
able is of interest to the researchers.
(c) Anything that becomes a means of collecting
information for your study is called a research tool 8. (d)
or a research instrument. For example, observa- Qualitative research methods mostly use open-ended
tion forms, interview schedules, questionnaires, techniques (such as interviews), to collect data. They
and interview guides are all classified as research may use non-statistical techniques to analyze the
tools. data.
(d) A non-parametric test (also called as distribu- The first statement uses the word ‘inductively’ that is
tion free test) does not assume anything about mostly used in qualitative research. The third state-
the underlying distribution. We should use non-­ ment reflects use of words or pictures and not exactly
parametric tests only if you have to know that the quantitative data. The fifth statement reflects that
assumptions like about normal distribution are data is better collected in ‘natural settings’.
being violated.
9. (a)
2. (b)
Cronbach (1949/1960) classified personnel selection
Random means every member has equal chance of tests into two broad categories, tests of maximum
being elected as member of sample. Probability sam- and tests of typical performance. The distinguish-
pling means that every member of the population ing feature of maximum performance tests is that
has a chance of being selected. It is more scientific they seek to assess how much or how well people
approach. It is mainly used in quantitative research. can perform at their best. Hence, candidates are
These sampling means are preferred as they help in encouraged to do well in order to earn the best score
being unbiased and objective. they can.
3. (d) On the other hand, ‘Tests of typical performance’
B denotes ‘analysis’ of conceptual elements. assess how the ability of a job candidate is evident on
D denotes ‘synthesis’ for creating innovative ideas. a day-to-day basis.
E asks for linking with internal or external standards. In short, these two types of tests assess what the can-
As per choices, (d) seems to be the best answer that didate chooses to do. Hence, they assess a candidate’s
‘reflective’ outcomes. motivation rather than his or her ability.
4. (a) 10. (a)
The following can be identified as the factors which To describe a single categorical variable, we use fre-
contributes towards positive feedback: quency tables. To describe the relationship between
It should be educative in nature. two categorical variables, we use a special type of
It should be given in a timely manner. table called a cross-tabulation (or crosstab). In a
Be sensitive to the individual needs of the student. cross-tabulation, the categories of one variable deter-
It should reference a skill or specific knowledge. mine the rows of the table, and the categories of the
Feedback should be given to keep students ‘on tar- other variable determine the columns. The cells of the
get’ for achievement. table contain the number of times that a particular
There should be scope for one-on-one conference combination of categories occurred.
to make it more effective. The table dimensions are reported as as R × C, where
5. (c) Answer is clear from the question itself. R is the number of categories for the row variable, and
C is the number of categories for the column variable.
6. (a) Answer is clear from the question itself. Tables of dimensions 2 × 2, 3 × 3, 4 × 4, etc., are
Formal evaluations are often informal and ungraded. called as square crosstabs.
Their aim is to provide both the students and teacher

Z03_MADAN 07_65901_2020_Paper 1 LAW.indd 18 23/12/22 5:11 PM


NTA-UGC NET/JRF Paper 1 2020 Z.19

Do you drink coffee? 18. (a)


Basis for
Comparison No Yes Total The basic style of communication should remain the
Gender Male 13 33 46 same. Persuasive communication is a form of commu-
nication that involves the act of convincing or guiding
       Female 13 32 45 listeners or readers towards the adoption of certain
Total 26 65 91 attitudes, actions, and ideas through emotional or
rational means. Persuasive skills include emotional
11. (b) Answer is available in the main passage itself. intelligence, active listening, interpersonal skills,
negotiation, logic and reasoning, etc.
12. (c) Answer is available in the main passage itself.
19. (c)
13. (d) Answer is available in the main passage itself.
Vertical communication occurs across different
14. (a) Answer is available in the main passage itself.
hierarchies in social institutions such as family or
15. (d) Answer is available in the main passage itself. management.
16. (b) Answer is available in the main passage itself. Horizontal communication means communication
There are four psychological barriers. These barriers between the members at the same level. Grapevine
are perception, homeostasis, conformity and com- communication (sort of informal also) may be a part
mitment, and personality factors. There is selective of it.
perception. This is when the person only chooses to Circular means ‘effective’ communication that needs
remember certain details about the change. They can- some feedback mechanism.
not look beyond the negative to see all the positives a
Transactional Model of communication is the
change could make. Perception also is a barrier when
exchange of messages between sender and receiver
the change agent may act in a way that someone in
where each take turns to send or receive messages. The
the organization may perceive as inappropriate.
model is mostly used for interpersonal communica-
Homeostasis simply wants to stay in an environment tion. Some times, as the element of feedback is there,
where it is comfortable. The key to this barrier is to it is also called as circular model of communication.
understand what the organization is going through at
each stage of the change and maintaining a certain 20. (a)
level of comfort as much as possible for the people There is skipping of one alphabet.
affected. B+2=D
D+2=F
17. (b) Then, for the first alphabet of second letter series, the
Average speed = Total Distance / Total Time letter between D and F is placed.
Total distance = 2 × 150 = 300 kms Thus, fourth series is KMO.
Time required during first phase = 150 / 50 = 3 hours 21. (a)
Time required during second phase = 150 / 30 = 1. The human life should exist in independent but
5 hours diversified forms in the society.
2. The earth intends to provide an integrating oppor-
Total time taken = 3 + 5 = 8 hours
tunity for all.
Thus, average speed = 300 / 8 = 37.5 kms / hour
22. (c)
Alternatively, this question can be solved directly with
The concept of human shared including the micro-
harmonic mean.
dynamics of participation is always crucial at impor-
For the journey where there are equal distances, the tant stages. The teacher as a facilitator is challenged
average speed can be calculated with help of har- by a range of social contingencies.
monic mean. (For example, from Chandigarh, Karnal
The role of a teacher is to share understanding in an
is exactly at middle point while moving to Delhi. Thus,
innovative manner, so the role of a teacher is never
there are two equal distances. In the second option,
secondary.
moving from Chandigarh to Karnal and then coming
back constitutes two ‘equal phases’ of journey). 23. (b)
Harmonic mean / Average speed Deduction means moving from universal to par-
= 2 / [(1/x + 1/y)] where x and y are two speeds ticular. Deduction is basically scientific approach to
do research as well. It can be identified more with
= 2 / [1/50 + 1/30] positivism.
= 2 / [ (3 + 5) / 150]
24. (c)
= (2 × 150) / 8 = 37.5 kms per hour
Functional communication is the means by which an

Here, total distance is not required while finding
individual spontaneously and independently commu-
solution.
nicates his/her wants and needs and socializes with

Z03_MADAN 07_65901_2020_Paper 1 LAW.indd 19 23/12/22 5:11 PM


Z.20 NTA-UGC NET/JRF Paper 1 2020

others. This communication can occur through a vari- The second is about Greek maxim – ‘know thyself’.
ety of forms, including speech, picture exchange, ges- That is to know yourself is to know the world.
tures, sign language and assistive devices. 30. (a)
25. (b) Statement I: Knowledge is a recollection of facts.
Let the age of son = X years Competencies are the person’s knowledge and behav-
The age of father = X + 30 years iours that lead them to be successful in a job.
ATQ, five years ago Statement II: Understanding is achieved through
X + 30 – 5 = 6(X – 5) effective communication.
X + 25 = 6X – 30 Thus, both statements are true.
OR 25 + 30 = 6X – X; 31. (d)
5X = 55 The heart attack is the least reported disease if we do
X = 55/5 = 11 years the total of all.
26. (a) The average number of heart attacks =
The words in question, reflect the ultimate goals in (320 + 240 + 360) / 3 = 920 / 3 = 306.67
life though there is a lot of semblance in these words.
32. (b)
(i) Nihsreyasa (fu%Js;le~) is the quest for oneself. It is
the search of innate adhyatmikity within oneself. Total number of patients in hospital A = 320 + 650 +
It is said to be the highest goal of mankind. 700 + 210 = 1880
(ii) Nirvana (fuokZ.k) is the goal of the Buddhist path. Total number of patients in hospital C = 360 + 1950
The literal meaning of the term is “blowing out” + 2100 + 630 = 5040
or “quenching”. Nirvana is the ultimate spiritual Increase in number = 5040 – 1880 = 3160
goal in Buddhism and marks the soteriological
release from rebirths in samsãra. Percentage increase in number = 3160 / 1880 × 100
(iii) The word apavarga means “liberation.” Pavarga = 168.96 %
means “material existence.” In material exist- 33. (b)
ence, one always works very hard but is ulti- We need to add number of patients in hospital B to
mately baffled. One then dies and has to accept those of A and C that have already been calculated.
another body to work very hard again. This is the Total number of patients in hospital B = 240 + 1350
cycle of material existence. Apavarga means just + 1400 + 420 = 3410
the opposite.
(iv) Moksha is the ultimate spiritual liberation. Total of numbers in hospitals A, B and C = 1880 +
3410 + 5040 = 10330
27. (b)
Let the principal amount = ` 1 Total number of blood sugar patients in all hospitals
Time period = 4 years = 650 + 1350 + 1950 = 3950
Amount = ` 2, Then Compound Interest = 2 – 1 = ` 1 Required percentage = 3950 / 10330 × 100
1 = 1(1+R/100)4 – 1 = 38.24 %
2 = (1+R/100)4 34. (a)
21/4 = 1 + R/100 Required percentage = (360 – 320) / 320 × 100
1.189 = 1 + R/100 = 40 / 320 × 100 = 12.5%
1.1892 – 1 = R/100
35. (c)
0.1892 × 100 = R
Required ratio = 210 : 630 = 1:3
R = 18.92 % (apprx)
Thus, 19% seems to be the best possible answer. 36. (d)
28. (b) The following are basically qualitative concepts about
1² + 3 = 4 environment:
3² + 3 = 12 1. Environmental awareness means being aware
5² + 3 = 28 about general issues of natural environment.
7² + 3 = 52 We need to make choices that benefit the earth,
9² + 3 = 84 rather than hurt it.
Thus, the answer is 84. 2. Environmental knowledge is the information
about environment and the ability to under-
29. (a)
stand and evaluate its impact on society and the
Human beings are conscious not only of the world environment.
around them but also of themselves: their activities, 3. Environmental attitudes are a psychological ten-
their bodies, and their mental lives. Thus, they are dency expressed by evaluative responses to the
self-conscious or self-aware. Self-consciousness can natural environment with some degree of favor
be understood as an awareness of oneself. or disfavor.

Z03_MADAN 07_65901_2020_Paper 1 LAW.indd 20 23/12/22 5:11 PM


NTA-UGC NET/JRF Paper 1 2020 Z.21

4. Environmental skills refer to the knowledge, UGC has decentralized its operations by setting up
abilities, values and attitude required to live in, six regional centres at Pune, Hyderabad, Kolkata,
develop and support a society which reduces the ­Bhopal, Guwahati and Bangalore with head office in
impact of human activity on the environment. New Delhi.
37. (a) 39. (a)
For sound intensity (the power of the sound waves Photochemical smogs are formed by the action of sun-
per unit of area), 0 decibels is equal to 1 pico watt per light on oxides of nitrogen, and the subsequent reac-
square meter. This corresponds approximately to the tions with hydrocarbons. Photochemical smogs were
faintest sound that can be detected by a person who first identified in Los Angeles in the mid-1940s. These
has good hearing. A quiet room has a normal sound smogs act as bronchial irritants and can also irritate
intensity of around 40 decibels, ten thousand times the eyes. Among the pollutants involved in photo-
louder than the faintest perceptible sound, and a chemical smogs are ozone, nitrogen dioxide and per-
thunderclap may have an intensity of 120 decibels, a oxyacyl nitrate (PAN).
trillion times louder than the faintest sound. 40. (d)
38. (a) 1. Montreal Protocol (1987) on Substances that
Let us look at the evolution of UGC over the period deplete the ozone layer were finalized to control
that might be important for other questions as well. the consumption and production of anthropo-
1. The present system of higher education dates back genic ozone-depleting substances (ODSs) and
to Mountstuart Elphinstone’s minutes of 1823. some hydrofluorocarbons (HFCs). September 16
2. Sir Charles Wood’s Dispatch of 1854, famously is celebrated as World Ozone Day. A UN study
known as the ‘Magna Carta of English Education ‘Scientific Assessment of Ozone Depletion: 2018’,
in India` suggested the formulation of a coher- has shown that the ozone layer is recovering at a
ent policy of education. Subsequently, the uni- rate of 1-3% per decade. The stratospheric ozone
versities of Calcutta, Bombay (now Mumbai) and layer protects life on earth from harmful UV
Madras were set up in 1857, followed by the uni- radiation.
versity of Allahabad in 1887. 2. Paris Agreement is a legally binding interna-
tional treaty on climate change. It was adopted by
The Inter-University Board (later known as the Asso-
196 Parties at COP 21 in Paris, on 12 December
ciation of Indian Universities) was established in 2015 and entered into force on 4 November 2016.
1925 to promote university activities, by sharing Its goal is to limit global warming to well below
information and cooperation in the field of education, 2, preferably to 1.5 degrees Celsius, compared
culture, sports and allied areas. to pre-industrial levels. Intended Nationally
In 1944, the Sargeant Report recommended the for-
Determined Contributions are (intended) reduc-
mation of a University Grants Committee, that was tions in greenhouse gas emissions declared by dif-
formed in 1945 to oversee the work of the three ferent nations so that they are more serious about
Central Universities of Aligarh, Banarasand Delhi. them.
In 1947, the Committee was entrusted with the 3. National Action Plan for Climate Change: It is
responsibility of dealing with all the then existing a Government of India’s programme launched in
Universities. 2008 to mitigate and adapt to the adverse impact
of climate change. The action plan was launched
In 1948, a report under the Chairmanship of Dr. S

in 2008 with eight sub-missions. Two missions
Radhakrishnan recommended that the University
were added later. These missions can be seen in
Grants Committee be reconstituted on the general
Unit 9.
model of the University Grants Commission of the
4. Kyoto Protocol: It is an international agree-
United Kingdom.
ment that aimed to reduce carbon dioxide (CO2)
In 1952, the Union Government decided that all cases
emissions and the presence of greenhouse gases
pertaining to the allocation of grants-in-aid from pub- (GHG) in the atmosphere. It speaks about ‘com-
lic funds to the Central Universities and other Uni- mon but differentiated responsibilities’. This pro-
versities and Institutions of higher learning might be tocol required that industrialized nations needed
referred to the UGC. Consequently, University Grants to lessen the amount of their CO2 emissions to
Commission (UGC) was formally inaugurated on 28 a previous year level. It was signed in 1997 as
December 1953. a part of UNFCCC, and was to be implemented
UGC was formally established only in November
from 2005. The Kyoto Protocol applies to the six
1956 as a statutory body of the Government of India greenhouse gases listed in Annex A: ­carbon diox-
through an Act of Parliament for the coordination, ide (CO2), Methane (CH4), nitrous oxide (N2O),
determination and maintenance of standards of uni- hydrofluorocarbons (HFCs), perfluorocarbons
versity education in India. (PFCs), and sulfur hexafluoride (SF6).

Z03_MADAN 07_65901_2020_Paper 1 LAW.indd 21 23/12/22 5:11 PM


Z.22 NTA-UGC NET/JRF Paper 1 2020

41. (b)
Additional Information:
Xuanzang (also popular as Hiuen Tsang), born in 602 L1 cache, or primary cache, is extremely fast but

A.D was the celebrated Chinese traveler. He has been relatively small, and is usually embedded in the
described therefore as the “Prince of Pilgrims.” processor chip as CPU cache. Level 3 (L3) cache is
His visited India during the reign of Harshavardhana. specialized memory developed to improve the per-
India is much indebted to this Chinese traveler for the formance of L1 and L2. L1 or L2 can be significantly
valuable accounts he left behind with many details faster than L3, though L3 is usually double the speed
of political, religious, economic, social conditions of of DRAM.
those days. The biography of Hiuen Tsang, written by 45. (a)
another Chinese, is also another valuable source for
Indian history. He remained at the Nalanda Univer- GIF is an image file format commonly used for images
sity for about five years. on the web and sprites in software programs. Unlike
the JPEG image format, GIFs use lossless compression
42. (a) that does not degrade the quality of the image.
Cyber-venting is the act of bad-mouthing about one’s 46. (a)
employer, senior executives, colleagues, etc. with the
intention of letting off the steam, using websites and Machine learning is a type of artificial intelligence
making such comments public for all to see them. that allows software applications to become more
accurate at predicting outcomes without being
43. (b) explicitly programmed to do so. Machine learning
GHz: Clock speed is the rate at which a processor exe- algorithms use historical data as input to predict
cutes a task and that is measured in Gigahertz (GHz). new output values.
A 2.6-Ghz processor, therefore, can run 2.6 billion 47. (a)
instructions in a second, while a 2.3-Ghz processor
can run 2.3 billion instructions per second. The ‘trophic status’ of lakes is the central concept
in the management of lakes. It describes the rela-
Gbps: GigaBits or GigaBytes per Second - One bil- tionship between nutrient status of a lake and the
lion bits or bytes per second. Gbps is a measurement growth of organic matter in the lake. Eutrophica-
of peripheral data transfer or network transmission tion is the process of change from one trophic state
speed. The correct abbreviation is “b” for bits and “B” to a higher trophic state by the addition of nutrient.
for bytes; however, “b” and “B” are often interchanged. Agriculture is a major factor in eutrophication of
GB: a unit of information equal to 1000 megabytes surface waters.
or 109 (1,000,000,000) bytes. This is gigabyte. A unit 48. (a)
for measuring computer memory. megabyte, MB, M
- a unit of information equal to 1000 kilobytes or 106 The evolution of UGC has been discussed in question
(1,000,000) bytes. number 38.
dpi (dots per inch): It is a measure of the resolution 49. (a)
of a printed document or digital scan. The higher the Jacques Delors worked on potential conflict situa-
dot density, the higher the resolution of the print or tions in the world. He played an important role in
scan. Typically, dpi is the measure of the number of coming out with the work “Learning: The Treasure
dots that can be placed in a line across one inch, or Within”. It posits an education that favours the emer-
2.54 centimeters. gence of the individual talents of each, preparing
44. (c) people for life as citizens, family and community
members, and also economic agents. Its image of

a successful education system is one built on four
Smaller, L0:
pillars:
faster and
costlier
registers

L1: on-chip L1
} CPU registers hold words retrieved
from L1 cache 1. Learning to know
2. Learning to do
}
(per byte) cache (SRAM) L1 cache holds cache lines retrieved
storage
off-chip L2 from the L2 cache memory 3. Learning to be
}
devices L2:
cache (SRAM) L2 cache holds cache line 4. Learning to live together
retrieved from main memory
L3:
main memory
50. (d)
}
Larger, (DRAM)
Main memory holds disk
slower, and blocks retrieved from The most of nuclear reactors use enriched uranium
cheaper
(per byte) L4: local secondary storage
local disks
(U-235) or reprocessed plutonium (Pu-239) as fuel

} (in the Uranium-Plutonium cycle), and only a hand-


(local disks)
storage Local disks hold files
devices retrieved from disks on ful have used thorium.
remote network servers
remote secondary storage
L5:
(distributed file systems, Web servers) Thorium itself will not split and release energy. Rather,
when it is exposed to neutrons, it will undergo a series

Z03_MADAN 07_65901_2020_Paper 1 LAW.indd 22 23/12/22 5:11 PM


NTA-UGC NET/JRF Paper 1 2020 Z.23

of nuclear reactions until it eventually emerges as an much Uranium. India has the world’s third largest
isotope of uranium called U-233, which will readily reserves of thorium. As discussed, thorium cannot
split and release energy next time it absorbs a neutron. be used as a fuel in its natural state. It needs to be
Thorium is therefore called fertile, whereas U-233 is converted into its usable “fissile” form after a series of
called fissile. China and India both have substantial reactions.
reserves of Thorium-bearing minerals and not as

Z03_MADAN 07_65901_2020_Paper 1 LAW.indd 23 23/12/22 5:11 PM


NTA-UGC NET/JRF
Paper 1 2021 - Set 1
INSTRUCTIONS To NTA-NET ASPIRANTS
Time Allowed: 1 hour  50 × 2 = 100 Marks

1. This paper consists of Fifty (50) objective type questions of Two (2) marks each.
2. All questions are compulsory.
3. Each item has four alternative responses marked (a), (b), (c) and (d). You have to darken the circle as indicated below
on the correct response against each item.
Example: a b c d where (c) is the correct response.
4. Your responses to the items are to be indicated in the OMR Sheet given at the end of the book.
5. Read instructions provided with each question carefully.
6. There are no negative marks for incorrect answers.

Information for Question 1 to 5: 4. Which year has the maximum exports?


Study the table carefully and answer the questions that (a) 2017 (b) 2018
follow: (c) 2019 (d) 2020
In the following table, total Exports and Imports of 5
5. Find out the difference between the average export
countries over 4 years (in Rs Crore) is given. Study the
and average import for the country P.
table carefully and answer the given questions. (I-import,
(a) 7.25 (b) 7.75
E-export)
(c) 8.25 (d) 8.50
Year 2017 2018 2019 2020 6. The number of characters in 8 bit ASCII code (Ameri-
Country I E I E I E I E can Standard Code for Information Interchange) is
P 25 30 24 32 28 36 22 32
(a) 128 (b) 256
(c) 512 (d) 64
Q 28 26 26 31 29 37 26 34
7. Match List I with List II:
R 24 32 28 25 32 36 28 34
S 22 28 18 28 16 21 23 32 List I List II
T 26 31 16 33 12 23 21 28 Steps in Inference Statements
A. Thesis (Pratijña) I. Because of smoke
1. Find out the ratio of export done by country S and T
during 2017-2020. B. Reason (Hetu) II. Wherever there is smoke,
(a) 109/115 (b) 111/115 there is fire, as in the kitchen
(c) 108/117 (d) 107/117 C. Example III. The mountain has fire
(Udaharan. a)
2. Which country has the maximum percentage of
profit increase from 2019 to 2020? (Profit = Export D. Application IV. The mountain also has
(Upanaya) smoke accompanied by fire,
- Import)
like the kitchen
(a) P (b) Q
(c) R (d) S Choose the correct answer from the options given below:

(a) A - II, B - I, C - III, D - IV
3. Which year has the minimum average import?
(b) A - III, B - I, C - II, D - IV
(a) 2017 (b) 2018
(c) A - III, B - II, C - I, D - IV
(c) 2019 (d) 2020
(d) A - III, B - IV, C - II, D – I

Z04_MADAN 07_65901_2021 paper.indd 24 23/12/22 5:13 PM


NTA-UGC NET/JRF Paper 1 2021 - Set 1 Z.25

8. Given below are two statements 12. Which one of the following is an important character-
Statement I: Exploration is particularly useful when istic of the researchers involved in qualitative study?
researchers lack a clear idea of the problems they will (a) Enthusiasm (b) Non-partisan
meet during the study. (c) Reflexivity (d) Transparency
Statement II: Through exploration, researchers 13. Given below are two statements, one is labelled as
develop concepts more clearly, establish priorities, Assertion A and the other is labelled as Reason R
develop operational definitions, and improve the final Assertion A: In classroom communication, teachers
research design. should acknowledge and take into account students’
In light of the above statements, choose the correct views.
answer from the options given below: Reason R: In a classroom, the probability of mes-
(a) Both Statement I and Statement II are false sage reception can be enhanced by establishing a
(b) Both Statement I and Statement II are true viewpoint.
(c) Statement I is false but Statement II is true In light of the above statements, choose the correct
(d) Statement I is true but Statement II is false answer from the options given below
9. Given below are two statements, one is labelled as (a) A is false but R is true
Assertion A and the other is labelled as Reason R (b) A is true but R is false
Assertion A: According to Naiyayikas, the fallacy of (c) Both A and R are true and R is the correct expla-
Savyabhicara occurs when the middle term leads to nation of A
different opposite conclusions. (d) Both A and R are true but R is NOT the correct
explanation of A
Reason R: When the middle term is irregular, it is dis-
tributively not related to the major term. 14. If ‘All men are mortal’ is given as True, then which of
In light of the above statements, choose the correct the following options can be validly inferred from it?
answer from the options given below: A. ‘No men is mortal’ is False
(a) A is false but R is true B. ‘Some men are mortal’ is True
(b) A is true but R is false C. ‘Some men are not mortal’ is True
(c) Both A and R are true and R is the correct expla- D. ‘Some men are not mortal’ is False
nation of A E. ‘Some men are mortal’ is False
(d) Both A and R are true but R is NOT the correct Choose the correct answer from the options given

explanation of A below:
10. In order to transform the regulatory system of Higher (a) A, B and D only
Education, the NEP proposes to set up HECI (Higher (b) A, B and C only
Education Commission of India). Which of the follow- (c) A, B, C and D only
ing are verticals of HECI? (d) B, C, D and E only
A. NHERC 15. INFLIBNET Centre, which is an autonomous Inter-
B. NAS University Centre of the University Grants Commis-
C. HEGC sion, maintains
D. NCIVE (a) e-Gyankosh (b) e-Pathshala
E. NAC (c) Shodhganga (d) Swayam Prabha
Choose the correct answer from the options given
16. Given below are two statements regarding the Router
below: Statement I: It enables data packets to be routed
(a) A, B and C only between different networks. Statement II: It works in
(b) A, C and E only the Data Link Layer.
(c) B, C and E only
In light of the above statements, choose the correct
(d) C, D and E only answer from the options given below:
11. United Nations Environmental Program (UNEP) was (a) Both Statement I and Statement II are false
the result of deliberations held during (b) Both Statement I and Statement II are true
(a) Earth Summit at Rio de Janeiro in 1992 (c) Statement I is false but Statement II is true
(b) Human Environmental Conference at Stockholm (d) Statement I is true but Statement II is false
in 1972
17. Survey research is based on which of the following?
(c) Kyoto Protocol 1997 (a) Constructivism (b) Hermeneutics
(d) Montreal Protocol 1987 (c) Interpretivism (d) Positivism

Z04_MADAN 07_65901_2021 paper.indd 25 23/12/22 5:13 PM


Z.26 NTA-UGC NET/JRF Paper 1 2021 - Set 1

18. The number of Sustainable Development Goals is 26. Which region of brain is involved in our ability to
(a) 13 (b) 15 learn new information, particularly if it is verbal?
(c) 17 (d) 19 (a) Cerebrum (b) Hippocampus
19. Global Citizenship Education promotes (c) Pons (d) Thalamus
(a) Common international laws for global citizenship 27. Match List I with List II
(b) International public relations
List I List II
(c) Issues pertaining to citizenship across countries
involving VISA A. Indian Institute of Advanced Study I. Noida, UP
(d) More peaceful, tolerant, inclusive, secure and B. Indian Biological Sciences and II. Shimla, HP
sustainable societies Research Institute

20. Marshall Mcluhan’s name is associated with the C. Indian Institute of Soil Science III. Bhopal, MP
assertion: D. I ndian Institute of Sugarcane IV. Lucknow, UP
(a) A message is a message Research
(b) The medium is the message Choose the correct answer from the options given

(c) The message determines the medium accurately below:
(d) The message is the medium (a) A - I, B - IV, C - II, D - III
21. How many terms are there in the series 168, 175, 182, (b) A - II, B - I, C - III, D - IV
____, 266? (c) A - III, B - IV, C - II, D - I
(a) 13 (b) 14 (d) A - IV, B - III, C - II, D – I
(c) 15 (d) 16 28. A total of 324 coins of 20 paise and 25 paise make a
22. Which of the following fractions is the smallest? sum of `71. The number of 20 paise coins is
(a) 11/16 (b) 29/40 (a) 124 (b) 125
(c) 5/8 (d) 65/80 (c) 144 (d) 200
23. Which among the following are non-parametric 29. Given below are two statements
statistics? Statement I: Policy action and implementation plans
A. t-test require sound database systems at the regional level
B. F-test only.
C. Spearman’s rank order correlation Statement II: The statistical system should ensure its
D. Mann-Whitney-Wilcoxon test impeccability with data architecture, security, quality,
E. Kendall coefficient of concordance cleaning and integration.
Choose the correct answer from the options given
In light of the above statements, choose the most
below: appropriate answer from the options given below:
(a) A, B and C only (b) B, C and D only (a) Both Statement I and Statement II are correct
(c) B, D and E only (d) C, D and E only (b) Both Statement I and Statement II are incorrect
(c) Statement I is correct but Statement II is incorrect
24. Identify the fallacy committed in the argument:
(d) Statement I is incorrect but Statement II is correct
Some birds are not beautiful creatures. All dogs are
beautiful creatures. 30. Match List I with List II
Therefore, no dogs are birds. List I List II
(a) Fallacy of Exclusive Premises
I/O device Data transfer rate
(b) Fallacy of the Illicit Process of Major Term
A. Disk I. 50 bps
(c) Fallacy of the Illicit Process of Minor Term
(d) Fallacy of the Undistributed Middle Term B. Mouse II. 1 Mbps
C. Laser printer III. 120 bps
25. The communication among persons working at differ-
ent levels who have no direct reporting relationship is D. Keyboard IV. 100 Mbps
called: Choose the correct answer from the options given

(a) Diagonal communication below:
(b) Horizontal communication (a) A - I, B - II, C - III, D - IV
(c) Intrapersonal communication (b) A - II, B - I, C - III, D - IV
(d) Upward communication (c) A - IV, B - II, C - I, D - III
(d) A - IV, B - III, C - II, D – I

Z04_MADAN 07_65901_2021 paper.indd 26 23/12/22 5:13 PM


NTA-UGC NET/JRF Paper 1 2021 - Set 1 Z.27

31. Which one of the following is an example of a techno- Choose the correct answer from the options given

logical barrier in communication? below:
(a) Complexity of ideas (a) A, B and E only (b) A, B and C only
(b) Low bandwidth of internet (c) A, C and E only (d) C, D and E only
(c) Personal attitude towards technology 38. Given below are two statements
(d) Physical locations of communicators Statement I: Small amount of atmospheric Nitrogen
32. Given below are two statements dioxide NO2 is produced from polluting sources
Statement I: The compound interest on `280 for 18 Statement II: Most of the atmospheric NO2 is produced
months at 10 % per annum is `44.3. Statement II: At due to photochemical reactions in the atmosphere
5.6 % rate of simple interest, a certain sum will be In light of the above statements, choose the most
doubled in 15 years appropriate answer from the options given below:
In light of the above statements, choose the correct (a) Both Statement I and Statement II are correct
answer from the options given below: (b) Both Statement I and Statement II are incorrect
(a) Both Statement I and Statement II are false (c) Statement I is correct but Statement II is incorrect
(b) Both Statement I and Statement II are true (d) Statement I is incorrect but Statement II is correct
(c) Statement I is false but Statement II is true
39. The average of ten number is 7. If each number is mul-
(d) Statement I is true but Statement II is false tiplied by 11, then the average of the new set of num-
33. The Post Office protocol is bers is:
(a) Protocol used for sending emails (a) 72 (b) 77
(b) Protocol used for transfer of files from one com- (c) 78 (d) 82
puter to another computer 40. Which of the following water borne diseases may be
(c) Protocol used to handle email attachments caused by virus, bacteria and protozoa?
(d) Protocol used when receiving emails from the (a) Cholera (b) Diarrhea
email server (c) Hepatitis (d) Typhoid
34. CBCS is 41. Given below are two statements
(a) Choice Based Credit System
Statement I: According to Piaget, the ability to solve

(b) Credit Based Choice Scores
conservation problems depends on having an under-
(c) Criteria Based Choice System
standing of three basic aspects of reasoning: identity,
(d) Criteria Based Creditable Scores compensation, and reversibility.
35. mooKIT platform uses, an open source content man- Statement II: The stage of classification does not

agement system. depend on a student’s ability to focus on a single
(a) DRUPAL (b) JOOMLA characteristic of objects in a set and group the objects
(c) Moodle (d) SAKAI according to that characteristic.
36. Following are some statements regarding File Trans- In light of the above statements, choose the most

fer Protocol (FTP). Choose the correct statements appropriate answer from the options given below:
A. It is used to access the world wide web (www) (a) Both Statement I and Statement II are correct
B. It is used to download data from file servers (b) Both Statement I and Statement II are incorrect
C. It is used for very small files (c) Statement I is correct but Statement II is incorrect
D. Files are transferred from one device to another (d) Statement I is incorrect but Statement II is correct
Choose the correct answer from the options given
42. Some of the types of hypothesis are as follows:
below: A. Descriptive
(a) A and B only (b) A and C only B. Null
(c) B and C only (d) B and D only C. Confounding
37. According to Urie Bronfenbrenner’s biological model D. Intervening
of human development, microsystem constitutes E. Explanatory (Causal)
____________. Choose the correct answer from the options given

A. Immediate family below:
B. School (a) A, B and D only (b) A, B and E only
C. Beliefs (c) A, B and C only (d) B, C and D only
D. Customs
E. Neighbourhood

Z04_MADAN 07_65901_2021 paper.indd 27 23/12/22 5:13 PM


Z.28 NTA-UGC NET/JRF Paper 1 2021 - Set 1

43. ________ messages have more or less same meaning involves creating change, not maintaining what normally
for the audience. happens. In addition, the changes sought are not dictated
(a) Abstract (b) Complex by leaders, but reflect purposes that leaders and followers
(c) Connotative (d) Denotative share. Moreover, change is toward an outcome that the
leaders and follower both want; a desired future or shared
44. Given below are two statements
purpose that motivates them toward this more preferable
Statement I: Truth and Falsehood are predicates of outcome. An important aspect of leadership is influencing
Arguments Statement II: Validity and Invalidity are others to come together around a common vision. Thus,
predicates of Statements leadership involves the influence of people to bring about
In light of the above statements, choose the most change toward a desirable future.
appropriate answer from the options given below: 46. What is the main theme of this passage?
(a) Both Statement I and Statement II are correct (a) Argument about leadership
(b) Both Statement I and Statement II are incorrect (b) Explanation of leadership construct
(c) Statement I is correct but Statement II is incorrect (c) Models of leadership
(d) Statement I is incorrect but Statement II is correct (d) Pros and cons of leadership
45. Which of the following state has maximum installed 47. The concept of leadership discussed in this passage is
wind power as per MNRE’s latest report? (a) Exceptional (b) Horizontal
(a) Andhra Pradesh (b) Karnataka (c) Individualistic (d) Verticle-topdown
(c) Maharashtra (d) Tamil Nadu
48. “Leadership studies are an emerging discipline”,
Read the passage carefully and answer the questions means:
46-50: (a) Leadership studies are a valid field of research
Leadership studies are an emergent discipline, and the (b) Leadership studies are an established field
concept of leadership will continue to change. Leadership (c) Leadership studies is an elaborate field
lore is defined as an influence relationship among lead- (d) Leadership studies is an evolving domain of
ers and followers who want real change and outcomes scholarship
that reflect their shared purpose. Leadership involves
influence. It occurs among people; those people who 49. What should be the outcome of leadership actions?
intentionally desire significant changes, and the changes (a) Movement towards common vision of the leader
reflect purpose shared by the leaders and their followers. and the followers
Influence means that the relationship among people (b) Movement towards the vision of the leader
is not passive; however, also part of this definition is that (c) Movement towards the vision of the majority
influence is multidirectional. It does not use orders or (d) Strengthening of the authority of leaders
threats to make somebody do something. The basic cul- 50. What variables are used for defining leadership?
tural values in America make it easiest to think of leader- (a) Power to affect relationship and desire for change
ship as something a leader does to a follower. However, in the true sense of the team
leadership has an effect in both directions. In most orga- (b) Power to change and control
nizations, superiors influence subordinates, but subordi- (c) Power to effect and desire for mandated change
nates also influence superiors. The people involved in the
(d) Power to get things done quickly by influence
relationship want real and important changes- leadership

Answer Keys
1. (a) 2. (d) 3. (b) 4. (d) 5. (b) 6. (b) 7. (b) 8. (b) 9. (c) 10. (b)

11. (b) 12. (c) 13. (c) 14. (a) 15. (c) 16. (d) 17. (d) 18. (c) 19. (d) 20. (b)

21. (c) 22. (c) 23. (d) 24. (b) 25. (a) 26. (a, b, d) 27. (b) 28. (d) 29. (c) 30. (d)

31. (b) 32. (a) 33. (d) 34. (a) 35. (a) 36. (d) 37. (a) 38. (a) 39. (b) 40. (b)

41. (c) 42. (b) 43. (c) 44. (b) 45. (d) 46. (b) 47. (b) 48. (d) 49. (a) 50. (a)

* There may be aberrations in 1-2 questions due to wrong framing of questions, being too subjective or they are data oriented but dates or years are
not mentioned.

Z04_MADAN 07_65901_2021 paper.indd 28 23/12/22 5:13 PM


NTA-UGC NET/JRF Paper 1 2021 - Set 1 Z.29

Hints and Solutions


Questions 1 to 5 are from Data Interpretation The total imports during different years are:

1. (a) Imports during 2017 = 25 + 28 + 24 + 22 + 26 =

125
Exports by country S during 2017–2020 = 28 + 28 +
21 + 32 = 109 Imports during 2018 = 24 + 26 + 28 + 18 + 16 =

Exports by country T during 2017–2020 = 31 + 33 + 112
23 + 28 = 115 Imports during 2019 = 28 + 29 + 32 + 16 + 12 =

Thus, required ratio of exports between S and T = 117
109 / 115 Imports during 2020 = 22 + 26 + 28 + 23 + 21 =
2. (d) 120
As per question, the profit percentage increase by dif-
Thus, the minimum average imports were in the year
ferent countries between 2019 and 2020 is to be cal- 2018. Thus, the answer is (b).
culated first. 4. (d)
For Country P As every figure is to be divided by 5 (number of years)
to calculate average exports, we may get the answer
Profit in 2019 = 36 – 28 = 8
directly by total as well, and thus save time.
Profit in 2020 = 32 – 22 = 10
The total exports during different years are:

Percentage increase = (10 – 8) / 8 × 100 = 2 / 8 × 100
Exports during 2017 = 30 + 26 + 32 + 28 + 31 =

= 25%
147
For Country Q
Exports during 2018 = 32 + 31 + 25 + 28 + 33 =

Profit in 2019 = 37 – 29 = 8 149
Profit in 2020 = 34 – 26 = 8 Export during 2019 = 36 + 37 + 36 + 21 + 23 = 153

Percentage increase = (8– 8) / 8 × 100 = 0 / 8 × 100 Export during 2020 = 32 + 34 + 34 + 32 + 28 = 160

= 0%
The maximum average exports were in the year 2020.

For Country R
Thus, the answer is (d).
Profit in 2019 = 36 – 32 = 4
5. (b)
Profit in 2020 = 34 – 28 = 6
The average exports by country P = 30 + 32 + 36 +
Percentage increase = (6 – 4) / 4 × 100 = 2 / 4 × 100
32 = 130 / 4 = 32.5
= 50% The average imports by country P = 25 + 24 + 28 +
As per question, the profit percentage increase by dif-
22 = 99 / 4 = 24.75
ferent countries between 2019 and 2020
Difference = 32.5 – 24.75 = 7.75
For Country S 6. (b) ICT
Profit in 2019 = 21 – 16 = 5
ASCII Code:
Profit in 2020 = 32 – 23 = 9
Computers work in binary code. Information is coded
Percentage increase = (9 – 5) /5 × 100 = 4 / 5 × 100
using 0s and 1s. Each 0 or 1 is called a bit. In the early
= 80% years of computer development, different computer
For Country T companies applied the binary system in their own
way.
Profit in 2019 = 23 – 12 = 11
Eventually, a set of standards was developed. Com-
Profit in 2020 = 28 – 21 = 7 puter manufactures agreed to use one code called
Percentage increase = (7 – 11) / 11 × 100 = Reduction the ASCII (American Standard Code for Information
Thus, country S has shown the maximum increase in profit. Interchange). ASCII is an 8-bit code. That is, it uses
3. (b) eight bits to represent a letter or a punctuation mark.
Eight bits are called a byte. A binary code with eight
As every figure is to be divided by 5 (number of years) digits, such as 1101 11012, can be stored in one byte
to calculate average imports, we may get the answer of computer memory.
directly by total as well, and thus save time.

Z04_MADAN 07_65901_2021 paper.indd 29 23/12/22 5:13 PM


Z.30 NTA-UGC NET/JRF Paper 1 2021 - Set 1

7. (b) Logical Reasoning Much time is spent on exploring the problem so


The knowledge through has been classified into that better insights are built for the future. It usually
doesn’t lead to a conclusive result. It starts on a gen-
1. svartha – for oneself – one’s own inferential knowl-
eral idea and we try to specify the related issues with
edge, through linking it with major and minor premises.
the topic of the research.
2. parartha – through justification of middle term –
Exploratory research is inexpensive, highly interactive
inferential knowledge for another.
and open–ended in nature. There is usually no prior
3. Pararthanumana – If the motive is for demon-
relevant information available from past researchers.
strating the truth of the conclusion to others it is called Since there is no standard for carrying out explor-
as Pararthanumana. Here, conclusion is arrived at atory research, it is usually flexible and scattered.
through justification of the middle term that leads to 9. (c) Logical Reasoning
it. A man having inferred the existence of fire in a hill
This question pertains to irregular behaviour of ‘Mid-
lays it down as a thesis and proves it as a conclusion
dle Term’. We can solve other questions, linked directly
following from the major and minor premises and
with Indian philosophy as in the following manner.
their combination into a third premise. The latter has
a five membered syllogism viz., pratijna, hetu, udaha- The fallacies in Anumana that can be termed as hetv-

rana, upanaya and nigamana. abhasha may occur due to the following:
1st member – Pratijna (preposition) — the hill is fiery
(i) Asiddha: This fallacy occurs due to unproved

hetu. It is also called as Paksadharmata.
2nd member – Hetu (reason) — because of smoke

(a) Ashrayasiddha: If Paksha (minor term) itself is

3rd member – Udaharana (example) — wherever

unreal, then there cannot be locus of the hetu. For
smoke exists, fire also exists as in kitchen.
example, The sky–lotus is fragrant, because it is a
4th member – Upanaya (application / subsumptive
lotus like any other lotus.
correlative) — there is smoke on the hill.
(b) Svarupasiddha: It is not possible that Hetu exist in

5th member – Nigamana (conclusion) — Hence, the
paksa. For example, sound is a quality, because it is visible.
hill is fiery
(c) Vyapyatvasiddha: This is termed as ‘conditional

Apart from above five, the few of the old Naiyayikas
hetu’. For example, ‘wherever there is fire, there is
speaks of other five members of syllogism: smoke’. The presence of smoke is because of wet fuel.
• Jijnasa — on the desire to know the truth.
(ii) Savyabhichara: This is termed as the fallacy of

• Samsaya — doubt about real nature of a thing.
irregular hetu.
• Shakyaprapti — the capacity of the pramanas to
(a) Sadharana: Here, the hetu is considered too

lead to true knowledge. wide. It is present in both sapaksa and vipaksa. ‘The
• Prayojana — the purpose of making an inference
hill has fire because it is knowable’.
• Samshaya vyudasa — the removal of all doubts the
(b) Asadharana: Here, the hetu is considered too

truth of an inference. narrow. It is present only in the Paksha. It is not pres-
ent in the Sapaksa and in the Vipaksha. For example,
Though Vatsyayana disagreed with the first five and

‘sound is eternal because it is audible’.
stated them as irrelevant.

(c) Anupasamhari:
8. (b) Research Aptitude
(iii) Satpratipaksa: In this case, one hetu is contra-

Statement II is the solution for Statement I. For this dicted by another hetu. In case, both have equal force,
purpose, we conduct exploratory study that is more then nothing should follow. For example, ‘sound
useful in qualitative research. is eternal, because it is audible’, and ‘sound is non–
In Statement II, there is list of steps in exploratory study. eternal, because it is produced’. The word, ‘audible’
Research is a continuous process that needs improve-
is counterbalanced by ‘produced’. Here, ‘audible’ and
ment with the passage of time, thus it is non– ‘produced’ carry the equal force.
exhaustive. Exploratory research mostly deals with (iv) Badhita: Here, the another proof (as defined by

qualitative data. Most of the researchers work on perception) definitely contradicts and disproves the
existing theories or formulations and build on them. middle term (hetu). For example, ‘fire is cold because
Some researchers are motivated to work on a problem it is a substance’.
that has not been studied very clearly to establish pri-
orities, develop operational definitions and improv-
ing the final research design.

Z04_MADAN 07_65901_2021 paper.indd 30 23/12/22 5:13 PM


NTA-UGC NET/JRF Paper 1 2021 - Set 1 Z.31

(v) Viruddha: Here, the virudha, instead of proving ‘UN Decade on Ecosystem Restoration 2021–2030’.
something it is proving the opposite. For example, A step further, UNEP can be further linked with
‘sound is eternal because it is produced’. UN Framework Convention on Climate Change
10. (b) Higher Education System (UNFCCC), Conference of Parties (COPs), Kyoto Pro-
New Education Policy, 2020 proposes a single regula- tocol, Paris Agreement and COP26 Glasgow meeting
tor for higher education institutions across the coun- asking for Zero Emissions. World seems to have set
try, that will be called as Higher Education Council of Year 2050 as the target year for Zero Emissions and
India (HECI). It will have four verticals for its various India has set Year 2070 for its own emissions during
roles. Glasgow COP26 Meeting.
(i) National Higher Education Regulatory Coun- 12. (c) Research Aptitude
cil (NHERC): It will function as the common, single Reflexivity: Research should be classified as a “Mis-
point regulator for the higher education sector includ- sion’, the researchers get in this mission and their own
ing teacher education. It however excludes medical personalities become part of it. Conducting qualitative
and legal education. research, such as fieldwork, changes a researcher in
(ii) National Accreditation Council (NAC) – a many ways. Through reflexivity, researchers acknowl-
‘meta–accrediting body’): It will be based primarily on edge the changes brought about in themselves as a
basic norms, public self–disclosure, good governance, result of the research process and how these changes
and outcomes. It will be carried out by independent have affected the research process. Thus, this reflexiv-
accrediting institutions supervised and overseen by ity is an iterative and empowering process. Reflexiv-
NAC. ity should be recognized as a significant part of the
research findings. Thus, it is an analytic attention to
(iii) Higher Education Grants Council (HEGC): It
the researcher’s role in qualitative research”, it is both
will look into funding and financing of colleges and
a concept and a process. As a concept, it refers to a
universities.
certain level of consciousness.
(iv) General Education Council (GEC): It will frame
There are the few basic principles in the linked quali-
expected learning outcomes for higher education
tative research:
programmes, also referred to as ‘graduate attributes’.
GEC will formulate a National Higher Education (i) Feminist principle
Qualification Framework (NHEQF). (ii) Rights–based research: the respect of human
11. (b) People, Development and Environment rights of participants during the research process.
Here, we have taken a broad view to deal with future (iii) Participatory research principles: It recognizes
questions. Since 1972, United Nations Environment that people have expert knowledge and deep insight
Programme (UNEP) has been the global authority into their own lives and communities.
to set the environmental agenda, the concept of ‘sus- 13. (c) Communication / Teaching Aptitude
tainable development’ for future generations and an This question is linked with feedback in the classroom
authoritative advocate for the global environment. to make it effective communication. A teacher should
UNEP is headquartered in Nairobi, Kenya. It has 193 be proficient in all four modes of communication – lis-
Member States and representatives from civil society, tening, speaking, reading, and writing – and should
businesses, etc. through the UN Environment Assem- know how to utilize this proficiency effectively in an
bly, the world’s highest–level decision–making body educational institute environment. Being able to do
on the environment. this has been proven to impact the success students
UNEP’s Medium–Term Strategy (MTS) articulates its achieve in their academic lives, as well as the teach-
role in delivering the promises of the ‘2030 Agenda er’s own career success.
for Sustainable Development and the United Nations 14. (a) Logical Reasoning
Conference on Sustainable Development (Rio+20)
If ‘all men are mortal’ is true, then ….
as well as its outcome document, “The Future We
Want.” To deal with such questions, we need to take help
from ‘square of opposition’.
Through its campaigns, particularly ‘World Environ-
ment Day’ (June 5), UNEP raises awareness and advo- Contradictories: Contradictory propositions cannot
cates for effective environmental action. The Day was be simultaneously true and false. If one is true, the
established in 1972 by the UN at ‘Stockholm Con- other is necessarily false and vice versa. That is, if ‘A’
ference on the Human Environment’ (5–16 June, is true, then ‘O’ is false; ‘E’ is false.
1972). This day is connected with the launch of

Z04_MADAN 07_65901_2021 paper.indd 31 23/12/22 5:13 PM


Z.32 NTA-UGC NET/JRF Paper 1 2021 - Set 1

Every S is P No S is P 4. e–ShodhSindhu – Consortium for Higher Educa-


A Contraries E tion e–Resources
5. ShodhShuddhi – Provides Plagiarism Detection
Software to Institutions
Subalterns Contradictories Subalterns
6. IRINS – Research Information Management Sys-
tem for Institutions
7. Vidya–Mitra – Integrated e–Content Portal
8. Vidwan – Expert Database & National Researcher
I Subcontraries O Network
Some S is P Some S is not P
9. e–PG Pathshala – an initiative of the Ministry of
Contrary: When two universal propositions differ Education under its National Mission on Education
only in quality. By definition, both opposite propo- through ICT (NME–ICT) being executed by UGC.
sitions can be both false and true at the same time.
10. e–Pathya: It is one the verticals of e–PG Pathshala
If one of them is true, the other must necessarily be
which is software driven course / content package
false. If one is false, the other may be true or false.
that facilitates students pursuing higher education
That is, if ‘E’ is true, then ‘A’ is true; ‘I’ is false
(PG level) in distance learning as well as campus
Sub–contraries: Contrasting propositions can be learning mode. it also facilitates offline access.
true simultaneously, but they cannot be false at the
11. e–Adhyayan (e–Books): e–Adhyayan is a plat-
same time. If one of them is true, the other may be
form to provide 700+ e–Books for the Post–Gradu-
true or false, but if one of them is false, the other must
ate Courses. All the e–Books are derived from e–PG
be true. That is, if ‘I’ is true, then ‘E’ is true; ‘O’ is true.
Pathshala courses. It also facilitates play–list of video
Sub – alternation: When two prepositions differ only content.
in quantity (one is universal and the other is special),
12. UGC–MOOCs: It is one of vertical to produce
such an opposition is said to be sub – alternation.
course on Post Graduate subjects in SWAYAM (Online
These propositions may be simultaneously true or
Courses, a Ministry of Education initiative). UGC
simultaneously false. That is, if ‘O’ is true, then ‘E’ is
is one of the national coordinator of SWAYAM, and
false; ‘I’ is true
INFLIBNET is technical partner for UGC–MOOCs.
Thus, if ‘all men are mortal’ is true, then we can validly
16. (d) Information and Communication Technology (ICT)
infer that No men are mortal is false’(Contrary). Some
men are mortal is true (Sub – alteration) and ‘Some Routers are networking devices. They are responsible
men are not mortal’ (contradictory proposition). for receiving, analysing, and forwarding data pack-
ets among the connected computer networks. On the
15. (c) Information and Communication Technology (ICT)
arrival of a data packet, the router inspects the des-
INFLIBNET – It is abbreviation of Information and tination address, consults its routing tables to decide
Library Network Centre. It is an Autonomous Inter– the optimal route and then transfers the packet along
University Centre (IUC) of UGC. It is located in this route.
Gandhinagar, Gujarat. It is a major National Pro-
A router can be used both in LANs (Local Area Net-
gramme initiated by the UGC in March 1991 as a
works) and WANs (Wide Area Networks). Routers are
project under the IUCAA, it became an independent
more expensive than other networking devices like
Inter–University Centre in June 1996. INFLIBNET is
hubs,bridges and switches. Routers are manufactured
involved in modernizing university libraries in India
by some popular companies such as Cisco, D–Link,
using the state–of–art technologies for the optimum
HP, Nortel etc.
utilisation of information. INFLIBNET is set out to be
a major player in promoting scholarly communication A router is a layer 3 or network layer device in OSI
among academicians and researchers in India. Model while data link layer works in the layer 2 of OSI
Model. Data link layer is responsible for maintaining
The following institutions and projects are linked
the data link between two hosts or nodes.
with it.
17. (d) Research Aptitude
1. Shodhganga – A Reservoir of Indian Theses
Survey Research
2. INDCAT – Online Union Catalogue of Indian
Universities Survey research a research method involving the use
of standardized questionnaires or interviews to collect
3. SOUL – State–of–art Integrated Library Manage-
data about people and their preferences, thoughts,
ment System

Z04_MADAN 07_65901_2021 paper.indd 32 23/12/22 5:13 PM


NTA-UGC NET/JRF Paper 1 2021 - Set 1 Z.33

and behaviors in a systematic manner. As surveys are (i) Dichotomous response – responses such as true/

used to collect data from a predefined subject(s), they false, yes/no, or agree/disagree. Is MSP required for
have been counted under ‘positivism’ approach. crops – Yes / No
The survey as a formal research method was pio-
(ii) Nominal response – more than two unordered

neered in the 1930–40s by sociologist Paul Lazarsfeld options. What is your industry of employment: manu-
to examine the effects of the radio on political opin- facturing / supply chain / consumer services / retail /
ion formation of the United States. This method has education.
since become a very popular method for quantitative (iii) Ordinal response – more than two ordered

research in the social sciences. options. What is your highest level of education: high
The survey method can be used for descriptive,
school / college degree / graduate studies/ PG / PhD.
exploratory, or explanatory research. (iv) Interval–level response – When respondents

Survey research has several inherent strengths com-
are presented with a 5–point or 7–point Likert scale,
pared to other research methods due to following semantic differential scale, or Guttman scale etc.
reasons. (v) Continuous response – When respondents enter

(i) Survey is an excellent vehicle to measure a wide
a continuous (ratio–scaled) value with a meaning-
variety of data – people’s preferences (politi- ful zero point, such as their age or tenure in a firm.
cal orientation), traits (self–esteem), attitudes These responses generally tend to be of the fill–in–the
(toward immigrants), beliefs (about a new law), blanks type.
behaviors (smoking), or factual information The survey questions should be stated in a very sim-

(income). ple language, preferably in active voice, and without
(ii) It is ideally suited for remotely collecting data
complicated words or jargon so that they are clearly
about a population. For example, a large nation understandable by a typical respondent. Questions
such as India or its state UP, MP, Maharashtra must not be biased and too much detailed.
that can be covered using mail–in, electronic Double–barreled questions are those that can have

mail, or telephone surveys. Researches can use multiple answers. For example, are you satisfied with
well planned sampling to ensure that the popula- the hardware and software provided for your work?
tion is adequately represented in a small sample.
Question should not be presumptuous such as on

(iii) Interviews may be the only way of reaching cer-
taxes – too high or too low. Avoid questions with
tain population groups such as the homeless or built–in presumptions.
illegal immigrants for which there is no sampling
‘Question sequencing’ should be proper, such as

frame available.
questions should flow logically from one to the next.
(iv) 
Survey research is economical in terms of
Role of interviewer should be motivational for respon-

researcher time, effort and cost than most other
dents. They should make a good questionnaire.
methods such as experimental research and case
research. Just pausing and waiting (without going into the

next question) may suggest to respondents that the
At the same time, survey research also has some

interviewer is waiting for more detailed response. It is
unique disadvantages. It is subject to a large number
called as ‘silent probe’.
of biases such as non–response bias, sampling bias,
social desirability bias, and recall bias, etc. The interviewer can try the psychotherapist’s trick

of repeating what the respondent said is called as
Depending on data collection techniques, survey

‘Reflection’.
research can be divided into two broad categories:
Five common biases in survey research are non–

(i) Questionnaire surveys – mail–in, group–admin-

response bias, sampling bias (excluding some peo-
istered, or online surveys), and
ple who don’t have some resources such as mobile
(ii) Interview surveys – personal, telephone, or
phone), social desirability bias (avoid negative opin-
focus group interviews (These have been dis- ions), recall bias (subjects’ motivation, memory, and
cussed in detail in second unit of Pearson title for ability to respond). Common method bias refers to
NTA–NET exam). the amount of spurious covariance shared between
Constructing a survey questionnaire (or interview-
independent and dependent variables that are mea-
ing) is an art. For the benefit of research students, sured at the same point in time such as in a cross–
the following responses to structured questions have sectional survey, using the same instrument, such as
been suggested. a questionnaire.

Z04_MADAN 07_65901_2021 paper.indd 33 23/12/22 5:13 PM


Z.34 NTA-UGC NET/JRF Paper 1 2021 - Set 1

18. (c) People, Development and Environment that message will be perceived. Modern electronic
Sustainable Development Goals (SDGs) communications (including radio, television, films,
and computers) would have far–reaching sociologi-
SDGs, also known as the Global Goals, were adopted
cal, aesthetic, and philosophical consequences, to the
by the United Nations in 2015 as a universal call to
point of actually altering the ways in which we experi-
action to end poverty, protect the planet, and ensure
ence the world.
that by 2030 all people enjoy peace and prosperity.
21. (c) Mathematical Reasoning and Aptitude
The 17 SDGs are integrated—they recognize that
action in one area will affect outcomes in others, and We can solve this question, by counting all the items
that development must balance social, economic and with the difference d (i.e 175 – 168 = 182 – 175)
environmental sustainability. Or by using the following formula of Arithmetic
Progression
GOAL 1: NO POVERTY
an = 266, a = 168 and d = 7
GOAL 2: ZERO HUNGER
Formula: an = a + (n – 1)d
GOAL 3: GOOD HEALTH AND WELL–BEING
266 = 168 + (n–1)7
GOAL 4: QUALITY EDUCATION n–1 = (266 – 168) / 7 = 98 / 7 = 14
GOAL 5: GENDER EQUALITY n = 14 + 1 = 15
GOAL 6: CLEAN WATER AND SANITATION Thus, (c) is the correct answer.
GOAL 7: AFFORDABLE AND CLEAN ENERGY 22. (c) Mathematical Reasoning and Aptitude
GOAL 8: DECENT WORK AND ECONOMIC GROWTH The highest denominator in options is 80. We will
GOAL 9: INDUSTRY, INNOVATION AND make every fraction equal to 80.
INFRASTRUCTURE 11/16 × 5/5 = 55/80
GOAL 10: REDUCED INEQUALITIES 29/40 × 2/2 = 58/80
GOAL 11: SUSTAINABLE CITIES AND COMMUNITIES 5/8 = 5/8 × 10/10 = 50 / 80
GOAL 12: RESPONSIBLE CONSUMPTION AND 65/80 – no change required.
PRODUCTION
Now every fraction has 80 as denominator, we need to
GOAL 13: CLIMATE ACTION compare only numerators to compare.
GOAL 14: LIFE BELOW WATER Thus 5/8 is the smallest fraction.
GOAL 15: LIFE ON LAND 23. (d) Research Aptitude
GOAL 16: PEACE, JUSTICE AND STRONG Non–parametric (and Parametric tests): First, we
INSTITUTIONS need to look at some basic difference between para-
GOAL 17: PARTNERSHIPS FOR THE GOALS metric and non–parametric tests.
In parametric tests, the parameters are assumed and
19. (d) Higher Education System the population distribution is always known. To cal-
Global Citizenship Education: Topics and learning culate the central tendency, a mean value is used.
objectives, should be counted as the first pedagogical In such tests, we assume normal distribution curve
guidance from UNESCO on global citizenship educa- (bell shaped curve). For example, if we were to mea-
tion. With such education, young people are able to sure the height of people (in a population), we take
solve problems, make decisions, think critically, com- some sample, we would see a typical bell–shaped
municate ideas effectively and work well with others. curve. This distribution is also called the Gaussian
This not only helps them personally and education- distribution.
ally, but eventually professionally as well. Thus, a In statistics, nonparametric tests are methods of sta-
global education in the classroom is paramount. tistical analysis that do not require a distribution to
20. (b) Communication meet the required assumptions to be analyzed (espe-
‘The medium is the message’ is an expression coined cially if the data is not normally distributed). Due
by Canadian educator Marshall McLuhan that is often to this reason, they are sometimes referred to as
interpreted to mean that the “media” used to commu- distribution–free tests.
nicate information have a significant impact on the Parametric tests are generally more powerful than the
messages they deliver. The form of a message (print, non–parametric tests, where smaller sample sizes are
visual, musical, etc.) determines the ways in which required in parametric tests.

Z04_MADAN 07_65901_2021 paper.indd 34 23/12/22 5:13 PM


NTA-UGC NET/JRF Paper 1 2021 - Set 1 Z.35

Types Of Non–Parametric Tests Intrapersonal communication: it is a communica-


• 1 Sample Sign Test tion within an individual almost all the times, such as
talking to oneself, listening to oneself and relating to
• 1 Sample Wilcoxon Signed Rank Test
oneself. It includes individual reflection, meditation,
• Friedman Test contemplation and praying to God.
• Goodman Kruska’s Gamma Upward communication: This communication is the
• Kruskal–Wallis Test process of information flowing from the lower levels
• The Mann–Kendall Trend Test of a hierarchy to the upper levels. It is the process by
• Mann–Whitney Test which lower–level company employees can directly
communicate with upper management to provide
• Mood’s Median Test feedback, complaints or suggestions regarding the
• Spearman Rank Correlation day–to–day operations of the company.
24. (b) Logical Reasoning 26. (a, b, d) People, Development and Environment
Exclusive premise: The fallacy of exclusive premises Cerebrum: Cerebrum, the largest and uppermost

is a syllogistic fallacy committed in a categorical syl- portion of the brain. The cerebrum consists of the
logism that is invalid because both of its premises are cerebral hemispheres and accounts for two–thirds of
negative. the total weight of the brain.
Illicit major: Illicit major is a formal fallacy commit-
Hippocampus: It is a small, curved formation in the

ted in a categorical syllogism that is invalid because
brain that plays an important role in the limbic sys-
its major term is undistributed in the major premise
tem. It is involved in the formation of new memories
but distributed in the conclusion.
and is also associated with learning and emotions.
Premise Type Distributed Pons: It is a portion of the hindbrain that connects the

Some birds are ‘O’ type Only predicate cerebral cortex with the medulla oblongata. It also
not beautiful (Particular serves as a communications and coordination centre
creatures. negative) between the two hemispheres of the brain.
Therefore, no dogs E type (Universal Both subject Thalamus: The thalamus functions as a relay

are birds. negative) and predicate
station in which sensory pathways of the spinal cord
Thus, in premises 1 some birds (major term / subject) and brainstem form synapses on their way to the cere-
are undistributed. In the conclusion dogs (subject) bral cortex. Specific locations in the thalamus are
and the birds (predicate) both are distributed. related to specific areas on the body surface and in the
Undistributed middle: The fallacy of the undistrib- cerebral cortex.
uted middle is a formal fallacy that is committed 27. (b) Higher Education System
when the middle term in a categorical syllogism is not
distributed in either the minor premise or the major Indian Institute of Advance Study (IIAS): This

premise. It is thus a syllogistic fallacy. research institute is based in Shimla. It was set up by
the Ministry of Education in 1964 and it started func-
Illicit minor: the fallacy of illicit minor term is undis- tioning in 1965.
tributed in the premise but is distributed in the con-
clusion (but not vice – versa).
Indian Biological Science and Research Institute
(IBRI): It is located in Noida, UP. IBRI is run by the
25. (a) Communication
academicians from different technological fields
Diagonal communication: The communication such as Pharmaceutics, Biotechnology, Bioinformat-
among the persons at different levels who have no ics, Information Technology and Mathematics. The
direct relationship is called diagonal relationship. mandate of IBRI is to undertake research and training
This communication is effective as hierarchical bind- from basics to industrial level in biosciences domains.
ings are removed and there is a free flow of informa-
Indian Institute of Soil Science (IISS): This i­ nstitute

tion, cutting across different positions.
is linked with ICAR (Indian Council of Agriculture
Horizontal communication: communication with Research). ICAR–IISS was established in 1988 at
people at the same level in a hierarchy of peers and ­Bhopal with a mandate of “Enhancing Soil Productiv-
colleagues is termed as horizontal communication. ity with Minimum Environmental Degradation”. IISS
This may combine both formal and informal has emerged as a leader in basic and strategic research
communication. on soils in the country.

Z04_MADAN 07_65901_2021 paper.indd 35 23/12/22 5:13 PM


Z.36 NTA-UGC NET/JRF Paper 1 2021 - Set 1

Indian Institute of Sugarcane Research (IISR): The Post Office Protocol provides access via an Inter-
IISR was established in Lucknow in 1952. It deals with net Protocol (IP) network for a user client application
basic issues linked with sugarcane. In 1954, its work- to a mailbox (maildrop) maintained on a mail server.
ing was taken care of by Government, then in 1969, it Thus, the protocol used when receiving emails from
was transferred to ICAR, New Delhi alongwith other the email server.
central agricultural research institutes. 34. (a) Higher Education System
28. (d) Mathematical Reasoning and Aptitude CBCS stands for the choice–based credit system.
Let number of 20 paise coins = X CBCS grading systems surpass the ‘conventional per-
Then number of 25 paise coins = 324 – X centage’ and grading means to evaluate a student’s
Total value = X (20/100) + (324 –X) 25/100 = ` 71 academic skills. CBCS follows a more standardised
X / 5 + (324 – X)/4 = ` 71 way of assessment and examination management.
X = 200 CBCS software grading system is uniform across all
Thus, the number of 20 paise coins is 200. educational institutes. It follows the semester pattern
and not the annual examination pattern.
29. (c) Higher Education system
UGC also follows a 10–point grading system, with 0
This question is linked with good governance, so
being absent/fail and 10 being outstanding.
linked with public. Data (statistics) and ICT are also
going to be part of it. There seems to be some aberra- Evaluation in choice Based Credit System in
tion in statement II. Higher Education
30. (d) Information and Communication Technology Letter Grades Grade Points
This is a direct question of matching.
O (Outstanding) 10
31. (b) Communication
A+ (Excellent) 9
Technological barrier: low bandwidth of internet is
A (Very Good) 8
example of technological barrier.
B+ (Good) 7
Psychological barrier: Complexity of ideas and per-
sonal attitude towards technology is an example psy- B (Above Average) 6
chological barrier. C (Average) 5
Geographical barriers: physical locations of com- P (Pass) 4
municators is a geographical barrier. F (Fail) or Absent (AB) 0
Semantic barrier: communication barriers related to
35. (a) Teaching Aptitude/ ICT
language and the meaning of the words.
DRUPAL: Drupal is a free software package that
32. (a) Mathematical Reasoning and Aptitude
allows an individual or a community of users to easily
Statement I publish, manage and organize a wide variety of con-
Amount = 280 (1+10/100)(1+5/100) tent on a website.
= 280(11/10)(21/20) = ` 323.4 Drupal is used by mooKIT platform for open source
Compound Interest = Amount – Principal content management system.
JOOMLA: This uses to build, organize, manage and
= 323.4 – 280 = ` 43.4
publish content for websites, blogs, Intranets and
The given answer is ` 44.3. mobile applications.
Statement II Moodle: Moodle is a free and open–source learning
This question is based on simple interest, so interest management system (LMS) written in PHP and dis-
will be ` 5.6 on ` 100 every annum. In 15 years, total tributed under GNU (General Public licence). Moo-
SI will be 15 × 5.6 = ` 84. Thus, amount will not be dle is used for blended learning, distance education,
doubled, but much less than that. flipped classroom and other e–learning projects in
Thus, both statements are not correct. schools, universities, workplaces and other sectors
33. (d) Information and Communication Technology SAKAI: Sakai is a free, community source, educa-
tional software platform designed to support teach-
In computing, the Post Office Protocol is an applica- ing, research and collaboration. Systems of this type
tion–layer Internet standard protocol used by e–mail are also known as Course Management Systems
clients to retrieve e–mail from a mail server. POP ver- (CMS), Learning Management Systems (LMS), or Vir-
sion 3 is the version in common use. tual Learning Environments (VLE).

Z04_MADAN 07_65901_2021 paper.indd 36 23/12/22 5:13 PM


NTA-UGC NET/JRF Paper 1 2021 - Set 1 Z.37

36. (d) Information and Communication Technology problems depends on having an understanding of
File Transfer Protocol (FTP): FTP is a standard com- three basic aspects of reasoning: identity, compensa-
munication protocol used for the transfer of computer tion and reversibility.
files from a server to a client on a computer network. Jean Piaget mentioned the following stages for cogni-
FTP is built on a client–server model architecture tive development:
using separate control and data connections between (i) Sensory Period (0–2 years)
the client and the server. FTP users may authenticate (ii) Pre – operational period (2–7 years)
themselves with a clear–text sign–in protocol, nor-
(iii) Concrete operational period (7 – 11 years)
mally in the form of a username and password. FTP
is often secured with SSL/TLS (FTPS) or replaced (iv) Formal operation period (11 – 15 years)
with SSH File Transfer Protocol (SFTP) for secure The stage of classification depends on students’ abil-
transmission. ity to focus on a single characteristic of objects in a set
37. (a) People, Development and Environment and group of objects according to that characteristic.
Thus, statement II is incorrect.
According to Urie Bronfenbrenner’s biological model
of human development, microsystem constitutes 42. (b) Research Aptitude
immediate family, schools and neighbourhood. Descriptive Hypothesis: This contains only one
variable so it is called a univariate hypothesis. This
38. (a) People, Development and Environment
typically states the existence, size, form or distribu-
Statement I: small amount of atmospheric Nitrogen tion of some variable. It describes the situation or col-
dioxide is produced from polluting sources. lects the data, the same as it has happened without
NO₂ is an intermediate in the industrial synthesis of adding our own assumptions as a researcher.
nitric acid. It is largely used in the production of fer- Null Hypothesis: the null hypothesis (often denoted
tilisers also. At higher temperatures it is a reddish– H₀) is that two possibilities are the same. The null
brown gas. It can be fatal if inhaled in large quantity. hypothesis is that the observed difference is due to
Statement II: Most of the atmospheric NO2 is chance alone. Using statistical tests, it is possible to cal-
produced due to photochemical reactions in the culate the likelihood that the null hypothesis is true.
atmosphere. Explanatory Hypothesis: This hypothesis guides the
Nitrogen dioxide, NO2, present in air is known to be cause and effect relationship between two variables.
photochemically active due to the presence of an For example, when salaries increase, then spending
unpaired electron. NO2 molecule reaches a higher on food items also increase.
level of energy when it absorbs a photon of light with Intervening and confounding variables add to the
energy, which converts the ground state molecule to impact of independent variables on dependent
electronically excited state molecule. NO2, Ozone and variables.
PAN show photochemical reactions and photochemi- 43. (c) Logical Reasoning
cal smog is formed.
The denotative meaning of a word is simply the
39.(b) Mathematical Reasoning and Aptitude commonly accepted meaning or the definition that
If each averaged number, X, is multiplied by an you would find if you were to look up that word in a
another number, Y, then new average will be multipli- dictionary.
cation of those numbers, that is X.Y. Thus, answer to Typically, the connotative meaning of a word has
this question will be 7×11 = 77. more of an emotional association and is more likely
40. (b) People, Development and Environment to trigger an emotional response than the denotative
Diarrhoea is a type of waterborne disease which is meaning.
caused by virus, bacteria and protozoa. The main 44. (b) Logical Reasoning
symptoms of diarrhea are frequent loose, watery Truth and falsehood are predicates of a statement and
stools and a pressing urge to have a bowel movement. validity and invalidity are predicates of an argument.
The most common causes of acute and persistent
45. (d)
diarrhea are infections, travelers’ diarrhea, and side
effects of medicines. Many viruses cause diarrhea, 46. (b)
including norovirus link and rotavirus link. Viral gas- 47. (b)
troenteritis is a common cause of acute diarrhea. 48. (d)
41. (c) Teaching Aptitude 49. (a)
According to Piaget, the ability to solve conservation 50. (a)

Z04_MADAN 07_65901_2021 paper.indd 37 23/12/22 5:13 PM


NTA-UGC NET/JRF
Paper 1 2021 - Set 2
INSTRUCTIONS To NTA-NET ASPIRANTS
Time Allowed: 1 hour  50 × 2 = 100 Marks

1. This paper consists of Fifty (50) objective type questions of Two (2) marks each.
2. All questions are compulsory.
3. Each item has four alternative responses marked (a), (b), (c) and (d). You have to darken the circle as indicated below
on the correct response against each item.
Example: a b c d where (c) is the correct response.
4. Your responses to the items are to be indicated in the OMR Sheet given at the end of the book.
5. Read instructions provided with each question carefully.
6. There are no negative marks for incorrect answers.

Information for Question 1 to 5: 4. Find out the average number of students who quali-
Study the given table carefully and answer the questions fied in 2020 (from all states).
that follow. (a) 84550 (b) 84580
Number (N) of candidates (in lakhs) appearing for (c) 85375 (d) 86770
a University Entrance examination from four different
states (A, B, C, D) and the percentage (P) of candidates 5. Find out the difference between the total number of
clearing the same over the years are provided in the table students who qualified in 2017 and the total number
below: of students who qualified in 2018 (from all states):
(a) 31750 (b) 31880
State A B C D
(c) 32220 (d) 32340
Year N P(%) N P(%) N P(%) N P(%)
6. The characteristics of a normal distribution curve are:
2016 2.31 32 1.64 42 2.60 46 3.3 29 A. It reflects an asymmetrical distribution
2017 2.02 44 1.72 41 2.45 36 3.1 32 B. It is bell-shaped
2018 1.98 39 2.02 37 2.20 33 2.9 31 C. It does not show standard deviations
2019 1.85 28 2.10 43 2.50 42 2.7 34 D. The mean, mode and median coincide at the cen-
2020 2.20 33 1.90 46 2.55 30 3.0 35 tre of the distribution
E. The curve is based on an infinite number of
1. Find out the average number of non-qualified candi- observations
dates in 2019 (from all states).
Choose the correct answer from the options given

(a) 135320 (b) 137270
below:
(c) 139870 (d) 144025 (a) A, B and C only
2. In which year did the highest number of candidates (b) B, C and D only
clear the entrance exam from State B? (c) B, D and E only
(a) 2016 (b) 2018 (d) C, D and E only
(c) 2019 (d) 2020
7. In India, according to Niti Ayog’s SDG Index (2019-
3. Find out the total number of students who have 2020), which is the best performing state on SDG-9
cleared the entrance exam from 2016 to 2020 in State (Industry, Innovation and Infrastructure)
A. (a) Gujarat (b) Karnataka
(a) 355460 (b) 358660 (c) Kerala (d) Telangana
(c) 360890 (d) 364420

Z04_MADAN 07_65901_2021 paper.indd 38 23/12/22 5:13 PM


NTA-UGC NET/JRF Paper 1 2021 - Set 2 Z.39

8. Which of the following statements are true? In light of the above statements, choose the most

A. Simplex data transmission is in two-directions appropriate answer from the options given below
only (a) Both Statement I and Statement II are correct
B. Half-duplex data transmission is in both direc- (b) Both Statement I and Statement II are incorrect
tions but not at the same time (c) Statement I is correct but Statement II is incorrect
C. Full-duplex data transmission is in both direc- (d) Statement I is incorrect but Statement II is correct
tions simultaneously
D. Serial data transmission is when data is sent, 8 bit 12. An extraneous variable is also known as
at a time, over a single wire (a) Artifact
E. Parallel data transmission is when one bit of data (b) Contiguous factor
is sent down over a single wire (c) Dependent variable
(d) Intervener
Choose the correct answer from the options given

below: 13. Identify the correct sequence of wind energy poten-
(a) A, B and C only (b) B and C only tial at 100m above ground level in decreasing order.
(c) C and D only (d) C, D and E only A. Karnataka
B. Andhra Pradesh
9. Match List I with List II:
C. Gujarat
List I List II D. Tamil Nadu
Types of Design Characteristics E. Andaman and Nicobar Islands
A. S
 olomon Four Group I. It is a cross-sectional design Choose the correct answer from the options given

Design below
B. P
 ost-test only Control II. Contains an additional (a) A > B > C > E > D
Group Design set of control and (b) C > A > B > D > E
experimental groups
(c) C > E > B > D > A
C. One-shot Case Study III. Omits the pre-tested
(d) D > C > A > B > E
groups altogether
D. Correlational Design IV. Observation of a single
14. What is the World Wide Web (www)?
group at one point of time (a) Collection of webpages stored on websites
to observe the change (b) Hypertext processor
(c) Interconnected networks
Choose the correct answer from the options given

below: (d) Web browser
(a) A -I , B -II , C -III , D -IV 15. University of the ancient period which has been
(b) A -II , B -III , C -IV , D -I declared by UNESCO as a heritage site is:
(c) A -III , B -IV , C -I , D -II A. Mithila B. Vallabhi
(d) A -IV , B -I , C -II , D -III C. Pushpagiri D. Takshashila
E. Nalanda
10. Given below are two statements
Statement I: The number 89325324 is divisible by 66. Choose the correct answer from the options given

below:
Statement II: The set of natural numbers is also called (a) A and B only (b) B and C only
the set of whole numbers.
(c) C and D only (d) D and E only
In light of the above statements, choose the correct
16. Which of the following types of power plants provide
answer from the options given below:
electricity consistently running 24 hrs per day, 7 days
(a) Both Statement I and Statement II are false
a week?
(b) Both Statement I and Statement II are true
(a) Geothermal (b) Hydro
(c) Statement I is false but Statement II is true
(c) Solar (d) Wind
(d) Statement I is true but Statement II is false
17. Name the fallacy committed in the argument below:
11. Given below are two statements:
“All men who understand women are potentially per-
Statement I: Personal competencies determine how fect husbands.
we manage ourselves
All potentially perfect husbands are men of infinite
Statement II: Social competencies determine how we patience.
handle relationships

Z04_MADAN 07_65901_2021 paper.indd 39 23/12/22 5:13 PM


Z.40 NTA-UGC NET/JRF Paper 1 2021 - Set 2

Therefore, some men of infinite patience are men who


23. Given below are two statements
understand women.” Statement I: DVDs use a single spiral ‘track’ to store
(a) Exclusive premises data working from the centre to the edge.
(b) Existential fallacy
Statement II: DVDs are Read-only devices
(c) Illicit major
In light of the above statements, choose the correct
(d) Undistributed middle
answer from the options given below
18. Match List I with List II: (a) Both Statement I and Statement II are false
List I List II (b) Both Statement I and Statement II are true
(c) Statement I is false but Statement II is true
A. Lay down standards of legal education I. NCTE
(d) Statement I is true but Statement II is false
B. Maintaining the quality of technical II. AICTE
education 24. If the selling price of `24 results in a discount of 20%
on the list price, what selling price would result in a
C. Maintenance of standards in teacher III. NMC
education
30% discount on list price?
(a) 18 (b) 20
D. Access to quality and affordable medical IV. BCI
(c) 21 (d) 27
education
25. Pick the correct option to represent the given
Choose the correct answer from the options given
argument
below: All artists are egoists
(a) A -I , B -II , C -IV , D -III
Some artists are paupers
(b) A -II , B -III , C -IV , D -I
Therefore, some paupers are egoists
(c) A -III , B -IV , C -I , D -II
(d) A -IV , B -II , C -I , D -III Choose the correct answer from the options given

below
19. What is Ethernet? (a) A II - I figure (b) A II - II figure
(a) A collision-detection protocol (c) A II - III figure (d) A II - IV figure
(b) A protocol used by many wired LAN’s
26. Which of the following statements are correct?
(c) A protocol used for address translation
A. If A’s speed is more than B’s speed, then to travel
(d) A security protocol
the same distance, A will take more time than B
20. The satellite used for the transmission and telecast of B. Two speeds, 1500 m/min and 90 km/hr are equal
SWAYAMPRABHA is called C. By walking at a speed of 18 km/hr, the distance
(a) EDUSAT Satellite covered in 2 minutes and 30 seconds is 750
(b) GSAT-15 Satellite meters
(c) GSAT-3 Satellite
Choose the correct answer from the options given

(d) SP-EDU-15 Satellite below:
21. Which of the following are Central Universities? (a) A and B only (b) A and C only
A. Dr. Babasaheb Ambedkar Open University (c) B and C only (d) C only
B. Jamia Millia Islamia 27. Which of the following are advantages of observation
C. Panjab University as a tool of data collection?
D. Punjabi University A. Observation takes place only in natural settings
E. University of Mumbai B. Observation methods demand the active partici-
Choose the correct answer from the options given
pation of subjects in the programme
below: C. Observations are known for directness
(a) A and B only (b) B and C only D. Artificiality can be minimised in observational
(c) C and D only (d) C and E only studies
22. What is the name for the technique which uses images E. Recording devices can be used in observational
that are often more evocative than words, and more studies for accuracy
precise and potent in triggering a wide range of asso- Choose the correct answer from the options given

ciations and thinking? below:
(a) Brainstorming (b) Mind-mapping (a) A, B and C only (b) A, B and E only
(c) Story boarding (d) Synectics (c) B, C and D only (d) C, D and E only

Z04_MADAN 07_65901_2021 paper.indd 40 23/12/22 5:13 PM


NTA-UGC NET/JRF Paper 1 2021 - Set 2 Z.41

28. In agricultural regions, groundwater can have signifi- (c) Statement I is false but Statement II is true
cant concentrations of which pollutant? (d) Statement I is true but Statement II is false
(a) Cadmium (b) Lead 34. The first education commission in Independant India
(c) Nitrate (d) Selenium is:
29. Which of the following Green House bases were the (a) National Knowledge Commission
target gases whose emission was to be covered under (b) Secondary Education Commission
the first commitment period of the Kyoto protocol? (c) The Education Commission
A. SO2 B. CO2 (d) University Education Commission
C. N2O D. NH3
35. The dynamics of classroom interaction can be effec-
E. CH4 tively managed through
Choose the correct answer from the options given
(a) Interpreting perceived meanings
below: (b) Language complexity
(a) A, B and C only (b) A, B, C and D only (c) Repetitive symbolism
(c) B, C and E only (d) B, C, D and E only (d) Verbosity
30. Match List I with List II 36. Match List I with List II
List I List II List I List II
A. System Software I. Library Management System A. If ‘A’ is True I. ‘E’ is True; ‘O’ is True
B. Application Software II. Firewall B. If ‘E’ is True II. ‘O’ is False; ‘E’ is False
C. Security Software III. Compiler C. If ‘I’ is False III. ‘A’ is False; ‘I’ is False
D. Embedded Software IV. Automatic fridge D. If ‘O’ is False IV. ‘E’ is False; ‘I’ is True
Choose the correct answer from the options given
Choose the correct answer from the options given below:

below: (a) A -I , B -II , C -III , D -IV
(a) A -I , B -III , C -IV , D -II (b) A -II , B -III , C -I , D -IV
(b) A -II , B -IV , C -I , D -III (c) A -III , B -I , C -II , D -IV
(c) A -III , B -I , C -II , D -IV (d) A -IV , B -II , C -III, D-I
(d) A -III , B -IV , C -I , D -II
37. Which of the following are myths about
31. Which of the following are in the “Six Thinking Hats communication?
System”, a form of parallel thinking? A. Good communicators are born, not made
A. Brown Hat B. White Hat B. Stage fright is not a communication apprehension
C. Violet Hat D. Red Hat C. Communication skills have nothing to do with
E. Black Hat relationships
Choose the correct answer from the options given
D. Communication has its own barriers
below: E. Practice leads to the right habits of
(a) A, B and C only (b) B, C and D only communication
(c) B, D and E only (d) C, D and E only Choose the correct answer from the options given

32. How tall is a tree that is 6 meters shorter than a wall below:
that is seven times higher than the tree? (a) A, B and C only (b) A, D and E only
(a) 1 meters (b) 1.25 meters (c) B, C and D only (d) C, D and E only
(c) 2 meters (d) 7 meters 38. Match List I with List II
33. Given below are two statements:
List I List II
Statement I: One’s perceptions and beliefs are the
A. Ashrayasiddha I. Sound is eternal because it is
basis of generalisation in communication.
visible
Statement II: Generalisations of the environment
B. Svarupasiddha II. All real are momentory; sound
develop through experiences. is real; sound is momentory
In light of the above statements, choose the correct C. Vyapyatvasiddha III. All objects are eternal because
answer from the options given below: they are knowable
(a) Both Statement I and Statement II are false
D. Anupasamhari IV. Sky lotus is fragrant because it
(b) Both Statement I and Statement II are true belongs to the class of lotus

Z04_MADAN 07_65901_2021 paper.indd 41 23/12/22 5:13 PM


Z.42 NTA-UGC NET/JRF Paper 1 2021 - Set 2

Choose the correct answer from the options given


(a) A -I , B -III , C -II , D -IV
below: (b) A -II , B -IV , C -I , D -III
(a) A -I , B -III , C -II , D -IV (c) A -III , B -II , C -IV , D -I
(b) A -II , B -IV , C -I , D -III (d) A -IV , B -I , C -III , D -II
(c) A -III , B -II , C -IV , D -I 43. Match List I with List II
(d) A -IV , B -I , C -II , D -III
List I List II
39. The middle term of the inference is related to the
major term neither as a cause nor as an effect but Elements Description
because they are uniformly related to each other in of Effective
our experience, is known as Communication
(a) Pakshdharmata A. Common frame I. Communications with people of
(b) Purvavat of reference similar background
(c) Samanyatodrshta B. Congruence II. Use of understandable
(d) Sheshavat language
C. Language III. Messages of common interest
40. Given below are two statements:
relevance between encoder and decoder
Statement I: A variable measured at the nominal level
D. Compatible IV. C
 ompatibility between the
can be used in higher-level statistics if it is converted environment receiver and the sender
into another form.
Choose the correct answer from the options given

Statement II: The result of the conversion process is
below:
called dummy variable.
(a) A -I , B -II , C -III , D -IV
In light of the above statements, choose the correct (b) A -II , B -III , C -IV , D -I
answer from the options given below:
(c) A -III , B -IV , C -I , D -II
(a) Both Statement I and Statement II are false
(d) A -IV , B -III , C -II , D -I
(b) Both Statement I and Statement II are true
(c) Statement I is false but Statement II is true 44. What number comes next in this sequence?
(d) Statement I is true but Statement II is false 6, 4, 2.5, 1.5, ?
(a) 0.5 (b) -0.5
41. According to Robbins et al, the sequence of barriers to
(c) 1 (d) -1
effective communication in an organisation is
A. Language 45. Gender Parity Index in higher education in India is:
B. Emotions (a) 0.98 (b) 0.99
C. Selective perception (c) 1.1 (d) 1
D. Information load Read the given passage and answer the questions
E. Filtering 46-50:
Choose the correct answer from the options given
Despite the fact that social security programmes in India
below: are not responses to structural adjustment but have a long
(a) A, B, D, E, C (b) B, C, D, A, E history of their own. Social expenditure in India is never-
(c) D, A, E, C, B (d) E, C, D, B, A theless particularly vulnerable to budget cuts. The social
sector is a major spending area of the government, com-
42. Match List I with List II: prising poverty reduction interventions, health, educa-
List I List II tion, nutrition, social assistance and social welfare. Most
departments of government are in some way responsible
Thinking Hat System Concepts and Ideas
for spending under this broad head.
A. White Hat I. Judgement and caution-the The sector is highly divisible, thus facilitating incre-
logical negative mental and piecemeal reductions in real expenditure. It
B. Red Hat II. Data gathering-facts, figures, has a weak political constituency, dominated by techni-
information needs and gaps cal-expert persons, which the mass of evaluative research
C. Black Hat III. Provocations, alternatives that has historically been critical of state interventionism
and creativity-proposals has further weakened.
D. Green Hat IV. Intuition and emotions Such work has been put to uses other than those origi-
nally intended - not to reform the sector but as justifica-
Choose the correct answer from the options given

tion to abolish major components of it entirely. However,
below:

Z04_MADAN 07_65901_2021 paper.indd 42 23/12/22 5:13 PM


NTA-UGC NET/JRF Paper 1 2021 - Set 2 Z.43

since 1991, social sector expenditure has not declined as (c) Indirect implementation
much as had been anticipated. Despite, or because of, its (d) Structural rearrangement
departmental pervasiveness, it is an extremely-perhaps 48. The social sector in India is experiencing
uncuttable-low proportion of GNP: 2.4 per cent as com- (a) A vast amount of speculative literature
pared with 6.5 per cent in Malaysia, 12.2 per cent in
(b) Change in composition and patterns of social
Botswana and 15-25 per cent in OECD countries. Even
welfare
though social sector expenditure is also flawed by spatial
(c) Increased current debt account by certain states
patchiness, conflicting time trends in expenditure levels
to protect their social security measures
and composition and patterns specific to each component
of welfare, cuts have been widely resisted and certain (d) Upward revision of budget allocations in a few
Indian states have increased their current debt in order to specific areas
protect social expenditure. There is, fortunately, and nec- 49. The expert evaluation reports are used to
essarily, speculative literature predicting, sometimes with (a) Justify banishing the social security system
illustrations drawn from elsewhere, the likely outcomes (b) Reform the social welfare sector
of cuts in various types of social expenditure. But it is far (c) Strengthen the political constituency of social
too early for these to be evidence of the actual impact on security
social welfare is a long-term project of several decades. (d) Support the government spending on social
46 The government spending on poverty reduction pro- sector
grammes suffers from 50. The gist of the passage is that
(a) Down-sized expenditure in real terms (a) In actuality, diversification funds to areas other
(b) High demand from prospective beneficiaries than social welfare are justifiable
(c) Several social welfare programmes competing for (b) The social security system in India should be
funds strengthened
(d) Too many divisible allocations (c) The social welfare system must be abandoned
47. Social security programmes in India encounter the (d) There is no evidence as to the utility of social wel-
issue of fare measures in India
(a) Direct implementation
(b) Increased government intervention

Answer Keys
1. (d) 2. (c) 3. (c) 4. (c) 5. (d) 6. (b) 7. (a) 8. (b) 9. (b) 10. (d)
11. (a) 12. (a) 13. (b) 14. (a) 15. (d) 16. (a) 17. (b) 18. (d) 19. (d) 20. (b)
21. (*) 22. (b) 23. (d) 24. (d) 25. (c) 26. (c) 27. (d) 28. (c) 29. (c) 30. (c)
31. (c) 32. (a) 33. (b) 34. (d) 35. (a) 36. (b) 37. (a) 38. (d) 39. (c) 40. (b)
41. (d) 42. (b) 43. (d) 44. (c) 45. (d) 46. (a) 47. (d) 48. (c) 49. (a) 50. (b)

*Note: These answers are likely to be challengeable as per the official answer key provided by UGC.

Z04_MADAN 07_65901_2021 paper.indd 43 23/12/22 5:13 PM


Z.44 NTA-UGC NET/JRF Paper 1 2021 - Set 2

Hints and Solutions


1. (d) State B = 190000 × 46/100 = 87400
Non-qualified candidates (%) = 100 – Number of State C = 255000 × 30/100 = 76500
qualified candidates State D = 300000 × 35/100 = 105000
State-wise number of non-qualified candidates in The average number of candidates = (72600 + 87400
the year 2019 + 76500 + 105000)/4 = 341500/4 = 85375
State A = 185000 × (100 – 28) /100 = 185000 × 5. (d)
72/100 = 133200
Number of qualified candidates from all states
State B = 210000 × (100 – 43) /100 = 210000 × during the year 2017
57/100 = 119700
State A = 20000 × 44/100 = 88880

State C = 250000 × (100 – 42) /100 = 250000 ×
58/100 = 145000 State B = 172000 × 41/100 = 70520

State D = 270000 × (100 – 34)/100 = 270000 × State C = 245000 × 36/100 = 88200

66/100 = 178200 State D = 310000 × 32/100 = 99200

Total non-qualified candidates = 133200 + 119700 Thus, the total candidates = 88880 + 70520 + 88200

+ 145000 + 178200 = 576200 + 99200 = 346800
Total non-qualified candidates
Number of qualified candidates from all states
Required Average =
Given number of states during the year 2018
576200 State A = 198000 × 39/100 = 77220

=
= 144025
4 State B = 202000 × 37/100 = 74740

2. (c) State C = 220000 × 33/100 = 72600

Number of qualified candidates from State B dur- State D = 290000 × 31/100 = 89900

ing the given years Thus, the total candidates = 72220 + 74740 + 72600

Year 2016 = 164000 × 42/100 = 68880 + 89900 = 314460
Year 2017 = 172000 × 41/100 = 70520 Thus, difference = 346800 – 314460 = 32340
Year 2018 = 202000 × 37/100 = 74740 6. (b) Research Aptitude
Year 2019 = 210000 × 43/100 = 90300 The normal distribution curve is mostly used in
Year 2020 = 190000 × 46/100 = 87400 hypothesis testing in research. There are basically two
Thus, in the year 2019, the highest number of candi- types of hypothesis—Null hypothesis and Alternative
dates cleared the entrance exam from State B. hypothesis. The null hypothesis is generally consid-
ered to be the ‘accepted fact’ that there is no differ-
3. (c)
ence between dependent and independent variables
Number of qualified candidates from State A dur- or else we may assume that the population parameter
ing the given years and the sample statistic. The null hypothesis is often
Year 2016 = 231000 × 32/100 = 73920 an initial claim that is based on previous analysis or
Year 2017 = 202000 × 44/100 = 88880 special knowledge. The null hypothesis is considered
the opposite of the alternative hypothesis.
Year 2018 = 198000 × 39/100 = 77220
Researchers also come up with an alternative hypoth-
Year 2019 = 185000 × 28/100 = 51800
esis that they think explains a phenomenon. Research-
Year 2020 = 220000 × 33/100 = 72600 ers try to reject or disprove the null hypothesis by
Total number of qualified candidates during the years means of sampling, so that the alternative hypothesis
2016 to 2020 = 73920 + 88880 +77220 +51800 is accepted as true. For this work, researchers also
+72600 = 364420 take the help of confidence level / level of signifi-
4. (c) cance, which can be 1%, 5% or 10%. The area under
the entire normal distribution curve is 1, or is 100 per-
Number of qualified candidates from all the states
cent. The Z-table or T-table helps by telling us what
during the year 2020
percentage is under the curve at a particular point.
State A = 220000 × 33/100 = 72600

Z04_MADAN 07_65901_2021 paper.indd 44 23/12/22 5:13 PM


NTA-UGC NET/JRF Paper 1 2021 - Set 2 Z.45

Mean = Median = Mode 3. Full duplex data transmission: Full duplex


data transmission occurs in both directions
Symmetrical sides simultaneously.
4. Serial data transmission: Serial Transmission is
a form of signal transmission that sends informa-
tion one bit at a time over a single data channel.
Serial interfaces are generally used to connect
Asymptotic tail
data communications equipment (DCE) such as
modems to data terminal equipment (DTE) such
as computers and terminals and for connecting a
DCE to a DTE.
The characteristics of the normal distribution curve 5. Parallel data transmission: In parallel transmis-
are given as under: sion (e.g., 8 bits) each bit uses a separate wire to
1. It is bell shaped curve and symmetrical in its transfer data on a parallel link, a separate line is
appearance. used as a clock signal. This serves to inform the
receiver when data is available.
2. The height of normal distribution curve is at its
maximum at the mean. Hence the mean, mode and 9. (b) Research Aptitude
median coincide at the centre of the distribution. Solomon four group design: It is a research design
3. The curve is a based on an infinite number of that attempts to take into account the influence of
observations. pretesting on subsequent post-test results. Some
research design includes a pre-test, which is taken
4. Since, there is only one maximum point, the nor-
before exposure to a treatment and a post-test which
mal curve is unimodal, i.e., has only one mode.
is administered after exposure to a treatment.
5. The first and third quarter are equidistant from
The post-test only control design: It is a research
mean.
design in which there are at least two groups, one of
7. (*) People development and environment which does not receive a treatment or intervention
In India according to NITI Ayog’s SDG Index Gujarat and data are collected on the outcome measure after
and the Kerala is the best performing state on SDG 9 the treatment. The group does not receive the treat-
(Industry, Innovation and Infrastructure). ment of interest is the control group.
SDG India Index is an index prepare by the NITI Ayog The main features of post-test are as follows:
to measure the progress of the country and that of • This type of design has two randomly assigned
individual states based on the United Nations initi- groups an experimental group and a control group.
ated Sustainable Development Goals (SDG). The pur-
• Neither group is pretested before implementing the
pose the NITI Ayog index is to estimate the progress
treatment.
of Indian states and UTs through a single measurable
index that would serve as a policy tool and initiate • The treatment is applied only to the experimental
action at the state level. group.
Here, we may talk about Good Governance Index – One-shot case study: It is difficult to draw conclu-
2021 as well, under which Gujarat is at the first place. sions as there is no typical cause and effect relation-
Maharashtra is at second place and Goa at third place. ship between the intervention and the outcome.
Gujarat has excelled in economic administration, Correlational design: As the name indicates,
human resource development, public infrastructure the purpose of correlational studies is to explore
and facilities, social welfare and development and whether there is any relationship or interdepend-
judicial and public security. ence between two variables or characteristics and
8. (b) Information and Communication Technology to ascertain the degree of such relationships. The
value of correlational research is to discover rela-
1. Simplex data transmission: Simplex is the data
tionships among different phenomena with a view
transmission mode in which the data can flow
to predict and in some situations control their
only in one direction, i.e., the communication is
occurrence.
unidirectional.
10. (d) Mathematical Aptitude and Reasoning
2. Half duplex data transmission: Half duplex data
transmission is transmission in both directions but The number 89325324 is divisible by 66. This is a
not at the same time. direct question. Thus, statement I is true.

Z04_MADAN 07_65901_2021 paper.indd 45 23/12/22 5:13 PM


Z.46 NTA-UGC NET/JRF Paper 1 2021 - Set 2

The set of natural numbers is not same as the set of


the dependent variable changes in relation to a varia-
the whole numbers. The difference between natural tion in the independent variable.
numbers and whole numbers is as follows: If an extraneous variable is the real reason for an

Natural numbers include all the positive integers from
outcome instead of the independent variables, then
1 till infinity (∞) and are also used for counting pur- it is also known as a confounding variable because it
pose. The set of natural numbers is represented by the has confused or confounded the relationship, we are
letter ‘N’. 0 is not a natural number. interested in.
N = {1, 2, 3, 4, 5, 6, 7, 8, 9, 10… and so on }
Smoking Coronary heart disease

Whole numbers include all the natural numbers and


(Assumed cause) (Assumed effect)
the number 0. In other words, all natural numbers are Independent variable Dependent variable
whole numbers, but all whole numbers are not natu-
ral numbers. Affect the relationship

Whole Numbers = {0, 1, 2, 3, 4, 5, 7, 8, 9...}


• The age of the person
• The extent of his/her smoking
• The duration of smoking
• The extent of daily exercise, etc.
Whole numbers Extraneous variables
(0, 1, 2, 3, 4, 5, 6......)
Relationship between Independent, Dependent and Extraneous Variables

13. (b) People, Development and Environment


The country currently has the fourth highest wind

installed capacity in the world with total installed
Natural numbers capacity of 39.25 GW (as on 31st March 2021) and
(1, 2, 3, 4, 5, 6......) has generated around 60.149 billion units during
2020–21.

Potential of Wind Energy in India
The Government, through National Institute of Wind

Energy (NIWE), has installed over 800 wind-moni-
11. (a) Teaching Aptitude toring stations all over the country and issued wind
Competencies refer to skills or knowledge that led to
potential maps at 50m, 80m, 100m and 120m above
superior performance. ground level. The recent assessment indicates a gross
wind power potential of 302 GW in the country at 100
Personal competencies determine how we manage
metres and 695.50 GW at 120 metres above ground
ourselves while the social competencies determine level. Most of this potential exists in seven windy
how we handle relationships. Thus, both the state- states given below:
ments are correct.
Wind Energy Wind
12. (a) Research Aptitude S.
State Potential Potential at
No.
Extraneous variable: An extraneous variable is also
(1000 m GW) 120 m (GW)
known as artifact. In real life situations, there can be 1. Gujarat 84.43 142.56
many factors or variables that may affect the outcome. 2. Rajasthan 18.77 127.75
These variables are termed as extraneous variables.
3. Maharashtra 45.39 98.21
Extraneous variables are also affecting the dependent
variable, although these are not manipulated by the 4. Tamil Nadu 33.79 68.75
researcher. 5. Madhya 10.48 15.40
Pradesh
They may mask the relationship between independ-

ent variable and dependent variable. Extraneous 6. Karnataka 55.85 124.15
variable may directly affect the dependent variable or 7. Andhra 44.22 74.90
may combine with the independent variable to pro- Pradesh
duce an affect.
Thus, the correct sequence of wind energy potential

Therefore, extraneous variables must be controlled so
at 100 m above ground level in decreasing order are
that the experimenter can determine whether or not as:

Z04_MADAN 07_65901_2021 paper.indd 46 23/12/22 5:13 PM


NTA-UGC NET/JRF Paper 1 2021 - Set 2 Z.47

Gujarat > Karnataka> Andhra Pradesh > Tamil Nadu In certain circumstances, financial assistance is also

> Andaman and Nicobar Islands given to such places by this committee. So far (as of
14. (a) Information and Communication Technology July 2021) about 1154 sites have been declared as
World Heritage Sites all over the world which includes
World Wide Web: It is the component of the Internet
897 cultural, 218 natural, 39 mixed and 138 other
that combines audio, video and graphics with text is
sites.
also called the web or simply the WWW. It is a subset
of the Internet and is a collection of documents and The ancient Taxila and Nalanda University have been

application residing on the internet and is a collection declared as heritage sites by UNESCO.
of documents and applications residing on the Inter- Nalanda: Nalanda Mahavihar is a world heritage site

net servers around the world. Thus, the World Wide that is 98 km away from Patna. Nalanda stands out
Web (WWW) is the collection of webpages stored on as the most ancient university of the Indian subcon-
websites. tinent. It engaged in the organised transmission of
Hypertext Preprocessor (PHP): PHP is a popular knowledge interrupted period of 800 years. The his-
scripting language, used to create attractive, user- torical development of the site testifies the develop-
friendly and interactive web pages. PHP is open- ment of Buddhism into a religion and the flourishing
source which means it is well documented and can be of monastic and educational traditions.
easily downloaded for free from the web. It was cre- Takshashila: Takshashila (or Taxila) was an ancient

ated in the year 1994 by Rasmus Lerdorf. Indian educational centre that was founded in
Interconnected network: It is a set of computer 1000 BCE. This produced some illustrious historical
nodes, including, but not limited to, personal comput- personalities:
ers, mobile devices, and physical or virtual servers, (a) Kautilya who wrote Arthashastra

which are linked together to facilitate the transmis- (b) Panini- the compiler of ‘Ashtadhyayi’ the Sanskrit

sion of data between users. grammar
Web Browser: Web browser is an application software (c) Charaka the great Ayurvedic healer (author of

to access the World Wide Web. When a user requests a Charak Smhita)
web page from a particular website, the web browser
(d) Jivaka who once treated Buddha.

fetches the required content from the web server and
then it displays the page on the user’s device. But there was not much formal (or centralized) sys-

Web browsers are used on a wide variety of devices, tem of education in Takshashila. It did not award
including desktops, laptops, tablets, and smart- degrees to its students. The education system there
phones. The most used browser is Google Chrome, was flexible, and was customizable towards students.
with 65% of the global market share, followed by The faculty here worked with complete autonomy,
Safari at 18%. forming their own school of learning with their spe-
A web browser is not the same as a search engine. A cific set of rules and manner.
search engine is a website that provides links to other The manner of functioning of Takshashila differed

websites. However, in order to connect to a website’s drastically from Nalanda University that was more
server and display its web pages, the user must have a structured than Taxila.
web browser installed in the system. By some accounts, the University of Ancient Taxila

15. (d) Higher Education System was considered to be one of the earliest universities in
UNESCO World Heritage Sites are called special the world.
places that are selected by the World Heritage Sites Pushpagiri: This vihar (or set of monuments) was

Committee. The purpose of this program is to select an ancient Buddhist mahavihara. It is also termed as
and preserve such sites of the world which are impor- monastic complex located atop Langudi Hill in Jajpur
tant to humanity from the point of view of world cul- district of Odisha. The visit of Xuanzang to Pushpagiri
ture. For example, forest area, mountain, lake, desert, vihar indicates that it was an important Buddhist site
monument, university, building, city, etc. in ancient India. Like Nalanda, Vikramashila, Odanta-
Each heritage site is the property of the particular puri, Takshashila and Vallabhi, it was a major ancient
country in which the site is located. It is also in the centre of learning. It progressed between 3rd and
interest of the international community to preserve 11th centuries CE.
them for future generations and for the benefit of Valabhi: The Valabhi University was an important

humanity. In this way, the responsibility of their pro- centre of Buddhist learning. It championed the cause
tection rests with the whole world community. of Hinayana Buddhism between 600 ce and 1200 ce.

Z04_MADAN 07_65901_2021 paper.indd 47 23/12/22 5:13 PM


Z.48 NTA-UGC NET/JRF Paper 1 2021 - Set 2

Valabhi was the capital of the Maitraka empire during sional conduct, etiquettes, and exercises disciplinary
the period 480–775 ce. jurisdiction over the bar. It also sets standards for
Mithila: Ancient Mithila University was famous for legal education and grants recognition to universi-
Nyaya Shastra and logical sciences. ties whose degree in law will serve as a qualification
for students to enrol themselves as advocates upon
16. (a) People, Development Environment
graduation.
Geothermal power plants provide electricity con-
NCTE: The National Council for Teacher Educa-
sistently running 24 hours per day, 7 days a week,
tion, in its previous status since 1973, was an advi-
regardless of weather conditions. The power output
sory body for the Central and State Governments on
of a geothermal power plant is highly predictable and
all matters pertaining to teacher education, with its
stable. The disadvantages of geothermal power plant
Secretariat in the Department of Teacher Education
location is restricted and also runs the risk of trig-
of the National Council of Educational Research and
gering earthquakes. This is due to alterations in the
Training (NCERT). The National Council for Teacher
Earth’s structure as a result of constant digging.
Education as a statutory body that came into exist-
Nuclear power is the only energy source that deliv-
ence in pursuance of the National Council for Teacher
ers carbon-free, reliable energy 24 hours a day. Along Education Act, 1993 on the 17th August, 1995. The
with wind, solar, and energy storage, nuclear has a main objective of the NCTE is to achieve planned and
vital role in our carbon-free future. coordinated development of the teacher education
It is a Carbon-Free Energy. We can get 24/7/365 days.
system throughout the country, the regulation and
The advantages include: proper maintenance of Norms and Standards in the
• Nuclear energy is carbon-free.
teacher education system and for matters connected
therewith.
• Nuclear energy is reliable.

AICTE: All India Council for Technical Education
• Nuclear energy is powerful.
(AICTE) was set up in November 1945 as a national
– level Apex Advisory body to conduct a survey on
17. (b) Logical Reasoning
the facilities available for technical education and to
Existential fallacy: In modern times, presupposition promote development in the country in a coordinated
that a class has members is seen as unacceptable. and integrated manner.
In 1905, Bertrand Russell wrote an essay entitled,
NMC: National Medical Commission (NMC) is an
‘The Existential Import of Proposition’, in which he
Indian regulatory body of 33 members which regu-
called this Boolean approach ‘Peano’s interpreta-
lates medical education and medical professionals.
tion’. In an existential fallacy, we presuppose that a
It replaced the Medical Council of India on 25 Sep-
class has members even when we are not explicitly
tember 2020. The Commission grants recognition of
told so, that is, we assume that the class has existen-
medical qualifications, gives accreditation to medical
tial import.
schools, grants registration to medical practitioners,
Exclusive premise: The fallacy of exclusive premises monitors medical practice and assesses the medical
is a syllogistic fallacy committed in a categorical syl- infrastructure in India.
logism that is invalid because both of its premises are
19. (d) Information and Communication Technology
negative.
Ethernet: Ethernet is a family of wired computer net-
Illicit major: Illicit major is a formal fallacy commit-
working technologies commonly used in Local Area Net-
ted in a categorical syllogism that is invalid because
works (LAN), Metropolitan Area Network (MAN) and
its major term is undistributed in the major premise
Wide Area Networks (WAN). It was commercially intro-
but distributed in the conclusion.
duced in 1980S. Systems communicating over Ethernet
Undistributed middle: The fallacy of the undistrib- divide a stream of data into shorter pieces called frames.
uted middle is a formal fallacy that is committed Each frame contains source and destination addresses,
when the middle term in a categorical syllogism is not and error checking data so that damaged frames can be
distributed in either the minor premise or the major detected and discarded; most often, higher-layer proto-
premise. It is thus a syllogistic fallacy. cols trigger retransmission of lost frames.
18. (d) Higher Education System Ethernet is widely used in homes and industry, and
BCI: The Bar Council of India is a statutory body interworks well with Wi-Fi technologies. The Inter-
established under the section 4 of Advocates Act net Protocol is commonly carried over Ethernet, and
(1961) that regulates the legal practice and legal hence, it is considered one of the key technologies
education in India. It prescribes standards of profes- that make up the Internet.

Z04_MADAN 07_65901_2021 paper.indd 48 23/12/22 5:13 PM


NTA-UGC NET/JRF Paper 1 2021 - Set 2 Z.49

20. (b) Higher Education System National Law Schools, the All-India Institute of Medi-
SAWAYAM PRABHA satellite: The SWAYAM cal Science, and other autonomous institutes.
PRABHA is a group of 34 DTH channels devoted to 22. (b) Information and Communication Technology
telecasting high-quality educational programmes Mind mapping: A mind map is a diagram used to visu-
across the country on 24 × 7 basis using the GSAT-15 ally organize information. A mind map is hierarchical
satellite. Every day, there will be new content for at and shows relationships among pieces of the whole. It
least (4) hours which would be repeated 5 more times is often created around a single concept, drawn as an
in a day, allowing the students to choose the time of image in the centre of a blank page, to which associ-
their convenience. The channels are up linked from ated representations of ideas such as images, words
BISAG, Gandhinagar. The contents are provided by and parts of words are added. Major ideas are con-
NPTEL, IITs, UGC, CEC, IGNOU, etc. The INFLIBNET nected directly to the central concept, and other ideas
Centre maintains the web portal. branch out from those major ideas.
21. (*) Higher Education System Thus, mind mapping is the technique which uses
Central university: Central universities or union uni- images that are often more evocative than words and
versities in India are established by an Act of Parlia- are precise and potent in triggering a wide range of
ment and are under the purview of the Department of associations and thinking patterns.
Higher Education in MoE. In general, universities in 23. (d) Information and Communication Technology
India are recognised by UGC, which draws its power
Digital Versatile Disc (DVD): DVD is a digital opti-
from the UGC Act 1956.
cal disc data storage format invented and developed
As of 31 March 2021, the list of central universities in 1995 and released in late 1996. Currently allowing
published by the UGC includes 54 central universi- up to 17.08 GB of storage, the medium can store any
ties. E.g., Jamia Milia Islamia is a central university. kind of digital data and was widely used for software
State universities: State universities are run by the and other computer files as well as video programmes
respective state government of the states and union watched using DVD player. DVD offers higher storage
territories of India, and are usually established by a capacity than compact discs while having the same
local legislative assembly act. Panjab university is dimensions.
considered both central and state university. There were several formats developed for recording
Deemed universities: The Governor of the State is video on optical discs before the DVD. Optical record-
the chancellor of deemed universities. The status of ing technology was invented by David Paul Gregg and
a deemed to be university allows full autonomy in James Russell in 1963 and was first patented in 1968.
courses, syllabus, admissions and fees. According to Thus, DVDs use a single spiral track to store data
the new norms in January, 2019, applicant institu- working from the centre to the edge.
tion must be in operation for at least 20 years, have
24. (d) Mathematical Aptitude and Reasoning
100 teachers and 2,000 students, a minimum teacher-
student ratio of 1:20 to get deemed university status. The discount offered on the article = 20%
One-third of the students should be pursuing research SP = ` 24
or post-graduate courses. We can calculate the CP
Private universities: Private universities are 24 × (100 + 20)/100 = ` 30
approved by the UGC. They can grant degrees but are If 30% discount is offered, then
not allowed to have off-campus affiliated colleges.
SP = 30 × (100–30)/ 100 = 30 × 70 / 100 = ` 21
Autonomous institutes: Apart from the above uni-
25. (c) Logical Reasoning
versities, other institutions are granted the permis-
sion to autonomously award degrees. These institutes Mood: Mood depends the type of propositions (A, E,
do not affiliate colleges and are not officially called I or O). The mood of a syllogism is determined by the
universities but autonomous organisations or auton- ‘quantity’ and ‘quality’ of the three propositions.
omous institutes. They fall under the administrative Figure: The figure of a categorical syllogism is deter-
control of the Department of Higher Education, MoE. mined by ‘middle term’. It is a number which corre-
These organisations include the Indian Institutes of sponds to the placement of two middle terms.
Technology, the National Institutes of Technology, the Figure 1: When the middle term is on the left in
Indian Institutes of Science Education and Research, Premise 1 and on the right of Premise 2.
the Indian Institutes of Engineering Science and Figure 2: When the middle term is on the right in
Technology, the Indian Institutes of Management, the both premises.

Z04_MADAN 07_65901_2021 paper.indd 49 23/12/22 5:13 PM


Z.50 NTA-UGC NET/JRF Paper 1 2021 - Set 2

Figure 3: When the middle term is on the left in both but also to human health. It causes thyroid disorders,
premises. viral infections, preeclampsia (high blood pressure),
Figure 4: When the middle term is on the right in swelling, etc.
Premise 1 and on the left of Premise 2. 29. (c) People, Development and Environment
26. (c) Mathematical Aptitude and Reasoning Greenhouse gases are atmospheric gases that absorb
(a) If the speed of A is more than that of B, then to and emit radiation within the thermal infrared range,
travel the same distance, A will take less time thereby, contributing to the greenhouse effect and
than B. global climate change. Many different greenhouse gases
are produced as a result of human activities, including:
(b) In this statement, the speed is given in different
1. Carbon dioxide (CO2)
units. We need to convert km/hr to metre/ minute.
2. Methane (CH4)
Thus, 90 km/h = 90000 m / 60 min = 1500 m / min 3. Nitrous Oxide (N2O)
Thus, it can be concluded that the two speeds 1500 4. Hydrofluorocarbons (HFCs)
m/min and 90 km/h are equal. Hence, statement B is Perfluorinated compounds:
true. 5. Sulphur Hexafluoride (SF6)
(c) Speed = 18 km/h = 18 × 5/18 = 5 m/s 6. Nitrogen trifluoride (NF3)
Time = 2 min 30 sec = 150 sec 7. Perfluorocarbons (PFCs)
Distance = Speed / Time 8. Fluorinated Ether (HFE)
= 5 × 150 = 750 m 9. Per-Flouro Poly Ether (PFPE)
10. Chlorofluorocarbons (CFCs)
Thus, traveling at the speed of 18 km/h, the distance
covered in 2 min and 30 sec is 750 m. Hence, state- 11. Hydrochlorofluorocarbons (HCFC)
ment C is true.
Even water is a GHG.
27. (d) Information and Communication Technology At the Rio Earth Summit in 1992, international nego-

tiators adopted the United Nations Framework Con-
The advantages of observation as a tool of data collec-
vention on Climate Change (UNFCCC). UNFCCC aims
tion are:
to prevent dangerous human interference with the
1. Observations are known for directness. climate system by stabilizing atmospheric concentra-
2. Artificiality can be minimised in observation tions of GHGs at safe levels. The UNFCCC itself has not
studies. set any mandatory limits on GHG emissions for indi-
3. Recording devices can be used in observational vidual countries and has no enforcement mechanism.
studies for accuracy. Instead, the UNFCCC provides updates (the ‘protocol’)
that set mandatory emissions limits. The Kyoto Proto-
28. (c) People, Development and Environment col was adopted in 1997 and came into force in 2005. It
In agricultural areas, groundwater can contain sig- set GHG emission reduction targets for a set of industri-
nificant concentration of the nitrate pollutant. When alized countries. These targets have been implemented
pollutants released on the Earth’s surface enter the in the emission reduction commitment period with the
groundwater, it leads to groundwater pollution. Pol- first period from 2008 to 2012 and the second period
lution can result from workplace sanitation systems, from 2013 to 2020. More than 190 countries have rati-
landfills, waste from wastewater treatment plants, fied the UNFCCC and its Kyoto Protocol.
sewer leaks, petrol filling stations, or overuse of ferti- The Kyoto Protocol aims to cut GHG emissions in the

lizers in agriculture. Therefore, groundwater in agri- developed world by about 5% by 2012 compared to
cultural areas can contain significant amounts of the 1990 levels. In India, the Kyoto Protocol was imple-
nitrate pollutant. mented in February 2005. Kyoto Protocol applies to
Cadmium: Cadmium is a chemical element with the 6 greenhouse gases; Carbon dioxide (CO2), meth-
symbol Cd and atomic number 48. ane (CH4), nitrous oxide (N2O), hydrofluorocarbons
Lead: Lead is a naturally occurring toxic metal found (HFCs), perfluorocarbons (PFCs), sulphur hexafluor-
in the Earth’s crust. Its widespread use has resulted ide (SF6). The seventh gas nitrogen trifluoride (NF3)
in widespread environmental pollution, human expo- was added at a later stage.
sure and public health problems in many parts of the The Intergovernmental Panel on Climate Change

world. (IPCC) has been assisting the UNFCCC in setting
Selenium: Selenium is a toxic element and its high national goals to develop a scientific summary of
concentration is not only harmful to the environment the climate system and the physical basis of climate

Z04_MADAN 07_65901_2021 paper.indd 50 23/12/22 5:13 PM


NTA-UGC NET/JRF Paper 1 2021 - Set 2 Z.51

change. This summary is updated from time to time Green Hat 1. Creative • New ideas
and published in the IPCC Assessment Report. The • Different perspective
Sixth Report (AR6) was published in the year 2021. • Lateral thinking
The IPCC - AR establishes values for the Global Warm- Blue Hat 2. Process • Cool and calm
ing Potential of various green gases. GWP values • Control
describe the radiative effect of a given unit of GHG • Organisation and
relative to (in comparison to) one unit of CO2. process
According to the latest development, the world has
set the year 2050 as the threshold for achieving the Thus, the white hat, red hat and black hat are a form
Zero Emissions target. of parallel thinking in the six-thinking hat system.
30. (c) Information and Communication Technology 32. (a) Mathematical Aptitude and Reasoning
System software: System software provides the basic This is a direct question. It can be solved easily by rea-
functions for computer usage and helps to run the soning method.
computer hardware and system. Compiler is a type of
system software. They convert the source code into
another computer language called object code.
Application software: It helps the user to perform
singular and multiple tasks. Library management sys-
7m
tem is an application software.
Security software: Security software is any type of
software that secures and protects a computer, net-
work or any computing-enabled device. It manages
access control, provides data protection to our PC.
Firewall is an example of security software. 1m

Embedded software: Embedded software is a piece Let the height of tree = X meters
of software that is embedded in hardware or non-PC
Height of wall = 7X
devices. It is written specifically for the particular
hardware that it runs on and usually has processing 7X – 1 X = 6 meters
and memory constraints because of the device’s lim- 6 X = 6 meters
ited computing capabilities. Automatic fridge is an X = 1 meter
example of embedded software.
33. (b) Communication
31. (c) Research Aptitude
One’s perception and beliefs are the basis of generali-
The ‘Six Thinking Hats’ were described by Dr. Edward sation in communication.
Bono. These provide a means of effective thinking
Generalisations of the environment develop through
for parallel thinking groups in a broad and cohesive
experience.
manner.
Thus, both the statements are true.
In this, different hats are used as metaphors for each
direction. 34. (d) Higher Education System
Six Thinking Hats Soon after Independence, the University Education
Commission was set up in 1948 under the Chair-
White Hat 1. Information • Facts manship of Dr. S Radhakrishnan “to report on Indian
• Figures university education and suggest improvements and
• Data
extensions that might be desirable to suit the present
Red Hat 2. Emotion • Gut feel and future needs and aspirations of the country”.
• Intuitions
35. (a) Teaching Aptitude
• How it makes you feel
The dynamic of classroom interaction can be effectively
Black Hat 1. Negative • Why it can’t be done
• What’s hard
managed through interpreting perceived meanings.
• What won’t work Effective teaching: teaching is a planned activity and
Yellow Hat 2. Positive • Hope effective teaching depends on the following factors:
• The bright side (a) How clearly the students understand what they
• Why it will work are expected to learn?

Z04_MADAN 07_65901_2021 paper.indd 51 23/12/22 5:13 PM


Z.52 NTA-UGC NET/JRF Paper 1 2021 - Set 2

(b) How accurately their learning can be measured? must be present in the minor term (pakshadharmata).
Thus, the dynamics of interactions in the classroom If it is not, it is unproved. It is of three kinds.
can be effectively restricted to the interpretation of A. Ashraya asidha: The minor term is the locus of the

understood meanings. middle term. If the minor term is unreal, the middle
36. (b) Logical Reasoning term cannot be present in it. For example,’ the sky-
lotus is fragrant, because it is a lotus, like the lotus of
Contradictories: This is a relationship between two
a lake’.
propositions having the same subject matter, but dif-
fering in both quality and quantity. Under this prop- B. Svarupa asidha: Here the minor term is not

osition A - O and E - I are included. Contradictory unreal. But the middle term cannot be its very nature
propositions cannot be simultaneously true and false. be present in the minor term. For example, ‘sound is
If one is true, the other is necessarily false and vice a quality, because it is visible’. Here, visibility cannot
versa. That is, if ‘A’ is true, then ‘O’ is false; ‘E’ is false. belong to sound which is audible.
Contrary: When two universal propositions differ C. Vyapyatva asidha: Here, Vyapti is conditional

only in quality. By definition, both opposite propo- (sopadhika). We cannot say,’ wherever there is fire
sitions can be both false and true at the same time. there is smoke’. Fire smokes only when it is associated
If one of them is true, the other must necessarily be with wet fuel. A red-hot iron ball or clear fire does not
false. If one is false, the other may be true or false. smoke. Hence, ‘Association with wet fuel’ is a condi-
That is, if ‘E’ is true, then ‘A’ is true; ‘I’ is false. tion necessary to the aforesaid vyapti. Being condi-
tioned, the middle term becomes fallacious if we say:
Sub-contraries: When two particular propositions

‘The hill has smoke because it has fire’.
differ only in quality, the opposites are said to be
sub-contraries. Contrasting propositions can be true 39. (c) Logical Reasoning
simultaneously, but they cannot be false at the same A. Puravavat inference (a prior): Vatsyayana gives
time. If one of them is true, the other may be true or two types of interpretations to purvavat. It is an infer-
false, but if one of them is false, the other must be ence in which the effect is inferred from the cause.
true. That is, if ‘I’ is true, then ‘E’ is true; ‘O’ is true. The inference of rain seeing a cloud is the example
Sub-alternation: When two prepositions differ only in
given.
quantity (one is universal and the other is special), such B. Sheshavat inference (a posterior): Here we infer

an opposition is said to be subordinate. These proposi- the unperceived cause from a perceived effect.
tions may be simultaneously true or simultaneously
For example, when we see a river in flood and infer

false. That is, if ‘O’ is true, then ‘E’ is false; ‘I’ is true.
that there was heavy rain. It is case of Sheshavat
Every S is P No S is P inference.
A Contraries E
C. Samanyatodrista inference (commonly
seen): Samanyatodrista refers to the inference
based on merely experience. Seeing the Sun in the
East in the morning, in the middle in the afternoon
Subalterns Contradictories Subalterns and in the west at evening twilight, the movement
is inferred.
Psychologically, it has been classified into:

I Subcontraries O (i) svartha (one’s own inferential knowledge) and


Some S is P Some S is not P
(ii) parartha (inferential knowledge of another).

37. (a) communication D. Anupsamhari: All matter is eternal, because it is

These are myths about communications: knowable.
• Good communicators are born, not made. Thus, if the cause of the inference is related to the
• Stage fright is not a communication apprehension. end, neither as a cause nor as an action, but they are
equally related to each other in our experience, it is
• Communication skills have nothing to do with
said to be generalisable.
relationships.
40. (b) Research Aptitude
38. (d) Logical Reasoning The nominal scale is also called the classification
The name ‘Asidha’ is attached with first three. This is scale. It allows variables to be measured in one, two,
the fallacy of the unproved middle. The middle term and three subcategories according to their diversity.

Z04_MADAN 07_65901_2021 paper.indd 52 23/12/22 5:13 PM


NTA-UGC NET/JRF Paper 1 2021 - Set 2 Z.53

For example, gender can be classified into two subcat- able. Referring to a person by an adjective (a brain,
egories, male and female. A variable measured at the a diabetic) reduces the person to that one trait. Lan-
nominal level can be used in higher–level statistics if guage that provokes or stereotypes a person poisons
it is converted into another form. the communication process. Language that insults
The result of the conversion process is called dummy an individual (or group) on the basis of age, eth-
variable. nicity, sexual preference, and vulgarity should be
avoided.
41. (d) Communication
Organizational barriers: These barriers develop
According to Robbins et. al, the sequence of barriers
when an organization evolves. They can be attributed
to effective communication in an organisation is:
to the following conditions.
Filtering > Selective perception > Information load
1. Size of organization
> language > Emotions.
2. Physical distance between employees
Filtering: Filtering prevents the members of an organ-
ization from getting a complete picture of a situation. 3. Specialization of jobs and activities
The main objective here is to manage the reactions 4. Organizational culture, which impacts freedom
of a person. During upward communication, people and trust
filter out bad news more. To maximize your chances 5. Organizational rules and regulations
of sending and receiving effective communication, it’s 6. Power structure in the organization
helpful to deliver a message in a variety of ways and 7. Complexity in organizational structure
get information from multiple sources.
8. Inadequate facilities and opportunities
Selective perception: Selective perception refers to 9. Lack of cooperation between seniors and
the filtering of what we see and hear according to our subordinates.
needs. This process is often unconscious. We are bom-
barded every day with a plethora of stimuli of infor- 42. (b) Research Aptitude
mation, whether we want them or not. That’s why we Refer explanation for question 31.
choose the information we need. ‘Selective perception’ White hat: data gathering – facts, figures, informa-
is a time-saving measure. This is an essential tool in a tion needs and gaps.
complex culture, but it can also lead to mistakes. Red hat: intuition and emotions.
Information load: Every type of message reaches us Black hat: judgement and caution the logical negative.
in different ways every day. Some messages are social, Green hat: provocations, alternatives and creativity
some in the form of advertising, some are commercial, – proposals.
such as e-mail, memos, and voice mail, Conversations
43. (d) Communication
with colleagues. There are other personal messages
too, i.e., messages and conversations from our loved Elements of effective communication are:
ones and friends. Dealing with all this becomes a Common frame of reference: Compatibility between
very difficult task at times. This state of imbalance is the receiver and the sender. For example, common
known as information overload. language develops better understanding between the
Emotions: An effective communication requires a sender and receiver of the message and makes com-
sender and a receiver. They should be free to talk munication more effective.
and listen to each other. There could be poten- Congruence: Messages of common interest between
tial differences of opinion or personality between encoder and decoder.
them. Both of them may have to separate them- Language relevance: Use of understandable language.
selves from the flow of emotions if they want to Compatible environment: Communications with
achieve the goal of communicating clearly. A people of similar background.
receiver who is emotionally upset ignores or dis-
torts what the sender is saying. A sender who is 44. (c) Mathematical Aptitude and Reasoning
emotionally upset may be unable to effectively The series follows a difference pattern of –2, –1.5, –1,
present thoughts or feelings. –0.5, and so on.
Words may have different meanings to different peo- 6 4 2.5 1.5 1
ple, or they may not mean anything to another per-
son. This is called semantics.
Language Complexity: Effective communication is
clear, factual and goal-oriented. This is also respect- −2 −1.5 −1 −0.5

Z04_MADAN 07_65901_2021 paper.indd 53 23/12/22 5:13 PM


Z.54 NTA-UGC NET/JRF Paper 1 2021 - Set 2

45. (d) Higher Education System Bachao Beti Padhao’ which have profound impact on
If we define Gender Parity Index (GPI) in terms of education and society and gender inclusion. Student
Gross Enrolment Ratio then it shows the ratio for girls enrolment increased by 11.4% in the last five years,
(or women) to boys (or men) enrolled in educational i.e., from 2015–16 to 2019–20.
institutions. In higher education, it measures the There has been an increase of 18.2% in female enrol-
female to male ratio enrolled. It is a measure of rela- ment in higher education during this period. The
tive access to education. If the GPI is less than 1, girls government has focused more on girls’ education,
are more disadvantaged than boys in terms of access women empowerment and empowerment of socially
to the institution. If both are equal then both sexes backward classes. There has been an increase in the
have the same relative access. A GPI is greater than 1 population of women, scheduled castes and sched-
indicates greater relative accessibility of girls. uled tribes in higher education.
The Gender Parity Index (GPI) in higher education The GPI in higher education in the 2019–20 report
is calculated for the age group of 18–23 years. This is 1.01, which reflects an improvement in relative
ratio reflects progress towards gender equality. It also access to higher education for women in the eligible
serves as an important indicator of women’s empow- age group when compared to men, as opposed to 1.00
erment in society. The data provides state-wise gen- in 2018–19.
der equality index for all categories including SC and Questions 46–50 are taken from Comprehension
ST. passage:
The All India Survey on Higher Education for the
46. (a)
year 2019–20, released in June 2021, showed that 47. (d)
India experienced a significant improvement in GPI.
48. (c)
With due care, UGC has implemented schemes such
as ‘Post Graduate Indira Gandhi Scholarship Scheme 49. (a)
for Single Girl Child’ and Government of India ‘Beti 50. (b)

Z04_MADAN 07_65901_2021 paper.indd 54 23/12/22 5:13 PM

You might also like